Vous êtes sur la page 1sur 594

EBD_7209

nd
• Corporate Office : 45, 2 Floor, Maharishi Dayanand Marg, Corner Market,
Malviya Nagar, New Delhi-110017

Tel. : 011-49842349 / 49842350

By :
Disha Expert

Typeset by Disha DTP Team

DISHA PUBLICATION
ALL RIGHTS RESERVED

© Copyright Publisher
No part of this publication may be reproduced in any form without prior permission of the
publisher. The author and the publisher do not take any legal responsibility for any errors or misrepresentations
that might have crept in. We have tried and made our best efforts to provide accurate up-to-date information in this
book.

For further information about the books from DISHA,


Log on to www.dishapublication.com or email to info@dishapublication.com
CONTENTS
1. The Living World 1-7

2. Biological Classification 8-15

3. Plant Kingdom 16-26

4. Animal Kingdom 27-37

5. Morphology of Flowering Plants 38-47

6. Anatomy of Flowering Plants 48-59

7. Structural Organisation in Animals 60-69

8. Cell: The Unit of Life 70-80

9. Biomolecules 81-88

10. Cell Cycle and Cell Division 89-95

11. Transport in Plants 96-102

12. Mineral Nutrition 103-110

13. Photosynthesis 111-120

14. Respiration in Plants 121-128

15. Plant Growth and Development 129-136

16. Digestion and Absorption 137-145

17. Breathing and Exchange of Gases 146-153

18. Body Fluids and Circulation 154-161

19. Excretory Products and their Elimination 162-169

20. Locomotion and Movement 170-178

21. Neural Control and Coordination 179-188

22. Chemical Coordination and Integration 189-196


EBD_7209
23. Reproduction in Organisms 197-203

24. Sexual Reproduction in Flowering Plants 204-211

25. Human Reproduction 212-225

26. Reproductive Health 226-232

27. Principles of Inheritance & Variation 233-241

28. Molecular Basis of Inheritance 242-251

29. Evolution 252-260

30. Human Health and Disease 261-269

31. Strategies for Enhancement in Food Production 270-275

32. Microbes in Human Welfare 276-281

33. Biotechnology : Principles and Processes 282-288

34. Biotechnology and its Applications 289-294

35. Organisms and Population 295-302

36. Ecosystem 303-309

37. Biodiversity and its Conservation 310-315

38. Environmental Issues 316-323

HINTS & SOLUTION S-1- S-212

MOCK TESTS
Mock Test-1 with solutions MT- 1- MT- 10

Mock Test-2 with solutions MT- 11- MT- 20

Mock Test-3 with solutions MT- 21- MT- 30

Mock Test-4 with solutions MT- 31- MT- 42

Mock Test-5 with solutions MT- 43- MT- 55


1

Chapter
The Living World

7. The fungi, the filamentous algae, the protonema of mosses,


FACT/DEFINITION TYPE QUESTIONS
all easily multiply by _____.
1. Which of the following are unique features of living (a) budding (b) fission
organisms? (c) regeneration (d) fragmentation
(a) Growth and reproduction 8. Which of the following factors exclusively affects
(b) Reproduction and ability to sense environment reproduction in seasonal breeders, both plants and
(c) Metabolism and interaction animals?
(d) All of the above (a) Water (b) Temperature
2. Cell division occurs _______ in plants and _______ in (c) Photoperiod (d) All of these
animals. 9. The sum total of all the chemical reactions occurring in
(a) continuously, only upto a certain age our body is known as
(b) only upto a certain age, continuously (a) metabolism (b) growth
(c) continuously, never (c) regeneration (d) reproduction
10. Which of the following term is used to refer the number of
(d) once, twice
varieties of plants and animals on earth ?
3. In unicellular organisms, with respect to growth and
(a) Taxonomy (b) Identification
reproduction following can be true.
(c) Biodiversity (d) Classification
1. Growth and Reproduction are inclusive events.
11. ICBN stands for
2. unicellular organisms grow by cell division.
(a) International Code of Botanical Nomenclature
3. Both are exclusive
(b) International Congress of Biological Names
(a) Only 1 correct (b) Only 2 correct (c) Indian Code of Botanical Nomenclature
(c) Both 1 and 2 correct (d) Only 3 correct (d) Indian Congress of Biological Names
4. In majority of higher animals and plants, _____ and _____ 12. Binomial nomenclature means
are mutually exclusive events. (a) one name given by two scientists.
(a) growth; nutrition (b) one scientific name consisting of a generic and
(b) nutrition; consciousness specific epithet.
(c) growth; reproduction (c) two names, one latinized, other of a person.
(d) reproduction; consciousness (d) two names of same plant.
5. Non-living objects- 13. In printed scientific names, only the ______ is capitalized.
1. Grows from external surface by collecting substance (a) class (b) species
on it. (c) genus (d) family
2. Grows from internal surface like living. 14. Each category of taxonomic hierarchy refers to as a unit
3. Do not grow at all. of ______.
Which of the followings option is correct? (a) systematic (b) identification
(a) Only 1 correct (b) Only 2 correct (c) nomenclature (d) classification
(c) Only 3 correct (d) All 1, 2, 3 correct 15. Systematics refers to the
6. In multicellular organisms, _____ refers to the production (a) identification and classification of plants and
of progeny possessing features more or less similar to animals.
those of parents. (b) nomenclature and identification of plants and
(a) growth (b) reproduction animals.
(c) metabolism (d) consciousness (c) diversity of kinds of organisms and their relationship.
(d) different kinds of organisms and their classification.
EBD_7209
2 Biology

16. Taxon is a
(a) unit of classification.
STATEMENT TYPE QUESTIONS
(b) species. 29. Which of the following statement(s) is/are correct?
(c) highest rank of classification. (a) Only living organisms grow.
(d) group of closely related organisms. (b) Plants grow only up to a certain age.
17. The practical purpose of classification of living organisms is
(c) The growth in living organisms is from inside.
to
(d) All of the above.
(a) explain the origin of living organsims.
(b) trace the evolution of living organsims. 30. Growth in living organisms occurs by
(c) name the living organisms. (a) division of cells.
(d) facilitate identification of unknown organisms. (b) increase in biomass.
18. Which of the following is not a characteristic of life? (c) accumulation of materrial by external agency.
(a) Reproduction (d) both (a) and (b).
(b) Complex chemical organization 31. Which one of the following is not a correct statement?
(c) Adaptation to environmental changes (a) Botanical gardens have collection of living plants
(d) Differentiation from cells to tissues for reference.
19. Families are characterized on which of the following (b) A museum has collection of photographs of plants
features of plant species? and animals.
(a) External morphology (c) Key is a taxonomic aid for identification of specimens.
(b) Anatomy of parts (d) Herbarium is a store house that contains dried,
(c) Vegetative and reproductive parts pressed and preserved plant specimens.
(d) Seasonal similarities and variations 32. Which two points are known as the twin characteristics
20. The order generally ends with of growth?
(a) ales (b) aceae (i) Increase in mass
(c) eae (d) none of these
(ii) Differentiation
21. Which of the following taxonomic aid provides
(iii) Increase in number of individuals
information for the identification of names of species
found in an area? (iv) Response to stimuli
(a) Monograph (b) Manual (a) (i) and (ii) (b) (i) and (iv)
(c) Flora (d) Periodical (c) (ii) and (iii) (d) (i) and (iii)
22. Which one of the taxonomic aids can give comprehensive 33. Which of the following statements regarding
account of complete compiled information of any one nomenclature is correct?
genus or family at a particular time? (a) Generic name always begins with capital letter
(a) Taxonomic key (b) Flora whereas specific name with small letter.
(c) Herbarium (d) Monograph (b) Scientific name should be printed in italics.
23. Which one of the following taxonomical aid is used for (c) Scientific name when typed or handwritten should
identification of plants and animals based on similarities be underlined.
and dissimilarities? (d) All of the above
(a) Flora (b) Keys 34. Which of the following statements are correct about
(c) Monographs (d) Catalogues herbarium?
24. Herbarium is a (a) It is a store house of collected plant specimens that
(a) garden where medicinal plants are grown. are dried and preserved on sheets.
(b) garden where herbaceous plants are grown. (b) Herbarium sheets contain information about date and
(c) dry garden.
place of collection, names, family, collector’s name,
(d) chemical to kill plants.
25. The famous botanical garden ‘Kew’ is located in etc.
(a) England (b) Lucknow (c) It serves as quick referral systems in taxonomical
(c) Berlin (d) Australia studies.
26. Keys are generally _______ in nature. (d) All of the above
(a) physical (b) chemical 35. Which of the following statements regarding growth is
(c) analytical (d) qualitative incorrect?
27. Each statement in the key is called _______. (a) In plants, growth by cell division is seen only upto a
(a) lead (b) catalogues certain stage.
(c) manuals (d) monographs (b) Growth exhibited by non-living objects is by
28. The keys are based on contrasting characters generally accumulation of material on the surface.
in a pair called _______. (c) A multicellular organism grows by cell division.
(a) flora (b) couplet (d) Growth in in vitro culture of unicellular organisms
(c) both (a) and (b) (d) manuals can be observed by counting the number of cells.
The Living World 3

36. Which of the following statement(s) is/are correct for (iii) Photoperiod affects the process of reproduction.
Metabolism ? (iv) Human being is the only organism who has self
(a) It is the sum total of all physical reactions taking consciousness.
place inside a living system. (a) Only (i) (b) Both (ii) and (iii)
(b) All plants, animals, fungi and microbes exhibit (c) Both (i) and (iv) (d) All of these
metabolism. 42. Which of the following statement(s) is/are correct ?
(c) Isolated metabolic reactions in-vitro are not living (i) Classification is the providing of standardize names
but are living reactions. to the organisms such that a particular organism
known by the same all over the world.
(d) All of the above
(ii) Taxonomy is the correct description of an organism
37. Which one of the following statements is correct about
and to recognition in its scientific name.
biodiversity ? (iii) The system of naming with two components
(a) It is the occurrence of varied type of organisms on (binomial nomenclature) is proposed by Carolus
earth. Linneaus.
(b) Each different kind of plant, animal or organism (iv) Zoological names are based on International Code
represents a species. for Zoological Nomenclature (ICZN).
(c) The number of species that are known and described (a) Only (i) (b) Both (iii) and (iv)
range between 1.7–1.8 million. (c) Only (iv) (d) All of these
(d) All of the above 43. Which of the following taxonomic categories is being
38. Which of the following statements are not correct ? described by the given statements (i-iii) ?
(i) Lower the taxon, more are the characteristics that (i) It is the basic unit of classification.
the members within the taxon share. (ii) It is defined as the group of individuals which
(ii) Order is the assemblage of genera which exhibit a resemble in their morphological and reproductive
few similar characters. characters and interbreed among themselves and
(iii) Cat and dog are included in the same family Felidae. produce fertile offsprings.
(iv) Binomial nomenclature was introduced by Carolus (iii) Human beings belong to the species sapiens which
Linnaeus. is grouped in the genus Homo.
(a) (i), (ii) and (iii) (b) (ii), (iii) and (iv) (a) Species (b) Genus
(c) Order (d) Family
(c) (i) and (iv) (d) (ii) and (iii)
44. Which of the following statement(s) is/are not correct ?
39. Which of the following statements are correct?
(i) Genus comprises a group of related species which
(i) Genus comprises a group of related species.
has more characters in common in comparison to
(ii) Taxon represents a taxonomic group of individual species of other genera.
organisms. (ii) Three different genera such as Solanum, Datura and
(iii) Family comprises a group of related genera. Petunia are placed in the family malvaceae.
(iv) Taxonomic category class includes related orders. (iii) In case of plants, classes with a few similar characters
(a) (i), (ii), and (iv) are assigned to a higher category called phylum.
(b) (ii) and (iv) (a) Both (i) and (ii) (b) Only (iii)
(c) (i), (iii) and (iv) (c) Both (ii) and (iii) (d) All of these
(d) (ii), (iii) and (iv) 45. Which of the following taxonomical aids is being described
40. Which of the following statement(s) is/are not correct ? by the given statements (i-iv) ?
(i) Reproduction is the production of progeny (i) They generally established in educational institutes.
possessing features dissimilar to their parents. (ii) They have collections of preserved plant and animal
(ii) The fungi, the filamentous algae, the protonema of specimens for study and reference.
mosses, all multiply by budding. (iii) Insects are preserved in boxes after collecting, killing
(iii) Many organisms like mules, sterile worker bees do and pinning.
not reproduce. (iv) They often have collections of skeletons of animals
(iv) Reproduction is not an all-inclusive defining too.
(a) Herbarium (b) Museum
characteristic of living organisms.
(c) Zoological parks (d) Botanical gardens
(a) Only (i) (b) Both (i) and (ii)
46. Which of the following statements regarding growth is
(c) Both (ii) and (iv) (d) All of these
incorrect?
41. Which of the following statement(s) is/are correct ? (a) In animals growth is seen up to a certain age.
(i) All living organisms have ability to respond the (b) Increase in body mass is considered as growth.
environment stimuli which could be physical, (c) Growth by cell division occurs continuously
chemical or biological. throughout their life span in animals.
(ii) Plants respond to external factors like light, water, (d) Increase in mass and number of individuals is the
temperature, other organisms, pollutants, etc. characteristics feature of animal growth.
EBD_7209
4 Biology

47. Select the correct statements (i-v) regarding taxonomic 52. Dicotyledons like mango, brinjal and monocotyledons like
categories. Wheat and Rice are placed under a taxonomic category
(i) Each step or rank in hierarchy is called taxonomic known as
category. (a) Phylum – Angiospermae
(ii) Species is a group of individual organisms with (b) Division – Angiospermae
(c) Class – Angiospermae
fundamental similarities capable of breeding among
(d) Sereis – Polypetalae
themselves.
53. Amongst the given taxonomic aids, how many are
(iii) Taxonomic studies of all unknown organisms have associated with preservation of specimens?
led to the development of common categories like Monograph, Flora, Key, Museums, Botanical Gardens,
kingdom, phylum or division, class, order, family, Catalogue, Herbarium, Manual
genus and species. (a) One (b) Three
(iv) Lower the category, greater is the difficulty of (c) Two (d) Four
determining the relationship to other taxa at the same
level. ASSERTION/REASON TYPE QUESTIONS
(a) (i) & (ii) only (b) (ii) & (iv) only In the following questions, a statement of Assertion is followed
(c) (i), (ii) & (iii) only (d) All of the above by a statement of Reason.
48. Which of the following statements is incorrect? (a) If both Assertion and Reason are true and the Reason is
(a) The scientific name for humans is Homo sapiens. the correct explanation of the Assertion.
(b) Organisms placed in the same genus are least closely (b) If both Assertion and Reason are true but the Reason is
related. not the correct explanation of the Assertion.
(c) Moving from species to kingdom, more different (c) If Assertion is true but Reason is false.
(d) If both Assertion and Reason are false.
species are included in each higher category.
(d) Species that are in the same genus share very specific 54. Assertion : Taxon and category are different things.
Reason : Category shows hierarchical classification.
characteristics.
55. Assertion : The species is reproductively isolated natural
49. Study the following statements and select the correct population.
description of botanical garden. Reason : Prokaryotes cannot be kept under different
(i) Plant species are grown for identification purposes. species on the basis of reproductive isolation.
(ii) Labeling of each plant consists of its botanical name/ 56. Assertion: Information given on the label of herbarium
scientific name and its family. sheets does not include family.
(iii) Specimens are preserved in the jars and containers. Reason: Families are characterized on the basis of
(iv) It is a type of store house which contains dried, reproductive features of plant species only.
pressed and preserved plant specimens on sheet. 57. Assertion: Flora contains the actual account of habitat
and distribution of plants of a given area.
(v) Plant specimen contains a labeling of information
Reason : Flora helps in correct identification.
about date and place of collection. 58. Assertion : Panthera is a polytypic genera.
(a) (i) and (ii) only (b) (i), (ii) and (iii) only Reason : Panthera has specific epithets like leo, tigris
(c) (ii) and (iv) only (d) All the five statements. and pardus.
50. Which of the following is the correct sequence of the 59. Assertion : Biological concept of species is based on
various steps of Herbarium formation? reproductive isolation.
(1) Drying (2) Poisoning (3) Collection Reason : Most accepted species concept was given
(4) Labelling (5) Mounting (6) Deposition by Linnaeus.
\(7) Stitching 60. Assertion : Botanical Gardens are ex-situ conservation
strategy of plants.
(a) 3, 1, 2, 5, 7, 4, 6 (b) 3, 2, 4, 5, 6, 7, 1
Reason : National botanical Garden is situated at
(c) 3, 1, 2, 5, 7, 6, 4 (d) 3, 1, 2, 7, 5, 6, 4 Howrah.
51. Which of the following pair is correctly matched. 61. Assertion : Scientific names for plants have been
(i) Fungi – Regeneration standardized through ICBN.
(ii) Mossess – Fragmentation Reason : Naming system which used three word format
(iii) Planaria – Budding was given by Linnaeus.
(a) (i) and (ii) (b) Both (i) and (iii) 62. Assertion : Two plants A and B are treated as two
(c) only (ii) (d) only (iii) taxonomic species.
Reason : Both A and B are different in correlated
characters.
The Living World 5

MATCHING TYPE QUESTIONS (a) A – III; B – IV; C – I; D – II


(b) A – IV; B – III; C – I; D – II
63. Match the common name given in column I with their
(c) A – IV; B – III; C – II; D – I
taxonomic category order given in column II and choose
(d) A – IV; B – I; C – III; D – II
the correct combination from the options given below.
67. Match column I with column II and choose the correct
Column-I Column-II
option.
(Common name) (Taxonomic
Column-I Column-II
category–Order)
A. Family I. tuberosum
A. Wheat I. Primata B. Kingdom II. Polymoniales
C. Order III. Solanum
B. Mango II. Diptera D. Species IV. Plantae
E. Genus V. Solanaceae
C. Housefly III. Sapindales
(a) A – IV; B – III; C – V; D – II; E – I
D. Man IV. Poales (b) A – V; B – IV; C – II; D – I; E – III
(a) A – I; B – II; C – IV; D – III (c) A – IV; B – V; C – II; D – I; E – III
(b) A – IV; B – III; C – II; D – I (d) A – V; B – III; C – II; D – I; E – IV
(c) A – II; B – IV; C – I; D – III 68. Which one of the following animals is correctly matched
(d) A – III; B – IV; C – II; D – I with its particular named taxonomic category ?
64. Match the common name given in column I with their (a) Tiger – Tigris, the species
taxonomic category family given in column II and choose (b) Cuttle fish – Mollusca, a class
the correct combination from the options given below. (c) Humans – Primata, the family
Column-I Column-II
(Common Name) (Taxonomic category– (d) Housefly – Musca, an order
Family) 69. Which of the following taxonomic category of housefly
A. Man I. Poaceae is incorrectly matched?
B. Datura II. Anacardiaceae (a) Genus – Musca
C. Mango III. Solanaceae (b) Family – Muscidae
D. Wheat IV. Hominidae (c) Order – Primata
(a) A – IV; B – III; C – II; D – I (d) Class – Insecta
(b) A – IV; B – III; C – I; D – II
70. Which of the following organisms is not correctly matched
(c) A – I; B – II; C – III; D – IV with its particular ?
(d) A – I; B – III; C – II; D – IV
65. Match column-I (Biological name) with column-II (Class) (a) Human beings – Sapiens, species
and choose the correct option. (b) Lion – Panthera, genus
Column-I Column-II (c) Cats – Felidae, genus
(Biological name) (d) Datura – Solanaceae, family
(Class) 71. In the given columns, column I contains the terms and
A. Homo sapiens I. Dicotyledonae column II contains its description. Select the correct match
B. Musca domestica II. Mammalia from the option given below.
C. Mangifera indica III. Monocotyledonae
D. Triticum aestivum IV. Insects Column I Column II
(a) A – IV; B – II; C – I; D – III A. Growth I. Production of offspring.
(b) A – II; B – IV; C – III; D – I B. Reproduction II. Composed of one or more cells.
(c) A – II; B – IV; C – I; D – III
(d) A – II; B – I; C – IV; D – III
66. Match column-I with column-II and choose the correct C. Metabolism III. Increase in mass and increase in
combination from the options given below. number of individuals.
Column-I Column-II D. Cellular IV. Sum total of all chemical reactions
A. National Botanical I. Carolus Linneaus organization occurring in body.
Research Institute
B. Indian Botanical II. Taxon A B C D
Garden (a) I II III IV
C. Binomial III. Howrah (India) (b) III I II IV
Nomenclature
D. A unit of IV. Lucknow (c) III I IV II
classification (d) II IV III I
EBD_7209
6 Biology

72. Match the taxonomic categories given in column I with A B C D


their feature given in column II. (a) II III IV I
Column I Column II (b) I IV II III
(c) IV I III IV
A . Taxon I. Bas ic u nit o f clas s ificatio n
(d) IV I II III
B. Sp ecies II. A taxo no mic grou p of an y rank
C. Ph ylu m III. Div is io n is th e s ame categ ory in
cas e o f p lants
DIAGRAM BASED QUESTIONS
D. Genu s IV. Id en tified b as ed on a n umber o f 75. Consider following diagram and fill the following blank
s imilar characters
spaces/boxes.
E. Order V. Gro up o f related s pecies hav in g
mo re ch aracter in co mmon with C
o thers s pecies
A B C D E
(a) V II IV III I Phylum/Division
(b) III I IV II V
(c) II I III V IV
(d) III II IV I V Class
73. Select the correct taxonomic aids given in column I with
their feature given in columns II.
Column I Column II
A . Herbariu m I. In clud es th os e s pecimens which B
can b e eas ily clas s ified on th eir
o bs ervable characters .
B. Botan ical II. Pres erved s p ecimen s on s h eets Family
garden b ecome a s to re h ou s e for fu ture
u s e.
C. M u s eum III. Gen erally s et up in ed ucatio nal Genus
in s titu tes .
D. Zoo lo gical IV. In clud es tho s e animals which are
Park id entified bas ed o n their ag greg ates
A
o f characters .
E. Key V. A ll animals p ro vid ed s imilar (a) A-Strain, B-Order, C-Sub kingdom
con ditio ns to th eir natural hab itat. (b) A-species, B-order, C-kingdom
VI. In clud es an imals o f related o rders . (c) A-subspecies, B-Tribe, C-Domain
VII. In clud es collectio n o f liv in g p lants (d) A-species, B-Cohort, C-Subdivision
fo r reference.
VIII. It id entifies an imals an d p lants o n CRITICALTHINKING TYPE QUESTIONS
th e bas is o f their s imilarities an d
d is s imilarities . 76. Identify the correct sequence of taxonomic categories.
A B C D E (a) Species ® Genus ® Order ® Class ® Family ®
(a) I II VIII V III Phylum/Division ® Kingdom
(b) III I IV II V (b) Species ® Genus ® Family ® Class ® Phylum/
(c) II VII III V VIII Division ® Order ® Kingdom
(d) II III VII I IV (c) Species ® Genus ® Family ® Order ® Class ®
74. Select the correct Match. Phylum/Division ® Kingdom
Column I Column II (d) Species ® Genus ® Family ® Order ® Class ®
Phylum/Division ® Kingdom
A . Ex-s itu co ns ervation I. Central n ation al
77. Two plants can be conclusively said to belong to the
Herbarium
same species if they
B. Qu ick referral s ys tem II. M u s eum
(a) have same number of chromosomes.
C. Pres erved plants an d III. Flo ra (b) can reproduce freely with each other and form seeds.
A nimals (c) have more than 90 per cent similar genes.
D. A cco un t of h abitat IV. Roy al Botanical (d) look similar and possess identical secondary
an d d is tribu tio n o f Gard en, Kew. metabolites.
p lant in an area.
The Living World 7

78. ‘Taxa’ differs from ‘taxon’ due to being 89. Genus represents
(a) a higher taxonomic category than taxon. (a) an individual plant or animal.
(b) lower taxonomic category than taxon. (b) a collection of plants or animals.
(c) the plural of taxon. (c) group of closely related species of plants or animals.
(d) the singular of taxon. (d) none of the above.
79. Taxonomic hierarchy refers to 90. The taxonomic unit ‘phylum’ in the classification of
(a) step-wise arrangement of all categories for animals is equivalent to which hierarchial level in
classification of plants and animals. classification of plants ?
(b) a group of senior taxonomists who decide the (a) Class (b) Order
nomenclature of plants and animals. (c) Division (d) Family
(c) a list of botanists or zoologists who have worked on 91. Which of the following is less general in characters as
taxonomy of a species or group. compared to genus?
(d) classification of a species based on fossil record. (a) Species (b) Division
80. Two animals which are the members of the same order (c) Class (d) Family
must also be the members of the same __________. 92. If an organism is in the same class but not in the same
(a) class (b) family family then it may belong to same
(c) genus (d) species (a) genus (b) species
81. One of the most important functions of botanical (c) variety (d) order
gardens is that 93. Refer the botanical name of wheat "Triticum aestivum"
(a) they provide a beautiful area for recreation. and identify the statement which correctly describes it.
(b) one can observe tropical plants there. (a) The second word belongs to genus and starts with a
(c) they allow ex-situ conservation of germ plasm. small letter.
(d) they provide the natural habitat for wildlife. (b) Both the words "Triticum aestivum" denote the
82. The disadvantage of using common names for species is specific epithet.
that (c) The first word Triticum denotes the genus which
(a) the names may change. starts with a capital letter.
(b) one name does not apply universally. (d) The first word Triticum denotes the specific epithet
(c) one species may have several common names and while the second word denotes the genus.
one common name may be applied to two species. 94. Which of the following characters given below displays
(d) all of the above the description of lowest taxonomic category of organisms
83. The most important feature of all living systems is to in the plant and animal kingdom?
(a) utilize oxygen to generate energy. (a) It includes one or more than one order.
(b) replicate the genetic information. (b) It is a group containing one or more families.
(c) produce gametes. (c) It is a group of related organisms that resemble one
(d) utilize solar energy for metabolic activities. another.
84. The common characteristics between tomato and potato (d) It is a group of organisms that are closely related
will be maximum at the level of their__________. and share similar characteristics.
(a) genus (b) family 95. Which of the following shows the correct example of
(c) order (d) division taxonomic category - Genus?
85. Which one of the following is the first step in taxonomy ? (a) Potato, tomato and brinjal belong to Solanum.
(a) Naming (b) Description (b) Monkey, gorilla and gibbon are placed in Mammalia.
(c) Identification (d) Classification (c) Solanum, Petunia, and Datura are placed in Solanacea.
86. Which of the following is the only taxonomic categories (d) Mangifera indica, Solanum tuberosum, and Panthera
that has a real existence? leo.
(a) Phylum (b) Species 96. "X" being a higher category is the assemblage of families
(c) Genus (d) Kingdom which exhibit a few "Y" characters. The "Z" characters
87. In angiosperm, characters of flowers are used in are less in a number as compared to different genera
classification because included in a family. Identify "X", "Y", and "Z".
(a) flowers are attractive. (a) X - Order; Y - Similar; Z - Similar
(b) flowers are large. (b) X - Genus; Y - Similar; Z - Different
(c) character of flowers are conserved. (c) X - Species; Y - Different; Z - Similar
(d) none of the above.
88. As we go from species to kingdom in a taxonomic (d) X - Class; Y - Different; Z - Different
hierarchy, the number of common characteristics 97. Couplet represents the choice made between two opposite
(a) will decrease. options which results in
(b) will increase. (a) rejection of both the option.
(c) remain same. (b) acceptance of both the option.
(d) may increase or decrease. (c) either acceptance or rejection of both the option.
(d) acceptance of only one and rejection of the other.
EBD_7209
2

Chapter
Biological Classification

8. Which of the following is an example of amoeboid


FACT/DEFINITION TYPE QUESTIONS
protozoans ?
1. Which of the following characteristic(s) is/are used by (a) Trypanosoma (b) Paramecium
Whittaker for the classification of organisms ? (c) Gonyaulax (d) Entamoeba
(a) Mode of nutrition 9. Which of the following is a parasitic fungi on the mustard
(b) Thallus organisation plant ?
(c) Phylogenetic relationships (a) Albugo (b) Puccinia
(d) All of the above (c) Yeast (d) Ustilago
2. Organisms of which of the following kingdom do not 10. Which of the following is used extensively in biochemical
have nuclear membrane ? and genetic work ?
(a) Protista (b) Fungi (a) Agaricus (b) Alternaria
(c) Monera (d) Plantae (c) Neurospora (d) Mucor
3. Protists are 11. Which of the following is/are example(s) of
(a) single-celled eukaryotes. deuteromycetes?
(b) multicellular eukaryotes. (a) Alternaria (b) Colletotrichum
(c) single-celled prokaryotes. (c) Trichoderma (d) All of these
(d) single-celled akaryote. 12. Which group of fungi is commonly known as imperfect
fungi ?
4. Which of the following pigment is present in
cyanobacteria? (a) Phycomycetes (b) Ascomycetes
(a) Chlorophyll ‘a’ (b) Chlorophyll ‘b’ (c) Basidiomycetes (d) Deuteromycetes
(c) Chlorophyll ‘c’ (d) Chlorophyll ‘d’ 13. Bladderwort and Venus fly trap are examples of
5. Which of the following is the smallest living cell and can (a) insectivorous plants (b) parasitic plants
live without oxygen? (c) N2 – rich plants (d) aquatic plants
(a) Mycoplasma (b) Mycorrhiza 14. The subunit of capsid is called
(c) Euglena (d) Trypanosoma (a) core (b) nucleotide
6. Which of the following processes are involved in the (c) amino acid (d) capsomere
reproduction of protists ? 15. Which of the following is not a viral disease ?
(a) Binary fission and budding (a) AIDS and mumps (b) Small pox and herpes
(b) Cell fusion and zygote formation (c) Influenza (d) Cholera
(c) Spore formation and cyst formation 16. The symbiotic association between fungi and algae is
(d) All of the above called
7. Which of the following pairs come under the group (a) lichen (b) mycorrhiza
chrysophytes? (c) rhizome (d) endomycorrhiza
(a) Diatoms and Euglena 17. The genetic material of virus includes
(b) Euglena and Trypanosoma (a) only RNA.
(c) Diatoms and Desmids (b) only DNA.
(d) Gonyaulax and Desmids (c) RNA and DNA both
(d) RNA or DNA , i.e., one nucleic acid in a virus.
Biological Classification 9

18. Dikaryon formation is the characteristic feature of (c) Methane is their preferred carbon source.
(a) ascomycetes and basidiomycetes. (d) They are present in guts of several ruminant animals
(b) phycomycetes and basidiomycetes. (cow, buffaloes) and produce biogas (CH4) from the
(c) ascomycetes and phycomycetes. dung of these animals.
(d) phycomycetes and zygomycetes. 27. Which of the following statements is correct for both
19. Clamp connection is found in blue-green algae and bacteria ?
(a) basidiomycetes (b) ascomycetes (a) Both show anaerobic respiration.
(c) saccharomycetes (d) haplomycetes (b) Both have chlorophyll pigment.
20. Plasmogamy is the fusion of (c) Both are devoid of true nucleus.
(a) two haploid cells including their nuclei. (d) None of the above
(b) two haploid cells without nuclear fusion. 28. Which of the following statements is incorrect ?
(c) sperm and egg. (a) TMV has a double-stranded RNA molecule.
(d) sperm and two polar nuclei. (b) Most plant viruses are RNA viruses.
21. Which scientist classified plants into trees, shrubs and (c) The bacteriophage has double-stranded DNA.
herbs and animals into two groups based on absence or (d) Most animal viruses are DNA viruses.
presence red blood cells?
29. Which of the following statement is/ are correct for bacteria?
(a) Aristotle (b) R. H. Whittaker
(a) They are the members of the kingdom monera.
(c) D. J. Ivanowsky (d) W. M. Stanley
(b) They live in extreme habitats such as hot springs,
22. Fungi are filamentous with the exception of "X" which is
deserts, snow and deep oceans.
unicellular. Identify X.
(c) They show the most extensive metabolic diversity.
(a) Yeast (b) Albugo
(d) All of the above
(c) Mucor (d) Lichen
23. The bacteria which oxidize various inorganic substances 30. Which of the following statements is a characteristic
and use the released energy for the synthesis of food are feature of chrysophytes?
called _______________. (a) They are parasitic forms which cause diseases in
(a) Archaebacteria animals.
(b) Heterotrophic bacteria (b) They have a protein rich layer called pellicle.
(c) Photosynthetic autotrophic bacteria (c) They have indestructible cell wall layer deposited
(d) Chemosynthetic autotrophic bacteria with silica.
(d) They are commonly called dinoflagellates.
STATEMENT TYPE QUESTIONS 31. Which of the following statements is correct for
dinoflagellates flagella ?
24. Which of the following statements is not correct for
viruses? (a) A single flagellum lies in the transverse groove
between the cell plates.
(a) Viruses are obligate parasites.
(b) Viruses can multiply only when they are inside the (b) A single flagellum lies in the longitudinal groove
living cells. between the cell plates.
(c) Viruses cannot pass through bacterial filters. (c) Two flagella, one lies longitudinally and the other
(d) Viruses are made up of protein and DNA or RNA transversely in a furrow between the wall plates.
(never both DNA and RNA). (d) Flagella are absent.
25. Which of the following statements is correct for archaea? 32. Choose the correct statements (i – v) regarding
(a) Archaea resemble eukaryotes in all respects. mycoplasma
(b) Archaea have some novel features that are absent in (i) Mycoplasma has no cell wall.
other prokaryotes and eukaryotes. (ii) Mycoplasma is the smallest living organism known.
(c) Archaea completely differ from both prokaryotes and (iii) Mycoplasma cannot survive without O2.
eukaryotes. (iv) Mycoplasma are pathogenic in animals and plants.
(d) Archaea completely differ from prokaryotes. (v) A sort of sexual reproduction occurs in bacterium by
26. Which of the following statements is not correct for adopting a primitive DNA transfer from one bacterium
methanogens? to the other.
(a) They are archaebacteria. (a) Only (iii) (b) (i), (iii) and (v)
(b) They live in marshy areas. (c) (i), (ii), (iv), and (v) (d) All of the above
EBD_7209
10 Biology
33. Read the given statements and answer the question. (a) Archaebacteria (b) Cyanobacteria
(i) It includes unicellular as well as multicellular fungi. (c) Chrysophytes (d) Dinoflagellates
(ii) In multicellular forms, hyphae are branched and 38. Which of the following group of kingdom protista is being
septate. described in the statements given below ?
(iii) Conidiophore produces conidia (spores) (i) This group includes diatoms and golden algae.
exogenously in chain. (ii) They are microscopic and float passively in water
(iv) Sexual spores are ascospores produced currents (plankton).
endogenously in chain. (iii) Most of them are photosynthetic.
(v) Fruiting body is called ascocarp. (iv) They have deposits in their habitat; this accumulation
Identify the correct class of fungi which have all the above over billion of years is referred to as ‘diatomaceous
given characteristics. earth’.
(a) Phycomycetes (b) Sac fungi (a) Dinoflagellates (b) Chrysophytes
(c) Club fungi (d) Fungi imperfecti (c) Euglenoids (d) Slime moulds
34. T. O. Diener discovered a new infectious agent that was 39. The given statements are some characters of a particular
smaller than viruses and have the following group of Kingdom protista.
characteristics. (i) Most of them are fresh water organisms found in
(i) It causes potato spindle tuber disease. standing water.
(ii) It has free RNA. (ii) They have a protein rich layer (called pellicle) which
(iii) Molecular weight of RNA is low. makes their body flexible.
Identify the infections agent. (iii) They have two flagella, a short and a long one.
(a) Viruses (b) Viroids (iv) Though they are photosynthetic in the presence of
(c) Virion (d) Mycoplasma sunlight, but in the absence of sunlight they behave
35. Consider the following statements with respect to like heterotrophs by predating on other smaller
characteristic features of the kingdom. organisms.
(i) In animalia, the mode of nutrition is autotrophic. Identify the correct group on the basis of these characters.
(ii) In monera, the nuclear membrane is present. (a) Protozoans (b) Chrysophytes
(iii) In protista, the cell type is prokaryotic. (c) Slime moulds (d) Euglenoids
(iv) In plantae, the cell wall is present. 40. Which of the following class of fungi is being described
Of the above statements, which one is correct ? by the given statements ?
(a) (i) only (b) (ii) only (i) They are found in aquatic habitats and on decaying
(c) (iii) only (d) (iv) only wood in moist and damp places.
36. Which of the following are the characters of (ii) Mycelium is aseptate and coenocytic.
dinoflagellates? (iii) Asexual reproduction takes place by zoospores
(i) They are planktonic golden yellow algae with soap (motile) or by aplanospores (non-motile).
box like structure. (iv) Some common examples are Mucor, Rhizopus and
(ii) They are marine red biflagellated protista. Albugo.
(iii) They appear yellow, green, brown, blue and red in (a) Ascomycetes (b) Phycomycetes
colour. (c) Basidiomycetes (d) Deuteromycetes
(iv) They are biflagellated organisms with pellicle. 41. Which of the following statement(s) is/are correct about
(v) They are saprophytic (or) parasitic unicellular forms. ascomycetes ?
(a) (ii) and (iii) (b) (ii) and (v) (i) Neurospora, which is used in biochemical and
(c) (i), (ii) and (iii) (d) (ii), (iv) and (v) genetic work is a member of this class.
37. The given characters are seen in which of the following (ii) They are mostly multicellular, e.g., Yeast, or rarely
group? unicellular, e.g., Penicillium.
(i) Unicellular, colonial, filamentous, marine or terrestrial (iii) They are saprophytic, decomposers, parasitic or
forms. coprophilous.
(ii) The colonies are surrounded by a gelatinous sheath. (iv) Some examples are Aspergillus, Claviceps and
(iii) Some can fix atmospheric nitrogen in specialized cells Neurospora.
called heterocysts. (a) Both (i) and (ii) (b) Only (ii)
(iv) They often form blooms in water bodies. (c) (i), (iii) and (iv) (d) All of these
Biological Classification 11
42. Read the following statements and answer the question
ASSERTION/REASON TYPE QUESTIONS
given below
(i) They are saprophytic protists. In the following questions, a statement of Assertion is followed
(ii) Under suitable conditions, they form an aggregation by a statement of Reason.
(called plasmodium) which may grow and spread over (a) If both Assertion and Reason are true and the Reason is
several feet. the correct explanation of the Assertion.
(iii) During unfavourable conditions, the plasmodium (b) If both Assertion and Reason are true but the Reason is
differentiates and forms fruiting bodies bearing not the correct explanation of the Assertion.
spores at their tips. (c) If Assertion is true but Reason is false.
Which of the following class of protists is being described (d) If both Assertion and Reason are false.
by the above statements ? 47. Assertion : Euglena can be placed in the plant knigdom
(a) Euglenoids (b) Dinoflagellates due to the presence of chlorophyll.
(c) Slime moulds (d) Protozoans Reason : Euglena cannot be classified on the basis of two
43. Which of the following statement(s) is/are correct about kingdom system of classification.
class basidiomycetes ? 48. Assertion : Outside a living cell, viruses have must
(i) They are commonly known as imperfect fungi crystalline statements.
because only the asexual or vegetative phases of
these fungi are known. Reason : Viroids have a protein coat.
(ii) They grow in soil, on logs and tree stumps and in 49. Assertion : TMV is a virus which causes mosaic disease.
living plant bodies as parasites, e.g., rusts and smuts. Reason : TMV has RNA as genetic material.
(iii) The mycelium is branched and septate. 50. Assertion : Lichen is important for chemical industries.
(iv) Some common members are Agaricus, Ustilago and Reason : Litmus and Orcein are formed from lichens.
Puccinia. 51. Assertion : Yeasts such as Saccharomyces cerevisiae
(a) Only (i) (b) Both (ii) and (iii) are used in baking industry.
(c) (ii), (iii) and (iv) (d) All of these Reason : Carbon dioxide produced during fermentation
44. Read the following statements and answer the question. causes bread dough to rise by thermal expansion.
(i) Some members are saprophytes or parasites while a
large number of them are decomposers of litter and MATCHING TYPE QUESTIONS
help in mineral cycling.
(ii) They reproduce only by asexual spores known as 52. Match the class of fungi given in column-I with their
conidia. common name given in column-II and select the correct
(iii) Mycelium is septate and branched. option.
(iv) Alternaria, Colletotrichum and Trichoderma are Column-I Column-II
examples of this class. (Class of fungi) (Common name)
Which of the following class of fungi is being described A. Phycomycetes I. Sac fungi
by the above statements ? B. Ascomycetes II. Algal fungi
(a) Phycomycetes (b) Deuteromycetes C. Basidiomycetes III. Fungi imperfecti
(c) Basidiomycetes (d) Ascomycetes D. Deuteromycetes IV. Club fungi
45. Which of the following statement(s) is/are correct ? (a) A – II, B – I, C – IV, D – III
(b) A – II, B – IV, C – I, D – III
(i) Reproduction in fungi can take place by vegetative
means – fragmentation, fission and budding. (c) A – IV, B – I, C – II, D – III
(d) A – IV, B – III, C – II, D – I
(ii) Fusion of two nuclei is called plasmogamy.
53. Match the terms given in column-I with their examples
(iii) Fusion of protoplasms between two motile or non-
motile gametes is called karyogamy. given in column-II and choose the correct option
Column-I Column-II
(iv) Meiosis in zygote results in diploid spores.
(Terms) (Examples)
(a) Only (i) (b) Both (ii) and (iii)
A. Ascus I. Spirulina
(c) (ii), (iii) and (iv) (d) All of these
B. Basidium II. Penicillium
46. Which of the following statements regarding C. Protista III. Agaricus
cyanobacteria is incorrect?
D. Cyanobacteria IV. Euglena
(a) It is also called blue green algae.
E. Animalia V. Sponges
(b) They are chemosynthetic autotrophs. (a) A – II, B – III, C – IV, D – V, E – I
(c) It forms blooms in polluted water bodies. (b) A – I, B – II, C – III, D – V, E – IV
(d) It is unicellular, colonial or filamentous, marine or (c) A – II, B – V, C – III, D – I, E – IV
terrestrial bacteria. (d) A – II, B – III, C – IV, D – I, E – V
EBD_7209
12 Biology
54. Match the class of fungi given in column I with their 58. Match the type of protozoans given in column-I with their
examples given in column II and choose the correct option examples given in column-II and choose the correct option.
Column-I Column-II Column-I Column-II
(Class of fungi) (Examples) (Type of Protozoans) (Examples)
A. Ascomycetes I. Rhizopus A. Amoeboid protozoans I. Paramecium
B. Basidiomycetes II. Penicillium B. Ciliated protozoans II. Plasmodium
C. Deuteromycetes III. Ustilago C. Flagellated protozoans III. Amoeba
D. Phycomycetes IV. Alternaria D. Sporozoans IV. Trypanosoma
(a) A – IV, B – III, C – I, D – II (a) A – I, B – III, C – IV, D – II
(b) A – II, B – III, C – IV, D – I (b) A – III, B – I, C – II, D – IV
(c) A – IV, B – I, C – II, D – III (c) A – III, B – I, C – IV, D – II
(d) A – III, B – IV, C – II, D – I (d) A – III, B – IV, C – I, D – II
55. Match column I (Kingdom) with column II (Class) and
59. Match column-I (Characters/feature) with column-II
select the correct options
(examples) and choose the correct option.
Column-I Column-II
Column-I Column-II
(Kingdom) (Class)
A. Plantae I. Archaebacteria (Characters/features) (Examples)
B. Fungi II. Euglenoids A. Red dinoflagellates I. Rhizopus
C. Protista III. Phycomycetes B. Unicellular fungi used to II. Gonyaulax
D. Monera IV. Algae make bread and beer
(a) A – IV, B – III, C – II, D – I C. Source of antibiotics III. Yeast
(b) A – I, B – II, C – III, D – IV D. Bread mould IV. Penicillium
(c) A – III, B – IV, C – II, D – I (a) A – III, B – II, C – I, D – IV
(d) A – IV, B – II, C – III, D – I (b) A – II, B – III, C – I, D – IV
56. Match the scientists given in column I with their (c) A – II, B – III, C – IV, D – I
discovery given in column II and choose the correct (d) A – II, B – IV, C – III, D – I
option. 60. Which of the following pairs is not correctly matched ?
Column-I Column-II (a) Anabaena – Cyanobacteria
(Scientists) (Discovery) (b) Amoeba – Protozoa
A. Ernst Mayr I. Discovered Viroids (c) Gonyaulax – Dinoflagellates
B. Whittaker II. Gave the name virus
(d) Albugo – Chrysophytes
C. Pasteur III. Proposed five kingdom
61. Which of the following groups of protozoan is not
classification
correctly matched with its feature?
D. Diener IV. Darwin of the
(a) Amoeboid - Marine forms have silica shells on their
20th century
surface.
(a) A – IV, B – III, C – II, D – I
(b) Flagellated - Either free living or parasitic.
(b) A – III, B – IV, C – II, D – I
(c) Ciliated - Actively moving organisms due to presence of
(c) A – II, B – III, C – IV, D – I
cilia.
(d) A – I, B – II, C – III, D – IV
(d) Sporozoans - Move and capture their prey with the
57. Match column I (containing fungus name) with column II
help of false feet.
(common name) and choose the correct options.
Column-I Column-II 62. Select the correct match from the given option.
(Fungus name) (Commonly called) (a) Occurrence of dikaryotic stage - ascomycetes and
basidiomycetes.
A. Puccinia I. Yeast
B. Ustilago II. Mushroom (b) Saprophytes - They are autotrophic and absorb
soluble organic matter from dead substrates.
C. Agaricus III. Smut fungus
(c) Vegetative mean of reproduction in fungi -
D. Saccharomyces IV. Rust fungus
fragmentation, budding and sporangiophores.
(a) A – I, B – II, C – III, D – IV
(d) Steps involved in asexual cycle of fungi - plasmogamy,
(b) A – II, B – III, C – IV, D – I
karyogamy and meiosis in zygote resulting in haploid
(c) A – III, B – IV, C – I, D – II spores.
(d) A – IV, B – III, C – II, D – I
Biological Classification 13
66. The figure given below shows the structure of a
DIAGRAM TYPE QUESTIONS bacteriophage. Identify its parts labelled as A, B, C and D.
63. Refer to the given figures of bacteria cell and Nostoc and
choose the option which shows correct label for the
structures marked as A, B, C, D and E ?

A B C D
(a) Tail fibres Head Sheath Collar
(b) Sheath Collar Head Tail fibres
(c) Head Sheath Collar Tail fibres
(d) Collar Tail fibres Head Sheath
67. The given figure shows the structure of filamentous blue
(a) A – Cell wall, B – Cell membrane, C – Heterocyst, green algae. Nostoc with a structure marked as "X". Select
D – DNA, E – Mucilagenous sheath the option which shows the correct identification of "X"
(b) A – Cell wall, B – Cell membrane, C – DNA, with its feature.
D – Heterocyst, E – Mucilagenous sheath
(c) A – Mucilagenous sheath, B – Cell membrane,
C – DNA, D – Heterocyst, E – Cell wall
(d) A – Cell membrane, B – Cell wall, C – DNA,
D – Heterocyst, E – Mucilagenous sheath
64. Choose the correct names of the different bacteria given
below according to their shapes.

(a) Spores - Reproduction


(b) Heterocysts - Nitrogen fixation
(a) A – Cocci, B – Bacilli, C – Spirilla, D – Vibrio (c) Pellicle - Recycling of nutrition
(d) Mucilaginous sheath - Photosynthesis
(b) A – Bacilli, B – Cocci, C – Spirilla, D – Vibrio
68. The given figure shows some structures labelled as A, B,
(c) A – Spirilla, B – Bacilli, C – Cocci, D – Vibrio C and D. Which structure has the protein coat that
(d) A – Spirilla, B – Vibrio, C – Cocci, D – Bacilli encloses the nucleic acid?
65. Identify the figures A, B and C given below.

(a) A – Euglena, B – Paramecium, C – Agaricus


(b) A – Euglena, B – Planaria, C – Agaricus
(c) A – Planaria, B – Paramecium, C – Agaricus
(a) A (b) B
(d) A – Euglena, B – Paramecium, C – Aspergillus
(c) C (d) D
EBD_7209
14 Biology
77. Which of the following organisms is\are correctly
CRITICAL THINKING TYPE QUESTIONS
assigned its/their taxonomic group?
69. Bacteria lack alternation of generation because there is (a) Paramecium and Plasmodium belong to the same
(a) neither syngamy nor reduction division. kingdom as that of Penicillium.
(b) no distinct chromosomes. (b) Lichen is a composite organism formed from the
(c) no conjugation. symbiotic association of an alga and a protozoan.
(d) no exchange of genetic material. (c) Yeast used in making bread and beer is a fungus.
70. Yeast is not included in protozoans but are placed fungi (d) Nostoc and Anabaena are examples of protista.
because 78. Bacteria are found to be primitive organisms because they
(a) it has no chlorophyll. (a) are small, microscopic which are not seen with naked
(b) some fungal hyphae grow in such a way that they eye.
give the appearance of pseudomycelium. (b) cause serious diseases to human being, domestic
(c) it has eukaryotic organization. animals and crop plants.
(d) cell wall is made up of cellulose and reserve food (c) produce endospores which are very resistant to
material is starch. adverse conditions.
71. A virus can be considered a living organism because it (d) possess incipient nucleus and show amitotic
(a) responds to touch stimulus division.
(b) respires 79. Food can be kept for a longer time in cold house than in
(c) reproduces (inside the host) normal conditions because
(d) can cause disease (a) insect cannot enter.
72. Lichens indicate SO2 pollution because they (b) bacterial multiplication stops.
(a) show association between algae and fungi. (c) bacterial multiplication is reduced.
(b) grow faster than others. (d) there is plasmolysis at low temperature.
(c) are sensitive to SO2. 80. Mycorrhizae are useful for plants because they
(d) flourish in SO2 rich environment. (a) fix atmospheric nitrogen.
73. When a moist bread is kept exposed in air, it becomes (b) enhance absorption of nutrients from the soil.
mouldy and black because (c) kill insects and pathogen.
(a) spores are present in the water. (d) provide resistance against abiotic stresses.
(b) spores are present in the bread. 81. The most abundant prokaryotes helpful to humans in
(c) spores are present in the air. making curd from milk and in production of antibiotics are
categorised as
(d) the bread decomposes.
(a) cyanobacteria
74. In some viruses, RNA is present instead of DNA indicating
that (b) archaebacteria
(a) their nucleic acid must combine with host DNA (c) chemosynthetic autotrophs
before replication. (d) heterotrophic bacteria
(b) they cannot replicate. 82. A specimen of fungus is brought by a student for
(c) there is no hereditary information. identification. Upon close examination, he discovered that
its hyphae are completely septate and it has gills on the
(d) RNA can transfer heredity material.
underside of the pileus. To which fungal group does it
75. Ustilago causes plant diseases (called smuts) because most likely belong ?
(a) they parasitize on cereals. (a) Basidiomycetes (b) Zygomycetes
(b) they lack mycelium. (c) Ascomycetes (d) Chytrids
(c) they develop sooty masses of spores. 83. How many bacteria are produced in four hours if a
(d) their affected parts becomes completely black. bacterium divides once in half an hour ?
76. A fungus contains cells with two nuclei from different (a) 8 (b) 64
genomes. The nuclei do not fuse but divide independently (c) 16 (d) 256
and simultaneously as new cells are formed. This fungus
84. Fungi show asexual reproduction by all of the following
belongs to
kinds of spores except
(a) phycomycetes (b) zygomycetes
(a) conidia (b) oospores
(c) deuteromycetes (d) basidiomycetes
(c) sporangiospores (d) zoospores
Biological Classification 15

85. Assume that two normal hyphal cells of different fungal 90. Identify the basis of classification of fungi into
mating types unite. After a period of time, the cell between phycomycetes, ascomycetes, basidiomycetes and
these cells will dissolve producing a deuteromycetes.
(a) mycelium i. Fruiting bodies
(b) fruiting body ii. Nature of habitat
(c) zygote iii. Morphology of mycelium
(d) dikaryotic cell, which is also heterokaryotic iv. Mode of spore formation
86. Protozons are not included in kingdom animalia because (a) i & ii only (b) ii & iii only
they are (c) i, iii, & iv only (d) All of these
(a) mostly asymmetrical. 91. Which class of fungi lacks sex organs but the process of
(b) unicellular eukaryotes. plasmogamy is brought about by fusion of two vegetative
(c) heterotrophic in nature. or somatic cells of different strains or genotypes?
(d) multicellular prokaryotes. (a) Sac fungi (b) Bracket fungi
87. Bacteria and yeast are similar in all the following features (c) Imperfect fungi (d) Phycomycetes
except that 92. Refer to the statement and answer the question.
(a) both are unicellular. "Once the sexual stage of members of deuteromycetes
(b) both are prokaryotes. were discovered, they were often moved to X and Y."
(c) both are capable of causing fermentation. Identify X and Y.
(d) both produce spores. (a) X - Monera ; Y - Protista
88. Which of the following is the correct sequence of three (b) X - Basidiomycetes ; Y - Phycomycetes
steps in the sexual cycle of fungi? (c) X - Ascomycetes ; Y - Basidiomycetes
(a) Mitosis ® Meiosis ® Fertilization (d) X - Phycomycetes ; Y - Archaebacteria
(b) Plasmogamy ® Karyogamy ® Meiosis 93. A scientist "X" demonstrated that extract of infected
(c) Mitosis ® Plasmogamy ® Karyogamy plants of "Y" could cause infection in healthy plants and
(d) Karyogamy ® Plasmogamy ® Meiosis called the fluid as "Contagium vivum fluidum".
89. An "X" reproduces in such a great numbers that the water Identify X and Y.
may appear, producing a red tides and kills large marine X Y
animals like "Z". "X" belongs to "Y". Identify "X", "Y" (a) W. M. Stanley Potato
and "Z". (b) M. W. Beijerinek Tobacco
(a) X - Gonyaulax ; Y - Dinoflagellates; Z - Fishes (c) D. J. Ivanowsky Cauliflower
(b) X - Paramecium ; Y - Protozoa ; Z - (d) Pasteur Tomato
Crocodiles
(c) X - Trypanosoma ; Y - Protozoa ; Z - Frogs
(d) X - Plasmodium ; Y - Euglenoids ; Z - Oysters
EBD_7209
3

Chapter
Plant Kingdom

FACT/DEFINITION TYPE QUESTIONS 9. What is the number and position of insertions of flagella
in rhodopyceae class of algae ?
1. The natural system of classification for flowering plants (a) 2 - 8, equal, apical
was given by
(b) 2, unequal, lateral
(a) Carolus Linnaeus (b) Bentham and Hooker
(c) Engler and Prantl (d) R. H. Whittaker (c) 2 - 6, equal, lateral
2. Phylogenetic classification system is based on the (d) Flagella are absent in Rhodophyceae
(a) morphological characters of various organisms. 10. In class phaeophyceae, the plant body is usually attached
(b) anatomical characters of various organisms. to the substratum by a A and has a stalk, the B
(c) physiological characters of various organisms. and leaf like photosynthetic organ-the C .
(d) evolutionary relationships between the various (a) A – holdfast, B – stipe, C – frond
organisms. (b) A – stipe, B – holdfast, C – frond
3. Cytological information like chromosome number, (c) A – frond, B – stipe, C – holdfast
structure, behaviour are related with
(d) A – stipe, B – frond, C – holdfast
(a) numerical taxonomy (b) cytotaxonomy
11. Which of the following groups of plants play an important
(c) chemotaxonomy (d) all of these role in plant succession on bare rocks/soil ?
4. Flagellate isogametes and anisogametes are found in
(a) Algae (b) Bryophytes
(a) Spirogyra (b) Fucus
(c) Pteridophytes (d) Gymnosperms
(c) Volvox (d) Chlamydomonas
12. _______ represent the reproductive organs amongst
5. Fusion of two gametes which are dissimilar in size is termed
as _______. gymnosperms.
(a) isogamous (b) oogamous (a) Prothallus (b) Capsules
(c) anisogamous (d) agamous (c) Setae (d) Cones
6. Algin, carrageen and proteins are obtained from 13. In bryophytes, male and female sex organs are called
(a) red algae, brown algae, green algae respectively. ______ and ______ respectively.
(b) brown algae, red algae, green algae respectively. (a) microsporangia; macrosporangia
(c) red algae, green algae, brown algae respectively. (b) male strobili; female strobili
(d) green algae, brown algae, red algae respectively. (c) antheridia; archegonia
7. Which of the following class of algae is rarely found in (d) androecium; gynoecium
fresh water ?
14. Laminaria (Kelp) and Fucus (Rock weed) are examples of
(a) Chlorophyceae (b) Phaeophyceae
(a) green algae (b) brown algae
(c) Rhodophyceae (d) Both (a) and (b)
(c) red algae (d) golden brown algae
8. Which of the following class of algae is mostly found in
salt water ? 15. Pyrenoids in green algal cells are related to
(a) Phaeophyceae (b) Rhodophyceae (a) starch formation (b) protein storage
(c) Chlorophyceae (d) Both (a) and (b) (c) general metabolism (d) enzyme secretion
Plant Kingdom 17

16. The heterosporous pteridophyte belonging to the class 23. Which one of the following is the major difference
lycopsida is between mosses and ferns ?
(a) Selaginella (b) Psilotum (a) Ferns lack alternation of generation while mosses
(c) Equisetum (d) Pteris show the same.
17. Which of the following pteridophytes belong to class (b) Mosses are facultative aerobes while ferns are
pteropsida? obligate aerobes.
(a) Equisetum and Psilotum (c) Vascular bundles of ferns show xylem vessels while
those of mosses lack it.
(b) Lycopodium and Adiantum
(d) Sporophytes of ferns live much longer as compared
(c) Selaginella and Pteris
to the sporophytes of mosses.
(d) Pteris and Adiantum
24. What is the similarity between gymnosperms and
18. Cycas and Adiantum resemble each other in having angiosperms?
(a) seeds (b) motile sperms (a) Phloem of both have companian cells.
(c) cambium (d) vessels (b) Endosperm is formed before fertilization in both.
19. Protonema and leafy stage are the predominant stage of (c) Origin of ovule and seed is similar in both.
the life cycle of
(d) Both have leaves, stem and roots.
(a) moss (b) dicots
25. Which one of the following terms is correctly matched
(c) liverwort (d) gymnosperm with their definition in Pinus ?
STATEMENT TYPE QUESTIONS (a) Monoecious – Male (microsporangiate) and female
(megasporangiate) cones are produced on same
20. Which one of the following is a correct statement? plant.
(b) Monoecious – Male and female sporophylls borne
(a) Pteridophyte gametophyte has a protonemal and on same strobilus.
leafy stage. (c) Dioecious – Male and female cones are produced on
(b) In gymnosperms, female gametophyte is free-living. different plants.
(c) Antheridiophores and archegoniophores are present (d) Monoecious – Micro and megasporocarp develop
in pteridophytes. on same plant.
(d) Origin of seed habit can be traced in pteridophytes. 26. Which of the following statement(s) is/are correct about
21. Why rhodophyta exhibit a red colour ? mosses ?
(a) Since most rhodophyta grow at great depths, the (a) The predominant stage of its life cycle is the
chlorophyll can only absorb light in the red area of gametophyte which consists of two stages –
the spectrum. protonema and leafy stages.
(b) The wavelengths of light that are absorbed by (b) Leafy stage are attached to the soil through
chloro- phyll are passed to phycoerythrin (a red multi-cellular and branched rhizoids.
pigment). (c) Sex organs-antheridia and archegonia are produced
(c) Red pigment of rhodophyta absorbs all the light at the apex of the leafy shoots.
waves. (d) All of the above
(d) The light reaching the greatest depth in water is in 27. Which of the following statement(s) is/are correct ?
the blue-green region of the spectrum, is absorbed (a) In angiosperms, each embryo sac has a three-celled
by phycoerythrin. egg apparatus – one egg cell and two synergids,
22. Which of the following statement is incorrect ? three antipodal cells and two polar nuclei.
(a) Double fertilization is unique to gymnosperms and (b) All seed – bearing plants i.e., gymnosperms and
monocotyledons. angiosperms follow dipontic life patterns of plants.
(b) Sequoia, a gymnosperm, is one of the tallest tree (c) In gymosperms, roots in some genera have fungal
species. association in the form of mycorrhiza (Pinus) while
(c) Phaeophyceae members possess chlorophyll a and in some others (Cycas) small specialized roots called
c, carotenoids and xanthophylls. coralloid are associated with N 2 – fixing
(d) Moss is a gametophyte which consists of two stages cyanobacteria.
namely, protonemal stage and leafy stage. (d) All of the above
EBD_7209
18 Biology
28. Which one of the following statements concerning the Of the above statements
algae is incorrect ? (a) (i) and (ii) (b) (iii) only
(a) Most algae are photosynthetic. (c) (ii) and (iii) (d) (iii) and (iv)
(b) Algae can be classified according to their pigments. 33. Read the following statements and choose the correct
(c) All algae are filamentous. option.
(d) Spirogyra does not produce zoospores. (i) In rhodophyceae, food is stored in the form of
mannitol and laminarin.
29. Which of the following statements is/are correct ?
(ii) The ovules of gymnosperms are not enclosed by
(i) In Equisetum, the female gametophyte is retained
ovary wall.
on the parent sporophyte.
(iii) Salvinia is heterosporous.
(ii) In Ginkgo, male gametophyte is not independent.
(iv) In the diplontic life-cycle, the free living gametophyte
(iii) The sporophyte in Riccia is more developed than represents the dominant phase.
that in Polytrichum.
(a) (ii) and (iii) are correct but (i) and (iv) are incorrect.
(iv) Sexual reproduction in Volvox is isogamous.
(b) (ii) and (iv) are correct but (i) and (iii) are incorrect.
(a) Two (b) Three
(c) (iii) and (iv) are correct but (i) and (ii) are incorrect.
(c) Four (d) One
(d) (i) and (ii) are correct but (iii) and (iv) are incorrect.
30. Consider the following statements regarding
34. Consider the following statements regarding the major
gymnosperms and choose the correct option.
pigments and stored food in the different groups of algae
(i) In gymnosperms, the male and female gametophytes and choose the correct option
have an independent existence.
(i) In chlorophyceae, the stored food material is starch
(ii) The multicellular female gametophyte is retained and the major pigments are chlorophyll-a and d.
within the megasporangium.
(ii) In phaeophyceae, laminarian is the stored food and
(iii) The gymnosperms are heterosporous. major pigments are chlorophyll-a and b.
Of these statements
(iii) In rhodophyceae, floridean starch is the stored food
(a) (i) and (ii) are correct but (iii) is incorrect and the major pigments are chlorophyll-a, d and
(b) (i) and (iii) are correct but (ii) is incorrect phycoerythrin.
(c) (ii) and (iii) are incorrect but (i) is correct (a) (i) is correct, but (ii) and (iii) are incorrect
(d) (ii) and (iii) are correct but (i) is incorrect (b) (i) and (ii) are correct, but (iii) is incorrect
31. Which of the following statements with respect to algae (c) (i) and (iii) are correct, but (ii) is incorrect
are correct. (d) (iii) is correct, but (i) and (ii) are incorrect
(i) Fusion between one large, non-motile female gamete 35. Which of the following statement(s) is/are correct?
and a smaller, motile male gamete is termed as
(i) Green algae are the members of chlorophyceae.
oogamous.
(ii) Brown algae are found primarily in marine habitates
(ii) Fusion of two gametes dissimilar in size is termed as
oogamous. (iii) Some red algae are found in fresh water, mostly occur
in salt water, some are found in brackish water.
(iii) Fusion of two gametes similar in size is called
anisogamous. (iv) The food in red algae is stored as floridean starch.
(iv) In chlorophyceae, the major pigments are chlorophyll (v) Red alga may occur in both well-lighted regions close
a and b , and the food is stored as starch. to water-surface and also at great depths in oceans
(v) In rhodophyceae, the major pigments are chlorophyll where light penetration is little.
a and d , and the food is stored as mannitol. (a) (i) and (v) only
(a) (i) and (v) (b) (iii) and (v) (b) (ii), (iii) and (iv) only
(c) (i) and (ii) (d) (i) and (iv) (c) All of the above
32. Which of the following statements with respect to (d) None of the above
gymnosperms and angiosperms is/are correct? 36. Which of the following statement(s) is/are correct about
(i) The process of double fertilization is present in gemmae ?
gymnosperms. (i) These are specialised structures by which asexual
(ii) Angiosperms range in size from microscopic Wolffia reproduction take place in liverworts.
to tall trees of Sequoia. (ii) They are green, multicellular and asexual buds.
(iii) In gymnosperms, the seeds are not covered. (iii) They develop in small receptacles called gemma cups.
(iv) In gymnosperms, the male and female gametophytes (iv) They detach from parent body and germinate to form
have an independent free living existence. new individuals.
Plant Kingdom 19

(a) (i) and (ii) (b) (ii) and (iii) (a) Algae (b) Fungi
(c) (i), (ii) and (iii) (d) All of these (c) Bryophytes (d) Pteridophytes
37. Which of the following branch of science is being 41. Which of the following group of plant is being described
described by the given statements ? by the given statements ?
(i) It can easily done by using computers based on all
(i) The plant body is thalloid.
observable characters.
(ii) Numbers and codes are assigned to all the characters (ii) Asexual reproduction takes place by fragmentation
and the data are then processed. of thalli, or by the formation of specialized structures
called gemmae.
(iii) Each character is given equal weightage and at the
same time hundred of characters can be considered. (iii) The sporophyte is differentiated into a foot, seta
(a) Cytotaxonomy (b) Numerical taxonomy and capsule.
(c) Chemotaxonomy (d) b-taxonomy (iv) They grow usually in moist and shady habitats.
38. Which of the following statements about algae is/are (a) Liverworts (b) Moss
correct? (c) Fern (d) Gymnosperm
(i) Algae are chlorophyll – bearing simple, thalloid, 42. Which of the following statement(s) is/are correct about
heterotrophic and aquatic (both fresh water and pteridophytes ?
marine) organisms.
(i) The main plant body is a sporophyte which is
(ii) Algae reproduce by vegetative means only. differentiated into true roots and leaves.
(iii) Fusion of two gametes dissimilar in size is termed as (ii) The leaves are small (microphylls) as in ferns or large
oogamous. (macrophylls) as in Selaginella.
(iv) A few of the massive forms of algae such as kelps, (iii) Genera like Selaginella and Salvinia which produce
form massive plant bodies. two kinds of spores–macro (large) and micro (small)
(a) Only (i) (b) Both (i) and (iii) spores, are known as heterospores.
(iv) Common examples are Funaria, Polytrichum and
(c) Only (iv) (d) All of these
Sphagnum.
39. The following statements are associated with one class
(a) Both (i) and (ii) (b) Both (ii) and (iii)
of algae. Identify the class of algae.
(c) Both (i) and (iii) (d) All of these
(i) One or more storage bodies called pyrenoids located
43. Which of the following group of plant is being described
in the chloroplasts are present in the members of
by the given statements ?
this class.
(i) They are plants in which the ovules are not enclosed
(ii) They have a rigid cell wall made of an inner layer of
by any ovary wall and remain exposed before and
cellulose and an outer layer of pectose.
after fertilization.
(iii) Asexual reproduction is by flagellated zoospores (ii) The giant red wood tree Sequoia is one of the tallest
produced in zoosporangia. tree species of the group.
(iv) Chlamydomonas, Volvox, Ulothrix, Spirogyra and (iii) The roots are generally tap roots.
Chara are commonly found members of this class.
(iv) They are heterosporous and they produce haploid
(a) Chlorophyceae (b) Rhodophyceae microspores and megaspores.
(c) Phaeophyceae (d) None of these (a) Algae (b) Bryophytes
40. Refer to the following statement(s) and identify the group (c) Gymnosperms (d) Pteridophytes
of plant which is being described by the given statements? 44. Which of the following statement(s) is/are correct about
(i) They include various mosses and liverworts that are angiosperms ?
found commonly growing in moist shaded areas in (i) In angiosperms or flowering plants, the pollen grains
the hills. and ovules are developed in specialised structure
(ii) They lack true roots, stem or leaves. called flowers.
(ii) They are divided into two classes : the dicotyledons
(iii) The main plant body is haploid.
and the monocotyledons.
(iv) They produce a multicellular body sporophyte which (iii) The male sex organ in a flower is the pistil or the
is not free living but attached to the photosynthetic carpel.
gametophyte and derives nourishment from it. (iv) The female sex organ is the stamen.
EBD_7209
20 Biology
(a) Only (i) (b) Both (i) and (ii) Reason: Phaeophyceae possess chlorophyll a, c,
(c) Both (ii) and (iv) (d) All of these carotenoids and xanthophylls.
45. Read the following statements and answer the question. 50. Assertion: Mosses are evolved from algae.
(i) Sporophytic generation is represented only by the Reason: Protonema of mosses is similar to some green
one-celled zygote. algae.
(ii) Meiosis in the zygote results in the formation of 51. Assertion: Red algae contributes in producing coral reefs.
haploid spores. Reason: Some red algae secrete and deposit calcium
(iii) The dominant, photosynthetic phase in such plants carbonate over their walls.
is free-living gametophyte.
52. Assertion: Coconut tree is distributed in coastal areas
(iv) Many algae such as Volvox, Spirogyra and some
over a large part of the world.
species of Chlamydomonas represent this pattern.
Which of the following pattern of life cycle of plant is Reason: Coconut fruit can float and get dispersed over
described by the above statements ? thousands of kilometers before losing viability.
(a) Haplontic (b) Diplontic 53. Assertion: Red algae contributes in producing coral reefs.
(c) Haplo-diplontic (d) None of these Reason: Some red algae secrete and deposit calcium
46. Which of the following statement(s) is/are correct ? carbonate over their walls.
(i) Agar, one of commercial products obtained from 54. Assertion: The peristome is a fringe of teeth-like
Laminaria and Sargassum are used to grow projections found at the mouth of the capsule.
microbes and in preparations of ice-creams and jellies. Reason: It may be of two types nematodontous and
(ii) In phaeophyceae, major pigments are chl a, d and orthodontus.
phycoerythrin.
(iii) Pteridophytes classified into four classes : Psilopsida, MATCHING TYPE QUESTIONS
Lycopsida, Sphenopsida and Pteropsida.
(iv) Gemmae are green, multicellular, asexual buds, which 55. Match the column-I with column-II and select the correct
develop in small receptacles called gemma cups answer using the codes given below.
located on the thalli. Column-I Column-II
(a) Both (i) and (ii) (b) Both (ii) and (iv) (Group of (Examples)
(c) Both (iii) and (iv) (d) All of these Plant Kindgdom)
47. Which of the following statements is incorrect? A. Algae I. Solanum tuberosum
(a) Pyrenoids contain protein besides starch. B. Fungi II. Equisetum
(b) Sexual reproduction may be isogamous, oogamous C. Angiosperm III. Cycas
and anisogamous in green and brown algae.
D. Pteridophyte IV. Chlamydomonas
(c) Some of the members of algae also occur in
E. Gymnosperm V. Rhizopus
association with fungi (lichen) and animals (eg, on
sloth bear). (a) A – V; B – IV; C – I; D – II; E – III
(d) The leaves in pteridophytes are well adapted to (b) A – IV; B – V; C – I; D – II; E – III
withstand extremes of temperature, humidity and (c) A – IV; B – I; C – V; D – II; E – III
wind. (d) A – IV; B – I; C – V; D – III; E – II
56. Match the classes of pteridophytes given in column I
ASSERTION/REASON TYPE QUESTIONS
with their examples given in column II and choose the
correct option
In the following questions, a statement of Assertion is followed
by a statement of Reason. Column -I Column-II
(a) If both Assertion and Reason are true and the Reason is (Classes of pteridophytes) (Examples)
the correct explanation of the Assertion. A. Psilopsida I. Selaginella
(b) If both Assertion and Reason are true but the Reason is B. Lycopsida II. Psilotum
not the correct explanation of the Assertion. C. Sphenopsida III. Dryopteris
(c) If assertion is true but Reason is false. D. Pteropsida IV. Equisetum
(d) If both Assertion and Reason are false.
(a) A – II; B – I; C – IV; D – III
48. Assertion: Double fertilization is a characteristic feature
(b) A – I; B – II; C – IV; D – III
of angiosperms.
(c) A – II; B – I; C – III; D – IV
Reason: Double fertilization involves two fusions.
49. Assertion: Members of phaeophyceae vary in colour from (d) A – II; B – IV; C – I; D – III
olive green to various shades of brown.
Plant Kingdom 21

57. Match column-I with column-II and choose the correct 60. Match the column-I with column-II and choose the correct
option. option.
Column-I Column-II Column-I Column-II
A. Phaeophyceae I. Have an elaborate A. Smallest flowing plant I. Eucalyptus
mechanism B. Male sex organ in II. Wolffia
of spore dispersal flowering plant
B. Rhodophyceae II. first terrestrial plant with C. Female sex organ III. Stamen
vascular tissue-phloem in flowering plant
and xylem D. Tallest tree IV. Pistil
C. Mosses III. Asexual reproduction by (a) A – I; B – II; C – III; D – IV
biflagellate zoosposes
(b) A – IV; B – III; C – II; D – I
D. Pteridophytes IV. Polysiphonia, Porphyra,
(c) A – II; B – III; C – IV; D – I
Gracilaria
(d) A – II; B – IV; C – III; D – I
(a) A – III; B – IV; C – I; D – II
61. Match the column-I with column-II and choose the correct
(b) A – IV; B – III; C – I; D – II
option.
(c) A – IV; B – III; C – II; D – I
(d) A – IV; B – I; C – III; D – II Column-I Column-II
58. Match the column I with column II and choose the option A. Amphibian of the I. Sphagnum
which shows its correct combination. plant kingdom
Column-I Column-II B. Specialized structures II. Angiosperms
(Pattern of (Examples) in liverworts for
life cycle in plant) asexual reproduction
A. Haplontic I. Bryophytes, C. Monocotyledons and III. Bryophytes
life cycle Pteridophytes, dicotyledons
Ectocarpus, Polysiphonia, D. A plant which has IV. Gemmae
Kelps capacity to holding water
B. Diplontic II. Seed bearing plants (a) A – III; B – IV; C – I; D – II
life cycle (Gymnosperm and (b) A – III; B – IV; C – II; D – I
Angiosperm), Fucus (c) A – IV; B – III; C – II; D – I
C. Haplo-diplontic III. Many algae (Volvox, (d) A – III; B – II; C – IV; D – I
life cycle Spirogyra) and some 62. Which of the following pairs is incorrectly matched ?
species of (a) Chlorophyceae – Major pigments are chl a and b.
Chlamydomonas (b) Phaeophyceae – Cell wall is made up of cellulose
(a) A – III; B – II; C – I (b) A – I; B – II; C – III and algin.
(c) A – II; B – I; C – III (d) A – III; B – I; C – II (c) Rhodophyceae – Stored food is mannitol.
59. Match the column-I with column-II and choose the correct (d) Chlorophyceae – Cell wall is made up of cellulose.
option.
63. Select the correct match of the feature present in column
Column-I Column-II
I with its respective terms given in column II.
(System of (Characteristics)
Column-I Column-II
classification)
(features) (term)
A. Artificial system I. Based on few
A. Presence of tap roots (I) Bryophyte
of classification morphological
characters and coralloid roots
B. Natural system II. Based on evolutionary B. The synergids and (II) Pteridophytes
of classification relationships between the antipodal cells
various organisms degenerates after
C. Phylogenetic III. Based on natural affinities fertilization
system of among the organisms and C. The food is stored as (III) Red algae
classification consider external as well as floridean starch which
internal features. is very similar to
(a) A – II; B – I; C – III (b) A – I; B – III; C – II amylopectin and
(c) A – III; B – II; C – I (d) A – I; B – II; C – III glycogen in structure
EBD_7209
22 Biology
D. Presence of sporophyte (IV) Angiosperms E. Members of this group (V) Gymnosperms
which is not free living are used for medicinal
but attached to the purposes, as soil
photosynthetic binders and frequently
gametophytes and grown as ornamentals
derives nourishment (a) A – I; B – II; C – III; D – IV; E – V
from it (b) A – III; B – V; C – II; D – IV; E – I
(c) A – III; B – I; C – V; D – II ; E – IV
(d) A – V; B – IV; C – III; D – I; E – II
DIAGRAM TYPE QUESTIONS
64. The given figures (A, B, C, D) represent the members algae. Identify the correct option for the given diagrams.

S. No. A B C D

(a) Porphyra Fucus Dictyota Polysiphonia

(b) Polysiphonia Porphyra Dictyota Fucus

(c) Fucus Dictyota Porphyra Polysiphonia

(d) Porphyra Polysiphonia Fucus Dictyota

65. Refer to the given figure (A, B, C and D) and answer the question. Which of the following figures are the members of green alga?

(a) A, B and D (b) A, B and C (c) B, C and D (d) C, D and A


Plant Kingdom 23

66. The given figures represent the examples of bryophytes. In them few structures/parts are marked as A, B, C and D.

Identify the option which shows the correct labelling of A, B, C and D.

S . No. A B C D

(a) Gemma cup A rch eg on iop ho re Sp oro p hy te S ph ag nu m

(b) A rch ego nio ph ore Gemma cu p Gametop hy te S ph ag nu m

(c) A rcheg on ia A ntheridia Gemma cup S ph ag nu m

(d) A n th erid ia A rch ego nia Gemma cup S ph ag nu m

67. Which one of the following options correctly represents the type of life cycle patterns?
Sporophyte
(2n) Zygote Sporophyte
(2n) (2n)
Syngamy Meiosis Zygote (2n)
Meiosis
A Syngamy C
Zygote (2n) B Spores (n)
Gametogenesis Spores (n)
Gametogenesis Gametogenesis
Syngamy
Meiosis Gametophyte (n)
Gametophyte (n)
(n)
(a) A - Diplontic, B - Haplodiplontic, C - Haplontic (b) A - Haplodiplontic,B - Haplontic, C - Diplontic
(c) A - Haplontic, B - Diplontic, C - Haplodiplontic (d) A - Diplontic, B - Haplontic, C - Haplodiplontic
68. Identify the plants (A, B, C and D) and choose their correct names from the options given below.

A
C D

B
EBD_7209
24 Biology
S . No. A B C D

(a) Eq uisetu m Gin k g o S ela ginella Lyco po dium

(b) S ela ginella Equ isetu m S alvin ia Gink go

(c) Fu na ria Ad ia ntum S alvin ia Ricc ia

(d) C ha ra M a rcha ntia Fu cu s Pin us

69. The given figure shows the life cycle of an angiosperm. Few plants are marked as A, B, C, D and E. Identify the correct
labelling from the given options.
A B
Microspore
mother cells
Style Filament
Microsporangium
Ovary
Flower Microspores
Megaspore
mother cell
Megasporangium
(ovule)
Sporophyte GAMETOPHYTIC
SPOROPHYTIC (n)
(2n) GENERATION
GENERATION
Microspore
(pollen grain)
Embryo

E C

Gametes
Zygote D

(a) A–Stigma, B–Anther, C–Male gametophyte, D–Egg, E–Female gametophyte


(b) A–Stigma, B–Anther, C–Female gametophyte, D–Egg, E–Male gametophyte
(c) A–Stigma, B–Anther, C–Male gametophyte, D–Fertilized egg, E–Female gametophyte
(d) A–Stigma, B–Anther, C–Embryo sac, D–Egg, E–Female gametophyte

CRITICAL THINKING TYPE QUESTIONS


70. Which of the following example belong to the same class 72. Fern plant is a
of algae? (a) haploid gametophyte
(a) Chara, Fucus, Polysiphonia (b) diploid gametophyte
(b) Volvox, Spirogyra, Chlamydomonas (c) diploid sporophyte
(c) Porphyra, Ectocarpus, Ulothrix (d) haploid sporophyte
(d) Sargassum, Laminaria, Gracilaria 73. The unique feature of bryophytes compared to other plant
71. A bryophyte differs from pteridophytes in having groups is that
(a) archegonia. (a) they produce spores.
(b) lack of vascular tissue. (b) they lack vascular tissues.
(c) swimming antherozoids. (c) they lack roots.
(d) independent gametophytes. (d) their sporophyte is attached to the gametophyte.
Plant Kingdom 25

74. If there are 4 cells in an anther, what will be the number of 83. Protonema
pollen grains? (a) is a stage of gametophytic generation.
(a) 8 (b) 4 (b) is a creeping, green, branched and develops directly
(c) 16 (d) 12 from a spore.
75. Bryophytes are different from fungi in having (c) produces lateral bud which forms leafy plant body.
(a) land habit. (d) All of the above
(b) sterile jacket layers. 84. The spreading of living pteridophytes is limited and
(c) multiflagellate gametes. restricted to narrow geographical region because
(d) gametophytic plant body. (a) gametophytic growth needs cool, damp and shady
76. Moss peat is used as a packing material for sending flowers places.
and live plants to distant places because
(b) it requires water for fertilization.
(a) it reduces transpiration.
(c) due to absence of stomata in leaf and absence of
(b) it serves as a disinfectant. vascular tissue.
(c) it is easily available. (d) both (a) and (b)
(d) it is hygroscopic. 85. Classification of plants proposed by Carolus Linnaeus
77. If the cells of root in wheat plant have 42 chromosomes, was artificial because it was based on
then the no. of chromosome in the cell of pollen grain is (a) only a few morphological characters.
(b) evolutionary tendencies which are diverse.
(a) 14 (b) 21
(c) anatomical characters which are adaptive in nature.
(c) 28 (d) 42
(d) physiological traits alongwith morphological
78. A research student collected certain alga and found that characters.
its cells contained both chlorophyll a and chlorophyll d 86. Mosses are of great ecological importance because of
as well as phycoerythrin. On the basis of his observation, (a) its contribution to prevent soil erosion.
the students conclude that the alga belongs to
(b) its contribution in ecological succession.
(a) rhodophyceae (b) bacillariophyceae (c) its capability to remove CO from the atmosphere.
(c) chlorophyceae (d) phaeophyceae (d) both (a) and (b)
79. Fruits are not formed in gymnosperms because of 87. You are given an unknown plant to study in the laboratory.
(a) absence of pollination. You find that it has chlorophyll, no xylem. Its multicellullar
sex organs are enclosed in a layer of jacket cells. Its
(b) absence of seed.
gametophyte stage is free living. The plant probably
(c) absence of fertilization. belongs to
(d) absence of ovary.
(a) chlorophyceae (b) bryophyte
80. What is common in all the three, Funaria, Dryopteris and
(c) pteridophyte (d) gymnosperm
Ginkgo?
88. Mosses do not have ‘true leaves’ because their leaf-like
(a) Independent sporophyte structures lack
(b) Presence of archegonia (a) starch in their chloroplast.
(c) Well developed vascular tissues (b) vascular tissues.
(d) Independent gametophyte (c) chlorophyll.
81. In angiosperms, a mature male gametophyte is derived (d) cellulose in their cell walls.
from a pollen mother cell by 89. Place the following groups of plants in order, beginning
with those that first appeared on the earth and progressing
(a) three mitotic divisions.
toward those that appeared most recently in time.
(b) one meiotic and two mitotic divisions. (a) Gymnosperms, angiosperms, ferns, moss, algae
(c) two meiotic divisions. (b) Algae, moss, ferns, gymnosperms, angiosperms
(d) a single meiotic division. (c) Moss, algae, ferns, angiosperms, gymnosperms
82. If you are asked to classify the various algae into distinct (d) Algae, ferns, angiosperms, gymnosperms, moss
groups then which of the following characters you should 90. Chemotaxonomy is connected with
choose for the classification ? (a) classification of chemicals found in plants.
(a) Nature of habitat (b) use of phytochemical data in systematic botany.
(b) Structural organization of thallus (c) application of chemicals on herbarium sheets.
(c) Chemical composition of the cell wall (d) use of statistical methods in chemical yielding plants.
(d) Types of pigments present in the cell.
EBD_7209
26 Biology
91. In order to colonize land, plants needed to acquire which 94. Angiosperms have dominated the land flora primarily
of the following characteristics? because of their
(a) A mechanism for moving water throughout the plant. (a) power of adaptability in diverse habitat.
(b) A mechanism to prevent desiccation of tissues. (b) property of producing large number of seeds.
(c) An ability to screen ultraviolet radiation. (c) nature of self pollination.
(d) Both (b) and (c) (d) domestication of man.
92. Deep in the tropical rain forest, a botanist discovered an 95. A student was given a sample to observe under the
unusual plant with vascular tissues, stomata, a cuticle, microscope. He observed and found that the sample is
flagellated sperm, cone-like reproductive structures the most common type of spore involved in asexual
bearing seeds, and an alternation-of-generations life cycle. reproduction in algae. Identify the spore.
He was very excited about this discovery because it would (a) Zoospore (b) Endospore
be rather unusual for a plant to have both
(c) Hypnospore (d) None of these
(a) a cuticle and flagellated sperm.
96. Refer to the statement and answer the question.
(b) vascular tissues and alternation of generations.
“They usually reproduce vegetatively by fragmentation
(c) seeds and flagellated sperm. and asexually by non - motile spores and sexually by non
(d) alternation of generations and seeds. - motile gametes.”
93. A universal feature of the life cycle of plants is Identify the group of plants and its example.
(a) morphologically identical haploid and diploid stages. (a) Mosses, Funaria
(b) genetically identical haploid and diploid stages. (b) Red algae, Polysiphonia
(c) alteration of generations between heteromorphic (c) Brown algae, Laminaria
haploid gametophytes and diploid sporophytes. (d) Pteridophytes, Selaginella
(d) none of the above
4

Chapter
Animal Kingdom

10. Which of the following group of animals reproduces only


FACT/DEFINITION TYPE QUESTIONS
by sexual means?
1. When any plane passing through the central axis of the (a) Ctenophora (b) Cnidaria
body divides the organism into two identical halves, the (c) Porifera (d) Protozoa
organism is called ___________. 11. __________ is responsible for maintaining the current
(a) radially symmetrical of water in sponge.
(b) bilaterally symmetrical (a) Osculum (b) Porocytes
(c) asymmetrical (c) Spongocoel (d) Choanocytes
(d) metamerically segmented 12. Aquatic annelids (like Nereis) possess lateral appendages
2. Which of the following is not the common fundamental called ______________, which help in swimming.
feature for animal classification? (a) visceral hump (b) parapodia
(a) Germinal layers. (c) radula (d) spicules
(b) Pathway of water transport. 13. Which of the following belong to phylum arthropoda?
(c) Pattern of organization of cells. (a) Bombyx and Apis
(d) Serial repetition of the segments. (b) Laccifer and Anopheles
3. Animals like annelids, arthropods, etc. where the body (c) Locusta and Limulus
can be divided into identical left and right halves in only (d) All of the above
one plane, exhibit___________symmetry. 14. Which of the following is a living fossil?
(a) radial (b) bilateral (a) Balanoglossus (b) Echinus
(c) asymmetrical (d) non- symmetrical (c) Ancylostoma (d) Limulus
4. Which of the following is a fresh water sponge? 15. A file like rasping organ for feeding, called radula, present
(a) Sycon (b) Euspongia in the phylum __________.
(c) Spongilla (d) Pleurobrachia (a) arthropoda (b) mollusca
5. Few cnidarians like corals have a skeleton composed of (c) echinodermata (d) chordata
(a) calcium hydroxide (b) calcium sulphate 16. In phylum arthropoda, excretion takes place through
(c) calcium carbonate (d) sodium bicarbonate (a) nephridia (b) flame cells
6. Meandrina (brain coral) belongs to phylum (c) malphigian tubules (d) gills
(a) porifera (b) coelenterata 17. In phylum echinodermata, the adult echinoderms are
(c) ctenophora (d) platyhelminthes ______A_________ but larvae are _______B______ .
7. In ctenophora, the body bears _______ external rows of (a) A – radially symmetrical; B – bilaterally symmetrical
ciliated comb plates, which help in locomotion. (b) A – bilaterally symmetrical; B – radially symmetrical
(a) five (b) six (c) A – bilaterally symmetrical; B – asymmetrical
(c) seven (d) eight (d) A – metamerically segmented; B – asymmetrical
8. Flame cells present in platyhelminthes are specialized in 18. In which of the phylum, excretory organ like proboscis
(a) respiration and absorption. gland is present?
(b) osmoregulation and circulation. (a) Hemichordata (b) Chordata
(c) respiration and excretion. (c) Echinodermata (d) Annelida
(d) osmoregulation and excretion. 19. Which of the following is not a chordate character?
9. Polyp phase is absent in (a) Presence of paired pharyngeal gill slits
(a) Hydra (b) Aurelia (b) Ventral heart
(c) Physalia (d) Obelia (c) Solid and ventral nerve cord
(d) Presence of post-anal tail
EBD_7209
28 Biology
20. Which of the following possesses electric organs and (i) These are primitive multicellular animals and have
belongs to class chondrichthyes? cellular level of organization.
(a) Torpedo (b) Petromyzon (ii) Digestion is intracellular.
(c) Trygon (d) Exocoetus (iii) They have a water transport or canal system.
21. Which of the following possesses poison sting and (iv) They reproduce asexually by fragmentation and
belongs to class chondrichthyes? sexually by formation of gametes.
(a) Labeo (b) Myxine (a) Porifera (b) Ctenophora
(c) Coelenterata (d) Platyhelminthes
(c) Clarias (d) Trygon
31. Which of the following statements (i - v) are correct ?
22. Which of the following pairs of animals comprises ‘jawless
(i) The pelvic fins of female sharks bear claspers.
fishes’?
(ii) In Obelia, polyps produce medusae sexually and
(a) Mackerals and rohu medusae form the polyps asexually.
(b) Lampreys and hag fishes (iii) Flame cells in platyhelminthes help in osmoregulation
(c) Guppies and hag fishes and excretion.
(d) Lampreys and eels (iv) In non-chordates, central nervous system is ventral,
23. In amphibians, respiration occurs through solid and double.
(a) gills (b) lungs (v) Pinnae are present in mammals.
(c) skin (d) all of these (a) (ii), (iv) and (v) (b) (i), (iii) and (v)
24. In amphibians, heart is ________ chambered. (c) (iii), (iv) and (v) (d) (i), (ii) and (iii)
(a) two (b) three 32. Which one of the following statement regarding coelom
(c) four (d) none of these of given animals is correct?
25. Heart is three - chambered in reptiles, except (a) Round worms (aschelminthes) are pseudocoelomates.
(a) turtle (b) Chameleon (b) Molluscs are acoelomates.
(c) Naja (Cobra) (d) crocodile (c) Insects are pseudocoelomates.
26. Which of the following is a poisonous snake? (d) Flatworms (platyhelminthes) are coelomates.
33. Read the following statements and answer the question.
(a) Naja (Cobra) (b) Bangarus (Krait)
(i) They are exclusively marine, radially symmetrical,
(c) Viper (Viper) (d) All of these
diploblastic organisms with tissue level of
27. Which of the following is a chordate feature and not
organisation.
shared by the non-chordates ? (ii) Body bears eight external rows of ciliated comb plates,
(a) Metamerism (b) Axial organization which help in locomotion.
(c) Bilateral symmetry (d) Pharyngeal gill slits (iii) Digestion is both extracellular and intracellular.
(iv) Reproduction takes place only by sexual means.
STATEMENT TYPE QUESTIONS Which of the following phylum is being described by
28. Which of the following statements (i – v) are incorrect? above statements?
(i) Parapodia are lateral appendages in arthropods used (a) Platyhelminthes (b) Arthropoda
(c) Mollusca (d) Ctenophora
for swimming.
34. Which of the following phylum is being described by the
(ii) Radula in molluscs are structures involved in
given statements?
excretion. (i) They are bilaterally symmetrical,triploblastic,
(iii) Aschelminthes are dioecious. segmented and coelomate animals.
(iv) Echinoderm adults show radial symmetry. (ii) The body consists of head, thorax, abdomen and
(v) Ctenophorans are diploblastic. have jointed appendages.
(a) (i) and (ii) (b) (i) and (iii) (iii) Circulatory system is of open type.
(c) (i), (iv) and (v) (d) (iii) and (v) (iv) Excretion takes place through malphigian tubules.
29. Which of the following statements (i – v) are incorrect ? (a) Arthropoda (b) Annelida
(i) Circulatory system in arthropods is of closed type. (c) Mollusca (d) Echinodermata
(ii) Parapodia in annelids helps in swimming. 35. The following statement are associated with the
(iii) Phylum mollusca is the second largest animal phylum. occurrence of notochord. Identify the incorrect statement.
(iv) Aschelminthes are dioecious. (a) It is present only in larval tail in ascidians.
(a) (i) only (b) (iii) only (b) It is replaced by a vertebral column in adult frog.
(c) (i) and (iii) (d) (iii) and (iv) (c) It is absent throughout the life in humans from the
30. Which of the following phylum is being described by the very beginning.
given statements? (d) It is present throughout life in Amphioxus.
Animal Kingdom 29

36. Which of the following characteristics is correct for 42. Which of the follwoing statement(s) is/are correct?
reptilia? (i) Organ systems in different group of animals show
(a) Body covered with dry and cornified skin, scales various patterns of complexities.
over the body are epidermal, they do not have external (ii) The digestive system in platyhelminthes has only a
ears. single opening to the outside of the body that serve
(b) Body is covered with moist skin and is devoid of scales,
as both mouth and anus, and is hence called
the ear is represented by a tympanum, alimentary canal,
complete.
urinary and reproductive tracts open into a common
cloaca. (iii) In open type of circulatory system, the blood is
(c) Fresh water animals with bony endoskeleton and pumped out of the heart and the cells and tissues are
air-bladder regulate buoyancy. directly bathed in it.
(d) Marine animals with cartilaginous endoskeleton and (iv) In closed type, the blood is circulated through a
body is covered with placoid scales. series of vessels of varying diameters (arteries, veins
37. Which of the following statements is incorrect ? and capillaries).
(a) Prawn has two pairs of antennae. (a) Only (i) (b) Both (ii) and (iii)
(b) Nematocysts are characteristic feature of the phylum (c) (i), (iii) and (iv) (d) All of these
cnidaria. 43. Which of the following statement(s) is/are correct?
(c) Millipedes have two pairs of appendages in each (i) Animals in which the cells are arranged in two
segment of the body.
embryonic layers, an external ectoderm and an
(d) Animals that belong to phylum porifera are
internal endoderm, are called diploblastic animals.
exclusively marine.
38. Which of the following statement(s) is/are correct (ii) Notochord is an ectodermally derived rod like
regarding class aves? structure formed on the ventral side during
(i) The forelimbs are modified into wings and the embryonic development in some animals.
hindlimbs generally have scales and are modified for (iii) In some animals, the body cavity is not lined by
walking, swimming or clasping the tree branches. mesoderm, instead, the mesoderm is present as
(ii) Heart is completely four-chambered. scattered pouches in between the ectoderm and
(iii) They are warm- blooded (homoiothermous) animals endoderm and such a body cavity is called
i.e., they are able to maintain a constant body pseudocoelom.
temperature. (a) Only (i) (b) Both (i) and (ii)
(iv) They are oviparous and development is direct. (c) Both (i) and (iii) (d) All of these
(a) Both (i) and (iii) (b) Both (i) and (iv)
44. Which of the following statement(s) is/are correct
(c) (i), (ii) and (iii) (d) All of these
regarding phylum coelenterata?
39. Which one of the following categories of animals is
correctly described with no single exception in it? (i) They are aquatic, mostly marine, sessile or free-
(a) In chondrichthyes notochord is persistent swimming, radially symmetrical animals.
throughout life. (ii) They have a central gastro-vascular cavity with a
(b) All mammals are viviparous and possess diaphragm single opening called hypostome.
for breathing. (iii) Digestion is extracellular and intracellular.
(c) All sponges are marine. (iv) Examples are Sycon, Spongilla and Euspongia.
(d) All reptiles possess scales, have a three chambered (a) (i) and (ii) (b) (i) and (iv)
heart and are cold blooded (poikilothermal). (c) (i), (ii) and (iii) (d) All of these
40. Which of the following is an incorrect statement regarding 45. Which of the following statement(s) is/are correct
flatworms ? regarding phylum aschelminthes?
(a) They are acoelomates.
(i) The body is circular in cross-section hence the name
(b) They are bilaterally symmetrical.
roundworms.
(c) They lack a digestive system.
(d) They have a circulatory system. (ii) Alimentary canal is complete with a well-developed
41. Which of the following is not a characteristic of phylum muscular pharynx.
echinodermata ? (iii) Sexes are separate (dioecious), i.e., males and females
(a) They have a water vascular system. are distinct.
(b) They have an internal skeleton. (iv) Nephridia help in osmoregulation and excretion.
(c) They are protostomes. (a) (i) and (ii) (b) (iii) and (iv)
(d) They have bilateral symmetry at larval stage. (c) (i), (ii) and (iii) (d) All of these
EBD_7209
30 Biology
46. Which of the following statement(s) is/are correct and free swimming is called "Z".
regarding phylum mollusca? Identify X, Y, and Z.
(a) They are bilaterally symmetrical, triploblastic and X Y Z
coelomate animals. a. Coelenterate Polyp Medusa
(b) Body is covered by a calcareous shell and is
unsegmented with a distinct head, muscular foot and b. Cnidarian Medusa Polyp
visceral hump. c. Ctenophora Radula Hypostome
(c) The mouth contains a file-like rasping organ for d. Porifera Osculum Radula
feeding, called radula.
(d) All of the above ASSERTION/REASON TYPE QUESTIONS
47. Which of the following class is being correctly described
by given statements (i - iv)? In the following questions, a statement of Assertion is followed
(i) All living members of this class are ectoparasites on by a statement of Reason.
some fishes. (a) If both Assertion and Reason are true and the Reason is
(ii) They have a sucking and circular mouth without the correct explanation of the Assertion.
jaws. (b) If both Assertion and Reason are true but the Reason is
(iii) Circulation is of closed type. not the correct explanation of the Assertion.
(iv) They are marine but migrate for spawning to fresh (c) If Assertion is true but Reason is false.
water. After spawing, within a few days they die. (d) If both Assertion and Reason are false.
(a) Cyclostomata (b) Chondrichthyes 52. Assertion : The duck-billed Platypus and the spiny ant-
(c) Osteichthyes (d) Amphibia eater, both are egg-laying animals yet they are grouped
48. Which of the following class is being described by the under mammals.
given statements (i - iv)? Reason : Both of them have seven cervical vertebrae and
(i) They are found in a variety of habitats- polar ice- 12 pairs of cranial nerves.
caps, deserts, mountains, forests, grasslands and 53. Assertion : Torison can be seen in ctendium.
dark caves. Reason : Ctenidium acts as the respiratory organ.
(ii) Most unique mammalian characteristic is the 54. Assertion : Tapeworm, roundworm and pinworm are
presence of mammary glands by which the young endoparasites of human intestine.
ones are nourished. Reason : Improperly cooked food is the source of intestinal
(iii) Heart is four-chambered. infections.
(iv) Sexes are separate and fertilization is internal. 55. Assertion : Sponges have body organization of "cellular
(a) Reptilia (b) Aves level".
(c) Mammalia (d) Amphibia Reason : There is some physiological division of labour.
49. Which of the following statement(s) is/are correct for class 56. Assertion : Ambulacral system plays major role in
amphibia? locomotion of echinoderm.
(i) Body is divisible into head and trunk. Reason : Hydraulic pressure of fluid and contraction of
(ii) Respiration is through gills only. muscle of tube feet make possible movement of
(iii) The heart is two chambered i.e. one auricle and one echinoderm.
ventricle.
(iv) Fertilization is internal. MATCHING TYPE QUESTIONS
(a) Only (i) (b) Only (iv)
57. Match the types of animals given in column I with their
(c) (i), (ii) and (iii) (d) All of these
examples given in column II and choose the correct option.
50. Which of the following statement is incorrect?
Column -I Column -II
(a) Platyhelminthes has incomplete digestive system.
(Types of animals) (Examples)
(b) In coelenterates, the arrangement of cells is more
A. Limbless reptiles I. Elephant
complex.
B. Jawless vertebrates II. Lamprey
(c) Nereis is monoecious but earthworms and leeches
C. Flightless bird III. Ichthyophis
are dioecious.
D. Largest IV. Ostrich
(d) Simple and compound eyes are present in the animals
terrestrial animal
of those phylum whose over two-thirds of all named
E. Limbless amphibia V. Cobra
species on earth are arthropods.
(a) A – II; B – V; C – IV; D – I; E – III
51. Refer the following statement and answer the question.
(b) A – V; B – II; C – IV; D – I; E – III
'Name of "X" is derived from stinging capsules. It exhibits
(c) A – V; B – II; C – I; D – IV; E – III
metagenesis containing two body forms in which sessile
(d) A – V; B – IV; C – II; D – I; E – III
and cylindrical form is called "Y" and umbrella shaped
Animal Kingdom 31

58. Column-I contains organisms and column-II contains their Column -I Column- II
exeretory structures. Choose the correct match form the (Phylum) (Examples)
options given below. A. Echinodermata I. Ascidia, Doliolum
Column- I Column -II B. Hemichordata II. Asterias, Ophiura
(Organism) (Excretory structures) C. Urochordata III. Branchiostoma
A. Cockroach I. Nephridia D. Cephalochordata IV. Balanoglossus,
B. Cat fish II. Malpighian tubules Saccoglossus
C. Earthworm III. Kidneys (a) A – IV; B – II; C – I; D – III
D. Balanoglossus IV. Flame cells (b) A – II; B – IV; C – I; D – III
E. Flatworm V. Proboscis gland (c) A – II; B – IV; C – III; D – I
(d) A – II; B – I; C – IV; D – III
(a) A – I; B – III; C – II; D – IV; E – V
63. Match the phylum given in column - I with the special
(b) A – III; B – I; C – II; D – V; E – IV
features present in them given in column - II and choose
(c) A – II; B – I; C – III; D – V; E – IV
the correct option.
(d) A – II; B – III; C – I; D – V; E – IV Column -I Column- II
59. Match the characteristic feature/terms given in column I (Phylum) (Special features
with the phylum to which they belongs given in column II present)
and choose the correct option. A. Porifera I. Mammary glands
Column-I Column-II B. Mollusca II. Cloaca
(Characteristic (Phylum) C. Ctenophora III. Choanocytes
feature/term) D. Amphibia IV. Radula
A. Choanocytes I. Platyhelminthes E. Mammalia V. Comb plates
B. Cnidoblasts II. Ctenophora (a) A – III; B – IV; C – V; D – II; E – I
C. Flame cells III. Porifera (b) A – IV; B – III; C – V; D – II; E – I
D. Nephridia IV. Coelenterata (c) A – III; B – IV; C – II; D – V; E – I
E. Comb plates V. Annelida (d) A – III; B – V; C – IV; D – II; E – I
(a) A – II; B – I; C – IV; D – V; E – III 64. In which one of the following, the genus name, its two
(b) A – II; B – IV; C – I; D – V; E – III characters and its class/phylum are correctly matched?
(c) A – V; B – I; C – III; D – II; E – IV Genus Two characters Class/
(d) A – III; B – IV; C – I; D – V; E – II name phylum
60. Column I contains zoological names of animals and column (a) Ascaris (i) Body segmented Annelida
II contains their common name. Match the following and (ii) Males and
choose the correct option. females distinct
Column -I Column- II (b) Salamandra (i) A tympanum Amphibia
represents ear
A. Physalia I. Sea anemone
(ii) Fertilization is
B. Meandrina II. Brain coral
internal
C. Gorgonia III. Sea fan
(c) Pteropus (i) Skin possesses Mammalia
D. Adamsia IV. Portuguese man-of-war hair
(a) A – III; B – II; C – I; D – IV (ii) Viviparous
(b) A – IV; B – III; C – II; D – I (d) Aurelia (i) Cnidoblasts Coelenterata
(c) A – IV; B – II; C – III; D – I (ii) Organ level of
(d) A – II; B – III; C – I; D – IV organization
61. Match the organisms given in column-I with their common 65. In which one of the following the genus name, its two
name given in column-II and choose the correct option. characters and phylum are not correctly matched ?
Column -I Column -II
Genus name Two characters Phylum
(Organisms) (Comman name)
A. Pennatula I. Sea-lily (i) Body segmented
(a) Pila Mollusca
B. Antedon II. Sea- pen Mouth with radula
C. Echinus III. Sea-urchin (ii) Spiny skinned
D. Cucumaria IV. Sea - cucumber (b) Asterias Echinodermata
Water vascular system
(a) A – II; C – III; D – I; E – IV
(b) A – II; C – IV; D – I; E – III (iii) Pore bearing
(c) A – II; C – I; D – III; E – IV (c) Sycon Porifera
Canal system
(d) A – II; C – I; D – III; E – IV
(iv) Jointed appendages
62. Match the phylum given in column - I with their example (d) Periplaneta
Chitinous exoskeleton Arthropoda
given in column - II and choose the correct option.
EBD_7209
32 Biology
66. Match the animal name (column-I), with its characteristics Column-I Column-II
(column-II), and the phylum/class (column-III) to which it (Term/Feature) (Examples)
belongs. A. Gregarious pest (i) Hirudinaria
Co lumn - I Co lumn- II Colu mn -III B. Vector (ii) Planaria
(a) Ichthyo p his Terres trial Reptilia C. Oviparous with (iii) Sepia
(b) Limu lu s Bo dy Pis ces
indirect development
co vered D. Metameres (iv) Aedes
b y chitino us E. High regeneration (v) Locust
exo s keleto n capacity
A B C D E
(c) Ad amsia Rad ially Po rifera (a) (i) (ii) (iii) (iv) (v)
s y mmetrical (b) (iii) (v) (ii) (iv) (i)
(d) Petro myzo n Ecto paras ite Cyclo s tomata (c) (iii) (i) (v) (ii) (iv)
67. Which one of the following groups of animals is correctly (d) (v) (iv) (iii) (i) (ii)
matched with its characteristic feature without even a 70. Match the features given in column I with their examples
single exception ? given in column II and choose the correct match from the
(a) Reptilia : possess 3 - chambered heart with one option given below.
incompletely divided ventricle. Column-I Column-II
(b) Chordata : Possess a mouth provided with an upper (Features) (Examples)
and lower jaw. A. Pseudocoelomates (i) Hydra, Adamsia
(c) Chondrichthyes : Possess cartilaginous B. Diploblastic (ii) Ctenoplana, Aurelia
endoskeleton. C. Cellular level of (iii) Ascaris, Wuchereria
(d) Mammalia : Give birth to young one. organization
68. Column I contains the characteristics features and column D. Radial symmetry (iv) Sycon, Spongilla
II contains the function/ location. Select the correct match E. Metamerism (v) Pheretima, Neries
from the option given below. A B C D E
Column-I Column-II (a) (v) (ii) (iv) (iii) (i)
(Characteristic (Function/Location) (b) (iii) (i) (iv) (ii) (v)
feature) (c) (ii) (i) (iii) (v) (iv)
A. Water canal system (i) Sponges (d) (iii) (ii) (iv) (i) (v)
B. Comb plates (ii) Help in swimming
DIAGRAM TYPE QUESTIONS
C. Nephridia (iii) Present in mollusca
D. Jointed appendages (iv) Characteristics of 71. The given figures (A & B) shows the germinal layer.
roundworm
Ectoderm
E. Muscular foot (v) A body part of Mesoglea
Endoderm
arthropoda
(vi) Helps in reproduction
(vii) Platyhelminthes
(viii) Helps in osmoregulation
and excretion
(ix) Eight ciliated external
rows present in a body Mesoderm
A B
of ctenophora.
A B C D E The animals having structures shown in the figures are
(a) (i) (ix) (viii) (v) (iii) respectively called
(b) (iii) (i) (vi) (ii) (v) (a) diploblastic, triploblastic
(c) (ii) (v) (i) (iv) (ix) (b) triploblastic, diploblastic
(d) (iii) (vi) (iv) (v) (i) (c) diploblastic, diploblastic
69. Match the terms/feature given in column I with their (d) triploblastic, triploblastic
examples given in column II and select the correct match 72. Refer the figures A, B, C and D given below. Which of the
from the option given below. following options shows the correct name of the animals
shown by the figures A, B, C and D ?
Animal Kingdom 33

76. Identify the figure with its correct name and phylum.

A B C D
(a) A – Locust, B – Scorpion, C – Prawn, D – Pila
(b) A – Locust, B – Prawn, C – Scorpion, D – Pila
(c) A – Locust, B – Scorpion, C – Prawn, D – Snail
(d) A – Butterfly, B – Scorpion, C – Prawn, D – Pila (a) Sycon - Porifera
73. Refer the given figures A, B, C and D and identify the (b) Aurelia - Coelenterata
(c) Pleurobrachia - Ctenophora
option which shows their correct name. D
B C (d) Tapeworm - Platyhelminthes
A
77. Identify the figures A, B and C and choose the correct
option .

A B C D
(a) Pleurobrachia Cnidoblast Aurelia Adamsia
(b) Aurelia Adamsia Cnidoblast Pleurobrachia
(c) Cnidoblast Pleurobrachia Adamsia Aurelia
(d) Adamsia Aurelia Pleurobrachia Cnidoblast
74. Examine the figures A, B and C. In which one of the four
options all the items A, B and C are correctly identified ?
A B C
(a) A - Male Ascaris, B - Hirudinaria (leech), C- Nereis
(b) A - Female Ascaris, B - Nereis, C-Hirudinaria (leech)
(c) A - Female Ascaris B- Hirudinaria (leech), C - Nereis
(d) A - Male Ascaris, B - Nereis, C- Hirudinaria (leech)
78. Identify the animals shown in the given figures A, B and C
B C from options given below.
A
A B C
(a) Sycon Euspongia Spongilla
(b) Euspongia Spongilla Sycon
(c) Spongilla Sycon Euspongia
(d) Euspongia Sycon Spongilla A B C
75. Identify the figures and select the correct option. (a) A - Octopus; B -Asterias, C- Ophiura
(b) A - Asterias; B - Ophiura, C- Octopus
(c) A - Echinus; B - Octopus C - Ophiura
(d) A - Ophiura; B - Echinus, C- Octopus
79. Identify the figure with its correct name and phylum.

A B

C
(a) A - Pseudocoelomate; B - Coelomate, C-Acoelomate
(b) A - Coelomate, B - Pseudocoelomate, C- Acoelomate (a) Cucumaria – Echinodermata
(c) A - Coelomate; B- Acoelomate; C - Pseudocoelomate (b) Ascidia – Urochordata
(d) A - Coelomate; B- Acoelomate; C-Eucoelomate (c) Balanoglossus – Hemichordata
(d) Hirudinaria – Annelida
EBD_7209
34 Biology
80. The given figure shows some characteristic features 83. The given figures of animals (A & B) are distinguished on
marked as chordates. Identify the correct labelling A,B,C the basis of symmetry. Select the correct option which
and D. shows the type of symmetry and its description against
A
the animals.
B

C
D
(a) A-Notochord; B-Post-anal part; C-Gill slits; D-Nerve
cord A B
(b) A-Nerve cord; B-Notochord; C-Post-anal part; D-Gill (a) A : Biradial, organisms is divided into unequal halves
slits by any plane through the central axis.
(c) A-Notochord; B-Nerve cord; C-Gill slits; D-Post-anal (b) B: Bilateral, body is divided into equivalent right and
part left halves by only one plane.
(d) A-Gill slits; B-Post-anal part; C-Nerve cord; D- (c) A: Asymmetrical, organisms is not divided into equal
Notochord halves by any plane through the central axis.
81. Refer the figures A, B and C and choose the correct option (d) B: Radial, in which any plane passing through the
which shows animals that regulate buoyancy with the help central axis of the body divides the organism into two
of air bladder. identical halves.
84. The figure given below shows the germinal layers marked
as A, B, C and D. Identify the label showing undifferentiated
layer and its location?
B
A
C

A B C
(a) A and B (b) A and C
(c) B and C (d) All of the above. D
82. The given figures A, B, C and D are the examples of first (a) A, Between B & C (b) B, Between A & C
true land vertebrates. They are dominant in mesozoic era (c) C, Between C & D (d) D, Between A & B
and belong to phylum ‘X’. Identify ‘X’ and the animals 85. The figure given below is the characteristic structure of
which have four chambered heart. the phylum in which animals are aquatic, free swimming or
sessile, mostly marine, radially symmetrical. Identify the
phylum and correct function of the structure.

A B

C D (a) Ctenophora; Emission of light.


(a) X – Reptile; B (b) X – Reptile; A (b) Porifera; Feeding, respiration and excretion.
(c) X – Amphibia, C (d) X – Pisces; D (c) Cnidarian; Anchorage, Defense and food capturing
(d) Mollusca; Locomotion, transport of food and respiration.
Animal Kingdom 35

86. Which of the following feature is not correct regarding the 89. Identify the correct characteristic feature shown by the
figure given below? given figure?

(a) Diploblastic in nature.


(b) Having radial symmetrical body.
(c) Dioecious with direct development.
(a)It is an aquatic form. (d) Presence of sensory tentacles on anterior head region.
(b)Circulatory system is of open type. 90. Which of the following animals are bilaterally symmetrical?
(c)It possesses parapodia for swimming.
(d)Neural system consists of paired ganglia connected
by lateral nerves to a double ventral nerve cord.
87. Which of the following animal's body is covered by
calcareous shell and unsegmented with a distinct head,
muscular foot, and visceral hump?
1 2 3 4
(a) 1 & 2 (b) 2 & 4
(c) 3 & 4 (d) 1 & 3
(a) (b) CRITICAL THINKING TYPE QUESTIONS
91. Which of the following pairs of animals are similar to each
other pertaining to the feature stated against them?
(a) Pteropus and Ornithorhyncus - Viviparity
(b) Garden lizard and crocodile - Three chambered heart
(c) Ascaris and Ancylostoma - Metameric segmentation
(c) (d) (d) Sea horse and flying fish - Cold blooded (poikilothermal)
92. Which of the following group of animals belongs to the
same phylum?
88. Which of the following animal contains respiratory organs (a) Earthworm, pinworm, tapeworm
like, gills, book gills, book lungs or tracheal system? (b) Prawn, scorpion, Locusta
(c) Sponge, Sea anemone, starfish
(d) Malarial parasite, Amoeba, mosquito
93. Which of the following traits is not shared by both sea
anemones and jellyfish ?
(a) A medusa as the dominant stage in the life cycle.
(b) Possession of a gastro vascular cavity.
(a) (b) (c) Sexual reproduction.
(d) Nematocysts present on the tentacles.
94. The combination of a true coelom and repeating body
segmentation allows the annelids (unlike the anatomically
“simpler” worms) to do which of the following?
(a) Attain complex body shapes and thus locomote more
precisely.
(b) Move through loose marine sediments.
(c) Be hermaphroditic.
(d) Inject paralytic poisons into their prey.
(c) (d) 95. The transition from aquatic to terrestrial lifestyles required
many adaptations in the vertebrate lineage. Which of the
following is not one of those adaptations ?
(a) Switch from gill respiration to air-breathing lungs.
(b) Improvements in water resistance of skin.
(c) Alteration in mode of locomotion.
(d) Development of feathers for insulation.
EBD_7209
36 Biology
96. Which of the following features distinguish mammals from 104. A common characteristic of all vertebrates without
other vertebrates ? exception is
(a) Hairy skin and oviparity (a) the division of body into head, neck, trunk and tail.
(b) Hairy skin and mammary glands (b) body covered with exoskeleton.
(c) Mammary glands and teeth (c) the possession of two pairs of functional
(d) Pinna and teeth appendages.
97. Which of the following sets of animals give birth to young (d) the presence of well- developed skull.
ones? 105. The organisms attached to the substratum generally
(a) Platypus, Penguin, Bat, Hippopotamus. possess
(b) Shrew, Bat, Cat, Kiwi. (a) one single opening to the digestive canal.
(c) Kangaroo, Hedgehog, Dolphin, Loris. (b) cilia on the surface to create water current.
(d) Lion, Bat, Whale, Ostrich. (c) radial symmetry.
98. Which one of the following features is common in (d) asymmetrical body.
silverfish, scorpion, dragonfly and prawn?
106. Which of the following statements is without exception
(a) Three pairs of legs and segmented body.
in sponges ?
(b) Chitinous cuticle and two pairs of antennae.
(a) They all have calcareous spicules.
(c) Jointed appendages and chitinous exoskeleton.
(b) They have high regenerative power.
(d) Cephalothorax and tracheae.
(c) They are found only in marine water.
99. Which of the following is a correct match of a phylum
(d) They are all radially symmetrical.
with its three examples?
(a) Platyhelminthes–Planaria, Schistosoma, Enterobius 107. Which of the following characters is absent in all
(b) Mollusca – Loligo, Sepia, Octopus chordates?
(c) Porifera – Spongilla, Euplectella, Pennatula (a) Diaphragm
(b) Coelom
(d) Cnidaria – Bonellia, Physalia, Aurelia
(c) Pharyngeal gill clefts
100. Hemichordates have now been placed with the non-
(d) Dorsal nerve cord
chordates, close to echinoderms, because true
(a) notochord is absent. 108. Which of the following features in birds indicates their
(b) pharyngeal gill-slits are lacking. reptilian ancestory ?
(c) dorsal nerve cord is absent. (a) Eggs with a calcareous shell
(b) Scales on their hind limbs
(d) heart is lacking.
(c) Four-chambered heart
101. Which of the following is not a characteristic feature of
(d) Two special chambers-crop and gizzard in their
kingdom animalia ?
digestive tract
(a) Storage of carbohydrates as starch.
(b) Multicellularity. 109. Which of the following characteristic is probably most
(c) Obtaining nutrients by ingestion. responsible for the great diversification of insects on land?
(d) Having eukaryotic cells without walls. (a) Segmentation
102. Which of the following characteristic distinguish (b) Antennae
arthropoda from annelids and molluscs ? (c) Bilateral symmetry
(a) An external skeleton made of chitin (a (d) Exoskeleton
polysaccharide) and protein rather than a shell made 110. Which of the following is a connecting link between
chiefly of mineral salts. invertebrates and non-invertebrates?
(b) Subdivision of the legs into movable segments. (a) Sphenodon
(c) Distinct group of muscles, derived from many body (b) Balanoglossus
segments, that move the separate parts of the (c) Tadpole larva
exoskeleton. (d) Crocodile
(d) All of the above 111. A student brought home a strange animal which he found
103. Tracheae of cockroach and mammal are similar in having outside under a rock. It had moist skin, a complete
(a) paired nature. digestive tract, a ventral nerve cord, and had gone through
(b) non-collapsible walls. torsion. Identify the phylum of the animal.
(c) ciliated inner lining. (a) Porifera (b) Annelida
(d) origin from head. (c) Mollusca (d) Echinodermata
Animal Kingdom 37

112. Identify the correct characteristics of porifera. (a) (i) and (iii) only
(i) Commonly known as sea walnuts. (b) (ii) and (iv) only
(ii) Presence of ostia and collar cells. (c) (ii), (iii), (v) and (vi) only
(iii) Exhibit tissue level of characteristics. (d) All of these
(iv) It is the largest phylum of animal kingdom. 115. Refer the types of cells present in some animals. Each cell
(v) The body is supported by spicules and sponging is specialized to perform a single specific function except
fibers. (a) Cnidocytes
(vi) Contains cnidocytes which is used for defense, (b) Choanocytes
anchorage and capturing of prey. (c) Interstitial cells
(a) (ii), (v) only (d) Gastrodermal cells
(b) (i), (ii), (vi) only 116. Select the incorrect feature of mollusca from the given
(c) (i), (ii), (iii), (iv) only statements.
(d) All of these. (i) Terrestrial or aquatic animals having cellular system
113. A student was given a specimen to identify on the basis level of organization.
of the characteristics given below. (ii) Radial symmetrical and acoelomate animals and
(i) They are metamerically segmented. possesses two germinal layers.
(ii) They have closed circulatory system. (iii) A file like rasping organ called radula is present.
(iii) They have circular and longitudinal muscles for (iv) Usually dioecious and viviparous animals.
locomotion. (v) Examples include Pila, Octopus, and Dentalium.
Identify the specimen. (a) (i) and (ii) only
(a) Prawn (b) Pheretima (b) (ii) and (iv) only
(c) Wuchereria (d) Ctenoplana (c) (i), (ii) and (iv) only
114. Refer the following animals and identify those which have (d) All the five statements.
a fluid filled body cavity with a complete lining derived
from mesoderm.
(i) Sycon (ii) Butterfly
(iii) Nereis (iv) Sea fan
(v) Scorpion (vi) Pila
EBD_7209
Morphology of
5

Chapter
Flowering Plants

FACT/DEFINITION TYPE QUESTIONS 10. Pneumatophores are found in


(a) the vegetation which is found in marshy and saline
1. Which of the following plant parts elongates directly lake.
and leads to the formation of primary roots? (b) the vegetation which is found in saline soil.
(a) bud (b) radicle (c) xerophytic condition.
(c) plumule (d) root hair (d) hydrophytic condition.
2. The primary roots and its branches constitute the 11. Which of the following plants grow in swampy areas,
(a) fibrous root system. where the roots come out of the ground and grow
(b) tap root system. vertically upwards?
(c) adventitious root system. (a) Potato (b) Opuntia
(d) all of the above (c) Rhizophora (d) Grass
3. Fibrous root system is found in
12. Root differs from stem in having
(a) monocotyledonous plants.
(a) nodes and internodes (b) axillary buds
(b) dicotyledonous plants.
(c) bryophytes. (c) multicellular hairs (d) unicellular hairs
(d) gymnosperms. 13. Which of the following plant parts is generally green
when young and later often becomes woody and dark
4. Roots develop from parts of the plant other than radicle
brown?
are called
(a) tap roots (b) fibrous roots (a) stem (b) seed
(c) leaves (d) flower
(c) adventitious roots (d) nodular roots
5. Root hairs develop from 14. The regions of the stem where leaves are borne are called
____________ while ____________ are the portions
(a) region of maturation
between two ____________.
(b) region of elongation
(a) nodes, nodes and internodes
(c) region of meristematic activity
(b) nodes, internodes and nodes
(d) root cap
(c) internodes, nodes and nodes
6. The part of the root which is most active in water
(d) internodes, internodes and nodes
absorption is called
15. Which of the following gr oups of plants have
(a) root cap (b) maturation zone
underground stems?
(c) meristematic zone (d) zone of elongation
(a) Potato, ginger, turmeric, Euphorbia, zaminkand
7. Fibrous roots develop in maize from
(b) Potato, ginger, turmeric, zaminkand, Colocasia
(a) upper nodes (b) lower nodes
(c) Potato, Citrus, Opuntia, zaminkand, Colocasia
(c) upper internodes (d) none of these
(d) Potato, cucumber, watermelon, zaminkand,
8. Prop roots of banyan tree are meant for
Colocasia
(a) respiration.
16. Stem tendrils are found in
(b) absorption of water from soil.
(a) cucumber (b) pumpkins
(c) providing support to big tree.
(c) grapevines (d) all of these
(d) all of the above.
17. Which of the following is a modified stem for the
9. Stilt roots occur in __________ . protection of plants from browsing animals?
(a) groundnut (b) rice (a) Tendrils (b) Thorns
(c) sugarcane (d) wheat (c) Rhizome (d) Tuber
Morphology of Flowering Plants 39

18. A branch in which each node bearing a rossette of leaves 30. Pollen grains are produced within the _________of
and a tuft of roots is found in aquatic plants like stamen.
_________ and _________. (a) ovary (b) anther
(a) Hydrilla and Pistia (c) filament (d) connective
(b) Eichhornia and Hydrilla 31. Which one of the following structure is not associated
(c) Pistia and Eichhornia with gynoecium?
(d) Pistia and Vallisneria (a) Ovary (b) Style
19. In which of the following plants, a slender lateral branch (c) Stigma (d) Filament
arises from the base of the main axis, and after growing 32. During the post-fertilization period, the ovules develop
aerially arch downwards to touch the ground? into ___A_____ and the ovary matures into a ___B ____.
(a) Mint and jasmine (b) Banana and pineapple (a) A - seeds; B - fruit (b) A - fruit; B - seeds
(c) Grass and stawberry (d) Pistia and Eichhornia (c) A - flower; B - seed (d) A - seeds; B - flower
20. Which of the following is the green expanded part of 33. A scar on the seed coat through which the developing
leaf with vein and veinlets? seeds are attached to the fruit is called as ________.
(a) Petiole (b) Node (a) testa (b) tegmen
(c) Stipule (d) Lamina (c) hilum (d) micropyle
21. Leaves of dicotyledonous plants possess _________ 34. Cotyledons and testa respectively are edible parts in which
venation, while _________ venation is the characteristic of the following plant group?
of most monocotyledons. (a) walnut and tamarind
(a) reticulate and parallel (b) cashew nut and litchi
(b) parallel and reticulate (c) french bean and coconut
(c) reticulate and perpendicular (d) groundnut and pomegranate
(d) obliquely and parallel 35. Floral formula of tomato/tobacco is
22. Which of the following is an example of pinnately
(a) K4–5 A10G(2)
compound leaf ?
(a) Cucumber (b) Papaya (b) K 2+2 C 4A2+4G 1
(c) Cucurbita (d) Neem
23. The main purpose of phyllotaxy for the leaves is to provide (c) P 2 A 3G1
sufficient_______.
(a) soil (b) air (d) K(5) C(5)A5 G(2)
(c) water (d) light 36. Botanical name of Cauliflower is
24. Alternate type of phyllotaxy is found in (a) Brassica oleracea var. capitata
(a) china rose (b) mustard (b) Brassica campesteris
(c) sunflower (d) all of these (c) Brassica oleracea var. botrytis
25. Which type of function is performed by the fleshy leaves (d) Brassica oleracea var. gemmifera
of onion and garlic? 37. Botanical name of banana is
(a) Storage (b) Reproduction (a) Musa paradisica (b) Phaseolus vulgaris
(c) Photosynthesis (d) Protection (c) Ricinus communis (d) Ananas sativus
26. The flower is the reproductive unit in the ___________
meant for ___________ reproduction. STATEMENT TYPE QUESTIONS
(a) angiosperms and sexual
(b) gymnosperms and sexual 38. Which one of the following statements is correct?
(c) algae and asexual (a) Bulb of Allium cepa is a modified stem.
(d) pteridophytes and asexual (b) Cloves of Allium sativum are fleshy scale leaves.
27. When a flower has both androecium and gynoecium, it is (c) Corm of Colocasia is a modified root.
known as__________. (d) Tendril in Vitis vinifera is a modified axillary bud.
(a) asexual (b) bisexual 39. Which one of the following statements is not correct?
(c) unisexual (d) multisexual (a) Each stamen which represents the male
28. When a flower can be divided into two equal radial halves reproductive organ consists of a stalk or a filament
in any radial plane passing through the centre, it is known as and an anther.
(a) actinomorphic (b) zygomorphic (b) An actinomorphic flower can be dissected into two
(c) asymmetric (d) bisymmetric equal halves from any plane.
29. A sterile stamen is known as (c) Superior ovary is found in hypogynous flowers.
(a) staminode (b) anther (d) When stamens are attached to petals, they are
(c) pollen grain (d) filament epiphyllous as in brinjal.
EBD_7209
40 Biology
40. Which of the following statement(s) is/are correct about 46. Consider the following statements regarding the root
calyx? system of angiosperms and choose the correct option
(a) Calyx is the outermost whorl of the flower and are given below.
called sepals. (i) In monocots, the fibrous root system arises from the
(b) Sepals are green, leaf like structure and protect the base of the stem.
flower in the bud stage. (ii) The region of elongation is called the root hair region.
(c) The calyx may be gamosepalous (sepals free) or (iii) In sweet potato, the adventitious roots get swollen
polysepalous (sepals united). and store food.
(d) Both (a) and (b) (iv) The stems of maize and sugarcane have supporting
41. Which of the following statement(s) is/are correct about roots called prop roots.
venation? (a) (i) and (ii) are correct but (iii) and (iv) are wrong.
(i) The arrangement of veins and the veinlets in the (b) (ii) and (iii) are correct but (i) and (iv) are wrong.
lamina of leaf is called venation. (c) (ii) and (iv) are correct but (i) and (iii) are wrong.
(ii) Reticulate venation is the characteristic of monocots. (d) (i) and (iii) are correct but (ii) and (iv) are wrong.
(iii) When the veinlets form a network, the venation is 47. Which of the following statement (s) is/are not correct?
termed as reticulate venation. (i) Calyx and corolla are reproductive organs of a flower.
(iv) When the veins run parallel to each other within a (ii) Zygomorphic flower can be divided into two equal
radial halves in any radial plane.
lamina, the venation is termed as parallel venation.
(iii) Flowers without bracts are termed as bracteate.
(a) Only (i) (b) Both (i) and (ii)
(iv) Parthenocarpic fruit is formed after fertilization of the
(c) (i), (iii) and (iv) (d) All of these
ovary.
42. Which one of the following are not true for parietal
(v) In legumes, seed is non-endospermic.
placentation?
(vi) Radical buds develop on roots.
(a) Ovules are borne on central axis. (a) (i) , (ii), (iii) and (iv) (b) (i), (ii) and (v)
(b) Ovary is one-chambered but it becomes two- (c) (iii), (iv) and (vi) (d) (iv), (v) and (i)
chambered due to the formation of false septum. 48. Which of the following statements are correct about the
(c) Examples are mustard and Argemone. leaf?
(d) Both (b) and (c) (i) Leaf is a lateral, generally flattened structure borne
43. Which one of the following characteristics is not related on the stem.
to gynoecium? (ii) It develops at the node and bears a bud in its axil.
(a) It is the female reproductive part of the flower. (iii) Leaves originate from root apical meristems and
(b) It is composed of stamens. arranged in an acropetal order.
(c) Stigma is usually at the tip of the style and is the (iv) They are the most important vegetative organs for
receptive surface for pollen grains. reproduction.
(d) Each ovary bears one or more ovules attached to a (a) (i) and (ii) (b) (ii) and (iii)
flattened, cushion like placenta. (c) (i), (ii) and (iv) (d) all of these
44. Study the following statements and select the correct 49. Read the following statements and answer the question.
option (i) Gynoecium is situated in the centre, and other parts
(i) Buds are present in the axil of leaflets of the of the flower are located on the rim of the thalamus
compound leaf. almost at the same level.
(ii) Pulvinus leaf-base is present in some leguminous plants. (ii) Ovary is half-inferior.
(iii) In Alstonia, the petioles expand, become green and (iii) Examples are plum, rose and peach.
synthesize food. Which condition of flowers is being described by the
(iv) Opposite phyllotaxy is seen in guava. above statements ?
(a) (ii) and (iv) are correct but (i) and (iii) are incorrect. (a) hypogyny (b) perigyny
(b) (i) and (iii) are correct but (ii) and (iv) are incorrect. (c) epigyny (d) none of these
(c) (i) and (iv) are correct but (ii) and (iii) are incorrect. 50. Read the following statements and answer the question.
(d) (ii), (iii) and (iv) are correct but (i) is incorrect. (i) It is the pattern of arrangement of leaves on the stem
45. Which of the following statements are correct? or branch.
(i) From the region of elongation, some of the epidermal (ii) It is usually of three types - alternate, opposite and
cells form root hairs. whorled.
(ii) Pneumatophores are seen in Rhizophora. (iii) It is meant for getting maximum amount of light.
(iii) Adventitious roots are seen in the banyan tree. Which condition of plant is being described by the above
(iv) Maize and sugarcane have prop roots. statements?
(a) (i) and (iv) (b) (i), (iii) and (iv) (a) Phyllotaxy (b) Venation
(c) (iii) and (iv) (d) (ii) and (iii) (c) Inflorescencew (d) Aestivation
Morphology of Flowering Plants 41

51. Which of the following statement(s) is/are correct about 59. Assertion : In stem, pericycle take active part in
the fruit? secondary growth.
(i) Fruit is a mature or ripened ovary, developed before Reason : In dicots, pericycle has the capacity to produce
fertilization. lateral roots.
(ii) It consists of a wall or pericarp and seeds.
(iii) When pericarp is thick and fleshy, it is differentiated MATCHING TYPE QUESTIONS
into outer mesocarp, middle epicarp and inner endocarp.
60. Match column-I containing types of aestivation with their
(iv) In mango and coconut, the fruit is known as a berry.
examples given in column-II and choose the correct option.
(a) Only (i) (b) Both (ii) and (iii)
Column - I Column-II
(c) Only (ii) (d) All of these
(Type of aestivation) (Examples)
52. Which of the following statement(s) is /are correct?
A. Valvate I. Cotton
(i) Many plants belonging to the family fabaceae are
B. Twisted II. Calotropis
good ornamentals (Tulip, Gloriosa), source of
C. Imbricate III. Bean
medicine (Aloe) and vegetables (Asparagus).
D. Vexillary IV. Gulmohar
(ii) The plumule and radicle are enclosed in sheaths
(a) A – I; B – II; C – IV; D – III
which are called coleorhiza and coleoptile
(b) A – II; B – I; C – IV; D – III
respectively.
(c) A – II; B – IV; C – I; D – III
(iii) A flower having either stamens or carpels is unisexual.
(d) A – II; B – I; C – III; D – IV
(iv) Basal, alternate, linear, exstipulate with parallel
61. Match the following placentation types given in column
venation types of leaves is found in the family
I with their examples given in column II and choose the
liliaceae.
correct combination from the options given below.
(a) Only (i) (b) Both (i) and (ii)
(c) Both (iii) and (iv) (d) All of these
Column-I Column-II
53. Which of the following statements is correct?
(a) Imbricate aestivation is found in the papilionaceous (Placentation Types) (Examples)
family. A. Basal I. Dianthus
(b) Generally, sepals are green, leaf like and protect the B. Free central II. Pea
flowers in the bud stage. C. Parietal III. Lemon
(c) In cymose type of inflorescence, the main axis
D. Axile IV. Marigold
terminates in a flower, hence is limitless in growth.
(d) In axile placentation ovary is one chambered but it becomes E. Marginal V. Argemone
two cambered due to the formation of false septum.
(a) A – I, B – II, C – III, D – IV, E – V
ASSERTION/REASON TYPE QUESTIONS (b) A – II, B – III, C – IV, D – V, E – I
In the following questions, a statement of Assertion is followed (c) A – IV, B – I, C – V, D – III, E – II
by a statement of Reason. (d) A – IV, B – III, C – V, D – I, E – II
(a) If both Assertion and Reason are true and the Reason is 62. Match the following stem modifications given in column
the correct explanation of the Assertion. I with their examples given in column II and select the
(b) If both Assertion and Reason are true but the Reason is correct combination from the options given below.
not the correct explanation of the Assertion.
(c) If Assertion is true but Reason is false. Column-I Column-II
(d) If both Assertion and Reason are false. (Stem Modifications) (Found in)
54. Assertion : A simple leaf has undivided lamina. A. Underground stem I. Euphorbia
Reason : Leaves showing pinnate and palmate
B. Stem tendril II. Opuntia
venations have various type of incisions.
55. Assertion : Ginger has a prostrate-growing rhizome. C. Stem thorns III. Potato
Reason : Shoot growth is not effected by gravity. D. Flattened stem IV. Citrus
56. Assertion : Many plants are propagated vegetatively E. Fleshy cylindrical V. Cucumber
even though they bear seeds. stem
Reason : Potatoes multiply by tubers, apple by cutting etc.
57. Assertion : Ginger has a prostrate growing rhizome. (a) A – I, B – II, C – III, D – V, E – IV
Reason : Shoot growth is not effected by gravity. (b) A – II, B – III, C – IV, D – V, E – I
58. Assertion : In fabaceae family monocarpellary, unilocular (c) A – III, B – IV, C – V, D – I, E – II
ovary is present. (d) A – III, B – V, C – IV, D – II, E – I
Reason : In fabaceae, placentation is parietal.
EBD_7209
42 Biology
63. Match the andsoecium formula (given in column II) with 67. Matching colmun I with column II and choose the correct
their family (given in column I) and choose the correct option.
combination from the options given below. Column I Column II
Column-I Column-II A. Coleorhiza I. Grapes
(Family) (Androecium formula) B. Food storing tissue II. Mango
A. Brassicaceae I. A3 + 3 C. Parthenocarpic fruit III. Maize
B. Fabaceae II. A(5) D. Single seeded fruit IV. Radicle
C. Solanaceae III. A(9) + 1 developing from
D. Liliaceae IV. A2 + 4 monocarpellary
(a) A – IV; B – III; C – II; D – I superior ovary
(b) A – I; B – II; C – III; D – IV E. Membranous V. Endosperm
(c) A – II; B – III; C – IV; D – I seed coat
(d) A – III; B – IV; C – I; D – II (a) A – III, B – I, C – IV, D – II, E – V
64. Match column I with column II and choose the correct (b) A – IV, B – II, C – V, D – I, E – III
combination from the options given below.
(c) A – V, B – I, C – III, D – IV, E – II
Column-I Column-II
(d) A – IV, B – V, C – I, D – II, E – III
(Position of floral parts (Represented in)
on thalamus) 68. Match column-I with column-II and choose the correct
A. Hypogynous I. Ray florets of sunflower option.
B. Perigynous II. Brinjal Column-I Column-II
C Epigynous III Peach (Members of Fabaceae) (Economic importance)
(a) A – II, B – I, C – III (b) A – I, B – II, C – III A. Gram, sem, moong, I. Medicine
(c) A – III, B – II, C – I (d) A – II, B – III, C – I soyabean
65. Match column I with column II and choose the correct B. Soyabean,groundnut II. Ornamental
option. C. Indigofera III. Fodder
Column-I Column-II D. Sunhemp IV. Fibres
A. Bud in the I. Pitcher plant E. Sesbania, Trifolium V. Dye
axil of leaf and venus fly trap F. Lupin, sweet potato VI. Edible oil
B. Outer layer of II. Cacti G. Mulethi VII. Pulses
seed coat (a) A – I, B – II, C – III, D – IV, E – V, F – VI, G – VII
C. Spines III. Testa
(b) A – VII, B – VI, C – V, D – IV, E – III, F – II, G – I
(modified leaves)
(c) A – II, B – IV, C – VI, D – I, E – III, F – V, G – VII
D. Leaves modified IV. Simple leaf
to catch insects (d) A – I, B – III, C – V, D – VII, E – II, F – IV, G – VI
E. Fleshy leaves V. Garlic and onion 69. Match column - I, II and III and choose the correct option.
with stored food Column-I Column-II Column-III
(a) A – I, B – II, C – III, D – IV, E – V
(b) A – V, B – IV, C – III, D – II, E – I A. Marginal I. p. Sunflower,
Marigold
(c) A – IV, B – III, C – II, D – I, E – V
(d) A – IV, B – II, C – III, D – I, E – V
66. Match column-I with column-II and choose the option
which shows their correct combination.
B. Axile II. q. Dianthus,
Column-I Column-II Primrose
A. Gamosepalous I. Flower of lily
B. Polysepalous II. Sterile anther
C. Gamopetalous III. Free petals
D. Polypetalous IV. Free sepals C. Parietal III. r. Mustard,
E. Epiphyllous V. Fused petals Argemone
F. Staminode VI. Fused sepals
(a) A – IV, B – V, C – III, D – I, E – VI, F – II
(b) A – IV, B – III, C – V, D – I, E – II, F – VI
(c) A – VI, B – IV, C – III, D – V, E – I, F – II
D. Free central IV. s. China rose,
(d) A – VI, B – IV, C – V, D – III, E – II, F – I
tomato, lemon
Morphology of Flowering Plants 43

72. The given figures (A and B) show the modificaiton of


roots.Which of the following statements regarding the
figures is correct ?
E. Basal V. t. Pea

(a) A - V, t; B - II, s; C -I, r; D -III, q; E -IV, p


(b) A - I, t; B - II, s; C - III, r; D - IV, p; E - V, q
(c) A - V, p; B - II, s; C - I, q; D - III, r; E - IV, t
(d) A - V, p; B - III, q; C - II, s; D - I, t; E - IV, r
70. Match the column I with column II and choose the correct
option.
Column I Column II A B
A. Placentation (i) Arrangement of (a) Tap roots of carrot, turnip and adventitious root of
flowers sweet potato, get swollen and store food.
on the rachis
(b) Pneumatophores conducts water, minerals &
B. Aestivation (ii) Modified shoot for
sexual reproduction photosynthesis
C. Inflorescence (iii)Arrangement of (c) Pneumatophore is found in the plants that grow in
various whorls in the sandy soil.
bud (d) Turnip & carrot shows adventitious roots and sweet
D. Flower (iv) Arrangement of potato shows tap root.
ovules within an ovary 73. Which of the following option shows the correct
A B C D labelling of the parts of leaf marked as A, B, C and D.
(a) (i) (ii) (iii) (iv)
(b) (iii) (i) (ii) (iv)
(c) (iii) (i) (iv) (ii)
(d) (iv) (iii) (i) (ii)

DIAGRAM TYPE QUESTIONS


71. The given figure shows the regions of root tip with
labelling as A, B and C. Choose the option which shows
the correct labelling of A, B and C.
Region of mature cells

A B C D
(a) Lamina Axillary bud Stipule Leaf base
(b) Lamina Stipule Axillary bud Leaf base
(c) Lamina Axillary bud Stipule Pedicel
(d) Leaflet Axillary bud Stipule Leaf base
74. Identify the inflorescence shown by the given figures A
and B.

(a) A - Zone of elongation, B - Zone of meiosis, C -


Zone of mitosis.
(b) A - Zone of maturation, B - Zone of meristematic
activity, C - Zone of elongation. (a) A-Cymose, B-Racemose
(c) A - Zone of mitosis, B - Zone of elongation, C - Zone (b) A-Racemose, B-Cymose
of root cap. (c) A-Racemose, B-Racemose
(d) A - Region of maturation, B - Region of elongation, (d) A-Cymose, B-Cymose
C - Zone of meristematic activity.
EBD_7209
44 Biology
75. Identify the different types of aestivation (A, B, C and D) 79. Which one of the following options shows the correct
in corolla and select the correct option. labelling of the structure marked as A, B, C & D?

(a) A-Valvate, B-Twisted, C-Imbricate, D-Vexillary


(b) A-Vexillary, B-Valvate, C-Twisted, D-Imbricate
(c) A-Imbricate, B-Vexillary, C-Valvate, D-Twisted
(d) A-Twisted, B-Imbricate, C-Vexillary, D-Valvate
76. Given figures (A, B and C) show the position of floral
parts on thalamus. (given as I, II and III) Select the correct
combination.

A B C D
(a) Gynoecium Megasporophyll Ovule Thalamus
(b) Gynoecium Stamen Seed Thalamus
(c) Microsporophyll Stamen Ovule Thalamus
(d) Gynoecium Stamen Ovule Thalamus
80. The given figure shows the parts of mango and coconut.
Choose the option which shows the correct labelling of
A, B, C and D marked in the figures.

I. Hypogynous flower II. Perigynous flower


III. Epigynous flower
(a) A-I, B-II, C-III (b) A-I, B-III, C-II
(c) A-III, B-II, C-I (d) A-III, B-I, C-II
77. The given figures (A & B) show two types of compound
leaves. Choose the option which identity the correct
compound leaf and their example (c).

A B C D
(a) Epicarp, Mesocarp, Seed, Endocarp
(b) Epicarp, Mesocarp, Ovule, Endocarp
(c) Epicarp, Mesocarp, Ovary, Endocarp
(d) Epicarp, Mesocarp, Embryo, Endocarp
81. Which one of the options shows the correct labelling of
(a) A - Pinnately compound leaf, C - Neem the parts marked as A, B, C and D in a typical structure of
(b) A - Palmately compound leaf, C - Neem dicotyledonous seeds ?
(c) B- Pinnately compound leaf, C - Silk cotton
(d) B- Palmately compound leaf, C - Silk cotton
78. Identify the kind of phyllotaxy shown in the given figures
A, B, and C.

(a) A - Hilum, B - Micropyle, C - Radicle, D - Cotyledon,


E - Plumule
(b) A - Hilum, B - Micropyle, C - Plumule,
D - Cotyledon, E - Radicle
(a) A-Alternate, B - Opposite, C - Whorled (c) A - Micropyle, B - Hilum, C - Plumule,
(b) A- Whorled, B - Opposite, C -Alternate D - Cotyledon, E - Radicle
(c) A-Alternate, B - Whorled, C - Opposite (d) A - Hilum, B - Micropyle, C - Plumule, D - Radicle,
(d) A-Whorled, B -Alternate, C - Opposite E - Cotyledon
Morphology of Flowering Plants 45

82. The given figure shows a typical structure of


monocotyledonous seeds. Identify the parts A, B, C, D
and E marked in the given figures.

(C)
(a) A (b) B
(a) A - Endosperm, B - Embryo, C - Scutellum, (c) C (d) Both A and B
D - Coleorhiza, E - Coleoptile 85. The given figure shows the parts of flowering plant.
(b) A- Embryo, B - Endosperm, C - Scutellum, Which parts of the given figure were involved in the
D - Coleoptile, E - Coleorhiza following functions:
(c) A - Endosperm, B - Embryo, C - Scutellum, (i) Storing reserve food material
D - Coleoptile, E - Coleorhiza (ii) Synthesis of plant growth regulators.
(d) A - Embryo, B - Endosperm, C - Scutellum, (iii) Absorption of water and minerals from the soil.
D - Coleorhiza, E - Coleoptile (iv) Providing a proper anchorage to the plant parts
83. Identify the correct families of the given plant species
(A, B and C) 1

2
3
4

5
(A) (B) (C)
6
(a) A - Liliaceae, B - Compositae, C - Malvaceae
(b) A - Fabaceae, B - Solanaceae, C - Liliaceae (a) 5 and 6 (b) 1, 2 and 6
(c) A - Compositae, B - Malvaceae, C - Liliaceae (c) 1, 2, 3 and 4 (d) 2, 4, 5 and 6
(d) A - Solanaceae, B - Fabaceae, C - Liliaceae
84. The given figure (A, B, and C) shows different types of CRITICAL THINKING TYPE QUESTIONS
roots. Identify the root which is seen in wheat plant and
86. Floral features are mainly used in angiosperms
originate from the base of the stem?
identification because
(a) flowers are of various colours.
(b) flowers can be safely pressed.
(c) reproductive parts are more stable and conservative
than vegetative parts.
Main root
(d) flowers are good materials for identification.
87. Aleurone layer helps in
(a) storage of food in endosperm.
(b) protection of embryo.
Laterals (c) utilization of stored food.
(d) all of the above.
88. Fibrous root system is better adopted than tap root system
for
(a) transport of organic matter.
(A) (B) (b) absorption of water and minerals.
EBD_7209
46 Biology
(c) storage of food. 98. In flower (X), the gynoecium occupies the highest position
(d) anchorage of plant to soil. while the other parts are situated below it. The ovary in
89. Main function of leaf is such flowers is said to be Y. Identify X and Y and select
(a) exchange of gases the correct option.
(b) increase the beauty of a tree X Y
(c) manufacturing of food (a) Epigynous Inferior
(d) nerve impulse induction (b) Perigynous Superior
90. Rearrange the following zones seen in the regions of (c) Hypogynous Superior
root tip and choose the correct option. (d) Perigynous Half-inferior
(A) Root hair zone (B) Zone of meristems 99. Which of the following is not the characteristic features
(C) Root cap zone (D) Zone of maturation of fabaceae?
(E) Zone of elongation (a) Tap root system, compound leaves and raceme
(a) C, B, E, A, D (b) A, B, C, D, E inflorescence.
(c) D, E, A, C, B (d) E, D, C, B, A (b) Flowers actinomorphic, twisted aestivation and
91. Which is not a stem modification ? gamopetalous.
(a) Rhizome of ginger (b) Corm of Colocasia
(c) Stamens 10, introrse, basifixed, dithecous.
(c) Pitcher of Nepenthes (d) Tuber of potato
(d) Monocarpellary, ovary superior and bent stigma.
92. Most prominent function of inflorescence is
(a) dispersal of seeds. 100. How many plants in the list given below have marginal
(b) formation of more fruits. placentation?
(c) formation of pollen grains. Mustard, Gram, Tulip, Asparagus, Arhar, Sunhemp,
(d) dispersal of pollens. Chilli, Colchicine, Onion, Moong, Pea, Tobacco, Lupin
93. The character of flower which is represented by floral (a) Four (b) Five
formula but not by floral diagram is (c) Six (d) Three
(a) aestivation (b) placentation 101. Ginger is an underground stem. It is distinguished from
(c) position of gynoecium (d) adhesion of stamen root because
94. The mature seeds of plants such as gram and peas possess (a) it lacks chlorophyll.
no endosperm because
(b) it stores food.
(a) these plants are not angiosperms.
(b) there is no double fertilization in them. (c) it has nodes and internodes.
(c) endosperm is not formed in them. (d) it has xylem and vessels.
(d) endosperm gets used up by the developing embryo 102. Which one of the following is correct explanation for
during seed development. the given floral formula ?
95. Seeds are regarded as products of sexual reproduction %O K(5) C1+2+(2) A(9)+1 G1
+
because they
(a) can be stored for a long time. (a) Zygomorphic, bisexual, sepals five and
(b) are result of fusion of pollen tube. gamosepalous, petals five and papilionaceous,
(c) are result of fusion of gametes. anthers ten and monadelphous, ovary superior and
monocarpellary.
(d) give rise to new plants.
96. Which of the following is correct with reference to floral (b) Zygomorphic, unisexual, sepals five and
gamosepalous, petals five and polypetalous, anthers
character of the family solanaceae?
nine united and one free, ovary superior and
(a) Racemose, zygomorphic, unisexual, floral characters
monocarpellary.
(b) Racemose, zygomorphic, bisexual, polypetalous
(c) Zygomorphic, bisexual, sepals five and
(c) Axillary, bisexual, actinomorphic, epipetalous
gamosepalous, petals five and papilionaceous,
(d) Axillary, actinomorphic, bisexual, epipetalous
anthers ten and diadelphous, ovary superior and
97. Which of the following represents the floral characters
monocarpellary.
of liliaceae?
(d) Zygomorphic, bisexual, sepals five and united,
(a) Six tepals, zygomorphic, six stamens, bilocular ovary,
petals five and united, anthers ten and diadelphous,
axile placentation.
ovary superior and monocarpellary.
(b) Actinomorphic, polyphyllous, unilocular ovary, axile
103. The main function(s) of root system is/are
placentation.
(c) Tricorpellary, actinomorphic, polyandrous, superior (a) absorption of water and minerals from the soil
ovary, axile placentation. (b) storing reserve food material
(d) Bisexual, zygomorphic, gamophyllous, inferior ovary, (c) synthesis of plant growth regulators
marginal placentation. (d) All of the above
Morphology of Flowering Plants 47

104. The region of the root-tip whose cells undergo rapid X Y Z


elongation and enlargement and are responsible for the (a) Scutellum Embryo Radicle
growth of the root in length is called the (b) Embryo Scutellum Radicle
(a) region of maturation. (c) Scutellum Radicle Embryo
(b) region of elongation. (d) Radicle Embryo Scutellum
107. "X" is the outermost whorl of the flower and contains
(c) region of meristematic activity.
"Y". Y is green, leaf like and protect the other whorls of
(d) root hairs.
the flower.
105. The region of the root-tip which is involved in the formation Identify X and Y.
of root hairs by epidermal cells is called the (a) X - Calyx; Y - Sepals
(a) region of maturation. (b) X - Corolla; Y - Petals
(b) region of meristematic activity. (c) X - Gynoecium; Y - Fruit
(c) region of elongation. (d) X - Androecium; Y - Ovary
(d) none of the above. 108. It is a proteinous layer and the outer covering of
106. The X is small and situated in a groove at one end of the endosperm which separates the embryo. Identify the
endosperm. It consists of one large and shield shaped layer.
cotyledon known as Y and a short axis with a plumule and (a) Tegmen
a Z. Identify X, Y and Z. (b) Scutellum
(c) Hyaline layer
(d) Aleurone layer
EBD_7209
Anatomy of Flowering
6

Chapter
Plants

9. A plant tissue when stained showed the presence of


FACT/DEFINITION TYPE QUESTIONS
hemicellulose and pectin in cells wall of its cells. The tissue
1. A tissue is a group of cells which are is called
(a) similar in origin, but dissimilar in form and function. (a) collenchyma (b) sclerenchyma
(b) dissimilar in origin, form and function. (c) xylem (d) meristem
(c) dissimilar in origin, but similar in form and function. 10. Various functions like photosynthesis, storage, excretion
(d) similar in origin, form and function. performed by _____________.
2. Apical, intercalary and lateral meristems are differentiated (a) sclerenchyma (b) parenchyma
on the basis of (c) collenchyma (d) aerenchyma
(a) origin (b) function 11. Sclerenchyma
(c) position (d) development usually___________and_____________
3. Which of following helps bamboo and grasses to elongate ? protoplasts.
(a) Apical meristems (b) Lateral meristems (a) live, without (b) dead, with
(c) Secondary meristems (d) Intercalary meristems (c) live, with (d) dead, without
4. Which meristem helps in increasing girth? 12. The __________ occurs in layers below the epidermis in
dicotyledonous plants.
(a) Lateral meristem (b) Intercalary meristem
(a) parenchyma (b) sclerenchyma
(c) Primary meristem (d) Apical meristem
(c) collenchyma (d) aerenchyma
5. Cells of permanent tissues are specialized
13. Xylem functions as a conducting tissue for water and
(a) functionally.
minerals from _________to the ______and__________.
(b) only structurally.
(a) roots, stems, leaves (b) stems, roots, leaves
(c) both structurally and functionally.
(c) leaves, stems, roots (d) leaves, stems, leaves
(d) for mitosis. 14. Bast fibres are made up of _____________cells.
6. The apical meristem of the root is present (a) sclerenchymatous (b) chlorenchymatous
(a) in all the roots. (c) parenchymatous (d) aerenchymatous
(b) only in radicals. 15. Which one of the following have vessels as their
(c) only in tap roots. characteristic feature?
(d) only in adventitious roots. (a) Angiosperms (b) Gymnosperms
7. During the formation of leaves and elongation of stem, (c) Pteridophytes (d) Bryophytes
some cells ‘left behind’ from the shoot apical meristem, 16. An organised and differentiated cellular structure having
constitute the cytoplasm but no nucleus is called _________.
(a) lateral meristem (b) axillary bud (a) vessels (b) xylem parenchyma
(c) cork cambium (d) fascicular cambium (c) sieve tubes (d) tracheids
8. Which of the following is responsible for the formation of 17. A vascular bundle in which the protoxylem is pointing to
an embryonic shoot called axillary bud? the periphery is called __________.
(a) Lateral meristem (b) Apical meristem (a) endarch (b) exarch
(c) Intercalary meristem (d) Both (b) and (c) (c) radial (d) closed
Anatomy of Flowering Plants 49

18. In dicot root 28. Cork cambium and vascular cambium are
(a) vascular bundles are scattered and with cambium (a) the parts of secondary xylem and phloem.
(b) vascular bundles are arranged in a ring and have (b) the parts of pericycle.
cambium (c) lateral meristems.
(c) xylem and phloem radially arranged
(d) xylem is always endarch (d) apical meristems.
19. Pericycle of roots produces 29. Phellogen and phellem respectively denote
(a) mechanical support (a) cork and cork cambium,
(b) lateral roots (b) cork cambium and cork,
(c) vascular bundles (c) secondary cortex and cork,
(d) adventitious buds (d) cork and secondary cortex,
20. In stems, the protoxylem lies towards the 30. A narrow layer of thin walled cells found between phloem/
_____________ bark and wood of a dicot is
and the metaxylem lies towards the ____________ of (a) cork cambium (b) vascular cambium
the organ. (c) endodermis (d) both (a) & (c)
(a) centre; periphery 31. Cork is formed from
(b) periphery; centre (a) phellogen (b) vascular cambium
(c) periphery; periphery (c) phloem (d) xylem
(d) centre; centre 32. Main function of lenticel is
21. Anatomically fairly old dicotyledonous root is (a) transpiration (b) guttation
distinguished from the dicotyledonous stem by (c) gaseous exchange (d) both (a) & (c)
(a) presence of cortex.
STATEMENT TYPE QUESTIONS
(b) position of protoxylem.
(c) absence of secondary xylem. 33. Which of the following characteristic is correct about a
(d) absence of secondary phloem. monocot leaf?
22. Monocot leaves possess (a) Having reticulate venation.
(a) intercalary meristem (b) Absence of bulliform cells.
(b) lateral meristem (c) Mesophyll not differentiated into palisade and
(c) apical meristem spongy tissues.
(d) mass meristem (d) Well differentiated mesophyll cells are present.
23. What is true about a monocot leaf ? 34. Which is not correct about sclereids?
(a) Reticulate venation (a) These are parenchyma cells with thickened lignified
(b) Absence of bulliform cells from epidermis walls.
(c) Mesophyll not differentiated into palisade and spongy (b) These are elongated and flexible with tapered ends.
tissues (c) These are commonly found in the shells of nuts and
(d) Well differentiated mesophyll in the pulp of guava, pear, etc.
24. Lignin is the important constituent in the cell wall of (d) These are also called stone cells.
(a) phloem (b) parenchyma 35. Which one of the following option is correct about
(c) xylem (d) cambium bulliform/motor cell ?
25. Heartwood differs from sapwood in: (a) It is seen in grasses.
(a) presence of rays and fibres (b) It is large-sized, thin-walled colourless, vacuolate
(b) absence of vessels and parenchyma cells on the adaxial surface.
(c) It helps in rolling of leaf to minimise water loss when
(c) having dead and non–conducting elements
it is flaccid.
(d) being susceptible to pests and pathogens
(d) All of the above
26. How does autumn wood differ from spring wood ?
(a) Broad vessels & tracheids 36. All the following statements regarding sieve tube elements
are correct except that
(b) Narrow vessels & tracheids
(a) their end walls have perforated sieve plates which
(c) Red colour of xylem
become impregnated with lignin at maturity.
(d) Cambium
(b) they possess peripheral cytoplasm as well as a large
27. Best method to determine the age of tree is to
vacuole.
(a) measure its diameter
(c) distinct proteinaceous inclusions, the P-proteins are
(b) count number of leaves
seen evenly distributed throughout the lumen.
(c) count number of annual rings at base stem
(d) long, slender, tube-like structures arranged in
(d) number of branches longitudinal series.
EBD_7209
50 Biology
37. Which of the following statements is not correct for 42. Read the following statements and answer the questions.
stomatal apparatus? (i) It is made up of elongated, tapering cylindrical cells
(a) Inner walls of guard cells are thick and in elastic. which have dense cytoplasm and nucleus.
(b) Guard cells invariably possess chloroplasts and (ii) The cell wall is composed of cellulose and has pits
mitochondria. through which plasmodesmatal connections exist
(c) Guard cells does not possess subsidiary cells. between the cells.
(d) Stomata are involved in gaseous exchange. (iii) It is absent in most of the monocotyledons.
38. Which of the following statement is correct regarding Which part of plant tissue is being described by the above
simple permanent tissue ? statements?
(a) The collenchyma occurs in layers below the (a) Sieve tube elements (b) Companion cells
epidermis in monocotyledonous plants. (c) Phloem parenchyma (d) Phloem fibres
(b) Sclerenchyma cells are usually dead and without 43. Which of the following statement(s) is/are correct about
protoplasts. the ground tissue system?
(i) All tissues except epidermis and vascular bundles
(c) Xylem parenchyma cells are living and thin walled
and their cell walls are made up of lignin. constitute the ground tissue.
(ii) It consists of xylem and phloem.
(d) The companion cells are specialized
(iii) In leaves, it consists of thin – walled chloroplast
sclerenchymatous cells.
containing cells called mesophyll.
39. Which of the following statement is not correct about
(a) Only (i) (b) Both (i) and (iii)
xylem?
(c) Both (ii) and (iii) (d) All of these
(a) It is a conducting tissue for water and minerals from
44. Read the following statements and answer the question.
roots to the stem and leaves.
(i) They are present on the stem as epidermal hairs.
(b) It also provides mechanical strength to the plants (ii) They are usually multicellular.
parts.
(iii) They may be branched or unbranched and soft or
(c) It is composed of four different kinds of elements, stiff.
namely, tracheids, companion cells, xylem fibres and (iv) They help in preventing water loss due to
xylem parenchyma. transpiration.
(d) Gymnosperms lack vessels in their xylem. Which part of epidermal tissue system is being described
40. Which of the following statement(s) is/are correct about by the above statements?
epidermal tissue system? (a) Stomata (b) Guard cells
(a) It forms the outer-most covering of the whole plant (c) Epidermis (d) Trichomes
body and comprises epidermal cells, stomata and the 45. Which of the following statements are correct about
epidermal appendages - the trichomes and hairs. phloem?
(b) Epidermal cells are parenchymatous with a small (i) Phloem transports food materials, usually from roots
amount of cytoplasm lining the cell wall and a large to the other parts of the plant.
vacuole. (ii) It is composed of sieve tube elements, companion
(c) Epidermis is often covered with a waxy thick layer cells, phloem parenchyma and phloem fibres.
called the cuticle which prevents the loss of water. (iii) The companion cells are specialised parenchymatous
(d) All of the above cells which are closely associated with phloem
41. Read the following statements and select the correct one(s). parenchyma.
(i) In flowering plants, tracheids and vessels are the (iv) The first formed primary phloem consists of narrow
main water transporting elements. sieve tubes and referred to as protophloem and the
(ii) The presence of vessels is a characteristic feature of later formed phloem has bigger sieve tubes and
angiosperms. referred to as metaphloem.
(iii) Xylem fibres have highly thinned walls and their cell (a) Both (i) and (iv) (b) Both (ii) and (iii)
walls are made up of cellulose. (c) Both (ii) and (iv) (d) All of these
(iv) Xylem parenchyma store food materials in the form 46. Which anatomy of plants is being described by the
of starch or fat and other substances like tannins. statements given below ?
Which of the above statement(s) is/are correct? (i) The cortex consists of several layers of thin-walled
(a) Only (i) (b) Both (ii) and (iii) parenchyma cells with intercellular spaces.
(c) Both (iii) and (iv) (d) (i), (ii) and (iv)
Anatomy of Flowering Plants 51

(ii) The tangential as well as radial walls of the 51. Which of the following statement(s) is/are correct ?
endodermal cells have a depostion of water- (i) Uneven thickening of cell wall is characteristic of
impermeable, waxy material -suberin- in form of sclerenchyma.
casparian strips. (ii) Periblem forms cortex of the stem and the root.
(iii) Secondary growth takes place. (iii) Tracheids are the chief water transporting elements
(iv) Pith is small or inconspicuous. in gymnosperms.
(a) Dicotyledonous root (iv) Companion cell is devoid of nucleus at maturity.
(b) Monocotyledonous root
(v) The commercial cork is obtained from Quercus suber.
(c) Dicotyledonous stem
(a) (i) and (iv) only (b) (ii) and (v) only
(d) Monocotyledonous stem
47. Which of the following statement(s) is/are correct about (c) (iii) and (iv) only (d) (ii), (iii) and (v) only
dorsiventral (dicotyledonous) leaf ? 52. Which of the following statements are correct ?
(i) The adaxial (upper surface) bears more stomata than (i) Xylem transports water and minerals.
the abaxial (lower surface) epidermis. (ii) Gymnosperms lack sieve tubes and companion cells
(ii) Mesophyll, which possesses chloroplasts and carry in phloem.
out photosynthesis, is made up of parenchyma. (iii) The first formed primary xylem is called metaxylem.
(iii) Mesophyll is not differentiated into palisade and (iv) Phloem fibres (bast fibres) are made up of
spongy parenchyma. collenchymatous cells.
(a) Both (i) and (iii) (b) Only (ii) (a) (i) and (iii)
(c) Only (iii) (d) All of these (b) (i) and (ii)
48. Which of the following statement is correct about heart (c) (iii) and (iv)
wood/duramen ? (d) (i) and (iv)
(i) It does not help in water and mineral conduction. 53. Which type of plant tissue is being described by the given
(ii) It is dark coloured but soft. statements?
(iii) It has tracheary elements filled with tannins, resins, (i) It consists of long, narrow cells with thick and lignified
gums, oil, etc. cell walls having a few or numerous pits.
(iv) It is a peripheral part. (ii) They are dead and without protoplasts.
(v) They are sensitive to microbes and insects, hence (iii) On the basis of variation in form, structure, origin
least durable. and development, it may be either fibres or sclereids.
(a) (i) and (iii) (c) (ii) and (iii) (iv) It provides mechanical support to organs.
(b) (iv) and (v) (d) (iii) and (iv) (a) Parenchyma (b) Sclerenchyma
49. Read the following statements and answer the question. (b) Collenchyma (d) Chlorenchyma
(i) Cambium is very active and produces a large number 54. Which of the following statement(s) is/are not correct?
of xylary elements having vessels with wider cavities.
(i) Cork cambium is also called phellogen.
(ii) It is also called early wood.
(iii) It is lighter in colour and has lower density.
(ii) Cork is also called phellem.
Which type of wood is described by the above statements?
(iii) Secondary cortex is also called periderm.
(a) Sap wood (b) Heart wood
(c) Spring wood (d) Autumn wood
(iv) Cork cambium, cork and secondary cortex are
50. Read the following statements and answer the question.
collectively called phelloderm.
(i) It has a sclerenchymatous hypodermis, a large
number of scattered vascular bundles and a large (a) (iii) and (iv) (b) (i) and (ii)
parenchymatous ground tissue. (c) (ii) and (iii) (d) (ii) and (iv)
(ii) Vascular bundles are conjoint and closed. 55. Which one of the following statement is incorrect ?
(iii) Peripheral vascular bundles are generally smaller than (i) Epidermal cell has small amount of cytoplasm and a
the centrally located ones. large vacuole.
(iv) Phloem parenchyma is absent, and water- containing (ii) Waxy cuticle layer is absent in roots.
cavities are present within the vascular bundles. (iii) Root hairs are unicellular, while stem hairs / trichomes
Which plant anatomy is being described by the above are multicellular.
statements? (iv) Trichomes may be branched or unbranched, soft or
(a) Dicotyledonous root stiff and prevent transpiration.
(b) Monocotyledonous root (v) Guard cells are dumbell shaped in dicots and bean-
(c) Dicotyledonous stem shaped in monocots (e.g. grass).
(d) Monocotyledonous stem
EBD_7209
52 Biology
(a) Only (i) (b) Only (iv) MATCHING TYPE QUESTIONS
(c) Only (iii) (d) Only (v)
56. Which of the following statements is correct? 64. Match the elements of xylem given in column I with their
(a) Lenticels occur in most woody trees. character given in the column II and choose the correct
(b) Sclerenchymatous cells are usually present in cortex. option.
(c) The vascular tissue system is divided into three main Column-I Column-II
zones- cortex, pericycle and pith. A. Xylem vessels I. Store food materials
(d) The conjoint vascular bundles usually have the B. Xylem tracheids II. Obliterated lumen
xylem located only on the outer side of the phloem. C. Xylem fibres III. Perforated plates
57. Which of the following pair of match is not correct? D. Xylem parenchyma IV. Chisel-like ends
(a) Pith - Large and well developed in (a) A – IV, B – III, C – II, D – I
monocotyledonous root. (b) A – III, B – II,C – I, D – IV
(b) Root hairs - Helps in preventing water loss due to (c) A – II, B – I, C – IV, D – III
transpiration (d) A – III, B – IV, C – II, D – I
(c) Sieve tube elements - Its functions are controlled by 65. Match column-I with column-II and choose the correct
the nucleus of companion cells. option.
(d) Stomatal apparatus - Consists of stomatal aperture, Column -I Column -II
guard cells and surrounding subsidiary cells A. Bulliform cells I. Initiation of lateral roots
B. Pericycle II. Root
ASSERTION/REASON TYPE QUESTIONS C. Endarch xylem III. Grasses
In the following questions, a statement of Assertion is followed D. Exarch xylem IV. Dicot leaf
by a statement of Reason. E. Bundle sheath cells V. Stem
(a) If both Assertion and Reason are true and the Reason is (a) A – III, B – V, C – IV, D – I, E – II
the correct explanation of the Assertion. (b) A – II, B – V, C – I, D – III, E – IV
(b) If both Assertion and Reason are true but the Reason is (c) A – II, B – IV, C – I, D – III, E – V
not the correct explanation of the Assertion. (d) A – III, B – I, C – V, D – II, E – IV
(c) If Assertion is true but Reason is false. 66. Match the terms given in column I with their funciton
(d) If both Assertion and Reason are false. given in column II and choose the correct option.
58. Assertion: Cambium is a lateral meristem and cause growth Column-I Column-II
in width. (Term) (Functions)
Reason: Cambium is made up of fusiform and ray initials A. Meristem I. Photosynthesis, storage
in stem. B. Parenchyma II. Mechanical support
59. Assertion : Higher plants have meristematic regions for C. Collenchyma III. Actively dividing cells
indefinite growth. D. Sclerenchyma IV. Stomata
Reason : Higher plants have root and shoot apices.\ E. Epidermal tissue V. Sclereids
(a) A – I, B – III, C – V, D – II, E – IV
60. Assertion : In woody stems, the amount of heart wood
(b) A – III, B – I, C – II, D – V, E – IV
continues to increase year after year.
(c) A – II, B – IV, C – V, D – I, E – III
Reason : The cambial activity continues uninterrupted. (d) A – V, B – IV, C – III, D – II, E – I
61. Assertion : Vessels are more efficient for water 67. Match the followings and choose the correct option
conduction as compared to tracheids. Column-I Column-II
Reason : Vessels are dead and lignified. A. Cuticle I. Guard cells
62. Assertion: Bulliform cells are useful in the unrolling of B. Bulliform cells II. Outer layer
leaf. C. Stomata III. Waxy layer
D. Epidermis IV. Empty colourless cell
Reason: Bulliform leaves store water.
(a) A – III, B – IV, C – I, D – II
63. Assertion : Long distance flow of photoassimilates in plants (b) A – I, B – II, C – III, D – IV
occurs through sieve tubes. (c) A – III, B – II, C – IV, D – I
Reason : Mature sieve tubes have parietal cytoplasm and (d) A – III, B – II, C – I, D – IV
perforated sieve plates. 68. Match the names of the structures given in column-I with
the functions given in column-II, choose the answer which
gives the correct combination of the two columns :
Anatomy of Flowering Plants 53

Column-I Column-II and pointed cells, generally


(Structure) (Function) occurring in groups
A. Stomata I. Protection of stem D. Vessels IV. Long cylindrical tube like
B. Bark II. Plant movement structure and cells are
C. Cambium III. Secondary growth devoid of protoplasm.
D. Cuticle IV. Transpiration Characteristic feature of
V. Prevent the loss of water angiosperms
E. Xylem parenchyma V. Reduced form of
(a) A – V, B – III, C – I, D – IV
sclerenchyma cells with
(b) A – I, B – IV, C – V, D – III
highly thickened lignified
(c) A – II, B – IV, C – I, D – III cellular walls that form small
(d) A – IV, B – I, C – III, D – V bundles of durable layers of
69. Match column-I with column-II and choose the correct tissue in most plants.
option. (a) A – I, B – II, C – III, D – IV, E – V
Column-I Column-II
(b) A – III, B – V, C – II, D – IV, E – I
A. Spring wood or I. Lighter in colour
early wood (c) A – III, B – I, C – V, D – II, E – IV
B. Autumn wood or II. High density (d) A – V, B – IV, C – III, D – I, E – II
late wood III. Low density 72. In the given columns, column I contain structures of female
IV. Darker in colour reproductive system and column II contain its feature.
V. Larger number of xylem Select the correct match.
elements Column-I Column-II
VI. Vessels with wider cavity A. Lateral meristem I. Fascicular vascular
VII. Lesser number of xylem cambium, interfascicular
elements cambium and cork
VIII. Vessels with small cavity cambium.
Which of the following combination is correct ? B. Apical meristem II. Produces dermal tissue,
ground tissues and
(a) A – II, IV, VII, VIII; B – I, III, V, VI
vascular tissue.
(b) A – I, II, VII, VIII; B – III, IV, V, VI
C. Bast fibres III. Generally absent in primary
(c) A – I, III, V, VI; B – II, IV, VII, VIII phloem but found in
(d) A – I, III, VII, VIII; B – II, IV, V, VI secondary phloem.
70. Match column-I with column-II and select the correct D. Sap wood IV. Involved in the conduction
option from the codes given below. of water and minerals from
Column-I Column-II the root to leaf.
A. Stele I. Innermost layer of cortex (a) A – I, B – II, C – III, D – IV
B. Endodermis II. Suberin (b) A – III, B – I, C – II, D – IV
C. Casparian strips III. All the tissues exterior to (c) A – I, B – IV, C – III, D – II
vascular cambium (d) A – II, B – IV, C – III, D – I
D. Bark IV. All the tissues inner to
endodermis DIAGRAM TYPE QUESTIONS
(a) A – IV, B – I, C – II, D – III
73. The given figure shows apical meristem of root apex with
(b) A – III, B – II, C – I, D – IV
few part marked as A, B and C. Identify the correct labelling
(c) A – I, B – II, C – III, D – IV
of A, B and C.
(d) A – IV, B – II, C – I, D – III
71. Match the terms given in column I with their features
given in column II and choose the correct option.
Column-I Column-II
(Terms) (Features)
A. Fibres I. Cells are living and thin
walled with cellulosic cell
wall, store food materials in
the form of starch or fat
B. Sclereids II. Main water conductive cells
of the pteridophytes and
the gymnosperms
C. Tracheids III. Thick walled, elongated
EBD_7209
54 Biology
(a) A – Vascular structure, B – Protoderm, C – Root cap (a) A – Parenchyma, Photosynthesis, Storage and
(b) A – Cortex, B – Endodermis, C – Root cap Secretion.
(c) A – Cortex, B – Protoderm, C – Root cap (b) B – Sclerenchyma Scleriods; Transport food material
(d) A – Tunica, B – Protoderm, C – Root cap
(c) C – Collenchyma; Provides mechanical support to
74. Identify A, B and C in the given figure of shoot apical
organs.
meristem
(d) D – Sclerenchyma Fibres; Provide Mechanical
support to the growing parts of the plant such as
young stem and petiole of a leaf.
76. The given figures are types of elements (A and B) which
constitute one type of complex tissue (c) of a plant . Identify
A, B and C.
A C
B

(a) A – Leaf primordium, B – Shoot apical meristem,


C – Axillary bud
(b) A – Leaf primordium, B – Shoot apical meristem,
C – Apical bud
(c) A – Root hair primordium, B – Root apical meristem, (a) A – Tracheid, B – Vessel, C – Xylem
C – Axillary bud (b) A – Vessel, B – Tracheild, C – Phloem
(d) A – Root hair primordium, B – Root apical meristem, (c) A – Fibre, B – Tracheid, C – Bark
C – Terminal bud (d) A – Fibre, B – Sclereid, C – Casparian strips
77. In the given figure of phloem tissue, identify the marked
75. Identify the types of simple tissue indicated by A, B,
part (A, B and C) which help in maintaining the pressure
C and D and their function. gradient in the sieve tubes.

(A) (B)

(a) A (b) B
(c) C (d) None of the above
78. The given diagrams show stomatal apparatus in dicots
and monocots. Which one is correct option for A, B and C?

(C) (D)
Anatomy of Flowering Plants 55

(a) A – Root hair, B – Epiblema, C – Cortex,


D – Endodermis, E – Passage cell, F – Pericycle,
G – Pith, H – Phloem, I – Metaxylem.
(b) A – Root hair, B – Epiblema, C – Cortex,
D – Endodermis, E – Passage cell, F – Pith,
G – Pericycle, H – Metaxylem, I – Phloem.
(c) A – Root hair, B – Epiblema, C – Cortex,
D – Endodermis, E – Pericycle, F – Phloem,
(a) A – Epidermal cells; B – Subsidiary cells; C – G – Protoxylem, I – Metaxylem
chloroplast (d) A – Root hair, B – Cortex, C – Epiblema,
(b) A – Guard cells; B – Subsidiary cells; C – Stomatal D – Pericycle, E – Endodermis, F – Pith, G – Phloem,
pore H – Protoxylem, I – Metaxylem
(c) A – Guard cells; B – Epidermal cells; C – Guard cells 81. The given figure shows the T.S of dicot root. Some parts
(d) A – Epidermal cells; B – Subsidiary cells; C – Guard are marked as A, B, C, D, E, & F. Choose the option which
cells shows the correct labelling of marked part.
79. Identify types of vascular bundles in given figures A, B and C.
E
B

(A)
(B)
D
F

(C)
(a) A – Epiblema, B – Root hair, C – Cortex,
A B C D – Endodermis, E – Pith, F – Pericycle
(a) Rad ial; Co njo int clos ed; Con jo int op en (b) A – Cortex, B – Pith, C – Epiblema, D – Endodermis,
(b) Con jo int clos ed; Co njo int op en; Rad ial E – Root hair, F – Pericycle
(c) Con jo int op en; Co njo int clos ed; Rad ial (c) A – Epiblema, B – Endodermis, C – Cortex, D – Root
hair, E – Pith, F – Pericycle
(d) Bico llateral; Co ncen tric; Rad ial
(d) A – Cortex, B – Epiblema, C – Pith, D – Endodermis,
80. Choose the correct labelling of (A – J) in the given figure E – Root hair, F – Pericycle
of T.S. of monocot root. 82. T.S. of dicot stem is given below, certain parts have been
A marked by alphabets (A – I). Choose the option which
shows their correct labelling.
B A
B
C
C
D D
E
E
F F
G
G

H
H
I

I
EBD_7209
56 Biology
(a) A – Epidermis, B – Epidermal hair, C – Parenchyma,
D – Starch sheath, E – Hypodermis (collenchyma),
F – Vascular bundle, G – Bundle cap, H – Medulla or
pith, I – Medullary rays
(b) A – Epidermal hair, B – Epidermis, C – Hypodermis
(collenchyma), D – Parenchyma, E – Endoderm is
(Starch Sheath), F – Pericycle, G – Vascular bundle,
H – Medullary rays, I – Medulla or pith
(c) A – Epidermal hair, B – Epidermis, C – Hypodermis
(collenchyma), D – Starch sheath, E – Parenchyma,
F – Vascular bundle, G – Bundle cap, H – Medulla or
pith, I – Medullary rays
(d) A – Epidermal hair, B – Epidermis, C – Parenchyma, (a) A–Epidermis, B–Spongy mesophyll,
D – Hypodermis (collenchyma), E – Starch sheath, C– Palisade mesophyll, D – Stomata, E– Guard cells,
F – Vascular bundle, G – Bundle cap, H – Medulla or F–Phloem, G–Metaxylem, H–Protoxylem
pith, I – Medullary rays (b) A–Epidermis, B–Palisade mesophyll, C– Spongy
83. The given figure shows T.S. of monocot stem. Identify mesophyll, D–Sub-stomatal cavity, E–Stoma,
the correct labelling of A to F marked in the given figure. F–Phloem, G– Xylem, H– Bundle sheath
(c) A – Epidermis, B – Palisade mesophyll,
C–Spongy mesophyll, D–Stomata, E– Guard cells,
F–Epidermis, G– Xylem, H–Phloem
(d) A–Epidermis, C–Palisade mesophyll,
C–Spongy mesophyll, D– Stomata, E– Guard cells,
F–Phloem, G– Metaxylem, H– Protoxylem
85. T.S. of monocot leaf is given below, certain parts have
been marked by alphabets (A – G). Which one is the option
showing there correct labelling?
A

B
C

D
(a) A – Epidermis, B – Hypodermis, C – Vascular
E
bundles, D – Phloem, E – Xylem, F – Ground tissue
(b) A – Cuticle, B – Epidermis, C – Sclerenchymatous F
G
sheath, D – Sclerenchymatous hypodermis, (a) A – Adaxial epidermis, B – Xylem, C – Mesophyll,
E – Parenchymatous sheath, F – Phloem D – Sub-stomatal cavity, E – Abaxial epidermis,
(c) A – Cuticle, B – Epidermis, C – Sclerenchymatous F – Stoma, G – Phloem
(b) A – Adaxial epidermis, B – Abaxial epidermis,
hypodermis, D – Sclerenchymatous sheath,
C – Xylem, D – Sub-stomatal cavity, E – Stoma,
E – Parenchymatous sheath, F – Phloem
F – Mesophyll, G – Phloem
(d) A – Cuticle, B – Epidermis, C – Sclerenchymatous (c) A – Adaxial epidermis, B – Phloem, C – Mesophyll,
hypodermis, D – Sclerenchymatous sheath, D – Sub-stomatal cavity, E – Abaxial epidermis,
E – Parenchymatous sheath, F – Protoxylem F – Xylem, G – Stoma
84. T.S. of dicot leaf passing through the midrib is given below. (d) A – Adaxial epidermis, B – Xylem, C – Stoma,
Certain parts have been marked by alphabets (A to H). D – Sub-stomatal cavity, E – Abaxial epidermis,
Choose the option showing their correct labelling. F – Phloem, G – Mesophyll
Anatomy of Flowering Plants 57

86. The given figure shows the secondary growth in a dicot 88. Which one of the following option shows the correct
stem. Their parts are marked as A, B, C, D, E & F. Choose labelling of the parts marked as A, B, C and D in the given
the correct labelling of the parts marked as A to F. figure of a typical dicot root?

E
(a) A – Primary phloem, B – Vascular cambium,
F
C – Secondary phloem, D – Primary xylem
(a) A – Phellem, B – Phellogen, C – Medullary rays,
(b) A – Secondary phloem, B – Vascular cambium,
D – Secondary xylem, E – Secondary phloem,
F – Cambium ring C – Primary phloem, D – Primary xylem
(b) A – Phellem, B – Phellogen, C – Medullary rays, (c) A – Primary phloem, B – Primary xylem,
D – Secondary phloem, E – Secondary xylem, C – Secondary phloem, D – Vascular cambium
F – Cambium ring (d) A – Secondary phloem, B – Primary xylem,
(c) A – Phellogen, B – Phellem, C – Medullary rays, C – Primary phloem, D – Vascular cambium
D – Secondary xylem, E – Secondary phloem,
F – Cambium ring 89. Which of the following figure is a type of permanent tissue
(d) A – Phellem, B – Phellogen, C – Cambium ring, having many different types of cell?
D – Secondary xylem, E – Secondary phloem,
F – Medullary rays
87. Which one of the followings option shows the correct
labelling of the parts marked as A, B, C and D in the given
figure of a lenticel?

(a)

Irregularly Thickened Primary Cell Wall


Collenchyma

(a) A – Epidermis, B – Secondary cortex, C – Cork


cambium, D – Cork Sclerenchyma
(b) Tissue
(b) A – Pore, B – Cork cambium, C – Secondary cortex,
D – Cork
(c) A – Pore, B – Cork, C – Complimentary cells,
D – Cork cambium
(d) A – Epidermis, B – Complimentary cells, C – Cork
cambium, D – Secondary cortex
EBD_7209
58 Biology
95. As secondary growth proceeds, in a dicot stem, the
Trachea types by cell wall
perforation plate thickness of
thickening
(a) sapwood increases.
(b) heartwood increase.
(c) both sapwood and heartwood increases.
(c) (d) both sapwood and heartwood remains the same.
96. The trees growing in desert will
trachea (a) show alternate rings of xylem and sclerenchyma.
(b) have only conjunctive tissue and phloem is formed
tracheid
annular spiral reticulate by the activity of cambium.
libriform fiber
(c) show distinct annual rings.
(d) not show distinct annual rings.
97. Some vascular bundles are described as open because
these
(a) are surrounded by pericycle but not endodermis.
(d) (b) are capable of producing secondary xylem and
phloem.
(c) possess conjunctive tissue between xylem and
Parenchyma phloem.
(d) are not surrounded by pericycle.
CRITICAL THINKING TYPE QUESTIONS 98. Apical and intercalary meristems are primary meristems
90. Trees at sea do not have annual rings because because
(a) soil is sandy. (a) they occur in the mature region of roots and shoots
(b) there is climatic variation. of many plants.
(c) there is no marked climatic variation. (b) they made up of different kinds of tissues.
(d) there is enough moisture in the atmosphere. (c) they involved in secondary growth.
91. One of the primary function of the ground tissue in a (d) they appear early in life of a plant and contribute to
plant is the formation of the primary plant body.
(a) photosynthesis. 99. Sieve tubes are suited for translocation of food because
they possess
(b) to protect the plant.
(a) bordered pits.
(c) to anchor the plant.
(b) no ends walls.
(d) water and sugar conduction.
(c) broader lumen and perforated cross walls.
92. When we peel the skin of a potato tuber, we remove
(d) no protoplasm.
(a) periderm (b) epidermis
100. A piece of wood having no vessels (trachea) must be
(c) cuticle (d) leaves
belonged to
93. Why grafting is successful in dicots ?
(a) teak (b) mango
(a) In dicots vascular bundles are arranged in a ring.
(c) pine (d) palm
(b) Dicots have cambium for secondary growth. 101. In land plants, the guard cells differ from other epidermal
(c) In dicots vessels with elements are arranged end to end. cells in having
(d) Cork cambium is present in dicots (a) cytoskeleton. (b) mitochondria.
94. The length of different internodes in a culm of sugarcane (c) endoplasmic reticulum. (d) chloroplasts.
is variable because of 102. Cambium is considered as a lateral meristem because
(a) size of leaf lamina at the node below each internode (a) it gives rise to lateral branches.
(b) intercalary meristem (b) it causes increase in girth.
(c) shoot apical meristem (c) it increases height and diameter of a plant.
(d) position of axillary buds (d) it adds bulk to a plant.
Anatomy of Flowering Plants 59

103. Tissue(s) present in an annual ring is/are 109. A student was given a tissue to observe under the
(a) secondary xylem and phloem. microscope. He observes the tissue and concludes that
(b) primary xylem and phloem. the tissue is a type of simple plant tissue and provides
mechanical support to young stem and petiole of leaf.
(c) secondary xylem only.
Identify the tissue.
(d) primary phloem and secondary xylem.
(a) Parenchyma (b) Collenchyma
104. The vessel elements of angiosperms differ from other
(c) Sclerenchyma (d) Xylem parenchyma
elements of xylem in having
110. In an experiment, a student cut a transverse section of
(a) simple pits on their radial walls.
young stem of a plant which he has taken from his school
(b) bordered pits on their lateral walls.
garden. After observing it under the microscope how would
(c) simple and bordered pits on their end walls. he ascertain whether it is a monocot stem or a dicot stem?
(d) simple perforation on their end walls. (a) With the help of bulliform cells.
105. Which of the following are present in monocot root ?
(b) With the help of casparian strips.
(a) conjoint, collateral, open polyarch vascular bundle.
(c) With the help of vascular bundles.
(b) exodermis, endarch, tetrarch closed vascular bundles.
(d) With the help of stomatal apparatus.
(c) suberized exodermis, casparian strip, passage cells,
cambium. 111. Which of the following process helps the trichomes in
(d) suberized exodermis, polyarch xylem, pith. preventing water loss?
106. Tissues are classified into two main groups, namely (a) Where companion cells helps in maintaining the
meristematic and permanent tissues on the basis of pressure gradient in the sieve tubes.
(a) whether the cells being able to divide or not. (b) Where plants absorb water through the roots and then
(b) position of the cells. give off water vapor through pores in their leaves.
(c) whether they are living or dead. (c) Where activity of cork cambium builds pressure on
(d) none of the above the remaining layers peripheral to phellogen and
107. A common structural feature of vessel elements and sieve ultimately these layers dies and slough off.
tube elements are (d) None of the above
(a) pores on lateral walls. (b) presence of p-protein.
(c) enucleate condition. (d) thick secondary walls.
108. Gymnosperms are also called soft wood spermatophytes
because they lack
(a) cambium (b) phloem fibres
(c) thick-walled tracheids (d) xylem fibres
EBD_7209
Structural Organisation in
7

Chapter
Animals

FACT/DEFINITION TYPE QUESTIONS 9. The chondrocytes of connective tissue are


1. The kind of epithelium which forms the inner walls of (a) fibre secreting cells (b) bone forming cells
blood vessels is (c) cartilage cells (d) bone eating cells
(a) cuboidal epithelium 10. The shape of a person’s ear is mainly due to
(b) columnar epithelium (a) dense regular connective tissue
(c) ciliated columnar epithelium (b) dense irregular connective tissue
(d) squamous epithelium (c) elastic cartilage
2. The ciliated columnar epithelial cells in humans occur in (d) fibrocartilage
(a) Eustachian tube and stomach lining 11. Tendons and ligaments are the examples of
(b) bronchioles and fallopian tubes (a) areolar connective tissue
(c) bile duct and oesophagus (b) adipose tissue
(d) fallopian tubes and urethra (c) dense regular connective tissue
3. Mast cells are associated with (d) loose connective tissue
(a) exocrine glands 12. Muscle tissue cells are contractile, which means they
(b) endocrine glands (a) are responsible for the production and secretion of
(c) areolar connective tissue enzymes.
(d) neural tissue (b) are specialized in contraction and relaxation.
4. Epithelial tissue is distinguished from connective tissue, (c) help in the movement of involuntary organs only.
muscular, or nervous tissue by its (d) all of the above
(a) large extracellular matrix. 13. In all connective tissues, except which of the following,
(b) contractibility. the cells secrete fibres of collagen or elastin protein ?
(c) ability to carry action potentials. (a) Bone
(d) basement membrane. (b) Cartilage
5. Compound squamous epithelium is found in (c) Areolar connective tissue
(a) stomach (b) intestine (d) Fluid connective tissue
(c) trachea (d) pharynx 14. Three essential components of most neurons are
6. General function of gland is to (a) simple epithelium, extracellular matrix and nerves.
(a) protect the body. (b) axon, dendrites and cell body.
(b) control the function of epithelial tissues. (c) nerve cells, synapse and neuroglia.
(c) produce and secrete specialized compounds to (d) mylein sheeth, node of Ranvier and Schwann cells.
control and maintain different body functions. 15. The supportive skeletal structures in the human external
(d) help to maintain blood pressure and nerve actions. ears and in the nose tip are examples of
7. Which of the following type of cell junction is not found (a) ligament (b) areolar tissue
in animal tissues ?
(c) bone (d) cartilage
(a) Adhering junction (b) Tight junction
16. The fibres of which of the following muscles are fusiform
(c) Gap junction (d) Plasmodesmata and do not show striations
8. The only type of cell seen in a tendon is (a) Skeletal muscles (b) Cardiac muscles
(a) muscle fibres (b) reticular cells
(c) Both (a) and (b) (d) Smooth muscles
(c) collagenous cells (d) fibroblasts
Structural Organisation in Animals 61

17. Intercalated discs are the communication junctions 28. Phallic organs in cockroach are related to
between the cells of (a) male excretory system.
(a) cardiac muscles (b) striped muscles (b) male reproductive system.
(c) adipose tissue (d) nerve and striated (c) female excretory system.
muscles (d) female reproductive system.
18. Smooth muscles are______. 29. Gizzard (proventriculus) in cockroach lies between
(a) voluntary, branched, uninucleate (a) oesophagus and stomach
(b) voluntary, multinucleate, cylindrical (b) crop and mesenteron
(c) involuntary, cylindrical, multinucleate (c) mesenteron and ileum
(d) involuntary, spindle shaped, uninucleated, tapering (d) oesophagus and crop
19. Neuroglia are 30. A frog never drinks water but absorbs it through one of
(a) excitable cells of neural tissue. its respiratory organ. Identify the organ.
(b) supporting and non-excitable cells of neural tissue. (a) Skin (b) Lung
(c) two to three times in volume of neural tissue. (c) Buccal cavity (d) None of the above.
(d) protective and excitable cells of neural tissue. 31. Which one of the following is not a characteristic feature
20. Which one of the following pairs of str uctures of frog?
distinguishe a nerve cell from other types of cell ? (a) The skin is moist and slimy
(a) Vacuoles and Fibres (b) Each of the fore limbs and hind limbs end in five digits
(b) Flagellum and Medullary sheath (c) Hepatic portal and renal portal systems are present
(c) Nucleus and Mitochondria (d) Skin, buccal cavity and lungs are the respiratory
(d) Cell body and Dendrites organs
21. Spermathecae in Pheretima is located in the segments: 32. The sensory papillae in frogs are associated with
(a) 14 to 18 (b) 10 to 13 (a) smell (b) hearing
(c) 6 to 10 (d) 6 to 9 (c) respiration (d) touch
22. In earthworms setae are present in all segments except STATEMENT TYPE QUESTIONS
(a) first and the last segments
(b) first segment and the clitellum 33. Which statement regarding anal cerci and anal style in
(c) first segment cockroach is correct?
(d) clitellum and last segments (a) Anal cerci are absent but anal styles are present in male.
23. About how many times does the nymph of the (b) Anal cerci are absent and anal styles are present in
Periplaneta americana undergo moulting before female.
becoming an adult? (c) Anal styles are absent and anal cerci are present in
(a) 4 (b) 2 male.
(c) 17 (d) 13 (d) Anal styles are absent and anal cerci are present in
24. Cockroaches are brown or black bodied animals that are female.
included in class _______ of phylum _______. 34. Select the correct statement regarding Periplaneta
(a) reptilia; annelida (b) insecta; arthropoda americana
(c) insecta; annelida (d) reptilia; arthropoda (a) There are 16 very long malpighian tubules present at
25. Male cockroach can be identified from the female by the the junctions of midgut and hindgut.
presence of (b) Grinding of food is carried out only by the mouth parts.
(a) long antennae (b) wingless body (c) Nervous system is located ventrally and consists of
(c) elongated abdomen (d) anal styles segmentally arranged ganglia joined by a pair of
26. In cockroach, the testes are present in longitudinal connectives.
(a) 3, 4, 5 abdominal segments (d) Females bear a pair of short thread like anal styles.
(b) 4, 5, 6 abdominal segments 35. Which of the following statement is incorrect regarding
(c) 5, 6, 7 abdominal segments cuboidal epithelium ?
(a) It is an epithelial tissue.
(d) 6, 7, 8 abdominal segments
(b) It is composed of a single layer of cube-like cells.
27. The mouth parts of cockroach are
(c) They are found in the walls of blood vessels and air
(a) cutting and biting type.
sacs of lungs.
(b) piercing and sucking type.
(d) Secretion and absorption are the main functions of
(c) sucking and rasping type.
these tissue.
(d) sucking and siphoning type.
EBD_7209
62 Biology
36. Which of the following statement(s) is/are correct (i) Head is formed by the fusion of 6-segments.
regarding compound epithelium ? (ii) Mouth parts are biting and chewing type.
(a) It is made of more than one layer of cells and thus (iii) Crop is the part of mid gut.
has a limited role in secretion and absorption. (a) Only (ii) is correct
(b) Their main function is to provide protection against (b) (ii) and (iii) are correct
chemical and mechanical stresses. (c) (i) and (ii) are correct
(c) They cover the dry surface of the skin, moist surface (d) Only (iii) is correct
of buccal cavity, pharynx, inner lining of ducts of 42. Read the following statements and answer the question.
salivary glands and pancreatic ducts. (i) It is made of a single thin layer of flattened cells with
(d) All of the above irregular boundaries.
37. Which of the following statement(s) is/are correct about (ii) They are found in the walls of blood vessels and air
muscle tissue ? sacs of lungs.
(a) Each muscle is made of many long, cylindrical fibres (iii) They are involved in functions like forming a diffusion
arranged in parallel arrays. boundary.
(b) Muscle fibres contract (shorten) in response to Which of the following characteristics of tissue is being
stimulation, then relax (lengthen) and return to their described by the above statements ?
uncontracted state in a coordinated fashion. (a) Squamous epithelium (b) Columnar epithelium
(c) Muscles play an active role in all movements of the (c) Ciliated epithelium (d) Compound epithelium
body. 43. Which of the following type of tissue is being described
(d) All of the above by the given statements ?
38. Which of the following statements is not correct regarding (i) They are named because of their special function of
neural tissue ? linking and supporting other tissues/organs of the
(a) It exerts the greatest control over the body’s body.
responsiveness to changing conditions. (ii) They include cartilage, bone, adipose and blood.
(b) Chondrocytes, the unit of neural system are excitable (iii) They provide strength, elasticity and flexibility to
cells. the tissue.
(c) Neuroglial cells protect and support neurons. (iv) They also secrete modified polysaccharides, which
(d) When a neuron is suitably stimulated, an electrical accumulate between cells and fibres and act as matrix.
disturbance is generated. (a) Epithelial tissue (b) Connective tissue
39. Which of the following statement is correct regarding (c) Muscle tissue (d) Neural tissue
Female reproductive system of earthworm ? 44. Which of the following statement (s) is/are correct ?
(a) It consists of two large ovaries, lying laterally in the (i) Loose connective tissue contains fibroblasts,
6th – 7th abdominal segments. macrophages and mast cells.
(b) Each ovary is formed of a group of five ovarian (ii) Adipose tissue is a type of dense connective tissue
tubules or ovar ioles, containing a chain of located mainly beneath the skin.
developing ova. (iii) Tendons and ligaments are examples of dense
(c) A pair of spermatheca is present in the 5th segment irregular connective tissue.
which opens into the genital chamber. (iv) Cartilage, bones and blood are various types of
(d) None of the above specialized connective tissue.
40. Which of the following statement(s) regarding cell (a) Only (i) (b) Both (ii) and (iv)
junctions is/are correct ? (c) Both (i) and (iii) (d) (i), (iii) and (iv)
(a) Tight junctions help to stop substances from leaking 45. Read the following statements and answer the question.
across a tissue. (i) They have a hard and non-pliable ground substance
(b) Adhering junctions perform cementing to keep rich in calcium salts and collagen fibres.
neighbouring cells together. (ii) They support and protect softer tissues and organs.
(c) Gap junctions facilitate the cells to communicate with (iii) Osteocytes are present in the spaces called lacunae.
each other by connecting the cytoplasm of adjoining (iv) They also interact with skeletal muscles attached to
cells, for rapid transfer of ions, small molecules and them to bring about movements.
sometimes big molecules. Which of the following type of tissue is being described
(d) All of the above by above statements ?
41. Consider the following statements regarding cockroach (a) Cartilage (b) Bone
and mark the correct option. (c) Blood (d) Neurons
Structural Organisation in Animals 63

46. Which of the following type of muscle tissue is being (ii) They absorb nitrogenous waste products and
described on the basis of given statements ? convert them into uric acid which is excreted out
(i) These muscle fibres taper at both ends and do not th rough hindgut. Hence, th is insect called
show striations. ammonetelic.
(ii) The wall of internal organs such as the blood vessels, (iii) In addition, fat body, nephrocytes and uricose glands
stomach and intestine contain this type of muscle also help in excretion.
tissue. (a) Only (i) (b) Both (ii) and (iii)
(iii) They are ‘involuntary’ as their function cannot be (c) Both (i) and (iii) (d) All of these
directly controlled. 51. Which of the following statement(s) is/are correct
(a) Skeletal muscle (b) Smooth muscle regarding cockroaches ?
(c) Cardiac muscle (d) All of these (i) The body of the cockroach is segmented and
47. Which of the following statement(s) is/are correct about divisible into three distinct regions – head, thorax
nervous system of cockroach ? and abdomen.
(i) It consists of a series of fused segmentally arranged (ii) Blood vascular system is of closed type.
ganglia joined by paired longitudinal connectives (iii) They are monoecious and both sexes have well
on the ventral side. developed reproductive organs.
(ii) There are six ganglia lie in the thorax, and three in the (iv) The development of P. americana is
abdomen. paurometabolus, meaning there is development
(iii) The sense organs are antennae, eyes, maxillary through nymphal stage.
pulps, labial pulps and anal cerci etc. (a) Both (i) and (ii) (b) Both (ii) and (iii)
(iv) Each eye consists of about 5000 hexagonal ommatidia. (c) Both (i) and (iv) (d) All of these
(a) Both (i) and (iii) (b) Only (ii) 52. Read the following statements.
(c) Both (i) and (iv) (d) All of these (i) It is a contractile tissue present only in the heart.
48. Which of the following statement(s) is/are correct (ii) Cell junctions fuse the plasma membranes of these
regarding respiratory system of cockroach ? cells and make them stick together.
(i) It consists of a network of trachea, that open through (iii) Communication juntions at some fusion points allow
12 pairs of small holes called spiracles present on the cells to contract as a unit, i.e., when one cell
the lateral side of the body. receives a signal to contract, its neighbours are also
(ii) Thin branching tubes carry oxygen from the air to all stimulated to contract.
the parts. Which of the following type of tissue is being described
(iii) The opening of the spiracles is regulated by by the above statements ?
sphincters. (a) Skeletal muscle (b) Cardiac muscle
(iv) Exchange of gases take place at the tracheoles by (c) Smooth muscle (d) Cartilage
diffusion. 53. Which of the follwing statement(s) is/are correct ?
(a) Only (i) (b) (i), (ii) and (iii) (i) Cockroaches are brown or black bodied animals that
(c) (ii), (iii) and (iv) (d) All of these are included in class insecta of phylum arthropoda.
49. Which of the following statement is/are correct regarding (ii) Males bear a pair of short, thread like anal styles
digestive system of cockroach ? which are absent in females.
(i) Mouth opens into a short tubular pharynx, leading (iii) Heart of cockroach consists of elongated muscular
to a narrow tubular passage called oesophagus. tube lying along mid dorsal line of thorax and
(ii) Gizzard helps in grinding the food particles. abdomen.
(iii) A ring of 6-8 blind tubules called hepatic or gastric (iv) The nymph grows by moulting about 13 times to
caecae is present at the junction of foregut and reach the adult form.
midgut, which secrete digestive juice. (a) Only (i) (b) Both (ii) and (iii)
(iv) At the junction of midgut and hindgut is present (c) Both (i) and (ii) (d) All of the above
another ring of 100-150 yellow coloured thin 54. Which of the following statements regarding frog is not
filamentous malphigian tubules which help in removal correct?
of excretory products from haemolymph. (a) Fertilization is external and takes place in water.
(a) Only (i) (b) Both (ii) and (iii) (b) External ear and tympanum can be seen externally.
(c) Both (i) and (iv) (d) All of these (c) In females the ureters and oviduct open separately
50. Which of the following statement(s) is/are correct in the cloaca.
regarding excretory system of cockroach ? (d) Copulatory pad on the first digit of the fore limbs
(i) Excretion is performed by malphigian tubules. and vocal sac are present in male frog.
EBD_7209
64 Biology

ASSERTION/REASON QUESTIONS (a) A – V; B – IV; C – II; D – III; E – I


(b) A – III; B – IV; C – V; D – II; E – I
In the following questions, a statement of Assertion is followed (c) A – V; B – IV; C – III; D – II; E – I
by a statement of Reason.
(a) If both Assertion and Reason are true and the Reason is (d) A – III; B – IV; C – V; D – I; E – II
the correct explanation of the Assertion. 64. Match column-I (type of epithelium) with column-II
(b) If both Assertion and Reason are true but the Reason is (Description) and choose the correct option.
not the correct explanation of the Assertion. Column-I Column-II
(c) If Assertion is true but Reason is false. (Types of epithelium) (Description)
(d) If both Assertion and Reason are false. A. Squamous I. It is composed of a
55. Assertion: Typhlosole increases the effective area of epithelium single-layer of cube-like
absorption in the intestine. cells
Reason: Typhlosole, present in the intestine, is the B. Cuboidal II. Having cilia on their free
characteristic feature of cockroach. epithelium surface
56. Assertion : The squamous epithelium is made of a single C. Columnar III. It is composed of a single
thin layer of flattened cells with irregular boundaries. epithelium layer of tall and slender
Reason : They are found in walls of blood vessels and air cells
sacs of wings. D. Ciliated IV. It is made up of a single thin
57. Assertion : Connective tissues are most aboundant and epithelium layer of flattened cells with
help in linking and supporting other tissues organ of the irregular boundaries
body.
(a) A – IV; B – I; C – III; D – II
Reason : In all connective tissues except blood the cells
secrete collagen or elastin. (b) A – I; B – IV; C – III; D – II
58. Assertion : Basophils cells secrete histamine. (c) A – IV; B – I; C – II; D – III
Reason : Histamine is a vasoconstrictor. (d) A – IV; B – III; C – I; D – II
59. Assertion : Numerous minute pores called nephridio pores 65. Match the types of connective tissue given in column-I
open on the surface of the body of earthworm. with their examples given column-II and choose the correct
Reason : In each body segment, there are rows of S-Shaped option.
setae present. Column-I Column-II
60. Assertion: In cockroaches the development is (Types of connective) (Examples)
paurometabolous.
Reason : Nymph grows by moulting about 13 times to A. Loose connective I. Tendons and ligaments
reach the adult form.
tissue
61. Assertion : Intercalated discs are important regions of
B. Dense regular II. Skin
cardiac muscle cells.
tissue
Reason : Intercalated discs function as boosters for muscle
C. Dense irregular III. Cartilage, bones, blood
contraction waves.
tissue
62. Assertion : Cartilage (protein matrix) and bone (calcium
D. Specialized IV. Fibroblasts, macrophages
matrix) are rigid connective tissue.
connective tissue and mast cells
Reason : Blood is connective tissue in which plasma is
(a) A – I; B – IV; C – II; D – III
the matrix.
(b) A – I; B – IV; C – III; D – II
MATCHING TYPE QUESTIONS (c) A – IV; B – I; C – II; D – III
(d) A – IV; B – II; C – I; D – III
63. Match the epithetial tissue given in column-I with its 66. Match the terms given in column-I with their feature given
location given in column-II and choose the correct option. in column-II and choose the correct option.
Column I Column II Column-I Column-II
(Epithelial tissue) (Location) (Terms) (Features)
A. Cuboidal I. Epidermis of skin A. Exocrine gland I. They help to stop substances
B. Ciliated II. Inner lining of blood vessels from leaking across a tissue
C. Columnar III. Inner surface of gall bladder B. Endocrine gland II. Hormones are secreted
D. Squamous IV. Inner lining of fallopian tube directly into the fluid bathing
E. Keratinized V. Lining of pancreatic duct the gland
squamous C. Tight junctions III. They perform cementing to
keep neighbouring cells
together.
Structural Organisation in Animals 65

D. Adhering junctions IV. Secretes mucus, saliva,


DIAGRAM TYPE QUESTIONS
earwax, oil, milk, digestive
enzymes and other cell 71. The diagram given below represents the reproductive
products organ of male cockroach. Choose the correct labelling of
(a) A – IV; B – II; C – I; D – III the part of marked as A, B, C and D.
(b) A – II; B – IV; C – I; D – III
(c) A – IV; B – II; C – III; D – I
(d) A – IV; B – I; C – II; D – III
67. Match column-I with column-II and choose the correct A
option. B
C
D
Column-I Column-II
A. Periplaneta I. Hepatic caecae
americana (a) A – 8th sternum, B - Anal cercus, C - 10th tergum,
D - Anal style
B. A ring of 6-8 blind II. Phylum arthropoda
(b) A - 10th tergum, B - Anal cercus, C - Anal style,
tubules
D - 8th sternum
C. Vascular system III. Spiracles
(c) A - Anal style, B - Anal cercus, C - 10th tergum,
D. 10 pairs of small IV. Malpighian tubules D- 8th sternum
holes (d) A - Anal cercus, B - 8th sternum, C - 10th tergum,
E. Excretion V. Open type D - Anal style.
(a) A – I; B – II; C – III; D – IV; E – V 72. In the given diagram of areolar connective tissue, the
(b) A – II; B – I; C – V; D – III; E – IV different cells and parts have been marked by alphabets
(c) A – II; B – I; C – III; D – V; E – IV (A, B, C & D). Choose the answer in which these alphabets
(d) A – III; B – IV; C – II; D – V; E – I correctly match with the parts and cells they indicate.
68. Find the incorrectly matched pair.
(a) Unicellular glandular cells - Goblet cell Matrix
(b) Saliva - Exocrine secretion
A
(c) Fusiform fibres - Smooth muscle
(d) Cartilage - Areolar tissue B
69. Which of the following types of connective tissue is
mismatched with its matrix ? C
(a) Areolar – Loosely packed matrix of protein fibres
(b) Bone – Mineralized matrix D
(c) Cartilage – Highly vascular matrix
(d) Blood – Liquid matrix (a) A-Adipocyte, B-Collagen fibres, C-Microfilament,
70. Match the description given in column I with their examples D-Mast cells
given in column II and choose the correct option. (b) A-Macrophage, B-Collagen fibres, C-Microfilament,
Column I Column II D-Mast cells
(Description) (Example) (c) A-Macrophage, B-Collagen fibres, C-Microtubule,
A. Aquatic respiratory I. Skin D-RBC
organ (d) A-Macrophage, B-Fibroblast, C-Collagen fibres,
B. Organ which acts II. Ureter D-Mast cells
urogenital duct and 73. Identify figures-I and II.
opens into the cloaca
C. A small median chamber III. Cloaca Fibroblast Collagen Fibre
that is used to pass faecal Matrix
matter, urine and sperms
to the exterior
D. A triangular structure IV. Sinus venosus
which joins the right
atrium and receives blood
through vena cava
(a) A – I; B – II; C – III; D – IV
(b) A – II; B – IV; C – I; D – III Matrix
Collagen Fibroblast
(c) A – II; B – I; C – III; D – IV Fibre
(d) A – IV; B – II; C – I; D – III Fig-I Fig-II
EBD_7209
66 Biology
Figure I Figure II 76. The figure given below shows the head region of
(a) Dense regular Dense irregular cockroach. Identify A to F.
connective tissue, connective tissue
(b) Loose irregular Loose regular
connective tissue, connective tissue Antennae A
(c) Adipose tissue, Specialized connective tissue B
(d) Connective tissue Areolar tissue
proper
74. The intercellular material of the given figure is solid and D
resists compression. Identify the figure and the label C
marked as A & B. E F

(a) A- Compound eye, B-Ocellus, C-Maxilla, D-


Mandible, E-Labrum, F-Labium
A (b) A- Ocellus, B-Compound eye, C-Mandible, D-
Maxilla, E-Labrum, F-Labium
B (c) A- Ocellus, B-Compound eye, C-Mandible, D-
Maxilla, E-Labium, F-Labrum
(d) A- Ocellus, B-Compound eye, C-Maxilla, D-
Mandible, E-Labrum, F-Labium
Fig
77. The given figure shows the digestive system of
Fig. cockroach with few structures marked as A, B, C and D.
A B
(a) Cartilage Collagen Chondrocyte
(b) Cartilage Collagen Chondroclast
(c) Bone Microtubule Chondroclast
(d) Bone Collagen fibres Osteoblast

75. The following figure shows the extarnal features of


cockroach with few structures labelled as A, B, C, D, and E.

Identify structures A to D.
Identify A to E . (a) A-Gizzard, B-Crop, C-Hepatic caecae, D-Malpighian
(a) A- Mesothorax, B-Pronotum, C-Metathorax, tubules
D-Tegmina, E-Anal style (b) A-Crop, B-Gizzard, C-Hepatic caecae, D-Malpighian
(b) A- Pronotum, B-Metathorax, C-Mesothorax, tubules
D-Tegmina, E-Sterna (c) A- Crop, B-Gizzard, C-Malpighian tubules, D-Hepatic
(c) A- Pronotum, B-Mesothorax, C-Metathorax, caecae
D-Tegmina, E-Anal cerci (d) A- Gizzard, B-Crop, C-Malpighian tubules, D-Hepatic
(d) A- Pronotum, B-Mesothorax, C-Metathorax, caecae
D-Tegmina, E-Anal style
Structural Organisation in Animals 67

78. The given figure shows open circulatory system of 81. The following figures A, B and C are types of muscle
cockroach with structure marked as A, B and C. Which tissue. Identify A, B and C.
structure is a 13 pair of wing shaped involuntary muscles
and mantain blood circulation?

(A) (B) (C)


(a) A – Smooth muscle, B – Cardiac muscle, C – Skeletal
muscle
(b) A – Skeletal muscle, B – Smooth muscle, C – Cardiac
muscle
(c) A – Cardiac muscle, B – Smooth muscle, C – Skeletal
muscle
(a) A (b) B (d) A – Smooth muscle, B –Skeletal muscle , C – Cardiac
(c) C (d) Both A nad B muscle
79. Figure given below shows reproductive system of female 82. The given figure shows the nephridial system of
cockroach. The correct labellings indicated by alphabets earthworm and answer the question.
(A, B & C) are respectively

Select the option which shows the correct identification


and the function of the structure marked as "X".
(a) Lateral heart. It is a blood pumping organ.
(a) A-Spermatheca, B-Collaterial glands, C-Gonapophyses (b) Calciferous glands. They neutralize the humic acid
(b) A-Phallic gland, B-Collaterial glands, C-Gonapophyses present in humus.
(c) A-Spermatheca, B-Seminal vesicle, C-Gonapophyses (c) Nephridia. It regulates the volume and composition
(d) A-Spermatheca, B-Collateral glands, C-Tegmina of the body fluids.
80. Identify the figure with its correct function (d) Blood glands. They produce blood cells and
haemoglobin which is dissolved in blood plasma.
83. The figure given below shows the alimentary canal of
cockroach with few structures labelled as 1, 2, 3 and 4. Identify
the correct identification of the structure which is also called
proventriculus and helps in grinding the food particles.

1
2
3
Fig :. Adipose connective tissue
(a) Areolar connective tissue – Serves as a support
framework for epithelium. 4
(b) Adipose tissue – Store fats and act as heat insulators.
(c) Dense regular tissue – Provide flexibility. (a) 1 (b) 2
(d) Dense irregular tissue – Provide strength and elasticity. (c) 3 (d) 4
EBD_7209
68 Biology
84. Refer th e given figure and identify the correct
CRITICAL THINKING TYPE QUESTIONS
characteristic feature.
87. The blood of cockroach contains no respiratory pigment.
It means that
(a) cockroach does not respire.
(b) respiration is anaerobic.
(c) oxygen goes directly into tissues by diffusion.
(d) oxygen goes directly into tissues by intracellular
capillary system.
(i) It is a type of loose connective tissue. 88. If the head of cockroach is cut off, it will still be alive for
(ii) It contains fibroblast, macrophages, collagen fibres as long as one week. Their is because of
and mast cells. (a) the body which is covered with hard chitinous
(iii) The cells of this tissue are specialized to store fats. exoskeleton.
(iv) The wall of internal organs such as the blood vessels, (b) head which holds a bit of nervous system.
stomach and intestine contains this type of tissue. (c) head which is of no use.
(a) (i) & (ii) (b) (i) & (iii) (d) food capturing appratus which is found elsewhere.
(c) (ii) & (iii) (d) (iii) & (iv) 89. What will happen if ligaments are cut or broken?
85. A student was given a sample of two tissues. He observes (a) No movements at joints.
(b) Bones will become fixed.
the tissues under the microscope and draws their figures
(c) Bones will become unfixed.
(1 and 2) as shown below.
(d) Bone will move freely at joints.
90. Four healthy people in their twenties faced injuries resulting
in damage and death of a few cells given below. Which of
the cells are least likely to be replaced by new cells?
(a) Liver cells
(b) Neurons
1 2 (c) Malpighian layer of the skin
Identify the tissues (1 and 2). (d) Osteocytes
(a) 1: Columnar cells bearing cilia; 2: Unicellular glandular 91. In cockroach head can move in all directions due to
epithelium (a) absence of neck.
(b) fusion of all 6 segments of head.
(b) 1: Cuboidal cells bearing cilia; 2: Multicellular
(c) flexible neck.
glandular epithelium
(d) head is small and light weight.
(c) 1: Compound cells bearing cilia; 2: Unicellular 92. What external changes are visible after the last moult of a
glandular epithelium cockroach nymph?
(d) 1: Columnar cells bearing cilia; 2: Multicellular (a) Development of anal cerci.
glandular epithelium (b) Development of both forewings and hind wings.
86. Refer the given figure of female reproductive system of (c) Development labium.
cockroach and identify the correct labels (marked as 1, 2, (d) Mandibles become harder.
3 and 4) which are collectively called genital pouch. 93. Choose the correct sequence of alimentary canal of
Cockroach
(a) Gizzard ® Crop ® Malphigian tubules ® Hepatic
caeca ® Rectum.
(b) Gizzard ® Hepatic caeca ® Crop ® Rectum ®
Ovary Malphigian tubules.
1 (c) Crop ® Gizzard ® Hepatic caeca ® Malphigian
Spermatheca
Common tubules ® Rectum.
oviduct (d) Crop ® Hepatic caeca ® Gizzard ® Rectum ®
or vagina
2
Malphigian tubules.
94. Lack of blood supply and presence of the noncellular
3
basement membrane are the characteristics of the
gonapophyses
4 (a) muscular tissue
(b) fluid connective tissue
(a) 1 & 2 (b) 1 & 3 (c) epithelial tissue
(c) 2 & 4 (d) 3 & 4 (d) nervous tissue
Structural Organisation in Animals 69

95. The secretions of endocrine glands are released directly 104. Non-ciliated simple columnar epithelium often contains
(a) into the skin surface (b) into the blood stream _______, which increase the surface area for secretion
(c) into a gland duct (d) into the brain tissue and absorption.
96. Cartilage tissues are likely to be slow in healing following (a) flagella (b) collagen fibres
an injury because (c) microvilli (d) all of these
(a) cartilage cells cannot reproduce. 105. What are the three basic components of connective
(b) they lack direct blood supplies. tissues?
(c) the intercellular material is missing. (a) Ground substance, cells and basement membrane
(d) cartilage cells are surrounded by fluids. (b) Cartilage, intercellular matrix and serum
97. When cardiac muscle cells are damaged by a heart attack, (c) Cells, protein fibers and ground substance
they are usually replaced by (d) Collagen, elastin and reticular fibers
(a) connective tissue cells 106. A student was given a sample of tissue. He observes and
(b) new smooth muscle cells concludes the following characters.
(c) new cardiac muscle cells (i) The cells are composed of a single layer of tall and
(d) epithelial cells slender cells.
98. Which of the following vertebrate tissues would be an (ii) Their nuclei are located at the base.
excellent source of collagen? (iii) Free surface may have microvilli.
(a) Liver (b) Nerve (iv) It is found in the lining of stomach and intestine
(c) Muscle (d) Tendon (v) They help in secretion and absorption.
99. Which of the following is involved in the production of Based on the above features identify the epithelium.
new blood cells ? (a) Cuboidal epithelium
(a) Adipose cell (b) Bone marrow (b) Columnar Epithelium
(c) Liver (d) Matrix (c) Squamous epithelium
100. The major functions of loose connective tissue include (d) Glandular epithelium
(a) occupying spaces between organs and supporting 107. Which of the following animal's sensory system does not
epithelia. has eyes but does possess light and touch sensitive
(b) supporting and surrounding blood vessels and organs (receptor cells) to distinguish the light intensities
nerves and to feel the vibrations in the ground?
(c) cushioning organs, storing lipids and facilitating (a) Frog (b) Snake
diffusion. (c) Earthworm (d) Cockroach
(d) All of the above 108. A student was given a specimen to identify on the basis
101. Nervous tissue cells that play several supporting roles of the characteristics given below.
but do not transmit impulses are called (i) They are metamerically segmented.
(a) glial cells (b) dendrites (ii) Presence of closed circulatory system.
(c) nerve cells (d) neurons (iii) They have circular and longitudinal muscles for
102. Hair present in the skin are locomotion.
(a) epidermal in origin and made of dead cells. Identify the specimen.
(b) epidermal in origin and made of living cells. (a) Frog (b) Pheretima
(c) dermal in origin and made of living cells. (c) Cockroach (d) Rabbit
(d) dermal in origin and made of dead cells. 109. Which of the following animals maintain ecological
103 In which one of the following preparations, cell junctions balance?
come across most frequently ? (a) Frog (b) Rabbit
(a) Ligament (b) Tendon (c) Earthworm (d) Cockroach
(c) Cartilage (d) Ciliated epithelium
EBD_7209
8

Chapter
Cell : The Unit of Life

11. The best material for study of structure of cell membrane


FACT/DEFINITION TYPE QUESTIONS
is
1. Who proposed the theory that "Cells arise only from the
(a) RBC of human (b) RBC of frog
pre-existing cells"?
(c) cheek cell of human (d) liver cell of rat
(a) Mohl (b) Virchow
(c) Haeckel (d) Brown 12. In which of the following the cells are held together by a
2. An organalle devoid of membrane covering is Ca-pectate layer?
(a) vacuole (b) ribosome (a) Primary cell wall (b) Secondary cell wall
(c) peroxisome (d) lysosome (c) Middle lamella (d) Tertiary cell wall
3. Difference between the prokaryotic and eukaryotic cells 13. Which one of the following structures between two
in having adjacent cells is an effective transport pathway?
(a) cell wall (b) nuclear membrane
(c) ribosome (d) none of these (a) Plasmodesmata (b) Plastoquinones
4. Membranous extensions in blue green algae are known
(c) Endoplasmic reticulum (d) Plasmalemma
as
14. Which of the following will determines the shape of the
(a) phytochrome (b) chromatophore
cells and provides a strong structural support to prevent
(c) mesosome (d) pneumatophore
the bacterium from bursting or collapsing?
5. Extension of plasma membrane in prokaryotic cell is
(a) mesosome (b) haploid (a) Plasmids (b) Cell wall
(c) ribosome (d) none of these (c) Mesosome (d) Cell membrane
6. Polysome is a chain of 15. Which one of the following is not considered as part of
(a) oxysomes (b) sphaerosomes the endomembrane system?
(c) ribosomes (d) dictyosomes (a) Golgi complex (b) Peroxisome
7. Integral proteins of cell membrane occur on/in (c) Vacuole (d) Lysosome
(a) inner surfaces 16. Function of RER is
(b) outer surfaces (a) autolysis
(c) phospholipid matrix (b) protein synthesis
(d) inner and outer surfaces
(c) lipid synthesis
8. Active transport across biomembrane involves
(a) production of ATP (b) requirement of energy d) carbohydrate synthesis
(c) production of toxin (d) release of energy 17. Which of the following cell organelles were discovered
9. The membrane of the erythrocytes has approximately after the introduction of electron microscope?
___% of proteins and ___% lipids. (a) Mitochondria
(a) 42, 50 (b) 52, 40 (b) Endoplasmic reticulum
(c) 50, 50 (d) 60, 40 (c) Ribosomes
10. The lipid component of the membrane mainly consists (d) Both (b) and (c)
of __________. 18. _______________ is the important site of formation
(a) polysaccharides (b) phosphoglyceride of glycoproteins and golycolipids.
(c) monosaccharaides (d) both (a) and (c) (a) SER (b) Lysosome
(c) Golgi apparatus (d) Mitochondria
Cell : The Unit of Life 71

19. Golgi apparatus is concerned with 31. Centrioles and centrosomes occur in the cells of
(a) excretion (b) secretion (a) green plants
(c) ATP synthesis (d) RNA synthesis (b) animals
20. Lysosomes contain (c) bacteria and cyanobacteria
(a) carbohydrates (b) hormones (d) both (b) and (c)
(c) nucleic acids (d) hydrolases. 32. Nucleolus is
21. Function of contractile vacuole in Amoeba is (a) rounded structure found in cytoplasm near nucleus.
(a) excretion and osmoregulation. (b) rounded structure inside nucleus and having rRNA.
(b) digestion and respiration. (c) rod-shaped structure in cytoplasm near the nucleus.
(c) osmoregulation and transportation. (d) none of the above.
(d) none of the above. 33. Satellite means
22. Both the membranes of mitochondrion are (a) terminal part of the chromosome beyond secondary
(a) structurally different but functionally similar. constriction.
(b) structurally as well as functionally different. (b) terminal part of the chromosome beyond primary
(c) structurally similar but functionally different. constriction.
(d) structurally different but functionally similar. (c) terminal part of chromosome beyond tertiary
constriction.
23. Plastids are found in
(d) none of the above
(a) all animal cells.
34. Chromosomes having equal or almost equal arms are
(b) some animal cells.
called
(c) all plant cells.
(a) metacentric (b) acrocentric
(d) all plant cells and euglenoides.
(c) polycentric (d) acentric.
24. Plastids storing fat are called
(a) Elaioplasts (b) Sphaerosomes
STATEMENT TYPE QUESTIONS
(c) Aleuroplasts (d) Pyrenoids
25. Grana are 35. Which of the following statement is correct regarding
(a) protein storing plastids. vacuole?
(b) coloured plastids. (a) It is membrane-bound and contains storage proteins
and lipids.
(c) stacks of thylakoids.
(b) It is membrane-bound and contains water and
(d) individual thylakoids present in stroma. excretory substances.
26. A component of cytoskeleton is (c) It lacks membrane and contains air.
(a) microtubule (b) bone (d) It lacks membrane and contains water and excretory
(c) chitin (d) cartilage. substances.
27. The cytoskeleton is a proteinaceous network of fibres in 36. Which of the following statement of a bacterial cell is/
the cytoplasm. It is involved in are correct?
(i) Mesosome is formed by the extensions of plasma
(a) mechanical support. (b) motility.
membrane into the cell.
(c) maintenace of cell-shape. (d) all of these (ii) The pili are elongated tubular structures made up
28. Axoneme with 9 + 2 microtubular arrangement occurs in of a protein.
(a) cilia (b) flagella (iii) Flagellum is composed of filament, hook and basal
(c) both (a) and (b) (d) centriole body.
(iv) Ribosomes are about 30 nm by 50 nm in size.
29. Prokaryotic and eukaryotic flagella differ in the
(a) (i), (ii) and (iii) (b) All of the above
(a) type of movement and placement. (c) (ii) and (iv) (d) None of the above
(b) location and mode of functioning. 37. Protein synthesis in an animal cell occurs
(c) microtubular structure and function. (a) on ribosomes present in cytoplasm as well as in
(d) microtubular organization and type of movement. mitochondria.
(b) on ribosomes present in the nucleolus as well as in
30. Basal bodies are associated with the formation of
cytoplasm.
(a) phragmoplast (b) cilia and flagella (c) only on ribosomes attached to the nuclears
(c) cell plate (d) kinetochore envelope and endoplasmic reticulum.
(d) only on the ribosomes present in cytosol.
EBD_7209
72 Biology
38. According to widely accepted “fluid mosaic model” cell (ii) Human RBC is about 7.0 mm in diameter.
membranes are semi-fluid, where lipids and integral (iii) Cytoplasm is the main area of cellular activities.
proteins can diffuse randomly. In recent years, this model (iv) Various chemical reactions occur in cytoplasm to
has been modified in several respects. In this regard, keep the cell in the living state.
which of the following statements is incorrect? (a) (i), (ii), (iii) and (iv) (b) Only (i) and (ii)
(a) Proteins in cell membranes can travel within the (c) Only (iv) (d) None of the above
lipid bilayer. 45. Study the following statements on cilium or flagellum
(b) Proteins can also undergo flip-flop movements in and answer the question.
the lipid bilayer. (i) Cilium / Flagellum contains an outer ring of nine
(c) Proteins can remain confined within certain doublet microtubules surrounding two singlet
domains of the membrane. microtubules.
(ii) Cilia are smaller which work like oars, causing the
(d) Many proteins remain completely embedded within
movement of either the cells or surrounding fluid.
the lipid bilayer.
(iii) Flagella are comparatively longer and responsible
39. In prokaryotes, chromatophores are
for cell movement.
(a) specialized granules responsible for colouration of (iv) Cilium and flagellum are covered with plasma
cells. membrane.
(b) structures responsible for organizing the shape of Which of the above statements are correct?
the organism.
(a) (i) and (ii) (b) (i), (ii), (iii) and (iv)
(c) inclusion bodies lying free inside the cells for (c) (i) and (iv) (d) (ii) and (iii)
carrying out various metabolic activities.
46. Which of the following is not the function of cell wall?
(d) internal membrane system which becomes
(i) Provides shape to the cell.
extensive and complex in photosynthetic bacteria.
(ii) Protects the cell from mechanical damage and
40. Which of the following statement is false? infection.
(a) The ribosomes of a polysome translate the mRNA (iii) Helps in cell to cell interaction.
into protein.
(iv) Provides barrier to undesirable macromolecules.
(b) Mitochondria divide by fragmentation.
(a) Only (i) (b) Only (iv)
(c) All cell arise from pre-existing cells. (c) Only (ii), (iii) and (iv) (d) None of the above
(d) The lipid component of the membrane mainly
47. Read the folowing statements and identify the correct
consists of phosphoglycerides.
option.
41. Which of the following is incorrect ?
(i) Contractile vacuole takes part in osmoregulation
(a) Mycoplasma is the smallest cell (0.3 µ).
and excretion.
(b) Bacteria are 3 to 5 µm in size.
(c) The largest cell is the egg of an ostrich. (ii) Food vacuole is formed by engulfing the food
(d) Nerve cells are some of the smallest cells. particles.
42. Select the incorrect statement about prokaryotic (iii) The vacuole is bound by a double membrane called
ribosomes. tonoplast.
(a) 50S and 30S subunits unite to form 70S ribosomes. (iv) Vacuole can occupy upto 90 percent of the volume
(b) Polysome/polyribosome consists of many of the cell.
ribosomes only. (a) (i) and (ii) (b) (ii) and (iv)
(c) Ribosome is the site of protein synthesis. (c) (i), (ii) and (iv) (d) None of the above
(d) Polysome indicate the synthesis of identical 48. Which of the following statements are incorrect ?
poolypeptide in multiple copies. (i) Plant cells have centrioles which are absent in
43. Select the statements which are related to Schwann. almost all animal cells.
(i) He reported that cells have a thin outer layer which (ii) Ribosomes are the site of protein synthesis.
is today known as plasma membrane (iii) The middle lamella is a layer mainly of calcium
(ii) Cell wall is a unique character of the plant cell. carbonate which holds the different neighbouring
(iii) Body of plants and animals are composed of cells cells together.
and products of cells. (iv) In animal cell, steroidal hormones are synthesized
(a) Only (i) (b) Only (iii)
by smooth endoplasmic reticulum.
(c) (i) and (iii) (d) All of these
Of the above statements
44. Which of the following statements is/are correct ?
(i) The shape of the cells may vary with the function (a) (i) and (iii) (b) (iii) and (iv)
they perform. (c) (ii) and (iv) (d) (i) and (iv)
Cell : The Unit of Life 73

49. Which of the following statements is/are correct ? (i) It is a membrane bound space found in the cytoplasm.
(i) The endomembrane system includes plasma (ii) It is bound by a single membrane called tonoplast.
membrane, ER, Golgi complex, lysosomes and (iii) It contains water, sap, excretory products and other
vacuoles. materials not useful to the cell.
(ii) ER helps in the transport of substances, synthesis (iv) It has higher concentration of sap than the
of proteins, lipoproteins and glycogen. cytoplasm.
(iii) Ribosomes are involved in protein synthesis. (a) Golgi apparatus (b) Lysosomes
(iv) Mitochondria help in oxidative phosphorylation and (c) Endoplasmic reticulum (d) Vacuoles
generation of ATP. 54. Read the statements given below with regard to the
(a) (ii), (iii) & (iv) (b) (i) only functions performed by Golgi apparatus ?
(i) Transport and chemically modify the materials
(c) (ii) only (d) (iii) only
contained within it.
50. Choose the wrong statements regarding bacterial cell
(ii) Performs the function of packaging materials.
(i) Glycocalyx is the outermost envelope in bacteria. (iii) Important site of formation of glycoproteins and
(ii) The glycocalyx could be a loose sheath called glycolipids.
capsule. Which of the following is the correct answer ?
(iii) The glycocalyx may be thick and tough called slime (a) (i) is wrong but (ii) and (iii) are correct
layer. (b) (ii) is wrong but (i) and (iii) are correct
(iv) A special structure formed by the plasma membrane (c) (ii) and (iii) are wrong but (i) is correct
is called mesosome. (d) All are correct.
(v) Small bristle like fibres sprouting out of the cell 55. Which of the following statements are correct?
are called fimbriae. (a) Na+/K+ pump is an example of active transport.
(a) (i) and (iii) (b) (i) and (ii) (b) In plant cells lipid like steroidal hormones are
(c) (ii) and (iii) (d) (i) and (iv) synthesized in SER.
51. Which of the following statements are correct ? (c) In plant cells, the vacuoles can occupy up to 10%
(i) In prokaryotic cells, a special membranous of the volume of the cell.
structure formed by the extension of the plasma (d) Chlorophyll and leucoplast are responsible for
membrane into the cell is known as polysome. trapping light energy essential for photosynthesis.
(ii) The smooth endoplasmic reticulum is the major site
for synthesis of glycoproteins. ASSERTION/REASON TYPE QUESTIONS
(iii) RuBisCO is the most abundant protein in the whole In the following questions, a statement of Assertion is followed
biosphere. by a statement of Reason.
(iv) Mitochondria, chloroplasts and peroxisomes are (a) If both Assertion and Reason are true and the Reason is
not considered as part of endomembrane system. the correct explanation of the Assertion.
Of the above statements (b) If both Assertion and Reason are true but the Reason is
(a) (iii) and (iv) (b) (i) and (ii) not the correct explanation of the Assertion.
(c) (ii) and (iii) (d) (i) and (iv) (c) If Assertion is true but Reason is false.
52. Consider the following statements and choose the (d) If both Assertion and Reason are false.
correct statement. 56. Assertion : Centrosomes and centrioles are related to
(i) The endomembrane system includes mitochondria, each other.
chloroplast and peroxisomes. Reason : Centrosome usually contains two cylindrical
(ii) Smooth endoplasmic reticulum is the major site for structures called centrioles.
synthesis of lipid. 57. Assertion : Cells vary greatly in their shape.
(iii) Rough endoplasmic reticulum is actively involved
Reason : The shape of cell does not depend on the
in protein synthesis.
function they perform.
(iv) Mitochondrial matrix possesses single circular
58. Assertion : The middle lamella is a layer made up of
DNA, a few RNA and 80S ribosomes.
calcium pectate.
Of the above statements.
(a) (i) and (iii) (b) (ii) and (iv) 59. Assertion : The acrocentric chromosome has
(c) (iii) and (iv) (d) (ii) and (iii) centromere at the terminal position.
53. Given below are some characters of a cell organelle. Reason : The metacentric chromosome has centromere
Identify the correct organelle which shows all the slightly away from the middle of the chromosome.
characters described above.
EBD_7209
74 Biology
60. Assertion : The quasifluid nature of lipid enables lateral 66. Match column-I (scientists) with column-II (discovery)
movement of proteins within the overall bilayer. and select the correct option.
Reason : This ability to move within the membrane is Column-I Column-II
called fluidity and is important for cell growth. A. Leeuwenhoek I. First saw and described a
61. Assertion : Pili are nonmotile appendages of bacteria. living cell
Reason : Pili take part in conjugation. B. Robert Brown II. Presence of cell wall is
62. Assertion : A plant cell bursts if placed in water. unique to plant cells
C. Schleiden III. Discovered the nucleus
Reason : High turgor pressure causes bursting of plant
D. Schwann IV. All plants are composed of
cells.
different kind of cells
MATCHING TYPE QUESTIONS (a) A – I; B – III; C – IV; D – II
(b) A – I; B – III; C – II; D – IV
63. Match column-I with column-II and choose the correct (c) A – III; B – I; C – IV; D – II
option. (d) A – I; B – IV; C – II; D – III
Column-I Column-II 67. Match column I (cell type) with column II (size) and
A. Tonoplast I. Contain digestive choose the correct option.
enzyme Column-I Column-II
B. Contractile vacuole II. Store metabolic gases (Cell type) (Size)
C. Food vacuole III. Excretion A. Viruses I. 1-2 mm
D. Air vacuole IV. Transport of ions in B. PPLO II. 10-20 mm
plants C. Eukaryotic cell III. About 0.1 mm
(a) A – IV; B – III; C – I; D – II D. Bacterium IV. 0.02 - 0.2 mm
(b) A – II; B – III; C – IV; D – I (a) A – I; B – II; C – III; D – IV
(c) A – IV; B – II; C – III; D – I (b) A – IV; B – III; C – II; D – I
(d) A – I; B – III; C – II; D – IV (c) A – I; B – III; C – II; D – IV
64. Match column-I with column-II and choose the correct
(d) A – IV; B – II; C – III; D – I
option.
Column-I Column-II 68. Match column-I (cell organelle) with column-II membrane
(Chromosome) (Position of and select the correct option from the codes given below.
Centromere) Column-I Column-II
A. Metacentric I. At the tip A. Mitochondria I. Without membrane
B. Submetacentric II. Almost near the tip B. Lysosomes II. Single membrane
C. Acrocentric III. At the middle C. Ribosomes III. Double membrane
D. Telocentric IV. Slightly away from the (a) A – I; B – II; C – III
middle
(b) A – III; B – I; C – II
(a) A – III; B – IV; C – II; D – I
(c) A – III; B – II; C – I
(b) A – IV; B – III; C – II; D – I
(c) A – I; B – II; C – III; D – IV (d) A – II; B – III; C – I
69. Match column-I with column-II and select the correct
(d) A – IV; B – III; C – I ; D – II
65. Match the items given in column-I with their role given option.
in column-II and choose the correct option. Column - I Column - II
Column-I Column-II A. Golgi apparatus I. Storage
A. SER I. Increase the surface B. Mitochondria II. Photosynthesis
area C. Vacuoles III. Transport
B. Golgi apparatus II. Store oils or fats D. Grana IV. Secretion
C. Cristae III. Excretion V. Respiration
D. Peroxisome IV. Photorespiration (a) A – IV; B – V; C – I: D – II
E. Elaioplasts V. Synthesis of lipid
(a) A – V; B – III; C – I; D – IV; E – II (b) A – I; B – II; C – IV: D – III
(b) A – V; B – III; C – II; D – IV; E – I (c) A – IV; B – I; C – II: D – III
(c) A – II; B – III; C – I; D – IV; E – V (d) A – I; B – II; C – III: D – IV
(d) A – III; B – IV; C – I; D – V; E – II
Cell : The Unit of Life 75

70. Match column-I with column-II and select the correct 75. Which of these is wrongly matched?
option. (a) Chloroplasts – Chlorophyll
Column - I Column - II (b) Elaioplasts – Starch
A. RER I. Intracellular and extracellular (c) Chromoplasts – Carotenoids
digestion (d) Amyloplasts – Carbohydrates
B. Cell wall II. Provide structural support to 76. Which one of the following pairs is NOT correctly
the cell matched?
C. Flagella III. Protein synthesis and (a) Cristae – The tubular structure formed by
secretion the folding of th e in ner
D. Lysosomes IV Responsible for cell movement membrane of the mitochondrion.
(b) Plasmodesmata – The membrane surrounding the
(a) A – III; B – II; C – IV; D – I
vacuole in plants.
(b) A – II; B – III; C – IV; D – I (c) Grana – Membrane bound discs in
(c) A – I; B – III; C – II; D – IV chloroplasts that contain
chlorophylls and carotenoids.
(d) A – IV; B – II; C – III; D – I
(d) Middle lamella – Layer between adjacent cells
71. Match column-I and column-II and select the correct walls in plants derived from cell
answer plate.
Column-I Column-II 77. Which of the following pair are correctly matched ?
A. Bacteria without walls I. Lysosome (I) Amyloplasts – Store proteins
B. Small circular DNA II. Mycoplasma cells (II) Mitochondrion – ‘Power house’ of the
C. Flattened sacs in III. Thylakoid cell
a chloroplast (III) Stroma – Chlorophyll pigment
(IV) Axoneme – 9 + 2 array
D. A vesicle in which IV. Plasmid
(a) (I) and (III) only (b) (II), (III) and (IV) only
hydrolytic enzymes
(c) (III) and (IV) only (d) (II) and (IV) only
are stored 78. Which one of the following cellular parts is correctly
(a) A – III; B – IV; C – II; D – I matched with their characters ?
(b) A – II; B – IV; C – III; D – I (a) Centrioles – Sites for active RNA synthesis.
(b) Lysosomes – Optimally active at a pH of about 8.5.
(c) A – I; B – II; C – III; D – IV (c) Thylakoids – Flattened membranous sacs forming
(d) A – IV; B – III; C – I ; D – II the grana of chloroplasts.
72. Which of the following pair are correctly matched? (d) Ribosomes – Those on chloroplasts are larger
(80S) while those in the cytoplasm are smaller
A. Microtubules – Structural components of cilia
(70S).
B. Centrioles – Store hydrolytic enzymes 79. Match Column I with Column II and choose the correct
C. Amyloplasts – Store oil protein and starch in plants option.
(a) A, B and C (b) A and B Column I Column II
(c) A only (d) A and C A. Centrioles I. Non-membrane bound
organelle which helps in
73. Choose the incorrect match. cell division
(a) Nucleus — RNA B. Fimbriae II. Special str ucture of
(b) Lysosome — Protein synthesis bacteria which help them to
attach with rocks in stream
(c) Mitochondria — Respiration
and also to host tissue
(d) Cytoskeleton — Microtubules C. Endomembrane III. Includes those organelles
74. Which one of the following combination is mismatched? system whose functions are
(a) Glycocalyx – May be capsule or slime layer coordinated
D. Mitochondria IV. Divide by fission and site
(b) Pili – Reproduction of aerobic respiration
(c) Cell wall – Protective, determines shape, prevents (a) A – I; B – II; C – III; D – IV
from bursting (b) A – III; B – I; C – II; D – IV
(d) Flagella, pili and fimbriae – Surface structures of (c) A – III; B – I; C – IV; D – II
bacterial cell (d) A – I; B – IV; C – III ; D – II
EBD_7209
76 Biology
80. Which of the following terms is NOT correctly matched (b) A – Acrocentric chromosome, B – Telocentric
with its feature? chromosome, C – Metacentric chromosome,
(a) Osmosis – Movement of water by diffusion. D – Submetacentric chromosome
(b) Nucleoplasm – Site of active synthesis of ribosomal (c) A – Submetacentric chromosome, B – Metacentric
RNA. chromosome, C – Telocentric chromosome,
(c) Mesosome – Infolding of cell membrane and D – Acrocentric chromosome
characteristics of eukaryotes.
(d) A – Metacentric chromosome, B – Submetacentric
(d) Pili – Elongated tubular surface structures (made
chromosome, C– Acrocentric chromosome,
of special protein) of bacteria.
D – Telocentric chromosome.
DIAGRAM TYPE QUESTIONS 83. Which of the following option correctly match A, B, C,
and D indicated in the given sectional view of
81. Identify the components labelled A, B, C and D in the chloroplasts.
given section of cilia/flagella showing different parts.
Choose the option which shows the correct labelling of
parts.

(a) A - Thylakoid, B-Stromal lamella, C - Stroma,


D - Granum
(b) A - Granum, B - Thylakoid, C - Stromal lamella,
(a) A – Plasma membrane, B – Interdoublet bridge, D - Stroma
C – Central microtubule, D – Radial spoke (c) A - Thylakoid, B - Granum, C - Stromal lamella,
(b) A – Plasma membrane, B – Arm, C – Central D - Stroma
microtubule, D – Radial spoke (d) A - Granum, B - Thylakoid, C - Stroma, D - Stromal
(c) A – Plasma membrane, B – Interdoublet bridge, lamella
C – Hub, D – Radial spoke 84. The given diagram shows the sectional view of a
(d) A – Plasma membrane, B – Interdoublet bridge, mitochondrion.
C– Hub, D – Arm
82. The given diagram shows the types of chromosomes Ribosomes DNA (B)
Tubuli
(labelled as A, B, C & D) based on the position of
centromere. Which one is the correct option for the (E)
labelled chromosomes. A, B, C and D ?

(A)
(D) (C)
Identify the parts labelled as A, B, C, D & E
(a) A - Outer membrane, B - Inner membrane,
C - Matrix, D - Inter- membrane space, E - Crista
A B C D (b) A - Outer membrane, B - Inner membrane, C - Inter-
membrane space, D - Matrix, E - Crista
(a) A – Telocentric chromosome, B – Acrocentric (c) A - Outer membrane, B - Inner membrane,
chromosome, C – Submetacentric chromosome, C - Matrix, D - Crista, E - Inter - membrane space
D – Metacentric chromosome (d) A - Outer membrane, B - Inner membrane, C- Crista,
D - Matrix, E - Inter-membrane space
Cell : The Unit of Life 77

85. Identify the components labelled as A, B, C, D and E in (b) A - Desmosome, B - Rough endoplasmic reticulum,
the diagram given below from the list (i) to (viii). C - Golgi apparatus, D - Mitochondrion,
E - Ribosomes
A
(c) A - Plasmodesmata, B - Smooth endoplasmic
B reticulum, C - Golgi apparatus, D - Mitochondrion,
E - Ribosomes
(d) A - Tight junction, B - Rough endoplasmic reticulum,
C - Golgi apparatus, D - Mitochondrion,
E -Ribosomes
87. The diagram given below represent a filuid mosaic model
of plasma membrance. Match the components marked
C as A, B, C, D and E in the diagram below from the list (i)
to (vii).
D

(i) Cristae of mitochondria


(ii) Inner membrane of mitochondria
(iii) Cytoplasm
(iv) Smooth endoplasmic reticulum
(v) Rough endoplasmic reticulum
(vi) Mitochondrial matrix
(vii) Ribosome
(viii) Nucleus
A B C D E (i) Sugar (ii) Protein
(a) (viii) (v) (vii) (iii) (iv) (iii) Lipid bilayer (iv) Integral protein
(b) (i) (iv) (vii) (vi) (iii) (v) Cytoplasm (vi) Cell wall
(vii) External protein
(c) (vi) (v) (iv) (vii) (i)
(a) A - (i), B - (ii), C - (iii), D - (iv), E - (v)
(d) (v) (i) (iii) (ii) (iv) (b) A - (ii), B - (i), C - (iii), D - (iv), E - (v)
86. The following diagram shows some of the missing (c) A - (i), B - (ii), C - (iii), D - (iv), E - (vi)
structures in a plant cell marked as A, B, C, D and E. (d) A - (i), B - (ii), C - (iii), D - (vii), E - (v)
Choose the option with their correct names.
88. Identify the cell organelle given below. Which is an
important site of formation of glycoproteins &
glycolipids?

(a) Rough endoplasmic reticulum


(a) A - Plasmodesmata, B - Rough endoplasmic (b) Smooth endoplasmic reticulum
reticulum, C - Golgi apparatus, D - Mitochondrion, (c) Golgi body
E - Ribosomes (d) Mitochondria
EBD_7209
78 Biology
89. The following diagram represents a structure Nucleus
chromosome. Identify the structures marked as A, B and Nuclear pore Rough
C. endoplasmic
reticulum

Ribosome

Smooth
Endoplamic
reticulum
(a) A - Satellite, B - Primary constriction,
C - Acrocentric (a) Golgi apparatus– Protein synthesis
(b) A - Satellite, B - Secondary constriction,
C - Metacentric (b) Golgi apparatus– Formation of glycolipids
(c) A - Satellite, B - Centromere, C - Telocentric (c) Rough endoplasmic reticulum– Protein synthesis
(d) A - Satellite, B - Centromere, C - Submetacentric
90. The figure below shows the structure of a mitochondrion (d) Rough endoplasmic reticulum– Formation of
with its four parts labelled (A), (B), (C) and (D). glycoproteins
92. In the given figure of animal cell, one organelle is
marked as A. Select the correct identification and
function of the organelle ‘A’ from the given option.

Select the part correctly matched with its function.


(a) Part (D): Outer membrane – Gives rise to inner
membrane by splitting.
(b) Part (B): Inner membrane – Forms infoldings called
A
cristae.
(c) Part (C): Cristae – Possess single circular DNA (a) Endoplasmic reticulum– Synthesis of lipids.
(b) Mitochondria– Produce cellular energy in the form
molecule and ribosomes.
of ATP.
(d) Part (A): Matrix – Major site for respiratory chain (c) Golgi body– Provides packaging material.
enzymes. (d) Lysosomes– Secrete hydrolytic enzymes.
91. Which one of the following organelle given below is 93. The given figures show two types of cell. Which
structures are common to both the cells?
correctly matched with its function ?
Cell : The Unit of Life 79

CRITICAL THINKING TYPE QUESTIONS


96. Microtubules are absent in
(a) mitochondria (b) centriole
(c) flagella (d) spindle fibres
97. Golgi apparatus is absent in
(a) higher plants
(b) yeast
(c) bacteria and blue-green algae
(d) None of the above
98. Most of water, in mature plant cells occurs in
(a) nucleus (b) cell wall
Cell 1 Cell 2
(c) vacuoles (d) cytoplasm
99. Which of the following lacks cell wall?
(a) Gametes (b) Amoeba
(a) Nucleus and cell wall
(c) Mycoplasma (d) All of these
(b) Nucleus and cytoplasm 100. What would happen if lysosomes get ruptured in a
cell?
(c) Ribosomes and flagella (a) Cell dies (b) Cell shrinks
(d) Ribosomes and cell wall (c) Cell swell up (d) Nothing would happen
101. Cell sap is a
(Directions for 94 and 95): Refer the given figure of animal
(a) living content of cytoplasm.
cell with few structure marked as W, X, Y and Z. On the basis
(b) nonliving content of cytoplasm.
of this figure answer the following questions.
(c) nonliving content of vacuole.
(d) living content of vacuole.
102. The fluidity of membranes in a plant in cold weather may
be maintained by
(a) increasing the number of phospholipids with
unsaturated hydrocarbon tails.
(b) increasing the proportion of integral proteins.
W (c) increasing concentration of cholesterol in
membrane.
X (d) increasing the number of phospholipids with
saturated hydrocarbon tail.
103. Which of the following is absent in prokaryotes?
(a) DNA (b) RNA
(c) Plasma membrane (d) Mitochondria
Y 104. In which method of transport, plasma membrane does
Z not require carrier molecule?
(a) Active transport (b) Facilitated diffusion
(c) Simple diffusion (d) Na+ – K+ pump
94. Which cellular structure helps in transferring genetic
105. pH of vacuolar cell sap is
information from one generation to another?
(a) W (b) X (a) neutral and isotonic.
(c) Y (d) Z (b) alkaline and isotonic.
95. Which function is carried out by the cell organelle 'X'? (c) acidic and hypertonic.
(a) helps control the movement of substance in and out
(d) equal to cytoplasm and isotonic.
of the cell
(b) passes information from the parent cell to newly 106. Which of the following pair lack the unit membrane?
formed cell (a) Nucleus and E.R.
(c) maintains the proper shape of the cell and serves as (b) Mitochondria and chloroplast
a protective barrier
(c) Ribosome and nucleolus
(d) helps the cell to make food with the help of
(d) Golgi body and lysosome
chlorophyll and sunlight
EBD_7209
80 Biology
107. The main organelle involved in modification and routing by the student is listed in the table below, on the basis of which
of newly synthesized proteins to their destinations is the student infers that the samples contain the organelles.
(a) chloroplast (b) mitochondria
Sample A Sample B
(c) lysosome (d) endoplasmic reticulum
Make energy available for Generates ATP and
108. Active transport differs from passive transport in that
cellular metabolism synthesizes sugar
active transport
Absent in cell that carry Present in plant cell
(a) requires energy. oxygen throughout the body
(b) always requires input of ATP. Called the energy currency of Source of all the food
(c) moves molecules against a concentration gradient. cell energy
(d) both (a) and (c) 114. Identify the part highlighted in the sample A and B and
109. Microtubules, motor proteins, and actin filaments are explain why they were called as semi-autonomous
all part of the organelles?
(a) mechanism of photosynthesis that occurs in (a) Sample A - Mitochondria, Sample B - Chloroplast;
chloroplasts. because both the organelles are double membrane
bound structure.
(b) rough ER in prokaryotic cells. (b) Sample A- Mitochondria, Sample B - Chloroplast;
(c) cytoskeleton of eukaryotic cells. because they both are capable of synthesis of their
(d) process that moves small molecules across cell own proteins only.
membranes. (c) Sample A - Mitochondria, Sample B - Chloroplast;
because they are capable of synthesis of their own
110. You are asked to examine a cell using a powerful light proteins and contain their own DNA.
microscope. The image you see has a clearly defined (d) Sample A- Mitochondria, Sample B - Chloroplast;
nucleus and mitochondria. It also has a large central because they contain their own DNA to transfer the
vacuole and chloroplasts. From what group of organisms genetic information from one generation to another.
did this cell most likely come? 115. Explain why the samples were belonged to eukaryotic
(a) Bacteria (b) Protists cell and not prokaryotic cell? Because,
(c) Fungi (d) Plants (a) eukaryotic cell have membrane bound organelles.
(b) eukaryotic cell have non - membrane bound
111. The best way to identify a cell as either prokaryotic or
organelles.
eukaryotic is to determine whether
(c) eukaryotic cell are smaller and multiply more
(a) it came from a single-celled or multicellular rapidly than prokaryotic cells.
organism. (d) eukaryotic cell are larger and multiply more rapidly
(b) it has a nucleus. than prokaryotic cells.
(c) it has a plasma membrane. 116. A student placed two cells in the same solution in two
different containers. The observation was given in the table.
(d) it has cytosol.
112. Which of the following organelles is directly connected Container Observation
to the outer membrane of the nucleus in a eukaryotic 1 Cell burst
cell? 2 Cell does not change its
(a) Mitochondrion (b) Lysosome shape
(c) Golgi apparatus (d) Endoplasmic reticulum Which structure maintains the shape of the cell present
in container 2 and provides the most significant
113. Centrifugation of a cell results in the rupture of the cell
difference between the two cells?
membrane and the contents compacting into a pellets in
(a) Nucleus (b) Cell wall
the bottom of the centrifuge tube. Bathing this pellet (c) Chloroplast (d) Cell membrane
with a glucose solution yields metabolic activity 117. The cell theory was given in year 1839 by Schleiden and
including the production of ATP. One of the contents of Schwann. According to this theory all organisms are
this pellet is most likely which of the following? composed of cell and cells are the basic unit of life.
(a) Cytosol (b) Mitochondria How did this theory help in the field of science?
(c) Lysosomes (d) Golgi bodies (a) It helped to study the working of cells.
(b) It helped in curing diseases caused by cell.
Directions for Q 114 and 115: A student was given cell (c) It helped in restating the earlier theories on cell.
samples (A and B) to identify parts which are highlighted. He (d) It helped in introducing the use of microscopes to
observed the samples under the microscope and list down the study cell.
function of the part of cell sample. The information collected
9

Chapter
Biomolecules

11. Glycerol is a
FACT/DEFINITION TYPE QUESTIONS
(a) tetrahydroxy propane (b) trihydroxy propane
1. The acid used for preliminary separation of biomolecule (c) trihydroxy butane (d) tetrahydroxy butane
in a living tissue is 12. A fat has
(a) trichlorobenzoic acid (b) benzoic acid (a) 3 glycerol and one fatty acid molecule.
(c) trichloroacetic acid (d) acetic acid (b) one glycerol and 3 fatty acid molecules.
2. Biomolecules are (c) 3 glycerol and 3 fatty acid molecules.
(a) organic compounds (d) one glycerol and one fatty acid molecule.
(b) inorganic compounds 13. Lecithin is a
(c) volatile compounds (a) phospholipid (b) carbohydrate
(d) both (a) and (b) (c) protein (d) amino acid
3. Which of the following is the example of acidic amino 14. A nucleotide is formed of
acid ? (a) purine, pyrimidine and phosphate
(a) Lysine (b) Glutamic acid (b) purine, sugar and phosphate
(c) Aspartic acid (d) Both (b) and (c) (c) nitrogen base, sugar and phosphate
4. All the following amino acids are aromatic, except (d) pyrimidine, sugar and phosphate
(a) tyrosine (b) phenylalanine 15. Building block of nucleic acid is ________.
(c) tryptophan (d) valine (a) nucleotide (b) nucleoside
5. The most basic amino acid is (c) amino acid (d) fatty acid
(a) arginine (b) histidine 16. Inulin is a polymer of
(c) glycine (d) glutamine (a) glucose (b) galactose
6. The simplest amino acid is _________. (c) fructose (d) arabinose
(a) glycine (b) proline 17. Primary structure of protein is due to the presence of
(c) leucine (d) tryptophan ______.
7. The charged molecule which is electrically neutral is (a) hydrogen bonds (b) peptide bonds
known as (c) – S – S – linkages (d) ionic bonds
(a) amino acid (b) zwitterion 18. Most abundant protein in human body is
(c) amide (d) peptide (a) collagen (b) myosin
8. Unsaturated fatty acids have (c) actin (d) albumin
(a) palmitic acid (b) oleic acid 19. Quaternary structure is present in ________.
(c) high melting point (d) one or more double (a) haemoglobin (b) histone
bonds (c) globulin (d) elastin
9. Which of the following is/are essential fatty acid(s)? 20. The bond between phosphate and hydroxyl group of
(a) Linoleic acid (b) Linolenic acid sugar in nucleic acid is _______.
(c) Arachidonic acid (d) All of these (a) glycosidic bond (b) peptide bond
10. Phospholipids are important cell membrane constituents, (c) ester bond (d) none of these
because they 21. Turn over number of enzyme depends upon
(a) contain glycerol. (a) size of enzyme molecule.
(b) can form bilayers in water. (b) number of the active sites.
(c) combine covalently with protein. (c) concentration of substrate molecule.
(d) contain polar and non-polar portions. (d) molecular weight of as enzyme.
EBD_7209
82 Biology
22. The Km value of the enzyme is the value of the substrate 31. Select the incorrect statement.
concentration at which the reaction reaches to (a) Ribozymes are nucleic acids with catalytic power.
(a) zero (b) 2 Vmax (b) Proteins are homopolymer made of amino acids.
(c) ½ Vmax (d) ¼ Vmax (c) Inulin is a polymer of fructose.
23. Ribozyme is a/an (d) Glycogen is stored in liver and muscles.
(a) enzyme whose cofactor is RNA. 32. Pick out the correct statement.
(b) RNA with enzyme activity. (a) Chitin is a homopolymer.
(c) enzyme by which RNA is formed. (b) Collagen is the most abundant protein in the whole
(d) conjugated protein having ribose sugar. of the biosphere.
24. Feedback inhibition of enzymes is affected by (c) Proteins are linear chains of amino acids linked by
(a) enzyme (b) substrate peptide bonds.
(c) end product (d) intermediate end (d) In a polysaccharide, the individual monosaccharides
products are linked by a phosphodiester bond.
25. Co-enzyme is 33. Which of the following statement is wrong regarding
(a) always a protein. chitin?
(b) often a metal. (a) It is a storage of polysaccharide.
(c) always an inorganic compound. (b) It is a heteropolysaccharide.
(d) often a vitamin. (c) It is a constituent of arthropods and fungal cell wall.
26. Which one of the following statements is correct, with (d) It is a second most abundant carbohydrate on earth.
reference to enzymes ? 34. Which of the following statement is incorrect regarding
(a) Holoenzyme = Apoenzyme + Coenzyme secondary metabolites ?
(b) Coenzyme = Apoenzyme + Holoenzyme (a) Plant tissues produce only secondary metabolities.
(c) Holoenzyme = Coenzyme + Co-factor (b) Secondary metabolities have restricted distribution
(d) Apoenzyme = Holoenzyme + Coenzyme in the plant kingdoms.
27. Inorganic catalyst work efficiently at ________ (c) Abrin and ricin are secondary metabolities used as a
temperature and ________ pressure. drugs.
(a) high, low (b) low, low (d) Some secondary metabolites have ecological
(c) low, high (d) high, high importance.
35. Which of the following statements about enzymes is
STATEMENT TYPE QUESTIONS incorrect?
28. Which of the following statement is not true about amino (a) Enzymes are denatured at high temperature but in
acid? certain exceptional organisms. They are effective
(a) The amino group and carboxylic group of an amino even at 80° - 90°C.
acid are attached to both amino and carboxylic groups (b) Enzymes require optimum pH for maximal activity.
to alpha carbon. (c) Most enzymes are proteins but some are lipids.
(b) Amino acids have an N-terminus, C-terminus, and (d) Enzymes are highly specific.
R groups. 36. Which statement is incorrect about coenzyme ?
(c) Amino acid can be classified by the number of peptide (a) Every coenzyme is a cofactor and every cofactor is a
bond. coenzyme.
(d) Essential amino acids are not synthesized in the (b) Every coenzyme is a cofactor and every cofactor is
body, therefore have to be provided in the diet. not a coenzyme.
29. Which of the following statement is incorrect ? (c) Most of the coenzymes are nucleotides and are
(a) Long chain molecules of fatty acids are formed by composed of vitamins.
polymerization of 2 carbon compounds. (d) Coenzymes are the active constituents of enzyme.
(b) Lipid molecules are soluble in water. 37. Study the given statements and select the correct option.
(c) In lipid, R group may be –CH3 group, –C2H5 group (i) Nitrogen bases are open chain hydrocarbons.
or higher number of –CH2 group (1 to 19 carbon). (ii) A nucleoside differ from nucleotide in not having
(d) Oils have lower melting temperature. phosphate.
30. Select the incorrect statement. (iii) One turn of B-DNA has ten base pairs.
(a) Amino acids are substituents methanes. (iv) Length of one turn of DNA is 3.4 Å.
(b) Glycerol is a trihydroxy propane. (a) Statements (i) and (ii) are correct.
(c) Lysine is a neutral amino acid. (b) Statements (i) and (iv) are correct.
(d) Lecithin is a phospholipid. (c) Statements (ii) and (iii) are incorrect.
(d) Statements (i) and (iv) are incorrect.
Biomolecules 83

38. Which of the following statements are correct?


ASSERTION/REASON TYPE QUESTIONS
(i) Chitin, a complex or heteropolysaccharide occuring
in exoskeleton consists of NAG.
(ii) Glucosamine and N-acetyl glucosamine are modified In the following questions, a statement of Assertion is followed
sugar. by a statement of Reason.
(iii) Cellulose shows blue colour when treated with I2. (a) If both Assertion and Reason are true and the Reason is
the correct explanation of the Assertion.
(iv) Starch in plants and glycogen in animals are store
houses of energy. (b) If both Assertion and Reason are true but the Reason is
(v) Right end of polysaccharide is called reducing end not the correct explanation of the Assertion.
while left end is called non-reducing end. (c) If Assertion is true but Reason is false.
(d) If both Assertion and Reason are false.
(a) (i), (ii) and (iii) (b) (iii), (iv) and (v)
(c) All except (iii) (d) All except (v) 43. Assertion: A coenzymes or metal ions that is very
39. Select false statements for an enzyme promoting a chemical tightly bound to enzyme protein is called prosthetic
reaction by group.
(i) lowering the energy of activation. Reason: A complete, catalytically active enzyme together
(ii) causing the release of heat, which acts as a primer. with its bound prosthetic group is called apoenzyme.
(iii) increasing molecular motion. 44. Assertion: Glycosidic bonds are formed by dehydration.
(iv) changing the free energy difference between Reason: In polysaccharides, individual monosaccharide
substrate and product. is linked by glycosidic bond.
(a) (i) and (iv) (b) (ii) and (iii)
45. Assertion: Human diet should compulsorily contain
(c) (ii), (iii) and (iv) (d) (iii) and (iv)
40. The steps in catalytic cycle of an enzyme action are given glycine, serine and tyrosine.
in random order. Reason: Essential amino acids can be synthesized in the
(i) The enzyme releases the products. Now enzyme is human body.
free to bind another substrate. 46. Assertion : Unsaturated fats are more reactive compared
(ii) The active sites, now in close proximity of substrate with the saturated fats.
breaks the bond of substrate and forms E-P complex. Reason : Unsaturated fats have only single bonds in their
(iii) Binding of substrate induces the enzyme to alter its structure.
shape fitting more tightly around the substrate. 47. Assertion : In a DNA molecule, A–T rich parts melt before
(iv) The substrate binds to the active site of enzyme (i.e.,
G–C rich parts.
fitting into the active site).
The correct order is Reason: In between A and T there are three
(a) (i), (ii), (iii), (iv) (b) (iv), (iii), (ii), (i) H–bond, whereas in between G and C there are two H-
(c) (i), (iii), (ii), (iv) (d) (i), (ii), (iv), (iii) bonds.
41. Which of the following statement(s) is/are incorrect? 48. Assertion: The amino acid glycine comes under the
(i) Nearly 1/3rd of all enzymes requires the presence of category of nonessential amino acids.
metal ions for catalytic function. Reason: This is due to the fact that it can not be
(ii) Metal ions form coordinate bond with side chain at synthesised in the body.
active site of metalloenzyme and at the same time
form one or more coordinate bonds with the MATCHING TYPE QUESTIONS
substrate.
(iii) NAD and NAD (coenzymes) contain niacin (vit.B3) 49. Match column I (organic compound) with column II
(iv) Coenzymes are organic compounds but their (examples) and choose the correct combination from the
association with the apoenzyme is only transient, given options.
usually occurring during catalysis. Column-I Column-II
(a) All of these (b) Only (iii) and (iv) (Organic Compounds) (Examples)
(c) Only (iv) (d) None of these A. Fatty acid I. Glutamic acid
42. Which of the given option is correct for the following
B. Phospholipid II. Tryptophan
statements ?
(i) The metabolic pathway in which acetic acid is C. Aromatic amino acid III. Lecithin
converted into cholesterol is an endothermic one. D. Acidic amino acid IV. Palmitic acid
(ii) Anabolic pathway is endergonic while catabolic (a) A – I; B – II; C – III, D – IV
pathway is exergonic. (b) A – IV; B – III; C – II, D – I
(iii) Without metabolism there can not be a living state. (c) A – II; B – III; C – IV, D – I
(a) All are correct (b) All are wrong (d) A – III; B – IV; C – I, D – II
(c) (i) and (ii) are correct (d) Only (iii) is correct
EBD_7209
84 Biology
50. Match the biomoecules given in column I with their 54. Match column I with column II and choose the correct
examples given in column II and choose the correct answer. option.
Column I Column II Column-I Column-II
(Biomolecules) (Examples) A. Cotton fibre I. Chitin
A. Carbohydrates I. Trypsin B. Exoskeleton of II. Glycogen
cockroach
B. Protein II. Cholesterol
C. Liver III. Starch
C. Nucleic acid III. Insulin
D. Peeled potato IV. Cellulose
D. Lipid IV. Adenylic acid (a) A – I; B – IV; C – III; D – II
(a) A – III; B – I; C – IV, D – II (b) A – IV; B – I; C – II; D – III
(b) A – II; B – III; C – IV, D – I (c) A – IV; B – I; C – III; D – II
(c) A – III; B – IV; C – I, D – II (d) A – I; B – IV; C – II; D – III
(d) A – IV; B – I; C – II, D – III 55. Match column I (function) with column II (Types of
51. Match the protein given in column I with its function enzymes) and select the correct option.
given in column II and choose the right option. Column I Column II
Column I Column II (Function) (Types of enzymes)
(Proteins) (Functions) A. Enzymes catalysing I. Isomerases
A. Collagen I. Glucose transport breakdown without
B. Trypsin II. Hormone addition of water.
B. Enzyme catalyzes the II. Oxidoreductase
C. Insulin III. Intercellular ground
conversion of
substance
an aldose sugar to
D. GLUT-4 IV. Enzyme a ketose sugar.
(a) A – III; B – IV; C – II; D – I C. Enzyme where III. Ligases
(b) A – IV; B – I; C – II; D – III catalysis involves
(c) A – II; B – IV; C – I; D – III transfer of electrons.
(d) A – III; B – IV; C – I; D – II D. Enzyme catalysing IV. Lyases
52. Match the column-I (component) with column-II (% of bonding of two
the total cellular mass) and identify the correct option. components with the
Column-I Column-II help of ATP.
(Component) (% of the total (a) A – I; B – IV; C – III; D – II
cellular mass) (b) A – I; B – IV; C – II; D – III
A. Water I. 3 (c) A – IV; B – I; C – II; D – III
B. Proteins II. 70-90 (d) A – IV; B – I; C – III; D – II
56. Which one of the following pairs of nitrogenous bases of
C. Carbohydrates III. 2
nucleic acids, is wrongly matched with the category
D. Lipids IV. 5-7
mentioned against it ?
E. Nucleic acids V. 10-15 (a) Adenine, thymine – Purines
(a) A – V; B – II; C – III; D – IV; E – I (b) Thymine, uracil – Pyrimidines
(b) A – II; B – V; C – I; D – III; E – IV (c) Uracil, cytosine – Pyrimidines
(c) A – III; B – I; C – IV; D – V; E – II (d) Guanine, adenine – Purines
(d) A – V; B – IV; C – III; D – II; E – I 57. Mark the odd pair in the followings
53. Match column I (category) with column II (secondary (a) Amino acid – Protein
metabolites)and choose the correct option. (b) Nucleotide – DNA
Column I Column II (c) Glycerol – Fatty acid
(Category) (Secondary metabolites) (d) Monosaccharide – Cellulose
A. Pigments I. Concanavalin A 58. Which one of the following is wrongly matched?
B. Terpenoides II. Monoterpenes, Diterpenes (a) Fungi – Chitin
(b) Phospholipid – Plasma membrane
C. Alkaloids III. Morphine, Cadeine
(c) Enzyme – Lipopolysaccharide
D. Lectins IV. Carotenoids, Anthocyanin
(d) ATP – Nucleotide derivative
(a) A – IV; B – II; C – III; D – I 59. Find out the wrongly matched pair
(b) A – IV; B – III; C – II; D – I (a) Primary metabolite – Ribose
(c) A – I; B – IV; C – III; D – II (b) Secondary metabolite – Anthocyanins
(d) A – I; B – III; C – II; D – IV (c) Chitin – Polysaccharide
(d) Cellulose – Heteropolymer
Biomolecules 85
60. Choose the mismatch pair 66. Identify the following molecule.
(a) Glutamic acid – Acidic
(b) Lysine – Basic
(c) Valine – Charged
(d) Phenylalanine – Aromatic
61. Find out the odd one from the given pair.
(a) Polysaccharide – Glycosidic bond
(b) Protein – Peptide bond
(c) Fat – Ester HO
(d) Polynucleotide – Hydrogen bond
(a) Phospholipid (b) Lecithin
62. Which of the following is wrongly matched ?
(c) Cholesterol (d) Oleic acid
(a) Ribozyme – Proteinaceous in nature.
(b) Apoenzyme – The protein part of enzyme. 67. Refer the given structure of adenylc acid. In this identify
(c) Co-enzyme – Loosely attached organic A.
cofactor of haloenzyme O
|| O
(d) Co-factors – Non-protein part of haloenzyme. HO – P – OCH2 N-base
63. Find out the correct match. |
OH A
(a) Inulin – Polymer of glucose
(b) Starch – Spiral secondary structure
(c) Cellulose – Component of cell wall
(d) Glycogen – Monosaccharide and reserved food of
plants (a) Glycosidic bond (b) Phosphate bond
(c) Ester bond (d) Ionic bond
DIAGRAM TYPE QUESTIONS 68. The adjoining graph shows change in concentration of
substrate on enzyme activity. Identify A, B and C.
64. Which of the two groups (marked as 1, 2, 3, & 4) of
following formula involved in peptide bond between Y
different amino acids? A
V
H2
| C
1
H 2 N—C—COOH 3
|
R4
[S] X
(a) 2 and 3 (b) 1 and 3 B
(c) 1 and 4 (d) 2 and 4
65. What kinds of the structures of proteins are shown in the
A B C
given figure (A, B, C and D)?
(a) Ki Km Vmax
Vmax
(b) Km Ki
N C 2
Vmax
(c) Vmax Km
2
A B Vmax
(d) Km Vmax
2

69. The given structural formulas represent amino acids


(labelled by as X, Y and Z)
N

C
C D H CH3 CH2 – OH
(a) A = 1º structure, B = 2º structure, C = 3º structure, X Y Z
D = 4º structure
(b) A = 4º structure, B = 2º structure, C = 3º structure, Identify the correct name of X, Y and Z.
D = 1º structure (a) Alanine, serine and glycine respectively.
(c) A = 1º structure, B = 4º structure, C = 3º structure, (b) Tyrosine, cysteine and glutamic acid respectively.
D = 2º structure (c) Glycine, alanine and serine acid respectively.
(d) A = 4º structure, B = 3º structure, C = 2º structure, (d) Cysteine, glutamic acid and tyrosine respectively.
D = 1º structure
EBD_7209
86 Biology
70. The structural formula given below belongs to 73. Which one of the given graph shows the effect of pH on
the velocity of a typical enzymatic reaction (V) ?
CH2OH

V V
(a) (b)
HO OH pH pH

OH OH
(c) V (d) V
(a) glucose (b) ribose
(c) sucrose (d) deoxyribose pH pH
71. The given graph shows concept of activation energy with 74. The curve given below shows enzymatic activity with
labelled 1, 2, 3, & 4. Co-relate the statements I, II, III & IV relation to three conditions (pH, temperature and substrate
with 1, 2, 3 & 4. concentration.) Identify the correct representation of two
axes (x and y).
3

1
Energy

A+B 4

}
C 2

Progress of the reaction


x - axis y-axis
I. Segment representing the energy of activation. (a) Enzymatic activity pH
II. Segment representing the amount of free energy (b) Temperature Enzymatic activity
released by the reaction. (c) Substrate concentration Enzymatic activity
III. Transition state. (d) Enzymatic activity Temperature
IV. Segment would be the same regardless of whether 75. The given diagrammatic representation shows one of the
the reaction were uncatalysed or catalysed. categories of small molecular weight organic compounds
in the living tissues. Identify the category shown and
I II III IV one blank component marked as “X” in it.
(a) 1 3 2 4 HOCH2 O X
(b) 1 2 3 2
(c) 1 3 2 4
(d) 1 2 4 3 OH OH
72. What is denoted by X and Y in the given graph ? Category Component (X)
(a) Cholesterol Guanine
(b) Amino acid NH2
Y
Potential energy

(c) Nucleotide Adenine


X (d) Nucleoside Uracil
76. Which one of the following structural formulae (A & B) of
two organic compounds is correctly identified along with
its related function?
Progress of reaction

X Y O
||
(a) Activation energy Activation energy with A. O CH2 – O – C – R
without enzyme enzyme || |
(b) Activation energy Activation energy R2 – C – O – CH
|
with enzyme without enzyme CH2 – O – P – O – CH2 – CH2
(c) Substrate concentration Substrate concentration | |
with enzyme without enzyme OH N
|
|

(d) Substrate Substrate concentration CH3 | CH3


CH3
concentration with enzyme
without enzyme
Biomolecules 87

82. Relation between amino acid and protein is similar to the


one found between
(a) glucose and fructose
N (b) thymine and uracil
B. (c) nucleosides and nucleic acid
(d) nucleotides and nucleic acid
N 83. Nucleotides are building blocks of nucleic acids. Each
nucleotide is a composite molecule formed by
(a) B : Adenine — A nucleotide that makes up nucleic
(a) base-sugar-phosphate.
acids. (b) base-sugar-OH.
(b) A: Triglyceride — Major source of energy (c) (base-sugar-phosphate)n.
(c) B : Uracil — A component of DNA (d) sugar-phosphate.
(d) A: Lecithin — A component of cell membrane 84. Antiparallel strands of a DNA molecule means that
77. The given graph shows the effect of substrate (a) the phosphate groups of two DNA strands, at their
concentration on the rate of reaction of the enzyme ends share the same position.
greengram-phosphatase. What does the graph indicate? (b) the phosphate groups at the start of two DNA
strands are in opposite position (pole).
(c) one strand turns clockwise.
(d) one strand turns anti-clockwise.
Velocity

85. Which of the following is not an attribute of enzymes ?


(a) They are substrate specific in nature.
(b) They are proteinaceous in nature.
O Substrate concentration (c) They are used up in the reaction.
(d) They speed up rate of biochemical reaction.
(a) The rate of enzyme reaction is directly proportional 86. “All enzymes are proteins.” This statement is now
to the substrate concentration. modified because an apparent exception to this biological
(b) Presence of an enzyme inhibitor in the reaction truth is
mixture. (a) arylsulfatase (b) dehydrogenase
(c) Formation of an enzyme-substrate complex. (c) ribozyme (d) nitroreductase
(d) At higher substrate concentration the pH increases. 87. Carbohydrates, the most abundant biomolecule on earth,
are produced by
78. Which of the following shows zwitter ionic form?
(a) some bacteria, algae and green plant cells.
R R (b) fungi, algae and green plant cells.
+ | + | (c) all bacteria, fungi and algae.
(a) H3 N - CH - COOH (b) H3 N - CH - COO - (d) viruses, fungi and bacteria.
88. Which one of the following biomolecules is correctly
R characterized?
| (a) Lecithin - A phosphorylated glyceride found in cell
(c) H 2 N - CH - COO - (d) None of these membrane.
(b) Palmitic acid - An unsaturated fatty acid with 18
CRITICAL THINKING TYPE QUESTIONS carbon atoms.
(c) Adenylic acid - Adenosine with a glucose phosphate
79. According to weight percentage, the first three elements molecule.
in human body are (d) Alanine amino acid - Contains an amino group and
(a) C > H > O (b) C > O > N an acidic group anywhere in the molecule.
(c) O > N > C (d) O > C > H 89. The effectiveness of an enzyme is affected least by
80. In the composition of cellular mass, arrange the (a) temperature.
components- proteins(P), carbohydrates(C), lipids(L) and (b) concentration of the substrate.
nucleic acids(N) in decreasing order of mass percentage. (c) original activation energy of the system.
(d) concentration of the enzyme.
(a) C > N > P > L (b) P > N > C > L
90. Which of the following set is correctly match to the
(c) P > C > L > N (d) P > N > L > C
category mentioned against them ?
81. If all the peptide bonds of a protein are broken down, (a) Lysine, glycine, thiamine – amino acids.
then what would remain? (b) Arachidonic acid, acetic acid, palmitic acid – Fatty
(a) Amino acids (b) Peptides acids
(c) Polypeptides (d) Oligopeptides (c) Thymidine, uridine, cytidilic acid – Nucleosides
(d) Cellulose, inulin, glycogen – Polysaccharides
EBD_7209
88 Biology
91. The stored form of sugar in animal is a 103. When triglycerides are solid at ordinary room temperature,
(a) homopolysaccharide (b) heteropolysaccharide they are called
(c) oligosaccharide (d) diasaccharide (a) oils (b) gases
92. Select the type of enzyme involved in the following (c) fats (d) none of these
reaction. 104. Natural lipids are readily soluble in
(a) oil (b) water
S - G + S' ® S + S'- G (c) mercury (d) none of these
(a) Dehydrogenase (b) Transferase 105. Enzyme that break nucleic acids into nucleotides belongs
(c) Hydrolase (d) Lyase to which of the following class ?
93. Transition state structure of the substrate formed during (a) Oxidoreductases (b) Transferases
an enzymatic reaction is. (c) Hydrolases (d) Lyases
(a) permanent but unstable. (b) transient and 106. Enzymes are sensitive to
unstable. (a) light (b) pH
(c) permanent and stable. (d) transient but stable. (c) temperature (d) both (b) and (c)
94. For a protein to have a quaternary structure, it must 107. Primary structure of polypeptide is stabilized or secondary
(a) have four amino acids. structure of polypeptide is maintained by
(b) consist of two or more polypeptide subunits. (a) disulphide bonds.
(c) consist of four polypeptide subunits. (b) ionic bonds.
(d) have at least four disulphide bridges. (c) hydrogen bonds.
95. The quaternary structure of human haemoglobin is best (d) hydrophobic interactions.
described as a 108. The catalytic efficiency of two different enzymes can be
(a) dimer of identical subunits. compared by the
(b) dimer of different subunits. (a) Km value.
(c) tetramer of four different subunits. (b) pH optimum value.
(c) formation of the product.
(d) tetramer of two different subunits.
(d) molecular size of the enzyme.
96. At temperature near freezing point, the enzymes are
109. What will happen when the cofactor is removed from the
(a) inactivated (b) activated
enzyme?
(c) slightly activated (d) slightly inactivated
(a) Catalytic activity of the enzyme is lost.
97. A nucleoside differs from nucleotide is not having
(b) Enzyme preserves in a temporarily inactive state.
(a) sugar (b) nitrogen
(c) The substurate molecules are not closely related to
(c) phosphate (d) phosphate and sugar
enzymes molecules.
98. Hexokinase (Glucose + ATP ® Glucose 6-P + ADP)
(d) Both b and c
belongs to the category
110. "When glucose is degraded to lactic acid in skeletal
(a) transferases (b) lyases
muscles energy is liberated".
(c) oxidoreductases (d) isomerases
Which of the following conclusion is correctly associated
99. Sugar-phosphate backbone in nucleic acid
with the above statement? enzyme?
(a) is hydrophilic in nature. (a) It is a catabolic pathway which releases energy.
(b) is negatively charged. (b) It is an anabolic pathway which releases energy.
(c) shows polarity. (c) The energy, liberated during this degradation, is
(d) all of the above. trapped and stored in the form of chemical bonds
100. On hydrolysis, a nucleoside would not yield (ADP).
(a) purine (b) pyrimidine (d) None of the above.
(c) pentose sugar (d) phosphoric acid 111. Why the velocity of enzymatic reaction is not exceeded
101. The information in a genetic nucleic acid resides in the by any further rise in the concentration of the substrate?
(a) number of nucleotides. (a) The enzyme molecules are equal to the substrate
(b) kinds of nucleotides. molecules.
(c) sequence of nucleotides. (b) The enzyme molecules are fewer than the substrate
(d) all of the above. molecules
102. Km is related to (c) The substrate molecules are not closely resembled
(a) temperature (b) ES complex to the enzyme molecules.
(c) pH (d) none of these (d) Both b and c.
Cell Cycle and
10

Chapter
Cell Division

10. Which of the following cellular structures always


FACT/DEFINITION TYPE QUESTIONS
disappears during mitosis and meiosis?
1. In cell cycle, DNA replication takes place in ______ (a) Plastid and nuclear membrane
(a) G1 phase (b) G2 phase (b) Nucleolus and nuclear membrane
(c) Mitotic metaphase (d) S phase (c) Endoplasmic reticulum and mitochondria
2. Which phase of cell cycle is known as quiescent stage ? (d) Endoplasmic reticulum and plasma membrane
(a) M phase (b) G0 phase 11. In meiosis, division is
(c) G1 phase (d) S phase (a) Ist reductional and IInd equational.
3. Interphase includes all of the following except ______ (b) Ist equational and IInd reductional.
(c) both reductional.
(a) G1 phase (b) Anaphase
(d) both equational.
(c) S phase (d) G2 phase
12. Interkinesis is a
4. Which of the following phase is called the resting phase (a) stage between meiosis I and meiosis II.
during which cell is preparing for division by undergoing (b) stage between two mitotic divisions.
both cell growth and replication of DNA? (c) interphase.
(a) M – phase (b) Prophase (d) both (b) and (c)
(c) Go phase (d) Interphase 13. Synapsis occurs between
5. Which of the following phase follows S and G2 phases of (a) spindle fibres and centromeres.
interphase? (b) mRNA and ribosomes.
(a) Prophase (b) Metaphase (c) a male and female gamete.
(d) two homologous chromosomes.
(c) Anaphase (d) Telophase
14. During mitosis, nuclear envelope, nucleolous begin to
6. In mitosis, nucleolus and nuclear membrane disappear
form and ER to reappear at ______
at
(a) prophase (b) late metaphase
(a) interphase (b) prophase (c) anaphase (d) telophase
(c) metaphase (d) telophase 15. The synaptonemal complex appears
7. What is the stage of mitosis in which chromosomes are (a) between homologous chromosomes.
arranged on the equator of spindle? (b) in zygotene stage.
(a) Anaphase (b) Prophase (c) composed of DNA + protein.
(c) Metaphase (d) Telophase (d) All of the above
8. Best stage to observe shape, size and number of 16. Chromosome synapsis or bivalent formation occurs in
chromosomes is ______ ______
(a) interphase (b) metaphase (a) leptotene (b) zygotene
(c) prophase (d) telophase (c) pachytene (d) diplotene
9. In anaphase of mitosis, 17. Crossing over occurs between
(a) sister chromatids of homologous chromosomes.
(a) chromosomes get arranged in middle of cell.
(b) non-sister chromatids of homologous chromosomes.
(b) nuclear envelope disappears.
(c) sister chromatids of non-homologous chromosomes.
(c) chromosome fibres become clear.
(d) non-sister ch romatids of non-homologous
(d) chromosomes aggregate at opposite poles to form chromosomes.
daughter nuclei.
EBD_7209
90 Biology
18. Crossing over occurs during ______ 28. Which of the following statement is not true for
(a) leptotene (b) pachytene homologous chromosome pairs ?
(c) diplotene (d) diakinesis (a) They come from only one of the individual’s parents.
19. Recombination involves (b) They usually contain slightly different versions of
(a) crossing over the same genetic information.
(b) chromosome duplication (c) They segregate from each other during meiosis I.
(c) spindle formation (d) They synapse during meiosis I.
(d) cytokinesis 29. Which of the following statement(s) is/are true ?
20. Chiasmata are first seen in ______ (a) Cell plate represents the middle lamella between the
(a) leptotene (b) zygotene walls of two adjacents cells.
(c) pachytene (d) diplotene (b) At the time of cytokinesis, organelles like
21. In which stage, the chromosomes appear thin and long mitochondria and plastids get distributed between
thread-like ? the daughter cells.
(a) Zygotene (b) Leptotene
(c) Cytokinesis in plant cell is centrifugal and takes place
(c) Pachytene (d) Prophase
by cell-plate formation while animal cells by
22. Terminalization occurs during ______
furrowing/cleavage and is centripetal.
(a) mitosis (b) diakinesis
(d) All of the above
(c) meiosis II (d) cytokinesis
30. Significance of mitosis involves
23. Electron micrographs of which stage indicate that
(a) the growth of multicellular organism.
chromosomes accompanied by the formation of complex
(b) cell repair.
structure called synaptonemal complex?
(c) production of diploid daughter cells with identical
(a) Zygotene (b) Pachytene
genetic complement.
(c) Diplotene (d) Diakinesis
(d) all of the above
24. The microtubules of the mitotic spindle attach to a
31. Which of the following statement(s) is/are correct about
specializes structure in the centromere region of each
S-phase (synthetic phase) ?
chromosome called the __________.
(i) It occurs between G1 and G2 phase.
(a) Telomere (b) Centriole
(ii) It marks the period during which DNA replicates.
(c) Chromatin (d) Kinetochore
(iii) At the end of this phase, DNA is doubled but the
25. Cell plate grows from
number of chromosomes remains unchanged.
(a) wall to centre (b) centre to walls
(c) one wall to another (d) simultaneously (iv) As the DNA is doubled in this phase number of
chromosomes is also doubled.
STATEMENT TYPE QUESTIONS (v) Centrioles replicate in this phase.
(vi) Amount of DNA changes from 2C to 4C.
26. Select the correct statement with respect to mitosis.
(a) (i), (ii), (iv), (v), (vi) (b) (i), (ii), (iii), (v), (vi)
(a) Chromatids separate but remain in the centre of the (c) All of the above (d) Only (iv)
cell in anaphase. 32. Which of the following statements related to G0 stage of
(b) Chromatids start moving towards opposite poles in the cell cycle are correct?
telophase. (i) It is a quiescent stage.
(c) Golgi complex and endoplasmic reticulum are still (ii) In this phase, cell cycle is stopped.
visible at the end of prophase. (iii) G0 cells do not grow or proliferate but metabolically
(d) Chromosomes move to the spindle equator and get active.
aligned along equatorial plate in metaphase. (iv) G0 cells can divide in response to some stimulus.
27. Which one of the following precedes re-formation of the (a) (i), and (ii) (b) (i), (ii) and (iii)
nuclear envelope during M phase of the cell cycle ? (c) Only (i) and (iv) (d) All of the above
(a) Decondensation from chromosomes, and re- 33. Which one is correct about bivalent ?
assembly of the nuclear lamina. (i) Bivalent are tetrads.
(b) Transcription from chromosomes, and re-assembly (ii) A bivalent means 4 chromatids and 2 centromere.
of the nuclear lamina. (iii) One bivalent consists of 2 h omologous
(c) Formation of the contractile ring, and formation of chromosomes each and sister chromatids.
the phragmoplast. (iv) Bivalents formation occurs in zygotene.
(d) Formation of the contractile ring, and transcription (a) All of these (b) Only (iii)
from chromosomes. (c) (iii) and (iv) (d) Only (iv)
Cell Cycle and Cell Division 91

34. Which one is correct about crossing over/genetic 41. Assertion: The final stage of meiotic prophase 1 is
recombination? diplotene.
(i) It occurs in tetrad stage which occurs in pachytene Reason: Diplotene is marked by terminalisation of
of prophase I of meiosis I. chiasmata.
(ii) It occurs between non-sister chromatids of 42. Assertion: The crossing over is an enzyme-mediated
homologous chromosomes. process.
(iii) It is recombinase enzyme mediated process. Reason: The enzyme involved in crossing over is lyase.
(iv) It is also the reciprocal transfer of genes between 43. Assertion: Metaphase II begins with splitting of
the non-homologous chromosomes centromere of each chromosome into two.
(a) All of the above (b) All except (iv) Reason: In Anaphase II chromosomes align at the equator.
(c) (ii), (iii) and (iv) (d) (iii) and (iv)
MATCHING TYPE QUESTIONS
35. Read the following statements about cell division and
select the correct statements. 44. Match the description (given in column I) with correct
(i) M phase represents the phase when actual cell stage of prophase I (given column II) and choose the
division occurs and I phase represents the phase correct option.
between two successive M phase. Column I Column II
(ii) In the 24 hours, average duration of cell cycle of a A. Chromosomes are moved I. Pachytene
human cell, cell division proper lasts for only about to spindle equator
an hour. B. Centromere splits and II. Zygotene
(iii) M phase constitutes more than 95% of the duration chromatids apart
of cell cycle. C. Pairing between homologous III. Anaphase
(a) (i) and (ii) (b) (ii) and (iii) chromosomes takes place
(c) (i) and (iii) (d) (i), (ii) and (iii) D. Crossing between homologous IV. Metaphase
ASSERTION/REASON TYPE QUESTIONS chromosomes
(a) A – I; B – II; C – III; D – IV
In the following questions, a statement of Assertion is followed
by a statement of Reason. (b) A – II; B – III; C – IV; D – I
(a) If both Assertion and Reason are true and the Reason is (c) A – IV; B – III; C – II; D – I
the correct explanation of the Assertion. (d) A – III; B – I; C – IV; D – II
(b) If both Assertion and Reason are true but the Reason is 45. Match the terms (given in column I) with their explanation
not the correct explanation of the Assertion.
(given in column II) and choose the correct combination
(c) If Assertion is true but Reason is false.
from the options given below.
(d) If both Assertion and Reason are false.
Column I Column II
36. Assertion: The stage between two mitotic divisions is
called interkinesis. (Terms) (Explanation)
A. Terminalization I. Pairing of homologous
Reason: Interkinesis is generally short lived.
chromosomes.
37. Assertion: Diplotene is characterized by the presence of
B. Synapsis II. Point of attachment between
chiasmata.
homologous chromosomes.
Reason: Diplotene can last for months and years in
C. Chiasmata III. Nuclear protein complex that
oocytes of some vertebrates.
helps in adherence of sister
38. Assertion: Interphase occupies 75-95% of the total chromatids and then
generation time. homologous chromosomes.
Reason: Interphase (I-phase) is the long non-dividing D. Synaptonemal IV. Shifting of chiasmata
phase. outwards
39. Assertion: Small disc-shaped structures at the surface of complex towards the ends of a
the centromeres are called kinetochores. bivalent.
Reason: Kinetochores serve as the sites of attachment of
(a) A – IV; B – I; C – II; D – III
spindle fibres to the centromeres.
40. Assertion: Karyokinesis follows cytokinesis. (b) A – II; B – III; C – IV; D – I
Reason: Karyokinesis is the division of cytoplasm into (c) A – II; B – IV; C – III; D – I
two daughter cells. (d) A – IV; B – I; C – III; D – II
EBD_7209
92 Biology
46. Match the description given in column-I with their stapes 53. Which of the following pair is correctly matched ?
given in column-II and identify the correct answer. (a) Anaphase I – Homologous chromosomes are
Column-I Column-II separated.
A. Initiation of the assembly I. Anaphase (b) Metaphase I – Pairing of maternal and paternal
of mitotic spindle chromosomes.
B. Proteins are synthesized in II. Prophase (c) Interphase – A nuclear envelop encloses each
preparation for mitosis while cell haploid set of chromosomes.
growth continues. (d) Prophase I – Non-homologous chromosomes are
C. Spindle fibres attach to III. Interphase separated.
kinetochores of chromosomes.
D. Movement of chromatids IV. Metaphase DIAGRAM TYPE QUESTIONS
towards opposite poles
The correct match is 54. Given below is a schematic break-up of the phases / stages
(a) A – II; B – III; C – IV; D – I of cell cycle with few parts labelled as A, B, C & D.
(b) A – III; B – II; C – I; D – IV
(c) A – I; B – III; C – II; D – IV
(d) A – IV; B – III; C – I; D – II
47. Select the correctly matched pair.
A. S phase – DNA replication
B. Zygotene – Synapsis
C. Diplotene – Crossing over
D. Meiosis – Both haploid and diploid cells
(a) A and B (b) C and D
(c) B and D (d) A and C
48. Find the correctly matched pairs and choose the correct
option.
A. Zygotene – Pairing of homologous
chromosomes Which one of the following is the correct indication of
B. Pachytene – Dissolution of the complex the stage/phase in the cell cycle?
synaptonemal takes place (a). C - Karyokinesis (b) D - Synthetic phase
C. Diplotene – Bivalent chromosomes appear as
(c) A - Cytokinesis (d) B - Metaphase
tetrads
D. Diakinesis – Terminalization of chiasmata 55. Which stages of cell division do the following figures A
takes place and B represent respectively?
(a) A and C (b) A and D
(c) A and B (d) B and C
49. Choose the mismatch pair.
(a) Karyokinesis – Division of centromere
(b) Cytokinesis – Division of cytoplasm
(c) S-phase – DNA synthesis
(d) Synapsis – Pairing of homologous
chromosomes
50. Find out the wrongly matched pair of the stage of Fig. A Fig. B
prophase I with their feature. (a) A - Metaphase ; B - Telophase
(a) Zygotene – Synaptonemal complex
(b) Pachytene – Mutation (b) A - Telophase ; B - Metaphase
(c) Diplotene – Chiasmata (c) A - Late Anaphase ; B - Prophase
(d) Diakinesis – Terminalization (d) A - Prophase ; B - Anaphase
51. Mark the incorrect pair.
(a) Interphase – S-phase 56. Given below is the representation of a certain event at a
(b) M phase – Metaphase particular stage of a type of cell division. Which stage is
(c) Meiosis I – Diplotene shown by the given figure?
(d) Meiosis II – Diakinesis
52. Which of the following pair is correctly matched ?
(a) G1 – Second growth phase
(b) S – Duplication phase
(c) G2 – Synthesis phase
(d) G0 – Quiescent stage
Cell Cycle and Cell Division 93

(a) Prophase I during meiosis. (a) Late prophase – Chromosomes move to spindle
(b) Prophase II during meiosis. equator.
(c) Prophase of mitosis. (b) Metaphase – Spindle fibres attached to kinetochores,
(d) Both prophase and metaphase of mitosis. centromeres split and chromatids separate
57. A stage in cell division is shown in the figure. Select the (c) Metaphase – chromosomes moved to spindle
answer which gives correct identification of the stage equator chromosomes made up of two sister
with its characteristics. chromatids
(d) Anaphase – centromeres split and chromatids
separate and start moving away
60. Choose the diagram which correctly depicts anaphase I.

(a) Late anaphase Chromosomes move away from


equatorial plate, Golgi complex
is not present.
(a) (b)
(b) Cytokinesis Cell plate is formed,
mitochondria distributed
between two daughter cells.
(c) Telophase Endoplasmic reticulum and
nucleolus not reformed yet
(d) Telophase Nuclear envelop reforms,
golgi complex reforms
58. The following diagram is of a typical cell cycle indicating
formation of two cells from one cell.
(c) (d)
61. Identify the given figures (A, B & C) showing meiotic
phases and select the correct option.
X Y
G0

es is
Cytokin se
p ha
e
Telo phas ase
a h
An etap
se
ha
M

M
op

Z
Ph

Pr
as

A B
e

Choose the correct option showing the correct


identification of X, Y, & Z.
(a) X - G1; Y - S; Z - G2 (b) X - G2; Y - S; Z - G1
(c) X - G0; Y - S; Z - G2 (d) X - G1; Y - G0; Z - S
59. A stage of mitosis is shown in the given diagram. Identify
stage with its characteristics?

C
(a) A-Metaphase B-Anaphase C-Telophase
(b) A-Metaphase-I B-Anaphase-I C-Telophase-I
(c) A-Metaphase-II B-Anaphase-II C-Telophase-II
(d) A-Anaphase-I B-Metaphase-I C-Telophase-I
EBD_7209
94 Biology
62. Identify the phases from the graph given below that shows (b) Pachytene and interphase (just prior to prophase I).
the change in DNA content during various phases (A to (c) Pachytene and S phase (of interphase just prior to
D) of mitotic cell cycle. prophase I).
(d) Zygotene and S phase (of interphase prior to
prophase I).
DNA content
68. Significance of meiosis lies in
4C (a) reduction of chromosome number to one half.
(b) maintaining consistancy of chromosome number
2C during sexual reproduction.
A B C D
(c) production of genetic variability.
(d) all of the above.
Time 69. How many meiotic divisions are required to produce 100
A B C D pollen grains ?
(a) 25 (b) 50
(a) G2 G1 S M (c) 100 (d) 125
70. Mitosis and meiosis take place respectively in
(b) G2 S G1 M (a) meristem and gametangia.
(b) gametangia and meristem.
(c) G1 S G2 M (c) permanent tissues and secretory tissues.
(d) secretory tissues and permanent tissues.
(d) M G1 S G2 71. If you are provided with root-tips of onion in your class
and are asked to count the chromosomes, which of the
63. The diagram shows a cell whose diploid chromosome following stages can you most conveniently look into?
number is four. Which one of the following option shows (a) Metaphase (b) Telophase
correct stage of cell ? (c) Anaphase (d) Prophase
72. Which of the following is the result when karyokinesis is
not followed by cytokinesis ?
(a) Synaptonemal complex
(b) Syncytium
(c) Recombination nodules
(a) Metaphase (d) Terminalization
(b) Anaphase of mitosis 73. Which of the following occurs only once in life cycle ?
(c) Anaphase I of meiosis (a) Replication of histone.
(d) Anaphase II of meiosis (b) Replication of chromosomes.
(c) Mitosis
CRITICAL THINKING TYPE QUESTIONS (d) Meiosis
64. In the somatic cell cycle 74. “G0” state of cells in eukaryotic cell cycle denotes
(a) In G1 phase, DNA content is double the amount of (a) check point before entering the next phase.
DNA present in the original cell. (b) pausing in the middle of a cycle to cope with a
(b) DNA replication takes place in S-phase. temporary delay.
(c) a short interphase is followed by a long mitotic phase. (c) death of a cell.
(d) G2 phase follows mitotic phase. (d) exit of cells from cell cycle.
65. In which stage of the cell cycle, histone proteins are 75. Mitosis occurs in
synthesized in a eukaryotic cell? (a) haploid individuals. (b) diploid individuals.
(a) During G2 stage of prophase. (c) Both (a) and (b) (d) in bacteria only.
(b) During S-phase. 76. Mitosis is the process by which eukaryotic cells
(c) During entire prophase. (a) expose the genes for protein synthesis.
(d) During telophase. (b) become specialized in structure and function.
66. How many mitotic divisions are needed for a single cell to (c) multiply.
make 128 cells? (d) grow.
(a) 7 (b) 14 77. The major event that occurs during the anaphase of
(c) 28 (d) 64 mitosis, which brings about the equal distribution of
67. During which stages (or prophase I substages) of meiosis chromosomes is
do you expect to find the bivalents and DNA replication (a) replication of the genetic material.
respectively? (b) splitting of the chromatids.
(a) Pachytene and interphase (between two meiotic (c) splitting of the centromeres.
divisions). (d) condensation of the chromatin.
Cell Cycle and Cell Division 95

78. In animal cells, cytokinesis involves 88. At which stage, the homologous chromosomes separate
(a) the separation of sister chromatids. due to repulsion, but are yet held by chiasmata ?
(b) the contraction of the contractile ring of (a) Zygotene (b) Pachytene
microfilament. (c) Diplotene (d) Diakinesis
(c) depolymerization of kinetochore microtubules. 89. A triploid nucleus cannot undergo meiosis because
(d) a protein kinase that phosphorylates other enzymes. (a) crossing over does not occur.
79. Which of the following will show simple cell division? (b) karyokinesis can not occur.
(a) Microspore mother cells (c) chromosomes do not replicate.
(b) Megaspore mother cells (d) not all of the chromosomes can form homologous
(c) Archesporial cells pairs.
(d) All of the above. 90. Crossing over
80. In Meiosis, (a) is important in genetic recombination.
(a) division of nucleus twice but DNA replicates only (b) makes a cell become cancerous.
once. (c) is a key process that occurs during mitosis.
(b) division of nucleus twice and DNA replicates twice. (d) is an important mechanism of chromosome repair.
(c) division of nucleus once and replication of DNA is 91. It is important that the centromere not divide till the end
also once. of metaphase because it
(d) division of nucleus once and DNA - replication is
(a) is connected with nuclear membrane.
twice.
(b) contains genes that control prophase.
81. Four daughter cells formed after meiosis are
(c) holds the replicated DNA molecules together.
(a) genetically similar (b) genetically different
(d) produces spindle fibres.
(c) anucleate (d) multinucleate
92. Which of the following does not lead to genetic
82. Number of bivalents are 8 in prophase I. What is the
number of chromosomes during anaphase II ? variability?
(a) 8 (b) 4 (a) Random fertilization.
(c) 13 (d) 32 (b) Crossing over during meiosis.
83. In meiosis, the daughter cells differ from parent cell as (c) Division of chromosomes during anaphase of
well as amongst themselves due to mitosis.
(a) segregation, independent assortment and crossing (d) Orientation of chromosomes during metaphase I of
over. meiosis.
(b) segregation and crossing over. 93. Which of the following carry the same genetic information?
(c) independent assortment and crossing over. (a) Sister chromatids
(d) segregation and independent assortment. (b) X and Y chromosomes
84. The separation of two chromatids of each chromosome at (c) All autosomes
early anaphase is initiated by (d) Homologous chromosomes
(a) the interaction of centromere with the chromosomal 94. 'X' ensures the production of 'Y' phase in the life cycle of
fibres. sexually reproducing organisms whereas fertilization
(b) the elongation of metaphasic spindle. restores the 'Z' phase.
(c) the force of repulsion between the divided Identify X, Y and Z.
kinetochores. . (a) X- Mitosis, Y - haploid, Z - haploid
(d) All of the above. (b) X- Mitosis, Y - diploid, Z - diploid
85. Cell would normally proceed to mitosis without interruption (c) X- Meiosis, Y - haploid, Z - diploid
(a) once it has entered the S phase. (d) X- Meiosis, Y - diploid, Z - diploid
(b) once it has entered the G2 phase. 95. Which of the following is responsible for the formation of
(c) at any time during cell division activity. the new cell wall and also represents the middle lamella
(d) none of the above between the walls of two adjacent cells?
86. Identify the meiotic stage in which the homologous (a) Cell plate (b) Cell inclusion
chromosomes separate while the sister chromatids remain (c) Cell membrane (d) None of the above
associated at their centromeres. 96. The following events occur during the process of meiosis.
(a) Metaphase I (b) Metaphase II Arrange the following events of meiosis in the correct
(c) Anaphase I (d) Anaphase II sequence.
87. In telophase of mitosis, the mitotic spindle breaks down I. Terminalization
and nuclear membranes form. This is essentially the II. Crossing over
opposite of what happens in III. Synapsis
(a) prophase (b) interphase IV. Disjunction of genomes
(c) metaphase (d) S phase (a) I, II, III, IV (b) II, III, IV, I
(c) III, II, I, IV (d) IV, I, II, III
EBD_7209
11

Chapter
Transport in Plants

8. A cell swells up when kept in


FACT/DEFINITION TYPE QUESTIONS
(a) hypotonic solution (b) hypertonic solution
1. In a fully turgid cell (c) isotonic solution (d) any of the three
(a) ys will be negative and yp will be positive. 9. Bulk flow of substances over the longer distances through
(b) yp will be negative and ys will be positive. the vascular tissue is called
(c) Both yp and ys will be positive. (a) simple diffusion (b) facilitated diffusion
(d) Both ys and yp will be negative. (c) active transport (d) translocation
2. Which of the following pairs of the cell structures are 10. Movement that is aided by cytoplasmic streaming and
important for determining the movement of molecules in occurs from cell to cell through plasmodesmata is called
or out of the plant cell? ________.
(a) apoplast (b) symplast
(a) Tonoplast + Vacuolar membrane
(c) active transport (d) translocation
(b) Tonoplast + Cell membrane
11. Casparian strip is made up of
(c) Cell wall + Cell membrane
(a) lignin (b) pectin
(d) Cell wall + Tonoplasts
(c) suberin (d) cellulose
3. The process in which water moves out of the cell and the
cell membrane of a plant cell shrinks away from its cell wall 12. At which cell layer, water movement through the apoplast
is known as pathway is restricted and is facilitated towards symplast
pathway?
(a) diffusion (b) osmosis
(a) Cortex (b) Pericycle
(c) plasmolysis (d) bulk flow
(c) Epidermis (d) Endodermis
4. Phenomenon of plasmolysis occurs when
13. Which of the following organism helps in the absorption
(a) cells are kept in hypertonic solution. of water and mineral ions from the soil?
(b) cells are kept in hypotonic solution. (a) Nostoc (b) Anabaena
(c) cells are kept in hypotonic solution. (c) Mycorrhiza (d) Spirullina
(d) none of the above. 14. A pressure that is responsible for pushing up water to
5. When a cell is plasmolysed, it becomes small height in the stem is called
(a) flaccid and its TP becomes zero. (a) positive root pressure
(b) turgid and its TP becomes zero. (b) turgor pressure
(c) turgid and TP becomes equal to OP. (c) pressure gradient
(d) flaccid and DPD becomes zero. (d) negative root pressure
6. Seed increase in its volume by the absorption of water 15. Guttation is loss of impure water which is the result of
through (a) osmosis (b) diffusion
(a) osmosis (b) diffusion (c) root pressure (d) transpiration
(c) imbibition (d) plasmolysis 16. Which of the following is the most acceptable theory for
7. The process by which water is absorbed by solids like movement of water through plants?
colloids causing them to increase in volume is called (a) Cohesion theory (b) Passive transport
_______. (c) Root pressure (d) Capillarity
(a) osmosis (b) plasmolysis 17. The force responsible for upward conduction of water
(c) imbibition (d) diffusion against gravity comes from _______.
Transport in Plants 97

(a) transpiration (b) translocation 29. Water in the adjacent xylem moves into the phloem by the
(c) respiration (d) photosynthesis process of
18. Guard cells help in (a) facilitated diffusion (b) acute transport
(a) protection against grazing (c) simple diffusion (d) osmosis
(b) transpiration 30. Phloem tissue is composed of sieve tube cells, which form
(c) guttation long columns with holes in their end walls called ______.
(d) fighting against infection (a) tracheids (b) sieve elements
19. Which of the following wall of guard cells is thick? (c) sieve plate (d) companion cell
(a) Side wall (b) Middle wall STATEMENT TYPE QUESTIONS
(c) Inner (d) Outer
20. When a root absorbes minerals from a region of lower 31. Find out the incorrect statements.
concentration to a region of higher concentration, and (a) The process of plasmolysis is usually irreversible.
need energy then this type of absorption is called
(b) The pressure exerted by the protoplasts due to entry
(a) passive absorption (b) facilitated diffusion of water against the rigid walls is called yp.
(c) active absorption (d) osmosis (c) The T.P. is responsible for enlargement and extension
21. In root endodermis there is one way active transport of growth of cells.
ions because of presence of (d) Plant cells swell in hypotonic solutions.
(a) pericycle (b) suberin layer
32. Which of the following the statements regarding
(c) cortex (d) cellulosic layer mycorrhizae is incorrect?
22. Sinks are related to (a) Mycorrhizal fungi form a network around the young
(a) transport of minerals (b) stomata root and they penetrate the root cells.
(c) hydathodes (d) phytochrome (b) Mycorrhizae helps the plant to absorb water and
23. Translocation of food in flowering plants occurs in the minerals.
form of (c) Root provides sugar and nitrogenous organic
(a) starch (b) glyceraldehyde compounds to the mycorrhizae.
(c) glucose (d) sucrose (d) Pinus seed can germinate and establish without
24. During fruit development, photosynthesizing leaves would mycorrhizae.
be the __________ and the fruit would be __________ 33. Which of the following statement is incorrect regarding
(a) sink, sink (b) source, source stomata?
(c) sink, source (d) source, sink (a) It helps in exchange of oxygen and carbon dioxide in
25. Bidirectional translocation of minerals takes place in the leaf.
(a) xylem (b) phloem (b) It is open in the day time and close during the night.
(c) parenchyma (d) cambium (c) Opening or closing of the stomata is a change in the
26. _______is mainly water and sucrose, but other sugars, turgidity of the guard cells.
hormones and amino acids are also _______ through (d) The inner wall of each guard cell, towards the stomatal
phloem. aperture is thin and rigid.
(a) xylem sap, loaded 34. Which of the following statement is correct?
(b) phloem sap, translocated (a) Unlike water, all minerals cannot be passively
(c) xylem sap, translocated absorbed by roots.
(d) phloem sap, loaded (b) Most of the minerals enter the root by active transport.
27. The hypothesis accepted for the translocation of sugar (c) Ions are absorbed from soil by both passive and active
from source to sink is__________. transport.
(a) pressure gradient (d) All of the above
(b) pressure flow hypothesis 35. Which of the following statements is/are not incorrect?
(c) mass flow hypothesis (i) Water and minerals, and food are generally moved
(d) both (b) and (c) by a mass or bulk flow system.
28. The process of loading at the source produces a (ii) Bulk flow can be achieved either through a positive
_________. condition in the phloem. hydrostatic pressure gradient or a negative
(a) hypertonic (b) isotonic hydrostatic pressure gradient.
(c) hypotonic (d) hydroponic (iii) The bulk movement of substances through the
conducting tissues of plants is called translocation.
EBD_7209
98 Biology
(iv) Xylem translocates organic and inorganic solutes,
ASSERTION/REASON TYPE QUESTIONS
mainly from roots to the aerial parts of the plants.
(v) Phloem translocates water, mineral salts, some In the following questions, a statement of Assertion is followed
organic nitrogen and hormones, from the leaves to by a statement of Reason.
other parts of the plants. (a) If both Assertion and Reason are true and the Reason is
(a) (ii), (iii) and (v) (b) (ii), (iii) and (iv) the correct explanation of the Assertion.
(c) (iv) and (v) (d) (ii) and (v) (b) If both Assertion and Reason are true but the Reason is
36. Which of the following statements is/are correct? not the correct explanation of the Assertion.
(i) The apoplastic movement of water occurs exclusively (c) If Assertion is true but Reason is false.
through the cell wall without crossing any (d) If both Assertion and Reason are false.
membranes. 40. Assertion : In symport both molecules cross the
(ii) The apoplastic movement occurs from cell to cell membrane in the same direction at the same time.
through the plasmodesmata. Reason : In antiport both molecules move in the opposite
(iii) Endodermis is impervious to water because of a band direction.
of suberised matrix. 41. Assertion : More is the number of solute molecules, the
(iv) Symplastic movement may be aided by cytoplasmic lower is y w .
streaming which occurs in Hydrilla leaf and Reason : Presence of solute particles reduces the free
chloroplast. energy of water and decreases the water potential.
(a) (i) and (ii) (b) (ii) and (iv) 42. Assertion : Osmosis is a special type of diffusion through
(c) (i), (iii) and (iv) (d) (i), (ii) and (iv) a semipermeable membrane.
37. Which of the following factors affect transpiration? Reason : The net rate and direction of osmosis depends
upon the pressure gradient.
(i) Number and distribution of stomata.
43. Assertion : Stomata are usually open in the day time and
(ii) Percent of open stomata.
close during the night.
(iii) Water status of the plant. Reason : The opening or closing of stomata is due to the
(iv) Canopy structure. change in the turgidity of the guard cells.
(a) (i) and (ii) (b) (i), (ii) and (iii) 44. Assertion : Cohesion, adhesion and surface tension give
(c) (ii) and (iv) (d) (i), (ii), (iii) and (iv) high tensile strength to water.
38. Which of the following statements, (i -v) regarding Reason : Capillarity is aided by small diameter of the
transpiration is/are correct? tracheary elements.
(i) It creates transpiration pull for absorption and 45. Assertion : Ions are absorbed from the soil only byactive transport.
transport of plants. Reason : The proteins present in the membranes of root
(ii) It supplies water for photosynthesis. hair cells passively pump ions from the soil into the
(iii) It transports minerals from the soil to all parts of the cytoplasm of the epidermal cells.
plants.
MATCHING TYPE QUESTIONS
(iv) It heats leaf surfaces, sometimes 10 to 15 degrees.
(v) It maintains the shape and structure of the plants by 46. Match column-I with column-II and find out the correct
keeping cells turgid. option from the codes given below.
(a) Only (ii) (b) Only (iii) Column - I Column - II
(c) (i), (ii), (iii) and (v) (d) All
A . Isotonic I. External s olution is more
39. The following statements are associated with translocation concentrated
of mineral ions. B. Hypotonic II. Shrinkage of protoplasm
(i) Mineral ions are slowly remobilised.
(ii) Younger leaves export most of their minerals content C. Hypertonic III. Solution is more dilute
to older leaves. than the cytoplasm
(iii) Elements most readily mobilised are P, S, N and K.
D. Plas mo lys is IV Two s olutions h ave the
(iv) Some elements that are structural components like s ame os molarity
calcium are not remobilised.
Which of the above statements are correct? (a) A – II; B – I; C – IV; D – III
(b) A – IV; B – III; C – I; D – II
(a) (i) and (iii) (b) (i) and (ii)
(c) (iii) and (iv) (d) (ii) and (iii) (c) A – III; B – I; C – IV; D – II
(d) A – II; B – III; C – IV; D – I
Transport in Plants 99

47. Match the name of the activities given under column-I


DIAGRAM TYPE QUESTIONS
with the description of activity given under column-II and
choose the correct option. 50. In the given figure, chamber A and B are separated by a
Column-I Column-II semipermeable membrane. Study the given figure and
A. Cohesion I. The ability to rise in choose the right option.
their tubes.
B. Adhesion II. Loss of water
vapour from plant
parts.
C. Tensile strength III. Mutual attraction
Semi-
between water
permeable
molecules membrane
D. Capillarity IV. Attraction of water
molecules to polar
(a) Chamber A has higher water potential and water will
surfaces
move from A to B.
V. An ability to resist a
(b) Chamber B has lower solute potential and water will
pulling force.
move from A to B.
(a) A – I; B – II; C – III; D – V
(c) Chamber A has higher solute potential and water will
(b) A – II; B – I; C – IV; D – III move from B to A.
(c) A – III; B – IV; C – V; D – I (d) Chamber B has lower water potential and water will
(d) A – IV; B – V; C – II; D – III move from B to A.
48. Match the name of the activities given under column-I 51. Study the experiment shown below :
with the description of activity given under column-II and
choose the correct option.
X
Column-I Column-II
A. Transpiration I. Anaerobic respiration in Stands
yeast
B. Guttation II. Active absorption of water Beaker Thistle funnel

C. Exudation III. Loss of water vapour Y Sugar solution


Semipermeable membrane
from plant parts
Pure water
D. Fermentation IV. Loss of liquid water from
leaves
V. Loss of water from After a few days, which of the following will have occured ?
injured plant parts (a) A rise in level X and Y
(a) A – I; B – II; C – III; D – V (b) A drop in level X and level Y
(b) A – II; B – I; C – IV; D – III (c) A rise in level X and a drop in level Y
(c) A – III; B – IV; C – V; D – I (d) A drop in level X and a rise in level Y
(d) A – IV; B – V; C – II; D – III 52. Choose the option which shows the correct labelling of
49. Match column-I with column-II and find out the correct the parts marked as A, B, C, D and E in the given figure of
answer from the code given below. water movement in the leaf.
Column-I Column -II
A. Diffusion I. Hydrophilic substances
B. Osmosis II. Shrinkage of protoplasm C
C. Imbibition III. Semipermeable A
membrane
D. Plasmolysis IV. Free movement of ions B
and gases
(a) A – II; B – I; C –IV; D – III
(b) A – IV; B – III; C – I; D – II D
E
Diffusion into
(c) A – III; B – I; C – IV; D – II surrounding air
(d) A – II; B – III; C – IV; D – I
EBD_7209
100 Biology
(a) A - Tracheids, B - Phloem, C - Mesophyll, D - Stomatal (a) water moves from cell A to cell B.
pore, E - Guard cell (b) there is no movement of water between cell A and
(b) A - Phloem, B - Xylem, C - Palisade, D - Guard Cell, cell B.
E - Water pore (c) water moves from cell B to cell A.
(c) A - Xylem, B - Phloem, C - Palisade, D - Guard cell, E (d) equal amount of water is simultaneously exchanged
- Stomatal pore between cell A and cell B.
(d) A - Phloem, B - Xylem, C - Mesophyll cell, D - guard 56 . A plant cell placed in pure water will
cell, E - Water pore (a) expand until the osmotic potential or solute potential
53. Based on Munch’s pressure-flow hypothesis shown in reaches that of water.
the given figure which of the following conditions would (b) becomes more turgid until the pressure potential of
increase the rate of translocation? cell reaches its osmotic potential.
(c) become more turgid until osmotic potential reaches
that of pure water.
(d) becomes less turgid until the osmotic potential
reaches that of pure water.
57. A boy is studying transport of a certain type of molecules
into cell. He finds that transport slows down when the
cells are poisoned with a chemical that inhibits energy
Sieve-tube elements
production. Under normal circumstances the molecules
studied by the boy is probably transported by
(a) simple diffusion (b) osmosis
(c) active transport (d) facilitated diffusion
58. Osmosis is a form of diffusion in which
(a) the solvent moves through a semipermeable
Companion sink (root cell)
cell membrane from its region of higher chemical potential
(a) An increase in the humidity in the outside air. to its region of lower chemical potential.
(b) A decrease in phloem unloading at the sink. (b) the solvent moves through a semipermeable
membrane from its region of lower chemical potential
(c) An increase in sucrose production at the source.
to its region of higher chemical potential.
(d) A decrease in photosynthesis.
(c) the solute moves through a semipermeable membrane from
54. The given figure represents symplastic and apoplastic
a region of higher concentration to lower concentration.
pathways of water & ion absorption & movement in roots.
Few parts are marked as A, B, C & D. At the endodermis, (d) the solute moves through a semipermeable membrane
water movement through the apoplast pathway is from a region of lower concentration to higher
obstructed by which part (marked as A, B, C & D)? concentration.
59. Water will move from its region of higher chemical potential
to its region of lower chemical potential until
(a) equilibrium is reached.
(b) amount of both solvent and solute in both regions
become equal.
(c) solvent amount in both regions become equal.
(d) solute amount in both regions become equal.
60. Read the given statement and answer the question :
“Osmosis is the diffusion of a solution of a weaker concentration
(a) A (b) B
when both are separated by semipermeable membrane”.
(c) C (d) D
What is the error in the statement ?
CRITICAL THINKING TYPE QUESTIONS (a) The movement of solvent molecule is not specified.
(b) There is no mention of DPD.
55. Cell A and cell B are adjacent plant cells. In cell A, (c) Behaviour of semipermeable membrane is not
ys = – 20 bars and yp = 8 bars. In cell B, ys = – 12 bars and specified.
yp = 2 bars. Then (d) The exact concentration of solutions are not
indicated.
Transport in Plants 101

61. The net direction and rate of osmosis depends on both (a) xylem (b) phloem
the ‘X’ and ‘Y’. Identify ‘X’ and ‘Y’. (c) sieve tube (d) casparian strip
(a) X - Solute; Y - Solvent 69. The movement of water from one cell of the cortex to the
(b) X - Pressure potential; Y - Solute potential adjacent one in roots is due to
(c) X - Water potential; Y - Pressure gradient (a) water potential gradient.
(d) X - Pressure gradient; Y - Concentration gradient (b) chemical potential gradient.
62. Bacteria cannot survive in a highly salted pickle because (c) turgor pressure.
(a) salt inhibits reproduction of bacteria. (d) mass flow.
(b) they become plasmolysed and death occurs. 70. ‘X’ breaks the continuity of the ‘Y’ pathway and forces
(c) nutrients in pickle cannot support life. water and solutes to cross the endodermis by passing
(d) enough light is not available for photosynthesis. through the plasma membrane. Identify ‘X’ and ‘Y’.
63. A cell is said to be flaccid when (a) X - Suberin; Y - Tonoplast
(a) there is no net flow of water towards the inside or (b) X - Suberin; Y - Symplast
outside. (c) X - Casparian strip ; Y - Tonoplast
(b) the external solution balances the osmotic pressure (d) X - Casparian strip ; Y - Apoplast
of the cytoplasm. 71. The path of water from soil upto secondary xylem is
(c) water flows into the cell and out of the cell and are in (a) Soil ® Root hair cell wall ® Cortex ® Endodermis
equilibrium. ® Pericycle ® Protoxylem ® Metaxylem
(d) the external solution is more dilute than the (b) Metaxylem ® Protoxylem ® Cortex ® Soil
cytoplasm. ® Root hair
64. Process of imbibition results in (c) Cortex ® Root hair ® Endodermis ® Pericycle
(a) increase in the volume of imbibant but without ® Protoxylem ® Metaxylem
development of pressure. (d) Pericycle ® Soil ® Root hair ® Cortex ® Endodermis
(b) decrease in the volume of imbibant and development ® Protoxylem ® Metaxylem
of pressure. 72. A Botanist discovered a mutant plant that was unable to
(c) no change in volume of imbibant but pressure produce materials that form casparian strip. This plant would
develops. (a) unable to transport water or solutes to the leaves.
(d) increases in volume of imbibant and development of (b) unable to use its sugar as a sugar sink.
pressure.
(c) able to exert greater root pressure than the normal
65. Seeds when soaked in water, they imbibe because of plant.
(a) OP inside the seed is low. (d) unable to control amounts of water and solutes it
(b) OP of water is high. absorbs.
(c) DPD of seed is very much low. 73. Which of the following is correct regarding guttation?
(d) water potential gradient between the seed coat and (a) It occurs through stomata.
water. (b) It occurs through hydathodes.
66. A bottle filled with previously moistened mustard seeds (c) It occurs mostly during night and early morning.
and water was screw capped tightly and kept in a corner. It (d) Both (b) and (c)
blew up suddenly after about half an hour. The
74. Which one of the following is not related to guttation?
phenomenon involved in this is
(a) Water is given out in the form of droplets.
(a) diffusion (b) imbibition
(b) Water given out is impure.
(c) osmosis (d) D.P.D.
(c) Water is given out during daytime.
67. Dry wooden stakes, if driven into a small crack in a rock
and then soaked, can develop enough pressure to split (d) Guttation is a process of universal occurrence.
the rock. Such a pressure is built up through the 75. Which of the following compound is used to study water
phenomenon of loss from a leaf and turns colour on absorbing water?
(a) imbibition (b) transpiration (a) Calcium chloride (b) Magnesium chloride
(c) turgor pressure (d) plasmolysis (c) Cobalt chloride (d) Sodium chloride
68. A boy has taken fresh twig from a tree and then he placed 76. Stomata in angiosperms open and close due to
it into a coloured water. After a few hours he cut the surface (a) their genetic constitution.
of the twig and examine it with a magnifying glass to study (b) effect of hormone.
the path of water movement. This experiment demonstrates (c) changes of turgor pressure in guard cells.
that movement of water occurs through (d) pressure of gases inside the leaves.
EBD_7209
102 Biology
77. Stomata closes because 81. Transpiration facilitates
(a) guard cells lose turgidity and becomes flaccid (a) electrolyte balance
(b) of increased turgidity of the guard cells brought (b) absorption of water by roots
about by exposure to light. (c) opening of stomata
(c) O.P. of the guard cell increases (d) excretion of minerals.
(d) of the movement of water from neighbouring cells 82. Which of the following will affect the active uptake of
into guard cells. water?
78. Arrange the following events in a correct order that explains (a) Transpirational power of the root hairs.
the mass flow of materials in the phloem? (b) Typical tissue organization.
(i) Water diffuses into the sieve tube elements. (c) Tension due to transpiration.
(ii) Leaf cells produce sugar by photosynthesis. (d) Osmotic concentration of the cell sap of the leaves.
(iii) Solutes are actively transported into the sieve 83. In part A of a plant, sugars are actively transported into
elements. the phloem tissue. In part B, sugars are actively
(iv) Sugar is transported from cell to cell in the leaf. transported out of the phloem. Which way will the phloem
(v) Sugar moves down the stem. sap move under these conditions?
(a) (ii) – (iv) – (iii) – (i) – (v) (a) From A to B.
(b) (ii) – (iv) – (i) – (iii) – (v) (b) From B to A.
(c) (i) – (ii) – (iii) – (iv) – (v) (c) First from A to B; then, once the pressure builds up,
(d) (iv) – (ii) – (i) – (iii) – (v) from B to A.
79. If a stem is girdled, (d) First from B to A; then, once the pressure builds up,
(a) root dies first. from A to B.
(b) shoot dies first. 84. If the external solutions balance the osmotic pressure of
(c) both die together. cytoplasm, it is said to be
(d) none of the above. (a) isotonic (b) hypotonic
80. Stomata opens when (c) atomic (d) hypertonic
(a) guard cells swell due to a decrease in their water
potential.
(b) guard cells swell up due to an increase in their water
potential.
(c) guard cells swell by endosmosis due to efflux of
potassium ions.
(d) guard cells swell by endosmosis due to influx of
hydrogen ions (protons).
12

Chapter
Mineral Nutrition

(a) SO42– (b) Mg2+


FACT/DEFINITION TYPE QUESTIONS
(c) K+ (d) Mo
1. Hydroponics refers to the plant development 9. _________ is an constituent of the ring structure of
(a) without soil. chlorophyll and helps to maintain the ribosome structure.
(b) in saline soil. (a) Manganese (b) Magnesium
(c) in water without soil. (c) Molybdenum (d) Copper
(d) without soil with alkaline pH. 10. Which of the following essential element is required for
2. Which of the following scientist for the first time photochemical reaction involved in photolysis of water?
demonstrated the experiment on hydroponics? (a) Cu2+ (b) Cl –
(a) Von sachs (b) Arnon (c) Zn 2+ (d) Mg2+
(c) Knop (d) Skoog 11. Which of the following elements are constituents of
3. More than ‘X’ elements of the ‘Y’ discovered so far are protein?
found in different plants. Identify ‘X’ and ‘Y’. (a) Nitrogen and phosphorus
(a) X - 0; Y - 110 (b) X - 80; Y - 105 (b) Nitrogen and chlorine
(c) X- 60; Y - 105 (d) X- 70; Y - 115 (c) Phosphorus and boron
4. The essential elements which are required by plants in (d) Chlorine and potassium
large amounts are called________ and those required in 12. Which of the following elements in plants are relatively
very small amount by the plants are called_________. immobile and are a part of the structural component of the
(a) micronutrients, macronutrients cell?
(b) bulky elements, trace elements (a) Sulphur and calcium
(c) macronutrients, micronutrients (b) Sulphur and potassium
(d) trace elements, bulky elements (c) Calcium and magnesium
5. The amount of trace elements per kg dry matter is (d) Potassium and magnesium
(a) 10 m mole (b) above 10 m mole 13. Which of the following element is necessary in plants for
(c) less than 10 m mole (d) 100 m mole protein synthesis and also it is a constituent of hormones
6. The major role of minor elements inside living organisms and many of the vitamins?
is to act as (a) Calcium (b) Phosphorus
(a) binder of cell structure. (c) Nitrogen (d) Magnesium
(b) co-factors of enzymes. 14. Which of the following is not caused by deficiency of
(c) building blocks of important amino acids. mineral nutrition?
(d) constituent of hormones. (a) Necrosis (b) Chlorosis
7. Boron in green plants assists in (c) Etiolation (d) Shortening of
internodes
(a) sugar transport
15. Any mineral ion concentration in tissues that reduces the
(b) activation of enzymes
dry weight of tissues by about _______ is considered
(c) acting as enzyme cofactor
toxic.
(d) photosynthesis
(a) 10% (b) 20%
8. Which of the following element activate an enzyme, (c) 30% (d) 40%
nitrogenase during nitrogen metabolism?
EBD_7209
104 Biology
16. The process by which mineral is absorbed is called 27. Pigment present in the root nodules of legume is
(a) passive absorption (b) active absorption (a) chlorophyll-c (b) fucoxanthin
(c) both (a) and (b) (d) none of these (c) phycoerythrin (d) leghaemoglobin
17. Denitrification is carried out by 28. The primary amino acid from which other 17 amino acids
(a) Nitrosomonas (b) Pseudomonas are formed through the process of transamination is
(c) Nitrobacter (d) Nitrococcus (a) glycine (b) aspartic acid
18. The bond in molecular nitrogen (N2) is difficult to break, (c) glutamic acid (d) arachidonic acid
because it is a 29. Asparagine and glutamine are the two most important
(a) double ionic bond. (a) amino acid (b) amides
(b) quadraplex hydrogen bond. (c) imino acid (d) proteins
(c) triple covalent bond. 30. Nitrogen fixation is a process of
(d) triple ionic bond. (a) converting nitrogen in the air to form a usable form
19. Which of the following represents the abiotic mode to by plants.
convert nitrogen to nitrogen oxides into the soil? (b) recycling nitrogen from organic matter in the soil.
(a) Lightening (b) Temperature (c) absorbing nitrogen from the soil.
(c) Ammonification (d) Nitrification (d) conversion of NO3 to N2 .
20. Conversion of N º N to NH3 occurs in plant cell by
STATEMENT TYPE QUESTIONS
(a) free-living bacteria (b) symbiotic bacteria
(c) anaerobic microbes (d) enzyme 31. Which of the following statements is not correct about
21. The enzyme _______ which is capable of nitrogen macro- nutrients?
reduction is present exclusively in prokaryotes. Such (a) They are present in plant tissues in excess of 100 m
microbes are called_____. mole per kg of dry matter.
(a) hydrogenase, N2-fixers (b) They include C, H, O, N, P, S, K, Ca, Mg.
(b) nitrogenase, N2-fixers (c) Some elements attained from CO2 and H2O while the
(c) hydrogenase, aerobic microbes others are absorbed from the soil.
(d) nitrogenase, aerobic microbes (d) C, H & O are mainly obtained from CO2 and H2O.
22. Frankia produces nitrogen fixing nodules on the roots 32. Which of the following statement is incorrect ?
of (a) Soil supplies minerals, harbours N2-fixing bacteria
(a) leguminous plants (b) n o n - l e g u m i n o u s and other microbes, holds water, supplies O2 to root
plants at acts as matrix that stabilises the plant.
(c) Cycas (d) monocot (b) Both macro and micronutrients forms component of
23. Which one of the following is free-living, anaerobic fertilizers and are applied as per need.
nitrogen-fixer ? (c) Weathering and breaking down of rock enrich the
(a) Beijernickia (b) Rhodospirillum soil with dissolved ions and inorganic salts.
(c) Rhizobium (d) Azotobacter (d) Denitrification is not done by bacteria Pseudomonas
and Thiobacillus.
24. The nodule in a plant root where nitrogen fixing bacteria
live forms from cells of the 33. Which of the following statements about Rhizobium
legume nodule formation is not correct ?
(a) epidermis (b) cortex
(a) Rhizobium can only fix nitrogen after it becomes a
(c) endodermis (d) vascular cylinder
bacteroid within a root cortex cell.
25. Which of the following plants will enrich the soil with
(b) Rhizobium induces invagination of root hairs.
nitrogen?
(c) Within an infection thread, Rhizobium is still
(a) Corn (b) Alfalfa
extracellular to the plant.
(c) Wheat grass (d) Beets
(d) The infection thread can fuse with any root cell of an
26. Leghaemoglobin helps in appropriate legume species.
(a) transport of food in plant. 34. Which of the following statements about nitrification is
(b) nitrogen fixation. not correct ?
(c) protecting nitrogenase from O2. (a) Nitrobacter oxidizes nitrite to nitrate.
(d) nodule formation. (b) Nitrosomonas and Nitrosococcus conver t
ammonium ions to nitrite.
Mineral Nutrition 105

(c) Nitrification reactions are energy-producing (a) (iii) and (iv) (b) (i) and (iv)
(exergonic) reactions. (c) (i) and (iii) (d) All of these
(d) Heterotrophic plants are more directly dependent on 39. Which of the following statement(s) is/are correct?
the nitrifying bacteria for usable nitrogen than (i) Conversion of organic nitrogen to NH4+ by soil
autotrophic plants. microbes is called ammonification.
35. Which of the following statements about nitrogen fixation
(ii) Ammonia is first oxidized to nitrite by Nitrosomonas
is correct ?
and Nitrosococcus.
(i) Nitrogenase is only catalytic under anaerobic
(iii) The nitrite is further oxidized to nitrate with the help
conditions.
of the bacterium Thiobacillus.
(ii) The energy for nitrogen fixation can be provided by
(iv) In leaves, nitrate is reduced to form ammonia that
either photosynthesis or respiration.
finally forms the –NH2 group of amino acids.
(iii) In nitrogen fixation, nitrogen is reduced by the
(v) Nitrosomonas, Nitrosococcus and Nitrobacter are
addition of three successive pairs of hydrogen atoms.
Chemoautotrophs.
(iv) Most nitrogen fixing microbes are aerobic.
(a) (i), (ii) and (iii) (b) (ii), (iii), (iv) and (v)
(a) (i) and (ii) only (b) (ii) and (iv) only
(c) (i), (iii) and (v) (d) (i), (ii), (iv) and (v)
(c) (i), (ii) and (iii) only (d) All of the above
40. Which of the following statements are incorrect ?
36. Read the following statement and answer the question
(i) The morphological changes are indicative of certain
Infected thread carries the bacteria to the inner ‘X’ cells.
element deficiencies and are called deficiency
The bacteria get modified into rod-shaped bacteroids and
symptoms.
cause inner ‘X’ and ‘Y’ cells to divide. Division and growth
of ‘X’ and ‘Y’ cells lead to nodule formation. (ii) The part of plants that show the deficiency
symptoms depend on the mobility of the element in
Identify ‘X’ and ‘Y’.
the plant.
(a) X-pericycle, Y - cortical
(iii) Deficiency symptoms appear first in the young
(b) X - cortical, Y - pericycle tissues whenever the element are relatively mobile.
(c) X - endodermis, Y - cortical (iv) The deficiency symptoms of Cl, Z, N, O, are visible
(d) X - epidermis, Y -pericycle first in the senescent leaves.
37. Which of the following statements are correct? Of the above statements.
(i) Magnesium competes with iron and manganese for (a) (iii) and (iv) (b) (i) and (iii)
uptake and with iron for binding with enzymes. (c) (i) and (iv) (d) (ii) and (iv)
(ii) Magnesium inhibit calcium translocation in shoot 41. Read the following statements (i to v) and answer the
apex. following question.
(iii) Excess of manganese may induce deficiencies of iron, (i) Nitrogen is very essential for the sustenance of life.
magnesium and calcium. (ii) N2 - fixation requires a strong reducing agent.
(iv) Symptoms of manganese toxicity may actually be (iii) N2 - fixation is accomplished with the help of nitrogen
the deficiency symptoms of iron, magnesium and fixing microbes, mainly Frankia.
calcium. (iv) The enzyme nitrogenase which plays an important
(a) (i), (ii) and (iii) (b) (i) and (ii) role in biological N2 fixation is very sensitive to carbon
(c) (iii) and (iv) (d) (ii), (iii) and (iv) dioxide.
38. How many of the given statements are correct? (v) The energy, ATP, required is provided by the
(i) The deficiency of any element can cause multiple respiration of the host cells.
symptoms. How many of the above statements are incorrect?
(ii) Same symptoms may be caused by the deficiency of (a) (i), (ii) and (iii) (b) (iii) and (v)
one or several different elements. (c) (iii) and (iv) (d) (ii), (iv) and (v)
(iii) The concentration of the essential element below 42. Which of the following statements are correct ?
which plant growth is retarded is termed as critical (i) Solution culture/hydroponics contains all essential
concentration. minerals except one, the usefulness of which is to be
(iv) Chlorosis is the loss of chlorophyll due to deficiency determined.
of N, K, Mg, Fe, S, Mn, Zn Mo. (ii) Na, Si, Co and selenium are beneficial element
(v) Different plants respond differently to the deficiency required by higher plants.
of the same element. (iii) Zn is the activator of nitrogenases while Mo is the
EBD_7209
106 Biology
activator of alcohol dehydrogenase. 47. Assertion : Deficiency of sulphur causes chlorosis in plants.
(iv) Zn is needed for auxin synthesis. Reason : Sulphur is a constituent of chlorophyll, protein
(a) (i), (ii), (iii) (b) (i), (ii), (iv) and nucleic acids.
(c) All of these (d) None of these 48. Assertion : Macromtrients are generally present in plant tissues
43. Ion transport in root occurs in large amounts (in excess of 10 mmole kg-1 of dry matter)
(i) passively through channels. Reason : Macronutrient includes iron manganese copper,
(ii) actively through channels. molybdenum, zinc, boron, chlorine and nickel
(iii) actively through carriers. 49. Assertion : Nitrogen is the essential nutrient element
(iv) through both symplast and apoplast. requird by plants in the greatest amount.
(a) (i) and (iii) (b) (ii), (iii) and (iv) Reason : Nitrogen is absorbed mainly as NO3- though
(c) (i), (iii) and (iv) (d) (iii) and (iv) some are also taken up NO2– or NH4+
44. Refer the given statements and answer the question. 50. Assertion : Chlorosis is the loss of chlorophyll leading to
(i) The element must be absolutely necessary for yellowing of leaves.
supporting normal growth and reproduction. Reason : This symptom is caused by the deficiency of
(ii) The requirement of the element must be specific and elements N,K, Mg, S, Fe and Mo.
not replaceable by another element. 51. Assertion :Ammonia is first oxidized to nitrate by the
(iii) The element must be directly involved in the bacteria Nitrosomonas or Nitrococcus.
metabolism of the plant. Reason : These nitryfying Bacteria are chemoheterotrophs .
The above statements apply to
(a) Criteria for hydroponics (b) Criteria for essentiality MATCHING TYPE QUESTIONS
(c) Role of micronutrients (d) Role of macronutrients
52. Match column-I with column-II and choose the correct
45. Which of the following mineral is associated with the option.
characters/functions given below ?
Column - I Column -II
(i) Helps in formation of middle lamella.
(Nutrients) (Functions)
(ii) Needed in mitotic spindle formation.
A. Mg2+ I. Activator of
(iii) Accumulates in older leaves.
dehydrogenase
(iv) Involves in normal functioning of the cell membranes.
B. Zn 2+ II. Activator for both Ru BP
(v) Activate certain enzymes.
carboxylase-
(vi) Plays an important role in regulating metabolic
activities. oxygenase and PEP are
(a) K+ (b) Fe3+ C. K+ III. Required for all
(c) NO3 – (d) Ca2+ Phosphorylation
46. The functions given below are performed by which of the reactions
following mineral ? D. H2PO4 – IV. Plays an important role
(i) An important constituent of proteins involved in in opening and closing
ETS. of stomata
(ii) Activator of catalase. (a) A – II, B – IV, C – I, D – III
(iii) Essential for chlorophyll synthesis. (b) A – II, B – I, C – IV, D – III
(a) N (b) Mg (c) A – III, B – I, C – IV, D – II
(c) Fe (d) Cd (d) A – III, B – I, C – II, D – IV
53. Match the items given in column-I with their examples
ASSERTION/REASON TYPE QUESTIONS given in column-II and choose the correct answer.
Column-I Column-II
In the following questions, a statement of Assertion is followed A. Free living aerobic I. Anabaena and Nostoc
by a statement of Reason.
(a) If both Assertion and Reason are true and the Reason is nitrogen fixers
the correct explanation of the Assertion. B. Anaerobic nitrogen II. Pseudomonas and
(b) If both Assertion and Reason are true but the Reason is fixers Thiobacillus
not the correct explanation of the Assertion. C. Nitrogen fixing III. Nitrosomonas and
(c) If Assertion is true but Reason is false. cyanobacteria Nitrococcus
(d) If both Assertion and Reason are false.
Mineral Nutrition 107

D. Denitrifying bacteria IV. Azotobacter and C. Magnesium III. Nitrate reductase


Beijernickia D. Molybdenum IV. Cysteine
E. Nitrifying bacteria. V. Rhodospirillum (a) A – I, B – II, C – III, D – IV
(a) A – IV, B – V, C – I, D – II, E – III (b) A – III, B – IV, C – I, D – II
(b) A – V, B – IV, C – I, D – III, E – II (c) A – III, B – I, C – II, D – IV
(c) A – IV, B – V, C – II, D – III, E – I (d) A – II, B – IV, C – I, D – III
(d) A – IV, B – III, C – I, D – II, E – V 57. Match the column-I with column-II and choose the correct
54. Match the column-I with column-II and choose the correct option.
option. Column-I Column-II
Column-I Column-II A. P I. Found in some amino acids
A. Manganese I. Component of various
enzymes and participate B. S II. All phosphorylation reaction
in nitrogen metabolism. C. I III. Not important for plants
B. Zinc II. Required for pollen D. Mn IV. Required for photolysis of
germination and water
carbohydrate (a) A – II, B – I, C – III, D – IV
translocation (b) A – I, B – II, C – III, D – IV
C. Molybdenum III. Helps in splitting of (c) A – I, B – III, C – IV, D – II
water to liberate oxygen (d) A – II, B – III, C – I, D – IV
during photosynthesis
58. Which of the following bacteria is correctly matched with
D. Boron IV. Needed in the synthesis
their function ?
of auxin
A. Nitrosomonas – Nitrite to nitrate
(a) A – I, B – IV, C – III, D – IV
B. Thiobacillus – Dentrification
(b) A – III, B – II, C – I, D – IV
C. Nostoc – Free-living nitrogen-fixer
(c) A – III, B – IV, C – I, D – II
D. Azotobacter – Anaerobic nitrogen-fixer
(d) A – IV, B – III, C – II, D – I
(a) A and B (b) C and D
55. Match the column-I containing minerals with the functions
(c) B and C (d) B and D
given in column-II and choose the correct combination
given. 59. Which of the following is the mismatched pair?
Mineral elements Form that is absorbed by plant
Column-I Column-II
(a) Nitrogen NO3–
(Minerals) (Functions)
(b) Phosphorus H2PO4–
A. K I. Stomatal opening
(c) Sulphur H2SO4
B. Mo II. Constituent of cell membrane (d) Iron Fe3+
C. P III. Photolysis of water 60. Which of the following is an incorrect match of essential
D. Mn IV. Free ion element and function?
(a) Manganese - Structural component of chlorophyll.
V. Component of nitrogenase
and nitrate reductase (b) Calcium - Component of the middle lamella.
(c) Zinc - Enzyme activator.
A B C D
(d) Iron - Component of ferredoxin.
(a) I, IV V II III
61. Find the incorrectly matched pair.
(b) I, V IV III II
(a) Rhizobium ® Alfalfa
(c) I , V IV II III
(b) Frankia ® Alnus
(d) IV I III II, V
(c) Rhodospirillum ® Aerobic
56. Match the column-I with column-II and choose the correct (d) Bacillus ® Free -living
combination from the option given below.
Column -I Column-II DIAGRAM TYPE QUESTIONS
A. Zinc I. Chlorophyll
62. The given figure shows a typical set up with their parts
B. Sulphur II. IAA
marked as A, B and C. Identify A, B and C and determine
which experiment is demonstrated in the given figure?
EBD_7209
108 Biology
Nutrient solution is sent to the elevated end of the tube
from the reservoir by X and it flows back to the
reservoir due to Y . Identify X and Y.
(a) pump, pump (b) gravity, gravity
(c) pump, gravity (d) gravity, pump
64. Refer the figure given below and select the option which
gives correct labelling for all the four blanks A, B, C and D.
B
A

A B C D
(a) Funnel for Aerating Nutrient Hydroponics
adding water tube solution
and nutrients
(b) Funnel for Aerating Nutrient Aeroponics
adding water tube solution
A B C D
only
(c) Funnel for Aerating Water Tissue (a) K Ammonification Animal Plant
adding tube culture biomass biomass
nutrients only (b) NH3 Ammonification Plant Animal
(d) Funnel for Aerating Water Hydroponics biomass biomass
adding water tube (c) CO2 Denitrification Animal Plant
and nutrients biomass biomass
63. The given diagram shows hydroponic/soilless plant (d) CHO Nitrification Plant Animal
production. Plants are grown in a tube or trough placed on biomass biomass
a slight incline. The arrows indicate the direction of flow 65. The given diagram shows the development of root nodule
of nutrient solution. in soyabean. Thus structures are marked as A, B, C and D.

D
A

C
B

Identify the correct labelling of A, B, C & D.


(a) A-Rhizobial bacteria; B-Cortex cell; C-Outer cortex;
D-Infection thread containing virus.
(b) A-Rhizobial bacteria; B-Cortex cell; C-Inner cortex
and pericycle cells; D-Infection thread containing
bacteria
(c) A-Rhizobial bacteria; B-Endodermal cell; C-Inner
Mineral Nutrition 109

endodermis; D-Infection thread containing virus X Y Z


(d) A-Nitrosomonas bacteria; B-Cortex cell; C-Inner cortex (a) Magnesium Phosphorus Nitrogen
and pericyele cells; D-Infection thread containing (b) Phosphorus Magnesium Nitrogen
bacteria. (c) Phosphorus Nitrogen Magnesium
CRITICAL THINKING TYPE QUESTIONS (d) Magnesium Nitrogen Phosphorus
74. Some bacteria such as ‘X’ and ‘Y’ occur in soil which
66. Plants absorb nitrogen from soil mainly in the form of reduce nitrate to nitrogen by the process of ‘Z’. Identify
(a) N2-gas (b) nitric acid ‘X’, ‘Y’ and ‘Z’
(c) nitrite (d) nitrate (a) ‘X’-Nitrogen, ‘Y’-Pseudomonas, ‘Z’-Ammonification
67. Which of the following element is involved in plants for (b) ‘X’-Nitrosomonas, ‘Y’-Thiobacillus, ‘Z’-Ammo-
protein synthesis? nification
(a) Potassium (b) Calcium (c) ‘X’-Pseudomonas, ‘Y’-Thiobacillus, ‘Z’-Nitrification
(c) Iron (d) Zinc (d) ‘X’-Pseudomonas, ‘Y’-Thiobacillus, ‘Z’-
68. An important essential element which is required by plants Dentrification
in the greatest amount is 75. Biological nitrogen fixation is the
(a) nitrogen (b) iron (a) reduction of nitrogen to ammonia by living
(c) sulphur (d) copper organisms.
69. The minerals involved in the synthesis of DNA and RNA, (b) oxidation of nitrogen to ammonia by living organism.
for maintenance of the turgidity of cells and for the (c) conversion of nitrogen to ammonia by UV radiation.
activation of the enzyme catalase are respectively (d) conversion of ammonia to nitrogen by electrical
(a) potassium, magnesium, chlorine discharge.
(b) sulphur, potassium, iron 76. All are free-living nitrogen fixers except
(c) phosphorus, potassium, chlorine (a) Azotobacter (b) Beijernickia
(d) magnesium, potassium, iron (c) Anabaena (d) Rhizobium
70. The term critical concentration means 77. Plant absorbs nitrogen from the soil in the form of
(a) essential element concentration below which plant (a) ammonia (b) N2
remains in the vegetative phase. (c) nitrite (d) nitrate
(b) essential element concentration below which the 78. Which of the following is an anaerobic N 2 fixing
plant growth is retarded. bacterium?
(c) essential element concentration above which the (a) Azotobacter (b) Bacillus
plant growth is stunted. (c) Rhodospirillum (d) Beijernickia
(d) non-essential element concentration below which 79. N2 + 8e– + 8H+ + 16 ATP ® 2NH4 + H2 + 16ADP + 16Pi
plant growth is retarded. The above equation refers to
71. Which one of these do plants require for the formation of (a) ammonification (b) nitrification
adenosine triphosphate?
(c) nitrogen fixation (d) denitrification
(a) N, Cu (b) N, Ca
(c) N, P (d) N, K 80. Nitrogen fixation by organisms requires conditions that
72. In an active process, the entry or exit of ions to and from are
the symplast requires (a) aerobic (b) anaerobic
(a) ATP (b) cyclic AMP (c) saturated with sunlight (d) free of water
(c) NADH (d) NADPH 81. Which of the following bacteria can fix nitrogen for plants
73. A small aquatic plant was put in each of the petridishes - such as clover and beans ?
X, Y & Z, containing different culture solutions. After six (a) Denitrovibri (b) Rhizobium
weeks the plant in dish X had the same number of leaves
(c) Pseudomonas (d) Nitrobacter
as it had previously & were all small and yellowish. Plant
in dish Y had more leaves of normal size and dark green 82. At physiological pH, for the formation of ammonium ion,
colour. Plants in dish Z had more leaves of normal size but ammonia is
very pale. Which of the following show the element (a) protonated (b) deprotonated
missing in the culture? (c) carbonylated (d) decarbonylated
EBD_7209
110 Biology
83. Which of the following expression describes nitrogen 89. Minerals are known to enter the plant root by means of a
fixation? number of mechanisms, including all except one of the
(a) N2 + 3H2 ® 2NH3 following. Which one of the following is a mechanism for
(b) 2NH+4+ 2O2 + 8e– ® N2 + 4H2O moving minerals into roots?
(c) 2NH3 ® N2 + 3H2 (a) Foliar feeding (b) Active transport
(d) 2N2 + glucose ® 2 amino acids (c) Proton (H+) pump (d) Cation exchange
84. A gardner purchases a commercial fertilizer. The label says 90. The most abundant gas in our atmosphere cannot be
that it is 10-20-10. This label refers to the utilized by plants directly in its atmospheric form and is,
therefore, captured by certain bacteria that live
(a) percentage of nitrogen, phosphate and potassium.
symbiotically in the nodules of roots. Identify the gas?
(b) percentage of nitrogen, carbon and oxygen.
(a) Oxygen (b) Nitrogen
(c) rate at which nitrogen is released from the fertilizer.
(c) Neon (d) Hydrogen
(d) ratio of organic to inorganic matter in the fertilizer.
91. Legume’s roots have swellings called nodules that
85. The deficiencies of micronutrients, not only affects growth
(a) produce antibiotics that protect the plant from soil
of plants but also vital functions such as photosynthetic
bacteria.
and mitochondrial electron flow. Among the list given
below, which group of three elements shall affect most, (b) provide a steady supply of sugar to the host plant.
both photosynthetic and mitochondrial electron (c) increases the surface area for water uptake.
transport? (d) contain nitrogen-fixing bacteria.
(a) Co, Ni, Mo (b) Ca, K, Na 92. In plants a common symptom caused by deficiencies of
(c) Mn, Co, Ca (d) Cu, Mn, Fe Cu, K, Ca and Mg is the
86. Which of the following can fix atmospheric nitrogen ? (a) formation of anthocyanin.
(a) Albugo (b) Cystopus (b) bending of leaf tip.
(c) Saprolegnia (d) Anabaena (c) poor development of vasculature.
87. Which of the following is a free living aerobic non- (d) appearance of dead necrotic tissues.
photosynthetic nitrogen-fixer? 93. A boy notices that the young leaves of his tomato plants
(a) Rhizobium (b) Azotobacter are very yellow. Deficiency of which of the following
(c) Azospirillum (d) Nostoc nutrient does this suggest ?
88. In plant nutrition, elements are classified as major or minor (a) Nitrogen (b) Carbon
nutrients depending on (c) Water (d) Iron
(a) their availability in the soil. 94. Which of the following groups contain no species that
(b) their relative production in the ash obtained after are able to fix nitrogen ?
burning the plants. (a) Cyanobacteria in the ocean and fresh water.
(c) the relative amounts required by the plants. (b) Soil bacteria including Rhizobium.
(d) their relative importance in plant growth. (c) Cyanobacteria in lichens.
(d) Aerobic bacteria in the genera Bacillus and
Pseudomonas.
13

Chapter
Photosynthesis

FACT/DEFINITION TYPE QUESTIONS (b) Cladophora


(c) purple & green sulphur bacteria
1. Photosynthesis is a/an
(d) blue green algae
(a) physio-chemical process.
7. Which one represents the correct empirical equation of
(b) physical process.
photosynthesis?
(c) chemical process.
(a) C6H12O6 + 6O2 ® 6CO2 + 6H2O + energy
(d) energy wasting process.
(b) C6H12O6 + 6O2 + 6H2O ® 6CO2 + 12H2O + energy
2. Photosynthesis is important because
(a) it is the primary source of food on earth. Light
(c) 6CO2 + 6H2O ¾¾¾® 6O2 + C6H12O6
(b) it is responsible for release of O2 into the atmosphere Chlorophyll
by green plants.
Light
(d) 6CO2 + 12H2O ¾¾¾® 6O2 + C6H12O6 + 6H2O
(c) it is responsible for release of water vapour into the
atmosphere. Chlorophyll
(d) both (a) and (b) 8. How can we separate leaf pigments of any green plant?
3. Half leaf experiment proves that (a) Column chromatography
(a) light is essential for photosynthesis. (b) Paper chromatography
(b) CO2 is essential for photosynthesis. (c) Electrophoresis
(c) O2 releases during photosynthesis. (d) Radio-isotopes
(d) chlorophyll is essential for photosynthesis. 9. What is/are the function(s) of accessory pigments?
4. One of the earliest experiments on photosynthesis was (a) They enable a wider range of wavelength of incoming
done in 1770 by Joseph Priestley. He demonstrated that light to be utilized for photosynthesis.
(a) sun is the ultimate source of energy. (b) They absorb light and transfer the energy to reaction
(b) water is essential for life. centre.
(c) plants & animals “restore” the air for each other. (c) They protect reaction centre from photo-oxidation.
(d) chlorophyll captures light energy. (d) All of the above
5. Contribution of Ingen-Housz in elucidation of process of 10. The light harvesting complex (LHC) is made up of
photosynthesis is that (a) one molecule of Chl a.
(a) only green parts of plants exposed to light can convert (b) very few molecules of Chl a.
foul air (CO2) into pure air (O2). (c) hundreds of pigment molecules bound to proteins.
(b) green plants convert light energy into chemical (d) Chl a + Chl c + protein + DNA.
energy
11. In a plant cell, which of the following pigments participates
(c) plants have the capacity to purify foul air. directly in the light reactions of photosynthesis?
(d) sunlight is the ultimate source of energy for plants (a) Chlorophyll a (b) Chlorophyll b
and animals.
(c) Chlorophyll d (d) Carotenoids
6. The experiment material used by Van Neil, to prove that
12. In PS-I, the reaction centre Chl a has absorption maxima at
O2 comes out from water was studied on
_____________, while in PS-II the reaction centre Chl a
(a) Chlorella pyrenoidosa has absorption maxima at ___________.
EBD_7209
112 Biology
(a) P680, P700 (b) P700, P680 (a) 2-carbon organic compound
(c) P800, P600 (d) P700, P900 (b) 3-carbon organic compound
13. An energy diagram for the transfer of electrons in the (c) 4-carbon organic compound
light reactions of photosynthesis in plants is (d) 5-carbon organic compound
(a) cyclic photo-phosphorylation 23. In Calvin cycle, RuBisCO incorporates CO2 into ribulose,
(b) Z-band 1, 5-bisphosphate which rapidly splits into
(c) Z-Scheme (a) 2 molecules of 3-PGA (b) 2 molecules of 2-PGA
(d) non-cyclic photo-phosphorylation (c) 3 molecules of 3-PGA (d) 3 molecules of 2-PGA
14. Electrons are picked up by an electron acceptor which 24. The total requirement of ATP & NADPH for each molecule
passes them to an electron transport system consisting of of CO2 fixed & reduced in photosynthesis in the Calvin
(a) phytochromes (b) cytochromes cycle is
(c) Z-scheme (d) redox potential scale (a) 2 ATP & 2 NADPH (b) 2 ATP & 3 NADPH
15. Splitting of water is related with (c) 3 ATP & 2 NADPH (d) 4 ATP & 3 NADPH
(a) photosystem I 25. Which is the primary CO2 fixation product in C4 plant?
(b) photosystem II (a) 3-phosphoglyceric acid (b) Oxaloacetic acid
(c) both (a) and (b) (c) Phosphoenol pyruvate (d) RuBP
(d) cyclic photo-phosphorylation 26. Site of photosynthesis in C4 plant is
16. Correct equation that represents the photolysis of water (a) mesophyll cells (b) bundle sheath cells
is (c) Both (a) and (b) (d) Cytosol
(a) 2H2O ® 4H+ + O2 + 4e– (b) H2O ® 4H+ + O2 + 4e– 27. The primary CO2 acceptor in C4 plant is
(c) 4H2O ®4H+ + O2 + 4e– (d) 2H2O ®4H+ + 2O2 +2e– (a) RuBP (b) phosphoenol pyruvate
17. The chemiosmotic mechanism mediates (c) PEP carboxylase (d) PGA
(a) ATP synthesis. 28. Chloroplast movement is influenced by
(b) splitting of water. (a) light exposure (b) dark condition
(c) reduction of NADP+. (c) atmospheric condition (d) number ofmesophyll cells
(d) flow of electrons from PS - II to PS - I 29. Fixation of CO2 molecule through Hatch and Slack pathway
18. ATP synthesis, in photosynthesis involves the requires an enzyme called _________ .
(a) establishment of a protein gradient. (a) PEPcase (b) RuBisCO
(b) oxidation of water. (c) RuBP carboxylase (d) oxygenase
(c) reduction of NADP+. 30. Bundle sheath cells
(d) flow of electrons. (a) are rich in RuBisCO.
19. In photosynthesis, protons accumulate in the (b) are rich in PEP carboxylase.
(a) inner membrane space of mitochondria. (c) lack RuBisCO.
(b) matrix of mitochondria. (d) lack both RuBisCO and PEP carboxylase.
(c) lumen of thylakoid. 31. In the leaves of C4 plants, malic acid formation during
(d) stroma of thylakoid. CO2 fixation occurs in the cells of
20. The light-driven synthesis of ATP & NADPH, provides (a) bundle sheath (b) phloem
energy and reducing power for the (c) epidermis (d) mesophyll
(a) conversion of inorganic carbon into organic carbon. 32. By looking at which internal structure of a plant can you
(b) fixation of CO2 into trioses. tell whether a plant is C3 or C4?
(c) for the production of sugars. (a) Kranz anatomy
(d) all of the above. (b) Distribution of mesophyll cells
21. No. of carbons in the primary CO2 fixation product of C4 (c) Bundle sheath cells only
plant is (d) Both (a) and (b)
(a) 2 (b) 3 33. In C4 plants, the process by which C4 acid is converted
into C3 acid in the bundle sheath cell is known as
(c) 4 (d) 5
(a) carboxylation (b) regeneration
22. Which was the first CO2 fixation product formed, in the
Calvin experiment, using radioactive labelled 14C in green (c) reduction (d) decarboxylation
algal?
Photosynthesis 113

34. A process that creates an important difference between (a) Pigments are substances that have an ability to
C3 & C4 plants is called____________. absorb light, at specific wavelengths.
(a) Calvin benson cycle (b) photosynthesis (b) Chlorophyll b is the chief pigment associated with
(c) photorespiration (d) transpiration photosynthesis.
35. Photorespiration is a wasteful process because (c) Leaf pigments can be separated by chromatography.
(a) there is no synthesis of sugars. (d) Accessory pigments protect chlorophyll a from
(b) there is no synthesis of ATP or NADPH. photo-oxidation.
(c) there is no synthesis of phosphoglycerate. 42. Which of the following statement is incorrect?
(d) both (a) and (b) (a) Photosystem-I receives electrons from photosystem-II.
36. Photorespiration (b) Photosystem-II receives electrons from photolytic
(a) occurs because oxygen rather than carbon dioxide dissociation of water.
links to the rubisco enzyme in the Calvin cycle. (c) Formation of NADPH is associated
(b) occurs more in C4 than in C3 plants under identical with photosystem -II.
conditions. (d) Reaction centre of photosystem I is P700.
(c) describes the uptake of CO2 & the release of oxygen 43. Which among the following sentence is incorrect about
in chloroplasts. light reaction?
(d) All of the above (a) It is also known as ‘photochemical’ phase.
37. The principle of limiting factors was proposed by
(b) It includes light absorption, water splitting, oxygen
release, and the formation of high-energy chemical
(a) Blackman (b) Hill intermediates.
(c) Arnol (d) Liebig
(c) Reaction centre consist of single molecule of chl a
38. Plant factors affecting photosynthesis include but 2 molecules of chl b.
(a) number, age, size, and orientation of leaves, (d) The pigments are organised into two discrete
mesophyll cells and chloroplast, internal CO2 conc., photochemical light harvesting complexes (LHC)
the amount of chlorophyll .
within PS - I and PS - II.
(b) nature of leaves, size of mesophyll cells and light.
44. Which one of the following statement correctly describes
(c) mesophyll cells, distribution and temperature. the cyclic photophosphorylation?
(d) quantity of chlorophyll, size of leaves and CO2. (a) Cyclic photophosphorylation has both PS-I and PS-II.
STATEMENT TYPE QUESTIONS (b) Cyclic phosphorylation produces neither ATP nor
NADPH + H+.
39. Which one of the following is incorrect about the activities (c) Water is the ultimate source of e – in cyclic
associated with PS - I and PS - II in non-cyclic phosphorylation.
photophosphorylation ? (d) Electrons are cycled in cyclic photophosphorylation.
(a) Water is oxidised in PS - II, but not in PS - I. 45. All of the following statements are incorrect for non-cyclic
(b) Photons (light) are needed to activate both PS - I and electron transport system, except
PS - II. (a) Electron transport between PS -II to PS-I produces
(c) Photolysis of water, formation of ATP + NADPH + H ATP by substrate level phosphorylation.
occur. (b) In PS-II, the oxidation of two water molecules
(d) Production of NADPH + H+ is associated with PS - produces four electrons, four proteins, & a single O2.
II, but not with PS - I. (c) Water is oxidized & electrons are released by PS-I.
40. Which of the following statement is incorrect? (d) PS - II reduces NADP+ to NADPH.
(a) Site of photosynthesis is mesophyll cells of 46. Which of the following statement is incorrect ?
chloroplast. (a) H2S, not H2O, is involved in photosynthesis of
(b) In Z-scheme, movement of electrons is uphill in terms purple sulphur bacteria.
of redox potential scale. (b) Light and dark reactions are stopped in the absence
(c) In Z-scheme of photosynthesis, the electrons flow of light.
from H2O to NADP+. (c) Calvin cycle occurs in the grana of chloroplast.
(d) ATP synthesis is linked to development of a proton (d) ATP is pr oduced during ligh t reaction via
gradient across a membrane. chemiosmosis.
41. Which of the following statement is incorrect regarding 47. Which of the following statement(s) is/are correct about
pigments? RuBisCO?
EBD_7209
114 Biology
(a) It catalyzes the fixation of CO2. 53. Why C4 plants are special ? Because,
(b) It has oxygenation & carboxylation both activity. (i) they have a special type of leaf anatomy.
(c) It is the most abundant protein on earth. (ii) they tolerate higher temperatures.
(d) All of the above (iii) they show a response to high light intensities.
48. Which of the following statements given below is incorrect ? (iv) they lack a process called photorespiration.
(a) The C4 plants respond to higher temperatures while (v) they have greater productivity of biomass.
C3 plants have a much lower temperature optimum. (a) (i) and (ii) (b) (i), (iii) and (iv)
(b) Tropical plants have a higher temperature optimum (c) (i), (ii), (iii) and (iv) (d) All of these
than the plants adapted to temperate climates. 54. Consider the following statements with respect to
(c) Some C3 plants are allowed to grow in CO2 enriched photosynthesis and identify the correct statements.
atmosphere that leads to higher yields. (i) The first carbon dioxide acceptor in C4 cycle is PGA.
(d) Water stress causes the stomata to remain open (ii) In C 3 plants, the first stable product of
hence enhancing the CO2 availability. photosynthesis during dark reaction is RuBP.
49. Which one of the following statement is incorrect in (iii) Cyclic photophosphorylation results in the formation
relation to photorespiration? of ATP.
(a) It is a characteristic of C3 plants. (iv) Oxygen which is liberated during photosynthesis
(b) The RuBP binds with O2 to form one molecule of comes from water.
phosphoglycerate and phosphoglycolate. (a) (i) and (ii) (b) (i) and (iii)
(c) There is synthesis of ATP or NADPH. (c) (iii) and (iv) (d) (ii) and (iii)
(d) It occurs in daytime only.
50. Which of the following statement best support the fact ASSERTION/REASON TYPE QUESTIONS
that photorespiration commonly occurs in C3 plants? In the following questions, a statement of Assertion is followed
(a) C3 plants don’t possess Kranz anatomy. by a statement of Reason.
(b) C3 plants have usually high CO2 compensation (a) If both Assertion and Reason are true and the Reason is
species. the correct explanation of the Assertion.
(c) C3 plants are less efficient in photosynthesis. (b) If both Assertion and Reason are true but the Reason is
(d) C3 plants are characterized by RuBP oxygenase not the correct explanation of the Assertion.
activity under high oxygen supply. (c) If Assertion is true but Reason is false.
(d) If both Assertion and Reason are false.
51. Which of the following statements (i-iv) regarding
55. Assertion : 6 molecules of CO2 and 12 molecules of
“Splitting of water” is/are correct. NADPH+ + H+ and 18 ATP are used to form one hexose
(i) It is photolysis of water which provides H+ ions for molecule.
synthesis of NADPH. Reason : Light reaction results in formation of ATP and
(ii) It provides electrons for photophosphorylation & NADPH2.
activation of NADP+. 56. Assertion: C 4 pathway of CO 2 fixation is found in some
(iii) O2 is evolved during this process. tropical plants.
(iv) It replenishes O2 consumed by living beings and
Reason: In this pathway, CO 2 is fixed by 3C compound.
combustion.
57. Assertion : Mitochondria helps in photosynthesis
(a) One (b) Two
Reason : Mitochondria have enzymes for dark reaction.
(c) Three (d) All
58. Assertion: Bacterial photosynthesis occurs by utilizing
52. Identify the correct statements for ATP synthase. wavelength longer than 700 nm.
(i) This enzyme consists of two parts: hydrophobic Reason: Here reaction centre is B-890.
membrane bound portion called F0 & a portion that 59. Assertion: Six molecules of CO2 and twelve molecules of
sticks out into stroma called F1. NADPH+ + H+ and 18 ATP are used to form one hexose
(ii) F0 appears to form a channel across the membrane molecule.
through which proton can pass. Reason: Light reaction results in the formation of ATP
(iii) The conformational change in the F1 portion of the and NADPH2.
complex synthesizes ATP. 60. Assertion : Cyclic pathway of photosynthesis first
(iv) The proton motive force that drives the synthesis of appeared in some eubacterial species.
ATP is associated with this enzyme. Reason : Oxygen started accumulating in the atmosphere
after the non-cyclic pathway of photosynthesis evolved.
(a) (i), (ii) and (iii) (b) (i), (ii) and (iv)
(c) (i) and (iv) only (d) All of these
Photosynthesis 115

61. Assertion : C4 photosynthetic pathway is more efficient The correct option is


than the C3 pathway. (a) A – I; B – II; C – III; D – IV; E – V
Reason : Photorespiration is suppressed in C4 plants. (b) A – III; B – I; C – II; D – V; E – IV
MATCHING TYPE QUESTIONS (c) A – IV; B – II; C – V; D – III; E – I
(d) A – V; B – IV; C – III; D – II; E – I
62. Match the scientests given in column-1 with their work, 64. Match the column-1 with column II and choose the correct
given in column-II & select the correct answer using the answer using the codes given below.
codes given below. Column -I Column -II
Column-I Column-II A. Emerson effect I. C4 cycle
A. Priestley I. Determined the action B. Hill reaction II. Photolysis
spectrum of chlorophyll C. Calvin cycle III. C3 cycle
B. Jan Ingenhousz II. Provided evidence that in D. Hatch & Slack cycle IV. Photosystem I & II
green parts of plant glucose
(a) A – I; B – II; C – III; D – IV
is made & stored as starch
C. Sachs III. Plants purify air only in the (b) A – I; B – III; C – IV; D – I
presence of light (c) A – III; B – IV; C – I; D – II
D. Engelmann IV. Demonstrated that (d) A – IV; B – II; C – III; D – I
photosynthesis is 65. Which of the following pair is mismatched?
essentially a light-dependent (a) Photosystem I – Uses the P700 molecule in its
reaction photocenter.
E. Niel V. Revealed the essential role of (b) Antenna complex – Contains hundreds of pigment
air in the growth of plants molecule.
(a) A – V; B – III; C – II; D – I; E – IV (c) PGA – 3- carbon compound.
(d) Dark reaction – Takes place in the grana of the
(b) A – V; B – I; C – II; D – III; E – IV
chloroplast.
(c) A – V; B – I; C – IV; D – III; E – II 66. Match the parts given in column I with the events given
(d) A – V; B – I; C – III; D – IV; E – II in column II and choose the correct combination from the
options given below.
63. The given figure shows the graph of light intensity (on x-
Column-I Column-II
axis) on the rate of photosynthesis (on y-axis). Few points
are marked as A, B, C, D and E. A. Grana of chloroplast I. Kreb's cycle
B. Stroma of chloroplast II. Light reaction
C. Cytoplasm III. Dark reaction
D. Mitochondrial matrix IV. Glycolysis
(a) A – IV; B – III; C – II; D – I
(b) A – I; B – II; C – IV; D – III
(c) A – IV; B – I; C – III; D – II
(d) A – II; B – III; C – IV; D – I
67. Match column-I with column-II and choose the correct
Match the marked alphabets given in column I with these combination from the options given below.
correct interpretation given in column II. Column -I Column -II
Column - I Column - II A. Oxygen evolving I. Pigments
A. Limiting factor I. Some factor other than complex ferric oxalate
in region A light intensity is B. Proton gradient II. High oxygen
becoming concentration
the limiting factor C. Absorb light at III. ATP synthesis
B. B represents II. Light is no longer specific wavelengths.
limiting factor D. Photorespiration IV. Photolysis of water
C. C represents III. Light intensity (a) A – IV; B – III; C – I; D – II
D D represents IV. Maximum rate of (b) A – IV; B – I; C – III; D – II
photosynthesis (c) A – II; B – I; C – IV; D – III
E. E represents V. Saturation point for (d) A – II; B – IV; C – III; D – I
light intensity
EBD_7209
116 Biology
68. Which of the following pair is not correctly matched ? 72. The given figure shows the diagramatic representation of
(a) C3 plant – Maize (b) C4 plant – Kranz anatomy the Hatch & Slack pathway few labelling are marked as P,
(c) Calvin cycle – PGA Q and R.
(d) Hatch and Slack pathway – Oxaloacetic acid
Atmospheric CO2
69. Which of the following is incorrectly matched?
Mesophyll Plasma
(a) Sorghum — Kranz anatomy membrane
cell
(b) PS - II — P700 Cell wall

(c) Photorespiration — C3 plants HCO3 Phosphoenol-
pyruvate
(d) PEP carboxylase — Mesophyll cells
Regeneration
DIAGRAM TYPE QUESTIONS Fixation
Plasmo- P R
70. The given figure of calvin cycle shows the carbon desmata
assimilation in C3 plants. Choose the correct labelling of
the carbohydrate molecule (Marked as I, II and III) Bundle Transport Transport
involved in the Calvin cycle. sheath cell Fixation by
Calvin cycle
C4 acid
CO 2
Q C3 acid

(I)

Which of the following option shows the correct labeling of P,


Q, and R

(II) P Q R
(a) C3 acid Red u ctio n C4 acid
(b) Fixation C4 acid Regen eratio n
?
(III) (c) C4 acid Decarbo xylatio n C3 acid
(d) Carb oxylatio n C3 acid Redu ction

73. The diagram below represents an experiment with isolated


(I) (II) (III) chloroplasts. The chloroplasts were first made acidic by
(a) RuBP Triose PGA soaking them in a solution at pH 4. After the thylakoid
phosphate space reached pH 4, the chloroplasts were transferred to
(b) PGA RuBP Triose a basic solution at pH 8. The chloroplasts are then placed
phosphate in the dark. Which of these compounds would you expect
to be produced?
(c) PGA Triose RuBP
phosphate
(d) RuBP PGA Triose
phosphate phosphate
71. The given diagram represents the Calvin cycle.

GP(PGA) Q

P
triose R hexose
starch
phosphate phosphate

RuBP S
(a) ATP
At which stage (inducated by P, Q, R and S)is CO2 (b) NAD
incorporated? (c) G3P
(a) P (b) Q (d) C6H12O6
(c) R (d) S
Photosynthesis 117

74. The diagram given below shows ATP synthesis through 76. Study the given graph which shows the action spectrum
chemiosmosis. of A. Superimposed on B spectrum of chlorophyll a.
Identify A & B in the graph.

A
B

Light absorbed
400 500 600 700
Wavelength of light in nanometres (nm)

A B
(a) Rate of respiration Action spectrum
(b) Rate of respiration Absorption
Which option shows the correct labelling of A, B, C and (c) Rate of photosynthesis Action spectrum
D in the diagram ?
(d) Rate of photosynthesis Absorption
(a) A - F1, B - Thylakoid membrane,
C - Photosystem (I), D - Photosystem (II) 77. The given figure shows the diagrammatic representation
(b) A - F0, B - Thylakoid membrane, of a section of chloroplast. Few plants are marked as A, B, C,
D & E.
C - Photosystem (I), D - Photosystem (II)
(c) A - F1, B - Thylakoid membrane,
C - Photosystem (II), D - Photosystem (I) A
(d) A - F0, B - Thylakoid membrane,
B
C - Photosystem (II), D - Photosystem (I)
C
75. Given below is the pathway (2-scheme) of light reaction.
Identify the blanks indicated by A, B, C and D.
D
C E
A
D Combination of which parts is responsible for trapping
e – acceptor
the light energy & also the synthesis of ATP and NADPH?
Light B ADP + iP
ATP (a) A, B, C (b) B, C, D
Electron transport (c) C, D, E (d) A, D, E
system
78. Which one of the following correctly identifies X and Y
and and their functions in the given figure of chloroplast ?
LHC
LHC
H2 O → 2e – + 2H + + [O] X
Y
EBD_7209
118 Biology

X Y 84. Which of following ratio is correct for the production of


Structure Function Structure Function one molecule of glucose through 6 rounds of Calvin cycle?
CO2 ATP NADPH2
Photolysis
(a) Grana Stroma CO2 fixation (a) 1 2 2
of water
Photolysis of (b) 6 18 12
(b) Grana CO2 fixation Stroma (c) 6 12 18
water
Photolysis (d) 5 6 9
(c) Stroma Grana CO2 fixation
of water 85. Which of the following combination is correct for C4
Photolysis of plants?
(d) Stroma CO2 fixation Lamellae
water Mesophyll Bundle Sheath
(a) PEPcase C4-cycle RuBisCO C3-cycle
CRITICAL THINKING TYPE QUESTIONS (b) PEPcase Calvin cycle RuBisCO C4-cycle
79. In non-cyclic reactions of photosynthesis, electrons from (c) RuBisCO C4-cycle PEPcase C3-cycle
chlorophyll molecules in photosystem-I are used in the (d) RuBisCO C2-cycle PEPcase C3-cycle
formation of NADPH. What is the source of such 86. In an experiment, the CO2 available to a C4 plant was labelled
electrons? with a radioactive isotope and the amount of radioactivity
in the chloroplast was measured. As photosynthesis
(a) Light
preceeded, in which of the following molecules did the
(b) NADPH radioactivity first appear?
(c) Photosystem-I (a) Oxaloacetic acid (b) PEP
(d) Photosystem-II, which splits water molecule (c) Malic acid (d) RuBP
87. Which of the following plant species have highest
80. The reactions of Calvin cycle not directly dependent on
photosynthetic yield?
light, but they usually do not occur at night. Why? (a) Species that perform photorespiration
(a) Night is often too cold for these reactions to occur. (b) Species possessing C3 pathway
(b) CO2 concentration in night is too high for these (c) Species possessing C4 pathway
reactions to occur. (d) Same for all
(c) Plants usually open their stomata at night. 88. According to Blackman’s law of limiting factor, at any given
time, photosynthesis can be limited by
(d) Calvin cycle is dependent on the products of light
(a) light only (b) CO 2 concentration
reaction. only
81. To make 100 molecules of glucose, how many molecules (c) both light and CO2 concentration
of ATP & NADPH are required? (d) either by light or by CO2
(a) 1800 and 1200 respectively. 89. During monsoon, the rice crop of eastern states of India
shows lesser yield due to limiting factor of
(b) 1200 and 1800 respectively.
(c) 1800 and 600 respectively. (a) CO2 (b) light
(d) 200 and 600 respectively. (c) temperature (d) water
82. Chloroplasts are disrupted and the stroma separated from 90. Why, at higher light intensities, gradually photosynthesis
the lamellae. The isolated stroma will fix CO2 if it is supplied rate does not show further increase?
with (a) Higher light intensity activate more chlorophylls.
(b) Higher light intensity causes more transpiration.
(a) O2 (b) RuBisCO
(c) No need of more sugar formation.
(c) light (d) ATP + NADPH (d) Other factors become limiting.
83. The correct sequence of Calvin cycle is 91. Under water stress, the rate of photosynthesis declines
(a) Decarboxylation ® Oxidation ® Regeneration because of
(a) stomatal closure leading to decrease in CO2 supply.
(b) Decarboxylation ® Regeneration ® Oxidation
(b) reduced water potential that decreases leaf surface
(c) Carboxylation ® Reduction ® Regeneration areas for photosynthesis.
(d) Carboxylation ® Reduction ® Regeneration (c) both (a) and (b) (d) turgidity of leaf.
Photosynthesis 119

92. In C4 (sugarcane plant) plant,14CO2 is fixed in malic acid 98. During the light stage of photosynthesis, the
in which the enzyme that fixes CO2 is photoactivated pigment removes an electron from the
(a) fructose phosphatase hydroxylation derived from the water molecule.
(b) ribulose biphosphate carboxylase The fate of the free hydroxyl radical is that it
(c) phosphoenol pyruvic acid carboxylase (a) is broken down into oxygen and a free radical of
(d) ribulose phosphate kinase hydrogen.
93. What will happen if the supply of oxygen is decreased to
(b) is used to raise the activation level of chlorophyll by
an illuminated wheat plant?
donating a positive charge.
(a) Its photosynthesis would decrease. (c) is used to produce adenosine triphosphate from
(b) Its respiration process would stop. adenosine diphosphate.
(c) All physiological process would stop. (d) reduces carbon dioxide to sugar.
(d) Its photosynthesis would increase. 99. The function of water in photosynthesis is to
94. A student sets up an experiment on photosynthesis as (a) absorb light energy.
follows: (b) supply electrons in the light dependent reaction.
He takes soda water in a glass tumbler and add chlorophyll (c) transport H+ ions in the light independent reactions.
extracts into the contents and keeps the tumbler exposed (d) provide O2 for the light-independent reactions.
to sunlight hoping that he has provided necessary 100. Which of the following is not concern with cyclic
ingredients for photosynthesis to proceed (viz., CO2, H2O, photophosphorylation?
chlorophyll and light). What do you think what will
happen after, say, a few hours of exposure of light? (a) Liberation of oxygen.
(a) Photosynthesis will take place and glucose will be (b) Synthesis of ATP.
produced. (c) It occurs in certain photosynthetic bacteria.
(b) Photosynthesis will take place and starch will be (d) Electron expelled from P700 return to it after passing
produced which will turn the mixture turbid. through different electron acceptor
(c) Photosynthesis will not take place because CO2 101. Cyclic and non-cyclic flow of e– is used in plants to
dissolved in soda water escapes into the atmosphere. (a) meet the ATP demands of Calvin-cycle.
(d) Photosynthesis will not take place because intact (b) avoid producing excess NADPH + H+.
chloroplasts are needed for the process. (c) balance ATP and NADPH + H+ ratio in chloroplasts.
95. The electrons that are released by the photolysis of water (d) All of the above
during non-cyclic photophosphorylation, ultimately end 102. Which one of the following event occurs both during
up in
cyclic and non-cyclic modes of photophosphorylation?
(a) glucose (b) ATP
(a) Involvement of both PS - I and PS - II.
(c) H2O (d) NADPH
(b) Formation of ATP.
96. ADP is phosphorylated and NADP is reduced, this
(c) Release of O2.
happens during.
(d) Formation of NADPH.
(a) dark phase of photosynthesis
(b) light phase of photosynthesis 103. In a crop field a weedicide is used to remove weeds in
order to increase the yield. But the effect of this weedicide
(c) Photorespiration
is that, it blocks electron transport from photosystem II to
(d) Calvin cycle photosystem I. This will result in
97. During light reaction, as electrons move through
(a) enhancement of dark reaction.
photosystems, protons are transported across the
membrane. This happens because of (b) failure of ATP synthesis.
(a) the primary acceptor of e– (located towards the outer (c) lack of reduction of NADP+.
surface of the membrane) transfers its electron not (d) both (b) and (c)
to an e– carrier but to H carrier. 104. Cooperation of the two photosystems of the chloroplast
(b) the primary acceptor of e– transfers only its e– to e– is required for
carrier. (a) ATP synthesis.
(c) the primary acceptor of e– transfers only H+ to the (b) reduction of NADP+.
next carrier.
(c) enhancement of dark reaction.
(d) NADP - reductase is present in grana.
(d) generation of protein motive force.
EBD_7209
120 Biology
105. Reduction of NADP+ into NADPH during light reaction (c) light energy to kinetic energy.
occurs in stroma because (d) solar energy to potential energy.
(a) NADP reductase enzyme is located on the stroma 114. Accessory pigments
side of membrane. (a) play no role in photosynthesis.
(b) PS - I reduces NADP+ to NADPH +H+ in the stroma. (b) release e– and get oxidized.
(c) The pH of the stroma remains constant. (c) transfer of e– to NADP.
(d) Both (a) and (b) are correct. (d) allow plants to harvest visible light of wider range
106. Assume the thylakoid membrane within a chloroplast is wavelengths.
punctured so that there is no separation between lumen & 115. Chlorophyll is suited for the capture of light energy
stroma of thylakoid. Which of these process would be because
most affected? (a) certain wavelengths of light raise it to an excited
(a) the splitting of water state.
(b) the synthesis of ATP (b) in its excited state chlorophyll gives off electrons.
(c) reduction of NADP+ (c) chlorophyll’s structure allows it to attach to
(d) the flow of electrons from PS - II to PS - I thylakoid membranes.
107. Two groups of isolated thylakoids are placed in an acidic (d) all of the above
bathing solution so that H+ diffuses into the thylakoids. 116. The pigment molecules responsible for photosynthesis
They are then transferred to a basic bathing solution, and are located in the
one group is placed in the light, while the other group is (a) cytoplasm of the cell.
kept in the dark.
(b) matrix of the mitochondria.
Select the choice given below that describes what you
(c) thylakoid membrane of the chloroplast.
expect each group of thylakoids to produce.
(d) All of the above
In Light In Dark 117. Light reaction of photosynthesis results in the formation
(a) ATP only Nothing of
(b) ATP, O2 ATP only (a) O2 (b) NADPH + H+
(c) ATP, O2 , glucose ATP, O2 (c) ATP (d) All of these
(d) ATP, O2 O2 118. During photosynthesis, electrons are continuously lost
from the reaction centre of PSII. By which process these
108. By which of the following complex, proton is pumped to electrons are replaced?
reach ATP synthase, to participate in ATP synthesis.?
(a) Sunlight
(a) Cytochrome b6 f (b) Cytochrome c oxidase
(b) Photolysis of water
(c) Cytochrome a - a3 (d) Cytochrome bc
(c) Release of oxygen
109. Number of carbons in the primary CO2 fixation products in
(d) Redox reaction
C3 plant is
119. Breakdown of water during the photosynthesis molecule
(a) 3 (b) 4
leads to release of
(c) 5 (d) 6
(a) electron and proton
110. How many molecules of RuBP & CO2 respectively are
(b) electron and oxygen
required for production of 6 molecules of 3-PGA?
(c) proton and oxygen
(a) 3 and 2 (b) 2 and 3
(d) electron, proton and oxygen
(c) 3 and 3 (d) 3 and 1
120. In Kranz anatomy, the bundle sheath cells have
111. Which of the following does not participate in the process
of photosynthesis? (a) thin walls, many intercellular spaces and no
chloroplasts.
(a) Red algae (b) Green algae
(b) thick walls, no intercellular spaces and large number
(c) Brown algae (d) None of these
of chloroplasts.
112. Which part of the plant do not perform photosynthesis?
(c) thin walls, no intercullular spaces and several
(a) Cactus stem (b) Guard cell of stomata chloroplasts.
(c) Mesophyll cells of leaf (d) Leaf epidermis (d) thick walls, many intercellular spaces and few
113. Photosynthesis is the transformation of chloroplasts.
(a) light energy to chemical energy.
(b) chemical energy to light energy.
14

Chapter
Respiration in Plants

(c) Kreb’s cycle


FACT/DEFINITION TYPE QUESTIONS
(d) alcoholic fermentation
1. Respiration is a/an 9. Conversion of phosphenol pyruvic acid to pyruvic acid
(a) anabolic + exergonic (b) catabolic + exergonic and ADP to ATP are examples of
(c) catabolic + endergonic (d) anabolic + endergonic (a) photophosphorylation.
2. ATP is (b) oxidative phosphorylation.
(a) an energy currency (c) photoelectric phosphorylation.
(b) a nucleotide (d) substrate level phosphorylation.
(c) formed in both respiration and photosynthesis 10. When oxygen is not available to a muscle cell, NADH
formed during glycolysis does not pass electrons to the
(d) all of the above
ETS. Instead, it passes hydrogen atoms to
3. Cellular respiration includes the various pathways by
(a) acetyl CoA (b) pyruvic acid
which carbohydrates and other metabolites are broken
(c) fructose (d) ADP
down with the consecutive buildup of 11. Which one of the following process releases a carbon
(a) ATP (b) protein dioxide molecule?
(c) vitamins (d) none of these (a) Glycolysis
4. During the process of respiration, which of the followings (b) Lactic acid fermentation
are released as products? (c) Alcohol fermentation
(a) CO2, H2O and O2 (b) CO2, O2 and energy (d) Hydrolysis of glycogen
(c) CO, H2O and energy (d) CO2, H2O and energy 12. In the fermentation of one glucose molecule, there is a net
5. Glycolysis occurs in the ________ and gain of ______________ molecules of ATP.
produces________, which in the presence of O2 enters (a) one (b) two
the____________. (c) six (d) eight
(a) cytosol; pyruvate; mitochondrion 13. In an anaerobic condition, yeast cells breakdown glucose
(b) cytosol; glucose; mitochondrion into
(c) mitochondrion; pyruvate; chloroplast (a) CO2 + H2O (b) C2H5OH and CO2
(d) chloroplast; glucose; cytosol (c) CO2 + lactic acid (d) CO2 + pyruvic acid
6. The enzymes, involved in the chemical reactions of 14. The formation of acetyl coenzyme A from pyruvic acid
glycolysis are located produces (in addition to acetyl coenzyme A) one molecule
(a) in the fluid matrix of cytoplasm. of A , one molecule of B , and one molecule
(b) in the mitochondrial matrix. (ion) of C , Identify A, B and C respectively.
(c) in the nuclear sap. A B C
(d) on the cristae of a mitochondria. (a) H
+
CO 2 NADH
7. In which of the following reaction of glycolysis, a molecule CO 2 +
(b) H NADH
of water is removed from the substrate ? +
(a) Frucoste-6-phosphate ® Fructose 1, 6-phosphate (c) NADH O2 H
(b) 3-phosphate glyceraldehyde ® (d) CO 2 NADH H
+

1, 3-biphosphoglyceric acid
15. Which of the following are not used in the conversion by
(c) PEP ® Pyruvic acid pyruvate to acetyl CoA?
(d) 2-phosphoglycerate ® PEP (i) Oxidative dehydrogenation
8. Decarboxylation is not involved in (ii) Oxidative dehydration
(a) electron transport system (iii) Oxidative phosphorylation
(b) glycolysis (iv) Oxidative decarboxylation
EBD_7209
122 Biology
(a) (i), (ii) and (iii) (b) (i) and (ii) 27. A major site for synthesis of ATP from ADP and Pi is
(c) (ii) and (iv) (d) (i) and (iii) (a) F1 headpiece in mitochondria.
16. Which one of the following reactions is an example of (b) F0.
oxidative decarboxylation? (c) F1 – F0.
(a) Conversion of succinate to fumarate. (d) mitochondria.
(b) Conversion of fumarate to malate. 28. The F1 headpiece is a
(c) Conversion of pyruvate to acetyl CoA. (a) peripheral membrane protein complex.
(d) Conversion of citrate to isocitrate.
(b) integral membrane protein complex.
17. Which one of the following is essential for the respiration
as well as photosynthesis ? (c) transmembrane protein.
(a) Ubiquinone (b) Cytochrome (d) carrier protein.
(c) RuBisCO (d) Plastocyanin 29. How many ATP molecules can be produced through
18. In which one of the following do the two names refer to oxidative phosphorylation of 2NADH2 and 3 FADH2?
tricarboxylic acid cycle? (a) 15 (b) 24
(a) a-ketoglutaric acid and Krebs cycle (c) 6 (d) 12
(b) Malic acid cycle and Kornberg cycle 30. The main purpose of electron transport chain is to
(c) Citric acid cycle and Krebs cycle (a) cycle NADH + H+ back to NAD+
(d) Oxaloacetic acid and Kornberg cycle (b) use the intermediates from TCA cycle
19. Citrate synthase, an enzyme of TCA cycle is located in (c) breakdown pyruvate
(a) cytosol in prokaryotes. (d) all of the above
(b) mitochondrial matrix in eukaryotes. 31. Chemiosmotic theory of ATP synthesis in the chloroplasts
(c) both (a) and (b) and mitochondria is based on the
(d) none of the above (a) membrane potential.
20. In citric acid cycle, energy bond (GTP) is produced as a
(b) accumulation of Na ions.
result of conversion of
(c) accumulation of K ions.
(a) citric acid to a-keto glutaric acid.
(b) succinic acid to malic acid. (d) proton gradient.
(c) succinyl - CoA to succinic acid. 32. The correct sequence of electron acceptor in ATP
(d) succinic acid to succinyl - CoA. synthesis is
21. Out of 38 ATP molecules produced per glucose, 32 ATP (a) cyt a a b c (b) cyt b c a a3
molecules are formed from NADH/FADH2 in (c) cyt b c a3 a (d) cyt c b a a3
(a) respiratory chain 33. Which of the following is amphibolic in nature?
(b) Krebs cycle (a) Glycolysis
(c) oxidative decarboxylation (b) Oxidative decarboxylation of pyruvate
(d) EMP (c) TCA cycle
22. Acceptor of acetyl Co-A in Kreb’s cycle is (d) Oxidative phosphorylation
(a) malic acid (b) fumaric acid 34. Refer the given equation and answer the question.
(c) a-keto glutaric acid (d) oxaloacetic acid
23. Total number of ATP consumed during Kreb’s cycle is 2(C 51H 98 O 6 ) + 145O 2 ¾¾
® 102CO 2 + 98H 2 O + Energy
(a) 0 (b) 1 The R.Q of above reaction is
(c) 2 (d) 3 (a) 1 (b) 0.7
24. Electron Transport System (ETS) is present in (c) 1.45 (d) 1.62
(a) inner mitochondrial membrane. 35. If R. Q. is less than 1.0 in a respiratory metabolism, it
(b) mitochondrial matrix. means that
(c) chlorophyll. (a) carbohydrates are used as respiratory substrate.
(d) chloroplast. (b) organic acids are used as respiratory substrate.
25. In the electron transport system, the final acceptor of (c) the oxidation of the respiratory substrate consumed
proton is more oxygen than the amount of CO2 released.
(a) cytochrome b (d) the oxidation of the respiratory substrate consumed
(b) cytochrome a3 less oxygen than the amount of CO2 released.
(c) oxygen 36. The overall goal of glycolysis, Kreb’s cycle and the
(d) ubiquinone (substance A) electron transport system is the formation of
26. Terminal cytochrome of respiratory chain which donates (a) ATP in one large oxidation reaction
electrons to oxygen is (b) sugars
(a) cyt. b (b) cyt. c (c) nucleic acids
(c) cyt. (d) cyt. a3 (d) ATP in small stepwise units
Respiration in Plants 123

(a) O2 must always be available for respiration.


STATEMENT TYPE QUESTIONS
(b) O2 combines with carbon to form CO2.
37. Which of the following statement regarding the process (c) O2 combines with hydrogen to form H2O.
of glycolysis is correct? (d) Air is inhaled and exhaled only from stomata.
(a) Glucose undergoes complete oxidation to form two 44. Which of the following statement(s) concerning ATP
molecules of pyruvic acid. synthesis is/are correct?
(b) Glucose undergoes partial oxidation to form one (a) ATP can be synthesized through substrate level
molecule of pyruvic acid. phosphorylation, photophosphorylation an d
(c) Glucose undergoes complete oxidation to form one oxidative phosphorylation.
molecule of pyruvic acid. (b) The proton-motive force is the establishment of
(d) Glucose undergoes partial oxidation to form two proton gradients and electrochemical potentials
molecules of pyruvic acid. across the inner membrane.
38. Which of the following is incorrect regarding the Kreb’s
(c) Proton-motive force is essential for back flow of H+
cycle?
from outer chamber of matrix of mitochondria through
(a) It is also known as tricarboxylic acid cycle.
proton channel (F0) of F0 – F1 particle to produce
(b) It occurs in mitochondria.
ATP.
(c) It starts with six carbon compound.
(d) All of the above
(d) It does not involve any decarboxylation step.
39. Select the incorrect statement about NADH during cellular 45. Which of the following statement is/are the correct events
respiration. in aerobic respiration?
(a) It is synthesized in glycolysis. (i) The complete oxidation of pyruvate by the stepwise
(b) It is transferred into the mitochondria. removal of all the hydrogen atoms, leaving three
(c) It undergoes oxidative phosphorylation. molecules of O2.
(d) It is reduced to NAD+. (ii). The complete oxidation of pyruvate by the stepwise
40. Which of the following statement is correct in relation to removal of all the hydrogen atoms, leaving three
the ETS? molecules of CO2.
(a) It is present in the mitochondrial matrix. (iii) The passing on of the electrons removed as part of
(b) Oxidation of one molecule of NADH gives rise to 2 the hydrogen atoms to molecular O 2 with
molecules of ATP, while that of one molecule of simultaneous synthesis of ATP.
FADH2 produces 3 molecules of ATP. (iv) The passing on of the electrons removed as part of
(c) Oxygen acts as the final hydrogen acceptor. the hydrogen atoms to molecular O 2 with
(d) In respiration, light energy is utilized for the simultaneous synthesis of ADP.
production of proton gradient. (a) (i) and (iii) (b) (i) and (iv)
41. Which of the following statement about cellular energy- (c) (ii) and (iii) (d) (ii) and (iv)
harvesting pathway is incorrect? 46. Which of the following statements (i to v) regarding
(a) Pyruvate oxidation can only occur under aerobic glycolysis are correct.
conditions. (i) It is ten enzymatic reactions that convert a six-carbon
(b) Autotrophs can produce their own food but must molecule to a three carbon pyruvate and result in a
obtain energy from it by glycolysis & cellular net gain of 2 ATP molecules.
respiration. (ii) Glucose undergoes partial oxidation to form one
(c) Fermentation usually occurs under aerobic molecule of pyruvic acid.
conditions. (iii) Glucose is phosphorylated to give rise to glucose - 6
(d) All of the above - phosphate by the activity of the enzyme
42. Which of the following statement regarding pyruvate phosphofructokinase.
during aerobic respiration is incorrect? (iv) The scheme of glycolysis was given by Gustav
(a) It is formed by the glycolytic catabolism of Embden, Otto Morrison, and J. Parnas and is often
carbohydrates in the mitochondrial matrix. referred to as the EMP pathway.
(b) It enters mitochondrial matrix & undergoes oxidative
(v) ATP is utilized at two steps: first in the conversion
decarboxylation.
of glucose into glucose 6-phosphate & second in
(c) The conversion of pyruvic acid to acetyl CoA is
the conversion of fructose - 6- phosphate to fructose
catalysed by pyruvic dehydrogenase.
1 ,
(d) Two molecules of NADH are produced from the
6-disphosphate.
metabolism of two molecules of pyruvic acid.
(a) (i), (iv) and (v) (b) (iii) and (v)
43. Which of the following is a more accurate statement about
respiration ? (c) (iv) and (v) (d) (ii) and (iv)
EBD_7209
124 Biology

ASSERTION/REASON TYPE QUESTIONS Column - I Column - II


A. Complex I I. Cytochrome bc 1 complex
In the following questions, a statement of Assertion is followed
by a statement of Reason. B. Complex II II. NADH dehydrogenase
(a) If both Assertion and Reason are true and the Reason is C. Complex III III. ATP synthetase
the correct explanation of the Assertion.
D. Complex IV IV. FADH2 dehydrogenase
(b) If both Assertion and Reason are true but the Reason is
E. Complex V V. Cytochrome c oxidase
not the correct explanation of the Assertion.
(c) If Assertion is true but Reason is false. (a) A – III; B – V; C – I; D – IV; E – II
(d) If both Assertion and Reason are false. (b) A – II; B – V; C – I; D – IV; E – III
47. Assertion : Glycolysis occurs in cytoplasm. (c) A – II; B – IV; C – I; D – V; E – III
Reason : Enzymes for glycolysis are found in cytoplasm. (d) A – IV; B – I; C – II; D – V; E – III
It is common in aerobic/anaerobic respiration. 54. Match the number of carbon atoms given in column - I
48. Assertion : The atmospheric concentration of CO2 at with that of the compounds given in column - II and select
which photosynthesis just compensates for respiration the correct option.
is referred to as CO2 compensation point. Column - I Column - II
Reason : The CO2 compensation point is reached when A. 4C compound I. Acetyl CoA
the amount of CO2 uptake is less than that generated B. 2C compound II. Pyruvate
through respiration because the level of CO 2 in the C. 5C compound III. Citric acid
atmosphere is more than that required for achieving CO2 D. 3C compound IV. a- ketoglutaric acid
compensation point.
V. Malic acid
49. Assertion : Stomata are absent in submerged (a) A – II; B – V; C – III; D – I
hydrophytes.
(b) A – V; B – I; C – IV; D – II
Reason : Respiration occurs by means of air chambers in (c) A – III; B – I; C – IV; D – II
submerged plants. (d) A – V; B – III; C – I; D – II
50. Assertion : Glycolysis is the first step of respiration in 55. Which of the following is incorrectly matched?
which glucose completely breaks into CO2 and H2O. (a) Protein ® Degarded by proteases
Reason : In this process, there is net gain of twenty four (b) Fats ® Fatty acid + PGAL
molecules of ATP. (c) Kreb’s cycle ® Carboxylation
51. Assertion : The inner membrane of mitochondria contains (c) Respiratory pathway ® Amphibolic
systems involving electron transport.
Reason : The mitochondrial matrix contains enzymes of DIAGRAM TYPE QUESTIONS
Kreb's cycle. 56. The given figure shows the few steps of the pathway are
indicated by P, Q, R and S major pathway of anaerobic
MATCHING TYPE QUESTIONS
respiration.
52. Match the column-I with column-II and choose the correct
combination from the options given below.
Column-I Column-II
A. Inner mitochondrial membrane I. Krebs cycle
B. Pyruvic acid is converted into II. ETC
CO2 and ethanol.
C. Cytoplasm III. Fermentation
D. Mitochondrial matrix IV. Glycolysis
(a) A – IV; B – III; C – II; D – I
(b) A – I; B – II; C – IV; D – III
(c) A – II; B – I; C – III; D – IV
(d) A – II; B – III; C – IV; D – I
+ CO2
53. In the given columns, column-I contain complexes and
column-II contain their alternative names. Select the correct
match from the option given below.
Respiration in Plants 125

Identify P, Q, R and S. X Y Z
P Q R S (a) GTP NADH2 CO2
(a) +
Ethanol Lactic acid PEP (b) FADH2 NADH2 GTP
NAD
(b) Ethanol + Lactic acid ATP (c) NADH2 FADH2 GTP
NAD
(c) Lactic acid Ethanol Glucose ADP (d) CO2 NADH2 ADP
(d) NAD Lactic acid Ethanol DHAP 59. In the given figure of electron transport chain - identify
P, Q, R, S and T.
57. Refer the figure of citric acid cycle and choose the correct Inner membrane
of mitochondria
combination of labelling (P, Q, R, S and T) the number of Q
P
carbon compounds in the substrate molecules, involved R
2H
in the given figure.
2H FMN S
T
2e
FeS Fes

T UQ 2e T
H2 UQ
2e
Cy b 2e
FeS

T UQ 2e T
H2 UQ
Oxaloacetic (P) (Q) 2e
Cy c – 1/2O2
+
2e 2H
Cy c Cy a – a1 H2O

(a) P - Matrix, Q - Outer membrane, R - RMNH2, S -


(R) NADH2, T - 2H
(T)
(b) P - Inter membrane space, Q - Matrix, R - NADH + H+,
S - NAD+, T - 2H+
(S) (c) P - Outer membrane, Q - Cristae, R - NAD, S - NADH
+ H+, T - H
(d) P - Cristae, Q - Outer chamber, R - NADH + H+, S -
(a) (P) 4C, (Q) 6C, (R) 5C, (S) 4C, (T) 4C NAD, T - 2H+
(b) (P) 6C, (Q) 5C, (R) 4C, (S) 3C, (T) 2C 60. The given figure represents the interelationship among
(c) (P) 2C, (Q) 5C, (R) 6C, (S) 4C, (T) 4C metabolic pathways showing the respiration mediated
breakdown of different organic molecules to CO2 and H2O.
(d) (P) 4C, (Q) 6C, (R) 4C, (S) 4C, (T) 5C
Now identify A to D.
58. Refer the figure and identify X, Y and Z
A Carbohydrates C
Ox
alo
ac Fatty Monosaccharides
eti + Glycerol E.g. Glucose Amino acids
c acids
ac
id
=
4C Glucose-6-phosphate

Fructose 1, 6 bis-phosphate

Glyceraldehyde
B 3-phosphate

Pyruvic acid

Kreb’s
cycle

H2 O CO2
EBD_7209
126 Biology
(a) It causes the breakdown of citric acid.
(b) It combines with glucose to produce carbon dioxide.
(c) It combines with carbon from organic molecules to
produce carbon dioxide.
(d) It combines with hydrogen from organic molecules
to produce water.
67. During glycolysis, glucose split into
(a) two pyruvic acid molecules.
(b) two coenzyme A molecules.
(c) two lactic acid molecules.
(d) one lactic acid plus one ethanol molecule.
68. Which one is correct sequence in glycolysis?
(a) G 6-P ® PEP ® 3-PGAL ® 3-PGA
(b) G 6-P ® 3-PGAL ® 3-PGA ® PEP
(c) G 6-P ® PEP ® 3-PGA ® 3-PGAL
(d) G 6-P ® 3-PGA ® 3-PGAL ® PEP
69. Which of the following is correct sequence in Kreb’s
cycle?
(a) Isocitric acid ® Oxalosuccinic acid ® a-ketoglutaric
acid
P Q R S (b) Oxalosuccinic acid ® Isocitric acid ® a-ketoglutaric
acid
(a) 1,3 di PGA 3 PGA Fr.1,6 d i P Fr. 6 P
(c) a-ketoglutaric acid ® Isocitric acid ® Oxalosuccinic
(b ) 3 PGA 1,3 di PGA Fr. 1,6 d i P Fr.6 P acid
(c) Fr. 1,6 d i P Fr. 6 P 3 PGA 1,3 d i PGA (d) Isocitric acid ® a-ketoglutaric acid ® Oxalosuccinic
(d ) Fr.6 P Fr. 1,6 d i P 3 PGA 1,3 d i PGA acid
70. In glycolysis, there is one step where NADH + H+ is
formed fr om NAD + , this is when 3-
CRITICAL THINKING TYPE QUESTIONS phosphoglyceraldehyde (PGAL) is converted to 1, 3-
62. Respiration substrates are bisphosphyglycerate (BPGA). This reaction shows
(i) the compounds that are oxidized to utilise energy. (a) oxidative dehydrogenation
(ii) the compounds that are reduced to utilise energy. (b) oxidative phosphorylation
(iii) the compounds that are oxidized to release energy. (c) oxidative dehydration
(iv) the compounds that are reduce to release energy. (d) oxidation reduction
(a) (i) only (b) both (i) and (ii) 71. If hexokinase, an enzyme that catalyzes the first step
(c) (iii) only (d) Both (iii) and (iv) reaction in glycolysis is blocked then what will be its
63. Respiration is the breakdown of impact on glycolytic pathway ?
(a) C-C bonds (b) C-H bonds (a) Glycolysis will speed up.
(c) H-H bonds (d) C-N bonds (b) Glycolysis will slow down.
64. Energy accumulate in ATP in (c) Glycolysis will stop.
(a) disulphide bond (d) Glycolysis will occurs normally.
(b) hydrogen bonds 72. How many ATP molecules could maximally be generated
(c) high energy phosphate bond from one molecule of glucose, if the complete oxidation of
(d) ester bond one mole of glucose to CO2 and H2O yields 686 kcal and
65. Life without air would be the useful chemical energy available in the high energy
(a) reductional. phosphate bond of one mole of ATP is 12 kcal ?
(b) free from oxidative damage. (a) Thirty (b) Fifty seven
(c) impossible. (c) One (d) Two
(d) anaerobic. 73. How many molecules of ATP are produced during
66. What is the function of molecular oxygen in cellular glycolysis?
respiration? (a) 2 (b) 4
(c) 6 (d) 8
Respiration in Plants 127
74. The reasons for the involvement of different enzyme in 82. Single turn of citric acid cycle yields
each step of glycolysis is that (a) 2FADH2, 2NADH2, 2GTP
(a) each step occurs in a different compartment of a cell. (b) 1FADH2, 2NADH2, 1GTP
(b) each step occurs in a different cells. (c) 1FADH2, 4NADH2, 1GTP
(c) each step involves a different chemical reaction. (d) 1FADH2, 1NADH2, 2GTP
(d) each step involves a different change in potential 83. Which one of the following is complex V of the ETS of
energy. inner mitochondrial membrane?
75. For bacteria to continue growing rapidly when they are (a) NADH dehydrogenase
shifted from an environment containing O2 to an anaerobic (b) ATP synthetase
environment, they must
(c) Succinate dehydrogenase
(a) produce more ATP per mole of glucose during (d) Ubiquinone
glycolysis.
84. O2 is used by
(b) produce ATP during oxidation of glucose.
(a) citric acid cycle
(c) increase the rate of glycolysis.
(b) electron transport chain
(d) increase the rate of TCA cycle.
(c) substrate level phosphorylation
76. In alcoholic fermentation, NAD+ is produced during the
(d) ATP synthase
(a) reduction of acetyldehyde to ethanol.
85. In mitochondria, protons accumulate in the
(b) oxidation of glucose.
(a) outer membrane (b) inner membrane
(c) oxidation of pyruvate to acetyl coA.
(c) inter membrane space (d) matrix
(d) hydrolysis of ATP to ADP.
86. The chemiosmotic coupling hypothesis of oxidative
77. Fermentation takes place
phosphorylation proposes that adenosine triphosphate
(a) under anaerobic conditions in many prokaryotes and (ATP) is formed because
unicellular eukaryotes.
(a) high energy bonds are formed in mitochondrial
(b) under aerobic conditions in many prokaryotes and
proteins.
unicellular eukaryotes.
(b) ADP is pumped out of the matrix into the
(c) under anaerobic conditions in all prokaryotes and
unicellular eukaryotes. intermembrane space.
(d) under aerobic conditions in all prokaryotes and (c) a proton gradient forms across the inner membrane.
unicellular eukaryotes. (d) there is a change in the permeability of the inner
78. In the conversion of pyruvic acid to acetyl coenzyme A, mitochondrial membrane toward adenosine
NAD+ is diphosphate (ADP).
(a) oxidized 87. Reduced form of ubiquinone is
(b) reduced (a) ubiquinine (b) ubiquinol
(c) broken into one-carbon units (c) ubiquitine (d) all of the above
(d) isomerized 88. Cytochrome oxidase contain
79. Initial step of TCA cycle to yield citric acid starts with the (a) Fe (b) Mg
condensation of (c) Zn (d) Cu
(a) acetyl group with OAA & H2O & catalyzed by the 89. Acetyl CoA is formed by the breakdown of
enzyme citrate synthase. (a) fats (b) fatty acid
(b) acetyl group with pyruvate & H2O & catalysed by (c) glycerol (d) PGAL
the enzyme citrates synthase. 90. In some succulent plants like Opuntia the R. Q. is zero
(c) acetyl group with OAA & H2O & is independent of because
the enzyme. (a) CO2 is released without any absorption of O2.
(d) none of the above. (b) O2 is absorbed but CO2 is not released.
80. End products of Krebs cycle from one molecule of glucose (c) there is often compensation point in these plants
is due to thick phylloclades.
(a) 2ATP, 2NADH, 2FADH2, CO2 and H2O (d) none of the above
(b) 2ATP, 8NADH, 2FADH2, CO2 and H2O 91. In alcoholic fermentation
(c) 8ATP, 4NADH, FADH2, CO2 and H2O (a) oxygen is the electron acceptor.
(d) ATP, 4NADH, FADH2, CO2 and H2O (b) triose phosphate is the electron donor while
81. In Kreb’s cycle, the conversion of succinyl CoA to acetaldehyde is the electron acceptor.
succinic acid requires (c) triose phosphate is the electron donor while pyruvic
(a) Acetyl CoA + GDP + iP (b) CoA + GTP + iP acid is the electron acceptor.
(c) Acetyl CoA+ GTP + iP (d) GDP + iP (d) there is no electron donor.
EBD_7209
128 Biology
92. Which one of the following is not included in glycolysis 97. Conversion of phosphoglyceraldehyde to 1, 1-biphosp-
? hoglyceric acid is a
(a) Substrate level phosphorylation occur. (a) primary reaction only
(b) The end products are CO2 and H2O. (b) redox reaction only
(c) ATP is formed. (c) primary and redox reaction
(d) ATP is used. (d) substrate level phosphorylation reaction
93. Inside an active mitochondrion, most electrons follow 98. In respiration, largest amount of energy is produced in
which pathway ? (a) anaerobic respiration
(a) Glycolysis ® NADH ® Oxidative phosphorylation (b) Krebs’ cycle
® ATP ® O2
(c) glycolysis
(b) Krebs’ cycle ® FADH2 ® ETS ® ATP
(d) none of the above
(c) ETS ® Krebs’ cycle ® ATP ® O2
99. In which one of the following reactions, substrate level
(d) Krebs’ cycle ® NADH + H+ ® Electron transport phosphorylation does not occur?
chain ® O2 (a) 1, 3-biphosphoglyceric acid ® 3, phosphoglyceric
94. The expressions given below shows the summary acid
equations I, II and III. (b) Glucose-6-phosphate ® Fructose 6 phosphate
(c) Succinyl CoA ® Succinic acid
(I) (d) Phosphoenol pyruvic acid ® pyruvic acid
100. Which of the following processes make direct use of
(II) C6H12O6 + NAD+ + 2ADP + 2Pi
oxygen?
® 2C3H4O3 + 2ATP + 2NADH + 2H+
(a) Glycolysis
(III) Pyruvic acid + 4NAD+ + FAD+ + 2H2O + ADP + Pi
(b) Fermentation
® 3CO2 + 4NADH + 4H+ + ATP + FADH2
(c) Electron transport
Categorise the summary equations under respective
(d) Krebs citric acid cycle
phases.
101. Lactic acid is formed by the process of
I II III (a) fermentation (b) HMP pathway
(a) Krebs’ cycle Glycolysis Fermentation (c) glycolysis (d) none of these
(b) Glycolysis Krebs’ cycle Fermentation 102. Oxidative phosphorylation refers to
(c) Fermentation Krebs’ cycle Glycolysis (a) anaerobic production of ATP.
(d) Fermentation Glycolysis Krebs’ cycle (b) the citric acid cycle production of ATP.
95. Fermentation is represented by the equation (c) production of ATP by chemiosmosis.
(a) C6H12O6 + 6O2 ® 6CO2 + 6H2O + 673 k cal (d) alcoholic fermentation.
(b) C6H12O6 ® 2C2H5OH + 2CO2 + 18 k cal 103. Cytochrome oxidase is a/an
(a) exoenzyme (b) endoenzyme
Light
(c) 6CO2 + 12H2O ¾¾¾¾¾® C6H12O6 + 6H2O + (c) proenzyme (d) coenzyme
Chlorophyll
6O2
(d) 6CO2 + 6H2O ® C6H12O6 + 6O2
96. The common phase between aerobic and anaerobic
respiration is called
(a) tricarboxylic acid cycle
(b) Oxidative phosphorylation
(c) Embden, Meyerhof, Parnas cycle (Glycolysis)
(d) Fermentation
\\node-d-19\E\vinod\2017\Amit\NCERT Xtract-12th-Biology\Biology\division chapter\Volume - I

Plant Growth and


15

Chapter
Development

9. The hormones which was first isolated from human urine


FACT/DEFINITION TYPE QUESTIONS
is.
1. Growth is accompanied by (a) indole-3-acetic acid (b) gibberellin
(a) anabolic process and energy (c) ethylene (d) kinetin
(b) catabolic process which occurs at the expense of 10. Which of the following is not an influence of auxins?
energy. (a) Apical dominance (b) Parthenocarpy
(c) metabolic processes which occur at the expense of (c) Phototropism (d) Fruit ripening
energy 11. Auxin herbicide is —
(d) energy only
(a) NAA (b) IAA
2. Germination takes place when the
(c) 2, 4-D (d) IBA
(a) previously dormant embryo is activated.
12. Gibberellin was first extracted from —
(b) cotyledons emerge above the ground.
(c) hypocotyl or epicotyl emerges above the ground. (a) Gibberella fujikuroi (b) algae
(d) vascular tissues begin to transport fluids. (c) bacteria (d) roots of higher plants
3. The method that renders the seed coat permeable to 13. Dwarfness can be controlled by treating the plant with —
water so that embryo expansion is not physically retarded (a) cytokinin (b) gibberellic acid
is called (c) auxin (d) antigibberellin
(a) vernalization (b) stratification 14. Hormone responsible for bolting is —
(c) denudation (d) scarification (a) IAA (b) kinetin
4. In arithmetic growth rate, when length of the organ is (c) ABA (d) GA
plotted against time, the nature of graph curve will be — 15. Which one is the first Gibberellins to be discovered and
(a) linear (b) sigmoidal remains the most intensively studied form?
(c) parabolic (d) hyperbolic (a) GA1 (b) GA2
5. The measure of the ability of the plant to produce new (c) GA3 (d) GA0
plant material is referred as —
16. Which of the following is not a function of cytokinin?
(a) efficiency index (b) absolute growth rate
(c) arithmetic growth (d) linear growth (a) Promotes apical dominance.
6. In which of the following process, the cell loose their (b) Promotes chloroplast development.
protoplasm to form tracheary element? (c) Promotes movement of nutrients.
(a) Dedifferentiation (b) Redifferentiation (d) Delay leaf senescence.
(c) Differentiation (d) Plasticity 17. The most abundant natural cytokinin that is isolated from
7. The ability of plant to follow different pathways and corn kernels and coconut milk is —
produce different structures in response to environment (a) carotenoid derivative
and phases of life is termed as — (b) pyrimidine derivative
(a) elasticity (b) growth efficiency (c) inetin
(c) plasticity (d) heterophylly (d) zeatin
8. Which one of the following is chemically identical to 18. Which of the following synthetic phytohormone was
ABA? discovered as a breakdown product of DNA?
(a) Indole-3-acetic acid (b) Kinetin (a) Kinetin (b) 2, 4-D
(c) Dormin (d) 2, 4-D (c) NAA (d) Thidiazuron
EBD_7209
130 Biology
19. Highest concentration of cytokinin is found in 31. Day neutral plants relate to
(a) area of continuous growth and meristematic region. (a) loss of activity during day time.
(b) meristematic region only. (b) flowering in all possible photoperiods.
(c) mature leaves. (c) overactive during day time.
(d) ripened fruit. (d) no flowering in any photoperiods.
20. Gaseous hormone is 32. Effect of low temperature which shortens vegetative
(a) ethylene (b) ethane period and hastens flowering is called _______.
(c) acetylene (d) benzaldehyde (a) photoperiodism (b) transpiration
21. Senescence in plants can be promoted by applying. (c) vernalization (d) photolysis
(a) auxins (b) cytokinins 33. Seed dormancy
(c) GA (d) ethylene (a) is the temporal delay to the process of germination.
22. Which of the following hormone is concerned with (b) is the permanent delay to the process of
climacteric respiration? germination.
(a) Ethylene (b) Auxin (c) minimizes seedling survival by preventing
(c) GA1 (d) Cytokinin germination.
23. Presence of which of the following compound (d) occurs due to the presence of growth inhibitors
accelerates abscission of flower and fruits? only.
(a) Acetaldehyde (b) Ethephon 34. 6-furfuryl amino purine, 2, 4-dichlorophenoxy acetic
(c) Ascorbic acid (d) Zeatin a ci d an d i ndole-3 acetic acid ar e exa mples
24. Lead abscission, fruit fall, bud dormancy occurs by which respectively for
of the following phytohormone? (a) synthetic auxin, kinetin and natural auxin.
(a) Auxin (b) Cytokinins (b) gibberellin, natural auxin and kinetin.
(c) Gibberellins (d) Abscisic acid (c) natural auxin, kinetin and synthetic auxin.
25. Which of the following hormone closes stomata in (d) kinetin, synthetic auxin and natural auxin.
response to water stress?
(a) IAA (b) ABA
STATEMENT TYPE QUESTIONS
(c) NAA (d) GA3
26. ABA is antagonistic to 35. Which of the following statement is not the
(a) GA (b) cytokinin characteristic of growth of an organism?
(c) ethylene (d) auxin (a) It is an irreversible permanent increase in size of an
27. The movement of plant and its parts in response to light organ / its part / an individual cell.
is called (b) It is accompanied by metabolic processes.
(a) photorespiration (b) photosynthesis (c) It is quantitative and intrinsic.
(c) phototropism (d)
(d) None of the above
photophosphorylation
36. Which of the following statement is not correct about
28. Photoperiodism is
the conditions for plant growth?
(a) recurrence of day and night.
(a) Oxygen helps in releasing metabolic energy
(b) effect of day length on flowering of a plant. essential for growth activities.
(c) flowering plant. (b) Nutrients are required by plants for the synthesis
(d) growth curvature in response to light. of protoplasm and act as source of energy.
29. The essential distinction between long-day and short- (c) Light and gravity affect certain stages of growth.
day plants is that flowering in long day plants is promoted
(d) Water oxidizes glucose to provide energy.
only when the day length exceeds a certain duration, called
the ______________. 37. Which one of the following statement regarding auxin is
correct?
(a) critical day length (b) short-long day length
(a) IAA and IBA are natural but NAA, 2 4-D and 2, 4, 5-
(c) long-short day length (d) photoperiod
T are synthetic auxins.
30. Plants requiring exposure to light for less than critical
(b) IAA and NAA are natural but IBA, 2, 4, 5-T and 2, 4-
period in order to flower are called ________.
D are synthetic auxins.
(a) long day plants.
(c) NAA and 2, 4, 5-T are natural but IAA, IBA and 2, 4-
(b) day neutral plants.
D are synthetic auxins.
(c) intermediate day plants.
(d) IAA, NAA, IAB, 2, 4-D and 2, 4, 5-T are synthetic
(d) short day plants.
auxins.
Plant Growth and Development 131

38. Which of the following statement is correct regarding (iv) Abscissic acid enables seeds to withstand
ethylene? desiccation.
(a) It delayes senescence. (a) (i) and (ii) only (b) (ii) and (iii) only
(b) It decreases the respiration rate during fruit ripening. (c) (i) and (iii) only (d) (ii), (iii) and (iv) only
(c) It breaks seed and bud dormancy. 44. Refer the following statements.
(d) It inhibits flowering in mango. (i) Promotes flowering in pineapple.
39. Which of the following is an incorrect statement? (ii) Used to prepare weed free lawn.
(a) Long day plants flower if the night length is shorter (iii) Promotes the abscission of older mature leaves and
than a critical period. fruits.
(b) Short-day plants flower when night length exceeds a The above functions are carried out by
critical dark period. (a) GA (b) C2H4
(c) Day-neutral plants are insensitive to day length. (c) ABA (d) Auxin
(d) The process of flowering does not occur in day- 45. Read the following statements (i-iv) r egarding
neutral plants. “ethephon” and answer the question which follows them.
40. Which of the following statement regarding vernalisation (i) Ethephon is sprayed in aqueous solution and is
is correct? readily absorbed and transported within the plant.
(a) Vernalisation refers to the promotion of flowering by (ii) It hastens fruit ripening in tomatoes and apples.
a period of low temperature. (iii) It can be used to induce fruit thinning in cotton, cherry
(b) The spring variety of crops are normally planted in and walnut.
the spring and come to flower before the end of the (iv) It is used to promote female sex expression in
growing season. cucumber and increase yield.
(c) It is not seen in biennial plants. How many of the above statement(s) is/ are correct?
(d) Subjecting cold treatment to sugar beet will result in (a) One (b) Two
flowering. (c) Three (d) All
41. Identify the correct and incorrect statements from the
following. ASSERTION/REASON TYPE QUESTIONS
(i) 17,500 new cells are produced per hour by a single In the following questions, a statement of Assertion is followed
maize root apical meristem. by a statement of Reason.
(ii) With the help of length, growth of pollen tube is (a) If both Assertion and Reason are true and the Reason is
measured. the correct explanation of the Assertion.
(iii) The growth of the leaf is measured in terms of volume. (b) If both Assertion and Reason are true but the Reason is
(iv) Cells in a watermelon may increase in size by upto not the correct explanation of the Assertion.
3,50,000 times. (c) If Assertion is true but Reason is false.
(d) If both Assertion and Reason are false.
(a) (i), (ii), (iii) are correct and (iv) is incorrect.
46. Assertion : Cytokinins are antisenescent.
(b) (i), (ii), (iv) are correct and (iii) is incorrect.
Reason : Effect of cytokinins is antagonistic to ethylene.
(c) (ii), (iii) are correct and (i), (iv) are incorrect.
47. Assertion : 2, 4-D is extensively used in agricultural and
(d) (i), (iv) are correct and (ii), (iii) are incorrect.
horticultural practices.
42. Which of the following statements on phytohormones
Reason : 2, 4-D is a herbicide.
& their action are correct?
48. Assertion : Auxins help to prevent fruits and leaves drop
(i) Cytokinins specially help in delaying senescence.
at early stages.
(ii) Auxins are involved in regulating apical dominance.
Reason : Auxins promote the abscission of older mature
(iii) Ethylene is specially useful in enhancing seed
leaves and fruits.
germination.
49. Assertion : The most widely used compound as source
(iv) Gibberellins are responsible for immature falling of
of ethylene is ethephon.
leaves.
Reason : Ethephon hastens fruit ripening in tomatoes and
(a) (i) and (iii) only (b) (i) and (iv) only
apples and accelerates abscission in stems and leaves.
(c) (i) and (ii) only (d) (ii) and (iii) only
50. Assertion : Decapitation is widely used in tea plantation
43. Which of the following statements are correct ? and hedge-making.
(i) Cytokinins promote senescence. Reason : Removal of shoot tips usually results in the
(ii) Auxins control apical dominance. growth of lateral buds.
(iii) Gibberellins promote shoot elongation.
EBD_7209
132 Biology
55. Match Column-I (Discovery) and Column-II (Scientists)
MATCHING TYPE QUESTIONS
and select the correct option.
51. Match the plant hormones given in column I with their
function/other name given in column II and choose the Column - I Column - II
correct combination. (Discovery) (Name of scientists)
Column -I Column -II A. Foolish seedling I. Cousins
(Plant hormone) (Function/other name) disease of rice
A. Zeatin I. Flowering hormone
B. Florigen II. Synthetic auxin B. Crystallized the II. F.W. Went
C. IBA III. Cytokinin Kinetic
D. NAA IV. Natural auxin C. Release of ethylene III. Skoog and Miller
(a) A – III; B – IV; C – I; D – II gas
(b) A – II; B – I; C – IV; D – III
(c) A – I; B – II; C – III; D – IV D. Bioassay of Auxin IV. E. Kurosawa
(d) A – III; B – I; C – IV; D – II (a) A – III; B – IV; C – I; D – II
52. Match the growth regulators given in column I with their
(b) A – IV; B – III; C – II; D – I
action given in column II and choose the correct
combination from the options given below. (c) A – IV; B – III; C – I; D – II
Column-I Column-II (d) A – IV; B – I; C – III; D – II
(Growth Regulator) (Action) 56. Refer the functions of the growth hormones given below.
A. Abscisic acid I. Delays leaf senescence I. Cell division II. Cell enlargement
B. Ethylene II. Inhibits seed III. Pattern formation IV. Tropic growth
germination V Flowering VI. Fruiting
C. Cytokinin III. Herbicide VII. Seed germination VIII. Response to wound
D. Auxin IV. Hastens fruit ripening
IX. Response to stresses of biotic and abiotic origin
(a) A-II; B-IV; C-I; D-III. (b) A-I; B-II; C-III; D-IV
(c) A-II; B-III; C-IV; D-I. (d) A-II; B-I; C-III; D-IV Identify the functions of growth promoters and growth
53. Match the plant hormones listed in column-I with their inhibitors from the above.
major role listed in column-II. Select the correct option
from the codes given below. Functions of growth Functions of growth
Column-I Column-II promoters inhibitor
A. Auxin I. Fruit ripening (a) I, II, VII, IX III, IV, V, VI, VII
B. Cytokinins II. Apical dominance (b) VIII, IX I, II, III, IV, V, VI, VII
C. Abscisic acid III. Antagonistic to GAs (c) I, II, III, IV, V, VI, VII VIII, IX
D. Ethylene IV. Stomatal opening and
closing (d) I, II, III, IV, V, VI, VII, IX VIII
V. Growth of lateral buds
(a) A – IV; B – V; C – III; D – I 57. Which one of the following pairs is not correctly
matched?
(b) A – II; B – IV; C – III; D – 1
(a) Abscisic acid — Stomatal closure
(c) A – II; B – V; C – III; D – I
(d) A – III; IV B – V; C – II; D – I (b) Gibberellic acid — Leaf fall
54. Match the growth regulators given in column-I with the (c) Cytokinin — Cell division
processes given in column-II and choose the correct (d) IAA — Cell wall elongation
combination. 58. Which one of the following pairs is incorrectly
Column-I Column-II matched?
(Growth regulators) (Processes) (a) Adenine derivative - Kinetin
A Auxin I. Colouring test in lemon (b) Carotenoid derivative - ABA
B Gibberellin II. Cell division test (c) Terpenes - IAA
in plants
(d) Indole compounds - IBA
C Cytokinin III. Avena curvature test
D Ethylene IV. Dwarf corn test 59. Which one of the following is incorrectly matched?
(a) A – III; B – IV; C – II; D – I (a) IAA - Indole compound
(b) A – I; B – IV; C – II; D – III (b) GA - Terpenes
(c) A – IV; B – III; C – I; D – II (c) ABA - Carotenoid derivatives
(d) A – II; B – I; C – IV; D – III (d) C2H4 - Adenine derivatives
Plant Growth and Development 133

DIAGRAM TYPE QUESTIONS


60. The picture given below shows a graph drawn on the
parameters of growth versus time. Identify A, B, C marked
in the given graph.

C I II III IV
Size
Fig : Experiment used to demonstrate that tip of the coleoptile
is the source of auxin. Arrows indicate direction of light
B
Time Which two coleoptiles will both bend towards the light
source?
(a) I and II (b) I and IV
A B C (c) II and III (d) III and IV
(a) Expo nen tial Lo g ph as e Statio nary p h as e 63. Which of the following shows the correct graph of
ph as e arithmetic growth?
(b ) Statio nary Lag phas e Lo g p has e
ph as e (a) ((b))
(c) Log p has e Statio nary Lo g arithmic ph as e
Growth
p has e
(d ) Log p has e Lag p has e Statio nary p h as e
Time

61. The given diagram represents the germination and seedling


development in beam.
Growth

Young leaves
Time
Cotyledons
B
C (c)c) (d)(
Soil line D
Growth Growth

A Time Time

Identify A, B, C and D marked in the given figure. 64. The graph given below shows a geometrical growth rate.

A B C D
(a) Plumule Cotyledons Hypocotyl Epicotyl

(b) Radicle Seed coat Epicotyl Hypocotyl

(c) Hypocotyl Cotyledons Epicotyl Root hair

(d) Root hair Cotyledons Plumule Hypocotyl

62. The following diagram shows four coleoptiles set up (I,


II, III & IV) at the start of an experiment.
EBD_7209
134 Biology
Which of the following statement regarding the above 67. The picture below shows three different types of plants
graph is incorrect? (marked as P-I, P-II and P-III) which flower on the basis
(a) The initial growth is slow, thereafter exponential of their critical photoperiod. Now identify these plants
phase and then stationary phase. (P-I, II and III).
(b) A sigmoidal curve is a characteristic of living organism
growing in a natural environment.
(c) With limited nutrient supply, the growth rate
increases rapidly leading to a exponential phase.
(d) Geometrical growth is typical for all cells, tissues and
organs of a plant.
65. The given figure shows the comparison of growth rate
of two leaves (A and B) over the period of one day. Both
the leaves A and B have increased their area in a given
time to produce A’ and B’ leaves respectively. If AGR =
absolute growth rate and RGR = relative growth rate,
then select the correct option.
B'

P-I P-II
A' P-III
10 cm2 55 cm2 (a) P-I = Long day plant; P-II = Short day plant;
P-III = Day neutral plant
(b) P-I = Short day plant; P-II = Long day plant;
2
50 cm P-III = Day neutral plant
2
5 cm (c) P-I = Short day plant; P-II = Short day plant;
A B P-III = Day neutral plant
(d) P-I =Long day plant; P-II = Long day plant;
P-III = Day neutral plant
Time = 1 day Time = 1 day
CRITICAL THINKING TYPE QUESTIONS
A-Leaf B-Leaf 68. Which of the following points is shown by cell at the
AGR RGR AGR RGR root or shoot apex ?
(a) 1% 1 2% 2 (a) Rich in protoplasm, possesses large conspicuous
triploid nucleus.
(b) 100% 5 10% 5 (b) Cell wall is cellulosic, primary in nature and with
(c) 5 100% 5 10% abundant plasmodesmata.
(d) 0.5 100% 1.5 100% (c) Rich in protoplasm with large conspicuous nucleus.
(d) Both (b) and (c).
66. Maryland Mammoth Tobacco is a short day plant. Its 69. Maximal size in terms of wall thickening and
critical duration of darkness is 10 hours. Under which protoplasmic modification are achieved by
of the following conditions will Maryland Mammoth (a) cells of divisional phase.
tobacco not flower ? (b) cells of maturation phase.
(c) cells of elongation phase.
(d) cells of meristematic tissue.
70. Arithmetic growth is expressed as
(a) Lt = L0 + rt (b) L0 = L0 + rt
(a) (b) (c) (d) (c) W1 = W0 ert (d) W0 = W1 ert
24- hour critical critical critical critical 71. A sigmoidal curve is obtained in geometrical growth
clock level level level level because
light dark (a) it has lag, log and then stationary phase.
Plant Growth and Development 135

(b) one daughter cell remains meristematic while the 79. Ethylene is used for
other daughter cell differentiates and matures. (a) retarding ripening of tomatoes.
(c) of the effect of environment on mitosis. (b) ripening of fruits.
(d) none of the above (c) slowing down ripening of apples.
72. Absolute growth rate is defined as the (d) both (b) and (c).
80. Skoog and Miller stimulated cell division in tobacco
(a) synthesis of new intercellular and extracellular
plant with degraded DNA. The active ingredient in
materials.
stimulants, which resembles cytokinins, was modified
(b) measurement & the comparison of total growth per (a) adenine (b) auxin
unit time (c) terpenes (d) carotenoids
(c) growth of the given system per unit time. 81. Which hormone is used to induce rooting from cut end
(d) increased growth per unit time. of the stem?
73. The exponential growth can be expressed as W1 = W0ert. (a) Kinetin (b) Indole butyric acid
What is ‘r’ in the expression ? (c) GA3 (d) Abscisic acid
(a) Relative growth rate and depends on final size. 82. Plant hormone which is translocated to other parts for
(b) Absolute growth rate & depends on initial size. growth of the plant is
(c) Relative growth and also referred to as efficiency (a) indole-3-acetic acid (b) gibberellins
index. (c) cytokinins (d) none of these
83. Flowering in pineapple is promoted by
(d) None of the above
(a) GA1 (b) C2H4
74. A primary root grows from 5 cm - 19 cm in a week. (c) NAA (d) Kinetin
Calculate the relative growth rate over the period.
84. Which one of the following promotes the abscission of
(a) 20% (b) 40% older mature leaves and fruits?
(c) 60% (d) 80% (a) Indole compounds
75. De-differentiation is (b) adenine derivatives
(a) regaining the lost capacity of division by living cells. (c) derivatives of carotenoids
(b) the ability of plant to produce different structures (d) gaseous hormones
in response to environment. 85. Removal of apical (terminal) bud of a flowering plant
(c) the intrinsic factor affecting plant growth. (or pruning of a flowering plant) leads to
(d) none of the above. (a) formation of new apical buds.
76. Which of the following represents the correct sequence (b) formation of adventitious roots on the cut side.
of the development process in a plant cell? (c) early flowering (or stopping of floral growth).
(a) Cell division ® Elongation ® Senescence ® (d) promotion of lateral branches.
Maturation 86. Which one of the following hormone is used to eradicate
(b) Meristematic cell ® Maturation elongation ® Death dicotyledonous weeds, but does not have its effect on
mature monocotyledonous plants?
(c) Cell division ® Elongation ® Maturation ®
(a) 2, 4-D (b) IAA
Plasmatic growth
(c) NAA (d) IBA
(d) Cell division ® Differentiation ® Maturation ®
87. The chemicals from an unlabelled bottle of plant
Senescence
hormone when applied to a plant, a curvature induced by
77. A phytohormone is light is found in the stem. The chemical is most probably a
(a) an ion that alters turgor pressure. (a) auxin (b) gibberellin
(b) small molecules of diverse chemical composition. (c) kinetin (d) ABA
(c) a pigment responds to environmental changes. 88. Which of the following acids is a derivative of terpenes?
(d) a secondary metabolic compound. (a) Gibberellic acid (b) Naphthalene acetic acid
78. Experiments done by Charles Darwin and his son on plant (c) Abscissic acid (d) ‘a’ and ‘c’
phototropism showed that 89. The fruits can be left on the tree longer so as to extend
(a) auxin is produced in the tip of the coleoptile. the market period. This is due to which function of GA ?
(b) the tip of the coleoptile is the light receptor of the (a) Bolting (b) Delays senescence
plant. (c) Internodal elongation (d) Parthenocarpy
(c) within coleoptiles, auxin moves laterally away from 90. Which of the following hormones causes fruits like apple
the source of the light. to elongate and improve its shape?
(d) more cell elongation takes place on the shaded side (a) GA (b) ABA
of the plant. (c) NAA (d) 2, 4D
EBD_7209
136 Biology
91. Which of the following hormone regulates the transition (a) Abscisic acid (b) Cytokinin
from juvenile to adult phases in confers? (c) Ethylene (d) Gibberellin
(a) Cytokinins (b) GA0 98. A long day plant flowers only if exposed to a light period
(c) ABA (d) IAA (a) more than its critical day length.
92. Which form of gibberellic acid is used to speed up the (b) less than its critical day length.
malting process in brewing industry? (c) equal to its critical day length.
(a) GA1 (b) GA2 (d) slightly less than its critical day length.
(c) GA3 (d) None of these 99. What type of plants will flower when short photoperiods
93. Farmers in a particular region were concerned that pre- are followed by long photoperiods?
mature yellowing of leaves of a pulse crop might cause (a) Long-short day plants (b) Short-long day plants
a decrease in the yield. Which treatment could be most (c) Intermediate plants (d) None of these
beneficial to obtain maximum seed yield? 100. One set (I) of a plant was grown at 12 hours day and 12
(a) Removal of all yellow leaves and spraying the hours night period cycles and it flowered while in the
remaining green leaves with 2, 4, 5-trichlorophenoxy other set (II) night phase was interrupted by flash of light
acetic acid. and it did not produce flower. Under which one of the
(b) Frequent irrigation of the crop. following categories will you place this plant (Set II)?
(c) Treatment of the plants with cytokinins along with (a) Long day (b) Darkness neutral
small doses of nitrogenous fertilizer. (c) Day neutral (d) Short day
(d) Application of iron and magnesium to promote 101. Which one of the following is not a mechanical method
synthesis of chlorophyll. for breaking seed dormancy?
94. Ethylene is not directly applied in the field as a gas due (a) Abrasions with knives
to its high diffusion rate. Which compound is used to (b) Abrasions with sandpaper
overcome this limitation? (c) Vigorous shaking into the flask
(a) Ethephon (b) Agent orange (d) By the action of microbes
(c) Benzaldehyde (d) 2, 4, 5-T 102. Which of the following effects is brought about by
95. Climacteric in the plants means gibberellins but not by auxins?
(a) increase in respiration rate before the ripening phase. (a) Inhibition of leaf abscission.
(b) decrease in respiration rate before the ripening (b) Stimulation of cambial activity.
phase. (c) Stimulation of fruit development.
(c) increase in respiration rate after the ripening phase. (d) Breaking of dormancy in leaf buds.
(d) decrease in respiration rate after the ripening phase. 103. Differentiation of shoot is controlled by
96. Abscisic acid is a (a) high gibberellin : cytokinin ratio.
(a) gaseous hormone (b) stress hormone (b) high auxin : cytokinin ratio.
(c) volatile hormone (d) liquid-gas mixture (c) high cytokinin : auxin ratio.
97. Which one of the following phytohormones is produced (d) high gibberellin : auxin ratio.
under water deficient condition and plays an important
role in the tolerance response of plants to drought?
16

Chapter
Digestion and Absorption

FACT/DEFINITION TYPE QUESTIONS 8. Crypts of lieberkuhn are present in


(a) intestine (b) stomach
1. Teeth of adult man, not present in milk dentition are (c) oesophagus (d) all of these
(a) molars (b) premolars 9. The sphincter of Oddi is present between
(c) canines (d) incisors (a) oesophagus and cardiac stomach.
2. Mammalian teeth are (b) pyloric stomach and duodenum.
(a) acrodont (b) homodont (c) hepatic duct and cystic duct.
(c) thecodont (d) polyphyodont (d) hepatopancreatic duct and duodenum.
3. Which of the following is a freely movable muscular organ 10. Which of the following does not produce any digestive
that is attached to the floor of the oral cavity by the enzyme ?
frenulum? (a) Acini of pancreas (b) Liver
(a) Jaws (b) Teeth (c) Stomach (d) Duodenum
(c) Tongue (d) Salivary glands 11. The number of salivary glands present in human beings
4. The function of tongue is to is
(a) grind and crush the food. (a) 5 pairs (b) 4 pairs
(b) position food for swallowing. (c) 3 pairs (d) 2 pairs
(c) add enzymes and moisture to the food. 12. Brunner’s glands are found in
(d) protect the opening of the pharynx during (a) submucosa of stomach
swallowing. (b) wall of rectum
5. A "1" flap called "2" prevents the entry of food into the (c) submucosa of duodenum
glottis – opening of the "3" – during "4". Identify 1, 2, 3, (d) mucosa of ileum
and 4. 13. Function of gall bladder is
(a) storage of bile.
1 2 3 4 (b) formation of enzymes.
(a) Cartilaginous Epiglottis Wind pipe Swallowing (c) synthesis of bile.
(d) formation of bile salts.
(b) Bony Glissons Stomach Digestion
14. Where does bile go after it leaves the gall bladder?
capsule (a) Duodenum (b) Jejunum
(c) Bony Pharynx Oral cavity Ingestion (c) Ileum (d) Caecum
(d) Cartilaginous Oesophagus Trachea Mastication 15. Which of the following is the structural and function unit
of the largest digestive gland?
6. The innermost layer of the digestive tract is the (a) Kupffer cells
(a) serosa membrane (b) Hepatic lobules
(b) mucosa membrane (c) Glissons capsule
(c) submucosa membrane (d) Crypts of Lieberkuhn
(d) lumen 16. The common bile duct in human is formed by the joining
7. ____ are needed in the diet as components of teeth and of
bone, regulators of acid-base balance and water balance, (a) pancreatic duct and bile duct.
and parts of certain enzymes. (b) cystic duct and hepatic duct.
(a) Carbohydrates (b) Fats (c) cystic duct and pancreatic duct.
(c) Minerals (d) Nucleic acids (d) hepatic duct and pancreatic duct.
EBD_7209
138 Biology
17. The substrate for amylase is 31. Which of the following enzyme digest milk protein in
(a) proteins (b) fats alkaline medium?
(c) starch (d) cane sugar (a) Pepsin (b) Trypsin
18. Digestive enzymes responsible for breaking down (c) Rennin (d) Chymotrypsin
disaccharides includes 32. Most of the fat digestion occurs in
(a) pepsin, trypsin and trypsinogen. (a) rectum (b) stomach
(b) amylase, pepsin, and lipase. (c) colon (d) small intestine
(c) sucrase, lactase, and maltase. 33. Muscular contraction in alimentary canal is called —
(d) pepsin, trypsin, and chymotrypsin. (a) deglutition (b) mastication
19. Salivary amylase is also known as (c) peristalsis (d) none of these
(a) ptyalin (b) gastrin 34. Enterokinase takes part in conversion of
(c) glyoxylase (d) pepsin (a) pepsinogen to pepsin
20. pH of saliva is (b) trypsinogen to trypsin
(a) 6.5 (b) 8 (c) protein into polypeptides
(c) 7 (d) 9.5 (d) caseinogen into casein
21. Maltase converts 35. HCl of gastric juice
(a) maltose to glucose at pH greater than 7. (a) inactivates ptyalin and activates pepsin.
(b) maltose to glucose at pH lesser than 7.0. (b) activates ptyalin and inactivates pepsin.
(c) maltose to alcohol. (c) inactivates both ptyalin and pepsin.
(d) starch to maltose at pH higher than 7.0. (d) activates both ptyalin and pepsin.
22. Which digestive organ mechanically and chemically 36. Rennin acts on milk protein and changes
transforms a food bolus into chyme? (a) caesinogen into caesin.
(a) Oesophagus (b) Stomach (b) caesin into paracaesin.
(c) Small intestine (d) Large intestine (c) caseinogen into paracaesin.
23. The disaccharidases are secreted with (d) paracaesin into caesinogen.
(a) saliva (b) gastric juice 37. Which cells of gastric mucosa secrete pepsinogen?
(c) intestinal juice (d) pancreatic juice (a) Parietal (b) Oxyntic
24. Succus entericus is a term used for (c) Chief cells (d) Goblet
(a) the junction of ileum and colon 38. Which enzyme initiates protein digestion ?
(b) inflammation of intestine (a) Pepsin (b) Trypsin
(c) vermiform appendix (c) Aminopeptidase (d) Carboxypeptidase
(d) digestive juice of intestine 39. Trypsin changes
25. Enzyme trypsin is secreted by (a) fats into fatty acids.
(a) duodenum (b) liver (b) proteins into peptones.
(c) pancreas (d) stomach (c) starch and glycogen into maltose.
26. The protein coated, water soluble fat globules are called (d) maltose into its components.
(a) chylomicrons (b) micelles 40. Most of the chemical digestion of food in humans is
(c) chyle (d) monoglycerides completed in the —
27. Milk protein is acted upon by a gastric enzyme in infant (a) small intestine (b) appendix
mammals. The enzyme is ___________. (c) ascending colon (d) stomach
(a) rennin (b) caesinogen 41. End product of protein digestion is —
(c) pepsinogen (d) pepsin (a) sugar (b) amino acid
28. Pancreatic juice helps in the digestion of (c) nucleotide (d) ammonia
(a) proteins and fats 42. Which one of the following is the constituents of the
(b) proteins and carbohydrates pancreatic juice while poured into the duodenum in
(c) fats and carbohydrates humans?
(d) proteins, fats and carbohydrates (a) Maltase (b) Enterokinase
29. Oxyntic cells are located in (c) Trypsinogen (d) Chymotrypsin
(a) Islets of langerhans. 43. Which homeostatic function of the liver is controlled and
(b) gastric epithelium and secrete pepsin. monitored in the pancreas?
(c) kidneys and secrete renin. (a) Deamination of amino acids
(d) gastric epithelium and secrete HCl. (b) Release of glucose
30. Enzyme trypsinogen is changed to trypsin by (c) Release of iron
(a) gastrin (b) enteropeptidase (d) Removal of toxins
(c) enterogastrone (d) secretin
Digestion and Absorption 139

44. Diglycerides on digestion give 54. Jaundice may be caused by retarded function of —
(a) one fatty acid and one glycerol (a) lungs (b) kidneys
(b) two fatty acid and one glycerol (c) heart (d) liver
(c) two glycerol and one fatty acid 55. ____i_____ is a diseased condition in which a person
(d) two glycerols only passes out watery stool frequently while
45. Semi - digested food in intestine is called ______ii_______ is known as infrequent elimination of
(a) chyle (b) chyme dry stool.
(c) bolus (d) none of these (a) i- Constipation, ii- Diarrhoea
46. Which of the following enzyme is not present in pancreatic juice? (b) i- Diarrhoea, ii- Constipation
(a) Chymotrypsinogen (b) Steapsin (c) i- Diarrhoea, ii- Vomiting
(c) Nuclease (d) Enterokinase (d) i- Constipation, ii- Vomiting
47. Emulsification refers to
(a) action of bile on breaking large fat droplets STATEMENT TYPE QUESTIONS
(b) action of gastric lipase on proteins 56. How do nutrients, absorbed by the small intestine, travel
(c) action of salivary amylase on starch to the individual cells of the human body?
(d) action of pancreas on fat droplets (a) The nutrients are absorbed from the small intestine
48. Protection of the walls of the stomach against the action
into the blood and move through the circulatory
of its own digestive juice
system to the body cells.
(a) is controlled by a centre in the medulla of the brain.
(b) The nutrients move from the small intestine directly
(b) results from the nervous reactions of the lining of
the stomach. to the liver and then move through the lymphatic
(c) results from the presence of an anti-enzyme chemical system to the body cells.
formed by the gastric gland. (c) The small intestine forces the nutrients into the
(d) results from the neutralizing, buffering and coating kidneys, where the nutrients are then dissolved in
mucus covering its inner surface. fluids used by the body cells.
49. A good source of lipase is (d) The body cells send nerve impulses indicating a lack
(a) saliva (b) gastric juice of nutrients to the small intestine, and the small
(c) bile (d) pancreatic Juice intestine sends the nutrients back to the cells.
50. Glucose, some amino acids and sodium are absorbed in 57. Which of the following statements is correct regarding
to blood by ________. chylomicrons ?
(a) simple diffusion (a) They are undigested proteins.
(b) active transport (b) They are undigested carbohydrates.
(c) facilitated transport (c) They are fat droplets coated with phospholipids.
(d) the statement is incorrect since they are absorbed in (d) They are fat droplets coated with glycerol and
lymph and not in blood.
proteins.
51. The ______ is primarily a storage chamber within the
58. Which of the following statements is correct regarding
digestive system, while the ______ reabsorbs water, ions,
enterokinase?
and generates the faeces.
(a) buccal cavity; midgut (a) It is a hormone that prevents the secretion of gastric
(b) crop; midgut juice.
(c) stomach; hindgut (b) It is an enzyme that activates the enzymes of
(d) buccal cavity; hindgut pancreatic juice.
52. The main function of lacteals in the villi of small intestine (c) It is an enzyme that activates the proteolytic enzymes
is the absorption of of succus entericus.
(a) amino acids and glucose (d) It is a hormone that prevents the secretion of
(b) glucose and vitamins pancreatic juice.
(c) water and mineral salts
59. Which of the following statement is incorrect ?
(d) fatty acids and glycerol
53. Under which of the following circumstances will insulin (a) Brunner’s glands are submucosal.
be secreted? (b) Rugae are the irregular folds of inner gastric mucosa.
(a) The blood sugar level in the liver is low. (c) Glisson’s capsule is the connective tissue sheath of
(b) The blood sugar level in the hepatic portal vein is hepatic lobule.
low. (d) Mesothelium or serosa lies in close proximity of the
(c) The blood sugar level in the islets of Langerhans is high. sub-mucosa.
(d) The glycogen level in the skeletal muscle is high.
EBD_7209
140 Biology
60. Which of the following is not the function of HCl in (iii) Food is digested completely in the stomach.
stomach? (iv) The large intestine is shorter than the small intestine.
(a) Breaking down proteins into peptones. (v) Inside the large intestine, most of the water from the
(b) Killing the bacteria ingested with food and drinks. undigested food is absorbed by the body.
(c) Promoting the formation of pepsin. (a) (i) and (ii) only
(d) Softening fibrous food elements. (b) (iii) and (v) only
61. What is common among amylase, renin and trypsin? (c) (i), (iv) and (v) only
(a) All are proteins. (d) (ii), (iii) and (iv) only
(b) All are proteolytic enzymes. 68. Which of the following statements are correct regarding
(c) Produced in stomach. jaundice?
(d) Act at pH lower than 7. (i) It indicates liver damage.
(ii) It involves infrequent elimination of dry stool.
62. What will happen if bile duct gets choked ?
(iii) It involves yellowing of skin and eyes.
(a) Faeces become dry.
(iv) It increases absorption of food.
(b) Acidic chyme will not be neutralized.
(a) (i) and (ii) only (b) (i) and (iii) only
(c) There will be little digestion in intestine.
(c) (i), (ii) and (iii) only(d) (i), (iii) and (iv) only
(d) Little digestion of fat will occur. 69. Which of the following statements is correct regarding
63. Which one of the following statements is true regarding absorption of fatty acids?
digestion and absorption of food in humans? (a) It is absorbed into the bloodstream at the small
(a) About 60% of starch is hydrolysed by salivary intestine.
amylase in our mouth. (b) It is absorbed into the lymph vessels at the small
(b) Oxyntic cells in our stomach secrete the proenzyme intestine.
pepsinogen. (c) It is absorbed into the lymph vessels from the blood
(c) Fructose and amino acids are absorbed through stream.
intestinal mucosa with the help of carrier ions like (d) It is absorbed into the bloodstream at the large
Na+. intestine.
(d) Chylomicrons are small lipoprotein particles that are
transported from intestine into blood capillaries. ASSERTION/REASON TYPE QUESTIONS
64. Which of the following statement is true ? In the following questions, a statement of Assertion is followed
(a) Pepsin cannot digest casein. by a statement of Reason.
(b) Trypsin can digest collagen. (a) If both Assertion and Reason are true and the Reason is
(c) Pepsin cannot digest collagen. the correct explanation of the Assertion.
(d) Chymotrypsin can digest casein. (b) If both Assertion and Reason are true but the Reason is
65. Which of the following statements is incorrect? not the correct explanation of the Assertion.
(a) Saliva lubricates the food. (c) If Assertion is true but Reason is false.
(b) Tongue helps in chewing of food. (d) If both Assertion and Reason are false.
(c) Salivary glands have protein digesting enzymes. 70. Assertion : Starch is hydrolysed by ptyalin to maltose.
(d) Bile does not contain any digestive enzymes. Reason : Sucrase hydrolyses sucrose to lactose.
66. Which of the following is not the function of digestive 71. Assertion : Presence of HCl in stomach is essential for
system? the process of digestion.
(a) It allows your body to get the nutrients and energy Reason : HCl kills and inhibits the growth of bacteria in
it needs from the food you eat. stomach.
72. Assertion : Chewing is one of the most important process
(b) It breaks down large food molecules into smaller
of digestion in animals.
molecules that can be used by cells.
Reason : Chewing helps in enzyme action.
(c) It converts foods to larger substances that can be
73. Assertion : Glucose, Na+ and amino acids are absorbed
absorbed and used by the cells of the body.
actively.
(d) It converts food into soluble and diffusible products
Reason : Na+, glucose and amino acids move against the
that can be absorbed by the blood.
concentration gradient and hence require energy.
67. Which of the following statements are true regarding 74. Assertion : Water and electrolytes are almost fully
human digestive system? absorbed in the large intestine.
(i) Food is not digested in the gullet. Reason : In large intestine, haustral contractions (slow
(ii) There is no digestive juice in the mouth. segmenting movements) roll the forming faeces over and
over, causing absorption of water and electrolytes.
Digestion and Absorption 141

MATCHING TYPE QUESTIONS 80. Match column I (organs) with column II (functions) and
choose the correct option.
75. Choose the correct matching pair. Column-I Column-II
(a) Renin – Protein (Organs) (Functions)
(b) Trypsin – Starch
A. Mouth I. Reclaims water and salts
(c) Invertase – Sucrose
(d) Amylase – Lactose B. Stomach II. Carries out most of the
76. Identify the correct set which shows the name of the digestion and absorption of
enzymes from where it is secreted and substrate upon nutrients.
which it acts C. Small intestine III. Releases amylase enzyme
(a) Pepsin – Stomach wall – Protein that break down
(b) Ptyalin – Intestine – Maltose carbohydrates.
(c) Chymotrypsin – Salivary gland –Lactose D. Large intestine IV. An acidic compartment that
(d) Ptyalin – Pancreas – Lipid begins to break proteins
77. Select the correct match of the digested products in into larger polypeptides.
humans with their absorption site and mechanism. (a) A – I; B – II; C – III; D – IV
Column-I Column-II (b) A – III; B – IV; C – II; D – I
(a) Fructose, Na+ – Small intestine, passive
(c) A – II; B – III; C – I; D – IV
absorption
(b) Glycerol, fatty acids – Duodenum, move as (d) A – IV; B – I; C – II; D – III
chylomicrons 81. The given pair represent substrate-enzyme pair. Which
(c) Cholesterol, maltose – Large intestine, of the following pair is (are) correct?
active absorption A. Maltose – Lactase
(d) Glycine, glucose – Small intestine, B. Protein – Pepsin
active absorption
C. Carbohydrate – Lipase
78. Which one of the following is the correct matching of the
site of action of the given substrate, the enzyme acting D. Casein – Renin
upon it and the end product? (a) A and B only (b) B and C only
Lipase (c) A and D only (d) B and D only
(a) Stomach : Fats ¾¾¾ ¾® Micelles
82. Match column I (food type) with column II their (enzymes)
Tryp sin
(b) Duodenum : Triglycerides ¾¾¾¾ ® and choose the correct option.
Monoglycerides Column-I Column-II
µAmylase (Food type) (Enzymes)
(c) Small intenstine : Starch ¾¾¾¾¾
®
Disaccharide A. Starch I. Nucleases
(Maltose) B. Protein II. Lipase
Pepsin C. Fats III. Amylase
(d) Small intestine : Proteins ¾¾¾¾ ® Amino acid
79. Which one of the following four secretions is correctly D Nucleic acid IV. Trypsin
matched with its source, target and nature of action? (a) A – III; B – IV; C – II; D – I
Secretion Source Target Action (b) A – I; B – II; C – III; D – IV
(a) Enterokinase Duodenum Gall Release of bile (c) A – II; B – III; C – IV; D – V
bladder juice (d) A – IV; B – I; C – II; D – III
(b) Gastrin Stomach Oxyntic Production of 83. Which of the following monomers are produced, when
lining cells HCl lipids are broken down by lipase?
(c) Salivary Salivary Mouth Breakdown of A. Nucleotides
amylase gland starch into
B. Amino acids
maltose
(d) Rennin Saliva Small Emulsification C. Glycerol
intestine of fats D. Fatty acids
(a) A and C only (b) C and D only
(c) C only (d) D only
EBD_7209
142 Biology
84. Match the following digestive processes given in column
I with their correct description given in column II and
then choose the correct option.
Column-I Column-II
(Digestive processes) (Description) S
A. Ingestion I. Elimination of
digestible solids.
B. Mechanical digestion II. Enzymatic
degradation of food
stuffs into simpler (a) A= amylase, B = maltase, C = lactase, D = invertase
molecules. (b) A = amylase, B = maltase, C= invertase, D= lactase
C. Chemical digestion III. Taking food into the (c) A = amylase, B = invertase, C = maltase, D= lactase
digestive systems (d) A = amylase, B = lactase, C = maltase, D = invertase.
D. Defecation IV. Chewing, mixing, 88. The diagram given below shows the human digestive
churning and system. Few structures are marked as I, II, III and IV.
segmentation of Which region of the human digestive system releases
food. bile juice?
(a) A – I; B – II; C – III; D – IV
(b) A – IV; B – I; C – II; D – III
(c) A – III; B – IV; C – II; D – I
(d) A – I; B – III; C – IV; D – II
85. Match the following organic molecules given in column I
with their correct description given in column II and then
choose the correct option.
Column-I Column-II
(I)
(Organic molecules) (Description)
A. Carbohydrates I. It is made of fatty (II)
acids and glycerol (III)
B. Proteins II. It is mostly ingested
in the form of starch
C. Nucleic acids III. It is built of long (IV)
chains of amino
acids
D. Lipids IV. It is made of ribose
(a) I (b) II
or deoxyribose
sugars and (c) III (d) IV
nitrogenous bases. 89. The given figure shows a section of small intestinal
(a) A – I; B – II; C – III; D – IV mucosa showing villi. What is the function of structure
(b) A – IV; B – I; C – II; D – III marked as I in the given figure?
(c) A – III; B – IV; C – I; D – II
(d) A – II; B – III; C – IV; D – I
86. Which of the following substances involved in organic
molecule digestion is correctly matched with their (I)
descriptions?
(a) Salivary amylase- It begins lipid digestion in mouth.
(b) Trypsin- It emulsifies fats for digestion.
(c) Bile- It digests proteins in small intestine.
(d) Maltase- It is a brush border enzyme that acts on
disaccharides.

DIAGRAM TYPE QUESTIONS


(a) To absorb amino acids.
87. The given flowchart shows the fate of carbohydrate
(b) To carry blood.
during digestion in the human alimentary canal. Identify
(c) To transport fat
the enzymes acting at stages indicated as A, B, C and D.
(d) To transport glucose
Digestion and Absorption 143

90. Refer the given diagram of digestive system to answer


the question.

x
93. Sphincter of Oddi controls the flow of digestive juice by
guarding which duct?
(a) 1 (b) 2
(c) 3 (d) 4
94. Which two ducts are responsible for the formation of a
duct that carry bile from the gall bladder and conduct it
Which of the following is associated with the structure into the first section of the small intestine?
marked as "X"? (a) 1 and 2 (b) 2 and 3
(a) It is a small blind sac which hosts some symbiotic (c) 3 and 4 (d) 4 and 1
microorganisms. 95. Which of the following best describes the role of the
(b) The undigested, unabsorbed substances enter into structure marked as Y in the given figure?
this structure through ileo-ceacal valve.
(c) It helps in mechanical churning and chemical
digestion of food.
(d) Both (a) and (c) Y
Directions (Qs. 91 and 92): Refer the given figure and answer
the questions. This figure shows the diagrammatic
representation of T.S of gut with few structures marked as 1, 2,
3 and 4

1
(a) Serves a minor role in the chemical digestion of fats.
Inner-circular 2
Outer-longitudinal (b) Mucus is secreted to protect the oesophagus from
the stomach enzymes.
3
(c) Digestive enzymes are secreted as food passes from
4
the oesophagus to the stomach.
(d) Connects the mouth to the stomach and has no
function in chemical digestion.

CRITICAL THINKING TYPE QUESTIONS


91. Which of the following structure forms glands in the
stomach and crypts in between the bases of villi? 96. In pancreatic juice, which of the followings are secreted
(a) 1 (b) 2 in inactive form as proenzymes ?
(c) 3 (d) 4 (a) Trypsin, chymotrypsin & carboxypeptidases
92. Which of the following part is made up of a thin (b) Pepsin, trypsin and chymotrypsin
mesothelium with some connective tissue? (c) Trypsin only
(a) 1 (b) 2 (d) Trypsin and chymotrypsin only
(c) 3 (d) 4 97. If the bile-pancreatic duct is blocked then which of the
Directions (Qs. 93 and 94): The given diagram shows the duct following will not be affected ?
systems of liver, gall bladder and pancreas in which few (a) Digestion of proteins
structures are marked as 1.2.3 and 4. On the basis of this figure (b) Emulsification of fats
answer the questions. (c) Level of blood glucose
(d) Digestion of starch
EBD_7209
144 Biology
98. Which of the following is the correct chronological order 107. In case of taking food rich in lime juice, the action of
for flow of food from mouth to anus? ptyalin on starch is
(a) Oesophagus ® Stomach ® Small intestine ® Large (a) enhanced (b) reduced
intestine (c) unaffected (d) stopped
(b) Large intestine ® Oesophagus ® Stomach ® Small
108. If the chyme of a person who had orally consumed only
intestine
starch as food is analysed before it enters the duodenum,
(c) Small intestine ® Large intestine ® Oesophagus ®
stomach it will show the presence of
(d) Stomach ® Small intestine ® Large intestine ® (a) maltose and glucose
Oesophagus (b) dextrin and maltose
99. Which of the following occurs in the duodenum? (c) starch, dextrin and maltose
(a) Absorption of vitamins and minerals. (d) starch, dextrin and glucose
(b) Mixing of food with pancreatic juice and bile. 109. An adolescent human below 17 years of age normally has
(c) Mastication of food.
dental formula as
(d) Absorption of water.
100. From deep to superficial, what are the tunics of the 2, 1, 2, 0 2, 1, 2, 2
(a) (b)
intraperitoneal portions of the alimentary canal? 2, 1, 2, 0 2, 1, 2, 2
(a) Serosa, muscularis, submucosa and mucosa
2, 1, 3, 2 2, 2, 3, 2
(b) Mucosa, submucosa, muscularis and serosa (c) (d)
(c) Adventia, muscularis, submucosa and mucosa 2, 1, 3, 2 2, 2, 3, 2
(d) Mucosa, submucosa, muscularis and adventia 110. Which of the following is the correct chronological order
101. If for some reason our goblet cells becomes non- for the enzyme activity of some enzymes taking part in
functional then this will adversely affect protein digestion?
(a) production of somatostatin.
(a) Pepsin ® Trypsin ® Peptidase
(b) secretion of sebum from the sebaceous glands.
(c) maturation of sperms. (b) Pepsin ® Peptidase ® Trypsin
(d) smooth movement of food down the intestine. (c) Trypsin ® Pepsin ® Peptidase
102. The digestion of butter begins with (d) Peptidase ® Trypsin ® Pepsin
(a) saliva 111. What will happen if the secretion of parietal cells of gastric
(b) gastric juice
glands is blocked with an inhibitor?
(c) pancreatic juice
(a) Enterokinase will not be released from the duodenal
(d) intestinal juice
mucosa and so trypsinogen is not converted into
103. Removal of gall bladder in man would lead to
(a) impairment of digestion of fats. trypsin.
(b) impairment of digestion of protein. (b) Gastric juice will be deficient in chymosin.
(c) jaundice (c) Gastric juice will be deficient in pepsinogen.
(d) increased acidity in intestine. (d) In the absence of HCl secretion, inactive pepsinogen
104. During prolonged fasting is not converted into the active enzyme pepsin.
(a) first fats are used up, followed by carbohydrates 112. Which enzymes are likely to act on the baked potatoes
from liver and muscles and proteins in the end. eaten by a man, starting from the mouth and as it moves
(b) first carbohydrates are used up, followed by fat and down the alimentary canal?
proteins towards end. (a) Salivary maltase ® Carboxypeptidase ®
(c) first lipids, followed by proteins and carbohydrates Trypsinogen
towards end.
(b) Pancreatic amylase ® Salivary amylase ® Lipases
(d) none of the above.
(c) Disaccharidase like maltase ® Lipases ® Nucleases
105. A healthy person eats the following diet - 5 gm raw sugar,
4 gm albumin, 10 gm pure buffalo ghee adultrated with 2 (d) Salivary amylase ® Pancreatic amylase ®
gm vegetable ghee (hydrogenated vegetable oil) and 5 Disaccharidases
gm lignin. How many calories he is likely to get? 113. A person who is on hunger strike and is surviving only
(a) 144 (b) 126 on water, will have
(c) 164 (d) 112 (a) less amino acids in his urine.
106. If pH of stomach is 7 then which component of food would (b) more glucose in his blood.
be affected? (c) less urea in his urine.
(a) Starch (b) Protein
(d) more sodium in his urine.
(c) Fat (d) Sucrose
Digestion and Absorption 145

114. If the parietal cells of the gut epithelium become partially 119. Which part is infected when a person is suffering from
non-functional, what is likely to happen? diarrhoea ?
(a) The pancreatic enzymes, specially the trypsin and (a) Small intenstine (b) Large intestine
lipase will not work efficiently. (c) Stomach (d) None of these
(b) The pH of stomach will fall abruptly. 120. Which process transported the bolus into the pharynx
(c) Steapsin will be more effective. and then into the oesophagus?
(d) Proteins will not be adequately hydrolysed by pepsin (a) Deglutition (b) Peristalsis
into proteases and peptones. (c) Vomiting (d) All of these
115. What happens to the food when it enters from oesophagus 121. If the epiglottis does not function correctly, then what
to stomach? will happen to the human being ?
(a) Food mixes with juices and protein digestion starts. (a) One might congest.
(b) Food mixes with juices and carbohydrate digestion (b) Peristalsis will discontinue.
starts. (c) Acid reflux disease will damage the oesophagus.
(c) Food quickly moves into small intestine. (d) Swallowing will be difficult or impossible.
122. Study the statement given below and answer the question.
(d) Food quickly moves into large intestine.
“Even though the contents of stomach are very acidic,
116. The hydrochloric acid secreted by the gastric glands
the stomach usually does not cause damage to itself”.
activates the enzyme _____i_____ while th e
Identify the reasons (i to iv) given below which are
_____ii_____ secreted by gastric glands protects the
responsible for the above statement.
lining of the stomach from the action of acid.
(i) The release of gastric juices is controlled to avoid
(a) i- pepsin, ii- mucus
too high a concentration.
(b) i- pepsin, ii- trypsin
(ii) The stomach lining is quick to repair itself.
(c) i- amylase, ii- trypsin
(iii) Mucus forms a thick protective coating for the
(d) i- amylase, ii- rennin
stomach.
117. Which digestive processes takes place in the mouth?
(iv) Food and water dilute the gastric juices
(a) Both chemical and physical digestion.
(a) (i), (ii) and (iii)
(b) Chemical digestion only.
(b) (ii), (iii) and (iv)
(c) Physical digestion only.
(c) (i) and (iv) only
(d) Neither chemical nor physical digestion.
(d) All of the above
118. Following are given parts of small and large intestine.
123. If the large intestine becomes irritated and peristalsis
(i) Cecum (ii) Colon
increases then which of the following will result?
(iii) Jejunum (iv) Rectum
(a) Indigestion (b) Diarrhoea
(v) Duodenum (vi) Ileum
(c) Constipation (d) Vomiting
Choose the arrangement that lists the structures in the
order food passes through them from the small intestine
to the anus.
(a) (i) – (ii) – (iii) – (iv) – (v) – (vi)
(b) (v) – (iii) – (vi) – (i) – (ii) – (iv)
(c) (iv) – (v) – (ii) – (iii) – (i) – (vi)
(d) (iii) – (v) – (vi) – (ii) – (i) – (iv)
EBD_7209
Breathing and Exchange
17

Chapter
of Gases

FACT/DEFINITION TYPE QUESTIONS (c) inspiratory capacity


(d) functional residual capacity
1. Which of the following has no specialized respiratory 11. The total volume of air a person can expire after normal
structures ? inspiration is called
(a) Crab (b) Earthworm (a) residual volume
(c) Salmon (d) Ant (b) vital capacity
2. The structure which prevents the entry of food into the (c) expiratory capacity
windpipe is (d) functional residual capacity
(a) gullet (b) glottis 12. Functional residual capacity can be represented as
(c) tonsil (d) epiglottis (a) TV + ERV (b) ERV + RV
3. Lungs are present in (c) RV + IRV (d) ERV + TV + IRV
(a) mediastinum (b) pleural cavity 13. The total lung capacity is represented by
(c) pericardial cavity (d) thoracic cavity (a) Tidal volume + Vital capacity
4. The structure which does not contribute to the breathing (b) Tidal volume + Residual volume
movements in mammals is (c) Vital capacity + Residual volume
(a) larynx (b) ribs (d) Inspiratory + Expiratory reserve volumes
(c) diaphragm (d) intercostal muscles 14. When 1200 mL air is left in the lungs, it is called
5. Intercostal muscles are found attached with (a) vital capacity
(a) diaphragm (b) ribs (b) tidal volume
(c) pleura (d) lungs (c) residual volume
6. Membrane separating air in pulmonary alveoli from blood (d) inspiratory reserve volume
capillaries is 15. The volume of air breathed in and out during normal
(a) alveolar epithelium breathing is called
(b) cardiac epithelium (a) vital capacity
(c) endothelium of blood capillaries (b) inspiratory reserve volume
(d) both (a) and (c) (c) expiratory reserve volume
7. The nose produces ____________ to put moisture in (d) tidal volume
the air and catch dust and bacteria. 16. In anaerobic respiration
(a) mucus (b) bronchi (a) O2 is taken in. (b) CO2 is taken in.
(c) cilia (d) oxygen (c) O2 is given out. (d) CO2 is given out.
8. The smallest and thinnest tube in the lung is 17. Every 100 ml of oxygenated blood delivers following amount
(a) trachea (b) larynx of O2 to the tissues under normal physiological contition.
(c) bronchi (d) bronchiole (a) 5 ml (b) 25 ml
9. The path that leads from the throat to the lungs is known as (c) 50 ml (d) More than 50 ml
(a) trachea (b) oesophagus 18. What will be the pO2 and pCO2 in the atmospheric air as
(c) epiglottis (d) larynx compared to that of alveolar air ?
10. The amount of air remaining in the air passages and alveoli (a) pO2 lesser and pCO2 higher
at the end of quiet respiration is called (b) pO2 higher and pCO2 lesser
(a) tidal volume (c) Both pO2 and pCO2 lesser
(b) inspirating reserve volume (d) Both pO2 and pCO2 higher
Breathing and Exhange of Gases 147

19. As blood becomes fully O2 saturated, haemoglobin (a) 5ml of CO to the alveoli
combines with____ molecule(s) of oxygen. (b) 6ml of CO2 to the plasma
(a) 1 (b) 2 (c) 4ml of CO2 to the alveoli
(c) 4 (d) 8 (d) 7ml of CO to the plasma
20. What percentage of oxygen is carried by haemoglobin ? 32. Asthma is caused due to
(a) 90 (b) 98 (a) infection of lungs.
(c) 23 (d) 73 (b) spasm in bronchial muscles.
21. Partial pressure of oxygen in inspired and expired air is (c) bleeding into pleural cavity.
……. and ……. mm of Hg. (d) infection of trachea.
(a) 100, 46 (b) 158, 40 33. Emphysema developes mainly because of
(a) allergy or hypersensitization.
(c) 158, 90 (d) 100, 95
(b) spasm of the smooth muscles of bronchioles.
22. p50 value denotes
(c) cigarette smoking.
(a) percentage of oxygen
(d) inflammation of the alveoli
(b) partial pressure of O2
(c) percentage of haemoglobin 34. Respiratory mechanism is controlled by
(a) central nervous system
(d) percentage of oxyhaemoglobin
(b) sympathetic nervous system
23. Most oxygen is carried by the blood __(i)__. Most carbon
(c) parasympathetic nervous system
dioxide is carried by the blood __(ii)__. (d) autonomic nervous system
(a) (i) attached to haemoglobin. (ii) in the form of 35. The respiratory centre, which regulates respiration, is
bicarbonate ions. located in
(b) (i) dissolved in plasma . (ii) dissolved in plasma. (a) cerebral peduncle (b) vagus nerve
(c) (i) in the form of H+ ions . (ii) in the form of bicarbonate (c) pons varolii (d) medulla oblongata
ions. 36. The pneumotaxic centre that can moderate the function
(d) (i) attached to haemoglobin. (ii) attached to of respiratory rhythm centre is located in
haemoglobin. (a) dorsal side of medulla
24. Which of the following gases makes the most stable (b) ventral side of medulla
combination with the haemoglobin of RBCs? (c) aortic arch and carotid artery
(a) CO2 (b) CO (d) pons varolii
(c) O2 (d) N 37. Which of the following normally contains the highest
25. In mammals, carbon dioxide is transported from tissues to concentration of oxygen ?
respiratory surface by (a) Body cells
(a) plasma (b) R.B.C. (b) Inhaled air
(c) W.B.C. (d) both (a) and (b) (c) Air in the pulmonary trunk
26. __(i)__ in CO2 in your blood, which causes __(ii)__ in (d) Blood entering the lungs
pH, would cause your breathing to speed up. 38. The volume of the air that leaves the lungs in human is
(a) (i) An increase, (ii) a rise measured by
(b) (i) An increase, (ii) a drop (a) respirometer
(c) (i) A decrease, (ii) a rise (b) spirometer
(d) (i) A decrease, (ii) a drop (c) sphygmomanometer
27. Carbonic anhydrase is found in high concentration in (d) monometer
(a) leucocytes (b) blood plasma 39. Which of the following organ receives electrical messages
(c) erythrocytes (d) lymphocytes from the brain for breathing in and out?
28. Dissociation of oxyhaemoglobin can be promoted by (a) Diaphragm (b) Bronchi
(a) low pCO2 (b) high pCO2 (c) Bronchioles (d) Alveoli
(c) low body temperature (d) high blood pH 40. Which of the following group are supported by incomplete
29. Which of the following structures is the actual gas cartilaginous rings?
exchange surface in the mammalian respiratory system ? (a) Pharynx, primary & tertiary bronchi and initial
(a) Bronchus (b) Alveolus bronchioles.
(c) Bronchiole (d) Trachea (b) Trachea, primary & secondary bronchi, and initial
30. Residual air mostly occurs in bronchioles.
(a) alveoli (b) bronchus (c) Larynx, primary, secondary and tertiary bronchi, and
(c) nostrils (d) trachea initial bronchioles.
31. Every 100 ml of deoxygenated blood delivers (d) Trachea, primary, secondary and tertiary bronchi, and
approximately _________________. initial bronchioles.
EBD_7209
148 Biology
41. Thoracic chamber is formed (A) by the vertebral column, 49. Which one of the following is the correct statement for
(B) by the sternum, (C) by the ribs and on the (D) side by respiration in humans?
the dome shaped diaphragm. (a) Workers in grinding and stone-breaking industries
Identify A, B, C and D. may suffer from lung fibrosis.
(a) A – dorsally, B – ventrally, C – laterally, D – lower (b) About 90% of carbon dioxide (CO2) is carried by
(b) A – ventrally, B – laterally, C – dorsally, D – upper haemoglobin as carbaminohaemoglobin.
(c) A – laterally, B - ventrally, C – dorsally, D – lower (c) Cigarette smoking may lead to inflammation of bronchi.
(d) A – dorsally, B - laterally, C – ventrally, D – upper (d) Neural signals from pneumotaxic centre in pons region
of brain can increase the duration of inspiration.
STATEMENT TYPE QUESTIONS 50. Mark the correct statement.
(a) Volume of residual air is higher than tidal volume.
42. Which of the following statement is correct? (b) Volume of complementary air is higher than tidal
(a) Tracheal rings are of hyaline cartilage . volume.
(b) Dorsal side of thoracic chamber is formed by sternum. (c) Volume of supplementary air is lower than residual
(c) Expiration occurs when there is negative pressure in capacity.
lungs. (d) All of the above
(d) All of the above 51. Which of the following statements is correct?
43. Which of the following statement is incorrect? (a) Inspiration is a passive process whereas expiration
(a) Diffusion membrane is made up of 3-major layers. is active.
(b) Solubility of CO2 is higher than O2 by 25 times. (b) Inspiration is active process whereas expiration is passive.
(c) Breathing volumes are estimated by spirometer. (c) Inspiration and expiration are active process.
(d) High conc. of hydrogen ion s favours (d) Inspiration and expiration are passive process.
oxyhaemoglobin formation. 52. Which of the following conditions is responsible for
44. With reference to human respiration, which is correct? increase in ventilation rate of lungs ?
(a) Pulmonary ventilation is equal to alveolar ventilation. (a) Increase of CO2 content in inhaled air.
(b) Alveolar ventilation is more than pulmonary (b) Increase of CO2 content in exhaled air.
ventilation. (c) Decrease in O2 content of inhaled air.
(c) Alveolar ventilation is less than pulmonary (d) Decrease in O2 content of exhaled air.
ventilation. 53. What happens during breathing?
(d) Pulmonary ventilation is less than alveolar (i) Size of our chest increases.
ventilation. (ii) Size of our chest decreases.
45. Which of the following statement correctly defines Bohr (iii) Carbon dioxide is removed out from body.
effect? (iv) Air enters our lungs.
(a) Rise in p50 with a decrease in CO2 conc. (a) (i) and (ii) (b) (i) and (iv)
(b) Rise in p50 with decrease in pH. (c) (ii) and (iii) (d) (ii) and (iv)
(c) Rise in p50 with increase in O2. 54. Air travels through
(d) Fall in p50 with decrease in pH. (i) Nose (ii) Skin
46. Which one of the following statement is correct? (iii) Trachea (iv) Mouth
(a) All animals require a medium for cellular respiration. (a) Both (i) and (ii) (b) Both (i) and (iv)
(b) In all animals oxygen is transported by blood. (c) only (i) (d) only (ii)
(c) All animals need oxygen for respiration. 55. Which of the following statements are true/false?
(d) All of the above (i) The blood transports CO2 comparatively easily
47. The correct statement in respect of protein haemoglobin because of its higher solubility.
is that it (ii) Approximately 8 -.9% of CO2 is transported being
(a) maintains blood sugar level. dissolved in the plasma of blood.
(b) acts as an oxygen carrier in the blood. (iii) The carbon dioxide produced by the tissues, diffuses
(c) functions as a catalyst for biological reactions. passively into the blood stream and passes into red
(d) forms antibodies and offers resistance to diseases. blood corpuscles and react with water to form H2CO3
48. Identify the correct statement with reference to transport (iv) The oxyhaemoglobin (HbO2) of the erythrocytes is
of respiratory gases by blood ? basic.
(a) Haemoglobin is necessary for transport of carbon (v) The chloride ions diffuse from plasma into the
dioxide and carbonic anhydrase for transport of
erythrocytes to maintain ionic balance.
oxygen.
(a) (i), (iii) and (v) are true; (ii) and (iv) are false.
(b) Haemoglobin is necessary for transport of oxygen and
(b) (i), (iii) and (v) are false; (ii) and (iv) are true.
carbonic anhydrase for transport of carbon dioxide.
(c) Only oxygen is transported by blood. (c) (i), (ii), and (iv) are true; (iii) and (v) are false.
(d) Only carbon dioxide is transported by blood. (d) (i), (ii) and (iv) are false; (iii) and (v) are true.
Breathing and Exhange of Gases 149

56. Read the following statements and select the correct


MATCHING TYPE QUESTIONS
option.
(i) Carbonic anhydrase is present in the erythrocytes . 63. Match the column I (Animals) with column II (Mode of
(ii) In erythrocytes the carbon dioxide combine with respiration) and choose the correct option.
water and is transported. Column - I Column - II
(a) Statement (i) is correct and is responsible for
(Animals) (Mode of respiration)
statement (ii).
(b) Statement (i) is not correct but statement (ii) is A. Earthworm I. Pulmonary
correct. B. Human II. Branchial
(c) Both statement (i) and (ii) are incorrect.
C. Prawn III. Tracheal
(d) Statement (i) is correct but not involved in statement (ii).
57. Which one of the followings statements is not correct? D. Insects IV. Cutaneous
(a) Total volume of air a person can expire after a normal (a) A – I; B – II; C – III; D – IV
inspiration is called expiratory capacity. (b) A – IV; B – II; C – I; D – III
(b) Binding of oxygen with haemoglobin is primarily (c) A – IV; B – I; C – II; D – III
related to partial pressure of CO2.
(d) A – III; B – II; C – IV; D – I
(c) Every 100 ml of deoxygenated blood delivers
approximately 4 ml of CO2 to the alveoli. 64. Match the disorders given in column - I with symptoms
(d) Every 100 ml of oxygenated blood can deliver around under column - II. Choose the answer which gives the
5 ml of O2 to the tissues under normal physiological correct combination.
conditions. Column-I Column-II
(Disorders) (Symptoms)
ASSERTION/REASON TYPE QUESTIONS
A. Asthma I. Inflammation of nasal
In the following questions, a statement of Assertion is followed tract
by a statement of Reason. B. Bronchitis II. Spasm of bronchial
(a) If both Assertion and Reason are true and the Reason is
muscles
the correct explanation of the Assertion.
(b) If both Assertion and Reason are true but the Reason is C. Rhinitis III. Fully blown out alveoli
not the correct explanation of the Assertion. D. Emphysema IV. Inflammation of
(c) If Assertion is true but Reason is false. bronchi
(d) If both Assertion and Reason are false.
V. Cough with blood
58. Assertion: A sigmod curve is obtained when percentage
saturation of haemoglobin with O2 is plotted against the strained sputum
pO2. (a) A – IV; B – II; C – V; D – I
Reason: Every 100 mL of oxygenated blood can deliver (b) A – V; B – III; C – II; D – I
around 5mL of O 2 to the tissues under n ormal (c) A – III; B – I; C – V; D – IV
physiological conditions. (d) A – II; B – IV; C – I; D – III
59. Assertion: If two men, expire the same volume of air after 65. Listed below are four respiratory capacities (A-D) in
normal inspiration, they have the same expiratiory capcity. column I and four jumbled respiratory volume of a normal
Reason: Expirtory capacity includes tidal valume and human adult in column II.
inspiratory reserve volume. Column -I Column-II
60. Assertion: A rise in pCO2, H+ ions and temperature shifts (Respiratory capacities) (Respiratory volumes)
the HbO2 dissociation curve to right. A. Residual volume I. 2500 mL
Reason: A rise in pCO2 or fall in pH decreases oxygen
B. Vital capacity II. 3500 mL
affinity fr haemoglobin.
61. Assertion: The abdominal muscles are primarily involved C. Inspiratory reserve volume III. 1200 mL
in generating pressure gradient between the lungs and D. Inspiratory capacity IV. 4500 mL
the atmosphere. Which one of the following is the correct matching of two
Reason: The strength of inspiration and expiration can capacities and volume?
be increased by additional muscles in diaphragm and (a) A – I; B – II; C – III; D – IV.
intercostal muscles. (b) A – IV; B – III; C – I; D – II.
62. Assertion: Emphysema is the permanent abnormal (c) A – II; B – III; C – IV; D – I.
inflation of air space of teminal bronchioles or alveolar
(d) A – III; B – IV; C – I; D – II.
sacs.
EBD_7209
150 Biology
66. Match the column I (Organs) with column II (Functions) (a) To prevent food from entering into trachea.
and choose the correct option. (b) To filter and warm the air.
Column - I Column - II (c) To help in exchange of gases.
(Organs) (Functions) (d) To catch dust and bacteria.
A. Nose I. Stops food from going
70. In the given diagarm of human respiratory system which
down into lungs.
marked label (I, II, III & IV) is the common passageway
B. Epiglottis II. Produces sound.
where the nasal and oral cavities meet?
C. Pharynx III. Traps bacteria as well as
dust.
D. Larynx IV. Allows air to pass from I
nose to oesophagus. II
III
(a) A – II; B – III; C – I; D – IV
(b) A – III; B – I; C – IV; D – II
(c) A – I; B – IV; C – II; D – III
(d) A – IV; B – II; C – III; D – I IV
67. Which one of the following four organs of respiratory
system is correctly matched with its characteristics?
A. Bronchi – Two branches of the trachea that (a) I ® nasal-cavity (b) II ® pharynx
brings air into the lungs. (c) III®trachea (d) IV®lungs
B. Trachea – Small flap that prevents food from 71. In which one of the options given below, the one part A,
entering B, C or D is correctly identified along with its function?
C. Diaphragm – Dome shaped muscle that pushes
on the lungs during exhalation.
D. Alveoli – Pair of organs that inflate as you
inhale and deflate as you exhale. D
(a) A and B only (b) C and D only
(c) A and C only (d) B and D only C
A
DIAGRAM TYPE QUESTIONS B
68. The given diagram represents the human respiratory
system with few structures labelled as I, II, III and IV. (a) A- Alveolar cavity- It is the main site of exchange of
respiratory gases.
I
(b) B- Red blood cell- It transports mainly carbon dioxide.
(c) C- Arterial capillary- It passes oxygen to tissues.
II III
(d) D- Capillary wall- Here, exchange of O2 and CO2 takes
IV
heart
place.
72. In the given diagram of human respiratory system, few
parts are marked as I, II, III, IV, V & VI. Choose the correct
combination of labelling from the given options.
The exchange of gases takes place in which labelled
structure?
(a) I ® trachea (b) II ® Bronchi
(c) III ® bronchioles (d) IV ® alvedi
69. In the given diagram of human respiratory system what is
the funciton of structure marked as X ?
X

heart
Breathing and Exhange of Gases 151

(a) I- Nose, II- Bronchus, III- Larynx, IV- Diaphragm, 75. The given figure shows the respiratory system. Identify
V- Trachea, VI- Lung the correct structure marked as 1, 2, 3 and 4 whose
(b) I- Nose, II- Larynx, III- Bronchus, IV- Lung, contraction initiated the inspiration which in turn increases
V- Diaphragm, VI- Trachea the volume of thoracic chamber in the antero-posterior
(c) I- Mouth, II- Trachea, III- Larynx, IV-Lung, axis.
V- Diaphragm, Vi- Bronchus.
(d) I- Mouth, II- Diaphragm, III- Trachea, IV- Bronchi,
V-Larynx, VI- Lung
73. In the given figure, label A represents ____________
while label B represents _________.
heart 3
4
1
A 2

B (a) 1-lung (b) 2-Diaphram


(c) 3-pleural (d) 4-Alveoli
76. The figure given below shows the mechanism of
breathing. Identify the stage (X) of breathing explained &
A, B and C marked in the figure.

(a) A- Trachea, B- Bronchus


(b) A- Alveolus, B- Bronchiole
Ribs and
(c) A- Bronchiole, B- Trachea
sternum Volume of
(d) A- Trachea, B- Bronchiole
A thorax
74. Refer the given figure and answer the question.
B
% saturation of
haemoglobin Rib cage

100 Diaphragm
80 C
60
(a) X – Expiration, A– raised, B – decreased, C – relaxed
40
(b) X – Inspiration, A – raised, B – decreased, C – relaxed
20 (c) X – Expiration, A – raised, B – increased, C –
contracted
2 4 6 8 10 12 14 (d) X - Inspiration, A– raised, B – increased, C –
partial pressure contracted
of oxygen (Kpa)
CRITICAL THINKING TYPE QUESTIONS
Which of the following statement is correct regarding the
above figure? 77. About 70% of CO2 is transported as
(a) When percentage saturation of haemoglobin is (a) carbonic acid (b) darboxyhaemoglobin
plotted against the partial pressure of oxygen, a (c) bicarbonates (d) carbamino compounds
sigmoid curve is obtained. 78. Oxyhaemoglobin dissociates into oxygen and
(b) Binding of oxygen with haemoglobin is primarily deoxyhaemoglobin at
related to partial pressure of carbon monoxide. (a) low O2 pressure in tissue.
(c) The given graph illustrates the amount of HbO2 as (b) high O2 pressure in tissue.
similar to Hb at different pO2.
(c) equal O2 pressure inside and outside tissue.
(d) None of the above.
EBD_7209
152 Biology
(d) all times irrespective of O2 pressure. V. Alveolar pressure decreases.
79. Even when there is no air in it, human trachea does not (a) I – II – III – IV – V (b) III – IV – II – V – I
collapse due to presence of (c) II – V – I – III – IV (d) IV – V – II – I – III
(a) bony rings (b) turgid pressure 89. Which of the following are the stages of respiration in
(c) chitinous rings (d) cartilaginous rings correct order?
80. Dissociation curve shifts to the right when A – Gaseous transport B – Cellular respiration
(a) pH increases. C – Tissue respiration D – Breathing
(b) CO2 concentration increases. (a) A – D – C – B (b) D – A – C – B
(c) O2 concentration decreases. (c) D – A – B – C (d) D – C – B – A
(d) 2, 3 D-P-G decreases. 90. Vital capacity of our lungs is
81. During inspiration in mammals, the diaphragm becomes (a) total lung capcity minus residual volume.
(a) dome shaped (b) raised highly (b) inspiratory reserve volume plus tidal volume.
(c) flat (d) static (c) total lung capacity minus expiratory reserve volume.
82. During oxygen transport, the oxyhaemoglobin at the tissue
(d) inspiratory reserve volume plus expiratory reserve
level liberates oxygen to the cells because in tissue
volume.
(a) O2 concentration is high and CO2 is low.
91. Functional residual capacity in human is the amount of air
(b) O2 concentration is low and CO2 is high.
(a) that can be filled in lungs by forceful inspiration.
(c) O2 tension is high and CO2 tension is low.
(b) that can be breathed out after forceful expiration.
(d) O2 tension is low and CO2 tension is high.
(c) that remains in the lungs after normal expiration.
83. Combining of haemoglobin with O2 in lungs can be
promoted by (d) that remains in the lungs after forceful expiration.
(a) decreasing CO2 concentration in blood. 92. If a person is suffering from emphysema, his/her
(b) decreasing O2 concentration in blood. (a) bronchioles are found damaged.
(c) increasing CO2 concentration in blood. (b) alveolar walls are found damaged.
(d) introducing CO2 into blood. (c) the plasma membrane is found damaged.
84. When CO2 concentration in blood increases, breathing (d) the respiratory muscle is found damaged.
becomes 93. Number of RBCs per unit volume of blood is likely to be
(a) slow and deep. higher in a person living at high altitudes, because
(b) faster and deeper. (a) air is clean and unpolluted.
(c) shallower and slow. (b) more sunshine is available.
(d) there is no effect on breathing. (c) air is less dense.
85. What would happen if human blood becomes acidic (d) vegetation gives out more O2.
(low pH)? 94. Blood analysis of a patient reveals an unusually high quantity
(a) Oxygen carying capacity of haemoglobin increases. of carboxyhaemoglobin content. Which of the following
(b) Oxygen carrying capacity of haemoglobin decreases. conclusions is most likely to be correct? The patient has been
(c) RBCs count increases. inhaling polluted air containing usually high content of
(d) RBCs count decreases. (a) chloroform (b) carbon dioxide
86. When, under certain conditions, the P 50 value of (c) carbon monoxide (d) carbon disulphide
haemoglobin rises, the affinity of the pigment of combining 95. Common feature in the alveoli of lungs and villi of intestine
with O2 will in mammals is that both
(a) remain same (b) rise (a) provide a large surface area.
(c) fall (d) first rise and then fall
(b) have ciliated epithelium.
87. Low oxygen tension in the blood causes
(c) are suited for diffusion of gases.
(a) coughing (b) yawning
(d) have rich supply of blood vessels and lymph ducts.
(c) hiccuping (d) sneezing
96. Presence of large number of alveoli around alveolar ducts
88. Arrange the following events in the correct order as they
opening into bronchioles in mammalian lungs is
occur during inspiration:
(a) inefficient system of ventilation with little of residual air.
I. Air flows into the lungs.
II. Alveolar volume increases. (b) inefficient system of ventilation with high percentage
of residual air.
III. Thoracic volume increases.
(c) an efficient system of ventilation with no residual air.
IV. Pleural pressure decreases.
(d) an efficient system of ventilation with little residual air.
Breathing and Exhange of Gases 153

97. Maximum amount of oxygen is exchanged from the blood (a) nasal cavity ¾¾ ® pharynx ¾¾ ® larynx ¾¾ ®
in the ............... trachea ¾¾ ® bronchioles ¾¾ ® bronchi ¾¾ ®
(a) capillaries-surrounding tissue cells. alveoli
(b) arteries of the body. (b) nasal cavity ¾¾ ® pharynx ¾¾ ® larynx ¾¾ ®
(c) left auricle of the heart. trachea ¾¾ ® bronchi ¾¾ ® bronchioles ¾¾ ®
alveoli
(d) capillaries surrounding the alveoli.
(c) nasal cavity ¾¾ ® larynx ¾¾ ® pharynx ¾¾ ®
98. After taking a long deep breath, we do not respire for
trachea ¾¾ ® bronchi ¾¾ ® alveoli
some seconds due to
(d) nasal cavity ¾¾ ® larynx ¾¾ ® pharynx ¾¾ ®
(a) more CO2 in blood (b) more O2 in blood
trachea ¾¾ ® bronchioles ¾¾ ® alveoli
(c) less CO2 in blood (d) less O2 in blood
104. The chloride shift in red blood cells is the movement of
99. A large proportion of oxygen is left unused in the human
blood even after its uptake by the body tissues. This O2 (a) cations into the cell to balance its uptake of chloride
ions.
(a) helps in releasing more O2 to the epithelium tissues.
(b) chloride ions from the blood into the glomerular
(b) acts as a reserve during muscular exercise.
filtrate for excretion.
(c) raises the pCO2 of blood to 75 mm of Hg.
(c) chloride ions out of the cell to balance the release of
(d) is enough to keep oxyhaemoglobin saturation at 96%. hydrogencarbonate ions.
100. Which one of the following is a possibility for most of us (d) chloride ions into the cell to balance its uptake of
in regard to breathing, by making a conscious effort? hydrogencarbonate ions.
(a) One can consciously breathe in and breathe out by 105. External gills, tracheae, and lungs all share which of the
moving the diaphragm alone, without moving the following sets of characteristics ?
ribs at all.
(a) Part of gas-exchange system, exchange both CO2
(b) The lungs can be made fully empty by forcefully and O2; increase surface area for diffusion.
breathing out all air from them.
(b) Used by water breathers; based on countercurrent
(c) One can breathe out air totally without oxygen. exchange; use negative pressure breathing.
(d) One can breathe out air through Eustachian tubes (c) Exchange only O2; are associated with a circulatory
by closing both the nose and the mouth. system; found in vertebrates.
101. The urge to inhale in humans results from (d) Found in insects; employ positive-pressure pumping
(a) rising pCO2 (b) rising pO2 based on crosscurrent flow.
(c) falling pCO2 (d) falling pO2 106. Which of the following equation shows the role of
102. The pharynx functions as a ___i___, whereas the larynx carbonic anhydrase enzyme in the transport of carbon
functions as a ___ii___. dioxide by blood?
(a) i- common passage for air and food; ii- passageway
for food only. (a) ¾¾
® H 2 CO3 ¬¾
CO2 + H 2O ¬¾
¾ ¾¾
® HCO3– + H +
¾
(b) i- passageway for air only; ii- passageway for air and
Oxidation
food.
(c) i- common passageway for air and food; ii- (b) C6H12O6 + 6 O2 6 CO2 + 6 H 2O
passageway for air only. Reduction
(d) i- block to bacteria; ii- passage for air and food.
(c) C6 H12 O6 ® 2 C6 H5OH + 2 CO 2
103. In man and mammals, air passes from outside into the
lungs through (d) None of the above.
EBD_7209
18
Body Fluids and

Chapter
Circulation

9. Rh factor is named after


FACT/DEFINITION TYPE QUESTIONS
(a) monkey (b) Drosophila
1. Lymph differs from blood in (c) rat (d) man
(a) absence of RBC (b) absence of WBC 10. When the blood of a foetus is agglutinated by its mother's
(c) excess of water (d) absence of protein Rh antibodies, then which of the following condition arises
2. Lymph is modified blood that contains as a severe anaemia?
(a) RBC and WBC (a) Immunization
(b) RBC, WBC and protein (b) Ectopic pregnancy
(c) WBC and all protein
(c) Erythroblastosis foetalis
(d) all contents of blood except RBC and certain protein
(d) Both (b) and (c)
3. In blood
(a) WBCs are more than RBCs 11. __________ plays an important role in blood clotting.
(b) RBCs are more than WBCs (a) Sodium (b) Chlorine
(c) RBCs are less than platelets (c) Calcium (d) Potassium
(d) Platelets are less than WBCs 12. Which of the following cations is required for the
4. Which one engulfs pathogens rapidly? conversion of prothrombin into active thrombin by
(a) Acidophils (b) Monocytes thromboplastin?
(c) Basophils (d) Neutrophils (a) Cu2+ (b) Fe3+
5. What is true about leucocytes ? (c) Fe 2+ (d) Ca2+
(a) Their sudden fall in number is indication of blood cancer
13. Clotting of blood is to
(b) These are produced in thymus
(a) seal up wounds and prevent blood loss.
(c) These are enucleated
(d) These can squeeze out through the capillary walls (b) slow down the movement of blood inside the blood
6. Which of the following is considered as the soldiers of vessels.
body? (c) trap oxygen inside the blood so that it can be carried
(a) Lungs (b) Capillaries around more easily.
(c) Red blood cells (d) White blood cells (d) trap harmful germs inside the blood and prevent them
7. Which of the following is responsible for ABO grouping? from harming the body.
(a) Presence or absence of clotting factors. 14. Fish has ___(i)____ and ___(ii)___ circulatory system.
(a) (i)- open, (ii)- single
(b) Compatibility of blood groups durin g blood
transfusion. (b) (i)- open, (ii)- double
(c) Presence or absence of surface antigens (A and B) on (c) (i)- closed, (ii)- single
WBCs. (d) (i)- closed, (ii)- double
(d) Presence or absence of two surface antigens (A and B) 15. Open type blood vascular system is found in
on the RBCs. (a) earthworm (b) lizard
8. What is correct for blood group ‘O’? (c) cockroach (d) toad
(a) No antigens but both a and b antibodies are present 16. Which of the following set of animals has an incomplete
(b) A antigen and b antibody double circulation system ?
(c) Antigen and antibody both absent (a) Frog and crocodile (b) Shark and whale
(d) A and B antigens and a, b antibodies (c) Lizard and pigeon (d) Toad and lizard
Body Fluids and Circulation 155

17. Heart is covered by 29. Sinu-atrial node in human heart is embedded in the wall of
(a) peritoneum (b) pleural membrane (a) sinus-venosus
(c) pericardium (d) visceral membrane (b) interatrial septum
18. Nodal tissue/musculature in human heart has (c) right atrium
(a) the ability to generate action potential due to any (d) atrioventricular septum
external stimuli. 30. Which term does not apply to human heart?
(b) two patches, one in atrium and other in ventricle. (a) Pacemaker (b) Four chambered
(c) purkin je fibres throughout th e ventricular (c) Mitral valve (d) Neurogenic
musculature. 31. What is the function of valve?
(d) atrioventricular node, also called pacemaker. (a) To prevent blood from flowing into the aorta.
19. Pulmonary vein, carrying oxygenated blood, opens into (b) To push blood into the ventricle.
(a) left auricle (b) right auricle (c) To push blood into the aorta.
(c) left ventricle (d) right ventricle (d) To prevent backflow of blood into the left ventricle.
20. The pacemaker of the human heart is 32. A blockage in aortic valve would directly reduce blood
flow to the _______.
(a) SA node (b) tricuspid valve
(a) heart (b) liver
(c) AV node (d) SV node
(c) lungs (d) brain
21. Which one of the following blood vessel is without
valves? 33. In mammals the blood from the right ventricle goes to
(a) systemic aorta (b) precavals
(a) Artery (b) Pulmonary aorta
(c) truncus arteriosus (d) pulmonary aorta
(c) Vein (d) Systemic aorta
34. The heart sound ‘dupp’ is produced when
22. Contraction of the ventricle in the heart begins by the
(a) tricuspid valve is opened.
command from
(b) mitral valve is opened.
(a) Purkinje fibres (b) AV node
(c) mitral valve is closed.
(c) chordae tendinae (d) SA node
(d) semi-lunar valves at the base of aorta get closed.
23. The opening of auricles into ventricles on the right side is
35. The first heart sound is
guarded by which valve?
(a) ‘LUBB’ sound produced at the end of systole.
(a) Tricuspid (b) Bicuspid
(b) ‘DUP’ sound produced at the end of systole.
(c) Semilunar (d) Mitral valve
(c) ‘LUBB’ sound produced at the beginning of systole.
24. Bicuspid valve allows blood from (d) ‘DUP’ sound produced at the beginning of systole.
(a) right auricle to left ventricle. 36. Closure of which of the following valves makes louder
(b) right auricle to right ventricle. sound of heart beat?
(c) left auricle to left ventricle. (a) Mitral valve (b) Seminular valve
(d) post caval to heart. (c) Auriculo-ventricular valve
25. ‘Bicuspid’ term is applied to (d) Tricuspid valve
(a) a valve in heart and a bone of pelvic girdle. 37. Cardiac cycle in man takes about
(b) a muscle in upper arm and a valve in heart. (a) 0.5 seconds (b) 1.0 second
(c) a valve in heart and tooth surface. (c) 1.2 seconds (d) 0.8 seconds
(d) a valve in heart. 38. The pattern of contraction and relaxation of the heart is
26. ‘Bundle of His’ is a part of which one of the following referred to as
organs in humans? (a) blood pressure (b) arterial flow
(c) blood flow (d) cardiac cycle
(a) Brain (b) Heart
39. ‘P’ wave of ECG occurs before the
(c) Kidney (d) Pancreas
(a) onset of ventricular contraction.
27. Which of the following has the thickest walls? (b) end of arterial contraction.
(a) Right ventricle (b) Left ventricle (c) beginning of atrial contraction.
(c) Right auricle (d) Left auricle (d) none of the above.
28. A four-chambered heart is found in 40. QRS is related to
(a) mammals only (a) ventricular contraction
(b) humans only (b) auricular contraction
(c) all vertebrates (c) cardiac cycle
(d) some reptiles, all birds and mammals (d) auricular relaxation
EBD_7209
156 Biology
41. The largest artery in the body is
STATEMENT TYPE QUESTIONS
(a) aorta (b) precaval
(c) vena cava (d) pulmonary artery 53. Mark the incorrect statement regarding normal ECG.
42. Pulmonary artery arises from (a) Patient is connected to 3 electrical leads (one to each
(a) right atrium (b) left atrium wrist and to the left ankle).
(c) right ventricle (d) left ventricle (b) ‘T’ wave represents atrial repolarization.
43. Which circulation provides nutrients and oxygen to (c) Q marks the beginning of ventricular systole.
cardiac muscle tissue?
(d) R represents ventricular depolarization.
(a) Pulmonary circulation
54. Given below are four statements (a-d) regarding human
(b) Systemic circulation
blood circulatory system
(c) Lymphatic circulation
(d) Coronary circulation (i) Arteries are thick-walled and have narrow lumen as
compared to veins.
44. Which structures are directly involved in the pulmonary
circulation? (ii) Angina is acute chest pain when the blood
(a) Right atrium, aorta and left ventricle. circulation to the brain is reduced.
(b) Left ventricle, aorta and inferior vena cava. (iii) Persons with blood group AB can donate blood to
(c) Superior vena cava, right atrium and left ventricle. any person with any blood group under ABO system.
(d) Right ventricle, pulmonary arteries and left atrium. (iv) Calcium ions play a very important role in blood
45. Which structures are directly involved in the "systemic clotting.
circulation"? Which two of the above statements are correct?
(a) Superior vena cava, right ventricle and left ventricle. (a) (i) & (iv) (b) (i) & (ii)
(b) Right ventricle, pulmonary arteries and left atrium. (c) (ii) & (iii) (d) (iii) & (iv)
(c) Left ventricle, aorta and inferior vena cava. 55. What is true about RBCs in humans?
(d) Right atrium, pulmonary trunk and left ventricle. (a) They carry about 20–25 per cent of CO2.
46. Which of the following carries oxygenated blood?
(b) They transport 99.5 per cent of O2.
(a) Renal vein (b) Hepatic portal vein
(c) They transport about 80 per cent oxygen only and
(c) Hepatic vein (d) Pulmonary vein
the rest 20 per cent of it is transported in dissolved
47. Which of the following parts of heart possess semilunar
valves? state in blood plasma.
(i) Aorta (ii) Vena cava (d) They do not carry CO2 at all.
(iii) Pulmonary artery (iv) Pulmonary vein 56. Find out the incorrect statement from the following.
(a) (i) and (iii) (b) (i), (ii) and (iii) (a) Veins are typically larger in diameter than arteries.
(c) (ii) and (iii) (d) (i), (iii) and (iv) (b) Because of their small size, capillaries contain blood
48. In adult man, normal BP is that is moving more quickly than in other parts of
(a) 100/80 mm Hg (b) 120/80 mm Hg the circulatory system.
(c) 100/120 mm Hg (d) 80/120 mm Hg (c) The walls of arteries are elastic, enabling them to
49. Pulse beat is measured from stretch and shrink during changes in blood pressure.
(a) capillaries (b) arteries (d) Veins contain more blood than any other part of the
(c) veins (d) nerves circulatory system.
50. Coronary artery disease (CAD) is often referred to as 57. Which one of the following statements is correct regarding
(a) heart failure (b) cardiac arrest blood pressure?
(c) atherosclerosis (d) angina (a) 190/110 mmHg may harm vital organs like brain and
51. Hardening of the arteries due to deposition of cholesterol kidney.
is called (b) 130/90 mmHg is considered high and requires
(a) thrombosis (b) atherosclerosis treatment.
(c) rhinitis (d) angina
(c) 120/80 mmHg is considered an ideal blood pressure.
52. In arteriosclerosis
(d) 105/50 mmHg makes one very active.
(a) arterial walls become very thin and weak so that the
blood oozes out of the walls. 58. Cardiac output is/are
(b) sex linked heredity is involved. (i) Product of heart rate and stroke volume.
(c) blood coagulates even in the arteries. (ii) Product of auricular and ventricular volume.
(d) arterial walls become inelastic and thickened. (iii) A process in which blood pumped in one minute.
Body Fluids and Circulation 157

Which of the above statements is true ? (b) RBC as an average life span of 120 days after which
(a) Only (i) (b) Only (ii) they are destroyed in the spleen.
(c) Both (i) and (ii) (d) Both (i) and (iii) (c) Neutrophils and basophils are phagocytic cells
59. Which of the following statements is/are correct for blood which destroy foreign organisms entering the body.
group? (d) A reduction in the number of WBC leads to clotting
(i) Blood group O is universal donor. disorders which will lead to excessive loss of blood
from the body.
(ii) Blood group AB is universal acceptor.
(iii) Blood group A contains antigen B and anti-A ASSERTION/REASON TYPE QUESTIONS
antibodies.
(iv) Blood group B contains antigen B and anti-A In the following questions, a statement of Assertion is followed
antibodies. by a statement of Reason.
(a) If both Assertion and Reason are true and the Reason is
(a) Only (i) and (ii). (b) Only (i), (ii) and (iii).
the correct explanation of the Assertion.
(c) Only (i), (ii) and (iv). (d) Only (i), (iii) and (iv). (b) If both Assertion and Reason are true but the Reason is
60. Which of the following statements is correct regarding not the correct explanation of the Assertion.
joint diastole? (c) If Assertion is true but Reason is false.
(a) Both atria relax but ventricles contract. (d) If both Assertion and Reason are false.
(b) Both ventricles relax but atria contract. 66. Assertion : SAN is called the pacemaker of heart.
(c) Both semilunar valves are open. Reason : The SAN can generate the maximum number of
(d) Tricuspid and bicuspid valves are open. action potentials and is responsible for initiating and
61. Which of the following statements is correct regarding maintaining the rhythmic contractile activity of the heart.
pulmonary artery ? 67. Assertion : Lub is a heart sound which is produced during
(a) It carries deoxygenated blood from heart to lungs. each cardiac cycle.
(b) It carries deoxygenated blood from lungs to heart. Reason : It is associated with the closure of the tricuspid
and bicuspid valves.
(c) It carries oxygenated blood from heart to lungs.
68. Assertion : Saline water is not given to patients with
(d) It carries oxygenated blood from lungs to heart. hypertension.
62. Which one of the statement is correct with reference to Reason: Saline water can cause vomiting and may drop
the circulation of blood in a mammal? blood pressure suddenly causing cardiac arrest.
(a) Left auricle receives oxygenated blood. 69. Assertion : Blood pressure is arterial blood pressure.
(b) Pulmonary artery carries oxygenated blood to the Reason : Blood pressure is measured by
lungs. sphygmomanometer.
(c) Blood from the lungs is returned to heart through 70. Assertion: WBCs accumulate at the site of wounds by
2-veins, one from each lung. diapedesis.
(d) None of the above. Reason: It is squeezing of leucocytes from endothelium.
63. Which of the following statement is correct regarding 71. Assertion : Blood coagulates in uninjured blood vessels.
lymph? Reason : Uninjured blood vessels release an anticoagulant
(a) It transports oxygen to brain. heparin.
(b) It transport CO2 to lungs. 72. Assertion : Prothrombinase enzyme acts as antiheparin.
Reason : Heparin prevents coagulation of blood in blood
(c) It returns interstitial fluid to blood.
vessels.
(d) It returns RBCs and WBCs to lymph nodes.
73. Assertion : When there is a fall in the blood pressure due
64. Which of the folowing statement best defines artery? to loss of blood volume, this is compensated by
(a) It carries blood from one visceral organ to another vasoconstriction of veins.
visceral organ. Reason : Veins hold the extra amount of blood which can
(b) It carries oxygenated blood to the different organs. be shifted to the arteries as required.
(c) It carries blood away from the heart to different
organs. MATCHING TYPE QUESTIONS
(d) It breaks up into capillaries which reunite to form a 74. In which of the following options the types of heart is
vein. correctly matched to their respective group of animals?
65. Which of the following statement is correct regarding (a) Two chambered heart- Fishes and amphibians
blood cells? (b) Three chambered heart- Amphibians and birds
(a) A healthy individual has 12 - 16 gms of haemoglobin (c) Four chambered heart- Birds and mammals
in every 200ml of blood. (d) Three chambered heart- Reptiles and mammals
EBD_7209
158 Biology
75. Which one of the following is a matching pair? 79. Match each area of the heart (column-I) with the structure
(a) Lubb-Sharp closure of AV valves at the beginning of (column-II) from which it receives blood.
ventricular systole. Column-I Column-II
(b) Dup-Sudden opening of semilunar valves at the (Area of heart) (Receives blood from)
beginning of ventricular diastole. A. Right atrium I. Left atrium
(c) Pulsation of the radial artery-Valves in the blood B. Right ventricle II. Vena cavae
vessels. C. Left atrium III. Right atrium
(d) Purkinje fibres-Initiation of the heart beat. D. Left ventricle IV. Pulmonary veins
76. In a standard ECG, which one of the following alphabets (a) A – II; B – III; C – IV; D – I
is the correct representation of the respective activity of (b) A – I; B – II; C – III; D – IV
the human heart? (c) A – IV; B – I; C – II; D – III
(a) P-Depolarization of the atria (d) A – III; B – IV; C – I; D – II
(b) R-Repolarization of ventricles 80. Match the column I with column II and choose the correct
(c) S-Start of systole answer from the option given below.
(d) T-End of diastole Column-I Column -II
77. Match the following descriptions (given in column-II) of A. Cardiac cycle I. 72 / min
each type of blood cell to their names (given in column-I). B. Plasma II. 120/80 mmHg
Column-I Column-II C. Systolic/Diastolic III. 0.8 seconds
(Blood cell) (Description) D. Haemoglobin IV. 12 -16 gms in every 100 ml
A. Erythrocyte I. Most abundant white of blood
blood cell, and the main E. Heart beat V. 55 % of the blood
phagocytic cell of the (a) A – I; B – II; C – III; D – IV; E – V
blood. (b) A – III; B – V; C – II; D – IV; E – I
B. Eosinophil II. Least abundant white (c) A – III; B – I; C – V; D – II; E – IV
blood cell; releases (d) A – V; B – IV; C – III; D – I; E – II
histamine granules.
C. Lymphocyte III. Resist infections and DIAGRAM TYPE QUESTIONS
are associated with
81. Given below is the ECG of a normal human.
allergic reactions.
Which one of its components is correctly interpreted?
D. Neutrophil IV. Blood cell that contains
haemoglobin and
transports oxygen.
E. Basophil V. Specialized antibody-
producing white blood
cells. (a) Peak P and Peak R together - Systolic and diastolic
(a) A – IV; B – III; C – V; D – I; E – II blood pressures
(b) A – I; B – II, C – III; D – IV; E – V (b) Peak P - Initiation of left atrial contraction only
(c) Complex QRS - One complete pulse
(c) A – II; B – III; C – I; D – V; E – IV
(d) Peak T - Initiation of total cardiac contraction
(d) A – IV; B – I; C – II; D – III; E – V
82. The given diagram represents human heart with four
78. Which of the following term (column-I) is correctly
chambers labelled as I, II, III & IV?
matched with its functions (column-II)?
Column-I Column-II
(Term) (Functions)
(a) Plasma Straw coloured fluid that
consists of suspended blood
cells I
III
(b) Heart Site where exchange of II
nutrients and gases occur
(c) Capillary A red colour fluid th at IV
provides food to the cells
(d) Blood Pumping station of the body
Body Fluids and Circulation 159

Which labelled structure receives carbon dioxide rich 86. The given figure represents the pathway of blood
blood from the body? throughout the body.
(a) I- Left atrium (b) II- Left ventricle
(c) III- Right atrium (d) IV- Right ventricle Lungs
83. In the given diagram of human heart the mitral valve is
located between which two parts of the heart? (3) (4)

A Heart

(1) (2)
E
Body
B tissues
D C
Identify the correct match of marked number 1, 2, 3 and 4.
(a) 1- Artery (b) 2- Pulmonary vein
(a) D and E (b) B and C (c) 3- Pulmonary artery (d) 4- Vein
(c) D and C (d) E and B 87. The given figure represents diagrammatic presentation
of an ECG. Each peak in the ECG is identified with a letter
84. In the human heart, blood from the lungs enters the heart
from P to T that corresponds to a specific electrical activity
through the left atrium, pumps into the left ventricle, out
of the heart.
the aorta and through the body, and then returns into the
right atrium, pumps into the right ventricle and exits to
the lungs.
Using the diagram, which set of letters (A, B, C, D, E)
correctly represents the process describe above?

A Which of the following letter shows the incorrect activity


of heart?
(a) QRS - Depolarization of the ventricles
(b) R wave - Marks the beginning of the systole.
E (c) P- Electrical excitation (or depolarization) of the atria
B
(d) T wave - Return of the ventricles from excited to
normal state (repolarization).
D C
CRITICAL THINKING TYPE QUESTIONS

(a) E, D, A, B, C (b) B, C, A, E, D 88. What is true about leucocytes ?


(c) C, D, A, B, E (d) D, C, A, E, B (a) Their sudden fall in number is indication of blood
85. Refer the given figure of human heart and identify the cancer.
chamber (marked as 1, 2, 3 & 4) which receives most of the (b) These are produced in thymus.
blood returning from the brain. (c) These are enucleated.
(d) These can squeeze out through the capillary walls.
89. Which is the correct route through which pulse making
impulse travels in the heart?
(a) SA node ® AV node ® Bundle of His ® Purkinje
4
1 fibres
(b) AV node ® Bundle of His ® SA node ® Purkinje
3 2 fibres ® heart muscles
(c) AV node ® SA node ® Purkinje fibres ® Bundle of
His ® heart muscles
(a) I- Left atrium (b) II- Left ventricle (d) SA node ® Purkinje fibres ® Bundle of His ® AV
(c) III- Right ventricle (d) IV- Right atrium node ® heart muscles
EBD_7209
160 Biology
90. If nerves of heart are cut, then heart will 99. If a person is loosing blood continuously due to injury
(a) beat rhythmically (b) stop then what will be the effect on his pulse and BP initially?
(c) beat arhythmically (d) shrink (a) Pulse and BP both will fall.
91. Which one of the following will be the cardiac output (in (b) Pulse will fall and BP will rise.
litres per minute) of a heart that has a stroke volume of (c) Both pulse and BP will rise.
0.07 litres and is beating at a rate of 90 per minute? (d) Pulse will rise and BP will fall.
(a) 63.30 (b) 63.00 100. During systemic circulation, blood leaves the ___(i)____
(c) 00. 63 (d) 06.30 and _________(ii)______.
92. The blood returning to the heart through pulmonary vein
(a) (i)- left ventricle, (ii)- goes directly to the aorta
has more
(b) (i)- right ventricle, (ii)- goes directly to the aorta
(a) RBC (b) haemoglobin
(c) oxygen (d) nutrient (c) (i)- right ventricle, (ii)- moves to the lungs.
93. First heart sound (LUBB) coincides with which wave of (d) (i)- left ventricle, (ii)- moves to the lungs
ECG? 101. In normal human blood pressure, what does the "120"
(a) R-wave (b) T-wave represent?
(c) P-wave (d) Q-wave (a) The maximum pressure that can be sustained by the
arteries before bursting.
94. What happens when the pacemaker becomes non-
functional? (b) The lowest pressure in the arteries that will keep
someone from fainting when they stand up.
(a) Only the auricles will contract rhythmically.
(c) The highest ideal arterial pressure for someone at
(b) The cardiac muslces do not contract in a coordinated
rest.
manner rhythmically.
(d) The pressure in the veins.
(c) Only ventricles will contract rhythmically.
102. If a person had two-chambered heart (one atrium, and
(d) Auricles and ventricles contract simultaneously.
one ventricle) with the vena cava entering the atrium and
95. Maximum amount of oxygen is lost from the blood in the the aorta leaving the ventricle then which of the following
(a) capillaries surrounding the tissue cells. statement would be correct?
(b) arteries of the body. (a) No oxygen would reach the cells.
(c) capillaries surrounding the alveoli. (b) No blood would reach the head and neck.
(d) left auricle of the heart. (c) The blood would be unable to carry food to the cells.
96. A drop of each of the following, is placed separately on (d) There would be no blood circulation.
four slides. Which of them will not coagulate? 103. Sinus venosus is/are present in which of the following
(a) Whole blood from pulmonary vein animals?
(b) Blood plasma (i) Fishes (ii) Amphibians
(c) Blood serum (iii) Reptiles (iv) Birds
(d) Sample from the thoracic duct of lymphatic system (v) Mammals
97. If due to some injury, the chordae tendinae of the tricuspid (a) (i) only (b) (i) and (iv) only
valve of the human heart is partially non-functional, what (c) (i), (ii) and (iii) (d) (i), (iii) and (iv)
will be the immediate effect?
104. In order for the blood to flow from right ventricle to left
(a) The flow of blood into the pulmonary artery will be ventricle in mammalian heart, it must flow through
reduced.
I. Right ventricle II. Pulmonary veins
(b) The flow of blood into the aorta will be slowed down.
III. Left atrium IV. Lungs
(c) The ‘pacemaker’ will stop working.
V. Pulmonary arteries
(d) The blood will tend to flow back into the left atrium.
(a) I- V- IV- II- III (b) I- II- III- IV- V
98. Which of the following set of events can occur
(c) III- V- I- II- IV (d) III- II- I- IV-V
simultaneously?
105. Which of the following is not correct for blood pressure?
(a) Auricular depolarization, ventricular depolarization,
auricular systole. (i) It is not affected by atherosclerosis.
(b) Ventricular depolarization, auricular systole, (ii) It is typically lower in veins than in arteries.
ventricular diastole. (iii) Diastolic pressure is higher than systolic pressure.
(c) Auricular depolarization, ventricular repolarization, (iv) It is usually refers to as venous pressure of the
auricular diastole. systemic circulation.
(d) Auricular depolarizaion, ventricular diastole, (a) (i) & (iii) (b) (i), (ii) & (iii)
auricular systole. (c) (ii) & (iii) (d) (i), (iii) & (iv)
Body Fluids and Circulation 161

106. Mother-foetus Rh blood type incompatibility disorder can 108. Which area allows the atria to completely empty as the
occur if the mother is _____ and her foetus is ____. ventricles fill with blood?
(a) Rh positive; Rh positive (a) A-V node (b) A-V bundle
(b) Rh positive; Rh negative (c) S-A node (d) Purkinje fibers
(c) Rh negative; Rh positive 109. When a leopard runs after you, the increase in heart rate
(d) Rh negative; Rh negative is probably due to the __________.
107. "X" is a fibrous tissue of the membranous septum of the (a) hypothalamus
heart just above the septal cusp of the tricuspid valve. It (b) sympathetic nerves
separates the atrium and the ventricle of the same side. (c) increase in blood pressure
Identify "X". (d) medullary accelerator centre
(a) Sino atrial node 110. The state of heart when it is not pumping blood effectively
(b) Atrioventricular septum enough to meet the needs of the body is called _____.
(c) Atrioventricular node (a) angina (b) cardiac arrest
(d) Interventricular septum (c) heart attack (d) heart failure
EBD_7209
Excretory Products and
19

Chapter
Their Elimination

FACT/DEFINITION TYPE QUESTIONS (c) double the number of Bowman’s capsules.


(d) sum of Bowman’s capsules an d Malpighian
1. Uricotelic mode of passing out nitrogenous wastes is found in corpuscles.
(a) reptiles and bird 8. Glomerulus and Bowman’s capsule constitute
(b) birds and annelids (a) nephrotome (b) renal corpuscle
(c) amphibians and reptiles (c) renal capsule (d) malpighian tubule
(d) insects and amphibians 9. In which part of the excretory system of mammals you
2. Ammonia is the main nitrogenous excretory material in can first use the term urine for fluid it contains?
(a) amphibians (b) turtles (a) Bowman’s capsule (b) Loop of Henle
(c) tadpoles (d) reptiles (c) Collecting tubule (d) Ureter
3. Mechanism of uric acid excretion in a nephron is 10. Columns of Bertini in the kidneys of mammals are formed
(a) osmosis (b) diffusion as extensions of
(c) ultrafiltration (d) secretion (a) Cortex into medulla
4. Why bony fishes, aquatic amphibian and aquatic insects (b) Cortex into pelvis
are called ammonotelic animals? (c) Medulla into pelvis
(a) They excrete ammonia as their excretory product. (d) Pelvis into ureter
(b) They excrete ammonia as a least toxic nitrogenous 11. Blood vessel leading to glomerulus is called
waste product. (a) renal artery (b) renal vein
(c) They excrete uric acid in the form of pellet and paste (c) efferent arteriole (d) afferent arteriole
with a minimum loss of water. 12. Which one of the following is not a part of a renal pyramid?
(d) These animals have nephridia as their excretory organ (a) Loops of Henle
which helps to remove nitrogenous waste and (b) Peritubular capillaries
maintain a fluid and ionic balance.
(c) Convoluted tubules
5. Excretion of nitrogenous waste product in semi-solid form
occur in (d) Collecting ducts
(a) amniotes (b) desert animals 13. The efferent arteriole emerging from the glomerulus forms
a fine capillary network around the renal tubule called the
(c) ureotelic animals (d) uricotelic animals
______________.
6. The projections of renal pelvis are called
(a) vasa recta (b) loop of Henle
(a) hiluses
(c) collecting duct (d) peritubular capillaries
(b) calyces
14. Juxta-glomerular apparatus is formed by cellular
(c) medullary pyramids
modification in the
(d) renal columns
(a) afferent arteriole and DCT
7. The number of nephrons in a kidney is equal to
(b) efferent arteriole and PCT
(a) the number of Bowman’s capsules.
(c) afferent arteriole and PCT
(b) sum of Bowman’s capsules and glomeruli.
(d) efferent arteriole and DCT
Excretory Products and Their Elimination 163

15. Which of the following accessory excretory structure (a) 100 ml/minute, i.e., 180 liters per day.
eliminates NaCl, lactic acid and urea? (b) 125 ml/minute, i.e., 180 litres per day.
(a) Kidney (b) Liver (c) 120 ml/minute, i.e., 100 litres per day.
(c) Sebaceous gland (d) Sweat gland (d) 130 ml/minute, i.e., 120 litres per day.
16. Which of the following components of blood does not 26. The maximum reabsorption of useful substances back into
enter into the nephron? the blood from filtrate in a nephron occurs in
(a) Urea (b) Water (a) PCT (b) Loop of Henle
(c) Glucose (d) Plasma protein (c) DCT (d) collecting duct
17. Kidney helps in the conservation of useful materials and 27. A fall in glomerular filtration rate (GFR) activates
excretion of wastes and therefore they receive 20% of the
(a) adrenal cortex to release aldosterone.
heart’s output of blood (as much as the heart and brain
(b) adrenal medulla to release adrenaline.
combined). On a percentage basis which substance is most
completely reabsorbed by the kidneys? (c) juxta - glomerular cells to release renin.
(a) Water (b) Glucose (d) posterior pituitary to release vasopressin.
(c) Urea (d) Sodium 28. The part of the nephron that helps in active reabsorption
18. The site and principal mechanism for the passage of of sodium is
glucose into the bloodstream in the human kidney is the (a) bowman’s capsule
(a) collecting duct, by active secretion. (b) distal convoluted tubules
(b) distal convoluted tubule, by passive diffusion. (c) ascending limb of Henle’s loop
(c) glomerulus, by selective reabsorption. (d) proximal convoluted tubules
(d) proximal convoluted tubule, by selective 29. Which region of the kidney nephron is the main site of
reabsorption. amino acid reabsorption?
19. The part of the nephron impermeable to water is (a) Glomerulus
(a) proximal tubule (b) Bowman’s capsule
(b) distal tubule (c) Proximal convoluted tubule
(c) ascending limb of Henle’s loop (d) Distal convoluted tubule
(d) collecting duct 30. Which of the following hormone is secreted from kidney ?
20. Reabsorption of chloride ions from glomerular filtrate in (a) ANF (b) Erythropoietin
kidney tubule occurs by (c) Rennin (d) Aldosterone
(a) active transport (b) diffusion 31. Which of the following is directly responsible for
(c) osmosis (d) brownian movement increasing glomerular blood pressure and hence GFR?
21. Colloidal osmotic pressure in blood plasma is mainly due (a) Aldosterone (b) ANF
to (c) Angiotensin II (d) Renin
(a) albumin (b) globulin 32. In the renal tubules the permeability of the distal
(c) fibrinogen (d) sodium chloride convoluted tubule and collecting duct to water is
22. The ascending loop of Henle is permeable for controlled by
(a) ammonia (b) glucose (a) aldosterone (b) vasopressin
(c) sodium (d) water (c) growth hormone (d) renin
23. Loop of Henle takes part in absorption of 33. The function of renin is
(a) potassium (b) glucose (a) degradation of angiotensinogen
(c) water (d) urea (b) stimulation of corpus luteum
24. In comparison to blood plasma, percentage of glucose in (c) to reduce blood pressure
glomerular filtrate is
(d) vasodilatation
(a) higher (b) equal 34. Volume of urine is regulated by
(c) lower (d) nil (a) aldosterone
25. Glomerular filtration rate (GFR) in a healthy individual is (b) aldosterone and ADH
approximately (c) aldosterone, ADH and testosterone
(d) ADH alone
EBD_7209
164 Biology
35. The hormone that promotes reabsorption of water from 43. Which one of the following statements in regard to the
glomerular filtrate is excretion by the human kidneys is correct?
(a) oxytocin (b) vasopressin (a) Ascending limb of Loop of Henle is impermeable to
(c) calcitonin (d) relaxin electrolytes.
36. Juxta-glomerular cells of renal cortex synthesizes an (b) Descending limb of Loop of Henle is impermeable to
enzyme called water.
(a) ADH (b) oxytocin (c) Distal convoluted tubule is incapable of reabsorbing
(c) renin (d) urochrome HCO3–.
37. Which one is an important constituent of renin (d) Nearly 99 per cent of the glomerular filtrate is
angiotensinogen-aldosterone system? reabsorbed by the renal tubules.
(a) JGA cell (b) Macular cell 44. Which one of the following statements is correct with
(c) Erythropoetin (d) Plasma cell respect to kidney’s function and regulation?
38. The voluntary response to the distension of urinary (a) During summer when body loses lots of water by
bladder is evaporation, the release of ADH is suppressed.
(a) polyurea (b) micturition (b) When someone drinks lot of water, ADH release is
(c) mellitus (d) menstruation suppressed.
(c) Exposure to cold temperature stimulates ADH
STATEMENT TYPE QUESTIONS
release.
39. Which of the following statement is correct? (d) An increase in glomerular blood flow stimulates
(a) Vasa recta is not present in cortical nephrons. formation of Angiotensin II.
(b) Maximum number of nephrons in kidney are juxta- 45. Which of the following statement is correct regarding
medullary type. urine formation?
(c) DCT of many nephrons open into collecting tubule. (a) Filtration and reabsorption takes place before
(d) All of the above secretion.
40. Which of the following statement is not correct with (b) Filtration an d secretion takes place before
respect to human kidney? reabsorption.
(a) The peripheral region is called cortex and central (c) Secretion takes place before reabsorption and
medulla. filtration.
(b) Malpighian capsules are present in the cortex region. (d) Reabsorption takes place before filtration and
(c) Blood enters glomerulus through efferent arterioles. secretion.
(d) The concave part of kidney is called hilus. 46. Which of the following statement is incorrect?
41. If Henle’s loop were absent from mammalian nephron
(a) Counter-current flow of blood in vasa recta helps to
which of the following event is to be expected ?
retain the reabsorbed sodium in the renal medulla.
(a) There will be no urine formation.
(b) Glomerular filterate is protein free plasma.
(b) There will be hardly any change in the quality and
quantity of urine formed. (c) Vasa recta carry glomerular filterate from distal
convoluted tubule to the collecting duct.
(c) The urine will be more concentrated.
(d) The urine will be more dilute. (d) Glomerular filterate in Bowman’s capsule is isotonic
to the plasma.
42. Almost all the aquatic animals excrete ammonia as the
nitrogenous waste product. Which of the following 47. Which of the following statements are correct?
statement is not in agreement with this situation? (i) Glucose has high threshold value.
(a) Ammonia is easily soluble in water. (ii) Urine is concentrated in Henle’s loop.
(b) Ammonia is released from the body in a gaseous (iii) Haemodialyser removes urea, uric acid, glucose and
state. proteins.
(c) Ammonia is highly toxic and needs to be eliminated (iv) In glomerulus, urea, uric acid, water, glucose and
as and when formed. plasma proteins are filtered out.
(d) Ammonia gets converted into a less toxic form called
(a) (i), (iii) and (iv) (b) (ii), (iii) and (iv)
urea.
(c) (i) and (ii) (d) (i) and (iii)
Excretory Products and Their Elimination 165

48. Which of the following statements is/are true? MATCHING TYPE QUESTIONS
(i) Urine is hypertonic in distal convoluted tubule.
55. Match the excretory functions given in column-I with the
(ii) When the urine passes into the collecting tubule,
parts of the excretory system in column-II. Choose the
it becomes hypotonic. correct combination from the given options.
(iii) Urine is isotonic in proximal convoluted tubule. Column-I Column-II
(iv) Urine becomes more and more hypotonic as it passes (Function) (Parts of excretory systems)
through the Henle’s loop. A. Ultra filtration I. Henle’s loop
(a) (i) and (iv) only (b) (i), (ii) and (iii) only B. Concentration II. Ureter
(c) (ii) and (iii) only (d) (iii) only of urine
C. Transport of III. Urinary bladder
ASSERTION/REASON TYPE QUESTIONS urine
In the following questions, a statement of Assertion is followed D. Storage of urine IV. Malpighian corpuscle
by a statement of Reason. V. Proximal convoluted
(a) If both Assertion and Reason are true and the Reason is tubule
the correct explanation of the Assertion. (a) A – IV; B – I; C – II; D – III
(b) If both Assertion and Reason are true but the Reason is (b) A – IV; B – III; C – II; D – I
not the correct explanation of the Assertion. (c) A – V; B – IV; C – I; D – III
(c) If Assertion is true but Reason is false. (d) A – V; B – IV; C – I; D – II
(d) If both Assertion and Reason are false. 56. Match the disorders given in column-I with their feature
49. Assertion : Aquatic animals like whales and seals are given in column-II and choose the correct option.
said to be ureotelic animals. Column-I Column-II
Reason : It is because of the fact that their main (Disorders) (Feature)
A. Uremia I. Excess of protein in urine
nitrogeneous waste product is urea.
B. Hematuria II. Presence of high ketone
50. Assertion : Kidney maintains the osmotic concentration bodies in urine
of the blood. C. Ketonuria III. Presence of blood cells
Reason : Kidney eliminates either hypotonic or hypertonic in urine
urine according to the need of the body. D. Glycosuria IV. Presence of glucose in
urine
51. Assertion : In the descending limb of loop of Henle, the
E. Proteinuria V. Excess of urea in blood
urine is hypertonic, whereas in ascending limb of loop of
(a) A - V; B -III; C - II; D - IV; E - I
Henle, the urine is hypotonic. (b) A - IV; B - V; C - III; D - II; E - I
Reason : Descending limb is impermeable to sodium, while (c) A - V; B -III; C - IV; D - II; E - I
ascending limb is impermeable to water. (d) A - III; B -V; C - II; D - I; E - IV
57. Which of the following parts of the nephron given in
52. Assertion : Secreting hypotonic urine is effective in column I is correctly matched with their functions given
reducing urinary loss of water. in column II?
Reason : Hypotonic urine is more concentrated and higher Column-I Column-II
in osmotic pressure than the blood. (Parts of the nephron) (Functions)
53. Assertion : Aldosterone is a steroid hormone and is A. Proximal convoluted I. Sodium is reabsorbed
tubules actively in this region.
important in the control of sodium and potassium ion
B. Distal convoluted II. Sodium and water are
concentration in mammals. tubules reabsorbed under the
Reason : It upgrades sodium ion concentration in the influence of hormone
ECF by promoting reabsorption of sodium ions from renal in this region.
tubules and excretion of potassium ions in urine. C. Descending limb III. Primary site of glucose
54. Assertion : Main constituent of human urine is ammonia. and amino acid
reabsorption.
Reason : If human urine is allowed to stand for some time, D. Ascending limb IV. Major substance
it smells strongly of ammonia. reabsorbed here is
water by osmosis.
EBD_7209
166 Biology
(a) A – I; B – II; C – III; D – IV CO2 (18 litres/day) and
(b) A – IV; B – I; C – II; D – III also significant quantities
(c) A – III; B – IV; C – II; D – I of water every day
(d) A – III; B – II; C – IV; D – I
D. Kidney IV. Remove wastes (metabolic
58. Which of the following hormone/enzyme is/are correctly
by-products) and regulate
paired with its function?
I. Renin – Enzyme that catalyses the formation of pH, ion concentration,
angiotensin I. volume and osmolarity of
II. Aldosterone – Regulates water reabsorption at the blood
distal convoluted tubule. (a) A – I; B – II; C – III; D – IV
III. Anti-diuretic hormone (ADH) – It is a powerful (b) A – II; B – I; C – II; D – IV
vasoconstrictor that stimulates the secretion of (c) A – III; B – I; C – IV; D – II
aldosterone. (d) A – IV; B – IV; C – III; D – I
IV. Angiotensin II – Promotes reabsorption of sodium 62. Select the correct match of the types of neuron present in
at distal convoluted tubule. column I with its location given in column II.
(a) Only I (b) Only III Column I Column II
(c) I, II and III (d) II, III and IV
A. Fall in GFR I. Activate the JG cells to
59. Which of the following is correctly matched with its
function of a specific part of a human nephron? release renin
(a) Afferent arteriole – Carries the blood away from the B. Angiotensin II II. Increases the glomerular
glomerulus towards renal vein. blood pressure and
(b) Podocytes – Create minute spaces (slit pores) for thereby GFR
the filtration of blood into the Bowman’s capsule. C. Renin III. Carries out the conversion
(c) Henle’s loop – Reabsorption of the major substances of angiotensinogen in the
from the glomerular filtrate. liver to angiotensin I.
(d) Distal convoluted tubule – Reabsorption of K+ ions D. Aldosterone IV. Causes reabsorption of
into the surrounding blood capillaries.
Na+ and water from the
60. Select the option which shows correct matching of animal
distal parts of the tubule.
with there excretory organs and excretory product.
This also leads to an
Excretory Excretory increase in blood
Animal pressure and GFR.
organs product
(a) Housefly Renal tubules Uric acid E. An excessive loss of V. Activate osmoreceptors
(b) Labeo (Rohu) Nephridial tubes Ammonia fluid from the body which stimulate the
(c) Salamander Kidney Urea
hypothalamus to release
ADH from the
(d) Peacock Kidney Urea
neurohypophysis
61. Select the correct match of the types of organs given in (a) A – I; B – II; C – III; D – V
column I with their role in excretion given in column II. (b) A – III; B – V; C – II; D – I
Column I Column II (c) A – III; B – I; C – IV; D – IV
(Types of organ) (Role in excretion) (d) A – V; B – IV; C – III; D – II
A. Lungs I. Secretes bile-containing
substances like bilirubin, DIAGRAM TYPE QUESTIONS
biliverdin, cholesterol,
63. The label X and Y in the given diagram of human urinary
degraded steroid hormones,
system represents
vitamins and drugs.
B. Liver II. Eliminates water and salts
in sweat and substances
like sterols, hydrocarbons
and waxes through sebum. (X)

C. Skin III. Remove large amounts of

(Y)
Excretory Products and Their Elimination 167

(a) X- Urethra, Y- Ureter 66. The label X represents ___________ that function in
______________.
(b) X- Ureter, Y- Urethra
(a) Vasa recta- Reabsorption of water, minerals and
(c) X- Bladder, Y- Urethra digestive end products.
(d) X- Ureter, Y- Bladder (b) Henle's loop- Filtration of plasma leaving the blood.
(c) Vasa recta- Filtration of plasma leaving the blood.
64. Which is the correct order for the path taken by urine
(d) Henle's loop- Reabsorption of water, minerals and
after it leaves the kidney?
digestive end products.
67. Which blood component would not usually pass through
the membranes from region A to region B?
(a) Mineral salts (b) Red blood cells
(c) Urea (d) Water
68. After the blood enters the kidney, it travels to the
II ____________.
(a) A (b) B
(c) C (d) D
Directions for (Q. 69 to 72): Refer the given figure of nephron
III
showing blood vessels and duct and answer the questions.
I
(a) I ® II ® III (b) III ® I ® II
(c) II ® III ® I (d) II® I ® III
65. The given figure shows the longitudinal section of kidney
with few structures labelled as I, II, III & IV identify renal
vein in the given figure.

IV

I
II
III
69. Which parts in the above figure have minimum
reabsorption and play a significant role in the maintenance
of high osmolarity of medullary interstitial fluid?
(a) 1 and 2 (b) 3 and 4
(a) I (b) II (c) 5 and 6 (d) 7 and 8
(c) III (d) IV 70. Cells of which part is lined by simple cuboidal brush border
Directions for (Q. 66 to 68): Refer the given diagrammatic epithelium that increases the surface area for reabsorption?
representation of a nephron of human excretory system and Identify the name also.
answer the following questions. (a) 3, Descending limb of loop of Henle
(b) 5, Vasa recta
(c) 6, Proximal convoluted tubule
A (d) 7, Collecting duct
B
71. Which part is capable of reabsorption of HCO3 - and
C
selective secretion of hydrogen and potassium ions and
NH3 to maintain the pH and sodium-potassium balance in
E blood?
(a) 1 (b) 3
(c) 5 (d) 7
D
72. Which structures have an ability to produce the
concentrated urine?
(a) 1, 2, 3 (b) 3, 4, 5
X (c) 6, 7, 8 (d) 2, 4, 7
EBD_7209
168 Biology

CRITICAL THINKING TYPE QUESTIONS 82. In nephron water absorption is maximum in


(a) proximal convoluted tubule (PCT).
73. Workers in deep mines usually suffer from dehydration (b) ascending limb of Henle.
because (c) descending limb of Henle.
(a) water is lost due to evaporation. (d) distal convoluted tubule (DCT).
(b) water is lost due to defecation. 83. Human urine contains
(c) water is lost in the form of urine. (a) 95% water, 2.6% urea, 2% salts and some uric acid.
(d) water is lost along with salts in the form of sweat. (b) 99% water and 1% urea.
74. Which of the following sets of animals produce the same (c) 92% water and 8% salts.
substances as their chief excretory product?
(d) 90% water, 8% uric acid and 2% proteins.
(a) Fish, pigeon and frog
84. Glomerular filtrate contains
(b) Camel, housefly and snake
(c) Frog, monkey and dog (a) blood without blood cells and proteins
(d) Amoeba, ant and antelope (b) plasma without sugar
75. Filtration slits are formed by (c) blood with proteins but without cells
(a) endothelial lining of glomerular capillaries. (d) blood without urea
(b) inner epithelium of Bowman’s capsule. 85. __i___ and ___ii___ carries the waste products.
(c) basement membrane. (a) i- Renal artery, ii- Renal vein
(d) the participation of all of these. (b) i- Renal vein, ii- Urethra
76. The cells named podocytes occur in (c) i- Renal vein, ii- Ureter
(a) inner wall of Bowman’s capsule (d) i- Renal artery, ii-Ureter
(b) outer wall of Bowman’s capsule 86. The urine under normal conditions does not contain
(c) in the wall of glomerulus glucose because
(d) in the wall of Henle’s loop (a) the normal blood sugar is fructose.
77. Ultrafiltration occurs in a glomerulus when (b) glucose of blood is not filtered in the glomerulus.
(a) hydrostatic pressure exceeds osmotic pressure.
(c) glucose in glomerular filtrate is reabsorbed in the
(b) osmotic pressure exceeds hydrostatic pressure.
uriniferous tubules.
(c) capsular hydrostatic pressure exceeds glomerular
hydrostatic pressure. (d) glucose in glomerular filtrate is converted into
(d) colloidal osmotic pressure plus capsular pressure glycogen.
remain less than glomerular hydrostatic pressure. 87. When a person is suffering from poor renal reabsorption,
78. Filtration fraction is the ratio of which one of the following will not help in maintenance of
(a) glomerular filtration rate (GFR) to renal plasma flow blood volume?
(RPF) (a) increased ADH secretion.
(b) glomerular filtrate to urine (b) decreased glomerular filtration.
(c) haemoglobin to oxyhaemoglobin (c) increased arterial pressure in kidneys.
(d) O2 to CO2 (d) decreased arterial pressure in kidneys.
79. In a mammalian kidneys, Bowman’s capsules occur in 88. Atrial natriuretic factor (ANF) is released in response to
(i) while loops of Henle are situated in (ii) . the increase in blood volume and blood pressure. Which
(a) (i) – cortex, (ii) – medulla of the followings is not the function of ANF? It
(b) (i) – medulla, (ii) – cortex (a) stimulates aldosterone secretion.
(c) (i) – cortex, (ii) – pelvis (b) inhibits the release of renin from JGA.
(d) (i) – pelvis, (ii) – medulla (c) stimulates salt loss in urine.
80. Urine is hypertonic (d) inhibits sodium reabsorption from collecting duct.
(a) in Bowman’s capsule. 89. Through the thick segment of ascending limb of Henle’s
(b) in PCT. loop the
(c) in the middle of descending & ascending limb of (a) NaCl can pass by active transport from filtrate to the
Henle’s loop. interstitial fluid.
(d) at the end of ascending limb of Henle’s loop. (b) NaCl can pass by passive transport into interstitial
81. Diuresis is a condition characterized by fluid.
(a) increase in urine volume. (c) NaCl cannot pass from the filtrate to interstitial fluid.
(b) increased glucose excretion. (d) Water can pass freely from filtrate to interstitial fluid.
(c) decrease in urine volume. 90. Which of the following is likely to accumulate in
(d) decrease in electrolyte balance. dangerous proportion in the blood of a person whose
kidney is not working properly?
Excretory Products and Their Elimination 169
+
(a) Ammonia (b) Urea (c) Distal convoluted tubule : Reabsorption of K ions
(c) Lysine (d) Sodium chloride into the surrounding blood capillaries.
91. If excess water passes out from tissues without being (d) Afferent arteriole : Carries the blood away from the
restored by kidneys, the cells would glomerular towards renal vein.
(a) burst open and die 100. Which one of the following enables the mammalian kidney
(b) not be affected at all to regulate water reabsorption during states of
dehydration?
(c) extract water from plasma
(a) The cells of the tubules detect the osmotic pressure
(d) shrivel and die
of the blood.
92. There is no sugar in urine. The blood entering the kidney (b) Water is extracted from the glomerular filtrate in the
has more sugar than leaving the kidney because proximal tubules.
(a) sugar is used by kidney cells in metabolism. (c) The kidney produces a hypotonic urine.
(b) sugar is absorbed by bladder. (d) Hormones increase the permeability of the collecting
(c) sugar is absorbed by proximal convoluted tubule. ducts.
(d) sugar is absorbed in Loop of Henle. 101. If the afferent arteriole that supplies blood to the
93. In human beings the capsular urine entering the Proximal glomerulus becomes dilated,
Convoluted Tubule (PCT) is (a) the protein concentration of the filtrate decreases.
(a) isotonic to blood (b) hydrostatic pressure in the glomerulus decreases.
(b) hypotonic to blood (c) the glomerular filtration rate increases.
(c) hypertonic to blood (d) all of the above
(d) isotonic to sea water 102. "X" causes reabsorption of "Y" and water from the distal
94. If the diameter of afferent renal arteriole is decreased and parts of the tubule. This also leads to an increase in "Z"
and glomerular filtration rate (GFR).
that of efferent renal arteriole increased, ultra filtration will
Identify X, Y and Z.
(a) be faster
(a) X: ADH; Y: Na+; Z: Blood pressure
(b) be slower (b) X: ADH; Y: K+; Z: Blood pressure
(c) not take place (c) X: Aldosterone; Y: Na+; Z: Blood pressure
(d) take place in the same speed (d) X: Aldosterone; Y: K+; Z: Ionic concentration
95. What will happen if the stretch receptors of the urinary 103. Under normal conditions which one is completely
bladder wall are totally removed? reabsorbed in the renal tubule?
(a) There will be no micturition (a) Salts (b) Urea
(b) Urine will not collect in the bladder (c) Glucose (d) Uric acid
(c) Micturition will continue 104. Identify the parts mentioned below which constitute a
(d) Urine will continue to collect normally in the bladder part of single uriniferous tubule.
96. In a kidney machine, which of the following passes from i. Loop of Henle.
the blood to the dialysis fluid? ii. Collecting duct
(a) Glucose (b) Plasma protein iii. Bowman's capsule
(c) Red blood cells (d) Urea iv. Distal convoluted tubule
97. Excretion means (a) i, ii and iii (b) ii, iii and iv
(a) removal of substances which have never been a part (c) i, iii and iv (d) All of the above
of body. 105. A patient was diagnosed that one process of his excretory
(b) removal of faecal matter from the body. system was not functioning properly due to which his
(c) removal of substances not required in the body. urine is not concentrating.
(d) all of the above. Identify the organ of the excretory system on which
98. Urine of a human suffering from diabetes inspidus is concentration of urine depends.
(a) concentrated with glucose (a) Collecting duct
(b) concentrated without glucose (b) Bowman's capsule
(c) watery with glucose (c) Length of loop of Henle
(d) watery without glucose (d) Proximal convoluted tubules
99. Which one of the following correctly explains the function 106. Which are the following group of hormones participate in
of a specific part of a human nephron ? the regulation of the renal function?
(a) Podocytes : Create minute spaces (slit pores) for the (a) ADH, TSH and ANF
filtration of blood into the Bowman’s capsule. (b) PCT, TSH and ANF
(b) Henle’s loop : Most reabsorption of the major (c) ADH, DCT and Aldosterone
substances from the glomerular filtrate. (d) ADH, ANF and Aldosterone
EBD_7209
Locomotion and
20

Chapter
Movement

FACT/DEFINITION TYPE QUESTIONS (c) The tail and a head


(d) F-actin and G- actin, but both globular.
1. The macrophages in human body exhibit 8. Anisotropic band is
(a) ciliary movement (a) thick and dark (b) thin and dark
(b) amoeboid movement (c) thick and light (d) thin and light
(c) no movement 9. A-band of the myofibril contains
(d) movement with the blood flow only (a) only thick filaments
2. Striated muscle fibres are found in (b) only thin filaments
(a) urinary bladder (b) lungs (c) both thick and thin filaments
(d) no filaments
(c) gall bladder (d) leg muscles
10. Troponin
3. Smooth muscle is (a) produces sliding movement of microtubules
(a) found in walls of heart only. (b) contains globular head
(b) found in the walls of all the hollow organs except (c) binding to Ca+2 produces skeletal muscle contraction.
heart. (d) covers the active site of actin.
(c) attached to the bones only. 11. During muscle contraction in humans the
(d) found only in the walls of alimentary canal. (a) actin filaments shorten.
4. Cardiac muscles are different from that of skeletal (b) sarcomere does not shorten.
muscles as the former are (c) A-band remain same.
(d) A, H and I bands shorten.
(a) striated but involuntary. 12. A sarcomere is best described as a
(a) movable structural unit within a myofibril bounded
(b) non striated and involuntary.
by H zones.
(c) smooth or unstriated. (b) fixed structural unit within a myofibril bounded by Z
(d) voluntary in action. lines.
5. Striations in the striated muscles are due to (c) fixed structural unit within a myofibril bounded by A
(a) absence of myofilaments. bands.
(b) presence of myofilaments. (d) movable structural unit within a myofibril bounded
(c) specialized arrangement of myofilaments. by Z lines.
13. Which muscle protein acts as ATPase?
(d) projections of myosin.
(a) Actin (b) Troponin
6. Which set clearly identify striated muscles?
(c) Myosin (d) Tropomyosin
(a) Cylindrical, Syncytial and Unbranched 14. During resting stage the binding site of actin for myosin
(b) Spindle, Unbranched and Uninucleated remains masked by
(c) Cylindrical, Striped and Nucleated (a) troponin (b) G-actin
(d) Cylindrical, Striped and Branched (c) tropomyosin (d) meromyosin
7. Actin protein occurs in which of the following two forms 15. Which of the following is the store house of calcium in
? muscles?
(a) Polymeric F- actin and monomeric G- actin (a) Sarcosome
(b) Sarcoplasmic reticulum
(b) Monomeric F- actin and polymeric G-actin
(c) Creatine phosphate
(d) Sarcomere
Locomotion and Movement 171

16. Red muscle fibres are rich in 29. A cup shaped cavity for articulation of femur head is
(a) golgi bodies (b) mitochondria (a) acetabulum (b) glenoid cavity
(c) lysosomes (d) ribosomes (c) sigmoid notch (d) obturator foramen
17. During fatigue 30. Elbow joint is an example of
(a) muscles cannot relax. (a) hinge joint
(b) muscles fail to be stimulated. (b) gliding joint
(c) blood supply to muscles stops. (c) ball and socket joint
(d) nerve supply to muscles become inactive. (d) pivot joint
18. The axon terminals of a nerve cell and the sarcolemma of 31. In humans, coccyx is formed by the fusion of
a skeletal muscle cell join at the_______ (a) 3 vertebra (b) 4 vertebra
(a) motor unit (b) synaptic cleft (c) 5 vertebra (d) 6 vertebra
(c) action potential (d) neuromuscular junction 32. An example of gliding joint is
19. In mammals the lower jaw is made of (a) humerus and glenoid cavity
(a) maxilla (b) dentary (b) femur and tibio-fibula
(c) mandible (d) ethmoid (c) occipital condyle and odontoid process
20. Hyoid bone is located (d) zygapophyses of adjacent vertebrae
(a) at the top of the buccal cavity. 33. Identify the joint between sternum and the ribs in humans.
(b) at the floor of the buccal cavity. (a) Fibrous joint (b) Gliding joint
(c) in front of the skull. (c) Cartilaginous joint (d) Angular joint
(d) behind the skull. 34. The joint in our neck which allows us to rotate our head
21. Which of the following is a single U shaped bone, present left to right is
at the base of the buccal cavity and it is also included in (a) pivot joint (b) hinge joint
the skull? (c) saddle joint (d) ellipsoid joint
(a) Hyoid (b) Malleus 35. Accumulation of uric acid crystals in the synovial joint
(c) Sacrum (d) Scapula causes
22. Which of the following is not part of axial skeleton? (a) rheumatoidal arthritis
(a) Sacrum (b) Sternum (b) gout
(c) osteoarthritis
(c) Mandible (d) Humerus
(d) muscular dystrophy
23. The vertebral formula of human adult is
36. Which of the following is an autoimmune disorder ?
(a) C7T12L5S5Cd1 (b) C7T12L 5S5Cd5
(a) Myasthenia gravis
(c) C7T12L5S5Cd4 (d) C7T12L5S4Cd4 (b) Osteoporosis
24. Part of the body having a single pair of bones is called (c) Muscular dystrophy
(a) pelvic girdle (b) external ear (d) Gout
(c) wrist (d) lower jaw
25. Which of the following vertebrae are fused? STATEMENT TYPE QUESTIONS
(a) Cervical (b) Sacral 37. Which of the following statement is incorrect ?
(c) Lumber (d) Thoracic (a) All movements lead to locomotion.
26. Glenoid cavity is found in (b) Ciliary movement help in passage of ova through
(a) pelvic girdle (b) skull female reproductive tract.
(c) pectoral girdle (d) sternum (c) Microfilaments are involved in amoeboid
27. Function of long bones in adult mammals is to provide movement.
(a) support only. (d) In Paramecium, the cilia help in movement of food
(b) support and produce RBCs only. through cytopharynx and in locomotion as well.
(c) support and produce WBCs only. 38. Which of the following is not the feature of red muscle
fibres?
(d) support and produce RBCs and WBCs.
(a) They have plenty of mitochondria.
28. Number of floating ribs in human body is
(b) They have high content of myoglobin.
(a) two pairs (b) three pairs
(c) They have high amount of sarcoplasmic reticulum.
(c) five pairs (d) six pairs
(d) They are called aerobic muscles.
EBD_7209
172 Biology
39. What is not true about human skull? (b) The vertebral column has 10 thoracic vertebrae.
(a) It is dicondylic. (c) The joint between adjacent vertebrae is a fibrous
(b) It includes 6 ear ossicles. joint.
(c) It includes 14 facial bones. (d) The decreased level of progesterone causes
(d) Hyoid is not included in skull bones. osteoporosis in old people.
40. Which of the following statement is correct? 47. Which of the following statements is/are correct?
(a) All striated muscles are voluntary. (i) During muscle contraction, isotropic band gets
(b) Visceral muscles are faintly striated. elongated.
(c) Cardiac muscles are not striated. (ii). Acetylcholine is released when the neural signal
(d) All non-striated muscles are involuntary. reaches the motor end plate.
41. Read the following statements (A to D) and select the (iii) Muscle contraction is initiated by the signal sent
one option that contains both correct statements. by CNS via a sensory neuron.
A. Z-line is present in the centre of the light band. (iv) Repeated activation of muscle can lead to lactic acid
B. Thin filaments are firmly attached to the M-line. accumulation.
C. The central part of thick filaments, not overlapped (a) (i) and (iii) (b) (ii) and (iv)
by thin filaments is called Z-band. (c) (i), (ii) and (iii) (d) (ii), (iii) and (iv)
D. Light band contains only thin filaments. 48. According to the sliding filament theory of muscle
(a) A and D (b) B and C contraction,
(c) A and C (d) B and D (a) actin binds ATP and breaks it apart as actin pulls
42. During muscle contraction. against myosin.
(a) chemical energy is changed to electrical energy. (b) calcium ions are released from myosin as the
(b) mechanical energy is changed to chemical energy. filaments slide by.
(c) chemical energy is changed to physical energy. (c) the thick and thin filaments do not change length
(d) chemical energy is changed to mechanical energy. during this process.
43. Contractile tissues have the following features. (d) all of the above
(i) They are mesodermal in origin. 49. Read the following 4-statements (i – iv) and accordingly
(ii) They contain stretch receptors. mark the option that has both correct statements.
(iii) Rhythmic contractions are seen in them. (i) Cardiac fibres are branched with one or more nuclei.
(iv) They do not fatigue during the life of the animal. (ii) Smooth muscles are unbranched and cylindrical.
Which of the above are characteristics of sphincters? (iii) Striated muscles can be branched or unbranched.
(a) Only (i), (iii) and (iv) (iv) Involuntary muscles are non-striated.
(b) Only (i), (ii) and (iii) (a) (i) and (iv) (b) (ii) and (iii)
(c) Only (i), (ii) and (iv) (c) (iii) and (iv) (d) (i) and (iii)
(d) All of these
44. Which one of the following is the correct description ASSERTION/REASON TYPE QUESTIONS
of certain part of a normal human skeleton?
In the following questions, a statement of Assertion is followed
(a) Parietal bone and the temporal bone of the skull
by a statement of Reason.
are joined by fibrous joint.
(a) If both Assertion and Reason are true and the Reason is
(b) First vertebra is axis which articulates with the
the correct explanation of the Assertion.
occipital condyles.
(c) The 9th and 10th pairs of ribs are called the floating ribs. (b) If both Assertion and Reason are true but the Reason is
(d) Glenoid cavity is a depression to which the thigh not the correct explanation of the Assertion.
bone articulates. (c) If Assertion is true but Reason is false.
45. Select the correct statement regarding the specific (d) If both Assertion and Reason are false.
disorder of muscular or skeletal system. 50. Assertion-: Knee joint is an example of hinge joint.
(a) Myasthenia gravis – Autoimmune disorder which Reason : Femur, patella and fibula are associated with
inhibits sliding of myosin filaments. knee joint.
(b) Gout – Inflammation of joints due to extra 51. Assertion : The phase of muscle contraction occurs
deposition of calcium. when myosin binds and releases actin.
(c) Muscular dystrophy – Age related shortening of
Reason : Muscle contraction is initiated by a signal sent
muscles.
(d) Osteoporosis – Decrease in bone mass and higher by the peripheral nervous system via a motor neuron.
chances of fractures with advancing age. 52. Assertion : Recurrent activation of the muscles will
46. Select the correct statement with respect to locomotion become fatigue.
in humans. Reason : Anaerobic breakdown of glycogen in the muscles
(a) Accumulation of uric acid crystals in joints causes can lead to the accumulation of lactic acid.
their inflammation.
Locomotion and Movement 173
53. Assertion : Inflammation of a skeletal joint may immobilize (a) A and B only (b) A, B and C only
the movements of the joint. (c) A, B, and D only (d) All of the above
Reason :Uric acid crystals in the joint cavity and
ossification of articular cartilage lead to this. 60. Match column I (types of synovial joints) with column II
54. Assertion : Ball and socket joints are the most mobile (bones involved) and choose the correct option.
joints. Column I Column II
Reason : Synovial fluid is present here. (Type of synovial joint) (Bone involved)
55. Assertion : Arthritis or inflammation of a joint makes A. Ball and Socket joint I. Carpal and metacarpal
the joint painful. of thumb
Reason : Some toxic substances are deposited at the B. Hinge joint II. Humerus and pectoral
joint.
girdle
MATCHING TYPE QUESTIONS C. Pivot joint III. Knee
D. Saddle joint IV. Atlas and axis
56. Three of the following pairs of the human skeletal parts (a) A – I; B – II; C – III; D – IV
are correctly matched with their respective inclusive
(b) A – II; B – III; C – IV; D – I
skeletal category and one pair is not matched. Identify
(c) A – III; B – I; C – IV; D – II
the non-matching pair.
(d) A – IV; B – III; C – II; D – I
Pairs of skeletal parts Category
(a) Sternum and ribs – Axial skeleton 61. Which of the following option shows the correctly
matched bones (given in column I) with its pair (given in
(b) Clavicle and glenoid – Pelvic girdle cavity
column II)?
(c) Humerus and ulna – Appendicular skeleton
Column-I Column-II
(d) Malleus and stapes – Ear ossicles
A. Carpals I. Bones that form the
57. Which of the following pair shows the correct fingers and toes
characteristics with an example of a synovial joint in
B. Tarsals II. Bones that form wrist
humans?
C. Phalanges III. Bones that form the
Characteristics Examples
palms of the hands
(a) Fluid filled between – Skull bones
D. Metatarsals IV. Bones that form the
two joints, provides
ankles
cushion
(b) Fluid filled synovial – Joint between (a) A – II, B – IV, C – I, D – III
cavity between two atlas and axis (b) A – I, B – II, C – III, D – IV
bones (c) A – III, B – II, C – IV, D – I
(c) Lymph filled between – Gliding joint (d) A – IV, B – I, C – III, D – II
two bones, limited between 62. Which of the following functional characteristics of
movement carpals muscle is correctly matched with its appropriate
(d) Fluid cartilage – Knee joint descriptive term?
between two bones, I. Elasticity- Ability of a muscle fibre to recoil and
limited movements resume its resting length after being stretched.
58. Which of the following pairs of joints and its location is II. Excitability- Ability to respond to any change in the
correctly matched? environment (inside or outside the body)
(a) Hinge joint – Between vertebrae III. Extensibility- Ability to be stretched
(b) Gliding joint – Between zygapophyses of IV. Contractility- Ability to shorten forcibly when
the successive vertebrae adequately stimulated
(c) Cartilaginous joint– Skull bones (a) I and III only (b) II and IV only
(d) Fibrous joint – Between phalanges (c) I, II, and III only (d) All of these
59. Which of the following human skeletal parts are 63. Match the name of bone given in column I with their
correctly matched with their respective category? numbers given in column II.
Pairs of skeletal parts Category Column-I Column-II
A. Humerus and ulna – Appendicular skeleton A. Thoracic I. 8
B. Malleus and stapes – Ear ossicles B. Wrist bones II. 2 pairs
C. Sternum and ribs – Axial skeleton C. False ribs III. 12
D. Clavicle and glenoid – Pelvic girdle
D. Metatarsal IV. 3 pairs
cavity
EBD_7209
174 Biology
E. Skull (cranial & facial) V. 26 (a) Tibia and Tarsals
VI. 5 (b) Femur and Fibula
VII. 22 (c) Fibula and Phalanges
A B C D E (d) Tarsals and Femur
(a) I VI II V III 67. In the given diagram of skull, what does “I” represent?
(b) III I IV VI VII (a) Frontal bone (b) Temporal bone
(c) I VII III V II (c) Occipital bone (d) Parietal bone
(d) II V VII I IV
64. Match the column I with column II and select the correct
option.
Column-I Column-II
A. Striated appearance of I. Distribution pattern of
myofibril actin and myosin (I)
B. Store house of calcium II. Sarcoplasmic reticulum
ions
C. Energy released III. Helps myosin head to bind
from ATP hydrolysis to exposed active sites
on actin to form a cross
bridge 68. The diagram given below shows the pelvic girdle and
D. Globular head of IV. Active ATPase enzyme
lower limb.
meromyosin and has binding sites
for ATP and active sites IV V
for actin. I
A B C D
(a) I II III IV
(b) III I IV II II
(c) II I III IV
(d) III IV II I
65. Which of the following match is incorrect?
(a) 8th, 9th and 10th pairs of ribs - do not articulate III
directly with the sternum but join the sixth rib with
the help of hyaline cartilage.
(b) Glenoid cavity - articulates with the head of the
humerus to form the shoulder joint. Parts labelled as ‘I’, ‘II’, ‘III’, ‘IV’ and ‘V’ respectively
(c) Fibrous joint - flat skull bones which fuse end-to- indicate
end with the help of dense fibrous connective
tissues in the form of sutures, to form the cranium. (a) Ilium, Femur, Tibia, Pubis and Sacrum
(d) Increase in Ca++ level - leads to the binding of calcium (b) Pubis, Tibia, Femur, Ilium and Sacrum
with a subunit of troponin on actin filaments and (c) Ilium, Femur, Tibia, Pubis and Sacrum
thereby remove the masking of active sites for myosin. (d) Pubis, Femur, Tibia, Ilium and Sacrum
69. The given diagram of human skeleton system shows types
DIAGRAM TYPE QUESTIONS of ball and socket joint. Identify the joints which are
66. Given below is a diagram of the bones of the left human marked as I, II, III, IV and V
hindlimb as seen from the front. It has certain mistakes I
in labelling. Identify the two bones which are not
correctly labelled. II

III

Femur IV

V
Fibula
Tibia

Tarsals
Phalanges (a) I and II (b) II and IV
(c) III and IV (d) IV and V
Locomotion and Movement 175

70. The given diagram represents the bones of human arm. 73. The label X in the given figure of an act in filament
Identify the bones marked as I, II, III & IV. represents

IV
I
(a) actin (b) myosin
II
III (c) tropomyosin (d) troponin
74. The given figure represents the cross bridge cycle in
skeletal muscle. What does the step B in the figure represents?

I II III IV Myosin head ADP


(high-energy P1
(a) Clavicle Ulna Radius Humerus configuration)

(b) Humerus Radius Ulna Scapula


(c) Scapula Radius Ulna Clavicle A
(d) Humerus Ulna Radius Scapula Thin filament

71. Which two bones in the given figure form a hinge joint?
ADP Thick filament ADP
ATP
hydrolysis P1

D B
IV
I
II ATP
Myosin head
III ATP (low-energy
configuration)

(a) I and II (b) I and III (a) Attachment of myosin head to actin forming cross
bridge.
(c) I and IV (d) II and III
(b) Release of phosphate. Myosin changes shape to pull
72. The given diagram shows a human skeleton. Which of actin.
the following correctly identifies the bones marked as (c) Attachment of new ATP to myosin head. The cross
I, II, III & IV? bridge detaches.
(d) Splitting of ATP into ADP and Pi. Myosin cocks into
its high energy conformation.
75. Refer the following figure and answer the question.
Fusion of which of the following marked bones (1- 6)
are responsible for the formation of coxal bones?
I
II

III
IV

I II III IV
(a) Tibia Fibula Radius Ulna
(b) Radius Ulna Tibia Fibula
(c) Ulna Radius Tibia Fibula
(d) Fibula Tibia Radius Ulna
EBD_7209
176 Biology
(a) 1, 2, 3 (b) 4, 5, 6 (a) Humerus: Longest bone of upper extremity and is
(c) 1, 2, 5 (d) 3, 5, 6 characterized by presence of deltoid tuberosity for
76. The given figure shows right pectoral girdle and upper the attachment of muscles.
arm. Few parts are marked as 1, 2, 3 and 4. (b) Radius: It is a smaller bone and formed by sesamoid
bone.
(c) Ulna: The bone extending from the elbow to the
wrist on the side opposite to the thumb in humans.
(d) Femur: Longest and largest bone of body.

CRITICAL THINKING TYPE QUESTIONS


79. Which of the following represents the correct order of
vertebral regions from superior to inferior?
I. Sacrum II. Thoracic
III. Cervical IV. Lumbar
V. Coccyx
(a) I – II – III – IV – V (b) II – IV – I – III – V
(c) IV – I – II – V – I (d) III – II – IV – I – V
Which of the following options shows the correct 80. What is the correct order that a motor nerve impulse
labelling of marked parts? travels when triggering a muscle contraction?
1 2 3 4 (a) Motor nerve®synaptic cleft®sarcolemma®
(a) Clavicle Scapula Hu merus Radiu s sarcoplasmic reticulum®troponin.
(b) Scapu la Clav icle Radiu s Humeru s (b) Motor nerve®synaptic
cleft®sarcolemma®troponin ®sarcoplasmic
(c ) Sacru m Scapula Uln a Tib ia
reticulum.
(d) Rad iu s Clav icle Scapula Humeru s
(c) Motor nerve®sarcoplasmic reticulum®synaptic
77. The given figure shows the diagrammatic cross sectional cleft®sarcolemma®troponin.
view of a muscle with their parts marked as 1, 2, 3 and 4. (d) Motor nerve®sar colemma®sarcoplasmic
Which part is held together by a common collagenous reticulum® synaptic cleft®troponin.
connective tissue layer? 81. There are three blanks in the following statement. Mark
the correct option having suitable words for filling the
blanks.
4
The thin filaments of myofibril contain ....... ‘A’......actin
1 and two filaments of ....... ‘B’......protein along with
2 ....... ‘C’......protein for masking binding site for myosin.
3
‘A’ ‘B’ ‘C’
(a) 1F troponin tropomyosin
(a) 1 (b) 2
(b) 1F tropomyosin troponin
(c) 3 (d) 4
78. The given figure shows the structure of pectoral girdle (c) 2F troponin tropomyosin
and upper arm. Identify the structure marked as "X" and (d) 2F tropomyosin troponin
its feature. 82. In which option the number of bones of two
corresponding parts are not the same?
(a) Thigh and upper arm
(b) Sole and palm
X (c) Ankle and wrist
(d) Leg and arm
83. The intercalated discs of________ muscle________.
(a) smooth; provide strong mechanical adhesion and
rapid electrical communication
Locomotion and Movement 177

(b) skeletal; are the basis for all voluntary muscle action (d) The absence of myofibrils in the central portion of
(c) skeletal; make possible both fast twitches and slow A-band.
twitches 92. Eye-lid muscles have
(d) cardiac; provide strong mechanical adhesion and (a) thick fibres with abundant mitochondria.
rapid electrical communication (b) thick fibres without myoglobin.
84. Convexity of one bone articulate with concavity of other (c) thin fibres with myoglobin.
bone in (d) thin fibres with lesser mitochondria.
(a) pivot joint (b) hinge joint 93. Which of the following structures is correctly organized
(c) gliding joint (d) ball and socket joint from large to small?
85. All or None law is associated with (a) Muscle, Muscle cell, Myofibril, Sarcomeres,
(a) skeletal muscle fibre Filaments.
(b) neuron (b) Muscle, Muscle fibres, Sarcomeres, Filaments,
(c) cardiac muscle fibres Myofibrils.
(d) all of the above (c) Muscle, Sarcolemma, Myofibrils, Actin filaments,
86. Long uninucleate muscles are found in Myosin filaments.
(a) diaphragm (b) alimentary canal (d) Muscle cells, Myofibrils, Filaments, Sarcoplasm.
94. Muscle contraction is triggered
(c) tongue (d) eye
(a) when high levels of oxygen and sugar are released
87. Which of the following is not exclusively supplied with
by the sarcolemma.
involuntary muscles?
(b) when a surplus of ATP is released by a nerve motor
(a) Muscles of iris
unit.
(b) Muscles of the ducts of gland
(c) by release of a neurotransmitter at a synapse that
(c) Muscles of urethra
directly causes actin and myosin to slide.
(d) Muscular coats of blood vessel (d) by the nerve releasing a neurotransmitter, which
88. Muscles of alimentary canal are mainly triggers a flow of calcium that attaches to actin
(a) striated and myogenic filaments and exposes the myosin binding sites.
(b) striated and neurogenic 95. The joint between _____(i)____ and _____(ii)_____
(c) unstriated and neurogenic forms ball and socket joint.
(d) unstriated and myogenic (a) (i) - Humerus, (ii) - Ulna
89. Muscle A and muscle B are of the same size, but muscle A (b) (i) - Humerus, (ii) - Scapula
is capable of much finer control than muscle B. Which (c) (i) - Ulna, (ii) - Radius
one of the following is likely to be true for muscle A? (d) (i) - Ulna, (ii) - Scapula
(a) It contains fewer motor units than muscle B. 96. Given below are some events which occur during muscle
(b) It has larger sarcomeres than muscle B. contraction.
(c) It is controlled by more neurons than muscle B. i. ATP is hydrolyzed.
(d) It is controlled by fewer neurons than muscle B. ii. Myosin heads bind to actin.
90. A cricket player is fast chasing a ball in the field. Which iii. Hemoglobin concentration in muscle fibers
one of the following groups of bones are directly increases.
contributing in this movement? iv. Calcium concentration in the sarcomere increase.
(a) Femur, malleus, tibia, metatarsals v. I bands shorten and H zones disappear.
(b) Pelvis, ulna, patella, tarsals Select the correct events which occur during muscle
(c) Sternum, femur, tibia, fibula contraction.
(d) Tarsals, femur, metatarsals, tibia (a) i only (b) ii, iii & iv only
91. The H-zone in the skeletal muscle fibre is due to (c) i, ii, iv & v only (d) All of these.
(a) The central gap between myosin filaments in the 97. The striations that give skeletal muscle its characteristic
A-band. striped appearance are produced by
(b) The central gap between actin filaments extending (a) the T tubules.
through myosin filaments in the A-band. (b) sarcoplasmic reticulum.
(c) Extension of myosin filaments in the central (c) arrangements of myofilaments.
portion of the A-band. (d) a difference in the thickness of sarcolemma.
EBD_7209
178 Biology
98. What will happen if the sarcoplasmic reticulum of the 102. A person is suffering from an age related disorder "X". X
muscle fibres is damaged? is characterized by decreased bone mass and increased
(a) Binding of ATP to actin will be affected. chances of fractures. Identify X and its common cause.
(b) Release of inhibition on Z discs will stop. (a) Tetany, Increased levels of estrogen
(c) Exposure of myosin binding sites on the actin will (b) Osteoporosis, Decreased levels of estrogen
be affected. (c) Myasthenia gravis, Decreased levels of estrogen
(d) Transmission of action potential along the (d) Muscular dystrophy, Increased levels of estrogen
sarcolemma will increase. 103. A student was given sample of two muscles marked as 1
99. Lubrication occurs at hinge joints which need to be able and 2. When he compared the muscles he found that
to move without friction. Which one of the following muscle 1 contains large amount of myoglobin and utilize
substances aids lubrication? large amount of stored oxygen for ATP production
(a) Cartilage (b) Ligament whereas muscle 2 contains few myoglobin, mitochondria
(c) Synovial fluid (d) Connective tissue and high sarcoplasmic reticulum.
100. Which of the following is not the function of skeleton? Identify the correct conclusion regarding the muscle 1
(a) It allows movement. and 2 from the option given below.
(b) It supports the body. (a) Both the muscles are called aerobic muscles.
(c) It connects muscle to joints. (b) Both the muscles are called red fibers and depend
on aerobic process for energy.
(d) It protects the internal part of the body.
(c) Muscle 1 is called white fibers and whereas muscle
101. "X" is a large triangular flat bone situated in the dorsal
2 depends on aerobic process for energy.
part of the thorax between the "Y" and the seventh ribs.
Identify "X" and "Y". (d) Muscle 1 is called red fibers and aerobic muscles
whereas muscle 2 is called white fibers and depend
(a) X - Patella ; Y - Third
on anaerobic process for energy.
(b) X - Clavicle ; Y -Eight
(c) X - Scapula ; Y - Sixth
(d) X - Scapula ; Y - Second
Neural Control and
21

Chapter
Coordination

FACT/DEFINITION TYPE QUESTIONS 9. Which of the following ions are required for nerve
conduction ?
1. The nerves carrying impulses to CNS are known as (a) Ca++, Na+ and K+ (b) Ca++ and Mg++
(a) motor (b) efferent (c) Mg++ and K+ (d) Na+ and K+
(c) afferent (d) mixed 10. During rest, sodium pump of a nerve results in
2. The controlling centre of autonomic nervous system is (a) more Na+ pumped out than K+ ions taken in.
(a) hypothalamus (b) spinal cord (b) Na+ pumped in without exchange with any other ion.
(c) cerebellum (d) pons (c) exchanging equal amounts of Na+ and K+.
(d) more Na+ being pumped in than K+ ions pumped out.
3. A bipolar neuron has
11. A typical value of resting membrane potential is
(a) 2 dendrites and 1 axon
(a) – 40m V (b) – 60m V
(b) 2 axons and 1 dendrite (c) – 70m V (d) – 80m V
(c) 1 dendrite and 1 axon 12. The transmission of impulse through neurons is a
(d) 2 axons and 2 dendrites (a) physical phenomenon
4. Sodium – Postassium pump across membrane, actively (b) chemical phenomenon
transports (c) electro-chemical phenomenon
(a) 2 Na ions outwards and 3 K ions into the cell. (d) gravitational phenomenon
(b) 3 Na ions outwards and 2 K ions into the cell. 13. The resting potential occurs because
(c) 2 K ions outwards and 3 Na ions into the cell. (a) of reduced energy production by mitochondria.
(d) 3 K ions outwards and 2 Na ions into the cell. (b) the action potential depletes transmitter substance.
5. During conduction of nerve impulse (c) of the different concentrations of ions across the cell.
(a) Na+ moves into axoplasm (d) the action potential causes axoplasmic transport back
(b) Na+ moves out of axoplasm towards the cell body.
(c) K+ moves into axoplasm 14. The cerebral cortex is
(d) Ca++ moves into axoplasm (a) the outer layer of cerebrum, called white matter.
6. In the axons, the nerve impulse travels. (b) inner layer of cerebrum, called white matter.
(a) towards the cell body.
(c) the outer layer of cerebrum, called grey matter.
(b) away from the cell body.
(d) inner layer of cerebrum, called grey matter.
(c) away from synapse.
(d) in both direction. 15. The secretion of gastric juice is controlled by
7. Action potential of nerve cell is (a) cerebellum (b) ANS
(a) – 60 mV (b) –80 mV (c) cerebrum (d) medulla
(c) +20 mV (d) +30 mV 16. Centre for sense of smell is
8. During recovery, a nerve fibre becomes (a) cerebellum (b) olfactory lobes
(a) positively charged outside and negatively charged (c) pons (d) midbrain
inside. 17. Purkinje cells are found in
(b) positively charged on both-outside and inside. (a) cerebellar cortex (b) mammalian heart
(c) negatively charged outside and positively charged (c) voluntary cells (d) semicircular canal
inside. 18. Clusters of neuron cell bodies embedded in the white
(d) negatively charged on both-outside and inside. matter of the brain are referred to as
(a) nuclei (b) gyri
(c) sulci (d) ganglia
EBD_7209
180 Biology
19. Part of mammalian brain controlling muscular coordination 30. The region of vertebrate’s eye where the optic nerve
is passes out of the retina is called
(a) cerebrum (b) corpus callosum (a) yellow spot (b) optic chiasma
(c) medulla oblongata (d) cerebellum (c) fovea (d) blind spot
20. Pons connects 31. The amount of light that falls on retina is regulated by
(a) brain with spinal cord (a) lens (b) cornea
(b) cerebrum with cerebellum (c) iris (d) ciliary muscles
(c) two-lobes of cerebellum 32. For seeing the nearby objects, the lens becomes more
(d) two cerebral hemispheres convex by
21. Part of brain responsible for hearing is (a) relaxation of iris muscles.
(a) cerebellum (b) cerebrum (b) contraction of iris muscles.
(c) medulla (d) hypothalamus (c) contraction of ciliary muscles.
22. Corpus callosum connects (d) relaxation of ciliary muscles.
(a) two cerebral hemispheres 33. Space between cornea and lens is called
(b) two ventricles of brain (a) aqueous chamber (b) vitreous chamber
(c) two cerebellar hemispheres (c) canal of schlemm (d) fovea centralis
(d) two optic thalamus 34. In human eye, the blind spot contains
23. Which of the following cell in the central nervous system (a) rods
functionally equivalent to a Schwann cell? (b) cones
(a) astrocyte (b) neuron (c) both rods and cones
(c) oligodendrocyte (d) microglial cell (d) neither rods nor cones
24. Pons varolii in human brain lies 35. Which of the following is devoid of blood supply?
(a) anterior to optic chiasma
(b) posterior to medulla oblongata (a) Retina (b) Choroid
(c) ventral to cerebellum (c) Cornea (d) Scleroid
(d) dorsal to diencephalon 36. Eye ball is moved in the orbit by
(a) four rectus and two oblique muscles
25. Which of the following is a thin middle layer of cranial
(b) ciliary muscles
meninges?
(c) suspensory ligaments
(a) Duramater (b) Arachnoid (d) two rectus and four oblique muscles
(c) Piamater (d) Optic nerve 37. The blind spot is the region where
26. Which of the following group of functions was regulated (a) image is formed.
by part of hindbrain? (b) cones are numerous.
(a) Sexual behaviour, body temperature, drinking. (c) the optic nerve leaves out.
(b) Gastric secretion, cardiovascular reflexes, respiration. (d) image is formed during the dark.
(c) Memory and communication, cardiovascular reflexes, 38. Which part of the human eye adjust the focal length of
respiration. lens ?
(d) Gastric secretion, body temperature, Memory and (a) Aqueous humour (b) Ciliary body
communication. (c) Conjunctiva (d) Cornea
27. Which is not a reflex action? 39. Colour perception in man is due to the presence of
(a) Swallowing of food (a) rhodopsin pigment in rod cells.
(b) Shivering in cold (b) iodopsin pigment in cone cells.
(c) Salivation at choicest food (c) iodopsin pigment in rod cells.
(d) Closure of eyelid by flashing light (d) rhodopsin pigment in cone cells.
28. Twilight vision is also called
40. The cochlea of ear contains
(a) scotopic vision and is the function of rods.
(a) perilymph
(b) scotopic vision and is the function of cones.
(b) aqueous humour
(c) photopic vision and is the function of rods.
(c) perilymph and endolymph
(d) photopic vision and is the function of cones.
(d) only endolymph
29. The thinned-out portion of retina where only cones are
41. At the base of cochlea, the canal that ends at the oval
densely packed is called
window is
(a) blind spot (b) corpus luteum
(a) scala tympani (b) scala media
(c) macula lutea (d) fovea
(c) scala vestibuli (d) auditory
Neural Control and Coordination 181

42. The stereocilia of hair cells of organ of Corti are covered (a) The space between cornea and lens is filled with
with a thick elastic membrane called transparent gel.
(a) Reissner’s membrane (b) When all cones are stimulated equally, a sensation
(b) basilar membrane of no light (dark) is produced.
(c) tympanic membrane (c) Rhodopsin is purplish red protein, hence called
(d) tectorial membrane visual purple.
43. Macula of labyrinth is bathed in (d) The anterior transparent portion of choroid is called cornea.
(a) aqueous humour (b) vitreous humour 54. Which of the following statement is incorrect?
(c) perilymph (d) endolymph (a) The ear ossicle attached to tympanic membrane is
44. Bony labyrinth of ear contains a fluid known as malleus.
(a) endolymph (b) perilymph (b) Opsin (of rhodopsin) develops from vitamin A.
(c) aqueous humour (d) synovial fluid (c) The pressure on ear drum is equalized by Eustachian tube.
45. The sense of equilibrium is determined by (d) Otolith organ consists of saccule and utricle.
(a) basilar membrane of cochlea.
55. Which of the following statement is correct regarding
(b) tectorial membrane of cochlea.
cerebellum of brain?
(c) sensory crista of ampulla.
(d) sensory cells of organ of corti. (a) It is concerned with the maintenance of posture/
46. Receptor cells for balance in human ear are located in equilibrium.
(a) utricle, saccule and semicircular canal (b) It is responsible for olfactory functions.
(b) malleus, incus and stapes (c) It controls optic functions.
(c) organ of corti (d) both (a) and (c)
(d) Eustachian tube 56. Which of the following statement is an example of
47. In mammals, the organs of Corti is found in conditioned reflex?
(a) scala vestibule (b) scala tympani (a) Hand took up when piercing with a needle.
(c) scala media (d) middle ear (b) Driving a vehicle.
48. In which part of the mammalian ear, the nerve impulse for (c) Eyes closed when any thing enter into it.
hearing starts?
(d) In digestion food goes forward in alimentary canal.
(a) Eardrum (b) Ear ossicles
(c) Cochlea (d) Auditory nerve 57. Identify the main functions of the cerebrum of human
49. Anvil-shaped bone is brain from the given statement.
(a) malleus (b) incus (i) Control the contraction of voluntary muscles
(c) stapes (d) columella auris through the frontal lobe.
50. In mammalian cochlea, the thin-walled sloping roof of the (ii) Control the sensitivity, movement, memory,
scala media is referred to as vocabulary etc. through the frontal lobe.
(a) organ of Corti (b) scala tympani (iii) Control the temperature, taste, touch, pain etc.
(c) basilar membrane (d) reissner’s membrance through the parietal lobe.
51. In static condition, the body balance is sensed by (iv) Control the hearing and sense of smell through the
(a) crista (b) macula occipital and frontal lobes.
(c) both (a) and (b) (d) cochlear canal (a) (i), (ii), (iv) (b) (i) (iii), (iv)
(c) (i), (ii), (iii) (d) (ii), (iii), (iv)
STATEMENT TYPE QUESTIONS 58. During the propagation of a nerve impulse, the action
52. Which of the following statement is correct? potential results from the movement of
(a) Electrical synapses are more common in our neural (a) Na+ ions from extracellular fluid to intracellular fluid.
system than chemical synapses. (b) K+ ions from extracellular fluid to intracellular fluid.
(b) The new potential in post-synaptic neuron may be (c) Na+ ions from intracellular fluid to extracellular fluid.
either excitatory or inhibitory. (d) K+ ions from intracellular fluid to extracellular fluid.
(c) Hypothalamus is the major coordination centre for 59. All are None law (principle) states that
sensory and motor signaling. (a) all stimuli produce action potentials.
(d) The tracts of nerve fibres that connect two cerebral (b) any cell membrane can generate and propagate an
hemispheres are called corpora bigemina. action potential if stimulated to threshold value.
53. Which of the following statement is correct regarding the (c) potential difference can either be 0 or 100.
organ of sight-eye? (d) the property of action potential is independent of
the strength of depolarizing stimulus.
EBD_7209
182 Biology
60. Which statement regarding ‘stapes’ is correct? 66. Which of the following statement is correct?
(a) It lies in the auditory meatus. (a) The space within cochlea called scala media is filled
(b) It fits onto the oval window. with endolymph.
(c) It conducts sound vibrations to fenestra rotundus. (b) The vestibular apparatus is composed of two semi-
(d) It is analogus to columella auris. circular canals and the otolith organ consisting of
61. Which of the following statements are correct? the saccule and utricle.
(i) Somatic nervous system- Conducts impulses from (c) The external auditory meatus helps in equalizing the
CNS to skeletal muscles. pressures on either sides of the ear drum.
(ii) Autonomic nervous system- Conduct impulses from (d) The membranes constituting cochlea, the reissner's
CNS to internal organ muscles. and basilar, divide the surrounding perilymph filled
(iii) Central nervous system- Consists of brain and spinal cord bony labyrinth into an upper scala tympani and a
(iv) Peripheral nervous system- Consists of nerves lower scala vestibuli.
carrying impulses to brain and spinal cord only
ASSERTION/REASON TYPE QUESTIONS
(a) Only (ii) and (iii) (b) Only (iii) and (iv)
(c) Only (i), (ii) and (iii) (d) All of these In the following questions, a statement of Assertion is followed
62. Which of the following statements is correct regarding a by a statement of Reason.
myelinated nerve fibre? (a) If both Assertion and Reason are true and the Reason is
(i) It is always associated with an axon. the correct explanation of the Assertion.
(ii) It allows rapid conduction of nerve impulses. (b) If both Assertion and Reason are true but the Reason is
(iii) It allows slow conduction of nerve impulses. not the correct explanation of the Assertion.
(iv) It has nodes of Ranvier. (c) If Assertion is true but Reason is false.
(a) Only (i) and (ii) (b) Only (i), (ii), and (iii) (d) If both Assertion and Reason are false.
(c) Only (i), (ii), and (iv) (d) Only (i), (iii), and (iv)
67. Assertion : The axonal membrane of the neuron is more
63. Which of the following events is involved in the transfer permeable to sodium ions (Na+) and nearly impermeable
of information across a chemical synapse? to potassium ions (K+).
(i) Neurotransmitters bind to the postsynaptic receptors. Reason : In a resting state neuron does not conduct any
(ii) Calcium channels open in the presynaptic region. impulse.
(iii) Ion channels open in the postsynaptic membrane. 68. Assertion : The chemical stored in the synaptic vesicles
(iv) Direct flow of ions from one neuron to the next. are termed as neurotransmitter.
(a) (i) and (ii) only (b) (i), (ii) and (iii) only Reason : Synaptic vesicles release these chemicals in
(c) (ii), (iii) and (iv) only (d) All of the above synaptic cleft.
64. Which of the following statements is correct about rods 69. Assertion : The imbalance in concentration of Na+, K+
compared to cones? and proteins generates resting potential.
(i) Rods are most numerous in the fovea. Reason : To maintain the unequal distribution of Na+ and
(ii) Rods contain rhodopsin. K+, the neurons use electrical energy.
(iii) Rods produce general outlines of objects rather than 70. Assertion : Astigmatism is due to uneven curvature of
sharp images. lens.
(iv) Rods produce black and white but not colour images. Reason : It is treated with cylindrical lenses.
(a) (i), and (ii) only 71. Assertion : A cerebellum is related with skillful voluntary
(b) (ii), and (iiii) only movement and involuntary activity like body balance,
(c) (i), (ii) and (iii) only equilibrium etc.
(d) (ii), (iii) and (iv) only Reason : It is part of hind brain and it is situated behind
the pons.
65. Which of the following statements is correct?
72. Assertion : The brain stem contains centres for
(a) The internal ear receives sound waves and directs
controlling activities.
them to the ear drum.
Reason : Brain stem is very sensitive.
(b) The membranous canals are suspended in the
73. Assertion : The chemical stored in the synaptic vesicles
endolymph of the bony canals.
are termed as neurotransmitters.
(c) The ear ossicles increase the efficiency of
Reason : Synaptic vesicles release these chemicals in the
transmission of sound waves to the inner ear.
synaptic cleft.
(d) The malleus is attached to the oval window and the stapes
is attached to the temporal membrane of the cochlea.
Neural Control and Coordination 183

79. Match the column I (various phase of an action potential)


MATCHING TYPE QUESTIONS
with column II (ionic activity associated) and choose the
74. Select the answer with correct matching of the structure, correct option.
its location and function. Column I Column II
Structure Location Function A. Resting stage of I. Opening and then
a neuron closing of the sodium
(a) Eustachian Anterior Equalizes air pressure
channels
tube part of on either sides of B. Depolarization phase II. All voltage gated sodium
internal ear tympanic membrane in the generation of and potassium channels
(b) Cerebellum Mid brain Controls respiration an action potential. are closed.
and gastric secretions C. Repolarization phase III. The sodium channels
(c) Hypothalamus Fore brain Controls body in the generation of remain opened.
temperature, urge for action potential
eating and drinking D. Absolute refractory IV. Opening of potassium
(d) Blind spot Near the Rods and cones are phase. gates and the rushing
place where present but inactive of potassium
optic nerve here (a) A – II, B – I, C – IV, D – III
leaves the (b) A – I, B – II, C – III, D – IV
eye (c) A – III, B – IV, C – I, D – II
(d) A – IV, B – II, C – III, D – I
75. Which one of the following is mismatched ?
80. Match the given terms in column I with its definition given
(a) Cerebrum - Memory
in column II and choose the correct options.
(b) Medulla oblongata - Temperature regulation
Column I Column II
(c) Cerebellum - Equilibrium (Terms) (Definition)
(d) Olfactory lobes - Smell A. Semi-circular canal I. Spiral organ of Corti
76. Column I lists the parts of the human brain and column II B. Vestibule II. Fluid found in the
lists the functions. Match the two columns and identify scala vestibule and
the correct option. scala tympani
Column-I Column-II C. Cochlea III. Evaluates rotational
A. Cerebrum I. Controls the pituitary motion
D. Perilymph IV. Fluid found within the
B. Cerebellum II. Controls vision and hearing
organ of Corti
C. Hypothalamus III. Controls the rate of heart E. Endolymph V. Responds to gravity
beat and movements of the
D. Medulla oblongata IV. Maintains body posture head
(a) A – II, B – IV, C – I, D – III (a) A – III, B – V, C – I, D – II, E – IV
(b) A – IV, B – V, C – III, D – I (b) A – I, B – II, C – III, D – IV, E – V
(c) A – V, B – IV, C – III, D – II (c) A – II, B – III, C – IV, D – V, E – I
(d) A – V, B – V, C – I, D – II (d) A – IV, B – I, C – V, D – II, E – III
77. Find out the right matching from the following pairs. 81. Which of the following pair is not correctly matched?
(a) Rods - Twilight vision
(a) Accommodation .................. Pupil
(b) Ciliary body - Iris
(b) Colour perception .................. Cones (c) Retina - Optic chiasma
(c) Night blindness .................. Rods (d) Vitreous humour - Posterior compartment
(d) Binocular vision .................. Iris 82. Select the correct match of the types of neuron present in
78. Which one of the following is the correct difference column I with its location given in column II.
between rod cells and cone cells of retina. Column I Column II
Rod Cells Cone Cells A. Sclera I. The visible coloured
(a) Distribution More Evenly distributed portion of the eye.
concentrated all over retina B. Choroid II. Its diameter is
regulated by the
in centre of
muscle fibres of iris.
retina C. Pupil III. Composed of a dense
(b) Visual acuity High Low connective tissue
(c) Visual pigment Iodopsin Rhodopsin D. Fovea IV. Portion of the retina
contained where only the cones
(d) Over all Vision in Colour vision and are densely packed
E. Iris V. Contains many blood
function poor light detailed vision in
vessels and looks
bright light bluish in colour
EBD_7209
184 Biology
(a) A – I; B – II; C – III; D – IV; E – V Sympathetic Parasympathetic
(b) A – III; B – V; C – II; D – IV; E – I Feature
nervous system nervous system
(c) A – IV; B – I; C – V; D – II; E – IV Salivary Stimulates Inhibits secretion
(d) A – V; B – IV; C – III; D – I; E – II (a)
glands secretion
83. Which of the following terms is not correctly matched
(b) Pupil of eye Dilates Constricts
with its feature?
(a) Efferent neurons - Carries signals from CNS to the (c) Heart rate Decreases Increases
effector. Intestinal Stimulates Inhibits secretion
(d)
(b) Axon terminal - Possess neurotransmitter containing peristalsis secretion
vesicle. 87. In the given diagram which stage of conduction of nerve
(c) Limbic system - Along with the hypothalamus, it is impulse through nerve fibre is observed?
involved in the regulation of sexual behaviour,
expression of emotional reactions and motivation.
(d) Association areas - Present in cerebellum and
responsible for functions like in tersen sory
associations, memory and communication. (a) Polarization (b) Resting potential
84. Identify the correct match of types of neurons with its (c) Repolarization (d) Depolarization
location.
88. Given below is a diagrammatic cross section of a single loop
(i) Unipolar neuron - Embryonic stage
of human cochlea with few part labelled as A, B, C & D.
(ii) Bipolar neuron - Cornea of eyes
(iii) Multipolar neuron - Cerebral cortex
(a) (i) & (ii) only (b) (ii) & (iii) only
(c) (i) & (iii) only (d) All the three.
B
DIAGRAM TYPE QUESTIONS
85. The following diagram represent the reflex arc. Identify C
the parts labelled as A, B, C, D, E, F and G and choose the
D
correct option

A C
B
Which one of the following options correctly represents
the name of three different parts?
(a) A: Perilymph, B: Tectorial membrane C: Endolymph
(b) B: Tectorial membrane, C:Perilymph, D: Secretory cells
F E (c) C: Endolymph, D: Sensory hair cells, A: Serum
G D
(d) D: Sensory hair cells, A: Endolymph B: Tectorial
(a) A - sense organ; B - sensory nerve; C - dorsal horn; membrane
D - interneuron; E - ventral horn; F - motor nerve ;
G - effector 89. The given diagram chows the axon terminal and synapse
with few part labelled as A, B, C & D. Choose the correct
(b) A - sense organ; B - sensory nerve; C - ventral horn;
D - interneuron; E - dorsal horn; F - motor nerve; combination of labelling from the given options.
G - effector
Ac

Ca2+
t

(c) A - effector; B - motor nerve; C - dorsal horn;


ion

B
po
ten

D - interneuron; E - ventral horn; F - sensory nerve; Mitochondrion


tia

D
l

G - effector Axon of
presynaptic
neuron
(d) A - effector; B - motor nerve; C - ventral horn; A
D - interneuron; E - dorsal horn; F - sensory nerve;
G - sense organ.
86. Given below is a table comparing the effects of C
sympathetic and parasympathetic nervous system for four
ion channel
features (a – d). Which of the following feature is correctly (closed) ion channel (open)

described?
Neural Control and Coordination 185

(a) A- Synaptic vesicle, B- Axon terminal, C- Synaptic 92. What is the function of label X in the given diagram?
cleft, D- Postsynaptic membrane
(b) A- Axon terminal, B- Synaptic vesicle, C-
X
Postsynaptic membrane, D- Synaptic cleft.
(c) A- Synaptic vesicle, B- Synaptic cleft, C- Axon
terminal, D- Post synaptic membrane
(d) A- Post synaptic membrane, B- Axon terminal,
C- Synaptic vesicle, D- Synaptic cleft
90. The given diagram shows different parts of a human eye
with one part labeled as X. (a) It speeds up the impulse transmission.
(b) It provides electrical insulation.
(c) It conducts impulse towards the nerve cell body.
(d) It is the functional unit of nerve.
93. Refer the given figure of ear with few structure marked as
I, II, III & IV. Which labelled structure converts sound
waves into mechanical vibrations?
X

III
I

Which of the following statement is correct regarding label II


X? IV

(a) It is the opening in lens that permits light into the (a) I (b) II
inner chambers of the eye. (c) III (d) IV
(b) It is the coloured portion of vascular tunic. 94. Identify the correct options for the given blanks.
(c) It is a biconcave structure that changes shape to
Part of the ear Functions
bring objects into focus.
(d) It is thick, jelly-like substance in the posterior _____A_____ Contains receptors for balance
compartment of the eye. _____B_____ Increases the magnitude of
91. Which of the following options correctly represents the vibrations
name of 1, 2, 3 and 4 is the given diagram of neuron? _____C_____ Collects sound waves
1
(a) A- Semi-circular canals, B- Ear ossicles, C- Pinna
(b) A- Cochlea, B- Semi-circular canals, C- Tympanum
2 4 (c) A- Semi-circular canals, B- Cochlea, C- Tympanum
(d) A- Cochlea, B- Pinna, C- Ear ossicles
95. Which of the following statements is/are functions of
structure labelled as ‘X’ in the given diagram of eye?
3

(a) 1- Axon, 2- Dendrites, 3- Node of Ranvier, 4- Myelin


sheath
(b) 1- Dendrites, 2- Axon, 3- Node of Ranvier, 4- Myelin
sheath
(c) 1- Dendrites, 2- Cell body, 3- Myelin sheath, 4- Node
of Ranvier
(d) 1- Axon, 2- Cell body, 3- Dendrites, 4- Node of Ranvier

X
EBD_7209
186 Biology
I. It provides attachment points for muscles that move 98. Identify the name and the labelled part which is a naked
the eye. portion of myelinated axon and facilitate the rapid
II. It maintains the shape of the eye ball. conduction of nerve impulses.
(a) Dendrites, 1 (b) Schwan cell, 2
III. It helps during accomodation. (c) Synaptic knob, 3 (d) Nodes of Ranvier, 4
IV. It is responsible for eye colour. 99. Identify the part along which the sequence of impulse
(a) I and II (b) I, II and IV generation is repeated.
(a) 1 (b) 2
(c) II, III and IV (d) All of these
(c) 3 (d) 4
96. The given diagram represents the sectional view of 100. Refer the given figure which shows the axon terminal and
cochlea with few part labelled as I, II, III & IV.
synapse with their parts marked as 1 to 7. Identify the
III
correct parts whose constitution forms the structure of
synapse.
I

II

IV
The movement of which marked label causes hair cell
microvilli to bend back and forth.
(a) I (b) II (a) 1, 2, 3 (b) 3, 4, 5
(c) III (d) IV (c) 4, 5, 6 (d) 5, 6, 7
97. The primary function of the structure labelled as X in the 101. Refer the given figure of eye in which few parts are labelled
given figure is
as 1, 2, 3 and 4. Select the option which shows the correct
X identification of the part with its characteristics.

(a) movement of head (b) position of head


(c) hearing (d) vision
Direction (Qs. 98 and 99): Refer the given figure of neuron structure
with few parts labelled as 1, 2, 3, and 4 and answer the questions.

(a) 1 : Choroid, it contains ganglion cells, bipolar cells


and photoreceptor cells.
(b) 2 : Iris, it is responsible for controlling the diameter
and size of the pupil and thus the amount of
light reaching the retina.
(c) 3 : Blind spot, it is a yellowish pigmented spot called
macula lutea with a central pit called the fovea.
(d) 4 : Cornea, it is a transparent front part of the eye
that covers the iris, pupil, and anterior chamber.
102. "X" is spiral shaped structure consisting of hair cells that
serve as receptors for auditory stimuli.
Identify "X" and its label & location (marked as 1, 2, 3 and
4) from the given diagrammatic representation of the
sectional view of cochlea.
Neural Control and Coordination 187

111. Sequence of meninges from inner to outside is


(a) Duramater – Arachnoid – Piamater
(b) Duramater – Piamater – Arachnoid
(c) Arachnoid – Duramater - Piamater
(d) Piamater- Arachnoid - Duramater
112. Which of these is an example of conditioned reflex?

(a) X- Organ of corti, 2, 3. (a) Watering of mouth at the taste of food.


(b) X- Eustachian tube, 1, 2. (b) Withdrawal of hand on touching a hot plate.
(c) X- Semicircular canal, 3, 4. (c) Cycling.
(d) X- Crista ampullaris, 1, 4. (d) Flowing of tears while peeling and cutting onions.
113. You are watching a horror movie and you notice your
CRITICAL THINKING TYPE QUESTIONS heart is beating fast and mouth is dry. It is because of
(a) fight and flight response
103. For sound (good) reflex actions we require intact (b) sympathetic nervous system
(a) spinal cord (b) cerebellum (c) parasympathetic nervous system
(c) hypothalamus (d) medulla oblongata (d) both (a) and (b)
104. In which animal nerve cell is present but brain is absent? 114. During the transmission of nerve impulse through a nerve
(a) Sponge (b) Earthworm fibre , the potential on the inner side of the plasma
(c) Cockroach (d) Hydra
membrane has which type of electric charge?
105. Which one of the following is an example of a simple
reflex ? (a) First positive, then negative and again back to
(a) Closing of eyes if an object suddenly approaches positive.
them. (b) First negative, then positive and again back to
(b) Climbing stairs in dark. negative.
(c) Watering of mouth at the sight of delicious food. (c) First positive, then negative and continue to be
(d) Tying laces while talking to and looking at another negative.
person. (d) First negative, then positive and continue to be
106. The main cause of paralysis is positive.
(a) some defect in muscles
115. A person entering an empty room suddenly finds a snake
(b) complete destruction of motor nerves
right in front on opening the door. Which one of the
(c) complete destruction of sensory nerves
(d) none of the above following is likely to happen in his neuro-hormonal control
107. An example of autonomous nervous system is system?
(a) swallowing food (a) Hypothalamus activates the parasympathetic
(b) pupillary reflex division of brain.
(c) peristalsis of intestine (b) Sympathetic nervous system is activated releasing
(d) knee-jerk response epinephrine and norepinephrine from adrenal cortex.
108. In a nerve if sodium pump is blocked, which of the following (c) Sympathetic nervous system is activated releasing
is most likely to happen ? epinephrine and norepinephrine from adrenal
(a) Na+ and K+ will increases outside the cell. medulla.
(b) Na+ outside the nerve will increase.
(d) Neurotransmitters diffuse rapidly across the cleft and
(c) Na+ inside the nerve will increase.
(d) K+ inside the nerve will increase. transmit a nerve impulse.
109. A person feels no sensation when he puts his hand over 116. Identify the correct order in which vibrations travel
flame. The part of the brain which has damaged is through the auditory ossicles?
(a) cerebellum (b) medulla oblongata (a) Stapes-Malleus-Incus
(c) diencephalon (d) hypothalamus (b) Malleus-Incus-Stapes
110. An axon has four terminal ends connected with dendrites (c) Stapes-Incus-Malleus
of four different neurons. Its nerve impulse will.
(d) Incus-Stapes-Malleus
(a) become weak due to distribution into four.
(b) travel in all the four neurons with equal strength. 117. The part of an eye which acts like diaphragm of a
(c) pass on to one neuron only. photographic camera, is
(d) travel to none because the movement of impulse is (a) pupil (b) iris
from dendrite to axon. (c) lens (d) cornea
EBD_7209
188 Biology
118. Cornea transplantation is successful as cornea is 126. Trace the correct sequence for pathway of light through
(a) easily available (b) without blood supply the eye to the retina?
(c) easily preserved (d) easily stitched (a) Conjunctiva - Cornea - Aqueous humour - Pupil -
119. A person is wearing spectacles with concave lenses for Lens - Vitreous humour - Yellow spot.
correcting vision. While not using the glasses, the image (b) Cornea- Conjunctiva- Vitreous humour- Lens- Pupil
of a distant object in his case will be formed - Aqueous humour - Yellow spot
(a) on the blind spot. (b) behind the retina. (c) Conjunctiva- Cornea - Vitreous humour - Lens - Pupil
(c) on the yellow spot.(d) in front of the retina. - Aqueous humour - Yellow spot
120. In a similarity with photographic camera the retina acts as (d) Cornea - Conjunctiva - Aqueous humour - Pupil- Lens
- Vitreous humour - Yellow spot
(a) shutter (b) lens
127. Refer the following features and identify the correct part
(c) diaphragm (d) film
of the ear to which these are associated.
121. When we move from light to dark, we fail to see for some i. It is also called auditory tube.
time but soon the visibility becomes normal. It is called
ii. It connects the middle ear cavity with the pharynx.
iii. It helps in equalizing the pressures on either sides of
(a) accomodation (b) adaptation the ear drum.
(c) photoperiodism (d) mutation (a) Ear ossicles (b) Eustachian tube
122. Layers in wall of eye balls from inside outwards are (c) Semicircular canal (d) Vestibular apparatus
(a) retina, choroid, sclerotic 128. ‘X’ is an important part of ‘Y’ which lies at the base of the
(b) sclerotic, choroid, retina structure which is a major coordinating centre for sensory
(c) choroid, retina, sclerotic and motor signalling. It contains a number of centre which
(d) choroid, sclerotic, retina control body temperature, urge for eating and drinking.
Identify X and Y from the options given below
123. Arrange the cardiac muscle fibres, skeletal muscle fibres
(a) X -Cerebellum ; Y - Hindbrain
and the nerve fibres on the basis of velocity of impulse
conduction in descending order, i.e. from maximum to the (b) X -Hypothalamus ; Y - Forebrain
minimum. (c) X -Corpora quadrigemina ; Y - Midbrain
(a) Nerve fibres - cardiac muscle fibres - skeletal muscle (d) X -Pituitary gland ; Y - Forebrain
fibres 129. Impulse transmission across an electrical synapse is
always ________than that across a chemical synapse.
(b) Nerve fibres- skeletal muscle fibres - cardiac muscle
fibres (a) faster (b) slower
(c) intermittent (d) continuous
(c) Skeletal muscle fibres- cardiac muscle fibres - nerve
fibres 130. Unmyelinated nerve fibre is enclosed by an ‘X’ that does
not form a myelin sheath around the ‘Y’, and is commonly
(d) Cardiac muscle fibres- skeletal muscle fibres - nerve
found in ‘Z’ and the somatic neural systems. Identify ‘X’,
fibres
‘Y’ and ‘Z’.
124. Which part of internal ear recognizes the different
frequencies of sound ? X Y C
(a) Tectorial membrane (a) Schwann cells Axon Autonomous
(b) Basilar membrane (b) Nodes of Ranvier Cell body Synaptic knob
(c) Tympanic membrane (c) Synapse Dendrites Sympathetic
(d) Reissner’s membrane (d) Meninges Nerve impulse Peripheral
125. The correct logical sequence regarding flow of sound in
131. The ____ neuron receives signal from a sensory organ
human ear is
and transmits the impulse via a dorsal nerve root into the
(a) Ear ossicles- Oval window - Tympanum - Auditory _____. The ______ neuron then carries signals from CNS
canal - Cochlea to the ________.
(b) Auditory canal - Tympanum- Ear ossicles - Oval (a) afferent, CNS, efferent, effector
window - Cochlea (b) efferent, PNS, afferent, effector
(c) Cochlea- Tympanum- Ear ossicles - Oval window - (c) afferent, CNS, efferent, receptor
Auditory canal (d) efferent, PNS, afferent, receptor
(d) Tympanum - Cochlea - Oval window - Ear ossicles -
Auditory canal
Chemical Coordination
22

Chapter
and Integration

FACT/DEFINITION TYPE QUESTIONS 11. Anti-ageing hormone is


(a) thyroxine (b) melatonin
1. Anterior lobe of pituitary secretes (c) estrogen (d) testosterone
(a) ACTH, TSH and oxytocin 12. Sleep-wake cycle and menstrual cycle are maintained by
(b) STH, GH and ADH (a) progesterone (b) melatonin
(c) TSH, ADH and prolactin (c) oxytocin (d) MSH
(d) FSH, GH and LH 13. When amount of ADH decreases in blood, micturition
2. A gorilla like appearance with huge hands and legs is due (a) increases
to abnormal secretion of (b) decreases
(a) FSH (b) LH (c) remains unaffected
(c) LTH (d) GH (d) increases and then decreases
3. ACTH is secreted by 14. Which hormone helps in reabsorption of water from
(a) adrenal cortex (b) pituitary kidney?
(c) adrenal medulla (d) thyroid (a) ADH (b) STH
4. Which of the following hormone is required for the (c) ACTH (d) TTH
maintenance of corpus luteum ? 15. Which of the following occurs due to ADH deficiency?
(a) Progesterone (b) Estrogen (a) Increase urine output.
(c) FSH (d) LH (b) Increased water in urine.
5. Oxytocin hormone is produced by (c) Less urine.
(a) pituitary (b) adrenals (d) No urination.
(c) hypothalamus (d) thyroid 16. Which of the following hormone acts upon the renal tubule
6. Ovulation in humans is controlled by and blood capillaries ?
(a) FSH and LTH (b) FSH and G.H (a) Glucagon (b) Aldosterone
(c) LTH and LH (d) FSH and LH (c) Vasopressin (d) Glucocorticoids
7. Ovulation and formation of corpus luteum is controlled 17. Which endocrine gland stores its secretion in extracellular
by spaces before discharging it into blood?
(a) ICSH (a) Testis (b) Pancreas
(b) FSH (c) Thyroid (d) Adrenal
(c) thyroxine hormone 18. Which of the following hormone opposes parathormone ?
(d) luteinizing hormone (a) ADH (b) STH
8. MSH is secreted by (c) Thyroxine (d) Thyrocalcitonin
(a) pars intermedia (b) pars tuberalis 19. Abnormal secretion of thyroxine produces
(c) pars distalis (d) pars nervosa (a) acromegaly (b) Addison’s disease
9. Secretion of estrogen is controlled by (c) cretinism (d) goitre
(a) hCG (b) FSH 20. Thyroxine acts on every organ of the body, except
(c) progesterone (d) testosterone (a) adult brain (b) testis
10. Deficiency of vasopressin primarily results in (c) thyroid itself (d) bones
(a) increased volume of urine. 21. Which of the following is (are) not influenced by
(b) decreased volume of urine. parathyroid hormone ?
(c) excessive secretion of urochrome. (a) Kidney (b) Bone
(d) change in pH from acidic to alkaline range. (c) Small intestine (d) None of the above
EBD_7209
190 Biology
22. Which of the following gland is often referred in 34. Hormone which helps in implantation of embryo in uterus is
connection with AIDS? (a) estrogen (b) oxytocin
(a) Thymus (b) Thyroid (c) relaxin (d) progesterone
(c) Adrenal (d) Pancreas 35. Secretion of which of the following structure is
23. Thymus in mammals is mainly concerned with responsible in preparing the inner wall of uterus for
(a) regulation of body growth. implantation ?
(b) secretion of thyrotropin. (a) Ovary (b) Pituitary gland
(c) Corpus luteum (d) Ovarian follicle
(c) regulation of body temperature.
36. Which of the following is a gastro-intestinal hormone?
(d) immunological functions.
(a) Cholinesterase (b) Enterokinase
24. Which one of the following disease is caused by the under (c) Secretin (d) Interocrinin
secretion of cortisol hormone? 37. The hormone secretin is produced in
(a) Anaemia (b) Addison’s disease (a) pancreas and influences conversion of glycogen into
(c) Hyperglycemia (d) Mental retardation glucose.
25. Which hormone possesses anti-insulin effect? (b) small intestine and stimulates pancreas.
(a) Cortisol (b) Calcitonin (c) adrenal glands and accelerates heartbeat.
(c) Oxytocin (d) Aldosterone (d) testes and produces male secondary sexual
26. Triple ‘F’ gland for flight, fright and fight is characters.
(a) thyroid (b) thymus 38. Which hormone interacts with membrane bound receptor
(c) pituitary (d) adrenal and does not normally enter the target cell?
27. Glycogenesis is the result of the hormone secreted from (a) FSH (b) Estrogen
(a) alpha cells of pancreas. (c) Thyroxine (d) Cortisol
(b) beta cells of pancreas. 39. Which one of the following is not a second messenger in
hormone action ?
(c) thyroid gland.
(a) Calcium (b) Sodium
(d) adrenal gland.
(c) cAMP (d) cGMP
28. Diabetes insipidus is caused by hyposecretion of
40. Receptors for protein hormones are found
(a) insulin (b) vasopressin (a) inside nucleus (b) inside cytoplasm
(c) oxytocin (d) thymosin (c) on surface of ER (d) on cell surface
29. Blood glucose level in man is regulated by 41. Which one of the following pairs of hormones are the
(a) insulin only examples of those that can easily pass through the cell
(b) adrenaline membrane of the target cell and bind to a receptor inside
(c) glucagon and insulin it (mostly in the nucleus) ?
(d) all of the above (a) Somatostatin, Oxytocin
30. Source of somatostatin is the same as that of (b) Cortisol, Testosterone
(a) insulin and glucagon (c) Insulin, Glucagon
(b) vasopressin and oxytocin (d) Thyroxine, Insulin
(c) thyroxine and calcitonin 42. Steroid hormones are produced only by the
(d) somatotropin and prolactin (a) adrenal medulla and pancreas.
31. In human, testosterone is produced by (b) thyroid gland and pancreas.
(a) tunica albuginea (c) anterior and posterior pituitary.
(b) leydig cell (d) sex organs and adrenal cortex.
(c) seminiferous tubule
(d) sertoli cell STATEMENT TYPE QUESTIONS
32. The hormone that supports pregnancy and stimulates 43. Mark the correct statement regarding somatostatin.
mammary glands for the formation of alveoli for storing (a) It is secreted from anterior pituitary.
milk, is secreted from (b) It inhibits the release of growth hormone.
(a) anterior Pituitary (b) posterior pituitary (c) It is secreted from posterior pituitary.
(c) graafian follicle (d) corpus luteum (d) It stimulates STH secretion.
33. During pregnancy corpus luteum 44. What of the following is correct about calcitonin ?
(a) degenerates. (a) It contains iodine.
(b) changes to corpus albicans. (b) It is an amino acid.
(c) persists until parturition. (c) It is released from parathyroid.
(d) persists upto the middle of pregnancy. (d) It is released from thyroid gland.
Chemical Coordination and Integration 191

45. Which statement regarding PTH is correct? 54. Which one of the following statement is correct?
(a) It is a peptide hormone. (a) Endrocrine glands regulate neural activity and
(b) It stimulates bone resorption. nervous system regulates endocrine glands.
(c) It is hypercalcemic hormone. (b) Neither hormones control neural activity nor the
(d) All of the above. neurons control endocrine activity.
46. Which of the following statements is correct in relation to (c) Endocrine glands regulate neural activity, but not
the endocrine system? vice versa.
(a) Organs in the body like gastrointestinal tract, heart, (d) Neurons regulate endocrine activity, but not vice
kidney and liver do not produce any hormones. versa.
(b) Non-nutrient chemicals produced by the body in 55. Which of the following statement about the hormone
trace amount that act as intercellular messenger are action in humans is correct ?
known as hormones. (a) In females, FSH first binds with specific receptors
(c) Releasing and inhibitory hormones are produced by on ovarian cell membrane.
the pituitary gland . (b) FSH stimulates the secretion of estrogen and
(d) Adenohypophysis is under direct neural regulation progesterone.
of the hypothalamus. (c) Glucagon is secreted by b-cells of Islets of
47. Which of the following is correct for thyrocalcitonin ? langerhans and stimulates glycogenolysis.
(a) Produced by parathyroid and decreases Ca++ in ECF. (d) Secretion of thymosin is stimulated with ageing.
(b) Produced by thyroid and decreases Ca++ in ECF. 56. Which of the followings is the more scientific definition
(c) Produced by parathyroid and increases Ca ++ in ECF. of hormone?
(d) Produced by thyroid and increases Ca++ in ECF. (a) They are extracellular messengers.
48. One similarity between enzymes and hormones is that (b) They always act at distantly located target organ.
(a) both are proteins. (c) They are the products of well organized endocrine
(b) both can be used again and again. glands.
(c) both are used in minute amount. (d) They are non-nutrient chemicals that act as
(d) both act at a particular pH.
intercellular messengers.
49. Hormones of pituitary gland are
(a) some steroids and some proteins ASSERTION/REASON TYPE QUESTIONS
(b) all proteins/peptides.
(c) complex substances formed from proteins, steroids, In the following questions, a statement of Assertion is followed
carbohydrates. by a statement of Reason.
(d) all steroids. (a) If both Assertion and Reason are true and the Reason is
50. Norepinephrine the correct explanation of the Assertion.
(i) is released by sympathetic fibres.
(b) If both Assertion and Reason are true but the Reason is
(ii) is released by parasympathetic fibres.
(iii) increases the heart rate. not the correct explanation of the Assertion.
(iv) decreases blood pressure. (c) If Assertion is true but Reason is false.
Which of the above said statements are correct? (d) If both Assertion and Reason are false.
(a) (i) and (ii) (b) (i) and (iii) 57. Assertion : Mammary glands are apocrine glands.
(c) (ii) and (iii) (d) (ii) and (iv) Reason : The distal part containing secretory granules
51. Choose the correct statement about ‘neurohypophysis’ ? break down and leaves as a secretion.
(a) It stores and release hormones secreted by 58. Assertion : Hormone calcitonin has an antagonistic effect
hypothalamus. to that of parathormone.
(b) It secretes its own hormones.
Reason : Calcitonin decreases blood calcium level while
(c) It is poorly developed and functionless in humans.
(d) It stores th e hormones produced by parathormone increases blood calcium level.
adenohypophysis. 59. Assertion : The person with diabetes insipidus feels
52. Which of the following statement is correct regarding thirsty.
glucagon hormone ? Reason : A person with diabetes insipidus suffers from
(a) It has opposite effect to that of insulin. excess secretion of vasopressin.
(b) It converts glucose to glycogen. 60. Assertion: Failure of secretion of hormone vasopressin
(c) It is given to diabetic patients. causes diabetes mellitus in the patient.
(d) It is formed by b-cells of pancreas. Reason: Vasopressin increases the volume of urine by
53. A woman may develop beard and moustaches due to
increasing the reabsorption of water from the urine.
(a) hypersecretion of adrenal cortex.
(b) hypersecretion of thyroxine. 61. Assertion : Histamine is involved in allergic and
(c) hyposecretion of adrenaline. inflammatory reactions.
(d) hyposecretion of thyroxine. Reason : Histamine is a vasodilator.
EBD_7209
192 Biology
(a) A – V, B – III, C – IV, D – II
MATCHING TYPE QUESTIONS
(b) A – V, B – III, C – II, D – IV
62. Select the correct match of a hormone with its source and (c) A – I, B – II, C – IV, D – III
function. (d) A – III, B – V, C – IV, D – II.
Hormone Source Function 66. Match the hormone given in column-I with their function
in given in column-II.
(a) Vasopressin Posterior Increases loss of
Column-I Column-II
pituitary water through urine.
A. FSH I. Prepare endometrium for
(b) Nor-epinephrine Adrenal Increases heart beat,
implantation
medulla rate of respiration
B. LH II. Develops female secondary
and alertness.
sexual characters
(c) Glucagon Beta-cells Stimulates
C. Progesterone III. Contraction of uterine wall
of Islets of glycogenolysis.
langerhans D. Estrogen IV. Development of corpus
luteum
(d) Prolactin Posterior Regulates growth of
V. Maturation of Graafian follicle
Pituitary mammary glands
(a) A – V, B – IV, C – I, D – II
and milk formation
(b) A – IV, B – V, C – II, D – I
in females.
(c) A – IV, B – III, C – II, D – V
63. Which one of the following four glands is correctly
(d) A – V, B – I, C – II, D – IV
matched with the accompanying description?
67. Which one of the following pairs is incorrectly matched?
(a) Insulin–Diabetes mellitus (disease)
(a) Thyroid – Hyperactivity in young children causes (b) Glucagon–Beta cells (source)
cretinism. (c) Somatostatin–Delta cells (source)
(b) Thymus – Starts undergoing atrophy after puberty. (d) Corpus luteum–Relaxin (secretion)
(c) Parathyroid – Secretes parathormone which 68. Match the endocrine gland as a source with its respective
promotes movement of calcium ions from blood into hormone as well as the function.
bones during calcification. Source gland Hormone Function
(d) Pancreas – Delta cells of Islets of Langerhans secrete (a) Thyroid Thyroxine Regulates blood
a hormone which stimulates glycolysis in liver.
calcium level
64. Match the hormones given in column-I with the terms
(b) Anterior Oxytocin Contraction of uterine
given in column-II
pituitary muscles during
Column -I Column-II child birth
A. ADH I. Pituitary (c) Posterior Vasopressin Stimulates resorption
B. ACTH II. Mineralocorticoid pituitary of water in the distal
C. Aldosterone III. Diabetes mellitus tubules in the
D. Insulin IV. Diabetes insipidus nephron
(d) Corpus luteum Estrogen Supports pregnancy
E. Adrenaline V. Vasodilator 69. Which of the following pair of hormone and their disorder
(a) A – I, B – IV, C – II, D – III, E – V is correctly matched ?
(b) A – IV, B – II, C – I, D – III, E – V (a) Parathormone – Cretinism
(c) A – IV, B – I, C – II, D – III, E – V (b) Thyroxine – Ricket
(d) A – IV, B – I, C – III, D – II, E – V (c) Insulin – Diabetes insipidus
65. Column-I lists the endocrine structure and column-II lists (d) Cortisol – Cushing’s disease
the corresponding hormones. Match the two columns 70. Mark the hormone given in column I and the endocrine
and identify the correct option given below. cells given in column II.
Column-I Column-II
Column-I Column-II A. a-cell I. Inhibin
A. Hypothalamus I. Relaxin B. b-cell II. Glucagon
B. Anterior pituitary II. Estrogen C. Leydig cell III. Insulin
C. Testis III. FSH and LH D. Sertoli cells IV. Testosterone
D. Ovary IV. Testosterone (a) A – I, B – III, C – IV, D – II
V. Gonadotropin releasing (b) A – III, B – II, C – IV, D – I
(c) A – I, B – III, C – II, D – IV
hormone
(d) A – II, B – III, C – IV, D – I
Chemical Coordination and Integration 193

71. Match the following hormones given in column I with Which of the following gland is correctly matched with
their appropriate category of hormones given in column II. their secretions?
Column-I Column-II Hormones Their secretions
(Hormones) (Categories) A I Melatonin
A. FSH and LH I. Glucocorticoids B II Thymosin
B. Cortisol II. Mineralocorticoids C III Epinephrine
C. Androgen III. Gonadotropins D IV Aldosterone
D. Aldosterone IV. Gonadocorticoids E V Testosterone
(a) A – III, B – I, C – IV, D – II (a) I, II and III only (b) I, IV and V only
(b) A – I, B – II, C – III, D – IV (c) II, IV, and V only (d) II, III and V only
(c) A – II, B – III, C – IV, D – I 74. The hormone released by label "X" in the given figure
(d) A – IV, B – II, C – I, D – III helps to restore Y. Identify X and Y.
DIAGRAM TYPE QUESTIONS
72. The given figure shows the hormonal control of female
reproductive system in which few steps are marked as A,
B, C and D. Identify the correct labelling.
X

X Y
(a) Thyroxine Too much calcium in the blood.
(b) PTH Lowered levels of calcium in blood.
(c) Thymosin Decreased level of blood sugar.
(d) Adrenaline Excessive loss of sodium in extracellular
fluid.
75. The label X represents ___i_____ and the hormone
released by it is _____ii______. Identify (i) and (ii)
X

(a) A- GnRH, B - TSH, C - LTH, D - Uterus


(b) A - GnRH, B - FSH/LH, C - Estrogen or progesterone,
D - urerus
(c) A - GnRH, B - STH, C - LH, D - Uterus
(d) A - GnRH, B - ACTH, C - LH, D - Uterus Y
73. The given diagram represents the location of human (a) i- Adrenal cortex, ii- Epinephrine
endocrine glands I, II, III, IV and V. (b) i- Adrenal cortex, ii- Aldosterone
(c) i- Adrenal medulla, ii- Epinephrine
I (d) i- Adrenal medulla, ii- Aldosterone
76. Which of the following disease is caused due to over
secretion of the structure marked as X?
II

III

X
IV

(a) Gigantism
(b) Diabetes mellitus
V (c) Diabetes insipidus
(d) Grave's disease
EBD_7209
194 Biology
77. The Hypophyseal portal system transports releasing and 80. Identify the hormone represented by lines 1 and 2?
inhibiting hormones from the hypothalamus into which (a) 1- ACTH, 2- Aldosterone
of the following parts marked in I, II, III and IV. (b) 1- ACTH, 2- Adrenaline
I (c) 1- TSH, 2- Thyroxine
(d) 1- TSH, 2- Aldosterone

CRITICAL THINKING TYPE QUESTIONS


81. In hormone action, if receptor molecules are removed from
target organ, the target organ will
(a) continue to respond to hormone.
IV (b) not respond to hormone.
(c) continue to respond but requires higher
concentration.
III (d) continue to respond but in the opposite way.
82. The number of hormones secreted by anterior pituitary is
II (a) 3 (b) 4
(a) I (b) II
(c) 6 (d) 8
(c) III (d) IV
78. Which endocrine gland secretes the hormone that causes
the abnormalities shown in the below picture? 83. Which hormone is related to mineral metabolism but is
not a peptide / protein in nature ?
(a) PTH (b) ANF
(c) Aldosterone (d) All of the above
84. Pancreatic duct of a healthy dog is blocked. Which of the
following function of pancreas will not be affected ?
(a) Maintenance of normal blood sugar level.
(b) Carbohydrate digestion.
(c) Protein digestion.
(d) Neutralization of chyme.
85. A decrease in the level of estrogen and progesterone
causes
(a) Hypothalamus (a) growth and dilation of myometrium.
(b) Pancreas (b) growth of endometrium.
(c) Adenohypophysis (c) constriction of uterine blood vessels leading to
(d) Neurohypophysis sloughing of endometrium or uterine epithelium.
Direction (For Qs. 79 and 80) : (d) release of ovum from the ovary.
The endocrine glands A and B in the given figure represent 86. Which of the following endocrine glands grows to the
___ (i) ____ and ____ (ii) ____ respectively. maximum size at puberty and then diminishes gradually?
(a) Thymus (b) Pituitary
(c) Thyroid (d) Adrenal
87. Which one is different from the category of other three?
(a) Gastrin (b) Ptyalin
(c) Secretin (d) Glucagon
88. Which of the following disorders are caused by
hypersecretion of their concerned hormones ?
(a) gigantism and exophthalmic goitre
(b) tetany and myxoedema
(c) diabetes mellitus and goitre
(d) rickets and diabetes insipidus
79. Identify (i) and (ii)
89. Hormones produced by anterior lobe of pituitary
(a) i- Pituitary gland , ii- Adrenal gland
(a) control calcium level in blood.
(b) i- Hypothalamus, ii- Pituitary gland
(b) stimulate thyroid and other endocrine glands.
(c) i- Hypothalamus, ii- Adrenal gland
(c) initiate alarm reaction.
(d) i- Pituitary gland, ii- Hypothalamus
(d) regulate water balance in body.
Chemical Coordination and Integration 195

90. Growth hormone of pituitary is more effective in 101. Identify a hormone which is produced by the pituitary
(a) presence of thyroxine. gland in both males and females but functional only in
(b) absence of thyroxine. females.
(c) absence of insulin. (a) Vasopressin (b) Relaxin
(d) presence of adrenaline. (c) Prolactin (d) Somatotropic hormone
102. Low level of progesterone and estrogen in blood stimulate
91. Which of the following hormone helps a person who
(a) FSH-RH production
suffers from a marked fall in blood pressure ? (b) LH production
(a) Insulin (b) Thyroxine (c) GH production
(c) GH (d) Adrenaline (d) All of the above
92. Hormones involved in carbohydrate metabolism are 103. Some glands produce hormones that stimulate other
(a) insulin, glucagon, epinephrine and calcitonin endocrine glands. Which of the following hormones
(b) insulin, glucagon, epinephrine and glucocorticoids specifically acts to trigger secretion of hormones by
(c) insulin, glucagon, cortisol and melatonin another endocrine gland?
(d) insulin, glucagon, norepinephrine and melatonin (a) Thyroxine
93. Which of the following hormones are identical? (b) Progesterone
(a) ACTH and adrenaline (c) Adrenocorticotropic hormone (ACTH)
(d) Antidiuretic hormone (ADH)
(b) hCG and progesterone
104. Which hormone causes dilation of blood vessels,
(c) Calcitonin and Oxytocin
increased oxygen consumption and glucogenesis?
(d) Vasopressin and ADH. (a) Insulin (b) Adrenaline
94. A man suffering from diabetes mellitus drinks water more (c) Glucagon (d) ACTH
frequently as he has to eliminate from blood, the extra 105. In the homeostatic control of blood sugar level, which
(a) salts (b) sugar organs function respectively as modulator and effector?
(c) insulin (d) protein (a) Liver and islets of langerhans
95. Which hormone promotes cell division, protein synthesis (b) Hypothalamus and liver
and bone growth? (c) Hypothalamus and islets of langerhans
(a) GH (b) ADH (d) Islets of langerhans and hypothalamus
(c) ACTH (d) PTH 106. ADH, responsible for reabsorption of water and reduction
96. Injection of glucagon will of urine secretion, is synthesized by
(a) posterior pituitary gland
(a) cause goitre
(b) juxtaglomerular apparatus
(b) cause galactosemia (c) hypothalamus
(c) cause hypoglycemia (d) anterior pituitary gland
(d) increase blood sugar level 107. The hormones that initiate ejection of milk, stimulates milk
97. A person who has protruding eyes, tachycardia and production and growth of ovarian follicles, are respectively
higher body temperature is suffering from known as
(a) cretinism (b) hyperthyroidism (a) PRL, OT and LH (b) OT, PRL and FSH
(c) myxoedema (d) acromegaly (c) LH, PRL and FSH (d) PRH, OT and LH
98. Point out the odd one: 108. Hypothyroidism in adults and hyperparathyroidism in
(a) Corticotropin (b) Vasopressin children will respectively lead to
(c) Noradrenaline (d) Prolactin (a) myxoedema and cretinism
(b) Grave’s disease and Hashimoto’s disease
99. When a boy goes through puberty, the steroid hormone
(c) myxoedema and osteitis fibrosa cystica
testosterone puts hair on his chest by
(d) Addison’a disease and cretinism
(a) interacting with DNA in the nuclei of cells. 109. Feeling the tremors of an earthquake a scared resident of
(b) causing cells to change shape. seventh floor of a multistoryed building starts climbing
(c) altering the permeability of plasma membranes. down the stairs rapidly. Which hormone initiated this
(d) triggering nerve impulses in cells. action?
100. A patient of diabetes mellitus excretes glucose in urine (a) Gastrin (b) Thyroxine
even when he is kept in a carbohydrate free diet. It is (c) Adrenaline (d) Glucagon
because 110. A person is having problems with calcium and phosphorus
(a) fats are catabolised to form glucose. metabolism in his body. Which one of the following glands
(b) amino acids are catabolised in liver. may not be functioning properly?
(c) amino acids are discharged in blood stream from liver. (a) Thyroid (b) Parathyroid
(d) glycogen from muscles are released in the blood (c) Parotid (d) Pancreas
stream.
EBD_7209
196 Biology
111. A pregnant female deliver a baby who suffers from stunted (c) I - Thyroid, II - Cretinism, III - Adrenaline, IV -
growth, mental retardation/low intelligence quotient and Estrogen
abnormal skin. This is the result of (d) I - Pituitary, II - Myxodema, III - Thyroxine,
(a) low secretion of growth hormone IV - Progesterone
(b) cancer of the thyroid gland 118. Tim once suffered a severe allergic reaction to a bee sting.
(c) over secretion of pars distalis The sting caused him to suffer a near-fatal drop in blood
(d) deficiency of iodine in diet pressure called anaphylactic shock. Now he carries a kit
112. A health disorder that results from the deficiency of containing a syringe of _____________, which he can
thyroxine in adults and characterized by a low metabolic- inject to speed up his heart if he reacts to a bee sting.
rate, increase in body weight and tendency to retain water (a) insulin (b) melatonin
in tissues is (c) testosterone (d) epinephrine
(a) cretinism (b) hypothyroidism 119. Which of the following sets of vertebrate hormones are
(c) simple goitre (d) myxoedema all produced in the anterior pituitary gland ?
113. In a normal pregnant woman, the amount of total (a) Somatostatin, Vasopressin, Insulin
gonadotropin activity was assessed. The result expected (b) Prolactin, Growth hormone, Enkephalins
was (c) Oxytocin, Prolactin, Adrenocorticotropin
(a) high levels of FSH and LH in uterus to stimulate (d) Estrogen, Progesterone, Testosterone
endometrial thickening. 120. Given below is an incomplete table about certain
(b) high level of circulating HCG to stimulate estrogen hormones, their source glands and one major effect of
and progesterone synthesis. each on the body in humans. Identify the correct option
(c) high level of circulating FSH and LH in the uterus to for the three blanks A, B and C
stimulate implantation of the embryo.
(d) high level of circulating HCG to stimulate endometrial GLANDS SECRETION EFFECT ON BODY
thickening. A Oestrogen Maintenance of
114. STH or growth hormone (A) differs from insulin (B) in secondary sexual
which of the following action ? characters
(a) (A) causes glycogenesis but (B) causes glycolysis. Alpha cells of B Raises blood sugar
(b) (A) causes lipolysis but (B) causes lipogenesis. Islets of level
(c) (A) is catabolic for protein but (B) is anabolic. Langerhans
(d) (A) causes glycogenolysis but (B) causes
Anterior pituitary C Over secretion leads
glycogenesis.
115. When the B.P. is high and over loading of heart is present to gigantism
then which hormone is released for compensating this Options:
mechanism ? A B C
(a) Aldosterone (a) Placenta Glucagon Calcitonin
(b) A.D.H (b) Ovary Glucagon Growth hormone
(c) Atri-natriuretic factor
(c) Placenta Insulin Vasopressin
(d) Renin
116. The hormone ANP (Atrial Natriuretic Peptide) (d) Ovary Insulin Calcitonin
(a) stimulates the secretion of ADH and aldosterone. 121. Which of the following represent the correct options?
(b) inhibits the secretion of aldosterone but stimulates Name of the gland Substance produced Function
ADH secretion. A Insulin C
(c) stimulates the secretion of renin. B Glucagon D
(d) inhibits the secretion of ADH, aldosterone and renin. (a) A - Alpha cells, B - Hyperglycemia, C - Beta cells,
117. Pick the odd one out from each given words and then D - Hypoglycemia
matches it with correct options. (b) A - Beta cells, B - Alpha cells, C - Hypoglycemia,
I. Thyroid, Adrenal, Pituitary, Prostate D - Hyperglycemias
II. Cretinism, Goitre, Myxodema, Scurvy (c) A - Alpha cells, B - Hypoglycemia , C - Beta cells,
III. Insulin, Blood sugar, Adrenaline, Thyroxine D - Hyperglycemia
IV. Estrogen, Progesterone, Testosterone, Prolactin (d) A - Beta cells, B - Alpha cells, C - Hyperglycemia,
(a) I- Prostate, II - Scurvy, III - Blood sugar, IV- D - Hypoglycemia
Testosterone
(b) I - Adrenal, II - Goitre, III - Insulin, IV - Prolactin
Reproduction in
23

Chapter
Organisms

FACT/DEFINITION TYPE QUESTIONS 8. In yeast cell division results in a small cell called
(a) bud (b) clone
1. The period from birth to the natural death of an organism (c) branch (d) offspring
represents 9. Fleshy buds produced in the axil of leaves, which grow to
(a) gametophytic phase (b) sporophytic phase form new plants when shed and fall on ground are called
(c) life span (d) life cycle (a) Bulbs (b) Bulbils
2. Which ot the following has the longest life span ? (c) Tubers (d) Offset
(a) Butterfly (b) Crocodile 10. The term ‘Terror of Bengal’ is used for
(a) Bengal tiger (b) water hyacinth
(c) Parrot (d) Tortoise
(c) algal bloom (d) aquatic fauna
3. In all the methods of asexual reproduction
11. In which of the following pairs the plants can be vegetative
(a) offsprings produced are genetically identical to the
propagated by leaf buds?
parents.
(a) Bryophyllum and Kalanchoe
(b) offsprings produced are genetically different from
the parents. (b) Chrysanthemum and Agave
(c) offsprings produced may or may not be identical to (c) Agave and Kalanchoe
the parents. (d) Asparagus and Bryophyllum
(d) None of the above 12. In sexual reproduction, offsprings resemble the parents
4. A clone is a group of individuals obtained through (a) structurally but not functionally.
(b) functionally but not structurally.
(a) self-pollination (b) hybridization
(c) both structurally and functionally.
(c) asexual reproduction (d) cross-pollination (d) neither structurally nor functionally.
5. The term clone is used to describe such ___________and 13. Oestrus cycle is seen in
________ similar individual.
(a) cows and sheep (b) rats and deers
(a) chemically, genetically
(c) dogs and tiger (d) all of the above
(b) physiologically, chemically
(c) morphologically, genetically 14. The end of vegetative phase in plants which marks the
beginning of the reproductive phase can be easily seen
(d) morphologically, metabolically
in the higher plants when they come to
6. Asexual reproduction is common among
(a) flower (b) fertilize
(a) single celled organisms only.
(c) pollinate (d) None of the above
(b) plants only. 15. Sexual reproduction can be grouped into _______ distinct
(c) animals with simple organization. states.
(d) single celled animals, plants and animals with simple (a) two (b) three
organizations. (c) four (d) five
7. In protists and monerans, asexual reproduction occurs 16. In flowering plants both male and female gametes are
by non-motile. The method to bring them together for
(a) budding (b) binary fission fertilization is
(c) conidia (d) multiple fission (a) water (b) air
(c) pollination (d) apomixis
EBD_7209
198 Biology
17. The terms homothallic and monoecious are used to 25. Which of the following definitions about oviparity is
denote incorrect ?
(a) bisexual condition (b) unisexual condition (a) Only birds and reptiles are oviparous.
(c) staminate flowers (d) pistillate flowers (b) The large amount of yolk provides the nutrients for
the developing embryo.
18. Syngamy means
(c) The shell protects the egg from dehydration.
(a) fusion of similar spores. (d) Both oxygen and carbon dioxide can diffuse through
(b) fusion of gametes. the shell.
(c) fusion of dissimilar spores. 26. Select the incorrect statemen.
(d) fusion of cytoplasm. (a) Anisogametes differ either in structure, function or
behaviour.
19. Development of an egg without fertilization is called
(b) In oomycetes, female gamete is smaller and motile,
(a) gametogenesis (b) metagenesis while male gamete is larger and non-motile.
(c) oogenesis (d) parthenogenesis (c) Chlamydomonas exhibits both isogamy and
20. Which of the followings is a post-fertilization event in anisogamy and Fucus shows oogamy.
flowering plants ? (d) Isogametes are similar in structure, function and
(a) Transfer of pollen grains behaviour.
(b) Embryo development 27. Select the incorrect statements.
(a) Cucurbits and coconuts are monoecious plants.
(c) Formation of flower
(b) Papayas and date palms are dioecious plants.
(d) Formation of pollen grains (c) Leeches and tapeworms are bisexual animals.
21. Viviparity is found in (d) Sponges and coelenterates are unisexual animals.
(a) whale (b) lizards 28. Which of the following statements is/are correct ?
(c) frogs (d) birds (i) The genetic constitution of a plant is unaffected in
vegetative propagation.
STATEMENT TYPE QUESTIONS (ii) Rhizome in ginger serves as an organ of vegetative
reproduction.
22. Which of the following statement about animal (iii) Totipotency of cells enables us to micropropagate
reproduction is incorrect ? plants.
(a) Species that reproduce sexually cannot reproduce (a) (i) and (ii) (b) (ii) and (iii)
asexually. (c) (ii) only (d) All of these
(b) Viviparity, but not ovoviviparity, is common in 29. Which of the following statement(s) is/are incorrect
mammals. about internal fertilization?
(c) Male insects can remove spermatophores deposited (i) Male gametes are motile.
in a female by other males. (ii) Male gametes are non-motile.
(d) Oogenesis and spermatogenesis both occur (iii) Male gametes are produced in large number.
simultaneous in hermaphrodites. (iv) Male gametes are produced in small number.
23. Which of the following statements about animals that (v) There is a significant reduction in the number of eggs
utilize external fertilization is incorrect ? produced.
(a) They are divided equally between terrestrial and (a) (i), (iii) and (v) (b) (iii) and (iv)
aquatic species. (c) (ii) and (iv) (d) Only (v)
(b) Many produce large numbers of gametes to ensure 30. Which of the following statements is incorrect ?
successful reproduction. (i) Bamboo species flower only once in their life time,
(c) The behaviours associated with mating are often generally after 50-100 years and produce large
highly synchronized. number of fruits and die.
(d) The probability of any one egg being fertilized and (ii) In animals, the juvenile phase is followed by
developing into an adult can be low. morphological and physiological changes prior to
24. Which of the following statement is correct in the process active reproductive behaviour.
of fertilization ? (iii) The reproductive phase is of same duration in all
(a) Only one sperm reaches the egg and enters it. organisms.
(b) The entry of sperm activates the egg for completing (iv) Juvenile phase is the period of growth between the
meiosis. birth of an individual till it reaches reproductive
(c) Two haploid nuclei fuse and immediately divide to maturity.
produce two nuclei which are again haploid. (a) Only (i) (b) Only (ii)
(d) Only the acrosome of the sperm enters the egg. (c) Only (iii) (d) Only (iv)
Reproduction in Organisms 199

31. Which of these statement(s) is/are correct ? 36. Which of the following statements regarding the asexual
(i) Stamens are male reproductive part whereas carpels reproduction is incorrect?
are female reproductive parts. (a) Both mitotic and meiotic division occurs.
(ii) In Spirogyra, asexual reproduction takes place by (b) It does not contribute to evolution and speciation.
fragmentation.
(c) It is uniparental and usually occurs in unicellular
(iii) Vegetative propagation by leaves occurs in sweet
organisms.
potato.
(a) (i) and (ii) (b) (ii) and (iii) (d) There is no variation and the offsprings have the
(c) (i) and (iii) (d) All are correct same phenotype and genotype.
32. Which one of the following option is correct regarding
the given statements ? ASSERTION/REASON TYPE QUESTIONS
(i) The plant was introduced in India because of its In the following questions, a statement of Assertion is followed
beautiful flowers and shape of leaves. by a statement of Reason.
(ii) It can propagate vegetatively at a phenomenal rate (a) If both Assertion and Reason are true and the Reason is
and spread all over water body in a short period. the correct explanation of the Assertion.
(iii) It is very difficult to get rid off these plants. (b) If both Assertion and Reason are true but the Reason is
(iv) It is associated with flowering plants. not the correct explanation of the Assertion.
(a) Dahlia (b) Water hyacinth (c) If Assertion is true but Reason is false.
(c) Azolla (water fern) (d) Mosses (d) If both Assertion and Reason are false.
33. Read the following statements and select the correct ones. 37. Assertion: A plant can be retained and multiplied
(i) Conidia are the asexual propagules restricted to
indefinitely without any change or variation through
kingdom Fungi.
asexual reproduction.
(ii) A piece of potato tuber having at least one eye (or
node) is capable of giving rise to a new plant. Reason: Asexual Reproduction does not involve meiosis
(iii) Ginger propagates vegetatively with the help of its and syngamy.
underground roots. 38. Assertion: Zygote is the link between two generations.
(iv) Fleshy buds which takes part in vegetative Reason: Zygote is the product of two gametes and
propagation are called bulbils, present in Dioscorea, producer of next generation.
Agave, etc. 39. Assertion: Zygote is a single cell.
(a) (ii) and (iii) (b) (i) and (iv) Reason : Two haploid cell fused to form two diploid cell.
(c) (i), (ii) and (iv) (d) (i), (ii) and (iii) 40. Assertion: Ants, bees and termite show parthenogenesis.
34. Read the following statements regarding sexual Reason: Parthenogenesis is the process in which new
reproduction and selects the incorrect ones. organism is formed without fertilization.
(i) Sexual reproduction does not always require two 41. Assertion: Embryogenesis refers to the development of
individuals. embryo from the zygote.
(ii) Sexual reproduction generally involves gametic Reason: During Embryogenesis zygote undergoes cell
fusion. division (mitosis) and cell differentiation.
(iii) Meiosis never occurs during sexual reproduction.
(iv) External fertilization is a rule during sexual MATCHING TYPE QUESTIONS
reproduction.
(a) (i) and (iii) (b) (iii) and (iv) 42. Match column-I with column-II and select the correct
(c) (i), (ii) and (iii) (d) (ii), (iii) and (iv) answer using the codes given below.
35. Which of the following statements regarding the given Column-I Column-II
terms are correct? A. Animals which give I. Hydra
Runners, Tubers, Offsets birth to young one
(a) These all are incapable of giving rise to new B. Animal which produces II. Planaria
individuals and formation of these structures does bud
not involve two parents. C. An animal which III. Viviparous
(b) These all are capable of giving rise to new individuals shows regeneration
and formation of these structures does not involve D. Provides nutrition IV. Placenta
two parents. to the developing
(c) These all are capable of giving rise to new individuals embryo from the mother
but formation of these structures involves two (a) A – I; B – III; C – II; D – IV
parents for reproduction. (b) A – III; B – I; C – II; D – IV
(d) These are all incapable of giving rise to new (c) A – III; B – I; C – IV; D – II
individuals but formation of these structures
(d) A – III; B – IV; C – I; D – II
involves two parents for reproduction.
EBD_7209
200 Biology
43. Match the terms given in column-I with their examples in Column - I Column - II
column-II and choose the correct option. A. Sponge I. Tuber
Column-I Column-II B. Yeast II. Offset
(Terms) (Examples) C. Potato III. Gemmules
A. Binary fission I. Algae D. Water hyacinth IV. Budding
B. Zoospore II. Amoeba (a) A – IV; B – I; C – II; D – III
C. Conidium III. Hydra (b) A – III; B – I; C – IV; D – II
D. Budding IV. Penicillium (c) A – III; B – IV; C – I; D – II
E. Gemmule V. Sponge (d) A – III; B – IV; C – II; D – I
(a) A – I; B – IV; C – V; D – III; E – II 48. Match the definition (given in column I) with their correct
(b) A – II; B – I; C – IV; D – III; E – V term (given in column II) and choose the correct
(c) A – II; B – IV; C – III; D – V; E – I
combination from the options given.
(d) A – I; B – IV; C – III; D – II; E – V
44. Match column-I with column-II and select the correct Column-I Column-II
option. A. The pollen transferred I. Germination
Column -I Column- II from one flower to another
(Name of the (Haploid chromosome B. The process in which II. Pollination
organism) number in gamete) embryo develops
A. Ophioglossum (fern) I. 23 into seedling
B. Rice II. 24 C. Fertilized egg in humans III. Menstruation
C. Potato III. 12 gets implanted in
D. Man IV. 630 D. When egg in humans IV. Uterus
(a) A – I; B – II; C – III; D – IV is not fertilized process occur
(b) A – II; B – III; C – IV; D – I (a) A – II; B – I; C – IV; D – III
(c) A – III; B – IV; C – II; D – I (b) A – I; B – II; C – IV; D – III
(d) A – IV; B – III; C – II; D – I (c) A – I; B – II; C – III; D – IV
45. Match the organisms (given in column I) with their (d) A – I; B – III; C – II; D – IV
approximate life span (given in column II) and choose the 49. Which one of the following pair is correctly matched?
correct combination from the options given below. (a) Onion – Bulb
Column-I Column-II (b) Ginger – Sucker
(Organism) (Approximate (c) Chlamydomonas – Conidia
life span) (d) Yeast – Zoospores
A. Butterfly I. 60 years 50. Find out the incorrectly matched pair.
B. Crow II. 140 years
(a) Tuber – Potato
C. Parrot III. 15 years
(b) Rhizome – Ginger
D. Crocodile IV. 1 – 2 weeks
(a) A – IV; B – III; C – I; D – II (c) Bulbil – Agave
(b) A – II; B – III; C – IV; D – I (d) Leaf buds – Banana
(c) A – III; B – II; C – I; D – IV 51. Match the column I contain str uctures of male
(d) A – IV; B – III; C – II; D – I reproductive system and column II contains its feature.
46. Match the name of the organism given in column I with Select the correct option.
their chromosome number in meiocytes choose the Column-I Column-II
correct combination from the options given below. A. Gamete I. Result of fusion of male
Column-I Column-II and female gametes
(Name of the (Chromosome number B. Budding II. Division of body into
organism) in meiocyte) two equal halves
(2n) C. Fission III. Germ cells
A. House fly I. 20 D. Fertilization IV. The fusion of male and
B. Fruit fly II. 34 female gametes
C. Apple III. 8 E. Zygote V. An unequal division of
D. Maize IV. 12 organisms in which
(a) A – II; B – III; C – IV; D – I individual arises as an
(b) A – III; B – IV; C – II; D – I outgrowth from the
(c) A – IV; B – III; C – I; D – II parent
(d) A – IV; B – III; C – II; D – I (a) A - I; B - II; C - III; D - IV; E - V
47. Match the organisms given in column-I with their (b) A - III; B - V; C - II; D - IV; E - I
reproductive structure/mode of reproduction given in (c) A - III; B - I; C - V; D - II; E - IV
column-II and select the correct option from the codes
given below : (d) A - V; B - IV; C - III; D - I; E - II
Reproduction in Organisms 201

DIAGRAM TYPE QUESTIONS A B C D


(a) Tuber Rhizome Bulb Leaf buds
52. The given figures show the members of fungi and simple (b) Offset Sucker Stolon Leaf buds
plants such as algae which undergo asexual reproduction. (c) Offset Stolon Sucker Leaf buds
Identify the correct asexual reproductive structures found (d) Tuber Rhizome Bulbil Leaf buds
in the members A, B, C and D. 54. The given figure represents the events marked as (A, B, C
and D) in the life of general reproduction. Identify the
events A, B, C and D.

A C
Pre-fertilization Post-fertilization
(A) (B) B D event
event

Chlamydomonas Penicillium fertilization


2n

Mouth (a) A-Gamete transfer, B-Gametogenesis, C-Zygote


formation, D-Embryogenesis
(b) A-Gametogenesis, B-Gamete transfer, C-Zygote
formation, D-Embryogenesis
(C) (D) (c) A-Gametogenesis, B-Zygote formation, C-Gamete
transfer, D-Embryogenesis
(d) A-Gametogenesis, B-Gamete transfer, C-
Hydra Sponge Embryogenesis, D-Zygote formation.
(a) A-Zoogamete, B-Conidia, C-Bud, D-Gemmule 55. The given figure refers to which type of reproduction in
yeast?
(b) A-Zoospore, B-Conidia, C-Bud, D-Gemmule
(c) A-Zoospore, B-Conidiosporangium, C-Bud,
D-Gemmule
(d) A-Aplanospore, B-Conidia, C-Bud, D-Gemmule
53. The given figures show some examples of angiosperms
as A, B, C and D. All these are capable of giving rise to
new offsprings with the help of vegetative propagules.
Identify the correct unit of vegetative propagules present
in these angiosperms. (a) Binary fission (b) Budding
(c) Layering (d) Fusion
Bud Nodes 56. The given figures (A, B and C) are types of gametes of
Leaf scar Buds different organisms. Identify gametes (A, B and C)
Node respectively.

A Adventitious B
root

(A) (B) (C)

(a) Heterogametes, Isogametes, Homogametes


(b) Isogametes, Homogametes, Heterogametes
(c) Homogametes, Isogametes, Heterogametes
(d) Homo/Isogametes, Heterogametes, Heterogametes
C D
EBD_7209
202 Biology
57. The given figures (i to v) represent the process of binary
fission in Amoeba.

(i) (ii)

(iii) (iv)

(a) Tuber (b) Offsets


(v) (c) Sucker (d) Rhizome

CRITICAL THINKING TYPE QUESTIONS


Arrange the figures in the correct sequence and choose
the correct option. 61. If basal half of an onion bulb is removed and upper half is
(a) (iv) ® (iii) ® (i) ® (ii) ® (v) sown in the ground then the new plant will
(b) (iii) ® (iv) ® (i) ® (ii) ® (v) (a) emerge normally. (b) not emerge.
(c) (iii) ® (v) ® (ii) ® (iv) ® (i) (c) be without leaves. (d) be without flowers.
(d) (iv) ® (iii) ® (ii) ® (v) ® (i) 62. If soil around stem of potato plant is constantly removed
58. Which of the labelled parts (X and Y) in the transverse from very young stage then only roots remain in the soil
section of pea plant is/are diploid ? then the potato tubers in plant will
(a) be larger (b) be smaller
(c) be formed normally (d) not be formed
63. The site of origin of the new plantlets in potato, dahlia,
X ginger and banana is
Y (a) floral buds present on stem.
(b) internodes of modified stem.
(c) nodes of modified stem.
(a) X (b) Y (d) adventitious buds present on root.
(c) Both X and Y (d) None of these 64. Which is the most common method of reproduction in
59. The given figures P (plant Chara) and Q (earthworm) majority of fungi and bacteria ?
have label A, B, C and D as their reproductive organs. (a) Binary fission (b) Multiple fission
Select the option which correctly identifies male (c) Budding (d) Spore formation
reproductive organs of the two organisms. 65. A feature of reproduction that is common to Amoeba,
Spirogyra and yeast is that
(a) they reproduce asexually.
(b) they are all unicellular.
(c) they reproduce only sexually.
C (d) they are all multicellular.
A
66. Sexual reproduction involves
D (a) meiosis only.
(b) meiosis and fusion of gametes.
B (c) both mitosis and meiosis.
(d) all of the above
P Q 67. Some organisms are capable of asexual or sexual
(a) A and B (b) B and C reproduction. Under favourable conditions, reproduction
(c) A and C (d) B and D proceeds asexually. When conditions become more
60. In the given figure of water hyacinth, a structure is marked stressful reproduction switches to a sexual mode. Why?
as "X". This structure is involved in vegetative (a) Sexual reproduction is simple and more rapid allowing
propagation as a unit of vegetative propagules. Identify larger numbers of offspring to be produced.
the type of unit.
Reproduction in Organisms 203

(b) Sexual reproduction requires two separate 73. Meiocytes is observed in all except
individuals, who can mutually provide nutrient (a) human egg (b) blue-green algae
support during stress. (c) equisetum (d) dicot plant
(c) Sexual reproduction produces individuals with new 74. In a practical test, a student has to identify the organisms
combinations of recombined ch romosomes in which syngamy does not occur. In those organisms
increasing diversity. the female gamete undergoes development to form new
(d) Asexual reproduction requires more energy. organisms without fertilization. This phenomenon is called
68. Seeds are regarded to be the product of sexual "X".
reproduction because they Identify the organisms and the phenomenon "X".
(a) can be stored for a long period. (a) Frog, Parthenogenesis
(b) give rise to new plants. (b) Lizards, Gametogenesis
(c) are the result of fusion of male gamete with the female (c) Rotifers, Embryogenesis
gamete. (d) Honeybee, Parthenogenesis
(d) none of the above 75. Which of the followings sequences are correct for plants
69. After culturing the anther of a plant, a few diploid plants and animals?
were found along with haploid plants. The diploid plants (i) animals Juvenile phase ® Senescent
could have arisen from phase ® Reproductive phase
(a) generative cell of pollen. (ii) animals Juvenile phase ® Repr oductive
(b) cells of anther wall. phase ® Senescent phase
(c) vegetative cell of pollen. (iii) plants Reproductive phase ® Juvenile
(d) exine of pollen wall. phase ® Senescent phase
70. "X" is the vital link that ensures continuity of species (iv) plants animals Vegetative phase ® Reproductive
between organisms of one generation to the next. Every phase ® Senescent phase
sexually reproducing organism begins life as a single cell (a) (i) and (iii) (b) (i) and (iv)
"X". Identify "X". (c) (ii) and (iii) (d) (ii) and (iv)
(a) Zygote (b) Gamete 76. Offsprings of oviparous animals are at greater risk as
(c) Embryo (d) None of the above compared to offsprings of viviparous animals because
71. Which of the following has to occur if a diploid body has (a) proper embryonic care and protection is lesser.
to produce haploid gametes? (b) embryo is not developed.
(a) Meiosis (b) Mitosis (c) progenies are with more variation.
(c) Fertilization (d) Embryo transfer (d) progenies are larger.
72. Development of zygote depends on
(a) life cycle of the organism
(b) environment it is exposed to
(c) both (a) and (b)
(d) nutrition of organism
EBD_7209
Sexual Reproduction in
24

Chapter
Flowering Plants

FACT/DEFINITION TYPE QUESTIONS 11. Ovules are attached to a parenchymatous cushion called
(a) nucellus (b) obturator
1. Flowers are highly modified _______. (c) conducting tissue (d) placenta
(a) Root (b) Shoot 12. The point at which funiculus touches the ovule is
(c) Stem (d) Leaves (a) chalaza (b) hilum
2. A typical flower has ______different kinds of whorl. (c) raphe (d) endothelium
(a) two (b) three 13. Egg apparatus consists of
(c) four (d) five (a) egg cell and antipodal cells.
3. Anther is typically (b) egg cell and central cell.
(a) tetrasporangiate (b) bisporangiate
(c) egg cell and two synergids.
(c) trisporangiate (d) monosporangiate
(d) egg cell and one synergid.
4. The functions of tapetum is to
14. The most common type of ovule is __________ .
(a) produce ubisch bodies.
(b) produce pollen grains. (a) orthotropous (b) hemitropous
(c) provide nourishment to the developing pollen grains. (c) anatropous (d) campylotropous
(d) store and protect pollen grains. 15. Filiform apparatus is found in
5. Microsporogenesis occurs (a) synergids (b) anther wall
(a) on margins of leaves. (c) secondary nucleus (d) egg cell
(b) inside the ovule. 16. Polygonum type of embryo sac/typical female
(c) inside the anther. gametophyte of angiosperms is
(d) in essential floral organs. (a) 7-celled, 7-nucleate (b) 7-celled, 8-nucleate
6. Exine of pollen grain is formed of (c) 8-celled, 7-nucleate (d) 8-celled, 8-nucleate
(a) callose (b) pecto-cellulose 17. Transfer of pollen grains from the anther to the stigma of
(c) ligno-cellulose (d) sporopollenin another flower of the same plant is called
7. One of the most resistant known biological material is. (a) geitonogamy (b) xenogamy
(a) lignin (b) hemicellulose
(c) autogamy (d) cleistogamy
(c) sporopollenin (d) lignocellulose
8. Pollen grain is liberated at 18. Cleistogamous flowers are
(a) one celled stage. (a) wind pollinated (b) self-pollinated
(b) two celled stage. (c) cross-pollinated (d) insect pollinated
(c) three celled stage. 19. Both chasmogamous and cleistogamous flowers are
(d) two or three celled stage. present in
9. Pollen grains can be stored in liquid nitrogen at (a) Helianthus (b) Lommelina
_________. (c) Rosa (d) Gossypium
(a) 70°C (b) 100°C 20. Pollination by water occurs in
(c) – 196°C (d) 0°C (a) Vallisneria (b) Zostera
10. Ovule is (c) Satvia (d) All of these
(a) megasporangium 21. Bees are important to agriculture as they
(b) megasporophyll
(c) integumented megasporangium (a) produce wax (b) perform pollination
(d) rolled megasporophyll (c) prevent pollination (d) produce honey
Sexual Reproduction in Flowering Plants 205

22. Vegetative fertilization involves fusion of


STATEMENT TYPE QUESTIONS
(a) two polar nuclei
33. Which of the following statement is correct for the pollen
(b) a male gamete and a synergid
tube?
(c) a male gamete and antipodal cell
(a) It shows chemotactic movement.
(d) nucleus of a male gamete and secondary nucleus
(b) It shows only tip growth.
23. During double fertilization in plants, one sperm fuses with
(c) It is composed of three non-cellular zones.
the egg cell and the other sperm fuses with
(d) It shows radial cytoplasmic streaming.
(a) synergids cell (b) central cell
34. Which of the following statement is incorrect about
(c) antipodal cell (d) nucellar cell
emasculation?
24. Endosperm is generally
(a) During emasculation process, stigma is removed.
(a) diploid (b) triploid
(b) Emasculated flowers are bagged in order to prevent
(c) haploid (d) polyploid
self-pollination.
25. Milky water of green coconut is
(c) Emasculation is the removal of stamens before the
(a) liquid chalaza
maturation of selected bisexual flowers.
(b) liquid nucellus (d) It is one of the steps for artificial hybridization.
(c) liquid endosperm 35. Which one of the following statement is incorrect?
(d) liquid female gametophyte (a) When pollen is shed at two-celled stage, double
fertilization does not take place.
26. Scutellum is present in the embryo of
(b) Vegetative cell is larger than generative cell.
(a) pea (b) Ranunculus (c) Pollen grains in some plants remain viable for months.
(c) Triticum (d) None of these (d) Intine is made up of cellulose and pectin.
27. Perisperm is a 36. Which of the following statement about sporopollenin is
(a) degenerate part of synergids. incorrect?
(a) Exine is made up of sporopollenin.
(b) peripheral part of endosperm. (b) Sporopollenin is one of the resistant organic
(c) degenerate part of secondary nucleus. materials.
(d) remnant of nucellus. (c) Exine has apertures called germ pores where
28. False fruits (thalamus also contributes to fruit formation) sporopollenin is present.
are found in (d) Sporopollenin can withstand high temperatures and
strong acids.
(a) apple and pear (b) strawberry 37. Which one of the following events takes place after
(c) cashewnut (d) All of these double fertilization?
29. Seeds are adoptively important because (a) The pollen grain germinates on the stigma.
(a) they maintain dormancy. (b) The pollen tubes enter the embryo sac.
(c) Two male gametes are discharged into the embryo sac.
(b) they protect young plants during vulnerable stages.
(d) The PEN (Primary Endosperm Nucleus) develops
(c) they store food for young plants and facilitate into endosperm.
disperal. 38. Which one of the following statement is correct?
(d) All of the above (a) Sporogenous tissue is haploid.
30. The seed in which endosperm is used by embryo is called (b) Endothecium produces the microspores.
__________ seed. (c) Tapetum nourishes the developing pollen.
(d) Hard outer layer of pollen is called intine.
(a) single (b) albuminous
39. Which one of the following statement is correct?
(c) endospermic (d) non-endospermic (a) Geitonogamy involves the pollen and stigma of
31. Apomixis is the flowers of different plants.
(a) development of plants in darkness. (b) Cleistogamous flowers are always autogamous.
(b) development of plants without fusion of gametes. (c) Xenogamy occurs only by wind pollination.
(d) Chasmogamous flowers do not open at all.
(c) inability to perceive stimulus for flowering.
40. Which of the following statement(s) is/are correct about
(d) effect of low temperature on plant growth. self-incompatibility ?
32. Nucellar polyembryony is reported in species of (i) It is a device to prevent inbreeding.
(a) Brassica (b) Gossypium (ii) It provides a biochemical block to self-fertilization.
(iii) It ensures cross-fertilization.
(c) Triticum (d) Citrus
(iv) It is governed by pollen-pistil interaction.
EBD_7209
206 Biology
(v) It is governed by series of multiple alleles. shaped embryo is followed by globular enlarge.
(vi) It prevents self-pollen (from the same flower of other (ii) The part of the embryonal axis above the level of
flowers of the same plant) from fertilizing the ovules cotyledons is epicotyl while the part below the level
by inhibiting pollen germination of pollen tube of cotyledons is hypocotyl.
growth in the pistil. (iii) Monocot seeds possess a sin gle cotyledon
(a) (i), (ii) and (iii) (b) (i), (iv) and (v) represented by scutellum.
(a) (i) and (ii) (b) (ii) and (iii)
(c) All of the above (d) None of the above
(c) (i) and (iii) (d) (i) (ii) and (iii)
41. Which of the following statement(s) is/are incorrect ?
(i) Endosperm formation starts prior to first division of ASSERTION/REASON TYPE QUESTIONS
zygote.
(ii) Angiospermic endosperm is mostly 3N while In the following questions, a statement of Assertion is followed
gymnospermic one is N. by a statement of Reason.
(a) If both Assertion and Reason are true and the Reason is
(iii) The most common type of endosperm is nuclear.
the correct explanation of the Assertion.
(iv) Coconut has both liquid nuclear (multinucleate) and
(b) If both Assertion and Reason are true but the Reason is
cellular endosperm. not the correct explanation of the Assertion.
(v) Milky water of green tender coconut is liquid female (c) If Assertion is true but Reason is false.
gametophyte. (d) If both Assertion and Reason are false.
(a) (i) and (ii) (b) Only (iii) 46. Assertion : If a pollen mother cell has 42 chromosomes,
(c) Only (v) (d) Only (ii) the pollen has only 21 chromosomes.
42. Which of the following statements are correct for a typical Reason : Pollens are formed after meiosis in pollen mother
female gametophyte of a flowering plant? cell.
(i) It is 8-nucleate and 7-celled at maturity. 47. Assertion : Endosperm is a nutritive tissue and it is
(ii) It is free-nuclear during the development. triploid.
(iii) It is situated inside the integument but outside the Reason: Endosperm is formed by fusion of secondary
nucellus. nucleus to second male gamete. It is used by developing
(iv) It has an egg apparatus situated at the chalazal end. embryo.
(a) (i) and (iv) (b) (ii) and (iii) 48. Assertion : Photomodulation of flowering is a
(c) (i) and (ii) (d) (ii) and (iv) phytochrome regulated process.
43. Study the following statements and select the correct Reason : Active form of phytochrome (PFR) directly
option. induces floral induction in shoot buds.
(i) Tapetum nourishes the developing pollen grains. 49. Assertion : Insects visit flower to gather honey.
(ii) Hilum represents the junction between ovule and Reason : Attraction of flowers prevents the insects from
funicle. damaging other parts of the plant.
50. Assertion : Chasmogamous flowers require pollinating
(iii) In aquatic plants, such as water hyacinth and water
agents.
lily, pollination is by water.
Reason : Cleistogamous flowers do not expose their sex
(iv) The primary endosperm nucleus is triploid.
organs.
(a) (i) and (ii) are correct but (iii) and (iv) are incorrect.
(b) (i), (ii) and (iv) are correct but (iii) is incorrect. MATCHING TYPE QUESTIONS
(c) (ii), (iii) and (iv) are correct but (i) is incorrect.
(d) (i) and (iv) are correct but (ii) and (iii) are incorrect. 51. Match the biotic agent of cross pollination given in column-
I with their feature given in column-II and select the correct
44. Seeds are adaptively important because
answer using the codes given below.
(i) they maintain dormancy.
Column-I Column-II
(ii) they protect young plants during vulnerable stages. A. Zoophily I. Pollination by birds
(iii) they store food for young plants, and facilitate B. Ornithophily II. Pollination by insects
dispersal. C. Entomophily III. Pollination by bats
Identify the correct reasons. D. Chiropterophily IV. Pollination by animals
(a) (i) and (iii) (b) (ii) and (iii) (a) A – III; B – II; C – I; D – IV
(c) (i) and (ii) (d) All of the above (b) A – I; B – II; C – III; D – IV
45. Which of the given statements are true? (c) A – IV; B – I; C – II; D – III
(i) During the development of a dicot embryo heart (d) A – IV; B – II; C – I; D – III
Sexual Reproduction in Flowering Plants 207

52. Match the parts of gynoceium given in column I with (a) A – II; B – III; C – V; D – IV; E – I
their definition given in column II. Choose the correct (b) A – I; B – III; C – II; D – IV; E – V
combination from the options given below. (c) A – II; B – III; C – I; D – IV; E – V
Column-I Column-II (d) A – II; B – IV; C – V; D – I; E – III
A. Funicle I. Mass of cells within ovule 56. Match the items given in column-I with their examples
with more food given in column-II and identify the correct option.
B. Hilum II. Basal part of ovule Column-I Column-II
C. Integument III. One or Two protective layers A. Coleorhiza I. Grapes
of ovule B. Food storing tissue II. Mango
D. Chalaza IV. Region where body of ovule C. Parthenocarpic fruit III. Maize
fuses with funicle D. Single seeded fruit developing IV. Radicle
E. Nucellus V. Stalk of ovule
from monocarpellary superior
(a) A – I; B – II; C – III; D – IV; E – V
ovary
(b) A – V; B – IV; C – III; D – II; E – I
E. Membranous seed coat V. Endosperm
(c) A – IV; B – II; C – I; D – III; E – V
(a) A – III; B – I; C – IV; D – II; E – V
(d) A – I; B – III; C – V; D – II; E – IV
53. Match the items given in column-I with their examples (b) A – IV; B – II; C – V; D – I; E – III
given in column-II and choose the correct option given (c) A – V; B – I; C – III; D – IV; E – II
below. (d) A – IV; B – V; C – I; D – II; E – III
Column-I Column-II 57. Which of the following is a mismatched pair?
(Items) (Examples) (a) Microsporangium – Pollen sac
A. Ovary I. Groundnut, mustard (b) Megasporangium – Ovule
B. Ovule II. Guava, orange, mango (c) Microsporophyll – Stamen
C. Wall of ovary III. Pericarp (d) Megasporophyll – Filament
D. Fleshy fruits IV. Seed 58. Which of the following is a mismatched pair?
E. Dry fruits V. Fruit (a) Storage of pollen grains – – 196°C
(a) A – V; B – IV; C – III; D – II; E – I (b) Pollen allergy – Carrot grass
(b) A – I; B – II; C – III; D – IV; E – V (c) Chasmogamous flowers – Exposed anthers and
(c) A – I; B – III; C – II; D – IV; E – V stigmas
(d) A – V; B – IV; C – I; D – II; E – III (d) Xenogamy – Self-pollination
54. Match the items given in column-I with those given in
column-II and choose the correct option given below. DIAGRAM TYPE QUESTIONS
Column-I Column-II 59. The given figure shows a typical stamen (a) and three
A. Parthenocarpy I. Inactive state dimensional cut section of an anther. Identify A to D
B. Polyembryony II. Meiosis and syngamy are respectively marked in the figures (a & b)
absent
C. Apomixis III. Occurrence of more than
one embryo A
D. Dormancy IV. Seedless fruit D
(a) A – I; B – II; C – III; D – IV
C
(b) A – IV; B – III; C – II; D – I Line of
(c) A – IV; B – I; C – II; D – III dehiscence
(d) A – III; B – II; C – I; D – IV
55. Match the items given in column-I with those given in
column-II and chose the correct option given below.
Column-I Column-II
A. Tapetum I. Irregular in shape with B
abundant food reserve (b)
B. Exine II. Acts as nutritive layer (a)
C. Pollenkit III. Thick, rigid protective layer
(a) Anther, Petiole, Pollen sac and Megaspore
D. Vegetative cell IV. Involve in the formation of
(b) Anther, Petiole, Megasporangium and Pollen grains
microspores
(c) Anther, Pedicel, Megasporangium and Pollen grains
E. Sporogenous V. Oily and sticky layer, help
(d) Anther, Filament, Pollen sac and Pollen grains
tissue in pollination.
EBD_7209
208 Biology
60. The given diagram refers to a T. S. of anther. Identify A to 63. Diagram given below shows the stages in embryogenesis
E respectively in a typical dicot plant (Capsella). Identify the structures
A to D respectively
Connective
C
E A
A B
D
Zygote
B D
(a) Sporogenous tissue, tapetum, epidermis, middle
layer, endothecium Heart-Shaped
Embryo C
(b) Sporogenous tissue, epidermis, tapetum, middle layer,
Globular Mature
endothecium Embryo Embryo
(c) Sporogenous tissue, epidermis, middle layer, tapetum,
endothecium (a) Suspensor, Radicle, Plumule, Cotyledons
(d) Sporogenous tissue, tapetum, middle layer, (b) Hypophysis, Radicle, Plumule, Cotyledons
epidermis, endothecium (c) Suspensor, Plumule, Radicle, Cotyledons
61. Identify A, B, C, D and E structures marked in the given (d) Suspensor, Radicle, Plumule, Hypocotyls
figure of a mature embryo sac. 64. The given figure represent the L.S of a flower showing
Chalazas end growth of pollen tube. Few structures are marked as A, B,
A C, D & E. Identify A, B, C, D and E respectively.

D
C
B Pollen tube
Egg
E A
D
B
E Egg cell
Micropylar end Synergid
A B C D E
Antipodal Central cell Polar nuclei Synergids Acrosome (a) Antipodal cells, Polar nuclei, Stigma, Style, Chalaza
(a)
cells (b) Antipodal cells, Polar nuclei, Style, Stigma, Chalaza
Antipodal Central cell Polar nuclei Synergids Filiform (c) Antipodal cells, Polar nuclei, Stigma, Chalaza, Style
(b) cells apparatus (d) Antipodal cells, Polar nuclei, Chalaza, Stigma, Style
Synergids Central cell Polar nuclei Antipodal Filiform 65. In the given figure of pollen grain tetrad, identify the parts
(c) cells apparatus marked as A, B, C, D and E.
Synergids M egaspore Polar nuclei Synergids Filiform
(d) A
mother cell apparatus B
C
62. Which of the following figure, showing types of D
gynoecium, is associated with wind pollination ? E

(a) A - Germ pore, B - Generative cell, C - Intine, D -


Exine, E - Vegetative cell
(b) A - Germ pore, B - Generative cell, C - Exine, D -
Intine, E - Vegetative cell
(c) A - Intine, B - Exine, C - Germ pore, D - Generative
cell, E - Vegetative cell
(d) A - Exine, B - intine, C - Vegetative cell, D - Germ pore,
(a) (b) (c) (d) E - Generative cell
Sexual Reproduction in Flowering Plants 209

66. Identified A, B, C and D in the given figure of false fruit of 69. The given figure shows a diagrammatic view of a typical
apple. anatropous ovule, in which some parts are typical
anatropous ovule, in which some parts are marked as A,
B, C, & D. Identify the correct labelling of A, B, C & D
from the options given below.

(a) A – Mesocarp; B – Endocarp; C – Seed; D – Thalamus


(b) A – Seed; B – Thalamus; C – Mesocarp; D –
Endocarp B
(c) A – Thalamus; B – Seed; C – Endocarp; D – Mesocarp C
(d) A – Mesocarp; B – Endocarp; C – Seed; D – Thalamus
67. Choose the option showing the correct labelling A, B, C
and D in the given figure of a dicot embryo. D
(a) A – Chalazal pole; B – Micropyle; C – Embryo sac;
D –Nucellus
(b) A – Micropyle; B – Chalazal pole; C – Embryo sac;
D – Nucellus
(c) A – Micropyle; B – Chalazal pole; C – Nucellus;
D – Embryo sac
(d) A – Micropyle; B – Nucellus; C – Embryo sac;
D – Chalazal pole
70. The given figure shows the L.S. of a monocot embryo.
Choose the correct labelling for A, B, C and D marked in
(a) A – Hypocotyl; B – Cotyledons; C – Root cap; the figure from the options given below.
D – Radicle
(b) A – Cotyledons; B – Hypocotyl; C – Root cap; A
D – Radicle
(c) A – Cotyledons; B – Hypocotyl; C – Radicle;
D – Root cap
B
(d) A – Cotyledons; B – Radicle; C – Hypocotyl;
D – Root cap.
68. The given diagram shows two plants of the same species.
Identify the type of pollination indicated as P1, P2 and P3.
CC

P3
P1

P2
D

(a) A – Coleoptile; B – Scutellum; C – Epiblast;


D – Coleorhiza
P1 P2 P3
(b) A – Scutellum; B – Coleoptile; C – Coleorhiza;
(a) Allogamy Chasmogamy Cleistogamy D – Epiblast
(b) Autogamy Xenogamy Geitonogamy (c) A – Scutellum; B – Epiblast; C – Coleoptile;
(c) Autogamy Geitonogamy Xenogamy D – Coleorhiza
(d) Geitonogamy Allogamy Autogamy (d) A – Scutellum; B – Coleoptile; C – Epiblast;
D – Coleorhiza
EBD_7209
210 Biology

CRITICAL THINKING TYPE QUESTIONS 81. The total number of nuclei involved in double fertilization
in angiosperms are
71. The largest cell in a embryo sac is (a) two (b) three
(a) egg (b) central cell (c) four (d) five
(c) synergid (d) antipodal cell 82. Unisexuality of flowers prevents
72. Which one of the following is not related to other three? (a) geitonogamy but not xenogamy.
(a) Archaegonium (b) Oogonium (b) autogamy and geitonogamy.
(c) Ovule (d) Antheridium (c) autogamy but not geitonogamy.
73. In a fertilized ovule, n, 2n and 3n conditions occur (d) both geitonogamy and xenogamy.
respectively in 83. Albuminous seeds store their reserve food mainly in
(a) antipodal, egg and endosperm.
(a) perisperm (b) endosperm
(b) egg, nucellus and endosperm.
(c) cotyledons (d) hypocotyl
(c) endosperm, nucellus and egg.
84. Pollination occurs in
(d) antipodals, synergids and integuments.
(a) bryophytes and angiosperms.
74. Seed coat is not thin, membranous in
(a) coconut (b) groundnut (b) pteridophytes and angiosperms.
(c) gram (d) maize (c) angiosperms and gymnosperms.
75. Which of the following floral parts forms pericarp after (d) angiosperms and fungi.
fertilization ? 85. An advantage of cleistogamy is that
(a) Nucellus (b) Outer integument (a) it leads to greater genetic diversity.
(c) Ovary wall (d) Inner integument (b) seed dispersal is more efficient and wide spread.
76. Product of sexual reproduction generally generates (c) each visit of pollinator brings hundreds of pollen
(a) prologned dormancy. grains.
(b) new genetic combination leading to variation. (d) seed set is not dependent upon pollinators.
(c) large biomass. 86. Point out the odd one from the given options.
(d) longer viability of seeds.
(a) Nucellus (b) Embryo sac
77. Sequence of development during the formation of embryo
sac is (c) Micropyle (d) Pollen grain
(a) Archesporium ® Megaspore ® Megaspore mother 87. While planning for an artificial hybridization programme
cell ® Embryo sac. if the female parent have unisexual flowers, then which of
(b) Megasporocyte ® Archesporium ® Megaspore ® the following steps would not be relevant?
Embryo sac. (a) Bagging of female flower.
(c) Megaspore ® Megaspore moth er cell ® (b) Dusting of pollen on stigma.
Archesporium ® Embryo sac.
(c) Emasculation.
(d) Archesporium ® Megaspore mother cell ®
(d) Collection of pollen.
Megaspore ® Embryo sac.
78. Which of the following processes is necessary for the 88. In the embryos of a typical dicot and a grass, true
complete development of male gametophyte? homologous structures are
(a) One meiotic cell division and two mitotic cell (a) coleorhiza and coleoptile.
divisions. (b) coleoptile and scutellum
(b) One meiotic cell division and one mitotic cell (c) cotyledons and scutellum
division. (d) hypocotyl and radicle
(c) Two meiotic cell divisions and one mitotic cell
89. Total number of meiotic division required for forming 100
division.
(d) Two mitotic cell divisions. zygotes/100 grains of wheat is
79. Megaspores are produced from the megaspore mother (a) 100 (b) 75
cells after (c) 125 (d) 50
(a) meiotic division. 90. The endosperm found in angiospermic seed is different
(b) mitotic division. from that of gymnosperms in the sense that, in the former
(c) formation of a thick wall. (a) it is formed before fertilization while in the latter it is
(d) differentiation.
formed after fertilization.
80. How many meiotic division are required for the formation
(b) it is formed after fertilization.
of 100 functional megaspores?
(a) 100 (b) 50 (c) it is cellular while in the latter it is nuclear.
(c) 75 (d) 25 (d) it is nutritive while in the latter it is protective.
Sexual Reproduction in Flowering Plants 211

91. For artificial hybridization experiment in bisexual flower, 95. Multinucleate condition is present in
which of the following sequences is correct ? (a) quiescent centre
(a) Bagging ® Emasculation ® Cross-pollination ® (b) maize
Rebagging (c) meristematic tissue
(b) Emasculation ® Bagging ® Cross-pollination ® (d) liquid endosperm of coconut
Rebagging 96. Through which part of the embryo sac, does the pollen
(c) Cross-pollination ® Bagging ® Emasculation ® tube enter the embryo sac?
Rebagging
(a) Egg cell (b) Persistent synergid
(d) Self-pollination ® Bagging ® Emasculation ®
(c) Degenerated synergid (d) Central cell
Rebagging
97. What is the main function of filiform apparatus present at
92. If a diploid female plant and a tetraploid male plant are
the micropylar part of the ovule?
crossed, the ploidy of endosperm shall be
(a) tetraploid (b) triploid (a) It prevents the entry of more than one pollen tube
(c) diploid (d) pentaploid into the embryo sac.
93. In a seed of maize, scutellum is considered as cotyledon (b) It helps in the entry of pollen tube into an antipodal
because it cell.
(a) protects the embryo. (c) It helps the pollen tube to enter the ovule through
(b) contains food for the embryo. chalazal end.
(c) absorbs food materials and supplies them to the (d) It guides the entry of pollen tube into a synergid and
embryo. discharge the male gametes.
(d) converts itself into a monocot leaf.
94. How many pollen grains will be formed after meiotic
division in ten microspore mother cells?
(a) 10 (b) 20
(c) 40 (d) 80
EBD_7209
212 Biology

25

Chapter
Human Reproduction

8. In the process of spermatogenesis, first maturation


FACT/DEFINITION TYPE QUESTIONS
division is called _______.
1. The __________ lead to vas deferens that ascends to (a) mitotic division (b) reduction division
the ______ and loops over the __________. (c) amitotic division (d) None of the these
(a) prostate, stomach, urinary bladder. 9. Spermatids are transformed into sperm by a process
(b) epididymis, abdomen, urinary bladder. called_______.
(c) vas efferentia, abdomen, ureter. (a) spermiation (b) implantation
(d) urinary bladder, ejaculatory duct, abdomen. (c) insemination (d) spermiogenesis
2. The enlarged end of penis is covered by a loose fold of 10. In humans, male germs cells differentiate into _____ at
skin is called. the end of first meiotic division.
(a) glans penis (b) foreskin (a) spermatid
(c) hymen (d) urethral meatus (b) spermatogonium
3. Which of the following is a transporting tube leading (c) secondary spermatocyte
from the bladder to which brings urine outside the body (d) primary spermatocyte
via penis? 11. Increased secretion of which hormone start the process
(a) Ureter (b) Epididymis of sperm formation at the time of puberty?
(c) Ejaculatory duct (d) Urethra meatus (a) GH (b) TSH
4. Vasa efferentia are the ductules leading from (c) PRL (d) GnRH
(a) epididymis to urethra. 12. After birth, colostrum is released from mammary glands
(b) vas deferens to epididymis. which is rich in
(c) rete testis to vas deferens. (a) fat and low in proteins
(d) testicular lobules to rete testis. (b) proteins and low in fat
5. A sac shaped like an upside down pear with a thick lining (c) proteins, antibodies and low in fat
and muscles in the pelvic area where a fertilized egg or (d) proteins, fat and low in antibodies
zygote comes to grow into a baby is called _______. 13. Which of the following hormone maintains the function
(a) oviduct (b) uterus of male sex accessory gland and ducts?
(c) vagina (d) vulva (a) Estrogen (b) Androgen
6. Which of the following is a finger like structure and lies at (c) Progesterone (d) Luteinizing hormone
the upper junction of the two labia minora above the
14. Semen is a constituent of seminal plasma with _______.
urethral opening?
(a) ovum (b) sperm
(a) Clitoris (b) Oviduct
(c) zygote (d) follicle
(c) Ampulla (d) Chorionic villi
15. Ejaculation of human male contains about 200 – 300
7. Which of the following produces sperms in
million sperms, of which for normal fertility ____ %
spermatogenesis?
sperms must have normal shape and size and at least
(a) Sertoli cells.
____% must show energetic motility.
(b) Interstitial cells.
(a) 40, 60 (b) 50, 50
(c) Primary spermatocytes.
(c) 60, 40 (d) 30, 70
(d) Immature male germ cells.
Human Reproduction 213

16. Which of the following stage of oogenesis forms a 26. Fusion of haploid nucleus of sperm and that of ovum
membrane called zona pellucida surrounding it? lead to the formation of _______.
(a) Oogonia (b) Polar body (a) zygote (b) blastocysts
(c) Corpus luteum (d) Secondary oocytes (c) embryo (d) foetus
17. By which process sperms released from the 27. Which of the following differentiate into embryo?
seminiferous tubules? (a) Morula (b) Zygote
(a) Spermiation (b) Insemination (c) Trophoblast (d) Inner cell mass
(c) Spermatogenesis (d) Spermiogenesis 28. Trophoblast and inner cell mass are the arrangements of
18. Which of the following contains a fluid filled cavity blastomeres as outer and inner layers respectively in
called antrum? _______.
(a) Primary spermatocyte. (a) zygote (b) morula
(b) Primary follicle of ovary. (c) placenta (d) blastocysts
(c) Tertiary follicle of ovary. 29. The embryo with 8 to 16 blastomeres is called ________.
(d) Secondary spermatocyte. (a) zygote (b) morula
19. Menstruation is triggered by a sudden decline in the (c) placenta (d) blastocysts
amount of hormone secreted by corpus luteum. Identify 30. Presence of XX or XY chromosomes in zygote depends on
the hormone.
(a) the sperm carrying X chromosome fertilized the
(a) Luteinizing hormone ovum.
(b) Follicle stimulating hormone (b) the sperm carrying Y chromosome fertilized the
(c) Progesterone ovum.
(d) Estrogen (c) the sperm without any chromosome fertilized the
20. Level of which hormones are at their highest during the ovum.
luteal phase (second half of the cycle) of the menstrual (d) the sperm carrying X or Y chromosomes fertilized
cycle? the ovum.
(a) Estrogen 31. Which layer of blastocysts gets attached to the
(b) Progesterone endometrium?
(c) Luteinizing hormone (a) Trophoblast (b) Inner cell mass
(d) Follicular stimulating hormone (c) Umbilical cord (d) Both (a) and (c)
21. Which phase of menstrual cycle is also called 32. Finger like projection, called chorionic villi, appear on
proliferative phase? the _________ after the implantation.
(a) Luteal (b) Ovulatory (a) ampulla (b) trophoblast
(c) Follicular (d) Menstruation (c) infundibulum (d) inner cell mass
22. Which phase of menstrual cycle is also called secretory 33. Which of the following hormones is produced in women
phase? only during pregnancy?
(a) Luteal (b) Ovulatory (a) Relaxin (b) Estrogen
(c) Follicular (d) Menstruation (c) Oxytocin (d) Progesterone
23. Which of the following indicates pregnancy? 34. The placenta is formed from the _______ of the embryo
(a) Lack of menstruation. and the _______ of the mother.
(b) Occurrence of menstrual flow. (a) uterus, trophoblast
(c) When released ovum is not fertilized. (b) chorion, endometrium
(d) When Graafian follicle matures and endometrium (c) endometrium, chorion
regenerates through proliferation. (d) inner cell mass, endometrium
24. When semen is released by the penis into the vagina 35. Placenta acts as an
during copulation, then it is called _____. (a) Embryo (b) Corpus luteum
(a) ovulation (b) insemination (c) Exocrine gland (d) Endocrine tissue
(c) menstruation (d) gametogenesis 36. Primary germ layers are
25. At the time of implantation, the human embryo is called (a) ectoderm and inner cell mass only.
_______. (b) trophoblast, ectoderm and mesoderm.
(a) zygote (b) blastocysts (c) endoderm and mesoderm only.
(c) embryo (d) foetus (d) ectoderm, endoderm and mesoderm.
EBD_7209
214 Biology
37. Which of the following hormone acts on uterine muscle few days after birth, before breast milk comes in.
and causes its stronger contraction? (a) Placenta (b) Colostrum
(a) Relaxin (b) Estrogen (c) Egg yolk (d) Blood cells
(c) Oxytocin (d) Progesterone 47. Which gland releases a small amount of fluid just prior to
38. Colostrum ejaculation to decrease acidity in the urethra caused by
(a) is a hormone essential for milk secretion. urine?
(b) can be synthesized by the newborn infant but not (a) Prostate
by a foetus. (b) Glans penis
(c) stimulates further secretion of oxytocin for uterine (c) Seminal vesicle
contraction. (d) Bulbourethral gland
(d) is a source of antibodies essential to develop 48. By the end of how many weeks, major organ system are
resistance against diseases in new born babies. formed during the embryonic development?
39. Secretion of milk from the mammary gland towards the (a) 4 weeks (b) 8 weeks
end of the pregnancy is called_____. (c) 12 weeks (d) 24 weeks
(a) lactation (b) parturition 49. Which of the following induces foetal ejection reflex?
(c) fertilization (d) implantation (a) Initiation of lactation
40. Two types of cells present in the lining of seminiferous (b) Fully developed foetus and placenta
tubules are _______ and _______. (c) Expulsion of the baby out of the uterus.
(a) leydig cells, sertoli cells. (d) Transport of embryo in the fallopian tube.
(b) male germ cells, sertoli cells. 50. Identify the structure on the basis of the given statement
(c) spermatogonium, spermatids. which surrounds the primary sex organ of male
(d) primary oocyte, leydig cells. reproductive system.
41. The womb opens into vagina through _______. "It is responsible for maintaining the low temperature by
(a) cervix (b) hymen about 2 - 2.5º C from normal body temperature to mature
(c) clitoris (d) ampulla sperm."
42. First polar body is formed during the formation of (a) Penis (b) Scrotum
_______ and completion of ___ meiotic division. (c) Ureter (d) Urethra
(a) Primary oocytes, II 51. Which of the following is not a paired structure in male?
(b) Secondary oocytes, I (a) Urethra (b) Vas deferens
(c) Secondary spermatocytes, II (c) Epididymis (d) Ejaculatory duct
(d) Primary spermatocytes, I 52. Blastomeres are daughter cells formed in the process of
43. Second meiotic division in secondary oocyte results in (a) cleavage, when zygote undergoes mitotic division.
the formation of (b) fertilization, when sperm enters in the cytoplasm of
(a) first polar body and a diploid ovum. ovum.
(b) first polar body and a haploid ovum. (c) implantation, when blastocysts attached to the
(c) second polar body and a diploid ovum. uterine endometrium.
(d) second polar body and a haploid ovum (d) gametogenesis, when male and female gametes are
produced by testis and ovary respectively
44. In human female, menopause is a stage in which
(a) oogenesis starts at puberty. STATEMENT TYPE QUESTIONS
(b) menstruation starts at puberty.
(c) corpus luteum starts secreting progesterone for 53. Which of the following statements regarding sertoli cell
maintaining pregnancy. is correct?
(d) menstruation stops at the age of 50 years and (a) It is found in seminiferous tubule and secrete
reproductive capacity is arrested. testosterone hormone.
45. Which of the following process induces the completion (b) It is a place where spermatozoa is concentrated and
of the meiotic division of secondary oocyte? stored until ejaculation.
(a) Parturition (b) Implantation (c) It secretes spermatozoa activating substances
(c) Fertilization (d) Gametogenesis like fructose, citrate, inositol, prostaglandin and
protein.
46. _______ is a sticky white or yellow fluid secreted by the
breasts during the second half of pregnancy and for a (d) It is found in seminiferous tubule and function as
nurse cells for differentiating spermatozoa.
Human Reproduction 215

54. The sperm and the egg make different contributions to having a number of lobules containing number of
zygote. Which of the following statements about their alveoli.
contributions are true? (c) The cells of alveoli secrete milk which is stored in
(i) Sperm contributes most of the mitochondria. the cavity of the alveoli.
(ii) Egg contributes most of the cytoplasm. (d) Each milk gland or lobules has lactiferous ducts that
(iii) Both sperm and egg contribute haploid nucleus. drain into openings in the nipple.
(iv) Both sperm and egg contribute centrioles. 59. Select the correct statements regarding oogenesis.
(a) (i) and (ii) (i) It is initiated during the embryonic development
(b) (ii) and (iii) stage when millions of oogonia are formed within
(c) (iiii) and (iv) each ovary.
(d) All of these (ii) Graafian follicle releases primary oocyte from the
ovary by ovulation.
55. Read the following statements (i to iv) and answer the
following question. (iii) At puberty only 60,000 – 80,000 primary follicles are
left in each ovary.
(i) Each testes has highly coiled 250 compartments
called seminiferous tubules. (iv) Secondary oocyte within tertiary follicles grows in
size and completes its second meiotic division.
(ii) Erection of the penis due to presence of special
tissues facilitates insemination. (a) (i), (ii) and (iii) (b) (i) and (iii)
(iii) Immunologically competent cells are also present (c) (ii) and (iv) (d) all the four statements.
in the interstitial spaces of seminiferous tubules. 60. Which of the following statement is correct regarding
(iv) Testes lie outside the abdominal cavity in a thin menstruation?
pouch like skin called scrotum. (a) The menstrual fluid can easily clot.
(v) Bulbourethral gland is a single accessory gland. (b) The end of the cycle of menstruation is called
How many of the above statements are incorrect? menarche.
(a) (i), (ii) and (iii) (b) (iii) and (v) (c) At menopause in the female, there is especially
abrupt decrease in gonadotropic hormones.
(c) (i) and (v ) (d) (ii), (iv) and (v)
(d) In human female, menstruation can be deferred
56. Which of the following is true regarding the male
by the administration of combination of estrogen
reproductive system?
and progesterone.
(a) Sperms are diploid.
61. Which of the following is required for the increased
(b) It includes testes, accessory ducts and glands, and production of estrogen, progestogens, cortisol, prolactin
oviducts. and thyroxine etc. in the maternal blood?
(c) The scrotum keeps the testes warmer, thus helping it (i) Metabolic changes in the mother.
to promote the sperm formation.
(ii) Maintenance of pregnancy.
(d) Sertoli cells are found in seminiferous tubules
(iii) Supporting the foetal growth
and provide nutrition to germ cells.
(iv) Destruction of Graafian follicle
57. Read the following statements (i to v) and answer the
following question. (a) (iii) and (iv) (b) (i), (iv) and (v)
(i) This structure is also called womb. (c) (i), (ii) and (iii) (d) All the four statements.
(ii) Its shape is like an inverted pear. 62. Which of the following statement regarding female
reproductive system is (are) correct?
(iii) The process of fertilization takes place in this
structure. (i) Myometrium undergoes strong contraction at the
time of delivery of baby.
(iv) The wall of this structure has three layers of tissue.
(ii) Ovary is secondary female sex organ which
(v) It secretes several steroid hormones.
produces female gamete and steroid hormones.
Identify the correct characteristics feature regarding
(iii) Ovarian stroma is divided into two zones: inner
uterus from the above statements.
cortex and outer medulla.
(a) (i) and (iv) (b) (iii) and (v)
(iv) Infundibulum possess finger like projections
(c) (i), (ii) and (iv) (d) All the five statements. which help in collection of ovum after the release
58. Which of the following statements regarding mammary of secondary oocyte.
gland is incorrect? (v) A functional mammary gland is the characteristics
(a) They are paired glandular structure that lies over the of all the mammals (including male and female).
pectoral muscles. (a) (i) and (iv) (b) (i), (ii), (iii) and (v)
(b) Each gland has 100 – 500 lobulated milk glands each (c) (iii), (iv) and (v) (d) All the five statements
EBD_7209
216 Biology
63. Read the following statements (i to v) and answer the telolecithal.
question. 71. Assertion : Interstitial cell is present in the region
(i) It produces several hormones like hCG, hPL, outside the seminiferous tubule called interstitial spaces.
estrogens, progestogens etc. Reason : Interstitial cells provide nutrition to the sertoli
(ii) It differentiates into three embryonic membranes cells.
– ectoderm, endoderm and mesoderm. 72. Assertion : Testicular lobules are the compartments
(iii) It undergoes mitotic division. present in testis.
Reason : These lobules are involved in the process of
(iv) It is the organ, formed in the lining of the
fertilization.
uterus by the union of the uterine mucous
73. Assertion : Head of sperm consists of acrosome and
membrane with the membranes of the foetus.
mitochondria.
(v) It develops at a point of implantation and providing Reason : Acrosome contains spiral row of mitochondria.
oxygen and nutrients for the foetus and transfer of
waste products from the foetal to the maternal blood MATCHING TYPE QUESTIONS
circulation.
Identify the correct characteristics feature regarding 74. In the given columns, column I contain structures of
placenta from the above statements. male reproductive system and column II contains its
(a) (iii) and (v) (b) (i), (iv) and (v) feature. Select the correct match from the options given
(c) (i), (ii) and (iv) (d) all the four statements. below.
64. Which of the following statements regarding parturition Column I Column II
is incorrect? (Structure of Male (Features)
(a) Prolactin induces uterine contraction. Reproductive System)
(b) It is induced by neuroendocrine mechanism. A. Seminiferous tubule I. Network of
(c) Uterine contraction leads to expulsion of baby seminiferous tubule
through the birth canal. B. Rete testis II. Secon dary sexual
(d) Oxytocin plays an important role in the contraction characters
of fallopian tube. C. Leydig cells III. Meiosis and sperm
formation occurs
ASSERTION/REASON TYPE QUESTIONS D. Prepuce IV. Place of implantation
In the following questions, a statement of Assertion is V. Terminal skin of penis
followed by a statement of Reason. (a) A – I; B – II; C – III; D – V
(a) If both Assertion and Reason are true and the Reason is (b) A – III; B – I; C – II; D – V
the correct explanation of the Assertion. (c) A – III; B – I; C – IV; D – II
(b) If both Assertion and Reason are true but the Reason is
(d) A – II; B – IV; C – III; D – V
not the correct explanation of the Assertion.
(c) If Assertion is true but Reason is false. 75. Which of the following pair is incorrectly matched?
(d) If both Assertion and Reason are false. (a) Leydig cells – Testosterone
65. Assertion : During fertilization only head of (b) Spermatogenesis – Seminiferous tubules
spermatozoa enters egg. (c) Male reproductive system – Pelvis region
Reason : If several spermatozoa hit the egg at same time, (d) Spermatogonia – Mitotic division
all can enter the egg. 76. In the given columns, column-I contain structures of female
66. Assertion : In morula stage, cells divide without
reproductive system and column-II contain its feature.
increase in size.
Select the correct match from the option given below.
Reason : Zona pellucida remain undivided till cleavage
is complete. Column-I Column-II
67. Assertion : Death is one of the important regulatory (Structures of female (Features)
process on earth. reproductive system)
Reason : It avoids over-crowding caused by continuous A. Ampulla I. It undergoes cyclical
reproduction. changes during menstrual
68. Assertion : Corpus luteum degenerates in the absence cycle.
of fertilization. B. Labia majora II. It helps in collection of
Reason : Progesterone level decreases. ovum after ovulation.
69. Assertion : Clitoris is not remnant of penis in females. C. Oviduct III. Wider part of fallopian
Reason : It also have high blood supply and erectile tissue.
tube where fusion of
70. Assertion : Mammalian ova produces hyaluronidase.
male and female gametes
Reason : The eggs of mammal are microlecithal and
takes place.
Human Reproduction 217

D. Fimbriae IV. Larger hairy folds which fertilized ovum and other
extend down from the events of pregnancy.
mons pubis and surrounds C. Follicular phase III. Secretion of luteinizing
the vaginal opening. hormone at its maximum
E. Endometrium V. Also called fallopian level and induces
tubes, which extend from breakdown of mature
the periphery of each ovary follicle to release the female
to the womb. gamete
(a) A – I; B – II; C – III; D – V; E – IV D Ovulatory phase IV. Formation of mature
(b) A – III; B – I; C – II; D – V; E – IV Graafian follicle and
(c) A – III; B – IV; C – V; D – II; E – I regeneration of
(d) A – II; B – IV; C – III; D – V; E – I endometrium of uterus.
77. Match the column-I with column-II and select the correct (a) A – IV; B – II; C – III; D – I
option. Match from the options given below (b) A – III; B – I; C – II; D – IV
Column-I Column-II (c) A – III; B – I; C – IV; D – II
A. Primary oocyte I. It is formed when oogonia (d) A – I; B – II; C – IV; D – III
starts division and 79. In the given columns, column-I contain features of
temporarily arrested at developing child and column-II contain the time of their
prophase of meiosis I. occurrence. Select the correct match.
B. Secondary oocyte II. A large haploid cell which Column-I Column-II
retains bulk of nutrient rich (Features of (Time of there
cytoplasm of the primary developing child) occurrence)
oocyte. A. Heart sound I. By the end of the second
C. Primary follicle III. A large number of these month of pregnancy
degenerate durin g th e B. Foetus develops II. During the fifth month
phase from puberty to birth. limbs and digit
D. Oogonia IV. Gamete mother cell. C. Formation of III. First sign of growing
major organ system foetus
D. First movement IV. By the end of 12 weeks
E. Secondary follicle V. Surrounded by more layers
of foetus and
of granulosa cells and a
appearance of
new theca.
hair on head
F. Graafian follicle VI. Rupture to release ovum E. Body covered with V. By the end of 24 weeks
from the ovary. hair, eyelid separate,
(a) A – I; B – II; C – III; D – IV; E – V; F – VI eyelashes are formed
(b) A – III; B – I; C – IV; D – II; E – V; F – VI (a) A – I; B – II; C – III; D – IV; E – V
(c) A – VI; B – IV; C – V; D – II; E – I; F – III (b) A – III; B – I; C – IV; D – II; E – V
(d) A – II; B – IV; C – III; D – V; E – I; F – VI (c) A – II; B – I; C – III; D – V; E – IV
78. In the given columns, column-I contain various phases (d) A – III; B – IV; C – II; D – V; E – I
of menstrual cycle and column -II contain its features. 80. Match the hormones given in column-I with their
Select the correct match from the options given below.
functions given in column-II and select the correct
Column-I Column-II option.
(Phases of menstrual (Features)
Column-I Column-II
cycle)
A. Menstrual phase I. Breakdown of endometrial (Hormones) (Functions)
lining of uterus along with A. Luteinizing I. Develop corpus luteum
its blood vessels which hormone
form liquid that comes out B. Progesterone II. Essential for maintenance
of vagina. of uterine layer (called
B. Luteal phase II. A temporary endocrine endometrium)
gland is formed and C. Estrogen III. Develops female
secretes a hormone which secondary sexual
maintains endometrium
characters
and implantation of
EBD_7209
218 Biology
D. Follicle stimulating IV. Maturation of Graafian
hormone follicle DIAGRAM TYPE QUESTIONS
E. Oxytocin V. Causes uterine contraction. 83. The given figure shows the male reproductive system.
(a) A – I; B – V; C – III; D – II; E – IV Some structures are marked as A, B, C, and D.
(b) A – III; B – I; C – II; D – IV; E – V Identify the structure whose removal will cause the
sperm to be reacted with acidic urine in the urethra.
(c) A – I; B – II; C – III; D – IV; E – V
(d) A – I; B – II; C – III; D – V; E – IV (a) A
81. Match the column-I with column-II and select the correct A

option. (b) B B
C
Column-I Column-II
A Fertilization I. Mitotic division (c) C
B Implantation II. Embryo with 8 to 16
blastomeres (d) D D

C Cleavage III. Ampullary-isthmic junction 84. Given below is a diagrammatic sketch of a portion of
D Morula IV. Structure formed by the human male reproductive system. Select the correct set
continuous division of 8 to of the names of the parts marked as A, B, C, and D
16 blastomeres respectively.
E Blastocysts V. Embedding of blastocysts
in the endometrium
A
B
(a) A – I; B – II; C – IV; D – V; E – III
C
(b) A – III; B – I; C – IV; D – II; E – V D

(c) A – III; B – V; C – I; D – IV; E – II


(d) A – III; B – V; C – I; D – II; E – IV
82. Select the correct match of terms given in column-I with
their definition given in column -II.
Column-I Column-II A B C D
(a) Ureter Seminal Prostate Bulbourethral
(Terms) (Definition)
vesicle gland
A. Parturition I. Duration between
(b) Ureter Prostate Seminal Bulbourethral
pregnan cy (of about 9
month) and birth vesicle gland
B. Ovulation II. Attachment of zygote to the (c) Vas Seminal Prostate Bulbourethral
endometrium deferens vesicle gland
C. Gestation III. Childbirth (d) Vas Seminal Bulbourethral Prostate
deferens vesicle gland
D. Implantation IV. Stoppage of menstruation
85. Identify the figure (A) whose sectional view is given
E. Conception V. Release of egg from
below and match with its characteristics (B) and its
Graafian follicle
location (C).
VI. Process of milk secretion
VII.Formation of zygote by
fusion
of the egg and sperm
(a) A – I; B – II; C – VII; D – V; E – III
(b) A – III; B – I; C – IV; D – II; E – V
(c) A – III; B – V; C – I; D – II; E – VI
(d) A – III; B – V; C – I; D – IV, E – II
Human Reproduction 219

Which of the following options shows the correct labeling


A B C
of A–F?
Involved in (a) A®Myometrium, B®Isthmus, C®Endometrium,
Graafian
(a) the formation Ovary D® Perimetrium, E®Ampulla, F® Infundibulum
follicle
of ovum (b) A®Infundibulum, B®Perimetrium,
Involved in C®Endometrium, D®Myometrium,
Seminiferous
(b) the formation Testis E®Ampulla, F®Isthmus
tubule
of sperm (c) A®Endometrium, B®Myometrium,
C®Perimetrium, D®Isthmus, E®Ampulla,
Ovum
Process of Graafian F®Infundibulum
(c) surrounded
fertilization follicle (d) A®Perimetrium, B®Endometrium, C®Isthmus,
by sperm
D®Infundibulum, E®Ampulla, F®Myometrium
Female 88. Refer the figure of mammary gland with few structures
Mammary Involved in milk
(d) reproductive marked as A, B, C and D. Which structure contains
gland secretion
system clusters of milk secreting cells?
86. Given below is the diagrammatic sectional view of
A
seminiferous tubule with their parts marked as A, B, C,
and D. Select the option which shows the correct
identification of the structure with its characteristics. B
C
D

(a) A (b) B.
D (c) C (d) D
(a) A: Spermatozoa, secretes testicular hormones that
control spermatogenesis. 89. The figure given below shows the sectional view of
(b) B: Spermatogonium, it is also called male germ seminiferous tubule.
cells which undergo meiotic division to from
spermatozoa.
(c) C: Interstitial cells, present in the interstitial spaces
and store and transport the sperms from the testis A
to the outside through the urethra.
B
(d) D: Sertoli cells, it maintains low temperature of
the testis.
87. The given figure shows the diagrammatic sectional view C
of female reproductive system with few structures
marked as A, B, C, D, E and F.
D

A Which marked structure (A to D) undergoes second


B
C meiotic division to produce four equal haploid cells
(called spermatids)?
(a) A (b) B
(c) C (4) D
EBD_7209
220 Biology
90. The figure given below shows the structure of sperm.
Identify the correct feature corresponding to the marked
structure A, B, C and D.

A
A
B

B C
C D

(a) A: Tertiary; B: I; C: Fertilization; D: Primary oocyte.


(b) A: Primary; B: II; C: Ovulation; D: Secondary oocyte.
(a) A – Head: Its anterior portion is covered by a (c) A: Secondary; B: II; C: Fertilization; D: Primary
structure filled with enzymes that help in the fusion oocyte.
of male and female gametes. (d) A: Primary; B: I; C: Ovulation; D: Secondary oocyte.
(b) B – Middle piece: It contains a haploid nucleus. 93. The figure given below shows the various events
(c) C – Neck: It possesses few ribosomes which occurring during a menstrual cycle with few structures
produces energy for the process of fertilization. marked as A, B, C and D. Which of the following options
shows the correct labeling?
(d) D – Tail: It releases energy source for swimming of
sperm.
91. The figure given below shows the sectional view of A
ovary. Select the option which gives correct
identification of marked structure (A to D) and its feature.
A B

C
B

(a) A: Primary follicle, it is also called gamete mother C D E

cell. (a) A®LH, B®Ovulation, C® Menstruation,


(b) B: Corpus luteum, it cannot be formed and added D® Proliferative phase, E®Luteal phase
after birth. (b) A®FSH, B®Implantation, C®Follicular phase,
(c) C: Graafian follicle, mature follicle which ruptures D®Menstruation phase, E®Luteal phase
to release secondary oocyte. (c) A®Estrogen, B®Parturition, C®Luteal phase,
(d) D: Tertiary follicle, a large number of this follicle D®Follicular phase, E®Follicular phase
degenerates during the phase from birth to puberty. (d) A®Progesterone, B®Fertilization, C®
92. The given figure shows the diagrammatic representation Menstruation phase, D®Secretory phase, E®
of oogenesis. Identify the option which shows the correct Follicular phase
label marked as A, B, C and D.
Human Reproduction 221

94. In the given figure the structure of ovum is surrounded 96. The given figure shows the human foetus within the
by few sperms and some art are labelled as A, B, C and D. uterus with few structures marked as A, B, C and D.
B

C
A

D
Which of the following options shows the correct
labelling?
(a) A®Zona pellucida, B®Ovum , C®Cells of corona Which of the following options shows the correct
radiata, D®Perivitelline space labeling?
(b) A®Perivitelline space, B®Antrum, C®Zona (a) A®Umbilical cord with its veins, B® Chorionic villi,
pellucida D®Ovum C®Antrum, D®Plug of mucus in cervix
(c) A®Zona pellucida, B®Ootid, C®Cells of corona
radiata D®Perivitelline space (b) A®Umbilical cord with its vessels, B®Fimbriae,
(d) A®Cells of corona radiata, B®Morula, C® C® Oocyte, D®Plug of mucus in vagina
Perivitelline space D®Zona pellucida
(c) A®Umbilical cord with its vessels, B®Placental villi
95. Which of the following human developmental stage
becomes embedded in the uterine endometrium by a C®Yolk sac, D®Plug of mucus in cervix
process called implantation and leads to pregnancy? (d) A®Umbilical cord with its veins, B®Placental villi
C®Trophoblast, D®Plug of mucus in vagina
97. Study the given figure and conclude the correct
explaination from the options given below:

(a) (b)

(c) (d)

(a) All the sperm attaches with ovum.


(b) Transport of sperm towards the ovum.
(c) Ovum and surrounded sperms were going to take
part in fertilization.
(d) Sperm induces changes in the cells of corona radiata
and blocks the entry of other additional sperms.
EBD_7209
222 Biology
98. Identify the structure marked as “X” and its function in
the given figure of male reproductive system.
A

(a) “X” - LH, A (b) “X” - GH, B


(a) Rete testis: It helps seminiferous tubule to open into
(c) “X” - ACTH, C (d) “X” - FSH, D
vas efferentia.
(b) Bulbourethral gland: It secretes alkaline mucus for
101. Study the given figure and identify the correct event
lubricating the reproductive tract. occuring in this.
(c) Vas efferentia: They have contractile mechanism that
aids in the emission of seminal fluid.
(d) Seminal vesicle: It synthesizes and secrete testicular
hormone.
99. The given figure shows the diagrammatic sectional view
of female reproductive system with few structures marked
as A, B, C, and D.
Select the option which shows the correct identification
of the structure with its characteristics.

(a) Role of pituitary hormones levels.


A
(b) Events occurring in uterine tissues.
B
(c) Role of ovarian hormone levels and growth of
C ovarian follicles.
(d) Both (a) and (c).
D
102. The given figure shows the human foetus within the
uterus. Identify the marked label (A to D) through which
placenta is connected to the embryo.
B

(a) A: Infundibulum, funnel shaped structure


surrounded by finger like projection.
(b) B: Ampulla, wider part of oviduct where fertilization C
A
occurs.
(c) C: Isthmus, it has a narrow lumen and joins with
uterus.
(d) D: Fimbriae, it collects ovum before ovulation.
100. “X” is a hormone which is secreted under the influence D
of GnRH. Identify “X” and the correct marked structures
(A to D) from the figure given below on which “X” acts (a) A (b) B
to stimulate secretion of some factors to help in (c) C (d) D
spermiogenesis.
Human Reproduction 223

103. The given figure represent a stage of embryonic


development. Identify the stage with its feature.
B

(a) Blastocysts, ready to fertilize with sperm.


C
(b) Secondary oocyte, implants on endometrial layer of
uterus.
(c) Morula, formed by mitotic division of zygote. D
(d) Ovary, produce female gamete and secretes
(a) A (b) B
hormones like estrogen etc
(c) C (d) D
104. In the given figure, which two marked structure (A to D)
is present in the lining of seminiferous tubule? 107. The figure given below shows a flowchart on
spermatogenesis. Identify the correct label marked as A,
B, C and D.

B A

C
B

D
D
C
(a) A and D (b) B and D
(c) C and D (d) A and B
105. The given figure shows the structure of sperm whose tail
enables it to swim. Why the sperm does needs to swim?
(a) A: Ist meiotic division; B: 2nd meiotic division; C:
Differentiation; D: 23.
(b) A: 2nd meiotic division; B: Differentiation; C: Ist
meiotic division; D: 46.
(c) A: Differentiation; B: 2nd meiotic division; C: Ist
meiotic division; D: 46.
(d) A: Mitosis differentiation; B: Ist meiotic division;
C: 2ndmeiotic division; D: 23.

CRITICAL THINKING TYPE QUESTIONS


(a) In order to get the ovum for fertilization. 108. The feature of some structures of male reproductive
(b) It helps the foetus to come out of the uterus. system is given below. Identify the structure on the
(c) It helps blastocysts to attach in endometnum in the basis of the characteristics which surrounds the primary
process of implantation. sex organ of male reproductive system.
(d) It helps the sperm to enter into the cytoplasm of (a) Its enlarged end is called glans penis.
the ovum through the zona pellucida and plasma (b) It travels through the penis and carry semen as well
membrane. as urine.
106. The given diagram shows a human female's reproductive (c) It is responsible for maintaining the low temperature
system. Select the part (labelled as A, B, C & D) where by about 2 – 2.5º C from normal body temperature to
semen is released into the female by the penis during mature sperm.
copulation.
(d) Stores sperms prior to ejaculation.
EBD_7209
224 Biology
109. Read the following statement and answer the question. 116. Study the given statement and answer the question.
“The urethra originates from a structure (called ‘X’) “During ‘P’ phase of the menstrual cycle, if pregnancy
and extends through the male external genitalia (called doesn’t happen, the ‘Q’ withers and dies, usually around
‘Y’ which helps in introducing semen into the vagina) day 22 in a 28-day cycle. The drop in ‘R’ levels causes the
to its external opening called urethral meatus.” lining of the uterus to fall away. This is known as ‘S’.
Identify X and Y. Identify P, Q, R and S.
(a) X - Urinary bladder ; Y - Penis (a) P ® Menstrual, Q ® Graafian follicle, R® Estrogen,
S ® Menarche
(b) X - Vas efferentia ; Y - Penis
(b) P ® Luteal, Q ® Corpus luteum, R ® Progesterone
(c) X - Ejaculatory duct ; Y - Ureter
S ® Menstruation
(d) X - Bulbourethral gland ; Y - Ureter (c) P ® Ovulatory, Q ® Endometrium, R ® Follicle
110. Which of the following is not a uterine function? stimulating hormone, S ® Menopause
(a) Waste removal for the developing embryo. (d) P ® Follicular, Q ® Secon dary oocyte,
(b) Nutritional support of the growing embryo. R ® Luteinizing hormone S ®Menstruation
(c) Place of fusion of male and female gametes. 117. Which of the following hormones attains a peak level in
(d) Mechanical protection of the developing embryo. the middle of menstrual cycle?
111. Milk secreted from the cells of alveoli of mammary (a) LH and estrogen
lobes reaches nipple through lactiferous duct(L), (b) FSH and progesterone
mammary duct (M), mammary tubule (T) and mammary (c) FSH and LH
ampulla (A) in the following order. (d) Estrogen and progesterone
(a) TMAL (b) MTLA 118. Select the correct sequence of menstrual cycle.
(c) MTAL (d) ATML (a) Menstruation, Secretory, Follicular, New cycle.
112. A gonadotropin hormone, “X” acts on interstitial (b) Menstruation, Follicular, Luteal, New cycle.
cells and stimulates synthesis and secretion of “Y”. (c) Follicular, Menstruation, Luteal, New cycle.
Identify X and Y from the given option. (d) Luteal, Menstruation, Follicular, New cycle.
(a) X - LH ; Y - Androgen 119. Menstrual cycle is controlled by
(b) X - FSH ; Y - Testosterone (a) LH and FSH only
(c) X - TSH ; Y - Progesterone (b) estrogen, LH & FSH only
(d) X - GH ; Y - Estrogen (c) estrogen & progesterone only
113. Secretion of which of the following are essential for (d) LH, FSH, estrogen & progesterone.
maturation and motility of sperm? 120. Which of the following is the first change that occurs to
(a) Ureter, vas deferens, urinary bladder and prostate. the zygote after fertilization?
(a) It divides to form a hollow ball of cells, called the
(b) Seminal vesicle, vas deferens, ejaculatory ducts and
blastocyst.
ureter.
(b) It begins to secrete the hormones.
(c) Epididymis, seminal vesicle, vas deferens and
(c) It contacts the endometrial wall of the uterus and
prostate.
becomes buried inside it.
(d) Epididymis, ejaculatory ducts, vas efferentia and (d) It initiates the formation of a placenta.
seminal vesicle.
121. Fertilization can only occurs if
114. Which of the following shows the correct sequence of
(a) sperm reaches to the ampullary – isthmic junction
events leading to the formation of mature sperm? before the ovum.
(a) Spermatogonium ® Secondary spermatocyte ® (b) ovum reaches to the ampullary – isthmic junction
Primary spermatocyte ® Spermatids ® Sperms. before the sperm.
(b) Spermatogonium ® Spermatids ® Secondary (c) sperms are transported to the uterus and ovum to
spermatocyte ® Primary spermatocyte ® Sperms. the fallopian tube simultaneously.
(c) Spermatids ® Primary spermatocyte ® Secondary (d) sperm and ovum are transported simultaneously to
spermatocyte ® Spermatogonium ® Sperms. the ampullary – isthmic junction.
(d) Spermatogonium ® Primary spermatocyte ® 122. Which of the following does not occur in the time
Secondary spermatocyte ® Spermatids ® Sperms. during and immediately following fertilization?
115. How many ova are released during the middle of the (a) Fusion of the sperm and ovum nuclei.
menstrual cycle? (b) Division of the oocyte cell by meiosis
(a) One (b) Two (c) Implantation of the ovum in the uterus.
(c) Three (d) Four
(d) Digestion of cell layers around the oocyte by sperm.
Human Reproduction 225

123. The acrosome enables the sperm to (c) Middle piece: Produces energy from mitochondria
(a) help in motility. for tail movement which facilitate sperm motility.
(b) produce energy for activity. (d) Tail: Help in fertilization with the help of enzyme
present in acrosome.
(c) penetrate vitelline membrane of ovum.
131. “A” cells start division and enter in “B” stage of meiotic
(d) fertilize more than one ovum. division and get temporarily “C” at this stage, called “D”.
124. At the time of fertilization, chromosome number Identify A, B, C and D.
(a) is halved (b) remains haploid (a) A: Oogonia; B: Metaphase I; C: Arrested; D: Primary
oocyte.
(c) becomes diploid (d) does not change
(b) A: Oogonia; B: Anaphase I; C: Released; D:
125. Foetal ejection reflex in human female is induced by Secondary oocyte.
(a) placenta only. (c) A: Oogonia; B: Prophase I; C: Arrested; D: Primary
(b) fully developed foetus and placenta. oocyte.
(c) release of oxytocin from pituitary gland. (d) A: Oogonia; B: Telophase I; C: Released; D:
(d) release of full developed corpus luteum. Secondary oocyte.
126. Study the statement given below and answer the question. 132. What happens during fertilization in humans after many
“Vigorous contraction of the ‘X’ at the end of the ‘Y’ sperms reach close to the ovum?
causes expulsion of the foetus.” Identify X and Y. (a) Cells of corona radiata trap all the sperms except one
(a) X -Vagina ; Y - Fertilization (b) Only the closest sperm to the ovum penetrates the
(b) X - Uterus ; Y - Pregnancy zona pellucida.
(c) X - Placenta ; Y - Implantation (c) Secretions of acrosome helps one sperm enter
(d) X - Embryo ; Y - Ovulation cytoplasm of ovum through zona pellucida and
127. Each spermatogonium which is diploid contains how plasma membrane.
many chromosomes? (d) All sperms except the one nearest to the ovum lose
(a) 23 (b) 26 their tails.
(c) 46 (d) 48 133. Study the following statements and answer the question.
128. Which of the following group of cells involved in In a process called ‘A’, ‘B’ division starts as the zygote
spermatogenesis represent haploid cells? moves through the ‘C’ of the ‘D’ towards the ‘E’.
(a) Spermatogonium Identify A, B, C, D and E.
(b) Primary spermatocyte
(a) A®Blastulation, B®Meiotic, C®Ampulla ,
(c) Both (a) and (b) D®Fallopian tube, E®Uterus
(d) Secondary spermatocyte
(b) A®Parturition, B®Meiotic, C®Infundibulum,
129. After the transformation of spermatids into sperm, D®Uterus, E®Vagina
their heads become embedded in a cell called “X” and
(c) A®Implantation, B®Mitotic, C®Fimbriae, D®
are finally released from the “Y” by the process called
Ovary E®Cervix
“Z”. Identify X, Y and Z.
(d) A®Cleavage, B® Mitotic, C®Isthmus, D®
X Y Z Oviduct E®Uterus
(a) Spermatogonium Epididymis Insemination 134. What will happen to pregnancy if placenta fails to
(b) Leydig Vas deferens Parturition function during the gestation?
Seminiferous (a) The pregnancy would not continue.
(c) Sertoli Spermiation
tubule (b) The foetus would be born prematurely.
Seminiferous (c) There would be no effect on the pregnancy.
(d) Spermatocyte Spermiogenesis
tubule (d) The corpus luteum would continue produce hormone
as an alternative source until birth.
130. Select the option which shows the correct part of a
sperm with its corresponding function.
(a) Head: Stimulate Leydig cell to produce androgen
hormone.
(b) Neck: Essential for maturation and motility of sperm.
EBD_7209
26

Chapter
Reproductive Health

FACT/DEFINITION TYPE QUESTIONS (b) Lactational amenorrhoea


(c) Condoms
1. According to which of the following organization (d) Sterilization
"reproductive health means a total well-being in all aspects 9. Which of the following contraceptive also provides
of reproduction"? protection from contacting STDs and AIDS ?
(a) WHL (b) UNESCO
(a) Diaphragms
(c) WHO (d) WWW
2. Which one amongst the following is the first country in (b) Spermicidal foams
the world to initiate action plans and programmes at a (c) Condoms
national level to attain total reproductive health as social (d) Lactational amenorrhoea
goal? 10. The diaphragm, cervical cap and vaults are
(a) China (b) India (a) disposable contraceptive devices
(c) Japan (d) USA (b) Reusable contraceptives
3. The family planning programmes in India were initiated in (c) Non-medicated IUDs
(a) 1951 (b) 1961 (d) Cu-releasing IUDs
(c) 1971 (d) 1981 11. Which of the following groups of contraceptives are
4. RCH stands for included under barrier methods?
(a) Routine Check-up of Health (a) Condom, Vaults, IUDs, Cervical caps
(b) Reproduction Cum Hygiene (b) IUDs, Diaphragm, Abstinence, Injections
(c) Reversible Contraceptive Hazards (c) Condom, Diaphragm, Cervical caps, Vaults
(d) Reproductive and Child Health Care (d) Surgical methods, Oral contraceptives, Cervical caps,
5. The technique which makes use of amniotic fluid for the vaults
detection of prenatal disorder is called as 12. IUDs stands for
(a) leproscopy (b) amniocentesis (a) Intra Uterine Devices
(c) endoscopy (d) ultrasound (b) Internal Uterine Devices
6. ‘Saheli’ a new oral contraceptive developed by (c) Inseminated Uterine Devices
(a) All Indian Institute of Medical Science (d) Injected Uterine Devices
(b) Central Drug Research Institute 13. Which of the following ions plays an effective role in the
(c) Health Care Pvt. Ltd. activity of IUDs?
(d) Bharat Immunologicals & Biologicals corp. Ltd. (a) Iron (b) Zinc
7. What is the marriageable age for the females and males (c) Copper (d) Ammonium
respectively in India? 14. Which of the following contraceptives is one of the most
(a) 18, 18 (b) 18, 25 widely accepted method of contraception in India?
(c) 21, 18 (d) 18, 21 (a) Sterilization
8. Which of the following is traditional method of (b) Intra Uterine device
contraception? (c) Withdrawal or coitus interruption
(a) Implantation (d) Medical termination of pregnancy
Reproductive Health 227

15. Progestasert and LNG-20 are 26. Artificial insemination mean


(a) implants (a) transfer of sperms of husband to a test tube
(b) copper releasing IUDs containing ova.
(b) artificial introduction of sperms of a healthy donor
(c) non-medicated IUDs
into the vagina.
(d) hormone releasing IUDs
(c) introduction of sperms of a healthy donor directly
16. The most important component of the oral contraceptive into the ovary.
pills is (d) transfer of sperms of a healthy donor to a test tube
(a) progesterone (b) growth hormone containing ova.
(c) thyroxine (d) luteinizing hormone 27. The technique called gamete intra fallopian transfer (GIFT)
17. Tying up or removing a small part of fallopian duct is is recommended for those females
called (a) who cannot produce an ovum.
(a) vasectomy (b) ductus arteriosus (b) who cannot retain the foetus inside uterus.
(c) archidectomy (d) tubectomy (c) whose cervical canal is too narrow to allow passage
for the sperms.
18. In which year, MTP was legalised in India?
(d) who cannot provide suitable environment for
(a) 1971 (b) 1951
fertilization.
(c) 1981 (d) 1923
19. MTPs are considered relatively safe during the ______ STATEMENT TYPE QUESTIONS
weeks of pregnancy.
28. Which of the following statement is incorrect?
(a) 12 (b) 15
(a) According to 2001 census our population growth
(c) 18 (d) 20
rate was 1.7%.
20. STD/VD/UTI are
(b) Marriageable age for male and female is respectively
(a) devices which are used to delay the pregnancy.
18 and 21 years.
(b) infections which are caused by food contamination.
(c) An ideal contraceptive should be reversible.
(c) diseases which are transmitted through sexual
intercourse. (d) The problem of infertility in India lies most often in
(d) action plans and programmes to create awareness female partner.
about various reproductive related health and 29. Which of the following statement regarding natural
problems. methods of contraception is true?
21. Hepatitis B is transmitted through (a) They increase phagocytosis of sperms.
(a) blood transfusion (b) They employ barriers to prevent fertilization.
(b) intimate physical contact (c) They are the natural ways of avoiding chances of
fertilization.
(c) Sexual contact
(d) They are surgical and terminal methods.
(d) All of these
30. Which one of the following statements is correct regarding
22. World AIDS day is Sexually Transmitted Diseases (STD)?
(a) December 21 (b) December 1 (a) The chances of a 5 year boy contacting a STD are
(c) November 1 (d) June 11 very little.
23. To form embryo in vitro, the male gamete is transferred (b) A person may contact syphilis by sharing milk with
into female gamete directly. Such technique is the one who is already suffering from the same
called_________. disease.
(a) IUI (b) IUT (c) Haemophilia is one of the STD.
(c) ICSI (d) GIFT (d) Genital herpes and sickle-cell anaemia are both STD.
24. The transfer of zygote or early embryo (up to 8 31. Read the following statements and mark the correct option
blastomeres) into fallopian tube is called _________. (i) MTP was legalized in 1971.
(a) IVF and ET (b) ZIFT (ii) Inability to conceive or produce children even after
(c) GIFT (d) IUT 2 years of unprotected sexual cohabitation is called
infertility.
25. The Test-tube Baby Programme employs which one of
(iii) Surgical method of contraception prevents gamete
the following techniques?
formation.
(a) Intra Cytoplasmic Sperm Injection (ICSI)
(iv) MTPs are relatively safe up to 14 weeks of pregnancy.
(b) Intra Uterine Insemination (IUI)
(a) (i) and (ii) (b) (ii) and (iii)
(c) Gamete Intra Fallopian Transfer (GIFT)
(d) Zygote Intra Fallopian Transfer (ZIFT) (c) (iii) and (iv) (d) (i) and (iii)
EBD_7209
228 Biology
32. Choose the correct statements given below regarding (vi) Encouraging myths and misconceptions.
contraception. (a) All of these (b) (i), (ii), (iv) and (vi)
(i) Medical Termination of Pregnancy (MTP) during first (c) (i), (ii), (iii), (iv) and (v) (d) (ii) and (v)
trimester is generally safe. 37. Identify the incorrect statements and select the correct
(ii) Generally chances of conception are nil until mother option.
breast-feeds the infant upto two years. (i) Birth control pills are likely to cause cardiovascular
(iii) Intrauterine devices like copper-T are effective problems.
contraceptives. (ii) A woman who substitutes or takes the place of the
(iv) Contraception pills may be taken upto one week after real mother to nurse the embryo is called surrogate
coitus to prevent conception. mother
(a) (i) and (ii) (b) (ii) and (iii) (iii) Numerous children have been produced by in vitro
(c) (iii) and (iv) (d) (i) and (iii) fertilization but with some abnormalities.
33. What is true for an ideal contraceptive ? (iv) Woman plays a key role in the continuity of the family
(i) It should be user-friendly. and human species
(ii) It should be easily available.
(v) Foetal sex determination test should not be banned.
(iii) It should be ineffective and reversible with least side
effects. (a) (i) and (ii) (b) (ii) and (iv)
(iv) It should be effective and reversible with least side (c) (iii) and (v) (d) None of these
effects. 38. Which of the following are the reasons for population
(v) It should interfere with the sexual act of the user. explosion?
(a) All of these (b) (i), (ii) and (iii) (i) Increased health facilities
(c) (i), (ii) and (iv) (d) (i),(ii), (iv) and (v) (ii) Rapid increase in MMR
34. Which of the following statement are correct? (iii) Rapid increase in IMR
(i) Purpose of tubectomy is to prevent egg formation. (iv) Rapid decrease in MMR
(ii) The most important component of the oral (v) Decrease in number of people reaching reproducible
contraceptive pills is progesterone. age
(iii) Contraceptive oral pills help in birth control by (a) (i) and (iv) (b) (iii) and (v)
preventing ovulation. (c) (ii) and (iv) (d) (i) and (v)
(iv) Genital warts is a sexually transmitted disease caused 39. To avoid transmission of STDs, we should
by herpes virus. (i) Avoid sex with multiple partners.
(v) In India, there is rapid decline in infant mortality rate (ii) Always have unprotected sex.
and maternal mortality rate. (iii) Use condoms during coitus.
(a) (i), (ii) and (iii) (b) (ii), (iii) and (v)
(iv) Avoid sex with unknown partners.
(c) (iii), (iv) and (v) (d) (iv), (v) and (vi)
(v) Avoid sharing of needles.
35. Choose the correct statements –
(a) (i), (ii), (iii) and (v)
(i) According to the WHO, reproductive health is total
(b) (i), (iii), (iv) and (v)
well-being in the physical, social, emotional,
behavioural aspects of reproduction. (c) (i), (ii) and (iii)
(ii) According to the WHO, reproductive health is total (d) (i), (ii) and (iv)
well being in the physical, social and emotional 40. Identify the incorrect statement regarding ZIFT.
aspects of reproduction. (a) ZIFT is zygote intra fallopian transfer.
(iii) A reproductively healthy society has people with (b) It is one of the techniques known as assisted
physically and functionally normal reproductive reproductive technologies.
organs. (c) Through this process embryo is formed by injecting
(iv) Reproductively healthy societies have abnormal sex- ovum into the sperm.
related emotional and behavioural interactions. (d) Zygote or embryo up to 8 blastomeres is collected
(a) (i), (ii) and (iii) (b) (ii) and (iv) and transferred into the fallopian tube.
(c) (i) and (iii) (d) (i) only 41. Which of the following statements regarding IUDs is
36. Reproductive health in society can be improved by correct?
(i) Introduction of sex education in schools. (a) It suppresses the process of gametogenesis.
(ii) Increased medical assistance. (b) They once inserted need not be replaced.
(iii) Awareness about contraception and STDs. (c) They are generally inserted by the user itself.
(iv) Equal opportunities to male and female child. (d) It increases phagocytosis of sperms within the
(v) Ban on aminocentesis. uterus.
Reproductive Health 229

42. Select the correct statements regarding diaphragm as 49. Assertion : Copper-T is an effective contraceptive device
contraceptive device from the given options. in human females.
(i) They act as physical barrier for sperm entry. Reason: Copper-T prevents passage of sperms from
(ii) They are placed to cover the cervical region during vagina upwards into Fallopian tubes.
coitus.
(iii) They are introduced into the uterus to prevent MATCHING TYPE QUESTIONS
implantation. 50. Given below are four methods (A-D) of contraceptive in
(iv) They act as spermicidal agents to increase their column-I and their modes of action in achieving
contraceptive effectiveness contraception in column-II. Match the columns and select
(a) (i), (ii) only (b) (i), (iv) only the correct option :
(c) (ii), (iii) only (d) (iii), (iv) only Column -1 Column -II
43. Identify the correct function(s) of pills (Method) (Mode of Action)
(i) Inhibit ovulation and implantation. A. The contraceptive I. Prevents sperms
(ii) Alter the quality of cervical mucus to prevent or pill reaching cervix
retard the entry of sperms. B. Condom II. Prevents implantation
(iii) Prevent the ejaculated semen from entering the female C. Vasectomy III. Prevents ovulation
vagina. D. Copper T IV. Semen contains no
(iv) Inhibit spermatogenesis. sperms
(a) (i), (ii) and (iii) (b) (i) and (ii) (a) A – II; B – III; C – I; D – IV
(c) (ii), (iii) and (iv) (d) (iii) and (iv) (b) A – III; B – I; C – IV; D – II
44. Select the correct statements regarding MTP from the (c) A – IV; B – I; C – II; D – III
given options. (d) A – III; B – IV; C – I; D – II
(i) Always surgical. 51. Column I contains different types of IUD’s with their
(ii) Used as a contraceptive method. examples given in column II. Match the column and choose
(iii) Generally suggested during first trimester. the correct option
(a) (i) only (b) (i), (ii) only
Column-I Column-II
(c) (i), (iii) only (d) All of these.
A. Non-medicated IUDs I. Lippes loop
ASSERTION/REASON TYPE QUESTIONS B. Hormone releasing IUDs II. Multiload 375
C. Copper releasing IUDs III. CuT
In the following questions, a statement of Assertion is followed
IV. Cu7
by a statement of Reason.
V. LNG - 20
(a) If both Assertion and Reason are true and the Reason is
the correct explanation of the Assertion. VI. Progestasert
(b) If both Assertion and Reason are true but the Reason is (a) A – I; B – II; VI; C – III; IV; V
not the correct explanation of the Assertion. (b) A – I; B – V; VI; C – II; III; IV
(c) If Assertion is true but Reason is false. (c) A – II; B – III; VI; C – I; V; IV
(d) If both Assertion and Reason are false. (d) A – II; B – I; VI; C – III; IV; V
45. Assertion : Mother would be blamed for the birth of the 52. Match the column I with column II and choose the correct
girls in the family. option
Reason : Father is not responsible for the sex of the child. Column-I Column-II
46. Assertion : Amniocentesis is often misused. (A) Hepatitis B I. Vitamin E
Reason : Amniocentesis is meant for determining the (B) Saheli II. 7’April, 1948
genetic disorders in the foetus, but it is being used to (C) Normal functioning III. CDRI, Lucknow
determine the sex of the foetus, to kill the normal female of reproductive organs
foetus. (D) World Health IV. Detection of antibody/
47. Assertion : Over-population has become a serious organisation antigen
problem in the developing countries. (E) ELISA technique V. Hepatitis B virus
Reason : It does not exhaust natural resources, causes (a) A – V; B – III; C – I; D – II; E – IV
unemployment and pollution. (b) A – V; B – II; C – I; D – III; E – IV
48. Assertion : Cu-T and Cu-7 do not suppresses sperm- (c) A – V; B – III; C – IV; D – II; E – I
motility. (d) A – V; B – II; C – IV; D – III; E – I
Reason : Hormones released by them affect sperm motility.
EBD_7209
230 Biology
53. Select the correct match of the techniques given in column 57. Identify the figure given below.
I with its feature given in column II.
Column I Column II
A. ICSI I Artificially introduction of semen into the
vagina or uterus.
(a) Male condom (b) Female condom
B. IUI II Transfer of ovum collected from a donor
into the fallopian tube where fertilization
(c) Norplant (d) Copper T
C. IUT occur
III Formation of embryo by directly injecting
58. Which of the following option is correct regarding the
sperm into the ovum diagram given below?
D. GIFT IV Transfer of the zygote or early embryo
(with upto 8 blastomeres) into a fallopian
E. ZIFT tube.
V Transfer of embryo with more than 8
blastomeres into the uterus
(a) A – V; B – IV; C – I; D – III; E – IV
(b) A – I; B – II; C – III; D – IV; E – V
(c) A – III; B – V; C – II; D – IV; E – I
(a) It is a device made of rubber and inserted into the
(d) A – III; B – I; C – V; D – II; E – IV
female reproductive tract to cover the cervix during
54. Select the correct match from the given option. coitus.
(a) Non Medicated IUDs - Multiload 375
(b) It is a device made of thin rubber/ latex sheath and
(b) Saheli - Contains a non - steroidal preparation
are used to cover penis in the male.
(c) Lactational amenorrhea - Presence of menstruation
(d) Diaphragms, cervical caps, vaults - Cover the cervix (c) This device is inserted by doctors in the uterus
after the coitus is done through vagina and increases phagocytosis of
sperms within the uterus.
DIAGRAM TYPE QUESTIONS (d) It is a set of 6-small plastic capsules (called implant)
which are placed under the skin of a women’s upper
55. The process done in the given figure
arm and it prevent pregnancy.
59. In the given figure which marked number (1, 2, 3, 4) is tied
and cut to block fertilization

(a) Prevents egg from reachin g th e uterus for


implantation. 1
(b) Avoid insemination
(c) Inhibits ovulation
(d) Increases contraceptive efficiency
56. The given diagram shows the uterine tubes of four women 2
(P, Q, R and S).
3

(a) 1 (b) 2
(c) 3 (d) 4
60. Refer the given figure below and answer the question.
Which feature is correctly associated with the given
figure?

In which two women is fertilization impossible at present ?


(a) P and Q (b) Q and R
(c) R and S (d) S and P
Reproductive Health 231

(a) It is a male condom which is used to cover the penis 64. Which of the following disease is completely curable if
just before the coitus to prevent the entry of detected early and treated properly?
ejaculated semen into the female reproductive tract. (a) Syphilis (b) Hepatitis B
(b) It is a female condom which is used to cover the (c) Genital herpes (d) HIV infection
cervix and vagina just before the coitus. 65. Present increase in India’s population has not been due
(c) It is a condom which is used to cover penis in male to decline in
and vagina and cervix in female. (a) decrease in infant mortality rate
(d) It is one type of IUDs which makes the uterus (b) decrease number of people reaching reproductive
unsuitable for implantation and cervix hostile to the age
sperms. (c) decline in death rate
61. The given figure shows one of the elements releasing (d) decline in maternal mortality rate
intrauterine device. Select the option which shows the 66. The copper ions of IUDs
correct identification of the device and its feature. (a) suppress the motility and fertilization capacity of
sperms.
(b) make the uterus unsuitable for implantation.
(c) increase phagocytosis of sperms.
(d) make cervix hostile to sperms.
67. Progestogens in the contraceptive pill
(a) prevents ovulation
(b) inhibits estrogen
(c) checks attachment of zygote endometrium
(d) All of the above
68. Which of the following birth control measures can be
considered as the safest?
(a) The rhythm method
(b) The use of physical barriers
(a) CuT; suppress sperm motility and its fertilizing (c) Temination of unwanted pregnancy
capacity. (d) Sterilization techniques
(b) Cu7; make uterus unsuitable for the attachment of 69. The success of birth control programmes in controlling
blastocysts. population growth is dependent on
(c) Lippes loop; protect the users from contracting AIDS (a) use of contraceptives
and STDs. (b) tubectomy
(d) LNG - 20; acts as spermicidal means and decrease (c) vasectomy
the contraceptive efficiency. (d) acceptability of the above by the people
70. Assisted reproductive technologies (ART)
CRITICAL THINKING TYPE QUESTIONS (a) include social awareness programmes to educate
62. On which days of the menstrual cycle is ovulation people about reproductive health and diseases.
expected? (b) include research organization working on to produce
new and more effective contraceptives for birth
(a) 10th - 30th (b) 1st - 10th
control.
(c) 10th - 17th (d) 18th - 25th
(c) include a number of special techniques which assist
63. Refer the following statement and answer the question. infertile couples to have children.
"Inability of an individual to inseminate the female or due
(d) both (b) and (c)
to very low sperm counts in ejaculates leads to "A". It
71. Which of the following STDs are not completely curable?
could be corrected by "B". In "B" the "C" is collected and
artificially introduced either into the vagina or into the (a) Chlamydiasis, gonorrhoea, trichomoniasis
"D" (IUI - intra-uterine insemination) of the female." (b) Chancroid, syphilis, genital warts
Identify A to D. (c) AIDS, syphilis, hepatitis B
(d) AIDS, genital herpes, hepatitis B
A B C D 72. Intensely lactating mothers do not generally conceive
a. STD Embryo transfer Urine Fallopian tube due to the
b. MTP GIFT Ovum Uterus
(a) suppression of gonadotropins.
(b) hypersecretion of gonadotropins.
c. Infertility Artificial technique Semen Uterus
(c) supression of gametic transport.
d. Infertility ZIFT Sperm Fallopian tube (d) supression of fertilization.
EBD_7209
232 Biology
73. The best way to control population of a country is (a) (i), (ii), only (b) (ii), (iii) only
(a) to educate people. (c) (iii), (iv), (v) only (d) All of these
(b) to have better houses. 80. Given below are some examples of sexually transmitted
(c) to kill people on a large scale. diseases. Identify the one or more which specifically affect
(d) to practice and implement family planning. the sex organs.
74. Which of the following is correct regarding the (i) AIDS ((i) Syphilis
consequences of over population? (iii) Gonorrhea (iv) Genital warts
(a) It increase the poverty of a country. (a) (i) only (b) (i), (ii) only
(b) It leads to shortage of food supply. (c) (ii), (iii), (iv) only (d) All of these.
(c) It results in unemployment. 81. Which of the following principle will not help people to
(d) All of the above. become free from the infection of sexually transmitted
75. An individual undergoes sterilization process in which a diseases?
small incision was done on “X” and a part of vas deferens (i) Always use condoms in the course of coitus.
is removed or tied up. (ii) Avoid sex with unknown partners/multiple partners
Identify “X” and the intervention of this surgical process. (iii) Refer any one in case of doubt for early detection
(a) “X” - Fallopian tube; Prevent ovulation and and diagnose of disease and get complete treatment.
implantation. (iv) In case of doubt, go to a qualified doctor for early
(b) “X” - Testes; Prevent conception by blocking sperm detection and get complete treatment if diagnosed
entry. with disease.
(c) “X” - Epididymis; Inhibit maturation of sperm and (v) Involvement in sex with known partners/single
its transport. partners
(d) “X” - Scrotum; Gamete transport is blocked and (a) (i), (iv) only (b) (i), (ii), (iii) only
thereby prevents conception. (c) (i), (ii), (iv) only (d) All of these.
76. It is a disease which mainly affects mucous membrane of 82. Which of the following infections can also be transmitted
urinogenital tract, In males burning feeling on passing by sharing of injection needles, surgical instruments, etc.,
urine after a yellow discharge occurs that is accompanied with infected persons, transfusion of blood, or from an
by fever headache and feeling of illness its name is infected mother to the foetus too?
(a) Phenylketonuria (b) Gonorrhoea (a) Hepatitis B and HIV
(c) AIDS (d) None of these (b) Genital herpes and HIV
77. Emergency contraceptives are effective if used within (c) Syphilis and Hepatitis B
(a) 72 hours of coitus (d) Chlamydiasis and Trichomoniasis
(b) 72 hours of ovulation 83. Identify the correct reasons of infertility.
(c) 72 hours of menstruation (i) Drugs
(d) 72 hours of implantation (ii) Diseases
78. Identify the type of most popular contraceptive device (iii) Congenital
whose features are given below. (iv) Use of Contraceptives
(i) They do not interfere the act of coitus. (v) Immunological or psychological
(ii) These are effective barriers for insemination. (vi) Assisted reproductive technology
(iii) These help in reducing the risk of sexually transmitted (a) (i), (ii), (iii) (b) (iii), (iv), (vi)
diseases. (c) (i), (ii), (iii), (v) (d) All of these
(a) IUD (b) Condom 84. Which of the following assisted reproductive technology
(c) Injectable (d) Oral contraceptives has been used for the longest time period?
79. Study the given reasons on the basis of which pregnancy (a) In vitro fertilization
can be terminated. Identify the correct reasons. (b) Artificial insemination
(i) To get rid of unwanted pregnancies.
(c) Intracytoplasmic sperm injection
(ii) To prevent the fatality or harmfulness to the mother
(d) Gamete intra fallopian transfer
or to foetus or both due to the continuation of
85. Amniocentesis is a technique used to
pregnancy.
(iii) Termination of pregnancy is safe in each and every (a) determine errors in amino acid metabolism in embryo
case. (b) pin point specific cardiac ailments in embryo
(iv) If the foetus is male. (c) determine any hereditary genetic abnormality in
(v) It plays an important role in decreasing the embryo
population. (d) All of these
Principles of Inheritance
27

Chapter
& Variation

FACT/DEFINITION TYPE QUESTIONS 8. Mendel selected pea as material for his experiments
because
1. Mendel’s last law is
(a) it is an annual plant with comparatively short life
(a) segregation
cycle.
(b) dominance
(b) the flowers are self-pollinated.
(c) independent assortment
(c) the number of seeds produced is quite large.
(d) polygenic inheritance
2. The contrasting pairs of factors in Mendelian crosses are (d) all of the above.
called 9. Which of the following crosses would produce a
(a) multiple alleles (b) alleles genotypic ratio of 1 : 2 : 1 in F2 ?
(c) alloloci (d) paramorphs (a) AB × AB (b) Ab × ab
3. The ratio of phenotypes in F2 of a monohybrid cross is (c) Ab × Ab (d) ab × ab
(a) 3 : 1 (b) 1 : 2 : 1 10. Punnett square is used to know the
(c) 9 : 3 : 3 : 1 (d) 2 : 1 (a) outcome of a cross
4. Which of the following crosses will give tall and dwarf (b) probable result of a cross
pea plants in same proportions? (c) types of gametes
(a) TT × tt (b) Tt × tt (d) result of meiosis
(c) TT × Tt (d) tt × tt 11. The crossing of F1 to homozygous recessive parent is
5. A pure tall pea was crossed with a pure dwarf pea. All the called
plants of F1 were found to be tall. This is due to (a) back cross (b) test cross
(a) law of dominance. (c) F1 cross (d) all of these
(b) disappearan ce of factor for dwar fness in F 1 12. The test cross is used to determine the
generation. (a) genotype of the plant
(c) segregation of factors. (b) phenotype of the plant
(d) co-ordination. (c) both (a) and (b)
6. The F 2 generation of a cross produced identical (d) None of the above
phenotypic and genotypic ratio. It is not an expected 13. ABO blood group system is due to
Mendelian result, and can be attributed to
(a) multifactor inheritance
(a) independent assortment
(b) incomplete dominance
(b) linkage
(c) multiple allelism
(c) incomplete dominance
(d) epistasis
(d) none of the above
14. In humans, the dominance relationship between the A
7. The monohybrid genotypic ratio 1 : 2 : 1 in F2 generation
and B alleles of the ABO blood group gene is an example
indicates
of
(a) segregation
(a) complete dominance
(b) independent assortment
(b) incomplete dominance
(c) dominance
(c) codominance
(d) incomplete dominance
(d) epistasis
EBD_7209
234 Biology
15. The distance between the genes is measured by 25. It is well known that Queen Victoria of England was a
(a) angstrom (b) map unit carrier for haemophilia. Since this is an X-linked disease,
(c) Dobson unit (d) millimetre it can be predicted that
16. Linkage reduces the frequency of (a) all of her sons would have had disease.
(a) hybrids. (b) all her daugthers would have been carriers.
(b) all parental types. (c) her father must definitely have had haemophilia.
(c) homozygous recessive parents.
(d) haemophilia would have occurred in more of her male
(d) heterozygous recessive parents.
than her female descendents.
17. Distance between the genes and percentage of
recombination shows 26. The number of phenotypes in ABO blood groups is
(a) a direct relationship (a) 1 (b) 4
(b) an inverse relationship (c) 6 (d) 8
(c) a parallel relationship 27. Extra chromosome ‘X’ is present in which one of the
(d) no relationship following cases?
18. HbA and HbS alleles of normal and sickle celled RBC are (a) Down syndrome (b) Klinefelter syndrome
(a) dominant-recessive alleles. (c) Turner syndrome (d) Bleeder’s disease
(b) polygenic alleles. 28. The person with Turner’s syndrome has
(c) codominant alleles. (a) 45 autosomes and X sex chromosome
(d) multiple alleles.
(b) 44 autosomes and XYY sex chromosomes
19. Sex is determined in human beings
(a) by ovum. (c) 45 autosomes and XYY sex chromosomes
(b) at time of fertilization. (d) 44 autosomes and X sex chromosome
(c) 40 days after fertilization. 29. Mental retardation in man associated with sex
(d) seventh to eight week when genitals differentiate in chromosomal abnormality is usually due to
foetus. (a) increase in size of X-chromosome.
20. The ‘X’ body of Henking was observed in (b) increase in size of Y-chromosome.
(a) all sperms during spermatogenesis. (c) increase in number of Y-chromosome.
(b) all eggs during oogenesis.
(d) increase in number of X-chromosome.
(c) half of the sperms during spermatogenesis.
30. Three children in a family have blood types O, AB and B
(d) half of the eggs during oogenesis.
respectively. What are the genotypes of their parents?
21. In a dihybrid cross, F2 phenotypic ratio is 13 : 3. It is case of
(a) IA i and IBi (b) IAIB and i i
(a) complementary genes
(c) IBIB and IAIA (d) IAIA and IBi
(b) epistatic genes
(c) multigenic inheritance 31. A character which is expressed in a hybrid is called
(d) incomplete dominance (a) dominant (b) recessive
22. In sickle-cell anaemia, shape of RBCs under oxygen (c) co-dominant (d) epistatic
tension becomes 32. Mutations can be induced with
(a) biconcave disc like (b) elongated and curved (a) infrared radiations (b) I AA
(c) circular (d) spherical (c) ethylene (d) gamma radiations
23. Sickle cell anaemia is 33. Which Mendelian idea is depicted by a cross in which
(a) caused by substitution of valine by glutamic acid in the F1 generation resembles both the parents?
the beta globin chain of haemoglobin. (a) Law of dominance
(b) caused by a change in a single base pair of DNA. (b) Inheritance of one gene
(c ) characterized by elongated sickle like RBCs with a (c) Co-dominance
nucleus. (d) Incomplete dominance
(d) an autosomal linked dominant trait. 34. In XO type of sex determination
24. Sickel-cell anaemia is an example of (a) females produce two different types of gametes.
(a) sex-linked inheritance. (b) males produce two different types of gametes.
(b) deficiency disease. (c) females produce gametes with Y chromosomes.
(c) autosomal heritable disease. (d) males produce single type of gametes.
(d) infectious disease.
Principles of Inheritance & Variation 235

(iii) Phenotypic ratio of monohybrid F2 progeny in case


STATEMENT TYPE QUESTIONS
Mirabilis jalapa is 3 : 1.
35. Which one of the following cannot be explained on the (a) All the statements are true.
basis of Mendel’s law of dominance? (b) Statements (i) and (ii) are true, but statement (iii) is
(a) The discrete unit controlling a particular character is false.
called a factor. (c) Statements (i) and (iii) are true, but statement (ii) is
(b) Out of one pair of factors’ one is dominant and the false.
other is recessive. (d) Statements (ii) and (iii) are true, but statement (i) is
(c) Alleles do not show any blending and both the false.
characters recover as such in F2 generation.
41. Which of the following is incorrect regarding ZW-ZZ type
(d) Factors occur in pairs. of sex determination?
36. Select the correct statement from the ones given below
(a) It occcurs in birds and some reptiles
with respect to dihybrid cross.
(b) Females are homogametic and males are
(a) Tightly linked genes on the same chromosome show
heterogametic
higher recombinations.
(c) 1 : 1 sex ratio is produced in the offsprings
(b) Genes far apart on the same chromosome show very
few recombinations. (d) All of these
(c) Genes loosely linked on the same chromosome show 42. Identify the incorrect statement.
similar recombinations as the tightly linked ones. (a) In male grasshoppers, 50% of the sperms have no
(d) Tightly linked genes on the same chromosome show sex chromosome.
very few recombinations. (b) Usually female birds produce two types of gametes
37. Which one of the following conditions correctly describes based on sex chromosomes.
the manner of determining the sex in the given example? (c) The human males have one of their sex chromosomes
(a) Homozygous sex chromosomes (ZZ) determine much shorter than the other.
female sex in birds. (d) In domesticated fowls, the sex of the progeny
(b) XO type of sex chromosomes determine male sex in depends on the type of sperm that fertilizes the egg.
grasshopper. 43. Which of the following statements are correct?
(c) XO condition in human as found in Turner’s (i) Haemophilia is a sex-linked recessive disease.
syndrome, determines female sex.
(ii) Down’s syndrome is due to aneuploidy.
(d) Homozygous sex chromosomes (XX) produce male
(iii) Phenylketonuria is an autosomal dominant gene
in Drosophila.
disorder.
38. Which one of the following is an incorrect statement
regarding mutations? (iv) Phenylketonuria is an autosomal recessive gene
disorder.
(a) Deletion and insertion of base pairs cause frame-
shift mutations. (v) Sickle-cell anaemia is an X-linked recessive gene
(b) Cancer cells commonly show chromosomal disorder.
aberrations. (a) (i), (iii) and (v) (b) (i) and (iii)
(c) UV and gamma rays are mutagens. (c) (ii) and (v) (d) (i), (ii) and (iv)
(d) Change in a single base pair of DNA does not cause 44. Choose the correct statements given below regarding
mutation. Mendelian inheritance.
39. Which of the following statement is not correct of two (i) Mendel’s experiments had small sample size which
genes that show 50% recombination frequency? gave greater credibility to the data.
(a) The genes are tightly linked. (ii) A true breeding line shows a stable trait inheritance
(b) The genes show independent assortment. and expression for several generations.
(c) If the genes are present on the same chromosome, (iii) In a dissimilar pair of factors, one member of the pair
they undergo more than one crossovers in every dominates over the other.
meiosis. (iv) A recessive parental trait is expressed only in its
(d) The genes may be on different chromosomes. heterozygous condition.
40. Refer the given statements and select the correct option. (v) Two alleles of a gene are located on homologous
(i) Percentage of homozygous dominant individuals sites on homologous.
obtained by selfing Aa individuals is 25%. (a) (ii) only (b) (ii), (iii) and (v)
(ii) Types of genetically different gametes produced by (c) (i), (iii) and (v) (d) (i) and (v)
genotype AABbcc are 2.
EBD_7209
236 Biology
45. Which of the following statements are the correct? Reason : Effect of environment, other alleles did not
(i) Failure of segregation of chromatids during cell explained by the Mendel.
division results in aneuploidy. 52. Assertion : Test cross is used to determine an unknown
(ii) Chromosomal disorders are mainly determined by genotype within one breeding generation.
alteration or mutation in a single gene. Reason : Test cross is a cross between F1 hybrid and
(iii) Thalasemia and cystic fibrosis are Mendelian dominant parent.
disorders.
(iv) Sickle cell anemia is an X-linked trait. MATCHING TYPE QUESTIONS
(v) Haemophilia is an autosome linked recessive disease. 53. Match column-I with column-II and select the correct
(a) (i) and (iii) (b) (i), (iii) and (iv) answer using the codes given below.
(c) (iii) and (iv) (d) (ii) and (iii) Column-I Column-II
46. Which of the following statements are correct ? A. ABO blood groups I. Dihybrid cross
(i) Incomplete or mosaic inheritance is an example of
B. Law of segregation II. Monohybrid cross
pre-Mendelian concept of blending inheritance.
(ii) Test cross is a special type of back cross. C. Law of Independent III. Base pairs substitution
(iii) Chromosomal aberrations are commonly observed assortment
in cancer cells. D. Gene mutation IV. Multiple allelism
(iv) Thalassaemia is a Mendelian disorder. (a) A – II; B – I; C – IV; D – III
(a) (i) and (ii) (b) (ii), (iii) and (iv) (b) A – IV; B – I; C – II; D – III
(c) (ii) and (iv) (d) (i) and (iv) only
(c) A – IV; B – II; C – I; D – III
ASSERTION/REASON TYPE QUESTIONS (d) A – II; B – III; C – IV; D – I
54. Match column-I with column-II and select the correct
In the following questions, a statement of Assertion is followed
by a statement of Reason. answer using the codes given below.
(a) If both Assertion and Reason are true and the Reason is Column-I Column-II
the correct explanation of the Assertion. A. Turner syndrome I. Trisomy
(b) If both Assertion and Reason are true but the Reason is
B. Linkage II. AA + XO
not the correct explanation of the Assertion.
(c) If Assertion is true but Reason is false. C. Y-chromosome III. Morgan
(d) If both Assertion and Reason are false. D. Down's syndrome IV. Testis determining
47. Assertion : The genetic component of an organism is factor
called genotype. (a) A – II; B – I; C – IV; D – III
Reason : Genotype is the type of hereditary properties of (b) A – II; B – I; C – IV; D – III
an organism.
(c) A – IV; B – II; C – I; D – III
48. Assertion : Haemophilia is a recessive sex linked disease.
(d) A – II; B – III; C – IV; D – I
Reason : Haemophilia occurs due to mutation of a
55. Match column-I with column-II and find the correct
structural gene on chromosome 15.
answer.
49. Assertion : Persons suffering from haemophilia fail to
Column -I Column -II
produce blood clotting factor VIII.
A. Monoploidy I. 2n – 1
Reason : Prothrombin producing platelets in such persons
are found in very low concentration. B. Monosomy II. 2n + 1
50. Assertion : In humans, the gamete contributed by the C. Nullisomy III. 2n + 2
male determines whether the child produced will be male D. Trisomy IV. 2n – 2
or female. E. Tetrasomy V. n
Reason : Sex in humans is a polygenic trait depending VI. 3n
upon a cumulative effect of some genes on X-chromosome (a) A – V, B – I, C – IV, D – II, E – III
and some on Y-chromosome.
(b) A – V, B – II, C – IV, D – I, E – III
51. Assertion : Mendel’s law are able to predict accurately
the pattern of inheritance for a situation in which alleles (c) A – VI, B – V, C – III, D – IV, E – II
show the complete dominance. (d) A – II, B – I, C – III, D – VI, E – V
Principles of Inheritance & Variation 237

56. Match the column-I with column-II and choose the correct 59. Match column-I with column-II and select the correct
option. option from the codes given below.
Column -I Column -II Column-I Column-II
A. Incomplete dominance I. Drosophila A. Autosomal I. Down’s syndrome
B. Mendelian disorder II. Antirrhinum sp. recessive trait
B. Sex-linked II. Phenylketonuria
C. Transforming principle III. Griffith
recessive trait
D. Dihybrid cross IV. Haemophilia
C. Metabolic error III. Haemophilia
(a) A – I; B – IV; C – III; D – II linked to autosomal
(b) A – IV; B – II; C – III; D – I recessive
(c) A – II; B – III; C – IV; D – I D. Additional 21st IV. Sickle cell anaemia
(d) A – II; B – IV; C – III; D – I chromosome
57. Match column-I with their name given in column-II and (a) A – II; B – I; C – IV; D – III
choose the correct answer. (b) A – IV; B – I; C – II; D – III
Column -I Column -II (c) A – IV; B – III; C – II; D – I
(d) A – III; B – IV; C – I; D – II
A. Alfred Sturtevant I. Mapped position of
genes 60. Match column-I (Definition) with column-II (Terms) and
select the correct option from the codes given below.
B. Henking II. X-body
Column-I Column-II
C. Meischer III. Nuclein
(Definition) (Terms)
D. Morgan IV. Dihybrid crosses in A. A single trait I. Pleiotropy
Drosophila controlled by
(a) A – I, B – III, C – IV, D – II three or more than
(b) A – I, B – II, C – III, D – IV three alleles
(c) A – IV, B – I, C – II, D – III B. A single trait II. Multiple alleles
(d) A – III, B – II, C – IV, D – I controlled by
three or more than
58. Match the symbols used in human pedigree analysis three genes
(given in column-I) with their name (given in column-II)
C. A single gene III. Polygenic inheritance
and choose the correct option. exhibits multiple
Column -I Column -II phenotypic
A. I. Consanguineous mating expression
(a) A – II; B – III; C – I (b) A – III; B – II; C – I
B. II. Normal female (c) A – I; B – II; C – III (d) A – II; B – I; C – III

C. III. Mating DIAGRAM TYPE QUESTIONS


61. Study the pedigree chart given below and choose its
D. IV. Affected female correct representation.

E. V. Parents with male child

affected with disease


VI. Sex unspecified
(a) Inheritance of a condition like phenylketonuria as
an autosomal recessive trait.
(a) A – III, B – I, C – II, D – V, E – IV (b) The pedigree chart is wrong as this is not possible.
(b) A – II, B – I, C – VI, D – III, E – IV (c) Inheritance of a recessive sex-linked disease like
(c) A – III, B – IV, C – I, D – V, E – II haemophilia.
(d) A – III, B – I, C – VI, D – V, E – IV (d) Inheritance of a sex-linked inborn error of metabolism
like phenylketonuria.
EBD_7209
238 Biology
62. Which one of the following symbols and its 65. The experiment shown in the given figure has been carried
representation, used in human pedigree analysis is out by Morgan to show the phenomenon of linkage and
correct? recombination. If in cross I, genes are tightly linked and in
cross II, genes are loosely linked then what will be the
(a) = mating between relatives
percentage of recombinants produced in cross I and cross
II respectively?

(b) = unaffected male


Cross I

Parents
(c) = unaffected female
Yellow White Wild type
body eyes

(d) = male affected


F1 generation

63. The given figure represents the inheritance pattern of a Wild type Yellow White
body eyes
certain type of traits in humans.

Cross II
FEMALE MALE
(Mother) (Father)

Daughter Son White Miniature Wild type


body wings

Which one of the following conditions could be an


example of this pattern?
(a) Thalassemia
(b) Haemophilia Wild type White Miniature
(c) Phenylketonuria body wings
(d) Sickle cell anaemia
64. The given figure is a highly simplified representation of (a) 98.7% and 62.8% (b) 1.3% and 37.2%
the human sex chromosomes from a karyotype. The gene (c) 37.2 and 1.3% (d) 62.8% and 98.7%
a and b could be of 66. Study the pedigree chart of a certain family given below
and select the correct conclusion which can be drawn for
Gene a the character.

Gene b

(a) The female parent is heterozygous.


(b) The parents could not have had a normal daughter
(a) colour blindness and body height. for this character.
(b) attached ear lobe and Rhesus blood group. (c) The trait under study could not be colourblindness.
(c) haemophilia and red-green colour blindness. (d) The male parent is homozygous dominant.
(d) phenylketonuria and haemophilia.
Principles of Inheritance & Variation 239

67. Identify the type of inheritance shown in the diagram. 72. What is the probability of production of dwarf offsprings
in a cross between two heterozygous tall pea plants?
(a) Zero (b) 50%
(c) 25% (d) 100%
73. In a normal couple, half the sons are haemophilic while
half the daughters are carriers. The gene responsible for
it is located on
(a) X-chromosome of father.
(b) Y-chromosome of father.
(a) dominant X-linked (b) recessive X-linked (c) one X-chromosome of mother.
(c) dominant Y-linked (d) recessive Y-linked (d) both the X-chromosomes of mother.
68. Following is a pedigree for albinism (aa). What is the
74. What can be the blood group of offfspring when both
probability of II -1 to be a heterozygous ?
parents have AB blood group?
I (a) AB only (b) A, B and AB
(c) A, B, AB and O (d) A and B only
II 1 2 3
75. According to the law of independent assortment in a
dihybrid cross
III (a) there are four genotypes in F2.
(b) F2 contains 16 phenotypes.
1 1 (c) there is a single individual which is homozygous
(a) (b) recessive for both the characters.
3 2
(d) it is not possible to forecast the different phenotypes.
2 1
(c) (d) 76. In case of codominance, the offsprings of F1 generation
3 4 have the trait
CRITICAL THINKING TYPE QUESTIONS (a) of either of two parents.
(b) of both the parents.
69. In Mendelian dihybrid cross when heterozygous round
(c) of none of the parents.
yellow are self crossed, round green offsprings are
represented by genotype (d) in between the traits of two parents.
(a) RrYy, RrYY, RRYy (b) RrYY, RRyy, rryy 77. Inheritance of which of the following traits is shown in
(b) rrYy, rrYY (c) RrYY, Rryy, RRyy the above given cross?
70. A mutation is a (a) X-linked dominant trait
(a) sudden temporary change in an organism’s genetic (b) X-linked recessive trait
material. (c) Autosomal recessive trait
(b) change in phenotype followed by a change in
(d) Autosomal dominant trait
genotype.
(c) change in hereditary material directed by a changing X c
environment. XY XX
(Normal man) (Carrier woman)
(d) change in genotype which may result in a new
expression of a characteristic. c c
XX XX XY XY
71. Harmful mutations does not get eliminated from gene pool
(Carrier (Normal (Normal (Normal
because daughter) daughter) son) son)
(a) they are recessive and carried by homozygous
individuals. 78. Regarding ABO blood group, if one parent is homozygous
(b) they are recessive and carried by heterozygous and other is heterozygous, what are the chances that their
individuals. child will have ‘O’ blood group?
(c) they are formed repeatedly. (a) 25% (b) 50%
(d) they show genetic drift. (c) 75% (d) Zero
EBD_7209
240 Biology
79. Which one of the following correctly represents the (b) a gamete receiving only one of the two homologous
nature of blood in the ABO system of blood groups chromosomes during meiosis.
pertaining to the presence of antigens and antibodies? (c) presence of both genes on the same chromosome.
(a) Blood group A –Antibody A and antigen B (d) None of the above
(b) Blood group B–Antigen B and antibody A 87. In genetics the term test cross means
(c) Blood group AB–Both antibodies A and B (a) the crossing of F1 individual with homozygous
(d) Blood group O–No antigens and no antibodies recessive.
80. In Down’s syndrome, karyotyping has shown that the (b) crossing an F1 individual with either of the two
disorder is associated with trisomy of chromosome parents.
number 21 usually due to (c) crossing F1 individual with another F1 individual.
(a) non-disjunction during egg formation. (d) crossing F1 individual with that of F2.
(b) non-disjunction during sperm cell formation.
88. What proportion of the offsprings obtaied from cross
(c) addition of extrachromosome during cleavage of AABBCC × AaBbCc will be completely heterozygous for
zygote. all genes segregated indpendently?
(d) non-disjunction during egg cells production and (a) 1/8 (b) 1/4
sperm production.
(c) 1/2 (d) 1/16
81. In Drosophila, XXY represents a female but in human it
89. How would you test a pea plant whether it is a pure or
is an abnormal male. It shows that
hybrid for tallness ?
(a) Y-chromosome is essential for male sex in human.
(a) Cross the pea plant with another tall pea plant of
(b) Y-chromosome is essential for female sex
unknown genotype.
determination in Drosophila.
(b) Cross the pea plant with a pure tall pea plant.
(c) Y-chromosome is not essential for male sex
(c) Cross the pea plant with a homozygous dwarf pea.
determination in human.
(d) Cross the pea plant with any pea plant.
(d) All of the above
90. The law of segregation of characters is also called the law
82. A tobacco plant heterozygous for albinism (a recessive
of purity of gametes because
character) is self-pollinated and 1200 seeds are
subsequently germinated. How many seedings would (a) gametes have only one of the two alleles for each
have the parental genotype? character.
(a) 1250 (b) 600 (b) gametes cannot be contaminated.
(c) 300 (d) 2250 (c) both (a) and (b)
83. Multiple alleles are present (d) gametes are very different type of cells.
(a) at different loci in the same chromosome. 91. Mendel was successful in formulating the laws of
inheritance whereas his predecessors were not because
(b) in different chromosomes.
(a) he studied one clear-cut character at a time.
(c) at the same locus in one type of chromosomes.
(b) the characters studied by him were present on
(d) None of the above separate chromosomes.
84. Down’s syndrome is caused by an extra copy of (c) of the right choice of material.
chromosome number 21. What precentage of offspring (d) he kept accurate records of his experiments.
produced by an affected mother and a normal father? 92. In Drosophila, the sex is determined by
(a) 50% (b) 25% (a) the ratio of pairs of X-chromosomes to the pairs of
(c) 100% (d) 75% autosomes.
85. A monohybrid cross is the one in which (b) whether the egg is fertilized or develops
(a) only a single plant is involved for the experiment. parthenogenetically.
(b) a single pair of contrasting characters is considered (c) the ratio of number of X-chromosomes to the sets of
for the genetic results. autosomes.
(d) X and Y-chromosomes.
(c) a hybrid is crossed to a homozygous.
93. The F2 generation offspring in a plant showing incomplete
(d) None of the above dominance, exhibit
86. The law of segregation of characters postulated by (a) variable genotypic and phenotypic ratios.
Mendel can be related to (b) a genotypic ratio of 1 : 1.
(a) the presence of two genes for each character in a (c) a phenotypic ratio of 3 : 1.
somatic cell. (d) similar phenotypic and genotypic ratios of 1 : 2 : 1.
Principles of Inheritance & Variation 241

94. In Mendel’s experiments with garden pea, round seed 99. Sex determination in grasshoppers, humans, and
shape (RR) was dominant over wrinkled seeds (rr), yellow Drosophila is similar because
cotyledon (YY) was dominant over green cotyledon (yy). (a) females are hemizygous.
What are the expected phenotypes in the F2 generation (b) males have one X-chromosome and females have
of the cross RR YY × rryy ? two X-chromosomes.
(a) Only wrinkled seeds with green cotyledons.
(c) all males always have one Y-chromosome in all three
(b) Only wrinkled seeds with both yellow cotyledons.
species.
(c) Only round seeds with yellow cotyledons.
(d) Round seeds with yellow cotyledons and wrinkled (d) the ratio of autosomes to sex chromosomes is the
seeds with green cotyledons. same in all three organisms.
95. A man has enlarged breasts, sparse hairs on the body 100. Mendel’s rules do not correctly predict patterns of
and sex chromosomal formula XXY. He then suffers from inheritance for tightly linked genes or the inheritance of
(a) Down’s syndrome alleles that show incomplete dominance or epistasis. Does
(b) Edward’s syndrome this mean that his hypothesis are incorrect ?
(c) Turner’s syndrome (a) Yes, because they are relevant to only a small number
of organisms and traits.
(d) Klinefelter’s syndrome
(b) Yes, because not all data support his hypothesis.
96. A gene is said to be dominant if
(c) No, because he was not aware of meiosis or the
(a) it expresses its effect only in homozygous state.
chromosome theory of inheritance.
(b) it expresses its effect only in heterozygous condition.
(c) it expresses its effect both in homozygous and (d) No, it just means that his hypothesis are limited to
heterozygous condition. certain conditions.
(d) it never expresses its effect in any conditions. 101. Haemophilia is mentioned as a trait carried by the mother
97. Two organisms that are true-breeding for a certain genetic and passed to her sons. What is the pattern of inheritance
characteristic are mated and their offspring were analysed. for this trait ?
Which of the following statements about this situation is (a) Haemophilia is an allele carried on one of the mother’s
correct? autosomal chromosomes.
(a) Both parents are homozygotes. (b) Haemophilia is an allele carried on the Y-chromosome
(b) The offspring are either all homozygotes or all because more males have this genetic disorder than
heterozygotes. females.
(c) The offspring represent the F1 generation and the (c) Haemophilia is an allele carried on the X-chromosome
gametes produced by the offspring will carry only and can be directly inherited by the son from the
one allele for this gene. father or the mother.
(d) All of the above (d) Haemophilia is carried on the X-chromosome and
98. Why is the allele for wrinkled seed shape in garden peas can only be inherited by the son if the mother is a
considered recessive ? carrier.
(a) It “recedes” in the F2 generation when homozygous
parents are crossed.
(b) The trait associated with the allele is not expressed
in heterozygotes.
(c) Individuals with the allele have lower fitness than
that of individuals with the dominant allele.
(d) The allele is less common than the dominant allele.
(The wrinkled allele is a rare mutant).
EBD_7209
28

Chapter
Molecular Basis of
Inheritance
(a) amino acids (b) enzymes
FACT/DEFINITION TYPE QUESTIONS
(c) nucleotides (d) histone proteins
1. The two strands of DNA are held together by 9. Histones are rich in
(a) peptide bonds (b) phosphodiester bonds (a) alanine and glycine (b) lysine and arginine
(c) hydrogen bonds (d) S – S bonds (c) histidine and serine (d) cysteine and tyrosine
2. Nucleotide arrangement in DNA can be seen by 10. In Meselson and Stahl’s experiments, heavy DNA was
(a) X-ray crystallography distinguished from normal DNA by centrifugation in
(b) electron microscope (a) CsOH gradient (b) 14NH4Cl
(c) ultracentrifuge (c) 15NH4Cl (d) CsCl gradient
(d) light microscope 11. In Streptococcus pneumoniae
3. Chargaff’s rules are applicable to (a) virulent form is smooth.
(a) single stranded RNA. (b) virulent form is rough.
(b) single stranded DNA and RNA. (c) nonvirulent form is capsulated.
(c) single stranded DNA. (d) all forms are rough.
(d) double stranded DNA.
12. The scientists involved in discovery of DNA as chemical
4. One turn of DNA possesses basis of heredity were
(a) one base pair (b) two base pairs (a) Hershey and Chase
(c) five base pairs (d) ten base pairs (b) Griffith and Avery
5. Which of the following is correct for Watson and Crick’s (c) Avery, Mac Leod and McCarty
model of DNA. It is duplex with
(d) Watson and Crick
(a) 10 base pairs and 3.4 Å distance for every turn.
13. During infection of E. coli cells by bacteriophage T2,
(b) 10 base pairs and 3.4 Å distance for each turn of
(a) proteins are the only phage components that actually
spiral.
enter the infected cell.
(c) 20 base pairs and 34 Å for each turn.
(b) both proteins and nucleic acids enter the cell.
(d) None of the above
(c) only proteins from the infecting phage can also be
6. Information flow or central dogma of modern biology is
detected in progeny phage.
(a) RNA ® Proteins ® DNA
(d) only nucleic acids enter the cell.
(b) DNA ® RNA ® RNA
14. If a double stranded DNA has 20% of cytosine, what will
(c) RNA ® DNA ® Proteins
be the percentage of adenine in it?
(d) DNA ® RNA ® Proteins
(a) 20% (b) 40%
7. Nucleosome is
(c) 30% (d) 60%
(a) intron interrupted DNA.
15. In some viruses, RNA is present instead of DNA indicating that
(b) double helix DNA.
(a) their nucleic acid must combine with host DNA
(c) negatively charged DNA wrapped around positively
before replication.
charged histone octomer.
(b) they cannot replicate.
(d) satellite DNA.
(c) there is no hereditary information.
8. Genetic information is carried out by long chain molecule
made up of (d) RNA can act to transfer heredity.
Molecular Basis of Inheritance 243

16. A bacterium grown over medium having radioactive 35S 26. In Escherichia coli, lac operon is induced by
incorporates radioactivity in (a) lactose (b) promotor gene
(a) carbohydrates (b) proteins (c) b-galactosidase (d) I-gene
(c) DNA (d) RNA 27. Who proved that DNA is basic genetic material?
17. Leading strand during DNA replication is formed (a) Griffith (b) Watson
(a) continuously. (c) Boveri and Sutton (d) Hershey and Chase
(b) in short segments. 28. Lac operon is
(c) first. (a) arabinose operon (b) repressible operon
(d) ahead of replication. (c) inducible operon (d) overlapping genes
18. DNA replication is 29. Satellite DNA
(a) conservative and discontinuous. (a) is classified in many categories such as micro-
(b) semi-conservative and semi-discontinuous. satellites, minisatellites, etc. on the basis of base
(c) semi-conservative and discontinuous. composition length of segments and number of
repetitive units.
(d) conservative.
(b) normally does not code for any protein.
19. Methyl guanosine triphosphate is added at 5' end of
(c) shows polymorphism.
hn-RNA in a process of
(d) All of the above
(a) tailing (b) splicing
30. Which process is used for amplication or multiplication
(c) capping (d) None of these
of DNA for finger printing ?
20. Genetic code is
(a) Polymerse chain reaction (PCR)
(a) triplet, universal, ambiguous and degenerate.
(b) Nesslerisation
(b) triplet, universal, non-ambiguous and non-
(c) Southern blotting
degenerate.
(d) Northern blotting
(c) triplet, universal, non-ambiguous and degenerate.
31. Polymorphism in DNA sequence
(d) triplet, universal, ambiguous and non-degenerate.
(a) is the basis of genetic mapping of human genome.
21. Segments of mRNA removed during splicing are called
(b) arises due to mutation.
______.
(c) is the basis of DNA finger printing.
(a) introns (b) exons
(d) All of the above
(c) promotor regions (d) integrator regions
32. VNTRs are
22. Frame shift mutation occurs when
(a) Variable Number of Tandem Repeats.
(a) base is deleted or added.
(b) Very Narrow Tandem Repeats.
(b) base is added.
(c) Variable Non-cistronic Transposon Repeats.
(c) base is deleted.
(d) Valuable Non-cistronic Transposon Regions.
(d) anticodons are not present. 33. SNP which is pronounced as “snips” stands for
23. Initiation codon of protein synthesis (in eukaryotes) is (a) Small Nuclear Protein
(a) GUA (b) GCA (b) Single Nucleotide Particle
(c) CCA (d) AUG (c) Single Nucleotide Polymorphism
24. In eukaryotes, mRNA is synthesized with the aid of (d) Small Nicking Points
(a) RNA polymerase III. 34. Human Genome Project (HGP) is closely associated with
(b) RNA polmerase II. the rapid development of a new area in biology called as
(c) RNA polymerase I. (a) biotechnology (b) bioinformatics
(d) reverse transcriptase. (c) biogeography (d) bioscience
25. Lactose operon produces enzymes
(a) b-galactosidase, permease and glycogen synthetase. STATEMENT TYPE QUESTIONS
(b) b-galactosidase, permease and transacetylase. 35. Which of the following statement is correct about DNA
(c) permease, glycogen synthetase and transacetylase. polymerase ?
(d) b-galactosidase, permease and phosphoglucose (a) DNA polymerase can synthesize mRNA in the 3' to
isomerase. 5' direction.
EBD_7209
244 Biology
(b) DNA polymerase can synthesize DNA in the 5' to 3' 42. Find out the incorrect statement.
direction. (a) Uracil is present in RNA at the place of thymine.
(c) DNA polymerase can synthesize mRNA in the 5' to (b) The complex of DNA and protein in chromosome is
3' direction. called chromatin.
(d) DNA polymerase can synthesize DNA in the 3' to 5' (c) Heterochromatin is the most highly condensed form
direction. of chromatin.
36. Which of the following statement forms the basis of DNA (d) The process involved in the RNA formation on the
fingerprinting? DNA template is called replication.
(a) The relative proportions of purines and pyrimidines 43. Select the two correct statements out of the four (i–iv)
in DNA. given below about lac operon.
(b) Satellite DNA occurring as highly repeated short (i) Glucose or galactose may bind with the repressor
DNA segments. and inactivate it.
(c) The relative difference in the DNA occurrence in (ii) In the absence of lactose, the repressor binds with
blood, skin and saliva. the operator region.
(d) The relative amount of DNA in the ridges and (iii) The z-gene codes for permease.
grooves of the fingerprints.
(iv) This was elucidated by Francois Jacob and Jacque
37. Select the correct statement regarding protein synthesis. Monod.
(a) When the small subunit of the ribosome encounters
(a) (ii) and (iii) (b) (i) and (iii)
an mRNA the process of translation begins.
(b) Peptidase catalyses the formation of peptide bond. (c) (ii) and (iv) (d) (i) and (ii)
(c) UTRs are present between the start codon and stop codon. 44. How many of the given statements (i-iv) is/are correct?
(d) At the end of translation, the release factor binds to (i) In transcription, adenosine pairs with uracil.
the initiation codon. (ii) Regulation of lac operon by repressor is referred to
38. Which of the following statement is incorrect? as positive regulation.
(a) VNTR belong to a class of mini satellite DNA. (iii) The human genome has approximately 50,000 genes.
(b) DNA sequences work on the principle developed (iv) Haemophilia is a sex-linked recessive disease.
by F. Sanger. (a) Two (b) Three
(c) HGP was coordinated by US Department of Energy (c) Four (d) One
and the National Institute of Health.
45. Which of the following statements are correct?
(d) DNA fingerprinting involves identifying similarities
in repetitive DNA. (i) r-RNA provides the template for synthesis of
proteins.
39. Identify the incorrect statement about RNA.
(ii) t-RNA brings amino acids and reads the genetic code.
(a) RNA was the first genetic material to evolve in the
living systems. (iii) RNA polymerase binds to promoter and initiates
transcription.
(b) Apart from being a genetic material, it is also a
catalyst. (iv) A segment of DNA coding for polypeptide is called
(c) DNA evolved from RNA with chemical modifications. intron.
(d) RNA being a catalyst is non-reactive and stable. (a) (i) and (iii) (b) (i) and (ii)
40. Identify the incorrect statement. (c) (i), (ii) and (iii) (d) (ii) and (iii)
(a) In prokaryotes, the structural gene is polycistronic. 46. Which of the following statements about RNA
(b) In eukaryotes, structural genes have interrupted polymerase are correct?
coding sequences. (i) RNA polymerase I transcribes rRNAs.
(c) Eukaryotes have split gene arrangement. (ii) RNA polymerase II transcribes snRNAs.
(d) Intervening sequences appear in mature RNA. (iii) RNA polymerase III transcribes hnRNA.
41. Choose the incorrect statement regarding the
(iv) RNA polymerase II transcribes hnRNAs.
observations drawn from the human genome project.
(a) (i) and (ii) (b) (i) and (iii)
(a) Repetitive sequences are stretches of RNA.
(c) (ii) and (iii) (d) (i) and (iv)
(b) Less than 2 per cent of the genome codes for protein.
47. Select the incorrect statement(s).
(c) SNPs help in tracing human history.
(i) Six codons do not code for any amino acid.
(d) Repetitive sequences make up a very large portion
of the human genome. (ii) Codon is read in mRNA in a contiguous fashion.

244244244244244244244
244
Molecular Basis of Inheritance 245

(iii) Three codons function as stop codons. MATCHING TYPE QUESTIONS


(iv) The initiation codon AUG codes for methionine.
(a) (i) only (b) (ii) only 54. Match the enzymes (given in column I) with their function
(given in column II) and choose the correct combination
(c) (i), (ii) and (iv) (d) (i), (ii) and (iii)
from the given options.
48. Read the following statements and choose the incorrect
statements. Column - I Column - II
(i) Nitrogenous base is linked to the pentose sugar A. Helicase I. Joining of nucleotides
through a N-glycosidic linkage. B. Gyrase II. Opening of DNA
(ii) Phosphate group is linked to 5¢–OH of a nucleoside C. Primase III. Unwinding of DNA
through phosphoester linkage. D. DNA polymerase III IV. RNA priming
(iii) Two nucleosides are linked through 3¢–5¢N- (a) A – II; B – I; C – III; D – IV
glycosidic linkage. (b) A – II; B – I; C – IV; D – III
(iv) Negatively charged DNA is wrapped around (c) A – IV; B – III; C – I; D – II
positively charged histone octamer to form (d) A – II; B – III; C – IV; D – I
nucleosome. 55. Match the following and choose the correct combination
(v) The chromatin that is more densely packed and stains from the given options.
dark is called euchromatin. Column - I Column - II
(a) (i) only (b) (iv) only A. Splicing I. Lac operon
(c) (iii) and (v) (d) (i), (ii) and (iii)
B. Okazaki fragments II. Lagging strands
ASSERTION/REASON TYPE QUESTIONS C. Jacob and Monad III. Lactose
D. Inducer IV. Removal of intron
In the following questions, a statement of Assertion is followed
(a) A – IV; B – II; C – I; D – III
by a statement of Reason.
(b) A – II; B – I; C – IV; D – III
(a) If both Assertion and Reason are true and the Reason is (c) A – IV; B – III; C – I; D – II
the correct explanation of the Assertion. (d) A – II; B – III; C – I; D – IV
(b) If both Assertion and Reason are true but the Reason is 56. Match the column-I with column-II and choose the correct
not the correct explanation of the Assertion. combination from the given options.
(c) If Assertion is true but Reason is false. Column - I Column - II
(d) If both Assertion and Reason are false. A. Operator site I. Binding site for
49. Assertion : Adenine cannot pair with cytosine RNA polymerase
Reason : Adenine and cytosine do not have a perfect B. Promoter site II. Binding site for
match between hydrogen donor and hydrogen acceptor repressor molecule
sites. Hence, they cannot pair. C. Structural gene III. Codes for enzyme
50. Assertion : A single mRNA strand is capable of forming protein
a number of different polypetide chains. D. Regulator gene IV. Codes for repressor
Reason : The mRNA strand has terminator codon. molecules
51. Assertion : The genetic code is degenerate. (a) A – II; B – I; C – III; D – IV
Reason : Most amino acids are coded by more than one (b) A – II; B – I; C – IV; D – III
codon. (c) A – IV; B – III; C – I; D – II
52. Assertion : Replication and transcription occur in the (d) A – II; B – III; C – I; D – IV
nucleus but translation takes place in the cytoplasm. 57. Match the steps of protein by synthesis given in column-
Reason : mRNA is transferred from the nucleus into I with their feature given in column-II and select the correct
cytoplasm where ribosomes and amino acids are available combination from the given options.
for protein synthesis. Column - I Column - II
53. Assertion : DNA fingerprinting is very well known for its A. Termination I. Aminoacyl tRNA
application in paternity testing is case of disputes. synthetase
Reason : It employs the principle of polymorphism in DNA B. Translation II. Okazaki fragments
sequences as the polymorphisms are inheritable from C. Transcription III. GTP dependent
parent to children. release factor
D. DNA replication IV. RNA polymerase
EBD_7209
246 Biology
(a) A – II; B – I; C – III; D – IV 61. Match the scientists given in column-I with their work
(b) A – III; B – I; C – IV; D – II given in column-II and select the correct option.
(c) A – IV; B – III; C – I; D – II Column-I Column-II
(d) A – II; B – III; C – I; D – IV
58. Match the column-I with column-II and select the correct A. F. Meischer I. DNA double helix
combination from the given options. B. Griffith II. Nuclein
Column - I Column - II
A. Griffith I. Nucleoid C. Hershey and III. S. pneumoniae
Chase
B. Hershey and Chase II. Active chromatin
C. Prokaryotic DNA III. Transduction D. Watson and Crick IV. Bacteriophages
D. Euchromatin IV. Transformation
E. Wilkins and V. X-ray diffraction studies
(a) A – II; B – I; C – III; D – IV
Franklin
(b) A – III; B – I; C – IV; D – II
(c) A – IV; B – III; C – I; D – II (a) A – II; B – III; C –IV; D – I; E – V
(d) A – II; B – III; C – I; D – IV (b) A – V; B – IV; C –III; D – I; E – II
59. Match the codons given column I with their respective
amino acids given in column II and choose the correct (c) A – I; B – III; C –IV; D – II; E – V
answer. (d) A – I; B – IV; C –III; D – II; E – V
Column -I Column -II 62. Match column-I with column-II and select the correct
(Codons) (Amino acids) combination from the given options.

A UUU I. Serine Column-I Column-II


B GGG II. Methionine A. Sigma factor I. 5' – 3'
C UCU III. Phenylalanine B. Capping II. Initiation
D CCC IV. Glycine C. Tailing III. Termination
E AUG V. Proline
D. Coding strand IV. 5' end
(a) A – III; B – IV; C – I; D – V; E – II V. 3' end
(b) A – III; B – I; C – IV; D – V; E – II (a) A – III; B – V; C – IV; D – II
(c) A – III; B – IV; C – V; D – I; E – II (b) A – II; B – IV; C – V; D – I
(d) A – II; B – IV; C – I; D – V; E – III (c) A – II; B – IV; C – V; D – III
60. Match the enzymes given in column -I with its function (d) A – III; B – V; C – IV; D – I
given in column -II and select the correct option. 63. Match column-I (Scientists) with column-II (Discoveries)
and select the correct options.
Column - I Column - II
Column-I Column-II
A b-galactosidase I. Joining of DNA
(Scientists) (Discoveries)
fragments
A. Alec Jeffreys I. Lac operon
B Permease II. Peptide bond
formation B. F. Sanger II. Automated DNA
sequences
C Ligase III. Hydrolysis of
lactose C. Jacob and Monod III. DNA finger printing
D Ribozyme IV. Increase D. Avery, Mc Leod IV. Transforming principle
permeability of and McCarty
-galactosidase (a) A – II; B – III; C – IV; D – I
(a) A – II; B – I; C – IV; D – III (b) A – III; B – II; C – I; D – IV
(b) A – III; B – IV; C – I; D – II (c) A – III; B – II; C – IV; D – I
(c) A – II; B – IV; C – I; D – III (d) A – I; B – II; C – III; D – IV
(d) A – I; B – II; C – IV; D – III

246246246246246246246
246
Molecular Basis of Inheritance 247

DIAGRAM TYPE QUESTIONS (a) A–Hydrogen bonds, B–Pyrimidine, C–Hexose


(deoxyribose) sugar, D–5' end, E–Purine base
64. The given figure shows the structure of nucleosome with (b) A–Hydrogen bonds, B–Purine base, C–Hexose
their parts labelled as A, B & C. Identify A, B and C.
(deoxyribose) sugar, D–5' end, E–Pyrimidine
A B
(c) A–Hydrogen bonds, B–Pyrimidine, C–Pentose
(deoxyribose) sugar, D–5' end, E–Purine base
(d) A–Hydrogen bonds, B–Purine base, C–Pentose
C (deoxyribose) sugar, D– 5' end, E– Pyrimidine
67. The diagram given below shows an important concept
(proposed by C) in the genetic implication of DNA. The
process occuring in that concept are marked as A and B.
Core of histone molecules
Identify A, B and C.
(a) A – DNA; B – H1 histone; C – Histone octamer
(b) A – H1 histone; B – DNA; C – Histone octamer A B
DNA mRNA Protein
(c) A – Histone octamer; B – RNA; C – H1 histone
(d) A – RNA; B – H1 histone; C – Histone octamer (a) A-Translation, B - Transcription, C-Erwin Chargaff
65. Name the types of synthesis A and B occurring in the (b) A-Transcription, B - Translation, C-Francis Crick
replication fork of DNA as shown below. (c) A-Translation, B - Extension, C-Rosalind Franklin
(d) A-Transcription, B - Replication, C-James Watson
5¢ 3¢
68. Which one of the following correctly represents the
Template DNA manner of replication of DNA ?
(parental strands)
5'
3'
sis
B

he
Sy

nt
n

Sy
th

(a) 3'
es

A 5'
is

3¢ 5¢
5¢ 3¢ 5'
3'
(a) A - Continuous synthesis (synthesis of leading
strand); B - Discontinuous synthesis (synthesis of
lagging strand). 3'
5'
(b) A - Discontinuous synthesis (synthesis of leading
strand); B - Continuous synthesis (synthesis of (b) 5'
lagging strand). 3'

(c) A- Continuous synthesis (synthesis of lagging 3'


5'
strand); B - Discontinuous synthesis (synthesis of
leading strand).
3'
(d) A - Discontinuous synthesis (synthesis of lagging 5'
strand); B - Continuous synthesis (synthesis of
leading strand). (c) 5'
3'
66. The given figure represents the double stranded poly-
nucteotide chain. Some parts are labelled as A, B, C, D 5'
3'
and E. Identify the correct labelling of A, B, C, D & E.

(d)
3'
5'

5'
3'
3'
5'
EBD_7209
248 Biology
69. Given figure represent the DNA double helix model, A B X Y Z L
proposed by Watson and Crick (1953). Select the option (a) Repressor Inducer b-Galacto- Permease Tr ans- X
that shows correct measurement of A, B and C marked in sidase acetylase
the figure. (b) Repressor Inducer Permease b-Galacto-Trans- Y
sidase acetylase
(c) Inducer Repressor b-Galacto- Permease Tr ans- Z
sidase acetylase
(d) Inducer Repressor b-Galacto- Tr ans- Permease B
sidase acetylase

72. The given figure represent one of the step in the process
A
B of transcription in bacteria. Identify the step and label A,
B & C marked in the figure.

3¢ 5¢
5¢ 3¢
B
A
C C
(a) Initiation; A – DNA, B – RNA, C – Promoter
(a) A – 3.4 nm, B – 0.34 nm, C – 2 nm
(b) Termination; A – RNA, B – RNA polymerase,
(b) A – 34 nm, B – 3.4 nm, C – 20 nm
C – Rho factor
(c) A – 3.4 Å, B – 0.34 Å, C – 20 Å
(c) Elongation; A – RNA, B – RNA polymerase,
(d) A – 34 Å, B – 3.4 Å, C – 2 Å
C – Sigma factor
70. Given diagram represents the schematic structure of a
(d) Elongation; A – DNA, B – DNA polymerase,
transcription unit with some parts labelled as A, B, C and
C – RNA
D. Select the option which shows its correct labelling.
73. Identify the labels A, B, C and D in the given structure of
Transcription start site tRNA and select the correct option.
A B
C
3¢ 5¢
D OH – 3¢ A
5¢ 3¢

C
A B C D C
C
(a) Terminator Promoter Template Coding 5¢ – G
strand strand
(b) Promoter Terminator Coding emplate
strand strand
(c) Promoter Terminator Template Coding D B
strand strand
Variable arm
(d) Terminator Promoter Coding emplate
strand strand
71. The given figure shows lac operon model and its
C
functioning. Select the option which correctly labels A,
B, X, Y and Z marked in the figure and also identify the
label (L) which is primarily responsible for the hydrolysis
of the disaccharide, lactose, into galactose & glucose. A B C D
(a) Anticodon TyC loop AA binding DHU loop
y site
P i p o z a
(b) AA binding TyC loop Anticodon DHU loop
Transcription site loop
mRNA
(c) AA binding DHU loop Anticodon TyC loop
A X Y Z site loop
B
(d) AA binding DHU loop TyC loop Anticodon
site loop loop

248248248248248248248
248
Molecular Basis of Inheritance 249

81. During elongation of polypeptide chain, sigma factor is


CRITICAL THINKING TYPE QUESTIONS
(a) functionless.
74. In tertiary structure of DNA, what is a histone octamer ? (b) retained for specific function.
(a) A complex consisting of eight positively charged
(c) released for re-use.
histone proteins (two of each H2A, H2B, H3 and H4)
that aid in the packaging of DNA. (d) required during closing of chain.
(b) A complex consisting of eight negatively charged 82. Determination of one amino acid by more than one codon
histone proteins (two of each H2A, H2B, H3 and H4) is due to
that aid in the packaging of DNA. (a) redundancy of genetic code.
(c) A complex consisting of nine positively charged (b) continuous nature of genetic code.
histone proteins (H1 and two of each H2A, H2B, H3
(c) punctuation in genetic code.
and H4) that aid in the packaging DNA.
(d) A complex consisting of nine negatively charged (d) universal nature of genetic code.
histone proteins (H1 and two of each H2A, H2B, H3 83. Operon is a
and H4) that aid in the packaging of DNA. (a) sequence of three nitrogen bases determining a single
75. RNA polymerases used for the transcription of genes amino acid.
require a ______ template. (b) set of closely placed genes regulating a metabolic
(a) rRNA (b) DNA pathway in prokaryotes.
(c) RNA (d) mRNA
(c) segment of DNA specifying a polypeptide.
76. The most abundant type of RNA in the cell is
(d) gene responsible for switching on and switching off
(a) r RNA (b) mRNA
other genes.
(c) t RNA (d) hn RNA
84. Clover leaf secondary structure of tRNA has a loop for
77. In terms of DNA and RNA structure, what is a nucleotide ?
(a) A nucleotide is a heterocyclic base. (a) three nucleotides of a codon.
(b) A nucleotide is a sugar molecule covalently bonded (b) three nucleotides of an anticodon.
to a heterocyclic base. (c) no nucleotides.
(c) A nucleotide is a sugar molecule bonded to (d) both (a) and (b)
phosphate group and a heterocyclic base. 85. DNA replication is semi-conservative as
(d) A nucleotide is a heterocyclic base bonded to (a) only non-parent strand acts as template.
phosphate group.
(b) both strands of new molecule are synthesized de
78. DNA exists in a double-stranded form whereas RNA is novo.
mainly a single stranded molecule. What is the likely
reason for DNA being double stranded ? (c) one of the strand in each new molecule is parental
(a) RNA strands cannot form base pairs. and the other is new.
(b) Double stranded DNA is a more stable structure. (d) daughter strands are dispersive.
(c) DNA cannot exist in the single stranded form. 86. Mutations which alter nucleotide sequence within a gene
(d) It is easier to replicate double stranded DNA than are called
single stranded RNA. (a) frame shift mutations
79. Escherichia coli fully labelled with 15N is allowed to grow (b) base pair substitutions
in 14N medium. The two strands of DNA molecule of the (c) both (a) and (b)
first generation bacteria have (d) none of these
(a) different density and do not resemble with their parent
87. Which one of the following pair is a purine pair?
DNA.
(a) Uracil, Guanine
(b) different density but resemble with their parent DNA.
(c) same density and resemble with their parent DNA. (b) Cytosine, Thymine
(d) same density but do not resemble with their parent (c) Adenine, Guanine
DNA. (d) Adenine, Thymine
80. Which step of translation does not consume high energy 88. Which one of the following group of codons is called as
phosphate bond? degenerate codons?
(a) Translocation (a) UAA, UAG and UGA
(b) Peptidyl transferase reaction (b) GUA, GUG, GCA, GCG and GAA
(c) Amino acid activation (c) UUC, UUG, CCU, CAA and CUG
(d) Aminoacyl tRNA binding to A-site (d) UUA, UUG, CUU, CUC, CUA and CUG
EBD_7209
250 Biology
89. The two strands of a double helix model of DNA are held 96. What role does messenger RNA play in the synthesis of
together by hydrogen bonds between proteins ?
(a) sugar and phosphate groups. (a) It catalysis the process.
(b) sugar and nitrogenous bases. (b) It translates the genetic code to a specific amino
(c) phosphate groups and nitrogenous bases. acid.
(d) nitrogenous bases. (c) It provides the genetic blue print for the protein.
90. Transcription (d) It modifies messenger RNA molecules prior to protein
(a) starts at initiator region and ends at stop region. synthesis.
(b) starts at operator region and ends at telomeric end. 97. What is the main function of tRNA in relation to protein
(c) starts at promoter region and ends at terminator synthesis ?
region. (a) Initiates transcription
(d) starts at CAAT box and ends at TATA box. (b) Inhibits protein synthesis.
91. Consider the process that a cell uses to replicate its double- (c) Identifies amino acids and transport them to
strand DNA to make copies for daughter cells. Which ribosomes.
statement describes the DNA in daughter cells ? (d) proof reading.
(a) The double helix in one daughter cell consists of 98. Which of the following molecule contains the genetic
two strands that were originally in the parent cell, code?
while the double helix in the other daughter cell (a) DNA (b) mRNA
consists of two newly made strands. (c) tRNA (d) rRNA
(b) The two strands of the double helices in both 99. What sequence on the template strand of DNA
daughter cells consist of segments of new and corresponds to the first amino acid inserted into a protein ?
parental DNA. (a) TAC (b) UAC
(c) The double helices in each daughter cell consists of (c) UAG (d) AUG
one parental strand and one newly made strand. 100. Which of the following would you expect to find in an
(d) None of the above. inducible system ?
92. Nucleotides are linked by (a) A repressor protein, which is bound to DNA in
(a) hydrogen bonds. (b) phosphodiester bonds. absence of any other factor.
(c) peptic bonds. (d) ionic bonds. (b) A repressor protein, which is bound to DNA in the
93. A geneticist isolates a gene for a specific traits under presence of a co-repressor.
study, she also isolate the corresponding mRNA. Upon (c) An activator protein, which is bound to DNA in the
comparison, the mRNA is found to contain 1,000 fewer absence of any other factor.
bases than the DNA sequence. Did the geneticist isolate (d) An activator protein, which is bound to DNA only
the wrong DNA ? in the absence of air inhibitor.
(a) Yes, mRNA is made from a DNA template and should 101. What effect would you expect if gene expression of the
be the same length as the gene sequence. lac operon were completely repressed ?
(b) Yes, the mRNA should contain more bases than the (a) The cell would be more efficient without 'wasting'
DNA sequence because bases flanking the gene are the energy required for the low level of Lac Z, LacY,
also transcribed. and Lac A gene expression.
(c) No, the final mRNA contains only exons, the introns (b) Allolactose would accumulate within the cell and
were removed. become toxic.
(d) No, the mRNA was partially degraded after it was (c) Lactose would not be converted into the inducer
transcribed. and the operon could not be induced.
94. A DNA strand with the sequence AACGTAACG is (d) All of the above
transcribed. What is the sequence of the mRNA molecule
102. Which of the following is Not a goal of the human genome
synthesized ?
project ?
(a) AACGTAACG (b) UUGCAUUGC
(a) To sequence the genomes of selected model
(c) AACGUAACG (d) TTGCATTGC
organisms.
95. During translation, proteins are synthesized by
(b) To eliminate all diseases.
(a) ribosomes using the information on DNA.
(c) To consider social, ethical and legal aspects of
(b) lysosome using the information on DNA. genetic information.
(c) ribosome using the information on mRNA. (d) To develop computational tools for analyzing
(d) lysosome using the information on mRNA. sequence information.

250250250250250250250
250
Molecular Basis of Inheritance 251

103. In addition to the human genome sequence, draft or 105. DNA fingerprinting using Variable Number Tandem
finished genome sequences existed for eight model Repeats (VNTRs) is based on the observation that
organisms by 2002. Which of the following organisms are (a) every individual has unique alleles at each VNTR
not the part of that group of eight model organisms ? locus.
(a) Saccharomyces cerevisiae (b) the DNA of VNTR loci is more stable than that of
(b) Drosophila melanogaster loci which code for proteins.
(c) Oryza sativa (c) VNTR sequences show little variability.
(d) Quercus rubra (d) VNTR loci are highly polymorphic.
104. Each individual has a unique DNA fingerprint as 106. The okazaki fragments in DNA chain
individuals differ in (a) result in transcription.
(a) number of minisatellites on chromosome. (b) polymerize in the 3' to 5' direction and forms
(b) location of minisatellites on chromosome. replication form.
(c) size of minisatellites on chromosome. (c) prove semi-conservative nature of DNA replication.
(d) All of the above (d) polymerize in the 5' to 3' direction and explain 3' to 5'
DNA replication.
EBD_7209
29

Chapter
Evolution

7. What is common amongst whale, seal and shark?


FACT/DEFINITION TYPE QUESTIONS
(a) Homoiothermy
1. Stanley L. Miller conducted experiments before 1953 (b) Seasonal migration
on prebiotic earth environment using special apparatus. (c) Thick subcutaneous fat
The primary surprising products were
(d) Convergent evolution
(a) amino acids (b) peptides 8. Thorn of Bougainvillea and tendril of Cucurbita are
(c) nucleotides (d) simple sugars examples of
2. Who proposed that the first form of life come from pre- (a) retrogressive evolution.
existing non- living molecules? (b) analogous organs.
(a) Oparin and Haldane (c) homologous organs.
(b) de Vries and Haldane (d) vestigial organs.
(c) Darwin and Lamarck 9. Industrial melanism is an example of
(d) Louis Pasteur and Miller (a) drug resistance.
(b) darkening of skin due to smoke from industries.
3. How is extinction represented in a family tree diagram?
(c) protective resemblance with the surroundings.
(a) When a branch splits.
(d) defensive adaptation of skin against ultraviolet
(b) When a branch ends. radiations.
(c) When a branch shifts along x - axis. 10. Darwin’s finches are a good example of
(d) When a branch shifts along y - axis. (a) convergent evolution
4. Atmosphere of earth just before the origin of life (b) industrial melanism
consisted of
(c) connecting link
(a) water vapours, CH4, NH3 and oxygen. (d) adaptive radiation
(b) CO2, NH3, and CH2 11. Adaptive radiation refers to
(c) CH4, NH3, H2 and water vapours. (a) power of adaptation in an individual to a variety of
(d) CH4, O3, O2 and water vapours. environments.
5. Analogous organs are (b) adaptations due to Geographical isolation.
(a) different in origin but perform similar functions. (c) evolution of different species from a common
(b) common in origin and perform common functions. ancestor.
(c) common in origin but perform different functions. (d) migration of members of a species to different
geographical areas.
(d) different in origin and perform different functions.
12. The Finches of Galapogas islands provide an evidence
6. Homologous organs show in favour of
(a) divergent evolution. (a) biogeographical evolution
(b) convergent evolution. (b) special creation
(c) parallel evolution. (c) evolution due to mutation
(d) both (b) and (c). (d) retrogressive evolution
Evolution 253

13. The term ‘Survival of Fittest’ was used by


STATEMENT TYPE QUESTIONS
(a) Charles Darwin (b) Herbert Spencer
(c) Jean Baptiste (d) Hugo de Vries 24. Which of the following statement is/are correct?
(i) Adaptative ability is inherited.
14. Tasmanian Wolf is a marsupial while Wolf is a placental
mammal. This shows (ii) Adaptative ability has a genetic basis
(iii) Fitness is the end result of the ability to adapt and
(a) convergent evolution get selected by nature.
(b) divergent evolution (a) Only (ii) (b) Both (i) and (iii)
(c) inheritance of acquired characters (c) All of these (d) None of these
(d) None of these 25. Select the correct statement(s).
15. Darwin was most influenced by (i) Microbial experiment show the pre-existing
advantageous mutations when selected will result
(a) Lamarck’s theory of acquired characters.
in the observation of new phenotypes. Over few
(b) Weismann’s theory of germplasm. generation this would result in speciation.
(c) Wallace’s theory of origin of species. (ii) Neanderthal fossils represent a human relative.
(d) Essay on Population by Malthus. (iii) In 1938, a fish caught in South Africa happened to
16. Single step large mutation leading to speciation is also be a coelacanth (lobe fins) which was thought to be
called extinct. These animals evolved into the first
amphibians living on both land and water.
(a) Founder effect (b) saltation
(iv) Lichens can be used as water pollution indicators.
(c) branching descent (d) natural selection (v) Alfred Wallace, a naturalist, who worked in Malay
17. A population is in Hardy- weinberg equilibrium for a gene Archaepelago (present Indonesia) had also come to
with only two alleles. If the gene frequency of an allele similar conclusion on natural selection as reached
A is 0.7, the genotype frequency of Aa is by Darwinism.
(a) 0.21 (b) 0.42 (a) (i) and (ii) (b) (i), (ii), (iii) and (v)
(c) 0.36 (d) 0.7 (c) (iii) and (iv) (d) (iv) and (v)
26. Which of the following features are correct for Homo erectus
18. If the frequency of dominant allele is 60%, find out
?
the percentage of heterozygous individuals in the
(i) Had a large brain around 900 c.c.
population.
(ii) Probably ate meat.
(a) 48% (b) 50%
(iii) Appeared about 1.5 mya year ago.
(c) 47% (d) 45% (iv) Evolved from H. habilis.
19. Random genetic drift in a population probably results (a) (i) and (ii) (b) (ii) and (iii)
from (c) None of these (d) All of these
(a) highly genetically variable individuals. 27. Which of the following is/are incorrect about neanderthal
(b) interbreeding within the population. man?
(c) constant low mutation rate. (i) They had large brain around 900 c.c.
(d) large population size. (ii) They lived in near east and central Asia between
20. Natural selection can lead to 100000-40000 years back.
(a) stabilization (b) directional change (iii) They used hides to protect their body and buried
(c) disruption (d) all of these their dead.
21. The first mammals were like (iv) They had no religious feeling.
(a) chimpanzee (b) gorilla (a) All of these (b) None of these
(c) (i) and (iv) (d) (ii) and (iii)
(c) shrews (d) reptiles
28. Which of the following statement is correct?
22. Which one of the following was the first to stand erect ?
(a) Life appeared about 500 M. yrs after the formation of
(a) Peking Man (b) Australopithecus
earth.
(c) Java Man (d) Cro-Magnon man (b) Louis Pasteur belived that life appeared only from
23. The extinct human ancestor who ate only fruits and pre-existing life.
hunted with stone weapons was (c) Oparin advocated that life came from pre-existing
(a) Ramapithecus (b) Australopithecus non-living organic molecules.
(c) Dryopithecus (d) Homo habilis (d) All of the above
EBD_7209
254 Biology
29. Read the following three statements (i - iii) and mark (c) Variation due to genetic drift results in changed
the right option. frequency of genes and alleles in future generations.
(i) The thorns in Bougainvillea and tendrils in cucurbits (d) Genetic recombination helps in maintaining Hardy-
represent divergent evolution. Weinberg equilibrium.
(ii) The similarity in the eyes of Octopus and monkeys 35. Given below are four statements (i - iv) each with one or
is the result of convergent evolution. two blanks. Select the option which correctly fills up
(iii) The potato and sweet potato are the examples of the blanks in two out of four statements.
homology.
(i) Wings of butterfly and birds look alike and are the
(a) (i) and (ii) correct (b) (ii) and (iii) correct
results of __(A)____, evolution.
(c) (i) and (iii) correct (d) All (i) are correct.
(ii) Miller showed that CH4, H2, NH3 and ___(A)___,
30. What is true regarding industrialization in England?
when exposed to electric discharge in a flask
(a) The white-winged moths were completely wiped out resulted in formation of ___(B)_____.
after industrialization.
(iii) Vermiform appendix is a ___(A)____ organ and an
(b) Since lichens did not grow in polluted area, the
__(B)_____ evidence of evolution.
number of melanized moths got reduced.
(iv) According to Darwin, evolution took place due to
(c) After industrialization the white-winged moths did
___(A)___ and ___(B)__ of the fittest.
not survive due to predators.
(a) (iv) – (A) small variations, (B) survival, (i) – (A)
(d) All of the above
convergent
31. Which of the following is a correct?
(b) (i) – (A) convergent, (ii) – (A) oxygen, (B)
(a) Homo erectus lived in east and central Asia and used nucleosides
hides to protect their bodies.
(c) (ii) – (A) water vapour, (B) amino acids, (iii) – (A)
(b) Agriculture came around 18000 yrs back. rudimentary, (B) anatomical.
(c) The skull of modern human resembles more closely
(d) (iii) – (A) vestigial, (B) anatomical, (iv) – (A)
to baby chimpanzee than to adult chimpanzee.
mutations, (B) multiplication
(d) All of the above
36. Identify the correct conclusion regarding the given
32. Which of the following is not a false statement ? statements.
(a) Life originated in mesozoic era. i. There are patterns in the fossil record that suggest
(b) Earth was formed about 4.6 million years ago. other species have diverged from a single ancestor
(c) Ist living organism in primitive ocean was coacervate. species.
(d) Homologous organs show divergent evolution. ii. There are biogeographic patterns in the distribution
33. Which of the following statement is/are correct? of species, for instance distinct bird species on an
(i) Increase in melanized moths after industrialization island tend to resemble one another, suggesting a
in Great Britain is a proof for natural selection. common ancestor.
(ii) When more individuals of a population acquire a iii. There are common stages in the early embryological
mean character value, it is called disruption. development of organisms representing several
(iii) Changes in allelic frequency in a population will distinct vertebrate groups.
lead to Hardy-Weinberg equilibrium. iv. Anatomical structures, such as forelimbs, in
(iv) Genetic drift changes the existing gene or allelic different groups appear to be modified versions
frequency in future generations. of structures that might have been present in a
(a) (ii) only common ancestor.
(b) (iv) only (a) These are the example of a macro evolutionary
(c) Both (i) and (iv) process.
(d) Both (i) and (iii) (b) They are the evidences for Lamarck's theory of
34. Choose the wrong statement regarding Hardy-Weinberg inheritance.
principle. (c) They are the evidences for Darwin's theory of
(a) Allele frequencies in a population are stable and common descent.
constant from generation to generation.
(d) They are the examples of natural selection which
(b) Sum total of all the allelic frequencies in a occurs within populations, where artificial selection
population is 1. does not occur.
Evolution 255

37. Which of the following statements best describe the B. Gene flow II. Differences in survival and
theory of natural selection? reproduction among
(a) All organisms are equally suited to their environment. variant individuals.
(b) Random selection will determine which organisms C. Natural selection III. Immigration, emigration
survive. change allele frequencies.
(c) Organisms better adapted to their environment have D. Genetic drift IV. Source of new alleles.
greater reproductive success.
(a) A – I, B – II, C – III, D – IV
(d) Organisms that produce the most offspring are better
suited to their environment. (b) A – IV, B – II, C – III, D – I
(c) A – III, B – I, C – IV, D – II
ASSERTION/REASON TYPE QUESTIONS (d) A – IV, B – III, C – II, D – I
46. Match the column-I with column-II and choose the correct
In the following questions, a statement of Assertion is followed
by a statement of Reason. option.
(a) If both Assertion and Reason are true and the Reason is Column-I Column-II
the correct explanation of the Assertion. A. Human embryos I. Chemical evolution
(b) If both Assertion and Reason are true but the Reason is have gill
not the correct explanation of the Assertion. B. Oparin and II. Stimulation experiment
(c) If Assertion is true but Reason is false. Haldane
(d) If both Assertion and Reason are false.
38. Assertion : Organic compounds first evolved in earth C. Miller and Urey III. Wings of bird and butterfly
required for origin of life were protein and nucleic acid. D. Analogous organs IV. Ontogeny repeats
Reason : All life forms were in water environment only. phylogeny
39. Assertion : Among the primates, chimpanzee is the closest (a) A – III, B – IV, C – II, D – I
relative of the present day humans. (b) A – II, B – I, C – IV, D – III
Reason : The banding pattern in the autosome numbers 3
(c) A – IV, B – I, C – II, D – III
and 6 of man and chimpanzee is remarkably similar.
40. Assertion : Natural selection is the outcome of differences (d) A – IV, B – I, C – III, D – II
in survival and reproduction among individuals that show 47. Match column I (containing list of scientists) with column
variation in one or more traits. II (their contributions) and choose the correct option.
Reason : Adaptive forms of a given trait tend to become
more common; less adaptive ones become less common Column-I Column-II
or disappear. (Name of the Scientist) (Contributions)
41. Assertion : The earliest organisms that appeared on the
A. Charles Darwin I. Mutation theory
earth were non-green and presumably anaerobes.
Reason :The first autotrophic organisms were the B. Lamarck II. Germ plasm theory
chemoautotrophs that never released oxygen. C. Hugo de Vries III. Philosophie Zoologique
42. Assertion : Darwin's finches show a variety of beaks suited D. Ernst Haeckel IV. The Origin of species
for eating large seeds, flying insects and cactus seeds. E. August Weismann V. Biogenetic law
Reason : Ancestral seed-eating stock of Darwin's finches VI. Essay on population
radiated out from South American mainland to different (a) A – IV; B – III; C – I; D – V; E – II
geographical areas of the Galapagos Islands, where they (b) A – IV; B – III; C – V; D – I; E – VI
found competitor-free new habitats.
(c) A – IV; B – VI; C – V; D – III; E – I
43. Assertion : The primitive atmosphere was reducing one
i.e., without oxygen. (d) A – II; B – III; C – I; D – V; E – II
Reason : In the primitive atmosphere, oxygen was 48. Match column -I with column -II and select the correct
involved in forming ozone. option.
44. Assertion : Jave Ape-man, Peking man and Heidelberg Column-I Column-II
man are the fossils of Homo erectus. A. Mesozoic I. First amphibians
Reason : Homo erectus evolved from Homo habilis. B. Devonian II. Proliferation of reptiles
C. Palaeocene III. Raise of modern mammals
MATCHING TYPE QUESTIONS
D. Permian IV. Radiation of primitive
45. Match the evolution concepts given in column-I with mammals
column-II and select the correct answer. V. 160 million years
Column - I Column - II (a) A – V, B – IV, C – III, D – II
A. Mutation I. Change in population’s (b) A – V, B – I, C – IV, D – II
allele frequencies due to (c) A – V, B – I, C – II, D – III
chance alone. (d) A – V, B – I, C – IV, D – III
EBD_7209
256 Biology
49. Match column-I with column-II and select the correct
DIAGRAM TYPE QUESTIONS
option.
Column-I Column-II 54. The given diagram represents Miller's experiment. Choose
A. Saltation I. Darwin the correct combination of labelling A, B , C, D and E.
B. Formation of II. Louis Pasteur
life was preceded
C. Reproductive III. de Vries A
fitness
D. Life comes from IV. Oparin and haldane
pre-existing life Spark discharge
(a) A – III, B – IV, C – I, D – II
(b) A – IV, B – III, C – II, D – I Mixture of
(c) A – II, B – III, C – I, D – IV D gases
(d) A – I, B – IV, C – III, D – II (B) Water out
50. Match column-I with column-II and select the correct
Water in
option.
Column-I Column-II
A. Wallace I. Essay on population
B. Malthus II. Biston
C. Hardy-weinberg III. p2 + q2 + 2pq =1
law Water containing
Boiling organic compound
D. Industrial IV. Co-proposer of
water C
melanism Natural selection
(a) A – III, B – IV, C – II, D – I
(b) A – II, B – I, C – IV, D – III (a) A–Electrodes, B– NH3 + H2 + H2O + CH4, C– liquid
(c) A – IV, B – I, C – II, D – III water in trap, D–vacuum pump
(d) A – IV, B – I, C – III, D – II (b) A– Electrodes, B–NH4 + H2 + CO 2 + CH3 ,
C– liquid water in trap, D – Vacuum pump
51. Which one of the following scientists name is correctly (c) A– Electrodes, B–NH3 + H2O, C–liquid water in trap,
matched with the theory put forth by him? D – Tap
(d) A– Electrodes, B–NH3 + H2 + H2 O + CH4 ,
(a) Weismann – Theory of continuity of C – liquid water in trap, D– Vacuum pump
Germplasm 55. The diversity within the wild bird species in the diagram
(b) Pasteur – Inheritance of acquired below can best be explained by which process?
characters
(c) De Vries – Natural selection
(d) Mendel – Theory of Pangenesis
52. Select the incorrect match from the given option.
(a) Homo habilis-Their brain capacities were between
650-800cc
(b) Homo erectus - Its fossil was discovered in Java in
1891 about 1.5 mya
(c) Neanderthal man - They used hides to protect their
body and buried their dead.
(d) Ramapithecus - Arose in Africa and moved across
continents and developed into distinct races
53. Which of the following pairs of terms is not related?
(a) Natural selection
(a) Evolution - Natural selection
(b) Ecological succession
(b) Vestigial structure - Appendix (c) Adaptive radiation
(c) Analogous structures - Butterfly wings (d) Both (a) and (c)
(d) Adaptive radiations - Vertebrates hearts or brains
Evolution 257

56. The given diagram of marsupials of Australia provides an


example of

Tasmanian
Flying D
wolf
phalanger Banded anteater

Australian
Marsupials C

Marsupial rat
Koala
Kangaroo
B
(a) convergent evolution
(b) parallel evolution
(c) recapitulation
(d) divergent evolution
A
57. Following is the diagrammatic representation of the
operation of natural selection of different traits. Which of
the following options correctly identifies all the three
graphs A, B and C

A B C D
Peak gets (a) CarboniferousTriassic Cretaceous Quaternary
higher (b) Jurassic Permian Tertiary Cretaceous
and (c) Permian Jurassic Quaternary Tertiary
narrower (d) Cretaceous Quaternary Carboniferous Jurassic
59. The given figure shows an example of
(A)

Medium-
Phenotypes sized
Number of individuals

favoured individuals
with phenotype

by natural are favoured


selection Tendril

(B)

Two peaks form


(a) homologous organs
(b) convergent evolution
(c) divergent evolution
(C)
(d) both (a) and (c)
60. The diagram given below shows the skull of two different
mammals. Which of the following accurately describes
A B C the differences between these skulls?
(a) Directional Stabilizing Disruptive
(b) Stabilizing Directional Disruptive
(c) Disruptive Stabilizing Directional
(d) Directional Disruptive Stabilizing
58. Following is given the diagrammatic representation of
evolutionary history of vertebrates through geological
periods. Identify the geological periods (A, B, C and D)
A B
and select the correct option.
EBD_7209
258 Biology
(a) Skull A has more teeth than skull B.
(b) Skull A has more brain capacity than skull B.
(c) Skull A is of a human and skull B is of an ape.
(d) Skull A is of an ape and skull B is of human.
61. The following graph shows the range of variation among
population members for a trait determined by multiple
genes.
Proportion

If this population is subject to stabilizing selection for Which statement best explains the variation in the beaks
several generations, which of the distributions (a -d) is of these four species?
most likely to result? (a) Over time, an abundance of seeds for food led to
increased similarities between the species.
(b) Over time, an abundance of seeds for food led to
Proportion

Proportion

increased differences between the species.


(a) (b) (c) Competition of limited food resources led to
selection for similar traits.
(d) Competition for limited food resources led to
selection for different traits.
Proportion

Proportion

CRITICAL THINKING TYPE QUESTIONS


(c) (d) 64. Which one of the following describes correctly the
homologous structures?
(a) Organs that have no function now, but had an
62. The given bones in the forelimbs of three mammals important function in ancestors.
figure shows. (b) Organs appearing only in embryonic stage and
disappearing later in the adult.
(c) Organs with anatomical similarities, but performing
different functions.
(d) Organs with anatomical dissimilarities, but
performing same function.
65. Evolution is
(a) progressive development of a race.
(b) history and development of a race alongwith variations.
(c) history of a race.
(d) development of a race.
66. Select the correct order of geological time scale of earth.
(a) Palaeozoic ® Archaeozoic ® Coenozoic
(b) Archaeozoic ® Palaeozoic ® Proterozoic
(c) Palaeozoic ® Mesozoic ® Coenozoic
For these mammals, the number, position, and shape of (d) Mesozoic ® Archaeozoic ® Proterozoic
the bones must likely indicates that they may have 67. The first organisms were
(a) developed in a common environment. (a) chemoautotrophs (b) chemoheterotrophs
(c) autotrophs (d) eukaryotes
(b) developed from the same earlier species.
68. Diversity in the type of beaks of finches adapted to
(c) identical genetic makeup. different feeding habits on the Galapagos Islands, as
(d) identical methods of obtaining food. observed by Darwin, provides evidence for
63. The diagram below shows four species of birds that (a) intraspecific variations
evolved from an ancestral species that had a small pointed (b) intraspecific competition
beak. Today, all four species inhabit the same island. (c) interspecific competition
(d) origin of Species by natural selection
Evolution 259

69. According to Oparin, which one of the following was not (a) artificial selection.
present in the primitive atmosphere of the earth? (b) natural selection.
(a) Methane (b) Oxygen (c) convergent evolution.
(c) Hydrogen (d) Water vapour (d) homologous structures.
70. Evolution of different species in a given area starting from 77. Stabilizing selection favours
a point and spreading to other geographical areas is known (a) both extreme forms of a tract.
as.
(b) intermediate forms of a tract.
(a) adaptive radiation (b) natural selection
(c) environmental differences.
(c) migration (d) divergent evolution (d) one extreme form over the other extreme form and
71. The eye of octopus and eye of cat show different patterns over intermediates forms of a tract.
of structure, yet they perform similar function. This is 78. The term living fossil refers to a
an example of (a) life like fossil well preserved in amber.
(a) Homologous organs that have evolved due to (b) fossil formed from a living animal.
divergent evolution. (c) living animal which is about to the fossilized.
(d) living organisation showing some very primitive
(b) Analogous organ s that have evolved due to
characters.
convergent evolution.
79. Genetic drift in a new colony is known as the
(c) Analogous organs that have evolved due to divergent (a) natural selection (b) Founder effect
evolution. (c) branching descent (d) saltation
(d) Homologous organs that have evolved due to 80. According to Lamarck, a giraffe has a long neck because
convergent evolution. (a) a creator designed it that way.
72. Fossil remains of Archaeopteryx indicates that (b) catastrophes eliminated short-necked forms.
(a) it was a flying reptile from Triassic. (c) its ancestors stretched their necks to get food.
(b) it was a flying reptile from Permian. (d) ancestral giraffes with slightly longer necks than
(c) reptiles gave rise to birds during Permian. others got more food and left more surviving
offspring.
(d) reptiles gave rise to birds during Jurassic.
81. The finches of Galapagos islands provide an evidence in
73. Archaeopteryx is known as missing connecting link favour of
because it is a fossil and shows the characters of
(a) evolution due to mutation
(a) fishes and amphibians.
(b) retrogressive evolution
(b) birds and reptiles. (c) biogeographical evolution
(c) reptiles and mammals. (d) special creation
(d) chordates and nonchordates. 82. Hugo de Vries gave his mutation theory on organic
74. When two species of different genealogy come to evolution while working on
resemble each other as a result of adaptation, the (a) Pisum sativum
phenomenon is termed as ________. (b) Drosophila melanogaster
(a) convergent evolution (c) Oenothera lamarckiana
(b) divergent evolution (d) Althea rosea
83. What was the most significant trend in the evolution of
(c) microevolution
modern man (Homo sapiens) from his ancestors?
(d) co-evolution (a) Shortening of jaws
75. The concept of chemical evolution is based on (b) Binocular vision
(a) possible origin of life by combination of chemicals (c) Increasing cranial capacity
under suitable environmental conditions. (d) Upright posture.
(b) crystallization of chemicals. 84. According to Darwin, two different areas within a
continent have different species because they have
(c) interaction of water, air and clay under intense heat.
different
(d) effect of solar radiation on chemicals. (a) evolutionary mechanisms
76. The shape of the heals of Darwins finches, industrial (b) ancestors
melanism, or the changes in horse teeth are all (c) environments
examples of (d) evolutionary times
EBD_7209
260 Biology
85. The tendency of population to remain in genetic 89. According to Hardy-Weinberg principle, allele and
equilibrium may be disturbed by genotype frequencies in a population will remain constant
(a) random mating (b) lack of immigration from generation to generation in the absence of other
(c) lack of mutations (d) lack of random mating evolutionary influences. It makes several assumptions
86. One of the important consequences of geographical which were given below.
isolation is i. Random Mating
(a) random creation of new species.
ii. Sexual Reproduction
(b) no change in the isolated fauna.
(c) preventing speciation. iii. Non-overlapping Generations
(d) speciation through reproductive isolation. iv. Occurrence of Natural Selection
87. What is meant by the term Darwin fitness? v. Small size of population
(a) The ability to survive and reproduce Identify two assumptions which do not met for a
(b) High aggressiveness population to reach Hardy-Weinberg Equilibrium?
(c) Healthy appearance
(d) Physical straight (a) iv and v (b) ii and iv
88. Which one of the following changes involved is (c) iii, iv and v (d) i, ii and iii
irrelevant in the evolution of man? 90. Hardy-Weinberg Equilibrium is based on which of the
(a) Perfection of hand for tool making. following equations?
(b) Change of diet from hard nuts and hard roots to soft (a) p + pq + q = 1
food.
(b) p + 2pq + q = 1
(c) Loss of tail.
(d) Increase in the ability to communicate with others (c) p + p2q2 + q = 1
and develop community behaviour. (d) p2 + 2pq +q2 = 1
Human Health and
30

Chapter
Disease

9. Which of the following set includes bacterial diseases ?


FACT/DEFINITION TYPE QUESTIONS
(a) Tetanus, typhoid, tuberculosis
1. Which of the following disease is confirmed by ‘widal (b) Small pox, influenza, tetanus
test’? (c) Meningitis, measles, syphilis
(a) Tuberculosis (b) Typhoid (d) None of the above.
(c) Plague (d) Tetanus 10. Which of the following is a communicable disease?
2. The toxic substance, ‘haemozoin’, related to the high fever (a) Malaria (b) Diabetes
and chill, is released during which of the following (c) Hypertension (d) Kwashiorkar
disease? 11. Black water fever is a severe complication of
(a) Dengue (b) Malaria (a) Plasmodium falciparum
(c) Diphtheria (d) Phenumonia (b) P. vivax
3. Malignant malaria is caused by the which of the following (c) P. malariae
species of Plasmodium ? (d) all of the above
(a) vivax (b) malariae 12. Mumps is a viral disease and painful swelling of
(d) ovale (d) falciparum (a) parotid glands (b) thyroid
4. The pathogens of genera, Microsporum, Trichophyton (c) thymus (d) sublingual glands
and Epidermorphyton are responsible for 13. Which of the following disease is not transmitted through
(a) botulism (b) conjunctivitis contaminated water ?
(c) ringworms (d) skin allergy (a) Typhoid (b) Diphtheria
5. Identify the site where Wuchereria bancrofti is normally (c) Amoebiasis (d) Hepatitis A
found on human body. 14. In which of the following disease, there is always a time
(a) Lymphatic vessels of the lower limbs lag between the infection and appearance of the
symptoms of that particular disease?
(b) Muscles of the legs
(a) AIDS (b) Allergy
(c) Blood vessels of the thigh region
(c) Cancer (d) Alcoholism
(d) Skin between the fingers
15. Which of the following is the most accurate definition of
6. Food poisoning is caused by infection?
(a) Entamoeba histolytica (a) The presence of microorganism in a particular
(b) Escherichia coli location.
(c) Clostridium botulinum (b) The disruption of normal host functions by
(d) Corynebacterium diphtheriae microorganisms.
7. Amoebiasis is caused by (c) The survival and reproduction of microorganisms in
(a) Plasmodium vivax a host.
(b) Entamoeba gingivalis (d) The appearance of symptoms in a host, caused by
(c) Trypanosoma gambiense microorganisms.
(d) Entamoeba histolytica 16. Infection of Ascaris usually occurs by
8. Vector for kala azar is: (a) eating imperfectly cooked pork.
(a) sandfly (b) bedbug (b) tse-tse fly.
(c) louse (d) housefly (c) mosquito bite.
(d) drinking water containing eggs of Ascaris.
EBD_7209
262 Biology
17. The cells affected in leukaemia are (d) innate non-specific immunity
(a) plasma cells (b) erythrocytes 29. Organ transplant rejection may be prevented by using
(c) thrombocytes (d) leucocytes (a) antibodies
18. To which type of barriers under innate immunity, do the (b) new T cells
saliva in the mouth and the tears from the eyes, belong? (c) immunosuppressive drug
(a) Physical barriers (b) Cytokine barriers (d) immunostimulatory drug
(c) Cellular barriers (d) Physiological barriers 30. Antigen binding site in an antibody is found between
19. Which one of the following acts as a physiological barrier (a) two light chains
to the entry of micro organisms in human body? (b) two heavy chains
(a) Skin (c) one heavy and one light chain
(b) Epithelium of urogenital tract
(d) either between two light chains or between one heavy
(c) Tears
and one light chain depending upon the nature of antigen.
(d) monocytes
31. Inflammatory response in allergy is caused by
20. Immunoglobulins are
(a) histamines (b) antigen
(a) antigen (b) antibodies
(c) prothrombin (c) antibodes
(c) antiseptics (d) antibiotics
32. Resistance in body against diseases is given by
21. Antigens are present
(a) vaccinations (b) histamine
(a) inside the nucleus.
(c) immunoglobulins (d) antigens
(b) on cell surface.
(c) inside the cytoplasm. 33. The principle of vaccination is based upon which property
of immune system ?
(d) on the nuclear membrane.
22. A cell-coded protein that is formed in response to infection (a) Memory (b) Diversity
with most animal viruses is called (c) Specificity (d) All of these
(a) interferon (b) antigen 34. Which of the following results in fever when released in
(c) histone (d) antibody body during disease ?
23. The cytotoxic cells are produced by (a) Pyrogens (b) Antibodies
(a) T-Cells (b) B-Cells (c) Interferons (d) Interleukins
(c) memory Cells (d) mast Cells 35. The foetus gets immunized after receiving antibodies from
24. The letter T in T-lymphocyte refers to : mother through placenta. This type of immunization is called
(a) thymus (b) thyroid (a) active immunity (b) innate immunity
(c) thalamus (d) tonsil (c) passive immunity (d) humoral immunity
25. The exaggerated response of the immune system to certain 36. When a quick immune response is required due to infection
antigens is called of a deadly microbe, the patient is injected with
(a) primary response (a) protein of pathogen
(b) secondary response (b) inactivated or weakened pathogen
(c) immune suppression response (c) preformed antibodies
(d) allergy (d) vaccine
26. Passive immunity is defined as immunity 37. Which of the following vaccine has been produced from
(a) inherited from the parents. yeast by recombinant DNA technology ?
(b) achieved through vaccination. (a) Hepatitis A (b) Hepatitis B
(c) acquired through first exposure to the disease. (c) Hib (d) Oral polio
(d) achieved through the sera of other animals enriched 38. B-lymphocytes are associated with
in antibodies. (a) humoral immunity (b) production of heparin
27. When body cannot differentiate between its own and (c) cell mediated immunity (d) internal cleansing
foreign matter it is referred to as 39. Which of the following lymphoid organ provides the site
(a) passive immunity (b) active Immunity for the interaction of lymphocytes with the antigen ?
(c) autoimmunity (d) immunodeficiency (a) Bone marrow (b) Thymus
28. Short -lived immunity acquired from mother to foetus across (c) Spleen (d) All of these
placenta or through mother’s milk to the infant, is categorized 40. Which of the following is trapped in the lymph nodes and
as
responsible for the activation of lymphocytes present
(a) active immunity there and cause the immune response?
(b) passive immunity
(a) Antigen (b) Antibody
(c) cellular immunity (c) Pathogen (d) Lymph fluid
Human Health and Disease 263

41. Which of the following is an autoimmune disease ? 52. The use of antihistamine, adrenaline and steroids quickly
(a) Asthma (b) Rheumatoid arthritis reduce the symptoms of
(c) Cirrhosis (d) AIDS (a) fungal disease (b) viral disease
42. Hepatitis B and AIDS are (c) allergy (d) helminthes disease
(a) caused by Retro-viruses. 53. TB is cured by
(b) transmitted through sexual contact. (a) griseofulvin (b) ubiquinone
(c) congenital diseases. (c) rifampin (d) eucitol
(d) transmitted through infected blood. 54. LSD is obtained from
43. Human immuno deficiency virus (HIV) has a protein coat (a) Cannabis sativus (b) Erythroxylon cocca
and a genetic material which is (c) Claviceps purpurea (d) Papaver somniferum
(a) Single stranded DNA. (b) Single stranded RNA. 55. The drug “Belladonna” is obtained from
(c) Double stranded RNA. (d) Double stranded DNA. (a) Rauwolfia (b) Atropa
44. The cell in the human body invaded by the human (c) Capsicum (d) Solanum
immunodeficiency virus (HIV) is 56. Which of the following are hallucinogens ?
(a) B - cell (b) macrophage (a) Charas (b) Bhang
(c) erythrocyte (d) T- helper cell (c) Ganja (d) All of these
45. Which is not immunity related disease ? 57. Which of the following is a narcotic drug?
(a) AIDS (b) Polio (a) Heroin (b) Codeine
(c) Cancer (d) Small pox (c) Morphine (d) All of these
46. Which of the following endocrine glands is often referred 58. Cyclosporin is used as is
in relation with AIDS? (a) allergic eczema
(a) Thyroid (b) adrenal (b) immunosuppressant
(c) Thymus (d) Pancreas (c) prophylactic for viruses
47. AIDS is due to (d) prophylactic for marasmus
(a) reduction in number of helper T cells 59. Opium is obtained from
(b) lack of interferon (a) Papaver somniferum (b) Rauwolfia serpentina
(c) reduction in number of killer T cells (c) Cannabis sativus (d) Claviceps purpurea
(d) autoimmunity 60. From which part of Atropa belladonna is the
48. Which of the following non-infectious disease is a major drug’belladona’ obtained?
cause of death in human beings? (a) Stems (b) Flowers
(a) AIDS (b) Cirrhosis (c) Leaves (d) Dried leaves and roots
(c) Cancer (d) Asthma 61. Narcotic drugs like charas, bhang and ganja are extracted
49. Cancer cells are more easily damaged by radiation than from
normal cells because they are (a) hemp plant (b) coco plant
(a) non-dividing (b) starved of mutation (c) poppy seeds (d) hallucinogen
(c) different in structure (d) undergoing rapid 62. Hashish and charas are obtained from
division (a) Rauwolfia serpentina (b) Cannabis sativus
50. Metastasis is (c) Papaver somniferum (d) Claviceps purpurea
(a) part of regeneration. 63. Opium is obtained from which part of the plant Papaver
(b) transfer of cancer cells from one part of the body to somniferum ?
another. (a) Inflorescence (b) Male part (dried)
(c) fast mitosis in cancer cells (c) Seed (unripe) (d) Leave
(d) all of the above 64. Ergot, a drug is derived from fungus
51. Techniques like radiography (use of X-rays), CT scanning (a) Aspergillus (b) Phytopthora
and MRI are very useful to detect (c) Clavicep (d) Perenospora
(a) autoimmune disorders. 65. Cirrhosis of the liver is caused by
(b) disease caused by viral infection. (a) cocaine (b) LSD
(c) disease caused by bacterial infection. (c) alcohol (d) morphine
(d) disease caused by an uncontrolled division of 66. Atropine, an alkaloid, is obtained from
abnormal cells of internal organs. (a) Datura anaroxia (b) Atropa belladonna
(c) Hyocyamus niger (d) Withania somnifera
EBD_7209
264 Biology
67. LSD is obtained from (iii) The B-lymphocytes are responsible for rejection of
(a) bryophyte (b) pleridophyte the graft.
(c) brown algae (d) fungus (iv) The acceptance or rejection of a kidney transplant
depends on the specific interferons.
STATEMENT TYPE QUESTIONS (a) (i) and (ii) (b) (ii) and (iii)
(c) (iii) and (iv) (d) (i) and (iii)
68. Which of the following statement is correct ?
74. Which one of the following statements is correct with
(a) Injecting microbes during immunization induces
respect to AIDS?
passive immunity.
(a) The causative HIV retr ovirus enters helper
(b) Cell-mediated immune response is responsible for
T-lymphocytes thus reducing their numbers.
graft rejection.
(b) The HIV can be transmitted through eating food
(c) Colostrum during initial days of lactation provides
together with an infected person.
active immunity to infant.
(c) Drug addicts are least susceptible to HIV infection.
(d) None of the above
(d) AIDS patients are being fully cured cent per cent
69. Which of the following statements regarding drugs are
with proper care and nutrition.
correct ?
75. Select the correct statement.
(i) Heroin, commonly called smack, is obtained by (a) Cocaine is given to patients after surgery as it
acetylation of morphine. stimulates recovery.
(ii) Cocaine is obtained from the latex of Papaver (b) Barbiturates when given to criminals make them tell
somniferum. the truth.
(iii) Marijuana interferes with the transmission of (c) Morphine is often given to persons who have
dopamine. undergone surgery as a pain killer.
(iv) Morphine is an effective sedative and pain killer. (d) Chewing tobacco lowers blood pressure and heart
(a) (i) and (ii) (b) (i) and (iv) rate.
(c) (ii) and (iii) (d) (iii) and (iv) 76. Sporozoites of the malarial parasite are found in
70. What is true about T-lymphocytes in mammals ? (a) salivary glands of freshly moulted female Anopheles
(a) These are produced in thyroid. mosquito.
(b) These originate in lymphoid tissues. (b) saliva of infected female Anopheles mosquito.
(c) They scavenge damaged cells and cellular debris. (c) red blood corpuscles of humans suffering from
(d) There are three main types of T-lymphocytes - malaria.
cytotoxic T cells, helper T cells and suppressor T cells. (d) spleen of infected humans.
71. Which of the following is the correct statement regarding 77. At which stage of HIV infection does one usually show
the particular psychotropic drug specified? symptoms of AIDS?
(a) Barbiturates cause relaxation and temporary (a) Within 15 days of sexual contact with an infected
euphoria person.
(b) Hashish causes after thought perceptions and (b) When the infecting retrovirus enters host cells.
hallucinations. (c) When viral DNA is produced by reverse transcriptase.
(c) Opium stimulates nervous system and causes (d) When HIV replicates rapidly in helper T-lymphocytes
hallucinations. and damages large number of these.
(d) Morphine leads to delusions and disturbed 78. Select the correct statement with respect to diseases and
emotions. immunization.
72. Which of the following statements is correct? (a) Certain protozoans have been used to produce
(a) Malignant tumours may exhibit metastasis. hepatitis B vaccine.
(b) Patients who have undergone surgery are given (b) Injection of snake antivenom against snake bite is
cannabinoids to relieve pain. an example of active immunization.
(c) Benign tumours show the property of metastasis. (c) If due to some reason B-and T-lymphocytes are
(d) Heroin accelerates body functions. damaged, the body will not produce antibodies
73. The following four statements (i - iv) kidney transplant. against a pathogen.
Identify the two correct statements. (d) Injection of dead / inactivated pathogens causes
(i) Even if a kidney transplant is proper the recipient passive immunity.
may need to take immunosuppresants for a long time. 79. Common cold differs from pneumonia in that
(ii) The cell-mediated immune response is responsible (a) pneumonia is caused by a virus while the common
for the graft rejection. cold is caused by the bacterium Haemophilus
influenzae.
Human Health and Disease 265

(b) pneumonia pathogen infects alveoli whereas the 84. Assertion : Cocaine has a potent stimulating action on
common cold affects nose and respiratory passage central nervous system, producing a sense of euphoria
but not the lungs. and increased energy.
(c) pneumonia is a communicable disease whereas the Reason : It interferes with the transport of the neuro-
common cold is a nutritional deficiency disease. transmitter acetylcholine.
(d) pneumonia can be prevented by a live attenuated 85. Assertion: Inspire of exposure to large number of
bacterial vaccine whereas the common cold has no infectious agents humans are resistive to diseases.
effective vaccine. Reason: Humans are able to defend against most of the
80. Which one of the following in not a property of cancerous foreign agents due to the ability to fight disease-causing
cells ? organisms.
(a) They divide in an uncontrolled manner. 86. Assertion: Virus-infected cells secrete proteins known
(b) They show contact inhibition. as interferons.
(c) They compete with normal cells for vital nutrients. Reason: Interferons protect the non-infected cells from
(d) They do not remain confined in the area of formation. bacterial infection.
81. Which of the following statement is correct regarding the 87. Assertion: Artificially acquired passive immunity results
number of helper T cells and the viral concentration in the when antibodies or lymphocytes produced outside the
blood? host are introduced into a host.
(a) As the number of HIV increase, so do the numbers Reason: A bone marrow transplant given to a patient with
of helper T cells. genetic immunodeficiency is an example of artificially
(b) As the number of HIV increase, the number of helper acquired passive immunity.
T cells decreases. 88. Assertion: Antiretroviral drugs are very effective in
(c) As the concentration of HIV decreases, the number treatment against AIDS.
of helper T cells increases. Reason: AIDS virus is a retrovirus with ssDNA as genetic
(d) There is no relationship between the concentration material.
of HIV and the number of helper T cells. 89. Assertion: IgG is the most abundant class of lgs in the
82. Which of the following statement is incorrect? body.
(a) Rheumatoid arthritis is an autoimmune disease. Reason: IgG is mainly found in sweet, tears, saliva, mucus,
(b) The use of drugs like antihistamine, adrenaline, and colostrum and gastro-intestinal secretions.
steroids quickly reduces the symptoms of bacterial
MATCHING TYPE QUESTIONS
infection.
(c) Several genes (called cellular oncogenes) have been 90. Which of the following pairs is not correctly matched?
identified in normal cells which when activated under (a) Malaria – Plasmodium
certain conditions, could lead to oncogenic (b) Plague – Yersinia pestis
transformation of the cells. (c) Tuberculosis – Trichuris trichura
(d) The vaccine also generates memory – B and T cells (d) Sleeping – Trypanosoma
that recognize the pathogen quickly on subsequent sickness gambiense
exposure and overwhelm the invaders with a massive 91. Which one of the following is not correctly matched ?
production of antibodies.
(a) Culex pipiens – Filariasis
ASSERTION/REASON TYPE QUESTIONS (b) Aedes aegypti – Yellow fever
(c) Female Anopheles – Leishmaniasis
In the following questions, a statement of Assertion is followed (d) Glossina palpalis – Sleeping sickness
by a statement of Reason. 92. Which one of the following pair is correctly match?
(a) If both Assertion and Reason are true and the Reason is
(a) Bhang – Analgesic
the correct explanation of the Assertion.
(b) If both Assertion and Reason are true but the Reason is (b) Cocaine – Opiate narcotics
not the correct explanation of the Assertion. (c) Morphine – Hallucinogen
(c) If Assertion is true but Reason is false. (d) Barbiturate – Sedatives
(d) If both Assertion and Reason are false. 93. Which one of the following is the correct match for
83. Assertion : Active immunity is slow and takes time to diseases and its causative agents ?
give its full effective response. (a) AIDS – Bacillus
Reason : Injecting the microbes intentionally during (b) Syphilis – Treoponema pallidum
immunization or infectious organisms gaining access into (c) Malaria – Trypanosoma
body during natural infection induces active immunity. (d) Gonorrhoea – Virus
EBD_7209
266 Biology
94. Column I lists the components of body defence 97. In which one of the following options the two examples
and column II lists the corresponding descriptions. Match (given in column I) are correctly matched with their
the two columns and choose the correct option. particular type of immunity (given in column II)?
Column-I Column-II Column I Column II
(components of body defence) (Description) (Examples) (Type of immunity)
A. Active natural I. Injection of gamma (a) Saliva in mouth – Physical barriers
immunity globulins and tears in eyes
B. First line of II. Complement proteins (b) Mucus coating of – Physiological barriers
defence and interferons epithelium, urogential
C. Passive natural III. Direct contact with tract and HCl in stomach
immunity the pathogens that (c) Polymorphonuclear – Cellular barriers
have entered inside leucocytes and
D. Second line of IV. Surface barriers monocytes
defence V. Antibodies (d) Anti-tetanus and – Active immunity
transferred through anti-snake bite
the placenta injections
(a) A – IV; B – III; C – V; D – II 98. Which of the following pairs is not correctly matched ?
(b) A – III; B – IV; C – II; D – V (a) Cholera – Vibrio cholerae
(c) A – III; B – IV; C – V; D – II (b) German measles – Rubella virus
(d) A – V; B – III; C – II; D – I (c) Whooping cough – Bordetella pertussis
95. Match the disease given in column I with the appropriate (d) Tetanus – Pasteurella pestis
items (pathogen/ prevention/treatment) given in column II. 99. Select the correct match of the symptoms of diseases
Column-I Column-II given in column I with their respective pathogen of the
(Diseases) (Pathogen/prevention/treatment) diseases given in column II.
A. Amoebiasis I. Lassa virus Column I Column II
B. Diphtheria II. Use only sterilized A. Appearance of dry, scaly I. Entamoeba
food and water lesions on various parts histolytica
C. Cholera III. DPT vaccine of the body such as skin
D. Rabies IV. Use oral rehydration nails and scalp.
therapy B. Chronic inflammation II. Ascaris
(a) A – II; B – III;C – IV; D – I of the lymphatic vessel of lumbricoides
(b) A – I; B – II; C – III; D – IV lower limbs.
(c) A – II; B – IV; C – I; D – III C. Fever, chills, cough, III. Haemophilus
(d) A – II; B – I; C – III; D – IV headache and in severe cases influenzae
96. Which one of the following options gives the correct the lips and finger nails may
matching of a disease with its causative organism and turn gray to bluish in colour.
mode of infection? D. Constipation, abdominal pain IV. Wuchereria
Disease Causative Mode of Infection and cramps, stool with excess bancrofti
organisms mucous and blood clots.
(a) Elephantiasis Wuchereria With infected E. Internal bleeding, muscular
bancrofti water and food pain, fever, anaemia and V. Microsporum
(b) Malaria Plasmodium Bite of male blockage of intestinal
vivax Anopheles mosquito passage.
(c) Typhoid Salmonella With inspired air (a) A – I; B – II;C – III; D – IV; E – V
typhi (b) A – III; B – V;C – II; D – IV; E – I
(d) Pneumonia Streptococcus Droplet infection (c) A – III; B – I;C – V; D – II; E – IV
(d) A – V; B – IV;C – III; D – I; E – II
pneumoniae
Human Health and Disease 267

(d) A - Viral DNA introduced into cell; B - Viral RNA;


DIAGRAM TYPE QUESTIONS
C - Viral RNA incorporates into host DNA; D - New
100. Identify the molecules (i) and (ii) given below and select viral DNA produced
the right option giving their source and use. 102. Given figure shows the human lymphatic system with
some part marked as A, B, C and D identify the correct
CH3 part
N O
CH3
(i) O
H
O A
H
O B
OH
C

(ii)
O
H

Molecule Source Use


(a) (i) Cocaine Erythroxylum Accelerates D
coca the transport
(a) A - lymph nodes (primary lymphoid organ), B -
of dopamine thymus (primary lymphoid organ), C - spleen
(b) (ii) Heroin Cannabis Depressant (secondary lymphoid organ), D - bone marrow
sativa and slows (secondary lymphoid organ)
down body (b) A - lymph nodes (primary lymphoid organ), B -
functions thymus (secondary lymphoid organ), C - spleen
(c) (ii) Cannabinoid Atropa Produces (primary lymphoid organ), D - bone marrow (primary
belladonna hallucinations lymphoid organ)
(d) (i) Morphine Papaver Sedative and (c) A - lymph nodes (secondary lymphoid organ),
somniferum pain killer B - thymus (primary lymphoid organ), C-spleen
101. Refer the given figure showing the mode of action of (secondary lymphoid organ), D - bone marrow
AIDS virus and identify the sequences labelled as A, B, C (primary. lymphoid organ)
and D. (d) A- lymph nodes (primary lymphoid organ), B -
thymus (secondary lymphoid organ), C - spleen
AIDS virus
(secondary lymphoid organ), D - bone marrow
Virus infects (secondary lymphoid organ)
A normal animal cell 103. The diagram given below shows an antibody molecule
with their parts labelled as A, B, C, D, E & F. Identify the
C part marked as A, B, C, D, E and F.
B

D A

B
(a) A - Viral DNA introduced into cell; B - Viral DNA; D-chain
C - Viral DNA incorporates into host RNA; D - New C
viral RNA produced
(b) A - Viral RNA introduced into cell; B - Viral RNA;
C - Viral DNA incorporates into host DNA; D - New
viral DNA produced F E-chain
(c) A - Viral RNA introduced into cell; B - Viral DNA; Bond/Bridge
C - Viral DNA incorporates into host DNA; D - New
viral RNA produced
EBD_7209
268 Biology
(a) A-Antigen binding site; B-Variable region 110. In polio, the legs get paralyzed and atrophied due to :
(of L-Chain); C - Constant region (of L-Chain); D - (a) obstruction of muscles.
Light polypeptide chain (L-Chain); E-Heavy (b) death of some muscles.
polypeptide chain (H-Chain); F - Disulfide bond. (c) degeneration of bones.
(b) A - Antigen binding site; B-Constant region (d) shrinkage of muscles.
(of L-Chain); C - Variable region (of L-Chain); D -
111. Which of these may cause hypothermia in humans ?
Light polypeptide chain (L-Chain); E-Heavy
(a) Smoking (b) LSD
polypeptide chain (H-Chain); F - Disulfide bond.
(c) Dopamine (d) Alcohol consumption
(c) A-Antigen binding site; B-Variable region
(of L-Chain); C - Constant region (of L-Chain); D - 112. The main reason, why antibodies could not solve all the
Heavy polypeptide chain (L-Chain); E - Light problems of bacteria mediated diseases, is the
polypeptide chain (H-Chain); F - Hydrogen bond (a) Development of mutant strains resistant to
(d) A-Antigen binding site; B-Variable region antibodies.
(of L-Chain); C - Constant region (of L-Chain); D - (b) Inactivation of antibodies by bacterial enzymes.
Light polypeptide chain (L-Chain); E - Heavy (c) Decreased efficiency of the immune system.
polypeptide chain (H-Chain); F - Hydrogen bond (d) Insensitivity of the individual following prolonged
exposure to antibiotics.
CRITICAL THINKING TYPE QUESTIONS 113. Which part of the brain is not affected by alcohol?
(a) Cerebrum (b) Cerebellum
104. Which of the following pair of diseases is caused by virus?
(c) Medulla oblongata (d) Pons varolii
(a) Typhoid and tetanus
114. Smoking addiction is harmful because it produces
(b) AIDS and syphilis
polycyclic aromatic hydrocarbons which causes
(c) Rabies and mumps
(a) reduction in oxygen transport.
(d) Cholera and tuberculosis
(b) retardation of growth of foetus.
105. A certain patient is suspected to be suffering from acquired
immuno deficiency syndrome. Which diagnostic (c) increase in blood sugar level.
technique will you recommend for its detection? (d) cancer.
(a) WIDAL (b) ELISA 115. In alcoholics, liver gets damaged as it
(c) MRI (d) Ultrasound (a) secretes more bile.
106. A person suffering from a disease caused by Plasmodium (b) stores excess of glycogen.
experiences recurring chill and fever at the time when (c) accumulates excess of fats.
(a) The sporozoites released from RBCs are being rapidly (d) all of the above.
killed and broken down inside speen. 116. There is a patient having a disease in which a semi–solid
(b) The trophozoites reach maximum growth and give material oozes out and forms a tough membrane over it in
out certain toxins. air passage. The disease is called
(c) The parasite after its rapid multiplication inside RBCs (a) diphtheria (b) pertussis
ruptures them, releasing the stage to enter fresh (c) tetanus (d) TB
RBCs. 117. What is common between mumps, cholera and TB ?
(d) The microgametocytes and megagametocytes are (a) They are all bacterial diseases.
being destroyed by the WBCs. (b) They are all endemic diseases.
107. Hormone produced against allergic reaction is (c) They are all viral diseases.
(a) epinephrine (b) nor-epinephrine (d) They are all communicable diseases.
(c) glucocorticoid (d) mineralocorticocoid 118. Saline is given to a person suffering with cholera because
108. Vaccines produced through genetic engineering are (a) it causes lysis of bacterial cell wall.
considered safe because they (b) cholera results in severe diarrhoea leading to loss of
(a) are active form of antigens. salts.
(b) are the least active forms. (c) both (a) & (b)
(c) contain antibodies for coat proteins only. (d) saline helps to produce antitoxins.
(d) contain antibodies against whole antigen. 119. A person is injected with globulin against hepatitis. This
109. If you keep the sanitary system around yourself sound is
then the disease which will not most probably break out (a) naturally acquired active immunity.
is : (b) naturally acquired passive immnity.
(a) cholera (b) malaria (c) artificially acquired active immunity.
(c) beri-beri (d) scurvy (d) artificially acquired passive immunity.
Human Health and Disease 269

120. Damage to thymus in a child may lead to 124. Given below are some reasons which affect health of
(a) reduction in haemoglobin content of blood. human beings. Identify the correct reason.
(b) reduction in stem cell production. (i) Genetic disorders (ii) Infections
(iii) Life styles (iv) Mental state
(c) loss of antibody mediated immunity.
(a) (i), (ii) and (iii) (b) (ii), (iii) and (iv)
(d) loss of cell mediated immunity. (c) (i), (iii) and (v) (d) All of these
121. The best HLA (human leukocyte antigen) match for 125. A person is suffering from one disease and shows the
transplants in order of preference is following symptoms, like sustained high fever (39 to 40
(a) sibling > twin > parent > unrelated donor degree Celsius), weakness, stomach pain, constipation,
(b) twin > unrelated donor > parent > sibling headache and loss of appetite. His doctor confirmed that
(c) twin > sibling > parent > unrelated donor he is suffering from one bacterial infection.
(d) sibling > parent > twin > unrelated donor On which of the following facts (given below) his doctor
confirmed the bacterial infection.
122. Which one of the following pairs of diseases is viral as (i) Salmonella typhi bacteria are responsible for the
well as transmitted by mosquitoes? infection.
(a) Elephantiasis and dengue (ii) Streptococcus pneumonia is responsible for the
(b) Yellow fever and sleeping sickness infection.
(c) Encephalitis and sleeping sickness (iii) By seeing the report which shows the positive result
(d) Yellow fever and dengue of Widal test.
123. The pathogen Microsporum responsible for ringworm (iv) The pathogen is transmitted through the bite of
female Anopheles mosquito.
disease in humans belongs to the same kingdom of
(v) As a result of infection, the alveoli get filled with
organisms as that of
fluid leading to severe problem in respiration.
(a) Rhizopus, a mould (a) (i) and (iii) only
(b) Ascaris, a round worm (b) (ii), (iii) and (v) only
(c) Taenia, a tapeworm (c) (ii), (iii), (iv) and (v) only
(d) Wuchereria, a filarial worm (d) (i), (ii), (iv) and (v) only
EBD_7209
31
Strategies for

Chapter
Enhancement in Food
Production

FACT/DEFINITION TYPE QUESTIONS 8. A branch of science that deals with the maintenance of
hives of honeybees for the production of honey is called
1. India and China have more than 70% of world livestock ______.
population and produce the following percentage of world (a) aquaculture (b) pisciculture
farm. (c) apiculture (d) sericulture
(a) 10 % (b) 25 % 9. Which of the following species of honeybee is reared in
(c) 40 % (d) 50 % artificial hives?
2. When breeding is between animals of the same breed it is (a) Apis indica (b) Apis florea
called ______________, while crosses between different (c) Apis rohita (d) Apis dorsata
breeds are called ______________. 10. Which one of the following is a freshwater fish?
(a) out-breeding; inbreeding (a) Catla (b) Rohu
(b) inbreeding; out-breeding (c) Common carp (d) All of these
(c) out breeding; cross-breeding 11. Which one of the following is/are marine fish?
(d) cross-breeding; inbreeding (a) Rohu (b) Hilsa
3. Inbreeding depression
(c) Mackerel (d) Both (b) and (c)
(a) usually increases fertility only.
12. 33 percent of India’s GDP (Gross Domestic Product) comes
(b) usually reduces productivity only.
from ______________ and employs _____________
(c) usually reduces fertility and productivity.
percent of the population.
(d) usually increases fertility and productivity.
(a) industry; 70 (b) agriculture; 62
4. Which one of the following is a new breed of sheep
(c) export; 30 (d) agriculture; 75
developed in Punjab by crossing Bikaneri ewes and
Marino rams? 13. The new varieties of plants are produced by
(a) Hisardale (b) White Leghorn (a) selection and hybridization.
(c) Assel (d) Langshan (b) mutation and selection.
5. The 'mule' is the result of (c) introduction and mutation.
(a) inbreeding depression (d) selection and introduction.
(b) out-breeding 14. Which of the following is a variety of Brassica resistant
(c) cross-breeding to white rust disease?
(d) inter-specific hybridization (a) Himgiri (b) Pusa komal
6. MOET stands for (c) Pusa swarnim (Karan rai) (d) Pusa Sadabahar
(a) Multiple Ovulation and Embryo Transfer 15. Pusa Snowball K–1 is a variety of
Technology (a) wheat (b) chilli
(b) Multiple Ovulation Energy Transport Technology (c) cowpea (d) cauliflower
(c) Method of Ovulation Energy Transfer Technology 16. Which of the following processes is used for creating
(d) Method of Ovulation Energy Transport Technology genetic variation by changing the base sequence within
7. Which of the following stage is transferred to surrogate genes resulting in the creation of a new character or trait
mothers in livestock breeding experiments? not found in the parental type?
(a) Unfertilized eggs (b) Fertilized eggs (a) Selection (b) Hybridization
(c) 8 to 32 celled embryo (d) Frozen semen (c) Mutation (d) Breeding
Strategies for Enhancement in Food Production 271

17. Which of the following is generally used for induced (d) Inbreeding exposes harmful recessive genes that are
mutagenesis in crop plants? eliminated by selection.
(a) X-rays (b) UV (260 nm) radiations 28. Which of the following statement(s) is/are correct ?
(c) Gamma rays (from cobalt 60)(d) Alpha particles (a) Haploid culture technique was developed by Guha
18. Which one of the following technique is used for the and Maheshwari.
manipulation of plant species in order to create desired (b) A line consists of a group of individuals related by
plant types that are better suited for cultivation and give descent and with similar genotype.
better yields and are disease resistant?
(c) Mutation is a sudden heritable change in a character
(a) Out-breeding (b) Out-crossing
of an organism.
(c) Cross-breeding (d) Plant breeding
(d) All of the above
19. The entire collection (of plants/ seeds) having all the
29. Which of the following statements is correct
diverse alleles for all genes in a given crop is called
_________. collection. (a) A gene bank should not be regarded as a plant mu-
(a) germplasm (b) genome seum.
(c) genebank (d) genotype (b) The germplasm, stored in the gene bank are actively
20. In Abelmoschus esculentus (bhindi), resistance genes are utilized by breeders to develop novel varieties.
transferred from a wild species against yellow mosaic virus (c) The phase between 1960 - 1970 is often called the
and resulted in a new variety of A. Esculentus called _____ Green Revolution.
(a) Pusa swarnim (b) Himgiri (d) All of the above
(c) Pusa sadabahar (d) Parbhani kranti 30. Which of the following statement(s) is/are incorrect?
21. Which of the following factor(s) is/are responsible for (a) In 2000, maize hybrids that had twice the amounts of
resistance to maize stem borers? the amino acids, lysine and proline, compared to
(a) High aspartic acid (b) Low nitrogen content existing maize hybrids were developed.
(c) Low sugar content (d) All of the above (b) The Indian Agricultural Research Institute, New
22. Which of the following variety of wheat, having a high Delhi has released several vegetable crops that are
protein content, has been used as a donor for improving rich in vitamins and minerals.
cultivated wheat ? (c) Some of the diseases caused by fungi are black rot
(a) Himgiri (b) Atlas 66 of crucifers, tobacco mosaic, turnip mosaic etc.
(c) Sonalika (d) Kalyan Sona (d) Both (a) and (c)
23. A plant cell has potential to develop into full plant. This 31. Consider the following four statements (i – iv) and select
property of the plant cell is called ________ . the option which includes all the correct ones only.
(a) tissue culture (b) totipotency (i) Single cell Spirulina can produce large quantities of
(c) pleuripotency (d) gene cloning food rich in protein, minerals, vitamins, etc.
24. An explant is a
(ii) Body weight-wise, th e micro-organ ism
(a) dead plant. (b) part of the plant.
Methylophilus methylotrophus may be able to
(c) part of the plant used in tissue culture.
produce several times more proteins than the cows
(d) part of the plant that expresses a specific gene.
per day.
25. The technique of obtaining large number of plantlets by
(iii) Common button mushrooms are a very rich source
tissue culture method is called _______ .
of vitamin C.
(a) micropropagation (b) macropropagation
(iv) A rice variety has been developed which is very rich
(c) plantlet culture (d) organ culture
in calcium.
26. Protoplast of two different species are fused in _______.
(a) (iii) and (iv) only (b) (i), (iii) and (iv) only
(a) micropropagation (b) somatic hybridization
(c) (ii), (iii) and (iv) only (d) (i) and (ii) only
(c) clonal propagation (d) organography
32. Select the correct statement(s).
STATEMENT TYPE QUESTIONS (i) IARI has released a mustard variety rich in vitamin C.
(ii) Pusa Sawani, a variety of Okra is resistant to aphids.
27. Which of the following statements about breeding is
(iii) Hairiness of leaves provides resistance to insect
incorrect?
pests.
(a) By inbreeding, purelines cannot be evolved.
(b) Continued inbreeding, especially close inbreeding (iv) Agriculture accounts for approximately 33% of India's
reduces fertility and productivity. GDP and employs nearly 62% of the population.
(c) Cross-breeding allows desirable qualities of two (a) (i) and (ii) (b) (ii) and (iii)
different breeds to be combined. (c) (i), (iii) and (iv) (d) None of these
EBD_7209
272 Biology
33. Read the following statement(s) and answer the question. (a) Only (i) (b) Only (iii)
(i) Animal husbandry is the industrial practice of (c) Both (ii) and (iii) (d) All of these
breeding and raising livestock. 38. Which of the following statement is correct?
(ii) Dairy farm management deals with processes which (a) Sonalika and Kalyan sona were low yielding and
improve the quality and quantity of milk production. disease resistant crops.
(iii) Poultry is the class of domesticated birds used for (b) Saccharum barberi had high sugar content and
food or eggs. yield, but was originally grown in north India.
Which of the following statement (s) is/are correct? (c) Saccharum officinarum had thicker stems and higher
(a) Only (i) (b) Only (iii) sugar content, but did not grow well in north India.
(c) Both (ii) and (iii) (d) All of these (d) Both (a) and (c)
34. Which of the following statement(s) is/are correct
regarding inbreeding? MATCHING TYPE QUESTIONS
(i) It is the mating of more closely related individuals
within the same breed for 4-6 generations. 39. Match column-I with column-II and choose the correct
(ii) It decreases homozygosity . option.
(iii) It exposes harmful recessive genes that are eliminated Column -I Column -II
by selection. A. Pisciculture I. Micropropagation
(iv) Inbreeding depression results due to continued B. Apiculture II. Crop production
Inbreeding, especially close – inbreeding, may reduce C. Tissue culture III. Rearing of fishes
fertility and productivity. D. Green revolution IV. Fish-production
(a) (i) and (ii) (b) (iii) and (iv) E. Blue revolution V. Bee-keeping
(c) (ii), (iii) and (iv) (d) (i), (iii) and (iv) (a) A – III; B – V; C – IV; D – I; E – II
35. Which of the following includes all the characteristics (b) A – III; B – V; C – I; D – II; E – IV
described given below ? (c) A – V; B – III; C – I; D – II; E – IV
(i) In this method of breeding, the mating of animals (d) A – III; B – V; C – II; D – I; E – IV
occurs with same breed. 40. Match the column-I with column-II and choose the correct
(ii) They have no common ancestors on either side of option.
their pedigree upto 4 - 6 generations.
Column-I Column-II
(iii) It is best for those animals that are below average in
A. Many people I. Single cell proteins
productivity in milk production, growth rate in beef
cattle. have deficiencies
(iv) It helps to overcome inbreeding depression. as they cannot buy
(a) Inbreeding (b) Out-breeding fruits & vegetables
(c) Out-crossing (d) Cross-breeding B. Crops with higher II. Micropropagation
36. Which of the following statement(s) is/are correct? vitamins, proteins
(i) Fishery is an industry, which deals with catching, and fats
processing and marketing of fishes and other aquatic C. Growing microbes III. Somaclones
animals such as prawn, crab, lobster, edible oyster, as the alternative
etc. that have a high economic value. source of proteins
(ii) Both aquaculture and pisciculture are same. D. Capacity to IV. Hidden hunger
(iii) The development and flourishing of the fishery generate plant
industry is called silver revolution. from a single cell
(a) Only (i) (b) Both (ii) and (iii) or explant
(c) Both (i) and (iii) (d) All of these E. Production of V. Bio-fortification
37. Which of the following statement(s) is/are correct? thousand plants
(i) Out-breeding is the breeding of the unrelated animals
through tissue culture
which may be between individuals of the same breed
F. Genetically VI. Totipotency
but having no common ancestors for 4 - 6 generations
or between different breeds or different species. identical plants
(ii) Cross-breeding allows the desirable qualities of two (a) A – IV; B – V; C – VI; D – I; E – II; F – III
different breeds to be combined. (b) A – IV; B – V; C – VI; D – I; E – III; F – II
(iii) In interspecific hybridization, male and female (c) A – IV; B – V; C – I; D – VI; E – II; F – III
animals of two different related species are mated. (d) A – VI; B – V; C – I; D – IV; E – II; F – III
Strategies for Enhancement in Food Production 273

41. Choose the option showing the crop plants with its correct ASSERTION/REASON TYPE QUESTIONS
A. Semi Dwarf Wheat I. Sonalika
B. Semi Dwarf Rice II. Kalyan sona In the following questions, a statement of Assertion is followed
III. IR-8 by a statement of Reason.
IV. Jaya (a) If both Assertion and Reason are true and the Reason is
V. Taichung Native-1 the correct explanation of the Assertion.
VI. Ratna (b) If both Assertion and Reason are true but the Reason is
not the correct explanation of the Assertion.
(a) A – I, III, V; B – II, IV, VI (b) A – III, IV, V, VI; B – I, II
(c) If Assertion is true but Reason is false.
(c) A – I, II, IV; B – III, V, VI (d) A – I, II; B – III, IV, V, VI
(d) If both Assertion and Reason are false.
42. Match the crops given in column-I with their resistance 47. Assertion : Somatic embryos can be induced from any
to disease given in column-II and choose the correct cell in plant tissue culture.
option. Reason : Any living plant cell is capable of differentiating
Column-1 Column-II into somatic embryos.
(Crop) (Resistance to diseases) 48. Assertion : Fish meal is a rich protein source for poultry
A. Wheat I. Tobacco mosaic virus and cattle.
and leaf curl Reason : It is produced from the fins and tail (the non-
B. Brassica II. Bacterial blight edible parts)
C. Cowpea III. Leaf and stripe rust 49. Assertion : An important technique of genetic
D. Cauliflower IV. White rust engineering is protoplast culture.
Reason : It results in the production of genetically
E. Chilli V. Black rot and curl
modified crops.
blight black rot
50. Assertion : Meristem tissue culture can produced virus
(a) A – IV; B – III; C – II; D – I; E – V free plants from virus infected plants.
(b) A – III; B – IV; C – II; D – V; E – I Reason : In this technique the growth of the virus is
(c) A – IV; B – III; C – V; D – I; E – II inhibited during the growth of the host tissue.
(d) A – III; B – IV; C – I; D – V; E – II 51. Assertion : A major advantage of tissue culture is
43. Which of the following crop plant is not matching as protoplast fusion.
correct pair with its variety ? Reason : A hybrid is formed by the fusion of naked
(i) Wheat – Himgiri protoplasts of two plants.
(ii) Brassica – Pusa Gaurav
(iii) Cauliflower – Pusa Komal DIAGRAM TYPE QUESTION
(iv) Chilli – Pusa Sadabahar 52. Refer the given figures and answer the questions.
(v) Okra – Pusa Sawani
(a) Only (i) (b) (ii) and (iii)
(c) Only (iii) (d) (ii) and (iv)
44. Which of the following is not correctly matched with its
variety?
(a) Brassica (rapeseed mustard) – Pusa Gaurav
(b) Cowpea – Pusa Sadabahar
(c) Flat been –Pusa sem 2
(d) Okra (Bhindi) – Pusa A- 4 Which of the following statements is correct regarding
45. Which of the following is incorrectly matched ? the above figures?
(a) Explant – Excised plant part used (i) These are all Indian hybrid crops of low yielding variet-
for callus formation ies.
(b) Cytokinins – Root initiation in callus (ii) These are all Indian hybrid crops of high yielding variet-
(c) Somatic embryo – Embryo produced ies.
from a vegetative cells (iii) The production of the above crops led to dramatic
(d) Anther culture – Haploid plants increase in food production.
46. Which of the following pair is correctly matched? (iv) These crops are produced as a result of various
(a) Sericulture - Fish plants breeding technique.
(b) Pisciculture - Silk moth (a) (i), (ii), and (iii) (b) (ii), (iii) and (iv)
(c) Aquaculture - mosquitoes (c) (iii) and (iv) only (d) (i) and (iii) only
(d) Apiculture - Honey bee
EBD_7209
274 Biology
(c) Reduced yield, lower quality of produce and
CRITICAL THINKING TYPE QUESTIONS
poisonous produce.
53. Select the correct chronological order of the events (d) Reduced yield, lower quality of produce, increased
occuring during callus culture cost of production and poisonous produce.
(a) Callus ® Cell division ® Explant ® Addition of 61. Sharbati sonora variety of wheat was obtained by
cytokinin ® Cells acquire meristematic property. (a) X-ray treatment.
(b) Explant ® Callus ® Cell division ® Addition of (b) crossing with wild varieties of wheat.
cytokinin ® Cells acquire meristematic property. (c) hybridization between wild grasses.
(c) Explant ® Cell division ® Callus ® Addition of (d) irradiation of sonora 64 with gamma rays.
cytokinin ® Cells acquire meristematic property. 62. Plants derived sexually from the same plant are
(d) Callus ® Explant ® Cell division ® Addition of _______while those derived from somatic tissue from
cytokinin ® Cells acquire meristematic property. the same plant are ______________.
54. Which one of the following combination would a sugar- (a) identical, different (b) different, also different
cane farmer look for in the sugarcane crop ? (c) different, identical (d) identical, also identical
(a) Thick stem, long internodes, high sugar content and 63. Which of the following pair of hormones are required for
disease resistant. a callus to differentiate ?
(b) Thick stem, high sugar content and profuse flower- (a) Auxin and cytokinin
ing. (b) Auxin and ethylene
(c) Auxin and abscisic acid
(c) Thick stem, short internodes, high sugar content,
(d) Cytokinin and gibberellin
disease resistant.
64. In crop improvement programme, haploids are important
(d) Thick stem, low sugar content, disease resistant. because they
55. The biggest compulsion of plant breeding is (a) require one half of nutrients.
(a) infrastructure. (b) are helpful in study of meiosis.
(b) trained manpower. (c) grow better under adverse conditions.
(c) transfer of genes from unrelated sources. (d) form perfect homozygous.
(d) availability of desirable gene in the crop and its wild 65. Which of the following techniques used in animal
biotechnology are required for the rapid multiplication
relatives.
and production of animals with a desirable genotype ?
56. High milk yielding varieties of cows are obtained by (a) Protoplast fusion and embryo transfer.
(a) super ovulation (b) artificial insemination (b) Hybrid selection and embryo transfer.
(c) use of surrogate mothers (d) all of the above (c) In vitro fertilization and embryo transfer.
57. Crop improvement is possible through (d) All of the above
66. The way in which biotechnology is contributed to
(a) judicious combination of selection, introduction and
sustainable agriculture is
hybridization. (a) biofertilizer
(b) selection (b) single cell protein (SCP)
(c) scientific improvement of cultivated plants. (c) disease and insect resistant varieties
(d) introduction. (d) all of the above
58. In order to obtain virus-free plants through tissue culture 67. An improved variety of transgenic basmati rice
the best method is (a) does not require chemical fertilizers and growth hormones.
(b) gives high yield and is rich in vitamin A.
(a) meristem culture (b) protoplast culture
(c) is completely resistant to all insect pests and
(c) embryo rescue (d) anther culture
diseases of paddy.
59. Which of the following is the process of choosing parent (d) gives high yield but has no characteristic aroma.
organisms for the characteristic that is wanted in their 68. In tissue culture medium, the embryoids formed from
offspring? pollen grains is due to
(a) Active selection (b) Reproductive selection (a) cellular totipotency (b) organogenesis
(c) Selective breeding (d) None of the above (c) double fertilization (d) test-tube culture
60. Which of the following is the consequence of plant 69. Plants can be disease resistant by
diseases? (a) breeding with their wild relatives.
(a) Reduced yield and lower quality of produce. (b) colchicine treatment.
(b) Reduced yield, lower quality of produce and (c) hormone treatment.
increased cost of production. (d) heat treatment.
Strategies for Enhancement in Food Production 275

70. The greatest threat to genetic diversity in agricultural crops (c) capacity to generate hybrid protoplasts.
is (d) recovery of healthy plants from diseased plants.
(a) extensive use of insecticides and pesticides. 74. Hisardale is a new breed of sheep developed in Punjab
(b) extensive mixed cropping. by one of the breeding technique in which superior male
(c) introduction of high yielding varieties. of one breed is mated with superior females of another
(d) extensive use of fertilizers. breed.
71. Which of the following is not used for crop Identify the breeding technique from the option given be-
improvement? low.
(a) Inbreeding (b) Introduction (a) Inbreeding (b) Out crossing
(c) Hybridization (d) Mutations (c) Out breeding (d) Cross breeding
72. Somaclonal variation appears in 75. On the basis of the following process, answer the
(a) organisms produced through somatic hybridization. question.
(b) plants growing in highly polluted conditions.
(i) gene transfer
(c) apomictic plants.
(ii) transfer of cytoplasm
(d) tissue culture raised plants.
(iii) production of useful allopolyploids
73. Totipotency refers to the
(a) capacity to generate genetically identical plants. Which of the following term is used in the given process?
(b) capacity to generate a whole plant from any plant (a) Somatic hybrid (b) Single cell protein
cell explant. (c) Bio - fortification (d) None of the above
EBD_7209
Microbes in Human
32

Chapter
Welfare

FACT/DEFINITION TYPE QUESTIONS 10. Which one of the following is used in the manufacture of
alcohol ?
1. Which of the following bacteria converts milk into curd ? (a) Bacteria (b) Bread molds
(a) Propionibacterium (b) Lactobacillus (c) Yeasts (d) Slime molds
(c) Streptococcus (d) Bacillus 11. Conversion of sugar into alcohol during fermentation is
2. Which role is played by lactic acid bacteria (LAB) in our due to the direct action of
stomach ? (a) temperature
(a) Beneficial (b) Harmful (b) micro-organisms
(c) Neutral (d) All of these (c) zymase
3. Lactic acid bacteria convert milk into curd and improves (d) concentration of sugar solution
its nutritional quality by enhancing 12. Glucose fermentation by yeast yields
(a) vitamin A (b) vitamin B (a) ethanol + CO2 (b) ethanol + H2O
(c) vitamin C (d) vitamin D (c) methanol + CO2 (d) H2O + CO2
4. The bacterium that commonly lives in animal and human
intestine is 13. Streptokinase, used as a ‘clot buster’ is obtained from
(a) Bacillus anthracis (b) Vibrio cholerae (a) Streptococcus (b) Staphylococcus
(c) Escherichia coli (d) Corynebacterium (c) Lactobacillus (d) Saccharomyces
14. Cyclosporin A is used for
5. Which gas is responsible for the puffed-up appearance
of dough ? (a) dissolving blood clots
(b) lowering cholesterol level
(a) CO2 (b) O2
(c) immunosuppression
(c) SO2 (d) NO2
(d) enhancing tenderness of meat
6. In cheese microorganisms are required for
15. The bioactive molecule cyclosporin A is used in the treatment
(a) ripening only
of
(b) souring of milk only
(a) whooping cough (b) diphtheria
(c) souring and ripening (c) leprosy (d) organ-transplant patients
(d) development of resistance to spoilage 16. Statins are obtained from
7. Which of the following scientists sh owed that
(a) Streptococcus (b) Mucor javanicus
Saccharomyces cerevisiae causes fermentation forming
(c) Monascus purpureus (d) Clostridium butyricum
products such as beer and buttermilk ?
17. Statins, a bioactive molecule, inhibits the enzyme
(a) Louis Pasteur (b) Alexander Flemming
responsible for synthesis of
(c) Selman Waksman (d) Schatz
(a) carbohydrate (b) protein
8. Saccharomyces cerevisiae is employed in production of
(c) vitamins (d) cholesterol
(a) idli (b) beer
18. Monascus purpureus is a yeast used commercially in the
(c) bread (d) all of these
production of
9. Which bacterium helps in the production of ‘Swiss
(a) ethanol
cheese’?
(b) streptokinase for removing clots from the blood
(a) Propionibacterium sharmanii
vessels
(b) Trichoderma polysporum
(c) citric acid
(c) Saccharomyces cerevisiae
(d) blood cholesterol lowering statins
(d) Aspergillus niger
Microbes in Human Welfare 277

19. The large vessels for growing microbes on an industrial 31. The technology of biogas production was developed in
scale are called _________ . India mainly due to the efforts of
(a) petridish (b) digestors (a) Indian Agricultural Research Institute (IARI) and
(c) biogas vessel (d) fermentors Khadi and Village Industries Commission (KVIC).
20. Sewage purification is done by (b) National Botanical Research Institute (NBR1).
(a) microbes (b) fertilizers (c) Indian Council of Medical Research (ICMR).
(c) antibiotics (d) antiseptics (d) Indian Council of Agricultural Research (ICAR).
21. The chemical substances produced by some microbes 32. Insect resistant transgenic cotton has been produced by
which can kill or retard the growth of other microbes are inserting a piece of DNA from
called _________ . (a) an insect
(a) toddy (b) lactic acid
(b) a wild relative of cotton
(c) antibiotics (d) ethanol
22. Primary treatment of sewage is (c) a bacterium
(a) physical process (b) biological process (d) a virus
(c) chemical process (d) biochemical process 33. Microbe used for biocontrol of pest butterfly caterpillars
23. Primary sludge is used for the is
(a) preparation of compost (b) preparation of manure (a) Trichoderma sp.
(c) biogas production (d) all of these (b) Saccharomyces cerevisiae
24. Passage of effluents into oxidation tank is for (c) Bacillus thuringiensis
(a) primary treatment (b) secondary treatment (d) Streptococcus sp.
(c) tertiary treatment (d) both (a) and (b) 34. The free-living fungus Trichoderma can be used for
25. The amount of oxygen required by microbes in the (a) killing insects.
decomposition of organic matter is called _________ .
(b) biological control of plant diseases.
(a) chemical oxygen demand
(c) controlling butterfly caterpillars.
(b) biochemical oxygen demand
(c) total oxygen demand (d) producing antibiotics.
(d) dissolve oxygen 35. Baculoviruses (nucleopolyhedrovirus) does not show
26. BOD refers to (a) host specificity.
(a) bacteria oxygen demand (b) narrow spectrum applications.
(b) biochemical oxygen demand (c) effects on non-target pathogens.
(c) biochemical operation demand (d) utility in IPM programme.
(d) biological organism demand 36. Which of the following serve as biofertilizer in paddy
27. Sewage treatment process in which part of decomposer fields?
bacteria is recycled into starting of the process is called (a) Bacteria
__________ . (b) Yeast
(a) cyclic treatment (c) Cyanobacteria (blue-green algae)
(b) primary treatment (d) Fungi
(c) tertiary treatment 37. A free living nitrogen-fixing cyanobacterium which can
(d) activated sludge treatment also form symbiotic association with the water fern Azolla
28. During sewage treatment, biogases produced include is
which of the following gases ? (a) Anabaena (b) Tolypothrix
(a) Methane, oxygen, hydrogen sulphide (c) Chlorella (d) Nostoc
(b) Hydrogen sulphide, methane, sulphur dioxide 38. Which of the following plants is used as biofertilizer ?
(c) Hydrogen sulphide, nitrogen, methane (a) Nostoc (b) Funaria
(d) Methane, hydrogen sulphide, carbon dioxide (c) Volvox (d) Rhizopus
29. Ganga and Yamuna action plan is initiated by
39. Symbiotic asociation is exhibited by
(a) Ministry of environment and forest.
I. Mycorrhiza II. Rhizobium
(b) Ministry of agriculture.
(c) Ministry of wild-life conservation. III. Heterocyst IV. Yeast
(d) None of the above (a) I, III, IV (b) I, II, III, IV
30. Methanogenic bacteria are present in (c) II, III, IV (d) I, II
(a) anaerobic sludge 40. The symbiotic association between fungi and roots of
(b) rumen (a part of stomach) of cattle higher plants is called __________ .
(c) both (a) and (b) (a) lichen (b) mycorrhiza
(d) none of the above (c) biofertilizer (d) BOD
EBD_7209
278 Biology

STATEMENT TYPE QUESTIONS 48. Which of the following statement is correct ?


(a) Methanobacterium is an aerobic bacterium found in
41. Baculoviruses are excellent candidates for rumen of cattle.
(a) species-specific narrow spectrum pesticidal (b) Biogas (commonly called gobar gas) is pure methane.
applications. (c) Activated sludge sediment in settlement tanks of
(b) species-specific broad spectrum pesticidal sewage treatment plants is rich source of aerobic
applications. bacteria.
(d) Biogas is produced by the activity of aerobic bacteria
(c) species-specific narrow spectrum insecticidal
on animal waste.
applications.
49. Which one of the following is the most important role of
(d) species-specific broad spectrum insecticidal microorganism for the well-being of humans?
applications.
(a) Sewage treatment
42. Which one of the following statement regarding BOD is (b) Production of methane
true? (c) Biological control of plant disease
(a) The greater the BOD of waste water, more is its (d) Conversion of milk to curd
polluting potential.
50. Consider the following statements about organic farming.
(b) The greater the BOD of waste water, less is its
(i) It utilizes genetically modified crops like Bt cotton.
polluting potential.
(ii) It uses only naturally produced inputs like compost.
(c) The lesser the BOD of waste water, more is its
(iii) It does not use pesticides and urea.
polluting potential.
(iv) It produces vegetables rich in vitamins and minerals.
(d) The lesser the BOD of waste water, less is its
Which of the above statements are correct?
polluting potential.
43. Which one of the following is not true about antibiotics? (a) (ii), (iii) and (iv)
(a) First antibiotic was discovered by Alexander (b) (iii) and (iv) only
Flemming. (c) (ii) and (iii) only
(b) The term ‘antibiotic’ was coined by S. Waksman in (d) (i) and (ii) only
1942. 51. What are the advantages of gobar gas over conventional
(c) Some persons can be allergic to a particular antibiotic. utilization?
(d) Each antibiotic is effective only against one particular (i) It is most efficient source of energy.
kind of germ. (ii) It is used as good fertilizer.
44. Pollution from animal excreta and organic wastes from (iii) It reduces the chances of spreading of pathogens.
kitchen can be most profitably minimised by (a) (i) and (ii) only (b) (ii) and (iii) only
(a) storing them in underground storage tanks. (c) (i) and (iii) only (d) all of these
(b) using them for producing biogas. ASSERTION/REASON TYPE QUESTIONS
(c) vermiculture.
(d) using them directly as biofertilizers. In the following questions, a statement of Assertion is followed
45. Benefits of mycorrhizae are by a statement of Reason.
(i) resistance to root-borne pathogens. (a) If both Assertion and Reason are true and the Reason is
(ii) tolerance of salinity and absorption of phosphorus. the correct explanation of the Assertion.
(iii) tolerance to drought. (b) If both Assertion and Reason are true but the Reason is
(iv) overall increase in the plant growth and not the correct explanation of the Assertion.
development. (c) If Assertion is true but Reason is false.
(a) (i) and (ii) (b) (ii) and (iii) (d) If both Assertion and Reason are false.
(c) (iii) and (iv) (d) all of these 52. Assertion : Vitamins B2 is found in cereals, green
46. Organic farming includes vegetables, brewer's yeast, egg white, milk and liver.
(a) use of fertilizers and pesticides of biological origin. Reason : It can be commercially produced by some yeasts.
(b) IPM (integrated pest management programme). 53. Assertion : Use of fertilizers greatly enhances crop
(c) locally developed pest resistant varieties. productivity.
(d) all of the above Reason : Irrigation is very important in increasing crop
47. Which of the following statements is not true for stirred productivity.
tank fermentaion ? 54. Assertion : In crop rotation, the plants that are best
(a) Buffer needed to control pH. preferred are leguminous plants.
(b) Batch and feed possible. Reason : Such plants have root nodules containing
(c) Control dissolved oxygen. clostridium bacteria able to fix nitrogen.
(d) Easy in process sampling.
Microbes in Human Welfare 279

55. Assertion : Biopesticides help in overcoming the (a) A – III; B – I; C – V; D – II


disadvantages of chemical pesticides. (b) A – III; B – I; C – V; D – IV
Reason : Biopesticides are harmless as they do not cause (c) A – I; B – II; C – III; D – IV
any damage. (d) A – II; B – I; C – III; D – IV
56. Assertion : Biogas has been used as a fuel for the
61. Find out the pairs which are correctly matched.
purposes of lighting and cooking.
Reason : It does not cause pollution and is ecofriendly. Column-I Column-II
A. Cyanobacteria I. Biopesticides
MATCHING TYPE QUESTIONS B. Mycorrhiza II. Solubilization of
57. Which one of the following is wrongly matched? phosphate
(a) Yeast – Ethanol C. Bacillus III. Cry protein
(b) Streptomyces – Antibiotics thuringiensis
(c) Lactobacillus - Beer D. Single cell protein IV. Rhizobia
(d) Methanogens – Gobar gas (a) A and II (b) C and III
58. Match the columns and choose the correct option. (c) C and IV (d) A and III
Column-I Column-II 62. The given table contains type of microbe (Column I),
A. Aspergillus niger I. Ethanol Scientific name (Column II) and commercial product
B. Clostridium II. Statins butydicum (Column III). Some names are replaced by A, B, C and D.
C. Saccharomyces III. Citric acid Identify the correct names.
cerevisiae
D. Trichoderma IV. Butyric acid Type of Microbe Scientific Name Commercial
polysporum Product
E. Monascus V. Cyclosporin A Bacterium A Lactic acid
purpureus Fungus B Cyclosporin A
(a) A – IV; B – III; C – II; D – I; E – V C Monascus purpureus Statins
(b) A – V; B – IV; C – I; D – II; E – III
Fungus Penicillium notatum D
(c) A – III; B – IV; C – I; D – V; E – II
(d) A – III; B – IV; C – V; D – I; E – II (a) A – Lactobacillus, B – Trichoderma polysporum,
59. Match the items of column-I with column-II and choose C – Yeast (fungus), D – Penicillin
correct answer.
(b) A – Lactobacillus, B – Trichoderma polysporum,
Column-I Column-II
C – Yeast (algae), D – Penicillin
A. Lady bird I. Methanobacterium
B. Mycorrhiza II. Trichoderma (c) A – Lactobacillus, B – Trichoderma polysporum,
C. Biological control III. Aphids C – Yeast (prokaryote), D – Penicillin
D. Biogas IV. Glomus (d) A – Lactobacillus, B – Trichoderma polysporum,
(a) A – II; B – IV; C – III; D – I C – Agaricus (fungus), D – Penicillin
(b) A – III; B – IV; C – II; D – I
63. The given table contains type of microbe (Column I),
(c) A – IV; B – I; C – II; D – III
Scientific name (Column II) and commercial product
(d) A – III; B – II; C – I; D – IV (Column III). Some names are replaced by A, B, C and D.
60. Match the columns and choose the correct combination. Identify the correct names.
Column-I Column-II
Type of Microbe Scientific Name Commercial
A. Escherichia coli I. ‘nif ’ gene
Product
B. Rhizobium II. Digests (hydrocarbons
Bacterium A Clot buster
meliloti of crude) oil enzyme
C. Bacillus III. Human insulin
B Aspergillus niger Citric acid
thuringiensis production
D. Pseudomonas IV. Biocontrol of fungal Fungus Trichoderma C
polysporum
putida disease
Bacterium D Butyric acid
V. Biodegradable
insecticide
EBD_7209
280 Biology
(a) A – Streptococcus, B – Fungus, C – Cyclosporin-A, 65. Match column-I with column-II and choose the correct
D – Clostridium butylicum option
(b) A – Clostridium butylicum, B – Streptococcus, Column-I Column-II
C – Fungus, D – Cyclosporin-A A. Statins I. Yeast
(c) A – Cyclosporin-A, B - Clostridium butylicum B. Ethanol II. Blood-cholesterol
C – Streptococcus, D – Fungus lowering agent
(d) A – Fungus, B – Cyclosporin-A, C – Clostridium C. Dung III. Insect-resistant plant
butylicum, D - Streptococcus D. Bt-cotton IV. Biogas
64. Identify the blank spaces marked A, B, C and D from the (a) A – II; B – I; C – IV; D – III
table given below. (b) A – III; B – IV; C – I; D – II
Medical (c) A – I; B – II; C – III; D – IV
Type of Scientific
Product Application (d) A – IV; B – II; C – I; D – III
Microbe Name
product
DIAGRAM TYPE QUESTIONS
(i) fungus A Cyclosporin B
66. The diagram below shows a typical biogas plant. With
Monascus few structure labelled as A, B and C. Identify A, B and C.
(ii) C statin D
purpurens
(a) A–Trichoderma polyspora, B – Organ transplant
patients, C – Yeast (fungus), D – Lowering of blood
cholesterol.
(b) A–Lowering of blood cholesterol, B – Trichoderma
polyspora, C – Organ transplant patients, D – Yeast
(fungus).
(a) A – Sludge, B – Methane, Oxygen, C – Dung, water
(c) A – Yeast (fungus), B – Lowering of blood (b) A – Sludge, B – Methane, Carbon dioxide, C– Dung,
cholesterol, C – Trichoderma polyspora, D – Organ water
transplant patients. (c) A – Sludge, B – Ethylin, Carbon dioxide, C – Dung,
(d) A – Organ transplant patients, B – Yeast (fungus), water
C – Lowering of blood cholesterol, D – Trichoderma (d) A – Sludge, B – Methane, Carbon dioxide, C – Sewage
polyspora.
67. The given figure shows the sewage treatment with few steps are marked as 1, 2, 3, and 4. In which of the following options,
correct word for all the four numbers (1,2,3 and 4) are indicated.

(a) 1 – Large aeration tanks, 2 – Chemical agitation, 3 – High, 4 – Anaerobic


(b) 1 – Large aeration tanks, 2 – Mechanical agitation, 3 – Low, 4 – Anaerobic
(c) 1 – Large aeration tanks, 2 – Chemical agitation, 3 –Low, 4 – Aerobic
(d) 1 – Large aeration tanks, 2 – Mechanical agitation, 3 – High, 4 – Anaerobic

CRITICAL THINKING TYPE QUESTIONS 69. Microbes are a diverse group which includes.
I. Bacteria II. Mosses
68. Which of the following is the pair of biofertilizers? III. Protozoans IV. Fungi
(a) Azolla and blue green algae (a) I, III, IV (b) I, IV
(b) Nostoc and legume (c) I, II (d) III, IV
(c) Rhizobium and grasses 70. Microbes are present in
(d) Salmonella and E. coli I. soil II. air
III. water IV. thermal springs
Microbes in Human Welfare 281

(a) I, III, IV (b) I, II, III, IV (c) Putrefying microbes, methanogens, saprophytic
(c) I, II (d) III, IV microbes.
71. The following bacteria help in nitrogen fixation from (d) Decomposers, fermentative microbes, methanogens.
atmosphere. 81. Read the following statement.
I. Azotobacter II. Rhizobium “A drug used for ———— (A) ———— patients is
III. Azospirillum IV. Lactobacillus obtained from a species of the organism ———— (B) —
Identify the correct bacteria. ———.”
(a) I, III, IV (b) I, II, III, IV Identify A and B.
(c) II, III, IV (d) I, II, III A B
72. Which of the following is used as biofertilizer ? (a) Heart Penicillium
I. Cyanobacteria II. Yeast (b) Organ-transplant Trichoderma
III. Symbiotic bacteria IV. Free living bacteria (c) Swine flu Monascus
(a) I, II, III (b) I, II, IV (d) AIDS Pseudomonas
(c) I, III, IV (d) II, III, IV 82. Which one of the micro-organism is used for production
73. Some industrial products are given below which are of citric acid in industries?
synthesized from microbes. (a) Lactobacillus bulgaricus (b) Penicillium citrinum
I. Antibiotics II. Fermented beverages (c) Aspergillus niger (d) Rhizopus nigricans
III. Enzymes and chemicals IV. Bioactive molecules 83. Biogas can be a good substitute for
Choose the correct option. (a) fuel wood (b) petroleum and oil
(a) I, III, IV (b) I, II, III, IV (c) coal (d) charcoal
(c) I, III (d) I, II, III 84. A genetically engineered micro-organism used
74. Methanogens grow anaerobically on cellulosic material successfully in bioremediation of oil spills is a species of
and produces which of the following gases ? (a) Pseudomonas (b) Trichoderma
I. Methane II. Oxygen (c) Xanthomonas (d) Bacillus
III. Carbon dioxide IV. Hydrogen 85. Bacillus thuringiensis (Bt) strains have been used as
(a) I, III, IV (b) I, II, III, IV (a) biofertilizers
(c) II, III, IV (d) I, II (b) biometallurgical techniques
75. Cheese and yogurt are products of (c) biomineralization processes
(a) pasteurisation (b) fermentation (d) bioinsecticidal plants
(c) dehydration (d) distillation 86. Microbes are used in the
76. Antibiotics are drugs commonly used to cure diseases of I. primary treatment of sewage.
(a) fungi (b) viruses II. secondary treatment of sewage.
(c) protozoans (d) bacteria III. anaerobic sludge digester.
77. Which of the following is common to Azospirillum, IV. production of bioactive molecules.
Azotobacter, Anabaena, Nostoc and Oscillatoria? Choose the correct option showing the uses of microbes.
(a) Prokaryotes (b) Nitrogen-fixers (a) I, III, IV (b) I, II, III, IV
(c) Both (a) and (b) (d) Eukaryotes (c) II, III, IV (d) IV, I, II
78. Lactobacillus mediated change of milk to curd occurs due to 87. What would happen if oxygen availability to activated
(a) coagulation and partial digestion of milk fats. sludge flocs is reduced?
(b) coagulation and partial digestion of milk proteins. (a) It will slow down the rate of degradation of organic
(c) coagulation of milk proteins and complete digestion matter.
of milk fats. (b) The centre of flocs will become anoxic, which would
(d) coagulation of milk fats and complete digestion of cause death of bacteria and eventually breakage of
proteins. flocs.
79. Crystals of Bt-toxin produced by some bacteria do not (c) Flocs would increase in size as anaerobic bacteria
kill the bacteria themselves because would grow around flocs.
(a) bacteria are resistant to the toxin. (d) Protozoa would grow in large numbers.
(b) toxin is inactive. 88. A lake with an inflow of domestic sewage rich in organic
(c) toxin is immature. waste may result in
(d) bacteria enclose toxins in a special sec. (a) drying of the lake very soon due to algal bloom.
80. Choose the correct sequence of microbes involved in (b) an increased production of fish due to lot of
biogas production. nutrients.
(a) Fragmentative microbes, decomposers, (c) death of fish due to lack of oxygen.
methanogens. (d) incr eased population of aquatic food web
(b) Decomposers, methanogens, putrefying microbes. organization.
EBD_7209
Biotechnology : Principles
33

Chapter
and Processes

9. Which one of the following palindromic base sequences


FACT/DEFINITION TYPE QUESTIONS
in DNA can be easily cut at about the middle by some
1. Plasmid has been used as vector because particular restriction enzyme?
(a) both its ends show replication. (a) 5'.............CGTTCG.............3'
(b) it can move between prokaryotic and eukaryotic cells. 3'.............ATGGTA.............5'
(c) it is circular DNA which have capacity to join to (b) 5'.............GATATG.............3'
eukaryotic DNA. 3'.............CTACTA.............5'
(d) it has antibiotic resistance gene. (c) 5'.............GAATTC.............3'
2. Which of the following is a plasmid? 3'.............CTTAAG.............5'
(a) pBR 322 (b) Bam H I (d) 5'.............CACGTA.............3'
(c) Sal I (d) Eco R I 3'.............CTCAGT.............5'
3. Which of the following is known as specific molecular 10. DNA fragments generated by the restriction
endonucleases in a chemical reaction can be separated
scissors?
by
(a) Ligase
(a) polymerase chain reaction
(b) Helicase (b) electrophoresis
(c) Restriction endonuclease (c) restriction mapping
(d) DNA polymerase (d) centrifugation
4. The first restriction endonuclease reported was 11. Agarose extracted from sea weeds is used in
(a) Hind II (b) EcoRI (a) spectrophotometry
(c) Hind III (d) BamHI (b) tissue culture
5. Restriction enzymes belong to a larger class of enzymes, (c) PCR
which is called as (d) gel electrophoresis
(a) ligases (b) kinases 12. In genetic engineering, the antibiotics are used
(c) nucleases (d) polymerases (a) as selectable markers.
6. There is a restriction endonuclease called EcoRI. What (b) to select healthy vectors.
does .co part in it stand for ? (c) to keep the cultures free of infection.
(a) Colon (b) Coelom (d) as sequences from where replication starts.
13. During heat shock to the bacterium, the temperature used
(c) Coenzyme (d) coli
for giving thermal shock is
7. Restriction endonuclease - Hind II always cuts DNA
(a) 52°C (b) 100°C
molecules at a particular point by recognizing a specific
(c) liquid nitrogen (d) 42°C
sequence of
14. Which of the following enzyme is used in case of fungus
(a) six base pairs. (b) five base pairs. to cause release of DNA along with other
(c) four base pairs. (d) seven base pairs. macromolecules ?
8. The enzyme used for joining two DNA fragments is called (a) Lysozyme (b) Cellulase
(a) ligase (c) Chitinase (d) Amylase
(b) restriction endonuclease 15. During isolation of DNA, addition of which of the
(c) DNA polymerase following causes precipitation of purified DNA ?
(d) gyrase (a) Chilled ethanol (b) Ribonuclease enzyme
(c) DNA polymerase (d) Proteases
Biotechnology : Principles and Processes 283

16. Which of the following is a natural genetic engineer of 26. Stirred-tank bioreactors have been designed for
plants ? (a) addition of preservatives to the product.
(a) Yeast (b) purification of the product.
(b) Agrobacterium tumefaciens (c) ensuring anaerobic conditions in the culture vessel.
(c) E. coli (d) availability of oxygen throughout the process.
(d) Mycoplasma 27. A device in which substances are treated to stimulate
17. _______ is a procedure through which a piece of DNA is transformation by living cells is called __________.
introduced in a host bacterium. (a) assimilator (b) digester
(a) Transduction (b) Transformation (c) bioreactor (d) agitator
(c) Conjugation (d) R.D.T 28. After completion of biosynthetic stage, the product has
18. Which one of the following is used as vector for cloning to be subjected through a series of processes before it is
genes into higher organisms? ready to marketing as a finished product. This series of
(a) Baculovirus processes is called
(b) Salmonella typhimurium (a) upstream processing
(c) Rhizopus nigricans (b) downstream processing
(d) Retrovirus (c) elution
19. Which of the following are required to facilitate cloning (d) insertional inactivation
into a vector ? STATEMENT TYPE QUESTIONS
(a) Origin of replication (b) Selectable marker
(c) Cloning sites (d) All of these 29. Which of the following statement is incorrect regarding
20. In agarose gel electrophoresis PCR?
(a) DNA migrates towards the negative electrode. (a) In PCR, two primers are used.
(b) supercoiled plamids migrate slower than their nicked (b) Taq DNA polymerase is related for PCR.
counterparts. (c) Taq DNA polymerase is not thermostable.
(c) larger molecules migrate faster than smaller (d) Multiple copies of gene can be synthesized in PCR.
molecules. 30. Which of the following statement is incorrect ?
(d) ethidium bromide can be used to visualize the DNA. (a) EcoRI cuts the DNA between bases G and A.
21. For transformation with recombinant DNA, the bacterial (b) Each EcoRI restriction endonuclease recognizes a
cells must first be made ‘competent’ which means specific palindromic nucleotide sequences in DNA.
(a) should increase their metabolic reactions. (c) When cut by same restriction enzyme, the resultant
(b) should decrease their metabolic reactions. DNA fragments do not have the same kind of sticky-
(c) increase efficiency with which DNA enters the ends.
bacterium. (d) Making multiple identical copies of any template
(d) ability to divide fast. DNA is called cloning.
22. For transformation, micro-particles coated with DNA to 31. Which statement about restriction enzymes is incorrect ?
be bombarded with gene gun are made up of (a) Restriction enzymes cut DNA at specific sequence
(a) silver or platinum (b) platinum or zinc called recognition sites.
(c) silicon or platinum (d) gold or tungsten (b) A restriction enzyme always cut DNA to leave the
23. The polymerase enzyme used in PCR is same sequence at the ends.
(a) DNA polymerase I (c) Some restriction enzymes cut the two DNA strands
(b) Taq polymerase at slightly different points within their recognition
site to make a ‘sticky’ end.
(c) reverse transcriptase
(d) Restriction enzymes are exonucleases rather than
(d) restriction endonuclease
endonucleases.
24. The first step in the PCR is
32. Which of the following is not correct about pBR 322 vector ?
(a) denaturation (b) primer extension
(a) It was constructed by using DNA derived from
(c) annealing (d) cooling naturally occurring plasmids of E coli.
25. Which of the following processes is employed to check (b) It has two drug resistance genes-tetR and ampR.
the progression of restriction enzyme digestion ?
(c) It was developed by Bolivar and Rodriguez.
(a) PCR
(d) Selectable markers present in it can differentiate
(b) Gene gun recombinants from non-recombinants on the basis
(c) Micro-injection of their inability to produce colour in the presence of
(d) Agarose gel electrophoresis chromogenic substrate.
EBD_7209
284 Biology
33. Which of the following statement is not correct about (iii) The downstream processing and quality control
cloning vector ? testing vary from product to product.
(a) ‘Ori’ is a sequence responsible for controlling the (iv) Competent bacterial cell cannot take up the plasmid.
copy number of the linked DNA. (a) All of the above (b) None of the above
(b) Selectable marker selectively permitting the growth (c) (i), (ii), and (iii) (d) Only (iv)
of the non-transformants. 39. Which of the following statements are incorrect.
(c) In order to link the alien DNA, the vector needs to (i) Genetic engineering is also called recombinant DNA
have single recognition site for the commonly used technology.
restriction enzymes. (ii) Bacteriophage is not used as vector.
(d) The ligation of alien DNA is carried out at a (iii) MALAYALAM is a palindrome.
restriction site present in one of the two antibiotic
(iv) Ethidium bromide can not be used for staining DNA.
resistance genes.
(a) (i) and (ii) (b) (ii) and (iv)
34. Which of the following statement is incorrect ?
(c) All of these (d) None of these
(a) T-DNA transform normal plant cell into a tumor.
40. Which of the given statements (i-iv) are correct about
(b) Retroviruses in animals have the ability to transform
bioreactor?
normal cell into cancerous cells.
(i) Bioreactor provides the optimal conditions for
(c) T plasmid of Agrobacterium tumefaciens is modified
obtaining the desired product.
into cloning vector which is more pathogenic to
plants. (ii) Raw materials are biological which are converted into
specific products.
(d) Retrovirus have also been disarmed and are now
used to deliver desirable genes into animal cells. (iii) Stirred-tank reactor is horizontal in shape.
35. Which of the following statement is incorrect ? (iv) Large volume of culture cannot be processed.
(a) Hind-II always cuts DNA molecules at a particular (a) (i) and (ii) (b) (ii) and (iii)
point by recognizing a specific sequence of 4 base (c) (iii) and (iv) (d) All of these
pairs. 41. Molecular probes used for identification of recombinant
(b) Besides Hind-II, today we know more than 900 clone carrying the desired DNA insert can be
restriction enzymes. (i) denatured double stranded DNA probes.
(c) The name Eco-RI comes from Escherichia coli-13. (ii) double stranded RNA probes.
(d) Restriction endonuclease are used in genetic (iii) protein probes.
engineering to form recombinant molecules of DNA. (iv) single stranded DNA probes.
36. Which one of the following is a correct statement (a) (i) and (ii) (b) (ii) and (iii)
regarding DNA polymerase used in PCR ? (c) (i) and (iv) (d) All of these
(a) It is used to ligate introduced DNA in recipient cell.
(b) It serves as a selectable marker. ASSERTION/REASON TYPE QUESTIONS
(c) It is isolated from a virus. In the following questions, a statement of Assertion is followed
(d) It remains active at high temperature induced by a statement of Reason.
denaturation of double stranded DNA (a) If both Assertion and Reason are true and the Reason is
37. Which of the given statement is correct in the context of the correct explanation of the Assertion.
observing DNA separated by agarose gel (b) If both Assertion and Reason are true but the Reason is
electrophoresis? not the correct explanation of the Assertion.
(a) DNA can be seen in visible light. (c) If Assertion is true but Reason is false.
(d) If both Assertion and Reason are false.
(b) DNA can be seen without staining in visible light.
42. Assertion : The uptake of DNA during transformation is
(c) Ethidium bromide stained DNA can be seen in visible
an active, energy requiring process.
light.
Reason : Transformation occurs in only those bacteria,
(d) Ethidium bromide stained DNA can be seen under
which possess the enzymatic machinery involved in the
exposure to UV light. active uptake and recombination.
38. Which of the following statements (i-iv) are correct.
43. Assertion : In recombinant DNA technology, human
(i) In elution, the separated bands of DNA are cut out genes are often transferred into bacteria (prokaryotes) or
from agarose gel and extracted from the gel piece. yeast (eukaryote).
(ii) E. coli cloning vector pBR 322 shows several Reason : Both bacteria and yeast multiply very fast to
restriction, Ori antibiotic resistance genes and rop. form huge population to express the desired gene.
Biotechnology : Principles and Processes 285

44. Assertion : DNA is cleaved at specific site by all 50. Match column-I with column-II and identify the correct
endonucleases. option.
Reason : Endonucleases are found in viruses. Column - I Column - II
45. Assertion : Restriction endonucleases have been A. Restriction enzyme I. Jumping gene
designated as molecular scissors. B. Transposons II. Cloning vehicle
Reason : Fragments produced by restr iction C. Bacteriophage III. Hind III
endonucleases upon mixing join due to their sticky ends. D. Palindromes IV. MALAYALAM
46. Assertion : Insertion of recombinant DNA within the (a) A – III; B – I; C – II; D – IV
coding sequence of b-galactosidase results in colourless
(b) A – II; B – I; C – IV; D – III
colonies.
(c) A – IV; B – I; C – III; D – II
Reason : Presence of insert results in inactivation of
(d) A – I; B – IV; C – II; D – III
enzyme b-galactosidase known as insertional inactivation.
51. Match column-I with column-II and select the correct
MATCHING TYPE QUESTIONS combination from the option given below.
Column - I Column - II
47. Match column-I with column-II and select the correct
answer using the codes given below. A. Competent I. Thermus aquaticus
Column - I Column - II B. Taq DNA polymerase II. Antibiotic
A. Plasmid I. Selectable marker C. Ampicillin III. Micro-injection
B. amp II. Extrachromosomal DNA D. Ethidium bromide IV. DNA staining
C. Ti-plasmid III. Enzyme (a) A – III; B – I; C – II; D – IV
D. Chitinase IV. Agrobacterium (b) A – II; B – I; C – IV; D – III
tumefaciens (c) A – IV; B – I; C – III; D – II
(a) A – IV; B – I; C – II; D – III (d) A – I; B – IV; C – II; D – III
(b) A – II; B – I; C – IV; D – III 52. Match the following and choose the correct combination
(c) A – IV; B – I; C – III; D – II from the options given below.
(d) A – I; B – IV; C – II; D – III Column - I Column - II
48. Match the items of column-I with those in column-II and A. PCR I. Large scale culture
choose the correct option. B. Bioreactor II. To induce alien DNA in
Column - I Column - II host cell
A. Recombinant I. Vector C. Gene gun III. Restriction endonuclease
DNA technology D. EcoRI IV. Amplification of gene
B. Cloning vehicles II. Sealing enzyme
(a) A – IV; B – I; C – II; D – III
C. Macromolecular III. Electrophoresis
separation (b) A – II; B – I; C – IV; D – III
D. DNA ligase IV.. Genetic engineering (c) A – IV; B – I; C – III; D – II
(a) A – IV; B – I; C – II; D – III (d) A – I; B – IV; C – II; D – III
(b) A – II; B – I; C – IV; D – III 53. Match Column I with Column II and identify the correct
(c) A – IV; B – I; C – III; D – II option.
(d) A – I; B – IV; C – II; D – III Column - I Column - II
49. Match column-I with column-II and choose the correct A. Primers I. PCR
option. B. Separation and II. C2H5OH
Column - I Column - II purification of products
A. EcoRI I. Bacilius amyloliquefaciens C. Precipitation of DNA III. Uptake of foreign DNA
B. Bam HI II. Haemophilus influenza by bacterium
C. Hind III III. Escherichia coli D. Transformation IV. Down stream processing
D. pBR 322 IV. Artificial plasmid (a) A – IV; B – I; C – II; D – III
(a) A – III; B – I; C – II; D – IV (b) A – II; B – I; C – IV; D – III
(b) A – II; B – I; C – IV; D – III
(c) A – IV; B – I; C – III; D – II
(c) A – IV; B – I; C – III; D – II
(d) A – I; B – IV; C – II; D – III (d) A – I; B – IV; C – II; D – III
EBD_7209
286 Biology
54. Which of the following is incorrectly matched? 57. The given figure shows a simple stirred tank bioreactor
(a) Agarose – Sea weeds with their parts labelled as A, B, C and D. Identify A, B, C
(b) Thermus aquaticus – ‘T - DNA’ and D.
(c) Plasmid DNA – Vector
(d) Sal I – Restriction endonuclease A
B
C
DIAGRAM TYPE QUESTIONS
55. The given figure shows the E.Coli cloning vector PBR322
showing restriction sites. Some parts are labelled as A, B,
C & D. Choose the option showing the correct labelling.
Cla I Hind III D
A (a) A-Motor; B-pH control; C-Foam braker; D-Sterile air
Pvu I (b) A-pH control; B-Motor; C-Foam braker; D-Sterile air
(c) A-pH control; B-Sterile air; C-Motor; D-Foam braker
Pst I B (d) A-Motor; B-Sterile air; C-pH control; D-Foam braker
C 58. Identify the correct match for the given apparatus.
tetR
pBR 322 Sal I

D
rop

Pvu II
A B C D
(a) Hind I EcoR I ampR ori
(b) Hind I BamH I kanR ampR
(c) BamH I Pst I ori ampR
(d) EcoR I BamH I ampR ori
56. Which one of the following option is correct for A, B and Apparatus Function
C marked in the given diagram of recombinant DNA (a) Gene gun Vectorless direct gene transfer
technology. (b) Column Separation of chlorophyll
(Cutting both foreign chromatography pigments
DNA and vector (c) Sparged stirred tank Carry out fermentation
DNA at bioreactor process
specific Vector
(d) Respirometer Finding out rate of respiration
Foreign DNA point) DNA
A (plasmid) 59. Identify the correct match of the technique with their role
shown in the given figure.
Walls
B join foreign Largest Smallest
DNA to plasmid
34
2
1

C
E.coli (a) Electrophoresis – Differential migration of DNA
Cells fragments
Divide
(b) Column – Separation of chlorophyll
chromatography pigments
(a) A-Exonuclease; B-Ligases; C-Transformation
(c) Gene cloning – Technique of obtaining
(b) A-Endonuclease; B-gyrase; C-Transformation
identical copies of a particular
(c) A-Exonuclease; B-Hydrolase; C-Transduction DNA or a gene segment
(d) A-Restriction endonuclease; B-Ligases; (d) Microinjection – Technique of introducing
C-Transformation foreign genes into a host cell
Biotechnology : Principles and Processes 287

60. The figure given below shows three steps (A, B, C) of artificial means (not by breeding) comes under a branch.
Polymerase Chain Reaction (PCR). Select the option giving called ________ .
correct identification together with what it represents? (a) molecular biology (b) cytogenetics
Region to be amplified (c) genetic hybridization (d) genetic engineering
65. DNA fragments are separated using gel electrophoresis
(a) because DNA is pulled through the gel towards the
negative end of the field.
(b) because larger DNA fragments move faster through
the gel than smaller DNA fragments.
(c) to identify and isolate DNA fragments.
(d) to synthesize DNA for cloning.
66. DNA ligases are enzymes that can be used to
(a) chop a large DNA molecule into small fragments.
(b) copy DNA fragments.
(c) insert the DNA from one species into the DNA of
another species.
(d) separate DNA fragments based on their size.
67. Imagine a gel through which DNA fragments have moved
in response to an applied electrical current. The band on
this gel that is farthest from the top (that is, from the place
where the DNA fragments were added to the “well”)
Amplified
(~1billion times) represents the
(a) shortest fragments of DNA.
(a) B - Denaturation at a temperature of about 98°C (b) longest fragments of DNA.
separating the two DNA strands. (c) restriction enzyme used to cut the DNA into
(b) A - Denaturation at a temperature of about 50°C. fragments.
(c) C - Extension in the presence of heat stable DNA (d) ligase used to bind the DNA fragments together.
polymerase. 68. A biologist intends to use a polymerase chain reaction to
perform a genetic task. The biologist probably is trying to
(d) A - Annealing with two sets of primers.
(a) discover new genes.
CRITICAL THINKING TYPE QUESTIONS (b) clone a gene.
(c) cut DNA into many small fragments.
61. Restriction endonucleases
(d) isolate DNA from a living cell.
(a) are enzymes that process pre-RNA’s.
69. In genetic engineering, where genes can be inserted from
(b) are enzymes that degrade DNA. one organism into another or back into the original
(c) protect bacterial cells from viral infection. organism uses which of the following techniques?
(d) None of the above (a) Polymerase chain reaction
62. The same basic techniques can be used to analyze the (b) Gene gun
DNA from species as diverse as bacteria and humans (c) DNA hybridization
because: (d) Gel electrophoresis
(a) all cells are identical. 70. Which of the following is not necessary to execute a
(b) every organism has the same amount of DNA. polymerase chain reaction successfully?
(c) the DNA sequences of all organisms are the same. (a) All four DNA bases
(d) DNA has a consistent structure in all organisms. (b) Short DNA base primers
63. A gene is said to be cloned if (c) DNA polymerase
(a) the DNA sequence of the gene is known. (d) DNA library
(b) the function of the gene is known. 71. Biolistics (gene-gun) is suitable for
(c) there is a DNA probe for the gene. (a) DNA finger printing.
(d) the gene has been isolated and copied. (b) Disarming pathogen vectors.
64. Introduction of one or more genes into an organism which (c) Transformation of plant cells.
normally does not possess them or their deletion by using (d) Constructing recombinant DNA molecules.
EBD_7209
288 Biology
72. The linking of antibiotic resistance gene with the plasmid 81. Which of the following process require energy?
vector became possible with (a) Ligation (b) Transformation
(a) DNA ligase (b) endonucleases (c) Restriction digestion (d) Hybridization
(c) DNA polymerase (d) exonucleases 82. In agarose gel electrophoresis, DNA molecules are
73. Plasmid present in bacterial cells are separated on the basis of their
(a) circular double helical DNA molecules. (a) charge only (b) size only
(b) linear double helical DNA molecules. (c) charge to size ratio (d) all of these
(c) circular double helical RNA molecules. 83. Restriction endonucleases are enzymes which
(d) linear double helical RNA molecules. (a) make cuts at specific positions within the DNA
74. What must be done before placing DNA into the molecule.
electrophoretic chamber ?
(b) recognize a specific nucleotide sequence for binding
(a) It must be ground up with mortar and pestle.
of DNA ligase.
(b) It must be cut by restriction endonucleases.
(c) restrict the action of the enzyme DNA polymerase.
(c) It must be treated with RNAase .
(d) remove nucleotides from the ends of the DNA
(d) None of the above
molecule.
75. Which of the following has the ability to transform normal
84. What type of enzyme is used in recombinant DNA
cells into cancerous cells in animals ?
technology to split a specific sugar phosphate bond in
(a) Agrobacterium tumefaciens
each strand of a DNA double helix ?
(b) Retroviruses
(a) Esterase (b) Restriction enzyme
(c) DNA-viruses
(c) Lipase (d) Ligase
(d) Plasmids
76. What are the properties of a good vector? 85. The colonies of recombinant bacteria appear white in
contrast to blue colonies of non-recombinant bacteria
(a) It should be ideally more than 10 kb in size.
because of
(b) It should be able to replicate autonomously.
(a) insertional inactivation of alphaga-lactosidase in non-
(c) It should have suitable marker genes.
recombinant bacteria.
(d) It should not be easy to isolate and purify.
(b) insertional inactivation of alpha-galactosidase in
77. A kind of biotechnology involving manipulation of DNA
recombinant bacteria.
is called
(a) DNA replication (b) genetic engineering (c) inactivation of glycosidase enzyme in recombinant
bacteria.
(c) denaturation (d) renaturation
78. Restriction endonucleases hydrolyzes polynucleotide (d) non-recombinant bacteria containing beta-
from galactosidase.
(a) only the 5¢ end. 86. Genetic engineering is possible, because
(b) from either terminal. (a) we can cut DNA at specific sites by endonucleases
(c) at an internal phosphodiester bond. like DNase I.
(d) a phosphodiester bond within a specific sequence. (b) restriction endonucleases purified from bacteria can
79. Restriction-modification systems of bacteria exist to be used in vitro.
(a) protect bacteria from invading foreign DNA. (c) the phenomenon of transduction in bacteria is well
(b) promote conjugation. understood.
(c) help the bacterial chromosome to replicate. (d) we can see DNA by electron microscope.
(d) encourage recombination of new genetic material.
80. Which of the following is used as a best genetic vector in
plants?
(a) Bacillus thuringiensis
(b) Agrobacterium tumefaciens
(c) Pseudomonas putida
(d) All of the above
Biotechnology and its
34

Chapter
Applications

7. Bt toxin is harmful to insects like


FACT/DEFINITION TYPE QUESTIONS
(a) lepidoterans (tobacco budworm, armyworms)
1. Consumption of which one of the following foods can (b) coleopterans (beetles)
prevent the kind of blindness associated with vitamin ‘A’ (c) dipterans (flies and mosquito)
deficiency? (d) all of the above
(a) ‘Flavr Savr’ tomato (b) Canolla 8. RNA interference (RNAi) technique has been devised to
(c) Golden rice (d) Bt-Brinjal protect the plants from nematode is silenced by ______
2. Bacillus thuringiensis (Bt) strains have been used for produced by the host plant.
designing novel (a) dsDNA (b) ssDNA
(a) bio-fertilizers (c) dsRNA (d) target proteins
(b) bio-metallurgical techniques 9. Tobacco plants resistant to a nematode have been
(c) bio-mineralization processes developed by the introduction of DNA that produced (in
the host cells)
(d) bio-insecticidal plants
(a) both sense and anti-sense RNA.
3. The genetically-modified (GM) brinjal in India has been
(b) a particular hormone.
developed for
(c) an antifeedant.
(a) insect-resistance
(d) a toxic protein.
(b) enhancing shelf life
10. C-peptide of human insulin is
(c) enhancing mineral content
(a) a part of mature insulin molecule.
(d) drought-resistance
(b) responsible for formation of disulphide bridges.
4. Bt toxin kill the larvae of certain insects
(c) removed during maturation of pro-insulin to insulin.
(a) by binding of activated toxin on mid gut epithelial
cells, creating pores, leading to swelling and lysis. (d) responsible for its biological activity.
(b) by stopping transcription of larval cells. 11. The first human drug made using recombinant DNA
technology was
(c) by altering central dogma taking place in the cells of
(a) glyphosatase (b) TPA
gut of larva.
(c) insulin (d) erythropoietin
(d) by stopping protein synthesis.
12. E. coli are used in production of
5. Cry protein is obtained from
(a) rifampicin (b) LH
(a) Bacillus thuringiensis
(c) ecdysone (d) interferon
(b) Bacillus subtilis
13. The first clinical gene therapy was given in 1990 to a 4
(c) Clostridium welchi
years old girl with enzyme deficiency of
(d) E. coli
(a) adenosine deaminase (ADA)
6. Cry-genes have been introduced in
(b) tyrosine oxidase
(a) cotton and corn (b) rice (c) monamine oxidase
(c) potato and soyabean (d) all of the above (d) glutamate dehydrogenase
EBD_7209
290 Biology
14. In some children, ADA deficiency can be cured by 25. Biopiracy is related to
(a) bone marrow transplantation (a) bioresearches
(b) enzyme replacement therapy (b) traditional knowledge
(c) both (a) and (b) (c) biomolecules and genes discovered
(d) none of the above (d) all of the above
15. The site of production of ADA in the body is 26. The use of bioresources by multinational companies and
(a) bone marrow (b) lymphocytes other organizations without proper authorisation from the
(c) blood plasma (d) monocytes countries and people concerned without compensatory
payment is called
16. Genes of interest can be selected from a genomic library
by using (a) bioethics (b) biopiracy
(a) restriction enzymes (b) cloning vectors (c) bioterror (d) bioweapon
(c) DNA probes (d) gene targets 27. Which step of Government of India has taken to cater
17. DNA or RNA segment tagged with a radioactive molecule requirement of patent terms and other emergency
is called ___________ . provisions in this regard ?
(a) vector (b) probe (a) Biopiracy act
(c) clone (d) plasmid (b) Indian patents bill
18. The transgenic animals are those which have (c) RTI act
(a) foreign DNA in some cells. (d) Negotiable instruments act
(b) foreign DNA in all of their cells.
(c) foreign RNA in all of their cells. STATEMENT TYPE QUESTIONS
(d) both (a) and (c) 28. Which of the following statement is correct about Bt toxin ?
19. Today, transgenic models exist for many human diseases (a) Bt protein exists as active toxin in the Bacillus.
which includes
(b) The activated toxin enters the ovaries of the pest to
(i) Cancer (ii) Cystic fibrosis
sterilize it and thus prevent its multiplication.
(iii) Rheumatoid arthritis (iv) Alzhiemer’s disease
(c) The concerned Bacillus has antitoxins.
(a) (i) and (iii) only (b) (ii) and (iii) only
(d) The inactive protoxin gets converted into active form
(c) (i), (ii) and (iii) only (d) all of these
in the insect gut.
20. The protein a-1 antitrypsin is used to treat the
29. Find out the incorrect statement.
(a) cancer (b) rheumatoid arthritis
(a) Human protein used to treat emphysema is a-1
(c) Alzheimer’s disease (d) emphysema
antitrypsin.
21. Maximum number of existing transgenic animals is of
(b) Human insulin is being commercially produced from
(a) fish (b) mice
a transgenic species of Agrobacterium tumefaciens.
(c) cow (d) pig
(c) Rosie, the first transgenic cow, produced human
22. GEAC stands for
protein enriched milk.
(a) Genome Engineering Action Committee
(d) Cry I Ab endotoxins obtained from Bacillus
(b) Ground Environment Action Committee thuringiensis is effective against corn borers.
(c) Genetic Engineering Approval Committee
30. Which one of the following statement(s) is/are correct
(d) Genetic and Environment Approval Committee about Genetic Engineering Approval committee (GEAC) ?
23. How many varieties of rice has been estimated to be
(a) It will make decision regarding the validity of GM
present in India?
research.
(a) 2,000 (b) 20,000
(b) It will make the safety of introducing GM - organism
(c) 200,000 (d) 2,000,000
for public services.
24. Which variety of rice was patented by a U.S. company
(c) Its genetic modification of organism can have
even though the highest number of varieties of this rice is
unpredictable results when such organisms are
found in India ?
introduced into the ecosystem. Therefore, the Indian
(a) Sharbati Sonara (b) Co-667
government has set up organisation such as GEAC.
(c) Basmati (d) Lerma Roja
(d) All of the above
Biotechnology and its Applications 291

31. Which one of the following statement is correct? (iii) Bacillus thuringiensis can form cry protein during
(a) The proteins encoded by the genes cry I Ac and any phase of their growth.
cry II Ab control cotton bollworms. (iv) Bacillus thuringiensis is not harmed by self cry
(b) Protein encoded by cry I Ab controls corn borer. protein because of its occurrence as protoxin
(c) Both (a) and (b) (inactive).
(d) Proteins encoded by cry I Ac and cry I Ab control (v) Protoxin cry protein is changed into active cry
flies. protein in the stomach of insects due to alkaline pH
32. Which of the following is a correct statement? in stomach.
(a) “Bt” in Bt-cotton indicates that it is genetically (a) Only (iii) (b) (i) and (iv)
modified organism produced through biotechnology. (c) All of these (d) None of these
(b) Somatic hybridization involves fusion of two 37. Read the following four statements (i-iv) and answer the
complete plant cells carrying desired genes. question ?
(c) The anticoagulant hirudin is being produced from (i) The first transgenic buffalo, Rosie produced milk
transgenic Brassica napus seeds. which was human alpha-lactal albumin enriched.
(d) “Flavr Savr” variety of tomato has enriched the (ii) Restriction enzymes are used in isolation of DNA
production of ethylene which improves its taste. from other macro-molecules.
33. Which of the following statement(s) is/are correct? (iii) Downstream processing is one of the steps of R-
(a) The procedure for chemical safety testing / toxicity DNA technology.
is the same as that used for testing toxicity of drugs.
(iv) Disarmed pathogen vectors are also used in transfer
(b) Transgenic animals are more sensitive to the toxic of R-DNA into the host.
substances than non-transgenic animals.
Which are the two statements having mistakes?
(c) Golden rice, a genetically engineered rice has high
(a) (ii) and (iii) (b) (iii) and (iv)
vitamin A (retinol) content.
(d) All of the above (c) (i) and (iii) (d) (i) and (ii)
34. Which of the following statements is correct? 38. Read the following statements and choose the correct
statements.
(a) The current interest in the manipulation of microbes,
plants and animal has raised serious ethical issues. (i) Gene therapy has been tested on a large number of
(b) One possible risk of genetic engineering is the patients with a wider variety of inherited genetic
accidental production of dangerously resistant disorders, and in numerous cases it has produced a
microorganisms. complete cure.
(c) Although risks are possible, genetic engineering (ii) Genetic engineering has been used to produce
appears to offer more of contribution to human insulin for curing the diabetes.
welfare than threats. (iii) DNA hybridization is the base pairing of DNA from
(d) All of the above two different sources.
35. Select the correct statement. (iv) Genetic engineering is a technique of plant breeding.
(a) Genetic engineering works only on animals and has (a) (i) and (ii) (b) (ii) and (iii)
not yet been successfully used on plants. (c) (i), (ii) and (iii) (d) All of these
(b) There are no risks associated with DNA technology. 39. Which of the following statement(s) is/are incorrect?
(c) The first step in PCR is heat which is used to separate (i) Insulin was originally extracted from pancreas of
both the strands of target DNA. slaughtered pigs and cattle.
(d) DNA from one organism will not bond to DNA from (ii) Animal insulin is difficult to obtain.
another animal.
(iii) Animal insulin is identical to human insulin.
36. Which of the following statements (i - v) is/are incorrect ?
(iv) Non-human insulin caused some patients to develop
(i) Recombinant DNA technology is used to improve allergy.
crop plants by increasing their productivity, by
(v) Recombinant insulin is actually obtained from E. coli
making them more nutritious and by developing in bacterial cell.
disease resistant.
(a) Only (i) and (ii) (b) Only (iii) and (iv)
(ii) Bt cotton is resistant to bollworm infestation. (c) Only (iii) (d) Only (v)
EBD_7209
292 Biology
40. Transgenic animals are produced Reason : In this, production of pectin degrading
(i) to study how genes are regulated and how they affect Polygalactouronase is blocked.
the normal functions of body and its development. 46. Assertion : ELISA is based on the principle of antigen
(ii) to study diseases. antibody interaction.
Reason : Pathogen infection is usually detected by
(iii) to obtain useful biological products .
presence of antigens or detection of antibodies
(iv) to test vaccine safety and chemical safety. synthesized against the pathogen.
(a) (i), (ii), (iii) and (iv) (b) (i) and (iv) 47. Assertion : The GEAC (Genetic Engineering Approval
(c) (ii) and (iv) (d) Only (i) Committee) has been set up by the Indian Government.
41. When a patient with defective ADA was treated, which of Reason : Introduction of GMO could have unpredictable
the following steps was performed for gene therapy? result in the ecosystem.
(i) Lymphocytes were obtained from the patients.
MATCHING TYPE QUESTIONS
(ii) Lymphocytes are transferred to culture dishes.
(iii) Lymphocytes were transfected with normal ADA 48. Match the following and choose the correct combination
genes. from the options given below :
(iv) The transfected cells are returned to the patients. Column - I Column - II
(a) All of these (b) (iii) and (iv) A. Escherichia coli I. nif gene
(c) Only (iv) (d) SCID cannot be treated B. Bacillus thuringiensis II. Interferon
C. Rhizobium meliloti III. Bt toxin
42. The given statements are the steps in one type of gene
D. Agrobacterium tumefaciens IV. Vector
therapy.
(a) A – II; B – I; C – (IV); D – III
(i) Inject engineered cells into patients bone marrow.
(b) A – II; B – I; C – (III); D – IV
(ii) Viral DNA carring the normal allele inserts into
(c) A – II; B – III; C – I; D – IV
chromosome.
(d) A – IV; B – I; C – II; D – III
(iii) Let retrovirus infect bone marrow cells that have
49. Match the items given in column - I with those in column - II
removed from patient and cultured.
and choose the correct option.
(iv) Insert RNA version of normal allele into retrovirus.
Column - I Column - II
The correct sequence is A. GMO I. Increased shelf life
(a) (i), (ii), (iii) and (iv) (b) (iv), (iii), (ii) and (i) B. Flavr - Savr tomato II. Bioresources
(c) (i), (ii), (iv) and (iii) (d) (iv), (iii), (i) and (ii) C. Biopiracy III. rDNA
D. E.coli IV. Insulin
ASSERTION/REASON TYPE QUESTIONS
(a) A – III; B – I; C – II; D – IV
In the following questions, a statement of Assertion is followed (b) A – II; B – I; C – III; D – IV
by a statement of Reason. (c) A – II; B – III; C – I; D – IV
(a) If both Assertion and Reason are true and the Reason is (d) A – IV; B – I; C – II; D – III
the correct explanation of the Assertion. 50. Match column-I with column-II and identify the correct option.
(b) If both Assertion and Reason are true but the Reason is Column - I Column - II
not the correct explanation of the Assertion. A. Gene therapy I. Rhizobium
(c) If Assertion is true but Reason is false. B. Biofertilizer II. Cry gene
(d) If both Assertion and Reason are false. C. Bt cotton III. SCID
43. Assertion : Blood clotting is prevented by Hirudin D. Humulin IV. Diabetes
protein. (a) A – II; B – I; C – IV; D – III
Reason : The gene encoding for the Hirudin protein is (b) A – III; B – I; C – II; D – IV
transferred into Brassica napus, where accumulation of (c) A – II; B – III; C – I; D – IV
hirudin occurs in the seeds. (d) A – IV; B – I; C – II; D – III
44. Assertion : An application of tissue culture is the 51. Match column-I with column-II and choose the correct
production of transgenic plants. option.
Reason : A transgenic organism is one that contains and Column - I Column - II
expresses a transgene. A. Golden Rice I. Cry protein
45. Assertion : Flavr Savr tomato is transgenic tomato that B. Bt toxin II. Rich in vitamin A
is capable of remaining fresh and retaining the flavour for C. RNAi III. First trangenic cow
a longer time. D. Rosie IV. Gene silencing
Biotechnology and its Applications 293

(a) A – II; B – I; C – IV; D – III (c) transgenic cow – rosie which produces high fat milk
(b) A – II; B – I; C – III; D – IV for making ghee.
(c) A – II; B – III; C – I; D – IV (d) animals like bulls for farm work as they have super
(d) A – IV; B – I; C – II; D – III power.
52. Match column-I with column-II and choose the correct 56. Silencing of mRNA has been used in producing transgenic
option. plants resistant to
Column - I Column - II (a) bollworms (b) nematodes
A. Forensic science I. AIDS
(c) white rusts (d) bacterial blights
B. ELISA II. Radioactive DNA/RNA
57. Which one of the following techniques made it possible
C. Probe III. Emphysema
to genetically engineer living organism ?
D. a-1-antitrypsin IV. DNA fingerprinting
(a) Recombinant DNA techniques
(a) A – II; B – I; C – IV; D – III
(b) X-ray diffraction
(b) A – II; B – I; C – III; D – IV
(c) Heavier isotope labelling
(c) A – II; B – III; C – I; D – IV
(d) Hybridization
(d) A – IV; B – I; C – II; D – III
58. Which of the following Bt crops is being grown in India
DIAGRAM TYPE QUESTIONS by the farmers?
(a) Cotton (b) Brinjal
53. Which of the following is the correct set of the labels A, (c) Soyabean (d) Maize
B, C and D in the given figure of maturation of pro-insulin
59. A transgenic food crop which may help in solving the
into insulin ?
problem of night blindness in developing countries is
(a) golden rice (b) Bt soyabean
s–s
(c) flavr - savr tomato (d) starlink maize
A B
s-s 60. Which of these is used as vector in gene therapy for
SCID?
C (a) Arbovirus (b) Rotavirus
+
(c) Retrovirus (d) Parvovirus
61. The genetic defect, adenosine deaminase (ADA)
D deficiency may be cured permanently by
A B C D (a) administering adenosine deaminase through
(a) Proinsulin cell peptidases Insulin Free C-Peptide injection
(b) Insulin cell peptidases Free C-Peptide Proinsulin (b) bone marrow transplantation
(c) Insulin Free C-Peptide cell peptidases Proinsulin
(c) enzyme replacement therapy
(d) Insulin Proinsulin Free C-Peptide cell peptidases
(d) introducing isolated gene from marrow cells
CRITICAL THINKING TYPE QUESTIONS producing ADA into the cells at early embryonic
stages
54. Transgenic plants are the ones 62. Genetically engineered bacteria have been successfully
(a) generated by introducing foreign DNA into a cell used in the commercial production of
and regenerating a plant from that cell.
(a) human insulin (b) testosterone
(b) produced after protoplast fusion in artificial medium.
(c) thyroxine (d) melatonin
(c) grown in artificial medium after hybridization in the
field. 63. Main objective of production/use of herbicide resistant
(d) produced by a somatic embryo in artificial medium. GM crops is to
55. Transgenic animals has been successfully used for (a) eliminate weeds from the field without the use of
producing manual labour.
(a) transgenic mice for testing safety of polio vaccine (b) eliminate weeds from the field without the use of
before use in humans. herbicides.
(b) transgenic models for studying new treatments for (c) encourage eco-friendly herbicides.
certain cardiac diseases. (d) reduce herbicide accumulation in food articles for
health safety.
EBD_7209
294 Biology
64. Biotechnology deals with industrial scale production of 69. What is the disadvantage of using porcine insulin
biopharmaceuticals and biological products using (from pig) in diabetic patients?
genetically modified (a) It leads hypercalcemia.
(a) microbes only (b) It is expensive.
(b) fungi only (c) It may cause allergic reactions.
(c) plants and animals only (d) It can lead to mutation in human genome.
(d) all of the above 70. Which technique would to be completely curative in
65. Cry II Ab and cry I AC produce toxins that control SCID ?
(a) cotton bollworms and corn borer respectively. (a) Gene therapy in adult stage.
(b) cotton borer and cotton bollworms respectively. (b) Gene therapy in embryonic stage.
(c) tobacco budworms and nematodes respectively. (c) Bone marrow transplantation.
(d) nematodes and tobacco budworms respectively. (d) Enzyme replacement therapy.
66. Bt toxin is 71. In order for gene therapy to be most effective, genes
(a) intracellular lipids. should be inserted in
(b) intracellular crystalline protein. (a) WBC (b) RBC
(c) extracellular crystalline protein. (c) stem cells (d) all of these
(d) intracellular polysaccharide. 72. Which of the following is based upon the principle of
67. RNA interference involves antigen-antibody interaction?
(a) synthesis of mRNA from DNA. (a) PCR (b) ELISA
(b) synthesis of cDNA from RNA using reverse (c) r-DNA technology (d) RNA
transcriptase. 73. Deliberate alteration of genome for treatment of disease
(c) silencing of specific mRNA due to complementary is called
RNA. (a) transformation rescue
(d) interference of RNA in synthesis of DNA. (b) imprinting
68. The RNAi stands for (c) exon shuffle
(a) RNA inactivation (b) RNA initiation (d) gene therapy
(c) RNA interference (d) RNA interferon
Organisms and
35

Chapter
Populations

9. The benthic organisms


FACT/DEFINITION TYPE QUESTIONS
(a) live near the sea bottom.
1. The term ‘precipitation’ includes (b) found in open water.
(a) rain (c) always live at the depth of 50-100 m.
(b) snow (d) live outside water.
(c) hails 10. The process in which the body’s internal environment is
kept stable is known as
(d) all forms of water that fall to the ground.
(a) homeostasis (b) adaptation
2. What are the key elements that lead to so much variations
in the physical and chemical conditions of different (c) geometry (d) acclimatization
habitats? 11. Which of the following is the stage of suspended
development?
(a) Temperature and water
(a) Dormancy (b) Hibernation
(b) Light and soil
(c) Aestivation (d) Diapause
(c) Only temperature
12. To avoid summer - related problems such as heat and
(d) Temperature, water, light and soil dessication fish undergoes
3. A majority of organisms which are restricted to narrow (a) hibernation (b) diapause
range of temperature are called as (c) aestivation (d) none of these
(a) stenothermal (b) endothermal 13. _____ is any attribute of the organism (morphological
(c) ectothermal (d) eurythermal physiological, behavioural) that enables the organisms to
4. A few organisms can tolerate and thrive a wide range of survive and reproduce in its habitat.
temperature. Such animals are called (a) Exponential growth (b) Acclimatization
(a) stenothermal (b) eurythermal (c) Adaptation (d) Mutualism
(c) thermophilic (d) ectothermal 14. Seals have a thick layer of fat (blubber) below their skin
5. The salinity in sea water in parts per thousand (ppt) ranges that acts as an
between (a) thermostat (b) capacitor
(a) 5-15% (b) 30-35% (c) resistor (d) insulator
(c) 50-75% (d) more than 100% 15. Microbes present in hydrothermal vents where the
temperature far exceed 100°C is
6. Organisms that are restricted to a narrow range of salinity,
(a) cyanobacteria (b) archaebacteria
are called
(c) eubacteria (d) none of these
(a) ectohaline (b) osmoconformer
16. If the age distribution (per cent individuals of a given age
(c) euryhaline (d) stenohaline
or group) is plotted for the population, the resulting
7. Deep (>500m) in the oceans, the environment is perpetually structure is called a/an
dark and its inhabitants are not aware of the existence of a
(a) population density (b) ecological pyramid
celestial source of energy called_________.
(a) ATP (b) photosynthesis (c) population growth (d) age pyramid
17. The age of pyramid with narrow base indicates
(c) sun (d) light
(a) high number of young individuals.
8. Percolation and water holding capacity of soil is dependent
upon (b) low number of young individuals.
(c) high number of old individuals.
(a) soil composition (b) grain size
(d) low number of old individuals.
(c) aggregation (d) all of these
EBD_7209
296 Biology
18. Which of the following parameter is not a part of population 29. The interaction is detrimental to both the species, in
growth? (a) predation (b) commensalism
(a) Natality (b) Mortality (c) amensalism (d) competition
(c) Metapopulation (d) Emigration 30. A wasp pollinating a fig flower is an example of
19. _______________ is the number of individuals of the (a) commensalism (b) amensalism
population who left the habitat and have gone elsewhere (c) parasitism (d) mutualism
during the time period under consideration. 31. An interaction where one species is harmed while the other
(a) Natality (b) Mortality is unaffected is called
(c) Immigration (d) Emigration (a) commensalism (b) competition
20. The formula for logistic growth are (c) amensalism (d) parasitism
(a) dN / dt = rN (b) rN / dN = dt
STATEMENT TYPE QUESTIONS
éK - Nù éN-Kù
(c) dN / dt = rN ê ú (d) dN / dt = rN ê ú 32. Consider the following statements (A) - (D) each with one
ë K û ë N û or two blanks.
21. In growth pattern, (1 – N/K) is A. Lichens represent an intimate (i) relationship
(a) carrying capacity between a fungus and (ii) .
(b) intrinsic rate of natural increase B. The (iii) are associations between fungi and
(c) environmental resistance the roots of higher plants.
(d) biotic potential C. Plants need the help of (iv) for pollinating their
22. Interspecific interaction arise from the interaction of flowers and dispersing their seeds.
(a) individuals of a community. D. The (v) pollinates the fig inflorescence while
(b) populations and their regulatory factors. searching for suitable egg - laying sites.
(c) populations of two different species. Which one of the following options, gives the correct fill
ups for the respective blank numbers from (i) - (v) in the
(d) none of the above statements?
23. Thorns of Acacia and cactus are the most common (a) (i) Parasitic; (ii) - Cyanobacteria; (iii) - Mycorrhizae;
morphological means of (iv) - Wind; (v) - Bee
(a) reproduction (b) competition (b) (i) Mutualistic; (ii) - Cyanobacteria; (iii) -
(c) defence (d) economical importance Mycorrhizae; (iv) - Animals; (v) Wasp
24. Which of the following secondary compounds are (c) (i) Parasitic; (ii) - Cyanobacteria; (iii) - Mycorrhizae;
produced by plants for the purpose of defences against (iv) - Insect; (v) Bumblebees
grazers and browsers? (d) (i) Mutualistic; (ii) - Cyanobacteria; (iii) - Lichen;
(a) Strychnine (b) Caffeine (iv) - Wind; (v) Wasp
(c) Quinine (d) All of these 33. Consider the following statements (A)-(D) each with one
or two blanks.
25. Connell’s elegant field experiments are related to barnacle,
in which superior barnacle Balanus dominates the inter (A) Bears go into (i) during winter to
tidal area, and excludes the smaller barnacle Chathamalus
(ii) cold weather..
from that zone. This phenomenon is called
(a) competitive exclusion principle (B) A conical age pyramid with a broad base represents
(b) competitive release (iii) human population.
(c) interspecific competition (C) A wasp pollinating a fig flower is an example of
(d) none of the above (iv) .
26. Cuscuta is an example of (D) An area with high levels of species richness is known
(a) ectoparasitism (b) endoparasitism (v) .
(c) predation (d) brood parasitism as
Which one of the following options, gives the correct fill
27. _____________ in birds is an interesting example of ups for the respective blank numbers from (i) to (v) in the
parasitism in which the parasitic bird lays its eggs in the statements?
nest of its host and the host incubates them. (a) (i) - hibernation, (ii) - attract, (iii) - expanding,
(a) Bird parasitism (b) Breed parasitism (iv) - commensalism, (v) - biodiversity park
(c) Brood parasitism (d) Ectoparasites (b) (i) - hibernation, (ii) - escape, (iii) - expanding,
28. Which of the following is not an example of commensalism? (iv) - mutualism, (v) - hot spot
(a) Sea anemone and clown fish (c) (i) - aestivation, (ii) - escape, (iii) - stable,
(b) Epiphyte / Orchid on mango branch (iv) - commensalism, (v) - marsh
(c) Liver fluke and fleas (d) (i) - aestivation, (ii) - escape, (iii) - stable,
(iv) - mutualism, (v) - hot spot
(d) Cattle egret and grazing cattle.
Organisms and Populations 297
34. Which of the following statement is incorrect ? 39. Identify the correct statement.
(a) Habitat includes both biotic and abiotic factors. (a) The smaller animals have larger surface area relative
(b) Abiotic and biotic components interact constantly to their volume.
with each other. (b) Smaller animals are rarely found in polar region.
(c) Abiotic components alone can characterize the (c) Bear cannot migrate hence hibernate during winter.
habitat of an organism. (d) All of the above
(d) Major abiotic factors includes temperature, water, 40. Identify the incorrect statement
light and soil.
(i) Thermoregulation energetically least expensive process
35. Study the following statements and answer the question. for many organisms like shrews and humming birds.
(i) Mango trees cannot grow in temperate countries like (ii) 99% animals and nearly all plants cannot maintain
Canada and Germany. their constant internal environment.
(ii) Tuna fish are rarely caught beyond tropical latitude (iii) During the course of evolution, the costs and heights
in the ocean.
of maintaining a constant internal environment are
(iii) Snow Leopards are not found in Kerala. discarded.
Which of the following factor is responsible for the above (iv) In aquatic animals, the osmotic concentration of the
statements?
body fluids change with that of the ambient water
(a) Light (b) Water osmotic concentration.
(c) Temperature (d) Soil
(a) (i) and (ii) (b) (iii) and (iv)
36. Read the following statements (i to v) and answer the
question. (c) (i) and (iii) (d) (ii) and (iii)
(i) Temperature progressively decreases from pole to 41. Read the following statements regarding adaptation and
equator. choose the correct option.
(ii) Our intestine is a unique habitat for hundreds of (i) Many xerophytic plants have a thick cuticle on leaf
species of microbes. epidermis and sunken stomata to prevent
(iii) Average temperature exceeds 100°C in thermal transpiration.
springs and hydrothermal vents. (ii) Some xerophytic plants have special photosynthetic
(iv) In polar areas and high altitudes temperature goes to pathway (CAM) that enables their stomata to close
70°C. during day.
(v) Temperature goes to > 50°C in tropical desert in (iii) Opuntias has no leaves, they are reduced to spines.
summer.
(iv) All adaptation are genetically fixed in all organisms.
How many of the above statements are incorrect?
(v) In Opuntia, the pathway of photosynthesis is
(a) (i), (ii), (iii) (b) (i), (iv)
through C3 cycle.
(c) (ii), (iv), (v) (d) (iii), (iv)
(a) (i), (ii) and (iii) (b) Only (ii)
37. Read the following statements and choose the correct
option. (c) (iv) and (v) (d) All of these
(i) Light is essential for life to exist on the earth. 42. Which statement explains the concept of Allen’s rule?
(ii) Many species of small plants under the canopy to (a) Aquatic mammals have blubber as insulator.
tall trees in forest show optimal use of available light (b) Mammals of colder climate have shorter ears and limbs.
due to having large sized antenna and higher number (c) Mammals of humid and warmer region have more
of thylakoids. melanin in their skin.
(iii) UV rays are not harmful to many organisms. (d) The bears undergoes hibernation during winter.
(iv) Photoperiodic requirement is essential for many
plants for flowering. 43. Mark the incorrect statement.
(v) Red algae can live in deeper water of sea because of (a) Many fishes thrive in Antarctic water where
having pigment, phycoerythrin. temperature is always below zero .
(a) (i) and (iii) (b) (i), (iii) and (iv) (b) Microbes can survive in hot springs where
(c) only (iii) (d) (i), (ii), (iv) and (v) temperature exceeds 100°C.
38. Which of the following statement is false? (c) Fishes can survive even at a depth where pressure
exceeds 100 atm.
(a) Regulators are organisms that are able to maintain
homeostasis by physiological means or sometimes (d) Desert lizards have marvelous physiological ability
by behavioural means. to survive scorching heat of desert.
(b) All birds and mammals, and very few lower 44. Given below are some examples associated with a type of
vertebrates and in vertebrates are capable of adaptation.
osmoregulation and thermoregulation. (i) Basking by desert lizards in sun.
(c) Sweating and shivering are the means of (ii) Hiding in burrows by some animals.
thermoregulation by human. (iii) Wearing of woolen clothes.
(d) Plants are capable of thermoregulation. (iv) Thermal gaping
EBD_7209
298 Biology
Identify the correct option. (c) If Assertion is true but Reason is false.
(a) Scansorial adaptation (b) Cursorial adaptation (d) If both Assertion and Reason are false.
(c) Biochemical adaptation (d) Behavioural adaptation 50. Assertion : In sigmoid growth curve, population finally
45. Which of the following are the characteristics of expanding stabilizes itself.
population ? Reason : Finally, the death rate increases than the birth
(i) Pyramid - shaped age structure. rate.
(ii) An urn - shaped age structure. 51. Assertion : In a water body, Daphnia populations showed
(iii) Pre-reproductive and reproductive age groups distinct variations in their morphology at different
become more or less equal in size. seasons.
(iv) Rapidly growing population with high birth rate. Reason : Variations in temperature of water bodies at
(a) (i) and (iii) (b) (i) and (iv) different seasons influences cyclomorphosis in some
(c) (iii) and (iv) (d) (ii) and (iii) organisms.
46. Which of the following statement is incorrect regarding 52. Assertion : Predation is a type of interspecific interaction
predators? with a strategy based on feeding.
(a) It keeps prey population under control. Reason : A stable population is maintained through time
(b) It helps in maintaining species diversity. by the populations of predator and their prey and rarely
(c) It reduces intensity of competition among competing one population becomes abundant or scarce.
prey species. 53. Assertion : Species are groups of potentially
(d) Predator in nature are prudent because they do not interbreeding natural populations which are isolated from
exploit their prey. other such groups.
47. Which one of the following is categorised as a parasite in Reason : Distinctive morphological characters are
true sense? displayed due to reproductive isolation.
(a) Human foetus developing inside the uterus draws 54. Assertion : Cold blooded animals do not have fat layer.
nourishment from the mother. Reason : Cold blooded animals use their fat for metabolic
(b) The female Anopheles bites and sucks blood from process during hibernation.
humans.
(c) Head louse living on the human scalp as well as laying MATCHING TYPE QUESTIONS
eggs on human hair.
(d) The cuckoo (koel) lays its eggs in crow’s nest. 55. Match column-I with column-II and choose the correct
48. Which of the following is an incorrect statement? answer.
(a) The human liver fluke depends on two intermediate
hosts to complete its life cycle. Column-I Column-II
(b) The malarial parasite needs vector (mosquito to A. Pacific I. Produces a small number of
spread to other host). Salmon fish large sized offspring
(c) Parasites that feed on external surface of the host B. Mammals II. Produces a large number of
organism are called endoparasites. small sized offspring
(d) Cuscuta derives its nutrition from the host plant C. Oysters III. Breed only once in their
which it parasitises.
lifetime
49. Select the incorrect statement.
D. Birds IV. Breed many times during
(a) Overwhelming majority of animals and nearly all
plants maintain a constant internal temperature. their lifetime
(b) An orchid growing as an epiphyte on a mango branch (a) A – III, B – IV, C – II, D – I
is an example of commensalism. (b) A – I, B – IV, C – II, D – III
(c) In brood parasitism, the parasitic bird lays its eggs in (c) A – IV, B – II, C – I, D – III
the nest of its host and lets the host to incubate (d) A – II, B – IV, C – III, D – I
them.
56. Match the following
(d) In amensalism, one species is harmed whereas the
Population Example
other is unaffected.
A. Predation I. Cuscuta and hedge plants
ASSERTION/REASON TYPE QUESTIONS B. Commensalism II. Balanus and Chathamalus
C. Parasitism III. Cactus and moth
In the following questions, a statement of Assertion is followed D. Competition IV. Orchid and mango
by a statement of Reason.
(a) A – III, B – IV, C – I, D – II
(a) If both Assertion and Reason are true and the Reason is
(b) A – IV, B – III, C – II, D – I
the correct explanation of the Assertion.
(c) A – I, B – III, C – II, D – IV
(b) If both Assertion and Reason are true but the Reason is
not the correct explanation of the Assertion. (d) A – III, B – IV, C – II, D – I
Organisms and Populations 299
57. Match Column - I with Column - II and choose the correct
option.
A
Column I Column II

External level
A. Pacific salmon fish I Verhulst - pearl B
logistic growth
B. Nt = N0 ert II Breed only once in C
life time
C. Oyster III Exponential growth

IV A large number of
éK- Nù Internal level
D. dN/dt = rN ê ú small sized
ë K û (A) (B) (C)
offsprings
(a) Partial regulator Regulator Conformers
(a) A – IV; B – III; C – I; D – II (b) Regulator Conformers Partial regulator
(b) A – III; B – IV; C – I; D – II (c) Conformers Regulator Partial regulator
(c) A – III; B – I; C – IV; D – II (d) Regulator Partial regulator Conformers
(d) A – II; B – III; C – IV; D – I
60. What type of human population is represented by the
DIAGRAM TYPE QUESTIONS given age pyramid?

58. The given figure flows biome distribution with respect to Post-reproductive
annual temperature and precipitation. In this few parts are Reproductive
marked as A, B & C. Mark the correct identification from
the following picture. Pre-reproductive
Expanding Stable Declining
(a) Expanding population (b) Vanishing population
(c) Stable population (d) Declining population
61. A country with a high rate of population growth took
measures to reduce it. The figure below shows age-sex
pyramids of populations A and B twenty years apart.
Select the correct interpretation about them.

Age
70+
60 - 69
Male Female
50 - 59
age (in years)

40 - 49
(A) (B) (C)
(a) Tropical forest Temperate forest Coniferous forest 30 - 39
(b) Temperate forest Tropical forest Coniferous forest
20 - 29
(c) Temperate forest Coniferous forest Tropical forest
(d) Coniferous forest Tropical forest Temperate forest 10 - 19
59. The given figure shows the diagram match representation
0-9
of organismic response. Which option gives the correct
identification of three types of organisms (marked as A, B &
C) in response to abiotic factor?
15 12 9 6 3 0 3 6 9 12 15
‘A’
EBD_7209
300 Biology
62. The density of a population in a given habitat during a
given period, fluctuates due to changes in four basic
Age processes On this basis choose the correct option to fill
70+ up A and B boxes in the given diagram.
60 - 69
Male Female + Population –
50 - 59 A Density B
(N)

age (in years)


40 - 49

30 - 39 (a) A = Natality + Immigration, B = Mortality + Emigration


(b) A = Natality + Mortality, B = Immigration + Emigration
20 - 29 (c) A = Birth rate + Death rate, B = Mortality + Emigration
(d) A = Natality + Emigration, B = Mortality + Immigration
10 - 19
63. Identify I to IV which affect the population density.
0-9
Immigration
II
15 12 9 6 3 0 3 6 9 12 15
‘B’ I
Population Mortality
Natality
Density
III
(a) “B” is earlier pyramid and shows stabilized growth
IV
rate.
(b) “B” is more recent showing that population is very Emigration
young.
I II III IV
(c) “A” is the earlier pyramid and no change has occurred (a) Increase Decrease Increase Decrease
in the growth rate. (b) Decrease Increase Decrease Increase
(d) “A” is more recent and shows slight reduction in the (c) Increase Increase Decrease Decrease
growth rate. (d) Decrease Decrease Increase Increase

64. Study the population growth curves given below.

(i)
K
(ii)
Population
density (N)

Time (t)
Which options is the best for curve (i) and (ii) ?
S. No. Type of (i) curve Type of (ii) curve Equation for curve (i) Equation for curve (ii)

dN æK-Nö dN
(a) Logistic curve Logistic curve = rN ç ÷ = rN
dt è K ø dt
dN dN æK-Nö
(b) Exponential curve Logistic curve = rN = rN ç ÷
dt dt è K ø
dN æ K - Nö dN
= rN ç = rN
(c) Logistic curve Exponential curve dt è K ÷ø dt
dN dN æ K - Nö
= rN = rN ç
(d) Exponential curve Exponential curve
dt dt è K ø÷
Organisms and Populations 301

65. In laboratory experiments, two species of the protist 71. Regarding temperature and osmotic concentration nearly
Paramecium were grown alone and in the presence of the all plants are
other species. The following graphs show growth of (a) regulator (b) conformers
species 1 (left) and species 2 (right), both along and when (c) partial regulator (d) escaper in time
in mixed culture. 72. Very small animals are rarely found in polar regions because
Species 1 Species 2
(a) they have a smaller surface area relative to their volume.
Alone
(b) they have a larger volume relative to their surface area.
Alone (c) they have smaller metabolic rate.
Population (d) they have a larger surface area relative to their
individuals With With volume.
Species 2 Species I 73. The kangaroo rats of North American deserts do not need
Time to drink water because
Interpretation of these graphs shows that (a) they meet their water requirement through internal
fat oxidation when the water is a byproduct.
(a) competitive exclusion occurred in these experiments. (b) they are able to concentrate urine, to minimize water
(b) both species are affected by interspecific competition loss.
but species 1 is less affected. (c) they do not have sweat glands.
(c) both species are affected by interspecific competition (d) all of the above
but species 2 is less affected. 74. Many tribes living in the high altitude of Himalayas have a
(a) higher WBC count than people living in the plains.
(d) both species are affected equally by interspecific
(b) lower WBC count than people living in the plains.
competition.
(c) higher RBC count than people living in the plains.
CRITICAL THINKING TYPE QUESTIONS (d) lower RBC count than people living in the plains.
75. Population ecology is an important area of ecology because
66. Which one of the following do not account for the (a) it determines the interaction among organisms and
formation of major biomes? between the organisms and its physical environment.
(a) Annual variation in intensity of temperature. (b) evolutionary changes through natural selection take
place at the population level.
(b) Annual variation in duration of temperature.
(c) it links ecology to population genetics and evolution.
(c) Annual variation in precipitation. (d) it links different types of communities together.
(d) Annual variation in texture of soil. 76. Natural selection operates to evolve the desired tracts at
67. Many freshwater fishes cannot live for long in sea water (a) cellular level (b) species level
and vice-versa mainly because of the (c) community level (d) population level
(a) variation in light intensity. 77. If in a pond, there were 20 lotus last year and through
(b) change in the levels of thermal tolerance. reproduction 8 new plants are added, taking current
population to 28, the birth rate per year is
(c) osmosis. (a) 0.2 (b) 0.4
(d) spectral quality of solar radiation. (c) 0.6 (d) 0.8
68. Factors that are important for aquatic organisms include 78. If 4 individuals in a laboratory population of 40 fruitflies
(a) chemical composition of water died during a specified time interval (i.e., a week), the death
(b) pH of water rate in the population during that period is
(c) spectral quality of solar radiation (a) 1 (b) 0.1
(d) both (a) and (b) (c) 0.01 (d) 0.4
79. In an age pyramid, the number of individuals of
69. Many animals use the diurnal and seasonal variations in reproductive age is lesser than pre-reproductive but higher
light intensity and photoperiod as cues timing of than post reproductive ones. The population is
(a) for age only (a) growing (b) declining
(b) reproductive activities only (c) stable (d) can not be predicted
(c) migration only 80. The age structure of a population influences population
(d) all of these growth because
70. To a large extent the vegetation in any area is determined (a) younger females have more offsprings than do older
females.
by
(b) different age groups have different reproductive
(a) temperature and pH. capabilities.
(b) pH, mineral composition and light. (c) the more individuals that are immature the slower the
(c) pH, mineral composition and topography. population will grow.
(d) types of minerals in soil. (d) a shorter generation time results in slower population
growth.
EBD_7209
302 Biology
81. If N is the population density at time t, then its density at 87. When certain exotic species are introduced into
time t + 1 is geographical area, they become invasive and start
(a) Nt + 1 = Nt + [(B + I) + (D + E)] spreading fast because
(b) Nt + 1 = Nt – [(B + I) + (D + E)] (a) they have high reproductive rate.
(c) Nt + 1 = Nt + [(B + I) – (D + E)] (b) they produce chemicals to inhibit the growth of other
(d) Nt + 1 = Nt – [(B + I) – (D + E)] organisms.
82. The two basic processes which contribute to a increase in (c) there is no competition.
population density are (d) the invaded land does not have its natural predators.
(a) mortality and immigration 88. Which of the following adaptation do not lessen the impact
(b) natality and immigration of predation?
(a) Some species of insects and frogs are camouflaged.
(c) mortality and emigration
(b) Some animals are poisonous.
(d) mortality and emigration (c) Monarch butterfly is highly distasteful due to having
83. The integral form of the exponential growth equation is certain chemical in their bodies.
(a) Nt = N0e–rt (b) N0 = Ntert (d) Different feeding habits of finches.
89. Mac Arthur observed that five closely related species of
(c) Nt = N0ert (d) rN = Ntert Warblers living on the same tree were able to avoid
84. Assume that you have been studying a population of competition and co-exist due to
cattails at the edge of a pond. After 10 years of (a) cooperation in their foraging efforts.
observations, you notice that the population has remained (b) behavioural differences in their foraging activities.
steady. What is the most likely explanation?
(c) different kinds of insects they eat.
(a) The birth rate and death rate are both increasing at
the same rate. (d) all of the above
(b) The pond is drying up. 90. The Abingdon tortoise in Galapagos Islands became
extinct within a decade after goats were introduced on the
(c) The carrying capacity of pond has been reached. island. This is apparently due to
(d) Nutrients levels in pond are fluctuating. (a) lower intrinsic rate of goat.
85. Organisms with very high intrinsic growth rates have (b) the greater browsing efficiency of the goats.
(a) long generation times. (c) limiting resource.
(b) short generation times. (d) superiority of the goat.
(c) no courtship behaviour. 91. Gause’s ‘competitive exclusion principle’ states that
(d) no carrying capacities. (a) humans are the most widespread agents of
86. In exponential growth, the increase or decrease in disturbance.
population size during a unit period is (b) in a competition for similar resource both the
(a) (B + I ) – (D + E) participants are benefitted.
(b) (b + d) – N (c) in a competition, both the participants are excluded.
(c) N × (b – d) (d) two closely related species competing for the same
(d) r + N resources cannot co-exist indefinitely and
competitively inferior one will be eliminated
eventually.
36

Chapter
Ecosystem

FACT/DEFINITION TYPE QUESTIONS 9. The rate at which light energy is converted into chemical
energy of organic molecules is the ecosystem’s
1. An ecosystem is a (a) net primary productivity
(a) group of components that interact with one another. (b) gross secondary productivity
(b) group of interacting species in one place at one time. (c) net secondary productivity
(c) biological community and component of the physical (d) gross primary productivity
10. Which of the following most often limits the primary
environment with which the community interacts.
productivity of the ecosystem ?
(d) group of interacting chemicals and their cycles. (a) Solar radiation/light (b) Oxygen
2. Which of the following is/are example(s) of man-made (c) Consumers (d) Nitrogen
ecosystem? 11. What is the annual net primary productivity of whole
(a) Herbarium (b) Crop fields biosphere?
(c) Aquarium (d) Both (b) and (c) (a) 170 billion tons (dry weight) of organic matter.
3. Vertical distribution of different species occupying (b) 165 billion tons (dry weight) of organic matter.
different levels is called (c) 160 billion tons (dry weight) of organic matter.
(a) enumeration (b) stratification (d) 155 billion tons (dry weight) of organic matter.
(c) species composition (d) none of these 12. Which one of the following process help the water-
4. Which one of the following is not an abiotic component? soluble inorganic nutrients go down into the soil
(a) Temperature (b) Decomposers horizon and get precipitated as unavailable salts?
(a) Fragmentation (b) Leaching
(c) Water (d) Soil
(c) Catabolism (d) Humification
5. In an ecosystem abiotic components includes which of
13. How much portion of the photosynthetically active
the following ? radiation (PAR) is captured by the plants?
(a) Flow of energy (a) 5 – 10% (b) 7 – 10%
(b) Cycling of materials (c) 8 – 10% (d) 2 – 10%
(c) Consumers 14. Energy flow in an ecosystem is
(d) Flow of energy and cycling of materials (a) unidirectional (b) bidirectional
6. Two main structural features of an ecosystem are (c) multidirectional (d) all of these
(a) species composition and stratification 15. Each trophic level has a certain mass of living material at
(b) species composition and productivity a particular time is known as
(c) productivity and energy flow (a) catabolism (b) standing crop
(d) nutrient cycling and stratification (c) humification (d) primary productivity
7. Identification and enumeration of plant and animal species 16. Ecosystems are ______ because resources ____ move
of an ecosystem gives its from one ecosystem to another.
(a) open; can (b) closed; cannot
(a) productivity (b) stratification
(c) one-way; can (d) one-way; cannot
(c) species composition (d) all of these 17. Energy pyramids are used to represent energy transfer in
8. Primary productivity an ecosystem because energy is _______ between each
(a) is equal to the standing crop of an ecosystem. trophic level.
(b) is greatest in freshwater ecosystems. (a) gained
(c) is the rate of conversion of light to chemical energy (b) lost
in an ecosystem. (c) conserved
(d) is inverted in some aquatic ecosystems. (d) either conserved or gained
EBD_7209
304 Biology
18. The primary difference between sedimentary and (c) Food-chain length is limited by productivity.
atmospheric cycles is that in _____ cycles the nutrient (d) Food-chain length is a function of an ecosystem’s
does not _____. physical structure.
(a) sedimentary; leave the terrestrial environment 24. Which of the following is not true for a pyramid of
(b) sedimentary; leave the aquatic environment productivity?
(c) atmospheric; leave the aquatic environment (a) Only about 10% of the energy in one trophic level is
(d) sedimentary; enter the atmosphere passed into the next level.
19. Which one of the following pairs is a sedimentary type of (b) Because of the loss of energy at each trophic level,
biogeochemical cycle ? most food chains are limited to three to five steps.
(a) Phosphorus and carbon monoxide (c) The pyramid of productivity of some aquatic
(b) Oxygen and nitrogen ecosystems is inverted because of the large
(c) Phosphate and nitrogen zooplankton primary - consumer level.
(d) Phosphorus and sulphur (d) Eating grain-fed beef is an inefficient means of
obtaining the energy trapped by photosynthesis.
STATEMENT TYPE QUESTIONS 25. Which statement is true corresponding to the given food
chain ?
20. Consider the following statements.
Grass ® Rabbits ® Snakes ® Hawks
(i) An ecosystem is a functional unit of nature and
(a) Each predator population has a greater biomass than
comprises abiotic and biotic components.
its prey population.
(ii) Abiotic components are organic materials.
(b) Each prey population has a greater biomass than its
(iii) Biotic components are producers, consumers and predator population.
decomposers. (c) Each population is omnivorous.
(iv) Energy flow is bidirectional. (d) Each level of food chain returns inorganic nutrients
which two statements are correct ? and energy to the producer.
(a) (i) and (ii) (b) (ii) and (iv) 26. Which two of the following statements regarding food
(c) (iii) and (iv) (d) (i) and (iii) chains are correct ?
21. Which one of the following statement is correct about (i) Removal of 80% tigers resulted in increased growth
decomposition ? of vegetation.
(a) Decomposition rate is slower if detritus is rich in (ii) Removal of most carnivores resulted in increased
lignin and chitin, and quicker if detritus is rich in population of deer.
nitrogen and water - soluble substances like sugars. (iii) Length of food chain is limited to 3 – 4 trophic levels
(b) Decomposition rate is slower if detritus is rich in due to energy loss.
nitrogen and water - soluble substances like sugars, (iv) Length of food chain may vary from 2 – 3 trophic
and quicker if detritus is rich in lignin and chitin. levels.
(c) Decomposition rate is slower if detritus is rich in (a) (i) and (iv) (b) (i) and (ii)
cellulose, and quicker if detritus is rich in phosphorus. (c) (ii) and (iii) (d) (iii) and (iv)
(d) Decomposition rate is quicker if detritus is rich in 27. The given statements (i - iv) are the characteristics of
lignin, and lower if detritus is rich in sulphur. productivity.
22. Which one of the following statement is correct? (i) The rate of biomass production is called productivity
(a) Warm and moist environment favour decomposition and is expressed in terms of a kcal m –2.
whereas low temperature and anaerobiosis inhibit (ii) Net primary productivity is rate of production of
decomposition. biomass during photosynthesis.
(b) Warm and moist environment inhibit decomposition (iii) Net primary productivity minus respiration loss is
whereas low temperature and anaerobiosis favour called net primary productivity.
decomposition. (iv) Primary productivity depends only on the plant
(c) Warm moist environment and anaerobiosis inhibit species inhabiting a particular area.
decomposition whereas low temperature favours Identify the correct statements.
decomposition. (a) (i) and (iii) (b) (ii) and (iii)
(d) Warm and low temperature inhibit decomposition (c) (ii), (iii) and (iv) (d) (ii) and (iv)
whereas anaerobiosis favours decomposition. 28. Food chains differ from food webs in that
23. Which of the following is not a hypothesis for why food- (i) food chains are single sequence of who eats whom
chain length is limited? in a community.
(a) Food-chain length is limited by one organism’s ability (ii) food chains better represent the entire community.
to consume another. (iii) food webs represent the complex interaction among
(b) Food-chain length is limited because they are easily food chains.
disrupted by environmental perturbations. (iv) food chain is the flow of energy in a population.
Ecosystem 305

(a) (i) and (iii) (b) (i) and (iv) (ii) All biogeochemical cycles include both organisms
(c) (i), (ii) and (iii) (d) (i), (ii), (iii) and (iv) and non-living components.
29. Which of following is/are trend(s) in from ecological (iii) Most elements remain longest in the living portion
succession ? of their cycle.
(i) An increase in complexity of species. (iv) The chemical elements used by organisms in large
(ii) An increase in productivity. quantities cycle back and forth between organisms
(iii) An increase in community stability and species and environment.
diversity. (a) (i) and (iii) (b) (i), (ii) and (iv)
(iv) A decrease in non-living organic materials. (c) (i), (ii) and (iii) (d) All of these
(a) (i) and (iii) (b) (i) and (iv)
(c) (i), (ii) and (iii) (d) (i), (ii), (iii) and (iv) ASSERTION/REASON TYPE QUESTIONS
30. Which of the following statement(s) is/are correct?
In the following questions, a statement of Assertion is followed
(i) An important characteristic of all communities is that
by a statement of Reason.
their composition and structure constantly change
(a) If both Assertion and Reason are true and the Reason is
in response to the changing environmental
the correct explanation of the Assertion.
conditions.
(b) If both Assertion and Reason are true but the Reason is
(ii) The climax community remains stable as long as the
not the correct explanation of the Assertion.
environment remains changed.
(c) If Assertion is true but Reason is false.
(iiii) The entire sequence of communities that
(d) If both Assertion and Reason are false.
successively change in a given area are called sere(s).
35. Assertion : Pyramid of energy may be upright or inverted.
(a) (i), (ii) and (iii) (b) (i) and (ii)
Reason : Only 20% of energy goes to next trophic level.
(c) (ii) and (iii) (d) (i) and (iii)
36. Assertion : Net primary productivity is gross primary
31. Which of the following include(s) ecosystem services ?
productivity minus respiration.
(i) Purification of air and water by forests.
Reason : Secondary productivity is produced by
(ii) Forests mitigate droughts and flood.
heterotrophs.
(iii) Forests act as store house of carbon.
37. Assertion : In a food chain, members of successive higher
(iv) Forests influence hydrological cycle.
levels are fewer in number.
(a) (i) and (iii) (b) (i) and (iv)
(c) (i), (ii) and (iii) (d) All of these Reason : Number of organisms at any trophic level
32. Which of the following factors influence communities ? depends upon the availability of organisms which serve
(i) Climate as food at the lower level.
(ii) Species interaction 38. Assertion : A network of food chains existing together in
(iii) Feeding relationships among organisms an ecosystem is known as food web.
(iv) Succession Reason : An animal like kite cannot be a part of a food
(a) (i) and (iii) (b) (i) and (iv) web.
(c) (i), (ii) and (iii) (d) All of these 39. Assertion : Biotic community has higher position than
33. Choose the correct statements regarding ecological population in ecological hierarchy.
pyramids. Reason : Population of similar individuals remains isolated
(i) The relationship between organisms at different in the community.
trophic levels is expressed in terms of number,
biomass and energy. MATCHING TYPE QUESTIONS
(ii) Any calculations of energy content, biomass or 40. Match column-I with column-II and choose the correct
number has to include one group of organism at that option.
trophic level. Column - I Column - II
(iii) In most ecosystems, all the pyramids of number, A. Primary succession I. Autotrophs
biomass and energy are upright. B. Climax community II. Community that has
(iv) The pyramid of biomass in sea is generally inverted. completed succession
(v) Pyramid of energy is always inverted and can never
C. Consumer III. Colonization of a new
be upright.
environment
(a) (i), (iii) and (iv) (b) (i)
(c) (ii) and (v) (d) (i) and (v) D. Producer IV. Animals
34. Which of the following statement(s) is/are correct about (a) A – III; B – II; C – IV; D – I
biogeochemical cycle? (b) A – III; B – I; C – IV; D – II
(i) Gaseous exch an ges of phosphorus between (c) A – I; B – III; C – II; D – IV
organism and environment are negligible (d) A – II; B – III; C – IV; D – I
EBD_7209
306 Biology
41. Match the following and choose the correct option. DIAGRAM TYPE QUESTIONS
Column - I Column - II
A. Standing state I. Nitrogen, Carbon 46. Refer the given diagrammatic representation of trophic
levels in an ecosystem some spaces are marked A, B, C
B. Gaseous cycles II. Amount of nutrients and D. Identify A, B, C and D.
C. Standing crop III. Sulphur, Phosphorus
D. Sedimentary cycles IV. Living matter at different
trophic levels
(a) A – II; B – I; C – IV; D – III
(b) A – I; B – II; C – III; D – IV
(c) A – III; B – II; C – IV; D – I
(d) A – I, B – IV; C – III; D – II
42. Match the following and choose the correct option.
Column - I Column - II
A. Phosphorus I. Atmosphere
B. Carbon II. Producers
C. Goat III. Rock
D. Grasses IV. T2
(a) A – III; B – II; C – IV; D – I
(b) A – III; B – I; C – IV; D – II
(c) A – I; B – III; C – II; D – IV
(d) A – II; B – III; C – IV; D – I (a) A – Primary producer; B – Secondary consumer;
43. Match the following and choose the correct option. C – Man / Lion; D – Plants
Column - I Column - II (b) A – Secondary consumer; B – Primary producer;
A. Pioneer community on I. Crustose lichens C – Man / lion; D – Plants
lithosphere (c) A – Primary producer; B – Secondary consumer;
B. Ecological succession II. Mesophytes C – Plants; D – Man / lion
C. Climax community III. Ecosystem (d) A – Secondary consumer; B – Primary producer;
development C – Plants; D – Man / lion
D. Ecological pyramid IV. Elton 47. Refer the given nutrient cycle in a terrestrial ecosystem
(a) A – III; B – II; C – IV; D – I with few labels marked as A, B, C and D. Identify A, B, C
(b) A – III; B – I; C – IV; D – II and D.
(c) A – I; B – III; C – II; D – IV
(d) A – II; B – III; C – IV; D – I B A
44. Match the following and choose the correct option.
Column - I Column - II
Litter fall
A. Presence of 3-4 storey I. Blue-green algae Detritus
of plants grown in a
C
forest
Uptake
B. A biome having grasses II. Stratification Soil solution
with scattered trees Run off
C. Man made ecosystem III. Savannah D
D. Pioneer in hydrosere IV. Dam Rock minerals
(a) A – III; B – II; C – IV; D – I
(b) A – III; B – I; C – IV; D – II (a) A – Consumers; B – Decomposition; C – Producers
(c) A – I; B – III; C – II; D – IV D – Weathering
(d) A – II; B – III; C – IV; D – I (b) A – Consumers; B – Weathering; C – Producers;
45. Which of the following pair is not correctly matched ? D – Decomposition
(a) Autotrophs – Fungi (c) A – Producers; B – Consumers; C – Decomposition;
(b) Primary consumers – Zooplankton D – Weathering
(c) Secondary consumers – Fishes (d) A – Consumers; B – Producers; C – Decomposition;
(d) Decomposers – Flagellates D – Weathering
Ecosystem 307

48. In the given food web few organisms are marked as (1),
SC PC
(2), (3) and (4). Identify (1), (2), (3) and (4).
(c) PC (d) PP
PP

52. Two food chains are given below.


(i) Tree ® aphid ® insectivorous bird ® prey feed on
bird
(ii) Phytoplankton ® zooplankton ® plankton feeding
fish ® carnivorous fish
Which diagram is a pyramid of energy representing both
food chains?
Trophic level
(1) (2) (3) (4) 4
(a)
3
(a) Deer Rabbit Frog Rat
2
(b) Dog Squirrel Bat Deer 1
(c) Rat Dog Tortoise Crow Trophic level
(b) 4
(d) Squirrel Cat Rat Pigeon
3
49. Given below is an imaginary pyramid of numbers. What 2
could be one of the possibilities about certain organisms 1
at some of the different levels ?
(c) Trophic level
TC 10 4
3
SC 50 2
PC 500 1
PP 1 (d) Trophic level
4
(a) Level PC is “insects” and level SC is “small 3
insectivorous birds”. 2
(b) Level PP is “phytoplanktons” in sea and “whale” is 1
on top level TC. 53. The given diagram shows (I, II, III, and IV) the flow of
(c) Level PP is “peepal trees” and the level SC is “sheep”. materials between different trophic levels. Which arrow
is incorrect?
(d) Level PC is “rats” and level SC is “cats”.
50. Given below is one of the types of ecological pyramids. Decomposers
This type represents
Secondary consumers

II III
IV
Primary consumers

I Producers
(a) Pyramid of numbers in a grassland
(b) Pyramid of biomass in a fallow land (a) I (b) II
(c) Pyramid of biomass in a lake (c) III (d) IV
(d) Energy pyramid in a spring
CRITICAL THINKING TYPE QUESTIONS
51. Which of the given pyramids represent the variation in
biomass at different trophic levels in pond ecosystem? 54. Which one is the correct food chain ?
(a) Eagle ® Snake ® Grasshopper ® Grass ® Frog
SC SC
(b) Frog ® Snake ® Eagle ® Grasshopper ® Grass
(a) PC (b) PC (c) Grasshopper ® Grass ® Snake ® Frog ® Eagle
PP PP (d) Grass ® Grasshopper ® Frog ® Snake ® Eagle
EBD_7209
308 Biology
55. In parasitic food chain, the pyramid of number is 63. Which of the following could not be considered an
(a) linear (b) upright ecosystem ?
(c) inverted (d) inverted upright (a) A small pond
56. If CO2 is removed totally from the biosphere, which (b) All the fish in a coral reef
organisms will be affected first ?
(c) Earth
(a) Consumers
(d) A pile of dung in a pasture
(b) Secondary consumers
(c) Producers 64. There is no difference between
(d) Primary producers (a) secondary consumers and herbivores.
57. The correct order of the process of decomposition is (b) primary consumers and herbivores.
(a) Catabolism ® Fragmentation ® Leaching ® (c) first trophic level and herbivores.
Humification ® Mineralization (d) primary carnivores and second trophic level.
(b) Catabolism ® Fragmentation ® Humification ® 65. Which of the following contribute(s) to the carbon cycle ?
Leaching ® Mineralization
(a) Respiration and photosynthesis
(c) Fragmentation ® Humification ® Catabolism ®
Leaching ® Mineralization (b) Fossil fuel combustion
(d) Fragmentation ® Leaching ® Catabolism ® (c) Decompostion of dead organisms
Humification ® Mineralization (d) All of these
58. Productivity at the second trophic level is always 66. In an upright pyramid of biomass, the herbivores generally
(a) greater than the productivity at the first trophic level. occupy which of the following position ?
(b) less than the productivity at the first trophic level. (a) First position (b) Second position
(c) equal to the productivity at the first trophic level. (c) Third position (d) Fourth position
(d) extremely variable compared to the productivity at
67. The primary succession refers to the development of
the first trophic level.
communities on a
59. Which one of the following animals may occupy more
(a) freshly cleared crop field.
than one trophic levels in the same ecosystem at the same
time? (b) forest clearing after devastating fire.
(a) Sparrow (b) Lion (c) pond, freshly filled with water after a dry phase.
(c) Goat (d) Frog (d) newly-exposed habitat with no record of earlier
vegetation.
60. The transfer of energy from one trophic level to another
is governed by the 2nd law of thermodynamics. The 68. Which of the following compartments of the global
average efficiency of energy transfer from herbivores to ecosystem would be characterized by very slow
carnivores is movement of materials within the compartment ?
(a) 5% (b) 10% (a) Oceans (b) Fresh water
(c) 25% (d) 50% (c) Atmosphere (d) Land
61. The phosphorus cycle differs from the carbon cycle in 69. Which one of the following is not used for construction
that of ecological pyramids?
(a) the phosphorus does not enter living organisms, (a) Number of individuals (b) Rate of energy flow
whereas carbon does. (c) Fresh weight (d) Dry weight
(b) the phosphorus cycle does not include a gaseous 70. Which one of the following is not a function of an
phase, whereas the carbon cycle does. ecosystem?
(c) the phosphorus cycle includes a solid phase, (a) Energy flow (b) Decomposition
whereas the carbon cycle does not. (c) Productivity (d) Stratification
(d) the primary reservoir of the phosphorus cycle is the 71. Which one of the following is a primary consumer in maize
atmosphere, whereas the primary reservoir for the field ecosystem?
carbon cycle is in rock. (a) Lion (b) Grasshopper
62. Grasslands can support greater grazing rates by herbivores (c) Wolf (d) Phytoplankton
than forests because 72. In a food web, each successive trophic level has
(a) net production of grassland is greater. (a) increased total energy content
(b) more of the grassland is above the ground. (b) less total energy content
(c) grasslands receive more sunlight. (c) more total energy content
(d) grasslands produce less woody plant tissue. (d) non-estimated energy content
Ecosystem 309

73. First link in any food chain is a plant because (b) secondary productivity
(a) green plants can synthesize food. (c) net primary productivity
(b) they can eat everything. (d) net productivity
(c) food is stored at one place. 76. In ecological succession from pioneer to climax
(d) none of the above. community, the biomass shall
74. Which of the following is the most stable ecosystem ?
(a) decrease
(a) Mountain (b) Desert
(b) increase and then decrease
(c) Forest (d) Ocean
(c) no relation between these communities
75. Energy storage at consumer level is called
(a) gross primary productivity (d) increase continuously
EBD_7209
37
Biodiversity and

Chapter
Conservation

FACT/DEFINITION TYPE QUESTIONS 9. Exotic species


(a) are often endangered.
1. Biodiversity describes the study of diversity
(b) usually increase biodiversity.
(a) below species level.
(c) often enhance the habitat for native species.
(b) at species level.
(d) usually reduce biodiversity.
(c) at community level.
10. The highest number of species in the world is represented
(d) at all levels of biological organization.
by
2. Alpha diversity is present
(a) algae (b) lichens
(a) within community.
(c) fungi (d) mosses
(b) between community.
11. Which of the following is not the example of recent
(c) ranges of communities. extinction
(d) all of the above. (a) Steller’s sea cow (b) Dodo
3. Diversity between two communities is called (c) Quagga (d) Pigeon
(a) alpha diversity (b) beta diversity 12. Which vertebrate group is more vulnerable to extinction?
(c) gamma diversity (d) none of these
(a) Birds (b) Amphibians
4. Diversity of habitat over the total landscape is called
(c) Mammals (d) Fishes
(a) b-diversity (b) g-diveristy (gamma)
13. From origin of life to its diversification on earth, there
(c) landscape diversity (d) ecosystem diversity have occurred following number of episodes of mass
5. Which one of the following pairs of geographical areas extinction.
show maximum biodiversity in our country?
(a) Two (b) Three
(a) Sunderbans and Rann of Kutch
(c) Four (d) Five
(b) Eastern Ghats and West Bengal
14. The ‘Evil Quartet’ of biodiversity loss, does not include
(c) Eastern Himalaya and Western Ghats
(a) habitat loss
(d) Kerala and Punjab
(b) introduction of alien species
6. IUCN stands for
(c) o-exploitation
(a) Indian Union for Conservation of Nature
(d) hunting
(b) International Union for Conservation of Nature
15. Amazon rain forest, called the ‘Lungs of the Planet’,
(c) Indian Union for Chemical Nomenclature
contribute following per centage of oxygen by
(d) International Union for Conservation of Nutrients photosynthesis, to earth atmosphere
7. According to IUCN 2004, the total number of plant and (a) 20% (b) 35%
animal species described so far is slightly more than
(c) 42% (d) 50%
(a) 5 million (b) 7 million
16. The taxa believed likely to join the endangered category
(c) 1.5 million (d) 0.5 million
in near future is called
8. More than 70% of all the species recorded so far, are
(a) Rare (b) Extinct
(a) insects (b) plants
(c) Vulnerable (d) Living fossil
(c) animals (d) invertebrates
Biodiversity and Conservation 311

17. The Cichlid species of Lake Victoria were driven to, or 26. Animals and plants are best protected in
nearly to, extinction by the introduction of _________. (a) zoos (b) botanical gardens
(a) North American sturgeon (c) sanctuaries (d) national parks
(b) Nile perch 27. The total number of national parks and sanctuaries in
(c) Eels India are
(d) Bass (a) 82 national parks and 410 sanctuaries.
18. Which of the following is not currently a major cause of (b) 83 national parks and 421 sanctuaries.
the global reduction in biodiversity? (c) 85 national parks and 450 sanctuaries.
(a) Overexploitation (d) 87 national parks and 460 sanctuaries.
(b) Global warming 28. Which one of the following is an example of ex-situ
(c) Habitat destruction conservation?
(d) Introduction of foreign predators and disease
(a) National park (b) Wildlife sanctuary
19. Which of the following is the most serious threat to
(c) Seed bank (d) Sacred groves
biodiversity?
29. In cryopreservation germplasm is maintained at
(a) Competition from exotic species
(a) – 196° F (b) 0° F
(b) Commercial harvesting
(c) – 100° F (d) none of these
(c) Habitat loss
(d) Overexploitation STATEMENT TYPE QUESTIONS
20. Which of the following characteristics would cause a 30. Mark the correct statement
country to be considered a high-priority region for (a) Amazonian rain forest has greatest biodiversity on earth.
conservation efforts? (b) According to Robert May estimates, the global
(a) A high degree of endemism species diversity is 7 million.
(b) Having low species richness (c) Biodiversity is greatest in tropics.
(c) Having little natural habitat remaining (d) All of the above
(d) All of the above 31. Best way to preserve the wild life is
21. Which of the following is not the approach for in-situ (a) to kill the predators.
conservation ? (b) vaccinize the animals.
(a) Biosphere reserve (b) Sanctuary (c) to preserve natural habitat.
(c) Wild life safari park (d) Sacred groove (d) optimize the breeding habit.
22. The area where wild populations, traditional life styles 32. Which one of the following statement is correct for
and genetic resources are protected is called botanical garden?
(a) core zone (b) buffer zone (a) They provide a beautiful area for recreation.
(c) biosphere reserve (d) manipulation zone (b) One can observe tropical plants there.
23. Which of the following is considered a hot-spot of (c) They allow ex-situ conservation of germ plasm.
biodiversity in India? (d) They provide the natural habitat for wildlife.
(a) Indo-Gangetic Plain (b) Eastern Ghats 33. Which of the following is done in a wild life sanctuary?
(c) Aravalli Hills (d) Western Ghats (a) conservation of fauna
24. Hotspots of biodiversity means (b) conservation of flora
(a) areas of the earth that contain many endemic species. (c) utiliation of soil and flora
(b) species serves as proxy for entire communities in (d) prohibition of hunting
particular area. 34. Which one of the following does not represent
(c) species in particular niche/area. biodiversity of a geographical region?
(d) species diversity at particular area. (i) Genetic diversity present is in the dominant species
25. Endemic plants and animals are those which are of the region .
(a) cosmopolitan in distribution. (ii) Species endemic to the region.
(b) restricted to certain area. (iii) Endangered species found in the region.
(c) found in arctic region. (iv) The diversity in the organisms living in the region.
(d) gregarious in habit. (a) (i) & (ii) (b) (iii) & (iv)
(c) (ii) only (d) (iv) only
EBD_7209
312 Biology
35. Which one of the following shows concept of species- (v) Loss of habitat
area relationship? (a) (i) and (ii) only (b) (iv) and (v) only
(a) The number of species in an area increases with the (c) (i), (ii), (v) only (d) (ii), (iv), (v) only
size of the area.
(b) Larger species require larger habitat areas than do ASSERTION/REASON TYPE QUESTIONS
smaller species. In the following questions, a statement of Assertion is followed
(c) Most species within any given area are endemic. by a statement of Reason.
(d) The larger the area, the greater the extinction rate. (a) If both Assertion and Reason are true and the Reason is
36. Which one of the following statement is true for genetic the correct explanation of the Assertion.
diversity? (b) If both Assertion and Reason are true but the Reason is
(a) The total genetic information contained within all not the correct explanation of the Assertion.
individuals of species. (c) If Assertion is true but Reason is false.
(b) The total phenotypic information contained within
all individuals of a species. (d) If both Assertion and Reason are false.
(c) The variety of life-forms on earth. 43. Assertion: Communities that comprise of more species
(d) The variety of biotic communities in a region along tend to be more stable.
with abiotic components. Reason: A higher number of species results in less animal
37. Which of the following is not a step in understanding variation in total biomoss.
biodiversity? 44. Assertion : Species diversity decreases while ascending
(a) Naming the species a mountain.
(b) Looking at other related species Reason : With increase in altitude and rise in temperature
(c) Assessing the species geographic range diversity of species becomes less.
(d) Quantifying the species genome 45. Assertion: The presently occurring species extinction is
38. Which one is the most important human activity leading different from the mass extinctions that have occurred
to extinction of wildlife?
earlier.
(a) Alteration and destruction of the natural habitats.
Reason : Extinction in present day is due to natural causes,
(b) Hunting for commercially valuable wildlife products.
whereas earlier it was due to man-made causes.
(c) Pollution of air and water.
(d) Introduction of alien species. 46. Assertion : Tropical rain forests are disappearing fast
39. Sacred groves are found in from developing countries such as India.
(a) Khasi and Jaintia Hills in Meghalaya. Reason : No value is attached to these forests because
(b) Aravalli Hills of Rajasthan. these are poor in biodiversity.
(c) Western ghat regions of Karnataka and Maharashtra 47. Assertion: Diversity observed in the entire geographical
and Sarguja, Chanda and Bastar areas of Madhya area is called gamma diversity.
Pradesh. Reason : Biodiversity decreases from high altitude to low
(d) All of the above altitude.
40. Modern ex-situ conservation includes
(i) cryopreservation techniques MATCHING TYPE QUESTIONS
(ii) in vitro-fertilization
(ii) propagation of plants by using tissue culture methods 48. Match column-I with column-II and choose the correct
(a) Only (i) (b) Only (iii) option.
(c) (ii) and (iii) (d) All of these Column-I Column-II
41. Biodiversity loss occurs due to the reasons given below. A. Nile Perch in Lake I. Obvious reasons for
(i) Habitat loss and fragmentation Victoria biodiversity conservation
(ii) Co-extinction B. Narrowly utilitarian II. Habitat destruction
(iii) Over-exploitation C. Main cause for III. High endemism
(iv) Alien species invasion biodiversity loss
Identify the correct reasons. D. Hotspots IV. Alien species
(a) (i) and (ii) (b) (i), (ii) and (iii)
(a) A – II, B – I, C – IV, D – III
(c) (ii), (iii) and (iv) (d) (i), (ii), (iii) and (iv)
42. The major cause of loss of numbers of migratory birds is (b) A – IV, B – I, C – II, D – III
(i) Bad weather (ii) Urbanization (c) A – I, B – III, C – II , D – IV
(iii) Pesticides (iv) Fragmentation (d) A – II, B – I, C – III, D – IV
Biodiversity and Conservation 313

49. Match column -I with column- II and choose the correct 52. Match the following and choose the correct option.
option. Column-I Column-II
Column-I Column-II A. Over-exploitation I. Environmental damage
by humans and threat to native
A. Biodiversity I. Edward Wilson species
B. Introduction of Nile II. Decline in plant
B. In-situ conservation II. Co-extinction Perch in Lake Victoria population
C. Plant pollinator mutualism III. On-site conservation C. Less solar energy III. Extinction of more than
2000 species of native
D. Ex-situ conservation IV. Off-site conservation
birds
(a) A – II, B – I, C – IV, D – III D. Introduction of Water IV. Extinction of Cichlid fish
(b) A – IV, B – I, C – II, D – III Hyacinth in India
(c) A – I, B – III, C – II , D – IV E. Colonization of V. Extinction of Passenger
tropical pacific pigeon
(d) A – II, B – I, C – III, D – IV Islands
50. Match the following and then choose the correct option. (a) A – II, B – V, C – IV, D – III, E – I
Column-I Column-II (b) A – V, B – IV, C – II, D – I, E – III
(c) A – I, B – II, C – III, D – IV, E – V
A. Endemism I. Khasi and Jaintia hills
Meghalaya (d) A – IV, B – I, C – II, D – V, E – III
B. Hotspot of India II. Advanced ex-situ
DIAGRAM TYPE QUESTIONS
conservation
C. Sacred groove III. Species found in a 53. Given below are pie diagrams A, B and C related to
particular area only proportionate number of species of major taxa of
D. Cryopreservation IV. Zoological park and invertebrates, vertebrates and plants respectively.
Botanical gardens Critically study and fill in the blanks I, II, III and IV.
E. Ex-situ conservation V. Western Ghats Other animal
(a) A – III, B – V, C – I, D – II, E – IV groups
Mammals
(b) A – I, B – II, C – III, D – IV, E– V
Crustaceans Fishes Birds
(c) A – II, B – III, C – IV, D – V, E – I
I Insects
(d) A – V, B – I, C – IV, D – II, E – III
51. Match the following and choose the correct option. II Reptiles
Column-I Column-II
A B
A. Narrowly utilitarian I. Conserving biodiversity
Invertebrates Vertebrates
argument for major ecosystem
services
B. Broadly utilitarian II. Conserving biodiversity Mosses
argument for philosophical or Ferns and
spiritual need to realise allies
Birds
that every species has III
intrinsic value and moral IV
duty to pass our
biological legacy in good Lichens
order to future generation.
A B Algae C
C. Ethical argument III. Conserving biodiversity Plant
for direct economic (a) I- Molluscs, II-Amphibians, III-Fungi,
benefits like food, IV-Angiosperms
medicine, industrial (b) I- Molluscs, II-Amphibians, III-Angiosperms,
products etc. IV-Fungi
(a) A – I, B – II, C – III (b) A – III, B – I, C – II (c) I- Hexapoda, II-Amphibians, III-Fungi,
(c) A – II, B – I, C – III (d) A – I, B – III, C – II IV-Angiosperms
(d) I- Turtles, II-Amphibians, III-Fungi, IV-Angiosperms
EBD_7209
314 Biology
54. Using the figure, determine the percentage of bird species 58. If S is species richness, A is area, Z is slope of the line,
that will be lost if the island's inhabitable land area is and the C is Y-intercept, then the species richness will be
reduced from 100,000 km2 to 1 km2. shown as
200 (a) S = C + Az (b) S = C + AZ
100 (c) S = C. AZ (d) S = C. Az
50 59. Which animal has become extinct from India?
S : Number of bird species

(a) Snow Leopard (b) Hippopotamus


30 (c) Wolf (d) Cheetah
20 60. A threatened species category includes
Slope (a) only endangered species.
10
(b) only vulnerable species.
0.15
5 (c) endangered and rare species.
S = 1.89
(d) endangered, vulnerable and rare species.
2
0.001 0.01 0.1 1 10 100 1000 10,000 100,000 61. In your opinion, which is the most effective way to
conserve the plant diversity of an area?
A : Island land area (km2)
(a) 17 percent of the bird species will be lost. (a) By developing seed bank
(b) 20 percent of the bird species will be lost. (b) By tissue culture method
(c) All of bird species will be lost. (c) By creating botanical garden
(d) 93 percent of the bird species will be lost. (d) By creating biosphere reserve
55. Which of the following boxes show maximum, greater and 62. Animal species should be preserved mainly because
minimum diversity ? (a) zoologists want to study them.
A B (b) they are lovely creatures.
(c) they are useful to mankind.
(d) man cannot recreate a species of animals after its
destruction.
63. Management of biosphere for providing maximum benefit
to the present generation and also maintaining its potential
C
for future generations, is the theme of
(a) afforestation (b) conservation
(c) deforestation (d) population
64. All forms of life should be conserved because
(a) A- Minimum diversity, B - Greater diversity, C - (a) they maintain diverse genetic resources.
Maximum diversity (b) they have economic values.
(b) A - Maximum diversity, B - Greater diversity, (c) they are important for maintaining balance of nature.
C - Minimum diversity (d) they will be otherwise lost.
(c) A - Maximum diversity, B - Minimum diversity, 65. Species diversity increase as one proceeeds from
C - Greater diversity (a) high altitude to low altitude and high latitude to low
(d) A - Minimum diversity, B - Maximum diversity, latitude.
C - Greater diversity. (b) low altitude to high altitude and high latitude to low
latitude.
CRITICAL THINKING TYPE QUESTIONS (c) low altitude to high altitude and low latitude to high
latitude.
56. Following arrangement is correct from the point of view
of decreasing biodiversity in angiosperms (N), fungi (F), (d) high altitude to low altitude and low latitude to high
latitude.
pteridophytes (P) and algae (A).
66. Which one of the following pairs of organisms are exotic
(a) N > F > P > A (b) N > F > A > P
species introduced in India?
(c) F > N > P > A (d) F > N > A > P
(a) Nile perch, Ficus religiosa
57. India’s share in global species diversity is around
(b) Ficus religiosa, Lantana camara
(a) 8% (b) 14 %
(c) Lantana camara, Water hyacinth
(c) 17 % (d) 2.4 %
(d) Water hyacinth, Prosopis cinereria
Biodiversity and Conservation 315

67. The table below give the populations (in thousands) of 69. Why do migratory species present special preservation
ten species (A = J) in four areas (I – IV) consisting of the challenges?
number of habitats given within brackets against each (a) Because they are endemic, they are especially
area. Study the table and answer the questions. susceptible to habitat destruction.
(b) Their conservation may require international
Area and Species and their population (in thousands) in the area cooperation when they require habitats in different
Number of countries.
Habitats A B C D E F G H I J
(c) They are often prone to population number decline
I. (11) 2.3 1.2 0.52 6 – 3 1.1 9 – 10.3 during their long migratory journeys.
II. (11) 10.2 – 0.62 – 1.5 3 – 8.2 1.1 11 (d) They reside in biodiversity hotspots that are most
III. (13) 11.3 0.9 0.48 2.4 1.4 4.2 0.8 8.4 2.2 4.1 susceptible to habitat degradation.
IV. (12) 3.2 10.2 11.1 4.8 0.4 3.3 0.8 7.3 11.3 2.1 70. Biosphere reserves differ from national parks and wildlife
sanctuaries because in the former
Which area out of I to IV shows maximum species (a) human beings are not allowed to enter.
diversity? (b) people are an integral part of the system.
(a) I (b) II (c) plants are paid greater attention than the animals.
(c) III (d) IV (d) living organisms are brought from all over the world
68. The greatest threat to global biodiversity is and preserved.
(a) natural disasters such as storms 71. Sacred groves are especially useful in
(b) pollution (a) preventing soil erosion.
(c) overexploitation of natural resources (b) year-round flow of water in rivers.
(d) alteration of habitats by humans (c) conserving rare and threatened species.
(d) generating environmental awareness.
EBD_7209
38

Chapter
Environmental Issues

8. Euro II norms stipulate that sulphur be controlled at


FACT/DEFINITION TYPE QUESTIONS
_______ ppm in diesel and _______ ppm in petrol.
1. Pollution causes undesirable changes in which the (a) 350; 150 (b) 150; 350
following characteristics of air, water, land or soil ? (c) 350; 250 (d) 150; 250
(a) Physical (b) Chemical 9. All automobiles and fuel -petrol and diesel were to have
(c) Biological (d) All of these met the Euro III emission specifications in some cities
2. To improve the quality of environment (air, water and soil) from 1 April 2005 and have to meet the Euro-IV norms by
the Govt. of India passed the ‘Environment (Protection) (a) 1 April 2008 (b) 1 April 2009
Act’ in year (c) 1 April 2010 (d) 1 April 2012
(a) 1981 (b) 1986
10. Which of the following minimum percentage of impurities
(c) 1987 (d) 1974
by domestic sewage make the water unfit for human use ?
3. Which one of the following is a most efficient device to
eliminate particulate matters from the industrial (a) 1 (b) 5
emissions ? (c) 0.1 (d) 0.5
(a) Cyclonic separators 11. The amount of organic matter in sewage water can be
(b) Trajectory separators estimated by
(c) Incineration (a) measuring oxygen demand
(d) Electrostatic precipitator (b) weight of micro-organisms
4. The scrubber is used mainly to remove which of the (c) salt analysis
following gas/es from the exhaust after spraying water/
(d) calorimetery
lime ?
12. Full form of B.O.D. is
(a) CO2 (b) SO2
(a) Biological organism death
(c) O2 and N2 (d) CO and CO2
(b) Biotic oxygen demand
5. The diameter of particulate matter that causes greatest
harm to human health is (c) Biochemical oxygen demand
(a) £ 2.5 µm (b) £ 0.25 µm (d) Biological organisation day
(c) £ 1.0 µm (d) £ 0.1µm 13. In B.O.D. test oxygen plays an important role to
6. Which of the two expensive metals are used as catalysts (a) destroy inorganic matter
by catalytic converters and are fitted into automobiles for (b) destroy pollution
reducing emission of poisonous gases ? (c) destroy waste organic matter
(a) Platinum - palladium, rhodium (d) none of these
(b) Cadmium, rhodium 14. Presence of large amount of nutrients in water also cause
(c) Copper, cadmium excess growth of planktonic (free-floating) algae, called
(d) Lead, mercury ___________.
7. Which of the following level of sound may damage ear (a) biomagnification
drum and can impair the hearing ability permanently ? (b) eutrophication
(a) 80 dB (b) 100 dB (c) algal bloom
(c) 120 dB (d) 150 dB (d) biofortification
Environmental Issues 317

15. Which of the following species of plant is considered as 25. ___________ is a cyclical zero-waste procedure, where
the world’s most problematic aquatic weed ? waste products from one process are cycled in as nutrients
(a) Lantana for other processes.
(b) Eichhornia (a) Eutrophication
(c) Parthenium (carrot grass) (b) Green house effect
(d) Brown algae (c) Integrated organic farming
16. The waste water from Industries may contain toxic heavy (d) Biomagnification
metals having density of more than
(a) 5 g / cm3 (b) 10 g /cm3 STATEMENT TYPE QUESTIONS
(c) 2 g /cm3 (d) 15 g /cm3
26. Read the following statements.
17. Concentr ation of DDT for first tr opic level
A lake near a village suffered heavy mortality of fishes
(phytoplanktons) and top trophic level (fish eating birds)
within a few days. Consider the following reasons for this;
is _____ respectively in aquatic food chains if DDT is
0.003 pb in water. (i) Lots of urea and phosphate fertilizer were used in
the crops in the vicinity.
(a) 0.025 ppm, 25 ppm (b) 0.003 ppm, 2 ppm
(ii) The area was sprayed with DDT by an aircraft.
(c) 0.5 ppm, 2 ppm (d) 0.04 ppm, 2 ppm
(iii) The lake water turned green and stinky.
18. DDT causes egg shell thinning in birds because it inhibits
(a) calmodulin (b) calcium ATPase (iv) Phytoplankton populations in the lake declined
(c) magnesium ATPase (d) carbonic anhydrase intially thereby greatly reducing photosynthesis.
19. Natural ageing of lake by biological enrichment of its water Which two of the above were the main causes of fish
is called mortality in the lake ?
(a) biomagnification (b) eutrophication (a) (i) and (iii) (b) (i) and (ii)
(c) biodegradation (d) water logging (c) (ii) and (iii) (d) (iii) and (iv)
20. FOAM (Friends of the Arcata Marsh) is a group of citizens 27. Pollution in big cities can be controlled to a large extent by
responsible for the integrated process of (i) improving traffic condition and road.
(a) reducing eutrophication (ii) road side plantation
(b) sewage and water treatment (iii) proper disposal of garbage and domestic as well as
(c) radio- active waste treatment municipal wastes.
(d) minimizing global warming (iv) cannot be controlled
21. The ‘Polyblend’ is a/ an (a) (i) and (ii) (b) (ii) and (iii)
(a) recycled modified plastic waste used for laying the (c) (i) and (iv) (d) all of these
roads. 28. Which of the following statements about eutrophication
(b) electronic waste buried in the landfills. are ?
(c) plastic film- waste to make disinfectants. (i) It can be a naturally occurring process.
(d) fine powder of plastic waste used to make the plastic (ii) It is commonly found in standing rather than running
bags. water.
22. Green house gases include (iii) It can lead to oxygen depletion.
(a) CO2, CFC, CH4 and (NO)x (iv) It is commonly associated with high levels of
(b) CO2, O2, N2, NO2 and NH3 phosphates and nitrates.
(c) CH4, N2, CO2 and NH3 (a) (iii) and (iv) (b) (i), (ii) and (iii)
(d) CFC, CO2, NH3 and N2
(c) (ii), (iii) and (iv) (d) all of these
23. Without Green house effect the average temperature of
29. Mark the statement that describes the eco-friendly
earth surface would have been
disposal of municipal solid- waste.
(a) 18 °C (b) 8°C
(a) It should be burnt to completion.
(c) – 8°C (d) –18°C
(b) It should be dumped in open waste land.
24. Slash and burn agriculture, which is commonly known as
(c) It should be dumped in sanitary landfills.
Jhum cultivation in the north-eastern states of India, leads to
(d) It should be sorted out into bio-degradable,
(a) deforestation (b) reforestation non- bio-degradable and recyclable wastes and
(c) desertification (d) water-logging treated separately.
EBD_7209
318 Biology
30. Which of the following is the correct statement ? (iii) To control the emission of ozone depleting
(a) Cultural eutrophication is an accelerated form of substances.
eutrophication. (iv) Biosafety of genetically modified organisms.
(b) In the presence of prime contaminants, such as (a) (i) and (iii) (b) (iii) only
nitrates and phosphates, the growth of algae is (c) (iii) and (iv) (d) (i) and (iii)
arrested. 37. Which of the following statement is incorrect?
(c) The water from electricity generating units enhances (a) Noise pollution does not leave any residue in the
the growth of indigenous fauna and flora. environment.
(d) All of the above (b) Noise pollution creates nervous disorders.
31. ‘EcoSan’ toilets, being used in Kerala and Sri Lanka are (c) Plants are efficient absorbers of noise of low
hygienic and cost effective solution to human waste frequency.
disposal. Mark the correct statement regarding EcoSan. (d) Loss of hearing is a common disorder .
(a) Recycled human waste from this can be used as a 38. Which of the following would be most likely to help to
natural fertilizer. slow down the greenhouse effect?
(b) They are called composting toilets. (a) Ensuring that all excess paper packaging is burned
(c) They do not depend on water for the disposal of to ashes.
excreta. (b) Promoting the use of private rather than public
(d) All of the above transport.
32. Mark the correct statement. (c) Converting tropical forests into grazing land for
(a) The ozone of stratosphere is good ozone. cattle.
(d) Re-designing land-fill dumps to allow methane to be
(b) Troposphere, on top of stratosphere, is away from
collected.
earth surface.
(c) The thickness of ozone is measured in dB. ASSERTION/REASON TYPE QUESTIONS
(d) The wavelength of UV- B is more than that of UV- A.
33. Which of the following statement (s) is/are correct about In the following questions, a statement of Assertion is followed
noise pollution? by a statement of Reason.
(a) If both Assertion and Reason are true and the Reason is
(a) It is an undesired high level of sound.
the correct explanation of the Assertion.
(b) It is measured in dB.
(b) If both Assertion and Reason are true but the Reason is
(c) It can cause damage to heart, increase blood
not the correct explanation of the Assertion.
cholesterol and even raise blood pressure etc.
(c) If Assertion is true but Reason is false.
(d) All of the above
(d) If both Assertion and Reason are false.
34. After the conventional sedimentation, filtering and
39. Assertion : Methane, component of green house gases,
chlorine, lots of dangerous pollutants still remain. To
contributing to global warming is about 20 percent.
combat this, the biologists developed a series of six
Reason : Introduction of multi-point fuel injection engines
connected marshes where appropriate plants, algae, fungi
in automobiles has decreased methane content in the
and bacteria were seeded into this area, which
exhausts.
(i) Neutralize the pollutants
40. Assertion : A suspended particulate matter (SPM) is an
(ii) Absorb the pollutants important pollutant released by diesel vehicles.
(iii) Assimilate the pollutants Reason : Catalytic converters greatly reduce pollution
(a) All of these (b) None of these caused by automobiles.
(c) only (i) and (iii) (d) only (i) and (ii) 41. Assertion : Presently, the global atmosphere is warming
35. Mark the correct statement. up.
(a) The major contribution in green house gases is of Reason : The depletion of stratospheric ozone layer has
CH4. resulted in increase in ultraviolet radiations reaching the
(b) Global warming is because of ozone depletion. earth.
(c) When organic waste enters into a water body its 42. Assertion : Water pollutants are measured by BOD.
BOD increases. Reason : If BOD is more, the water is polluted.
(d) All of the above 43. Assertion : Eutrophication shows increase in productivity
36. Which of the following is/are correct regarding Montreal in water.
Protocol? Reason : With increasing eutrophication, the diversity of
(i) Persistent organic pollutants. the phytoplankton increases.
(ii) Global warming and climate change.
Environmental Issues 319

(a) A – II; B – III; C – I; D – IV


MATCHING TYPE QUESTIONS
(b) A – III; B – II; C – I; D – IV
44. Match column-I with column-II and choose the correct (c) A – II; B – III; C – IV; D – I
option. (d) A – II; B – I; C – III; D – IV
Column-I Column-II 48. Match column-I with column-II and select the correct
A. DDT I. CH4, CO2 option.
B. Platinum- palladium II. SO2 Column-I Column-II
and Rhodium A. Ahmed Khan I. Spreading information
C. Acid rain III. Biological and help on the practice
magnification of integrated organic
D. Global warming IV. Catalytic converter farming
B. Ramesh Chandra II. Protecting wildlife
(a) A – IV; B – III; C – II; D – I
Dagar
(b) A – I; B – III; C – II; D – IV
C. Amrita Devi Bishnoi III. A plastic sack
(c) A – II; B – III; C – IV; D – I manufacturer of
(d) A – III; B – IV; C – II; D – I Bangalore developed
45. Match column-I with column-II and select the correct polyblend
option. (a) A – III; B – I; C – I (b) A – II; B – I; C – I
Column-I Column-II (c) A – I; B – II; C – II (d) A – III; B – II; C – II
A. Catalytic converter I. Particulate matter 49. Which one of the following pairs is mismatched?
B. Electrostatic II. Carbon monoxide and (a) Fossil fuel burning - release of CO2
precipitator nitrogen oxides (b) Nuclear power - radioactive wastes
C. Earmuffs III. High noise level (c) Solar energy - green house effect
D. Land fills IV. Solid wastes (d) Biomass burning - release of CO2
(a) A – I, B – II, C – III, D – IV 50. Match the items of column-I with column-II and choose
(b) A – II, B – I, C – III, D – IV the correct option.
(c) A – IV, B – III, C – II, D – I Column-I Column-II
A. UV I. Biomagnification
(d) A – III, B – II, C – IV, D – I
B. Biodegradable
46. Match column-I with column-II and select the correct
organic matter II. Eutrophication
option.
C. DDT III. Snow blindness
Column-I Column-II
D. Phosphates IV. BOD
A. Environment I. 1987 (a) A – II, B – I, C – IV, D – III
(Protection) Act (b) A – III, B – II, C – IV, D – I
B. Air (Prevention and II. 1986 (c) A – III, B – IV, C – I, D – II
Control of Pollution) Act (d) A – III, B – I, C – IV, D – II
C. Water (Prevention and III. 1980 51. Match column-I with column-II and choose the correct
Control of Pollution) Act option.
D. Concept of Joint IV. 1974 Column-I Column-II
Forest Management A. Colloidal materials I. Typhoid, Jaundice,
of Govt. of India Cholera
(a) A – IV; B – III; C – II; D – I B. Water-borne diseases II. Irreparable computes
(b) A – II; B – I; C – IV; D – III and other electronic
(c) A – I; B – II; C – III; D – IV goods
(d) A – I; B – III; C – II; D – IV C. E-wastes III. Faecal matter bacteria,
cloth and paper fibres
47. Match column-I with column-II and select the correct
D. Manure IV. Troposphere
option.
E. Bad ‘Ozone’ V. Cattle excreta (dung)
Column-I Column-II
(a) A – I, B – III, C – II, D – V, E – IV
(Organisms) (Concentration of DDT)
(b) A – III, B – I, C – V, D – II, E – IV
A. Zooplankton I. 2 ppm
(c) A – III, B – I, C – II, D – V, E – IV
B. Small fish II. 0.04 ppm
(d) A – III, B – I, C – V, D – IV, E – II
C. Large fish III. 0.5 ppm
D. Fish-eating birds IV. 25 ppm
EBD_7209
320 Biology

DIAGRAM TYPE QUESTIONS


CFCs
52. According to size of air pollutants, range and types of 6%
chemical the device given below is best used to control N2 O
which of the following pollutants? 14%

CO 2
(d) CH4
60%
20%

(a) Large particulates 54. The given graph shows the effect of sewage discharge
(b) Charged particulate matter on some important characteristics of a river. Which of the
(c) Dissolved gases following is the correct label for A, B and C ?
(d) Fine particles
53. Which of the following figures shows correct relative

Concentration
contribution of greenhouse gases to global warming?

N2O
6%
CFCs
14%
Sewage Direction of Flow
B
CO2 Discharge
(a) CH4
60% (a) (A) Dissolved oxygen, (B) Point of sewage discharge,
20%
(C) BOD
(b) (A) BOD, (B) Point of treated water discharge,
(C) Dissolved oxygen
(c) (A) Dissolved oxygen, (B) Point of treated water
discharge, (C) BOD
CO2
6% (d) (A) BOD, (B) Point of sewage discharge,
CFCs (C) Dissolved oxygen
14%
55. The diagram below shows the effect of polluting a river
(b)
N 2O with untreated whey. What does graph X represent?
CH4 60%
20%

N2O
6%
CH4
14% (a) Bacterial count.
(b) Number of fish.
(c) CO2
CFCs 60% (c) Mass of curds.
20%
(d) Concentration of rennet.
Environmental Issues 321

56. The given diagram shows electrostatic precipitator. 60. In clean water, the concentration of
Identify A, B and C. (a) BOD is low but DO is high.
(b) Both BOD and DO are high.
(c) BOD is high but DO is low.
(d) Both BOD and DO are low.
61. Water pollution can be stopped best by
(a) treating effluents to remove injurious chemicals.
(b) rearing more fishes.
(c) cultivating useful water plants.
(a) A - Discharge corona, B - Negatively charged wire,
(d) spraying with DDT.
C - Collection plate grounded
62. Fishes die by sewage because
(b) A - Discharge corona, B - Positively charged wire, (a) of its bad smell.
C - Collection plate grounded (b) it replaces food material of fishes.
(c) A - Discharge corona, B - Negatively charged wire, (c) it increases oxygen competition among fishes.
C - Collection plate burnt (d) CO2 is mixed in large amounts in water.
(d) A - Uncharge corona, B - Positively charged wire, 63. A lake affected by high levels of artificial eutrophication
C - Collection plate never grounded will have
57. What does ‘x’ indicate in the given figure? (a) high nutrient levels, large phytoplankton
populations, and low oxygen levels at depth.
(b) high levels of nutrients, low phytoplankton levels,
high oxygen levels in surface waters.
(c) low nutrient levels, large phytoplankton populations,
X and low oxygen levels at depth.
(d) low nutrient levels, low phytoplankton populations,
and high oxygen levels at depth.
(a) Greenhouse effect (b) El Nino Effect 64. Two lakes, A and B are identical in all aspects except that
(c) Ozone hole (d) Marsh meadow stage lake A has higher temperature. Which of the following is
58. Which of the following phenomenon is represented by true ?
the given figure? (a) A has higher rate of oxygen dissolution.
(b) B has higher rate of oxygen dissolution.
(c) Oxygen dissolution of both is the same.
Sunlight
(d) Both the lakes have same BOD.
Green H
ouse
65. Today the concentration of green house gases is very
Gases
high because of
e
her (a) use of refrigerator.
osp
Atm (b) increased combustion of oils and coal.
Radiant heat (b) deforestation.
Trapped by
GHGs Earth Surface (d) all of the above
66. Which constituent of the atmosphere is likely to change
if the forest cover is removed ?
(a) O2 level is increased
(b) CO2 level is increased
(a) Green house effect (b) El Nino effect (c) O2 level is significantly increased
(c) Ozone hole (d) Eutrophication (d) CO2 level is significantly decreased
67. If the forest cover is reduced to half, what is most likely to
happen on a long term basis?
CRITICAL THINKING TYPE QUESTIONS (a) Tribals living in these areas will starve to death.
(b) Cattle in these and adjoining areas will die due to
59. CNG is better than petrol/ diesel, since lack of fodder.
(a) CNG burns more efficiently. (c) Large areas will become deserts.
(b) CNG burns completely. (d) Crop breeding programmes will suffer due to a
(c) CNG cannot be adulterated. reduced availability of variety of germplasm.
(d) All of the above
EBD_7209
322 Biology
68. Which of these is not an advantage of CNG over diesel? 76. Which of the following is not one of the prime health
(a) Burns more efficiently. risks associated with greater UV radiation due to depletion
(b) It is cheap. of stratospheric ozone?
(c) Cannot be siphoned off by thieves. (a) Increased liver cancer
(d) Easy to lay down pipelines for delivery. (b) Increased skin cancer
69. Motor vehicles equipped with catalytic converter are (c) Damage to eyes
advised to use unleaded petrol because (d) Reduced immune system
(a) lead causes inactivation of catalyst. 77. As a result of global warming, the sea level will
(b) lead reduces the emission of poisonous gases. (a) increase (b) decrease
(c) lead is a heavy metal. (c) remain the same (d) none of these
(d) lead decreases the efficiency of vehicle. 78. The most adverse effect of radioactive pollutant is
70. Which of the following steps is not taken for reducing (a) gene mutation (b) hepatitis
vehicular pollution? (c) polio (d) T.B.
(a) Use of unleaded petrol. 79. Drawback of DDT as pesticide is that
(b) Use of high-sulphur petrol and diesel. (a) it becomes ineffective after sometime.
(c) Use of catalytic converters in vehicles. (b) it is less effective than others.
(d) Application of stringent pollution-level norms for (c) it is not easily/rapidly degraded in nature.
vehicles. (d) its high cost.
71. Biomagnification refers to increase in concentration of 80. Global warming can be controlled by
the toxicant at successive trophic levels. This happens (a) reducing reforestation, increasing the use of fossil
because fuel.
(a) a toxic substance accumulated by an organism can (b) increasing deforestation, slowing down the growth
not be metabolized. of human population.
(b) a toxic substance accumulated by an organism can
(c) increasing deforestation, reducing efficiency of
not be excreted.
energy usage.
(c) a toxic substance is passed on to the next trophic
level. (d) reducing deforestation, cutting down use of fossil
fuel
(d) All of the above
72. Which of the following statements is/are correct about 81. Climate of the world is threatened by
‘Eco San’ toilets? (a) increasing concentration of atmospheric oxygen.
(a) It is a sustainable system for handling human excreta (b) decreasing amount of atmospheric oxygen.
or faecal matter by using dry ‘composting toilets’. (c) increasing amount of atmospheric carbondioxide.
(b) These are very useful for the rural areas where sewer (d) decreasing amount of atmospheric carbondioxide.
systems are not possible. 82. Which one of the following is not correct as regards to
(c) These toilets are hygienic, efficient, practical & most the harmful effects of particulate matter of the size 2.5
effective for the disposal of human waste. micro meters or less?
(d) All of the above (a) It can be inhaled into the lungs.
73. Which one of the following statement pertaining to (b) It can cause respiratory problems.
pollutants is correct?
(c) It can directly enter into our circulatory system.
(a) DDT is a non-biodegradable pollutant.
(d) It can cause inflammation and damage to the lungs.
(b) Excess fluoride in drinking water causes
osteoporosis, hardening of bones, stiff joints. 83. Algal blooms impart a distinct colour to water due to
(c) Excess cadmium in drinking water causes black foot (a) their pigments.
disease. (b) excretion of coloured substances.
(d) Methylmercury in water may cause "Itai Itai" disease. (c) formation of coloured chemicals in water facilitated
74. The effect of todays radioactive fall out will be harmful to by physiological degradation of algae.
children of future generation because
(a) infants are more susceptible to radiations. (d) absorption of light by algal cell wall.
(b) susceptibility to radiation increase with age. 84. In the coming years, skin related disorders will be more
(c) mutated genes are frequently recessive. common due to
(d) contamination of milk supply is not cumulative. (a) air pollution
75. Which of the following is not an environmental problem? (b) use of detergents
(a) Soil erosion (b) Water logging (c) water pollution
(d) depletion of ozone layer
(c) Desertification (d) Reforestation
Environmental Issues 323

85. Vast tracts of rainforest have been clear-cut and burned. 88. The presence of ozone in the atmosphere of earth
This practice has increased atmospheric carbon dioxide (a) is advantageous since it supplies O2 for people
levels significantly because travelling in jets.
(a) forest provided a place for the carbon dioxide to be (b) helps in checking the penetration of ultraviolet rays
used. to earth.
(b) burning of the forest released a large amount of (c) hinder higher rate of photosynthesis.
carbon dioxide into the atmosphere.
(d) has been responsible for increasing the average
(c) grasslands that replaced the forests can’t utilize as global temperature in recent years.
much carbon dioxide.
89. Nuclear power stations even with adequate radiation
(d) all of the above safety measures generate
86. In an aquatic ecosystem, maximum biomagnification is (a) thermal pollution of water bodies.
seen among
(b) thermal pollution of soil.
(a) fishes (b) phytoplanktons
(c) noise polllution.
(c) microscopic plant (d) zooplanktons
(d) all of the above
87. Soil fertility can be destroyed by
(a) cutting down forests
(b) acid rain
(c) overgrazing and over-irrigation
(d) all of the above
EBD_7209
Hints
&
Solutions
S-2 Biology
EBD_7209
Hints & Solutions S-3

Chapter 1 : The Living World Simultaneous Catabolism and Anabolism called as


Amphibolism.
1. (d) All living organisms share certain unified and basic 10. (c) The term biodiversity is used for the variety and
characteristics (includin energy utilization, regulation variability among all forms of living organisms like plants,
or homeostasis, growth, development, reproduction, animals, and micro-organisms present in a given region
adaptation) metabolism and interaction. under natural conditions. Biodiversity can be defined
2. (a) All cells arise from pre-existing cells by a process of as the totality of genes, species and ecosystem of a
cell division. Cell division is the phenomenon of region. India is very rich in biodiversity.
production of daughter cell from parent cell. It occurs 11. (a) ICBN (International Code of Botanical Nomenclature)
continuously in plants and only up to a certain age is one of the code of nomenclature which is
in animals. The continuous growth in plants termed independent of zoological and bacteriological
“unique”, while in animals, cells divide up to a certain nomenclature. The foundations of ICBN was found
stage. in Philosophia Botanica, a book written by C.
3. (c) In unicellular organisms, both Growth and Linnaeus.
Reproduction are inclusive events as unicellular 12. (b) Binomial nomenclature means that the scientific name
organisms simply grow by cell division, in which of any organism consist of a generic epithet and a
their population size also increases. Both the features specific epithet. Binomial nomenclature was
are exclusive in higher organisms. developed by Linnaeus.
4. (c) Growth and reproduction are mutually exclusive 13. (c) In printed scientific names, only the genus is
events in majority of the higher animals and plants. capitalized. Genus is an assembly of related species
Growth may be defined as a positive change in size, which evolved from a common ancestor and have
often over a period of time. Reproduction leads to certain common characters. Eg, Solanum tuberosum
production of progenies, possesing features more and Solanum melongena are two species which
or less similar to those of parents. belongs to the same genus of Solanum.
5. (a) Non-living objects have characteristic growth called 14. (d) Taxonomic hierarchy is the sequence of
extrinsic growth in which object grows after arrangements of taxonomic categories in a
accumulating substance over its surface. Living descending order during the classification of
organisms show intrinsic growth (i.e., grows from organisms. Each category of taxonomic hierarchy
inside). refers to as a unit of classification.
6. (b) Reproduction ensures the continuity of the species, 15. (c) Systematics, often used interchangeably with
generation after generation. Genetic variations are taxonomy, is the study of diversity of organisms,
created and inherited during reproduction. “Like their comparative and evolutionary relationships on
begets like” is the phrase to explain the similarity the basis of findings from various fields of biology.
between parents and offsprings. 16. (d) Taxon (Plural: taxa) is a grouping of organisms of
7. (d) Fragmentation is a sexual mode of reproduction in any level in hierarchial classification which is based
which an organism is split into fragments. Each of on some common characteristics. It represents real
these fragments develops into mature, fully grown biological objects placed in any category while
individuals that are clones of the original organism. category itself is an abstract term.
The fungi, the filamentous algae and the protonema 17. (d) Biological classification is the scientific arrangement
of mosses all easily multiply by fragmentation. of organisms in a hierarchial series of groups and
8. (c) Photoperiodism is the physiological reaction of subgroups on the basis of similarities and differences
organisms to the length of day or night. It occurs in in their traits. It helps in building evolutionary
plants and animals. It can also be defined as the pathways and in identifying new organisms.
developmental responses of plants to the relative 18. (d) The characteristics of life include: responsiveness
lengths of light and dark periods. Photoperiod to the environment; growth and change; ability to
exclusively affects the reproduction in seasonal reproduce; have a metabolism and breathe; maintain
breeders, both plants and animals. homeostasis; being made of cells; passing traits onto
9. (a) Catabolism – breakdown of substances eg., offspring. Differentiation from cells to tissues is not
Respiration a characteristic of life.
Anabolism – Formation of substances. eg., 19. (c) A family is a subdivision of an order consisting of a
Photosynthesis. group of related genera which in turn are composed
Catabolism + Anabolism = Metabolism. of groups of related species. Families are
These are defining features of all life forms. characterized on the basis of vegetative and
S-4 Biology

reproductive parts of the plants species. Suffix added nature. There are separate taxonomic keys specific for
in families of both plants and animals may be –aceae each taxonomic category such as Family, Genus, etc.
or –ae. 29. (c) Growth is the act or process, or a manner of growing;
20. (a) The order generally ends with ales. Order being a development; gradual increase. It is an exclusive
higher category is the assemblage of families which event in majority of the higher animals and plants. In
exhibit a few similar characters. plants, growth occurs continuously throughout their
21. (b) Manuals contain information for the identification life span and in animal, growth is seen only up to a
of names of species found in an area. They also certain age. In living organisms, growth is from
provide information about keys, description of family, inside. Therefore, it cannot be taken as a defining
genus and species. property of living organisms.
22. (d) Taxonomic keys are aids for rapid identification of 30. (d) Accumulation of material by external agency cause
unknown plants. Flora is an inventory of the plants extrinsic growth which can not be the feature of living
of a defined geographical region. Herbarium is a safe organism.
place for storing specimens as well as provide 31. (b) Biological museums have the collection of preserved
suitable atmosphere for research. Monograph is a animals and plants specimens for study and
comprehensive taxonomy treatment of a taxonomic references. Biological museums are generally set up
group, generally a genus or a family, providing all in educational institutes, i.e. schools and colleges.
taxnomic data relating to the group. The specimens may be preserved dry or in
23. (b) Key is a taxonomical aid used for identification of preservative solution in jars or containers. The
plants and animals based on the similarities and insects are preserved in insect boxes after collecting,
dissimilarities. The keys are based on set of killing and pinning. The larger animals, like mammals
contrasting characters, generally in a pair called and birds are stuffed and are chemically treated for
couplet. Each character of a couplet called lead. Keys long term preservation. Museums also have
are generally analytical in nature. collection of animal skeletons.
24. (c) Herbarium is the store house of dead, dried, pressed 32. (d) Increase in mass and increase in number of individuals
and preserved plant specimens on paper sheets, are twin characteristics of growth. Growth is defined
called herbarium sheets. The sheets, along with as increase in size and mass during the development
description of plant specimen, are arranged according of an organism over a period of time. It is measured as
to standard system of classification, and are stored an increase in biomass and is associated with cell division
for future use. All sheets carry labels having by mitosis, subsequent increases in cell size, and with
information about date and place of collection, the differentiation of cells to perform particular functions.
English, local and botanical names, family and 33. (d) All the statements regarding nomenclature are
collector’s name etc. The herbarium sheets can be correct. Nomenclature is giving distinct scientific
used as a quick reference for taxonomic studies. names to various structures including living
25. (a) The famous botanical garden 'Kew' is located in organisms for their identification. It is a set of rules
England. Royal Botanic Garden, Kew is a non- used for forming the names or terms in a particular
departmental public body in the United Kingdom field of arts or sciences. Nomenclature is only
sponsored by the Department for Environment, Food possible when the organism is described correctly
and Rural Affairs. It is an internationally important and we know to what organisms the name is attached
botanical research and education institution, it to (called identification).
employs 750 staff. 34. (d) Herbarium is a collection of plant parts that usually
26. (c) Key is a type of taxonomical aid used for the have been dried, pressed, preserved on sheets. The
identification of plants and animals based on their herbarium sheets also carry a label that provides
similarities and dissimilarities. Keys are generally information about date and place of collection.
analytical in nature. Herbarium serves the quick referral systems for
27. (a) Each statement in the key is called a lead. Catalogue taxonomical studies. It can also termed as Dry
is a list or record, systematically arranged and often Garden.
including descriptive material. 35. (a) In plants, growth by cell division occurs continuously
28. (b) The keys are based on contrasting characters throughout their life span.This continuos growth in
generally in a pair called couplet. Key is an important plant is axial (i.e., takes place on two axes) and
taxonomic aid used for identification of plants. Each unique. Plant growth consists of primary and
character of the couplet or statement in the key is secondary growth, on the basis of time when it occurs.
called as “lead”. Keys are generally analytical in
EBD_7209
Hints & Solutions S-5

36. (b) Metabolism is defined as the sum total of all the taxonomy. It is the group of individual organisms
chemical reactions occurring in our body. All plants, with fundamental similarities.
animals, fungi and microbes exhibit metabolism. 44. (c) Three different genera such as Solanum, Datura and
Isolated metabolic reactions in vitro are not living Petunia are placed in the family solanaceae. In case
things but are nonliving reactions. of plants, classes with a few similar characters are
37. (d) Biodiversity is the term used to describe the variety assigned to a higher category called division. Phylum
of life found on Earth and all of the natural processes. is used in case of animals.
This includes ecosystem, genetic and cultural 45. (b) All the given descriptions are related to museum.
diversity, and the connections between these and Museum is one of the taxonomic aids which may be
all species. The different aspects of biodiversity all defined as a place where large collection of specimens
have a very strong influence on each other. of plants and animals are preserved for its further
38. (d) Order being higher category is the assemblage of study and references. Museums are generally set up
families which exhibit a few similar characteristic. in educational institutes such as schools and
Dog (Canis familaris) and Cat (Felis domesticus) colleges. Museums often have collections of
belong to two different families—Cancideae and skeletons of animals too.
Felidae respectively. 46. (c) Growth may be defined as a positive change in size,
39. (c) The term ‘taxon’ is used to refer to any rank or level often over a period of time. It can occur as a stage of
or category of the classification. Based on certain maturation or a process toward fullness or fulfillment.
observable characteristics like insects represent a Growth by cell division occurs continuously
group of organisms which share some common throughout their life span in plants.
features like three pairs of jointed legs hence insects 47. (a) Taxonomic studies of all known organisms have led
can be easily recognized and classified. to the development of common categories like
40. (b) Reproduction refers to the production of progeny kingdom, phylum or division, class, order, family,
possessing features more or less similar to those of genus and species. Higher the category, greater is
parents. The fungi, the filamentous algae, the the difficulty of determining the relationship to other
protonema of mosses, all easily multiply by taxa at the same level.
fragmentation. 48. (b) Genus comprises a group of related species which
41. (d) All the given statements are correct. All living has more characters in common in comparison to
organisms have the ability to respond the species of other genera. Organisms placed in the
environment stimuli which could be physical, same genus are most closely related.
chemical or biological. Plant responds to external 49. (a) Statement (i) and (ii) are correct regarding botanical
factors like light, water, temperature etc. Photoperiod garden. A botanical garden (or botanic garden) is a
is defined as the developmental responses of plants garden dedicated to the collection, cultivation and
to the relative lengths of light and dark periods. It display of a wide range of plants labeled with their
exclusively affects the reproduction in seasonal botanical names. Botanic garden h olds th e
breeders, both plants and animals. Human being is documented collections of living plants for the
the only organism who has self - consciousness. purposes of scientific research, conservation, display
42. (b) Nomenclature is the providing of standardize names and education.
to the organisms such that a particular organism Statement (iii), (iv) and (v) are related to herbarium.
known by the same name all over the world. 50. (a) Herbarium is a storehouse of collected plant
Identification is the correct descripton of an organism specimens that are dried, pressed and preserved on
and its recognition in its scientific name. The branch paper sheets.
of science which deals with the study of principle
æ 1 1 2 ö
and procedure of classification known as taxonomy. Herbarium sheet is 41 ´ 39 cm2 çè 16 ´ 11 inches ÷ø .
2 2
It is of two types.
(i) Classical taxonomy – Based on observable It serves two primary functions–
morphological characters. (1) Accurate identification
(ii) Modern taxonomy – Based on external and (2) Alpha taxonomic research. (based on gross
internal features of organisms. morphology)
43. (a) All the given descriptions are related to taxonomic 51. (c) Fungi shows Fragmentation, budding, etc but not
category called species. Species, the lowest category regeneration while Planaria truely regenerate during
in the taxonomic hierarchy, is the basic unit of its life cycle.
S-6 Biology

Mossess shows fragmentation for propagation of 61. (c) International code for Botanical Nomenclature
their progenies. Thallus tip, protonema fragments (ICBN). Standardize all scientific names for plants.
and even rhizoids can grow into new moss thallus. ICBN came into existance in 1961. The naming system
52. (b) Division is a category higher than that of class. Term uses three words (Trinomial system) was proposed
phylum is used for animals while division is used for by Lamarck. Trinominal system of nomenclature
plants. Classes with few similar characteristics are followed by Polynomial system of nomenclature.
assigned to division. eg., Corvus splendens splendens (Indian crow)
53. (c) Herbarium and Botanical Gardens only have Gorilla gorilla gorilla (Gorilla)
preserved specimens. Rest all taxonomic aids have 62. (a) Species is a natural population of individuals which
living organisms or their listed descriptions. resemble one another in all essential morphological
54. (a) A category is a rank or level in the hierarchial and reproductive characters so that they are able to
classification of organisms. Taxon is a unit in interbreed freely in nature to produce fertile offspring.
classification which may represent any level of Two different taxonomic species essentially have
grouping of organisms based on certain common different characteristics.
characteristics. There is some confusion in the use 63. (b) A - IV; B - III; C - II; D - I
of taxon and category, for example Bryophyta is a 64. (a) A - IV; B - III; C - II; D - I
taxon while division is a category. 65. (c) A - II; B - IV; C - I; D - III
55. (b) The species is genetically distinct and reproductively 66. (b) A - IV; B - III; C - I; D - II
isolated natural population. Sexual reproduction is 67. (b) A - V; B - IV; C - II; D - I; E - III
absent in prokaryotes and some protists. In such 68. (a) Tiger and tigris both are from same genus with
cases morphological differences, cytotaxonomy and particular taxonomic category.
chemo-taxonomy are resorted too. 69. (c) Housefly, Musca domestica, is a fly of the order
56. (d) Herbarium sheet carry a label providing information Diptera. It is the most common of all domestic flies.
about date and place of collection, English local and Primate is a mammal of the order primata.
botanical names, family, collectors name etc. Families 70. (c) Cats belong to the family Felidae.
are characterized on the basis of both vegetative 71. (c) A – III, B – I, C – IV, D – II. Growth may be defined as
and reproductive features of plant species. a positive change in size, often over a period of time.
57. (b) Flora contains the actual account of habitat and Reproduction is the process by which living
distribution of plants of a given area. It provides the organisms produce young on of their own type.
index to the plant species found in a particular area. Metabolism is defined as the sum total of all the
58. (a) Panthera is a polytypic Genus and it contain several chemical reactions occurring in our body. Cellular
species like Panthera leo, Panthera tigris and organization refers to the components of a cell and
Panthera pardus. These species have several how these individual parts are arranged within the cell.
common features. Polytypic Genera have multiple 72. (c) A – II, B – I, C – III, D – V, E – IV. Taxon is a taxonomic
species while monotypic genera have one species group of any rank. Species, the lowest category in
only. the taxonomic hierarchy, is the basic unit of taxonomy.
59. (c) Biological concept of species is based on It is the group of individual organisms with
reproductive isolation. The most accepted concept fundamental similarities. Division is similar to phylum
of species proposed by Ernst Mayr. It is called and used in case of plants. Genus is a group of related
Biological concept of species. According to this species having more character in common with others
concept, species. Order is identified based on a number of
A species is (1) Closely related similar characters.
(2) Share common gene pool. 73. (c) A – II, B – I, C – III, D – V, E – IV.
(3) Interbreeding. 74. (d) A-IV, B-I, C-II, D-III
– exception of this concept are – Mule, Hinny, Tigon, 75. (b) There are 7 obligate hierarchial levels of taxonomic
Liger categories. These started from species, which is the
60. (c) Botanical Gardens provide ex-situ conservation unit/basis of taxonomical studies. It constitute the
strategy as it reside large variety of plants collected group of interbreeding organisms. This is natural
from various places, To enhance study, Research unit of taxonomical group. Group of families with
and Recreation, Botanical gardens plays important more or less similar characteristics called order. The
role. National Botanical Garden present at Lucknow, upppermost category in which Divisions/Phylum
while Indian botanical Garden situated at Howrah. present called Kingdom. It form the uppermost
category of hierarchial system in taxonomical studies.
EBD_7209
Hints & Solutions S-7

76. (d) Species is the lowest category in basic taxonomic 84. (b) Families are characterized on the basis of both
hierarchy and has the maximum common vegetative and reproductive features of plant
characterstics with other species under the same species. Tomato (Lycopersicon esculentum) and
genus. The genus is an aggregate or a group of potato (Solanum tuberosum) belongs to the same
closely related species. Family is the group of closely family solanaceae.
related genera, and has less common characterstics 85. (c) Identification is determining the correct place in a
than species or genus rank. system of classification and finding out the correct
Order is a higher taxon and is the assemblage of name of the organism. It is carried out with the help
families having similar characterstics. of keys. It not only assigns the organism to a
Class is a group of related orders. particular group, locate its correct name but also
Phylum: The classes with similar features are grouped provide information if the organism is new to
into phylum in animals and division in plants. systematics and requires giving a new name.
The phyla are grouped into still broader categories, 86. (b) Species is the lowest or basic taxonomic category
called kingdom. which consists of one or more natural populations
77. (b) Two plants can be conclusively said to belong to of individuals that resemble one another more closely
the same species if they can reproduce freely with each than individuals of other species, interbreed freely,
other and form seeds. The formed seed must be viable. have a distinct genetic set up and are reproductively
Two animals can be regarded as species when they isolated from others e.g., Panthera Leo (Lion),
can interbred each other and form fertile progency. Mangifera indica (Mango), Solanum tuberosum
78. (c) Taxa is a plural form of taxon. Taxon is a grouping of (Potato).
organisms of any level in hierarchial classification 87. (c) Classification is the process by which anything
which is based on some common characteristics. It grouped into convenient categories based on some
represents real biological objects placed in any easily observable characters. In angiosperms
category while category itself is an abstract term. characters of flowers are used in classification
79. (a) Taxonomical hierarchy (introduced by Linneaus) is because characters of angiospermic flowers are
arrangement of various taxonomic levels in the preserved for its further studies and references. In
descending order, starting from kingdom upto any organism, reproductive character (i.e., Floral
species. The hierarchy indicates the various levels characters) are more stable and less susceptible to
of kinship. The number of similar characters of enviromental influence than vegetative characters.
categories decreases from lowest rank to highest rank 88. (a) All the members of a taxonomic category possess
i.e., from species to kingdom. Thre are 7 obligate some similar characters which are different from
categories which constitute taxonomical hierarchy. those of others. The placement of individuals or
In addition to these obligate categories there are organisms in species, genus, family, order, class and
some optional categories are as follows. e.g., Tribe, phylum are determined by their specific similar
subclass, superclass, etc. characters and relationships. Maximum similarity
80. (a) Class is a group of related orders. The lizards, birds occurs in species which is also the lowest category
and cattle belong to class reptilia, aves and mammals in the hierarchy of categories. Similarity of characters
respectively. decreases with the ascent in hierarchy.
81. (c) Ex-situ conservation means off-site conservation. It 89. (c) Genus is the first higher category above the level of
is the process of protecting an endangered species species. A genus may have one to several species.
of plant or animal by removing it from an unsafe or A genus having a single species is called monotypic.
threatened habitat and placing it or part of it under Currently, the genus Homo is monotypic with a single
the care of man. Botanical garden is one of the species of Homo sapiens. A genus having two or
method of ex-situ conservation. more species is called polytypic. The genus Panthera
82. (d) The disadvantage of using common name for species is polytypic. Some species of this genus are P. leo
is that the names may change, one name does not (Lion), P. tigris (Tiger), P. onca (Jaguar) and P.
apply universally, one species may have several pardus (Leopard).
common names and one common name may be 90. (c) Phylum is a taxonomic category higher than class
applied to two species. and lower in rank to kingdom. The term phylum
83. (b) Replication of the genetic information causes transfer (coined by George Cuvier) is used for animals while
of genetic information from one generation to the the term division (coined by Eichler) is used for
next. It is very important feature of all living plants.
organisms.
S-8 Biology
91. (a) Species is the lowest taxonomic category. Class is a 5. (a) Mycoplasma are the simplest or smallest known
category made of one or more related orders organisms that completely lack a cell wall and can
possessing similar correlated characters. Family is survive without oxygen.
composed of one to many related genera. Division 6. (d) In protists, reproduction takes place by means of
comprises of several related classes. asexual and sexual methods. Asexual reproduction
92. (d) Order is a higher taxon and is the assemblage of occurs through binary fission (e.g. Euglena), cyst
families having similar characteristics. However, the formation (e.g., Entamoeba), budding, sporulation,
common characteristic will be fewer than at family or etc. In sexual reproduction, fertilization occurs
genus level. In mammals the common orders are through syngamy and conjugation.
primates (monkey, gorilla and human), carnivora, 7. (c) Chrysophytes are a group of diatoms, golden algae
rodentia and cetacea (whale and dolphin). (desmids) an d golden brown photosynthetic
93. (c) Genus is an assembly of related species which microscopic protists. Their body is covered by a
evolved from a common ancestor and have certain transparent siliceous shell.
common characters. In the botanical name of wheat 8. (d) Amoeboid protozoans are organisms that live in
(Triticum aestivum), the first word Triticum denotes fresh water, sea water or moist soil. They move and
the genus which starts with a capital letter. capture their prey by putting out pseudopodia (e.g.
94. (c) Species is the lowest category in taxonomic hierarchy. Amoeba). Some of them such as Entamoeba are
Species is a group of animals, plants or other living parasites.
things that all share common characteristics and that 9. (a) Albugo is the parasitic fungi on mustard and belongs
are all classified as alike in some manner. to the class phycomycetes and kingdom fungi.
95. (a) Potato, tomato and brinjal are three different species 10. (c) Neurospora belongs to class ascomycetes and
but all belong to the genus Solanum. Genus is an kingdom Fungi. It is used extensively in biochemical
assembly of related species which evolved from a and genetic work.
common ancestor and have certain common 11. (d) Deuteromycetes is also known as fungi imperfecti
characters. Solanum is a polytypic genera with more because the perfect (sexual) stage is either absent or
than one species. not reported. Alternaria, Colletotrichum and
96. (a) Order being a higher category is the assemblage of Trichoderma belong to deuteromycetes.
families which exhibit a few similar characters. The 12. (d) Deuteromycetes is commonly known as fungi
similar characters are less in number as compared to imperfecti due to absence of perfect sexual stage.
different genera included in a family. They are classed by the colour and structure of the
97. (d) Couplet represents the choice made between two conidia. Since most of the conidia structures look
opposite options which results in acceptance of only like ascomycetes type, they are believed to be
one and rejection of the other. derived from ascos which does not have the ability
to reproduce sexually.
Chapter 2 : Biological Classification
13. (a) Bladderwort and venus fly trap are partially
1. (d) R.H. Whittaker (1969) proposed a five kingdom antotrophs. They trap insects for obtaining mtrogen
classification. The main criteria for classification used and are called insectivorous plants e.g. itcher plant.
by him include cell structure, thallus organisation, 14. (d) The capsomere is a subunit of the capsid, an outer
mode of nutrition, reproduction and phylogenetic covering of protein that protects the genetic material
relationships. of a virus. Capsomeres self-assemble to form the
2. (c) Monera is the group of prokaryotes. They are capsid.
basically unicellular, may be mycelial, colonial and 15. (d) Cholera is a bacterial disease caused by the pathogen,
filamentous. They do not contain any organized Vibrio cholerae.
nucleus with distinct membrane. 16. (a) The symbiotic association of fungi and algae is called
3. (a) Haeckel created the kingdom Protista to include all Lichen. In lichen, there are two components i.e., algal
unicellular eukaryotic micro-organisms. They have partner called phycobiont and fungal partner called
a typical eukaryotic structure with membrane bound mycobiont.
organelles and nucleus. 17. (d) Viruses are obligate intracellular parasite which can
4. (a) The cyanobacteria or blue-green algae are the largest reproduce only by invading and taking over other
and most diverse group of photosynthetic bacteria. cells as they lack the cellular machinary for self
They have chlorophyll ‘a’ similar to green plants. reproduction. Viruses have either DNA or RNA as
They are the first organisms to make atmosphere the genetic material. Viruses having RNA as the
acrobic. genetic material are known as Retroviruses.
EBD_7209
Hints & Solutions S-9

18. (a) In fungi (ascomycetes an d basidiomycetes), 28. (a) Tobacco mosaic virus (TMV) is a single stranded
kayrogamy is delayed and occurs just before meiosis. RNA virus that infects a wide range of plants,
In the stage intervening between plasmogamy and especially tobacco and other members of the family
karyogamy the cells often contain two nuclei or Solanacea.
dikaryons (n + n). Such cells are called dikaryotic 29. (d) Bacteria are microscopic organisms placed under
cells. The phase is known as dikaryophase. kingdom monera whose single cells have neither a
19. (a) In many members of basidiomycetes, cell division is membrane-enclosed nucleus nor other membrane-
accompanied by clamp connection. These are bridge enclosed organelles like mitochondria and
(hook) like connections. They function as bypass chloroplasts.
hyphae through which nuclei migrate to make all of 30. (c) Chrysophytes belong to the kingdom protista.
mycelium dikaryotic. Examples are diatoms and golden algae. Their walls
20. (b) Plasmogamy is the first stage of sexual reproduction are embedded with silica and thus the walls are
in which the cytoplasm of two sex cells fuse with indestructible.
each other. The nuclei of sex cells come close to 31. (c) Dinoflagellates are mostly marine and photosynthetic
each other but do not fuse. Thus, the resulting cell organism. They appear yellow, green, brown, blue
becomes binucleate or dikaryon. or red depending on the main pigments present in
21. (a) Aristotle was the earliest scientist to attempt a more their cells. Most of them have two flagella; one lies
scientific basis of classification. He classified plants longitudinally and the other transversely in a furrow
into trees, shrubs and herbs on the basis of simple between the cell plates.
morphological characters and animals into two groups 32. (c) Mycoplasma refers to a genus of bacteria that lack
based on absence of presence of red blood cells. cell wall around their cell membrane. Without a cell
22. (a) Yeast being a unicellular fungus does not show wall, they are unaffected by many common antibiotics
filamentous nature. It is a microscopic fungus such as penicillin or other beta-lactam antibiotics
consisting of a single oval cell that reproduces by that target cell wall synthesis. They can be parasitic
budding. or saprotrophic.Mycoplasma are the facultative
23. (d) Chemosynthetic autotrophic bacteria use the energy anaerobes, they grow best where there is little or no
obtained by the oxidation of chemicals for the oxygen.
synthesis of food. They play a great role in recycling 33. (b) Sac fungi is a common name of ascomycetes which
of nutrients like nitrogen, phosphorous, iron and produce spores in a distinctive type of microscopic
sulphur. sporangium called an ascus. They are characterized
24. (c) Virus is a small infectious agent that replicates only by well-developed thallus and production of
inside the living cells of other organisms. Viruses ascospores. They are the largest class of fungi.
can infect all types of life forms, from animals and Examples of sac fungi are yeasts, morels, truffles,
plants to microorganisms, including bacteria and and Penicillium.
archaea. Viruses can pass through bacterial proof 34. (b) Viroids cause potato spindle tuber disease (PSTV),
filters as they are smaller than bacteria. chrysanthemum stunt, citrus exocortis, cucumber
25. (b) Archaebacteria are found in most harsh habitats. pale fruit etc. They cause persistent infections, i.e.,
Their cell wall is adapted to tolerate extreme never recovered.
conditions (wall contains protein and non-cellulosic 35. (d) Kingdom animalia includes all heterotrophic,
polysaccharides). Cell membranes are characterized multicellular complex eukaryotes. They depend either
by the presence of branched chain lipids that make upon plants or on other animals for their food
them highly resistent to heat and low pH. requirements. Kingdom monera includes prokaryotic
26. (c) Methanogens are archaebacteria that live in marshy organisms lacking membrane bound nucleus. Hence,
area produce methane as a metabolic byproduct in nuclear membrane cannot be found in Monera.
anoxic conditions. Methane is not their preferred Protista is a eukaryotic kingdom.
carbon source. 36. (c) Dinoflagellates are mostly single-celled organisms
27. (c) Cyanobacterial cells (blue-green algal cells) are larger classified in the kingdom protista. Dinoflagellates
and more elaborate than bacteria. In both the characteristically have two flagella for locomotion
organisms the cell structure is typically prokaryotic and most have a rigid cell wall of cellulose encrusted
type one envelope organisation with peptidoglycan with silica. Their cell wall is divided into two halves
wall, naked DNA, Blue- grenalgal have chlorophyll, called theca that may fit as two halves of a soap box
bacteria is devold of chlorophyll. or a petri dish. Some species (e.g., Noctiluca miliaris)
are bioluminescent.
S-10 Biology
37. (b) Cyanobacteria are aquatic and photosynthetic, i.e., 47. (b) Euglena has chlorophyll so it can be regarded as
they live in the water, and can manufacture their own plant. It is not possible to classify Euglena on the
food. They are quite small and usually unicellular, basis of two kingdom system of classification.
though they often grow in colonies large enough to 48. (c) Viruses maintain an inert crystalline structure outside
see. They are the first organisms to make the living cell viroids do not have protein coat
atmosphere aerobic. 49. (a) Nostoc is a cyanobacterium. Cyanobacteria are gram
38. (b) Chrysophytes are plant-like protists that can be (–) ve prokaryotes which perform oxygenic
found in marine and freshwater environments which photosynthesis like plants. Cyanobacteria can be
are often low in calcium. There are three main types unicellular (e.g. spirulina), colonial. (e.g. Nostoc) or
of chrysophytes: diatoms (bacillariophyta), golden- filamentous (e.g. Oscillatoria)
brown algae (chrysophyceae), and yellow-green 50. (a) Cell wall is a characteristic of bacteria, plants and
algae (xanthophyceae). fungi.
39. (d) Euglenoids is one of the best-known groups of 51. (c) Heterotrophs were the first to be evolved and fungi
flagellates. They are commonly found in freshwater, in the plant kingdom are heterotrophs. Yeast with its
especially when it is rich in organic materials, with a characteristic absence of chlorophyll and its
few marine, and endosymbiotic members. Most vegetative propagation through budding and
euglenoids are unicellular. Many euglenoids have saprophytic nature shows it is a fungus.
chloroplasts and produce energy through the 52. (a) Phycomycetes, ascomycetes, basidiomycetes and
process of photosynthesis, but others feed by deuteromycetes are four classes of fungi which are
phagocytosis, or strictly by diffusion. This group is divided on the basis of the septation of the mycelium
known to contain carbohydrate paramylon. and the characteristics features of reproduction.
40. (b) Phycomycetes is a large and important class of Phycomycetes, ascomycetes, basidiomycetes and
parasitic or saprophytic fungi, the algal or alga like deuteromycetes are respectively commonly known
fungi. The plant body ranges from an as algal like fungi, sac fungi, club fungi and fungi
undifferentiated mass of protoplasm to a well-
imperfecti.
developed and much-branched mycelium. Mycelium
53. (d) Ascus is a sac, typically cylindrical in shape, in which
is aseptate and coenocytic. Asexual reproduction
the spores of ascomycetes fungi develop. Basidium
takes place by spores and sexual reproduction by
is a microscopic club-shaped spore-bearing structure
the formation of conidia or sporangia.
produced by certain fungi. Euglena, Spirulina and
41. (c) Ascomycetes are commonly known as sac-fungi.
sponges are the examples of protista, cyanobacteria
They are mostly multicellular e.g., Penicillium or
and animalia respectively.
rarely unicellular, e.g., Yeast.
54. (b)
42. (c) All the described statements are related to slime
55. (a) Archaebacteria, euglenoids, phycomycetes and algae
moulds. Slime moulds are saprophytic protists. They
are classes of monera, protista, fungi and plantae
are a simple organism that consists of an acellular
respectively.
mass of creeping jelly-like protoplasm containing
nuclei, or a mass of amoeboid cells. When it reaches 56. (a) Ernst Mayr was a renowned taxonomist, tropical
a certain size it forms a large number of spore cases. explorer, ornithologist, and historian of science. His
43. (c) Basidiomycetes are commonly known as club fungi. work contributed to the conceptual revolution that
Other commonly known forms of basidiomycetes are led to the modern evolutionary synthesis of
mushrooms, bracket fungi or puffballs. Mendelian genetics, systematics, and Darwinian
44. (b) Deuteromycetes is an artificial group without any evolution, and to the development of the biological
common relationship. They are commonly known as species concept. Robert Harding Whittaker was a
fungi imperfecti due to absence of perfect sexual distinguished American plant ecologist. He was the
stage. They have septate hyphae and reproduce first to propose the five-kingdom taxonomic
asexually by means of conidia. classification of the world's biota into the Animalia,
45. (a) Fusion of two nuclei is called karyogamy. Fusion of Plantae, Fungi, Protista, and Monera in 1959. Louis
protoplasms between two motile or non-motile Pasteur was a French chemist and microbiologist
gametes called plasmogamy. Meiosis in zygote results renowned for his discoveries of the principles of
in haploid spores. vaccination, microbial fermentation and
46. (b) Cyanobacteria are photosynthetic (containing a blue pasteurization. He gave the name virus. Theodor Otto
photosynthetic pigment) autotrophs. They are Diener discovered the viroid, a plant pathogen one
prokaryotic and represent the earliest known form of fiftieth of the size of the smallest viruses.
life on the Earth.
EBD_7209
Hints & Solutions S-11

57. (d) Puccinia, Ustilago, Agaricus and Saccharomyces end plate is present to which tail fibres are attached.
are commonly called as rust fungi, smut fungi, 67. (b) Heterocysts are pale-yellow, thick-walled cells that
mushroom and yeast respectively. are capable of nitrogen fixation and therefore play a
58. (c) Protozoans are single celled microscopic eukaryotic major part in the nitrogen cycle.
organisms that are noted for their ability to move 68. (a) The structure marked as A, B, C and D are
independently. Protozoans are classified on the basis respectively head, collar, sheath and tail fibres. In
of their locomotion - amoeboid (Amoeba), ciliated head (structure marked as A), the protein coat (called
(Paramecium), flagellated (Trypanosoma) and the capsid) made of small subunits called
Sporozoans (Plasmodium). capsomeres, protect the nucleic acid.
59. (c) Rhizopus is a genus of common saprophytic fungi 69. (a) True sexual reproduction is absent in bacteria, but
on plants and specialized parasites on animals. They there occurs genetic recombination, i.e., bringing
are found on a wide variety of organic substrates, together of genetic material of two bacterial cells,
including "mature fruits and vegetables", jellies, i.e., transformation, transduction, conjugation.
syrups, leather, bread, peanuts and tobacco. 70. (b) The plant body of fungi typically consists of
Rhizopus stolonifer is more commonly known as branched and filamentous hyphae, which form a net
black bread mold. Gonyaulax is one of the like structure, known as mycelium. In yeast, the plant
dinoflagellates responsible for the advent of red body is unicellular but sometimes cells remain
tides. Yeast is a unicellular fungus, used to make attached in short chains, forming a pseudomycelium.
bread and beer. Penicillium is a genus of 71. (c) A virus is an infective agent that typically consists
ascomycetous fungi of major importance in the of a nucleic acid molecule in a protein coat. It is too
natural environment as well as food and drug small to be seen by light microscopy, and is able to
production. Some members of the genus produce multiply only within the living cells of a host. It can
penicillin, a molecule that is used as an antibiotic. be considered living organisms because it reproduces
60. (d) Albugo is the parasitic fungi which is found on inside the host cell.
mustard. It is the member of phycomycetes that are 72. (c) Lichens are formed by symbiotic relationship
found in aquatic habitats and on decaying wood in between algae or cyanobacteria and fungi. Lichens
moist and damp places. typically grow in harsh environments most lichens,
61. (d) Amoeboid protozoans move and capture their prey especially epiphytic fruticose species and those
by putting out pseudopodia (false feet). containing cyanobacteria, are sensitive to
62. (a) Saprophytes are heterotrophic and absorb soluble pollutants. Hence, they have been widely used as
organic matter from dead substrates. Vegetative mean SO2 pollution indicator organisms.
of reproduction in fungi are fragmentation, budding 73. (b) When moist bread is kept exposed in air, it becomes
and fission. Steps involved in sexual cycle of fungi mouldly and black because spores are present in the
are plasmogamy, karyogamy and meiosis in zygote bread.
resulting in haploid spores. 74. (d) Viruses are defined as infectious nucleoproteins. A
63. (b) The correct labeling in the figures of bacterial cell complete virus particle is called virion whose main
and Nostoc are - A- cell wall, B - cell membranes, C - function is to deliver its DNA or RNA gesnome into
DNA, D-heterocysts, E - mucilaginous sheath. the host cell, so that genome can be expressed by
64. (a) A - Cocci (spherical), B - Bacilli (rod shaped), C - the host cell. In a particular virus either DNA or RNA
Spirilla (spiral), D - Vibrio (comma shaped). These is genetic material, never both are present in a virus.
are all the shapes of the bacteria. Hence, viruses are:
65. (a) Euglenoids include flagellates like Euglana which (i) Double stranded DNA or ds DNA - Hepatitis B
have plant like characteristics (chlorophyll) in (ii) Single stranded DNA or ss DNA - coliphage
addition to some animal characteristics. (iii) Double stranded RNA or ds RNA - Reo virus,
Paramoecium is a ciliated protozoan, aquatic and wound Tumor virus
actively moving organisms because of the presence (iv) Single stranded RNA or ss RNA - Tobacco
of thousands of cilia. Agaricus (mushroom) belongs mosaic virus (TMV)
to class basidiomycetes of kingdom Fungi. 75. (d) Smut disease is caused by Ustilago species of
66. (c) The given diagram is of bacteriophage (viruses that basidiomycetes fungi. It is characterized by formation
infect the bacteria). They have tadpole-like structure, of black coloured chlamydospores or teleutospores
i.e., with head and tail. Nucleic acid generally DNA (called smut spores) due to which the affected part
is present inside the head. Tail is having hollow core becomes black.
end is surrounded by tail sheath. At the end of tail,
S-12 Biology
76. (d) Basidiomycetes are commonly known as club fungi. 88. (b) The sexual cycle in fungi involves the following three
They are fungus whose spores develop in basidia. steps : plasmogamy, karyogamy and meiosis
They include the majority of familiar mushrooms and (i) Fusion of protoplasm between two motile or
toadstools. non-motile gametes called plasmogamy.
77. (c) Saccharomyces cervisiae is a yeast used in making (ii) Fusion of two nuclei called karyogamy.
bread (Baker’s yeast) and commercial production of (iii) Meiosis in zygote resulting in haploid spores.
ethanol. Paramecium and Plasmodium are of animal 89. (a) A Gonyaulax reproduces in such a great numbers
kingdom while Penicillium is a fungi. Lichen is that the water may appear red, producing a red tides
composite organism formed from the symbiotic and kills large marine animals like fishes. Gonyaulax
association of an alga and a fungus. Nostoc and belongs to dinoflagellates.
Anabaena are examples of kingdom monera. 90. (c) Fruiting bodies, morphology of mycelium and mode
78. (d) Bacteria are microscopic organisms whose single of spore formation are the basis of classification of
cells have neither a membrane-enclosed nucleus nor fungi into phycomycetes, ascomycetes,
other membrane-enclosed organelles like mitochondria basidiomycetes and deuteromycetes.
and chloroplasts. Bacteria are found to be primitive 91. (b) Bracket fungi belong to basidiomycetes. This class
organisms because they do not have well-defined of fungi lacks sex organs but the process of
nucleus and shows amitotic division. plasmogamy is brought about by fusion of two
79. (c) Food can be kept for a longer time in cold house vegetative or somatic cells of different strains or
than in normal condition because bacterial genotypes.
multiplication is reduced in cold house. 92. (c) Deuteromycetes is commonly known as fungi
80. (b) Mycorrhizae is symbiotic association between fungi imperfecti. Once the sexual stage of members of
and roots of higher plants, e.g., in the roots of Pinus. deuteromycetes was discovered they were often
This association provides the fungus with food. moved to ascomycetes and basidiomycetes.
Fungus help in the absorption of water and nutrient. 93. (b) M. W. Beijerinek demonstrated that extract of
81. (d) The most abundant prokaryotes helpful to humans infected plants of tobacco could cause infection in
in making curd from milk and in production of healthy plants and called the fluid as "Contagium
antibiotics are the heterotrophic bacteria. vivum fluidum".
Lactobacillis bacteria convert milk into curd.
Chapter 3 : Plant Kingdom
82. (a) The pileus, or cap, is a common characteristic of the
club fungi (basidiomycetes). The mycelium is 1. (b) The natural system of classification for higher plants
branched and septate. was given by Bentham and Hooker in their Genera
83. (d) Plantarum (1862 - 1883). The characters employed
in this system include those of study of form
30 min 30 min 30 min
1 ¾ ¾ ¾¾
® 2 ¾ ¾ ¾¾
® 4 ¾ ¾ ¾¾
®8 (morphology), internal structur e (anatomy),
30 min
¾ ¾ ¾¾
® 16 ¾ ¾ ¾¾
30 min
® 32 ¾ ¾ ¾¾
30 min
® 64 development (embryology), reproduction, cell
30 min 30 min structure (cytology), life processes, (physiology),
¾ ¾ ¾¾
® 128 ¾ ¾ ¾¾
® 256
behaviour and biochemistry.
84. (b) In fungi, asexual reproduction takes place by spores- 2. (d) Phylogenetic system of classification indicates the
called conidia or sporangiospores or zoospores, and evolutionary as well as genetic relationships among
sexual reproduction is by oospores, ascospores and organisms. It is based on fossil record, biochemical,
basidiospores. anatomical, morphological, embryological,
85. (d) When two hyphal cells of different mating types fuse, physiological, genetics, karyotype and other studies.
they form a dikaryotic cell which is heterokaryotic. 3. (b) Cytotaxonomy is the classification of organisms
86. (b) Protozoans comes under the kingdom protista based on cellular structure and function, especially
(single-celled eukaryotes). They are heterotrophs on the structure and number of chromosomes.
and live as predators or parasites. They are divisible 4. (d) Isogamous and anisogamous type of reproduction
into four major groups — zooflagellata, sarcodina, are found in Chlamydomonas. Fusion of flagellated
sporozoa and ciliata. gametes of similar size is called isogamous whereas
87. (b) Bacteria are prokaryotes while yeast belongs to the fusion of two gametes of different size is called
kingdom Fungi (they are multicellular eukaryotes). anisogamous.
Yeast is exception in fungi because it is unicellular 5. (c) Fusion of two gametes which are of dissimilar size is
but forms pseudomycelium. called anisogamous. Fusion of flagellate gametes of
similar size is called isogamous. Oogamous is the
EBD_7209
Hints & Solutions S-13

fusion between one large, non - motile female gamete 17. (d) Pteridophytes are classified into four classes :
with a smaller, motile male gamete. Agamous does Psilopsida (Psilotum), Lycopsida (Selaginella,
not involve the fusion of male and female gametes in Lycopodium), Sphenopsida (Equisetum) and
reproduction. Pteropsida (Dryopteris, Pteris, Adiantum).
6. (b) Certain marine brown and red algae produce large 18. (b) Cycas (a gymnosperm) and Adiantum (known as
amounts of hydrocolloids (water holding Maiden hair fern, a pteridophyte) resemble each other
substances), e.g., algin (brown algae) and carrageen in having motile sperms. Seeds, cambium are common
(red algae) which are used commercially. Chlorella in gymnosperms but absent in pteridophytes. True
and Spirullina are unicellular green algae, which are vessels are absent in both pteridophytes and
rich in proteins and used as food supplements. gymnospems.
7. (b) The members of phaeophyceae or brown algae are 19. (a) The predominant stage of the moss is the presence
found primarily in marine habitats. The common forms of gametophyte which consists of two stages – the
are Ectocarpus, Dictyota, Laminaria, Sargassum first stage is protonema and the second stage is leafy
and Fucus. stage. Protonema stage develops directly from the
8. (b) Rhodophyceae are commonly called as red algae spores and leafy stage develops from the secondary
because of the predominance of the red pigment (r - protonema as a lateral bud.
phycoerythrin) in their body. They are mostly found 20. (d) Gametophyte of bryophytes bears protonemal and
in salt water. leafy stage. In gymnosperm, female gametophyte is
9. (d) Flagellate forms and stages are completely absent in not free living. Antheridiophores and
class rhodophyceae (red algae). archegoniophores are present in Marchantia which
10. (a) In class phaeophyceae, the plant body is usually is a bryophyte. Origin of seed habit started in
attached to the substratum by a holdfast (A), and has pteridophyte.
a stalk, the stipe (B) and leaf like photosynthetic organ 21. (b) Rhodophyta are commonly called as red algae
- the frond (C). because of the predominance of the red pigment (r –
11. (b) Bryophytes play an important role in plant phycoerythrin) in their body. They occur in both
succession on bare rocks/soil. Bryophytes include well lighted regions close to the surface of water and
the various mosses and liverworts that are found in also at great depths in ocean where relatively little
moist shaded areas in the hills. Moss stage is the light penetrates. They exhibit a red colour because
second stage in biological succession on a bare rock the wavelengths of light that are absorbed by
that slowly changes rocky substratum into fertile chlorophyll are passed to phycoerythrin.
soil. Important members of moss stage are Tortula, 22. (a) Double fertilization is a process, unique to flowering
Grimmia and Polytrichum. plants (angiosperms). The gymnosperms do not
12. (d) In gymnosperms, the reproductive structures are exhibit double fertilization or triple fusion like
mostly in the form of compact cones except female angiosperms.
organs of Cycas. There are two types of sporophylls, 23. (d) Mosses and ferns belong to bryophtes and
usually segregated to form distinct cones or strobili pteridophytes respectively. In bryophytes, the
(male and female cones). dominant phase in the life cycle is the gametophytic
13. (c) In bryophytes, sex organs are of two types, male plant body. However, in pteridophytes, the main plant
antheridium and female archegonium. They are body is a sporophyte, which is differentiated into
multicellular and jacketed, i.e., covered by jacket of true root, stem and leaves. These organs possess
sterile cells. well – differentiated vascular tissues.
14. (b) Laminaria (kelp) and Fucus (rock weed) are the 24. (d) Gymnosperms (including conifers) and angiosperms
examples of brown algae. Brown algae are the common both have advanced support tissues true leaves,
name of phaeophyceae. They are found primarily in stems and roots they also are the two groups that
marine habitats and show great variation in size and produce pollen and seeds.
form. 25. (a) Monoecious is a plant species in which male
15. (a) Pyrenoids are the rounded bodies found in the (microsporangiate) and female (megasporangiate)
chloroplast of green algae and are the centres of organs are found on the same plant (for example
conversion of glucose to starch and also collection pinus). Dioecious plant is a plant species in which
of starch. male and female organs appear on separate
16. (a) Selaginella is a member of lycopsida, which produce individuals.
two kinds of spores-macro (large) and micro (small) 26. (d) 27. (d)
spores. Thus, known as heterosporous pteridophytes.
S-14 Biology
28. (c) Algae are defined as chlorophyllous, thalloid flagellate unicells, filaments, and more. They also
avascular plants with no cellular differentiation. The reproduce in a variety of ways, though all have a
size and form of algae is highly variable. Not all the haploid life-cycle, in which only the zygote cell is
algae are filamentous. The size ranges from the diploid. The zygote will often serve as a resting spore,
microscope unicellular forms to colonial forms and able to lie dormant though potentially damaging
to the filamentous forms. environmental changes such as desiccation.
29. (a) Statement (i) and (ii) are correct. 40. (c) Bryophytes are small, non-vascular plants, such as
Riccia is liverwort in which simplest sporophyte mosses, liverworts and hornworts. Bryophytes do
consists of capsule only while Polytrichum is moss not have seeds or flowers. Instead they reproduce
in which sporophyte consists of foot, seta & capsule. via spores.
Volvox is a fresh water green colonial alga. 41. (a) Liverworts (belongs to bryophytes) is a flowerless,
Reproduction is both sexual and asexual in Volvox. spore-producing plant - with the spores produced in
Sexual reproduction is of oogamous type. small capsules. Liverworts are small, green, terrestrial
30. (d) In gymnosperms, the male and female gametophytes plants. They do not have true roots, stems, or leaves.
never grow independently but develop within the Instead, they have an above ground leaf-like
sporophyte structures. structure, known as a thallus, and an underground
31. (d) Isogamous fertilization is the fusion of two similar structure, known as a rhizoid. Most liverworts are
gametes while anisogamous fertilization is fusion of found in moist environments and they tend to be
two morphologically dissimilar motile gametes. less resistant to desiccation than their relatives, the
In rhodophyceae, the major pigments are chlorophyll mosses.
‘a’ and ‘d’ but the stored food is floridean starch. Like mosses, many species of liverworts reproduce
32. (b) Double fertilization is a characteristic of angiosperms by making gemmae.
not gymnosperms. But in both of these two groups, 42. (c) The leaves are small (microphyll) as in Selaginella
gametophytic phase is highly reduced and is retained or large (macrophyll) as in ferns. Funaria,
inside sporophytic structures. Sequoia is the tallest Polytrichum and Sphagnum are common examples
gymnosperm not angiosperm. Tallest angiosperm of mosses.
belong to the genus Eucalyptus. 43. (c) Gymnosperms are seed-bearing vascular plants, such
33. (a) In rhodophyceae (red algae) food is stored in the as cycads, ginkgo, yews and conifers, in which the
form floridean starch. Whereas mannitol and laminarin ovules or seeds are not enclosed in an ovary.
are stored food material of class phaeophyceae (brown Gymnosperm seeds develop either on the surface of
algae). In the diplontic life cycle, sporophyte represents scale or leaf-like appendages of cones or at the end
the dominant phase. of short stalk. The largest group of living
34. (d) In chlorophyceae, the stored food material is starch gymnosperms are the conifers (pines, cypresses and
and the major pigments are chlorophyll- a and b. In
relatives) and the smallest is ginkgo, a single living
phaeophyceae, laminarian is the stored food and
plant species found in China.
major pigments are chlorophyll a, c and fucoxanthin.
44. (b) In angiosperms, the male sex organ is the stamen
35. (c)
and the female sex organ is the pistil or the carpel.
36. (d) Gemmae are a means of asexual reproduction found
Each stamen consists of a slender filament with an
in many bryophytes (e.g, liverworts). They are 1 to
another at the tip. The anther, following meiosis,
many celled, specially produced clonal plant
produce pollen grains. Pistil consists of an ovary
fragments. They are green, multicellular, asexual buds
enclosing one to many ovules.
which develop in small receptacles (called gemma
45. (a) Haplontic life cycle is a simplest and most primitive
cups) located on the thalli. Gemmae become detached
type of life-cycle found in all chlorophyceae. In
from the parent body and germinate to form new
haplontic pattern of life cycle, the somatic phase
individuals.
37. (b) (plant) is haploid (gametophyte) while the diploid
38. (c) Algae are chlorophyll bearing simple, thalloid, phase (sporophyte) is represented by zygote. During
autotrophic and largely aquatic organisms. They germination, the zygote (2n) divides meiotically
reproduce by vegetative, asexual and sexual producing haploid (n) zoospores, which develop into
methods. Fusion of two gametes dissimilar in size is individual plant. Here the unicellular or filamentous
termed as anisogamous. gametophyte (n) alternates with one-celled zygote
39. (a) The members of chlorophyceae include free- or sporophyte (2n).
swimming unicellular species, colonies, non-
EBD_7209
Hints & Solutions S-15

46. (c) Agar, one of commercial products obtained from Angiosperms comprises those that have flowers and
Gracilaria and Gelidium are used to grow microbes produce seeds enclosed within a carpel, including
and in preparation of ice-creams and jellies. In herbaceous plants, shrubs, grasses, and most trees.
phaeophyceae, major pigments are chlorophyll a, c Example - Solanum tuberosum.
and fucoxanthin. Pteridophytes are the vascular plants (those having
47. (d) The leaves in gymnosperms are well adapted to xylem and phloem tissues). They include the highly
withstand extremes of temperature, humidity and diverse true ferns and other graceful, primarily forest-
wind. In conifers, the needle like leaves reduces dwelling plants. Example - Equisetum.
surface area. Their thick cuticle and sunken stomata Gymnosperms are seed-bearing vascular plants in
also help to reduce water loss. which the ovules or seeds are not enclosed in an
48. (b) Double fertilization is a characteristics feature of ovary. Example - Cycas.
angiosperms. It involves two fusions in which one 56. (a) Pteridophytes are the vascular plants (those having
male gamete fuses with egg cell to from zygote and xylem and phloem tissues) that reproduce by
the other male gamete fuses with the diploid releasing spores rather than seeds, and they include
secondary nucleus to produce the triploid primary the highly diverse true ferns and other graceful,
endosperm nucleus (PEN). primarily forest-dwelling plants. The pteridophytes
49. (a) Phaeoph yceae possesses chlor ophyll a, c, are further classified into four classes: psilopsida
carotenoids and xanthophylls. Members of (eg, Psilotum), lycopsida (eg, Selaginella),
phaeophyceae show variations in colour from olive sphenopsida (eg, Equisetum) and pteropsida (eg,
green to different shades of brown depending upon Dryopteris).
the amount of the xanthophyll pigments. 57. (a) 58. (a)
50. (a) Mosses that belong to the bryophytes have evolved 59. (b) Artificial system of classification is based on the
from algae. The fact that protonema has a thallus like few morphological characters. Natural system of
body shows that mosses have evolved from algae. classification is based on the natural affinities among
51. (a) Red algae generally grow attached to rocky stones. the organisms and considers external as well as
Some deep water red algae are calcareous and build
internal features. Phylogenetic system of
up hard stony thalli responsible for the production
of lime stones and coral reefs. classification is based on the evolutionary
52. (a) Coconut tree is distributed in coastal areas since it is relationships between the various organisms.
adapted to saline water. 60. (c)
53. (a) Red algae generally grow attaced to rocky stones. 61. (b) Bryophytes are called as the amphibian of the plant
Some deep water red algae are calcareous and build kingdom because these plants can live in soil but are
up hard stony thalli responsible for the production dependent on water for sexual reproduction. Gemma
of lime stones and coral reefs. is specialised structure in liverworts for asexual
54. (b) The peristome teeth are present at the mouth of the reproduction. Monocotyledons and dicotyledons
capsule. The teeth may be solid cellular tissue or
are two groups of angiosper ms. Species of
composed only of the thickened portions of the cell
walls of adjacent cells. When the teeth of peristome Sphagnum, a moss provide peat that have been long
are solid structures composed of bundles of dead been used as a fuel.
cells, it is termed nematodontous peristome are solid 62. (c) Rhodophyceae is red algae in which food is stored
structures composed of bundles of dead cells, it is as floridean starch which is very similar to
termed nematodyntous peristome. It is found in amylopectin and glycogen in structure.
polytrichum, Pogonatum and Tetraphis. Peristome 63. (d) 64. (a)
when composed of thin, membranous, transversely 65. (a) The options A B ,C and D are Volvox
barred teeth, and each tooth is made up of the Chlamydomonas Laminaria and Chara respectively.
thickened portions of the cell walls of adjacent cells. A, B and D belong to Chlorophycae (green algae)
Such a peristome is called orthodontous.
where as C belongs to Phaeophycae (brown algae)
55. (b) Algae are a simple, non-flowering, and typically
66. (b)
aquatic plant of a large assemblage that includes the
67. (d) In haplontic cycle, there is a single dominant phase
seaweeds and many single-celled forms. Example -
i.e., haploid gametophyte which is free living
Chlamydomonas.
photosynthetic phase. In diplontic life cycle, diploid
Fungi is a diverse group of eukaryotic single-celled
sporophyte is dominant vegetative phase. The
or multinucleate organisms comprisin g th e sporophyte is independent photosynthetic phase of
mushrooms, moulds, mildews, smuts, rusts, and the plant. Haplo-diplontic life cycle is intermediate
yeasts. Example - Rhizopus.
S-16 Biology
condition where both phases are multicellular and 78. (a) In green algae (chlorophyceae), the photosynthetic
often free living. pigments are chlorophyll a and b, carotenes and
68. (b) The correct name of the plants A, B, C and D are xanthophylls. In phaeophyceae (brown algae), the
respectively Selaginella, Equisetum, Salvinia and pigments are chlorophyll a, c and carotenes and
Ginkgo. The first three plants belong to fucoxanth. Rhodophyceae are red algae and contain
pteridophytes and the fourth one belongs to the major pigment as chlorophyll a and phyeoerythrim.
gymnosperms. Phycoerythrin gives red colour to rhodophyceae.
69. (a) The life cycle of an angiosperm is defined by the 79. (d) The ovules are not enclosed inside the ovary. Instead
formation of the seed and its development to a full- they are borne naked on the leafy sporophylls, and
grown plant which, in turn, produces seeds. hence the name gymnosperms (gymnos- naked
Angiosperms are vascular plants with flowers that sperma- seed) is given Double fertilization is absent
produce seeds enclosed in an ovule—a fact that is in gymnosperms.
recognized as the angiospermic condition. 80. (b) The female sex organ archegonium is formed in
70. (b) Chlamydomonas, Volvox, Ulothrix, Spirogyra and bryophytes (Funaria), pteridophytes (Dryopteris)
Chara are green algae. The common forms of brown and gymnosperms (Ginkgo).
algae are Ectocarpus, Dictyota, Laminaria, 81. (b) In angiosperms, a mature male gametophyte is derived
Sargassum and Fucus. Polysiphonia, Porphyra, from a pollen mother cell by one meiotic and two
Gracilaria and the Gelidium are members of red mitotic divisions. In the pollen sac (microsporangium)
algae. of the anther, haploid microspores are formed by
71. (b) Bryophytes completely lack vascular tissue system meiosis. Mitosis then follows to produce a two-celled
and absorb water by generally body surface. pollen grain with a small generative cell and a large
Whereas pteridophytes are the first terrestrial plants vegetative cell. This generative cell will undergo
to possess vascular tissues -xylem and phloem. further mitosis to form two male gametes (nuclei).
72. (c) Fusion of male gamete with the egg results in the The pollen tube grows through a spore in the pollen
formation of zygote. This zygote produces a multi- grain, with the tube (vegetative) nucleus at its tips
cellular well differentiated sporophyte which is the and the male nuclei behind.
dominant phase of the pteridophytes. 82. (d) Algae are eukaryotic organisms that have no roots
73. (d) In bryophytes, zygotes donot undergo reduction stems or leaves but do have chlorophyll and other
division immediately. They produce a multicellular pigments for carrying out photosynthesis
body called a sporophyte. The sporophyte is not a 83. (a) A protonema is a creeping, green, filamentous,
free-living but attached to the photosynthetic thread-like chain of cells which is produced by the
gametophyte and derives nourishment from it. germination of the spores and forms the earliest stage
meiotic division (the haploid phase) of a bryophyte life cycle. When
74. (c) 1 microspore mother cell ¾¾¾¾¾¾® 4 a moss first grows from the spore, it grows as a
haploid microspores or pollen grains protonema which develops into a leafy gametophore.
4 microspore mother cells = 4 × 4 pollen grains 84. (d) Pteridophytes are the vascular plants (those having
= 16 pollen grains. xylem and phloem tissues) that reproduce by
75. (b) Bryophytes have multicellular jacketed sex organs releasing spores rather than seeds, and they include
to protect gametes against drying effects of air and the highly diverse true ferns and other graceful,
retention of zygote inside archegonium to provide primarily forest-dwelling plants. The spreading of
nourishment to developing embryo. living pteridophytes is limited and is restricted to
76. (d) Species of Sphagnum, a moss, provides peat that narrow geographical r egion because its
have long been used as a packing material for sending gametophytes require cool, damp, shady places to
flowers and live plants to distant places because it is grow and also it requires water for fertilization.
hygroscopic. 85. (a) Plant classification proposed by Carolus Linneaus
77. (b) Number of chromosome in root cells of wheat plant was artificial because it was based on a few
is morphological characters such as habit, colour,
2n = 42 number and shape of leaves, etc.
42 86. (d) Mosses along with lichens are the first organisms to
n= = 21 colonise rocks and hence, are of great ecological
2
importance. They decompose rocks making the
The number of chromosome in the cell of pollen grain
substrate suitable for the growth of higher plants.
is 21 because pollen grains are haploid (n) in nature.
Since mosses form dense mats on the soil, they
EBD_7209
Hints & Solutions S-17

reduce the impact of falling rain and prevent soil - fragmentation, asexually by non-motile spores and
erosion. sexually by non-motile gametes. Red algae is the
87. (b) According to the features described, the unknown common name of Rhodophyta. They are called red
plant belongs to bryophytes. Bryophytes refer to a algae because of the presence of red pigment, r-
group of plants comprising the mosses, liverworts, phycoerythrin.
and hornworts. They do not have a true vascular
Chapter 4 : Animal Kingdom
system and are unable to pull water and nutrients up
from the ground at any significant distance. This 1. (a) Radial symmetry is found in coelenterates,
distinguishes bryophytes from ferns and flowering ctenophores and echinoderms. The right and left, or
plants. the dorsal and ventral sides, in such symmetry are
88. (b) Mosses are non-vascular plants which means they not differentiated. This type of symmetry is
cannot transport water and nutrients to the aerial advantageous to sessile (attached) animals as they
parts of the plant. can feed from all directions.
89. (b) The correct sequence of the plants group which 2. (b) Pathway of water transport is not the common
appeared first on the earth to appear most recently fundamental feature for animal classification.
in time is: Algae ® Moss ® Fern ® Gymnosperms 3. (a) Bilateral symmetry is a symmetrical arrangement, as
® Angiosperms of an organism or a body part, along a central axis,
90. (b) Chemotaxonomy is based on chemical products so that the body is divided into equivalent right and
particularly secondary metabolites. Various families left halves by only one plane.
of plants have been identified on the basis of 4. (c) Sponges are pore bearing animals belong to phylum
raphides (Crystals of calcium oxalates). porifera. They have a water transport or canal system.
91. (d) In order to colonize the land, the plants needed to Spongilla is a fresh-water sponge.
acquire the mechanisms to prevent desiccation of 5. (c) Cnidarians are aquatic, mostly marine, sessile or free-
tissues and the ability to screen ultraviolet radiation. swimming, radially symmetrical animals. The
Because there are several successful groups of presence of the stinging capsules or nematocytes is
plants that lack mechanisms for moving water the characteristic feature of cnidarians and they are
throughout the plant. present on the tentacles and the body for anchorage,
92. (c) No known seed plant has flagellated sperms. defense and for the capture of prey. Some of the
93. (c) Life cycle of any sexually reproducing plant consists cnidarians, e.g., corals have a skeleton composed of
of two morphological phases i.e., haploid CaCO3.
gametophyte and diploid sporophyte that regularly 6. (b) Meandrina (brain coral) belongs to phylum
alternate with each other. This phenomenon is termed coelenterate. Coelenterates are radially symmetrical,
as alternation of generation. diploblastic aquatic animals with diffused nervous
94. (a) Angiosperms are highly evolved and well adapted system.
land plants. They have both vessels and tracheids 7. (d) Ctenophores are commonly known as sea-walnuts
in xylem for better conduction of water. Roots are or comb-jellies. They are exclusively marine, radially
modified into tap roots, adventitious roots, symmetrical, diploblastic organisms with tissue level
pneumatophores etc. to suit the desired climate. of organization. The body bears eight external rows
Sex organs are highly developed, sporophylls are of ciliated comb plates, which help in locomotion.
organized into flowers and the flowers are highly 8. (d) Flame cells function like a kidney, removing waste
coloured or attract pollinators at different times and materials. Flame cells are ciliated cells that form part
places. Insect pollination is more prevalent because of the excretory and osmoregulatory system of
it is more efficient and leads to less wastage of pollen platyhelminthes, rotifers, and nemertine worms. This
grains as compared to wind pollination. So the system, known as a protonephridium, consists of
flowers are made attractive to attract a variety of branching tubules that open to the exterior through
insects. Seeds are more protected as they are excretory pores; flame cells occur at the ends of the
enclosed inside a fruit. All these adaptations have tubules, into which their cilia project.
made angiosperms more adaptive in diverse habitats. 9. (b) Cnidarians exhibit two basic body forms called polyp
95. (a) Asexual reproduction (in algae) is by the production and medusa. Polyp is a sessile and cylindrical form
of different types of spores, and the most common like Hydra, Adamsia, etc. whereas, the latter is
type is zoospore. Zoospores are flagellated (motile) umbrella-shaped and free-swimming like Aurelia or
and on germination give rise to new plants. jelly-fish.
96. (b) The red algae usually reproduce vegetatively by 10. (a) Ctenophora is a small phylum of marine coelenterates
S-18 Biology
like free swimming and biradially symmetrical animals. marine with streamlined body and have cartilaginous
In ctenophora, sexes are not separate. Reproduction endoskeleton. Some of them have electric organs
takes place only by sexual means. In Cnidaria, which (e.g., Torpedo) and some possess poison sting (e.g.,
exist in both forms, polyps produce medusae Trygon).
asexually and medusae form the polyps sexually. 21. (d) Trygon belongs to chondricthyes, possesses poison
Porifera (sponge) reproduce asexually by sting. Chondricthyes is also called cartilaginous
fragmentation and sexually by formation of gametes. fishes and includes all members of shark, skates,
Protozoans come under protista which reproduce rays and chimareae.
asexually as well as sexually. 22. (b) Lampreys and hag fishes (Myxine) are unusual,
11. (d) Choanocytes (also known as "collar cells") are cells jawless fish that comprise the order cyclostomata,
that line the spongocoel. The water current of the so named because of the circular shape of the mouth.
body of sponge is maintained by the movement of The brains of lampreys and hagfishes differ a lot,
flagella of choanocytes. but they also show a large number of similarities, as
12. (b) Nereis is a genus of polychaete worms in the family do all craniate brains.
nereidae. It possesses setae and parapodia. 23. (d) In amphibia, respiration occurs through gills, lungs
Parapodia are paired, lateral appendages extending and skins. With the exception of a few frog species
from the body segments. that lay eggs on land, all amphibians begin life as
13. (d) Phylum arthropoda is the most numerous phyla of completely aquatic larvae. Respiratory gas exchange
all living organisms, both in number of species and is conducted through thin, gas-permeable skin and
in number of individuals. the gills. As amphibian larvae develop, the gills (and
14. (d) Limulus commonly known as king crab belongs to in frogs, the tail fin) degenerate, paired lungs
phylum arthropoda. It is a living fossil and large sized develop, and the metamorphosing larvae begin
marine arthropod. making excursions to the water surface to take air
15. (b) The radula is an anatomical structure that is used by breaths.
molluscs for feeding, sometimes compared rather 24. (b) In amphibians, the heart is three chambered (two
inaccurately to a tongue. It is a minutely toothed, ventricles and one auricle). Accessory chambers are
chitinous ribbon, which is typically used for scraping sinus venosus and truncus arterious.
or cutting food before the food en ters the 25. (d) Crocodile belongs to class reptilia. Reptiles have
oesophagus. The radula is unique to the molluscs, usually three chambered heart but crocodile have
and is found in every class of mollusc except the four-chambered heart.
bivalves. 26. (d) Naja, Viper and Bungarus are poisonous snakes.
16. (c) Malpighian tubule is the organ of excretion in insects The poison of snake is called venom. Bungarus is
and many other arthropods. It lies in the abdominal the highly poisonous land snake.
body cavity. 27. (d) Chordates show the presence of nerve cord,
17. (a) Echinoderms are a phylum of marine animals. The notochord and pharyngeal gill slits.
adults are recognizable by their (usually five-point) 28. (a) Aquatic annelids like Nereis possess lateral
radial symmetry, whereas the larvae are bilaterally appendages (called parapodia) which help in
symmetrical. For example starfish, sea urchins, sand swimming. Radula is the rasping organ for feeding
dollars, and sea cucumbers, as well as the sea lilies which is present in the mouth of molluscs.
or "stone lilies". 29. (a) Circulatory system in arthropods is of open type
18. (a) Phylum hemichordata consists of a small group of i.e., blood does not flow in definite vessels. Irregular
worm-like marine animals with organ system level of spaces known as lacunae or sinuses, filled with blood
organization. They are bilaterally symmetrical, are present.
triploblastic and coelomate animals. Excretion takes 30. (a) Porifera is commonly referred to as sponges. They
place through proboscis gland. are multicellular organisms that have bodies full of
19. (c) The chordate is the phylum which includes humans pores and channels allowing water to circulate
and other vertebrates. However, not all chordates through them. Water canal system is the
are vertebrates. All chordates have the following characteristic feature of the porifera.
features at some point in their life (in the case of 31. (c) Statements (iii), (iv) and (v) are correct. The pelvic
humans and many other vertebrates, these features fins of male sharks bear claspers. In cnidarians (e.g.
may only be present in the embryo): Pharyngeal slits, Obelia) polyps produce medusae asexually and
dorsal nerve cord, notochord, Post-anal tail. medusae form the polyps sexually.
20. (a) Animals that belong to class chondrichthyes are 32. (a) Acoelomates are animals that have no body cavity
EBD_7209
Hints & Solutions S-19

or coelom. The examples are poriferans, excretory tube removes body wastes from the body
coelenterates, ctenophores, platyhelminthes, etc. cavity through the excretory pore.
Pseudocoelomates are animals that have false or 46. (d) Mollusca are the second largest phylum after
pseudocoelom. Examples are aschelminthes. arthropoda and includes predominantly marine
Coelomates are animals that have true coelom animals.
enclosed by mesoderm on both sides. Examples: 47. (a) Cyclostomata is a group of chordates that includes
annelida to arthropoda. Hence, roundworms are the living jawless fishes: the lampreys and hagfishes.
pseudocoelomates, molluscs and insects are The name cyclostomata means "round mouths".
coelomates while flatworms are acoelomates. The Their mouths cannot close due to the lack of a jaw,
charactersties features given above describe so they have to constantly cycle water through the
tenophora. mouth.
33. (d) Ctenophores, also known as comb jellies/sea 48. (c) Mammals are tetrapod that have hair, a four-
gooseberries/sea walnuts, or Venus's girdles, are chambered heart, a diaphragm, and mammary glands.
voracious predators. 49. (a) As the name suggests, amphibians can live in aquatic
34. (a) as well as terrestrial habitats. Respiration is by gills,
35. (c) Notochord is a flexible rod like structure that forms lungs and through skin. The heart is three chambered
the main support of the body in the lowest chordates. (two auricles and one ventricle) and the fertilization
It is not absent in humans throughout their life. is external.
Notochord is present in embryonic stage and get 50. (c) Nereis is dioecious but earthworms and leeches are
changed or replaced by vertebral column in the adult. monoecious.
36. (a) In option (a) all the characteristics belong to class 51. (a) Name of coelenterate is derived from stinging
reptilia. In options (b), (c) and (d) the characteristics capsules. It exhibits metagenesis containing two
belong to the classes amphibia, osteichthyes and body forms in which sessile and cylindrical form is
chondrichthyes respectively. called polyp and umbrella shaped and free –
37. (d) Animals belong to phylum porifera are mostly marine swimming is called medusa.
except a few which are found in fresh water. E.g. 52. (b) Both duck billed platypus and spiny ant eaters are
Spongilla, Euspongia. mammals because of their constant body temperature
38. (d) Aves are warm blooded, oviparous, bipedal flying and presence of diaphragm.
vertebrates with an exoskeleton of feathers. 53. (b) Ctenidium is a gill situated on the right side of the
Forelimbs are modified as wings for flying and hind branchial chamber. It helps in respiration by beating
limbs are adapted for walking, perching and cilia. During development, ctenidium shifts from left
swimming. side to right side which is called "torison". It is
39. (a) Platypus is an anoviparous mammal. Spongilla and characteristic feature of gastropods.
Euspongia are fresh water sponges. crocodiles have 54. (b) Tapeworm, roundowrm & pinworm ar e all
four chambered heart. In the members of class endoparasites. The main cause of the intecstinal
chondrichthyes, the notochord is persistent infection is impeoperly woked food. However
throughout life. taprworm infection occur by eating improperly
40. (d) Sponges, cnidarians, flatworms and ctenophorans cooked food, roundworm is transmitted by
do not have circulatory systems. contaminated food & water and pinworm or ringworm
41. (c) Species of the ph ylum echinodermata are is transmitted through food or imporoper sanitary
deuterostomes. condition.
42. (c) The digestive system of platyhelminthes is 55. (b) Sponges are multicellular but they have cellular level
incomplete. A complete digestive system has two of body organization i.e., true tissue, movable parts,
openings, mouth and anus. or appendages are not formed. Although, there is
43. (c) Notochord is a mesodermally derived rod like some physiological division of labour, accompanied
structure formed on dorsal side during embryonic with structural differentiation amongst body cells.
development in some animals. But here similar cells are arranged neither in
44. (c) Sycon, Spongilla and Euspongia are examples of permanent layer nor masses to form tissues.
phylum Porifera. Physalia, Adamsia, Pennatula, 56. (a) The water vascular system is a unique organ system
Gorgonia and Meandrina are examples of phylum that functions in locomotion, feeding, respiration and
coelenterata (Cnidaria). excretion. Ambulacral canal is connected to outside
45. (c) Nephridia present in phylum- annelida help in through external tube feet. Hydraulic pressure of fluid
osmoregulation and excretion. In aschelminthes, an
S-20 Biology
and contraction of muscle of tube feet make possible made of chitin. Pila is a genus of large freshwater
movement of Echinoderm. snails with an operculum.
57. (b) 58. (d) 59. (d) 60. (c) 61. (c) 62. (b) 63. (a) 73. (b) Aurelia is a genus of scyphozoan jellyfish (Cnidaria,
64. (c) Pteropus (flying fox) belongs to class mammalia. The Scyphozoa). Adamsia is a genus of sea anemones. It
skin of mammals is unique in possessing hair. They is usually found growing on a gastropod shell
are viviparous with exception (Ornithorhynchus is inhabited by the hermit crab, Pagurus prideaux. A
oviparous). cnidocytes (also known as a cnidoblast or
65. (a) Molluscans are soft bodied animals. Their body is nematocyte) is an explosive cell containing one giant
unsegmented with a distinct head, muscular foot and secretory organelle that defines the phylum Cnidaria
visceral hump. In Pila, the buccal cavity contains a (corals, sea anemones, hydrae, jellyfish, etc.).
rasping organ, the radula with transverse rows of Pleurobranchus is a genus of sea slugs, specifically
teeth. side-gill slugs, marine gastropod mollusc.
66. (d) The sea lamprey (Petromyzon marinus) is a parasitic 74. (a) All the figures (A, B, and C) are the examples of
lamprey. Ichthyophis is a limbless amphibian, belongs porifera phylum. A, B and C are respectively Sycon,
to apoda class of amphibia. Limulus (horse shoe crab) Euspongia (also called horny sponge) an d
is a living fossil and long sized marine arthropod. Spongilla. They are primitive, sessile, aquatic, water
Adamsia is a genus of sea anemones in the family dwelling filter feeders that pump water through their
Hormathiidae. bodies to filter out particles of food matter.
67. (c) Chondrichthyes are the cartilaginous fish with a 75. (b) Acoelomates : The animals which do not have coelom
flexible skeleton made of cartilage rather than bone. e.g., sponges, coelenterates, ctenophorans and
68. (a) A - (i); B - (ix); C - (viii); D - (v); E - (iii) flatworms. Pseudocoelomates : Body cavity is not
69. (d) A - (v); B - (iv); C - (iii); D - (i); E - (ii) lined by mesoderm continuously, but it is present as
70. (b) A - (iii); B - (i); C - (iv); D - (ii); E - (v) scattered pouches in between the ectoderm and
Pseudocoelomates is a group of invertebrates with a endoderm. Aschelminthes (round worms) are
three-layered body that has a fluid-filled body cavity pseudocoelomates.
called pseudocoelom. E.g, roundworms, rotifers. Eucoelomates (Coelomates) : The animals which
Diploblastic animals are with two germ layers - possess true coelom. True coelom is found in
ectoderm and endoderm, e.g, cnidaria. In cellular level annelids, echinoderms and chordates.
of organization, the cells are arranged as loose cell 76. (d) The given figure is of tapeworm. Tapeworm belongs
aggregates, e.g, porifera. In radial symmetry, to phylum Platyhelminthes which are mostly
symmetrical arrangement of parts of an organism is endoparasites, bilaterally symmetrical, triploblastic
around a single main axis, so that the organism can and acoelomate animals with organ level of
be divided into similar halves by any plane that organization.
contains the main axis. The body plans of 77. (b) Ascaris belongs to nematode phylum which is a non
echinoderms, ctenophores, cnidarians, and many - segmented roundworms / threadworms/ pinworms.
sponges and sea anemones show radial symmetry. Nereis is a genus of polychaete worms. It possesses
Metamerism is the phenomenon of having a linear setae and parapodia for locomotion. Hirudinaria, also
series of body segments fundamentally similar in known as cattle leech, belongs to annelida phylum. It
structure, e.g, annelida. acts as a parasite on cattle.
71. (a) Diploblastic animals have two germinal layers, outer 78. (a) Octopus (also called devil fish) belongs to molluscs.
ectoderm and inner endoderm, e.g., porifera and Asterias (Pentaceros) is a genus of the Asteriidae
coelenterate. Triploblastic animals have three family of sea stars. Ophiura albida is a species of
germinal layers – outer ectoderm, middle mesoderm brittle star in the order Ophiurida of Echinodermata
and inner endoderm, e.g., platyhelminthes, phylum.
aschelminthes, annelida, arthropoda, molluscs, 79. (c) Balanoglossus is a deuterostome, and resembles the
echinodermata and chordata. Ascidians or sea squirts, in that it possesses
72. (a) Locusts are the swarming phase of certain species branchial openings, or "gill slits". It has notochord
of short-horned grasshoppers in the family Acrididae. in the upper part of the body and has no nerve chord.
Scorpions are predatory arthropod animals having 80. (b) Cnidoblast are use for anchorage, defense and for
eight legs and are easily recognised by the pair of the capture of prey.
grasping pedipalps and the narrow, segmented tail. 81. (a) The given figures (A, B and C) are respectively
Prawn is an arthropod (invertebrate having jointed hippocampus, catla (both are bony fishes) and
limbs and a segmented body with an exoskeleton salamander (amphibian). Hippocampus have a
EBD_7209
Hints & Solutions S-21

skeleton made up of bony plates, they use gills to chambered heart. Ascaris and Ancylostoma are
breath and have an inflatable bladder to help regulate segmented roundworms.
their buoyancy in the water. Catla is a member of 92. (b) Prawn, scorpion and locusta belong to phylum
ostoichthyes and possesses air bladder to regulate Arthropoda. Arthropoda is the largest phylum of
buoyancy. animalia, and includes the insects arachnids,
82. (a) The given figures (A, B, C and D) are respectively crustaceans and others. It is a group of animals with
chameleons, crocodile, turtle and cobra. Reptiles are features including bilaterally symmetrical,
the first true land vertebrates and dominant in triploblostic tube within tube plan, organ system
Mesozoic era. They are cold blooded vertebrates. level of organization and metemerically segmented
Heart in reptiles are usually three chambered but four body.
chambered in crocodiles. 93. (a) Medusa and polyp are the stages in the life cycle of
83. (b) Animals (like annelida and amphibia) that are cnidarians. Medusa is a sexual free swimming form
bilaterally symmetric have mirror symmetry in the and polyp is asexual form. The jellyfish have both a
sagittal plane, which divides the body vertically into medusa and polyp stage in their life cycle, but the
left and right halves, with one of each sense organ sea anemones do not have the medusa stage and
and limb pair on either side. Bilateral symmetry spend the life cycle as polyps.
developed due to cephalization. 94. (a) The segmentation of the annelids allows more
84. (a) Germinal layers marked as A, B, C and D is mesoglea, complex coordinated movement.
ectoderm, endoderm and mesoderm respectively. 95. (d) For transition from aquatic to terrestrial life, the
"A" label represent Mesoglea layer, which is development of feathers for insulation was not
undifferentiated and present in between the required. The amphibians and reptiles do not have an
ectoderm (B) and the endoderm (C). insulation layer and the mammals have hair for
85. (c) The given figure shows the structure of cnidoblast. insulation.
Cnidoblast is the characteristic feature of porifera 96. (b) The most unique mammalian characteristic is the
phylum. A cnidoblast (also known as a cnidocytes presence of milk producing glands (mammary glands)
or nematocyte) is an explosive cell containing one by which the young ones are nourished. The skin of
giant secretory organelle that defines the phylum mammals is unique in possessing hair.
Cnidaria (corals, sea anemones, hydrae, jellyfish, etc.). 97. (c) Penguin, kiwi and ostrich all belong to class Aves
They are used for prey capture and defense from (i.e. birds) under phylum chordata and they do not
predators. give birth to their young ones. They are oviparous
86. (b) The given figure (Nereis) is an example of annelida. while kangaroo, hedgehog, dolphin and loris, all
Annelida is a group commonly referred to as belong to class mammalia and are viviparous.
segmented worms, and they are found worldwide 98. (c) Silverfish, scorpion, dragonfly and prawn belong to
from the deepest marine sediments to the soils. phylum arthropoda. Their body is covered by
Circulatory system is of closed type. chitinous exoskeleton and have jointed appendages.
87. (c) Octopus belongs to mollusca. Body of octopus is 99. (b) Loligo, Sepia and octopus are examples of phylum
covered by calcareous shell and unsegmented with Mollusca.
a distinct head, muscular foot, and visceral hump. Enterobius is an example of nematode. Pennatula is
88. (c) Figure c (scorpion) is an example of arthropoda. a colonial coral, belongs to coelenterate phylum.
Arthropoda contains respiratory organs like, gills, Bonellia (the green spoon worm) is a marine worm
book gills, book lungs or tracheal system. (phylum Echiura) noted for displaying exceptional
89. (d) The given figure shows the examples of Mollusca. sexual dimorphism and for the biocidal properties of
These animals are triploblastic and bilaterally a pigment in its skin.
symmetrical in nature. They are usually dioecious 100. (a) Hemichordates have now been placed with the non-
and oviparous with indirect development. chordates because true notochord is absent in them
90. (b) Animal 2 and 4 are bilaterally symmetrical. Bilateral but a buccal diverticulum is present in pre-oral region
symmetry is a symmetrical arrangement, as of an which is often called stomochord.
organism or a body part, along a central axis, so that 101. (a) In animals, carbohydrates are stored in the form of
the body is divided into equivalent right and left glycogen while in plants, carbohydrates are stored
halves by only one plane. in the form of sucrose. So, option (a) belongs to
91. (d) Sea horse and flying fish are cold blooded animals. kingdom Plantae.
Ornithorhyncus is oviparous. Crocodile has four 102. (d) An arthropod has a segmented body covered by an
exoskeleton made from chitin and other chemicals.
S-22 Biology
This exoskeleton serves as protection and provides belongs to phylum Hemichordata which was earlier
places for muscle attachment. Arthropods must moult considered as a sub-phylum under phylum chordata.
because their exoskeletons do not grow with them. But, now it is placed as a separate phylum under
The body feature from which the phylum takes its non-chordata because notochord and post-anal tail
name is the jointed appendages, which include are absent in it.
antennae and mouthparts as well as walking legs. 111. (b) The animal found by the boy belongs to mollusca
103. (b) Trachae act as passage of air during respiration in phylum. Mollusca are the second largest phylum after
both cockroach and mammals. In cockroach, the arthropoda and include predominantly marine
cuticular lining is spirally thickened forming taenidia animals. They are triploblastic, bilaterally symmetrical,
which prevents the trachael tubes from collapsing. schizocoelic and unsegmented protostomes. They
In mammals, cartilaginous ring supporting the walls have moist skin, a complete digestive tract, a ventral
of the trachae prevent their collapsing. nerve cord, and had gone through torsion.
104. (d) The sub-phylum vertebrata (or craniata) have a well- 112. (a) Porifera is commonly referred to as sponges. They
developed nervous system that is differentiated into are multicellular organisms that have bodies full of
brain and spinal cord. Brain is protected by a brain pores and channels allowing water to circulate
box called cranium, so they are also called as craniata. through them, consisting of jelly-like mesohyl
105. (c) The organism attached to the substratum possess sandwiched between two thin layers of cells.
radial symmetry in all vertical planes. All the animals 113. (b) Pheretima belongs to annelida phylum. Annelida is
belonging to cnidaria (e.g., jellyfish) and a group commonly referred to as segmented worms,
echinodermata (e.g., and they are found worldwide from the deepest marine
starfish) are radially symmetrical, and typically sediments to the soils in our city parks and yards.
sessile in their adult form. In radial symmetry, the 114. (c) Butterfly, Nereis, scorpion and pila are animals which have
parts in an organ or organism when cut through the a fluid filled body cavity with a complete lining derived
centre in any direction produces two halves that are from mesoderm, Hence, called as coelomate animals.
mirror images of each other. 115. (c) Interstitial cell refers to any one of a number of
106. (b) Sponges may have calcareous or siliceous spicules. different types of cells characterized by their
All sponges are not marine, some are freshwater interstitial nature (i.e., their interposition between
living also. Sponges may be asymmetrical or other cells that were usually characterized earlier or
bilaterally symmetrical, besides being radially more completely).
symmetrical. So, these characters are with exception. Cnidocytes is present in coelenterates as an organ
The character without exception is the regenerative used for prey capture and defense from predators.
power of sponges. Choanocytes are flagellated cells, present in porifera
All sponges have a good power of regeneration. and function as the sponge's digestive system.
They can regrow any part of the body lost or cut off. Gastrodermal cells are present in cnidarians and helps
Small fragments can grow into a complete sponge. in digestion.
107. (a) Diaphragm is a membrane that separates thoracic 116. (c) (i) Terrestrial or aquatic animals having organ
cavity from abdominal cavity. It is present only in system level of organization
mammals. All other chordates do not have diaphragm (ii) Bilateral symmetrical and coelomate animals
as their body cavity is not divided into thoracic and and possesses three germinal layers.
abdominal cavities. (iii) A file like rasping organ called radula is present.
108. (c) Four chambered heart in birds indicates their reptilian Radula is an anatomical structure that is used
ancestry. by molluscs for feeding, sometimes compared
109. (d) Exoskeleton is probably the most responsible for the rather inaccurately to a tongue.
great diversification of insects on land. An (iv) Usually dioecious and oviparous animals.
exoskeleton is the external skeletons that supports (v) Examples include Pila, Octopus, and Dentalium.
and protects an animal's body, in contrast to the
Chapter 5 : Morphology of Flowering Plants
internal skeleton (endoskeleton) of, for example, a
human. Examples of exoskeleton animals include 1. (b) The direct elongation of the radicle leads to the
insects such as grasshoppers and cockroaches, and formation of primary roots which grows inside
crustaceans such as crabs and lobsters. the soil. The radicle is the first part of a seedling
110. (b) Balanoglossus is a connecting link between (a growing plant embryo) to emerge from the seed
invertebrates and non-invertebrates. Balanoglossus during the process of germination. The radicle is
the embryonic root of the plant, an d gr ows
EBD_7209
Hints & Solutions S-23

downward in the soil (the shoot emerges from differentiated as the node. The space between two
the plumule). successive nodes is called internode.
2. (b) Tap root or primary roots develop from the radicle. It 15. (b) Underground stems are non green stems that may
forms lateral branches or secondary roots which are take part in perennation, store food or help in
further branched to form tertiary roots. This type of vegetative propagation. Underground stems of
root system is seen in the mustard plant. potato, ginger, turmeric, zaminkand, Colocasia are
3. (a) Fibrous root system is found in monocotyledonous modified stem to store food in them.
plants. In monocotyledons, primary root is short lived 16. (d) Tendrils are spirally coiled, thread - like sensitive
and replaced by a large number of roots which structures which develop from axillary buds and can
originate from the base of the stem and constitute coil around a support and help the plant in climbing.
the fibrous root system. E.g., cucumber, pumpkins, water melon and
4. (c) Root developing from any part of the plant other than grapevines.
the radicle is called adventions roots. Adventitious 17. (b) Thorn is a stiff, sharp-pointed woody projection on
roots are found in some plants like grass, Monstera the stem or other part of a plant. Thorns are found in
and banyan tree. many plants such as Citrus, Bougainvillea. They
5. (a) Root consists of four regions: root cap, region of protect plants from grazing animals.
maturation, region of elongation and region of 18. (c) A lateral branch with short internodes and each node
meristematic activity. Root hairs develop from the bearing a rosette of leaves and a tuft of roots is found
region of maturation. Region of maturation is the in aquatic plants like Pistia and Eichhornia.
area of origin of lateral roots. Root hair region is 19. (a) In mint and jasmine, a slender lateral branch arises
called piliferous zone. This root hair absorbs water from the base of the main axis and after growing
and minerals from the soil. aerially for some time arch downwards to touch the
6. (b) Water absorption takes place through root hairs. ground.
The latter are present only in the maturation zone. 20. (d) Lamina on the leaf blade is usually flattened green
7. (b) In maize, Pandamus, etc., adventitious roots develop photosynthetic part consisting leaf veins and
from the lower nodes of the stem to provide veinlets. Its upper surface is called adaxial while the
mechanical support and are called stilt roots. lower surface is called abaxial. The shape margin,
8. (c) Some plants are profusely branched and branches apex, surface and extent of incision of lamina vary in
are thick and heavy. From these branches, roots arise different leaves.
and hang downwards in the air and later penetrate the soil 21. (a) Leaves of dicotyledonous plants possess reticulate
and function as prop (for support) to the branch, e.g., Ficus venation while parallel venation is the characteristics
bengalensis (Banyan). of most monocotyledonous. In reticulate venation,
9. (c) In some plants, roots arise from lower nodes of stem the main veins of leaf form numerous irregular
and enter the soil and become stronger. Such roots branches and as a result a net like arrangements is
are called shift roots. They protect the plant against formed. Reticulated venation is the most common
winds, e.g., sugarcane, maize, screwpine. vein formation in leaves. It can be found in the leaves
10. (a) Pneumatophores or respiratory roots are erect, short of maple trees, oak trees and rose bushes.
root showing vertical and negatively geotrophic (grow In parallel venation veins are arranged parallel to
in an upward direction). It protrudes some distance each other.
above substratum and occurs in certain halophytes, 22. (d) Pinnate are feather like leaves. The segmentation of
which grow in saline marshes (mangroves). the leaf-blade is towards the mid-rib, so that the
11. (c) Pneumatophores or respiratory roots are short leaflets are borne laterally on midrib or rachis.
vertical and negatively geotrophic (grow in upward Example, Neem.
direction). Examples, – Rhizophora, Heritiera and 23. (d) The arrangement of mature leaves on the stem or its
Avicennia. branches is called phyllotaxy. These are meant for
12. (d) A root differs from stem, without any exception, by getting maximum amount of light for photosynthesis.
the presence of unicellular root hairs and the absence 24. (d) In alternate (or spiral) type of phyllotaxy, only one
of nodes and internodes. leaf is borne at each node and leaves are arranged
13. (a) Stem is generally green when young and later often alternatively giving a spiral form. Example chinarose,
become woody and dark brown. mustard and sunflower.
The stem bears buds which may be terminal or axillary. 25. (a) In onion and garlic, scale - leaves store food and
14. (b) A node is formed at the place from where a leaf arises, water and are, therefore, thick and fleshy.
i.e., the place of origin of a leaf on the stem apex is 26. (a) The flower is the reproductive unit in angiosperms
S-24 Biology
and is meant for sexual reproduction. Flower is the 36. (c) Cauliflower has the scientific name Brassica oleracea
seed-bearing part of a plant, consisting of belonging to the variety botrytis.
reproductive organs (stamens and carpels) that are 37. (a) Botanical name of banana is Musa paradi-sica.
typically surrounded by a brightly coloured corolla 38. (a) Bulb of Allium cepa is a modified stem.
(petals) and a green calyx (sepals). 39. (d) When stamens are attached to the petals, they are
27. (b) When both the essential whorls are present in the epipetalous as in brinjal, or epiphyllous when
same flower, it is described as bisexual, e.g., attached to the perianth as in the flowers of lily.
cruciferae, malvaceae etc. 40. (d) Calyx or sepals are the outermost lower most non-
28. (a) On the basis of symmetry, flower may be essential but protective whorl of the flower. Sepals
actinomorphic (radial symmetry) or zygomorphic are green, leaf like and protect the flower in bud stage.
(bilateral symmetry). In actinomorphic symmetry, When the sepals are free from one another, the calyx
flower can be divided into two equal halves in any is said to be polysepalous (mustard) and when the
radial plane passing through the centre. Examples - sepals are wholly or partially united with one another
mustard, Datura and chilli. by their margins then the calyx is said to be
29. (a) Staminode is a sterile or abortive stamen, frequently gamosepalous.
resembling a stamen without its anther. 41. (c) Reticulate venation is the characteristic of dicots. In
30. (b) Pollen grains are produced within the anther of reticulate venation the main vein a of leaf forms
stamen. Anther is the pollen-bearing part at the upper numerous irregular branches and as a result a net
end of the stamen of a flower. Most anthers occur at like arrangement is formed.
the tip of a slender, stem-like filament and have two 42. (a) In parietal placentation, the ovules develop on the
lobes. Each lobe contains two pollen sacs. When inner wall of the ovary or on peripheral part.
pollen matures in the pollen sacs, the lobes of the 43. (b) Androecium (male whorl) is composed of stamens.
anthers burst open in the process known as Each stamen which represents the male reproductive
dehiscence to release the pollen. organ consists of a stalk or a filament or an anther.
31. (d) Gynoecuim or pistil is the female whorl that is 44. (a) A bud is present in the axil of petiole in both simple
differentiated into the ovary, style and stigma. and compound leaves, but not in the axil of leaflets
Androecium is the male reproductive organ of the compound leaf. In Australian Acacia, the
consisting of stamen. Each stamen is distinguishable petioles expand, become green and synthesize food.
into anther and filament joined by a connective. 45. (d) The zone of elongation region of a root increases
32. (a) During the post fertilization period, the ovules length of the root. The external cells possess the
develop into seeds (A) and the ovary matures into a power of absorption of water and minerals salts from
fruit (B). the soil. The root hair zone represents the zone of
Seed is a fertilized and ripened ovule and the differentiation or maturation because different types
characteristics of gymnosperms and angiosperms. of primary tissues differentiate or mature in this
Fruit is the part of a seed-bearing plant that contains region. Maize and sugarcane have stilt roots.
the fertilized seeds capable of generating a new plant. 46. (d) The cells of the elongation zone gradually
Fruit develops from the female part of the plant. differentiate and mature. Hence, this zone proximal
Apples, peaches, tomatoes, etc. are fruits. to the region of elongation, is called the region of
33. (c) Hilum is the scar on a seed marking the point of maturation. From this region some of the epidermal
attachment of the developing seed vessel to the fruit. cells form very fine and delicate, thread like structures
34. (d) Cotyledons and testa respectively are the edible parts called root hairs. Stems of maize and sugarcane have
in ground nut and pomegranate. Cotyledon is an supporting roots coming out of the lower nodes of
embryonic leaf in seed-bearing plants, one or more the stem called stilt roots.
of which are the first leaves to appear from a 47. (a) Statements - i, ii, iii and iv are not correct.
germinating seed. Testa is the outer layer of seed (i) Calyx and corolla are helping or accessory whorls
coat. It is thick, hard and leathery, whereas tegmen whereas andr oecium an d gynoecium are
is the inner layer of seed coat which is thin and reproductive organs of a flower.
papery. (ii) Actinomorphic flowers can be divided into two equal
35. (d) Tomato and tobacco both belong to the family radial halves in any radial plane and zygomorphic
Solanaceae. Solanaceae has some identifying flowers can be divided into two similar halves in one
characteristics-bicarpellary, syncarpous superior particular plane.
ovary, axile plancentation, fruit is berry or capsule. (iii) Flowers without bracts are termed as ebracteate and
EBD_7209
Hints & Solutions S-25

flowers with bract, reduced leaf found at the base of effect of gravity. Rhizome of ginger contains nodes,
the pedicel are called as bracteates. internodes and scaly leaves. Buds are emerges from
(iv) Parthenocarpic fruit is formed without fertilization of axils of scaly leaves. Response to light by plants is
the ovary. called phototropism. In this sense, shoot shows
48. (a) Statement (i) and (ii) are correct about leaf. positive phototropism and root shows negative
(iii) Leaves originates from the shoot apical phototropism.
meristems and arranged in an acropetal order. 58. (c) In fabaceae ovary is present. Placentation is marginal
(iv) Leaves are the most important vegetative with many ovules.
organs for photosynthesis. 59. (c) Pericycle is the outermost layer of stele. In dicot
49. (b) According to the position of gynoecium, the flowers stems, pericycle strengths the stem and provide
are of three kinds: perigynous, hypogynous and protection for the vascular bundles. In angiosperms
epigynous. The given statements describe the (dicots) pericycle gives rises to lateral roots
perigyny condition of flowers. contribute to the vascular cambium often diverging
In perigynous condition of flower, the calyx, corolla into a work cambium.
and androecium arises from the around of ovary. In 60. (b) Valvate, twisted, imbricate and vexillary are types of
these flowers the ovary is semi inferior. e.g, plum, aestivation.
rose and peach. A: Valvate aestivation - In this sepals or petals or
50. (a) The statements describe the phyllotaxy condition of tepals just touch one another without any
plants. Phyllotaxy is the arrangement of leaves on overlapping, e.g. Calotropis.
an axis or stem. Phyllotaxy facilitate the leaves to B: Twisted aestivation - In this one margin of each
obtain maximum light for photosynthesis. petal overlaps the margin of an adjacent petal and
51. (c) Fruit is a mature or ripened ovary, developed after the other margin being overlapped by margin of
fertilization. When pericarp is thick and fleshy, it is another adjacent petal, e.g. China rose, cotton.
differentiated into outer epicarp, middle mesocarp C: Imbricate aestivation: If the margin of sepals or
and inner endocarp. In mango and coconut, the fruit petals overlap one another but not in any particular
is known as a drupe. direction as in Cassia and Gulmohar, the aestivation
52. (c) Tulip, Gloriosa, Aloe and Asparagus belong to is called imbricate.
family liliaceae. The plumule and radicle are enclosed D: Vexillary aestivation - It is the characteristics
in sheaths are called coleoptile and coleorhiza aestivation of corolla of pea when posterior petal is
respectively. outermost. E.g, bean.
53. (b) Vexillary aestivation is found in papilionaceous 61. (c) 62. (d) 63. (a)
family. In cymose type of inflorescence, the main axis 64. (d) When the calyx, corolla and androecium arise below
terminates in a flower, hence is limited in growth. In the ovary, the flower is called hypogynous. In these
parietal placentation ovary is one chambered but it flowers the ovary is superior, e.g., mustard, China
becomes two cambered due to formation of the false rose and brinjal. In perigynous condition of flower,
septum. the calyx, corolla and androecium arise from the
54. (b) A leaf having a single or undivided lamina is called around of ovary. In these flowers the ovary is semi
simple leaf. The lamina can have different types of inferior, e.g., plum, rose and peach. When the calyx,
incisions, which may reach upto half (-fid), more than corolla and androecium arise from the top of the
half (-partite) or near the base or midrib (-sect). ovary, the flower is called epigynous. In these flowers
Depending upon the pinnate or palmate venation, the ovary is inferior, e.g., guava, cucumber and the
the incisions are known as pinnatifid, palmatifid, ray florets of sunflower.
pinnatipartite, palmatipartite, pinnatisect and 65. (c) Simple leaf is a leaf whose blade is not divided to the
palmatisect etc. midrib even though lobed. Simple is a leaf with entire
55. (b) Ginger is horizontal in position and generally lamina or when incised, the incisions do not touch
branched and producing aerial leaves or shoots the midrib. Testa is the outer layer of seed coat. It is
above ground and adventitious roots on lower side thick, hard and leathery. Cacti are succulent plant
in favourable season. Thus, shoot growth is not with a thick fleshy stem which typically bears spines,
effected by gravity. lacks leaves, and has brilliantly coloured flowers.
56. (c) Plants do propagate more vegetatively since they Pitcher plants and Venus fly trap are insectivorous
multiply faster vegetatively. plants. They have modified leaves to catch insects.
57. (b) Ginger is an example of rhizome (e.g. prostrate stem Garlic and onion have fleshy leaves which store food.
creeping horizontally under soil surface). It has no
S-26 Biology
66. (c) A: Gamosepalous: Sepals united 71. (d) In the given figure, the root tip shows their different
B: Polysepalous: Sepals free regions which are marked as A, B and C. The correct
C: Gamopetalous: Petals free labelling of A, B and C are region of maturation,
D: Polypetalous: Petals united region of elongation and region of meristematic
E: Epiphyllous: Stamens are attached to the perianth activity respectively.
as in flower of lily. 72. (a) Roots are modified in some plants to perform
F: Staminode: A sterile or abortive stamen, frequently functions other than absorption and conduction of
resembles a stamen without its anther. water and minerals. Figure A shows tap roots of
67. (d) Coleorhiza is the sheath that envelops the radicle in turnip, carrot and adventitious roots of sweet potato
certain plants and that is penetrated by the root in and figure B shows pneumatophores in Rhizophora.
germination. Endosperm is the part of a seed which Tap roots system is the characteristics of dicot
acts as a food store for the developing plant embryo, plants. Tap roots (develop from radicle) and
usually containing starch with protein and other adventitious root (develop from any part of the plant
nutrients. Grape is a parthenocarpic fruit. other than the radicle) get swollen and store foods.
Parthenocarpic fruit is formed without the fertilization Pneumatophore is a specialized structure developed
in the ovary. Mango is a single seeded fruit from the root in certain plants growing in swamps
developing from monocarpellary superior ovary. and marshes. These types of roots come out of the
Maize has a membranous seed coat. ground and grow vertically upwards. Pneumatophore
68. (b) Many plants belonging to the fabaceae family are helps to get oxygen for respiration.
sources of pulses (gram, arhar), edible oils (soyabean, 73. (a) Leaf is the main photosynthetic organ of plants. In
ground nut), dye (indigofera), fibres (sun hemps), the given figure of leaf, the part marked as A, B, C
fodder (Sesabania), ornamental (lupin, sweet pea), and D are lamina, axillary bud, stipule and leaf base,
and medicine (mulaithi). respectively.
69. (a) The figures show the types of placentation. Lamina or leaf blade is green and expanded portion
Placentation is the arrangement of the placenta or of the leaf. In the middle of the lamina, a strong vein
placentae in the ovary of a flower. The function of called midrib is present which extends from its base
placentation is to transfer nutrients from maternal or apex. Axillary bud is borne at the axil of a leaf and
tissue to a growing embryo. is capable of developing into a branch shoot or flower
A: Marginal placentation is found in monocarpellary cluster. Stipule is a small leaf-like appendage to a
ovary. In this ovary is unilocular and ovules arranged leaf, typically borne in pairs at the base of the leaf
along margin of unilocular ovary. Examples - pea, stalk. Leaf base is the lower part of the lamina, and is
Clitoria, groundnut. attached to petiole or stem.
B: Axile placentation is found in bi- or multicarpellary 74. (b) In racemose inflorescence, the main axis continues
and multilocular ovary. Ovules are arranged along to grow. The flowers are borne laterally in an acropetal
the central axis of placenta and the number of succession, which means the older flowers are at the
chambers corresponds to the number of carpels. base and younger flowers are near the apex. E.g.,
Examples - lemon, tomato, Hibiscus, cotton. radish, mustard, Amaranthus.
C: Parietal placentation is found in bi- or In cymose inflorescence, the axis terminates in a
multicarpellary ovary but unilocular. Ovules are flower, hence, is limited in growth. The flowers are
arranged along periphery or the inner wall of the borne in a basipetal order, which means older flowers
ovary, and number of placentae corresponds to the are at the apex and younger flowers are near the
number of carpels. Examples, Cucurbita, Argemone. base. E.g. cotton, jasmine, Calotropis.
D: Fr ee central placentation is found in 75. (a) Aestivation is the arrangement of petals and sepals
multicarpellary, syncarpous ovary. Ovules are borne with respect to one another in a floral bud before it
along the central axis which is not connected with opens. The given figure shows different types of
the ovary wall by septum. Example, Dianthus roam aestivation. Figures A, B, C and D show valvate,
Primrose. twisted, imbricate and vexillary aestivation.
E: Basal placentation is found in monocarpellary but In valvate aestivation, sepals or petals or tepals just
unilocular. In this placentation, the placenta develops touch one another without overlapping, e.g.
at the base of the ovary and a single ovule is attached Calotropis. In twisted aestivation, one margin of
to it. Examples - sunflower, marigold. each petal overlaps the margin of an adjacent petal
70. (d) and the other margin is overlapped by margin of
another adjacent petal, e.g. China rose, cotton. If the
EBD_7209
Hints & Solutions S-27

margin of sepals or petals overlap one another but Components of whorls are concentrically arranged
not in any particular direction as in Cassia and on a thalamus.
Gulmohar, the aestivation is called imbricate. Vexillary 80. (a) In mango and coconut, the fruit is known as drupe.
is the characteristics aestivation of corolla of pea Drupe develops from monocarpellary superior ovaries.
when posterior petal is outermost. In mango, the pericarp is differentiated into an outer
76. (a) Based on the position of floral parts on thalamus, thin epicarp, middle fleshy edible mesocarp and an
flowers are divided into three types shown in the inner stony hard endocarp. In coconut, the mesocarp
figures. Figures A, B and C are respectively is represented by the fibrous part.
hypogynous, perigynous and epigynous flower. 81. (b) In the given figure of dicotyledonous seeds, the
When the calyx, corolla and androecium arise below structure marked as A, B, C, D and E are respectively
the ovary, the flower is called hypogynous. In these hilum, micropyle, plumule, cotyledon and radicle.
flowers the ovary is superior. e.g., cruciferae, liliaceae. Hilum is the scar on a seed marking the point of
In perigynous condition of flower, the calyx, corolla attachment of the developing seed vessel to the fruit.
and androecium arise around the ovary. In these Micropyle is a small opening or pore present just
flowers the ovary is semi inferior. e.g., Saxifraga. below the hilum. It is the way of entry of water into
When the calyx, corolla and androecium arise from the seed. Plumule is the developing bud of a plant
the top of the ovary, the flower is called epigynous. embryo, situated above the cotyledons and
In these flowers the ovary is inferior. e.g., umbiliferae, consisting of the epicotyl and immature leaves.
compositae. Cotyledon is an embryonic leaf in seed-bearing
77. (a) The given figure shows types of compound leaves. plants, one or more of which are the first leaves to
A compound leaf consisting of several or many appear from a germinating seed. Radicle is the first
distinct parts (leaflets) is joined to a single stem. They part of a seedling (a growing plant embryo) to emerge
are of two types - pinnately (figure A) and palmately from the seed during the process of germination.
compound leaves (figure B). The radicle is the embryonic root of the plant, and
In pinnately compound leaves, a number of leaflets grows downward in the soil (the shoot emerges from
are present on a common axis, the rachis, which the plumule).
represents the midrib of the leaf as in neem. In 82. (c) In the given figure of monocotyledonous seeds, the
palmately compound leaves, the leaflets are attached structure marked as A, B, C, D and E are respectively
at a common point i.e. at the tip of petiole as in silk endosperm, embryo, scutellum, coleoptile and
cotton. coleorhiza. Endosperm is the part of a seed which
78. (a) Phyllotaxy is the arrangement of leaves on a stem or acts as a food store for the developing plant embryo,
axis. Phyllotaxy is usually of three types which are usually containing starch with protein and other
shown in the given figures A (alternate), B (opposite) nutrients.
and C (whorled). Embryo is the part of a seed which develops into a
In alternate type, one leaf is borne at a node and plant, consisting (in the mature embryo of a higher
leaves are arranged alternately giving a spiral form, plant) of a plumule, a radicle, and one or two
e.g, mango, mustard and tobacco. In opposite cotyledons. Scutellum is the large shield like
arrangements, each node give rise to two leaves, cotyledon of the embryo of certain monocots. It is
arranged opposite to each other, e.g, Calotropis and specialized for the absorption of food from the
guava plants. In whorled arrangement more than two endosperm. Coleoptile is the first leaf above the
leaves are formed from each node, e.g, Nerium. ground, forming a protective sheath around the stem
79. (d) In the given figure of flower, the structure marked as tip. It surrounds the plumule. Coleorhiza is the sheath
A, B, C and D are respectively gynoecium, stamen, that envelops the radicle in certain plants and that is
ovule and thalamus. penetrated by the root in germination.
Gynoecium constitutes the inner essential whorl 83. (b) The given plants A (Pisum sativum), B (Solanum
flower comprising carpels. Carpel is the unit of nigrum) and C (Allium cepa) belong to fabaceae,
gynoecium. Each carpel contains ovary and stigma. solanacea and liliaceae families respectively.
Ovules are produced within ovary. Stamen is the 84. (b) Figure A, B and C are respectively tap root, fibrous
male fertilizing organ of a flower, typically consisting root and adventitious root. The root branches of the
of a pollen-containing anther and a filament. Ovule primary root are called as Taproot system. The
is the part of the ovary of seed plants that contains monocotyledons will have the primary root as short-
the female germ cell and after fertilization becomes lived and has many numbers of small lateral roots.
the seed. Thalamus is an expanded tip of the pedicel. These roots develop at the stem base and they form
S-28 Biology
a root system called as fibrous root system, which is piliferous zone. In cell maturation zone, secondary
observed in the wheat plant (monocot). In some growth takes place. It is the area of origin of lateral
plants like banyan tree, monstera, and grass, parts roots.
of the plant other than radicle will be responsible for 91. (c) Underground stems of potato, ginger and Colocasia
the development of root system called as are modified to store food in them whereas leaves of
adventitious root system. certain insectivorous plants such as pitcher plant,
85. (a) All the functions are carried out by the root system venus-fly trap are modified leaves.
of the parts (marked as 5 and 6) of flowering plants. 92. (b) The arrangements of flowers on the floral axis
The flowering plant consists of an axis, root system (peduncle) are called as inflorescence. Most
and shoots system. The flowering plants consist of prominent function of the inflorescence is the
a long cylindrical axis which is differentiated into an formation of more flowers.
underground root system. Root is the descending, 93. (c) The character of flower which is represented by floral
non-green, underground part lacking nodes, formula but not by the floral diagram is the position
internodes, leaves and buds. Root system consists of gynoecium.
of primary roots (5) and secondary roots (6). The 94. (d) Gram and peas are exalbuminous (non-endospermic)
main functions of the root system are absorption of seeds because they usually store reserve food
water and minerals from the soil, providing a proper materials in cotyledons. In these seeds, the
anchorage to the plant parts, storing reserve food endosperm is used up and not present in mature
material and synthesis of plant growth regulators. seeds.
Shoot system is an aerial system, usually above the 95. (c) Morphologically, seed (ripened ovule) is the
soil and originates from the plumule. It consists of integumented mature, megasporangium which is
flower (1), fruits (2), stem (3), leaves (4) and branches. developed from a fertilized ovule and is with an
86. (c) The flower is the most characteristic structure of the embryo (future plant, enclosed by seed coat.)
angiosperms. It is a complex unit consisting typically 96. (c) Floral features of family solanaceae
of those parts which directly or indirectly contribute Inflorescence: Solitary and axillary
to the process of reproduction. Flowers: Actinomorphic, bisexual flowers
87. (c) The major part of the grain is occupied by a large Calyx: Calyx is composed of five sepals that are united
endosperm which is rich in starch. The endosperm and persistent. Aestivation is valvate.
has one to three layered peripheral protein layer Corolla: Corolla consists of five united petals with
called aleurone layer which separates the embryo valvate aestivation.
with endosperm. Androecium: It consists of five epipetalous stamens.
88. (d) Fibrous root system is better adopted than tap root Gynoecium: It consists of bicarpellary, syncarpous
system for anchorage of plants to soil. In fibrous superior ovary with axile placentation.
root system, the roots originate from the base of the Fruits: Berry
stem. E.g, wheat plant. Tap root system is the primary Seeds: Numerous, endospermous
root which is short lived and is replaced by a large
Floral formula: Å K(5) C(5).A5 G(2)
number of roots (called fibrous root).
89. (c) Leaf is a green, flattened outgrowth of the plant 97. (c) Liliaceae is the characteristics of monocotyledonous
arising from the node of the stem and is specialized plants. Floral characters of this family are:
to perform the process of photosynthesis. Therefore, tricarpellary, actinomorphic, polyandrous, superior
leaf is known as the kitchen or food factory of the ovary, axile placentation.
plant. 98. (c) Hypogynous flowers (Superior ovary): In this, the
90. (a) The correct sequence of the zones seen in the root ovary occupies the highest position on the thalamus,
tip is: CBEAD i.e. root cap zone, zone of meristems, while other floral parts are situated below it. In such
zone of elongation, root hair zone and zone of flowers, the ovary is superior. E.g., China rose,
maturation. mustard, etc.
Root cap zone protects the tender apex of the root as Perigynous flowers (Half inferior ovary): In this,
it makes its way through the soil. Zone of meristems the ovary is situated at the centre and other floral
have cells of very small size, thin walled and with parts are arranged on the rim of the thalamus. The ovary
dense protoplasm. They divide repeatedly. The cells here is said to be half inferior. E.g., plum, rose, peach.
in zone of elongation undergo rapid elongation and Epigynous flowers (Inferior ovary): In this, the
enlargement and are responsible for the growth of thalamus grows around the ovary fusing with its
the roots in length. Root hair region is called the wall. The other floral parts are present above the
EBD_7209
Hints & Solutions S-29

ovary. Hence, the ovary is said to be inferior. E.g., origin is called a tissue. A plant is made up of different
flowers of guava and cucumber. kinds of tissues. The complex tissues are, however,
99. (b) In fabaceae family, flower is zygomorphic, imbricate composed of different kinds of cells and they perform
aestivation and polypetalous. different types of functions.
100. (c) Marginal placentation is a placentation with ovules 2. (c) Apical, intercalary and lateral meristems are
borne on the wall along the ventral suture of a simple differentiated on the basis of position. Apical meristem
ovary. Gram, arhar, sunhemp, moong, pea & lupin belong is situated at the shoot apex and the root apex.
to Fabaceae family that bears marginal placentation. Intercalary meristem is present at the base of internodes,
101. (c) In roots, node and internodes are absent. e.g., in grasses or at the base of leaves e.g., in Pinus or
102. (c) The given floral formula in the question belongs to at the base of nodes, e.g., mint. Lateral meristems are
the family fabaceae. This family was earlier called present along the lateral sides of stem and roots.
Papilonoideae, a subfamily of family leguminosae.
103. (d) Root system generally grows beneath the ground 3. (d) Intercalary meristem develops between regions of
into the soil. Functions of root system are as follows: mature or permanent tissue (at the base of the grass
– It provides great anchorage and support to the leaf). The cells of this tissue possess the ability to
plant. Huge trees such as mango, red wood stand divide and produce new cells, as do apical and lateral
erect due to the root. meristems. Intercalary meristem helps the bamboo
– The root hairs absorb nutrients, water and and grasses to elongate.
oxygen from the soil and conduct them to the 4. (a) Lateral meristem divides only periclinally or radially
upper parts of the plant. and is responsible for increase in girth or diameter. It
– Some of the tap roots are specially modified for includes vascular cambium and cork-cambium.
storage of carbohydrates and water. 5. (c) Divisions of cells in both primary as well as
104. (b) Region of elongation lies above the region of secondary meristems, the newly formed cells become
meristematic activity. Cells in this region undergo structurally and functionally specialized and lose the
rapid elongation and enlargement and are ability to divide. Such cells are termed as permanent
responsible for the growth of the root in length. or mature cells and constitute the permanent tissues.
105. (a) Region of maturation lies above the region of
elongation. The cells of the elongation zone gradually 6. (a) The apical meristem, or growing tip, is a completely
differentiate and mature. Epidermal cells of this region undifferentiated meristematic tissue found in the
form delicate thread like root hair, which helps in the buds and growing tips of roots in plants. Its main
absorption of water and minerals from the soil. function is to begin growth of new cells in young
106. (b) X – embryo, Y – scutellum, C – radicle seedlings at the tips of roots and shoots (forming
The embryo is small and situated in a groove at one buds, among other things). The apical meristem of
end of the endosperm. It consists of one large and the root is present in all roots.
shield shaped cotyledon known as scutellum and a 7. (b) Axillary buds are present in the axils of leaves and
short axis with a plumule and a radicle. are capable of forming a branch or a flower.
107. (a) Calyx of the flower contains sepals which are green 8. (c) Apical meristem is a completely undifferentiated
in colour and look like leaf and protect the other meristematic tissue found in the buds and growing
whorls of the flower. Corolla is present inside the tips of roots in plants. Its main function is to begin
calyx and made up of petals. Petals are different in growth of new cells in young seedlings at the tips of
color and shapes and protects the other whorls roots and shoots (forming buds, among other
present inside it. It attracts insects for pollination things). The axillary bud is an embryonic shoot which
due to its color. Androecium is present inside the lies at the junction of the stem and petiole of a plant.
corolla and made up of stamens. Each stamen As the apical meristem grows and forms leaves, it
contains filament, anther and a connective. leaves behind a region of meristematic cells at the
108. (d) The aleurone layer is the outermost layer of the node between the stem and the leaf.
endosperm, followed by the inner starchy endosperm. 9. (a) Collenchyma is living mechanical tissue, found
This layer of cells is sometimes referred to as the beneath the epidermis (i.e., hypodermis) of
peripheral endosperm. It lies between the pericarp herbaceous dicot stem.
and the hyaline layer of the endosperm. The intercellular spaces in this tissue are absent
Chapter 6 : Anatomy of Flowering Plants because in intercellular spaces at the corner of cells
thickenings of cellulose and pectin develop due to
1. (d) A group of structurally similar or dissimilar cells that which the cell wall become rigid and thick at corners.
perform a common function and have a common
S-30 Biology
10. (b) Parenchyma is the commonest simple tissue. This is towards the periphery of the organ. This type of
the most primitive tissues from which other tissues primary xylem is called endarch.
are evolved and hence also called as fundamental 21. (b) Anatomically fairly old dicotyledonous root is
tissue. The main function of parenchyma is storage distinguished from the dicotyledonous stem by
of food. position of cortex. Protoxylem is the first-formed xylem
Photosynthesis, respiration, secretion, assimilation, developing from procambium and consisting of narrow
etc. are some of the important processes which occur cells with annular, spiral, or scalariform wall thickenings.
in parenchymatous cells. 22. (a) Intercalary meristems are derived from apical
11. (d) Sclerenchyma consists of long, narrow cells with thick meristems and separated from the same by permanent
and lignified cell walls having a few or numerous cells. They are responsible for localised growth.
pits. They are usually dead and without protoplasts. 23. (c) In monocot leaves, the mesophyll cells are
They are simple dead mechanical tissue occuring in undifferentiated.
mature organs of plants body. They are chiefly 24. (c) Lignin is the important constituent in the cell wall of
distributed in cortex, pericycle, xylem and phloem xylem. Xylem consists of four different types of
region. elements- tracheids, vessels, and xylem parenchyma
12. (c) The collenchyma occurs in layers below the and xylem fibres. Wall of tracheids are highly
epidermis or outer layer of cells in young stems and thickened by the deposition of lignin, except at
in leaf veins in dicotyledonous plants. Collenchyma certain points called as pits.
cells are elongated cells with irregularly thick cell 25. (c) Heartwood differs from sapwood in having dead and
walls that provide support and structure. Their thick non-conducting elements. In old trees, the inner
cell walls are composed of the compounds cellulose region that comprises dead elements with highly
and pectin. lignified walls is called heartwood. Heartwood does
13. (a) Xylem is a complex permanent tissue mainly not conduct water but gives mechanical support to
responsible for conduction of water and minerals the stem. On the other hand, the peripheral region,
from the roots to the top of plants (unidirectional). which is lighter in colour are called sapwood. It is
14. (a) Bast fibres (phloem fibres) are sclerenchymatous involved in the conduction of water and minerals
dead cells that provide mechanical strength. They from root to leaf.
have thick wall with simple pits. 26. (b)
15. (a) The presence of vessels is a characteristic feature of 27. (c) By counting the number of annual rings at base
angiosperms. Vessels help in conduction of water stem of a tree the age of the tree can be determined.
and dissolved salts form roots to the different parts 28. (c) Cork cambium and vascular cambium are lateral
of the shoot and provides mechanical support. meristem. Lateral meristems are responsible for
16. (c) A mature sieve element possess a peripheral increase in thickness of the axis.
cytoplasm and a large vacuole but lacks nucleus. 29. (b) Phellogen and phellem are respectively called as cork
Sieve tube consists of row of cells arranged one cambium and cork. Phellogen is defined as the
above the other to form long pipes. Sieve tube occurs meristematic cell layer responsible for the
in angiosperms. The functions of sieve tubes are development of the periderm. Cells that grow inwards
controlled by the nucleus of companion cells. from the phellogen are termed phelloderm, and cells
17. (b) In roots, the protoxylem lies towards periphery and that develop outwards are termed phellem or cork.
metaxylem lies towards the centre. Such arrangement 30. (b) Vascular cambium is located between the xylem and
of primary xylem is called exarch. the phloem in the stem and roots of a vascular plant,
18. (c) The xylem and phloem strands alternate with each and is the source of both the secondary xylem growth
other separated by parenchymatous cells. Such kinds (inwards, towards the pith) and the secondary
of vascular bundles are called radial and found mainly phloem growth (outwards).
in roots. 31. (a) In hypodermis or outer cortical cells, a layer becomes
19. (b) Pericycle in roots is active in the formation of root meristematic which is known as cork cambium or
branches or lateral roots. phellogen. Phellogen cuts off cells on both sides.
20. (a) The first formed primary xylem elements are called The cells cut off on outer side are phellem or cork
protoxylem and the later formed primary xylem is cells and cells cut off on inner side are phelloderm or
called metaxylem. In stems, the protoxylem lies secondary cortex.
towards the centre (pith) and the metaxylem lies 32. (a) Lenticels are some loosely arranged areas in the
EBD_7209
Hints & Solutions S-31

periderm formed due to rapid activity of phellogen. plant, which may be thickened by a cuticle. It
Lenticels are characteristics of woody stem. They consists of epidermis and epidermal outgrowth.
help in gaseous exchange and transpiration. Epidermis is the superficial layer covering the entire
33. (c) In monocotyledonous leaf, all the cells of mesophyll surface of the primary plant body. All the epidermal
are alike. Reticulate venation is absent. There is no cells are living (parenchymatous) and contain
differentiation of mesophyll into palisade and spongy vacuolated protoplasm. Stomata are the structures
parenchyma. Bulliform cells are present. present in the epidermis of leaves. Epidermal
outgrowths are of two kinds- trichomes and
34. (a) Sclereids (stone cells) are sclerenchymatous cells
emergences.
which are lignified, extremely thick walled so that the
lumen of the cells is almost obliterated. Sclereids are 41. (d) Xylem fibres have highly thickened walls and
most abundant in soft tissues like cortex, phloem, obliterated central lumens whereas xylem
medulla, fleshy fruits, seed coats and fruit walls. parenchyma are thinned wall and their cell walls are
made up of cellulose.
35. (d) All the statements are correct regarding Bulliform or
motor cells. They are large, bubble-shaped, empty 42. (c) The described statements are associated with the
colourless epidermal cells that occur in groups on phloem parenchyma. Phloem parenchyma is living
the upper surface of the leaves of many grasses. and has thin cell walls. These cells form the packing
Loss of turgor pressure in these cells causes leaves tissue between all the other types of cells. These
to "roll up" during water stress. cells stores compounds such as starch.
36. (a) The end walls of sieve tube elements are perforated 43. (b) It consists of simple tissues such as parenchyma,
in a sieve like manner to form the sieve plates. A collenchyma and sclerenchyma.
mature sieve element possesses a peripheral
44. (d) The described statements are associated with
cytoplasm and a large vacuole but lacks a nucleus.
trichomes. Trichomes are hair-like outgrowth from
37. (c) The stomatal aperture, guard cells and th e an epidermal cell of a plant, as a bristle, prickle, root
neighbouring (and encircling) cells is called a
hair, etc. They are ephemeral or persistent and their
stomatal complex or stomatal apparatus. Stomata
cell wall is made up of cellulose. Trichomes serve a
consist of an opening or stomatal pore, and two
variety of functions on the basis of their location.
kidney-shaped guard cells. The guard cells are used
As root hairs (and as leaf hairs in epiphytes),
for opening and closing the stomatal pore, in order
to regulate the evapotranspiration and gas exchange. trichomes absorb water and minerals. As leaf hairs,
These guard cells are kidney shaped in dicot and they reflect radiation, lower plant temperature, and
dumb-bell shaped in monocots. The epidermal cells reduce water loss. They also provide defense against
surrounding the guard cells are specialized and are insects.
known as subsidiary cells or accessory cells. Stomata 45. (a) Phloem transports food materials, usually from leaves
are often more common on the lower leaf surfaces. to the other parts of the plant. The companion cells
38. (b) Sclerenchyma is simple dead supportive tissue with are specialised parenchymatous cells which are
highly thick walled cells composed of cellulose or closely associated with sieve tube elements.
lignin with little or no protoplasm. Collenchyma are 46. (a) All the statements describe the anatomy of
present beneath the epidermis of young stem, petiole dicotyledonous root. Dicot root consists of a single
and midrib of leaves etc. in dicotyledonous plants.
layer of epiblema which bears unicellular root hairs.
Xylem parenchyma cells are living and this walled,
Endodermis is followed by one or more layers of
and their cell walls are made up of cellulose.
pericycle. Inner to pericycle lies radially arranged
Companion cells are specialized parenchymatous
vascular bundles. This arrangement keeps the xylem
cell, which are closely associated with sieve tube
elements. bundles in direct contact with the outer tissue of the
roots which conduct water absorbed by root hairs
39. (c) Xylem is composed of four different kinds of
to the inside. Xylem is exarch.
elements, namely, tracheids, vessels, xylem fibres and
xylem parenchyma whereas companion cells are 47. (b) The abaxial epidermis bears more stomata than the
present in phloem. adaxial epidermis. Mesophyll has two types of cells-
40. (d) All the statements are correct about epidermal tissue the palisade parenchyma and the spongy
system. It is the outer protective layer of cells of a parenchyma.
48. (a) Statement (i) and (iii) are correct about heartwood/
S-32 Biology
Duramen. pore. Guard cells are dumbbell shaped in monocots
Heartwood is the older, non-living central wood of a (e.g grasses) and bean shaped in dicots.
tree or woody plant, usually darker and harder than 56. (a) Lenticels are raised pores in the stem of a woody
the younger sapwood. The heartwood does not plant that allows gas exchange between the
conduct water but it gives mechanical support to atmosphere and the internal tissues. Parenchymatous
the stem. cells are usually present in cortex. The ground tissue
system is divided into three main zones- cortex,
Sapwood is the peripheral region of the secondary
pericycle and pith. The conjoint vascular bundles
xylem. It is lighter in colour and involved in the
usually have the phloem located only on the outer
conduction of water and minerals from root to leaf.
side of the xylem.
49. (c) The described statements are associated with spring
57. (a) Root hairs - Helps absorbs water and minerals from
wood or early wood. Spring woods are the softer
the soil.
more porous portion of an annual ring of wood that
58. (b) Fusiform initials are vertically elongated cells that
develops early in the growing season. It consists of produce xylem and phloem elements.
large thin-walled xylem cells.
Ray initial are isodiametric and produce
50. (d) All the given statements represent the anatomical parenchymatous rays in secondary xylem and
features of monocotyledonous stem. Monocot stem phloem.
is characterised by epidermis (2-3layered), 59. (a) The root apex and shoot apex are meristematic in
hypodermis and undifferentiated ground tissue stem. nature. These meristematic tissues are embryonic in
origin. It is primary in origin because it developes
Vascular strand is numerous and scattered. Vascular
from embryonic tissues. It is also primary in function
bundles are conjoint, collateral and closed. because it forms the primary structure of the plant
51. (d) Statements (ii), (iii) and (v) are correct. cell, the root apex and shoot apex, that live till the
(i) Uneven thickening of the cell wall is not the death of the whole plant. Hence, plants have the
characteristics of sclerenchyma. feature of indefinite growth.
(iv) A mature sieve elements are devoid of nucleus 60. (a) In woody trees, the central portion of stem is dark in
at maturity. colour. It is hard and tough due to deposition of
52. (b) The first formed primary xylem elements are called resins, tannins, gums and formation of tyloses. This
protoxylem and the later formed primary xylem is central hard portion is called heart wood. It is formed
called metaxylem. Phloem fibres (bast fibres) are made by secondary growth. Due to cambial activity
secondary xylem becomes non-functional and forms
up of sclerenchymatous cells.
heart wood or duramen. It is more durable and little
53. (b) The given statements are associated with
susceptible to attack of pathogens. The cambial
sclerenchyma plant tissue. Sclerenchyma cells are
activity continues in this region.
strengthening tissue in a plant, formed from cells
with thickened, typically lignified, walls. Such cells 61. (b) Vessels are more efficient for water conduction as
occur in many different shapes and sizes, but two compared to tracheids. Vessels resemble tracheids
main types occur: fibres and sclereids. Fibres are very much in structure and function. But unlike
greatly elongated cells whose long, tapering ends tracheids these are like long tubes arranged in vertical
interlock, thus providing maximum support to a plant. row formed of cylindrical cells arranged to end with
their end walls completely dissolved. These are also
They can be found almost anywhere in the plant
dead and lignified.
body, including the stem, the roots, and the vascular
bundles in leaves. Sclereids are extremely variable in 62. (b) In isobilateral leaves, the upper epidermis contains
shape and are present in various tissues of the plant specialized cells, i.e., bulliform or motor cells. They
such as the periderm, cortex, pith, xylem, and phloem. are highly vacuolate and can store water, if avail-
able. However, in case of water deficiency the
54. (a) Secondary cortex is also called phelloderm. Cork
bulliform cells lose water and become flaccid. As a
cambium. cork and secondary cortex are collectively
result the leaf gets rolled up to reduce the exposed
called periderm.
surface. The bulliform cells are also useful in the
55. (d) Guard cells are specialized cells in the epidermis of unrolling of leaf during its development.
leaves, stems and other organs that are used to
63. (a) Sieve tubes are the conducting elements of phloem
control gas exchange. The guard cells are produced
(a vascular tissue which conducts organic food in
in pairs with a gap between them that forms a stomatal
EBD_7209
Hints & Solutions S-33

plant body) which are elongated tubular channels soft and succulent, found chiefly in the softer parts
formed by end to end union of numerous cells. The of leaves, pulp of fruits, bark and pith of stems, etc.
septa between individual sieve tube cells or sieve It involves in photosynthesis, storage and secretion.
elements are bulged out. They are called sieve plates C - Collenchyma: They are simple, living and
possessing a number of perforations (sieve pores or mechanical tissue. They are present beneath the
sieve pits) and helps in conduction of food. epidermis of young stem, petiole and midrib of leaves
64. (d) A - Xylem vessel: They are a long straight chain etc. It provides mechanical support to the growing
made of tough long dead cells known as vessel ele- parts of the plant such as young stem and petiole of
ments or vessel members. They have a lignified cell a leaf.
wall and a large central cavity. Vessel members are D - Sclerenchyma: Sclerenchyma cells are the
interconnected through perforations in their com- principal supporting cells in plant tissues that have
mon walls. ceased elongation. These are chiefly distributed in
B - Xylem tracheids: Tracheid is an elongated, cortex, pericycle, xylem and phloem region. Based
tapering xylem cell having lignified, and pitted, intact on the size and shape, sclerenchyma cells are of two
types- fibres and sclereid.
walls which lacks perforations in the cell wall.
Tracheids have pits where the cell wall is modified E - Epidermal tissue: It is the outer protective layer of
into a thin membrane, across which water flows from cells of a plant, which may be thickened by a cuticle.
tracheid to tracheid. It consists of epidermis and epidermal outgrowth.
Stomata are the structures present in the epidermis
C - Xylem fibres: They have highly thickened walls
of leaves.
and obliterated central lumens. They may either be
67. (a) A - Cuticle: It is a protecting film covering the
septate or aseptate.
epidermis of leaves, young shoots and other aerial
D - Xylem parenchyma: These cells are living and plant organs without periderm. It consists of lipid
thin walled, and their cell walls are made up of and hydrocarbon polymers impregnated with wax,
cellulose. They store food materials in the form of and are synthesized exclusively by the epidermal
starch or fat and other substances like tannins. cells.
65. (d) A - Bulliform cells: They are large, bubble-shaped, B - Bulliform cells: They are large, bubble-shaped,
empty colourless epidermal cells that occur in groups empty colourless epidermal cells that occur in groups
on the upper surface of the leaves of many grasses. on the upper surface of the leaves of many grasses.
B - Pericycle: It is a thin layer of plant tissue between Loss of turgor pressure in these cells causes leaves
the endodermis and the phloem. Initiation of lateral to "roll up" during water stress.
roots and vascular cambium during the secondary C - Stomata: They are minute aperture structures on
growth takes place in these cells. plants found typically on the outer leaf skin layer,
C - Endarch xylem: When protoxylem lies towards also known as the epidermis. They consist of two
specialized cells, called guard cells that surround a
the centre (pith) and metaxylem lies towards the
tiny pore called a stoma.
periphery of the organ, it is called endarch xylem.
This type of xylem occurs in stems. D - Epidermis: It is the outer layer of tissue in a plant,
except where it is replaced by periderm. It is a single-
D - Exarch xylem: When protoxylem lies towards the
layered group of cells that covers plants leaves,
periphery and metaxylem lies towards the centre, it
flowers, roots and stems. It forms a boundary
is called exarch xylem. This type of xylem occurs in between the plant and the external environment.
roots.
68. (d) A - Stomata: They are minute aperture structures on
E - Bundle sheath cells: They are layer of cells in plants found typically on the outer leaf skin layer,
plant leaves and stems that forms a sheath also known as the epidermis. They consist of two
surrounding the vascular bundles. These cells are specialized cells, called guard cells that surround a
found in dicot leaf. tiny pore called a stoma. It helps in transpiration and
66. (b) A - Meristems: It is a region of plant tissue, found gaseous exchange. Transpiration is the transport of
chiefly at the growing tips of roots and shoots and water through an actual, vegetated plant into the
in the cambium, consisting of actively dividing cells atmosphere. Transpiration is an important part of
forming new tissue. the evapotranspiration process, and a major
B - Parenchyma: It is the cellular tissue, typically mechanism of the water cycle in the atmosphere.
S-34 Biology
B - Bark: Bark is the outermost layers of stems and and are shorter than fibres.
roots of woody plants. Plants with bark include trees, C - Tracheids: Tracheids are the more ancient
woody vines, and shrubs. Bark refers to all the tracheary elements, the main water conductive cells
tissues outside of the vascular cambium. It serves of the pteridophytes and the gymnosperms. They
as protection of stem against damage from parasites, are elongate cells with tapering ends. Their cell walls
herbivorous animals and diseases, as well as characteristically bear spiral (pteridiophytes), pitted
dehydration and fire. or bordered-pitted (gymnosperms) thickening. Water
C - Cambium: It is a cellular plant tissue from which moves from cell to cell through the side walls.
phloem, xylem, or cork grows by division, resulting D - Vessels: Vessel is a long cylindrical tube like
(in woody plants) in secondary thickening. structure made up of many cells called vessels
D - Cuticle: It is a protecting film covering the members.
epidermis of leaves, young shoots and other aerial E - Xylem parenchyma: Xylem parenchyma cells are
plant organs without periderm. It consists of lipid living and thin walled and their cell walls are made
and hydrocarbon polymers impregnated with wax, up of cellulose. They store food materials in the form
and are synthesized exclusively by the epidermal of starch or fats and other substances like tannins.
cells. In addition to its function as permeability barrier 72. (a) A: Lateral meristem is a meristem that is arranged
for water and other molecules (prevent water loss), parallel to the sides of an organ and that is responsible
the micro and nano-structure of the cuticle confer for increase in diameter of the organ. Fascicular
specialised surface properties that prevent vascular cambium, interfascicular cambium and cork
contamination of plant tissues with external water, cambium are the examples of lateral meristem.
dirt and microorganisms.
B: Apical meristem is a completely undifferentiated
69. (c) Spring wood is the part of an annual ring of wood, meristematic tissue found in the buds and growing
characterized by large, thin-walled cells, formed tips of roots in plants. Its main function is to begin
during the first part of the growing season. It is lighter growth of new cells in young seedlings at the tips of
in colour with low density. It produces a larger roots and shoots (forming buds, among other
number of xylem elements having vessels with wider things). During the formation of the primary plant
cavities. body, specific region of apical meristem produces
Autumn wood or late wood is the wood element dermal tissue, ground tissue and vascular cambium.
formed during winter when cambial activity is more. C: Bast fibres or phloem fibres are made up of
It is darker in colour with high density. Wood sclerenchymatous cells. These are generally absent
elements are lesser in amount and have narrow lumen. in primary phloem but found in secondary phloem.
70. (a) A - Stele: It is the central core of the stem and root of D: Sapwood is the peripheral region of the secondary
a vascular plant, consisting of the vascular tissue xylem. It is involved in the conduction of water and
(xylem and phloem) and associated supporting tissue. minerals from root to leaf.
B - Endodermis: It is an inner layer of cells in the 73. (c) In the given figure of root apical meristem, the part
cortex of a root and of some stems, surrounding a marked as A, B and C is respectively cortex,
vascular bundle.
protoderm and root cap.
C - Casparian strips: It is a band of cell wall material
The apical meristem is a completely undifferentiated
deposited on the radial and transverse walls of the
meristematic tissue found in the buds and growing
endodermis, and is chemically different from the rest
tips of roots in plants. Its main function is to begin
of the cell wall - the cell wall being made of lignin and
growth of new cells in young seedlings at the tips of
without suberin - whereas the Casparian strip is made
roots and shoots (forming buds, among other
of suberin and sometimes lignin.
things).
D - Bark: Bark is the outermost layers of stems and
roots of woody plants. Plants with bark include trees, 74. (a) In the given figure of shoot apical meristem, the parts
woody vines, and shrubs. marked as A, B and C are respectively leaf primordium;
shoot apical meristem and axillary bud (present in
71. (b) A - Fibres: Fibres are thick walled, elongated and
the axils of leaf and are capable of forming a branch
pointed cells, generally occurring in groups, in
or a flower). The shoot apical meristem is a population
various parts of the plants.
of cells located at the tip of the shoot axis. It produces
B - Sclereids: Sclereids is a thick-walled lignified lateral organs, stems tissues and regenerates itself.
sclerenchyma cell. Sclereids are variable in shape
75. (a) Figure A, B, C and D are respectively parenchyma,
EBD_7209
Hints & Solutions S-35

collenchyma, sclerenchyma fibres and sclerenchyma bundles are respectively radial, conjoint closed and
sclereid. conjoint open. When xylem and phloem are arranged
Parenchyma: It involves in photosynthesis, storage in the radius of the plant in alternative manner then it
and secretion. Collenchyma: It provides mechanical is called radial vascular bundle. Radial VBs are always
support to the growing parts of the plant such as closed and present in all types of roots. In conjoint
vascular bundle, xylem and phloem are situated at
young stem and petiole of a leaf. Sclerenchyma fibres:
the same radius of vascular bundles. Such VBs are
It provides mechanical strength to various plant
common in stems and leaves.
organs and specialised to tolerate stress of bending,
sheering, compression and pull. Sclerenchyma 80. (c) In the given figure of T. S of monocot root, the part
sclereid: It forms protective covering of testa in many marked as A, B, C, D, E, F, G, H and I are respectively
legume seeds. root hairs, epiblema, cortex, endodermis, pericycle,
phloem, protoxylem, pith and metaxylem.
76. (a) The given figures A (tracheid) and B (vessel) are
Monocot root do not undergo any secondary
types of xylem (C). Xylem is the vascular tissue in
growth. It consists of thin walled cells in the epiblema;
plants which conducts water and dissolved nutrients
unicellular root hairs, passage cells and casparian
upwards from the root and also helps to form the
thickenings in the endodermis; parenchyma cells in
woody element in the stem. It consists of tracheids,
the pericycle; conjuctive tissue; distinct pith and
vessels, parenchyma cells, and woody fibres.
radial vascular bundles with polyarch condition and
Tracheid is an elongated, tapering xylem cell having an exarch xylem. Cuticle and stomata are absent.
lignified, and pitted, intact walls which lacks 81. (d) In the given figure of T.S of dicot root, the parts
perforations in the cell wall. Tracheids have pits where marked as A, B, C, D, E and F are respectively cortex,
the cell wall is modified into a thin membrane, across epiblema, pith, endodermis, root hairs and pericycle.
which water flows from tracheid to tracheid. The cells
Dicot root consists of a single layer of epiblema which
die when mature, leaving only their lignified cell walls.
bears unicellular root hairs. Endodermis is followed
Tracheids are found in all vascular plants. It is
by one or more layers of pericycle. Inner to pericycle
adapted for conduction and support. lies radially arranged vascular bundles. This
Vessels are a long straight chain made of tough long arrangement keeps the xylem bundles in direct
dead cells known as vessel elements or vessel contact with the outer tissue of the roots which
members. The vessel cells, devoid of protoplasm, conduct water absorbed by root hairs to the inside.
are arranged end to end and the cell walls have Xylem is exarch.
disappeared. This makes a tube. They have a lignified 82. (b) In the given figure of dicot stem, the parts marked as
cell wall and a large central cavity. A, B,C, D, E, F, G, H and I are respectively epidermal
77. (c) In the given fig. of phloem tissue A, B & C are hair, epidermis, hypodermis, parenchyma,
respectively sieve tube, phloem parenchyma and endodermis, pericycle, vascular bundle, medullary
campanion cells. Companion cells (c) are specialized rays and medulla or pith. Dicot stems with primary
parenchymal cells adjacent to a sieve tube in the growth have pith in the centre, with vascular bundles
phloem of flowering plants. It is beliveved to regulate forming a distinct ring visible when the stem is
the flow of nutrients through the tube. viewed in cross section. The outside of the stem is
covered with an epidermis, which is covered by a
78. (d) Stomata are tiny pores, and composed of two bean
waterproof cuticle. The epidermis also may contain
shaped epidermal cells called guard cells which
stomata for gas exchange and multicellular stem hairs
enclose stomatal pore.
called trichomes. A cortex consisting of hypodermis
Stomata are mostly present in epidermal layers of (collenchyma cells) and endodermis (starch
leaves and in other aerial parts like young stems, containing cells) is present above the pericycle and
floral parts etc. Guard cells in dicots are kidney vascular bundles. Inner to endodermis is present a
shaped and in monocots are dumb bell shaped. The few layers of heterogenous pericycle. Vascular
guard cells may be surrounded by varying number bundles are conjoint, collateral and open. Medullary
of specialized epidermal cells called subsidiary cells rays are present in between vascular bundles.
or accessory cells. Cambium helps in secondary growth.
79. (a) Vascular bundles (VBs) is a strand of conducting 83. (a) In the given figure of monocot stem, the structure
vessels in the stem or leaves of a plant, typically marked as A, B, C, D, E and F are respectively
with phloem on the outside and xylem on the inside. epidermis, hypodermis, vascular bundle, phloem,
In the given figures A, B and C the types of vascular xylem and ground tissue. Monocot stem is
S-36 Biology
characterised by epidermis (2- 3 layered), hypodermis sclerenchyma, xylem tissues and parenchyma
and undifferentiated ground tissue stem. Vascular tissues. Collenchyma, sclerenchyma and
strand is numerous and scattered. Vascular bundles parenchyma tissues are types of simple permanent
are conjoint, collateral and closed. tissues. Figure c (xylem tissue) is a type of permanent
84. (b) In the given figure of dicot leaf, the part marked as A, tissue having many different types of cell. Xylem is
B, , D, E, F, G and H are respectively epidermis, one of the two types of transport tissue in vascular
palisade mesophyll, spongy mesophyll, sub-stomal plants, phloem being the other. The most distinctive
cavity, stoma, phloem, xylem and bundle sheath. xylem cells are the long tracheary elements that
Dicot leaf is characterised by upper and lower transport water. Tracheids and vessel elements are
epidermis, cuticle, mesophyll cells (present between
distinguished by their shape; vessel elements are
upper and lower epidermis) and vascular bundles.
shorter, and are connected together into long tubes
Mesophyll possess chloroplast, is divided into
palisade and spongy parenchyma. Vascular bundles that are called vessels.
are scattered in spongy parenchyma and the 90. (c) Annual ring is a ring in the cross section of the stem
vascular bundles in midrib region is largest. VBs are or root of a temperate woody plant, produced by
conjoint, collateral and closed. one year's growth. Trees at sea do not have annual
85. (a) In the given figure of monocot leaf, the structure rings because there is no marked climatic variation.
marked as A, B, C, D, E, F and G are respectively In climates with well-marked alternations of seasons
adaxial epidermis, xylem, mesophyll, sub-stomal (either cold and warm or wet and dry), the wood cells
cavity, abaxial epidermis, stoma and phloem. produced when water is easily available and growth
Monocot leaf (isobilateral) is somewhat similar to is rapid (generally corresponding to the spring or
dorsiventral leaf in having epidermis, cuticle, and wet season) are often noticeably larger and have
mesophyll cells. Stomata are present on the both thinner walls than those produced later in the season
the surfaces of epidermis. Mesophyll cells are when the supply of water has diminished and growth
undifferentiated into palisade an d spongy is slower. There is thus a sharp contrast between the
parenchyma. Vascular bundles are conjoint, small, thick-walled late-season wood cells produced
collateral, closed with phloem towards lower side one year, and the large, thin-walled cells of the spring
and xylem towards upper side. wood of the following year results. Where the climate
86. (a) In the given figure of secondary growth of a dicot is uniform and growth continuous, as in wet, tropical
stem, the parts marked as A, B, C, D, E and F are forests, there is usually little or no gross visible
respectively phellem, phellogen, medullary rays, contrast between the annual rings, although
secondary xylem, secondary phloem and cambium differences exist. When rings are conspicuous, they
ring. may be counted in order to obtain a reasonably
accurate approximation of the age of the tree. They
87. (d) Most of the cells of phellem are dead. But at some
are also reflective (by their range of thickness) of the
places living cells are also found. Suberin is not
climatic and environmental factors that influence
deposited in these places, These places are known
growth rates.
as Lenticels. Lenticels are certain loosely arranged
areas in the periderm formed due to rapid activity of 91. (a) All tissues except epidermis and vascular bundles
constitute the ground tissue. In leaves the ground
phellogen. Lenticels appears on the outer surface of
tissue consists of thin-walled chloroplast containing
the plant either in small points or in the form of areas
cells called mesophyll cells that actively involve in
of protruberance.
the process of photosynthesis.
Usually they are formed below the stomata. These 92. (a) Potato is a stem tuber. When we peel the skin of a
cells are also known as complementary cells/ potato tuber, we remove periderm. Periderm is the
complementary tissue and helps in gaseous corky outer layer of a plant stem formed in secondary
exchange and transpiration. thickening or as a response to injury or infection.
88. (a) In the given figure of a typical dicot root, the marked 93. (b) In most dicot stems and roots, secondary growth
structure A, B, C and D are respectively primary occurs due to addition of secondary tissues
phloem, vascular cambium, secondary phloem and (secondary cortex, secondary phloem and secondary
primary xylem. xylem) formed by the activity of vascular cambium in
89. (c) Figure a, b, c and d are respectively collenchyma, the stelar region inside pericycle and cork cambium
EBD_7209
Hints & Solutions S-37

in the extrastelar regions (region outside pericycle growth.


i.e., cortex) (ii) Intra-fascicular cambium or fascicular
94. (b) Intercalary meristem occurs between mature tissues cambium : This cambium occurs inside the
and is the separated region of apical meristem. It is vascular bundle. Except intrafascicular cambium
found in between the plant organs. By the activity all cambia are secondary in origin.
of this meristem, length of the plant organs increases. 103. (c) The periodical activity of the cambium, thus results
95. (c) In a very old stem, the secondary xylem elements in in distinct growth layers or rings of secondary xylem.
the inner part turn darker in colour and is called the These are called growth rings. The growth rings of
heartwood or duramen. It is very strong and durable spring wood and autumn wood produced in a year
and imparts great amount of mechanical strength to constitute an annual ring.
the stem. The sapwood or alburnum is the light 104. (d) Vessel is a long cylindrical tube-like structure made
coloured peripheral part of the secondary xylem up of many cells called vessel-members, each with
consists of dead tracheids. Vessels and fibres are lignified walls and a large central cavity. Vessel
some living cells. It helps in the conduction of water members are interconnected through perforations in
and solutes. their common walls. The presence of vessels is a
96. (d) The trees growing in desert will not show distinct characteristic feature of angiosperms.
annual rings because climatic variations (autumn and
105. (d) The internal structure of a typical monocotyledon
spring seasons) are absent in deserts.
root is similar to dicotyledon root. Number of xylem
97. (b) Open means presence of cambium during secondary bundles are more than six (polyarch) in
growth. Vascular cambium divides to form secondary monocotyledon root (exceptionally the number of
xylem towards inner side while secondary phloem xylem bundles are two to six in onion). Pith is well
towards outside. developed in monocotyledon root. Monocot roots
98. (d) Primary meristems are those meristems which do not undergo secondary growth.
originate from the embryonic meristem. They are 106. (a) Plant tissues are broadly classified on the basis of
located at the tips of stems, roots and appendages. stages of development into two categories:
Apical and intercalary meristems are primary meristem
(i) Meristematic tissue - They have ability to divide.
because they appear early in life of a plant and
contribute to the formation of the primary plant body. (ii) Permanent tissue - They have lost the ability to
99. (c) Phloem is a conducting tissue that transports food divide.
materials, usually from leaves to other part of the 107. (c) In plant conducting tissue xylem has an important
plants. In phloem, sieve elements arranged one above integral cell as xylem vessel which is without nucleus.
the other in distinct linear rows and have sieve plates The phloem on other hand has a row of sieve tubes
(oblique or transverse perforated septa) on their end which are also without nucleus at maturity.
walls. These are associated with companion cells. 108. (d) Soft wood is non-porous, homoxylous
100. (c) Teak, mango and palm belong to angiosperms in gymnospermic wood rich in tracheids and xylem
which presence of vessels is a characteristic feature. parenchyma, vessels and fibres are absent and resin
Pine is a gymnosperm which lack vessels in their canals are present, e.g., Deodar (Cedrus), Pine (Pinus)
xylem. 109. (b) Collenchyma is a type of simple tissue which occurs
101. (d) Guard cells differ from epidermal cells in having in layers below the epidermis of dicotyledonous
chloroplast. The cell wall of guard cells are not plants. It consists of living usually elongated cells
uniform, inner walls are thicker than the outer walls, with unevenly thickened walls (due to deposition of
epidermal cells are uniformly thin. cellulose, hemicellulose and pectin) and acts as
102. (b) Lateral meristem increases the width of plant organ support especially in areas of primary growth. This
so it is responsible for secondary growth. Lateral tissue provides mechanical support to the rowing
meristem includes vascular or cork cambium. parts of the plants such as young stem and petiole
Lateral meristems are both primary and secondary in of a leaf.
origin (mostly secondary in origin). There are two 110. (c) By observing the arrangements of vascular bundles,
examples of primary lateral meristem : the student will discover the nature of stem. If the
(i) Marginal meristem : It occurs at the margin of vascular bundles are arranged in a ring then it is a
leaf. Its activity increases the width of leaf so dicot stem and if the vascular bundles are scattered,
total growth of leaf is called intercalary marginal
S-38 Biology
then it is a monocot stem. 11. (c) Tendons, which attach skeletal muscles to bones
and ligaments which attach one bone to another are
111. (b) Trichomes is a small hair or other outgrowth from examples of dense regular connective tissues in
the epidermis of a plant stem, typically unicellular which collagen fibres are present in rows between
and glandular. Trichomes that often cover the plant many parallel bundles of fibres.
body are the result of divisions of epidermal cells. 12. (b) Muscle tissue plays the vital role of providing
They may be branched or unbranched and soft or movement and heat generation to the organs of the
stiff. The trichomes help in preventing water loss body. Muscle cells are specialized for contractility
and electrical conductivity.
due to transpiration. Transpiration is a process where
13. (d) In all connective tissue except blood (a fluid
plants absorb water through the roots and then give connective tissue), the cells secrete fibres of
off water vapor through pores in their leaves. structural proteins called collagen or elastin.
14. (b) A neuron (also known as a neurone or nerve cell) is
Chapter 7 : Structural Organisation in Animals an electrically excitable cell that processes and
1. (d) Squamous epithelium is formed of thin discoidal and transmits information through electrical and chemical
polygonal cells that fit like tiles in a floor, so is also signals. These signals between neurons occur via
called pavement epithelium. It is found in the walls synapses, specialized connections with other cells.
of blood vessels, in the alveoli of lung for exchange A typical neuron possesses a cell body (soma),
of gases, and in Bowman’s capsule of nephron for dendrites, and an axon.
ultra filtration. 15. (d) Cartilage is a type of connective tissue which is
2. (b) Ciliated columnar epithelium is usually found in the present in human external ears, the nose tip, ribcage
respiratory tract and fallopian tubes. joints, etc.
3. (c) Mast cell are a cell found in connective tissue that 16. (d) Cells of smooth muscle tissue are spindle shaped
contains numerous basophilic granules and release and pointed at their ends. Striations are absent due
substances such as heparin and histamine in to different arrangement of actin and myosin
response to injury or inflammation of tissues. filaments. Smooth muscle is the earliest form of muscle
4. (d) Basement membrane is a thin extracellular to evolve.
supporting layer that separates a layer of epithelial 17. (a) Cardiac muscles are involuntary, cross striated &
cells from the underlying lamina propria and is non-fatigued fibres. Cell junctions fuse the plasma
composed of the basal lamina and reticular lamina membranes of cardiac muscle cells and make them
5. (d) Compound epithelium covers the dry surface of the stick together. Communication junctions
skin, the moist surface of buccal cavity, pharynx, (intercalated discs) at some fusion points allow the
inner lining of ducts of salivary glands and pancreatic cells to contract as a unit, i.e., when one cell receives
ducts. a signal to contract, its neighbouring cells are also
stimulated to contract.
6. (c) Gland is an organ in the human or animal body which
secretes particular chemical substances for use in 18. (d) Smooth muscle is an involuntary, spindle shaped,
the body or for discharge into the surroundings. uninucleated, tapering and non-striated muscle.
7. (d) Plasmodesmata are narrow channels that act as Smooth muscle is responsible for the contractility of
intercellular cytoplasmic bridges to facilitate hollow organs, such as blood vessels, the
communication and transport of materials between gastrointestinal tract, the bladder, or the uterus. Its
plant cells. Adhering, tight and gap junctions are structure differs greatly from that of skeletal muscle,
specialized junctions that provide both structural and although it can develop isometric force per cross-
functional links between the individual cells of animal sectional area which is equal to that of skeletal
tissues. muscle.
8. (d) A tendon is a fibrous connective tissue which 19. (b) Neurons forms the structural and functional unit of
attaches skeletal muscle to bone. Tendons may also nervous tissue. They are excitable cells. While the
attach muscles to structures such as eyeball. A neuroglial cells constitute the rest of the neural
tendon help in the movement of bone or structure. system that protect and support neurons and are
9. (c) Chondrocytes are the only cells found in cartilage. They non-excitable.
produce and maintain the cartilaginous matrix, which 20. (d) Each neuron has an enlarged portion, the cell body
consists mainly of collagen and proteoglycans. The (perikaryon), containing the nucleus and from the
chondrocytes lie between the muscle dfibres. It is found
cell body extend several processes (dendrites)
in the epiglottis (part of the larynx) and the pinnae (the
external ear flaps of many mammals including humans). through which impulses enter from their branches.
10. (c) Elastic cartilage or yellow cartilage is a type of 21. (d) Spermathecae are present in sixth, seventh, eighth
cartilage present in the outer ear, Eustachian tube and nineth segments of the earthworm. They store
and epiglottis. It contains elastic fibre networks and the sperms received from another earthworm during
collagen fibres. The principal protein is in elastic copulation.
cartilage is elastin.
EBD_7209
Hints & Solutions S-39

22. (d) Except the first, the last and clitellar segment in each on the basement membrane. Being multi-layered,
segment bears a ring of tiny curved, chitinous compound epithelia have little role in secretion or
structure known as setae. Setae helps in locomotion absorption, but they provide protection to
and copulation. underlying tissues against mechanical, chemical,
23. (d) In cockroaches, the development is paurometabolus, thermal or osmotic stresses. Compound epithelia may
which means development through nymphal stage. be stratified or transitional.
The nymph grows by moulting about 13 times to 37. (d) Muscle tissue is contractile tissue grouped into
coordinated systems for greater efficiency. In
reach the adult form.
humans the muscle systems are classified by gross
24. (b) Cockroaches are usually brown or black. appearance and location of cells. The three types of
Cockroaches belong to kingdom animalia, phylum muscles are striated (or skeletal), cardiac, and smooth
arthropoda, class insecta and order Blattaria. Some (or non-striated).
species invade human dwellings and are considered
38. (b) Neurons, the unit of neural system are excitable cells
pests. Others are beneficial to the environment as
while chondrocytes are cartilagenous cells.
important recyclers of decayed organic material.
39. (d) Female reproductive system consists of the large
25. (d) Both the sexes of cockroach have anal cerci which
ovaries, lying laterally in the 2nd–6th abdominal
are jointed structures. But in the male, in addition,
segments. Each ovary is formed of a group of eight
there is a paired unjointed needle-like anal style, which
ovarian tubules or ovarioles, containing a chain of
serve to distinguish between the male and the female.
developing ova. A pair of spermatheca is present in
26. (b) Testes are paired, 3-4 lobed, situated dorsolaterally the 6th segment which opens into the genital chamber.
in the 4th, 5th, 6th abdominal segments.
40. (d) A cell junction (or intercellular bridge) is a type of
27. (a) Grasshoppers, locusts and cockroaches have biting structure that exists within the tissue of some
and cutting type of mouth parts. The mouth parts of multicellular organisms, such as animals. Cell
mosquito are piercing and sucking type; and of house- junctions consist of multiprotein complexes that
fly is siphoning type. provide contact between neighbouring cells or
28. (b) The phallic organs (also called conglobate gland) between a cell and the extracellular matrix. There are
are related to male reproductive system of cockroach. three major types of cell junctions: adherens
This organ is present below the utricular gland in junctions, desmosomes and hemidesmosomes
the 6th segment. It communicates with exterior by a (anchoring junctions); gap junctions (communicating
pore close to gonophore. junction); tight junctions (occluding junctions).
29. (b) Gizzard has an outer layer of thick circular muscles 41. (c) Crop is the part of foregut which is used for storing
and thick inner culicle forming six highly chitinous of food.
plate called teeth. It helps in grinding food particles.
42. (a) Squamous epithelium is an epithelium whose most
30. (a) The frog never drinks water but absorbs it through superficial layer consists of flat, scale-like cells called
the skin. The skin is always maintained in a moist
squamous epithelial cells. The epithelium may be
condition.
composed of one layer of these cells, in which case
31. (b) Frog having a pair of forelimb and hindlimb is it is referred to as simple squamous epithelium or it
appended to the trunk. The forelimb consists of the may possess multiple layers, referred to then as
proximal upper arm, the middle fore arm, wrist and a stratified squamous epithelium.
hand bearing four fingers. The hindlimb consists of 43. (b) Connective tissue is a group of tissues in the body
proximal thigh, the middle shank, a long ankle and a that maintains the form of the body and its organs
foot bearing five toes. and provides cohesion and internal support. The
32. (d) Frog has different type of sense organs like organs connective tissues include several types of fibrous
of touch (sensory papillae), taste (taste buds), smell tissue that vary only in their density and cellularity,
(nasal epithelium), vision (eyes) and hearing as well as the more specialized and recognizable
(tympanum with internal ears). variants-bone, ligaments, tendons, cartilage, and
33. (d) In cockroaches, males bear a pair of short, thread adipose (fat) tissue.
like anal styles which are absent in females. In both 44. (d) Adipose tissue is a type of loose connective tissue
sexes, the 10th segment bears a pair of jointed located mainly beneath the skin. (For more refer
filamentous structures called anal cerci. answer 66)
34. (d) Anal style is a pair of short, thread-like structure 45. (b) All the given statements are associated with bone.
present in males only. In both sexes, the 10th segment Bone is a rigid body tissue consisting of cells
bears of pair of jointed filamentous structures called embedded in an abundant, hard intercellular material.
anal cerci. This material consists of two important components
35. (c) They are commonly found in ducts of glands and - collagen and calcium phosphate. It forms the major
tubular parts of nephrons in kidneys. portion of the human skeleton. Bone support and
36. (d) Compound Epithelium consists of more than one protect the various organs of the body, produce red
layer of cells. Only the cells of the deepest layer rest and white blood cells, store minerals and also enable
S-40 Biology
mobility. 60. (a) The development of cockroaches is
46. (b) The smooth muscle fibres are involuntary, taper at paurometabolous which means development
both ends and do not show striation. The wall of through nymphal stage.
internal organs such as the blood vessels, stomach 61. (a) Cardiac muscle cells are short cylindrical cells joined
and intestine contains this type of muscle tissue. end to end and by side branching to form a network.
47. (c) There are three ganglia lie in the thorax, and six in the Intercalat discs are the dense junctions formed in
abdomen. Each eye consists of about 2000 hexagonal between the cardiac muscle cells where they meet
ommatidia. each other. Intercalated discs are the specialised
48. (c) In cokroach, respiratory system consists of a network regions of the cell membranes. As cardiac muscle
of trachea, that open through 10 pairs of small holes possesses considerable rhythmicity and generates
called spiracles present on the lateral side of the body. its own wave of excitation, these discs function as
49. (d) Digestive system of cockroach is divided into - boosters for muscle contraction wave.
foregut, midgut and hindgut. The mouth opens into 62. (b) Cartilage comprises of mucopolysaccharide called
a short tubular pharynx which further opens into a chondroctin sulphate. Bone is a hard connective
narrow tubular structure called oesophagus. The tissue. Blood is a fluid connective tissue.
oesophagus opens into a sac-like structure called 63. (c) Cuboidal epithelium is a single layer of cube shaped
crop where food is stored. The crop is followed by cells with a centrally located nucleus. Ciliated
gizzard and proventriculus. The gizzard helps in epithelium is a type of bodily tissues that is lined
grinding the food particles. The entire foregut is lined with "ciliated" cells, which are basically cells that
by cuticle. At the junction of foregut and midgut, a have small, hair-like protrusions known as "cilia" that
ring of 6 - 8 blind tubules are present. These are can either help the cells move along the tissue or can
called gastric or hepatic cecae which secrete digestive help debris and waste move along the surface of the
juice. The hindgut is broader than the midgut. The cells. Columnar epithelial cells occur in one or more
hindgut is differentiated into ileum, colon and rectum. layers. The cells are elongated and column-shaped.
The rectum opens through anus. Malpighian tubules The nuclei are elongated and are usually located near
are the excretory organs which absorb nitrogenous the base of the cells. Squamous cells have the
wastes and convert them into uric acid. Uric acid is appearance of thin, flat plates. The shape of the
excreted out through the hindgut. nucleus usually corresponds to the cell form and
50. (c) Cockroaches absorb nitrogenous waste products help to identify the type of epithelium. Keratinized
and convert them into uric acid which is excreted squamous epithelium consists of multiple layers of
out-through hindgut. Hence they called as uricotelic. squamous cells. The apical layer of cells are dead
51. (c) Blood vascular system of cockroach is an open type. and filled with the protein keratin.
They are dioecious and both sexes have well 64. (b) (Refer answer 63)
developed reproductive organs. 65. (c) Loose connective tissue (which includes areolar
52. (c) tissue, reticular tissue, and adipose tissue) holds
53. (c) organs in place and attaches epithelial tissue to other
54. (b) External ear are absent in frog and only tympanum underlying tissues. It also surrounds the blood
can be seen externally. vessels and nerves. Cells called fibroblasts are widely
55. (d) Typhlosole is the characteristic feature of earthworm. dispersed in this tissue. Dense regular tissue includes
It can be defined as an extra flap of tissue or an tendons and ligaments. Tendon connects skeletal
infolding along the inner wall of the intestine. The muscle with a bone. Ligaments attach one bone to
typhlosole in earthworms increases the surface area another. Dense irregular tissue includes skin.
of the intestine for efficient secretion and absorption Specialized connective tissue includes cartilage,
during digestion. bones and blood. Cartilage is a rubbery tissue made
56. (b) by cells called chondroblasts. It does not have any
57. (b) In all connective tissues except the cell secretes fibers blood vessels within it so healing is not easy if it is
of structural proteins called collagen or elastin. The damaged and may not occur at all if the damage is
fibres provide strength elasticity and flexibility to extensive enough. Bone is a rigid organ that
the tissues. constitutes part of the vertebral skeleton. It support
58. (d) Mast cells secrete histamine are especially prevalent and protect the various organs of the body, produce
in the connective tissue of skin and respiratory tract red and white blood cells, store minerals and also
and in surrounding blood vessels. Histamine is a enable mobility. Blood delivers necessary
vasodilator and lower blood pressure. It is also substances such as nutrients and oxygen to the cells
involved in allergic reactions. and transports metabolic waste products away.
59. (c) In each body segment of earthworm, except the first 66. (a)
last and clitellum, there are rows of S-shaped setae 67. (b)
embedded in the epidermal pits in the middle of each 68. (d) Cartilage is a soft skeletal tissue (a specialized
segment. Their principle role in locomotion. connective tissue) which forms the endoskeleton of
the vertebrates. It is a vascular and nutrient are
EBD_7209
Hints & Solutions S-41

diffused through the matrix. 78. (c) Alary muscles (structure marked as C) are located in
69. (c) pairs in the chambers of cockroach heart. They are
70. (a) 1 - A; 2 - B; 3 - C; 4 - D 13 pairs of wing shaped involuntary muscles which
71. (a) In the given structure of male reproductive system expand and contract to facilitate the flow of
of cockroach, the label marked as A to D are haemolymph through the heart.
respectively 8th sternum; anal cercus; 10th tergum 79. (a) A - Spermatheca; B - Collaterial gland;
and anal style. C - Gonapophyses
Anal style is a pair of short, thread-like present in 80. (b) The figure represents the image of adipose tissue.
males. In both sexes, the 10th segment bears of pair Adipose is a loose connective tissue that fills up
of jointed filamentous structures called anal cerci. space between organs and tissues and provides
72. (d) The given figure represents loose connective tissue structural and metabolic support. It is part of the
(areolar tissue). Loose connective tissue has cells and nutrient glue that holds us all together. Adipose
fibres loosely arranged in a semi-fluid ground tissue is often referred to as fat.
substances, for example, areolar tissue present beneath 81. (b) The given figures A, B and C are skeletal muscle,
the skin. Often it serves as a support framework for smooth muscle and cardiac muscle respectively. Skeletal
epithelium. It contains fibroblasts (cells that produce muscle fibres occur in muscles which are attached to
and secrete fibres), macrophages and mast cells. the skeleton. They are striated in appearance and are
73. (a) The figure I & II represent dense regular and dense under voluntary control. Smooth muscle fibres are
irregular connective tissue respectively. In the dense located in walls of hollow visceral organs, except the
regular connective tissues, the collagen fibres are heart, appear spindle-shaped, and are also under
present in rows between many parallel bundles of involuntary control. Cardiac muscle cells are located in
fibres. Tendons and ligaments are example of this the walls of the heart, appear striated, and are under
tissue. Dense irregular connective tissue has involuntary control.
fibroblasts and many fibres (mostly collagen) that 82. (d) Blood glands are present on the 4th, 5th and 6th
are oriented differently. This tissue is present in the segments. They produce blood cells and
skin. haemoglobin which is dissolved in blood plasma.
74. (a) The given figure represents the image of cartilage. Blood cells are phagocytic in nature.
Cartilage is an important structural component of Lateral hearts are present in 7th and 9th segments
the body. It is a firm tissue but is softer and much and connects dorsal blood vessel with ventral blood
more flexible than bone. It is a connective tissue found vessel. The Nephridia is an organ which occurs in
in joints between bones e.g. the elbows, knees and pairs and performs a function similar to the vertebrate
ankles; ends of the ribs; between the vertebrae in kidney. Nephridia remove metabolic wastes from an
the spine; ears and nose; bronchial tubes or airways. animal's body.
Cartilage is made up of specialized cells called 83. (c) In the given figure the marked label 1, 2, 3 and 4 are
chondrocytes (B). These chondrocytes produce oesophagus, crop, gizzard and malpighian tubules.
large amounts of extracellular matrix composed of Gizzard is a short muscular organ which is used for
collagen (A) fibres, proteoglycan, and elastin fibres. grinding food.
There are no blood vessels in cartilage to supply the 84. (a) The given figure shows the image of areolar tissue, a
chondrocytes with nutrients. type of loose connective tissue. Loose connective
75. (c) A - Pronotum; B - Mesothorax; C - Metathorax; tissue (which includes areolar tissue, reticular tissue,
D - Tegmina; E - Anal cerci and adipose tissue) is the most common type of
76. (b) A - Ocellus; B - Compound eye; C - Mandible; connective tissue in vertebrates. It holds organs in
D - Maxilla; E - Labrum; F - Labium. place and attaches epithelial tissue to other
77. (b) A - Crop; B - Gizzard; C - Hepatic cecae; D - underlying tissues. It also surrounds the blood
Malpighian tubule vessels and nerves. Areolar tissues are present
Crop is a sac-like structure which store food. The beneath the skin and serve as a support framework
crop is followed by gizzard and proventriculus. The for epithelium. It contains fibroblast, macrophages,
gizzard has an outer layer of thick circular muscles collagen fibres and mast cells.
and thick inner cuticle; forming six highly chitinous 85. (d) Tissue 1 is columnar cells bearing cilia and tissue 2
plate called teeth. Food particles are ground in the is multicellular glandular epithelium.
gizzard. At the junction of foregut and midgut, a Ciliated columnar epithelial cells possess fine hair-
ring of 6 - 8 blind tubules are present. These are like outgrowths, cilia on their free surfaces. These
called gastric or hepatic cecae and secrete digestive cilia are capable of rapid, rhythmic, wavelike beatings
juice. Malpighian tubules are the excretory organs in a certain direction. This movement of the cilia in a
which absorb nitrogenous wastes and convert them certain direction causes the mucus, which is secreted
into uric acid. Uric acid is excreted out through the by the goblet cells, to move (flow or stream) in that
hindgut. direction. Ciliated epithelium is usually found in the
S-42 Biology
air passages like the nose. It is also found in the 96. (b) Due to the lack of blood vessels, cartilage grows
uterus and fallopian tubes of females. Columnar and repairs more slowly than other tissues.
epithelium with goblet cells is called glandular 97. (a) Cardiac muscle cells or cardiac myocytes are the
epithelium. Some parts of the glandular epithelium muscle cells (myocytes) that make up the cardiac
consist of such a large number of goblet cells that muscle. When these cells are damaged by a heart
there are only a few normal epithelial cells left. attack, they are usually replaced by connective
86. (d) Label 3 and 4 are respectively genital chamber and muscle cells.
vestibulum which is collectively called genital pouch. 98. (d) Collagen is the most abundant protein in the human
87. (d) Without respiration, no animal can survive. body. In the dense regular connective tissues, the
Cockroach is not as simple as Hydra where oxygen collagen fibres are present in rows between many
can diffuse directly into the cells. There is a system parallel bundles of fibres. Tendons, which attach
of tubes in cockroach, the tracheal system, with the skeletal muscles to bones are example of this tissue.
help of which oxygen goes directly into the tissues 99. (b) Bone marrow is the flexible tissue in the interior of
by intracellular capillary system. bones. In humans, red blood cells are produced by
88. (b) If the head of cockroach is cut off, it will still alive for cores of bone marrow in the heads of long bones in
as long as one week, because the head holds a bit of a process known as haematopoiesis. Bone marrow
a nervous system while the rest is situated along te is also a key component of the lymphatic system,
ventral part of the body. producing the lymphocytes that support the body's
89. (c) Ligaments attach a bone with another bone in immune system.
movable/synovial joints. Broken or cut ligaments 100. (d) Loose connective tissue is a category of connective
make movement at joints very painful and restricted. tissue which includes areolar tissue, reticular tissue,
90. (b) Neurons are the basic structural and functional units and adipose tissue. It holds organs in place and
of the nervous system. Neurons are least likely to be attaches epithelial tissue to other underlying tissues.
replaced by new cells as they have least regeneration 101. (a) The neuroglial cells are non-excitable cells that
power. protect and support neurons.
91. (c) The head is triangular in shape which lies anteriorly 102. (d) Hair is a protein filament that grows from follicles
at right angles to the longitudinal body axis. The found in the dermis or skin. While hair is growing
head is formed by the fusion of six segments. The beneath the epidermis, its outer covering is soft. Once
flexible neck facilitates movement in all directions. it goes past the epidermis, the outside layer hardens
92. (b) The development of cockroach is paurometabolous into keratin. Inside the follicle, the hair is growing
meaning that their development is through nymphal and is “connected” to blood vessels and nerves.
stage. The nymphs look very much like adults. The Outside the skin, the hair is essentially dead.
nymph grows by moulting about 13 times to reach 103. (d) Cell junctions come across most frequently in the
adult form. The last nymphal stage has wingpads preparation of ciliated epithelium. A cell junction is a
while adult cockroaches have wings. structure within a tissue of a multicellular organism.
93. (c) In cockroach, oesophagus opens into a sac-like Cell junctions are especially abundant in epithelial
structure called crop (used for storing of food). Crop tissues. They consist of protein complexes and
followed by gizzard that helps in grinding the food provide contact between neighbouring cells,
particles. A ring of 6-8 blind tubules (called hepatic between a cell and the extracellular matrix, or they
of gastric caecae) is present at the junction of foregut built up the paracellular barrier of epithelia and control
and midgut, which secrete digestive juice. the paracellular transport.
Malpighian tubules, present at the junction of midgut 104. (c) The columnar epithelium is composed of a single
and hindgut helps in the removal of excretory layer of tall and slender cells. Their free surface have
products. Hindgut is differentiated into ileum, colon microvilli found in the lining of stomach and intestine
and rectum. The rectum opens out through anus. and help in secretion and absorption.
94. (c) Epithelial tissue covers the body surface, or lines 105. (c) Connective tissue fills the spaces between organs
the body cavity. The cells rest upon a non-cellular and tissues, and provides structural and metabolic
basement membrane which is secreted by epithelial support for other tissues and organs. The three basic
cells. It is not n ourished by blood, so non- components of connective tissue are cells, protein
vascularized. fibres and ground matrix.
95. (b) On the basis of the mode of pouring of secretion, 106. (b) Columnar epithelium is an epithelium consisting of
gland are divided into – (a) exocrine, and (b) one or more layers of elongated cells of cylindrical
endocrine glands. or prismatic shape. Columnar epithelial cells resemble
Exocrine glands secrete mucus, saliva, oil, digestive elongated boxes. They have oval nuclei that usually
enzymes and other cell products. These products are situated in the lower part of the cell. The cells are
are released through ducts or tubes. In contrast, closely joined together and rest on a basement
endocrine glands do not have ducts. Their products membrane, known as the basal lamina. Some cells
called hormones are secreted directly into the fluid have tiny hair-like structures known as cilia on their
bathing the gland. upper surfaces. These tiny fronds beat in unison to
EBD_7209
Hints & Solutions S-43

waft particles across the cell surface. Columnar cells studies on the RBC cell membrane enabled the
with cilia can be found in the respiratory tracts. Other scientists to infer the possible structure of plasma
cells have minute finger-like projections called membrane. The plasma membrane of the RBC
microvilli. These help to increase the surface area consists of a complex, well-ordered group of lipids
available for absorption and can be found in the lining
of the intestines. and proteins stretched over the outer surface of the
107. (c) Sensory system of earthworm does not have eyes cell in the form of a lipid bilayer punctuated by
but does possess light and touch sensitive organs penetrating or attached proteins. This membrane has
(receptor cells) to distinguish the light intensities numerous properties that arise in part from specialized
and to feel the vibrations in the ground. interactions between specific membrane proteins or
108. (b) The given specimen shows the characteristics of lipids, or both.
earthworm (Pheretima). An earthworm is a tube- 12. (c) Middle lamella is the cementing layer between the
shaped, segmented worm commonly found living in cells. It is made up of Ca and Mg pectates.
soil that feeds on live and dead organic matter. 13. (a) Plasmodesmata is an effective transport pathway
"Earthworm" is the common name for the largest
present between two adjacent cells. They are
members of oligochaeta in the phylum annelida.
microscopic channels which traverse the cell walls
109. (a) Frog maintains ecological balance because these
serve as an important link of food chain and food of plant cells and some algal cells, enabling transport
web in the ecosystem. and communication between them.
14. (b) Cell wall is a very tough, flexible and rigid layer that
Chapter 8 : Cell : The Unit of Life surrounds the cell membrane. Cell wall determines
1. (b) Rudolf Virchow (1855) first explained that cells the shape of the cells and provides a strong structural
divided and new cells are formed from pre-existing support to prevent the bacterium from bursting or
cells (omnis cellula-e-cellula). It is also know as law collapsing.
of cell lineage. 15. (b) The endomembrane system includes endoplasmic
2. (b) Ribosome is sub-spherical granular structure which reticulum, golgi complex, lysosomes and vacuoles.
is devoid of any covering membrane. 16. (b) Rough endoplasmic reticulum (RER) is found
3 (b) The prokaryotic cells do not have nuclear membrane throughout the cell but its density is higher near the
while eukaryotic cell have well organized nuclear nucleus and the Golgi apparatus. Ribosomes present
membrane. on the rough endoplasmic reticulum are called
'membrane bound' and are responsible for the
4 (b) In some prokaryotes like cyanobacteria or Blue-Green
assembly of many proteins (in a process called
algae, there are other membranous extensions in the
translation (a step in protein synthesis).
cytoplasm called chromatophores.
17. (d) Endoplasmic reticulum and ribosomes are the cell
5. (a)
organelles which were discovered after the
6. (c) Several ribosomes may attach to a single mRNA and
introduction of electron microscope.
forms a chain called polysomes. The ribosomes of a
18. (c) The Golgi apparatus (also known as the Golgi complex,
polysome translate the mRNA into proteins.
Golgi body) is an organelle, identified in 1897 by the
7. (d) Integral proteins occur inside the lipid bilayer
Italian physician Camillo Golgi and named after him in
passing to its diferent depths. Some intrinsic proteins
1898. Golgi apparatus is the important site of formation
run throughout the thickness of the membrane. They
of glycoproteins and glycolipids. It functions as a
are called transmembrane proteins.
factory in which proteins received from the ER are
8. (b) An energy dependent process, in which ATP is
further processed and sorted for transport to their
utilised is called an active transport.
ultimate endpoints: lysosomes, the plasma membrane.
e.g., Na+ / K+ Pump.
19. (a) Golgi apparatus principally performs the function of
9. (b) The ratio of protein and lipids varies considerably in
packaging material to be delivered either to the intra
different cell types. The membrane of the erythrocytes
– cellular targets or secreted outside the cell.
has approximately 52% of proteins and 40% lipids.
20. (d) Lysosomes are originated by golgi complex and
10. (b) The lipid component of the membrane is mainly
contain many hydrolytic enzymes (hydrolases –
consists of phosphoglycerides. Phosphoglyceride
lipases, proteases, carbohydrases) which are
are glycerol-based phospholipids and main
component of biological membranes. Each optically active at the acidic pH.
glycerophospholipid molecule consists of a small 21. (a) Contractile vacuole is a sub-cellular structure
polar head group and two long hydrophobic chains. (or organelle) which is found primarily in protists
11. (a) RBC (red blood cell) of the human is the best material (for eg, Amoeba) and unicellular algae. Contractile
vacuole is responsible for excretion an d
for the study of structure of cell membrane. Chemical
osmoregulation. It absorbs excess water and wastes
S-44 Biology
from a microorganism's cell and excretes them into is located at the base of eukaryotic cilia and flagella,
the environment by contracting. Contractile which is a continuation of the 9 outer-axonemal
vacuoles function in a periodic cycle by expanding microtubule doublets, but with the addition of a C
while collecting water and contracting to release the tubule to form a centriole-like triplet. Basal bodies
water. may be self-replicating and serve as a nucleating
22. (b) The mitochondrion is a double membrane-bound centre for axonemal assembly, which are anchored in
organelle found in most eukaryotic cells. It contains the cytoplasm by rootlets.
outer and inner membran es composed of 31. (b) A centriole is a barrel - shaped cell structure found in
phospholipid bilayers and proteins. The two most animal eukaryotic cells, though absent in higher
membranes have different properties. Because of this plants and most fungi.
double-membrane organization, there are five distinct 32. (b) Nucleolus is a small rounded body within a resting
parts to a mitochondrion- the outer mitochondrial nucleus that contains RNA and proteins. It is usually
membrane, the intermembrane space (the space associated with a specific chromosomal site and
between the outer and inner membranes), the inner involved in ribosomal RNA synthesis and the
mitochondrial membrane, the cristae space (formed formation of ribosomes.
by infolding's of the inner membrane), and the matrix 33. (a) Satellite is a small chromosomal segment separated
(space within the inner membrane). Mitochondria are from the main body of the chromosome by a
also called as the power house of the cell. secondary constriction. In humans it is usually
23. (d) Plastids are found in all plant cells and in euglenoides. associated with the short arm of an acrocentric
24. (a) Elaioplasts are a type of leucoplast that is specialized chromosome. They play a vital role in the formation
for the storage of lipids in plants. Elaioplasts house of the nucleolus after cell division. In humans,
oil body deposits as rounded plastoglobuli, which chromosomes number 13, 14, 15, 21 and 22 are
are essentially fat droplets. examples of SAT (satellite) chromosomes.
25. (c) Grana is a stacked membranous structure within the 34. (a) Centromeres consist of a complex combination of
chloroplasts of plants and green algae that contains proteins and DNA. They are essential to the division
the chlorophyll (green pigment). It is the site of the and ensure the accurate segregation of
light reactions of photosynthesis. The sac-like chromosomes. Metacentric chromosomes have the
membranes that make up grana are known as centromere in the centre, such that both sections are
thylakoids. of equal length. Human chromosome 1 and 3 are
26. (a) Microtubules are the largest cytoskeletal filaments metacentric.
in cells and found throughout the cytoplasm. They 35. (b) Vacuole is a membrane bound organelle, found in both
are filamentous intracellular structures that are animal and plant cells but are much larger in plant
responsible for various kinds of movements in all cells. Vacuoles are essentially enclosed compartments
eukaryotic cells. They are involved in nucleic and which are filled with water containing inorganic and
cell division, organization of intracellular structure, organic molecules including enzymes in solution,
and intracellular transport, as well as ciliary and vacuoles might store food or any variety of nutrients
flagellar motility. They are formed by the a cell might need to survive. They can even store
polymerization of a dimer of two globular proteins, waste products so the rest of the cell is protected
alpha and beta tubulin. from contamination.
27. (d) The cytoskeleton is a series of intercellular proteins 36. (a) Ribosomes are about 15 nm by 20 nm in size.
that help a cell with shape, mechanical support, and 37. (a) Protein synthesis is the process by which amino acids
movement. Cytoskeleton has three main structural are linearly arranged into proteins through the
components: microfilaments, intermediate filaments, involvement of ribosomal RNA, transfer RNA,
and microtubules. messenger RNA, and various enzymes. In animal cells,
28. (c) Axoneme with 9 + 2 microtubular arrangement occurs in protein synthesis occurs on ribosomes present in
cilia and flagella. Cilia are hair like growths of the cell cytoplasm as well as in mitochondria.
membrane. Flagella are comparatively longer and 38. (b) The quasi-fluid nature of lipid enable lateral movement
responsible for movement. The axoneme usually has nine of proteins within the overall bilayer. This ability to
pair of centrallylocated microtubules. Such an arrangement
move within the membrane is measured as its fluidity.
of axonemal microtubules is referred to as the 9 + 2 array.
39. (d) In some prokaryotes like cyanobacteria, there are
29. (d) Microtubular organization and type of movement
membranous extensions into the cytoplasm called
differentiates prokaryotic flagella or eukaryotic flagella.
chromatophores which contain pigments.
30. (b) Basal body is a short cylindrical array of
40. (b) Mitochondria divide by fission.
microtubules. It is formed from a centriole and
41. (d) Nerve cells are the longest cells of the body.
associated with the formation of cilia and flagella. It
EBD_7209
Hints & Solutions S-45

42. (b) Prokaryotes have 70S ribosomes, each consisting of part of the endomembrane system. The mitochondrial
a small (30S) and a large (50S) subunit. Polyribosomes matrix possesses single circular DNA molecule, a few
(or polysomes) are a cluster of ribosomes, bound to a RNA molecule, ribosomes (70S) and the components
mRNA molecule in translation during the process of required for the synthesis of proteins.
protein synthesis. It was first discovered and 53. (d) The given characters are related to cell organelle -
characterized by Jonathan Warner, Paul Knopf, and vacuole. Vacuole is a membrane bound organelle,
Alex Rich in 1963. Polysomes indicate the synthesis found in both animal and plant cells but are much
of identical polypeptide in multiple copies. larger in plant cells. Vacuoles are essentially enclosed
43. (d) All the given statements are related to Schwann compartments which are filled with water containing
Theodor. Schwann was a German physiologist. His inorganic and organic molecules including enzymes
contributions to biology include the development in solution, though in certain cases they may contain
of cell theory, the discovery of Schwann cells in the solids which have been engulfed. Vacuoles might
peripheral nervous system, the discovery and study store food or any variety of nutrients a cell might
of pepsin, the discovery of the organic nature of need to survive. They can even store waste products
yeast, and the invention of the term metabolism. so the rest of the cell is protected from contamination.
44. (a) All the statements are correct. 54. (d) All the functions described are performed by Golgi
45. (b) All the given statements regarding cilia and flagella apparatus.
are correct. The Golgi apparatus is an organelle found in most
46. (d) All the given function is related to cell wall. Cell wall eukaryotic cells. It was identified in 1897 by the Italian
is a very tough, flexible and fairly rigid layer that physician Camillo Golgi and named after him in 1898.
surrounds some types of cells. It surrounds the cell Golgi apparatus is the important site of formation of
membrane and provides these cells with structural glycoproteins and glycolipids.
support and protection. It also acts as a filtering 55. (a) In animal cells lipid like steroidal hormones are
mechanism. synthesized in SER. In plant cells, the vacuoles can
47. (c) The vacuole is bound by a single membrane called occupy up to 90% of the volume of the cell.
tonoplast. Chlorophyll and carotenoid pigments are responsible
48. (a) The centrioles occur in nearly all animal cells, but for trapping light energy essential for
absent in plant cells with few exceptions such as photosynthesis.
zoospores of algae, sperm cells of ferns, and motile 56. (a) The centrosome (occur only in animals cells) is the
algae. Middle lamella is a thin, amorphous and main place where cell microtubules get organized.
cementing layer between two adjacent cells. It is Centrosome usually contains two cylindrical
made up of pectates of calcium and magnesium. structures called centrioles. Centrioles are composed
49. (a) Endomembrance system is a grouping of some of groupings of microtubules arranged in a 9 + 3 pattern.
membrane organelles which function in close The pattern is so named because a ring of nine
coordination with one another, viz., endoplasmic microtubule "triplets" are arranged at right angles to
reticulum, Golgi complex, lysosomes and vacuoles. one another. Centrioles, found in animal cells, help to
50. (c) In bacteria, a sticky, gelatinous material called organize the assembly of microtubules during cell
glycocalyx collects outside the cell wall to form an division. Centrioles replicate during the interphase
additional surface layer. Glycocalyx covering of the stage of mitosis and meiosis.
bacterial cell may be loose, thin sheath or thick and 57. (c) Cells vary greatly in their shape. They may be disc-
tough coat. Loose glycocalyx covering is termed slime like, polygonal, columnar, cuboid, thread like, or even
layer while tough and thick type is known as capsule. irregular. The shape of the cell may vary with the
51. (a) The special membranous structure formed by the function they perform e.g., xylem vessels are long as
extension of prokaryotic plasma membrane is known they have to form long water conducting channels.
as mesosome while polysome is structure formed by 58. (b) Middle lamella is a thin, amorphous and cementing
combination of many ribosomes. layer between the primary walls of two adjacent cells.
SER is the major site of synthesis of lipids. The site Middle lamella is the first layer which is formed from
of protein synthesis is RER. cell plate between newly formed daughter cells at
52. (d) Statements (ii) and (iii) are correct. The the time of cytokinesis. Middle lamella is absent on
endomembrane system include endoplasmic the outer side of surface cells. It it made up of calcium
reticulum (ER), golgi complex, lysosomes and and magnesium pectates.
vacuoles. Since the functions of the mitochondria, 59. (d) Based on the position of the centromere, the
chloroplast and peroxisomes are not coordinated with chromosomes can be classified into four types:
the above components, these are not considered as metacentric (centromere middle), sub-metacentric
S-46 Biology
(centromere slightly away from middle), acrocentric like hormones and lipids. It also distributes those
(centromere close to end) and telocentric (terminal products throughout the cell and to other places in
centromere). The metacentric chromosome has the organism.
middle centromere forming two equal arms of the Golgi apparatus is also involved in the transport of
chromosome. In acrocentric chromosome the lipids around the cell, and the creation of lysosomes.
centromere is situated close to its end forming one Cristae are the infoldings or inward projections of
extremely short and one very long arm. the inner membrane of the mitochondrion, which are
60. (b) The structure of cell membrane was proposed by studded with proteins and increase the surface area
Singer and Nicolson (1972) widely accepted as fluid for chemical reactions to occur like cellular
mosaic model. According to this, the quasi-fluid respiration.
nature of liped enables lateral movement of proteins Peroxisome is a small organelle present in the
within the overall bilayer. This ability to move within cytoplasm of many cells, which contains the reducing
the membrane is measured as its fluidity. The fluid enzyme catalase and usually some oxidases. This
nature of the membrane is important for functions organelle is involved in photorespiration.
like cell growth, formation of intercellular junctions, Elaioplasts, a type of leucoplast, is specialized for
secretion, endocytosis, cell division etc. the storage of lipids in plants. Elaioplasts house oil
61. (b) Pili and fimbriae are non motile appendages that are body deposits as rounded plastoglobuli, which are
out growths of membrane. Pili are present in F+ essentially fat droplets.
bacteria and are used to transport F factor (fertility 66. (a) Leeuwenhoek – He is commonly known as "the
factor) from F+ bacteria to F– bacteria during Father of Microbiology", and considered to be the
conjugation. first microbiologist. First saw and described a living
62. (d) Plant cells have cell wall to couuteract turgor cell.
pressure (T.P) by exerting exactly equal and opposite Robert Brown was a Scottish botanist and palaeobotanist
wall pressure. Wall pressure stops entry of water who made important contributions nucleus and
into plant cells beyond a certain limit thus prevents cytoplasmic streaming.
their bursting. Schleiden was a German botanist and co-founder of
63. (a) Tonoplast is a membrane separating a vacoule from the cell theory, along with Theodor Schwann and
the surrounding cytoplasm in a plant cell. It controls Rudolf Virchow. He discovered that all plants are
ionic movement in and around the cell. Contractile composed of different kind of cells.
vacuole is a sub-cellular structure found in unicellular Schwann - He discovered that presence of cell wall
algae. It absorbs excess water and wastes from a is unique to plant cells.
microorganism’s cell and excretes them into the 67. (b) Viruses – 0.02 - 0.2 micrometre
environment. Food vaculole is a membrane-unclosed PPLO ( Pleuro Pneumonia like Organisms) – About
cell vacuole with a digestive function. It contains 0.1 micrometre
material taken up in by the process of phagocytosis. Eukaryotic cells – 10-20 micrometre
Air vacuole is a small cavity or space in the tissues. Bacterium – 1-2 micrometre
It stores metabolic gases.
68. (c) Mitochondria are double membrane cell organelles
64. (a) Metacentric chromosomes have the centromere in with the outer membrane and inner membrane.
the centre, such that both sections are of equal Lysosomes are a single membrane bound vesicular
length. Human chromosome 1 and 3 are metacentric. structure. It contains many hydrolytic enzymes and
Sub metacentric chromosomes are a chromosome commonly called as suicidal bag of the cells.
with the centromere so placed that it divides the Ribosome is a granular structure, without membrane.
chromosome into two arms of strikingly unequal It consists of RNA and proteins and is the protein
length. builders or the protein synthesizers of the cell.
Acrocentric chromosomes have a centromere which 69. (a) Golgi apparatus is a cellular organelle which is
is located almost near the tip leading to one very involved in in the modifying, sorting and packaging
long and one very short section. Human of proteins for secretion. Mitochondria are known
chromosomes 13, 15, 21, and 22 are acrocentric. as the power house of the cell as they are the site of
Telocentric chromosomes have centromere which is chemical reactions that transfer energy from organic
located at the terminal end. Telomeres may extend compounds in ATP. Vacuole is a membrane bound
from both ends of the chromosome. Humans do not organelle and filled with water containing inorganic
possess telocentric chromosomes. and organic molecules including enzymes in solution.
65. (a) SER (smooth endoplasmic reticulum) found in both Vacuoles might store food or any variety of nutrients
animal cells and plant cells, makes cellular products a cell might need to survive. Grana are a stacked
EBD_7209
Hints & Solutions S-47

membranous structure within the chloroplasts of chlorophyll and provide food to all organisms
plants and green algae that contains the chlorophyll through photosynthesis.
and is the site of the light reactions of 76. (b) Plasmodesmata are cytoplasmic bridges between two
photosynthesis. adjacent cells which connect the neighbouring cells.
70. (a) RER (rough endoplasmic reticulum) bears ribosomes 77. (d) Amyloplasts store carbohydrates (starch), e.g.,
on their surface. RER is involved in protein synthesis potato whereas proteins are stored by aleuroplasts.
and secretion. Cell wall is a characteristic feature of Chlorophyll pigments are presents in the thylakoid.
plant cell, bacteria, fungi, algae and some archaea. 78. (c) Centrioles serve as basal bodies for cilia and flagella.
The major function of the cell wall is to provide They are concerned with spindle formation during
rigidity, tensile strength, structural support, cell division. Lysosomes are membrane-bound
protection against mechanical stress and infection. organelles containing many hydrolytic enzymes,
Flagella are a slender thread-like structure, especially which are optimally active at an acid pH (near pH
a microscopic whip-like appendage which enables 5). Thylakoids are membrane-bound organelles
many protozoa, bacteria, spermatozoa, etc. to swim. found within chloroplasts. The thylakoid membrane,
It is responsible for cell movement. Lysosome is a forms many flattened, fluid-filled tubules that
single membrane bound vesicular structure and enclose a single convoluted compartment. These
contains many hydrolytic enzymes. It is commonly tubules tend to stack on top of each other to form a
called as suicidal bag of the cells and involved in structure called a granum.
intracellular and extracellular digestion. 79. (a) Centriole is a non-membrane bound organelle which
71. (b) Mycoplasma cells are bacteria without walls. helps in cell division. It is composed mainly of a
Plasmid is a genetic structure in a cell that can replicate protein called tubulin. Fimbria is a special structure
independently of the chromosomes. It is typically a of bacteria. It is small bristle like fibres sprouting out
small circular DNA strand in the cytoplasm of a of the cell and help bacterias to attach with rocks in
bacterium or protozoan are saclike membranes that make stream and also to host tissue. Endomembrane
up grana in a chloroplast. system includes those organelles (like ER, golgi body,
Lysosomes are a single membrane bound vesicular lysosome, vacuole) whose functions are coordinated.
structure. It contains many hydrolytic enzymes and Mitochondria are a double membrane cell organelle
commonly called as suicidal bag of the cells. It is which divide by fission and site of aerobic
involved in intracellular and extracellular digestion. respiration.
72. (c) Pair A is correct. 80. (c) Mesosome is special structure, formed by the
Centrioles – It is composed mainly of a protein called extensions of plasma membrane. It helps in the
tubulin. formation of cell wall, DNA replication and
Lysosomes – It stores hydrolytic enzymes. distribution to daughter cells. They also help in
respiration, secretion processes, to increase the
Amyloplasts – They are responsible for the synthesis
surface area of the plasma membrane and enzymatic
and storage of starch granules, through the
content. It is the characteristics of prokaryotes.
polymerization of glucose.
81. (a) In the given figure of cilia/flagella, the marked part
Elaioplasts – It stores oil and fats.
as A, B, C and D are respectively plasma membrane,
Aleuroplasts – It stores proteins. interdoublet bridge, central microtubule, and radial
73. (b) Protein synthesis takes place in ribosomes, which are spoke.
attached to surface of endoplasmic reticulum by 82. (a) The figures (A, B, C and D) are respectively
ribophorin-I and ribophorin-II. About 50 hydrolytic telocentric chromosomes, acrocentric chromosomes,
enzymes are found in the lysosome. They include sub metacentric chromosomes and metacentric
proteases, nucleases, glycosidases, lipases, chromosomes. These chromosomes are classified on
phospholipases, phosphatases and sulphatases. All the basis of location of centromere.
lysosomal enzymes are acid hydrolases and optimally 83. (b) In the given diagram of chloroplast, the parts marked
active at pH-5.0. as A, B, C and D are respectively granum, thylakoid,
74. (b) Pili are orgonelles of adhesion allowing bacteria to stromal lamella and stroma.
colonize and resist flushing. Granum is a stacked membranous structure that
75. (b) Elaioplasts are colourless plastids which store lipids. contains the chlorophyll and is the site of the light
Chromoplasts are yellow or reddish in colour because reactions of photosynthesis. Thylakoids are saclike
of the presence of carotenoid pigments. Chloroplasts membranes that make up grana. Stromal lamella are
are the most common type of plastids and are of the extensions that run from one granum, through
prime biological importance as they contain the stroma, into a neighbouring granum.The space
S-48 Biology
between the inner membrane and the thylakoid length. Human chromosome 1 and 3 are metacentric.
membrane is filled with stroma, a matrix containing 90. (b) The mitochondrial inner membrane forms infoldings
dissolved enzymes, starch granules, and copies of known as cristae, which allow greater surface area
the chloroplast genome. for protein such as cytochrome to function properly
84. (a) In the given diagram of mitochondria, the structure and efficiently.
marked as A, B, C, D and E are respectively outer 91. (c) The given figure shows the structure RER (rough
membrane, inner membrane, matrix (space within the endoplasmic reticulum). RER is involved in protein
inner membrane), intermembrane space (the space synthesis and responsible for the assembly of many
between the outer and inner membranes) and crista proteins (in a process called translation).
(formed by infolding's of the inner membrane). 92. (b) In fig., (A) shows the cell organelle mitochondria.
Mitochondria are a membrane-bound organelle The mitochondria are bounded by two membranes,
found in most eukaryotic cells and also called as the i.e., outer membrane and inner membrane.
power house of the cell. Mitochondria are referred as “power house” of the
85. (a) In the given diagram of endoplasmic reticulum, the cell as they produce 95% of ATP. This energy is
components marked as A, B, C, D and E are produced during the breakdown of food molecules
respectively nucleus (viii), rough endoplasmic which involve glycolysis, oxidative decarboxylation
reticulum (v), ribosomes (vii), cytoplasm (iii) and and oxidative phosphorylation (Krebs cycle and
smooth endoplasmic reticulum (iv). respiratory chain).
93. (d) Cell 1 is a bacterial cell and cell 2 is a plant cell. Cell
Endoplasmic reticulum is a network of membranous
wall and ribosomes are present in both plant and
tubules within the cytoplasm of a eukaryotic cell, bacterial cells. Nucleus is not present in the bacterial
continuous with the nuclear membrane. It usually cells.
has ribosomes attached and is involved in protein 94. (c) The cellular structure Y (nucleus) helps in transfer
and lipid synthesis. of genetic material from one generation to another.
86. (a) In the given figure of plant cell, the structure marked Nucleus is a dense organelle present in most
as A, B, C, D and E are respectively plasmodesmata, eukaryotic cells, typically a single rounded structure
rough endoplasmic reticulum, Golgi apparatus, bounded by a double membrane, containing the
mitochondrion and ribosomes. genetic material.
Plant cells are eukaryotic cell and have certain 95. (a) The cell membrane (marked as X) helps control entry
distinguishing features, including chloroplasts and exit of substances into the cell. Nucleus passes
(Photosynthesis takes place) cell walls (allow plants the information from the parent cell to the new cell. It
to have strong, upright structures) and intracellular is also called the control center of the cell. A cell wall
vacuoles (help regulate how cells handle water and is the tough, rigid layer present outside of the plant
storage of other molecules). cell. It provides structural support and protection to
87. (a) The given diagram shows the fluid mosaic model of the cell. Chloroplast occurs in the plant cells and
cell membrane. In the diagram the components marked contains chlorophyll that helps the cell in making
as A, B, C, D and E are respectively (i) sugar, (ii) food.
96. (a) Microtubules occur in the cytoplasm of all eukaryotic
protein, (iii) lipid bilayer, (iv) integral proteins, and
cells. Besides cytoplasm, microtubules occur in many
(v) cytoplasm. The fluid mosaic model was devised
specialized cellular structures like cilia, flagella, basal
by SJ Singer and GL Nicolson in 1972, describes the bodies, centrioles, astral rays, spindle apparatus,
cell membrane as a two-dimensional liquid in which chromosome fibres etc.
phospholipid and protein molecules diffuse easily. 97. (c) Bacteria and blue green algae are unicellular
88. (c) The given cell organelle is Golgi body. Golgi organisms which are devoid of membrane bound
apparatus is found in most eukaryotic cells. It was nucleus and organelles. Golgi apparatus is membrane
identified in 1897 by the Italian physician Camillo bound organelle absent in prokaryotes.
Golgi and named after him in 1898. Functions as a 98. (c) In plant cells, the vacuoles can occupy upto 90
factory in which proteins received from the ER are percent of the volume of the cell. The vacuole is
further processed and sorted for transport to their bound by a single membrane called tonoplast.
eventual destinations: lysosomes, the plasma 99. (d) Cell wall is a characteristic feature to cells of plants,
membrane, or secretion. bacteria, fungi, algae and some archaea. It is located
89. (b) The figure shows the structure of metacentric (C) outside the cell membrane. The major function of the
chromosomes. The parts marked as A and B are cell wall is to provide rigidity, tensile strength,
respectively are satellite and secondary constriction. structural support, protection against mechanical
Metacentric chromosomes have the centromere in stress and infection. It also help in diffusion of gases
the centre, due to which both sections are of equal in and out of the cell. Gametes, Amoeba and
Mycoplasma lack cell wall.
EBD_7209
Hints & Solutions S-49

100. (a) Lysosomes are membrane-enclosed organelles and energy to do so. That energy does not always have
consist of hydrolytic enzymes which capable of to be directly supplied in the form of ATP.
breaking down all types of biological polymers (like 109. (c) The cytoskeleton supports the cell and allows
proteins, nucleic acids, carbohydrates, and lipids). movement of the entire cell and microtubules, motor
If lysosomes get ruptured in a cell, the cell dies due proteins and actin filaments are part of the
to release of hydrolytic enzymes which destroys the cytoskeleton.
cells. 110. (d) We can eliminate bacteria because they lack
101. (c) Cell sap is a non-living fluid manufactured by the organelles completely. Of the organelles listed the
cytoplasm of the living plant cell. It consists of water only unique one is the chloroplast, which is found
and various substances that are often in the form of exclusively in plants.
a colloidal suspension. The cell sap adds osmotic 111. (a) The only definitive characteristic listed the presence
properties and turgor of the cell, according to the or absence of a nucleus.
elasticity of plant tissues and organs. It also serves 112. (d) This connection allows material produced in the
as a receptacle for the water and various substances nucleus to directly enter the ER via the nuclear pores.
that participate in cell metabolism and as a place for 113. (b) The pellet is undergoing cellular respiration, a
the deposit of the end products of metabolism. function that occurs in the mitochondria. You can
102. (a) The concept of membrane fluidity refer to the fact also assume that the single membrane of the cell
that both lipids and proteins may have considerable itself is ruptured, other single membrane-bound
freedom of lateral movements within the bilayer. The organelles would be ruptured as well.
fluidity of membranes in a plant in cold weather may 114. (c) Part in sample A and B are respectively mitochondria
be maintained by increasing the number of and chloroplast. They were called as semi-
phospholipids with unsaturated hydrocarbon tails. autonomous organelles because they are capable of
103. (d) Mitochondria are present in the living eukaryotic synthesis of their own proteins and contain their
cells and absent in prokaryotic cells like bacteria and own DNA; within the cell they are partially
blue-green algae. This organelle is known as the dependent upon cellular metabolism.
powerhouse of the cell as they are the site of chemical 115. (a) The samples belong to eukaryotic cells because
reactions th at transfer energy from organic eukaryotic cell is a type of cell which have enclosed
compounds in the form of ATP. membrane bound organelles and nucleus. And both
104. (c) Simple diffusion is a process in which plasma the mitochondria and chloroplast are membrane
membrane does not require any carrier molecule to bound organelles. Whereas prokaryotic cells are
pass small non- charged molecules or lipid soluble morphologically the most primitive cells and they
molecules between the phospholipids to enter or leave don't have a membrane bound nucleus and
the cell. The particles move from areas of high organelles. Prokaryotic cells are generally smaller in
concentration to areas of low concentration (they size and multiply more rapidly than the eukaryotic
move down their concentration gradient). Oxygen and cells.
carbon dioxide and most lipids enter and leave cells 116. (b) The cell wall protects the cell, maintains its shape
by simple diffusion. and provides supports and strength to it. Nucleus
105. (c) Vacuole is a non - living reservoir, bounded by a controls the cell's activities. Chloroplast is a cell
selectively permeable membrane, the tonoplast. It is organelle that makes food for the plant cell. The cell
not a air filled cavity but it is filled with a highly membrane holds the parts of the cell together and
concentrated solution called vascular sap or cell sap. also separates the cell from its surroundings.
pH of vacuolar cell sap is acidic and hypertonic. 117. (a) Cell theory explains the general principles of
106. (c) Cell organelles like ribosomes and nucleolus lack the construction for all living things by focusing on
unit membrane. Ribosomes are found individually in studying the working of cells. Th e use of
the cytoplasm and also line the membranes of the microscopes to study cell was in practice way before
rough endoplasmic reticulum. It is composed of RNA the cell theory was formulated.
and ribosomal protein. Nucleolus is a part of nucleus
and characterized by the absence of limiting Chapter 9 : Biomolecules
membrane, presence of chromatin and granules and 1. (c) In order to study the various biomolecules found in
fibrils of RNA and proteins.
107. (d) Endoplasmic reticulum bearing ribosomes on their living tissues, the living tissues are ground in
surface is called rough endoplasmic reticulum (RER). trichloroacetic acid using a mortar and a pestle to
RER is involved in protein synthesis and are form a thick soup. It is then strained to obtain
responsible for the assembly of many proteins (in a two fractions : acid soluble and acid insoluble
process called translation). fraction. Chemicals present in both the fractions are
108. (d) Active transport is the transport of ions or molecules further separated by various analytical techniques
against a concentration gradient and it requires
and identified.
S-50 Biology
2. (a) Biomolecules are organic compounds normally by a covalent bond. Nucleosides are compounds
present as essential components of living organisms. formed from a nitrogen base and pentose sugar.
3. (d) Glutamic acid and aspartic acid are acidic amino acid 15. (a) DNA and RNA are made up of nucleic acids. They
because they bear one amino acid group and two are the vehicles of genetic inheritance. Building
blocks of nucleic acids are called nucleotides. Each
carboxylic group. Lysine is a basic amino acid.
nucleotide consists of three parts: a sugar (ribose
4. (d) Aromatic amino acid possess cyclic structure with a for RNA and deoxyribose for DNA), a phosphate,
straight side chain bearing carboxylic and amino and a nitrogenous base. These nucleotides linked
group. Valine is a neutral amino acid. It contains equal together with covalent bonds to form a sugar-
number of amino and carboxylic groups. phosphate backbone with extended nitrogenous
5. (a) Arginine, lysine and histidine are basic amino acids bases.
with two amino groups and one carboxylic group. 16. (c) Inulin is a heterogeneous collection of fructose
Arginine is the most and histidine is the least basic polymers. It consists of glucosyl moiety and fructosyl
amino acid. moiety, which are linked by b (2, 1) bonds. It is a
soluble dietary fibre which is a naturally occurring
6. (a) Glycine is the simplest amino acid with lowest
oligosacch ar ide belonging to a group of
molecular weight. The side chain in glycine is
carbohydrates known as fructans. Unlike most
repr esented by carbohydrates, inulin is non-digestible.
H (Hydrogen) atom (not alkyl group). Due to 2-
Inulin is naturally present in many different foods
opposite H-atoms, this amino acid is symmetrical or such as asparagus, leek, onions, banana, wheat and
achiral. All other amino acids are assymetrical or chiral, garlic. Higher concentrations exist in herbs.
and can exist as D- and L-stereoisomers. 17. (b) Proteins are polypeptide. A polypeptide is formed
7. (b) Zwitterion is a neutral molecule with a positive and a of a linear row of amino acids joined by peptide bonds.
negative electrical charge, though multiple positive This constitutes the primary structure (1°).
and negative charges can be present. Amino acids
18. (a) The collagen is the most abundant protein of animal
are the best known examples of zwitterions.
world.
8. (d) Saturated fatty acids do not have any double bond
19. (a) Quaternary structure is found only in multimeric
while unsaturated fatty acids have one or more
protein. In quaternary structure, more than one
double bonds.
polypeptide chains are involved to form a large
9. (d) Essential fatty acid are some polyunsaturated acid
multiunit protein. e.g., haemoglobin.
which can not be synthesized in the animal body.
Therefore it must be supplied with food to avoid 20. (c) Glycosidic bonds are bonds established between
their deficiency. Linoleic acid, linolenic acid and aldose or ketose group of carbohydrate with
arachidonic acids are some examples of essential fatty alcoholic or nitrogen group of another organic
acids. compound.Two amino acid molecules can be
10. (d) Phospholipids are composed of phosphate group and covalently joined through a substituted amide
one or more fatty acids. They have hydrophilic (polar) linkage, termed a peptide bond.
phosphate group and long hydrophobic (non-polar) 21. (b) Turn over number is the number of substrate
hydrocarbon ‘tails’. The phospholipids readily form molecules which can be catalyzed by a single
membrane like structure in water. molecule of an enzyme in a unit time. Turnover number
11. (b) Glycerol is a trihydroxy propane compound and has of enzyme is dependent on active site, rapidity of
three hydroxyl groups that are responsible for its reaction and separation of end products.
solubility in water and its hygroscopic nature. It is a 22. (c) The concentration of substrate at which velocity of
colourless, odourless, viscous liquid that is widely enzymatic action reaches half of its maximum value,
used in pharmaceutical formulations. is called Km value or Michaelis Menten constant.
12. (b) Fat is a group of natural esters of glycerol and various 23. (b) A ribozyme is an RNA molecule that catalyzes a
fatty acids, which are solid at room temperature and chemical reactions, similar to the action of protein
are the main constituents of animal and vegetable enzymes.
fat.
24. (c) Feedback inhibition of enzymes is affected by end
13. (a) Lecithin is a choline containing phospholipid.
product. Feedback inhibition occurs when the end
14. (c) A nucleotide is a compound made of a nitrogen base, product of a reaction interferes with the enzyme that
a pentose sugar and phosphate all linked together helped produce it. The inhibitor does this by binding
EBD_7209
Hints & Solutions S-51

to a second active binding site that's different from 35. (c) Almost all enzymes are proteins. Certain RNAs have
the one attached to the initial reactant. The enzyme also been reported to have enzyme property. They
then changes its shape and can't catalyze the reaction are called ribozymes or RNA enzymes. No enzyme
anymore. This type of inhibition is done as a exists which is made up of lipid.
regulatory mechanism to meet the metabolic needs 36. (a) Coenzymes are organic non-protein molecules that
of the cell or organisms. bind with the protein molecule (apoenzyme) to form
25. (d) Coenzyme is a non-proteinaceous organic substance the active enzyme (holoenzyme). They function as
that usually contains a vitamin or mineral and intermediate carriers of electrons, specific atoms or
combines with a specific protein, the apoenzyme, to functional groups that are transferred in the overall
form an active enzyme system. reaction. Every coenzyme is a cofactor is not a
26. (a) Holoenzyme is conjugated enzyme which consists coenzyme.
of a protein part called apoenzyme and a non-protein 37. (d) Nitrogen bases are heterocyclic compounds. Length
called cofactor. Coenzyme are also organ ic
of one turn of DNA is 34Å.
compounds but their association with apoenzyme
38. (c) Starch can hold iodine (I2) molecules in its helical
is only transient and acts as cofactors.
secondary structure but cellulose being non helical,
27. (d) Inorganic catalyst work efficiently at h igh
can not hold I2. Thus, cellulose does not shows blue
temperature and high pressure. Inorganic catalysts
colour when treated with I2.
speed up reactions, but they do not have carbon-
39. (c) Enzymes are biological molecules (proteins) that act
hydrogen atoms. An example of this is magnesium
as catalysts and help complex reactions occur.
sulphate, which is a compound used to speed up
Enzymes bind temporarily to one or more of the
some reactions in the chemistry lab.
reactants the substrate(s) of the reaction they
28. (c) Amino acids are the building blocks proteins. Amino
catalyze by lowering the amount of activation energy
acid can be classified on the basis of characteristics
needed and thus speed up the reaction.
of their side chains.
40. (b) The correct sequence of the steps in catalytic cycle
29. (b) Lipid molecules are insoluble in water because lipid
of an enzyme actions is : iv, iii, ii, i.
molecules are hydrophobic, but they are readily
41. (d) 42. (a)
soluble in nonpolar organic solvents like chloroform
43. (c) Prosthetic group is an organic substances which is
and benzene. dialyzable, thermostable and firmly attached to the
30. (c) Lysine is a basic amino acid. protein or apoenzyme portion during the complete
31. (b) A protein is a heteropolymer and not a homopolymer. catalytic cycle. Apoenzyme is a protein that forms
32. (c) Options (a), (b) and (d) are incorrect. an active enzyme system by combination with a
Chitin is a structural heteropolysaccharide found in coenzyme and determines the specificity of this
the fungal walls as fungus cellulose and as chitin in system for a substrate.
the exoskeloton of arthropods. 44. (b) In polysaccharides, individual monosaccharide is
Collagen is the most abundant protein in animal world linked by glycosidic bond. This bond is formed
while RuBisCo is the most abundant protein in the between two carbon atoms of two adjacent
whole of the biosphere. monosaccharides. A glycosidic bond is a type of
covalent bond that joins a carbohydrate (sugar)
In a polysaccharide, the individual mono-saccharides
molecule to another group, which may or may not be
are linked by a glycosidic bond. Phosphodiester
another carbohydrate. Glycosidic bonds are formed
bonds help in polymerization of nucleotides to form by dehydration.
polynucleotide. 45. (d) Essential amino acids are those which are taken from
33. (a) Chitin is a long-chain polymer of a N- food and not synthesized in the body whereas non-
acetylglucosamine, a derivative of glucose. It is a essential amino acids need not be supplied in the diet
characteristic component of the cell walls of fungi, and are synthesized in the body. Glycine, serine and
exoskeletons of arthropods such as crustaceans (e.g., tyrosine are non-essential amino acids.
crabs, lobsters and shrimps) and insects, the radulae 46. (c) Compound having double bond in their structure are
of molluscs, and the beaks and internal shells of more unstable compound in comparison to single
cephalopods, including squid and octopuses. bond holder compounds. Unsaturated fats those have
34. (c) Abrin and ricin are toxin secondary metabolities. The double bonds in their structures are more reactive
than saturated fats.
secondary metabolities which are used as drugs are
47. (c) In a DNA molecule, A-T rich parts melt before G-C
vinblastin, curcumin, etc.
rich parts because there are two H-bond between A
S-52 Biology
and T whereas in between G and C, there are three H- cells which are restricted by their cell walls).
bond. 51. (a) Collagen is a class of extracellular proteins abundant
48. (c) Non-essential amino acids are those amino acids in higher animals. They are specially found in the
which need not be supplied in the diet because they skin, bone, cartilage, tendon, and teeth, forming
can be synthesised by the body, particularly from strong insoluble fibres. It serves as connective tissue
carbohydrate metabolites, Glycine is one such non between cells, yielding gelatin when denatured by
essential amino acid. On the contrary, essential amino boiling and Trypsin is an enzyme found in the
acids are those amino acids which can not be digestive system of many vertebrates, where it
synthesised in the animal body and must be supplied hydrolyses proteins. It is produced in the pancreas
with food in adequate amounts. Out of twenty amino as the inactive protease trypsinogen. Insulin is a
acids, eight are considered essential in human diet. pancreatic hormone which regulates the metabolism
49. (b) Palmitic acid is a solid saturated fatty acid, obtained of carbohydrates and fats by promoting the
from palm oil and other vegetable and animal fats. absorption of glucose from the blood to skeletal
Lecithin is a group of phospholipids. It is composed muscles and fat tissue and by causing fat to be stored
of units of choline, phosphoric acid, fatty acids, and rather than used for energy. GLUT – 4, also known
glycerol. It occurs in animal and plant tissues and as glucose transporter type 4, is a protein that in
egg yolk, Phospholipids are fat derivatives in which humans is encoded by the GLUT4 gene. GLUT4 is
one fatty acid has been replaced by a phosphate the insulin-regulated glucose transporter found
group and one of several nitrogen-containing primarily in adipose tissues and striated muscle
molecules. Tryptophan is an example of aromatic (skeletal and cardiac). The first evidence for this
amino acids. It is an essential amino acid which is a distinct glucose transport protein was provided by
constituent of most proteins. It is a precursor of the David James in 1988. The gene that encodes GLUT4
neurotransmitter serotonin, melatonin, a hormone was cloned and mapped in 1989.
related to sleep, and the B vitamin niacin. Tryptophan 52. (b) A – II, B – V, C – I, D – III, E – IV
is also necessary for normal growth and development
of infants. Glutamic acid is a non- essential acidic Component % of the total
amino acid. The carboxylate anions and salts of cellular mass
glutamic acid are known as glutamates. Glutamate is A Water II 70 – 90
an important neurotransmitter that plays the principal B Proteins V 10-15
role in neural activation.
C Carbohydrates I 3
50. (a) Insulin is a type of carbohydrate. Basically it is a
peptide hormone produced by beta cells of pancreas. D Lipids III 2
It regulates the metabolism of carbohydrates and E Nucleic acid IV 5-7
fats by promoting the absorption of glucose from
the blood to skeletal muscles and fat tissue and by 53. (a) Carotenoids and anthocyanin are types of pigments.
causing fat to be stored rather than used for energy. Carotenoids are naturally occurring pigments
Trypsin is an proteinaceous enzyme which is found synthesized by plants, algae, and photosynthetic
in the digestive system where it hydrolyses proteins. bacteria. These richly coloured molecules are the
It is produced in the pancreas as the inactive protease sources of the yellow, orange, and red colours of
trypsinogen. many plants. Anthocyanin is a water soluble blue,
Adenylic acid is a nucleic acid. It is a compound violet or red flavonoid pigment found in plants.
consisting of an adenosine molecule bonded to one Terpenoids are large class of organic compounds
acidic phosphate group. It is present in most DNA including terpenes, diterpenes, and sesquiterpenes.
and RNA. It typically exists in a cyclic form with the They have unsaturated molecules composed of
phosphate bonded to the nucleoside at two points. linked isoprene units, generally having the formula
Cholesterol is a sterol (or modified steroid), a lipid (C5H8) n.
molecule. It is biosynthesized by all animal cells Alkaloids are nitrogenous organic compounds of
because it is an essential structural component of plant origin which have distinct physiological
cell membranes that is required to maintain both actions on humans. They include many drugs
membrane str uctural integrity and fluidity. (morphine, quinine) and poisons (atropine,
Cholesterol enables animal cells to (a) not need a cell strychnine).
wall (like plants & bacteria) to protect membrane Concanavalin A is member of legume lectins
integrity/cell-viability and thus be able to (b) change (carbohydrate-binding protein) family. It binds
shape and (c) move about (unlike bacteria and plant specifically to certain structures found in various
sugars, glycoproteins, and glycolipids, mainly
EBD_7209
Hints & Solutions S-53

internal and non-reducing terminal a-D-mannosyl structure, quaternary structure, tertiary structure and
and a-D-glucosyl groups. secondary structure.
54. (b) Cellulose, an insoluble substance, is the main 66. (c) Cholesterol is a 4-ringed structure (3-cyclohexane
constituent of plant cell walls and of vegetable fibres ring and
such as cotton. It is type of a polysaccharide which 1-cyclopentane ring) found only in animals.
consists of chains of glucose monomers. Chitin is a 67. (c) Phosphate is bound to pentose sugar by ester bond.
derivative of glucose, and is a characteristic 68. (c) In the given graph which shows the change in the
component of the cell walls of fungi, the exoskeletons concentration of substrate on enzyme action
of arthropods such as crustaceans (e.g., crabs, activity, letter marked as A, B and C is respectively
lobsters and shrimps) and insects. Glycogen is a Vmax, Km, and Vmax/2.
polysaccharide which is deposited in the bodily (Vmax) is the maximum velocity of the enzyme which
tissues as a store of carbohydrates. It forms glucose determines the maximum speed of enzymatic reaction
on hydrolysis and found primarily in the liver and during which the substrate concentration is
muscle tissue. Starch is an odourless, tasteless white increased until a constant rate of product formation
substance occurring widely in plant tissue. It is is achieved. This is called maximum velocity (Vmax)
obtained chiefly from cereals and potatoes and of the enzyme. Km, the Michaelis – Menten constant
functions as a carbohydrate store and is an important is the substrate concentration at which an enzyme
constituent of the human diet. attains half its maximal velocity.
55. (c) Enzymes are generally classified on the basis of the 69. (c) The correct name of the structural formula of the
type of reactions that they catalyse, like given amino acids - X, Y and Z are respectively
oxidoreductases, transferase, hydrolases, glycine, alanine and serine.
isomerases, lyases and ligases. 70. (a) The given structural formula represents glucose
56. (a) Adenine is purine while thymine is pyrimidine. (C6H12O6).
57. (c) Three molecules of fatty acid linked with one 71. (b) The graph shows the concept of activation energy.
molecule of glycerol to form a simple lipid called Activation energy is the least amount of energy
triglycerides. required to activate atoms or molecules to a state in
58. (c) Enzyme is made up of proteins. They are catalysts which they can undergo a chemical reaction. The label
(biocatalysts) that speed up chemical reactions in the marked as 1, 2, 3 and 4 in the graph are respectively
cell without themselves undergoing any permanent correlated with the statements I, II, III and IV.
chemical change. 72. (b) In the given graph, X and Y are respectively
59. (d) Cellulose is a polysaccharide and homopolymer that activation energy with enzyme and activation energy
consists of a long unbranched chain of glucose units. without enzyme.
60. (c) Valine is a neutral essential amino acid which 73. (c) Parabolic curve in option (c) correctly represents the
contains equal number of amino and carboxylic group. effect of pH on the velocity of a typical enzymatic
61. (d) Phosphodiester bonds help in polymerization of reaction (V). Enzyme shows its highest activity at a
nucleotides to form polynucleotide. particular pH called the optimum pH. Activity
62. (a) Ribozymes (ribonucleic acid enzymes) are also called declines both below and above the optimum value.
catalytic RNA. They are RNA molecules capable of 74. (b) In the given curve the relation of temperature and
catalysing specific biochemical reactions, similar to enzyme activity is shown.
the act of protein enzymes. Ribozyme, discovered in 75. (d) A combination of a nitrogen base with a pentose
1982, demonstrated that RNA can be both genetic sugar is known as nucleoside. The nitrogen base
material (like DNA) and a biological catalyst (like combines with the sugar molecule at its carbon atom
protein enzymes). Examples of ribozymes include the 1¢ in a glycosidic bond (C – N – C) by one of its
hammerhead ribozyme, the VS ribozyme, Leadzyme nitrogen atoms (usually 1 in pyrimidines and 9 in
and the hairpin ribozyme. purines). Depending upon the type of pentose sugar,
63. (c) Options (a), (b) and (d) are wrong. nucleosides are differentiated into ribonucleosides
(a) Inulin is a polymer of fructose. and deoxyribonucleosides.
(b) Starch forms helical secondary structure. 76. (d) Lecithin is a fat like substance called a phospholipid,
(d) Glycogen is glucosan homopolysaccharide which which is a part of plasma membrane. Adenine is a
is the major reserve food of animals, fungi and some nitrogenous base.
bacteria. 77. (a) This graph represents the effect of substrate
64. (b) The different amino acids are attached through–Co– concentration on the velocity of enzyme action in
NH bond, called peptide bond or amide bond. which the rate of engyme reaction is directly
65. (c) The type of protein structure shown in the given proportional to the substrate concentration.
figures A, B, C and D are respectively primary
S-54 Biology

R photosynthesis. During photosynthesis carbon


H dioxide is reduced into carbohydrates by water and

78. (b)
+ O shows zwitter ionic form. Zwitterions oxygen is liberated.
H3N
O 88. (a) Palmitic acid is one of the most common saturated
fatty acids found in animals and plants. It has 16
are those ions that are electrically neutral overall but
carbons including the carboxyl carbon. Adenylic acid
contain non adjacent regions of positive and negative
is a nucleotide consisting of adenine, ribose or
charges. They are sometimes referred to as dipolar
deoxyribose, and a phosphate group. It is a
ions. Free amino acid found in cells is an example of
zwitterion. General structure of an amino acid constituent of DNA or RNA. It is also called
exposes two parts, or groups, of the molecule that adenosine monophosphate. Amino acids are organic
can function as an acid/base pair, the –COOH and acids (with carboxylic group COOH) having amino
–NH2 groups. group (–NH2) generally attached to carbon or carbon
79. (d) According to weight percentage, the first three elements next to carboxylic group. The carbon also bears a
in human body are O > C > H. % Weight of O (oxygen), variable alkyl group (R) or hydrogen or hydrocarbon.
C (carbon) and H (hydrogen) in human body are 65.0, In alanine it is represented by methyl group.
18.5 and 0.5. 89. (c) The effectiveness of an enzyme is affected least by
80. (b) In the comparison of cellular mass, the decreasing the original activation energy of the system. Because
order of the components is: activation energy is the least amount of energy
P (Protein):10 – 15% >N (Nucleic acid): 5 – 7 % > C required to start chemical reaction.
(Carbohydrates): 3% > L (Lipid): 2% 90. (d) Thiamine is nitrogen base. Acetic acid is a weak acid,
81. (a) Amino acids are building blocks of protein which Cytidylic acid is a nucleotide.
are joint by peptide bond. 91. (a) Homopolysaccharides composed of glucose which
82. (d) Relation between the protein and amino acid is similar includes glycogen and starch, the storage
to one found in the nucleotides and nucleic acid. In carbohydrates of animals and plant respectively.
the first case, proteins are polysaccharide and amino 92. (b) Transferase is a class of enzyme that catalyse the
acids are the building blocks of proteins. Similarly transfer of a group of atoms from one molecule to
nucleotides are the building blocks of nucleic acid. another. The given reaction shows the involvement
83. (a) Nucleotides are the building blocks of nucleic acid. of transferase enzyme.
Each nucleotide consists of three parts: a sugar 93. (b) Transition state structure formed during an
(ribose for RNA and deoxyribose for DNA), a enzymatic reaction is transient and unstable.
phosphate, and a nitrogenous base. 94. (b) Quaternary structure is the three-dimensional
structure of a multi-subunit protein. For a protein to
84. (b) The two strands of DNA run in opposite directions
have a quaternary structure, it must consist of two
to one another with the hydrogen bonds between
or more polypeptide subunits.
them. One strand of DNA has 5¢-3¢ direction and the
95. (d) Haemoglobin consists of 4 subunits. Two of these
other strand has 3¢-5¢ direction. So they are
are identical to each other. Hence, two subunits of a
antiparallel. This direction is determined by the
type and two subunits of b type together constitute
presence of a free phosphate or OH group at the end
the human haemoglobin (Hb).
of the strand. If the strand has phosphate group at
96. (a) Temperature affects activity of enzyme. At very low
the 5¢ end then a free OH group present at the 3¢ end.
temperature (almost near freezing point) the enzymes
85. (c) Enzymes increase the rate of reaction without being
are inactivated whereas high temperature denatures
consumed in the reaction.
enzymes permanently. Temperature ranges for maximum
86. (c) All enzymes are proteinaceous in nature. This
functioning of enzyme is 25 – 40 degree Celsius.
statement is now modified due to exception of
97. (c) A nucleoside consists of a nitrogenous base
ribozyme. Ribozymes are RNA molecules which
covalently attached to a sugar (ribose or deoxyribose)
catalyses specific biochemical reactions.
but without the phosphate group. A nucleotide
87. (a) Carbohydrates are organic compounds synthesized
consists of a nitrogenous base, a sugar (ribose or
in the chlorophyll containing cells of some bacteria,
deoxyribose) and one to three phosphate groups.
algae and green plant cells, during photosynthesis.
Certain photoautotrophic bacteria, e.g. green sulphur Nucleoside = Sugar + Base
bacteria and purple sulphur bacteria contain Nucleotide = Sugar + Base + Phosphate
pigments like chlorobium chlorophyll and 98. (a) Transferases catalyse reactions which involve group
bacteriochlorophyll respectively that helps them in transfer from one molecule to another.
EBD_7209
Hints & Solutions S-55

99. (d) The backbone is constructed from alternating ribose 109. (a) When the cofactor is removed from the enzyme,
sugar and negatively charged phosphate molecules catalytic activity of the enzyme is lost. Cofactors
which are highly polar. Because the backbone is were defined as an additional substance apart from
polar, it is hydrophilic which means that it likes to be protein and substrate that is required for enzyme
immersed in water. The DNA is antiparallel means activity.
that the two strands of DNA have opposite chemical 110. (a) The above statement shows the catabolic pathway.
polarity, or stated another why their sugar- Catabolic pathways involve the breakdown of
phosphate backbones run in opposite directions. nutrient molecules into usable forms (building
blocks). In this process, energy is either stored in
100. (d) Nucleoside is the combination of pentose sugar with
energy molecules for later use, or released as heat.
nitrogenous bases (purines or pyrimidines). So, on
The energy, liberated during this degradation, is
hydrolysis it does not yield phosphoric acid. trapped and stored in the form of chemical bonds
101. (c) Nucleotides are the building blocks of nucleic acid (ATP).
(DNA and RNA), therefore the information in a 111. (a) With the increase in substrate concentration, the
genetic nucleic acid resides in the sequences of velocity of the enzymatic reactions rises at first. The
nucleotide. The biological functions of nucleotides reaction ultimately reaches a maximum velocity which
are: storage of data (as part of DNA/RNA); energy is not exceeded by any further rise in concentration
currency (ATP); cellular communication (cAMP; of the substrate. This is because the enzyme
ATP allosteric regulator) and co-enzyme catalysis. molecules are fewer than the substrate molecules
102. (b) Km of an enzyme is the substrate concentration that and after saturation of these molecules there are no
gives half maximal velocity. Enzyme-substrate (ES) free enzyme molecules to bind with the additional
complex formation is essential in enzymatic reaction. substrate molecules.
103. (c) Fats is a group of natural esters of glycerol along Chapter 10 : Cell Cycle and Cell Division
with various faty acids. They are generally solid at
ordinary room temperature. Fats occur naturally in 1. (d) S-phase is a synthetic phase in which DNA molecules
food and play a important role in human nutrition. replicate and synthesis of histone proteins occurs.
Fats store energy in the body, protectbody tissues 2. (b) G0 phase is non-cycling quiescent stage of cell which
by insulating them, cushion internal organs, and leaves the cell cycle in the early half of G1 phase due
transport fat-soluble vitamins in the blood. Fats are to absence of cycling factors.
soft, low-melting solids, with a density less than that 3. (b) Interphase is the longest phase of the mitotic cycle,
last more than 95% of the duration of cell cycle. It
of water.
has three sub-phases : G1, S and G2. Interphase is
104. (d) Natural lipids are readily soluble in non-polar
the most suitable period to carry out FISH for
hydrocarbons such as benzene and light petroleum. detecting and locating gene mutations &
105. (c) Nucleases, which belong to the class of enzymes chromosome abnormalities.
called hydrolases, are usually specific in action. 4. (d) Interphase is called the resting phase during which
Nucleases is an enzyme that cleaves nucleic acids. cell is preparing for division by undergoing both cell
Hydrolases is one of the six main classes of enzymes growth and replication of DNA.
that catalyze a hydrolytic cleavage reaction. 5. (a) Prophase is the first stage of mitosis which follows S
106. (d) Enzymes are sensitive to temperature, enzyme and G2 phases of the interphase. In the S and G2
concentration, substrate concentration, product phases, the new DNA molecules formed are not
concentration, activations, inhibitors and pH. Each distinct but interwined. Prophase is marked by the
enzyme shows its highest activity at a particular initiation of condensation of chromosomal material.
temperature and pH called the optimum temperature 6. (b) Prophase is the stage of mitosis in which the
and pH respectively. Activity declines both below following events takes place like,
and above the optimum value. (i) The nucleolus disappears.
107. (c) Primary structure of polypeptide is stabilized or (ii) In the cytoplasm, the mitotic spindle, consisting of
secondary structure of polypeptide is maintained by microtubules and other proteins, forms between the
hydrogen bonds. two pairs of centrioles as they migrate to opposite
108. (a) The catalytic efficiency of two different enzymes can poles of the cell.
be compared by the Km value. Km is the Michaelis- (iii) The nuclear envelope disappears at the end of
Menten constant. It is the substrate concentration prophase. Which indicates the beginning of the sub
at which an enzyme attains half its maximal velocity. stage called prometaphase.
S-56 Biology
(iv) Chromosomes become more coiled and can be homologous pair. It brings about gene recombination
viewed under a light microscope. and also produces genetic variation. The chromatids
(v) Each duplicated chromosome is seen as a pair of are the unit of crossing over.
sister chromatids joined by the duplicated but un- 18. (b) Crossing over is the exchange of chromosomes
separated centromere. segments between non-sister chromatids of
7. (c) In metaphase of mitosis, centromeres of all the homologous pair. It brings about gene recombination
chromosomes lie over the equator to form an apparent and also produces genetic variation. Pachytene is
plate called equatorial plate. the third stage of the prophase of meiosis during
8. (b) Metaphase is the best stage to count the number of which the homologous chromosomes become short
chromosomes and study their morphology. and thick and divide into four distinct chromatids.
9. (d) Anaphase is the stage of mitosis or meiosis when 19. (a) Recombination involves the crossing over. Crossing
chromosomes are split and the sister chromatids over occurs during pachytene stage. It is exchange
move to opposite poles of the cell. of chromosomes segments between non-sister
10. (b) Nucleolus and nuclear membrane always disappears chromatids of homologous pair. Crossing over brings
during mitosis and meiosis. At the beginning of about gene recombination and also produces genetic
mitosis, the chromosomes condense, the nucleolus variation.
disappears, and the nuclear envelope breaks down, 20. (d) The points of attachment between homologous
resulting in the release of most of the contents of the chromosomes after their separation in diplotene
nucleus into the cytoplasm. At the end of mitosis, appear X-shaped. They are called chiasmata.
the process is reversed. In diakinesis stage of 21. (b) Leptotene is the first stage of prophase I of meiosis
prophase I of meiosis I, nucleolus and nuclear I. Leptotene is of very short duration in which
membrane disappear and chromosomes set free in progressive conden sation and coiling of
the cytoplasm. chromosomal fibres takes place. In leptotene, the
nuclear membrane enlarges and chromosomes
11. (a) Meiosis I is known as reductional division due to
become distinct. Chromosomes appear as long and
reduction in the number of chromosomes. Meiosis II
slender threads bearing beads like structure called
is called equational division because of maintaining
chromomeres.
the same number of chromosomes.
22. (b) Terminalization of chiasmata occurs in diakinesis
12. (a) Interkinesis is a brief interphase which sometimes
stage of meiosis I. It is the disappearance of
occurs between meiosis I and meiosis II.
chiasmata of sliding towards the tips of the
13. (d) The process of pairing of homologous chromosomes
chromosomes due to tight condensation.
i s
23. (a) Electron micrographs of zygotene stage of prophase
called synapsis. Synapsis occurs during zygotene
I of meiosis I indicate that chromosomes
stage of meiosis-I.
accompanied by the formation of complex structure
14. (d) Durin g telophase of mitosis, the nucleolous
called synaptonemal complex. During this stage
condense and reappear. The nuclear envelope is
chromosomes start pairing together (a process called
assembled around the chromatin mass and ER reform
synapsis).
again thus forming two daughter nuclei.
24. (d) Kinetochore is the portion of the chromosome
15. (d) The synaptonemal complex is formed at the region
centromere to which the mitotic spindle fibres
of synapsis. It is a protein structure that forms
(microtubules) attach. It is the location on the
between homologous chromosomes (two pairs of
centromere where the spindle fibre attaches.
sister chromatids) during meiosis and is thought to
25. (b) In plant cells, wall formation starts from the centre of
mediate chromosome pairing, synapsis, and
the cell and grows outward to meet the existing lateral
recombination. This stage appears in zygotene stage
walls (centrifugal).
of prophase I of meiosis I.
26. (d) (i) Chromatids separate but remains in the centre
16. (b) Synapsis is the pairing of two homologous
of the cell in metaphase.
chromosomes during meiosis. It allows matching-up
(ii) Chromatids start moving towards opposite
of homologous pairs prior to their segregation, and
poles in anaphase.
possible chromosomal crossover between them. The
(iii) Golgi complex and endoplasmic reticulum are
process of attachment of homologous chromosomes
still visible at the end of telophase.
leads to the formation of bivalents. These two events
27. (b) At the beginning of M phase or mitotic phase, the
occur in zygotene stage of prophase I.
17. (b) Crossing over is exchange of chromosomes nuclear membrane and nucleolus disappear. The
segments between non-sister chromatids of decondensation of chromosomes occur. But when
EBD_7209
Hints & Solutions S-57

nuclear envelop restart forming then nuclear lamina among DNA molecules.
assemble and chromosomes start condensing. 35. (a) Statements (i) and (ii) are correct.
28. (a) A homologous chromosome is a set of one maternal (iii) Interphase constitutes more than 95% of the
chromosome and one paternal chromosome that pair duration of the cell cycle.
up with each other inside a cell during meiosis. 36. (d) Interkinesis or interphase II is a period of rest that
29. (d) All the given statements are correct. cells of some species enter during meiosis, between
meiosis I and meiosis II. No DNA replication occurs
30. (d) Mitosis a type of cell division which results in two
during interkinesis however does occur during the
daughter cells each having the same number and Interphase I stage of meiosis. Interkinesis is generally
kind of chromosomes as the parent cells have. The short lived.
significance of mitosis are: 37. (b) Diplotene is the longest and most active subphase
(i) It helps in growth and development of multi- of prophase I of meiosis. The beginning of diplotene
cellular organisms from a single-celled zygote. is recognized by the dissolution of the synaptonemal
(ii) The number of chromosomes remains the same complex and the tendency of the recombined
in all the cells produced by mitosis. Thus, the homologous chromosomes of the bivalents to
daughter cells retain the same characters as separate from each other except at the sites of
those of the parent cell. crossovers. These X shaped structures are called
(iii) It helps the cell in maintaining proper size. chiasmata. Diplotene can last for months and years
(iv) It helps in restoring wear and tear in body in oocytes of some vertebrates.
tissues, replacement of damaged or lost part, 38. (a) The cell cycle is divided into two basic phase : (i)
healing of wounds and regeneration of detached Interphase, (ii) M-phase. The M phase represents
parts (as in tail of lizards). the phase when the actual cell division of mitosis
(v) It is a method of multiplication in unicellular occurs and the interphase represents the phase
organisms. between two successive M phase. In the 24 hour
average duration of cell cycle of human cell, cell
(vi) If mitosis remains unchecked, it may result in
division proper lasts for only about an hour. The
uncontrolled growth of cells leading to cancer
interphase lasts more than 95% of the duration of
or tumour. cell cycle.
31. (b) During S or synthesis phase, replication or 39. (c) During metaphase discontinuous fibres radiate out
duplication of chromosomal DNA and synthesis of from two spindle poles and get connected to the
histone proteins takes place. If the initial amount of disc shaped structure at the surface of the centromere
DNA is denoted as 2C then it increases to 4C. called kinetochores. These are called chromosome
However, there is no increase in the chromosome fibres or spindle fibres. A kinetochore is a complex
number; if the cell has diploid or 2n number of protein structure that is analogous to a ring for the
chromosomes at G1, even after S phase the number microtubule hook; it is the point where microtubules
of chromosomes remains the same, i.e., 2n. attach themselves to the chromosome.
32. (d) All the statements regarding Go stage of the cell are 40. (d) Cytokinesis is the division of cytoplasm, cytokinesis
correct. Go stage or resting phase is a period in the follows karyokinesis. Karyokinesis is the division of
cell cycle in which cells exist in a quiescent state. Go nucleus. The M phase starts with nuclear division,
phase is viewed as either an extended G1 phase, where corresponding to the separation of daughter
the cell is neither dividing nor preparing to divide, or chromosomes and leads to division of cytoplasm.
a distinct quiescent stage that occurs outside of the 41. (d) The final stage of meiotic prophase I is diakinesis.
cell cycle. Some (like cells, nerve and heart muscle This is marked by terminalisation of chiasmata.
cells), become quiescent when they reach maturity During this phase the chromosomes are fully
condensed and the meiotic spindle is assembled to
(i.e., when they are terminally differentiated) but
prepare the homologous chromosomes for
continue to perform their main functions for the rest
separation. Diakinesis represents end of prophase I
of the organism's life. and transition to metaphase.
33. (a) All the statements regarding bivalents are correct. 42. (c) Crossing over is the exchange of genetic material
A bivalent, sometimes referred to as a tetrad, is a pair between two homologous pair of chromosomes.
of associated homologous chromosomes which are Crossing over is also an enzyme-mediated process
held together by a complex after chromosome and the enzyme involved is called recombinase.
replication. During meiosis the process of synapsis 43. (d) During Metaphase II the chromosomes align along
occurs in which bivalents are formed. the equator. The microtubules from opposite poles
34. (b) Crossing over leads to recombination between of the spindle get attached to the kinetochores of
homologous chromosomes. Recombination is the sister chromatids. Anaphase II occurs with the
rearrangement of genetic information within and simultaneous splitting of the centromere of each
S-58 Biology
chromosome move towards the opposite poles of varities.
the cell. 51. (d) Diakinesis is the final stage of prophase I of meiosis
44. (c) A – IV, B – III, C – II, D – I I.
Metaphase – Chromosomes are moved to spindle 52. (d) G1 – First growth phase, S-Synthesis phase, G2 -
fibre. Second growth phase.
Anaphase – Centromere splits and chromatids 53. (a) Pair of anaphase I with their feature is correctly
apart. matched.
Zygotene – Pair ing between homologous Metaphase I: Spindle apparatus appear and the
chromosomes takes place. chromosomes are arranged on equatorial plate, with
Pachytene – Crossing between homologous the centromeres towards the pole. Spindle fibres
chromosomes occurs. become attached to the centromeres of the two
45. (a) Terminalization which occurs in diakinesis stage of homologous chromosomes.
meiosis I, is the disappearance of chiasmata of sliding Interphase: It is the time during which cell is preparing
towards the tips of the chromosomes due to tight for division by undergoing both cell growth and
condensation. Synapsis is the pairing of two DNA replication.
homologous chromosomes. It allows matching-up Prophase I: It is the lengthy phase when compared
of homologous pairs prior to their segregation, and with mitotic phase. It is subdivided into 5 sub-
possible chromosomal crossover between them. The phases: leptotene, zygotene, pachytene, diplotene
process of attachment of homologous chromosomes and diakinesis.
leads to the formation of bivalents. Chiasmata is a 54. (b) In schematic break-up of the phases/ stages of cell
point at which paired chromosomes remain in contact cycle, D-synthetic phase is the correct indication of
during the first metaphase of meiosis, and at which the stage/phase in the cell cycle.
crossing over and exchange of genetic material occur 55. (c) The figure A represents late anaphase while figure B
between the strands. Synaptonemal complex is represent prophase stage of mitosis.
formed at the region of synapsis. It is a protein
56. (a) Given diagram represents crossing over that takes
structure th at forms between homologous
place in pachytene stage of prophase - I during
chromosomes (two pairs of sister chromatids) and is
meiosis-I.
thought to mediate chromosome pairing, synapsis,
and recombination. 57. (d) Telophase is the stage of reconstitution of nuclei.
The chromosomes that have reached their respective
46. (a) A – II, B – III, C – IV, D – I
poles decondense and lose their individuality and
Anaphase – Movements of chromatids towards collect in a mass in the two poles. Nuclear envelope
opposite poles. assemble around chromatin mass. Nucleolus, golgi
Prophase – Initiation of assembly of mitotic complex and ER reform.
spindle. 58. (a) The correct identification of X, Y and Z are
Interphase – Proteins are synthesized in respectively G1, S and G2. During G1 phase, cell stores
preparation for mitosis while cell ATP for cell division and synthesis of protein, RNA,
growth continues. amino acids, ATP and nucleotides occur. S phase,
Metaphase – Spindle fibres attach to the also called invisible stage of M phase, includes the
kinetochores of chromosomes. synthesis of DNA and histone proteins. In G2 phase
47. (a) Pairs A and B are correctly matched. synthesis of RNA and proteins continues and
Diplotene – Chiasmata formation of macromolecules for spindle and
Meiosis – Haploid cells organelle formation occurs. In this the quantity of
DNA within the cells has increased to 4C but the cell
48. (b) Pachytene is characterized by occurrence of
is still considered diploid.
crossing over. Bivalent chromosomes now clearly
appear as tetrads. Diplotene follows pachytene and 59. (c) Metaphase chromosome is made up of two sister
involves dissolution of synaptonemal complex. chromatids which are held together by the
centromere. Chromosomes are moved to spindle
49. (a) Karyokinesis is the first step of M-phase of cell cycle.
equator and gets aligned along metaphase plate
It brings about division of nucleus to form two
through spindle fibres to both poles.
daughter nuclei.
60. (d) In anaphase I, the homologous chromosomes break
50. (b) During pachytene stage of meiosis-I crossing over
apart (called disjunction) while sister chromatids
leads to recombination of traits. This is highly useful
remains associated at their centromere. At the end
for breeders in development of new improved
of anaphase I, two groups of chromosomes are
EBD_7209
Hints & Solutions S-59

produced at two poles, having half the number of in S phase. S Phase is the synthesis phase during
parental chromosomes. which replication of chromosomes occur.
61. (b) The given figures (A, B and C) showing meiotic phases 65. (b) During S phase, replication of chromosomal DNA
are respectively metaphase I, anaphase I and telophase and synthesis of histone proteins takes place. During
I. this time, the amount of DNA for cell doubles.
66. (a) In mitosis, a single cell divides to form two daughter
Metaphase I: Spindle apparatus appear and the
cells. So, the number of mitotic divisions can be
chromosomes are arranged on equatorial plate, with
calculated by 2n.
the centromeres towards the pole. Spindle fibres
where, n is the number of division of cell.
become attached to the centromeres of the two
2n = 128
homologous chromosomes. 2n = 27
Anaphase I: The homologous chromosomes break n=7
apart while sister chromatids remains associated at Thus, 7 mitotic divisions are needed for a single cell
their centromere. At the end of anaphase I, two to make 128 cells.
groups of chromosomes are produced at two poles, 67. (d) During zygotene and S phase, the formation of
having half the number of parental chromosomes. bivalents and DNA replication occurs respectively.
Telophase I: The homologous chromosome pairs A bivalent, is a pair of associated homologous
complete their migration to the two poles as a result chromosomes held together by a complex after
of spindle action. As a result a haploid set of chromosome replication. During meiosis the process
chromosomes is present at each pole, with each of synapsis occurs in which bivalents are formed S
chromosome still having two chromatids. A nuclear phase is the synthesis phase during which replication
envelope reforms around each chromosome set, the of choromosomes occur.
spindle disappears, and cytokinesis follows. 68. (d) Meiosis results in four daughter cells and each with
62. (c) The given figure shows (I - phase). Interphase (called having half the number of chromosomes of the parent
resting phase) is the time during which cell is preparing cell. Meiosis begins with a parent cell that is diploid,
i.e it has two copies of each chromosome. The parent
for division by undergoing both cell growth and DNA
cell undergoes one round of DNA replication
replication in a systemic manner. According to the
followed by two separate cycles of nuclear division.
given figure, the phases marked as A, B, C and D are
The process leads to production of four daughter
respectively G1, S, G2 and M phase.
cells that are haploid, which means that they contain
G1 phase: It is also called antephase as during the half the number of chromosomes of the diploid parent
phase the cell stores ATP for cell division. In this cell. The significance of meiosis are:
phase, synthesis of protein, RNA, amino acids, ATP (i) It maintains the same chromosome number in the
and nucleotides occur. sexually reproducing organisms. From a diploid cell,
S phase: It is also called invisible stage of M phase. haploid gametes are produced which in turn fuse to
In this stage, synthesis of DNA and histone proteins form a diploid cell.
occurs. (ii) It restricts the multiplication of chromosome number
G2 Phase: In this phase, synthesis of RNA and proteins and maintains the stability of the species.
continues and formation of macromolecules for spindle (iii) Maternal and paternal genes get exchanged during
and organelle formation occurs. In this the quantity of crossing over which results in variations among the
DNA within the cells has increased to 4C but the cell is offspring.
still considered diploid. (iv) All the four chromatids of a homologous pair of
M phase: In M phase, mitosis occurs when the chromosomes segregate and go over separately to
nucleus of the cell divides into two identical nuclei four different daughter cells. This leads to variation
with the same number and type of chromosomes, in the daughter cells genetically.
followed by cytokinesis when the cytoplasm, for 69. (a) One cell produces 4 daughter cells after meiotic
both plant and animal cells, divides, thus creating division. Thus, for the formation of 100 pollen grains,
two daughter cells that are genetically equal and
100
approximately identical in size. = 25 PMC are needed and each will undergo
63. (c) The given figure shows anaphase I of meiosis. In 4
anaphase I, the homologous chromosomes break one reduction/meiotic division.
apart while sister chromatids remains associated at 70. (a) Mitosis division occurs in somatic cells while meiosis
their centromere. At the end of anaphase I, two occurs in reproductive cells. Here, meristem is a
groups of chromosomes are produced at two poles, somatic cell while gametangia is a reproductive cell.
having half the number of parental chromosomes. 71. (a) Metaphase is a stage of mitosis in the eukaryotic
64. (b) In the somatic cell cycle, DNA replication takes place cell cycle in which chromosomes are at their most
S-60 Biology
condensed and coiled stage. Metaphase is the best changing 2n DNA content to n DNA during
stage in mitosis for analysing the chromosomes and anaphase II will remain same i.e. 8.
to study their morphology. 83. (a) In meiosis, the daughter cells differ from the parent
72. (b) In some organisms karyokinesis is not followed by cell as well as amongst themselves due to
cytokinesis as a result of which multinucleate segregation, independent assortment and crossing
condition arises leading to the formation of over.
syncytium (e.g., liquid endosperm in coconut). 84. (c) The separation of two chromatids of each
73. (d) Meiosis happens only once in the life cycle of an chromosome at early anaphase is initiated by the
organism. Meiosis forms the basis of sexual force of repulsion between the divided kinetochores.
reproduction and can only occur in eukaryotes. In 85. (a) Cell would normally proceed to mitosis without
Meiosis, the diploid cells’ genome, which is interruption once it has entered the S phase.
composed of ordered structures of coiled DNA (called 86. (c) During Anaphase I, the homologous chromosomes
chromosomes) is replicated once and separated separate, after breaking apart (the process is called
twice, producing four sets of haploid cells each disfunction) while sister chromatids remain
containing half of the original cells chromosomes. associated at their centromeres.
74. (d) When cells are not to divide after G1 phase and start 87. (a) The mitotic spindle forms and the nuclear membrane
undergoing differentiation into specific types of cells disappear during prophase.
such cells are said to be in G0 phase/G0 state. 88. (c) Diplotene is the longest and the most active sub-
75. (c) Mitosis occurs in both diploid and haploid cells while phase of prophase I of meiosis. In diplotene, the
meiosis is found only in diploid cells. Mitosis is the homologous chromosomes separate due to repulsion
cell division that takes place in somatic cells. It is a as the nucleoprotein complex of synapsed
process by which a cell separates its duplicated chromosomes dissolves, but are yet held by
genome into two identical halves. chiasmata.
89. (d) The triploid nucleus formed in the embryo sac of a
76. (c) Mitosis a type of cell division by which eukaryotic
seed plant by fusion of a sperm nucleus with two
cells multiply. This division results in two daughter
cells and each having the same number and kind of polar nuclei or with a nucleus formed by the prior
chromosomes as the parents cells have. fusion of the polar nuclei. A triploid nucleus cannot
undergo meiosis because not all of the chromosomes
77. (c) Anaphase is characterized by splitting of the
can form homologous pairs.
centromeres and separation of chromatids.
90. (a) Crossing over is the exchange of genetic material
Chromatids move to opposite poles from the
between homologous chromosomes. It leads to
equatorial plates.
recombination between homologous chromosomes.
78. (b) In animal cell cytokinesis involves the contraction 91. (c) Centromere is the constricted region of the chromatid
of the contractile ring of microfilament. Contractile containing kinetochore. It is joined in cells during
ring of microfilaments develops peripherally below prophase and metaphase. It is important that the
the cell membrane in the equatorial region.
centromere not divide till the end of the metaphase
79. (c) Archesporial cell is a somatic cell where mitosis will because it holds the replicated DNA molecules
occur. together.
80. (a) Meiosis involves two sequential nuclear and cell 92. (c) Mitosis does not introduce genetic variations as
division. It results in four daughter cells each with mitosis is a mode of cell division in which the daughter
half the number of chromosomes of the parent cell, cells are genetically similar to the parent cell because
as in the production of gametes and plant spores. their nuclei have the same number and type of
Meiosis begins with a parent cell that is diploid, chromosomes as are present in the parent cell.
meaning it has two copies of each chromosome. The 93. (a) Sister chromatids are the two identical strands of a
parent cell undergoes one round of DNA replication duplicated chromosome. So, it carries the same
followed by two separate cycles of nuclear division. genetic information.
The process results in four daughter cells that are 94. (c) Meiosis ensures the production of haploid phase in
haploid, which means they contain half the number the life cycle of sexually reproducing organisms
of chromosomes of the diploid parent cell. whereas fertilization restores the diploid phase.
81. (b) During meiosis, daughter cells have the half number 95. (a) The formation of new cell wall begins with the
of chromosomes with respect to parent cells. formation of a simple precursor, called cell plate that
Therefore daughter cell will be genetically different. represents the middle lamella between the walls of
82. (a) Meiosis II maintains the haploid number of two adjacent cells.
chromosomes obtained after meiosis I while 96. (c) The correct sequence of the events in the meiosis is:
III, II, I, IV.
EBD_7209
Hints & Solutions S-61

III. Synapsis: Synapsis is the pairing of two solid particles which adsorb water or any other liquid
homologous chromosomes that occurs during are called imbibibants. The liquid which is imbibed is
meiosis. It allows matching-up of homologous known as imbibate. Examples are absorption of water
pairs prior to their segregation, and possible by seeds and dry wood.
chromosomal crossover between them. It
occurs in zygotene sage. 7. (c) Imbibition is a process in which water is absorbed
II. Crossing over: Crossing over is exchange of by solids like colloids and leads to swelling and
chromosomes segments between non-sister increase the volume of the cell. It involves three
chromatids of homologous pair. Crossing over characteristics like volume change, heat production
occurs during pachytene stage. and pressure development.
I. Terminalization: Terminalization of chiasmata
8. (a) A solution whose osmotic concentration (solute
occurs in diakinesis stage of meiosis I. It is the
disappearance of chiasmata of sliding towards potential) is less than that of another solution or cell
the tips of the chromosomes due to tight sap is called hypotonic solution. If a cell is placed in
condensation. such a solution, water starts moving into the cell by
IV. Disjunction of genomes: The process of the process of endosmosis, and cell becomes turgid.
breakdown of homologous chromosomes is 9. (d) The movement of organic food or solute in soluble
called disjunction. This process occurs in form, from one organ to another organ is called
anaphase I. translocation of organic solutes. The process of
Chapter 11 : Transport in Plants translocation requires expenditure of metabolic
energy and the solute moves at the rate of 100 cm/hr.
1. (a) A cell at full turgor has its osmotic potential (ys) and The translocation of solutes occurs in a mass along
pressure potential (yp) equal but opposite in sign. with cell sap through the sieve tubes from a region
where solute potentials (ys) are always in negative of higher turgor pressure to low turgor pressure (i.e,
values. along a turgor pressure gradient).
2. (c) Cell wall and cell membrane are the cellular structures 10. (b) The symplastic movement of water occurs from cell
which play an important role in determining the to cell through the plasmodesmata.
movement of molecules in or out of the plant cell.
11. (c) The casparian strips are formed due to deposition of
Cell wall is the rigid layer of polysaccharides lying
wax like substance called suberin.
outside the plasma membrane of the cells of plants,
12. (d) Apoplastic movement of water occurs exclusively
fungi, and bacteria. In the algae and higher plants it
through the cell wall and intercellular spaces, without
consists mainly of cellulose. Cell membrane,
crossing any membrane. It is continuous throughout
composed of lipids and proteins, is the
the plant, except at the band of suberised matrix called
semipermeable membrane which surrounds the
casparian strips of the endodermis in the roots.
cytoplasm.
13. (c) Some plants have additional structures associated
3. (c) The shrinkage of the protoplast of a living cell from
with them that help in water and minerals absorption.
its cells wall due to exosmosis under the influence of
Mycorrhiza is a symbiotic association of a fungus
a hypertonic solution is called plasmolysis.
with a root system. In mycorrhiza a large number of
4. (a) Phenomenon of plasmolysis occurs when cells are fungal hyphal are associated with young root and
kept in hypertonic (containing more solutes) solution. also extend into soil. The hyphal have large surface
Plasmolysis is a process in which cell loses water area for absorption. The hyphal absorbs water and
(due to exosmosis) leading to shrinkage of plasma minerals and handed over them to root. Root
membrane or protoplast away from its cell wall. provides the fungus with sugar and nitrogenous
5. (a) When a cell is plasmolysed, it becomes flaccid, its compound.
TP (turgor pressure) becomes zero and space 14. (a) A pressure that is responsible for pushing up water
between cell wall and the plasma membrane is to small height in the stem is called positive root
occupied by hypertonic solution. pressure. Root pressure is a type of osmotic
6. (c) Imbibition is the process of absorption of water by pressure. It occurs within the cells of a root system
hydrophilic surfaces of a substance without forming that causes sap to rise through a plant stem to the
a solution. It is a type of diffusion by which movement leaves. Root pressure occurs in the xylem of vascular
of water takes place along a diffusion gradient. The
S-62 Biology
plants when the soil moisture level is high either at the plants.
night or when rate of transpiration is low during the 25. (b) The relation between source and sink is variable.
day. The direction of movement of organic solutes in
15. (c) The process of guttation takes place due to the root phloem can be upwards or downwards i.e.,
pressure, developed in cortex cells of root. bidirectional.
16. (a) Cohesion of water and transpiration pull theory is 26. (b) Phloem sap is mainly water and sucrose, but other
the most widely accepted theory put forth by Dixon sugars, hormones and amino acids are also
and Jolly in 1894, and further supported by Renner translocated through phloem.
(1911, 1915), Curtis and Clark (1951), Bouner and 27. (d) Pressure flow hypothesis (also called mass flow
Golston (1952), Kramer and Kozlowskl (1960). It is hypothesis, proposed by Ernst Munch) is the
also known as Dixons cohesion theory, or Cohesion accepted hypothesis for the translocation of sugar
tension theory. from source to sink. This hypothesis explains the
17. (a) The force which is responsible for the upward transport the food materials from the phloem.
conduction of water against the gravity comes from 28. (a) The process of loading at the source produces a
the transpiration. Transpiration is a process in which hypertonic condition in the phloem. Hypertonic
plants absorb water through the roots and then give condition occurs when glucose, which is prepared
off water vapour through pores (called stomata) in with the help of photosynthesis at the source, is
their leaves. converted into sucrose and the latter is moved into
18. (b) Guard cells help in transpiration (a process in which the companion cells and then into the living phloem
water losses from a plant in the form of water sieve tube cells by active transport.
vapour).These cells are paired epidermal cells that 29. (d) Water in adjacent xylem moves into the phloem by
control the opening and closing of stomata with the the process of osmosis (diffusion of a solvent
changes in the turgidity of the cell. through a semipermeable membrane from a less
19. (c) Inner wall of guard cell towards the stomatal aperture concentrated to a more concentrated solution until
is thick and elastic. These cells regulate the rate of both the solutions are of the same concentration).
transpiration by opening and closing the stomata. As osmotic pressure builds the phloem sap will move
20. (c) Active absorption takes place by the activity of root to areas of lower pressure.
itself, particularly root hairs. The factor responsible 30. (c) Phloem tissue is composed of sieve tube cells, which
for water absorption is present within the roots. It form long columns with holes (through which food
utilizes metabolic energy and water moves from the material is conducted) in their end walls called sieve
region of higher water potential towards the region plates.
of lower water potential. 31. (a) Plasmolysis is the shrinkage of plasma membrane or
21. (b) In root endodermis there is one way active transport protoplast from the cell wall due to exosmosis. The
of ions because of the presence of suberin. process of plasmolysis is reversible which is known
22. (a) Sinks are related to transport of materials. Basically deplasmolysis.
sinks are the growing regions of plants (e.g, apical 32. (d) Statement (d) is incorrect. Pinus seeds cannot
and lateral meristems, young leaves, developing germinate and establish without the presence of
fruits, flowers and seeds, and the storage organs). mycorrhiza, because pinus trees have an obligate
23. (d) Translocation of food in the flowering plants occurs association with the mycorrhiza.
in the form of sucrose. It is transported by the 33. (d) Statement (d) is incorrect. The inner wall of each
vascular tissue phloem from a source (regions of guard cell, towards the stomatal aperture is thick and
excess carbohydrates, primarily mature leaves) to elastic.
sinks (regions where the carbohydrate is needed). 34. (d) All the given statements are correct.
24. (d) During food development, photosynthesizing leaves 35. (c) Statements (iv) and (v) are not correct.
would be the source and the fruit would be sink.
(iv) Xylem is associated with the translocation of
They (source and sink) both may be reversed
mainly water, mineral salts, some organic
depending on the seasons or the requirements of
nitrogen and hormones from roots to the aerial
EBD_7209
Hints & Solutions S-63

parts of the plants. during the night. The immediate cause of the opening
(v) Phloem translocates a variety of organic and or closing of the stomata is a change in the turgidity
of the guard cells. The inner wall of each guard cell,
inorganic solutes mainly from the leaves to
towards the pore or stomatal aperture, is thick and
other parts of the plants. elastic. When turgidity increases within the two
36. (c) Statement (i), (iii) and (iv) are correct and (ii) is guard cells surraunding each stomatal pore, the thin
incorrect. outer walls bulge out and force the inner walls into a
(ii) The apoplastic movement of water occurs crescent shape. The opening of the stoma is also
due to the orientation of the microfibrils in the cell
exclusively through the intercellular spaces and
walls of the guard cells. When the guard cells lose
the walls of the cells.
turgor, due to water loss, the elastic inner walls regain
37. (d) All the given factors affect transpiration. their original shape, the guard cells become flaccid
Transpiration is directly proportional to the light and the stoma closes.
intensity, temperature, wind velocity, leaf surface 44. (b) Transpiration pull or cohesion - tension theory was
area, root - shoot ratio and number, distribution and given by Dixon and Joly in 1894. It was further
percent of stomata and inversely proportional to CO2 improved by Dixon in 1914. This theory is based
concentration and atmospheric humidity. upon the cohesive and adhesive properties of water
molecules, which forms the unbroken continuous
38. (c) Statement (i), (ii), (iii) and (v) are correct and (iv) is water column in the xylem.
incorrect regarding transpiration.
Water molecules are held together by strong
(iv) Transpiration cools leaf surfaces, sometimes 10 cohesion force which is due to hydrogen bonds
– 15 degrees, by evaporative cooling. amongst them. There is another force of adhesion
39. (c) Statements (iii) and (iv) regarding translocation of which holds water to the walls of xylem vessels. Water
mineral ions are correct. molecules are attracted to one another more more
than the water molecules in the gaseous state. It
(i) Mineral ions are frequently remobilised, produces surface tension that accounts for high
particularly from older senescing parts. capillarity through tracheids and vessels. Water
(ii) Older drying leaves export much of their mineral column is present in tracheary elements. There is a
contents to younger leaves. continuous column of water from roots through the
stem and to the leaves. These tracheary elements
40. (b) Some carrier proteins allow transport of only two
form this continuous system through their
types of molecules to move together. This is called
unthickened areas. Since, a large number of tracheary
cotransport. Symport and antiport are two types of
elements are present together, no breakage in the
cotransport. Symport allows both the molecules to
continuity of water occurs even if there is a blockage
move in the same direction and antiport allows both
of one or few of them.
the molecules to move in opposite direction.
41. (a) Water molecules posses kinetic energy water 45. (d) Minerals are present in the soil in the form of ions.
molecules are in random motion. In liquid and Ions cannot directly cross the cell membrane. They
gaseous form the greater the concentration of water are absorbed from the soil by both passive and active
in a system, greater is its kinetic energy or water transport. Specific proteins present in the membranes
potential. Pure water has greatest water potential of root hair cells actively pump ions from the soil
i.e., zero. When a solute is added to pure water, it into the cytoplasm of the epidermal cells. Like all
decreases the free energy of water and thus cells, the endodermal cells have many transport
decreases the water potential (y w ) . proteins embedded in their plasma membrane; they
let some solutes cross the membrane, but not others.
42. (c) Diffusion of water from its pure state into a solution
Transport proteins of endodermal cells are control
Where the two are separated by a semi-permeable
points, where a plant adjusts the quantity and types
membrane is termed as osmosis. The direction and
of solutes that reach the xylem.
rate of osmosis depend upon the sum of two forces,
pressure gradient (gradient of yr ) and concentration 46. (b)
gradient (gradient of ys ). The net force or gradient 47. (c)
is determined by the difference in the water potentials 48. (c)
of solutions separated by a semipermeable
49. (b) Diffusion is the movement of ions, atoms or
membrane.
molecules from a region of higher concentration to a
43. (a) Normally stomata are open in the day time and close
region of lower concentration. Osmosis is the
S-64 Biology
diffusion of a solvent through a semipermeable Water potential = Osmotic potential + Pressure
membrane from a less concentrated to a more potential
concentrated solution until both the solutions are of
the same concentration. Imbibition is the process of yw = y s + y p
increase of the volume of a solid due to absorption For cell A
of water by hydrophilic colloids. Plasmolysis is a
process in which cell loses water (due to exosmosis) yW(A) = -20 + 8
leading to shrinkage of plasma membrane or
= 12 bars
protoplast away from its cell wall.
For cell B
50. (a) The given diagram represents the process of osmosis
where chamber B has lower concentration of water yW(B) = -12 + 2
hence possesses lower water potential. Osmosis will
occur from higher yw to lower yw so, it will occur = – 10 bars
from chamber A to chamber B. yB > y A
51. (c) The given experiment shows the process of osmosis i.e., – 10 > – 12
by using thistle funnel. X and Y are respectively
Thus, movement of water will occur from cell B to
sugar solution and water. After a few days of
cell A.
experiment, sugar level rises and water level drop
down. Level of sugar solution in the stem of thistle 56. (b) When a plant is placed in pure water then the cell
funnel rises due to the movement of water molecules becomes more turgid until the pressure potential of
in thistle funnel through semi-permeable membrane. the cell reaches its osmotic potential.
This demonstrates that the water molecules move 57. (c) Active transport uses energy (ATP) to pump
from low concentration (from beaker) to the high molecules against a concentration gradient. Cells
concentration (inside thistle funnel). undergoing active transport bear abundant
52. (c) The given figure shows the water movement in the mitochondria to provide ATP, needed to power active
leaf. The parts marked as A, B, C, D and E in the transport. So, the production of ATP is blocked or
figure is respectively xylem, phloem, palisade, guard decreased and active transport is blocked or slow
cells and stomatal pore. down.
53. (c) In the given figure which shows the pressure flow 58. (d)
hypothesis, the rate of translocation increases due 59. (a) Water will move from its region of higher chemical
to increase in sucrose production at the source. This potential to its region of lower chemical potential
hypothesis explains the translocation of sugar from until the equilibrium is reached.
source to sink in the phloem.
60. (a) In the given statement of osmosis, the movement of
54. (b) In the given figure which shows the symplastic and solvent molecule is not specified.
apoplastic pathways of water and ion absorption
61. (d) Osmosis occurs in response to a driving force. The
and movement in roots, the structure marked as A,
B, C and D are respectively xylem, casparian strip, net direction and the rate of osmosis depend on both
phloem and cortex. Casparian strip (marked as B), the pressure gradient and concentration gradient.
impermeable substances made of suberin and 62. (b) Salted pickle is an example of hypertonic solution
sometimes lignin is a band of cell wall material whose osmotic potential is more than that of another
deposited on the radial and transverse walls of the solution or cell sap. If a cell is placed in such a
endodermis. It obstructs the water movement solution, water comes out of the cell by the process
through the apoplastic pathway at the endodermis. of exosmosis and cell becomes flaccid. Due to
It forces water through the endodermal cell and in exosmosis, the protoplasm shrinks i.e., bacterial cell
such a way regulates the amount of water getting to is plasmolysed and death occurs.
the xylem. 63. (c) A cell is said to be flaccid when water flows into the
55. (c) Chemical potential of water is water potential. Water cell and out of the cell and are in
always move from higher water potential to lower 64. (d) The phenomenon of imbibition has three important
water potential. characteristics i.e., increase in volume, production
EBD_7209
Hints & Solutions S-65

of heat and development of imbibitional pressure. the leaves through the special pore (always open)
65. (d) Seeds when soaked in water, they imbibe because of like structures (called hydathodes or water stomata).
the presence of the water potential gradient between 75. (c) Cobalt chloride is used to study water from a leaf
the seed coat and water. Seeds and other such and turns colour on absorbing water.
materials have almost no water hence they absorb 76. (c) Opening and closing of stomata occurs due to
water easily. Imbibition is a process of absorption of changes in turgor pressure in guard cells. The
fluid by a solid body without resultant chemical transpiration is regulated by the movement of guard
change in either of the material. cells of stomata.
66. (b) The blow of a bottle packed with moistened mustard 77. (a) Stomata closes because guard cells lose turgidity
seed and water is due to the process of imbibition. and becomes flaccid. This may occur when the plant
Imbibition is a process in which water is absorbed has lost an excessive amount of water. In addition, it
by solids like colloids and leads to swelling and generally occurs daily as light levels drop and the
increase the volume of the cell. It involves three use of CO2 in photosynthesis decreases.
characteristics like volume change, heat production 78. (a) The correct order of the events takes place in the
and pressure development. mass flow in the phloem is: (ii) - (iv) - (iii) - (i) - (v)
67. (a) (ii) Leaf cells produce sugar by photosynthesis -
68. (a) The upward conduction of water takes place through (iv). Sugar is transported from cell to cell in the leaf -
the xylem. It can be proved by immersing the cut end (iii). Solutes are actively transported into the sieve
of a branch in an aqueous solution of a dye, i.e., elements - (i). Water diffuses into the sieve tube
eosine or basic fuschsin. In such an experiment only elements - (v). Sugar moves down the stem.
xylem vessels and trachieds get stained. 79. (a) If a stem is girdled, root dies first as the food
69. (a) The movement of water from one cell of the cortex in synthesized by leaves is not able to reach to the
the adjacent one in roots is due to the water potential roots.
gradient. 80. (a) Stoma opens when guard cells are turgid and swell
70. (d) X and Y are casparian strip and apoplast respectively. due to a decrease in their water potential. This
Casparian strip (a impermeable substances made of turgidity is caused by the accumulation of K+
suberin and sometimes lignin, present on the radial (potassium ions) in the guard cells. As K+ levels
and transverse walls of the endodermis) breaks the increase in the guard cells, the water potential of the
continuity of the apoplast pathway and forces the guard cells drops and water enters the guard cells.
water and solutes to cross the endodermis by passing 81. (b) Transpiration helps in the movement of water and minerals
through the plasma membrane. absorbed by the roots to the other parts of the plants.
71. (a) Path of water absorption is as follow 82. (c) The rate of absorption of water is almost directly
Soil solution ® Root hairs ® Epiblema/epidermis proportional to the rate of transpiration.
® Cortex ® Endodermis (passage cell) ® Pericycle 83. (a) Sap will flow from high to low pressure areas in the
cell ® Protoxylem ® Metaxylem. plant. Adding sugars to the sap causes water to enter
72. (d) A mutant plant, discovered by the Botanists was the phloem, thus increasing pressure. Removing
incapable to produce material for making casparian sugars causes water to leave the phloem, thereby
strip. So such plant would unable to control the reducing pressure.
amounts of water and solutes it absorbs. This is 84. (a) Isotonic solution is the external solution which
because casparian strip allows the plants to regulate balances the osmotic pressure of the cytoplasm.
the movement of material through apoplast.
Chapter 12 : Mineral Nutrition
73. (d) Statements (b) and (c) are correct regarding guttation.
Guttation is a loss of water in the form of liquid from the 1. (c) In 1860, Julius von Sachs, a prominent German
uninjured margins of the leaves of those plants which botanist, demonstrated for the first time that plants
grows in moist places. It occurs through hydathodes could be grown to maturity in a defined nutrient
(special type of stomata called water stomata). solution in complete absence of soil. This technique
of growing plants in a nutrient solution is known as
74. (d) Guttation is not a universal process. Normally,
hydroponics or soilless growth.
guttation is found in herbaceous plants like grasses
2. (a) Experiment on hydroponics was demonstrated by
Balsam, Colocasia etc. and in some of the plants of
Von Sachs. Hydroponics is the process of growing
cucurbitaceae family. It occurs from the margins of
S-66 Biology
plants in nutrient rich sand, gravel, or liquid but active, as it requires metabolic energy. Hence, it is
without soil. called active absorption.
3. (c) Most of the minerals present in soil can enter plants 17. (b) Denitrification is a process by which soil anaerobic
through roots. In fact, more than sixty elements of microbes convert nitrate (NO3–) or nitrite (NO2–) to
the 105 discovered so far found in different plants. the gases, nitrous oxide (N2O) and molecular nitrogen
4. (c) (N2), which are then lost in the atmosphere. It is
5. (c) Micronutrients or trace elements are essential carried out by bacteria Pseudomonas and
elements required by plants in traces only less than Thiobacillus.
10 mmole kg–1 of dry matter. These are eight in 18. (c)
numbers (Zn, Mn, B, Cu, Mo, Cl, Ni, Fe). 19. (a) In nature, lightening and ultraviolet radiation provide
6. (b) Microelements or minor elements are mostly involved enough energy to convert nitrogen to nitrogen oxides
in the functioning of enzymes as cofactors or metal (NO, NO2, N2O).
activators. 20. (d) In the process of biological nitrogen fixation by free
7. (a) Boron is required for uptake and utilization of Ca2+, living and symbiotic nitrogen fixers, the dinitrogen
membrane functioning, pollen germination, cell molecule (Nº N) is progressively reduced step-by-
elongation, cell differentiation and carbohydrate step to ammonia (NH3) by addition of pairs of
(sugar) translocation. hydrogen atoms. The overall process occurs in
8. (d) presence of enzyme nitrogenase which is active in
9. (b) Magnesium is an essential mineral nutrient for life anaerobic condition.
and is present in every cell type in every organism. In 21. (b)
plants, magnesium is necessary for synthesis of 22. (b) Frankia is symbiont in root nodules of several non-
chlorophyll and photosynthesis. It is a constituent of legume plants like Casuarina, Myrica and Alnus.
the ring structure of the chlorophyll and helps to 23. (b) 24. (b) 25. (b)
maintain the ribosome structure. 26. (c) Leghaemoglobin is red colour haemoglobin like
10. (b) Chlorine is absorbed by plants in the form of chloride pigment which absorbs oxygen and protect
anion (Cl–). It is essential for the water– splitting nitrogenase enzyme from the oxygen.
reaction in photosynthesis, a reaction that leads to 27. (d) 28. (c) 29. (b) 30. (a)
oxygen evolution. 31. (a) Macronutrients are those essential elements which
11. (a) Nitrogen and phosphorous are the constituents of are present in easily detectable quantities, at least 10
the proteins. m mole kg–1 of dry matter.
12. (a) 13. (c) 32. (d) Denitrification is the process of converting nitrates
14. (c) Etiolation is not caused by deficiency of mineral and ammonia of soil to molecule nitrogen with the
nutrition. It is the form and growth of seedlings help of aerobic bacteria like Pseudomonas and
grown in darkness. It results in a pale, unusually tall Thiobacillus.
and slender appearance, dramatically different from 33. (d)
the stockier, green appearance of seedlings grown 34. (d) Nitrification is the conversion of ammonia into nitrite
in the light. Necrosis, chlorosis and shortening of and nitrates by soil bacteria (nitrifying bacteria).
internodes are caused by deficiency of mineral Heterotrophic plants are less dependent on nitrogen
nutrition. obtained from nitrification since they receive some
15. (a) Any mineral ion concentration in tissues that nitrite and nitrate through their parasitic or
reduces the dry weight of tissues by about 10% is carnivorous nutritional modes.
considered toxic. 35. (b) 36. (b)
16. (c) Elements are absorbed from the soil solution in the 37. (c) Manganese competes with iron and magnesium for
form of ions. Uptake of mineral ions, can be divided in uptake, and with magnesium for binding with
two main phases. enzymes. Manganese also inhibits calcium
In the first phase, uptake of mineral ions occurs freely translocation in shoot apex.
between external solution and free space or outer
38. (d)
space (intercellular space and cell wall) of cell. It is
39. (d) The nitrite is further oxidised to nitrate with the help
called passive absorption and it does not require
of the bacterium Nitrobacter.
any metabolic energy.
40. (a) The deficiency symptoms tend to appear first in the
In the second phase, ions are taken into the inner
young tissues whenever the elements are relatively
space (cytoplasm and vacuole).This entry or exit is
immobile and are not transported out the mature
EBD_7209
Hints & Solutions S-67

organs, for example, elements like sulphur and photoautotrophic nitrogen-fixing bacteria.
calcium are a part of the structural component of the 62. (a) The given experimental set up shows the process of
cell and hence are not easily released. The deficiency hydroponics. Hydroponic is process of growing
symptoms of nitrogen, potassium and magnesium plants in a soilless mineral nutrient solution. The A,
are visible first in the senescent leaves. B, and C are respectively funnel for adding water,
41. (c) N2 - fixation is accomplished with the help of aerating tube and nutrient solution respectively.
nitrogen-fixing microbes, mainly Rhizobium.
The enzyme nitrogenase which plays an important 63. (c)
role in biological N2 fixation is very sensitive to 64. (b) The given figure represents nitrogen cycle which
oxygen. shows the relationship between the three main
42. (b) Zinc (Zn2+) is an activator of alcohol dehydrogenase nitrogen pools - atmospheric soil and biomass. The
and Mo of nitrogenase during nitrogen metabolism. blanks in the figure marked as A, B, C and D are
43. (c) 44. (b) 45. (d) 46. (c) respectively N2, ammonification, plant biomass and
47. (c) Due to deficiency of sulphur plant shows chlorosis animal biomass.
(i.e., yellowing due to degradation of chlorophyll) 65. (b)
followed by anthocyanin development. The younger 66. (d) Plants absorb nitrogen from the soil in the form of
leaves show chlorosis before older ones. Sulphur is nitrate (NO3–) and ammonium ( NH +4 ) ions, both of
not the constituent of chlorophyll. The main which are water soluble. Nitrate ions are absorbed
constituent of chlorophyl is magnesium. quickly by plant roots but leach easily. Ammonium
48. (c) Micronutrients include iron manganese, copper, ions are attracted to soil particles and move slowly
olybdenum, zinc, boron, chlorine and nickel. through the soil to plant roots.
49. (b) Nitrogen is one of the major constituents of proteins 67. (a) Potassium helps to maintain an anion-cation balance
nucleic acids vitamins and hormones. in cells and is involved in protein synthesis, opening
50. (a) and closing of stomata, activation of enzymes and in
51. (d) Ammonia is first oxidized to nitrite by the bacteria the maintenance of the turgidity of cells.
Nitrosomonas and or Nitrococcus. The nitrite is 68. (a) Nitrogen is the mineral element that plants require in
further oxidized to nitrate by Nitrobacter. These steps greatest amounts. It serves as a constituent of many
are called nitrification. These nitrifying bacteria are plant cell components, including amino acids and
chemoautotroph's. nucleic acids. Therefore, nitrogen deficiency rapidly
52. (b) 53. (a) 54. (c) 55. (a) inhibits plant growth.
56. (d) Zinc is used in the synthesis of a type of auxin, IAA 69. (d)
(indole-3- acetic acid). Sulphur is present in two amino 70. (b) Critical concentration of the nutrient may be defined
acids - cysteine and methionine and is the main as the minimum tissue content of the nutrient that is
constituent of several coenzymes, vitamins (thiamine, corrected with maximal growth or yield. As the
biotin, Coenzyme A) and ferredoxin. Magnesium is a nutrient concentration of the tissue increases
constituent of ring structure called chlorophyll and beyond the adequatic zone, growth or yield decline
helps to maintain the structure of ribosome. because of its toxicity.
Molybdenum is a component of several enzymes 71. (c) Nitrogen is the major constituents of proteins, nucleic
like nitrate reductase and nitrogenase which takes acids, vitamins and hormones. Phosphorous is a
part in the nitrogen metabolism.
constituent of cell membranes, certain proteins, all
57. (a)
nucleic acids and nucleotide and is required for all
58. (c) Nitrosomonas converts the ammonia to nitrogen
phosphorylation reactions. Therefore, nitrogen and
while the Azotobacter is aerobic free-living nitrogen
phosphorous are utilized by plants for the formation
fixing bacteria residing in soil.
of adenosine triphosphate (ATP).
59. (c) Plants obtain sulphur in the form of sulphate (SO42–).
60. (a) Magnesium is a constituent of the ring structure of 72. (a) The entry or exit of ions to and from the symplast
chlorophyll. Function of manganese is to activate required the expenditure of metabolic energy (ATP),
many enzymes involved in photosynthesis, which is an active process.
respiration and nitrogen metabolism. The best 73. (a)
defined function of manganese is the splitting of 74. (d) Denitrification (Z) is the process of converting nitrates
water to liberate oxygen during photosynthesis. and ammonia of soil to molecule nitrogen with the
61. (c) Rhodospirillum is a free-living, anaerobic
S-68 Biology
help of aerobic bacteria like Pseudomonas (X) and during photosynthetic phosphorylation.
Thiobacillus (Y). Manganese is absorbed by the plants when it is in
75. (a) Biological nitrogen fixation is a process in which the bivalent form. Manganese participates in the
nitrogen gas (N2) from the atmosphere is incorporated photolysis of water in pigment system II during
into the tissue of certain plants. Only a select group photosynthesis and thus it helps in the electron
of plants is able to obtain N this way, with the help transport from water to chlorophyll.
of soil microorganisms. Iron is mainly available in the ferrous form and it is
76. (d) Azotabacter and Beijernickia are free living aerobic absorbed in the ferric form, also. It is a part of
nitrogen fixing bacteria. Anabaena is both free living catalases, peroxidases, cytochromes etc. and plays
and symbiotic nitrogen fixing cyanobacteria. a role in electron transport system in photosynthesis.
Rhizobium is symbiotic nitrogen fixing bacteria. 86. (d) Anabaena is known for its nitrogen fixing abilities.
77. (d) Nitrogen is the mineral element required by plants in They are capable of transforming atmospheric
the greatest amount. It is absorbed from the soil by nitrogen into fixed nitrogen (inorganic compounds
the plant in the form of nitrate. usable by plants).
78. (c) Rhodospirillum is a free-living anaerobic photo- 87. (b) Azotobacter is a free living non photosynthetic
autotrophic nitrogen fixing bacteria. aerobic nitrogen fixer.
79. (c) Nitrogen fixation requires a strong reducing agent 88. (c) Only a few elements have been found to be absolutely
and energy in the form of ATP. N2 -fixation is essential for plant growth and metabolism. These
accomplished with the help of nitrogen fixing elements are further divided into two broad
microbes, mainly Rhizobium. categories based on their quantitative requirements:
80. (b) The enzyme nitrogenase catalyses the conversion macronutrients and micronutrients.
of atmospheric nitrogen to ammonia, the first stable 89. (a) Foliar feeding is not a mechanisms or moving minerals
product of nitrogen fixation. This enzyme is highly into roots. It is a technique of feeding plants by
sensitive to the molecular oxygen, therefore it applying liquid fertilizer directly to their leaves. Then
requires anaerobic conditions. Therefore, nitrogen plants absorb essential elements through their
fixation by organisms require anaerobic condition. leaves. The absorption takes place through their
81. (b) Rhizobium can fix nitrogen for plants like clover and stomata and also through their epidermis.
beans. Rhizobia are soil bacteria that induce the 90. (b) Nitrogen gas cannot be directly absorbed or utilized
formation of special structures, called nodules, on directly by plants because nitrogen itself is
the roots of their host plants. Inside these nodules, unreactive, and cannot be used by green plants to
the rhizobia fix nitrogen by converting dinitrogen make protein. Nitrogen gas therefore needs to be
(the nitrogen gas that makes up 80% of the air one converted into nitrate compound in the soil by
breathe) into ammonia. Ammonia is toxic, so it is nitrogen-fixing bacteria in soil, root nodules or
rapidly assimilated into organic compounds, most lightning.
of which the bacteria pass to the plant to fulfill its 91. (d) Nodules, formed in leguminous roots, contains
nutritional need for nitrogen. nitrogen fixing bacteria. These bacteria can fix
82. (a) 83. (a) atmospheric nitrogen into biologically usable forms.
84. (a) Commercial fertilizers is a substance which contains 92. (d) The kind of deficiency symptoms shown in plants
one or more known plant nutrients and that is used includes chlorosis, necrosis, and stunted plant
for its plant nutrient content. Important minerals used growth, premature fall of leaves and buds, and
in it are nitrogen, phosphorous and potassium which inhibition of cell division. Chlorosis is the loss of
usually meet nutrients deficiencies in a more balanced chlorophyll leading to yellowing in leaves. This
manner and require less labour to apply than straight symptom is caused by the deficiency of elements
fertilizers used separately. The label representing 10 like N, K, Mg, S, Fe, Mn, Zn and Mo. Likewise,
- 20 - 10 on its packet denotes the percentage of necrosis, or death of tissue, particularly leaf tissue,
nitrogen, phosphorous and potassium. occurs due to the deficiency of Cu, K, Ca and Mg.
85. (d) Copper is absorbed on the clay particles as divalent 93. (d) A lack of mitrogen in the soil causes the yellowing
cations, from where it can be absorbed by the plants of leaves in tomato plant.
by exchange mechanism. It is constituent of 94. (d)
plastocyanin which takes part in electron transport
EBD_7209
Hints & Solutions S-69

Chapter 13 : Photosynthesis 8. (b) Paper chromatography is an analytical method that


is used to separate coloured chemicals or substances,
1. (a) Photosynthesis is a physiochemical process by which especially pigments. This can also be used in
green plants and some other organisms use sunlight secondary or primary colours in ink experiments. Leaf
to synthesize nutrients from carbon dioxide and pigments of any green plants can be separated by
water, and generates oxygen as a by-product. using paper chromatography. A chromatographic
2. (d) Photosynthesis is the most important anabolic separation of the leaf pigments shows that the colour
process on earth. It is defined as the transformation we see in leaves is due to a single pigment but due to
of photonic energy (i.e. light or radiant energy) into four pigments: chlorophyll a (bright or blue green in
chemical energy by the given parts of the plants. In the chromatogram), chlorophyll b (yellow green),
the process of photosynthesis, light energy drives xanthophylls (yellow) and carotenoids (yellow to
the synthesis of carbohydrates from carbon dioxide yellow orange).
and water with the generation of oxygen. 9. (d) Accessory pigments are light-absorbing compounds
3. (b) Half leaf experiment proves that CO2 is essential for that traps light energy and channels it to chlorophyll
photosynthesis. a, the primary pigment, which initiates the reactions
4. (c) Joseph Priestley (in 1970) performed a series of of photosynthesis. Accessory pigment includes the
experiments that revealed the essential role of air in the carotenoids, phycobiliproteins, and chlorophylls b,
growth of green plants. c, and d. Indeed, they not only enable a wider range
5. (a) Jan Ingenhousz showed that sunlight is essential to of wavelength of incoming light to be utilized for
the plant process that somehow purifies the air fouled photosynthesis but also protect chlorophyll a from
by burning candles or breathing animals. Ingenhousz the photo-oxidation.
in an experiment with an aquatic plant showed that in 10. (c) The light-harvesting complex (or antenna complex)
bright sunlight, small bubbles were formed around the is an arrangement of various proteins and chlorophyll
green parts while in the dark they did not form any molecules embedded in the thylakoid membrane of
bubbles. Later he identified the bubbles as oxygen. plants, which transfer light energy to one chlorophyll
Hence, he showed that it is only the green parts of a molecule at the reaction centre of a photosystem.
the plants that could release oxygen. It is used by plants and photosynthetic bacteria to
6. (c) By studying purple sulphur bacteria and green sulphur collect more of the incoming light than would be
bacteria, Cornelius Van Niel was the first scientist to captured by the photosynthetic reaction centre alone.
demonstrate that photosynthesis is a light-dependent The main function of the light-harvesting complexes
redox reaction in 1931, in which hydrogen from an is to gather light energy and to transfer this energy
oxidizable compound reduces carbon dioxide to cellular to the reaction centres for the photo-induced redox
materials. This can be expressed as: 2 H2A + CO2 ® processes.
2A + CH2O + H2O, where A is the electron acceptor. 11. (a) Chlorophyll a is the pigment that participates directly
His discovery predicted that H2O is the hydrogen in the light reaction of photosynthesis. It absorbs
donor in green plant photosynthesis and is oxidized the wavelengths of violet-blue and orange-red light
to O2. and functions as a primary electron donor during
7. (d) Photosynthesis is the process by which plants use the election transport chain in photosynthesis. It
the energy from sunlight to produce sugar (fuel) used also functions in transporting energy to the reaction
by all living things. The photosynthesis equation is centre where P680 and P700 are located. The 'light-
a chemical representation of the process of dependent reactions' is the first stage of
photosynthesis which takes place in the photosynthesis, in which plants capture and store
chloroplasts. Plants take in carbon dioxide and water energy from sunlight. In this process, light energy is
to produce glucose (carbohydrate) and oxygen. The converted into chemical energy, in the form of the
following is the chemical equation, which explains energy-carrying molecules ATP and NADPH.
this process: 12. (b) In PS - I, the reaction centre Chlorophyll a has
sunlight
absorption maxima at P700 while in PS - II the reaction
6CO2 + 12H2O ¾¾¾¾¾® C6H12O6 + 6O2 centre Chlorophyll a has absorption centre maxima
chlorophyll
+ 6H2O at P680.
13. (c) An energy diagram for the transfer of electrons in
S-70 Biology

the light reaction of photosynthesis in plants is Z- the first CO2 fixation product was a 3-carbon organic
scheme. Plastocyanin transfers electrons to P700 acid.
reaction centre of PS I. On getting excited, P700 hand 23. (a) In Calvin cycle, carboxylation is the most crucial step
over electrons to a special electron acceptor molecule, where CO2 is utilised for the carboxylation of RuBP.
which passed downhill to energy rich molecule. This In this CO2 reacts with ribulose 1,5 biphosphate to
is called Z scheme due to its characteristic zig zag yield two molecules of 3-phosphoglycerate, a
shape. This shape is formed when all the carriers are reaction catalyzed by the chloroplast enzyme ribulose
placed in a sequence on a redox potential scale. biphosphate carboxylase-oxygenase, referred to as
14. (b) Electrons are picked up by an electron acceptor RuBisCO.
which passes them to an electron transport system 24. (c) For every CO2 molecule entering the Calvin cycle, 3
consisting of cytochromes. Cytochromes are molecules of ATP and 2 molecules of NADPH are
compounds which consist of haem bonded to a required.
protein and function as electron transfer agents in 25. (b) In C4 plants, a 4 - C compound oxaloacetic acid (OAA)
many metabolic pathways, especially cellular is the first stable product, and phosphoenol pyruvate
respiration. (PEP) is the CO2 acceptor. This reaction is catalyzed
15. (b) PS II is located on the inner surface of appressed by the enzyme PEP carboxylase or PEP case in
parts of grana thylakoids. It comprises of about 200 mesophyll cells of the leaf.
chlorophylls, 50 caretonoids and one molecule P680. 26. (c) In C4 plants, photosynthesis occurs in chloroplast
P680 of PS-II absorbs light energy, gets excited and of mesophyll and bundle sheath cells. While in C3
transfers its electrons to electron acceptor and plants photosynthesis occurs only in mesophyll
becomes a strong oxidant. It paves the way for light cells.
dependent splitting of water called photolysis. 27. (b) In C4 plants, a 4 - C compound oxaloacetic acid (OAA)
16. (a) Photolysis of water is a process of breakdown of is the first stable product, and phosphoenol pyruvate
water molecules into hydrogen (H+) and oxygen [O] (PEP) is the CO2 acceptor. This reaction is catalyzed
and electrons by the influence of light during the by the enzyme PEP carboxylase or PEP case in
light reactions of photosynthesis. The hydrogen mesophyll cells of the leaf.
released from the water molecule is transferred to 28. (a) Chloroplast movement is influenced by light
the hydrogen acceptor. NADP becomes reduced to exposure. Light-induced changes in the cellular
form NADPH . The splitting of water is associated distribution or orientation of chloroplasts have been
2
with the PSII. observed in nearly all green plants including algae,
17. (a) Chemiosmotic hypothesis has been put forward by mosses, ferns, and angiosperms. Under low light
Mitchell (1961) to explain the mechanism of ATP conditions, chloroplasts accumulate along the cell
synthesis. walls that are perpendicular to the incident light.
18. (a) ATP synthesis is linked to the development of a Under high light conditions, they accumulate along
proton gradient accros the membranes of thylakoid. the walls that are parallel to the incident light. These
19. (c) When electrons are transported through the electron are the regions of plant leaf cells where internal
transport system (ETS), the protons accumulate fluence rates of light are the highest and lowest,
inside the thylakoid membranes. Lumen of thylakoid respectively and it is believed likely that the light-
becomes enriched with H+ ion due to photolytic induced chloroplast movements serve an adaptive
splitting of water. function.
20. (d) The light driven synthesis of ATP and NADPH 29. (a) In C4 plants, a 4 - C compound oxaloacetic acid
provides energy and reducing power for the (OAA) is the first stable product, and phosphoenol
conversion of inorganic carbon into organic carbon; pyruvate (PEP) is the CO2 acceptor. This reaction is
for the production of sugars and fixation of CO2 into catalyzed by the enzyme PEP carboxylase or PEPcase
trioses. in mesophyll cells of the leaf.
21. (c) Number of carbons in the primary CO2 fixation 30. (a) In C4 plants, bundle sheath cells are rich in RuBisCO,
product of C4 plant is 4. The C4 acid, oxaloacetic but lack PEPcase. The CO2 released in the bundle
acid (OAA) is formed in the mesophyic cells. sheath cells enters the C3 or the Calvin pathway,
22. (b) The use of radioactive 14C by Melvin Calvin in algal common to all plants. Thus, the basic pathway that
photosynthesis studieshad led to the discovery that
EBD_7209
Hints & Solutions S-71

results in the formation of sugars, the Calvin pathway 37. (a) The principle of limiting factors was formulated by
is common to the C3 and C4 plants. Blackman (1905). It states that when a process is
31. (d) In C4 plants, C4 cycle occurs in mesophyll cells and conditioned as to its rapidity by a number of separate
C3 - cycle occurs in a bundle sheath. factors, the rate of process is limited by the pace of
32. (d) C4 plants possess Kranz type of leaf anatomy. The slowest factor.
mesophyll is undifferentiated and its cells occur in 38. (a) During photosynthesis in green plants, light energy
concentric layers around vascular bundles. C4 plants is captured and used to convert water, carbon
possess dimorphic chloroplasts. While in C3 plants dioxide, and minerals into oxygen and energy-rich
leaf anatomy is not of Kranz type. Only one type of organic compounds. Factors which influence the rate
chloroplasts are found. of photosynthesis are of two kinds - external
33. (d) In C4 plants , the process by which C4 acids are (environmental) or internal. The amount of light, the
converted into C3 acids in the bundle sheath cell is carbon dioxide supply, temperature, water supply,
known as decarboxylation. Shortly after the and the availability of minerals are the most important
discovery of the C4 cycle the Crassulacean acid environmental factors that affect the rate of
metabolism (CAM) pathway was dissected and photosynthesis in land plants. Plants factors affecting
elucidated as a C4 variant. This pathway is found in photosynthesis includes number, age, size and
desert succulents and epiphytes. In the typical C4 orientation of leaves, mesophyll cells and
cycle, the fixation reaction occurred in a mesophyll chloroplast, internal CO2 concentration and the
cell and the decarboxylation reaction occurred in a amount of chlorophyll.
bundle sheath cell, Decarboxylation is a chemical 39. (d) Photosystem (PS) - I and II are two pigments system
reaction that removes a carboxyl group and releases of light reaction. Non-cyclic photophosphorylation
carbon dioxide (CO2 ). Enzymes that catalyze is the light-requiring part of photosynthesis in higher
decarboxylation are called decarboxylases. plants, in which an electron donor is required, and
34. (c) Light induced CO2 liberation from a C2 compound oxygen is produced as a waste product. It consists
(glycolic acid) of dark phase is called of two photoreactions, resulting in the synthesis of
photorespiration. It occurs in green cells only. ATP and NADPH2. The hydrogen needed for the
Photorespiration is absent in C4 plants and is present reduction of NADP (nicotinamide adenine
in C3 plants. Photorespiration involves three cell dinucleotide phosphate) is made available from the
organelles–Peroxisomes, chloroplasts and breakdown of water.
mitochondria. 40. (b) In Z-scheme, the movement of electrons is downhill
35. (d) Photorespiration is the process by which in the in terms of an oxidation reduction or redox potential
presence of light plant consumes oxygen and scale.
releases carbon dioxide (instead of fixing carbon 41. (b) Chlorophyll a is the chief pigment associated with
dioxide) during photosynthesis, resulting in a photosynthesis.
decrease in photosynthetic output since no ATP is 42. (a) PS-I is located on the outer surface of non appressed
produced and carbon (as well as nitrogen in the form parts of the grana thylakoids and fret channels. This
of ammonia) is lost inevitably. In this pathway there system does not receive electrons from photosystem
is no synthesis of sugar, ATP or NADPH, due to II. This system is not directly involved with the
which photorespiration refers to as a wasteful photo-oxidation of water and evolution of molecular
process. This process reduces efficiency of oxygen.
photosynthesis in C3 plants. 43. (c) The single chlorophyll a molecule forms the reaction
36. (a) Light induced CO2 liberation from a C2 compound centre. The reaction centre is different in both the
(glycolic acid) of dark phase is called photosystems.
photorespiration. It occurs in green cells only. It is 44. (d) Cyclic photophosphorylation involves only
absent in C 4 plants and present in C3 plants. photosystem I. In each flow of electrons, 2 molecules
Photorespiration occurs because oxygen rather than of ATP are synthesized. NADPH is not produced. It
carbon dioxide links to the rubisco enzyme in the is not connected with photolysis of water, so no
Calvin cycle. It involves three cell organelles - oxygen is evolved.
peroxisomes, chloroplast and mitochondria. 45. (b) Photophosphorylation (discovered by Arnon) is of two
types- cyclic and non-cyclic phosphorylation. Non
S-72 Biology

cyclic photophosphorylation involves the mechanism, the tendency of rubisco enzyme to photo
participation of both PS I and PS II. The first step is respire, or waste energy by using oxygen to break
photo-oxidation of water resulting in splitting of water down carbon compounds to CO2, is minimized. Most
into H+, e– and release of O2. It involves sequence C4 plants have a special leaf anatomy (called Kranz
of electron transfer where NADP+ is reduced by PS- anatomy) in which the vascular bundles are
I, PS-I is reduced by PS-II and PS-II is reduced by surrounded by bundle sheath cells, shows a response
water so that the electro transport is non-cyclic. to highlight intensities, lacks photorespiration and
46. (c) The Calvin cycle takes place in the stroma of have a greater productivity of biomass. Examples of
chloroplasts of mesophyll cell because enzymes and C4 plants include sugarcane, maize, sorghum,
intermediates of the Calvin cycle are located in the amaranth, etc.
stroma of chloroplasts. 54. (c) In C3 plants the primary CO2 acceptor is RuBP and
47. (d) Rubisco is an enzyme present in chloroplasts, the initial stable product is PGA. While in C4 plants,
involved in fixing atmospheric carbon dioxide during the primary CO2 acceptor is PEP and the first stable
photosynthesis and in oxygenation of the resulting product is OAA.
compound during photorespiration. It is the most 55. (b) Six molecules of CO2 enter Calvin cycle to produce
abundant enzyme on the earth and catalyzes the one hexose molecule whereas 18 ATP, 12 NADPH +
carboxylation of CO2 to ribulose 1, 5-bisphosphate, H+ molecules are used up. The light reaction of
initiates photorespiration when the CO2/O2 ratio is photosynthesis results in ATP and NADPH 2
low. formation.
48. (d) Water stress causes the stomata to close hence 56. (b) C4 pathway is an adaptation of tropical plants to
reducing the CO2 availability. reduce/avoid the photorespiratory loss. In C4
pathway, first acceptor of CO 2 is a 3 carbon
49. (c) Light-induced CO2 liberation from a C2 compound compound - phosphoenol pyruvate.
of dark phase is called photorespiration. This 57. (d) Mitochondria helps in cellular respiration by
process is considered to nullify the result of transferring energy from organic compounds to ATP.
photosynthesis as there is no synthesis of ATP or Chloroplast helps in photosynthesis. Dark reaction
NADPH. takes part in the stroma of the chloroplast.
50. (d) Photorespiration (C2 cycle) is enhanced by bright 58. (b) In bacteria, photosynthesis utilizes light wavelength
more than 700 nm and their reaction centre is B-890.
light, high temperature, high oxygen and low CO2
59. (b) Light reaction or Hill reaction results in the formation
concentration. In C 3 plants, RuBisCO, acts as of ATP and NADPH2, 6CO2 , 6H2 O, ATP and
ribulose biphosphate oxygenase under low NADPH2 are utilised to produce one molecule of
atmospheric concentration of C2 and increased glucose.
concentration of O 2 . Thus, photorespiration is 60. (b) Photosynthetic bacteria have a substance called
favoured by C3 plants.
51. (d) All the statements regarding splitting of water is bacteriochlorophyll. The bacterio-chlorophyll
correct. In photosynthesis, water splitting donates pigment absorbs light in the extreme UV and infra-
electrons to power the electron transport chain in
red parts of the spectrum which is outside the range
photosystem II.
52. (d) All the statements regarding ATP synthetase are used by normal chlorophyll, seen in plants.
correct. ATP synthase is an important enzyme that 61. (a) C4 pathway/Hatch and Slack pathway ensures the
provides energy for the cell to use through the Calvin cycle to be operated only in bundle sheath
synthesis of adenosine triphosphate (ATP). ATP is cell. It is an adaptation to photorespiratory loss.
the most commonly used "energy currency" of cells Therefore, C4 plants are adapted to photorespiratory
loss.
from most organisms.
62. (a)
53. (d) C4 plants utilize the C4 carbon fixation pathway in
63. (b)
which the CO 2 is first bound to a
phosphoenolpyruvate in mesophyll cell resulting in 64. (d) A : Emerson effect is the increase in the rate of
photosynthesis after chloroplasts are exposed
the formation of four-carbon compound
to light of wavelength 670 nm (far red light) and
(oxaloacetate) that is shuttled to the bundle sheath
700 nm (red light). It involves both photosystem
cell where it will be decarboxylated to liberate the
I and II.
CO2 to be utilized in the C3 pathway. In this
EBD_7209
Hints & Solutions S-73

B : Hills reaction (or photolysis of water) is the light- producing carbon dioxide and water as waste
dependent transfer of electrons by chloroplasts products, and ADP is converted to energy-rich ATP.
in photosynthesis that results in the cleavage of 67. (a) A : Photolysis of water - oxygen evolving complex
water molecules and liberation of oxygen. Hill ferric oxalate
reaction was proposed by Robert Hill. B : ATP synthesis - Proton gradient concentration
C : Calvin cycle (also called C3 pathway) is the set C : Pigments - Absorbs light at specific wavelengths
of chemical reactions that take place in stroma of D : High oxygen - Photorespiration
the chloroplasts during photosynthesis. The 68. (a) Maize is a C4 plant. It possesses Kranz anatomy
cycle is light-independent because it takes place which is absent in C3 plants.
after the energy has been captured from sunlight. 69. (b) Reaction center in PS - II is P680. PS-II is located on
D : Hatch & Slack cycle (also known as C4 pathway) the inner surface of appressed parts of grana
is a metabolic cycle involved in the non-light- thylakoid. PS-II is inolved only in non-cyclic electron
requiring phase of photosynthesis in certain transport.
plants having specific metabolic and anatomical 70. (d) Calvin cycle (also known as the Calvin-Benson cycle)
modifications in their mesophyll and bundle is the set of chemical reactions that take place in
sheath cells which facilitate the temporary stroma of the chloroplasts during photosynthesis.
fixation of carbon dioxide (CO2) into four-carbon The cycle is light-independent because it takes place
organic acid. These acids are next broken down after the energy has been captured from sunlight.
to three-carbon organic acids along with CO2 in The Calvin cycle proceeds in three stages:
bundle sheath cells, where this unbound CO2 is 1. Carboxylation - CO2 is covalently linked to a
then fixed into carbohydrates in a normal Calvin carbon skeleton (RuBP)
cycle pathway.
2. Reduction - carbohydrate is formed at the
65. (d) Dark reaction is a thermochemical reaction. It takes expense of ATP and NADPH
place in the stroma of the chloroplast. Dark reaction,
3. Regeneration - the CO2 acceptor RuBP reforms
also called CO2 fixation or carbon assimilation, occurs
at the expense of ATP.
even in the presence of light but it doesn’t require
In the given figure of Calvin cycle, the labelling
light. Dark reaction consists of 3 phases-
of carbohydrate molecule as I, II and III are
carboxylation, reduction and regeneration of CO2
respectively RuBP, PGA phosphate and triose
acceptor.
phosphate.
66. (d) A : Light reaction occurs in the grana of the
71. (a) In the given diagram of Calvin cycle, CO2 is
chloroplast. It is the first stage of photosynthesis
incorporated at stage P.
in which plants capture and store energy from
72. (c) In the given diagrammatic representation of Hatch
sunlight. In this process, light energy is
and Slack pathway (also known as C4 pathway), the
converted into chemical energy, in the form of
steps marked as P, Q and R are respectively C4 acids,
the energy-carrying molecules ATP and NADPH.
decarboxylation and C3 acids. It is a metabolic cycle
It was observed by Arnon.
involved in the non-light-requiring phase of
B : Dark reaction is a thermochemical reaction, which
photosynthesis in certain plants having specific
takes place in the stroma of the chloroplast. It is
metabolic and anatomical modifications in their
not a light driven reaction but are dependent on
mesophyll and bundle sheath cells which facilitate
the products of light reactions (ATP and
the temporary fixation of carbon dioxide (CO2) into
NADPH).
four-carbon organic acid. These acids are then broken
C : Glycolysis takes place in the cytoplasm. It is a
down to three-carbon organic acids along with CO2
metabolic process that occurs in nearly all living
in bundle sheath cells, where this freed CO2 is then
cells in which glucose is converted in a series of
fixed into carbohydrates in a normal Calvin cycle
steps to pyruvic acid and during which energy
pathway.
is released in the form of ATP.
73. (a) ATP synthase works via a concentration gradient;
D : Krebs cycle is the sequence of reactions by
high (H+) in the lumen, and low (H+) in the stroma.
which most living cells generate energy during
When the chloroplast is bathed in pH 4, the entire
the process of aerobic respiration. It takes place
thing will have a higher [H+]; once it is put in the
in the mitochondria (matrix), using up oxygen and
S-74 Biology
basic solution, you’ll still have the high [H+] in the independent. But they usually do not occur in night
lumen but no longer in the stroma. This is exactly because this cycle is dependent on the products of
what normally occurs via the light reactions the light reactions. The Calvin cycle proceeds in three
(cytochrome b6f pumps protons into the lumen when stages: carboxylation [CO2 is covalently linked to a
light activates the photosystems). So if you have a carbon skeleton (RuBP)], reduction (carbohydrate
proton gradient, you don’t need light to drive ATP is formed at the expense of ATP and NADPH) and
synthesis. regeneration (the CO2 acceptor RuBP reforms at the
74. (d) In the given diagram of ATP synthesis through the expense of ATP.
chemiosmosis, the correct labelling of the A, B, C 81. (a) 18 ATP and 12 NADPH molecules will be required to
and D are respectively F0, thylakoid membrane, make one molecule of glucose through the Calvin
photosystem II and photosystem I. Th e Pathways. Therefore to make 100 molecules of
chemiosmotic theory (Mitchell 1961) explains how glucose 1800 ATP and 1200 NADPH molecules are
ATP is generated in the chloroplast. ATP synthesis required.
in chloroplasts (photophosphorylation) proceeds 82. (d) Chloroplast is disrupted and the stroma separated
according to a mechanism that is basically similar to from the lamella, the isolated stroma will fix CO2 if it
that in mitochondria. is supplied with ATP+NADPH.
75. (d) Given figure represents Z scheme of light reaction. 83. (d) Calvin cycle (also called C3 pathway) is the set of
The whole scheme of transfer of electrons, starting chemical reactions that take place in stroma of the
from the PS II, uphill to the acceptor, down the electron chloroplasts during photosynthesis. The cycle is
transport to PS I, excitation of electrons transfer to light-independent because it takes place after the
another acceptor, and finally down hill to NADP+ energy has been captured from sunlight. The correct
causing it to be reduced to NADPH + H+ is called the sequence of Calvin cycle is: Carboxylation '
Z scheme, due to its characteristic shape. Reduction' Regeneration.
76. (d) Given graph shows the action spectrum of 84. (b) 6CO2, 18ATP and 12NADPH2 are required for the
photosynthesis superimposed on absorption production of one molecule of glucose, 18 ADP and
spectrum of chlorophyll a. 12 NADP through 6 rounds of Calvin cycle.
77. (a) In the given figure of chloroplast section the parts 85. (a) PEPcase; C4 cycle in mesophyll and RuBisco, C3
marked as A, B, C, D and E are respectively stromal cycle in bundle sheath is correct for C4 plants.
lamella, grana, stroma, starch granule and lipid 86. (a) In C4 plant, the first stable product is oxaloacetic
droplets. Stromal lamella (A), grana (B) and stoma acid (OAA), which is a 4 C-compound where PEP
(C) are considered as membrane system in the acts as the CO2 acceptor. So, the radioactivity will
chloroplast and are responsible for the trapping the first appear in OAA.
light energy and also the synthesis of ATP and 87. (c) Plant species possessing C4 pathway have highest
NADPH. photosynthetic yield.
78. (a) In the given figure of chloroplast X and Y are 88. (d) According to Blackmans law of limiting factor, at any
respectively grana and stroma. Grana are a given time photosynthesis can be limited either by
membranous structure within a chloroplast made up light or CO2. Blackman proposed the law of limiting
of stacks of thylakoids. Stroma is the matrix of the factors in 1905 to determine the rate of
chloroplast which is a thick fluid in between grana photosynthesis. According to this law, when a
where various enzymes, molecules and ions are process depends on a number of factors, its rate is
found, and where the dark reaction (or carbohydrate limited by the pace of the slowest factor.
formation reactions) of photosynthesis occurs. 89. (b) According to law of limiting factors given by
Grana and stroma are involved in photolysis of water Blackman, when a process depends upon a number
and CO2 fixation respectively. of factors which are separate, the rate of the process is
79. (d) In non-cyclic reaction of photosynthesis, electrons limited by the pace of the slowest factor. Rate of yield is
from chlorophyll molecules in photosystem I are used dependent on light as photosynthesis is dependent
in the formation of NADPH. The source of those on light. During monsoon, the light is dim and so this
electrons is photosystem II which splits water molecule. reduces rate of photosynthesis and hence yield.
80. (d) Calvin cycle that take place in stroma of the 90. (c) As the intensity of light increases, the rate of
chloroplasts during photosynthesis, is light- photosynthesis also increases. But at higher light
EBD_7209
Hints & Solutions S-75

intensity, the rate of photosynthesis decreases, molecules ATP and NADPH.


because of the following reasons– 97. (a) During light reaction, as electrons move through the
(i) other factors required for photosynthesis photosystems, protons are transported across the
become limiting. membrane because of the primary acceptor of e–
(ii) destruction of chlorophyll occurs, so there will (located towards the outer surface of the membrane)
be no carbohydrate (sugar) formation. transfers its electrons not to an e– carrier but to H
91. (c) Under water stress, the rate of photosynthesis carrier.
declines because of stomatal closure leading to 98. (a) The water molecule is split as shown in the equation :
decrease in CO2 supply and reduced water potential H 2O ® 1
O 2 + 2 H + + 2e – forming oxygen and
2
that decrease leaf surface areas for photosynthesis.
hydrogen radicals.
92. (c) In C4 (sugarcane plant) plant, 14CO2 is fixed in malic
99. (b) The function of water in photosynthesis is to supply
acid in which the enzymes that fixes CO2 is
electrons in the light dependent reaction. The light-
phosphoenol pyruvic acid car boxylase.
dependent reactions take place in the membranous
Phosphoenolpyruvate carboxylase catalyzes the
sections of the chloroplast - thylakoids (grana/
addition of bicarbonate (HCO 3 – ) to
lamellae) which offer a large surface area to absorb
phosphoenolpyruvate (PEP) to form the four-carbon
light energy. The main function of these reactions is
compound oxaloacetate and inorganic phosphate:
to provide a source of ATP and reduced NADP, which
PEP + HCO3– Oxaloacetate + Pi
are used to reduce CO2 in the light independent
This reaction is used for carbon fixation in CAM
reactions
(Crassulacean acid metabolism) and C4 organisms,
100. (a) Liberation of oxygen is not concerned with cyclic
as well as to regulate flux through the citric acid cycle
photophosphorylation. This system is not concerned
(also known as Krebs or TCA cycle) in bacteria and
with photo-oxidation of water but only concerned with
plants.
the production of ATP. Non-cyclic
93. (d) If the supply of oxygen is decreased to an illuminated
photophosphorylation is the light-requiring part of
wheat plant, its photosynthesis would increase.
photosynthesis in higher plants, in which an electron
94. (d) According to the experiment performed by the donor is required, and oxygen is produced as a waste
student, photosynthesis will not take place because product. It consists of two photoreactions, resulting
intact chloroplast is needed for the process of in the synthesis of ATP and NADPH.
photosynthesis. Photosynthesis is the process by
101. (d) Cyclic and non-cyclic flow of e– is used in plants
which green plants create energy from sunlight. It
- to meet the demands of Calvin cycle,
occurs at the cellular level in the leaves of plants and
- avoid producing excess NADPH + H+, and
is the way in which they produce oxygen and
- balance ATP and NADPH + H+ ratio in
carbohydrates. The oxygen is released into the
chloroplast
atmosphere, and the carbohydrates, simple sugars,
are used by the plant for growth. In order to carry 102. (b) Formation of ATP occurs during cyclic and non -
out photosynthesis, green plants need several cyclic photophosphorylation. The net product of
ingredients. Chlorophyll, the pigment in plants that cyclic photophosphorylation is ATP, NADPH and
makes them green, is essential to the photosynthetic O2 and in non - cyclic photophosphorylation
process. This chemical is produced naturally by all it is ATP only.
green plants and its role in photosynthesis is to 103. (d) During the electron transport from photosystem II
absorb light. to photosystem I, ATP is synthesized and NADP+
95. (d) The electrons that are released by the photolysis of is reduced to NADPH. So, the blockage of electron
water during non-cyclic photophosphorylation transport will stop ATP synthesis and the reduction
ultimately end in NADPH. of NADP+ to NADPH.
96. (b) "ADP is phosphorylated and NADP is reduced" 104. (b) The cooperation of two photosystems is required for
occurs during light phase of the photosynthesis. the upward electron transport from water to NADP+of
The 'light-dependent reactions' is the process by the two photosystems. Photosystem I is more directly
which plants capture and store energy from sunlight. associated with the reduction of NADP+, whereas the
In this process, light energy is converted into photosystem II is more closely associated with the
chemical energy, in the form of the energy-carrying
S-76 Biology
extraction of electrons from water. photosynthesis. Accessory pigment includes the
105. (d) Reduction of NADP+ into NADPH during light carotenoids, phycobiliproteins, and chlorophylls b,
reaction occurs in stroma because NADP reductase c, and d. Indeed, they not on enable a wider range of
enzyme is located on the stroma side of membranes wavelength of incoming light to be utilized for
and PS I reduces NADP+ to NADPH + H+ in the photosynthesis but also protect chlorophyll a from
stroma. the photo-oxidation.
106. (b) According to chemiosmotic hypothesis (Mitchell 115. (d) The “tails” of chlorophyll molecules are associated
1961) the ATP is synthesized on ATPase complexes with the thylakoid membranes of the chloroplasts.
located on the non appressed portions of thylakoid When excited by light, the chlorophyll moves into
membranes particularly towards margins. Therefore, an excited state and passes electrons to acceptor
if the thylakoid membrane within a chloroplast is molecules. This begins to set up the proton gradient
punctured so that there is no separation between across the membrane that will drive ATP synthesis.
lumen and stroma of thylakoid, synthesis of ATP 116. (c) Pigments are chemical compounds which reflect only
will be affected the most. certain wavelengths of visible light. More important
107. (b) than their reflection of light is the ability of pigments
108. (a) With the help of cytochrome b6f, proton is pumped to absorb certain wavelengths. Because they interact
to reach ATP synthetase to participate in ATP with light to absorb only certain wavelengths,
synthesis. pigments are useful to plants and other autotrophs
109. (a) 3-phosphoglyceric acid (3-PGA), a 3C-compound is organisms which make their own food using
the first stable product in C3 plant. photosynthesis. Photosynthetic pigments are
110. (c) One molecule of RuBP and one molecule of CO2 are located in the membranes of the thylakoid membranes
required to produce two molecules of 3-PGA. in specific areas called quantasomes. The
Therefore, for the production of 6 molecules of 3- quantasomes are mainly present in grana lamellae.
PGA, 3 molecules of RuBP and 3 molecules of CO2 117. (d) During light phase, ATP & NADPH are produced
are required. whereas oxygen is byproduct during light phase.
111. (d) The algae have the chlorophyll pigment and make 118. (b) The electrons generated by photolysis of water are
their food by photosynthetic process in which used to replace the electrons lost by P680 of PS-II.
oxygen is released as a byproduct. Although all 119. (d) Breakdown of water during the photosynthesis
algae essentially contain chlorophyll a pigment but molecules leads to release of electron, proton and
in some forms the green colour of chlorophyll oxygen.
pigment is masked by other pigments. For example, 120. (b) Kranz anatomy is a special structure in the leaves of
In phaeophyceae (brown algae), a brown pigment - plants that have a C4 pathway of carbon dioxide
fucoxanthin and in rhodophyceae (red algae) a red fixation. The leaves contain a ring of mesophyll cells,
pigment-r-phycoerythrin masks the green colour of containing a few small chloroplasts concerned with
the chlorophyll. the initial fixing of carbon dioxide, surrounding a
112. (d) Photosynthesis occurs particularly in specialized cell sheath of parenchyma cells (the bundle sheath)
called mesophyll cells. These cells contain which has large chloroplasts involved in the Calvin
chloroplast, which is the actual sites for cycle. In Kranz anatomy, the bundle sheath cells
photosynthesis. The cactus stem and guard cell of have thick walls, no intercellular spaces and large
stomata contain chloroplasts, so they can number of chloroplast.
manufacture food by photosynthesis. The epidermal
cells do not contain chloroplasts, so they do not Chapter 14 : Respiration in Plants
perform photosynthesis. 1. (b) Respiration is an enzyme controlled process of
113. (a) Green plants capture solar energy (light) from sun biological oxidation of food materials in a living cell,
and convert it into chemical energy (food) with the
using molecular oxygen (O2), producing CO2 and
help of CO2 taken from atmosphere, water from soil
H2O and releasing energy in a small steps and storing
and releases oxygen.
it in biologically useful forms (generally ATP). Hence,
114. (d) Accessory pigments are light-absorbing
respiration is a catabolic, exothermic and oxidative
compounds that traps light energy and channels it
to chlorophyll a which initiates the reactions of process.
EBD_7209
Hints & Solutions S-77

The reaction for respiration is: 12. (b) In the fermentation of one glucose molecule, there is
C6H12O6 + 6O2 ® 6CO2 + 6H2O + Energy net gain of two molecules of ATP.
13. (b) Yeast cell perform alcoholic fermentation under
2. (d) ATP is a nucleotide found in the mitochondria of all
anaerobic conditions where incomplete oxidation of
plant and animal cells. It is the major source of
glucose is achieved by sets of reaction in which
energy for cellular reactions and is commonly known
pyruvic acid is converted to CO2 and ethanol
as the energy currency of the cell. This energy is
(C2H5OH). The enzymes, pyruvic acid decarboxylase
released during its conversion to ADP. and alcohol dehydrogenase catalyse these reactions.
3. (a) 14. (d) In the presence of an enzyme complex, pyruvate
4. (d) Respiration is a metabolic process in which energy dehydrogenase complex, sulphur containing CoA
is produced with the intake of oxygen and the release and NAD + , pyruvate undergoes oxidative
of carbon dioxide from the oxidation of complex decarboxylation or both oxidation (removal of
organic substances along with water. hydrogen) and decarboxylation (removal of CO2). It
produces a 2-carbon active acetate group or acetyl
5. (a) Glycolysis occurs in the cytoplasm and does not
CoA, NADH + H+ and CO2.
use oxygen. Thus, it is an anaerobic pathway. In this
process, glucose is partially oxidized/converted into Pyruvate + NAD+ + CoA Pyruvate dehydrogenase Acetyl CoA
two molecules of pyruvate/ pyruvic acid. In aerobic (3 – C) Complex (2 – C)
respiration pyruvate enters mitochondrion through + NADH + H+ + CO2­
a specific transport protein. It undergoes oxidative
15. (a) 16. (c)
decarboxylation and dehydrogenation to produce
17. (b) Cytochrome is a compound which consists of haem
CO2 and NADH.
bonded to a protein and function as electron transfer
6. (a) Glycolysis occurs in the cytosol, where the glucose agents in many metabolic pathways, especially
molecule is converted into two molecules of the 3C
cellular respiration.
compound, pyruvate. The enzymes are located in
the fluid matrix of cytoplasm. 18. (c) Tricarboxylic acid cycle (TCA cycle), also called
Krebs’ cycle and citric acid cycle, is the process by
7. (d) During glycolysis, dehydration occurs in the
which living cells break down organic molecules in
presence of enzyme enolase and cofactor Mg2+.
the presence of oxygen to produce the energy they
2-phosphoglycerate loses a molecule of water and
need to grow and divide. In all organisms except
is changed into phosphoenol pyruvate (PEP).
bacteria the TCA cycle is carried out in the matrix of
8. (b) Decarboxylation is the removal or loss of a carboxyl intracellular structures of mitochondria.
group from an organic compound, usually resulting
19. (c) Enzymes of citric acid cycle are located in the cytosol
in the formation of carbon dioxide. It does not involve
in prokaryotes and mitochondrial matrix in
in glycolysis.
eukaryotes.
9. (d)
20. (c) During the conversion of succinyl CoA to succinic
10. (b) In human muscle cell lactic acid fermentation occurs acid a molecule of GTP is synthesized. This is a
where pyruvic acid produced in glycolysis is reduced substrate level phosphorylation. In the presence of
by NADH2 to form lactic acid without producing enzyme succinyl CoA synthase, succinyl CoA is
carbon dioxide. In fermentation process less energy hydrolysized to CoA and succinate are formed. The
is released. energy liberated during the process is used in
Lactate dehydrogenase synthesis of ATP in plants, and GTP in animals.
CH3COCOOH + NADH2 CH3CHOHCOOH
Pyruvic acid FMN, Zn2+ Lactic acid 21. (a)
+
NAD 22. (d) Oxaloacetic acid is the acceptor of acetyl coA in
Krebs’ cycle.
11. (c) Alcoholic fermentation is a process in which
23. (a) Krebs’ cycle does not consume any ATP molecules.
molecules such as glucose etc are converted into
It generates 2ATP/2GTP molecules thr ough
cellular energy and thereby produce ethanol and
substrate level phosphorylation.
carbon dioxide as metabolic waste products.
S-78 Biology
24. (a) Electron transport chain is a series of coenzymes electronegative compound to the more
and cytochromes that takes part in the passage of electropositive O2. On the basis of redox potential
electrons from a chemical to its ultimate acceptor. It cytochrome acceptor order is cyt b c a a3.
takes place on the cristae of mitochondria found on 33. (c) Krebs cycle (also called as tricarboxylic acid cycle,
the inner surface of the membrane of mitochondria. TCA) is amphibolic or having both catabolic and
25. (c) In the electron transport system, the final acceptor anabolic rates, for e.g. acetyl CoA entering the cycle
of proton is oxygen. Oxygen functions as the terminal is completely broken down to form CO2 and hydrogen
acceptor of electrons and is reduced to water along showing catabolic role. In anabolic role, acetyl CoA
with the hydrogen atoms in mitochondrial matrix. It is related to synthesis and breakdown of fatty acids,
drives whole process by removing hydrogen from steroids, carotenoids, terpenes and aromatic
system. compounds.
26. (d) Terminal cytochrome of respiratory chain is cyt a3. 34. (b) The ratio of the volume of CO2 liberated to the
cyt a3 posseses two copper centres. It helps in volume of oxygen absorbed per molecule during
transfer of electrons to oxygen. respiration is called Respiratory Quotient (RQ). The
27. (a) F1 head piece is a peripheral membrane protein value of RQ indicates the types of respiratory
complex of mitochondria which is the major site of substrate.
synthesis of ATP from ADP and Pi.
Volume of CO2 evolved
28. (a) The F head piece is a peripheral membrane protein RQ =
1 Volume of O 2 consumed
complex of mitochondria.
29. (d) Oxidation of molecule of NADH gives rise to 3 102
RQ = = 0.7
molecules of ATP, while that of one molecule of 145
FADH2 produces 2 molecules of ATP.
35. (c) The ratio of the volume of carbon dioxide evolved to
Therefore 2 NADH2 = 6 ATP molecules that of oxygen consumed by an organism, tissue, or
3 FADH2 = 6 ATP molecules cell in a given time is called respiratory quotients.
Total = 12 ATP molecules 36. (d)
37. (d) Glycolysis occurs in cytoplasm, is a common phase
30. (a) The electron transport chain is a process in which
of aerobic and anaerobic respiration. It is a partial
the NADH and FADH2 produced during glycolysis,
breakdown of glucose molecule into two molecules
b-oxidation, and other catabolic processes are
of pyruvic acid.
oxidized with a release of energy in the form of ATP.
The main purpose of electron transport chain is to 38. (d) Krebs’ cycle is the process by which living cells
cycle NADH + H+ back to NAD+. break down organic molecules in the presence of
oxygen to produce the energy they need to grow
31. (d) The enzyme responsible for oxidative
phosphorylation is ATP synthase. ATP synthase is and divide. It cycle involves decarboxylation steps.
located in the F1 component of F0 – F1 or elementary 39. (d) Cellular respiration is the mechanism of breakdown
particles. ATP synthase becomes active in ATP of food materials within the cell to release energy
formation only where there is a proton gradient and the trapping of this energy for synthesis of ATP.
having higher concentration of H+ or protons on the In cellular respiration, NADH is oxidised to NAD+.
F0 side as compared to F1 side. Increased proton 40. (c) ETS (Electron transport system or chain) is a series
concentration is produced in the outer chamber of of coenzymes and coenzymes that takes part in the
outer surface of inner mitochondrial membrane by passage of electrons from a chemical to its ultimate
pushing of protons with the help of energy liberated acceptor. ETS is present in the inner mitochondrial
by passage of electrons from one carrier to another. membrane, oxidation of one molecule of NADH gives
32. (b) The correct sequence of electron acceptor in ATP rise to 3 molecules of ATP, while that of one molecule
synthesis is cyt b c a a3. These are arranged in order of FADH2 produces 2 molecules of ATP. In
of their increasing redox potential and electron flow respiration, energy of oxidation - reduction is utilized
through the chain in step wise manner from the more for the production of proton gradient.
EBD_7209
Hints & Solutions S-79

41. (c) Fermentation represents anaerobic mode of 51. (b) Electron transport system is a series of co-enzymes
respiration. In fermentation the incomplete oxidation and cytochromes that takes part in the passage of
of glucose is achieved by sets of reactions where electrons from a chemical to its ultimate receptor.
The mitochondrial matrix contains all the soluble
pyruvic acid is converted to CO2 and ethanol.
enzymes of the citric acid or Kreb's cycle and those
42. (a) Pyruvate, is formed by the glycolytic catabolism of involved in the oxidation of fatty acids.
carbohydrates in the cytosol, after that it enters 52. (d) Electron transport takes place on the cristae of
mitochondrial matrix and undergoes oxidative mitochondria found on the inner surface of the
decarboxylation by a complex set of reactions membrane of mitochondria. Conversion of pyruvic
catalyzed by pyruvic dehydrogenase. acid into CO2 and ethanol is called fermentation.
43. (c) 44. (d) Glycolysis occurs in cytoplasm, is a common phase
45. (c) The complete oxidation of pyruvate by the stepwise of aerobic and anaerobic respiration. It is a partial
removal of all the hydrogen atoms, leaving three breakdown of glucose molecule into two molecules
molecules of CO2. The passing on of the electrons of pyruvic acid. Krebs’ cycle is the process by which
removed as part of the hydrogen atoms to molecular living cells break down organic molecules in the
O2 with simultaneous synthesis of ATP. presence of oxygen to produce the energy they need
to grow and divide. It takes place in the mitochondrial
46. (a) Glucose undergoes partial oxidation to form two
matrix.
molecules of pyruvic acid, Glucose is phosphorylated
to give rise to glucose-6-phosphate by the activity 53. (c) 54. (b)
of the enzyme hexokinase. The scheme of glycolysis 55. (c) Krebs’ cycle involves decarboxylation.
was given by Gustav Embden, Otto Meyerhof, and 56. (a) P, Q, R and S in the given major pathway of anaerobic
J. Parnas, and is often referred to as the EMP pathway. respiration are respectively NAD+, ethanol, lactic
47. (a) Glycolysis occurs in cytoplasm as all necessary acid and PEP respectively.
enzymes are found in it. This process is common in 57. (a) 58. (c) 59. (b) 60. (b) 61. (d)
aerobic/anaerobic respiration. In this process, one
62. (c) Respiratory substrate is a organic substance that
glucose molecule is converted into two moles of can be broken down in respiration to release energy.
pyruvic acid. Carbohydrates, fats, and proteins are examples of
48. (c) Compensation point is that value or point in the light respiratory substrates. The most common among
intensity and atmospheric CO2 concentration when them is carbohydrates, especially glucose. Different
the rate of photosynthesis is just equivalent to the substrates have different energy values.
rate of respiration in the photosynthetic organ. So
that there is no net gaseous exchange. 63. (a) Respiration is the breakdown of C—C bonds. In
49. (b) The stomata are absent since gaseous exchange respiration, the breaking of the C—C bond of complex
takes place through diffusion in submerged plants. organic molecules through oxidation provides carbon
50. (d) Glycolysis is the process of breakdown of glucose or skeleton for the synthesis of a large number of other
similar hexose sugar into two molecules of pyruvic essential plant products, such as polysaccharide,
acid through a series of enzyme mediated reactions, proteins, fats, nuclei acids, pigments, cytochromes
releasing energy (ATP) and reducing power etc.
(NADH2). It is the first step of respiration, which 64. (c) ATP is the major source of energy for cellular
occurs inside the cytoplasm and is independent of reactions and is commonly known as the energy
O2. In glycolysis, two molecules of ATP are consumed currency of the cell. This energy is released during
during double phosphorylation of glucose to form
its conversion to ADP. Energy accumulates in ATP
fructose 1, 6 diphosphate. Four molecules of ATP are
in high energy phosphate bonds.
produced in the conversion of 1, 3-
diphosphoglycerate to 3-phospho-glycerate and 65. (d) In anaerobic respiration, oxygen is not used in the
phosphenol pyruvate to pyruvate whereas, two breakdown of respiratory substrate.
molecules of NADH2 are formed during oxidation of 66. (d) The oxygen obtained from cellular respiration
glyceraldehyde 3-phosphate to combines with the hydrogen obtained from the
1,3-diphosphoglycerate. Since, each NADH is
oxidation of organic molecules to form water.
equivalent to 3 ATP, so net gain in glycolysis is 8 ATP.
S-80 Biology
67. (a) Glycolysis is a partial breakdown of glucose molecule 87. (b) 88. (a) 89. (b)
into two molecules of pyruvic acid. It occurs in 90. (b) In succulent plants like Opuntia there is an
cytoplasm and a common phase of aerobic and incomplete oxidation of the carbohydrates resulting
anaerobic respiration. in the formation of organic acids. CO2 is not released.
68. (b) Glucose – 6-phosphate ® 3- 2C6H12O6 + 3O2 ® 3C4H6O5 +3H2O+386kcal.
phosphoglyceraldehyde ®
CO2 0
3-phosphoglyceric acid ® phosphoenol pyruvate R.Q. = = =0
O2 3
® pyruvic acid.
69. (a) 70. (a) 91. (b)
71. (c) If hexokinase is blocked then glucose will not convert 92. (b) In glycolysis, the end product is pyruvic acid.
into glucose-6-phosphate which result in the blockage 93. (d)
of whole glycolytic pathway. 94. (d) Fermentat1ion is the process in which a substance
72. (b) 12 kcal of energy present in one molecule of ATP & breaks down into a simpler substance.
on oxidation of one mole of glucose into CO2 and Microorganisms like yeast and bacteria usually play
H2O energy released is 686 kcal. So no. of ATP which a role in the fermentation process, creating beer, wine,
can store this energy would be = 57.1 = 57 ATPs. bread, yogurt and other foods. Glycolysis occurs in
73. (a) Glycolysis is a partial breakdown of glucose molecule cytoplasm, is a common phase of aerobic and
into two molecules of pyruvic acid. Number of ATP anaerobic respiration. It is a partial breakdown of
formed during glycolysis is 4 ( 2 ATP for each PGA). glucose molecule into two molecules of pyruvic acid.
Net gain of ATP during glycolysis is 2 (4 – 2= 2). Krebs’ cycle is the process by which living cells
break down organic molecules in the presence of
74. (c) 75. (c)
oxygen to produce the energy they need to grow
76. (a) Alcoholic fermentation is a process in which and divide.
molecules such as glucose etc are converted into
95. (b) 96. (c) 97. (d)
cellular energy and thereby produce ethanol and
carbon dioxide as metabolic waste products. During 98. (b) In Krebs’ cycle, number of ATP molecules are
alcoholic fermentation, NAD+ is produced when produced in large amount i.e. 24. Therefore it
acetaldehyde is reduced to ethanol. generates large amount of energy.
77. (a) 78. (b) 79. (a) 80. (b) 81. (d) 99. (b) 100. (c) 101. (a) 102. (b) 103. (b)
82. (c) Chapter 15 : Plant Growth and Development
83. (b) ATP synthetase is a complex V of the electron
1. (c) Growth is defined as a permanent or irreversible
transport chain. When the electrons are shunted increase in the dry weight, size, mass or volume of a
over the carriers via complex I to IV in the ETC, the cell, organ or organism. Generally, growth is
are coupled to ATP synthetase for the formation of accompanied by metabolic processes (both anabolic
ATP from ADP and Pi. and catabolic), that occur at the expense of energy.
84. (d) 85. (c) 2. (b) Seed germination is the process by which a plant grows
86. (d) Chemiosmotic theory postulated by the British from a seed. Germination is the initial phase of growth
and emergence from the ground. Germination takes
biochemist, Peter Mitchell (1920-22) to explain the
place when the cotyledons emerge above the ground.
formation of ATP in the mitochondrial electron
3. (d) Scarification is the method that renders the seed coat
transport chain. As electrons are transferred along
permeable to water so that expansion of embryo is
the electron carrier system in the inner mitochondrial not physically retarded. It breaks seed dormancy. It
membrane, hydrogen ions (protons) are actively may be mechanical abrasions or chemical (weakening
transported into the space between the inner and of seed coat by acids, water, solvent etc).
outer mitochondrial membranes, which thus contains 4. (a) Arithmetic growth is a type of growth in which the
a higher concentration of protons than the matrix. rate of growth is constant and increase in growth
This creates an electrochemical gradient across the occurs in arithmetic progression i.e., 2, 4, 6, 8 etc. Here
inner membrane, through which protons move back after mitosis, only one daughter cell continues to divide
into the matrix. and other takes part in differentiation and maturation.
Here a linear curve is obtained with positive value.
EBD_7209
Hints & Solutions S-81

5. (a) The rate of growth can be measured by an increase supported by Kurosawa. Yahuta and Sumuki finally
in size or area of an organ of the plant like leaf, flower, isolated and crystallize these chemicals and named
fruit etc. in a unit time. The rate of growth is also them as gibberellic acids or gibberellins (GA3).
called efficiency index. 13. (b) When GA is sprayed on rosette plants, there is a
6. (c) Differentiation is maturation of cells derived from rapid expansion of internodes and flowers with long
optical meristems of root and shoot. It is applied to stalks are produced. This phenomenon is called
the qualitative differences between cells, tissues and bolting. If GA is sprayed on single gene dwarf plant,
organs. During differentiation cell undergoes few to genetic dwarfism can be overcome and plant
major anatomical and physiological changes both in becomes long, e.g., in maize and pea etc.
their cell walls and protoplasm. For example, to form 14. (d) Gibberellins (GAs) are plant hormones that regulate
tracheary element, the cells would lose their growth and influence various developmental
protoplasm. processes, including stem elongation, germination,
7. (c) In different phases of growth, plants follow different dormancy, flowering, sex expression, enzyme
pathways and form different kinds of structures in induction, and leaf and fruit senescence. GA is also
response to environment. The ability to change under responsible for bolting (internode elongation just
the influence of internal or external stimuli is called prior to flowering).
plasticity, e.g., heterophylly in cotton, coriander and 15. (c) GA3 was one of the first gibberellins to be discovered
larkspur. Occurence of different types of structures and remains the most extensively studied form.
on same plant in different growth phases or under 16. (a) Cytokinins are mildly basic growth hormones which
different environment conditions is known as are usually amino purine derivatives and promote
heterophylly. cell division in plants. Cytokinins inhibit apical
8. (c) During mid- 1960s, three independent researches dominance while auxins promote apical dominance.
reported the purification and chemical 17. (d) Zeatin is the most common type of naturally
characterization of three different kinds of inhibiters occurring cytokinin in plants which was isolated in
: inhibitor - B, abscission II and dormin. Later all the pure crystalline form from corn (Zea mays). It is also
three were proven to be chemically identical. It was isolated from coconut milk which is widely used in
named abscisic acid (ABA). ABA plays an important the preparation of culture medium and promotes
role in seed development, maturation & dormancy. multiple shoot generation from internodes and high
9. (a) Auxin is a weakly acidic hormone having an frequency thick spread roots.
unsaturated ring structure and promotes cell 18. (a) Kinetin (a modified form of adenine a purine) is a
elongation especially in shoots. Auxins was first type of synthetic cytokinin, a class of plant hormone
isolated from human urine. Auxins like IAA and IBA that promotes cell division. Kinetin was originally
have been isolated from plants. NAA and 2, 4 - D are isolated by Miller and Skoog et al. as a compound
synthetic auxins. from autoclaved DNA from Herring sperms that had
10. (d) Auxins inhibit the growth of axillary buds and cell division -promoting activity. Chemically Kinetin
promote apical dominance. It induces parthenocarpic was identified as 6-furfurylamino purine.
development of fruits and such fruits are seedless 19. (a) Cytokinins (CK) are a class of plant growth substances
and auxin is also responsible for phototropism and that promote cell division, or cytokinesis, in plant roots
geotropisms. Ripening and maturity of fruits are and shoots. Cytokinins are found in abundance in
related to ethylene. young roots, leaves and young fruits where rapid cell
11. (c) A synthetic auxin 2, 4 – D (2, 4 – dichlorophenoxy division occurs and are synthesized in the meristematic
acetic acid) is a famous herbicide or weedicide regions of the plants. Cytokinins have been implicated
(especially kills broad leaved weeds). They are widely in many plant activities; usually along with some other
used to kill dicotyledonous weeds and do not affect plant hormone such as auxins or ethylene. Among these
mature monocotyledonous plants. It is used to for are: mitosis, chloroplast development, differentiation
preparation of weed free lawns by gardeners. of the shoot meristem, stimulating the development of
12. (a) Gibberellin is very a important plant hormone which lateral buds and therefore branching, differentiation of
was first of all reported in Japan, where a disease in the tissues of the root, leaf formation and leaf
rice called ‘bakane disease’ or foolish seedling senescence.
disease spread, in which rice plants became 20. (a) Ethylene is the only gaseous phytohormone. It is a
abnormally long with significant internodal growth inhibitor and mainly concerned with
elongation. Sawada reported that this ‘bakane maturation and fruit ripening. It is a volatile gas
disease’ is caused by a chemical secreted by the formed by the complete combustion of carbon rich
fungus Gibberella fujikuroi and later it was substances like coal, petroleum etc.
S-82 Biology
21. (d) Ethylene promotes abscission and senescence of Mammoth tobacco Nicotiana tobaccum, Xanthium,
leaves, flowers etc. rice etc.
22. (a) Ethylene is highly effective in inducing fruit ripening 31. (a) Day neutral plants are those plants where day length
when it is produced in large amount which coincides does not influence flowering. These plants flower
with respiratory climacteric i.e., a brief rise to a very in both shorter and longer photoperiod. For
high level of respiration. This rise indicates the example, maize, cotton, tomato etc.
beginning of senescence and death. Ethylene 32. (c) Vernalization is the cooling of seed during
increases the permeability of cell because of which germination in order to accelerate flowering when it
the fruit is softened and entry of oxygen into fruit is is planted.
accelerated. Climacteric can be prevented by 33. (a) Seed dormancy is defined as a state in which seeds
eliminating oxygen from the atmosphere to prevent are prevented from germinating even under
respiration. environmental conditions, normally favourable for
23. (b) Ethephon is the compound used to supply ethylene. germination. These conditions are a complex
It is readily absorbed in aqueous solution and combination of water, light, temperature, gasses,
transported within the plant and it releases ethylene mechanical restrictions, seed coats, and hormone
slowly. Ethephon hastens fruit ripening in tomatoes structures.
and apples. It accelerates abscission of flowers and 34. (d) 6 - furfuryl amino purine, 2, 4 dichlorophenoxy acetic
fruits in cotton, cherry, walnut, etc. acid and indole - 3 acetic acid are the examples of
kinetin, synthetic auxins and natural auxins
24. (d) Abscissic acid (ABA), also known as abscisin II and
respectively.
dormin. It is usually found in vascular plants, some
fungi and some green algae. It acts as an inhibitor Kinetin stimulates cell division in plants. 2, 4
dichlorophenoxy acetic acid is a common systemic
because it opposes the growth of promoting effect
herbicide used in the control of broadleaf weeds.
of auxins, GA and cytokinins, thus keep their activity
Indole - 3 acetic acid induces cell elongation and cell
under control. Leaf abscission, fruit fall and dormancy
division and results in plant growth and
occurs due to abscissic acid.
development.
25. (b) In guard cells, the plant hormone abscisic acid (ABA) 35. (d) All the given statements are the characteristics of
inhibits stomatal opening and induces stomatal growth of an organism. Growth can be defined as an
closure through the coordinated regulation of ion irreversible permanent increase in size of an organ or
transport. its parts or even of an individual cell.
26. (a) ABA counteracts many effects of GA such as 36. (d) Water does not oxidize glucose to provide energy. It
induction of hydrolases and a-amylase in barley maintains the turgidity of growing cells and provides
seedlings. medium for enzymatic reactions.
27. (c) Phototropism is the orientation of a plant or other 37. (a) IAA and IBM are natural auxins but NAA, 2, 4 -D
organisms in response to light, either towards the and 2, 4, 5 T are synthetic auxins.
source of light (positive phototropism) or away from 38. (c) Ethylene promotes senescence and abscission of
it (negative phototropism). plant organs especially of leaves and flowers. It
28. (b) The effect of photoperiods or daily duration of light enhances the respiration rate during fruit ripening.
hours (and dark periods) on the growth and Ethylene is also used to initiate flowering and for
development of plants, especially flowering, is called synchronising fruit-set in pineapples. It also induces
photoperiodism. Photoperiodism was first studied flowering in mango.
by W.W Garner and H.A. Allard (1920) in Mary Land 39. (d) The process of flowering also occurs in day neutral
Mammoth (mutant variety of tobacco). plants but these plants do not require specific
29. (a) The period of daylight appears to initiate flowering photoperiods to flower. Such plants flower in almost
in long-day plants or inhibit flowering in short-day all the photoperiods ranging from few hours to 24
plants. In actual fact, long-day plants will not flower hours of uninterrupted light periods.
if the dark period exceeds a certain maximum and 40. (c) Vernalisation is also seen in biennial plants. Biennials
conversely short-day plants will not flower unless are monocarpic plants that normally flower and die
the dark period exceeds a certain minimum. These in the second season.
periods are termed critical dark periods and must be 41. (b) The growth of the leaf is measured in terms of surface
continuous to have effect. area.
30. (d) Short day plants (or long night plants) are those 42. (c) Statement (i) and (ii) are correct. Gibberellins are not
plants where flowering takes place when they are responsible for immature falling of leaves. Ethylene
exposed to shorter photoperiod i.e day length is less and abscisic acid generally have a negative role and
than the critical length. For examples, Maryland are responsible for immature falling of leaves, rapid
EBD_7209
Hints & Solutions S-83

fading of flowers, inhibiting germination etc. flowering in plants. It is produced in the leaves,
43. (d) Cytokinin delays senescence of leaves and other and acts in the shoot apical meristem of buds
organs by preventing drainage of nutrients, and growing tips.
checking degradation of photosynthetic pigments C : Indole-3-butyric acid (IBA) is a natural plant
proteins and nucleic acids. hormone in the auxin family and is an ingredient
44. (d) in many commercial horticultural plant rooting
45. (d) All the given statements regarding ethephon are products. It is used for stimulating plant growth
correct. Ethephon is a synthetic plant growth and root formation.
regulator that induces flowering and abscission by D : Naphthalene acetic acid (NAA) is a synthetic
promoting the release of ethylene and has been used plant hormone in the auxin family. It is toxic to
to cause early ripening (as of apples on the tree). plants at high concentrations. It induces the
46. (b) When cytokinins are added directly to the abscission formation of more fertile branches in cotton plants.
layer, senescence of the zone is retarded. They delay 52. (a) A : Abscissic acid (ABA), also known as abscisin
the degradation of protein and chlorophyll of the II and dormin. It is usually found in vascular
plant parts and hence delay senescence. As they plants, some fungi and some green algae. It was
act as anti senescent, they act antagonistic to discovered for its role in regulating abscission
ethylene which accelerate senescence. and dormancy. It simulates the closure of
47. (a) NAA (naphthalene acetic acid) and 2, 4-D or 2, stomata in the epidermis and increases the
4-dichlorophenoxyacetic acid are synthetic auxins. tolerance of plants to various kinds of stresses,
All these auxins have been used extensively in therefore it is also called as stress hormone.
agricultural and horticultural practices. They are B : Ethylene is a gaseous plant hormone.
widely used as herbicides. 2, 4-D is widely used to C : Cytokinins are found in abundance in young
kill dicotyledonous weeds, it does not affect mature roots, leaves and young fruits where rapid cell
monocotyledonous plants. It is used to prepare weed- division occurs and are synthesized in the
free lawns by gardeners. meristematic regions of the plants. They are
48. (b) Auxin delays abscission of young leaves and fruits. responsible for mitosis, chloroplast
Its effect is through nonformation of abscission zone development, and differentiation of the shoot
below a leaf or fruit. Abscission zone cuts off meristem, stimulating the development of lateral
nutrients and water supply. However, auxin buds and therefore branching, differentiation of
promotes the abscission of mature or older leaves the tissues of the root, leaf formation and leaf
and fruits. senescence.
49. (c) Ethylene is a simple gaseous PGR. The most widely D : Auxin is a plant hormone which causes the
used compound as source of ethylene is ethephon. elongation of cells in shoots and is involved in
Ethephon in an aqueous solution is readily absorbed regulating plant growth. They are also
and ‘transported within the plant and releases responsible for apical dominance.
ethylene slowly. Ethephon hastens fruit ripening in 53. (c) 54. (a) 55. (c)
tomatoes and apples and accelerates abscission in
56. (c) Phytohormones can be broadly classified into two
flowers and fruits (thinning of cotton, cherry, walnut).
groups based on their functions in a living plant
It promotes female flowers in cucumbers thereby
body - growth promoters and growth inhibitors.
increasing the yield.
Auxin, gibberellin and cytokinin are growth
50. (a) Apical dominance is the phenomenon by which promoters and ethylene and abscissic acid (ABA)
presence of apical bud does not allow the nearby are growth inhibitors.
lateral buds to grow. Apical bud inhibits the growth
Functions of growth promoters - I, II, III, IV, V, VI, VII
of lateral buds by releasing auxins. When the apical
bud is removed, the lateral buds sprout. This Functions of growth inhibitors - VIII, IX
produces dense bushy growth. The phenomenon is 57. (b) Gibberellin helps in cell growth of stem, leaves and
widely used in tea plucking and hedge-making. other aerial parts.
51. (d) A : Zeatin is derived from the purine base called 58. (c) The plant growth regulators (PGRs) are small, simple
adenine. Zeatin belongs to the family of plant-growth molecule of diverse chemical composition. They
hormones called cytokinins and was discovered in could be indole compounds (indole-3-acetic acid,
immature corn kernels from the genus Zea. (IAA, IBA); adenine derivatives (N6-furfurylamino
purine, kinetin) derivatives of carotenoids (abscisic
B : Florigen (or flowering hormone) is the
acid); terpenes (gibberellic acid, GA3) or gases
hypothesized hormone-like molecule
(ethylene, C2H4).
responsible for controlling and/or triggering of
S-84 Biology
59. (d) Ethylene is produced from amino acid methionine. seasonal changes in day length. Depending on the
Kinetin is a modified form of adenine, (purine). length of the photoperiod required for flowering, plants
60. (b) In the given graph of growth verses time, the phases are classified as short day, long day and day neutral
marked as A, B and C are respectively stationary plants. Plant I, II and II are respectively short day plant,
phase, lag phase and log phase. In lag phase, the long day plant and day neutral plant.
growth is slow, the growth increases during log Short day plants (or long night plants) are those plants
phase and becomes steady in the stationary phase. where flowering takes place when they are exposed to
61. (b) In the given figure of germination and seedling shorter photoperiod i.e day length is less than the
development in bean, the structure marked as A, B, critical length. For examples, Maryland Mammoth
C and D are respectively radicle, seed coat, epicotyl tobacco Nicotiana tobaccum, Xanthium, rice etc.
and hypocotyl. Long day plants or short night plants are those plants
Radicle is the part of a plant embryo that develops where flowering takes place when they are exposed to
into the primary root. Seed coat is the protective longer photoperiod i.e more than critical day length.
outer coat of a seed. Epicotyl is the region of an Example - sugar beet, wheat, radish etc. Day neutral
embryo or seedling stem above the cotyledon. plants are those plants where day length does not
Hypocotyl is the part of the stem of an embryo plant influence flowering. These plants flower in both
beneath the stalks of the seed leaves or cotyledons shorter and longer photoperiod. For example, maize,
and directly above the root. cotton, tomato etc.
62. (d) Auxin concentration increases in shaded area, i.e., 68. (d) The given two statements are shown by the cell at
auxins are collected in the opposite side of light. the root apex or shoot apexes.
Increased concentration of auxin is stimulatory for 69. (b) The period of growth is generally divided into three
shoot growth and for this reason, shaded side shows phases: meristematic (cells of apical meristem divide),
more growth than lighted side. Hence, bending of elongation (proteins, protoplasm, cell wall material
shoot takes place towards light. is synthesized) and maturation (secondary walls are
63. (b) The option (b) shows the correct graph of arithmetic laid down).
growth. Arithmetic growth refers to the situation Maximal size in term of wall thickening and
where a population increases by a constant number protoplasmic modification are achieved by cells of
of persons (or other objects) in each period being maturation phase.
analysed. In arithmetic growth, following mitotic cell 70. (a) Arithmetic growth is a type of growth in which the
division, only one daughter cell continues to divide rate of growth is constant and increase in growth
while the other differentiates and matures. On occurs in arithmetic progression i.e., 2, 4, 6, 8 etc.
plotting length of the organ against time, a linear Here after mitosis, only one daughter cell continues
curve is obtained. to divide, other takes part in differentiation and
64. (c) In the given graph of geometric growth rate, (c) is maturation. In this type of growth, a linear curve is
the correct statement. According to this graph, with obtained with positive value.
limited supply of nutrients, the growth rate increases Arithmetic growth is expressed as Lt = L0 + rt. Where,
rapidly leading to an exponential phase. Lt = Length after time t, L0 = Length at the beginning,
65. (c) (i) Absolute growth rate (AGR) is actual growth r = growth rate.
per unit time. 71. (a) In geometric growth, if total growth is plotted against
AGR of leaf A = 10 – 5 = 5 time, an S-shaped or sigmoid curve is obtained. It
AGR of leaf B = 55 – 50 = 5 consists of four parts-lag phase, log phase
(exponential phase) phase of diminishing growth and
(ii) Relative growth rate (RGR) is the growth per
stationary phase (steady growth for organs or
unit time as percentage of initial size.
organisms of indefinite growth).
5 72. (b) Absolute growth rate is defined as the measurement
RGR of leaf A = ´ 100 = 100%
5 and the comparison of total growth per unit time.
5 73. (c) In the expression W1 = W0ert, r is the relative growth
RGR of leaf B = ´ 100 = 10% rate and is also the measure of the ability of the plant
50
66. (a) Short day plants generally require light period of to produce new plant material, referred to as efficiency
less than 12 hours and continuous dark period of index.
a b o u t 74. (b) Relative growth rate is the measure of the ability of
14-1hours for subsequent flowering. the plant to produce new plant material.
67. (b) All the plants (I, II and III) show photoperiodism. Growth = 19 – 5 = 14 cm
Photoperiodism is the response of an organism to Period = 7 days
EBD_7209
Hints & Solutions S-85

Growth rate = 14/7 = 2 cm/day have been used extensively in agricultural and
Relative growth rate = 2/5 × 100 = 40% horticultural practices.
75. (a) Differentiation is a process in which cells undergo 84. (a) Indole compounds (e.g, IAA, IBA) help to prevent
few to major structural changes both in their cell fruit and leaf drop at early stage but promote the
walls and protoplasm. abscission of older mature leaves and fruits. Indole
76. (d) Development includes all changes that an organism is an aromatic heterocyclic organic compound. It has
goes through during its life cycle from germination a bicyclic structure, consisting of a six-membered
of the seed to senescence. The correct sequence of benzene ring fused to a five-membered nitrogen-
the developmental process in a plant cell is: containing pyrrole ring.
Cell division - Differentiation - Maturation - 85. (d) The phenomenon of apical dominance can be seen
Senescence in most of the vascular plants in which, in the
Cell division involves plasmatic growth. presence of apical bud, growth of lateral buds (formed
Differentiation is a process in which cells undergo just below the apex) is suppressed. At the removal
few to major structural changes both in their cell of apical bud, the lateral buds grow vigorously. It
walls and protoplasm. Maturation is the process that shows that apical bud suppresses the growth of
a living thing goes through as it ages and becomes lateral bud (axillary bud) just below it. This is known
ripe or fully developed. Senescence is the stage in as apical dominance. It is widely used in tea
the life history of an individual when the rate of plantation and hedge making.
metabolic activities declines that leads to ageing and 86. (a) Auxins are widely used as herbicides. 2, 4-
then eventually death. Dichlorophe-noxyacetic acid is a common systemic
77. (b) Plant hormones are chemicals that at low herbicide used in the control of broadleaf weeds. It
concentration, influences plant growth and is one of the most widely used herbicides in the world.
differentiation. They are signal molecules produced 2, 4 D, widely used to kill dicotyledonous weeds, do
within the plant, and occur in extremely low not affect mature monocotyledonous plants. It is
concentrations. used to prepare weed free lawns by gardeners.
78. (b) Experiment done by Charles Darwin and his son on 87. (a)
plant phototropism shows that the tip of the 88. (d) Terpenoids, which are derivatives of terpenes,
coleoptile is the light receptor of the plant that caused includes abscisic acid and gibberellin (plant growth
the bending of the entire coleoptile towards the light substances) and the carotenoid and chlorophyll
source. pigments. Gibberellins (GAs) are plant hormones that
regulate growth and influence various developmental
79. (b) Ethylene is a gaseous hormone. It is a natural plant
processes, including stem elongation, germination,
hormone which affects the growth, development,
dormancy, flowering, sex expression, enzyme
ripening, and senescence (aging) of all plants. It is
induction, and leaf and fruit senescence.
normally produced in small quantities by most fruits
and vegetables. Many fruits produce larger 89. (b) Ripening of citrus and apple fruits can be delayed
quantities of ethylene and respond with uniform with the help of gibberellins. The fruits can be left on
ripening when exposed to an external source of the tree longer to extend market period. It is also
ethylene. useful for safe and prolonged storage of fruits.
80. (a) Kinetin (cytokinin) is a modified form of adenine, a 90. (a) Gibberellins (GAs) causes fruits like apple to elongate
purine. and improve in shapes. GA are plant hormones that
regulate growth and influence various developmental
81. (b) IBA (indole butyric acid) is a natural plant hormone
processes, including stem elongation, germination,
in the auxin family and is an ingredient in many
dormancy, flowering, sex expression, enzyme
commercial horticultural plant rooting products. It is
induction, and leaf and fruit senescence. GA is also
used for stimulating plant growth and root formation.
responsible for bolting (internode elongation just
82. (a) IAA (Indole - 3 - acetic acid) is the most abundant prior to flowering).
and the basic auxin natively occurr ing and
91. (b) In conifers, gibberellin (GA) has been found to cause
functioning in plants. IAA is translocated to other
quicker early growth so that maturity is reached early.
parts for growth of the plant. It promotes elongation
It is useful for obtaining early and quicker yield of
of stems and roots.
economically important seeds.
83. (c) Flowering in pineapple is promoted by NAA 92. (c) Germinating barley grains produce more malt when
(Naphthalene acetic acid). NAA is a synthetic plant provided with GA3 probably by forming higher
hormone in the auxin family. NAA and 2, 4, D are quantities of a -amylase and other hydrolytic
synthetic auxin plant hormones. All these auxins enzymes.
S-86 Biology
93. (a) Removal of all yellow leaves and spraying the 102. (d) Breaking of dormancy in leaf buds is brought about
remaining of the leaves with 2, 4, 5 by gibberellin but not by auxin.
trichlorophenoxyacetic acid could be most beneficial to 103. (c) Differentiation of shoot is controlled by high
obtain maximum seed yield from the premature yellowing cytokinin: auxin ratio. The ratio of auxin to cytokinin
of leaves of a pulse crops with decreased yield. plays an important role in the effect of cytokinin on
94. (a) plant growth. Cytokinin alone has no effect on
95. (a) The climacteric in plants is a stage of fruit ripening parenchyma cells. When cultured with auxin but no
associated with increased ethylene production and cytokinin, they grow large but do not divide. When
a rise in cellular respiration. Apples, bananas, melons, cytokinin is added, the cells expand and differentiate.
apricots, tomatoes (among others) are climacteric When cytokinin and auxin are present in equal levels,
fruit. Citrus, grapes, strawberries are non-climacteric the parenchyma cells form an undifferentiated callus.
(they ripen without ethylene and respiration bursts). More cytokinin induces growth of shoot buds, while
96. (b) Abscisic acid is also called stress hormone as it more auxin induces root formation.
increases tolerance of plants to various kinds of
Chapter 16 : Digestion and Absorption
stresses and causes partial closure of stomata under
drought and thus acts as anti transpirant. 1. (b) Premolars are not present in milk dentition. The
97. (a) Abscissic acid (ABA), also known as abscisin II and premolar teeth are transitional teeth located between
dormin, is usually found in vascular plants, some the canine and molar teeth. In humans, there are two
fungi and some green algae. It is a multipurpose plant premolars per quadrant in the permanent set of teeth,
hormone which acts as an inhibitor because it making eight premolars total in the mouth.
opposes the growth of promoting effect of auxins, Milk dentition (also called deciduous teeth or
GA and cytokinins, thus keep their activity under temporary teeth) is the first set of teeth in humans
control. ABA is also known as stress hormone and other diphyodont mammals. They develop
because it is produced under conditions of stress during the embryonic stage of development. They
and causes partial closure of stomata under drought are usually lost and replaced by permanent teeth,
and thus acts as antitranspirants. but in the absence of permanent replacements, they
98. (a) Long day plant or short night plants are those plants can remain functional for many years.
where flowering takes place when they are exposed 2. (c) Thecodont are the teeth embedded in the sockets of
to longer photoperiod i.e. more than critical day the jaw bone. E.g., mammals
length. Examples - spring barley, sugar beet etc. 3. (c) Tongue is a freely movable muscular organ that is
99. (b) Short long day plants require short photoperiod for attached to the floor of the oral cavity by the frenulum.
initiation of flowering and long photoperiod for 4. (b) The tongue functions primarily to move food into
blossoming, eg, Tripholium repens. Long short day position for chewing and swallowing. It also
plants require long photoperiod for initiation of functions in tasting food and facilitating speech.
flowering and short photoperiod for blossoming, eg, 5. (a) A cartilaginous flap called epiglottis prevents the
Bryophyllum, Cestrum. Intermediate plants flower entry of food into the glottis - opening of the wind
within a definite range of photoperiod, eg, wild kidney pipe - during swallowing.
beans. 6. (b) The membranes of the digestive tract are from the
100. (d) Short day plants (or long night plants) are those inside to the outside: mucosa, submucosa, circular
plants where flowering takes place when they are longitudinal muscles and serosa.
exposed to shorter photoperiod i.e day length is less 7. (c) Minerals are naturally occurring inorganic
than the critical length. For examples, Maryland substances with a definite and predictable chemical
Mammoth tobacco (Nicotiana tobaccum), composition and physical properties. They are
Xanthium, rice etc. needed in the diet as a component of teeth and bone;
101. (d) Seed dormancy is defined as a state in which seeds regulators of acid base balance and water balance
are prevented from germinating even under and parts of certain enzymes.
environmental conditions normally favourable for 8. (a) Crypts of Lieberkuhn are tubular invaginations of
germination. These conditions are a complex the epithelium around the villi, lined largely with
combination of water, light, temperature, gasses, younger epithelial cells which are involved primarily
mechanical restrictions, seed coats, and hormone in secretion. They are present in the intestine. It
structures. consists of two secreting cells - Paneth cells (found
Role of microbes in breaking seed dormancy is not a in duodenum) and argentaffin cells.
mechanical method. Microorganisms present in the 9. (d) The opening of hepato-pancreatic duct in the
soil weaken and decompose the hard seed coat. duodenum is guarded by sphincter of Oddi.
EBD_7209
Hints & Solutions S-87

10. (b) Liver does not produce any digestive enzymes. They 22. (b) Mechanically and chemically, stomach transforms a
secrete alkaline bile juice. food bolus into chyme. The bolus of food is carried
11. (c) Salivary glands are exocrine glands, which secrete down the oesophagus by peristalsis, and finally
saliva and an enzyme (called salivary amylase or reaches the stomach. The stomach stores food for 4
ptyalin) that breaks down starch into maltose. In - 5 hours, then the food mixes thoroughly with the
human, there are three pairs of salivary glands - acidic gastric juices of the stomach by the churning
parotid gland (major and the largest among three), movements of its muscular wall and is called chyme.
submandibular gland and sublingual gland. They 23. (c) The disaccharidases are secreted with intestinal
secrete saliva to facilitate mastication and swallowing juice. These disaccharidases help in the digestion of
and to begin the digestion of starches. disaccharides.
12. (c) Brunner’s (duodenal) glands are confined to sub 24. (d) Succus entericus refers to the digestive juice of
mucosa of duodenum and opens into crypts of intestine. It is the alkaline secretion produced by the
Lieberkuhn. It secretes mucous only. intestinal glands in the wall of the duodenum. It
13. (a) Bile is stored temporarily in the gall bladder until it is consists of water, mucoproteins, and hydrogen
needed by the small intestine to emulsify fats. carbonate ions. It helps to neutralize the highly acidic
14. (a) Bile is stored temporarily in gall bladder until it is and proteolytic chyme which enters the small
needed by the small intestine to emulsify fats. After intestine from the stomach, and thus protects the
it leaves the gall blader, it enters into the duodenum. duodenum from damage.
15. (b) Liver is the largest digestive gland. Hepatic lobules 25. (c) The enzyme trypsin is secreted by pancreas.
are the structural and functional unit of liver. Liver is Trypsinogen is present in an inactive form in the
basically an organ of homeostasis. Which controls pancreatic juice. The enzyme Enterokinin
many metabolic activities essential for maintaining (enteropeptidase) secreted by the intestinal mucosa
constant blood composition. – activates trypsinogen into trypsin.
Kupffer cells (present in the lining of blood spaces 26. (a) The chylomicrons are formed inside enterocytes and
of liver) are phagocytic and involved in the are absorbed in lacteals.
breakdown of old red blood cells and ingestion of 27. (a) Rennin plays an important role in the coagulation of
potentially harmful bacteria. Glissons capsule is a milk.
thin layer of dense connective tissue which surrounds 28. (d) Pancreatic juice contains variety of enzymes that help
the liver lobule enclosing branches of portal vein, in the digestion of protein, fats and carbohydrates.
the hepatic artery, the bile duct and lymphatic 29. (d) Oxyntic cells are found on the inner wall of gastric
capsule. Crypts of Lieberkuhn (present in the glands and secrete HCl.
intestine) are tubular invagination of the epithelium 30. (b) Trypsinogen is present in an inactive form in the
around the villi, lined largely with younger epithelial pancreatic juice. The enzyme enterokinin
cells which are involved primarily in secretion. (enteropeptidase) secreted by the intestinal mucosa
16. (b) Bile is drained from the liver by a bile duct which is activates trypsinogen into trypsin.
formed by the joining of a cystic duct from the gall 31. (d) Chymotrypsin is a digestive enzyme which uses an
bladder and a common hepatic duct from different active serine residue to perform hydrolysis on the C-
liver lobes. terminus of the aromatic amino acids of other
17. (c) Saliva contains an enzyme called salivary amylase proteins. It is a protease enzyme that cleaves on the
that helps to digest the starch (carbohydrate) present C-terminal phenylalanine (F), tryptophan (W), and
in the food partially. tyrosine (Y) on peptide chains. It shows specificity
18. (c) The digestive enzymes sucrase, lactase, and maltase for aromatic amino acids because of its hydrophobic
are involved in the breakdown of disaccharides into pocket.
the monosaccharides glucose, galactose and 32. (d) Most of the fat digestion occurs in the small intestine.
fructose. 33. (c) Peristalsis is the wave like movement caused by the
19. (a) Another name for salivary amylase is ptyalin. This alternate contraction and relaxation of the muscles
enzyme is secreted by salivary gland. of the alimentary canal. It is found in the stomach,
20. (a) Saliva is a complex mixture and a hypotonic solution small intestine and large intestine.
that is continuously secreted from salivary gland. 34. (b) The enzyme Enterokinin (enteropeptidase) secreted
pH of saliva is 6.5. by the intestinal mucosa activates trypsinogen into
21. (a) Maltase is a digestive enzyme, produced by the trypsin.
intestine (crypts of Lieberkuhn). It acts on maltose, 35. (a) HCl does not directly help in digestion. HCl of gastric
maltotriose and alpha dextrin and convert them into juice inactivates ptyalin and activates pepsin. It
glucose at a pH greater than 7. converts proenzyme prorennin and pepsinogen into
active rennin and pepsin respectively by lowering
S-88 Biology
the pH of stomach between 1.5 - 2.5. HCl acidifies give two fatty acids and one glycerol. Lipase acts in
the food and stops the action of ptyalin. this conversion.
36. (b) Rennin, also known as chymosin, is a proteolytic 45. (b) Semi digested food in stomach or intestine is called
enzyme and synthesized by chief cells in the stomach. chyme. Chyle is a term for lymph having fat particles.
Its role in digestion is to curdle or coagulate milk in 46. (d) Enterokinase is present in the intestinal juice.
the stomach. It acts on milk protein and changes 47. (a) Emulsification is the process of break down of large
casein into paracasein. If milk were not coagulated, fat globules into smaller globules so that the
it would rapidly flow through the stomach and miss pancreatic enzymes can easily act on them.
the opportunity for initial digestion of its proteins. 48. (d) Protection of the walls of the stomach against the
37. (c) Chief (zymogen or peptic) cells secrete inactive action of its own digestive juice results from the
enzymes like pepsinogen, prorennin and lipase. neutralizing, buffering and coating mucus covering
38. (a) Protein digestion takes place in two different phases: its inner surface.
in the stomach and in the small intestine. HCl 49. (d) The pancreatic lipase (present in the pancreatic juice)
(hydrochloric acid) and pepsinogen secreted by and the intestinal lipase (present in the intestinal
stomach interact to create pepsin, an enzyme that juice) hydrolyse the fat molecules into triglycerides,
plays a very important role in protein digestion. diglycerides, monoglycerides, and ultimately into
Pepsin is one of three principal protein-degrading, glycerol.
or proteolytic, enzymes in the digestive system, the 50. (b) Active transport is the absorption of nutrients from
other two being chymotrypsin and trypsin. Pepsin lower concentrated region to higher concentrated
is an enzyme whose zymogen (pepsinogen) is region (i.e. against concentration gradient). It
released by the chief cells in the stomach and that requires energy.
degrades food proteins into peptides. 51. (c) The stomach stores food (and performs some
39. (b) Trypsin, produced in the pancreas as the inactive digestion too) before passing it on to the intestine.
protease trypsinogen, cleaves proteins into The small intestine (midgut) finishes the digestion and
peptones. Trypsin cleaves peptide chains mainly at carries out most of the nutrient absorption, while the
the carboxyl side of the amino acids lysine or arginine, large intestine (hindgut) reabsorbs water and ions.
except when either is followed by proline. It is used 52. (d) Lacteals is a lymphatic vessel in a villus of the small
for numerous biotechnological processes. intestine which conveys chyle from the intestine
40. (a) Most of the chemical digestion of food in humans is through the mesenteric glands to the thoracic duct.
completed in the small intestine. Small intestine is The main function of the lacteals in the villi of small
the place where most of the digestive enzymes are intestine is the absorption of fatty acids and glycerol.
secreted by the pancreas, and enter the small 53. (c) Insulin is secreted by the Islets of Langerhans in the
intestine via the pancreatic duct. The three major pancreas when the blood sugar level is high. This
classes of nutrients that undergo digestion are increases the rate of glucose uptake from the blood
proteins, lipids and carbohydrates. into the muscle cells.
41. (b) End product of protein digestion is amino acid. 54. (d) Jaundice is a yellowish pigmentation of the skin, the
Amino acids are organic acids that combine to form conjunctival membranes over the sclerae (whites of
proteins. Both amino acids and proteins are the the eyes), and other mucous membranes caused by
building blocks of life. When proteins are digested high blood bilirubin levels. Jaundice is often seen in
or broken down, amino acids are left. The human liver disease such as hepatitis or liver cancer.
body uses amino acids to make proteins to help the 55. (b) Diarrhoea is a diseased condition in which a person
body, like - break down of food, growth, repair of passes out watery stools frequently. It reduces the
body tissue and many other body functions. absorption of food while constipation is infrequent
42. (c) Pancreatic juice contains a variety of inactive elimination of dry stool. It is due to decreased
enzymes such as trypsinogen, chymotrypsinogen, peristalsis in colon.
and carboxypeptidases. 56. (a) The nutrients are absorbed from the small intestine
43. (b) Glucose is stored in the liver as glycogen. It can be into the blood and move through the circulatory
converted to free glucose by the process of system to the body cells.
glycogenolysis, which involves the activation of a 57. (d) Chylomicron is a small fat globule composed of
phosphorylase enzyme by the hormone glucagon. protein and lipid (fat/glycerol). They are found in
Glucagon is made by the pancreas and is released blood and lymphatic fluid where they serve to
when the blood sugar levels fall. transport fat from its point of entry in the intestine to
44. (b) Diglyceride is a glyceride consisting of two fatty liver and to adipose (fat) tissue. After a fatty meal,
acid chain covalently bonded to a glycerol molecule the blood is so full of chylomicrons that it looks
through ester linkages. On digestion, Diglycerides milky.
EBD_7209
Hints & Solutions S-89

58. (c) The enzyme Enterokinin (enteropeptidase) secreted of stomach. The activated pepsin converts proteins
by the intestinal mucosa activates proteolytic into proteases and peptides.
enzymes of succus entericus. 77. (d) Small intestine is the major area of absorption of
59. (d) Serosa, muscularis, submucosa and mucosa are the nutrients. Approximately 80% of absorption take
walls of alimentary canal. Serosa is the outermost place here. Glucose, fructose, fatty acids, amino acids
layer and is made up of mesothelium. Mesothelium (glycine etc.) are absorbed through mucosa into
or serosa lies in close proximity of muscularis layer. blood and lymph by active absorption.
60. (a) Pepsin is a protein- digesting enzyme. The activated 78. (c) In small intestine, carbohydrate-digestion is resumed.
pepsin converts proteins into proteases and The food gets mixed with pancreatic juice. It contains
peptides. the pancreatic amylase (amylopsin) that hydrolyses
61. (a) Amylase, renin and trypsin are all proteins. the polysaccharides into disaccharides.
62. (d) Bile helps in emulsification of fats. If bile duct gets 79. (c) Saliva secreted by the salivary glands contains a
choked, little digestion of fats will occur. digestive enzyme called salivary amylase. This
63. (c) About 30% of starch is hydrolysed by salivary enzyme breaks down starch into sugar at pH 6.8.
amylase into a disaccharide - maltose. Oxyntic or Starch ® maltose + Isomaltose + Limit dextrins
parietal cells, present in the gastric glands of stomach, 80. (b) Mouth releases saliva (secreted from salivary glands)
secrete HCl and intrinsic factor (factor essential of containing salivary amylase enzyme that break down
absorption of vitamin B12). Chylomicrons are small carbohydrates. Stomach is the most distensible and
protein coated fat globules, transported into the widest organ of the alimentary canal. It is an acidic
lymph vessels (lacteals) in the villi. environment where digestion of protein takes place
64. (d) Milk protein can be digested by pepsin and (by breaking proteins into larger polypeptides). Small
chymotrypsin. intestine carries out most of the digestion and
65. (c) Saliva secreted from salivary glands provides the absorption of nutrients. Large intestine reclaims water
chemical digestion, which helps to break down foods and salts.
in the mouth. 81. (d) The correct substrate for lactase is lactose and for
66. (c) Digestive system helps in breakdown of larger food lipase is fats.
particles into smaller ones for easy digestion. 82. (a) Amylase (secreted by salivary gland), trypsin and
67. (c) Statement (i), (iv) and (v) are correct regarding the lipase (secreted by exocrine part, acini of pancreas),
human digestive system. In the mouth, salivary and nucleases (secreted by intestine) acts on starch,
glands are present and secrete salivary juices. Food protein, fats and nucleic acid respectively.
is digested completely in the small intestine. 83. (b) The end products of fat digestion are fatty acids and
68. (b) Statement (i) and (iii) are correct regarding jaundice. glycerol.
84. (c) Ingestion means intake of food. Mechanical
Jaundice may be caused by the retarded function of
digestion comprises of mastication or chewing,
the liver. Jaundice is a yellowish pigmentation of the
liquefaction of food by digestive juices, swallowing
skin, the conjunctival membranes over the sclerae
and peristalsis. Chemical digestion includes the
(whites of the eyes), and other mucous membranes
enzymatic action on foods. Defecation is the
caused by high blood bilirubin levels. Jaundice is
elimination of faecal matter which is formed by
often seen in liver disease such as hepatitis or liver
undigested food along with water and excess of
cancer
digestive enzymes.
69. (b) Fatty acids are absorbed into lymph vessels at the
85. (d) Carbohydrate is mostly ingested in the form of starch
small intestine.
and its end product in the body is glucose which is
70. (c) Sucrase hydrolyses sucrose to glucose and fructose.
absorbed directly in the blood. Proteins are the main
71. (b) HCl, secreted from oxyntic cells, dissolves the bits building blocks of our body. It is made of long chain
of food and creates an acidic medium so that of amino acids. Nucleic acids occur in all body cells
inactivated pepsinogen gets converted into active and transmit coded information of all morphological
pepsin. and functional hereditary characteristics from one
72. (a) Chewing or mastication is a type of mechanical generation to the next. The molecules of nucleic acids
digestion. It is the most important process of are long chain polymers of nucleotide monomers. They
digestion in animals. It helps in enzyme action. are of two types - DNA and RNA. Each molecule is
73. (a) 74. (a) itself a complex molecule formed by linking of a
75. (c) Invertase is the enzyme that catalyses the hydrolysis nitrogenous base, pentose and a phosphate group.
of sucrose into glucose and fructose. 86. (d) Maltase is a brush border enzyme that acts on
76. (a) Pepsin is a protein- digesting enzyme, secreted in its disaccharides. Salivary amylase begins carbohydrate
inactive form (called pepsinogen) from gastric glands digestion in the mouth. Trypsin digests proteins in
S-90 Biology
the small intestine and bile emulsifies fats for helps break down fats in food, is produced by the liver. It
digestion. is stored and concentrated in the gallbladder until it is
87. (d) In the given fate of carbohydrate digestion in the needed to help digest food.
alimentary canal, the stages marked by A, B, C and D 95. (d) The given structure marked as Y is oesophagus.
are the enzymes which act on their respective food Oesophagus is a narrow, muscular thin tube that
types to convert them into simple molecules for pierces the diaphragm and enters the abdominal
digestion. A, B, C and D are respectively amylase, cavity. It connects the mouth to the stomach and
lactase, maltase and invertase. has no function in chemical digestion.
Starch Maltose 96. (a) Pancreatic juice contains variety of inactive enzymes
Lactose Glucose + Galactose like trypsinogen, chymotrypsinogen, and
Maltose Glucose + Glucose carboxypeptidases.
Sucrose Fructose + Glucose 97. (c) The level of glucose will not be affected because
88. (a) The label I represents liver. Liver releases bile which glucose level can be maintained by the insulin
is then stored in the gall bladder. hormone secreted from pancreas and hormones do
89. (c) The structure I represents lacteal. The function of not pass through duct.
lacteal is to absorb and transport fatty acids and 98. (a) Alimentary canal is a long continuous tube having
glycerol in the blood streams through thoracic duct. muscular wall and glandular epithelium extending
90. (c) The marked structure (X) is stomach. Stomach is the from mouth to anus.Arranged sequentially, it includes
most distensible and widest organ of the alimentary the following parts: buccopharyngeal cavity ®
canal which is differentiated into four parts - cardiac oesophagus ® stomach ® small intestine ® large
fundus, body and pyloric part. It acts as a food intestine.
reservoir and helps in mechanical churning and 99. (b) Mixing of food with pancreatic juice and bile occurs
chemical digestion of food. Caecum is a small blind in duodenum. Pancreatic juice and bile are released
sac which hosts some symbiotic microorganisms. through the hepato-pancreatic duct. These mixing
The undigested, unabsorbed substances enter into of food occur due to the movements generated by
the caecum of large intestine through ileo-ceacal the muscula layer of the small intestine.
valve which prevents the back flow of the faecal 100. (b) The walls of the alimentary canal are made up of four
matter. layers namely serosa muscularis, sub mucosa and
91. (d) The given figure shows the layers present in the mucosa. Serosa is the outermost layer while mucosa
wall of alimentary canal from oesophagus to rectum. being the innermost lining of the lumen of alimentary
Part 4 is mucosa. Mucosa is the innermost layer lining canal.
the lumen of alimentary canal. Mucosa forms gastric 101. (d) Goblet cells, found in the intestinal mucosal
glands in the stomach and crypts in between the epithelium, secretes mucus. The mucus lubricates
bases of villi in the intestine (crypts of Lieberkuhn). the food for its easy passage. So, if for some reason,
This layer also forms irregular folds (rugae) in the goblet cells become non-functional, it will adversely
stomach and small finger like foldings called villi in affect the smooth movement of food down the
the small intestine. intestine.
92. (a) Structure 1 is serosa. Serosa is the outermost layer 102. (b) Butter is a fat and digestion of fat begins with gastric
in the wall of alimentary canal from oesophagus to juice. Gastric juice contains a very small amount of
rectum. It is made up of thin mesothelium (epithelium gastric lipase.
of visceral organs) with some connective tissues. 103. (a) Bile is stored temporarily in the gall bladder until it is
93. (c) Hepato – pancreatic duct (marked as 3) is guarded needed by the small intestine to emulsify fats.
by sphincter of Oddi. The sphincter of Oddi (or Removal of gall bladder would lead to impairment of
Hepatopancreatic sphincter) is a muscular valve that digestion of fats.
controls the flow of digestive juices (bile and 104. (b) During prolonged fasting, first carbohydrate are used
pancreatic juice) through the ampulla of vater into up, followed by fat and proteins towards the end.
the second part of the duodenum. 105. (a) Physiological value of carbohydrates is 4.0 kcal/g,
94. (d) Ducts [marked as 1(duct from gall bladder) and 4 ( proteins 4.0 kcal/g and of fats is 9.0 kcal/g. Hence,
duct from liver)] are responsible for the formation of 5 g raw sugar will yield
common bile duct (marked as 2)].The common bile duct 5 × 4.0 = 20.0 kcal
is a small, tube-like structure formed where the common 4 g albumin (protein) will yield
hepatic duct (duct from liver) and the cystic duct (duct 4 × 4.0 = 16.0 kcal
from gall bladder) join. Its function is to carry bile from 10 + 2 g of fat will yield
the gallbladder and conduct it into the upper part of the 12 × 9.0 = 108.0 kcal
small intestine (the duodenum). The common bile duct is Total yield = 144 kcal.
part of the biliary system. Bile, a yellow-brown fluid which
EBD_7209
Hints & Solutions S-91

106. (b) For the activation of pepsinogen into pepsin, acidic stomach stores food for 4 – 5 hours, then the food
medium is required. So, if pH of stomach is made 7, mixes thoroughly with the acidic gastric juices of the
digestion of protein would be most affected. stomach by the churning movements of its muscular
Hydrochloric acid provides an acidic medium for wall and is called chyme.
pepsinogen. 116. (a) The gastric gland secretes hydrochloric acid, mucus
107. (b) The enzyme, salivary amylase breaks down starch and pepsin. The hydrochloric acid activates enzyme
into sugar at pH 6.8. pepsin that helps in digestion of protein, while mucus
108. (c) Saliva, secreted by the salivary glands contains a protects the lining of stomach from the action of
digestive enzyme called salivary amylase. This HCl.
enzyme breaks down starch into sugar at pH 6.8. 117. (a) Digestion of food begins in mouth. Both chemical
About 30% of starch is digested by amylase.Salivary and physical digestion takes place in mouth. The
amylase continues to act in the oesophagus, but its teeth cut the food into small pieces, chew and grind
action stops in the stomach as the contents become
it. So, teeth help in physical digestion.
acidic. Hence, carbohydrate-digestion stops in the
Salivary gland produces saliva that contains amylase
stomach.
enzyme. The amylase enzyme digests the starch
109. (b) The dental formula expresses the arrangement of
present in food into sugar, thereby helping in
teeth in each half of the upper jaw and the lower jaw.
chemical digestion.
110. (a) The correct chronological order for the enzyme
activity of some enzymes taking part in protein 118. (b) The small intestine is the largest, narrow and tubular
digestion is pepsin trypsin peptidase. part of alimentary canal. It is divided into three parts
Protein digestion takes place in two different phases: - proximal duodenum, middle jejunum and distal ileum.
in the stomach and in the small intestine. Two of the Large intestine is divided into caecum, colon and
substances secreted by the stomach , HCl rectum. The correct arrangement of the parts through
(hydrochloric acid) and pepsinogen, interact to create which food passes from the small intestine to the
pepsin, an enzyme that plays a very important role in anus is:
protein digestion. Pepsin is one of three principal Duodenum ® Jejunum ® Ileum ® Caecum ® Colon
protein-degrading, or proteolytic, enzymes in the ® Rectum
digestive system, the other two being chymotrypsin 119. (b) The absorption of water in large intestine is affected
and trypsin. Pepsin is an enzyme whose zymogen when a person is suffering from diahorrea. Diarrhoea
(pepsinogen) is released by the chief cells in the is a condition in which an individual losses excessive
stomach and that degrades food proteins into amount of water and salts from the body.
peptides. 120. (a) Deglutition (swallowing) is a succession of muscular
111. (d) Parietal cells secrete hydrochloric acid. HCl makes contractions from above downward or from the front
the medium acidic, which is required for activation backward. It propels food into the pharynx and then
of protein digesting enzyme called pepsin. into the oesophagus. Peristalsis is the involuntary
112. (d) Chemical process of digestion started in the oral constriction and relaxation of the muscles of the intestine
cavity by the hydrolytic action of the carbohydrate or another canal, creating wave-like movements which
(potato contains starch) splitting enzyme, the pushes the contents of the canal forward. Vomiting is the
salivary amylase. Carbohydrates in the chyme are ejection of stomach contents through the mouth. A
hydrolysed by pancreatic amylase into disaccharides. feeling of nausea precedes vomiting.
113. (c) Due to a long hunger strike and survival on water, a 121. (a) During swallowing the epiglottis covers the opening
person will have less urea in his urine because urea to the airways to prevent the entry of food or liquid,
comes to kidney as a waste product from liver which therefore if the epiglottis does not function property
is formed after the breakdown of protein, fat,
then one might get congestion.
carbohydrate during hunger.
122. (d) The mucus produced by the stomach prevents
114. (d) Parietal cells secrete hydrochloric acid (HCl). HCl
damage that might be caused by exposure to acids,
makes the medium acidic, which is required for
food and water dilute gastric juices, the stomach
activation of protein digesting enzyme called pepsin.
lining is quick to repair itself, and the release of gastric
If parietal cells become partially non-functional,
juices is controlled to avoid too high a concentration.
proteins will not be adequately hydrolysed by pepsin
Therefore all the options serve to protect the stomach
into proteases and peptones.
from damaging itself with its acidic contents.
115. (a) When food enters from oesophagus to stomach,
123. (b) The abnormal frequency of bowel movement and
food mixes with juices and protein digestion starts.
increased liquidity of the faecal discharge is known
The bolus of food is carried down the oesophagus
as diarrhoea. It reduces the absorption of food.
by peristalsis, and finally reaches the stomach. The
S-92 Biology
Chapter 17 : Breathing and Exchange of Gases 13. (c) Total lung capacity (TLC) is the total volume of air in
the lungs after a maximum inspiration.
1. (b) Earthworms do not have a special respiratory organ.
(RV + ERV + TV + IRV or VC + RV).
Exchange of gases takes place through their skin. In
them 14. (c) Residual volume (RV) is the volume of air remaining
in lungs even after a forcible expiration. It is about
2 (d) Epiglottis is a stiff flap like structure covering the 1100-1200 ml.
glottis. It acts as a trapdoor to the trachea and
15. (d) Tidal volume (TV) is the volume of air inspired or
prevents the entry of food going down to lungs.
3. (d) Lungs are conical in shape that take up most of the expired during a normal breathing.

chest and thoracic cavity. 16. (d) Anaerobic respiration uses electron acceptors other
than oxygen. Although oxygen is not used as the
4. (a) Though larynx is a part of air passage connecting final electron acceptor, the process still uses a
the pharynx with trachea it does not contribute to respiratory electron transport chain. It is a respiration
the breathing movements. A cartilaginous larynx without oxygen and produces energy, carbon dioxide,
(sound box or voice box) helps in sound production. lactic acid or alcohol.
5. (b) Intercostal muscle is a set of muscles attached to the 17. (a) Every 100 ml of oxygenated blood can deliver around
ribs within the chest cavity that control the movement 5 ml of O2 to the tissues under normal physiological
of the rib cage. conditions.
6. (d) Alveolar epithelium and endothelium of blood
18. (b) Pressure contributed by individual gas in a mixture
capillaries are two membranes which separate air in
of gases is called partial pressure and is represented
pulmonary alveoli from blood capillaries.
as pO2 for oxygen and pCO2 for carbon dioxide.
7. (a) The sticky mucus lining the nasal chamber moistens
On the basis of the table given below it can be
the air and filters dust particles.
concluded that pO2 is higher and pCO2 is lesser in
8. (d) Bronchiole is the smallest and thinnest tube in the the atmospheric air as compared to that of alveolar
lungs. It is the tiny branch of air tubes within the
air.
lungs that is a continuation of the bronchus. The
bronchioles, which connect to the alveoli (air sacs), Table: partial pressure of oxygen and carbon dioxide
are divided into 3 types: lobular bronchioles, terminal at different parts involved in diffusion in comparison
bronchioles, and respiratory bronchioles. The to those in the atmosphere.
terminal bronchioles are the final airways in the
Respiratory Atmospheric Blood Blood
conducting zone, with the respiratory bronchioles gas air
Alveoli
(deoxygenat ed) (Oxygenat ed)
T issues
beginning the respiratory zone. The primary function
O2 159 104 40 95 40
of the bronchioles is to conduct air from the bronchi
CO 2 0.3 40 45 40 45
to the alveoli, and to control the amount of air
distributed through the lung by constricting and 19. (c) Haemoglobin has 4 subunits, each of which binds to
dilating. 1 molecule of O2 for a total of 4 molecules of O2
9. (a) Trachea is the path that leads from the throat to the bound to 1 haemoglobin molecule.
lungs. The trachea, also called windpipe, is a tube 20. (b) About 98% of O2 is transported by RBC. O2 binds
that connects the pharynx and larynx to the lungs, with haemoglobin (red coloured iron containing
allowing the passage of air. The trachea extends from pigment present in th e RBCs) to form
the larynx and branches into the two primary bronchi. oxyhaemoglobin. This process is known as
At the top of the trachea the cricoid cartilage attaches
it to the larynx. This is the only complete ring, the oxygenation.
others are incomplete rings of supporting cartilage. 21. (b) Partial pressure of oxygen in inspired and expired air
10. (d) Functional residual capacity (FRC) is the volume of is 158 and 40 mm of Hg.
air remaining in the lungs after a normal expiration 22. (b) p50 value is the oxygen tension at half saturation
(ERV + RV). It is about 2300 ml. (50%) of blood. It is calculated from the measured
11. (c) Expiratory capacity (EC) is the volume of air expired oxygen tension and oxygen satur ation by
after a normal inspiration (TV + ERV). extrapolation along the oxygen dissociation curve
12. (b) Functional residual capacity is the volume of air to 50% saturation.
remaining in the lungs after a normal expiration (ERV 23. (a) Most oxygen is carried by haemoglobin and most
+ RV). It is about 2100-2300 ml. carbon dioxide is carried as bicarbonate ions.
EBD_7209
Hints & Solutions S-93

24. (b) CO makes the most stable combination with the 38. (b) Spirometer is an apparatus for measuring the volume
haemoglobin of RBCs. Haemoglobin has much more of air inspired and expired by the lungs. It measures
affinity (about 250 times) for CO than the oxygen. In ventilation, the movement of air into and out of the
the presence of CO, it readily combines to form a lungs. There are various types of spirometers which
stable compound called carbonmonoxyhaemoglobin use a number of different methods for measurement
(COHb). (pressure, transducers, ultrasonic, water gauge).
25. (d) CO2 is transported both by plasma and haemoglobin 39. (a) Diaphragm receives electrical messages from the
of blood. brain for breathing in and out. Diaphragm is a dome-
26. (b) An increase (i) in CO2 in blood, which causes a drop (ii) shaped muscular partition which separates the thorax
in pH, would cause your breathing to speed up. from the abdomen in mammals. It plays a major role
27. (c) Carbonic anhydrase is found in high concentration in breathing, as its contraction increases the volume
in erythrocytes (RBCs). Carbonic anhydrase is a zinc of the thorax and so inflates the lungs.
enzyme that speeds up the formation of carbonic 40. (d) Trachea is a straight tube which extends up to the
acid. mid-thoracic cavity. The trachea divides at the level
28. (b) The higher concentration of pCO2 stimulates of 5th thoracic vertebra into right and left primary
dissociation of oxyhaemoglobin. bronchi. Each bronchus undergoes repeated divisions
29. (b) Alveoli are the exchange surfaces in mammalian to form secondary and tertiary bronchi and bronchioles.
lungs. All the other structures listed are dead space They finally end up in very thin terminal bronchioles.
used to transport air to and from the alveoli. The tracheae, bronchi and the initial bronchioles are
30. (a) Residual air is the amount of air that remains in the supported by incomplete cartilaginous rings.
lungs after a maximal expiration. This air mostly 41. (a) Thoracic chamber is formed dorsally by the vertebral
occurs in alveoli (the main site of gas exchange). column, ventrally by the sternum, laterally by the
31. (c) Every 100 ml of deoxygenated blood delivers ribs and on the lower side by the dome shaped
approximately 4ml of CO2 to the alveoli. diaphragm. Thoracic cavity, also called chest cavity, is
32. (b) Asthma is a disease caused due to an allergic reaction the second largest hollow space of the body. It is
to foreign substances that affect the respiratory tract. enclosed by the ribs, the vertebral column, and the
sternum, or breastbone, and is separated from the
In this, the airways and lungs of a person can become
abdominal cavity (the body's largest hollow space) by
obstructed because they become narrow and cut off
a muscular and membranous partition, the diaphragm.
air flow. Bronchioles can constrict (narrow) because
42. (a) Trachea is a membranous tube supported by “C”
of muscle spasms.
shaped hyaline cartilage ring. The cartilage ring
33. (c) Emphysema is a chronic, irreversible disease of the
protects the trachea from collapse and injury.
lungs. It is characterized by abnormal enlargement of
air spaces in the lungs accompanied by destruction 43. (d) Oxyhaemoglobin is the bright red product which is
of the tissue which lines the walls of the air spaces. It formed when oxygen from the lungs combines with
is one of several diseases known collectively as haemoglobin in the blood. It travels through the
Chronic Obstructive Pulmonary disease (COPD). bloodstream to cells, where it breaks down to form
Smoking is the leading cause of emphysema. haemoglobin and oxygen, and the oxygen then
34. (a) Respiratory centers are the centers in the medulla passes into cells. Factors like high pO2, low pCO2,
oblongata and pons varolii that collects sensory lesser H+ concentration, and lower temperature
favours for the formation of oxyhaemoglobin.
information about the level of O2 and CO2 in the
blood and determines the signals to be sent to the 44. (b) Alveolar ventilation is more than pulmonary
respiratory muscles. ventilation.
35. (d) Respiratory centers that regulate respiration are 45. (b) Rise in p50 with decrease in pH is related to Bohr
located in medulla oblongata. Respiratory centre effect. Bohr effect is a physiological phenomenon
regulates normal breathing and also automatically first described in 1904 by the Danish physiologist
adjusts the breathing rate to varying requirements Christian Bohr. This effect states that haemoglobin's
of body during stress conditions. oxygen binding affinity is inversely related both to
36. (d) Pneumotaxic center controls medullary respiratory acidity and to the concentration of carbon dioxide.
centres, particularly inspiratory centre, so that duration 46. (a) All animals require a medium for cellular respiration.
of inspiration is controlled. It is located in pons varolii Cellular respiration is the process of oxidizing food
of brain. molecules, like glucose, to carbon dioxide and water.
37. (b) Inhaled air has not yet exchanged gases with lung The energy released is trapped in the form of ATP
capillaries, therefore it contains the highest for use by all the energy-consuming activities of the
concentration of oxygen. cell.
S-94 Biology
47. (b) About 97% of O2 is transported by RBC. O2 binds carbon dioxide and water, facilitating the transfer of
with haemoglobin (red coloured iron containing carbon dioxide from tissues to blood and from blood
pigment present in th e RBCs) to form to alveolar air.
oxyhaemoglobin. 57. (b) Binding of oxygen with haemoglobin is primarily
48. (b) Haemoglobin is an iron-containing protein in red related to partial pressure of O2.
blood cells that is responsible for transporting 58. (b) Oxygen-haemoglobin dissociation cur ve
oxygen to the tissues and removing carbon dioxide
demonstrates a progressive increase in the
from them. Carbonic anhydrase helps in transport of
percentage of haemoglobin bound to oxygen as blood
carbon dioxide.
pO2 increases which is called percent saturation of
49. (a) About 97% of oxygen is transported by RBCs in the haemoglobin. The total quantity of oxygen bound
blood. About 20 – 25% of carbon dioxide is carried
to haemoglobin in normal systemic arterial blood,
by haemoglobin as carbaminohaemoglobin. Cigarette
which is 97 percent saturated, is about 19.4 milliliters
smoking is the leading cause of emphysema. Neural
signals from the pneumotaxic centre in pons region of per 100 milliliters of blood. While passing through
brain can reduce the duration of inspiration and thereby the tissue capillaries, this amout is reduced, on
alter the respiratory rate. average, to 14.4 milliliters (pO2 of 40 mm Hg, 75
percent saturated 14.4 millititers (pO2 of 40 mm Hg,
50. (d) All the given statements are correct.
75 percent saturated haemoglobin). Thus, under
51. (b) Inspiration is the active intake of air from atmosphere
normal condition,s about 5 milliliters of oxygen are
into lungs while expiration (breathing out) is the
transported from the lungs to the tissues by each
passive expelling of air from the lungs.
100 milliliters of blood.
52. (a) Ventilation rate of lungs is the process that mixes
fresh inspired gas with alveolar gas. Increase of CO2 59. (c) Expiratory capacity is the total volume of air a person
content in inhaled air is responsible for increase in can expire after a normal inspiration. This includes
ventilation rate of lungs. If there is no ventilation at tidal volume and expiratory reserve volume (TV +
all, there will be no replenishment of oxygen and no ERV).
removal of CO2. PO2 will fall and PCO2 will rise towards 60. (a) A shift to the right of oxygen dissociation curve
the venous O2 and CO2 tensions. indicates dissociation of oxygen from haemoglobin.
53. (b) When we breathe in, air goes through our nose and In the tissues, low pO2 , high pCO 2 , high H+
mouth, down over windpipe and into our lungs. As a concentration (low pH or acidity) and higher
result, size of our chest increases. temperature favour dissociation of oxygen from the
When we breathe out, carbon dioxide is removed oxyhaemoglobin.
from our lungs. 61. (d) The diaphragm an d a specialised set of
54. (b) Air travels through nose and mouth. The respiratory musclesexternal and internal intercostals between the
system moves air in and out of the body – using ribs, help in generating pressure gradient. Besides
oxygen and eliminating carbon dioxide, a gas this we can increase the strength of inspiration and
produced when cells use oxygen. The respiratory expiration with the help of additional muscles in the
system includes the nose, mouth, trachea, bronchi,
abdomen.
diaphragm and lungs. Air enters the body through
either the open mouth or the nose. It travels down 62. (b)
the trachea to the lungs, where the oxygen in it 63. (c) Mode of respiration in earthworm, human, prawn and
passes into the bloodstream. insects are respectively cutaneous (skin), pulmonary
55. (a) About 20 – 25% of carbon dioxide is carried by (lung), branchial (branchial gills) and tracheal
haemoglobin as carbaminohaemoglobin. About 7% (trachea).
of CO2 is transported dissolved in the plasma of 64. (d) Asthma is a chronic lung disease that aggravates
blood. The oxyhaemoglobin of the erythrocytes is and narrows the airways. It causes recurring periods
acidic. of wheezing (a whistling sound when you breathe),
56. (a) Carbonic anhydrase is a zinc containing enzyme chest tightness, shortness of breath, and coughing.
present in the red blood cells. In erythrocytes (red Bronchitis is the inflammation of bronchi of the lungs
blood cells) the carbon dioxide combines with water and characterised by coughing up mucus, wheezing,
and is transported. This enzyme assists rapid inter- shortness of breath, and chest discomfort. Rhinitis
conversion of carbon dioxide and water into carbonic is the irritation and inflammation of the mucous
acid, protons and bicarbonate ions. This enzyme membrane inside the nose and leads to a stuffy nose,
catalyzes the decomposition of carbonic acid into runny nose, and post-nasal drip. It is usually
EBD_7209
Hints & Solutions S-95

triggered by airborne allergens such as pollen and 71. (a) The label A is correctly matched with its function.
dander. Emphysema is a chronic, irreversible disease Alveolar cavity is the main site of exchange of
of the lungs characterized by abnormal enlargement respiratory gases.
of air spaces in the lungs accompanied by 72. (a) The correct labelling of the structure marked in the
destruction of the tissue lining the walls of the air figure of respiratory system are:
spaces. I – Nose, II – Bronchus, III – Larynx, IV – Diaphragm,
65. (d) Residual Volume (RV), about 1,200 mL, is the volume V – Trachea, VI – Lung.
of air still remaining in the lungs after the expiratory Nose (nasal cavity) is the first part of the respiratory
reserve volume is exhaled. The Vital Capacity (VC), system whose small hair presents in the cavity help
about 4,500 mL, is the total amount of air that can be to filter particles of dust and other foreign matter.
expired after fully inhaling (VC = TV + IRV + ERV = Bronchus is any of the major air passageways of the
approximately 80 percent TLC). The value varies lungs. Larynx (voice box) holds the vocal cords and
according to age and body size. The Inspiratory responsible for producing voice, helping us swallow
Reserve Volume (IRV), about 2,500 mL, is the and breathe. Diaphragm is a dome-shaped muscular
additional air that can be forcibly inhaled after the partition separating the thorax from the abdomen and
inspiration of a normal tidal volume. The Inspiratory plays a major role in breathing, as its contraction
Capacity (IC), about 3,500 mL, is the maximum amount increases the volume of the thorax and so inflates
of air that can be inspired (IC = TV + IRV). the lungs. Trachea is a tube that connects the
66. (b) The small hair present in the cavity help to filter pharynx and larynx to the lungs, allowing the passage
particles of dust and other foreign matter. Epiglottis of air. Lungs are a pair of breathing organs located
is a leaf shaped cartilage which acts as a switch with the chest which remove carbon dioxide from
between the larynx and the oesophagus to permit air and bring oxygen to the blood.
to enter the airway to the lungs and food to pass 73. (d) The label A represents trachea while label B
into the gastrointestinal tract. The epiglottis also represents bronchiole. Tracheal (wind pipe) carriers
protects the body from choking on food that would air between larynx and bronchi and bronchioles are
normally obstruct the airway. Pharynx is a cone- the terminal part of bronchi.
shaped passageway leading from the oral and nasal 74. (a) The given graph shows the oxygen – haemoglobin
cavities in the head to the oesophagus and larynx. dissociation curve. The relationship between the pO2
The pharynx chamber serves both respiratory and and the percent saturation of haemoglobin when
digestive functions. Larynx (voice box) holds the represented on a graph is termed as oxygen –
vocal cords. It is responsible for producing voice, haemoglobin dissociation curve. It is S-shaped
helping us swallow and breathe. Air passes in and because of the behaviour of the Hb in different pO2.
out of the larynx each time the body inhales or 75. (b) The correct labelling are: 1 - Lung, 2 - Diaphragm, 3 -
exhales. Pleural membrane, 4 - Alveoli. Diaphragm is a dome-
67. (c) Trachea (also called windpipe) is a tube that connects shaped muscular partition separating the thorax from
the pharynx and larynx to the lungs, allowing the the abdomen in mammals. It plays a major role in
passage of air, and so is present in all air-breathing breathing, as its contraction increases the volume of
animals with lungs. the thorax and so inflates the lungs.
Alveoli – main site of gaseous exchanges. 76. (d) X – Inspiration, A – raised, B – increased, C –
contracted.
68. (d) In the diagram, label IV represents alveoli. Alveoli
are sac like structure present in lungs. They are the The given figure shows the inspiration stage of
site for exchange of gases. The label I represent breathing. Breathing is the process that moves air in
trachea, label II represents bronchi and label III and out of the lungs. Inspiration is an active process
represents bronchioles. which is due to muscle contraction. During
inspiration, the external intercostal muscles contracts
69. (a) The label X represents epiglottis. Epiglottis is a and the internal intercostal muscle relax. This pulls the
structure (containing elastic cartilage) that prevents ribcage up and down. At the same time the diaphragm
food from entering into trachea. muscle contracts which flattens the diaphragm. Both
70. (b) The label II represents pharynx. Pharynx is the actions increase the volume of thorax.
common passageway where the nasal and oral 77. (c) About 70% of carbon dioxide is transported as
cavities meet. The label I represents nasal cavity, sodium bicarbonate.
label III represents trachea and label IV represents 78. (a) Oxyhaemoglobin is the bright red haemoglobin which
lungs. is a formed by the combination of haemoglobin and
S-96 Biology
oxygen from the lungs. Oxyhaemoglobin transports alveolar walls are found to be damaged.
oxygen to the cells of the body. It dissociates into 93. (c) Number of RBCs per unit volume of blood is likely to
oxygen and deoxyhaemoglobin at low O2 pressure be higher in a person living at high altitudes, because
in tissue. air is less dense. At high altitude composition of air
79. (d) Trachea is a membranous tube supported by “C” remains the same as at sea level, but density of air
shaped cartilage ring. The cartilage ring protects the gradually deceases due to which arterial pO2 is also
trachea from collapse and injury. decreased. While ascending up a mountain one
80. (b) When the relationship between the pO2 and the inspires thin air and get less oxygen.
percent saturation of haemoglobin is represented on 94. (c) Presence of high quantity of carboxyhaemoglobin
a graph, then it is termed as oxygen - haemoglobin content through blood analysis shows that the
dissociation curve. A rise in pCO2, H+ ions (fall in patient has inhaled polluted air contacting usually
pH), temperature and diphosphoglyceric acid raises high content of carbon monoxide. CO makes the most
the p50 value and shifts the oxygen - dissociation stable combination with the haemoglobin of RBCs.
curve to right or vice versa.
Haemoglobin has much more affinity (about 250
81. (c) Inhalation is the active intake of air from atmosphere times) for CO than the oxygen. In the presence of
into lungs. During this process, the diaphragm CO, it readily combines to form a stable compound
contracts and flattens, causing an increase in vertical called carboxyhaemoglobin or
volume i.e. antero-posterior axis. carbonmonoxyhaemoglobin (COHb).
82. (d) During oxygen transport, when O2 tension is low 95. (a) Both Alveoli of lungs and villi of intestine in mammals
and CO 2 tension is high in the tissues, the provide a large surface area. The exchange of gases
oxyhaemoglobin at the tissue level liberates oxygen between the air and blood takes place across the
to the cells. walls of the alveoli. O2 from alveoli passes into
83. (b) Combining of haemoglobin with O2 in lungs can be capillaries and CO2 from other capillaries diffuses
promoted by decreasing O2 concentration in blood. into alveoli for being removed. Alveoli are the organs
84. (b) When carbon dioxide concentration in blood where the actual gaseous exchange occurs.
increases, breathing becomes faster and deeper due 96. (d) Alveoli is a tiny air-filled sacs arranged in clusters in
to decreased concentration of oxygen in tissues. the lungs. The alveoli are located in the respiratory
85. (b) If human blood becomes acidic, oxygen carrying zone of the lungs, at the distal termination of the
capacity of haemoglobin decreases. alveolar ducts and atria. These air sacs are the forming
86. (c) The affinity of the haemoglobin pigment of and termination point of the respiratory tract and
combining with O2 will fall, when the p50 value of provide total surface area of about 100 m2. It is the
the haemoglobin rises. place where exchange of oxygen and carbon dioxide
takes place. Therefore alveoli are an efficient system
87. (b) Yawning is caused due to low oxygen tension in the
of ventilation with residual air.
blood.
97. (a) Maximum amount of oxygen is exchange from the
88. (b) The correct order of the events during inspiration is:
blood in the capillaries – surrounding tissue cells.
III – IV – II – V – I
98. (c) Deep breathing helps expand the lungs and forces
Thoracic volume increases – Pleural pressure better distribution of the air into all sections of the
decreases – Alveolar volume increases – Alveolar lung. When we breathe fully and deeply, the
pressure decreases—Air flows into the lungs. diaphragm moves farther down into the abdomen
89. (b) The correct sequence of stages of respiration is: and our lungs are able to expand more completely
breathing- gaseous transport- tissue respiration- into the chest cavity. This means that more oxygen
Cellular respiration. is taken in and more carbon dioxide is released with
90. (a) Vital capacity (VC) is the volume of air that can each breath. Therefore, after taking a long deep
breathe in after a forced expiration or volume of air breath, we do not respire for some seconds due to
that can breathe out after a forced inspiration (ERV + less CO2 in blood.
TV + IRV). 99. (b) A large proportion of oxygen is left unused in human
91. (c) This includes RV and ERV (2500ml). blood even after its uptake by the body tissue, can
acts as reserve during muscular exercise because
92. (b) Emphysema is one of several diseases known our tissues are able to utilize only 25% of O2 carried
collectively as chronic obstructive pulmonary by arterial blood. Our venous blood is still 75%
disease (COPD) whose leading cause is smoking. If saturated with O2.
a person is suffering from emphysema, his/her
EBD_7209
Hints & Solutions S-97

100. (a) 3. (b)


101. (a) The urge to inhale in humans results from rising pCO2. 4. (d) Neutrophils are a type of leucocyte (WBCs) that can
It is the increase of blood levels of carbon dioxide take all types of stain (acid-basic-Neutral). It is most
(PCO2) that causes the urge to breathe in, not a lack of abundant (60 –70% of total WBCs) and most active
dissolved oxygen (PO2). type of WBCs i.e., they are the most actively
102. (c) Pharynx is the common passage at the back of the phagocytic in nature.
mouth for air and food while larynx is passageway 5. (d) Leucocytes are colourless, nucleated amoeboid cells
for air only. found in blood which are devoid of haemoglobin
103. (b) Air travels through nose and mouth. The respiratory and are capable of coming out of blood capillaries
system moves air in and out of the body by using through the process of diapedesis. Fall of WBC count
oxygen and eliminating carbon dioxide (a gas is called leucopenia, and occurs due to folic acid
produced when cells use oxygen). The respiratory deficiency and AIDS, etc.
system includes the nose, mouth, trachea, bronchi, 6. (d) White blood corpuscles are like soldiers because
diaphragm and lungs. The complete passage through they fight off the body's enemies -- harmful bacteria
which air passes from outside into the lungs are: and disease. White blood cells attack and kill germs
nasal cavity – pharynx – larynx – trachea – bronchi – in the body, and they also carry away dead cells.
bronchioles – alveoli. 7. (d) ABO grouping is the classification of human blood
104. (c) Most of the hydrogencarbonate ions formed in the based on the inherited properties of red blood cells
red blood cells diffuse out of the plasma along a (erythrocytes). It is determined by the presence or
concentration gradient and combine with sodium in absence of the antigens A and B, which are carried
the plasma to form sodium hydrogencarbonate. The on the surface of the red cells. Persons may thus
loss of negatively-charged hydrogencarbonate ions have type A, type B, type O, or type AB blood. The
leaves them with a more positive charge that is A, B, and O blood groups were first identified by
balanced by chloride ions diffusing into the red blood Austrian immunologist Karl Landsteiner in 1901.
cells from the plasma. 8. (a) Blood group ‘O’ is known as the universal donor
105. (a) The external gill, tracheae, and lungs are all examples since, it does not contain antigens.
of gas-exchange systems used by various animals 9. (a) Rh factor is named after the Rhesus monkey.
to exchange both CO2 and O2. One of the special Experiments by Karl Landsteiner and Alexander S.
properties of all respiratory systems is that they Wiener, showed that rabbits, when immunised with
increase surface area for diffusion. rhesus monkey red cells, produce an antibody that
106. (a) Equation given in option a shows the role of carbonic also agglutinates the red blood cells of many humans.
anhydrase enzyme in the transport of carbon dioxide 10. (c) Erythroblastosis foetalis is a type of haemolytic
by blood. In erythrocytes the carbon dioxide disease of new-borns due to ABO blood type
combines with water and is transported. Carbonic incompatibility. This occurs when a mother's blood
anhydrase, a zinc containing enzyme assists rapid type of A, B, or O is not compatible with a blood
inter-conversion of carbon dioxide and water into group of foetus. It develops in a foetus, when the IgG
carbonic acid, protons and bicarbonate ions. This molecules produced by the mother pass through the
enzyme catalyzes the decomposition of carbonic acid placenta. Among these antibodies are some which
into carbon dioxide and water, facilitating the transfer attack the red blood cells in the foetal circulation; the
of carbon dioxide from tissues to blood and from red blood cells are broken down and the foetus can
blood to alveolar air. develop reticulocytosis and anaemia. This foetal
disease ranges from mild to very severe, and foetal
¾¾
® H 2 CO3 ¬¾
CO2 + H 2 O ¬¾
¾ ¾¾
® HCO 3– + H +
¾ death from heart failure (hydrops fetalis) can occur.
carbon dioxide + carbonic acid bicarbonate +
water hydrogen ion
11. (c) Blood clotting (coagulation) is the process by which
blood vessels repair ruptures after injury. Injury repair
Chapter 18 : Body Fluids and Circulation actually begins even before clotting does, through
vascular spasm, or muscular contraction of the vessel
1. (a) walls, which reduces blood loss. Clotting itself is a
2. (d) Lymph is a clear yellowish, slightly alkaline, complex cascade of reactions involving platelets,
coagulable fluid containing white blood cells in a enzyme, and structural proteins. Calcium plays an
liquid resembling blood plasma. It is composed of important role by working together with vitamin K
fluid matrix, plasma, white blood corpuscles or and a protein called fibrinogen in the clotting
leucocytes. cascade.
S-98 Biology
12. (d) Thrombokinase hydrolyses prothrombin to thrombin 25. (c) Bicuspid term is applied to a valve in heart and tooth
in presence of Ca 2+.Thrombin converts soluble surface.
fibrinogen to insoluble fibrin. 26. (b) ‘Bundle of His’ is a typical cardiac muscle fibres,
13. (a) The clotting of blood is to seal up wounds and connecting the atria with ventricle.
prevent loss of blood. Platelets in blood releases 27. (b) Ventricle is one of two large chambers in the heart
clotting chemicals at the site of injury which forms a that collect and expel blood received from an atrium
clot and prevent bleeding. towards the peripheral beds within the body and
14. (c) Fish has closed and single circulatory system. lungs. Ventricles are far more thick walled than atria
Amphibians and reptiles have closed and incomplete and left ventricle is at least three times thicker than
double circulation. Birds and mammals have closed the right one.
and double circulation. 28. (d) Crocodiles, birds and mammals have 4-chambered
15. (c) Open type blood vascular system is a type of heart.
circulatory system in which nutrients and waste are 29. (c) SA (sino - atrial) node is called the natural pacemaker
moved through the body with the help of a fluid of the heart. It controls the heart rate by generating
which flows freely through the body cavity, rather electrical impulses and then sending them through
than being contained in veins. Many invertebrates the heart muscle, causing the heart to contract and
like insects (eg, cockroach) and shellfish have an pump blood throughout the body. It is made up of a
open circulatory system, with the exact composition group of cells (called myocytes) and positioned on
of the circulating fluid varying, depending on the the wall of the right atrium, at the centre of the heart
animal species involved. and near the entrance of the superior vena cava.
16. (d) Incomplete double circulation is found in amphibians These cells contract at a rate of about 70-80 times
and reptiles. The left atrium receives oxygenated per minute, which make up the natural heartbeat.
blood from the gills/lungs/skin and the right atrium 30. (d) Neurogenic heart is found in most crustaceans, some
gets the deoxygenated blood from other body parts. insects and annelids. The hearts of vertebrates and
However, they get mixed up in the single ventricle molluscs are myogenic.
which pumps out mixed blood. 31. (d) Valves of heart maintain unidirectional flow of blood
17. (c) Heart wall is formed of three layers: endocardium, (i.e from the atria to the ventricles and from the
myocardium and pericardium. ventricles to the pulmonary artery or aorta) and
18. (c) Nodal tissues are specialized cardiac musculature opening and closing of valve depends upon the
present in heart wall. This tissue has two patches- pressure on opposite sides. It prevents its
one present in the right upper corner of right atrium regurgitation in the opposite direction. These valves
(SA node) & other is seen in the lower left corner of act like one way doors and allows blood to flow either
right atrium close to the atrio-ventricular septum (AV forward into the next chamber or out of the heart via
node). A set of specialized muscle strands which one of two main blood vessels that carry blood away
originates in the AV node and pass downwards in from the heart. The valves close to prevent the back
the interventricular septum is called Purkinje fibres. flow. The heart has 4 valves: mitral valve and
19. (a) Pulmonary vein carries oxygen rich blood from the tricuspid valve (which control blood flow from the
lungs to the left atrium of heart. atria to the ventricles); and aortic valve and
20. (a) Sino-atrial node (SA node) initiates and maintains pulmonary valve (which control blood flow out of
contraction of heart by generating action potentials the ventricles).
(70-75/min). So it is called the pacemaker. 32. (c) A blockage in aortic valve would directly reduce
21. (a) Arteries are elastic vessels that transport blood away blood flow to the lungs.
from heart towards the other body tissue. They do 33. (d) In mammals the blood from the right ventricles goes
not have valves. to the pulmonary aorta.
22. (d) The SA node has the inherent power of generating a 34. (d) Lub is the first heart sound and dub is the second
wave of contraction and controlling the heartbeat. heart sound. Dub is associated with the closure of
Hence, it is known as the pacemaker. the semilunar valves at the beginning of diastole.
23. (a) Tricuspid valve is formed of three muscular flaps or 35. (c) Lub is the first heart sound, which is associated with
cusps, which guard the opening between the right the closure of the tricuspid and bicuspid valves at
atrium and the right ventricle. the beginning of systole.
24. (c) Bicuspid valve (mitral valve) guards the opening 36. (c) Closure of AV valve makes the louder sound of heart
between the left atrium and the left ventricle, made beat.
up of two flaps.
EBD_7209
Hints & Solutions S-99

37. (d) A cardiac cycle (atrial systole + ventricular systole + 50. (c) Coronary Artery Disease (CAD) or atherosclerosis
diastole) is completed in 0.8 seconds. is a disorder in which the deposition of calcium, fat,
38. (d) Cardiac cycle is defined as the complete cycle of cholesterol and fibrous tissue occurs in coronary
events in the heart from the beginning of one heart arteries which makes the lumen of arteries narrower
beat to the beginning of the next. It comprises three and thereby affect the blood supply.
stages – atrial systole, ventricular systole, and 51. (b) Hardening of arteries due to deposition of cholesterol
complete cardiac diastole. is called atherosclerosis.
39. (c) The P-wave indicates atrial depolarization, which 52. (d) Arteriosclerosis is the thickening, hardening and
causes atrial systole. During this wave, the impulse loss of elasticity of the walls of arteries. This process
of contraction is generated by the SA node. progressively restricts the blood flow to one's organs
40. (a) QRS-complex represents depolarization of ventricles and tissues and can leads to severe health risks. It is
(ventricular systole). caused by the deposition of fatty plaques, cholesterol
and some other substances in and on the artery walls.
41. (a) The largest artery is called the aorta. It is about 1
53. (b) ECG is the recording of electrical changes that
inch in diameter and receives the push of blood from
accompany the cardiac cycle. Normal ECG is composed
the left ventricle.
of P-wave, QRS-wave and T-wave. The P-wave indicates
42. (c) Pulmonary artery arises from right ventricle and is
atrial depolarization, which causes atrial systole. QRS-
the only artery that carries deoxygenated blood. wave represents ventricular depolarization. The T-wave
43. (d) Coronary circulation is a system of coronary vessels represents repolarization of ventricles.
that circulate blood to and from the cardiac 54. (a) Angina is an acute chest pain due to oxygen
musculature. deficiency to heart muscles. It occurs due to improper
44. (d) Pulmonary circulation involves circulation between blood flow. Persons with AB group are called
lungs and heart. The deoxygenated blood pumped Universal recipients because they can accept blood
into the pulmonary artery is passed on to lungs from from all groups. Persons with O Group are called
where oxygenated blood is carried by pulmonary Universal donors because they can donate blood to
veins into the left atrium. persons with any other blood group.
45. (c) Systemic circulation involves circulation of blood 55. (a) Blood transports oxygen from respiratory organs to
between heart and various body parts except lungs. the tissue cells and also transports carbon dioxide
The oxygenated blood from the left ventricle of the from the tissue cells to the respiratory membrane.
heart is passed through aorta, arteries, arterioles and About 97% of oxygen is transported by RBCs in the
capillaries and reaches the tissues. The blood while the remaining three percent of oxygen is
deoxygenated blood collected from the tissues by carried in a dissolved state through plasma. Nearly
venules, veins and vena cava is carried to the right 20-25% of carbon dioxide is transported by RBCs
atrium. The systemic circulation provides nutrients, whereas 70% is carried as bicarbonate. About 7% of
O2 and other essential substances to the tissues and CO2 is carried in dissolved state through plasma.
takes CO2 and other harmful substances away for 56. (b) The wall of the capillary is very thin. They form
elimination. capillary plexus between arterioles on one end and
46. (d) Pulmonary vein carries oxygen rich blood from the venules on others. Capillaries transports blood from
lungs to the right atrium heart. arteries to the veins.
47. (a) Semilunar valves are pocket-like structures attached 57. (c) Blood pressure is the measure of the force of blood
at the point at which the pulmonary artery and the pushing against blood vessel walls. The heart pumps
aorta leave the ventricles. Aorta and pulmonary artery blood into the arteries which carry the blood
possess semilunar valve. The pulmonary valve throughout the body. It is usually expressed in terms
guards the orifice between the right ventricle and of the systolic (maximum) pressure over diastolic
the pulmonary artery. The aortic valve protects the (minimum) pressure and is measured in millimetres
orifice between the left ventricle and the aorta. of mercury (mm Hg). The ranges of blood pressure
48. (b) In normal person, the normal blood pressure is 120/ are:
80 mm Hg. The normal systolic (pumping) pressure • Normal: Less than 120 over 80 (120/80)
is 120 mm Hg and normal diastolic (resting) pressure • Prehypertension: 120-139 over 80-89
is 80 mm Hg. • Stage 1 high blood pressure: 140-159 over 90-99
49. (b) Pulse beat can be measured where an artery comes • Stage 2 high blood pressure: 160 and above
close to the skin, such as your wrist, neck, temple over 100 and above
area, behind the knee or top of your foot. • High blood pressure in people over age 60: 150
and above over 90 and above.
S-100 Biology
58. (d) Cardiac output is the volume of blood pumped out embedded in the walls of the lower part of the heart.
by each ventricle per minute, i.e. stroke volume x 67. (b) Lub and dub are two heart sounds which occur due
heart rate. to the closure of cuspid valves and semilunar valves
59. (c) Persons with blood group A have the A antigen on respectively. Lub is the first heart sound which is
the surface of their RBCs, and antibodies to antigen formed due to closure of atrioventricular valves
B in their plasma. (tricuspid and bicuspid) at the beginning of
60. (d) In joint diastole, all chambers of the heart are in relaxed ventricular systole. It is a low pitched of long
state. When the tricuspid and bicuspid valves open, duration (0.15 sec).
blood from pulmonary vein and vena cava flows into 68. (c) Saline water increases the blood pressure due to the
left and right ventricles respectively through left and presence of NaCl in it. Hence, the reason for the
right atria. assertion is false.
Note: Semilunar valves are closed at this stage. 69. (b) Blood pressure is the arterial pressure of blood
61. (a) Pulmonary artery is the only artery that carries the exerted on the wall of arteries with each heart beat. It
de-oxygenated blood from heart to lungs. is measured form the brachial artery in the elbow pit.
62. (a) Blood cir culation involves the transport of It is expressed as
oxygenated blood from the lungs to the left atrium systolic pressure (mm/Hg)
by the pulmonary vein. From the left atrium, the =
diastolic pressure (mm/Hg)
oxygenated blood is pumped into the left ventricle,
from where it is taken to all body tissues by aorta. Arterial (superficial; arteries) blood pressure is
measured by sphygmomanometer.
63. (c) Lymph is a clear-to-white fluid made of white blood
cells, especially lymphocytes. The lymph is formed 70. (b) WBC’s are wandering cells capable of coming out of
when the interstitial fluid (the fluid which lies in the blood capillaries by amoeboid movement called
interstices of all body tissues) is collected through diapedesis.
lymph capillaries. It is then transported through 71. (d) When an injury is caused to a blood vessel, bleeding
lymph vessels to lymph nodes before emptying starts which is stopped by blood clotting. At the site
ultimately into the right or the left subclavian vein, of injury blood platelets release platelet factor - 3
where it mixes back with blood. and injured tissues release thromboplastin. The two
64. (c) Artery transport blood away from heart towards the combine to form prothrombinase enzyme which
other body tissue. Artery carries oxygen rich blood converts prothrombin to thrombin. The latter
except for pulmonary artery. Pulmonary artery is the stimulates formation of fibrin thread or clot. Blood
only artery that carries carbon dioxide rich blood contains an anticoagulant heparin which prevents
from heart to lungs. blood clotting in uninjured vessels.
65. (b) A healthy individual has 12 - 16 gms of haemoglobin 72. (b) Prothrombinase enzyme is necessary for blood
in every 100ml of blood. Neutrophils and monocytes clotting. It act as antiheparin. Coagulation of blood
are phagocytic cells which destroy foreign organisms in vessels is prevented by heparin, a quick acting
entering the body. A reduction in the number of anticoagulant. It inhibits conversion of prothrombin
platelets leads to clotting disorders which will lead to thrombin and is used in open-heart surgery.
to excessive loss of blood from the body. 73. (a) When the blood pressure of an individual decreases
66. (a) SAN is the natural pacemaker of the heart. It is one due to loss of blood volume, then vasoconstriction
of the major elements in the cardiac conduction of veins occurs. This shifts the little amount of blood
system, the system that controls the heart rate. The from veins to arteries.
SA node consists of a cluster of cells that are situated 74. (c) Fishes have two-chambered hearts. Amphibians and
in the upper part of the wall of the right atrium where reptiles have three chambered hearts. Birds and
the electrical impulses are generated. The electrical mammals have four chambered hearts.
signal generated by the SA node moves from cell to 75. (a) Lub is the first heart sound. It is associated with the
cell down through the heart until it reaches the closure of the tricuspid and bicuspid valves at the
atrioventricular node (AV node). The AV node serves beginning of systole and opening of semilunar
as a gate that slows the electrical current before the valves.
signal is permitted to pass down to the ventricles. 76. (a) The P-wave indicates atrial depolarisation. For more
This delay ensures that the atria have a chance to refer answer 49.
fully contract before the ventricles are stimulated. 77. (a)
After passing the AV node, the electrical current 78. (a) Plasma is a straw coloured fluid that consists of
travels to the ventricles along special fibers suspended blood cells. Heart is the pumping station
EBD_7209
Hints & Solutions S-101

of the body. Capillary is the site where exchange of 84. (b) In the given diagram of human heart, the parts
nutrients and gases takes place while blood is a red marked as A, B, C, D and E are respectively aorta, left
colour fluid that provides food to the cells. atrium, left ventricle, right ventricle and right atrium.
79. (a) Blood cir culation involves the transport of The correct sequence of the letters which represent
oxygenated blood from the lungs to the left atrium the process of blood flow described in the question
by the pulmonary vein. From the left atrium, the blood is B, C, A, E, D.
is pumped into the left ventricle, from where it is 85. (d) The label 4 represents right atrium. Right atrium
taken to all body tissues by aorta. From the body receives most of the blood returning from the brain.
tissues, the deoxygenated blood is transported back 86. (c) The label 1, 2, 3 & 4 represent vein, artery, pulmonary
to the heart, into the right atrium, by the vena cava. artery and pulmonary vein respectively.
From the right atrium, the blood is pumped into the 87. (b) The contraction starts shortly after the Q wave and
right ventricle, from where it is taken back to the marks the beginning of the systole.
lungs by the pulmonary artery for oxygenation. 88. (d) Leucocytes are colourless, nucleated amoeboid cells
80. (b) A - (iii), B - (v), C - (ii), D - (iv), E - (i) found in blood which are devoid of haemoglobin
Cardiac cycle refers to a complete heartbeat from its and are capable of coming out of blood capillaries
generation to the beginning of the next beat. The through the process of diapedesis. Fall of WBC
frequency of the cardiac cycle is described by the count is called leucopenia, and occurs due to folic
heart rate, which is typically expressed as beats per acid deficiency and AIDS etc.
minute. The duration of cardiac cycle is 0.8 seconds. 89. (a) The auricular contraction initiated by the wave of
Plasma is the pale yellow liquid component of blood excitation from the sino-atrial node (SA node)
that normally holds the blood cells in whole blood in stimulates the atrio-ventricular node, thereby leading
suspension. It makes up about 55% of the body's to the contraction of ventricles through the bundle
total blood volume. 120/80 mmHg is the normal value of His and Purkinje fibres.
of blood pressure. A blood pressure reading has a 90. (a) If nerves of heart are cut, then heart will beat
top number (systolic) and bottom number (diastolic). rhythmically.
A healthy individual has 12 - 16 gms of haemoglobin
91. (c) Cardiac output is the amount of blood flowing from
in every 100 ml of blood. Heart beat is a complete
the heart (i.e from the left ventricle into the aorta)
cardiac cycle, during which the electrical impulse is
over a given period of time. It can be calculated by
conducted and mechanical contraction occurs. Heart
stroke volume (the volume of blood pumped by heart/
beats 72 times per minute.
heartbeat) and heart rate (ventricular systole/min).
81. (c) ECG (Electrocardiogram) is a graphic sketching of
Stroke volume × Heart rate
the variations in electrical potential caused by the
Stroke volume - 0.07 litres
excitation of the heart muscle and detected at the
body surface. The normal ECG is a scalar Heart rate - 72 / minute
representation that shows deflections resulting from According to the formula: 0.07 × 72 = 5.04
cardiac activity. It comprises the P wave, QRS 92. (c) The blood returning to the heart through pulmonary
complex, T wave and U wave. vein carries oxygen rich blood.
QRS complex: It represents the depolarization of the 93. (a) Lub is the first heart sound. It is associated with the
ventricles which initiates the ventricular contraction. closure of the tricuspid and bicuspid valves at the
The contraction starts shortly after Q and marks the beginning of systole. First heart sound coincided
beginning of the systole. Hence QRS complex shows with R wave of ECG.
one complete pulse. 94. (b) SAN (called pacemaker) initiates and maintains
82. (c) The label III in the given figure represents right contraction of heart by generating action potentials
atrium. The right atrium receives carbon dioxide rich (70-75/min). When pacemaker is non-functional, the
blood from the body. The label I represents left atrium. cardiac muscles do not contract in a coordinated
It receives oxygenated blood from the lungs. manner.
The label II represents left ventricle. It pumps 95. (a) Maximum amount of oxygen is lost from the blood in
oxygenated blood to rest of the body. the capillaries surrounding the tissue cells. Capillaries
The label IV represents right ventricle. It pumps are the smallest blood vessels that make up the
deoxygenated blood to the lungs. microcirculation. Their endothelial linings are only
83. (b) Bicuspid valve (mitral valve) made up of two flaps one cell layer thick. These microvessels connect
guards the opening between the left atrium (B) and arterioles and venules, and help to enable the
the left ventricle (C). exchange of water, oxygen, carbon dioxide, and many
S-102 Biology
other nutrients and waste substances between the tissues, the deoxygenated blood is transported back
blood and the tissues surrounding them. to the heart, into the right atrium, by the vena cava.
96. (c) Blood serum does not coagulate because it does not From the right atrium, the blood is pumped into the
contain any clotting factors. Blood serum is the right ventricle, from where it is taken back to the
component that is neither a blood cell (serum does lungs by the pulmonary artery for oxygenation.
not contain white or red blood cells) nor a clotting 105. (d) Blood pressure is the arterial pressure of blood
factor. It is the blood plasma that includes all proteins exerted on the walls of arteries with each heartbeat.
not used in blood clotting (coagulation) and all the It is measured from the brachial artery and expressed
electrolytes, antibodies, antigens, hormones, and as systolic pressure/diastolic pressure (i. e. 120/80
any exogen ous substances (e.g., drugs and mmHg). Blood pressure is affected by atherosclerosis
microorganisms). (caused by deposits if calcium, fats, cholesterol, and
97. (a) The chordae tendinae are strong, fibrous strings fibrous tissue which make lumen narrow). Diastolic
which attach on one end to the edges of the tricuspid pressure is lower than systolic pressure. It refers to
and mitral valves of the heart and on the other end to as arterial pressure of the systemic circulation.
the papillary muscles (which is a small muscle within 106. (c) In order for the foetus to be Rh positive, the father
the heart that serve to anchor the valves). When the must also be Rh positive. As a result, when the
cusps close, the chordae tendinae prevent them from mother is Rh negative and the father is Rh positive
fluctuating back into the atrium cavity (on the upper (or his Rh type is unknown), doctors usually assume
chamber). If chordae tendinae of tricuspid valve is that there will be a serious medical problem.
partially non-functional due to some injury, then the 107. (b) The atrioventricular septum is a septum of the heart
flow of blood into the pulmonary artery will be between the right atrium (RA) and the left ventricle.
reduced. It is a small part of the membranous septum of the
98. (d) Auricular depolarization, ventricular diastole and heart just above the septal cusp of the tricuspid
auricular systole are the events that occur in cardiac valve.
cycle simultaneously. Cardiac cycle is the rhythmic 108. (a) The atrioventricular node has junctional fibres which
contraction (systole) and relaxation (diastole) of the will slightly delay the passage of the impulse,
heart chambers that corresponds to one heartbeat. allowing the ventricles sufficient filling time.
During the cardiac cycle, blood is pumped out of the 109. (a) The hypothalamus is involved with what could be
heart into the aorta and the pulmonary artery, and considered emotional responses such as a sudden
blood re-enters the heart from the vena cava and the increase in heart rate or irregular beat in situations of
pulmonary veins. fear or anxiety.
99. (d) If a person is losing blood continuously due to injury 110. (d) Heart failure occurs when the heart is unable to pump
then his pulse will rise and BP will fall. sufficiently to maintain blood flow to meet the body's
100. (a) During systemic circulation, blood leaves the left needs. Common causes of heart failure include
ventricle and goes directly to the aorta while during coronary artery disease including a previous
pulmonary circulation, blood leaves the right myocardial infarction (heart attack), high blood
ventricle and moves to the lungs. pressure, atrial fibrillation, valvular heart disease, and
101. (c) The systolic pressure, the highest pressure in the cardiomyopathy.
artery during each heartbeat of a healthy adult is
Chapter 19 : Excretory Products and Their Elimination
about 120 mm of Hg.
102. (a) A two-chambered heart with the vena cava entering 1. (a) An uricotelic organism produces uric acid as a result
the auricle and the aorta leaving the ventricle would of de-amination. Examples of such organism are birds
totally by pass the pulmonary circuit. As a result, and insects.
the blood would never reach the lungs and would
2. (c) The animals which excrete ammonia are called
never pick up a fresh supply of oxygen to take to the
ammonotelic animals. The example includes aquatic
cells.
invertebrates, bony fishes, aquatic amphibians,
103. (c) Sinus venosus is found only in fishes, amphibians
and reptiles. tadpoles, aquatic insects, etc.
104. (a) Blood cir culation involves the transport of 3. (d) Mechanism of uric acid excretion in a nephron is
oxygenated blood from the lungs to the left atrium called secretion. Certain chemicals in the blood that
by the pulmonary vein. From the left atrium, the blood are not removed by filtration from the glomerular
is pumped into the left ventricle, from where it is capillaries is removed by a process of urine formation
taken to all body tissues by aorta. From the body are called tubular secretion. Ions removed from the
EBD_7209
Hints & Solutions S-103

blood by tubular secretion include potassium, 13. (d) The efferent arteriole emerging from the glomerulus
hydrogen, and ammonium ions and foreign forms a fine capillary network around the renal tubule
substances like drugs, penicillin, creatinine and uric called the peritubular capillaries.
acid etc. 14. (a) Juxta-glomerular apparatus is a microscopic
4. (a) Bony fishes, aquatic amphibian and aquatic insects structure located between the vascular pole of the
are called ammonotelic animals because these animals renal corpuscle and the returning distal convoluted
excrete ammonia as their nitrogenous waste products. tubule of the same nephron. It is formed by cellular
Being the most toxic form ammonia requires large modification in the afferent arteriole and DCT.
amount of water for its elimination. Aquatic animals, 15. (d) Sweat glands are highly vascular and tubular glands
like crustaceans, bony fishes and amphibian larvae, that separate the waste products from the blood and
generally are ammonotelic since ammonia diffuses excrete them in the form of sweat. Sweat excretes
more easily through membranes and it is more water- excess salt and water from the body.
soluble than the other nitrogen wastes. 16. (d) Plasma protein of blood does not enter into the
5. (d) Excretion of nitrogenous waste product in semi-solid nephron.
form is found in uricotelic animals. 17. (b) Glucose is 100% reabsorbed and thus a healthy
6. (b) Hilus leads to funnel shaped cavity called renal pelvis person will excrete no glucose in the urine. Sodium
with projections called calyces. and water are usually over 99% reabsorbed. Urea is
7. (a) The number of nephron in a kidney is equal to the the main excretory product formed as a result of
number of Bowman’s capsule. protein breakdown.
8. (b) Malpighian corpuscle (renal corpuscle) comprises 18. (d) The proximal convoluted tubule is the site whereby
glomerulus and Bowman’s capsule. The malpighian selective reabsorption occurs, restoring the level of
corpuscle is named after Marcello Malpighi, an italian metabolities in the tubule and prevents excessive
physician and biologist. loss of useful substances.
9. (c) The collecting duct system contains of a series of 19. (c) The ascending limb of Henle’s loop is impermeable
tubules and ducts that connect the nephrons to the to water but allows transport of electrolytes like
ureter. It takes part in the maintenance of electrolyte sodium. So, the filtrate gets diluted.
and fluid balance through reabsorption and 20. (b) Diffusion is the movement of molecules from a high
excretion. These pr ocesses are regulated by concentration to a low concentration. Diffusion helps
aldosterone and antidiuretic hormone. Therefore it to reabsorb chloride ions from the glomerular filtrate
refers as the part of the excretory system which in kidney tubules.
contains fluids as urine. 21. (a) Colloidal osmotic pressure in blood plasma is mainly
10. (b) The renal column (or column of Bertini) is a medullary due to albumin.
extension of the renal cortex in between the renal 22. (c) The ascending limb is impermeable to water but
pyramids. It allows the cortex to be better anchored. allows transport of electrolytes like sodium. So, the
Each column consists of lines of blood vessels and filtrate gets diluted.
urinary tubes and a fibrous material.
23. (c) Loop of Henle is a long, U-shaped portion of the
11. (d) Glomerulus is a tuft of capillaries formed by afferent tubule takes part in the absorption of water and
arteriole(a fine branch of renal artery). conducts urine within each nephron of the kidney of
12. (c) Convoluted tubules is not a part of renal pyramid reptiles, birds, and mammals. The important function
but a portion of the nephron in the kidney that of the loop of Henle appears to be the recovery of
functions in concentrating urine and in maintaining water and sodium chloride from the urine which
salt, water, and sugar balance. Renal pyramids (also allows production of urine that is far more
called malpighian pyramids) consist mainly of tubules concentrated than blood, limiting the amount of water
that transport urine from the cortical (or outer) part of needed as intake for survival. Many species that
the kidney, where urine is produced, to the calyces or live in arid environments such as deserts have highly
cup-shaped cavities in which urine collects before it efficient loops of Henle.
passes through the ureter to the bladder. The point of 24. (b) In comparison to blood plasma, percentage of
each pyramid, called the papilla, projects into a calyx. glucose in glomerular filtrate is equal.
S-104 Biology
25. (b) The amount of the filtrate formed by the kidneys per for erythrocyte (red blood cell) precursors in the bone
minute is called glomerular filtration rate (GFR). GFR marrow. It is produced by interstitial fibroblasts in
in a healthy individual is approximately 125 ml/minute, the kidney in close association with peritubular
i.e., 180 litres per day. capillary and promixal convoluted tubule.
26. (a) PCT (proximal convoluted tubule) is a small tubular 31. (c) Angiotensin II is a powerful vasoconstrictor that
structure within the nephron and connects increases the glomerular blood pressure and thereby
Bowman's capsule with the proximal straight tubule. GFR.
It is essential for the reabsorption of water and 32. (b) Vasopressin regulates the permeability of DCT and
solutes from filtrate within the nephron. The lining collecting duct to water.
of the PCT contains many protein channels, which 33. (a) Renin converts angiotensinogen in blood to angiotensin
use both active and passive transport to move I and further to angiotensin II (a vasoconstrictor).
substances such as glucose and electrolytes across 34. (b) Aldosterone and ADH maintains the volume of urine.
the tubule's lining and into the interstitial fluid for
Aldosterone, produced by the adrenal cortex, causes
reabsorption. Nearly all of essential nutrients, and the retention of water in the body by increasing the
70-80 percent of electrolytes and water are levels of sodium and potassium ions in the blood,
reabsorbed by this segment. The epithelium of the which causes the body to reabsorb more water.
PCT also prevents waste products from being Antidiuretic hormone (ADH) produced by the
reabsorbed into the bloodstream.
hypothalamus and released by the posterior pituitary,
27. (c) The amount of the filtrate formed by the kidneys per causes more water to be retained by the kidneys
minute is called glomerular filtration rate (GFR). GFR when water levels in the body are low.
in a healthy individual is approximately 125 ml/minute, 35. (b) Vasopressin promotes reabsorption of water from
i.e., 180 litres per day. A fall in GFR can activate the the glomerular filtrate. Vasopressin, also known as
JG cells to release renin which can stimulate the antidiuretic hormone (ADH), is a neurohypophysial
glomerular blood flow and thereby the GFR back to hormone. It helps to retain water in the body and
normal. Renin converts angiotensinogen in blood to
constrict blood vessels. Vasopressin regulates the
angiotensin I and further angiotensin II. Angiotensin body's retention of water by acting to increase water
II being powerful vasoconstrictor increases the reabsorption in the collecting ducts of the nephron,
glomerular blood pressure and thereby GFR. which is the functional component of the kidney.
28. (c) Loop of Henle is a U- shaped tube that consists of
36. (c) A fall in glomerular blood flow/glomerular blood
an ascending and descending limb. Its primary role
is to concentrate the salts in the interstitium, the pressure/GFR activates the juxtaglomerular cells to
tissue surrounding the loop. Descending limb are release renin. For more refer answer 27.
thin walled and permeable to water but impermeable 37. (a) Juxtaglomerular apparatus (JGA) is an important
to salt and thus only indirectly contributes to the constituent of renin-angiotensinogen- aldosterone
concentration of the interstitium. The ascending system (RAAS). Renin-angiotensin-aldosterone
limbs are thick walled and impermeable to water. It system is a hormone system that helps regulate long-
actively reabsorbs the remaining 25% of the filtered
term blood pressure and blood volume in the body.
K+ and some amounts of Cl–, some Na+ is also reab-
It is a specialized region of a nephron where the
sorbed by diffusion due to electrostatic attraction of
reabsorbed Cl–. afferent arteriole and distal convoluted tubule (DCT)
29. (c) Glomerulus and Bowmans capsule are involved in comes in direct contact with each other.
ultrafiltration. In the proximal convoluted tubules the 38. (b) Micturition is the process by which the urine from
mitochondria provide energy for active transport. the urinary bladder is excreted.
The cells here are adapted for reabsorption. The 39. (a) Vasa recta is absent or highly reduced in cortical
amino acids diffuse into the cells and are actively nephrons.
transported to the intercellular spaces, where they 40. (c) Blood enters glomerulus through afferent arterioles.
diffuse into the surrounding capillaries. 41. (d) Henle’s loop is a U-shaped tube that consists of a
30. (b) Erythropoietin is a glycoprotein hormone that descending limb and ascending limb. Its primary role
controls erythropoiesis, or the formation of red blood is to concentrate the salt in the interstitium, the tissue
cell. It acts as a cytokine (protein signaling molecule) surrounding the loop. In the absence of Henle’s loop,
EBD_7209
Hints & Solutions S-105

the urine will be more dilute. 50. (a) Kidney plays an important role in maintaining the
42. (d) In ureotelic animals, ammonia is converted into less concentration and osmotic pressure of blood. When
toxic urea (for conservation of water). water intake of an animal is very high, the urine
43. (d) Ascending limb of Loop of Henle are thick walled excreted has to be hypotonic and lower in osmotic
and impermeable to water. Descending limb of loop pressure than their blood in order to remove the
of Henle is thin walled and permeable to water but excess water.
impermeable to salt and thus indirectly contributes On the contrary, when there is a shortage of water,
to the concentration of the interstitium. Distal the urine excreted needs to be hypertonic and higher
convoluted tubule is a portion between the loop of in osmotic pressure than their blood to reduce the
Henle and the collecting duct system which is loss of water with urine.
capable of reabsorption of HCO3– and selective 51. (a) Descending limb of loop of Henle is permeable to
secretion of hydrogen and potassium ions and NH3 water but not to sodium. Consequently water moves
to maintain the pH and sodium-potassium balance out into interstitium and concentration of sodium in
in the blood. tubular filtrate rises making filtrate hypertonic.
44. (b) Antidiuretic hormone (ADH) helps regulate water Ascending limb of loop of Henle is impermeable to
balance in the body by controlling the amount of water but permeable to sodium and makes the filtrate
water by the kidneys reabsorb while they are filtering hypotonic.
wastes out of the blood. When someone drinks lots 52. (d) Hypotonic urine means concentration of urine is less
of water, ADH release is suppressed. Therefore, if than that of blood. Hypertonic urine is more
there is too little ADH or the kidneys do not respond concentrated than blood and has high osmotic
to ADH, then too much water is lost through the pressure than the blood, therefore it helps in reducing
the loss of water with urine. The urine is filtered by
kidneys, the urine produced is more dilute than
the Bowman’s capsule. The tubules of nephrons
normal, and the blood becomes more concentrated. reabsorb a large quantity of water making urine more
This can cause excessive thirst, frequent urination, concentrated.
dehydration. If there is too much ADH, then water is 53. (a) Aldosterone is one of the important
retained, blood volume increases, and the person mineralocorticoids in humans secreted by adrenal
may experience nausea, headaches, disorientation, cortex. Its main function is to regulate sodium content
lethargy, and low blood sodium (hyponatremia). of the body . It increases sodium ion concentration
45. (a) Urine formation is the result of ultrafiltration or in the blood by absorbing sodium ions from renal
glomerular filtration of the blood plasma by the tubules. Excessive production of aldosterone causes
a disease aldosteronism. It symptoms include high
glomeruli, selective reabsorption by the tubules and
blood pressure, high blood volume.
secretion by the tubules. Glomerular filtration is the
54. (d) Urea is the chief nitrogenous constituent of human
first of the three processes that forms urine. Tubular
urine, though it possesses small amount of ammonia.
reabsorption is the second process in the formation But when the urine is allowed to stand for sometime,
of urine from filtrate and tubular secretion is the third bacterial degradation occurs and it leads to the production
process in the urine formation. of ammonia from urine. And thus it smells strongly.
46. (c) Vasa recta are a series of straight capillaries in the 55. (a) Ultrafiltration is a filtration under high pressure which
medulla. They lie parallel to the loop of Henle. occurs at the arterial end of the glomerulus. The
47. (c) Haemodialyzer removes urea, uric acid, glucose and filtrate from the glomerulus passes into the renal
proteins. In glomerulus, urea, uric acid, water, glucose capsule and moves in to th e renal tubule.
and plasma proteins are filtered out. Concentration of urine takes place in Henle's loop.
48. (d) Urine is hypotonic in distal convoluted tubule. When The primary role of the loop of Henle is to
the urine passes into the collecting tubule, it concentrate the salt in the interstitium, the tissue
becomes hypertonic. Urine becomes more and more surrounding the loop. Transport of urine takes place
hypotonic as it passes through the DCT. in ureter. Ureters are narrow tubes started as a pelvis
49. (a) Ureotelism is the process of excretion of urea. Aquatic within kidney opening into a common urinary bladder
animals like whales and seals are said to be ureotelic which opens outside through urethra. Urinary
animals because their major nitrogenous waste bladder is a sac like structure which stores urine
product is urea. temporarily.
S-106 Biology
56. (a) Uremia refers to increased level of urea and other blood pressure and thereby GFR. Angiotensin II also
nitrogenous waste compounds in the blood that are activates the adrenal cortex to release aldosterone.
normally eliminated by the kidneys. Hematuria is the Aldosterone causes reabsorption of Na+ and water
presence of blood in the urine. Ketonuria is the from the distal parts of the tubule. This also leads to
excretion of abnormally large amounts of ketone an in crease in blood pressure and GFR.
bodies in the urine. It is the characteristic of diabetes Osmoreceptors in the body are activated by changes
mellitus, starvation, or other medical conditions. in blood volume, body fluid volume and ionic
Glycosuria is a condition characterized by an excess concentration. An excessive loss of fluid from the
of sugar in the urine. It is typically associated with body can activate these receptors which stimulate
diabetes or kidney disease. Proteinuria is the the hypothalamus to release antidiuretic hormone
presence of abnormal quantities of protein in the (ADH) or vasopressin from the neurohypophysis.
urine, which may indicate damage to the kidneys. 63. (b) The label X represents ureters, while label Y
57. (d) Proximal convoluted tubule connects Bowman's represents urethra. Ureters are narrow tubes started
capsule with the proximal straight tubule and the as a pelvis within kidney opening into a common
primary site of glucose and amino acid reabsorption. urinary bladder which opens outside through
Distal convoluted tubules are responsible for the urethra. Urinary bladder is a sac like structure which
stores urine temporarily.
resorption of sodium, water and secretion of
hydrogen potassium. Descending limb of loop of 64. (c) The nephron carries the urine into the collecting
Henle is a place where reabsorption of major tubule of the kidney from where it is carried to ureter.
substances takes place with the help of osmosis. From ureter, the urine passes into the urinary bladder.
Ascending limb of loop of Henle is a place where The urinary bladder stores the urine until it is forced
sodium is reabsorbed actively. out of the body, through an opening known as the
urethra.The label II represents ureter, label III
58. (a) Renin is an enzyme that catalyses the formation of
represents urinary bladder and label I represents
angiotensin I.
urethra.
Aldosterone promotes reabsorption of sodium at
65. (b) The label II represents renal vein while label I
distal convoluted tubule. Anti-diuretic hormone
represent renal artery, label III represents renal pelvis
regulates water reabsorption at the distal convoluted and label IV represents renal column.
tubule. Angiotensin II is a powerful vasoconstrictor
66. (a) The label X represents vasa recta that function in
that stimulates the secretion of aldosterone.
the reabsorption of water, minerals and digestive end
59. (b) Afferent arterioles enters the glomerulus at the products.
vascular pole and divides into capillaries which
67. (b) In the given figure of nephron, red blood cells would
subsequently merge to form efferent arterioles.
not usually pass through the membranes from the
Henle's loop has minimum reabsorption. It plays a region A (Glomerulus) to region B (Bowman's
significant role in the maintenance of osmolarity of capsule). Capillaries of the glomerulus are lined by
the medullary interstitial fluid. i.e concentration of endothelial cells. These contain numerous pores
urine. In Distal convoluted tubules reabsorption of (called fenestrae) 70-100 nm in diameter. These pores
water and sodium takes place. It is also capable of allow for the free filtration of fluid, plasma solutes
reabsorption of HCO3– and selective secretion of and protein. However they are not large enough that
hydrogen and potassium ions. red blood cells can be filtered.
60. (c) Salamander (amphibia; caudata) excretes urea with 68. (a) Label represents glomerulus. After the blood enters
the help of kidneys. the kidney, it travels to the glomerulus.
61. (b) A - III, B - I, C - II, D - IV. 69. (b) 3 and 4 are respectively descending limb and
62. (a) A - I, B - II, C - III, D - IV, E - V. ascending limb of loop of Henle. They have minimum
The JGA plays a complex regulatory role. A fall in reabsorption and play a significant role in the
glomerular blood flow/glomerular blood pressure/ maintenance of high osmolarity of medullary
GFR can activate the JG cells to release renin which interstitial fluid.
converts angiotensinogen to angiotensin I and 70. (c) Part marked as 6 is proximal convoluted tubule (PCT).
further to angiotensin II. Angiotensin II, being a PCT is the convoluted portion of the vertebrate
powerful vasoconstrictor, increases the glomerular nephron that lies between Bowman's capsule and
EBD_7209
Hints & Solutions S-107

the loop of Henle. Cells of PCT are lined by simple the part of a kidney tubule which forms a long loop
cuboidal brush border epithelium that increases the in the medulla of the kidney, from which water and
surface area for reabsorption. The cells lining the salts are reabsorbed into the blood.
PCT absorb organic nutrients, plasma proteins, & 80. (b) Urine is hypertonic in the middle of descending and
ions from the tubular fluid & release them into the ascending limb of Henle’s loop.
interstitial fluid surrounding the renal tubule where 81. (a) Diuresis is characterized by increased urine volume.
this reabsorbed fluid enters the peritubular capillaries. ADH prevents diuresis by facilitating water
71. (d) Distal convoluted tubule (DCT) is marked as 7. The reabsorption from DCT and collecting duct.
DCT is an important site for the active secretion of 82. (a) PCT increases the surface area for reabsorption. PCT
ions, acids, drugs, & toxins and the selective reabsorbs most of the nutrients, and 70-80% of
reabsorption of sodium ions from the tubular fluid. electrolytes and water.
In the final portions of the DCT, an osmotic flow of 83. (a) Urine is a liquid produced by the kidneys to remove
water assists in concentrating the tubular fluid. waste products from the bloodstream. Human urine
72. (b) 3, 4 and 5 are respectively descending limb, ascending is transparent, yellowish in colour and variable in
limb of loop of Henle and vasa recta. These three chemical composition. It consists primarily of water
structures have an ability to produce the (95%), with organic solutes including urea (2.6%),
concentrated urine through the process of creatinine, uric acid, and trace amounts of enzymes,
countercurrent exchange system. Counter current carbohydrates, hormones, fatty acids, pigments, and
mechanism (the process due to which urine is made mucins, and inorganic ions such as sodium (Na+),
hypertonic) is regular exchange of Na+ ions between potassium (K+), chloride (Cl–), magnesium (Mg2+),
the ascending and descending limbs of kidney. calcium (Ca2+), ammonium (NH4+), sulfates (SO42–),
73. (d) Workers in deep mines usually suffer from and phosphates (PO43–).
dehydration because water is lost along with salts in 84. (a) The molecules that leave the blood & enter the
the form of sweat. glomerular capsule are called glomerular filtrate.
74. (c) Frog, monkey and dog, all are ureotelic animals. They Glomerular filtrate contains blood without blood cells
excrete their wastes in the form of urea. and proteins.
75. (b) The epithelial cells (podocytes) of the Bowman’s 85. (d) Renal artery and ureter carries the waste products.
capsule are arranged in an intricate manner so as to Renal vein carries pure blood back to the heart.
leave some minute spaces called filtration slits (slit 86. (c) Under normal conditions, urine does not contain
pores). glucose because glucose in glomerular filtrate is
76. (a) Podocytes are found in the inner wall of Bowman’s reabsorbed in the uriniferous tubules.
capsule. Podocyles send foot processes over the 87. (c) The kidneys control plasma volume by controlling
length of the glomerulus. how much water a person excretes. The plasma
volume has a direct effect on the total blood volume,
77. (d) Ultrafiltration is an entirely passive process. It occurs
which has a direct effect on one's blood pressure.
in glomerulus when colloidal osmotic pressure plus
Salt (NaCl) will lead to osmosis; the diffusion of water
capsular pressure remain less than glomerular
into the blood. When a person is suffering from poor
hydrostatic pressure.
renal reabsorption, increased arterial pressure in
78. (a) Filtration fraction is the ratio of the glomerular kidneys will not help to maintain the blood volume.
filtration rate (GFR) to the renal plasma flow (RPF) i.e
88. (a) ANF is secreted by heart under the influence of too
FF = GFR/RPF. The filtration fraction, therefore,
much blood in the circulatory system. ANF stimulates
represents the proportion of the fluid reaching the
the loss of sodium in urine while aldosterone absorbs
kidneys which passes into the renal tubules.
sodium from glomerular filtrate.
79. (a) In a mammalian kidney, Bowman' capsule occur in
89. (a) The ascending limb of loop of Henle has a thin and a
cortex while loops of Henle are situated in medulla. thick segment. The thin ascending limb is found in
Bowman capsule is a cup-like sac at the beginning the medulla of the kidney, and the thick ascending
of the tubular component of a nephron that performs limb can be divided into a part that is in the renal
the first step in the filtration of blood to form urine. A medulla and a part that is in the renal cortex. Through
glomerulus is enclosed in the sac. Loop of Henle is the thick segment of ascending limb of loop of Henle,
S-108 Biology
the NaCl can pass by active transport from the filtrate efferent renal arteriole increased, ultrafiltration will
to the interstitial fluid. The medullary ascending limb not take place.
remains impermeable to water. Sodium, potassium 95. (d) Urinary bladder is a sac like structure which stores
(K+) and chloride (Cl–) ions are reabsorbed by active urine temporarily. It is made up of smooth and
transport. K+ is passively transported along its involuntary muscles. The lumen of urinary bladder
concentration gradient through a K+ leak channel in is lined by transition epithelium, which has great
the apical aspect of the cells, back into the lumen of power of stretching. If the stretch receptors of the
the ascending limb. urinary bladder wall are totally removed, the urine
90. (b) One of the most important things the kidneys excrete will continue to collect normally in the bladder.
nitrogenous waste. As the liver breaks down amino 96. (d) During dialysis, as the patient's blood is passed
acids it also releases ammonia. The liver then quickly through dialysing solution, most of the wastes like
combines that ammonia with carbon dioxide, creating
urea present in it pass through selectively permeable
urea which is the primary nitrogenous end product of
cellulose tubes into the dialysing solution. The clean
metabolism in humans. The liver turns the ammonia
fluid is then pumped back into the vein of patient's
into urea because it is much less toxic. Therefore, urea
arm.
is likely to accumulate in dangerous proportion in the
blood of a person whose kidney is not working 97. (c) Excretion is the process of removing waste products
properly. produced in the cells of living organism.
91. (b) If the excess water passes out from the tissue without 98. (d) Deficiency of ADH causes diabetes insipidus which
being restored by kidneys, the cells would not be is characterized by excessive dilute urine, without
affected at all. Individual cells have no role to play in glucose.
this process. Excess water in the blood affects the 99. (a) Glomerular podocytes are highly specialized cells
osmoreceptors present in hypothalamus and volume with a complex cytoarchitecture and plays a major
receptors present in left atrium, ventricles and role in establishing the selective permeability of
pulmonary veins. This causes ADH release so that glomerular filtration barrier.
body hydration is regulated by removal of excess 100. (d) During states of dehydration, the hormone ADH
water by kidney. (anti-diuretic hormone) is produced, which increases
92. (a) The most important work of kidney is to extract waste the permeability of the collecting ducts and increases
from blood, balance body fluids, form urine, and aid water reabsorption.
in other important functions of the body. Usually 101. (c) Changes in the afferent arteriole pressure affects
urine contains no sugar but the blood entering the glomerular filtration rate. Increases in pressure will
kidney has more sugar than leaving the kidney increase filtration rate and decreases will decrease
because sugar is used by the kidney cells in filtration rate.
metabolism. 102. (c) Aldosterone causes reabsorption of Na+ and water
93. (a) In human beings, the capsular urine entering the from the distal parts of the tubule. This also leads to
proximal convoluted tubule is isotonic to blood. an increase in blood pressure and glomerular filtration
Because the plasma membranes in the proximal tubule rate (GFR).
are freely permeable to water, so that water and salt 103. (c) The cells lining the proximal convoluted tubule are
are removed in proportionate amounts. well adapted for reabsorption of materials from the
94. (c) Ultrafiltration, a type of membrane filtration, refers filtrate. They have abundant mitochondria and bear
to the passage of protein-free fluid from the numerous microvilli on the free side thus giving
glomerular capillaries into Bowman's space. Blood brush border appearance. The cells reabsorb entire
flows into these capillaries through the afferent glucose, amino acids, most of the inorganic ions,
arteriole and leaves through the efferent arteriole. much of the water as well as some urea from the
The blood pressure in the efferent arteriole is higher filtrate.
than the blood pressure in the afferent arteriole. This 104. (c) A uriniferous tubule/nephron is a long tubule
is because the efferent arteriole has a smaller diameter differentiated into four regions - Bowman's capsule,
than the afferent arteriole. If the diameter of the proximal convoluted tubule, loop of Henle and distal
afferent renal arterioles is decreased and that of convoluted tubule. Collecting ducts are the larger
EBD_7209
Hints & Solutions S-109

tubes, each receiving the collecting tubules of several thin filaments. Actin protein occurs in two forms -
nephrons. It does not form a part of the uriniferous polymeric F actin and monomeric G actin. In muscles,
tubule. fibrous actin (F-actin) is a helical polymer of a
globular polypeptide chain, G-actin.
105. (c) Concentration of urine depends upon the length of
8. (a) A-band is thick and dark. The length of A-band is
Henle's loop. Loop of Henle is the hairpin shaped about 1.5 m.
section of a kidney tubule situated between the 9. (c) A - band of myofibrils contains both thick and thin
proximal and distal tubules in the nephron. It consists filaments. A myofibril (also known as a muscle fibril)
of a thin descending limb which is permeable to water is a basic rod-like unit of a muscle. Myofibrils are
and a thick ascending limb which is impermeable to composed of long proteins such as actin, myosin,
water complex movements of ions and water across and titin, and other proteins that hold them together.
the walls of the loop enable it to function as a These proteins are organized into thin filaments and
countercurrent multiplier, resulting in the production thick filaments, which repeat along the length of the
myofibril in sections called sarcomeres. Muscles
of concentrated urine in the collecting duct.
contract by sliding the thin (actin) and thick (myosin)
106. (d) Antidiuretic hormone (ADH, or vasopressin), filaments along each other.
aldosterone (the principal mineralocorticoid secreted 10. (d) Troponin is a globular protein complex involved in
by the adrenal cortex) and atrial natriuretic factor muscle contraction. It occurs with tropomyosin in the
(ANF) are hormones that participate in the regulation thin filaments of muscle tissue. It covers the active site
of the renal function. of actin. Together with tropomyosin, troponin forms a
regulatory protein complex controlling the interaction
Chapter 20 : Locomotion and Movement of actin and myosin and that when combined with
calcium ions permits muscular contraction.
1. (b) Movement of leucocyte, macrophages and
cytoskeletal elements in our body exhibits amoeboid 11. (c) Increase in Ca++ level into the sarcoplasm leads to
movement. It is a type of movement which occur the binding of calcium with a subunit of troponin on
with the help of pseudopodia formed by cytoplasmic actin filaments and there by remove the masking of
streaming (as in Amoeba). active sites for myosin. Utilizing the energy from ATP
hydrolysis, the myosin head now binds to the
2. (d) Striated muscle tissue is also known as the skeletal exposed active sites on actin to form a cross bridge.
muscle or voluntary muscle, because it is attached This pulls the attached actin filaments towards the
mainly to the bones and skin, and is responsible for centre of ‘A’ band. The ‘Z’ line attached to these
the mobility of the body and limbs. The muscles are actins are also pulled inwards thereby causing a
fibrous, dense tissues, whose primary function is to shortening of the sarcomere, i.e., contraction. It is
allow the body to move by repeated contraction and clear from the above steps, that during shortening
relaxation. Besides movement, the muscle is also of the muscle, i.e., contraction, the ‘I’ bands get
responsible for maintaining posture, stabilizing the reduced, whereas the ‘A’ bands retain the length.
joints, and producing body heat through muscle
function. These muscles are found in legs. 12. (d) A sarcomere is a structural unit within a myofibril
bounded by Z lines that contain actin and myosin.
3. (b) Smooth muscle or "involuntary muscle" is found
13. (c) Myosin is a fibrous protein which forms (together
within the walls of organs and structures such as
with actin) the contractile filaments of muscle cells and
the esophagus, stomach, intestines, bronchi, uterus,
is also involved in motion in other types of cell. Myosin
urethra, bladder, blood vessels, and the arrector pili
comprises a family of ATP-dependent motor proteins
in the skin (in which it controls erection of body
and is best known for their role in muscle contraction
hair). Smooth muscle is responsible for the
and their involvement in a wide range of other eukaryotic
contractility of hollow organs, except the heart.
motility processes. They are responsible for actin-based
4. (a) Cardiac muscle fibres are striated but involuntary in motility. Therefore they acts as ATPase.
action while skeletal muscle fibres are striated but
14. (a) Troponin (having 3 subunits) is seen at regular
voluntary in action. intervals on tropomyosin. In the resting state a
5. (c) Striated muscle fibres bear striations in the form of subunit of troponin masks the binding sites for
alternate light and dark bands. myosin on the actin filaments.
6. (a) Striated muscles are cylindrical, syncytial and 15. (b) The sarcoplasmic reticulum tubules contain calcium
unbranched. ions which are released from the tubules on
7. (a) Actin is a protein that functions in the contractile stimulation by the impulse passing along the T
system of skeletal muscle, where it is found in the tubular network.
S-110 Biology
16. (b) A red muscles fibre is a muscle in which small dark known as innominate or pelvic bones), sacrum and
fibers predominate and myoglobin and mitochondria coccyx.
are abundant. Red muscle fibres contract and fatigue 25. (b) The five sacral vertebrae are fused in the adult,
more slowly than white fibres and generate ATP by forming one structure called the sacrum. It lies
aerobic catabolism of glucose and fats, utilizing between the innominate bones of the pelvic girdle.
myoglobin-bound O2.
26. (c) Each pectoral girdle has a glenoid cavity into which
17. (b) Fatigue (also called exhaustion, tiredness etc) is a
the head of humerus is articulated.
feeling of tiredness which is distinct from weakness,
and has a slow onset. They have physical or mental 27. (a) Long bones, like humerus, radius and ulna of forearm,
causes. Physical fatigue is the temporary inability of femur, tibia and fibula of shank in adult mammals
a muscle to maintain optimal physical performance, provides support.
and is made more severe by intense physical 28. (a) The number of floating ribs in human body is two
exercise. Mental fatigue is a temporary decrease in pairs. They are not connected to either the sternum
maximal cognitive performance resulting from of the cartilage of another rib. Their main function is
prolonged periods of cognitive activity. to protect the kidneys.
18. (d) The neuromuscular junction (NMJ) is the site of 29. (a) At the point of fusion of ilium, ischium and pubis,
communication between motor nerve axons and
there is a cavity called acetabulum to which the thigh
muscle fibres. It is composed of four specialized cell
bone articulates.
types: motor neurons, Schwann cells, muscle fibres
and the recently discovered kranocytes. 30. (a) Elbow joint is an example of hinge joint. The elbow is
19. (c) Mandible (also known as lower jaw) is the largest, a hinge joint; it can open and close like a door. Hinge
strongest and only movable bone of the face. joint is a form of diarthrosis (freely movable joint)
that allows angular movement in one plane only,
20. (b) Hyoid bone is a U-shaped bone seen below buccal
increasing or decreasing the angle between the bones
cavity. It lies at the base of the mandible
e.g. elbow joint, knee joint etc.
(approximately C3 ), where it acts as a site of
attachment for the anterior neck muscles. 31. (b) Four coccygeal vertebrae are fused to form one
21. (a) Hyoid is a horseshoe-shaped (or U shaped) bone coccyx.
situated in the anterior midline of the neck between 32. (d) Gliding joint permit only back and forth and side to
the chin and the thyroid cartilage. At rest, it lies at side movements. e.g. zygapophysis of adjacent
the level of the base of the mandible in the front and vertebrae.
the third cervical vertebra (C3) behind. The hyoid 33. (c) Cartilaginous joints are connected entirely by
bone provides attachment to the muscles of the floor cartilage (fibrocartilage or hyaline). These joints allow
of the mouth and the tongue above, the larynx below, more movement between bones than a fibrous joint
and the epiglottis and pharynx behind. The hyoid but less than the highly mobile synovial joint.
bone helps in tongue movement and swallowing. Cartilaginous joint is found in between the sternum
22. (d) Humerus is not a part of axial skeletal. It is a part of and the ribs in human. They also form the growth
appendicular skeletal. Humerus is the bone of the regions of immature long bones and the intervertebral
upper arm or forelimb, forming joints at the shoulder discs of the spinal column.
and the elbow.
34. (a) Pivot joint is a type of joint where one bone rotates
23. (c) The vertebral column, also known as the backbone
around another. Example includes the joint in our
or spine, is a bony skeletal structure found in
neck, which allows us to rotate our head left to right.
vertebrates. It is formed from individual bones called
vertebrae which house the spinal canal, a cavity that 35. (b) Gout is a sex controlled (autosomal) trait. Gout is the
encloses and protects the spinal cord. Vertebral accumulation of uric acid crystals in the region of
formula indicates the number of vertebrae in each joints which results in painfull movements.
segment of the spinal column; for humans it is C7, 36. (a) Myasthenia gravis is a chronic autoimmune disorder.
T12, L5, S5, Cd4 = 33. The letters C, T, L, S, and Cd It affects neuromuscular junction leading to fatigue,
denotes cervical, thoracic, lumbar, sacral, and weakening and paralysis of skeletal muscles.
coccygeal. 37. (a) All movements do not lead to locomotion.
24. (a) Part of the body having a single pair of bones is Locomotion is the voluntary movements resulting
pelvic girdle. Pelvic girdle, also called bony pelvis, is in a change in location.
a ring-like structure, located in the lower part of the 38. (c) Red muscle fibres are one of the two main types of
trunk. It connects the axial skeleton to the lower limbs. skeletal muscle, which contains abundant
The bony pelvis consists of the two hip bones (also mitochondria and myoglobin. Red muscle fibres
contract and fatigue more slowly than white fibres
EBD_7209
Hints & Solutions S-111

and generate ATP by aerobic catabolism of glucose 46. (a) Vertebral column has 12 thoracic vertebrate. The
and fats, utilizing myoglobin-bound O2. joints between adjacent vertebrae is cartilaginous
Sarcoplasmic reticulum is a system of membrane- joint which permits limited movements. Progesterone
bound tubules that surrounds muscle fibrils, is secreted by corpus luteum which supports
releasing calcium ions during contraction and pregnancy in females.
absorbing them during relaxation. 47. (b) (i). Isotropic bands are skeletal muscle cells that form
39. (d) Hyoid is a part of skull bones. Hyoid is a horse shoe the light bands (I bands) that contribute to the
shaped bone which supports tongue and provides striated pattern of the cells. During muscle
insertion to some tongue muscle. contraction, isotropic band does not get elongated.
40. (d) Smooth muscles are non-striated. Their activities are (iii). Muscle contraction is initiated by a signal sent
not under control of the nervous system and are by the central nervous system via a motor neuron.
therefore known as involuntary muscles. All striated
48. (c) The thick and thin filaments do not change length
muscles are not voluntary. Visceral muscles are
during muscle contraction.
located in hollow organs. They do not exhibit any
striation and are smooth in appearance. Cardiac 49. (d) Smooth muscles are not branched and cylindrical in
muscles are the muscles of heart. These muscles are appearance. Involuntary muscles are striated.
striated and involuntary in nature. 50. (d) Knee joint is an example of hinge joint. The knee
41. (a) (B) Thin filaments are firmly attached to 'Z' line. Z joint joins the thigh with the leg and consists of two
band is a thin membrane in a myofibril and seen articulations: one between the femur and tibia, and
on longitudinal section as a dark line in the one between the femur and patella. It is the largest
centre of the I band. The distance between Z joint in the human body. Hinge joint allows the
bands delimits the sarcomeres of striated movements only in a single plane. In this articular
muscle. end of one bone is deeper convex and that of other
(C) The central part of thick filament, not is deeper concave.
overlapped by thin filaments is called H zone. 51. (c) The phase of muscle contraction occurs when
42. (d) During muscle contraction, chemical energy is myosin binds and releases actin. Muscle contraction
changed into mechanical energy. is initiated by a signal sent by the central nervous
43. (b) Sphincters is a ring of muscle surrounding and system via a motor neuron. A motor neuron along
serving to guard or close an opening or tube like the with the muscle fibres connected to it constitutes a
anus or the openings of the stomach. These muscles motor unit.
do not fatigue during the life of an animal. 52. (a) Repeated activation of the muscles can lead to the
44. (a) (b) First vertebra is atlas. It is without the centrum accumulation of lactic acid due to anaerobic
and articulated to skull through occipital breakdown of glycogen in them, causing fatigue.
condyles for nodding movements. 53. (a) Painful inflammation of the synovial membrane of
the joints results in stiffening of joints and painful
(c) 11th and 12th pairs of ribs are called floating
movements Uric acid accumulation in the joints can
ribs.
lead to painful movement of joint.
(d) Glenoid cavity is the concavity in the head of 54. (b) Synovial fluid is a thick sticky fluid secreted by
the scapula that receives the head of the synovial membranes into the synovial cavity.
humerus to form the shoulder joint. Though the presence of synovial fluid is one of the
45. (d) (a) Myasthenia gravis is either an autoimmune or reasons behind the mobility of the joints, but the
congenital neuromuscular disease that leads most accurate reason is the arrangement of the bones
to fluctuating muscle weakness and fatigue. at the joint, the spheroidal ball-like end of one bone
(b) Gout is usually characterized by recurrent articulates here with the cup-shaped depression in
attacks of acute inflammatory arthritis-a red, another. This allows the bone with the ball head to
tender, hot, swollen joint. It is caused by be moved freely in many planes. Shoulder joints and
elevated levels of uric acid in the blood. The hip joints are the ball-and-socket joints.
uric acid crystallizes, and the crystals deposit 55. (c) Arthritis or inflammation of a joint makes the joint
in joints, tendons, and surrounding tissues. painful and may even immobilise the movements at
(c) Muscular dystrophy is a progressive the joint. This may result from a lack of the synovial
degeneration of skeletal muscles mostly due fluid at the joint. The ossification of the articular
to genetic disorder. cartilage, deposition of uric acid crystals in the joint
cavity or other changes at the joint.
S-112 Biology
56. (b) Clavicle and glenoid cavity is found in pectoral girdle. Compared to the tibia, the fibula is about the same
57. (b) Synovial joints are characterized by the presence of length, but is considerably thinner. The difference in
a fluid filled synovial cavity between the articulating thickness corresponds to the varying roles of the
surfaces of the two bones. Such an arrangement two bones; the tibia bears the body's weight from
allows considerable movement. These joints help in the knees to the ankles, while the fibula functions as
locomotion and many other movements. Ball and a support for the tibia. Phalanges are digital bones in
socket joint (between humerus and pectoral girdle), the hands and feet of most vertebrates. In primates,
hinge joint (knee joint), pivot joint (between atlas the thumbs and big toes have two phalanges while
and axis), gliding joint (between the carpals) and the other digits have three phalanges. The phalanges
are classed as long bones.
saddle joint (between carpal and metacarpal of
thumb) are some examples. 67. (b) 'I' represent temporal bone in the given figure. The
temporal lobe is one of the four major lobes of the
58. (b) Gliding joint is found between zygapophyses of the
cerebral cortex in the brain and located beneath the
successive vertebrae. In gliding joint articular ends
lateral fissure on both cerebral hemispheres of the
of two bones are either flat or slightly curved to allow
mammalian brain. The temporal lobes are involved in
sliding or gliding movement. Examples bones of palms
processing sensory input into derived meanings for
and sole, between pre-zygapophyses and post-
the appropriate retention of visual memories,
zygapophyses of vertebrae.
language comprehension, and emotion association.
59. (b) Clavicle and glenoid cavity belongs to pectoral girdle
68. (d) The label I, II, III, IV and e respectively are pubis,
rather than pelvic girdle. Hence, it is incorrectly
femur, tibia, ilium and sacrum.
paired.
69. (b) Shoulder joint (label II) and hip joint (label IV) are
60. (b) All the four joints given in column I are types of
examples of ball and socket joint (for more refer
synovial joints. Ball and socket joint is a type of
answer 70).
joint in which the ball-shaped surface of one rounded
bone fits into the cup-like depression of another 70. (b) The label I, II, III and IV respectively are humerus,
bone. The distal bone is capable of motion around radius, ulna and scapula.
an indefinite number of axes, which have one 71. (b) The elbow joint is a hinge joint. It is the joint between
common centre. It enables the bone to move in many the humerus and ulna.
planes (almost all directions). Hinge joint allows the 72. (b) In the given figure of human skeletal system, the
movements only in a single plane. In this particular marked bones (I, II, III and IV) are identified as radius,
end of one bone is deeper convex and that of other ulna, tibia and fibula respectively.
is deeper concave. In pivot joints, the axis of a convex Radius and ulna are two large bones of the forearm.
articular surface is parallel with the longitudinal axis Radius extends from the lateral side of the elbow to
of the bone. Saddle joints are similar to ball and the thumb side of the wrist and runs parallel to the
sockets joints but both ball and socket are poorly ulna, which exceeds it in length and size. Radius is a
developed and movements are comparatively less long bone, prism-shaped and slightly curved
free. It allows the bone with convex head to move in longitudinally. It articulates with the capitulum of
many directions. the humerus, and with the ulna at two points: the
61. (a) Tarsals (ankle), metatarsals (sole) and phalanges radial notch (lower) and the head (upper part) of the
(toes) are bones of hind limbs and carpel (wrist) a ulna. Ulna runs parallel to the radius, which is shorter
bone of forelimb. and smaller.
62. (d) All the characteristics of muscles are correctly The tibia and fibula are two bones of hind limbs.
matched with their appropriate terms. Tibia, also known as the shin bone, is the larger and
63. (b) A - III, B - I, C - IV, D - VI , E - VII stronger of the two lower leg bones. It forms the
64. (a) A - I, B - II, C - III, D - IV knee joint with the femur and the ankle joint with the
fibula and tarsus. The support and movement of the
65. (a) 8th, 9th and 10th pairs of ribs do not articulate directly tibia is essential to many activities performed by the
with the sternum but join the seventh rib with the legs, including standing, walking, running, jumping
help of hyaline cartilage. and supporting the body's weight. Fibula or calf bone
66. (c) In the given figure, fibula and phalanges are not is located on the lateral side of the tibia, with which
correctly labelled. The fibula is the long, thin and it is connected above and below. It is shorter, thinner
lateral bone of the lower leg. It runs parallel to the and slender.
tibia and plays a significant role in stabilizing the 73. (d) The label 'X' in the figure of actin filament is troponin.
ankle and supporting the muscles of the lower leg. Troponin is a complex of three regulatory proteins
EBD_7209
Hints & Solutions S-113

(troponin C, troponin I, and troponin T) that is integral 85. (d) All or none law is associated with skeletal muscle
to muscle contraction in skeletal muscle and cardiac fibre, neuron and cardiac muscle fibres. The all-or-
muscle, but not smooth muscle. Troponin is attached none law is the principle states that the strength by
to the protein tropomyosin and lies within the groove which a nerve or muscle fiber responds to a stimulus
between actin filaments in muscle tissue. In a relaxed is independent of the strength of the stimulus. If
muscle, tropomyosin blocks the attachment site for that stimulus exceeds the threshold potential, the
the myosin cross bridge, thus preventing contraction nerve or muscle fibre will give a complete response;
74. (b) Step A: Attachment of myosin head to actin forming otherwise, there is no response. It was first
cross bridge. established by the American physiologist Henry
Step B: Release of phosphate. Myosin changes Pickering Bowditch in 1871 for the contraction of
shape to pull actin. heart muscle.
Step C: Attachment of new ATP to myosin head. The 86. (b) Smooth muscles are long and uni-nucleated. They
cross bridge detaches. are found in urogenital tracts, digestive tract, lungs,
iris, blood vessel etc.
Step D: Splitting of ATP into ADP and Pi. Myosin
87. (c) Muscle of urethra is not exclusively supplied with
cocks into its high energy conformation.
involuntary muscles.
75. (a) Pelvic girdle consists of two coxal bones. Fusion of
bone 1(ilium), 2(pubis) and 3(ischium) are responsible 88. (c) Muscles of alimentary canal are smooth unstriated
for the formation of coxal bones. and are innervated by fibres of ANS. These muscles
are neurogenic.
76. (a) The correct labeling of marked parts (1, 2, 3 and 4) in
the given figure of right pectoral girdle and upper 89. (c) Fine motor control is accomplished by the presence
arm are respectively clavicle, scapula, humerus, and of smaller, more numerous motor units. Each motor
radius. unit requires an individual motor neuron.
90. (d) Tarsals, femur, metatarsal and tibia are directly
77. (d) 1, 2, 3 and 4 are respectively muscle fibre, sarcolemma,
contributing the movement when a cricket player is
blood capillary and fascicle. Each organized skeletal
chasing a ball in the field.
muscle in human body is made of a number of muscle
bundles or fascicles held together by a collagenous 91. (b) Central part of thick filament, not overlapped by thin
connective tissue layer called fascia. filaments is called the ‘H’ zone. ‘H’ zone is also called
78. (a) The structure marked as X is humerus. Humerus is Hensen’s Line.
the longest bone of upper extremity and is 92. (b) An eyelid is a thin fold of skin that covers and protects
characterized by presence of deltoid tuberosity for the eye. Eyelid muscles have thick fibre without
the attachment of muscles. myoglobin.
79. (d) Vertebral column is a sring like vertebrae which lies 93. (a) The correct organization of the structures from large
in the middorsal line of the neck and trunk. The correct to small is: muscle, muscle cells, myofibrils,
order of vertebral region from superior to inferior are sarcomeres, filaments.
Cervical-thoracic-lumbar-sacrum-coccyx. Muscle is a band or bundle of fibrous tissue that has
80. (a) The correct order that a motor nerve impulse travels the ability to contract, producing movement in or
when triggering a muscle contraction is: motor nerve maintaining the position of parts of the body. Muscle
® synaptic cleft ® sarcolemma ® sarcoplasmic cell is an elongated contractile cell that forms the
reticulum ® troponin. muscles of the body. Myofibril is any of the elongated
81. (d) A - 2F, B - tropomyosin, C - troponin contractile threads found in striated muscle cells.
Sarcomere is a structural unit of a myofibril in striated
The thin filaments of myofibril contain 2F actin and muscle, consisting of a dark band and the nearer
two filaments of tropomyosin protein along with the half of each adjacent pale band. A filament is a slender
troponin protein for masking binding sites for thread-like object or fibre, especially one found in
myosin. animal or plant structures.
82. (c) The number of bones of ankle and wrist are not the 94. (d) Muscle contraction is triggered by a nerve releasing
same as ankle have 7 bones and wrists have 8 bone. a neurotransmitter, which in turn triggers the
83. (d) Both of these characteristics are important for the sarcoplasmic reticulum to release calcium ions into
function of cardiac muscle. the muscle interior where they bind to troponin, thus
84. (d) Ball and socket joints have one end like a ball and causing tropomyosin to shift from the face of the
other like a cup shaped socket. In ball and socket actin filament to which myosin heads need to bind
joint, convexity of one bone articulate with concavity to produce contraction.
of other bone. Example shoulder joint and hip joint.
S-114 Biology
95. (b) The joint between shoulder (scapula) and the upper nervous system. Afferent neurons communicate with
arm (humerus) forms the ball and socket joint. specialized interneurons.
96. (c) Events i, ii, iv and v only occurs during muscle 2. (a) Hypothalamus is the controlling centre of autonomic
contraction. Concentration of myoglobin, red nervous system. Hypothalamus lies at the base of
coloured oxygen storing pigment, in muscle fibres the thalamus and contains a number of centres which
increases. control body temperature, urge for eating & drinking.
97. (c) The striations th at give skeletal muscle its 3. (c) A bipolar neuron has one dendrite and one axon. It
characteristic striped appearance are produced by is found in the retina of eye.
arrangements of myofilaments. 4. (b) The ionic concentration gradients across the resting
98. (c) If the sarcoplasmic reticulum of the muscle fibres is membrane are maintained by active transport of ions
damaged, the exposure of myosin binding sites on by sodium-potassium pump which transports 3 Na+
the actin will be affected. outwards for 2 K+ into the cell.
99. (c) Synovial fluid is a transparent viscid lubricating fluid 5. (a) During conduction of nerve impulse, Na+ moves into
secreted by a membrane of an articulation, bursa, or the axoplasm. Axoplasm is a jelly like substance that
tendon sheath. Synovial fluid is necessary for normal contains both inorganic and organic matter in a axon.
joint function. For an inactive neuron, the axoplasm has an overall
100. (c) Skeleton is the body part that forms the supporting negative charge. This is because proteins, amino
structure of an organism. Connecting muscles to joint acids, phosphates, and other negatively-charged
is not a function of skeleton. It is a function of tendon entities inside the cell cannot cross the selectively-
which is a flexible but inelastic cord of strong fibrous permeable cell membrane. Two types of positively-
collagen tissue which attaches muscle to a bone. charged ions, potassium (K+) and sodium (Na+), can
101. (d) Scapula (X) is a large triangular flat bone situated in cross the cell membrane through selective ion
the dorsal part of the thorax between the second (Y)
channels. Normally there are more potassium ions
and the seventh ribs.
inside the cell than outside, whereas there are more
102. (b) Decreased levels of estrogen is a common cause of
sodium ions outside the cell than inside.
osteoporosis. Osteoporosis is a condition in which
the bones become brittle and fragile from loss of 6. (b) Axon is a long fibre which transmits impulses away
tissue, typically as a result of hormonal changes or from the cell body.
deficiency of calcium or vitamin D. Osteoporosis is 7. (d)
characterized by low bone mass and deterioration of 8. (a) During recovery, a nerve fibre becomes positively
bone tissue. This leads to increased bone fragility charged outside and negatively charged inside.
and risk of fracture (broken bones), particularly of 9. (a) Sodium, calcium and potassium ions are required for
the hip, spine, wrist and shoulder. Osteoporosis is nerve conduction.
often known as "the silent thief" because bone loss 10. (a) Resting membrane potential is the potential
occurs without symptoms. difference existing in a resting neuron (unstimulated
103. (d) Red muscle fibre (or Type I, slow twitch muscle fibre) neuron). During resting condition, the concentration
is dense with capillaries and is rich in mitochondria of K+ ions is more inside the axoplasm while the
and myoglobin, giving the muscle tissue its concentration of Na + ions is more outside the
characteristic red color. It can carry more oxygen and axoplasm. As a result, the potassium ions move faster
sustain aerobic activity using fats or carbohydrates from inside to outside as compared to sodium ions.
as fuel. Slow twitch fibers contract for long periods Therefore, the membrane becomes positively charged
of time but with little force. outside and negatively charged inside. This is known
White muscle fibre (or Type II, fast twitch muscle as polarization of membrane or polarized nerve.
fibre) contract quickly and powerfully but fatigue 11. (c) The resting membrane potential of a neuron is about
very rapidly, sustaining only short, anaerobic bursts -70 mV. This means that the inside of the neuron is 70
of activity before muscle contraction becomes mV less than the outside. At rest, there are relatively
painful. They contribute most to muscle strength more sodium ions outside the neuron and more
and have greater potential for increase in mass. Type potassium ions inside that neuron.
II is anaerobic, glycolytic, "white" muscle that is least 12. (c) The transmission of impulse through neurons is a
dense in mitochondria and myoglobin. electro-chemical phenomenon.
13. (c) The resting potential occurs because of the different
Chapter 21 : Neural Control and Coordination concentrations of ions across the cell. The electrical
1. (c) Afferent neurons (also known as sensory, receptor potential difference across the resting plasma
neurons and afferent axons) carry nerve impulses membrane is called as the resting potential. In the
resting nerve fibres, the cytoplasm just beneath its
from receptors or sense organs toward the central
EBD_7209
Hints & Solutions S-115

membrane is electronegative relative to the layer of 25. (b) The meninges are three layers of protective tissue
extracellular fluid just outside the membrane. called the dura mater, arachnoid mater, and pia mater
14. (c) Each cerebral hemisphere is divided into two regions that surround the brain and spinal cord. The meninges
- the outer region and the inner region. The outer of the brain and spinal cord are continuous, being
region of cerebrum contains grey matter and is linked through the magnum foramen. The arachnoid
known as cerebral cortex, which contains cell bodies or arachnoid mater is the middle layer of the
of the neuron. The inner region of cerebrum contains meninges. In some areas, it projects into the sinuses
white matter and is known as cerebral medulla, which formed by the dura mater and transfer cerebrospinal
contains nerve fibres or axons of the neurons. fluid from the ventricles back into the bloodstream.
15. (d) Medulla oblongata helps in regulation of respiration, 26. (b) Hindbrain is composed of the pons, cerebellum and
heartbeat, blood pressure, circulation, peristaltic medulla oblongata. The hindbrain is one of the three
movements etc. major developmental divisions of the brain; the other
16. (b) Olfactory lobes receive the sensation of smell but two are the midbrain and forebrain. Gastric secretion,
relay the same to temporal lobe of cerebrum. cardiovascular reflexes, respiration was regulated by
17. (c) Purkinje cells are large neurons with many branching the medulla oblongata. Located in the brainstem,
extensions that is found in the cortex of the anterior to the cerebellum, the medulla oblongata is
cerebellum. It plays an important role in controlling a cone-shaped neuronal mass in the hindbrain,
motor movement. These cells were first discovered controlling a number of autonomic functions.
in 1837 by Czech physiologist Jan Evangelista 27. (b) Reflex action is the rapid, involuntary and
Purkinje. They are characterized by cell bodies that unconscious actions of the body brought about by
are flask - like in shape, by numerous branching any part of the CNS through sudden stimulation from
dendrites, and by a single long axon. Most Purkinje receptors. Shivering in cold is not a reflex action.
cells release a neurotransmitter called GABA which 28. (a) The rods contain the rhodopsin pigment that is
exerts inhibitory actions on certain neurons and highly sensitive to dim light. It is responsible for
thereby reduces the transmission of nerve impulses. scotopic (twilight) vision.
29. (d) Fovea is a small depression in the centre of macula
18. (a) Nuclei are areas of grey matter within the white matter,
lutea. It has only cone cells. They are devoid of rod
where nerve impulses are processed.
cells. Hence, fovea is the place of most distinct vision.
19. (d) Cerebellum (the second largest part of brain) helps
30. (d) The spot at the back of the eye, from where optic
in co-ordination of muscular activities.
nerve fibres leave is free from rods & cones. This
20. (c) Pons is a band of nerve fibres that lies between spot is devoid of the ability for vision and is called
medulla oblongata and midbrain. It connects the blind spot.
lateral parts of cerebellar hemisphere together. It 31. (c) The amount of light that falls on retina is regulated
carries impulses from one hemisphere of the by iris. Iris is the anterior part and lies behind the
cerebellum to another. cornea.
21. (b) 32. (c) For seeing the nearby objects, the lens becomes more
22. (a) Corpus callosum is a thick band of nerve fibres that convex by contraction of ciliary muscles. The ciliary
divides the cerebrum into left and right hemispheres. muscle is a ring of smooth muscle in the eye's middle
It connects the left and right sides of the brain layer (vascular layer) and controls accommodation
allowing for communication between both cerebral for viewing objects at varying distances and
hemispheres. The corpus callosum transfers motor, regulates the flow of aqueous humour into
sensory, and cognitive information between the brain Schlemm's canal. It changes the shape of the lens
hemispheres and also involved in several functions within the eye, not the size of the pupil which is
of the body including communication between brain carried out by the sphincter pupillae muscle and
hemispheres, eye movement, maintaining the balance dilator pupillae.
of arousal and attention, and tactile localization. 33. (a) Space between cornea and lens is called aqueous
23. (c) Oligodendrocytes in the CNS are analogous to chamber. The anterior chamber is the fluid-filled
Schwann cells in the PNS and form the myelin sheaths space between the iris and the cornea's innermost
around axons in the CNS. Astrocytes are supporting surface, the endothelium. The anterior chamber is
cells and may function in the transport of materials filled with a watery fluid known as the aqueous
from blood vessels to neurons. Microglia are believed humor, or aqueous. It is produced by a structure
to play a phagocytic role in the CNS. alongside the lens (called the ciliary body). The
24. (c) Pons varolii is situated in front of the cerebellum aqueous passes first into the posterior chamber
below the mid brain and above the medulla (between the lens and iris) and then flows forward
oblongata. The pons is concerned with the through the pupil into the anterior chamber of the
maintenance of normal rhythm of respiration. eye.
S-116 Biology
34. (d) Refer answer 30. interpreted. Two of the three fluid sections are canals
35. (c) Cornea is a transparent anterior portion of eye that and the third is a sensitive 'organ of Corti' which
lacks blood vessels and is nourished by lymph from detects pressure impulses which travel along the
the nearby area. auditory nerve to the brain. The two canals are called
36. (a) Eye movement refers to the voluntary or involuntary the vestibular canal and the tympanic canal.
movement of the eyes. It helps in acquiring, fixating 49. (b) Ear ossicles are three small bones - the incus (anvil),
and tracking visual stimuli. Eye ball is moved in the malleus (hammer), and stapes (stirrup) that lie in the
orbit by four rectus and two oblique muscles. mammalian middle ear, forming a bridge between the
37. (c) Refer answer 30. tympanum (eardrum) and the oval window.
38. (b) Ciliary body is a part of uvea. It is a circular structure 50. (d) Reissner's membrane (also called vestibular
just behind the iris composed of the ciliary muscle membrane) is a thin membrane inside the cochlea of
and ciliary processes which attach to the lens. The the inner ear. It separates scala media from scala
ciliary processes secrete the aqueous fluid, and the vestibuli. Together with the basilar membrane it
ciliary muscle modify focus by changing the shape creates a compartment in the cochlea filled with
of the lens. endolymph, which is important for the function of
39. (b) The cones contain the iodopsin pigment (visual the organ of Corti.
violet) and are highly sensitive to high intensity light. 51. (b) In static condition, the body balance is sensed by
They are responsible for photopic (daylight) vision macula. Macula provide information on head position
and colour visions. (static equilibrium), as well as linear acceleration and
40. (c) Cochlea of (auditory region of internal ear) is a long deceleration, a type of dynamic equilibrium. The
coiled tubular and blind outgrowth of sacculus. It macula consist of hair cells with hair bundles and
consists of perilymph and endolymph. supporting cells.
41. (c) Scala vestibuli is the lymph-filled spirally arranged 52. (b) An electrical synapse is a mechanical and electrically
canal, present in the bony canal of the cochlea. It is conductive link between two adjoining neurons that
separated from the scala media below by the is formed at a narrow gap between the pre- and
vestibular membrane and connected with the oval postsynaptic neurons known as a gap junction.
window and receives vibrations from the stapes. Electrical synapses are rare in our neural system.
42. (d) Organ of Corti consists of row of sensory hair cells. Hypothalamus contains a number of centres which
The hairs of these cells project upwards and lie in control body temperature, urge for eating and
close contact with tectorial membrane, which projects drinking. It also contains several groups of
above them. neurosecretory cells which secretes hormone called
43. (d) Macula (receptors on the utriculus and sacculus) hypothalamic hormones. Thalamus is a major
are the organs of equilibrium and posture of body.
coordinating centre for sensory and motor signalling.
They are bathed in endolymph.
The tracts of nerve fibres that connect two cerebral
44. (b) Labyrinth is divided into bony labyrinth and a
hemispheres are called corpus callosum.
membranous labyrinth. Bony labyrinth is filled with
53. (c) The space between the cornea and the lens is called
perilymph while membranous labyrinth is filled with the aqueous chamber and contains a thin watery
endolymph. fluid called the aqueous humor. When all the cones
45. (c) The sense of equilibrium is determined by sensory are stimulated equally, a sensation of white light is
cristae of ampulla. The crista ampullaris is the sensory produced. The anterior transparent portion of sclera
organ of rotation located in the semi-circular canal is called cornea.
of the inner ear. The function of the crista ampullaris 54. (b) The photopigments of the human eye are composed
is to sense angular acceleration and deceleration. of opsin (protein) and retinal. Upon light absorption,
46. (a) The receptor cells for balance in human ear are opsins can change their conformation from a resting
located in utricle, saccule and semi-circular canal. state to a signalling state. Then it activates the G
47. (c) Organ of Corti is found in scala media. It consists of protein, thereby resulting in a signalling cascade that
row of sensory hair cells. The hairs of these cells produces physiological responses. Retinal, also
project upwards and lie in close contact with tectorial called retinaldehyde or vitamin A aldehyde, is a
membrane, which projects above them. polyene chromophore which is bound to proteins
48. (c) The cochlea is the hearing or auditory portion of the called scotopsins and photopsins, and is the chemical
inner ear. It is a spiral-shaped cavity in the bony basis of animal vision.
labyrinth. The cochlea receives sound in the form of 55. (a) The cerebellum (also called little brain) is a structure
vibrations, which cause the stereocilia to move. The located at the back of the brain, underlying the
stereocilia then convert these vibrations into nerve occipital and temporal lobes of the cerebral cortex. It
impulses which are taken up to the brain to be contains over 50% of the total number of neurons in
EBD_7209
Hints & Solutions S-117

the brain. The cerebellum is involved in the perilymph filled bony labyrinth into an upper scala
maintenance of balance and posture, coordination vestibuli and a lower scala tympani.
of voluntary movements, motor learning and 67. (d) The axonal membrane of the neuron is more
cognitive functions. permeable to potassium ions (K+) and nearly
56. (b) Conditioned reflexes are not present at birth but impermeable to sodium ions (Na+). In a resting state
acquired as the result of experience. When an action neuron does not conduct any impulse. In the resting
is performed repeatedly, the nervous system learns state the period during which a neuron is not
to react automatically. conducting the fluids outside the cell membrane carry
57. (c) Statements (i), (ii) and (iii) are the main functions of a relatively high positive charge. The fluids inside
cerebrum. Cerebrum is the largest and most highly the cell membrane carry a less positive, or relatively
developed part of the human brain. The outer portion negative, charge.
of the cerebrum is covered by a thin layer of gray 68. (b) Neurotransmitters are the chemicals which allow the
tissue called the cerebral cortex. It is divided into transmission of signals from one neuron to the next
right and left hemispheres that are connected by the across synapses. They are also found at the axon
corpus callosum. endings of motor neurons, where they stimulate the
(iv) Cerebrum controls the hearing and sense of smell muscle fibres. These chemicals stored in the synaptic
through the temporal lobe. vesicles (or neurotransmitter vesicles) are released
58. (a) Action potentials are formed when a stimulus causes at the synaptic cleft. The release is regulated by a
voltage-dependent calcium channel. Vesicles are
the cell membrane to depolarize past the threshold
essential for propagating nerve impulses between
of excitation, causing all sodium ion channels to
neurons and are constantly recreated by the cell.
open. During the propagation of a nerve impulse, 69. (c) The imbalance in concentration of Na+, K+ and
the action potential results from the movement of proteins generates resting potential. The electrical
Na+ ions from the extracellular fluid to intracellular potential difference across the resting plasma
fluid. membrane is called as the resting potential. In the
59. (d) The all-or-none law is the principle states that the resting nerve fibres, the cytoplasm just beneath its
strength by which a nerve or muscle fibre responds membrane is electronegative relative to the layer of
to a stimulus is independent of the strength of the extracellular fluid just outside the membrane. If the
stimulus. If that stimulus exceeds the threshold two sides of the membrane are connected by
potential, the nerve or muscle fibre will give a galvanometer, the inner side is seen to possess a
complete response, otherwise, there is no response. negative potential of about 70 mV relative to the
60. (b) Stapes is one of the three ear essicles. It is attached outside. This is called resting membrane potential.
to the oval window of cochlea. It is homologus to To maintain the unequal distribution of Na+ and K+,
Columella auris. the neurons use chemical energy in the form of ATP to
61. (c) The statements (i), (ii) and (iii) are correct while the actively transport Na+ out of the cell and move K+ in.
statement (iv) is incorrect. The peripheral nervous 70. (b) Astigmatism is a kind of defect of vision in which
system consists of nerves carrying impulses to and the image of an object is distorted. It is because all
from brain and spinal cord. the light rays do not come to focus on retina. It is
62. (c) Conduction of impulses is very slow in unmyelinated due to abnormal curvature of the lens . It can be
fibres. corrected by wearing cylindrical lenses.
63. (b) Ions only flow directly from cell to cell in an electrical 71. (b) Hind brain consists of cerebellum located dorsally
synapse. medulla oblongate and ponsvarolii. It contains
64. (d) Rods and cones are light receptors present in retina. centres for maintenance of posture and equilibrium
Rods are absent in the fovea while cones are of the body and for the muscle tone. All activities of
numerous in the fovea. the cerebellum are involuntary but may involve
65. (c) The external ear (pinna) collects sound waves and learning in their early stages.
directs them to the ear drum. The membranous canals 72. (b) The brain stem consists of pons varoli, medulla
are suspended in the perilymph of the bony canals. oblongata, mid brain and diencephalon. The brain
The malleus is attached to the tympanic membrane and stem is the connection between brain and spinal cord.
the stapes is attached to the oval window of the cochlea. It contains centres for controlling many vital activities
66. (a) The vestibular apparatus is composed of three semi- like respiration, body temperature, urge for eating and
circular canals and the otolith organ consisting of drinking etc. It also carries nerve tracts between the
the saccule and utricle. The Eustachian tube helps spinal cord and the higher brain structure.
in equalizing the pressures on either sides of the ear 73. (b) The axon terminal of the neuron contains many
drum. The membranes constituting cochlea, the membrane bound vesicles called synaptic vesicles, in
reissner's and basilar, divide the surrounding its cytoplasm. Within these vesicles, chemical
S-118 Biology
substances such as adrenaline and acetylcholine 79. (a)
remain stored. These chemicals are called
neurotransmitters, because they help to transmit nerve 80. (a) Semi-circular canal is interconnected tubes located
impulses across the synapses. When a nerve impulse inside each ear. They are lined with cilia and filled
passes the axon terminal, its synaptic vesicles release with a fluid known as endolymph. With every
their stored chemicals to the synaptic cleft. These movement of head the endolymph moves the cilia.
diffuse through the cleft to reach the membrane of the This works as a type of motion sensor, as the
next neuron, stimulating the latter. This causes the movements of the cilia are communicated to the brain.
nerve impulse to be transmitted along the next neuron. The vestibule is the central part of the osseous
74. (c) Hypothalamus is the part of the forebrain that labyrinth and responds to gravity and movements
controls involuntary functions such as hunger, thirst, of head. Cochlea is the auditory portion of the inner
sweating, sleep, fatigue, sexual desire, temperature ear. It is a spiral-shaped cavity in the bony labyrinth.
regulation, etc. Perilymph is an extracellular fluid located within the
75. (b) The medulla oblongata is the part of the brainstem cochlea (part of the inner ear) in two of its three
that is situated between the pons and the spinal cord. compartments: the scala tympani and scala vestibuli.
The medulla contains the cardiac, respiratory, The major cation (positively charged ion) in perilymph
vomiting and vasomotor centres and therefore deals is sodium. Endolymph is the fluid contained in the
with the autonomic (involuntary) functions of membranous labyrinth of the inner ear.
breathing, heart rate and blood pressure.
81. (c) Optic chiasma is not the feature of retina.
Hypothalamus is responsible for temperature
regulation. 82. (b) Sclera the external layer composed of dense
76. (a) Cerebrum controls vision and hearing. Cerebellum connective tissue. It is the tough, opaque tissue that
maintains body posture. Hypothalamus controls the serves as protective outer coat for eye’s Choroid
pituitary gland. Medulla oblongata controls the rate contains many blood vessels and looks bluish in
of heart beat. colour. Pupil is a hole located in the centre of the iris
77. (b) Cone cells are responsible for colour vision. They of the eye that allows light to strike the retina. It
function best in relatively bright light, as compared appears black because light rays entering the pupil
to rod cells that work better in dim light. Cone cells are are either absorbed by the tissues inside the eye
densely packed in the fovea centralis, a rod-free area directly, or absorbed after diffuse reflections within
with very thin, densely packed cones which quickly
the eye that mostly miss exiting the narrow pupil.
reduce in number towards the periphery of the retina.
Accommodation of vision occurs by contraction and Fovea is a portion of retina where only the cones are
relaxation of ciliary muscles. Pupil determines the densely packed. Iris is the visible coloured portion
amount of light that enters the eye. Night blindness of the eye.
occurs due to vitamin A deficiency. It is due to a 83. (d) Association areas are the regions of the cerebral
disorder of the cells in the retina that are responsible cortex of the brain connected by numerous nerve
for vision in dim light. Binocular vision is a vision fibres to all parts of both cerebral hemispheres and
using two eyes with overlapping fields of view, coordinating such higher activities as learning and
allowing good perception of depth. Iris is responsible reasoning. Association areas are responsible for
for controlling the diameter and size of the pupil and functions like intersensory associations, memory and
thus the amount of light reaching the retina.
communication.
78. (d) Difference between rods and cones
84 (c) Bipolar neurons is a type of neuron which has two
Features Rods Cones extensions - one axon and one dendrite. Bipolar cells
i Dis trib u tio n M o re n umero us Les s nu merou s , are specialized sensory neurons for the transmission
and u niformly dis p ers ed and of special senses. As such, they are part of the
dis trib u ted tig htly p acked sensory pathways for smell, sight, taste, hearing and
in the retina tog ether at vestibular functions. They are found in the retina of
fov ea eye.
to giv e a s h arp 85. (a) A reflex arc is a neural pathway that controls an action
imag e reflex. It is a rapid, automatic response to a stimulus.
ii Vis u al acu ity Lo w High
Some (but not all) reflexes are subconscious, and
iii Vis u al p igment Rho do ps in o r Iod o ps in some (but not all) can be consciously controlled or
co ntained vis u al p urple inhibited. The parts marked as A to G in the given
figure are respectively sense organ, sensory nerve,
iv Ov er all Sp ecialis ed Sp ecialized fo r dorsal horn, interneuron, ventral horn, motor nerve
fu n ctio n fo r lig ht vis ion colo ur vis ion and effector.
EBD_7209
Hints & Solutions S-119

86. (b) 92. (a) The label X represents Node of Ranvier. It speeds
up the impulse transmission.
Sympathetic 93. (b) The label II represents tympanic membrane.
Parasympathetic
Features nervous Tympanic membrane converts sound waves into
nervous system
system mechanical vibrations.
(a) Salivary glands Inhibits Stimulates The structure I, II, III and IV are external auditory
secretions secretions canal, tympanic membrane, cochlea and eustachian
(b) Pupil of eye Dilates Constricts tube respectively.
(c) Heart rate Increases Decreases 94. (a) Semi-circular canal (A) - Contains receptor for
strength and strength balance
rate of heart and rate of Ear ossicles (B) - Increases the magnitude of
vibrations
beat heart beat
Pinna (C) - Collects sound waves
(d) Intestinal Inhibits Stimulates 95. (b) The label X represents sclera. The white of the eye is
peristalsis secretions secretions called sclera. Choroid is responsible for eye colour.
Ciliary muscles alters the shape of the lens during
87. (b) In the given figure, stage of resting potential is
accomodation.
observed. The electrical potential difference across 96. (d) The label IV represents basilar membrane. The
the resting plasma membrane is called as the resting movement of the basilar membrane is most important
potential. In the resting nerve fibres, the cytoplasm to hearing. Once the hair cell microvilli bend, the hair
just beneath its membrane is electronegative relative cells depolarize, thus inducing action potentials in
to the layer of extracellular fluid just outside the the cochlear nerve.
membrane. 97. (a) The label X represents to semi-circular canals of the
88. (a) In the given figure of cochlea, the correct labelling inner ear. The three semi-circular canals are involved
of A, B, C and D are respectively perilymph, tectorial in balance, primarily detecting movements of the
membrane, endolymph and organ of corti. head. The arrangement of the three semi-circular
The cochlea is the auditory portion of the inner ear. canals allows one to detect movements in all
It is a spiral-shaped cavity in the bony labyrinth and directions.
receives sound in the form of vibrations, which cause 98. (d) Nodes of Ranvier (4) are the gaps formed between
the stereocilia to move. The stereocilia then convert the myelin sheaths generated by different cells. A
these vibrations into nerve impulses which are taken myelin sheath is a multi-layered coating and largely
up to the brain to be interpreted. Two of the three composed of a fatty substance called myelin that
fluid sections are canals and the third is a sensitive wraps around the axon of a neuron and very
'organ of Corti' which detects pressure impulses efficiently insulates it. At nodes of Ranvier, the axonal
which travel along the auditory nerve to the brain. membrane is uninsulated and, therefore, capable of
The two canals are called the vestibular canal and generating electrical activity.
the tympanic canal. 99. (b) Part marked as 2 is axon. Axon (also known as nerve
89. (a) In the given figure of axon terminal and synapse, the fibre) is a long, slender projection of a nerve cell, or
correct labelling of A, B, C and D are respectively neuron that typically conducts electrical impulses
synaptic vesicles, axon terminal, synaptic cleft and away from the neuron's cell body. The function of
postsynaptic membrane. the axon is to transmit information to different
90. (b) In the given figure of human eye, the part marked as neurons, muscles and glands.
"X" is iris. The iris is a thin, circular structure in the 100. (c) The parts labelled as 4, 5 and 6 are respectively
pre-synaptic membrane, synaptic cleft and post-
eye which is responsible for controlling the diameter
synaptic membrane. These parts constitute the
and size of the pupil and thus the amount of light
structure of synapse. A synapse is a structure that
reaching the retina. The colour of the iris gives the
permits a neuron (or nerve cell) to pass an electrical
eye its colour.
or chemical signal to another cell (neural or
91. (c) In the given diagram of neuron, the part marked as I, otherwise). Synapses are essential to neuronal
II, III and IV are respectively dendrites, cell body, function. At a synapse, the plasma membrane of the
myelin sheath and nodes of Ranvier. Neuron is a signal-passing neuron (the presynaptic neuron)
specialized, impulse-conducting cell that is the comes into close apposition with the membrane of
functional unit of the nervous system, consisting of the target (postsynaptic) cell. Both the presynaptic
the cell body and its processes, the axon and and postsynaptic sites contain extensive arrays of
dendrites. molecular machinery that link the two membranes
together and carry out the signaling process. In many
S-120 Biology
synapses, the presynaptic part is located on an axon, 109. (d) If a person feels no sensation when he puts his hand
but some postsynaptic sites are located on a dendrite on a flame then the hypothalamus part of his brain is
or soma. damaged. Because hypothalamus contains a number
101. (b) The parts marked as 1, 2, 3 and 4 in the given figure of centres which control body temperature, urge for
of human eye are respectively cornea, iris, blind spot eating and drinking. It also contains several groups
and choroid. of neurosecretory cells which secretes hormone
Cornea (1), the anterior portion of sclera, is the called hypothalamic hormones.
transparent, dome-shaped window covering the front 110. (b) An axon has four terminals ends connected with
of the eye. It is a powerful refracting surface. Blind dendrites of four different neurons. Its nerve impulse
spot (3) is the area on the retina without receptors will travel in all the four neurons with equal strength.
that respond to light. Therefore an image that falls 111. (d) Brain is well protected by cranial meninges that are
on this region will not be seen. It is in this region that made up of an outer layer called dura mater, a thin
the optic nerve exits the eye on its way to the brain. middle layer called arachnoid, and an inner layer
Choroid (4) is the layer of blood vessels and called pia mater.
connective tissue between the sclera (white of the 112. (c) Conditioned reflexes are acquired as the result of
eye) and retina. experience. When an action is performed repeatedly,
the nervous system learns to react automatically.
102. (a) The marked label 1, 2, 3 and 4 are respectively
Walking, running, typing, riding a bicycle etc. are
Reissner's membrane, organ of corti, basilar
examples of learned conditioned activities.
membrane and tectorial membrane.
113. (d) The autonomic nervous system regulates the
Organ of corti (X) is a sensitive element in the inner
functions of internal organs (the viscera) such as
ear. It is a spiral shaped structure located on the the heart, stomach and intestines. It is divided into
basilar membrane (3) which contains hair cells that three parts: the sympathetic nervous system, the
acts as auditory receptors. The hair cells are present parasympathetic nervous system and the enteric
in rows on the internal side of the organ of corti. nervous system. The autonomic nervous system is
103. (a) For sound reflex actions, we require intact spinal most important in two situations: in emergencies that
cord. The spinal cord is the most important structure cause stress and require us to ‘fight’ or take ‘flight’
between the body and the brain. It extends from the (run away) and in non-emergencies that allow us to
foramen magnum where it is continuous with the ‘rest’ and ‘digest’.
medulla to the level of the first or second lumbar 114. (b)
vertebrae. It functions primarily in the transmission 115. (c) When a person who enters in empty room suddenly
of neural signals between the brain and the rest of finds a snake in front on opening the door, his
the body but also contains neural circuits that can sympathetic nervous system is activated and
independently control numerous reflexes and central releases epinephrine and norepinephrine from the
pattern generators. adrenal medulla.
104. (d) A primitive type of nervous system, without brain is 116. (b) The middle ear contains a flexible chain of three
found in the form of intraepidermal nerve net. middle bones called ear ossicles. The three ear
However, there is no brain in Hydra to coordinate
ossicles are malleus, incus, and stapes that are
the response.
attached to each other. The malleus is attached to
105. (a) Closing of eyes, if an object suddenly approached
tympanic membrane on one side and to incus on the
them, is an example of simple reflex.
106. (b) Paralysis involves the complete destruction of motor other side. The incus is connected with stapes.
nerves. Stapes, in turn, are attached with an oval membrane,
107. (c) Peristalsis of intestine is the example of autonomic fenestra ovalis, of internal ear. In ear, vibrations travel
nervous system. The autonomic nervous system through malleus, incus to stapes. The ear ossicles
(ANS) regulates the functions of internal organs (the increase the intensity of the sound waves.
viscera) such as the heart, stomach and intestines. It 117. (b) Iris is a thin, circular structure in the eye, responsible
is part of the peripheral nervous system and controls for controlling the diameter and size of the pupil and
some of the muscles within the body. thus the amount of light reaching the retina. The
Swallowing of food, pupillary reflex and knee jerk colour of the iris gives the eye its colour. In optical
response are the examples of reflex action. terms, the pupil is the eye's aperture and the iris is
108. (c) The ionic concentration gradients across the resting the diaphragm that serves as the aperture stop.
membrane are maintained by active transport of ions 118. (b) Cornea transplant is successful as cornea is without
by sodium-potassium pump which transports 3 Na+ blood supply. The cornea is the transparent, dome-
outwards for 2 K+ into the cell. Hence, if sodium shaped window covering the front of the eye. It is a
pump is blocked, sodium inside the nerve will powerful refracting surface, providing 2/3 of the eye's
increase. focusing power. Because there are no blood vessels
EBD_7209
Hints & Solutions S-121

in the cornea, it is normally clear and has a shiny tube) is a tube that links the nasopharynx to the
surface. middle ear. It is a part of the middle ear. It aerates the
119. (d) The described condition is case of myopia. In myopia middle ear system and clears mucus from the middle
(or near-sightedness), the eyeball is antero- ear into the nasopharynx.
posteriorly elongated so that the image of distant 128. (b) Hypothalamus is an important part of forebrain which
objects is formed in front of the retina. The defect lies at the base of the structure which is a major
can be removed by using concave glasses. coordinating center for sensory and motor signaling.
120. (d) In a similarity with photographic camera the retina It contains a number of centers which control body
acts as film. Retina is a light-sensitive layer of tissue, temperature, urge for eating and drinking.
lining the inner surface of the eye. The optics of the 129. (a) Impulse transmission across an electrical synapse is
eye create an image of the visual world on the retina always faster than that across a chemical synapse.
(through the cornea and lens), which serves much 130. (a) Unmyelinated nerve fibre is enclosed by an Schwann
the same function as the film in a camera. cells that does not form a myelin sheath around the
121. (b) Adaptation is the ability of the eye to adjust to axon, and is commonly found in autonomous and
various levels of darkness and light. the somatic neural systems.
122. (a) Layers in wall of eye balls from inside to outwards 131. (a) The afferent neuron receives signal from a sensory
are retina, choroid and sclerotic. Retina is a light- organ and transmits the impulse via a dorsal nerve
sensitive layer of tissue, lining the inner surface of root into the CNS. The efferent neuron then carries
the eye. The choroid is the layer of blood vessels signals from CNS to the effector.
and connective tissue between the sclera (white of
the eye) and retina. It is part of the uvea and supplies Chapter 22 : Chemical Coordination and
nutrients to the inner parts of the eye. Sclerotic is Integration
related to sclera which is commonly known as "the 1. (d) Anterior lobe of pituitary secretes follicle stimulating
white of the eye." It is the tough, opaque tissue that hormones, growth hormone and luteinizing hormone.
serves as the eye's protective outer coat. Six tiny 2. (d) GH (growth hormone), secreted from anterior
muscles connect to it around the eye and control the pituitary gland, stimulates growth of body by
eye's movements. The optic nerve is attached to the
synthesis and deposition of protein in tissue,
sclera at the very back of the eye.
increase glucose level in blood by decreased
123. (b) The velocity of nerve impulse is approximately
secretion of insulin, increased cell division ad
100 m. per second and velocity of cardiac impulse is
increased growth of bones by increased absorption
about 0.6 m. per second.
of calcium from intestine. Hyper secretion of GH leads
124. (b) The highest and the lowest frequencies of sound
to acromegaly in adults which is characterised by
are recognized by the outermost and the innermost
gorilla like appearance with huge hand and legs.
coils of the cochlea, respectively.
125. (b) The pinna of the external ear collects the sound 3. (b) Adrenocorticotrophic hormone (ACTH) is secreted
waves and directs them to the tympanic membrane by anterior pituitary.
(ear drum) via the external auditory canal. The ear 4. (d) LH is a luteinizing hormone which stimulates the non
drum then vibrates the sound waves and conducts - empty follicle to develop into corpus luteum. Corpus
them to the internal ear through the ear ossicles. The luteum secretes progesterone hormone during the
ear ossicles increase the intensity of the sound latter half of the menstrual cycle which maintain
waves. These vibrating sound waves are conducted pregnancy.
through the oval window to the fluid in the cochlea. 5. (a) Oxytocin hormone is produced by posterior lobe of
Consequently, a movement is created in the lymph. pituitary under the influence of nervous stimulation.
This movement produces vibrations in the basilar It causes contraction of the smooth muscles of the
membrane, which in turn stimulate the auditory hair uterus and myoepithelial cells lining the duct of the
cells. These cells gen erate a nerve impulse, mammary gland. Hence it is also called as birth
conducting it to the auditory cortex of the brain via hormone and milk ejection hormone.
afferent fibres. The auditory cortex region interprets 6. (d) Ovulation (release of graafian follicle from the ovary)
the nerve impulse and sound is recognized. is controlled by FSH and LH.
126. (a) The correct sequence for pathway of light through 7. (b) FSH (Follicle Stimulating Hormone), secreted by
the eye to the retina is: anterior pituitary gland, is a gonadotrophic hormone.
conjunctiva - cornea - aqueous humor - pupil - lens - It stimulates spermatogenesis and growth of
vitreous humor - yellow spot. seminiferous tubules in testes in male and maturation
127. (c) The given features are related to Eustachian tube. of Graafian follicle and secretion of estrogen and
The Eustachian tube (also known as the auditory progesterone from corpus luteum in female.
S-122 Biology
8. (a) Melanocyte Stimulating Hormone (MSH) is secreted calcium; bones- release of calcium and small
by pars intermedia. MSH acts on melanocytes to intestine-absorption of calcium.
regulate pigmentation of skin. 22. (a) Thymus is related to AIDS as it the first developing
9. (b) Secretion of estrogen is stimulated by FSH of anterior lymphoid organ whose main function is to develop
pituitary gland. immature T cells into immunocompetent T cells. AIDS
10. (a) Deficiency of ADH (vasopressin) causes diabetes (acquired immune deficiency syndrome) is a disease
insipidus (increased volume of urine). It is caused in which there is a severe loss of the body's cellular
due to decreased reabsorption of water from immunity, greatly lowering the resistance to infection
collecting tubules. and malignancy.
11. (b) Melatonin, secreted by pineal gland, is called as anti- 23. (d) Thymus secretes a peptide hormone called
ageing hormone. “thymosins” that imparts resistance to diseases.It
12. (b) Melatonin regulates the diurnal i.e. 24-hour rhythm helps in differentiation of T-lymphocytes, which
of body. e.g. maintenance of sleep-wake cycle, body provide cell mediated immunity.It promotes
temperature etc. It influences metabolism, production of antibodies for humoral immunity.
pigmentation and menstrual cycle. 24. (b) Deficiency of aldosterone and cortisol causes
13. (a) ADH, secreted by posterior pituitary gland, acts on Addison’s disease. It is characterized by bronze-like
kidney. It concentrates the urine by promoting the skin colouration, hyponatraemia, hyperkalaemia, low
reabsorption of water and salts into the cortical blood sugar, nervous depression, nausea, vomiting,
collecting ducts. Therefore, when the amount of ADH weakness and diarrhoea.
decreases in blood, micturition will increase. 25. (a) Cortisol is secreted by the middle region of adrenal
14. (a) ADH, secreted by posterior pituitary gland, acts on cortex. It increases the blood glucose level (which is
kidney. It concentrates the urine by promoting the anti-insulin effect) by converting proteins & fats into
reabsorption of water and salts into the cortical glucose.
collecting ducts. 26. (d) Adrenal gland (adrenaline hormone) is also called as
15. (b) ADH, secreted by posterior pituitary gland, acts on triple F gland for flight, fright and fight reaction which
kidney. It concentrates the urine by promoting the occurs in condition of emergency. Hence it is also
reabsorption of water and salts into the cortical called as emergency hormone.
collecting ducts. Therefore, when the amount of ADH 27. (b) Insulin is a hypoglycemic factor, secreted by b- cells.
decreases in blood, micturition will increase with It stimulates glycogenesis (conversion of glucose
increased water in urine. to glycogen).
16. (c) ADH (or vasopressin) is secreted by posterior 28. (b) Deficiency of ADH causes diabetes insipidus. It is
pituitary gland. It acts on kidney tubule and blood caused due to decreased reabsorption of water from
capillaries and concentrates the urine by promoting collecting tubules.
the reabsorption of water and salts into the cortical 29. (c) Glucagon and insulin regulates the level of blood in
collecting ducts. man. Both the hormone is secreted from alpha and
17. (c) In thyroid gland, thyroxine hormone is stored in beta cell of pancreas respectively. Glucagon is
extracellular spaces before discharging it into the secreted when the blood sugar level is low and it
blood. These hormones are stored in the follicular stimulates glycogen breakdown and glucose
cells in the form of colloidal suspension called synthesis in the liver by increasing blood glucose
thyroglobulin. concentration. Whereas rising level of blood glucose
18. (d) Thyrocalcitonin opposes parathormone. Calcitonin, stimulates insulin secretion.
secreted from parafollicular cells in thyroid gland, 30. (a) Insulin, glucagon and somatostatin are secreted from
plays a role in calcium and phosphorus metabolism. beta, alpha and delta cells of the pancreas
19. (c) Hypothyroidism of T3 and T4 during fetal life and respectively.
infancy leads to cretinism. It is a condition 31. (b) Leydig (interstitial) cells in the inter-tubular spaces
characterized by the retarded mental development, are stimulated to secrete male sex hormone called
stunted growth, delayed puberty, decreased heart androgens (mainly testosterone), under the influence
rate, pulse rate, blood pressure etc, reduced urine of ICSH of anterior pituitary.
output, pigeon’s chest etc. 32. (d) Corpus luteum secretes the hormone progesterone
20. (b) Thyroxine does not act on testis. It is a hormone which that supports pregnancy and stimulates mammary
is secreted from thyroid gland and plays an important glands for formation of alveoli for storing milk.
role in the body's metabolisms and calcium balance. 33. (d) During pregnancy corpus luteum persists up-to the
21. (d) All the organs or tissues listed are influenced by middle of pregnancy. Corpus luteum is a hormone-
parathyroid hormone like, kidneys- reabsorption of secreting structure that develops in an ovary after
EBD_7209
Hints & Solutions S-123

an ovum has been discharged in a process called Gastrointestinal tracts secretes four major peptide
ovulation. hormones – gastrin, secretin, cholecystokinin (CCK)
34. (d) Progesterone stimulates proliferation of endometrium and gastric inh ibitory peptide (GIP) while
of uterus and prepares it for implantation of juxtaglomerular cells of kidney secretes
blastocysts. erythropoietin, a peptide hormone. Releasing and
35. (c) Corpus luteum is a yellow, progesterone-secreting inhibitory hormones are produced by hypothalamus.
mass of cells that forms from an ovarian follicle after Neurohypophysis or posterior pituitary is under
the release of a mature egg. It prepares the inner wall direct neural regulation of the hypothalamus.
of the uterus for implanting the zygote (blastocysts) 47. (b) Thyrocalcitonin (TCT) is secreted by parafollicular
and maintains the pregnancy. cells of thyroid gland. It is a protein hormone that
36. (c) Secretin is a hormone which is released into the regulates (lowers) the blood calcium levels in blood
bloodstream by the duodenum under the influence plasma.
of stomach acid. It stimulates secretion of liver and 48. (c) Both enzymes and hormones are used in minute
pancreas. quantities.
37. (b) Secretin is a hormone which is released into the 49. (b) All the hormones secreted from pituitary gland are
bloodstream by the wall of the upper part of the small proteins or peptides.
intestine (the duodenum) under the influence of 50. (b) Norepinephrine is released by sympathetic fibres i.e,
stomach acid. It stimulates secretion of liver and rapidly secreted in response to stress of any kind
pancreas. and during emergency situations. It increases the
38. (a) Follicle Stimulating Hormone (FSH) produces its heart beat, the strength of heart contraction and the
effect by binding to its specific receptors present on rate of respiration.
the ovarian cell membrane. 51. (a) Neurohypophysis is a posterior part of pituitary
39. (b) In heart cells AMP acts as secondary messenger gland. It stores the hormones (ADH and oxytocin)
which is made in the response of adrenaline and it which is synthesised in the hypothalamus and
stimulates Ca 2+ ions to come out from th e releases them on their requirement.
sarcoplasmic reticulum of muscle fibres which causes 52. (a) Glucagon is secreted from the alpha cells of pancreas
muscle contraction. when the blood sugar level is low. It stimulates
glycogen breakdown and glucose synthesis in the
cGMP (Cyclic Guanosine Monophosphate) which
liver by increasing blood glucose concentration. It
acts as secondary messenger works on the action of
has opposite effect to that of insulin because insulin
acetylcholine, increase in flow of Ca2+ into muscle
is released when the level of blood sugar is high.
fibres & hence causes muscle relaxation. There is no
53. (a) The over secretion of androgenic corticoid, by
role of sodium in hormonal action.
adrenal cortex, in female causes adrenal virilism in
40. (d) Receptors for protein hormone are found on the cell which male type secondary sexual characters appear
surface. in female. In male, it causes gynaecomastia (enlarged
41. (b) Cortisol and testosterone are the steroidal hormones. breasts in male).
They can easily pass through both the plasma 54. (a) Endocrine glands regulate neural activity and
membrane and nuclear membrane. nervous system regulates endocrine glands.
42. (d) The sex hormones, glucocorticoids, and mineralo- 55. (a)
corticoids are steroid hormones. 56. (d) Hormones are chemical messengers of the body that
43. (b) Somatostatin also called growth inhibiting hormone transfers information from one set of cells to another.
inhibits release of growth hormone from pituitary. 57. (a) Based on the mode of secretion, the glands are of
44. (d) Thyrocalcitonin (TCT) is secreted by parafollicular three types : mesocrine, apocrine and holocrine.
cells of thyroid gland.
Mammary glands that are present in mammals to feed
45. (d) All the statements are correct regarding PTH. PTH
the young ones with milk are the example of apocrine
(Parathyroid Hormone), also called Collip's hormone,
glands. In apocrine glands, the secretion accumulates
is secreted from the parathyroid gland. It is
responsible for the following: as secretory granules in the distal part of the cell.
(i) Release of calcium by ones into the blood This part later breaks down and leaves as a secretion.
streams. 58. (a) Calcitonin or thyrocalcitonin is secreted by
(ii) Absorption of food by the intestine. parafollicular cells of thyroid stroma. It retards bone
(iii) Conservation of calcium by the kidneys. dissolution and stimulates excretion of calcium in
46. (b) Hormones are non-nutrient chemicals which act as urine. Thus, it lowers calcium level in extra cellular
intercellular messengers and are produced in trace fluid (ECF). Parathormone is secreted by chief cells
amounts. of parathyroid gland and is also known as Collip's
S-124 Biology
hormone. It maintains blood calcium level by flow and sodium to the kidney. Insulin is secreted
increasing its absorption from food in intestine and when the blood glucose level is high. Its deficiency
its reabsorption from nephrons in the kidney. will result in diabetes mellitus. Adrenaline is secreted
Maintenance of proper calcium level is in fact, a from adrenal medulla and acts as vasodilator.
combined function of parathormone and calcitonin. 65. (a) Hypothalamus releases gonadotrophic releasing
hormones. FSH and LH are gonadotrophic hormones,
When calcium level falls below normal parathormone
secreted from the anterior pituitary gland.
maintains it by promoting its absorption, Testosterone is the most abundant androgen released
reabsorption and also by demineralisation of bones. by the leydig cells. Estrogen is mainly secreted by
When blood calcium level exceeds above normal, follicular epithelial cells of granulosa membrane of
then calcitonin hormone increases excretion of Graafian follicle.
calcium in urine. 66. (a) FSH and LH are gonadotrophic hormones, secreted
59. (c) Vasopressin or antidiuretic hormone is secreted by from the anterior pituitary gland. Corpus luteum
posterior pituitary gland. The deficiency of secretes progesterone hormone. Estrogens are
vassopressin results in a disorder known as diabetes released from the ovarian follicle.
insipidus. The main symptoms of diabetes insipidus 67. (b) Glucagon is secreted by alpha cells.
are increase in thirst and increase in urination. 68. (c) Thyroid secretes thyroxine hormone and plays an
60. (d) Vasopressin or anti-diuretic hormone (ADH) reduces important role in the body's metabolisms and calcium
the volume of urine by increasing the reabsorption of
balance. Oxytocin is secreted from the posterior
water from the urine in the distal convoluted tubules,
collecting tubules and collecting ducts in the kidney. pituitary gland. It causes contraction of the smooth
It does so by rendering the walls of these tubules muscles of the uterus and myoepithelial cells lining
leads to diabetes insipidus (increased urination). the duct of the mammary gland. Hence it is also called
Although the volume of urine is increased. No glucose as birth hormone and milk ejection hormone. Corpus
appears in the urine of such patients. luteum secretes progesterone hormone which
Diabetes mellitus is a disease which is caused due to supports pregnancy.
the failure of insulin hormone secretion by the 69. (d) Over secretion of cortisol causes Cushing disease.
pancreatic islets. The osmotic effect of glucose in the This disease mainly occurs in females and causes
urine considerably increases the volume of urine, due obesity hypertension, glycosuria, etc.
to which thirst is also enhanced. In extreme cases, 70. (d) Glucagon is secreted from the alpha cells of pancreas
the patient suffers from coma and may die. when the blood sugar level is low. It stimulates
61. (a) Histamine is a derivative of the amino acid histidine glycogen breakdown and glucose synthesis in the
produced by damaged cells of vertebrates. When liver by increasing blood glucose concentration.
released, it has the effect of dilating capillaries and Whereas rising level of blood glucose stimulates
lowering blood pressure. Histamine is involved in insulin secretion from the beta cells of pancreas.
allergic and inflammatory reactions also. Leydig cells secretes testosterone hormone and
62. (b) Nor-epinephrine is secreted by adrenal medulla sertoli cells secretes inhibin.
region. It increases the heartbeat, rate of respiration 71. (a) Hormones and their appropriate categories are:
and alertness. A. FSH and LH- Gonadotropins
63. (b) Thymus starts functioning in the embryonic stage B. Cortisol- Glucocorticoids
itself, becomes active during childhood and C. Androgens- Gonadocorticoids
undergoes regression and gradually stops D. Aldosterone- Mineralocorticoids
functioning in old individuals. So, production of 72. (b)
thymosins decreases. As a result, immune responses 73. (b) Glands Their secretions (Hormones)
of old persons become weak. I. Pineal gland Melatonin
64. (c) ADH, secreted by posterior pituitary gland, acts on II. Thyroid gland Calcitonin and thyroxine
kidney. It concentrates the urine by promoting the III. Thymus Thymosin
reabsorption of water and salts into the cortical IV. Adrenal gland Aldosterone
collecting ducts. Its deficiency will results in diabetes V. Testis Testosterone
insipidus. ACTH (adrenocorticotrophic hormone) is 74. (b) The label X represents parathyroid gland. It helps to
secreted by anterior pituitary gland. It controls the restore lowered levels of calcium in blood.
structure and function of adrenal cortex especially Too much calcium in the blood is restored by
secretion of glucocorticoids and sex corticoids. calcitonin, secreted by thyroid gland.
Aldosterone is a type of mineralocorticoids. It is a Decreased level of blood sugar is restored by insulin,
salt retaining hormone which is secreted in response secreted by pancreas.
to increased potassium levels or decreased blood Excessive loss of sodium in extracellular fluid is
restored by aldosterone, secreted by adrenal gland.
EBD_7209
Hints & Solutions S-125

75. (b) The label X represents adrenal cortex and the 87. (b) Ptyalin is a salivary enzyme in the saliva that converts
hormone secreted by it is aldosterone. starch into dextrin and maltose.
The label Y represent adrenal medulla and the 88. (a) Gigantism and exophthalmic goitre are the disorders
hormone secreted by it is epinephrine. which occurs due to hyper secretion of growth
76. (d) The label X represents thyroid gland. The over- hormone (secreted from anterior pituitary gland) and
secretion of thyroid gland may lead to Grave’s thyroxine (secreted from the thyroid gland)
disease. Grave’s disease is categorized as an respectively.
autoimmune disorder. 89. (b) Hormones secreted from anterior pituitary glands
77. (b) The hypophyseal portal system transports releasing stimulates thyroid and other endocrine glands like
and inhibiting hormones from the hypothalamus into adrenal, parathyroid, ovary, testes, pancreas etc.
the anterior pituitary, which is labelled as II in the 90. (a) Growth hormone of pituitary is more effective in
given diagram. The label I represents hypothalamus, presence of thyroxine hormone.
label III represents posterior pituitary and label IV 91. (d) Adrenaline caused an increase in systolic blood
represents portal circulation. pressure, a decrease in diastolic blood pressure, and
78. (c) The given picture shows the disorders (gigantism an increase in heart rate. Therefore it is given to those
and dwarfism) which occurs due the abnormal patients who suffer from a marked fall in their blood
secretion of GH (growth hormones). GH is secreted pressure.
from the adenohypophysis (anterior pituitary gland). 92. (b) Carbohydrate metabolism represents the various
79. (a) (i) and (ii) are respectively pituitary gland and adrenal biochemical processes which is responsible for the
gland respectively. formation, breakdown and inter-conversion of
80. (a) The hormone representing by lines 1 and 2 are carbohydrates in living organisms. Hormones
respectively ACTH an d aldosterone. ACTH involved in this metabolism are insulin, glucagon,
(adrenocorticotrophic hormone) is secreted by epinephrine and glucocorticoids.
anterior pituitary gland. It controls the structure and 93. (d) Vasopressin and ADH are identical. Vasopressin or
function of adrenal cortex especially secretion of Anti-diuretic hormone (ADH) stimulates the
glucocorticoids and sex corticoids. Aldosterone is a reabsorption of water and electrolytes by DCT of
type of mineralocorticoids secreted from adrenal kidney and thereby reduces diuresis (loss of water
cortex. It is a salt retaining hormone which is secreted through urine).
in response to increased potassium levels or
94. (b) The prolonged hyperglycemia leads to diabetes
decreased blood flow and sodium to the kidney.
mellitus. Diabetes mellitus is defined as abnormal
81. (b) Hormones produce their effects by binding to the
high glucose level in blood, which results in release
specific receptors located in the target tissues.
of sugar in urine and formation of toxic ketone bodies.
82. (c) The number of hormones secreted by anterior
The individual suffering from diabetes mellitus drinks
pituitary are six- growth hormone, prolactin, TSH,
water frequently to remove sugar from blood.
ACTH, LH and FSH.
95. (a) Somatotropin or Growth Hormone (GH) is involved
83. (c) Aldosterone is a type of mineralocorticoids secreted
in growth and development of body. It targets the
from adrenal cortex. It is a salt retaining hormone
bone, muscle and adipose tissue.
which is secreted in response to increased potassium
96. (d) Glucagon is a hyperglycemic factor secreted by
levels or decreased blood flow and sodium to the
alpha-cells of pancreas. Its secretion is stimulated
kidney.
by low blood sugar level.
84. (a) Pancreas being an endocrine gland, releases insulin
97. (b) Hyperthyroidism is the over-activity of the thyroid
into the blood. Insulin maintains the normal blood
gland, resulting in a rapid heartbeat and an increased
sugar level.
rate of metabolism.
85. (c) Estrogen and progesterone hormones help to
98. (c) 99. (a) 100. (a)
maintain pregnancy. A decrease level of both the
hormones causes constriction of uterine blood 101. (c) In females, prolactin (PRL) regulates growth of
vessels leading sloughing of endometrium or uterine mammary glands and formation of milk after the birth
of baby.
epithelium in the menstrual period.
102. (a) FSH RH (Follicle Stimulating Hormone - Releasing
86. (a) Thymus starts functioning in the embryonic stage
Hormone) is secreted from hypothalamus when the
itself, becomes active during childhood and
level of progesterone and estrogen hormone is low
undergoes regression and gradually stops
in blood.
functioning in old individuals. So, production of
103. (c) ACTH stimulates the release of hormones from the
thymosins decreases. As a result, immune responses
adrenal cortex.
of old persons become weak.
104. (b)
S-126 Biology
105. (c) In the homeostatic control of blood sugar level, to control electrolyte homeostasis. Its main targets
hypothalamus and islets of Langerhans of pancreas are the kidney and the cardiovascular system but
functions as modulator and effector respectively. ANF interacts with many other hormones in order to
106. (c) ADH is synthesized by hypothalamus. regulate their secretion.
107. (b) Oxytocin (OT) hormon e is produced by 116. (d) ANP stimulates the loss of sodium in the urine and
hypothalamus under the influence of nervous promotes diuresis.
stimulation. It causes contraction of the smooth 117. (a) Except prostate, all the three glands (thyroid, adrenal
muscles of the uterus and myoepithelial cells lining and pituitary) are endocrine glands which secretes
the duct of the mammary gland. Hence it is also called hormones. Cretinism, goitre and myxoedema are
as birth hormone and milk ejection hormone. hormonal disorders whereas as scurvy is disease
Prolactin (PRL), secreted from anterior pituitary caused due to vitamin C deficiency. Insulin,
gland, acts to initiates and maintain milk secretion adrenaline and thyroxine are endocrine hormones
by the mammary gland, hence it is known as hormone secreted from pancreas, adrenal medulla and thyroid
of maternity and milk stimulating hormone. FSH gland respectively. Testosterone is a male hormone
(follicle stimulating hormone), secreted by anterior secreted from Leydig cells of testes whereas the other
pituitary gland, is a gonadotrophic hormone. It three (estrogen, progesterone and prolactin) are
stimulates spermatogenesis and growth of female hormones secreted from ovary, corpus luteum
seminiferous tubules in testes in male and maturation and anterior pituitary gland respectively.
of Graafian follicle and secretion of estrogen and 118. (d) Epinephrine stimulates the heart.
progesterone from corpus luteum in female. 119. (b) All of these hormones are produced in the anterior
108. (c) Hypothyroidism in adults leads to myxoedema and pituitary gland.
hyper-parathyroidism in children lead to osteitis 120. (b) Estrogen is secreted by ovary, alpha cells of islets of
fibrosa cystica. langerhans secrete glucagon and anterior pituitary
109. (c) Adrenaline hormone is also called as triple F gland secretes growth hormone. The over-secretion of
for flight, fright and fight reaction which occurs in growth hormone leads to gigantism.
condition of emergency. 121. (d)
110. (b) PTH (parathyroid hormone), also called Collip's
hormone, is secreted from the parathyroid gland. It Chapter 23 : Reproduction in Organisms
is responsible for the following functions like: release 1. (c) Life span is the period from the birth to the natural
of calcium by ones into the blood streams, absorption death of an organism.
of food by the intestine and conservation of calcium 2. (d)
by the kidneys.
3. (a) Asexual reproduction is a mode of reproduction by
111. (d) Iodine is essential for the normal rate of hormone
which offsprings arise from a single organism, and
synthesis in the thyroid. Deficiency of iodine in our
diet results in hypothyroidism and enlargement of inherit the genes of that parent only. In all the
the thyroid gland, commonly called goitre. methods of asexual reproduction offsprin gs
Hypothyroidism during pregnancy causes defective produced are genetically identical to the parents.
development and maturation of the growing baby 4. (c) Asexual reproduction is a process in which the new
leading to stunted growth (cretinism), mental organisms are produced without the formation and
retardation, low intelligence quotient, abnormal skin, fusion of gametes. In this case, all the genes and
deaf-mutism, etc. genetic contents are received by the offspring from
112. (d) Severe hypothyroidism leads to myxoedema. It is one parent through mitotic cell division.
also called as Gulls disease which develops in adults 5. (c) Clone is an organism or cell, or group of organisms
and characterized by puffy appearance due to or cells, produced asexually from one ancestor or
subcutaneous accumulation of fats, low BMR, low stock, to which they are genetically identical.
heart rate and body temperature, retarded sexuality.
6. (d) Asexual reproduction does not involve meiosis,
113. (b) HCG (Human Chorionic Gonadotropin) is a placental
ploidy reduction or fertilization, and the offspring is
hormone which maintains the corpus luteum for
continuous secretion of progesterone and estrogen a clone of the parent organism because of no
so as to maintain pregnancy. exchange of genetic material. Asexual reproduction
114. (b) Both help in protein synthesis, and are anabolic in is the primary form of reproduction for single-celled
nature. organisms such as the archaebacteria, eubacteria,
115. (c) Atrial- natriuretic factor (ANF) is a cardiac hormone and protists. Many plants and fungi reproduce
whose main function is to lower blood pressure and asexually as well.
EBD_7209
Hints & Solutions S-127

7. (b) In protists and monerans, asexual reproduction 18. (a) Homothallic and monoecious are used to denote
occurs by binary fission. Binary fission is the bisexual condition. The example includes fungi and
process which involves the division of the nucleus plants. Heterothallic and dioecious are used to denote
followed by that of cytoplasm, breaking the parent unisexual condition.
cell which splits into two daughter cells of 18. (b)
approximately equal size. It is seen in Amoeba and 19. (d) Parthenogenesis is a form of reproduction in which
Paramecium. the ovum develops into a new individual without
8. (a) In yeast, cell division results in a small cell called fertilization. It is common among insects and certain
bud. Bud is a small rounded outgrowth on an other arthropods.
asexually reproducing organism, such as a yeast or 20. (b) Embryo development is a post fertilization event in
Hydra, which is capable of developing into a new flowering plants. The zygote develops into the
individual. embryo and ovules develop into the seed. The ovary
9. (b) develops into the fruit which develops a thick wall
10. (b) The excessive growth of a floating aquatic plant, called pericarp that is protective in function.
water hyacinth (Eichhornia crasipes) caused havoc 21. (a) Viviparous animals give birth to young ones. This
in India by blocking our water ways. In India, it is
includes all placental mammals. The zygote develops
also known as ‘Terror of Bengal’. Water hyacinth
into a young one inside the body of the female. After
grows abundantly in nutrient rich water bodies and
attaining a certain stage of growth, the young ones
causes an imbalance in the aquatic ecosystem that
are delivered out of the body of the female organism.
result in the death of fishes and other aquatic
22. (a) Many animals reproduce both by aexual and sexual
organism.
means.
11. (a) Leaves of a number of plants develop or possess
23. (a) Since external fertilization can only take place in an
adventitious buds for vegetative propagation e.g., aquatic habitat, there are no terrestrial animals that
Bryophyllum, Kalanchoe, Adiantum caudatum, etc. use it.
12. (b) Sexual reproduction is mostly biparental. It involves 24. (b) Fertilization is a process of the union of male and
meiosis, gamete formation and usually fertilization female gametes, during sexual reproduction, to form
also, and introduces genetic variation in the a zygote (or fertilized egg). Millions of sperms reach
offsprings and play a role in evolution of species. the egg but only one enters it. The entry of sperm
13. (d) The females of placental mammals exhibit cyclical activates the egg for completing the meiosis.
changes in the activities of ovaries and accessory 25. (a) The birds and reptiles are not the only oviparous
ducts as well as hormones during the reproductive species. A group of mammals, the monotremes, are
phase. In non-primate mammals like cows, sheep, also egg layers.
rats, deers, dogs, tiger, etc. such cyclical changes 26. (b) In oomycetes female gamete is large and non-motile
during reproduction are called oestrus cycle whereas while male gamete is small and motile.
in primates (monkeys, apes and humans) it is called 27. (d) Sponges and coelenterates are bisexual animals.
menstrual cycle. 28. (d) All the given statements are correct.
14. (a) The end of vegetative phase in plants which marks 29. (c) Statement (ii) and (iv) are incorrect regarding internal
the beginning of the reproductive phase can be fertilization. Internal fertilization takes place inside
easily seen in the higher plants when they come to the female after insemination through copulation. In
flower. organisms exhibiting internal fertilization, the male
15. (b) Events in sexual reproduction may be grouped into gamete is motile and has to reach the egg in order to
3 distinct stages namely, the pre-fertilization, fuse with it. In these even though the number of
fertilization and the post-fertilization events. sperms produced is very large, there is a significant
17. (c) In flowering plants, both male and female gametes reduction in the number of eggs produced.
are non-motile. The method to bring them together for 30. (c) Statement (iii) is not correct. The reproductive phase
fertilization is called pollination. Pollination is the process is not of same duration in all organisms.
by which pollen is transferred from the anther (male part) 31. (a) Vegetative propagation occurs by buds or notches
to the stigma (female part) of the plant, thereby enabling in potato.
fertilization and reproduction. Pollination takes place in 32. (b) All the given statements are related to water
the angiosperms, the flower bearing plants. hyacinth. Water hyacinth is a free-floating tropical
S-128 Biology
American water plant which has been introduced as conidia, e.g., Penicillium.
an ornamental and in some warmer regions has (iii) Budding - by external budding, e.g., Hydra or
become a serious weed of waterways. by gemmules (internal buds), e.g., sponges.
33. (c) Statement (i), (ii), and (iv) are correct. 44. (d) 45. (d) 46. (d)
(iii) Ginger propagates vegetatively with the help of an 47. (c) A : Gemmule is a tough-coated dormant cluster of
underground stems (rhizomes). embryonic cells produced by a freshwater sponge
34. (b) Statement (iii) and (iv) are incorrect regarding sexual for development in more favourable conditions.
reproduction. B: Budding is a type of asexual reproduction in which
(iii) Meiosis occurs during sexual reproduction. Meiosis a daughter individual is formed from a small
is a type of cell division that results in four daughter projection, the bud, arising from the parent body,
cells each with half the number of chromosomes of e.g., yeast.
the parent cell, as in the production of gametes and C: Tuber is an underground stem structure which can
plant spores. take part in vegetative reproduction, e.g., Potato
(iv) External fertilization is not a rule during sexual D: Offset is a type of sub aerial or creeping stems.
reproduction. These are one internode long runners that occur in
35. (b) Runners, tubers and offsets are all the units of some aquatic plants. Breaking of offsets helps in
vegetative propagules in plants. These are all capable propagation, e.g., water hyacinth and water lettuce.
of giving rise to new offsprings. Since the formation 48. (a) A : Pollination is the process by which pollen is
of these structures does not involve two parents, transferred from the anther (male part) to the stigma
the process involved is asexual.
(female part) of the plant, thereby enabling
36. (a) When offspring is produced by a single parent with
fertilization and reproduction.
or without the involvement of gamete formation, the
reproduction is called asexual. As a result, the B: Germination is the process by which a plant grows
offspring that are produced are not only similar to from a seed, e.g., the sprouting of a seedling from a
one another but are also exact copies of their parent. seed of an angiosperm or gymnosperm. .
Only mitotic division occurs in asexual reproduction. C: Uterus is the enlarged, muscular, expandable portion
37. (a) Asexual mode of reproduction is the process in which of the oviduct in which the fertilized ovum implants
the genetic constituent remains the same. So due to and develops during prenatal development.
that offspring and parent are morphologically and
D: Menstruation is the process in a woman of
genetically same.
38. (a) Zygote is the product of two individual and if discharging blood and other material from the lining
succeeded then produce the gamete (male or female) of the uterus at intervals of about one lunar month
for next generation from puberty until the menopause, except during
39. (c) Two Haploid cell form one diploid cell by sexual pregnancy.
reproduction. In higher organism diploid is the most 49. (a) Onion - Bulb (Underground stem), Ginger - Rhizome,
common nature of cell. Chlamydomonas - Zoospore, Yeast - Ascospores
40. (a) 41. (a) 50. (d) Banana is vegetatively propagated by modified stem
42. (b) A-III; B-I; C-II; D- IV
(rhizomes).
Animals which give birth to young one are called
viviparous animals. Animals (like Hydra) which 51. (b) A - III, B - V, C - II, D - IV, E - I
produces bud as a small projection to form a complete A: Gamete is a mature haploid male or female germ cell
individual from that is a type of asexual reproduction which is able to unite with another of the opposite
called budding. Planaria shows regeneration. where sex in sexual reproduction to form a zygote.
formation of the whole body of an organism occurs B: Budding is a form of asexual reproduction in which
from a small fragment or the replacement of the lost new individuals form from outgrowths (buds) on the
part. Regeneration was first discovered in Hydra by
bodies of mature organisms. These outgrowths grow
Abraham Trembley in 1740. Placenta is a flattened
by means of mitotic cell division.
circular organ in the uterus of pregnant eutherian
mammals which nourish and maintain the foetus C: Fission is a form of asexual reproduction and cell
through the umbilical cord. division used by all prokaryotes (bacteria and
43. (b) A – II; B – I; C – IV; D – III; E – V archaebacteria) and some organelles within
Asexual reproduction takes place in many ways: eukaryotic organisms (e.g., mitochondria). In this a
(i) Binary fission, e.g., Amoeba. unicellular organism divides into two or more
(ii) Spore formation - by motile zoospores, e.g., independently maturing daughter cells.
Ulothrix, Chlamydomonas or by non-motile
EBD_7209
Hints & Solutions S-129

D : Fertilization involves the fusion of male and female gametogenesis, gamete transfer, zygote formation
gametes to form a zygote. and embryogenesis. Gametogenesis is the
E : Zygote is a diploid cell resulting from the fusion of development and production of the male and female
two haploid gametes; a fertilized ovum. germ cells required to form a new individual. The
52. (b) In the given figure of Chlamydomonas (A), gametes in human males are produced by the testes
Penicillium (B), Hydra (C) and sponge (D), the and gametes in human females are produced by the
asexual reproductive structure found in them are ovaries. After their formation, male and female
respectively zoospore, conidia, bud and gemmules. gametes must be physically brought together to
Zoospore is a motile asexual spore that uses a facilitate fusion (fertilization). Zygote formation is
flagellum for locomotion. They are produced inside universal in all sexually reproducing organisms which
the zoosporangia and generally naked (without cell depend on the type of life cycle of the organism and
wall). Conidia are non-motile spores produced singly the environment to which it is exposed.
or in chains by constriction at the tip or lateral side Embryogenesis refers to the process of development
of special hyphal branches, called conidiophores. of embryo from the zygote.
They are produced exogenously, dispersed by wind 55. (b) The type of reproduction shown in the given figure
and germinate directly by giving out germ tubes. of yeast is budding. In budding, a daughter
Bud is a small rounded outgrowth on an asexually individual is formed from a small projection, the bud,
reproducing organism, such as a yeast or Hydra, arising from the parent body. In yeast, the division
which is capable of developing into a new individual. is unequal and a small bud is produced that remains
Gemmule is a tough-coated dormant cluster of attached initially to the parent body. Later on the
embryonic cells produced by a freshwater sponge bud gets separated and matures into a new yeast
for development in more favourable conditions. organism.
53. (d) Vegetative propagation or reproduction is the 56. (d) The type of gametes shown in the figure A
formation of new plants from vegetative units (= (Cladophora, an algae), B (Fucus, an algae) and C
vegetative parts of the plant) such as buds, tubers, (Homo sapiens) are respectively homo/isogametes,
rhizomes, etc. These vegetative units are called heterogametes and heterogametes. Isogamete is a
vegetative propagules. In the given figures of gamete indistinguishable in form, size, or behaviour
members of angiosperms (A- Potato; B - Ginger; C - from another gamete with which it can unite to form
Agave and D - Bryophyllum), the correct vegetative a zygote. Heterogametes is either of a pair of gametes
propagules present are respectively tuber, rhizome, that differ in form, size, or behaviour and occur
bulbil and leaf buds. typically as large non-motile female gametes and
Tubers: These have buds over their nodes or eyes small motile sperms.
which produce new plantlets when a stem tuber or a 57. (c)
part of it having an eye is placed in the soil, e.g., 58. (c) Both X and Y are diploid in the given figure of
Artichoke, Potato (also called eyes on tuber). transverse section of pea plant.
Rhizomes are main underground stems which store 59. (b) In both the figures, the structure marked as B (in
food during unfavourable conditions. These have figure P) and C (in figure Q) are the male reproductive
organs. B and C is respectively antheridium and
buds for formation of new aerial shoots during
testes sac. An antheridium (or antheridia) is a haploid
favourable conditions. Examples are banana, ginger,
structure (or organ) producing and containing male
turmeric, Aspidium, and Adiantum. gametes (called antherozoids or sperms).
Bulbils are multicellular fleshy buds that take part in It is present in the gametophyte phase of cryptogams
vegetative propagation, e.g., Oxalis, Agave, like bryophytes and ferns, and also in the primitive
Pineapple (Ananas), Dioscorea (Yam), Lily, vascular psilotophytes. Testis sac is the pouch
Chlorophytum. In Agave, bulbils are modified floral containing the testes and their accessory organs.
buds that develop on the flowering axis. 60. (b) The marked structure in the given figure is offset.
Leaf buds: These leaf buds (adventitious) arise from Offsets are one internode long runners that occur in
some aquatic plants. Breaking of offsets helps in
the notches present at the margin of leaves in
propagation, e.g., Eichhornia (Water Hyacinth),
Bryophyllum.
Pistia (Water Lettuce).Vegetative propagation is a
54. (b) The events marked as A, B, C and D in the given form of asexual reproduction in plants, in which
figure of life of general reproduction is respectively multicellular structures become detached from the
S-130 Biology
parent plant and develop into new individuals that the haploid (n) small, male gametophyte covered by
are genetically identical to the parent plant. For two membranes outer ‘exine’ and inner ‘intine’ and
example, liverworts and mosses form small clumps divide mitotically to produce a bigger vegetative
of tissue (called gemmae) that are dispersed by cell(n) and a smaller generative cell(n). Therefore, if
splashing raindrops to form new plants. Bulbs, we culture the whole anther then we will get both
corms, offsets, rhizomes, runners, suckers, and haploid plantlets from pollen grains and diploid
tubers are all important means of vegetative plantlets from anther wall (vegetative cell).
reproduction and propagation in cultivated plants. 70. (a) A zygote is a eukaryotic cell formed by a fertilization
61. (b) Bud giving rise to new plant is present towards base event between two gametes. The zygote's genome
of the onion bulb. So, if basal half of the onion bulb is a combination of the DNA in each gamete, and
is removed then new plant will not emerge. contains all of the genetic information necessary to
62. (d) Tubers are formed from underground branches of form a new individual. In multicellular organisms, the
stem and if rools only remain in the soil. Then the zygote is the earliest developmental stage. Zygote
potato tubes in plants will not formed. is the vital link that ensures continuity of species
63. (c) between organisms of one generation to the next.
64. (d) Fungi reproduce by vegetative, asexual and sexual Every sexually reproducing organism begins life as
means. About 20% fungi propagate only by asexual a single cell called zygote.
means. Asexual reproduction takes place during 71. (a) Meiosis occurs during sexual reproduction. It is a
favourable conditions by the formation of a variety type of cell division that results in four daughter
of conidia or spores. The spores may be unicellular cells each with half the number of chromosomes of
(e.g., Aspergillus) or multicellular (e.g., Alternaria). the parent cell, as in the production of gametes and
Bacteria also reproduce mainly by asexual method plant spores. Therefore, meiosis has to occur if a
and therefore they have a dominant haploid phase diploid body has to produce haploid gametes.
in their life-cycle. Asexual reproduction in bacteria 72. (c)
takes place by binary fission, budding, conidia, cysts 73. (b) In diploid organisms, specialised cells called
and endospores. meiocytes undergo meiosis. Blue-green algae
65. (a) Common feature of reproduction among Amoeba, (cyanobacteria) are unicellular where the most
Spirogyra and yeast is that they all reproduce common method of multiplication is binary fission.
asexually. Asexual reproduction is common among 74. (d) Parthenogenesis is a form of reproduction in which
single celled organisms and in plants and animals an unfertilized egg develops into a new individual,
with relatively simple organization. occurring commonly among insects and certain other
66. (b) Sexual reproduction involves both meiosis and arthropods.
fusion of male and female haploid gametes. It is the 75. (d) The correct sequences are: (ii) and (iv)
production of new living organisms by combining In animals - Juvenile phase ® Reproductive phase
genetic information from two individuals of different ® Senescent phase
sexes. In higher organisms, one sex (male) produces In plants - Vegetative phase ® Reproductive phase
a small motile gamete which travels to fuse with a ® Senescent phase
larger stationary gamete produced by the other Juvenile phase is a stage of growth and maturity in
(female) organisms. the life of an individual before they can reproduce
67. (c) Some organisms are capable of asexual or sexual sexually. Vegetative phase is characterized by
reproduction. Under favourable conditions, meristems producing leaves.
reproduction proceeds asexually. When conditions 76. (a) Offspring's of oviparous animals are at greater risk
become more stressful reproduction switches to a as compared to offspring's of viviparous animals
sexual mode, because sexual reproduction produces because proper embryonic care and protection is less
individuals with new combinations of recombined in oviparous animals. Oviparous animals are those
chromosomes increasing diversity. animals that lay eggs, with little or no other embryonic
68. (c) Seed formation begins with the combination of a male development within the mother. These eggs mature
and female gamete, a process known as fertilization. and hatch after being expelled from the body.
Fertilization or syngamy, can occur when both male Examples, most fish, amphibians, reptiles, all birds etc.
and female gametophyle are fully nature. Whereas viviparous animals give birth to young lives.
69. (b) Anther, a male reproductive organ, is diploid (2n) in The fertilized eggs develop into a young one inside
chromosome numbers. As a result of microsporo- the body of the female organisms. Therefore, chances
genesis, tetrads of microspores are formed from a of the survival of the young ones in viviparous
single spore mother cell. They are known as pollen animals is more because of proper embryonic care
grains after their release from tetrads. Pollen grain is and protection inside the mother body.
EBD_7209
Hints & Solutions S-131

Chapter 24 : Sexual Reproduction in Flowering 9. (c) Pollen grains can be stored in liquid nitrogen (–196°C)
Plants to retain their viability for longer duration of time.
Such stored pollen can be used as pollen banks similar
1. (b) Flowers are highly modified shoot bearing nodes
to seed banks, in crop breeding programmes.
and modified floral leaves, which are meant
10. (a) Ovule is the part of the ovary of seed plants that
essentially for sexual reproduction in plants.
contains the female germ cell and after fertilization
2. (c) A typical flower has four different kind of whorl.
becomes the seed. An ovule consists of a
These are calyx (green leafly sepals) corolla (bright
megasporangium surrounded by one or two layers
colored petals), and androecium (male reproduction
of tissue called integuments. The megasporangium
organ) and gynoecium (female reproduction organ).
produces spores that develop into
3. (a) Typically, angiospermic anther is bilobed which is
megagametophytes. These megagametophytes
connected by connective and dithecous i.e., each
remain within the tissues of the ovule and produce
lobe has two theca. The bilobed structure of the
one or more egg cells.
anther is tetrasporangiate (four sporangia). In
transverse section, it appears as four sided tetragonal 11. (d) Placenta is the surface of the carpel to which the
structure consisting of two microsporangia in each ovules (potential seeds) are attached. The placenta
lobe. is usually located in a region corresponding
4. (c) The tapetum is a layer of nutritive cells found within somewhat to the margins of a leaf but is actually
the sporangium, particularly within the anther, of submarginal in position.
flowering plants. Its main function is to provide 12. (b) The point at which funiculus touches the ovule is
nutrition to the developing microspore mother cells called hilum. Thus, hilum represents the junction
and pollen grains. between the ovule and funiculus.
5. (c) Microsporogenesis is the process of formation of 13. (c) Egg apparatus consists of two synergids and one
microspores or pollen grains, from a pollen mother egg cell lying at the micropylar end. Synergids bear
cell through meiosis is micro sporogenesis Each cell prominent structure called ‘ filiform’ apparatus which
of sporogenous tissue serve as microspore mother are finger like projections. Synergids guide the path
cell (MMC). These MMCs undergo meiosis and form of pollen tube towards the egg, help in obtaining
microspore tetrad and become haploid, microspores nourishment from the outer nucellar cells and also
or pollen grains. As anthers mature microspores of function as shock absorbers during the penetration
the tetrad separate from each other and develops of pollen tube into the embryo sac. Cytoplasm of
into pollen grains. Each microsporangium contains egg is inactive, rich in ribosomes, and contains
numerous pollen grains which are released after plastids.
dehiscence of anther wall. 14. (c) In Anatropous ovule, the body of the ovule is
6. (d) Exine is tough, cutinized layer often with spinous completely turned at 180° angle, due to unilateral
outgrowth but sometimes smooth. Exine is composed growth of funiculus, so it is also called inverted
of sporopollenin which is resistant to physical and ovule. The chalaza and micropyle lie in straight line.
biological decomposition. It protect the pollens from The hilum and micropyle lie side by side very close
environmental extremes. to each other. This type of ovule is found in 80%
7. (c) Sporopollenin is the tough resistant biopolymer that families of angiosperms but not in Capsella. In this
coats the outer walls of pollen grains spores and ovule, micropyle is facing downward condition. This
related micro-organisms. It is considered “the most is the most common type of ovule so that it is
resistant organic material known” and accounts for considered as a "typical ovule" of angiosperms.
th e morphology and microstructure an d for 15. (a) Synergids bear prominent structure called ‘filiform’
geological preservation of polymorphs (an organic apparatus which are finger like projections. This
microfossils). apparatus is present in upper part of each synergid. This
8. (d) Pollen grain is liberated at 2 or 3 celled stage. If pollen apparatus is useful for the absorption and transportation
grain is released at the 2-celled stage (i.e., vegetative of materials from the nucellus to the embryo sac.
and generative cells) then generative cell divides
16. (b) A typical mature embryo sac of angiosperm is 7-
meiotically to produce two male gametes, but when
celled, 8 nucleate structure i.e., 3 antipodal cells, 3-
pollen grains are released at 3-celled stage then two
egg apparatus cells and one central cell (2 polar
male gametes are already present.
nuclei).
S-132 Biology
17. (a) Autogamy and geitonogamy are types of self - usually fuses with the two polar nuclei at the centre
pollination. Geitonogamy is the fertilization of a flower of the embryo sac, forming a primary endosperm cell
by pollen from another flower on the same (or a (its nucleus is often called the triple fusion nucleus).
genetically identical) plant. It occurs between About 70% of angiosperm species have endosperm
bisexual flowers or unisexual flowers of the same cells that are polyploid. These are typically triploid
plant. While autogamy is a kind of pollination in (containing three sets of chromosomes) but can vary
which the pollen from the anthers of a flower are widely from diploid (2n) to 15n.
transferred to the stigma of the same flower. 25. (c) Coconut water is the clear liquid inside young green
18. (b) In some plants, bisexual flowers are formed which coconuts. Milky water of green coconut is called
never open throughout the life. Such flowers are liquid endosperm.
called cleistogamous flowers such as Commelina 26. (c) A nature embryo in monocotyledons has a single
viola. So only self-pollination takes place in these cotyledon called ‘scutellum’, e.g. Triticum.
plants. Self pollination is a rule in cleistogamous 27. (d) In most of the angiosperm, entire part of the nucellus
flowers. is utilized by developing embryo sac but in some of
19. (b) the angiosperm some part of the nucellus remain
20. (a) Pollination by water (called hydrophily) occurs in inside the ovules. That part of the nucellus present
Vallisneria. In vallisneria (ribbon weed), female flower inside the seed in the form of a thin layer is known as
reaches on the surface of water and assumes perisperm.
horizontal position due to elongation of its stalk, 28. (d) False fruits are those in which addition to ovary other
male flowers or pollen grains are released on to the floral parts also contribute for its development is
surace of water. They are carried passively by water called false fruits, examples - apple, pear, strawberry
currents & some of which eventually reach to the and cashewnut.
stigma. 29. (d) Seeds can remain dormant for many years and
21. (b) Bees are important to agriculture as they perform germinate on return of favourable conditions.
pollination. Bees are responsible for over 80% of all Reproductive process is not dependent on water.
pollination done by insects. Plants pollinated by Seeds have sufficient food reserves to initiate embryo
insects are colorful with fragrance and abundant development and seedling development till the
nectar which attracts insects. photosynthesis process is initiated. Hard seed coat
22. (d) The fusion of male and female gametes is called also protects the embryo from environment extremes.
fertilization. Fertilization in angiosperms involves two Since seeds are the product of sexual reproduction it
nuclear fusions, one between one of the male gametes promotes diversity. Seeds have better adaptive
and the egg and the other between the second male strategies for dispersal to new habitats & help the
gamete and the secondary nucleus of the polar nuclei. species to colonise in other areas.
The first of these fusions called generative 30. (d) Seeds which do not have an endosperm at maturity,
fertilization or syngamy which leads to the formation are called non-endospermic or Ex–albuminous seeds.
of a zygote and the second known as vegetative The endospermic tissues are absorbed during the
fertilization which leads to the formation of a triploid development of embryo. The absorbed food materials
primary endosperm nucleus. from the endosperm is stored in cotyledons that is
23. (b) During double fertilization in plants one sperm fises why they become so large and fleshy, e.g., Capsella
with the egg cell and the other sperm fuses with two and all dicotyledons. But Castor seed is
polar nucles of the central cell to produce triploid endospermic.
primary endospersm cell (3n). Since the latter 31. (b) Apomixis is the production of seeds without
involves fusion of three haploid nudeus therefore it fertilization. The term apomixis was given by winker
is called triple fuxion. (1908). There are several ways of development of
24. (b) Endosperms is produced inside the seeds of most apomictic seeds. In some species the diploid egg
flowering plants around the time of fertilization. It is cell is formed without reduction division & develops
formed when the two sperm nuclei inside a pollen into the embryo without fertilization. In some species
grain reach the interior of an embryo sac or female the nucellar cells surrounding the embryo sac start
gametophyte. One sperm nucleus fertilizes the egg, dividing, protrude into the embryo sac & develop
forming a zygote, while the other sperm nucleus into embryos. Some apomictic plants are citrus,
EBD_7209
Hints & Solutions S-133

cactaceae, grasses, parthenium, etc. stages, and consequently no seeds are produced.
32. (d) Polyembryony is the state of occurrence of more 41. (c) The coconut water from tender coconut is nothing
than one embryo in a seed. It was observed by but free nuclear endosperm (made up of thousands
Leeuwen hoek in citrus (oran ge) seeds. of nuclei) and the surrounding white kernel is the
Polyembryony is commonly found in gymnosperms cellular endosperm. Female gametophyte is embryo
but it is also found in some of angiospermic plants sac.
such as orange, lemon and Nicotiana, etc. 42. (c) Statement (i) and (ii) are correct about typical female
33. (a) The movement of pollen tube towards embryosac gametophyte. Female gametophyte contains the egg
though style is chemotactic as it secretes pectinase cell and central cell that become fertilized and give
and other hydrolytic enzymes to create a passage rise to the embryo and endosperm of the seed,
for its entry into style. respectively. Female gametophyte development
34. (a) Emasculation is removal of anthers from the flower begins early in ovule development with the formation
bud before the anther dehisces in bisexual flowers. of a diploid megaspore mother cell that undergoes
35. (a) In over 60 per cent of angiosperms, pollen grains are meiosis.
shed at cell 2-celled stage. In the remaining species 43. (b) In a majority of water plants like water hyacinth and
the generative cell divides mitotically to give rise to water lily, flowers emerge above the water level and
the two male gametes before pollen grains are shed are pollinated by insects.
(3-celled stage.) 44. (d)
36. (c) Exine has apertures which are called germ pores (if 45. (b) During the development of a dicot embryo, the zygote
rounded) or germinal furrows (if elongated). These gives rise to the proembryo and subsequently to the
are the areas where sporopollenin is absent. globular heart-shaped and mature-embryo.
37. (d) Double fertilization forms a diploid zygote nucleus 46. (a) Pollen mother cells undergo meiosis and produce
and a triploid primary endosperm nucleus (PEN). pollen grains. The pollen grains have haploid number
After that, the two products of double fertilization of chromosomes.
i.e., zygote and PEN, develop into embryo and
47. (a) Male gamete (n) + secondary nucleus (2n) = primary
endosperm respectively.
endosperm nucleus which develops into endosperm (3n)
38. (c) Sporogenous tissue is always diploid, endothecium
Endosperem is the reserve food used by developing
is second layer of anther wall and perform the
embryo.
function of protection and help in dehiscence of
48. (a) Active form of PFR is responsible for inducing
anther to release the pollen. Hard outer layer of pollen
flowering. Phytochrome, protein pigment, exists in
is called exine but tapetum always nourishes the
developing pollen. two inter convertible forms.
Cells of the tapetum possess dense cytoplasm and Red
Pr Pfr
generally have more than one nucleus (polypoid). Far red
39. (b) Cleistogamous flowers do not expose their
reproductive parts. Anthers and stigma lie close to
Dark
each other. Pure autogamy occurs since there is no
49. (d) Honey bee visit flowers to gather nectar and turn it
chance of cross-pollination. Cleistogamy is the most
efficient floral adaptation for promoting self- into honey. Visiting of insects for nectar helps in
pollination. E.g., Viola mirabilis and Oxalis pollination.
autosella. 50. (b) The majority of angiosperms bear chasmogamous
40. (c) All the statements are correct about self - flowers, which means the flowers expose their mature
incompatibility. Self-incompatibility is a general name anthers and stigma to the pollinating agents. There
for several genetic mechanisms in angiosperms, is another group of plants which set seeds without
which prevent self-fertilization and thus encourage exposing their sex organs. Such flowers are called
outcrossing and allogamy. In plants with self - cleistogamous and the phenomenon is cleistogamy.
incompatibility, when a pollen grain produced in a 51. (c) A : Zoophily is a form of pollination whereby pollen
plant reaches a stigma of the same plant or another is transferred by vertebrates, particularly by
plant with a similar genotype, the process of pollen humming-birds and other birds, and bats, but
germination, pollen tube growth, ovule fertilization, also by monkeys, marsupials, lemurs, bears,
and embryo development is halted at one of its rabbits, deer, rodents, lizards and other animals.
S-134 Biology
B : Ornithophily is the pollination of flowering plants that are pollinated by insects. Vegetative cells
plants by birds. are involved in the formation of microspores. The
C : Entomophily is the pollination whereby pollen vegetative cells are bigger, has abundant food
or spores are distributed by insects. Several reserves and a large irregular shaped nucleus.
insects are reported to be responsible for the Sporogenous tissue is a group of compactly
pollination (potential or effective) of many plant arranged homogenous cells. It is irregular in shape
species, particularly bees, Lepidoptera with abundant food reserves.
(butterflies and moths), wasps, flies, ants and 56. (d) Coleorhiza is the sheath that envelops the radicle in
beetles. certain plants (grass or cereal grain) and that is
penetrated by the root in germination. Food storing
D : Chiropterophily is pollination of plants by bats.
tissue is endosperm. Endosperm is the part of a seed
Bat pollination is most common in tropical and
which acts as a food store for the developing plant
desert areas that have many night-blooming
embryo. Parthenocarpic fruit produced without the
plants.
formation of seeds due to lack of pollination,
52. (b) Funicle is a filamentous stalk attaching a seed or
fertilization and embryo development. Example-
ovule to the placenta. Hilum is the point at which
banana, pineapple, orange and grapefruit. Single
funiculus touches the ovule. Thus, hilum represents
seeded fruit developing from monocarpellary is
the junction between the ovule and funiculus.
mango. Membranous seed coat is present in maize.
Integuments are the protective covering or layers
57. (d) Male reproductive organ is called androecium and
present in the ovule. It encircles the ovule except at
their unit is called stamen. Stamen is also known as
the tip where a small opening called micropyle is
microsporophyll. A typical stamen differentiates into
organised. Chalaza is the basal swollen part of the
three parts- filament, anther and connective tissue.
nucellus (opposite the micropylar end) from where
The anther is bilobed and the lobe encloses 4 pollen
the mteguments originate. Nucellus is the central part
sacs or microsporangia. The integumented nucellus
of an ovule, containing the embryo sac.
or megasporangium is called ovule.
53. (a) Fruit is mature or ripened ovary, developed after
58. (d) When the pollination takes place in between the two
fertilization. The ovules after fertilization, develop
into seeds. The wall of the ovary develops into the different flowers of two different plants of the same
wall of fruit called pericarp. In fleshy fruits pericarp species then it is called xenogamy. This is real or true
is generally distinguished into three layers epicarp, cross-pollination. Genetically, as well as ecologically,
mesocarp and endocarp. Mesocarp is fleshy or it is cross-pollination.
fibrous. These fruits are indehiscent. e.g., guava, 59. (d) In the given figure of typical stamen (a) and three
orange, mango etc. In dry fruits pericarp is not dimensional cut section of anther (b), the structure
distinguished into three layers. They may be marked as a, b, c and d are respectively anther, petiole,
dehiscent, indehiscent and splitting e.g., groundnut, pollen sac and megaspore. Anther is the part of a
mustard, etc. stamen that contains the pollen. Petiole is the slender
54. (b) Parthenocarpy is the development of fruit without stalk by which a leaf is attached to the stem. Pollen
the formation of seeds due to lack of pollination, sac is one of the chambers of an anther or a male
fertilization and embryo development. Polyembryony cone in which pollen is produced; th e
is the formation of more than one embryo from a microsporangium of a seed plant. Whereas
single fertilized ovum or in a single seed. Apomixis is megaspore is a spore that gives rise to a female
the development of an embryo without the gametophyte and is larger than a microspore.
occurrence of fertilization. Parthenogenesis is one
60. (a) In the given figure of TS of anther, the structure
form of apomixis. Dormancy is a state of temporary
marked as A, B, C, D and E are respectively
metabolic inactivity or minimal activity therefore
sporogenous tissue, tapetum, epidermis, middle layer
helps an organism to conserve energy.
and endothecium. Sporogenous tissue is a group of
55. (a) Tapetum is a layer of nutritive cells found within the
compactly arranged homogenous cells. It is irregular
sporangium, particularly within the anther, of
flowering plants. Exine is the decay-resistant outer in shape with abundant food reserves. Tapetum is a
coating of a pollen grain or spore. It is made up of layer of nutritive cells found within the sporangium,
sporopollenin (most resistant organic material particularly within the anther, of flowering plants. It
known). Pollenkit is a sticky covering found on the is important for the development of pollen grains.
surface of pollen grains. It is especially common in Epidermis is the common anther covering. Cells of
EBD_7209
Hints & Solutions S-135

middle layers degenerate to provide nourishment to embryo sac that are destined to form endosperm.
the growing microspore mother cells. Endothecium The stigma is the receptive tip of a carpel, or of several
is the inner lining of a mature anther of a flower. fused carpels, in the gynoecium of a flower. Style is
61. (b) In the given figure of mature embryo sac, the a long, slender stalk that connects the stigma and
structure marked as A, B, C, D, and E are respectively the ovary. Chalaza is two spiral bands of tissue that
antipodal cells, central cells, polar nuclei, synergids suspend the yolk in the centre of the white (the
and filiform apparatus. albumen).
Embryo sac is the female gametophyte of a seed plant, 65. (d) Pollen grains represent the male gametophyte. The
containing the egg, synergids, and polar and outer part of the pollen is exine, which is composed
antipodal nuclei. In this, fusion of the antipodal and of a complex polysaccharide, sporopollenin. Inner
a pollen generative nucleus forms the endosperm. part is intine. The cell contains vegetative cell which
The embryo sac develops in the central portion of develop into the pollen tube and germ pore and
the ovule (nucellus), where the maternal generative cell (degenerative) are also present.
macrosporocyte, as a result of meiotic division, forms 66. (c) False fruit are those fruits in which addition to ovary
four haploid cells (a tetrad of macrospores), of which other floral parts (like thalamus) also contribute for
one develops (the rest atrophy). During the its development is called false fruits. Examples–
development of the embryo sac there are three apple, pear, strawberry and cashewnut. In the given
successive synchronous mitotic divisions of its figure of false fruit (apple), the structure marked as
nuclei, so that their number increases in the A, B, C and D are respectively thalamus, seed,
progression 1: 2: 4: 8 and they are distributed evenly endocarp and mesocarp.
along the ends of the growing embryo sac. Thalamus is a receptacle of a flower in which the
62. (b) Fig (b) is a wind pollinated plant showing compact embryo fruits and later the seeds are held. Seed is a
inflorescence and well expored stamens. Pollination fertilized and ripened ovule and the characteristics
by wind is more common amongst abiotic of gymnosperms and angiosperms. Endocarp is the
pollinations. Wind pollination also requires that the innermost layer of the pericarp which surrounds a
seed in a fruit. It may be membranous (as in apples)
pollen grains are light, small, dry and non-sticky so
or woody (as in the stone of a peach or cherry).
that they can be transported in wind currents. Both
Mesocarp is the middle layer of the pericarp of a
the stigmas and anthers are exserted. Anthers are
fruit, between the endocarp and the exocarp.
versatile, stigma is hairy, feathery or branched to
catch the wind borne pollen grains. 67. (b)
63. (a) The given figure shows the stages in embryogenesis 68. (c) P1-Autogamy; P2-Geitnogamy; P3-Xenogamy. If the
in a typical dicot (Capsella). The structure marked pollen grains are transferred from an anther to the
as A, B, C and D is respectively suspensor, radicle, stigma of the same flower, or different flowers of the
plumule and cotyledon. Suspensor is a suspending same plant is called self-pollination or autogamy.
part or structure as a group or chain of cells that is When the pollen grains are transferred to the stigma
produced from the zygote of a seed plant and serves of other flower of the same species is called cross-
to push the developing embryo into the endosperm. pollination or allogamy. It takes place in between
Radicle is the part of a plant embryo that develops two different flowers.
into the primary root. Plumule is the young shoot of Cross-pollination is of two types – Geitonogamy and
a plant embryo above the cotyledons, consisting of Xenogamy.
the epicotyl and often of immature leaves. Cotyledon Geitonogamy : When pollination takes place in
is an embryonic leaf in seed-bearing plants, one or between the two flowers of the same plant then it is
more of which are the first leaves to appear from a called geitonogamy.
germinating seed. When the pollination takes place in between the two
64. (a) In the given figure of LS of flower, the structure different flowers of two different plants of the same
marked as A, B, C, D and E are respectively antipodal species then it is called xenogamy.
cells, polar nuclei, stigma, style and chalaza. 69. (d) Anatropous ovule is a completely inverted ovule
Antipodal cells are the three haploid cells in the turned back 180 degrees on its stalk. It is inverted at
mature embryo sac of flowering plants that are an early stage of growth, so that the micropyle is
situated at the opposite end to the micropyle. Polar turned toward the funicle and the embryonic root is
nuclei are either of the two nuclei of a seed plant at the opposite end. In the given figure of anatropous
S-136 Biology
ovule, the structure marked as A, B, C and D are (outermost layer); the mesocarp (middle layer); and
respectively micropyle, nucellus, embryo sac and the endocarp (inner layer surrounding the ovary or
chalazal pole. the seeds). In a citrus fruit, the epicarp and mesocarp
Micropyle is the opening through which pollen make up the peel.
nuclei enter the ovule. Nucellus is the central part of 76. (b) Sexual reproduction leads to formation of new
an ovule, containing the embryo sac. Embryo sac is combination and appearance of variations. Genetic
the female gametophyte of a seed plant, containing recombination, interaction etc. during sexual
the egg, synergids, and polar and antipodal nuclei. reproduction provides vigour and vitality to the
Chalazal end is the basal part of a plant ovule opposite offsprings. They better adapt themselves to
the micropyle; where integument and nucellus are changing environmental conditions and also plays
joined. an important role in evolution.
70. (d) In the given figure of monocot embryo, the structure 77. (d) Sequence of development during the formation of
marked as A, B, C and D are respectively scutellum, embryo sac is:
coleoptile, epiblast and coleorhiza. Archesporium ® Megaspore mother cell ®
Scutellum is the large shield like cotyledon of the Megaspore ® Embryo sac
embryo of certain monocots. It is specialized for the 78. (a) Development of male gametophyte is called micro -
absorption of food from the endosperm. Coleoptile gametogenesis. One meiotic division and two mitotic
is the first leaf above the ground, forming a protective divisions are necessary for the complete
sheath around the stem tip. It surrounds the plumule. development of male gametophyte. Male
Epiblast is the outermost layer of an embryo before gametophyte when fully developed is a 3 nucleate
it differentiates into ectoderm and mesoderm. structure.
Coleorhiza is the sheath that envelops the radicle in 79. (a) Single megaspore mother cell (MMC) with dense
certain plants (grass or cereal grain) and that is cytoplasm and a prominent nucleus gets
penetrated by the root in germination. differentiated from nucellus near the micropylar
71. (b) Central cell is the cell in the venter of the archegonium region. This megaspore mother cell (MMC)
whose division produces the egg and usually also undergoes meiosis to form ‘4’ haploid cells (called
the ventral canal cell (as in cycads). It is the largest megaspores) and the process of formation is known
cell the in an embryo sac. as megasporogenesis.
72. (d) Antheridium is a male structure and the other three 80. (a) Megaspore mother cell (MMC) undergoes meiosis
archegonium, oogonium & ovule are female parts. to form four haploid cells (called megaspores) and
An antheridium is a haploid male reproductive the process of formation is known as
structure producing gametes, occurring in ferns, megasporogenesis. The MMC undergoes meiotic
mosses, fungi and algae. Archegonium is the female, division results in the production of four megaspores
egg-producing reproductive structure on the 100 functional megaspores are produced by 100
gametophytes of non-flowering land plants. The MMC, since three out of four megaspores degenerate
archegonium is comprised of an oogonium in each case.
surrounded by protective layers of thick-walled 81. (d) In the cytoplasm of the synergid pollen tube releases
pigmented cells. In seed plants, ovule is the structure the two male gametes. After reaching ovary the poller
that gives rise to female reproductive cells. tube enters the ovule. One of these male gametes
73. (b) In a fertilized ovule n, 2n and 3n conditions occur fuses with egg to form diploid zygote (2n) while the
respectively in egg, nucellus and endosperm. other uses with two polar nucles of the central cell to
produce triploid primary endosperm cell (PEC) (3n).
74. (a) The seed coat develops from integuments originally
So, in some angiosperms these two types of fusion
surrounding the ovule. It is thick and hard in coconut
occur in the same embryo sac. This phenomenon is
which protects the embryo from mechanical injury
called double fertilization.
and from drying out.
82. (c) Unisexuallity of flowers prevents autogamy but not
75. (c) Ovary wall forms pericarp after fertilization. Pericarp geitonogamy because autogamy is the transfer of
is the tissue that develops from the ovary wall of the pollen grains from anther to stigma of same flower
flower and surrounds the seeds. The pericarp is and geitonogamy is the fertilization of a flower by
typically made up of three distinct layers: the epicarp pollen from another flower on the same (or a
EBD_7209
Hints & Solutions S-137

genetically identical) plant. Therefore, geitonogamy events where a zygote and endosperm are both
occurs between bisexual flowers or unisexual flowers formed. Endosperm is a tissue in which the nutrient
of the same plant. substances necessary for the development of the
83. (b) Endosperm is the nutritive tissue which provides embryo are deposited. In angiosperms the endosperm
nourishment to the embryo in seed plant. Albuminous is formed after fertilization as a result of the merging
seeds retain a part of endosperm as it is not of the spermatozoid with the secondary (diploid)
completely used up during embryo development (e.g., nucleus of the embryo sac. In this case the cells of
wheat, maize, barley, castor, sunflower). the endosperm are triploid. By uniting sets of
84. (c) The transfer of pollen grains from anther to the stigma chromosomes from both parent plants the
is called pollination. This process of pollination endosperm becomes a physiologically active, viable
occurs only in gymnosperms and angiosperms. nutrient tissue. It is not only feeds the embryo but
85. (d) Cleistogamy is a self-fertilization that occurs within fosters embryonic growth and differentiation.
a permanently closed flower. In cleistogamous flower, In gymnosperms the endosperm is formed in the
the anther and stigma lies close to each other. When ovule during germination of the megaspore; it
anther dehisces in the flower buds, pollen grains consequently becomes the female prothallus
come in contact with the stigma to effect pollination. (gametophyte), with a haploid set of chromosomes.
Thus, cleistogamous flowers are invariably 91. (b) Artificial hybridization is one of the major approaches
autogamous as there is no chance of cross - pollen of crop improvement programme. The correct
landing on the stigma. Cleistogamous flowers sequence in artificial hybridization experiment in
produce assured seed set even in the absence of bisexual flower is:
pollinators. Emasculation ® Bagging ® Cross-pollination ®
86. (d) Nucellus, embryo sac & micropyle are all found in Rebagging
ovule while pollen grain is a male gametophyte. Emasculation is the removal of the anthers of a flower
87. (c) Artificial hybridization is one of the major approaches in order to prevent self-pollination or the undesirable
of crop improvement programme. While planning for pollination
artificial hybridization programmes, if the female of neighbouring plants. After that emasculated male
parent has unisexual flowers, then process of and female plants are kept in isolation by enclosing
emasculation (removal of the anthers of a flower in them in a bag in a process called bagging. When the
order to prevent self-pollination or the undesirable stigma of bagged flowers attains receptivity, mature
pollination of neighbouring plants) does not relevant. pollen grains collected from anthers of the male
The female flower buds are bagged before the flowers parent are dusted on the stigma, and the flowers are
open. When the stigma becomes receptive, rebagged and the fruits allowed develop.
pollination is carried out using the desired pollen 92. (a) Diploid female plant will have 2 polar nuclei (each
and the flower rebagged. haploid) with which one male gamete form tetraploid
88. (c) Cotyledons and scutellum are two homologous male plant (male gamete of tetraploid plant will be
structures in the embryo of a typical dicot and a diploid) fuses, making endosperm. So endosperm will
grass. Cotyledon is an embryonic leaf in seed-bearing be tetraploid.
plants, one or more of which are the first leaves to Male gamete + 2 polar nuclei Endosperm
appear from a germinating seed. Scutellum is the large (2n) (n ) (n ) (4n)

shield like cotyledon of the embryo of certain 93. (c) In a seed of maize, the scutellum is considered as
monocots. It is specialized for the absorption of food cotyledon because it absorbs food materials and
from the endosperm. supplies them to the embryo.
89. (c) 100 zygotes require 100 pollen grains and 100 embryo 94. (c) Each microspore mother cell gives rise to 4
sacs. 100 pollen grains are formed from 25 microspore microspores which develop into pollen grains.
mother cells while 100 embryo sacs are formed from 95. (d) Depending upon the mode of its formation,
100 functional megaspores which in turn are angiospermic endosperm is of three types nuclear,
produced by 100 megaspore mother cells since three cellular and helobial. Multinucleate condition is
out of four megaspores degenerate in each case. presen t in liquid endosper m of coconut.
90. (b) In angiospermic seed, the endosperm is formed after Multinucleate condition is also known as nuclear
fertilization. Angiosperms undergo two fertilization type of endosperm. Coconut has multicellular
S-138 Biology
endosperm (called coconut meal) in the outer part lamina. Some of these cells stop proliferation and
and free nuclear as well as vacuolated endosperm differentiate into primary spermatocytes. After they
(called coconut milk) in the centre. Nuclear proceed through the first meiotic division, two
endosperm is the most common type of endosperm. secondary spermatocytes are produced. The two
It is named so because it contains free nuclei in the secondary spermatocytes undergo the second meiotic
beginning. division to form four haploid spermatids. These
96. (c) After entering the ovule, the pollen tube is attracted spermatids differentiate morphologically into sperm
towards the micropylar end of the embryo sac. The by nuclear condensation, ejection of the cytoplasm
attractants are secreted by synergids or help cells. and formation of the acrosome and flagellum.
The pollen tube pierces one of the two synergids 8. (b) The immature male germ cells or primary
and bursts open into it. The synergid is spermatocyte duplicates its DNA and subsequently
simultaneously destroyed. undergoes meiosis I which is a reductional division
97. (d) Filiform apparatus in the form of finger-like projection to produce two haploid secondary spermatocytes.
from cell wall is present in the upper part of each 9. (d) Spermiogenesis is the final stage of spermatogenesis,
synergids. This apparatus is useful for the absorption
which involves the maturation of spermatids and, its
and transportation of materials from the nucellus to
transformation into mature, motile spermatozoa.
the embryo sac.
10. (c) Each primary spermatocyte duplicates its DNA and
Chapter 25 : Human Reproduction subsequently undergoes meiosis I to produce two
equal, haploid secondary spermatocytes. Each of the
1. (b) The epididymis leads to vas deferens that ascends
two secondary spermatocytes further undergoes
to the abdomen and loops over the urinary bladder.
meiosis II to produce two haploid spermatids.
2. (b) Foreskin is a double-layered fold of smooth muscle
Therefore, total four equal, haploid spermatids are
tissue, blood vessels, neurons, skin, and mucous
producted after second meiotic division.
membrane that covers and protects the glans penis
11. (d) Gonadotropin-releasing hormone (GnRH) is released
and the urinary meatus when the penis is not erect.
from the anterior pituitary. GnRH activity is very low
The foreskin is mobile, fairly stretchable, and acts as
during childhood, and is activated at puberty or
a natural lubricant.
3. (d) The urethra originates from the urinary bladder and adolescence. At the puberty increased secretion of
extends through the penis to its external opening GnRH start the process of sperm formation.
called urethral meatus. 12. (c)
4. (c) The vasa efferentia leave the testis and open into 13. (b) Androgen is a male sex hormone which is responsible
epididymis located along the posterior surface of for proper functioning of male sex accessory glands
each testis. The epididymis leads to vas deferens. and ducts.
Or we can say that the vasa efferentia or efferent 14. (b) Semen, or seminal fluid, is an alkaline fluid that
ducts connect the rete testis with the initial section contains spermatozoa embedded in seminal plasma.
of the epididymis. Semen is ejaculated by male reproductive system
5. (b) The uterus is a female reproductive organ located during orgasm.
between the bladder and the rectum, in the pelvic
15. (c) Ejaculation of human male contains about 200 – 300
area. The main purpose of the uterus is to nourish a
million sperms, of which for normal fertility 60 %
foetus prior to birth. In menstruating females, the
sperms must have normal shape and size and at
ovaries release eggs which travel via the fallopian
least 40 % must show energetic motility.
tubes to the uterus.
6. (a) Clitoris is a small, sensitive, erectile part of the female 16. (d) The zona pellucida is a glycoprotein thick non-
genitals at the anterior end of the vulva. It is cellular membrane surrounding the plasma membrane
homologous with the penis. of an oocyte or secondary oocytes.
7. (d) Immature male germ cells (also called as 17. (a) The mature spermatozoa are released from the
spermatogonia) produce sperms in the process of protective sertoli cells into the lumen of the
spermatogenesis. Spermatogonia proliferate seminiferous tubule and a process called spermiation
continuously by mitotic divisions around the outer then takes place, which removes the remaining
edge of the seminiferous tubules, next to the basal unnecessary cytoplasm and organelles.
EBD_7209
Hints & Solutions S-139

18. (c) In biology, antrum is a general term for a cavity or mass. The trophoblast layer then gets attached to
chamber, which may have specific meaning in the endometrium and the inner cell mass gets
reference to certain organs or sites in the differentiated as the embryo.
body.Tertiary follicle of ovary contains a fluid filled 28. (d) The blastocyst is a thin-walled hollow structure in
cavity called antrum and a secondary oocyte ready early embryonic development. It possesses an inner
for ovulation. cell mass (ICM) which subsequently forms the
19. (c) The corpus luteum secretes large amounts of embryo. The outer layer of the blastocyst consists
progesterone which is essential for maintenance of of cells collectively called the trophoblast. This layer
the endometrium and the pregnancy but its decrease surrounds the inner cell mass and a fluid-filled cavity
in secretion triggers the menstrual cycle. known as the blastocoel. The trophoblast gives rise
to the placenta.
20. (b) The ovulation (ovulatory phase) is followed by the
29. (b) Morula is an early stage embryo consisting of cells
luteal phase (latter phase of the menstrual cycle)
(called blastomeres) in a solid ball contained within
during which the remaining parts of the Graafian
the zona pellucida. The morula is produced by a
follicle transform as the corpus luteum, which
series of cleavage (mitotic) divisions of the early
produces progesterone. So progesterone is highest
embryo, starting with the single-celled zygote. Once
at luteal phase. the embryo has divided into 16 cells, it begins to
21. (c) The menstrual phase is followed by the follicular resemble a mulberry, hence the name.
phase. During this phase, the primary follicles in the 30. (d) Ovum contains the haploid set of chromosomes with
ovary grow to become a fully mature Graafian follicle one of the X chromosomes. The haploid set of
and simultaneously the endometrium of uterus chromosomes in the male gamete, sperm has either
regenerates through proliferation. the X or Y chromosome. Thus, the sex of the foetus
22. (a) Luteal phase is also called as the secretory phase of depends on the male gamete fertilizing the ovum.
menstruation cycle. The luteal phase begins with 31. (a) The outer wall of the blastocyst i.e., the trophoblast
the formation of the corpus luteum and ends in either gets attached to the endometrium of the uterus during
pregnancy or luteolysis. The main hormone implantation.
associated with this stage is progesterone, which is 32. (b) After implantation, the trophoblast develops finger-
significantly higher during the luteal phase than other like projections on its outside known called as
phases of the cycle. chorionic villi, which are surrounded by the uterine
23. (a) During pregnancy all events of the menstrual cycle tissue and maternal blood.
stop and there is no menstruation. 33. (a)
24. (b) The process of insemination is the release of semen 34. (b) The chorionic villi and the endometrium become
containing male gametes, the sperms, into the female interdigitated with each other and together form a
reproductive tract during coitus. structural and functional unit between developing
25. (b) During human embryogenesis, the blastocyst arises embryo (foetus) and maternal body called placenta.
from the morula in the uterus, after 5 days of 35. (d) Apart from being a nourishing medium for the
fertilization. The early embryo undergoes cell developing foetus, the placenta also acts as an
differentiation and structural changes to become the endocrine tissue, producing hormones like human
blastocyst. It is then prepared for implantation into chorionic gonadotropin (hCG), human placental
the uterine wall 6 days after fertilization. Implantation lactogen (hPL), estrogens, progestogens, etc.
marks the end of the germinal stage and the 36. (d) Primary germ layers are ectoderm, endoderm &
beginning of the embryonic stage of development. mesoderm. Many animals are primarily triploblastic,
as endoderm (inner) and ectoderm (outer) interact to
26. (a) In sexual reproduction in organisms, when the two
produce a third germ layer, called mesoderm (middle).
nuclei of male and female gametes fuse together the
Together, the three germ layers will give rise to every organ
immediate cell which is formed is called the zygote.
in the body, from skin and hair to the digestive tract.
The zygote goes on to become an embryo and later
stages of development. 37. (c) Oxytocin acts on the uterine muscle and causes its
27. (d) In the blastocyst, the blastomeres are arranged into strong contractions, which inturn further stimulates
an outer layer called trophoblast and an inner group the secretion of oxytocin making a kind of feedback
of cells attached to trophoblast called the inner cell loop. Thus, the resultant stimulatory reflex between
S-140 Biology
the uterine contraction and oxytocin secretion 46. (b) The milk produced during the initial few days of
continues resulting in stronger and stronger lactation is called colostrum which is somewhat
contractions. sticky and yellowish or greyish white in colour.
38. (d) Whatever milk is produced during the initial few days 47. (d) Bulbourethral gland, also called as Cowper's gland,
of lactation is called colostrum. This contains several is one of two small exocrine glands in the male
antibodies absolutely essential to develop resistance reproductive system. This gland releases a small
for the new-born babies. amount of fluid just prior to ejaculation to decrease
39. (a) The mammary glands of the female undergo the acidity in the urethra. Bulbourethral gland are
differentiation during pregnancy and starts homologous to Bartholin's glands present in females.
producing milk towards the end of pregnancy by the 48. (c) By the end of 12 weeks (first trimester), most of the
process called lactation. major organ systems are formed, for example, the
40. (b) Male germ cells and sertoli cells are two types of limbs and external genital organs are well-developed.
cells present in the lining of seminiferous tubules. 49. (b) Fully developed foetus and the placenta induce foetal
Male germ cells give rise to male gamete of an ejection reflex. Foetal ejection reflex is also called
organism that reproduces sexually. A sertoli cell (a mild uterine contraction.
kind of sustentacular cell) is a nurse cell of the 50. (b) Scrotum is a part of the external male genitalia located
testicles whose main function is to nourish the behind and underneath the penis. It is the small
developing sperm cells through the stages of muscular sac that contains and protects the testicles,
spermatogenesis. Sertoli cells also act as phagocytes, blood vessels, and part of the spermatic cord. The
consuming the residual cytoplasm during scrotum protects the testicles (or testes, the primary
spermatogenesis. male sex organ) from temperature changes. In order
41. (a) The womb (or uterus) opens into vagina through to insure normal sperm production, the scrotum
cervix. The cervix is the lower part of the uterus. In a keeps the testes at a temperature slightly cooler than
non-pregnant woman, the cervix is usually between the rest of the body by contracting or expanding.
2 and 51. (a) Urethra is not a paired structure in male. The urethra
3 cm long and roughly cylindrical in shape. The is a tube that connects the urinary bladder to the
narrow, central cervical canal runs along its entire urinary meatus for the removal of fluids from the
length, connecting the uterine cavity and the lumen body. In male, the urethra travels through the penis,
of the vagina. The opening into the uterus is called and carries semen as well as urine and in female, the
the internal os and the opening into the vagina is called urethra is shorter and emerges at the female external
the external os. urethral orifice above the vaginal opening.
42. (b) First polar body is formed during the formation of 52. (a) Blastomeres are a type of cell produced by cleavage
secondary oocytes and completion of Ist meiotic
(cell division) of the zygote after fertilization and are
division. In humans, the secondary oocytes are produced
an essential part of blastula formation.
when the primary oocytes complete meiosis I.
54. (b)
43. (d) Second meiotic division in secondary oocyte results
in the formation of a second polar body & a haploid 55. (c) Each testes has about 250 compartments called
ovum (ootid). testicular lobules.Each lobule contains one to three
highly coiled seminiferous tubules. Bulbourethral
44. (d) In human beings, menstrual cycles ceases around
glands are paired male accessory glands.
50 years of age; that is termed as menopause.
56. (d) Sperm are haploid male gametes. Oviducts are a part
45. (c) Fertilization (the fusion of male and female gametes)
induces the completion of the meiotic division of of female reproductive system. Scrotum maintains
secondary oocyte. The secondary oocytes will be testes at lower (2–2.5 degrees) than normal body
arrested at the stage of metaphase II of meiosis II temperature. Sertoli cells certainly nourish the
until fertilization takes place. Thus, when a sperm developing male germ cells.
cell fertilizes the ovum, the secondary oocyte rapidly 57. (c) The process of fertilization takes place in the
completes the remaining stages of meiosis II, giving ampullary part of fallopian tube. Uterus does not
rise to an ootid and an ovum, which the sperm cell secrete any steroid hormones. It's the ovary which
unites with. secretes the steroid hormones.
EBD_7209
Hints & Solutions S-141

58. (b) The glandular tissue of each breast is divided into penis. It is devoid of erectile tissue and high blood
15-20 mammary lobes containing clusters of cells supply as present in penis. Penis is the copulatory
called alveoli. organ of males.
70. (d) Hyaluronidase, a hydrolytic enzyme is an acrosomal
59. (b) Graafian follicle releases secondary oocyte from the content in mammalian sperm. It helps at the time of
ovary by the process of ovulation. Primary oocyte fertilization during the penetration of sperm into
within the tertiary follicle grows in size and completes ovum. Based on the amount of yolk mammalian eggs
its first meiotic division. are alecithal means egg without yolk. Microlecithal
60. (d) Menstrual cycle is the cycle of natural changes that eggs are with very little yolk e.g., sea urchin, starfish.
On the basis of distribution of yolk telolecithal eggs
occurs in the uterus and ovary as an essential part
are those eggs in which the yolk is concentrated
of making sexual reproduction possible. Menstrual
towards the vegetal pole and cytoplasm and nucleus
fluid cannot easily clot. The end of the cycle of lie near the animal pole, e.g., birds and reptiles.
menstruation is called menopause. During the 71. (c) Leydig cells, also known as interstitial cells, are found
follicular phase, gonadotropins (LH and FSH) adjacent to the seminiferous tubules in the testicle.
increase gradually and stimulate follicular They produce testosterone in the presence of
development as well as secretion of estrogen by luteinizing hormone (LH).
growing follicles. 72. (d) Testicular lobules are the compartments present in
61. (c) During pregnancy, the increased production of the the testes, but the lobules as whole are not involved
levels of hormones like estrogen, progestogens, in the process of fertilization. Fusion of male and
cortisol, prolactin, thyroxine in the maternal blood female gametes are called fertilization.
are essential for supporting the foetal growth, 73. (c) Head of a sperm has acrosome but the spiral row of
metabolic changes in the mother and maintenance mitochondria are present in the mid (connecting)
of pregnancy. piece of the sperm.
62. (a) Statement (i) and (iv) are correct. Ovary is the primary 74. (b) Seminiferous tubules are located in the testes, which
female sex organ which produces female gamete are the specific location of meiosis, and the
(ovum) and steroid hormones. Ovarian stroma is subsequent formation of sperms. Rete testis is a
divided into peripheral cortex and inner medulla. network of small tubules found in the part of the
Mammary gland is a paired structure that contains testicle that carries sperm. Leydig cells or interstitial
glandular tissue and variable amount of tissue. It is cells of Leydig produce the hormone called
the characteristic of all female mammals. testosterone which contributes to male secondary
63. (b) sexual characters. Prepuce is the foreskin, skin
64. (a) Parturition, also called birth or childbirth, is a process surrounding and protecting the head of the penis as
of bringing forth a child from the uterus, or womb. well as clitoris.
Oxytocin induces uterine contraction. 75. (d) A spermatogonium divides into two primary
65. (c) Fertilization is the fusion of male and female gamets spermatocytes. Each primary spermatocyte
to form zygote. During fertilization only head of the duplicates its DNA and subsequently undergoes
sperm enters egg. After that polyspermy is avoided meiosis I to produce two haploid secondary
by fertilization membrane. spermatocytes. Each of the two secondary
66. (a) Morula involves cleavage of cells till 32 cell stage is spermatocytes further undergoes meiosis II to
formed. It is still surrounded by Zona pellucida. produce two haploid spermatids.
67. (a) Death is the ultimate goal of every organism. This is 76. (d) The menstrual cycle is the series of changes a
caused by the wear and tear of organs which woman's body goes through to prepare for a
constitute the body of a living being. pregnancy. About once a month, the uterus grows a
68. (b) In female, Graafian follicle forms corpus luteum after new lining (endometrium) to get ready for a fertilized
ovulation. The cells of corpus luteum are called luteal egg. When there is no fertilized egg to start a
cells. The cytoplasm of luteal cells have yellow pregnancy, the uterus sheds its lining. This is the
granules called lutein which secrete the hormone monthly menstrual bleeding (also called menstrual
progesterone to maintain pregnancy if fertilization period).
takes place. In the absence of fertilization, corpus 77. (b) 78. (d) 79. (b) 80. (c)
luteum degenerates and forms corpus albicans and 81. (d) Fertilization is the fusion of male and female gametes
there is decrease in progesterone level also. which takes place in the ampullary isthmic junction
69. (c) Clitoris is a female reproductive organ. It is of fallopian tube. Implantation is the attachment of
homologous to penis of males. It is not remnant of the blastocysts in the endometrium. Cleavage is the
mitotic cell division which takes place on fertilized
S-142 Biology
egg. Morula is the solid mass of 32 cells formed from (proximal centriole and distal centriole). Distal
the zygote after successive mitotic division. centriole gives rise to axial filament of the sperm
Blastocysts is the developmental stage which which runs up to the end of the tail. The middle piece
embedded in the uterine endometrium by a process has a central filamentous core with many spirally
called implantation and leads to pregnancy. The arranged mitochondria which is used for ATP
blastocyst is a structure formed in the early production. The tail or flagellum performs the beating
development of mammals. movements that propel the spermatocyte.
82. (c) Parturition is the delivery of foetus or child birth. 91. (c) Oogonia are called as gamete mother cell. Corpus
Ovulation is the release of egg from Graafian follicle luteum is formed as a temporary endocrine structure
on 14th day during menstrual cycle. Gestation is the after the ovulation. It is involved in the production
duration of pregnancy and birth. Implantation is the of relatively high levels of progesterone and
attachment of the blastocysts to the endometrium moderate levels of estradiol and inhibin A to maintain
layer of the uterus. Conception is the formation of pregnancy. A large number of primary follicles
zygote by fusion of the egg and sperm. degenerates during the phase from birth to puberty.
83. (d) The marked structure A, B, C and D are respectively 92. (d) A - Primary; B - I; C - Ovulation; D - Secondary
called as seminal vesicles, urinary bladder, ejaculatory oocyte
duct and bulbourethral gland. Bulbourethral gland, 93. (a) A–LH, B–Ovulation, C–Menstruation,
also called Cowper's gland is a pea sized gland that D–Proliferative, E–Luteal
lies beneath the prostate gland and whose secretion LH is luteinizing hormone secreted from the anterior
neutralizes the acidity of the urine in the urethra. pituitary gland. In females, an acute rise of LH (called
84. (c) A-Vas deferens B- Seminal vesicle, C- Prostate, LH surge) triggers ovulation and development of
D- Bulbourethral gland. the corpus luteum. Ovulation is the process when a
85. (b) The given figure shows the sectional view of mature egg is released from the ovary, pushed down
seminiferous tubule. The seminiferous tubules are the fallopian tube, and is available for fertilization.
the site of the germination, maturation, and Menstruation is a woman's monthly bleeding.
transportation of the sperm cells within the male Menstrual blood flows from the uterus through the
testes. Seminiferous tubules are made up of columnar small opening in the cervix and passes out of the
sertoli cells surrounded by spermatogenic cells on body through the vagina. Proliferative and luteal are
the epithelial interior and stem cells exteriorly. phases of the menstrual cycle.
86. (b) Spermatogonia are undifferentiated germ cells which 94. (a) In the given figure of ovum surrounded by many
originate in seminiferous tubules and divide into two sperms, the parts labeled as A, B, C and D are
primary spermatocytes (a kind of germ cell) in the respectively zona pellucida, ovum, cells of corona
production of spermatozoa. radiata and perivitelline space. The zona pellucida is
87. (c) In the given figure of female reproductive system, a glycoprotein layer which surrounds the plasma
the marked structures (A to F) are the parts of uterus membrane of oocytes. It is essential for oocyte death
and fallopian tube. A to F are respectively and fertilization. Ovum, released from the ovaries, is
endometrium, myometrium, perimetrium, isthmus, capable of developing into a new organism when
ampulla and infundibulum. fertilized (united) with a sperm cell. Corona radiata is
88. (a) The structures marked in the figure of mammary gland the outermost, protective coat of an unfertilized egg
are A–mammary lobe, B–mammary duct, C–ampulla, of a female. It is quite thick and many of the sperms
and D–lactiferous duct. The mammary gland is a release hyaluronidase to break and penetrate it in
gland located in the breasts of females that is order to reach the zona pellucida, which is the layer
responsible for lactation. Mammary glands only next to it. It keeps adhering to the zona pellucida
produce milk after childbirth. Mammary lobe (A) even after the ovulation (release of the egg from the
contains clusters of cells called alveoli which secrete follicle) so that no more than one sperm can reach
milk which is stored in the cavities of alveoli. the ovum. The perivitelline space is the space
89. (b) 'B' are the secondary spermatocytes which further between the zona pellucida and the cell membrane of
undergoes meiosis II to produce two haploid an oocyte or fertilized ovum.
spermatids. 95. (c) Figure (c) refers to blastocysts stage. Blastocyst is
90. (a) The mammalian sperm cell consists of a head, neck, embedded in the uterine endometrium by a process
middle piece and a tail. The head contains the called implantation and leads to pregnancy. It
elongated haploid nucleus with densely coiled possesses an inner cell mass (ICM) which
chromatin fibres, surrounded anteriorly by an subsequently forms the embryo. The outer layer of
acrosome. Acrosome contains en zymes or the blastocyst consists of cells collectively called
penetrating the female egg. Neck contains centrioles the trophoblast. This layer surrounds the inner cell
EBD_7209
Hints & Solutions S-143

mass and a fluid-filled cavity known as the blastocoel. 108. (c) The primary sex organ of male reproductive system
The trophoblast gives rise to the placenta. is the pair of testis, which is surrounded by scrotum
96. (c) A–Umbilical cord with its vessels, B–Placental villi, to maintain the former at a lower temperature than
C–Yolk sac, D–Plug of mucus in cervix the normal body temperature to ensure proper
97. (c) The given figure shows that ovum is surrounded by spermatogenesis.
many sperms. These two gametes (ovum and sperms) 109. (a) The urethra originates from a structure [called urinary
were going to take part in fertilization. During bladder (X)] and extends through the male external
fertilization, a sperm comes in contact with the zona genitalia [called penis (Y) which helps in introducing
pellucida layer of ovum and induces the changes in semen into the vagina) to its external opening called
membrane that blocks the entry of additional sperm. urethral meatus.
98. (a) The structure marked as X is rete testis. The rete 110. (c) Male and female gametes are fused in ampullary-
testis is an anastomosing network of delicate tubules isthmic junction of fallopian tube. Fusion of gametes
located in the hilum of the testicle (mediastinum is not the function of uterine wall.
testis) that carries sperm from the seminiferous 111. (a)
tubules to the efferent ducts. 112. (a) In males, Luteinizing hormone (LH) stimulates Leydig
99. (b) The structure marked as A, B, C and D in the given cell to produce androgen testosterone hormone.
figure of female reproductive system are respectively 113. (c) Secretion of epididymis, seminal vesicles, vas
isthmus, ampulla, infundibulum and fimbriae. Isthmus deferens, and prostate are essential for maturation
is the narrower part of the tube that links to the uterus and motility of sperm.
and the interstitial part that transverses the uterine 114. (d) A spermatogonium divides into two primary
musculature. Ampulla is the part behind the spermatocytes. Each primary spermatocyte
infundibulum, where fertilization of ovum takes place. duplicates its DNA and subsequently undergoes
Infundibulum is a funnel shaped structure whose meiosis - I to produce two haploid secondary
edges possess finger like projection called fimbriae. spermatocytes. Each of the two secondary
Fimbriae help in the collection of ovum after spermatocytes further undergoes meiosis II to
ovulation. produce two haploid spermatids.
100. (d) X-Follicle stimulating hormone (FSH) acts on the 115. (a) Only one ovum is released in the middle of each
D-Sertoli cells and stimulates secretion of some menstrual cycle.
factors which help in the process of spermiogenesis. 116. (b) During luteal (P) phase of the menstrual cycle, if
101. (c) The given figure shows the role of ovarian hormones pregnancy doesn't happen, the corpus luteum (Q)
and growth of ovarian follicles in the various phases withers and dies, usually around day 22 in a 28-day
of menstrual cycle. cycle. The drop in progesterone (R) levels causes
102. (a) A-Umbilical cord with its vessels, is the cord through the lining of the uterus to fall away. This is known as
which placenta is connected to the developing fetus. menstruation (S).
103. (c) The given figure shows the morula stage of 117. (c) Both LH and FSH attain a peak level in the middle of
embryonic development. Morula is produced by a menstrual cycle (about 14th day) resulting in
series of cleavage (mitotic) divisions of the early ovulation.
embryo, starting with the single-celled. Once the 118. (b) The cycle starts with the menstrual phase. When
embryo has divided into 16 cells, it begins to menstrual flow occurs, it lasts for 3-5 days. The
resemble a mulbery, hence the name. menstrual phase is followed by the follicular phase
104. (a) A-Interstitial cells, B-Spermatogonia, C-Spermatozoa, or proliferative phase. The luteal or secretory phase
D-Sertoli cells. In the lining of seminiferous tubules, just follows the former and completes the cycle. Then,
interstitial cells and sertoli cells are present. all new cycle begins thereafter.
105. (a) The proper movement of tail facilitates the sperm 119. (d) Menstrual cycle is the cycle of natural changes that
motility ensuring its reach to the ovum for fertilisation. occurs in the lining of the uterus and ovary as an
106. (d) A-Endometrium, B-Fallopian tubule, C-Cervix, essential part of making sexual reproduction possible.
D-Vagina. Vagina is located posterior to urethra and LH, FSH, estrogen and progesterone hormones
anterior to rectum, it receives sperms during sexual controlled the menstrual cycle.
intercourse, serves as exit for blood during menstrual 120. (a) The zygote divides mitotically to form 8, 16 daughter
flow & serves as birth canal during child birth. cells called blastomeres. This stage is called morula
107. (d) The given flowch art shows the process of which continues to divide and transforms into
spermatogenesis. The marked steps A to D are blastocyst.
respectively mitosis differentiation, Ist meiotic division, 121. (d) The ampullary- isthmus junction is the particular
2nd meiotic division and 23. place where actually fertilization occurs. This is the
S-144 Biology
crucial place for the fertilization because the ovum starts as the zygote moves through the isthmus of
released by the ovary is transported to the ampullary- the oviduct (fallopian tube) towards the uterus.
isthmic junction. 134. (a) The pregnancy would be terminated if placenta is
122. (c) Implantation is not the immediate process. It takes 5 failed to develop.
days to occur after fertilization.
123. (c) The acrosome (head of the sperm) contains the Chapter 26 : Reproductive Health
necessary enzymes(hyaluronidase and acrosin) to 1. (c) The World Health Organization (WHO) is a
penetrate the membrane of the ovum. specialized agency of the United Nations (UN) that
124. (c) Fertilization through the process of the fusion of
is concerned with international public health. It was
haploid male and female gametes, ensures the
established on 7 April 1948, headquartered in Geneva,
restoration of the diploidy of the human foetus.
125. (b) Vigorous contraction of the uterus at the end of Switzerland.
pregnancy causes expulsion/delivery of the foetus 2. (b) India is the first country to initiate action plans and
(parturition). Parturition originates from the fully programmes at a national level to aware people of
developed foetus and the placenta which induces the issues of reproductive health and made it one of
mild uterine contractions called foetal-ejection reflex. the social issues to be tackled with good efforts.
126. (b) Vigorous contraction of the uterus at the end of the
3. (a) The programmes called ‘family planning’ for
pregnancy causes expulsion of the foetus.
reproductive health awareness were initiated in 1951.
127. (c) Each human somatic cell is diploid with a set of pair
of 23 chromosomes, so the total number of 4. (d) Reproduction-related areas are currently in operation
chromosomes is 46. Spermatogonia are also diploid in India come under the popular name ‘Reproductive
in nature having 46 chromosomes. and Child Health Care (RCH) programmes.
128. (d) Secondary spermatocytes are the result of meiotic 5. (b) Amniocentesis or amniotic fluid test or AFT is a
division, so they are the haploid ones.
medical procedure which is used in prenatal
129. (c) After the transformation of spermatids into sperm,
diagnosis of chromosomal abnormalities and foetal
their heads become embedded in a cell called sertoli
cells (X) and are finally released from the seminiferous infections. This is the same procedure used for sex
tubule (Y) by the process called spermiation (Z). determination.
130. (c) Sperm is the male reproductive cell and consists of a 6. (b) ‘Saheli’–an oral contraceptive for the females was
head, neck, middle piece and a tail. The head of the developed at Central Drug Research Institute (CDRI)
sperm comprises the nucleus (DNA) and tip contains in Lucknow, India.
acrosome which enables the sperm to penetrate the
7. (d) Marriageable age is the age at which a person is
egg. Neck contains centrioles which are proximal
allowed by law to marry. In India this age has been
centriole and distal centriole. Distal centriole gives
rise to axial filament of the sperm which runs up to set as 18 for woman and 21 for man (without parents’
the end of the tail. The middle piece contains the consent).
mitochondria which supplies the energy in the form 8. (b) Lactational amenorrh oea is the absence of
of ATP the tail needs to move. The tail moves with menstruation. It is the breast sucking of mother by
whip-like movements back and forth to propel the her child for a long time which is considered to
sperm towards the egg. contribute a gap for pregnancy. It is based on the
131. (c) Oogenesis is initiated during the embryonic fact that ovulation and the menses do not occur
development stage when some million oogonia (A) during the period of intense lactation following
are formed within each foetal ovary. No more oogonia
parturition.
are formed and added after birth. These cells start
division and enter into prophase-I (B) of the meiotic 9. (c) The use of condoms during coitus is some of the
division and get temporarily arrested (C) at that stage, simple precautions to avoid contacting STDs like
called primary oocytes (D). AIDS and syphilis.
132. (c) During fertilization in humans, when many sperms 10. (b) Diaphragms, cervical caps and vaults are reusable
reach close to the ovum, the secretion of acrosome barrier contraceptives made of rubber that are inserted
helps the sperm to enter into the cytoplasm of the
into the female reproductive tract to cover the cervix
ovum through the zona pellucida and the plasma
during coitus. They prevent conception by blocking
membrane. This induces the completion of the meiotic
division of the secondary oocyte. the entry of sperms through the cervix.
133. (d) The given statement explains the process of 11. (c) Condom, diaphragm, cervical caps, and vaults are
cleavage. In the process of cleavage, mitotic division included under barrier methods of contraception.
EBD_7209
Hints & Solutions S-145

Barrier methods of birth control are physical or 20. (c) Sexually transmitted diseases (STDs) are caused by
chemical barriers that prevent sperm from passing infections that are passed from one person to another
through the woman's cervix into the uterus and during sexual contact. Most STDs initially do not
fallopian tubes to fertilize an egg. Some methods also cause symptoms. Symptoms and signs of disease
protect against sexually transmitted disease (STDs). may include: vaginal discharge, penile discharge,
12. (a) IUDs stand for Intra Uterine Devices. These are ulcers on or around the genitals, and pelvic pain.
plastic or metal devices placed in the uterus. They 21. (d) 22. (b)
contain either copper or a common synthetic 23. (c) Intra cytoplasmic sperm injection (ICSI) is an in-vitro
hormone used in oral contraceptives. These include fertilization procedure in which a single sperm is
loop, copper-T, spiral etc. They prevent fertilization injected directly into an egg. This procedure is most
of the ess or implantation of egg. commonly used to overcome male infertility
13. (c) Copper ions play an important and effective role in problems, although it may also be used where eggs
the activity of IUDs. Copper IUDs primarily work by cannot easily be penetrated by sperm, and
disrupting sperm mobility and damaging sperm so occasionally in addition to sperm donation.
that they are prevented to take part in the process of 24. (b) ZIFT (Zygote intra fallopian transfer) is the process
fertilization. The increased copper ions in the cervical in which the zygote or early embryos (with upto 8
mucus inhibit the sperm's motility and viability, blastomeres) is transferred into the fallopian tube.
preventing sperm from traveling through the cervical
25. (d) Test tube baby programme employs zygote intra
mucus or destroying it as it passes through.
fallopian transfer (ZIFT) technique. In this technique
14. (b) Intra uterine device, the most widely accepted method fusion of ovum and sperm is done outside the body
of contraception in India, is a method of birth control. of woman to form zygote which is allowed to divide
It is designed for insertion into a woman's uterus so forming 8 blastomeres, then it is transferred into the
that changes occur in the uterus that makes it difficult fallopian tube of the woman.
for fertilization of an egg and implantation of a
26. (b) In artificial insemination (AI) technique, the semen
pregnancy. IUDs also have been referred to as "intra
collected either from the husband or a healthy donor is
uterine contraception (IUC).
artificially introduced either into the vagina or into the
15. (d) The hormone releasing Intra Uterine Devices (IUDs) uterus (IUI – intrauterine insemination) of the female.
such as Progestasert and LNG-20 increase
27. (a) Gamete Intra fallopian Transfer (GIFT) is
phagocytosis of sperms within the uterus and makes
recommended for those females who cannot produce
the uterus unsuitable for implantation & the cervix
an ovum. In this process, the eggs of the donor
hostile to the sperms.
woman are removed and in a form of mixture with
16. (a) Oral administration of small doses of either sperm transferred into fallopian tube of another
progestogens or progestogen–estr ogen woman who cannot produce ovum, but can provide
combinations (in the form of pills) is one of the suitable environment for fertilization. Thus in GIFT,
contraceptive methods used by the females. site of fertilization is fallopian tube, not laboratory.
17. (d) Female steriliztion (called tubectomy) prevents 28. (d) Infertility problems cannot be generalised. It is a false
fertilization by interrupting te passage troug fallopian statement to say that in India the infertility problems
tube. Eggs continues to be produced but they fail to lie in females more often.
pass into the uterus. In tubectomy, a small part of
29. (c) Natural method of contraception works on the
the fallopian tube is removed or tied up through a
principle of avoiding chances of ovum and sperm
small incision in the abdomen or through vagina.
meeting. include periodic abstinence, coitus
18. (a) Medical Termination of Pregnancy (MTP) was interrupts and lactational ammenorrhea.
legalised in 1971 by Government of India with some
30. (a) STDs are those infections or diseases which are
strict conditions to avoid its misuse.
transmitted through sexual intercourse. STDs are
19. (a) MTP is the intentional or voluntary termination of more common in15-24 years age group. Gonorrhoea,
pregnancy before term. MTPs are considered syphilis, genital herpes, chlamydiasis, genital warts,
relatively safe during the first trimester, i.e., up to 12 trichomoniasis, hepatitis-B and AIDS are some of
weeks of pregnancy. the common STDs. Haemophilia is a sex-linked
recessive disease, which shows its transmission from
S-146 Biology
unaffected carrier female to some of male progeny. and in vitro fertilized. The resulting zygote is placed
Sickle cell anaemia is an autoimmune linked recessive into the fallopian tube.
trait in which both the partners are carrier for the 41. (d) The Intra Uterine Device (IUD) is a small ‘T’ shaped
gene. device with a monofilament tail that is inserted into
31. (a) Surgical methods of contraception are generally the uterus by a health care practitioner. It increases
advised for male/female partner to prevent any more phagocytosis of sperm within the uterus and the
pregnancies. It blocks gametes transport and thereby copper ion released suppresses sperm motility and
prevent conception. MTPs are relatively upto first fertilizing capacity of sperms.
trimester i.e. first three months of the pregnancy. 42. (a) The diaphragm is one of a barrier method of
32. (d) Contraceptive pills have to be taken daily for a period contraception. It is a shallow, dome-shaped cup with
of 21 days starting preferably within the first five a flexible rim and made of silicon. It is inserted into
days of menstrual cycle. It helps in the prevention of the female reproductive tract to cover the cervix. To
conception. As long as the mother feeds breast the be effective in preventing pregnancy, diaphragms
child fully, chances of conception are almost nil only need to be used in combination with spermicide,
up to a maximum period of six months following which is a chemical that kills sperm.
parturition. 43. (b) Birth control pills (oral contraceptive) check
33. (c) An ideal contraceptive should be user-friendly, easily ovulation by inhibiting the secretion of follicle
available, effective and reversible with no or least stimulating (FSH) and luteinizing hormone (LH) that
side-effects. are necessary for ovulation such contraceptives are
34. (b) Purpose of tubectomy is to prevent the gamete transport. small oral doses of either progestogens or
Genital warts are a sexually transmitted disease, caused progestogen-estrogen combinbation and are used
by some types of human papilloma virus. by the females they are used in the form of tablets
and hence are popularly called the pills.
35. (c) Reproductive health is a crucial part of general health
and a central feature of human development. It is a 44. (c) MTP is medical termination of pregnancy. It is not
reflection of health during childhood, and crucial used as a contraceptive method. It is generally
during adolescence and adulthood, Reproductive performed to get rid of unwanted pregnancies due
healthy societies have normal sex related emotional to rapes, casual relationships etc.
and behavioural interactions. 45. (d) Mother is homogametic (produce X, X) and father is
36. (c) Reproductive health is defined as a state of physical, heterogametic (X and Y). So, father is responsible
mental, and social well-being in all matters relating for the sex of the child not mother.
to the reproductive system, at all stages of life. 46. (a)
Reproductive health in society can be improved by 47. (b) AIDS can be prevented by using condoms. Condoms
introducing sex education in schools, increasing are the barrier contraceptive method. Contraceptive
medical assistance, ban on amniocentesis, spreading planted under the skin have synthetic streroid
awareness on contraception and STDs and providing preparation. It works similar to the contraceptive pills.
equal opportunities to male and female. 48. (c) Cu-7 and Cu-T are intrauterine contraceptive devices
37. (c) for females. They do not suppress sperm motility.
38. (a) Increased health facilities and rapid decrease in Their mode of action is different. Cu-T and Cu-7
maternal mortality rate are among the various reasons discharge 50-75 micrograms of ionic copper into the
of population explosion. uterus daily. These copper ions interfere with life-
sustaining functions that regulate implantation in
39. (b) Sexually Transmitted Diseases (STDs) are infections
the uterus. No any hormone is released by them.
that are commonly passed from one person to another
49. (c) Intrauterine device (IUD) Copper- T is plastic or metal
during sexual contact. Except for having unprotected
sex, all the other reasons are helpful to avoid object placed in the uterus by a doctor. Copper- T
prevents the fertilization of the egg or implantation
transmission of STDs.
of the embryo. Their presence perhaps acts as a minor
40. (c) Zygote Intrafallopian Transfer (ZIFT) is an infertility
irritant and this makes the egg to move down the
treatment. It is used when a blockage in the fallopian fallopian tubes and uterus rather quickly before
tubes prevents the normal binding of sperm to the fertilization or implantation.
egg. Egg cells are removed from a woman's ovaries,
EBD_7209
Hints & Solutions S-147

50. (b) A contraceptive pill prevents ovulation, condom releasing eggs and by thickening cervical mucus.
prevents sperms to reach the cervix, vasectomy 59. (a) Th e given figure showing th e procedur e of
allows the semen to flow but the latter contains no vasectomy, in which a small part of the vas deferens
sperms. Copper-T prevents implantation in females. is removed or tied up through a small incision on the
51. (b) Intra Uterine Devices are available as the non- scrotum.
medicated IUDs (e.g., Lippes loop), copper releasing 60. (b) Condoms are barriers made of thin rubber/ latex
IUDs (CuT, Cu7, and Multiload 375). Progestasert, sheath that are used to cover the penis in the male or
LNG-20 are hormone releasing IUDs. vagina and cervix in the female.
52. (a) 61. (a) The given figure is that of copper CuT. CuT is a
53. (d) ICSI (Intracytoplasmic sperm injection) - Formation simple copper releasing IUD made of a flexible, "T"
of embryo by directly injecting sperm into the ovum shaped piece of plastic wrapped with a thin copper
IUI (intrauterine insemination) - Artificially containing wire. It makes the uterus and fallopian
introduction of semen into the vagina or uterus tubes produce fluid that kills sperm. This fluid
IUT (Intra uterine transfer) - Transfer of embryo with contains white blood cells, copper ions, enzymes,
more than 8 blastomeres into the uterus and prostaglandins. Copper ions prevent pregnancy
by inhibiting the movement of sperm, because the
GIFT (Gamete intra fallopian transfer) - Transfer of
copper-ion-containing fluids are directly toxic to
ovum collected from a donor into the fallopian tube
sperm. Even if an aggressive little spermatozoon
where fertilization occur
fertilizes an egg, the copper ion laden environment
ZIFT (Zygote intra fallopian transfer) - Transfer of prevents implantation of the fertilized egg, and thus
the zygote or early embryo (with upto 8 blastomeres) pregnancy.
into a fallopian tube.
62. (c)
54. (b) Saheli is a non- hormonal birth control pill. It is
63. (c) Inability of an individual to inseminate the female or
different from other birth control pills because it
doesn't contain any hormones. Instead of using the due to very low sperm counts in ejaculates leads to
hormone estrogen to prevent pregnancy, it contains infertility. It could be corrected by artificial technique.
a drug that blocks estrogen. In artificial technique the semen is collected and
artificially introduced either into the vagina or into
Non Medicated IUDs - Lippes loop
the uterus (IUI - intra-uterine insemination) of the
Lactational amenorrhea - Absence of menstruation female.
Diaphragms, cervical caps, vaults - Cover the cervix 64. (a) Syphilis is a chronic bacterial disease that is
during the coitus. contracted chiefly by infection during sexual
55. (a) The figure shows the process of tubectomy. This is intercourse, but also congenitally by infection of a
a surgical method to prevent pregnancy in women. developing foetus. Syphilis is completely curable if
In tubectomy small part of the fallopian tube is detected early and treated properly.
removed or tied through a small cut in the abdomen 65. (b) Rapid declines in death rate, maternal mortality rate
or through vagina. (MMR) and infant mortality rate (IMR) as well as an
56. (c) In R, the fallopian tubes have been blocked, and in increase in number of people in reproducible age are
S, they are cut out. Both the procedures completely probable reasons for the present increase in india’s
make it impossible for fertilization to occur. population.
57. (a) The given figure is of male condom. A male condom 66. (a) The Cu ions released suppress sperm motility and
is a thin sheath that covers the penis during the fertilizing capacity of sperms.
intercourse and protects against sexually transmitted
67. (a) Birth control pills (oral contraceptives) check
infection (STI) and preventing direct contact between
ovulation in female by inhibiting the secretion of
the penis and vagina, as well as collecting the semen
follicle stimulating hormone and luteinizing hormone
and preventing it from entering the vagina.
that are important for ovulation.
58. (d) The given figure is a contraceptive implant. It is a
68. (d) Sterilization techniques can be considered as the
small flexible tube which is inserted under the skin
safest birth control measures. It provides a permanent
(typically the upper arm) and prevents pregnancy
and sure birth control. It is called vasectomy in male
by releasing hormones that prevent ovaries from
and tubectomy in female.
S-148 Biology
69. (d) 80. (c) Syphilis, gonorrhea and genital warts are sexually
70. (c) Assisted reproductive technologies (ART) include transmitted disease which specifically affects the sex
a number of special techniques which assist infertile organs. Syphilis is caused by the spirochete
couples to have children. In-vitro fertilization is one bacterium Treponema pallidum. Gonorrhea is caused
such technique. by Neisseria gonorrhoeae, a bacterium that can
grow and multiply easily in mucus membranes of the
71. (d) AIDS, genital herpes and hepatitis B are sexually
body. Genital warts, which are also called
transmitted diseases which are not completely
condylomata acuminata or venereal warts, are
curable.
growths in the genital area caused by a sexually
72. (a) transmitted papilloma virus.
73. (d) Family planning allows individuals and couples to The AIDS (Acquired immunodeficiency syndrome)
anticipate and attain their desired number of children is a severe immunological disorder caused by the
and the spacing and timing of their births. It is retrovirus HIV. The illness alters the immune system,
achieved through use of contraceptive methods and making people much more vulnerable to infections
the treatment of involuntary infertility (like and diseases.
tubectomy, vasectomy). A woman's ability to space
81. (c) Sexually transmitted infections (STI), also referred
and limit her pregnancies has a direct impact on her
to as sexually transmitted diseases (STD) and
health and well-being as well as on the outcome of
venereal diseases (VD), are infections that are
each pregnancy.
commonly spread by sex, especially vaginal
74. (d) Overpopulation is an undesirable condition where intercourse, anal sex and oral sex. Most STIs initially
the number of existing human population exceeds do not cause symptoms. Symptoms and signs of
the carrying capacity of Earth. It is caused by number disease may include: vaginal discharge, penile
of factors like, reduced mortality rate, better medical discharge, ulcers on or around the genitals, and pelvic
facilities, depletion of precious resources etc. pain. Principle i, ii, and iv will not help people to
Consequences of overpopulations are depletion of become free from the infection of sexually transmitted
natural resources, degradation of environment, diseases.
shortage of food supply, rise in unemployment, high
82. (a) Hepatitis B is an infectious disease caused by the
cost of living, and increase of poverty.
hepatitis B virus (HBV) which affects the liver. HIV
75. (d) The described sterilization process is called (human immunodeficiency virus) is a virus that
vasectomy in which a small incision was done on attacks the immune system, the body's natural
the scrotum and a part of vas deferens is removed or defense system. Without a strong immune system,
tied up. Due to this gamete transport is blocked and the body has trouble fighting off disease. Both the
thereby prevents conception. diseases can also be transmitted by sharing of
76. (b) injection needles, surgical instruments, etc., with
77. (a) Emergency contraceptives are effective if used within infected persons, transfusion of blood, or from an
72 hours of coitus. infected mother to the foetus too.
78. (b) Condom is a barrier device that may be used during 83. (a) Infertility is defined as not being able to get pregnant
sexual intercourse to reduce the probability of despite having frequent, unprotected sex for at least
pregnancy and spreading sexually transmitted a year for most people and six months in certain
infections (STIs/STDs) such as HIV/AIDS. It is put circumstances. Reasons for infertility include drugs,
on an erect penis and physically blocks ejaculated diseases, and congenital problems, immunological
semen from entering the body of a sexual partner. or psychological problems.
Condoms are also used for collection of semen for Contraception is the deliberate use of artificial
use in infertility treatment. methods or other techniques to prevent pregnancy
79. (a) To get rid of unwanted pregnancies and to prevent as a consequence of sexual intercourse. Assisted
the fatality or harmfulness to the mother or to foetus reproductive technology (ART) is the technology
or both due to the continuation of pregnancy are the used to achieve pregnancy in procedures such as
reasons on the basis of which pregnancy can be fertility medication, artificial insemination, in vitro
terminated fertilization and surrogacy. It is reproductive
EBD_7209
Hints & Solutions S-149

technology used primarily for infertility treatments, up with each other but remains together and
and is also known as fertility treatment. segregate (or separate) so that offspring obtains one
factor from each parent.
84. (b) Artificial insemination is the assisted reproductive
8. (d) Mendel selected Garden pea as material for his
technology that has been used for the longest time
hybridization experiments because of the following
period. It can help treat certain kinds of infertility in
reasons:
both men and women. In this procedure, sperms are
(i) Hybridization or crossing in pea is easy.
inserted directly into a woman's cervix, fallopian tubes,
(ii) It has bisexual flowers.
or uterus. This makes the trip shorter for the sperm
(iii) It has a number of well defined contrasting
and bypasses any possible obstructions. Intra
characters.
uterine insemination (IUI), in which the sperm is
(iv) It shows predominantly self - fertilization.
placed in the uterus, is the most common form of
(v) It has a short life span.
artificial insemination.
9. (c) Selfing Ab × Ab
85. (c) F2 generation:
Chapter 27 : Principles of Inheritance & A b
Variation A AA Ab

1. (c) Mendel’s second or last law was “law of independent b Ab bb


Assortment”. It states that genes of different Phenotyppic ratio: 3 : 1
characters located in different pairs of chromosomes Genotypic ratio: AA : Ab : bb
are independent of one another in this segregation 1 : 2 : 1
during gamete formation. Ab × Ab would produce a genotypic ratio of 1:2:1 in
2. (b) Allele or allelomorph is a pair of contrasting F2 generation.
characters in Mendelian crosses. Alleles are slightly 10. (b) Punnett square is a checker-board used to show the
different forms of the same gene. result of a cross between two organisms. It was
3. (a) Monohybrid cross is the ratio which is obtained in devised by geneticist, R.C. Punnett (1927). It depicts
F2 generation when monohybrid cross is made & both genotype and phenotype of the progeny.
the offsprings of F1 generation are self bred. 11. (b) The crossing of F1 to homozygous recessive parent
Mendel crossed pure tall and dwarf plants. The plants is called test cross. Test cross is a cross between
belonged to F1 generation, all tall hybrid, were self- two individuals in which one individual shows the
pollinated. The plants of F2 generation were both dominant phenotype of a characteristic and the other
tall and dwarf, in approximate 3 : 1 ratio phenotypically individual who is homozygous recessive for that trait
and 1 : 2 : 1 genotypically. in order to determine the genotype of the dominant
4. (b) individual.
5. (a) According to law of dominance, only one character 12. (a) The test cross is used to determine the genotype of
or factor expresses itself in F1 generation and this the offsprings, to know whether the offspring is
character is called dominant. Therefore in this cross, homozygous or heterozygous.
character of tallness, which is observed in all the 13. (c) The mode of inheritance in case of multiple alleles is
plants of F1 generation, is the dominant character. called multiple allelism. A well known and simplest
6. (c) Incomplete dominance is the phenomenon where example of multiple allelism is the inheritance of ABO
dominant allele does not completely express itself. blood groups in human beings. In human population,
This phenomenon was first studied in flower colour 3 different alleles for this characters are found – IA,
of Mirabilis jalapa or four O’ clock plant. The IB and IO. A person is having only two of these three
phenotypic as well as genotypic monohybrid ratio alleles and blood type can be determined.
in F2 generation in incomplete dominance is 1 : 2 : 1 14. (c) Codominance is the phenomenon of two alleles
i.e., pure dominant : hybrid : pure recessive. F1 lacking dominant recessive relationship and both
generation expresses a phenotype which is express themselves in the organisms. The
intermediate between those of the parent, e.g., pink codominant alleles are able to express themselves
flowers are obtained when red and white flowered independently when present together. ABO blood
plants are crossed. groups are controlled by the gene I. Gene I has three
7. (a) Law of segregation (originated by Gregor Mendel) alleles IA, IB and i.
states that during the production of gametes the two 15. (b)
copies of each hereditary factor do not blend or mix 16. (a) Linkage is the tendency for alleles of different genes
S-150 Biology
to be passed together from one generation to the Sickle-cell disease, or sickle-cell anaemia, is a life-
next. It reduces the frequency of hybrids. long blood disorder characterized by red blood cells
17. (a) that assume an abnormal, rigid, sickle shape. Sickling
18. (c) Codominance is the phenomenon of two alleles decreases the cells flexibility and results in a risk of
lacking dominant recessive relationship and both various complications.
express themselves in the organisms. HbA and HbS 24. (c) Sickle cell anaemia is caused by a point mutation in
alleles of normal and sickle celled RBC show the beta globin chain of haemoglobin pigment of the
codominance. In codominance, the effect of both blood. The disease is controlled by a pair of allele
the alleles is equally conspicious and there is no HbA and HbS Homozygous dominant (HbA HbA )
mixing of the effect of the two alleles. Both the alleles is normal, heterozygous (HbA HbS) is carrier and
produce their effect independently, e.g., IA and IB, homozygous recessive (HbS HbS ) is diseased.
HbS and HbA. 25. (d) Since haemophilia is an X linked disease it can be
19. (b) A pair of X chromosomes is present in the female predicted that haemophilia would have occurred in
whereas X and Y chromosomes are present in the more of her male than her female descendants due to
male. There is an equal possibility of fertilization of criss cross inheritance. Haemophilia (also known as
the ovum (female gamete containing pair of X bleeder disorder) is a sex linked recessive disease
chromosomes) with the sperm (male gamete) carrying which occurs due to deficien cy of plasma
either X or Y chromosomes. If ovum fertilizes with a thromboplastin or antihaemophilia globulin during
sperm carrying X chromosome the zygote develops which the exposed blood does not clot. It transfers
into the female offspring and the fertilization of ovum from unaffected carrier female to some of the male
with sperm (carrying Y chromosome) results into the progeny. The possibility of female becoming a
male offspring. On this basis, it is evident that only haemophilic is extremely rare because mother of such
sperm plays an important role to determine the sex of female has to be atleast carrier and the father should
a child. be haemophilic.
20. (c) The X body of Henking was observed in half of the 26. (b) The three alleles IA, IB and i of gene I in ABO blood
sperms during spermatogenesis. During his group system can produce six different genotypes
experiments on insects, Henking found the traces of and four different phenotypes as shown below :
a nuclear structure all through the process of Genotype Phenotype
spermatogenesis which he named it as X body. He
IA IA – Blood group A
also observed that, after spermatogenesis, exact half
of the sperms received this X body, while the IA i
remaining half did not. Later, scientists found out IB IB – Blood group B
that this X body of Henking was actually a IB i
chromosome and hence, named it X-chromosome. IA IB – Blood group AB
21. (b) In a dihybrid cross, F2 phenotypic ratio (13 : 3) is a ii – Blood group O
case of epistatic genes. Epistate gene is a gene or 27. (b) Klinefelter's syndrome is a genetic disorder that
locus which suppresses the action of a gene at affects males. This syndrome occurs when a boy is
another locus. In dominant epistasis out of two pairs born with one or more extra X chromosomes due to
of genes the dominant allele, (i.e., gene A) of one union of nondisjunct XX egg and a normal sperm, or
gene masks the activity of other allelic pair (Bb). Since nondisjunct XY sperm with a normal egg. Having an
the dominant epistatis gene A exerts its epistatic extra X chromosome can cause a male to have some
influence by suppressing the expression of gene B
physical traits unusual for males.
or b, it is known as dominant epistasis. Example –
Dominant epistasis in dog. Similar phenomena have 28. (d) Turner’s syndrome is caused by the absence of
been seen in fruit colour in Cucurbita as summer X-chromosomes in females. Persons suffering from
squash and coat colour in chickens. turner ’s syndrome are monosomic for sex
22. (b) chromosomes i.e., possess only one X and no Y-
chromosome (XO). In other words they have
23. (b) Sickle cell anaemia is an autosomal linked recessive
chromosome number 2n – 1 = 45. They are phenotypic
trait that can be transmitted from parents to the
females but are sterile because they have under
offsprings when both the partners are carrier for gene
(or heterozygous). It is caused by a change in a single developed reproductive organs. They are dwarf
base pair of DNA. Sickle-cell anaemia is the name of about 4 feet 10 inches and are flat chested with wide
a specific form of sickle-cell disease in which there is spread nipples of mammary glands which never
homozygosity for the mutation that causes HbS. enlarge like those in normal woman. They develop
EBD_7209
Hints & Solutions S-151

as normal female in childhood but at adolescence sperms : gynosperms with X and androsperms
their ovaries remain underdeveloped. They lack without X. In grasshopper, the males lack Y-sex
female hormone estrogen. About one out of every chromosome and have only an X-chromosome
5,000 female births results in Turner’s syndrome. besides autosomes whereas females have a pair of
29. (d) X-chromosomes. Male produce sperm cells that
30. (a) Parents having genotype IAi and IBi have children contain either an X-chromosome or no sex
of the following blood group type – O, AB, and B. chromosome, which is designated as O.
31. (a) A character which is expressed in a hybrid is called 38. (d) Change in single base pair of DNA is also a type of
dominant character. It is an inherited character mutations called point mutations. It is a type of
expressed by a dominant gene in the F1 generation. mutation that causes the replacement of a single base
nucleotide with another nucleotide of the genetic
32. (d) Mutation is the process by which genetic variations
material, DNA or RNA. For example, a point mutation
are created through changes in the base sequence
is the cause of sickle cell disease.
within genes. It is possible to induce mutations
artificially through use of chemicals or radiations (like 39. (a) Tightly linked genes show more linkage than
gamma radiations). crossing over.
40. (b) 41. (b)
33. (c) In co-dominance, F1 generation resemble both the
parents. e.g., Blood group inheritance. 42. (d) In many birds, female has a pair of dissimilar
chromosomes ZW and male has two similar ZZ
34. (b) In ‘XO’ type of sex determination, males produce
chromosomes unlike mammals, the sex in birds is
two different types of gametes, e.g., grasshopper
where half of the sperms contain X-chromosomes decided by the Ova/eggs of the female and not by
the sperms of the male.
while the other half lacking ‘X’ chromosomes or
possessing ‘O’. 43. (d) Haemophilia is a sex-linked disease, which is marked
in heterozygous condition. It is a recessive character.
35. (c) According to Mendel’s law of Dominance, out of
Down’s syndrome is due to aneuploidy which arises
two contrasting allelomorphic factors only one
because of non-disjunction of two chromosomes of
expresses itself in an individual. The factor that
homologous pair. Phenylketonuria is an autosomal
expresses itself is called dominant while the other
recessive disorder in which the homozygous
which has not shown its effect in the heterozygous
recessive individuals lacks the enzyme phenylalanine
individual is termed as recessive. The option (c ) in
hydroxylase needed to change phenylalanine to
the given question cannot be explained on the basis
tyrosine. Sickle cell anaemi a is an autosome linked
of law of dominance. It can only be explained on the recessive trait that can be transmitted from parents
basis of Mendel’s Law of independent assortment, to the offsprings when both the partners are carrier
according to which in a dihybrid cross, the two for the gene (or heterozygous).
alleles of each character assort independently of the
44. (b) Statements (ii), (iii) and (v) are correct. Gregor Mendel,
alleles of other character and separate at the time of
conducted hybridization experiments on garden peas
gamete formation.
for seven years and proposed the laws of inheritance
36. (d) When two allelic pair are used for crossing, it is called in living organisms. His experiments had a large
dihybrid cross. Linkage is the inheritance of genes sampling size, which have greater credibility to the
of same chromosome together and capacity of these data. A recessive parental trait is expressed only in
genes to retain their parental combination in its homozygous condition.
subsequent generation. The strength of linkage 45. (a) Statements (i) and (iii) are correct. Mendelian
between two genes is inversely proportional to the disorders are mainly determined by alternation or
distance between the two. This means, two linked mutation in a single gene whereas chromosomal
genes show higher frequency of recombination if disorders are caused due to absence or excess or
the distance between them is higher and lower abnormal arrangement of one or more chromosomes.
frequency if the distance is smaller. Sickle cell anaemia is an autosome linked recessive
37. (b) XO type of sex chromosomes determine male sex in trait that can be transmitted from parents to the
grasshoppers. This type of sex-determination comes offspring when both the partners are carrier for the
under XX-XO type. Its common examples are gene. Haemophilia is sex-linked recessive disease
cockroaches, grasshoppers and bugs. The female which shows its transmission from unaffected carrier
has two homomorphic sex chromosomes XX and is female to some of the male progeny.
homogametic. It produces similar eggs, each with 46. (b) Statement (ii), (iii) and (iv) are correct.
one X-chromosome. The male has one chromosome (i) Incomplete or mosaic inheritance is not an example
only and is heterogametic. It produces 2 types of of pre - Mendelian concept of blending inheritance
S-152 Biology
because the parental types reappear in the F2 sequence. Mutations range in size; they can affect
generation. However, it is considered to be an anywhere from a single DNA building block (base
example of quantitative inheritance where only a pair) to a large segment of a chromosome that
single gene pair is involved and F2 phenotypic ratio includes multiple genes.
is 1: 2: 1, similar to genotypic ratio. 54. (d) Turner's syndrome is a genetic defect in which
47. (a) Genotype of the organism include all dominant and affected women have only one X chromosome (XO),
recessive characters. and causes developmental abnormalities and
infertility. Linkage is the tendency of alleles that are
48. (c) Haemophilia also known as bleeder disease is an located close together on a chromosome to be
example of recessive sex linked inheritance in human inherited together during meiosis. Morgan coined
beings. It is masked in heterozygous condition. The the term linkage to describe the physical association
person suffering from this disease lack factors VIII of genes on chromosomes. Y chromosomes are
and IX responsible for blood clotting. A small cut referred to as the testis determining factor (also known
may lead to bleeding till death. Men are affected by as sex determining factor). Down’s syndrome arises
this disease while women are the carriers. due to trisomy of chromosome 21. It is usually caused
by an error in cell division called "nondisjunction"
Mutation of a structural gene on chromosome
which results in an embryo with three copies of
number 15 causes Marfan syndrome. This disease chromosome 21 instead of the usual two.
results in formation of abnormal form of connective 55. (a) A-V, B-I, C-IV, D-II, E-III
tissues and characteristic extreme loosseness of
Monoploidy – n
joints.
Monosomy – 2n – 1
49. (c) Haemophilia bleeding disorder is a group of Nullisomy – 2n – 2
hereditary genetic disorders that impair the body’s Trisomy – 2n + 1
ability to control blood clotting or coagulation. In its Tetrasomy – 2n + 2
most common form, Hemophilia A, clotting factor 56. (d) Incomplete dominance (discovered by Correns) is
VIII is absent. In Haemophilia B, factor IX is deficient. the phenomenon where dominant allele does not
Factor VIII participates in blood coagulation; it is a completely express itself. The inheritance of flower
cofactor for factor IXa which, in the presence of Ca+2 colour in Antirrhinum sp. is a good example of
and phospholipids forms a complex that converts incomplete dominance. Haemophilia is a sex linked
factor X to the activated form Xa. Defects in this recessive disease. It is a type of Mendelian disorder
gene results in hemophilia A, a common recessive since its inheritance follows the principles of
X-linked coagulation disorder. Prothrombin Mendelian genetics and can be traced in a family by
producing platelets in such persons are not found in pedigree analysis. Transforming principle was given
very low concentration. by Griffith. Dihybrid cross is a cross when two allelic
pair are used for crossing. Several dihybrid crosses
50. (c) In human, the gamete contributed by the male
were carried out in Drosophila to study genes that
determines whehter the child produced will be male
were sex linked.
and female. Sex in humans is a polygenic trait
depending upon cumulative effect of some genes 57. (b) 58. (d)
present on Y-chromosome. Only sex in human is 59. (c) Sickle cell anaemia is an autosomal recessive trait.
monogenic trait. Haemophilia is a Sex linked recessive trait. Metabolic
51. (b) 52. (c) error linked to autosomal recessive: Phenylketonuria.
53. (c) ABO blood group is an example of multiple allelisms Phenylketonuria is an inherited disease due to faulty
because of presence of more than two alleles (IA, IB metabolism of phenylalanine. The affected individual
and i) of a gene. They are produced due to repeated lacks an enzyme that converts the amino acids
mutation of the same gene but in different directions. phenylalanine into tyrosine. It is characterized by
Law of segregation can be explained with the help of the presence of phenyl ketones in the urine and
monohybrid cross.. Law of independent assortment usually first noted by signs of mental retardation in
can be explained with the help of dihybrid cross. infancy. Down’s syndrome occurs due to presence
Law of independent assortment states that allele of an Additional 21st chromosomes.
pairs, which are independent of one another, separate 60. (a)
independently during the formation of gametes. Gene 61. (a) The pedigree chart shows the inheritance of a
mutation is a permanent alteration in the DNA condition like phenylketonuria as an autosomal
EBD_7209
Hints & Solutions S-153

recessive trait. Parents need to be heterozygous as 73. (c) Haemophilia is a sex linked disease in which the
two of their children are known to be sufferer of the patient continues to bleed even from a minor cut
disease. It cannot be recessive sex linked inheritance since he or she does not possess the natural
because then the male parent would also be a sufferer. phenomenon of blood cloting. Haemophilia is
62. (a) Pedigree analysis is a record of the occurrence of a genetically due to the presence of recessive gene h,
trait in several generations of a human family. In this carried by X-chromosome. A female becomes
male members are shown by squares and female by haemophiliac only when both its X-chromosomes
circles. Sibs are represented horizontally on a line in carry the gene (XhXh). However, such females
order of birth. It helps us in giving information about generally die before birth because the combination
genotype of an individual for trait under of these two recessive alleles is lethal. A female having
investigation. only one allele for haemophilia (XXh) appears normal
63. (b) The inheritance pattern of a particular trait shown in because the allele for normal blood clotting present
the given picture results in haemophilia. Haemophilia on the other X-chromosome is dominant. Such
is a group of inherited blood disorders in which the females are known as carriers. In case of males, a
blood does not clot properly. It is caused by a fault single gene for the defect is able to express itself as
in one of the genes that determine how the body the Y-chromosome is devoid of any corresponding
makes blood clotting factor VIII or IX. These genes allele (XhY) in which an organism has three times
are located on the X-chromosome. Haemophilia (3n) the haploid number (n) of chromosomes.
appears only in human male which can be transferred 74. (b) 75. (c)
to their grandson through their carrier daughter 76. (b) In co-dominance, both the genes are expressed for a
(Criss-cross inheritance). particular character in F1 hybrid progeny. There is
64. (c) In the given figure of sex chromosomes, gene a and no blending of characters, whereas both the
b present on X chromosomes represent disorders- characters are expressed equally. Examples : Co-
haemophilia and red green colour blindness. Both dominance is seen in animals for coat colour.
these disorders occur due to recessive sex linked When a black parent is crossed with white parent, a
genes present on sex chromosomes. Red green roan colour in F1 progeny is produced.
colour blindness is more common in males than 77. (b) In the given cross, disease is passed from carrier
females due to presence of only one X chromosomes. female to male progeny, this is known as criss-cross
The sufferers are not able to distinguish red and inheritance. The trait which shows criss-cross
green colour. inheritance is located on the sex chromosome. In
65. (b) The percentage of recombinants produced in cross I XXC, single recessive gene XC is present, that does
and cross II are respectively 1.3% and 37.2%. not cause the disease.
66. (a) On the basis of the given pedigree chart of a certain 78. (b) Blood group ‘O’ contains both antibodies but no
family, it can be concluded that the female parent antigens. The alleles IA and IB are codominant and
(shown by blank circle) is heterozygous where one are dominant over the allele IO(IA = IB > IO).
gene is dominant and other gene is recessive. If one parent is homozygous (IOIO) and other is
67. (a) The type of inheritance shown in the given diagram heterozygous (IAIO/IBIO), their progeny could be
is dominant X linked IA IO
68. (c) Parents are heterozygous normal otherwise II-2 could
IO I A IO IO IO
not be albino. Now cross between 2-heterozygous Patents
(Aa) produces AA (1) : Aa (2) : aa (1). Since II-1 is IO I A IO IO IO
normal, the chances of a heterozygous (normal) child
will be 2 out of three, i.e., 2/3. Genotype ratio = 1 : 1
Therefore, there is 50% chances that the child will
69. (d)
having ‘O’ blood group and 50% chances to have
70. (d) Mutation is the sudden inheritable discontinuous
blood group either ‘A’ or ‘B’.
variation which appears in an organism due to
79. (b) ABO system is one of the most important human
permanent changes in their genotypes. The term
“blood group systems”. The system is based on the
mutation was coined by Hugo de Vries (1901).
presence or absence of antigens A and B on the
71. (b) Recessive mutation is carried by heterozygous surface of red blood cells and antibodies against
carrier individuals. This is why they do not get these in blood serum. A person whose blood contains
eliminated from the gene pool. either or both of these antibodies cannot receive a
72. (c) transfussion of blood containing the corresponding
S-154 Biology
antigens as this would cause the red blood cells to should be crossed with aabb. As aabb will produce
clump. only one type of gamete thus difference in traits if
observed in their progenies will be due to the different
Type of gametes produced by hybrid dominant parent. This
Antigen Antibody % in society
blood group will indicate the genotype of the dominant parent.
A A Anti-B or 'b' 23.5 88. (a)
B B Anti-A or 'a' 24.5 89. (c) Purity and hybrid nature of the pea plant can be
tested by crossing the pea plant with a homozygous
AB A, B Absent 7.5
dwarf pea plant. This is known as test cross.
O None 'a' and 'b' 34.5
90. (a) The law of segregation states that when a pair of
80. (a) Down’s syndrome is also known as Mongolian contrasting factors or genes or allelomorphs are
idiocy or mongolism. In Langdon Down of England brought together in a heterozygote (hybrid), the two
(1866) studied the Mongolian idiocy and described members of the allelic pair remain together without being
the trisomic condition of their chromosomes. Down’s contaminated and when gametes are formed from the
syndrome, a very common congenital abnormality hybrid, the two separate out from each other and only
arises due to the failure of separation of 21st pair of one enters each gamete as seen in monohybrid and
autosomes during meiosis. Thus, an egg is produced dihybrid cross. That is why the law of segregation is
with 24 chromosomes instead of 23. A Down’s also described as law of purity of gametes.
syndrome has 3 autosomes in 21st pair instead of 2. 91. (a) Mendel was successful in formulating the laws of
Total number of chromosome in this case is 2n + 1 inheritance whereas his predecessors were not
(21st) = 47. because he studied one clear cut character at one
81. (a) In Drosophila, XXY represents a female but in time for his breeding experiment and took those traits
human it is an abnormal male, because Y which do not show linkage, interaction or incomplete
chromosomes are essential for determining the sex dominance.
of the individual. Normally male individual possess 92. (a) According to Bridges, in Drosophila Y-chromosome
X and Y chromosomes and female have XX is heterochromatic so it is not active in sex
chromosomes. Therefore individual having XXY determination. In Drosophila sex determination takes
genotype is an abnormal male. place by sex index ratio.
82. (b) No. of X-chromosomes X
Sex index ratio = =
83. (c) More than two alternative forms (alleles) of a gene in No. of set of autosomes A
a population occupying the same locus on a Gene of femaleness (Sxl- gene) (Sxl = Sex lethal gene)
chromosome or its homologue are known as multiple is located on X-chromosome and gene of maleness
alleles. is located on autosome. Gene of male fertility is
84. (a) Down’s syndrome is the example of autosomal located on Y-chromosome and in Drosophila, Y-
aneuploidy. Here, an extra copy of chromosome 21 chromosome plays additional role in
occurs. During gametogenesis, 50% gametes of the spermatogenesis and development of male
affected parent will have extra chromosome. Thus, reproductive organ, so Y-chromosome is essential
the percentage of affected offsprings produced from for the production of fertile male.
affected mother and normal father should be 50%. 93. (d) The F2 generation offsprings in a plant showing
85. (b) A monohybrid cross is the one in which a single pair incomplete dominance exhibit similar phenotypic and
of contrasting characters is considered at a given genotypic ratios of 1 : 2 : 1 i.e. pure dominant: hybrid:
time for the genetic results. pure recessive. In incomplete dominance, neither of the
86. (b) The law of segregation of characters postulated by two alleles are completely dominant over the other and
Mendel can be related to a gamete receiving only the dominant alleles do not completely express itself.
one of the two homologous chromosomes during 94. (d) Since round seed shape is dominant over wrinkled
meiosis. seed shape and yellow cotyledon is dominant over
87. (a) A test cross is a cross involving the mating of F1 green cotyledon so RRYY individuals is round yellow
individual with the homozygous recessive parent. It and rryy is wrinkled green.
is done to find out homozygous and heterozygous Round yellow seeds X Wrinkled green seeds
individuals. The offsprings will be 100% dominant if RRYY rryy
the individual was homozygous dominant and the ¯
ratio will be 50% dominant and 50% recessive in case F1 generation RrYy
of hybrid or heterozygous individual. Here, AaBb, F2 generation is obtained by selfing F1.
EBD_7209
Hints & Solutions S-155

RY Ry rY ry 100. (d) Mendel's rules do not correctly predict patterns of


inheritance for tightly linked genes or the inheritance
RY RRYY RRYy RrYY RrYY
of alleles that show incomplete dominance or epistasis.
Ry RRYy RRyy RrYy Rryy
This shows that these hypothesis are correct and limited
rY RrYY RrYy rrYY rrYy to certain condition.
ry RrYy Rryy rrYy rryy 101. (d) Haemophilia is an X-linked trait, and can only be
Expected phenotypes in F2 generation are round inherited by the son from his mother’s X-chromosome.
yellow seed 9, wrinkled yellow seed 3, round green The father contributes the Y-chromosome to his son
seed 3, wrinkled green seed 1. (not his X-chromosome) and thus cannot pass any of
95. (d) Klinefelter’s syndrome occurs due to trisomy of sex his X-linked alleles to his son.
chromosomes which results in (XXY) sex
chromosomes. Total chromosomes in such persons Chapter 28 : Molecular Basis of Inheritance
are 2n + 1 = 47 in place of 46. Klinefelter (1942) found 1. (c) In DNA, the two chains are held together by
that testes in such male remain underdeveloped in hydrogen bonds between pairs of bases which help
adulthood. They develop secondary sex characters to stabilize the interaction.
of female like large breasts and loss of facial hair. 2. (a) X-ray crystallography is the study of molecular
Characters of male develop due to Y-chromosome structure by examining diffraction patterns made by
and those like female due to XX chromosomes. x-rays beamed through a crystalline form of the
About one male child out of every 5,000 born, molecules. It is widely used in biochemistry to
develops Klinefelter’s syndrome. Such children are examine the molecular structure of molecules such
born as a result of fertilization of abnormal eggs (XX) as proteins and DNA.
with normal sperm with Y-chromosome or by 3. (d) Chargaff's rule are applicable to double stranded
fertilization of normal eggs with (X) chromosomes DNA because according to the Chargaff's rule,
by abnormal sperms with (XY) chromosome. They percentage of adenine is equal to the percentage of
are sterile males, mentally retarded and are eunuchs. thymine and percentage of guanine is equal to the
percentage of cytosine.
96. (c) A gene is said to be dominant if expresses its effect
4. (d) There are about 10 base pairs in each turn of DNA
both in homozygous and heterozygous condition. A
double helix.
dominant gene would lead to the expression of its
5. (a) According to Watson and Crick model of DNA, it is
phenotype irrespective of the fact whether its allelic a double helical molecule with 10 base pairs and 3.4
gene is dominant or recessive. Å distance for every turn.
97. (d) Given that both parents are true-breeding, the cross 6. (d) Central dogma term was proposed by Crick (1958). It
must be either AA ´ AA, AA ´ aa, or aa ´ aa. If you proposes unidirectional or one way flow of
work out these crosses, you will see that all of the information from DNA to RNA and then to protein
possible answers are true for each possible situation. (polypeptide).
Replication
98. (b) Allele for wrinkled shape of seed in garden pea plant
Transcription
is considered to be recessive because the trait Translation
DNA RNA Protein
(character) associated with the allele is not expressed
Reverse
in heterozygotes. Transcription
99. (b) Sex determination in grasshoppers, humans, and 7. (c) Nucleosome is the structural unit of a eukaryotic
Drosophila is similar because males have one X- chromosome, and thought to be present only during
chromosome & females have two X-chromosomes. interphase of cell cycle. It consists of DNA wrapped
In these three species, females have two X- around histone octamer.
chromosomes and males have one X-chromosome. 8. (c) Nucleotide is an organic molecule consisting of a
The ratio of X-chromosomes to autosomes is nucleoside (nitrogenous base and pentose sugar)
important (and different in each organism) in connected to a phosphate group. It forms the basic
Drosophila and grasshoppers, but not in humans. In structural unit of nucleic acids (such as DNA or RNA)
which carry the genetic information.
all three species, males have one Y-chromosome, but
the Y-chromosome is required for male fertility, not for 9. (b) Histones are positively charged, basic proteins,
enriched in the amino acids arginine and lysine. Thus,
Drosophila to be male (in Drosophila, male flies can
being basic, histones bind tightly to DNA which is
be XO). an acid H4.
S-156 Biology
10. (d) In Meselson and Stahl's experiments, heavy DNA chain. The information is transferred in the form of
was distinguished from n ormal DNA by triplet of bases coding for one amino acid. It is triplet,
centrifugation in CsCl gradient. When DNA is mixed universal, non-ambiguous and degenerate in nature.
with caesium chloride it will settle down at a particular 21. (a) In RNA splicing, intron sequences are removed by
height in centrifugation and heavier one higher up. process (known as splicing) and ligates the ends of
11. (a) Griffith (1928) described the phenonmenon of exon sequences together.
bacterial transformation. He experimented with the 22. (a) Frame shift mutation are those mutation in which the
smooth and rough strains of Streptococcus reading of the frame of the base sequence shifts
pneumoniae. Smooth strains of bacteria were virulent laterally either in forward direction due to addition
or pathogenic & cause pneumonia. of one or more nucleotides or in backward direction
12. (c) The scientists involved in the discovery of DNA as due to deletion of one or more nucleotides.
chemical basis of heredity were Avery, Mac Leod 23. (d) AUG is the initiation codon of protein synthesis in
and McCarty. They expanded the work of Griffith on eukaryotes. AUG always codes for methionine in
the process of transformation. eukaryotes.
13. (d) The experiment conducted by Hershey and Chase 24. (b) In eukaryotes, RNA polymerase are of 3 types:
proved that DNA is the genetic material and that (i) RNA polymerase I : Transcribes rRNA (285S,
during infection of E. coli cells by bacteriophage 18S, 5.8S). It is found in nucleolus.
T2, only nucleic acids (DNA or RNA) enter the cell. (ii) RNA polymerase II : Transcribes mRNA
Nucleic acids from the head pass through the hollow (hnRNA-heterogenous RNA). It is found in
tail and enter the bacterial cell. The remainder of the nucleoplasm.
phage remains on the outside of the bacterium as (iii) RNA polymerase III : Transcribes tRNA, 5S
"ghost". rRNA and SnRNA (small nuclear RNAs). It is
14. (c) The purines and pyrimidines are always in equal found in nucleoplasm.
amounts as per chargaff’s rule. So, if cytosine is 20%, 25. (b) Lactose operon (model proposed by Jacob and
the thymine will be 30% and thymine is equal to Monad) produces three enzymes - b- galactosidase
adenine. Then the percentage of adenine will be 30%. (z), permease (y), transacetylase (a). b- Galactosidase
15. (d) RNA and DNA both are genetic material and carry (z) is responsible for the splitting of lactose into
genetic information from one generation to other. A glucose and galactose. Permease (y) is required in
virus is a small parasite that cannot reproduce by entry of the lactose/galactose. Transacetylase (a)
itself. Most viruses have either RNA or DNA as their transfers an acetyl CoA to
genetic material. Once a virus infects a susceptible b- Galactosides.
cell, it can direct the cell machinery to produce more 26. (a) The inducer for Lac operon of Escherichia coli is
viruses. lactose (actually allolactose or metabolite of
16. (b) Few bacteriophages were grown in bacteria Lactose). This lac operon normally remains inactive.
containing 35S which was incorporated into the
When lac operon contacts with lactose, the lactose
cysteine and methionine amino acids of proteins and
acts as an inducer and combines with the repressor,
thus these amino acids with 35S formed the proteins
of phage. and the repressor is detached from operator gene.
17. (a) Leading strand during DNA replication is formed Thus RNA polymerase enzyme gets its passage and
continuously in 5' - 3' direction by continuous reaches to the structural genes and starts the
polymerization at the 3' growing tip. transcription.
18. (c) DNA replication is the process in which a double- 27. (d) Hershey & Chase (1962) discovered that DNA is the
stranded DNA molecule is copied to produce two genetic material of bacteriophage. They experimented
identical DNA molecules. In DNA replication each with T2 phage which attacks the bacterium E. coli.
new strand is half the original parent strand (hence Some virus made to grow on culture containing
called semiconservative) and one strand is radioactive sulphur and some on radioactive
synthesized continuously and other discontinuous phosphorus. Findings indicated that protein did not
(hence called discontinuous). enter the bacteria from the viruses but DNA from the
19. (c) In capping, unusual nucleotide (methyl guanosine virus particle enters bacteria as genetic material.
triphosphate) is added to 5' end of hn-RNA and forms 28. (c) Lac operon is an inducible operon. Inducible operon
cap. CCA segment is also added to t-RNA as terminal system regulates genetic material which remains
addition for specific function. switched off normally but becomes operational in
20. (c) Genetic code is the depiction of codon by which the the presence of inducer.
information in RNA is decoded in a polypeptide
EBD_7209
Hints & Solutions S-157

29. (d) Satellite DNA is a portion of DNA consisting of 37. (a) Peptidase catalyses the breaking of peptide bond.
short, repeating sequences of nucleotide pairs near The UTRs are present at both 5¢-end (before start
the region of the centromere. Normally it does not codon) and 3'-end (after stop codon). At the end of
code for any protein but shows polymorphisms. It is translation release factor binds to the stop codon,
classified in many categories like micro- or terminating translation and releasing the complete
minisatellites based on the composition, length of polypeptide from the ribosome.
segments and number of repetitive units. 38. (d) DNA fingerprinting involves identifying differences
in repetitive DNA. Since the DNA from every tissue
30. (a) Polymerase chain reaction (PCR) is a process used
of an in dividual show the same degree of
for the amplification (copy - small segments) of DNA.
polymorphism, they become very useful identification
It is a technique for enzymatically replicating DNA tool in forensic application.
without using living organisms, such as E. coli or 39. (d) RNA used to act as a genetic material as well as a
yeast. It is commonly used in the medical and catalyst (in some important biochemical reactions).
biological research labs for a variety of tasks, like But, RNA being a catalyst is reactive and unstable.
detection of hereditary diseases, the identification 40. (d) Introns or intervening sequences do not appear in
of genetic fingerprints, diagnosis of infectious mature or processed RNA.
diseases, cloning of genes, paternity testing etc. 41. (a) Repetitive sequences are stretches of DNA
31. (d) Polymorphism in DNA sequence is a variation at sequences that are repeated many times sometimes
genetic level. It arises due to mutation and is the hundred to thousand times. They are thought to have
basis of genetic mapping of human genome as well no direct coding functions, but they shed light on
as of DNA fingerprinting. chromosome structure, dynamics and evolution.
32. (a) In human genome, there are about 200,000 satellite 42. (d) The process involved in the RNA formation on the
loci. These simple tandem repeats of short sequences DNA template is called transcription.
are called ‘Variable Number Tandem Repeats’ 43. (c) Jacob and Monod proposed the lac operon of E.
(VNTRs). These repeats are inherited from the coli. The lac operon contains a promoter, an operator,
parents, and are used as genetic markers in a personal and three structural genes called Z, Y, and A, coding for
the enzyme, b galactosidase, permease and
identity test.
transacetylase respectively. The lac regulator gene,
33. (c) Single nucleotide polymorphism(SNP) is the most
designated as i gene, codes for repressor. In the absence
common type of genetic variation among people. of the inducer, the repressor binds to the lac operator,
Each SNP represents a difference in a single DNA preventing RNA polymerase from binding to the
building block, called a nucleotide. For example, a promoter and thus transcribing the structural gene.
SNP may replace the nucleotide cytosine (C) with 44. (a) Statement (i) and (iv) are correct.
the nucleotide thymine (T) in a certain stretch of Regulation of lac operon by repressor is referred to
DNA. as negative regulation. In negative regulation, a
34. (b) Human Genome Project (HGP) is closely associated repressor molecule binds to the operator of an operon
with the rapid development of a new area in biology and terminates transcription. In positive regulation,
called bioinformatics which is used for storage and an activator interacts with the RNA polymerase in
analysis of enormous amount of data. the promoter region to initiate transcription.
35. (b) DNA polymerase is an enzyme which is involved in Human genome contains some 20,000 - 25,000 genes
the replication and repair of DNA. It synthesizes new billion bases.
DNA strands using a DNA template in the 5' - 3' 45. (d) mRNA provides the template for synthesis of
direction. proteins. A segment of DNA coding for polypeptide
36. (b) DNA fingerprinting is a test to identify and evaluate is called exon.
the genetic information called DNA 46. (d) RNA pol I transcribes rRNAs, whereas the RNA pol
(deoxyribonucleic acid) in a person’s cells. DNA III is responsible for transcription of tRNA, 5srRNA
fingerprinting is a form of identification based on and snRNAs. RNA pol II transcribes hnRNA.
sequencing specific non-coding portions of DNA 47. (a) 3 codons do not code for any amino acid. Such
that are known to have a high degree of variability codons are called non-sense codons or terminator
from person to person. These sections are known as codon. Eg UAG, UAA & UGA.
tandem repeats. The test is used to determine whether 48. (c) Two nucleotides are linked through 3' – 5'
a family relationship exists between two people, to phosphodiester linkage to form a dinucleotide. The
identify organisms causing a disease, and to solve chromatin that is more densely packed and stains
criminal cases. dark is called heterochromatin.
S-158 Biology
49. (a) Adenine pairs with thymine and cytosine pairs with allows for the termination of translation by
guanine due to the perfect match of hydrogen donor recognizing the termination codon or stop codon in
and acceptor sites. an mRNA sequence. Amino acyl tRNA synthetase is
50. (a) A single mRNA strand is capable of forming different an enzyme which plays an important role in
polypetide chains because it has different reading translation during protein synthesis. This enzyme is
frame (the way through which reading of mRNA by responsible for the specific amino acylation of tRNA.
tRNA) Transcription is the process of transferring the
51. (a) Phenomenon in which more than one codon encodes information stored in DNA into a new molecule of
a single amino-acid is called degeneracy of genetic mRNA through the synthesis of RNA over the DNA
code. template. Transcription is carried out with the help
52. (a) In eukaryotes the replication and transcription takes of an RNA polymerase enzyme and a number of
place in the nucleus. mRNA came out from the accessory proteins (called transcription factors).
nucleus through the nuclear pore. In cytoplasm RNA polymerase enzyme is responsible for copying
translation occurs. In prokaryote there is no nuclear a DNA sequence into an RNA sequence. DNA
membrane, so replication, transcription and replication is the process in which a double-stranded
translation all occur in the cytoplasm. DNA molecule is copied to produce two identical
53. (b) DNA molecules. Okazaki fragments are short, newly
54. (d) Helicase is an enzyme which unwinds the DNA synthesized DNA fragments that are formed on the
strand by breaking the H - Bonding present between lagging (or discontinuous) template strand during
the nucleotide pairs. Gyrase catalyzes the breaking DNA replication.
and rejoining of bonds linking adjacent nucleotides 58. (c) Griffith described the phenomenon of bacterial
in circular DNA to generate supercoiled DNA helices. transformation. Hershey and Chase discovered that
The synthesis of RNA primer is done by primase RNA is the genetic material of bacteriophage.
enzyme. DNA polymerase III is involved in the Prokaryotic DNA is also called nucleoid. Nucleoid
synthesis of DNA from its deoxyribonucleoside is an irregularly-shaped region within the cell of a
triphosphate precursors. prokaryote (unicellular organisms) that contains all
55. (a) Splicing is a process in which introns and intervening or most of the genetic material. Euchromatin is a
sequences of non - essential nature are removed by chromosome material and comprises the most active
nuclease. Okazaki fragments are newly synthesized portion of the genome within the cell nucleus. It does
DNA fragments that is associated with the lagging not stain strongly except during cell division and
or discontinuous strand. Jacob and Monad were the represents the major genes and is involved in
first to elucidate a transcriptionally regulated system. transcription.
They proposed lac (lactose) operon. The lac operon 59. (a) A-III, B-IV, C-I, D-V, E-II
is an operon which is required for the transport and UUU – Phenylalanine
metabolism of lactose in Escherichia coli bacteria GGG – Glycine
and some other enteric bacteria. It has three adjacent UCU – Serine
structural genes, lacZ, lacY, and lacA. Inducer is a CCC – Proline
molecule that regulates gene expression. It attaches AUG – Methionine
to repressor and changes the shape of operator 60. (b) A-III, B-IV, C-I, D-II
binding site so that repressor no more remains b-galactosidase – Hydrolysis of lactose
attached to the operator. In the lac operon, allolactose Permease – Increases permeability to
is the actual inducer while lactose is the apparent b-galactosidase
inducer. Ligase – Joining of DNA fragments
56. (a) Operator site gives passages to RNA polymerase Ribozyme – Peptide bond formation
moving from the promoter to structural gene. 61. (a) F. Meischer discovered nuclein as an acidic
Promoter site is the initiation point for transcription substance present in nucleus. Griffith experimented
and the site for binding of RNA polymerase. with the smooth(S) and rough (R) strains of S.
Structural gene determines the amino acid sequence pneumoniae. Smooth strains of bacteria were virulent
on the segment of DNA molecule. Regulator gene or pathogenic and cause pneumonia. Rough strains
controls the activity of operator gene by producing were non-pathogenic or avirulent. Hershey and
repressor molecules. Chase discovered that RNA is the genetic material
57. (b) GTP dependent release factor is involved in the of bacteriophage (virus which infects bacteria).
termination. Termination requires the activities of Watson and Crick proposed the three dimensional
three release factors R1, R2, R3. A release factor structure of DNA based on X ray diffraction
EBD_7209
Hints & Solutions S-159

photographs of DNA taken by Rosalind Franklin and 70. (c)


M H F Wilkins. 71. (a) In the given figure of lac operon model (proposed
62. (b) Sigma factor is associated with the initiation of by Jacob and Monad), the labels A, B, X, Y and Z are
transcription. Sigma factor conferrs the specificity respectively repressor, inducer, b- galactosidase (z),
of RNA synthesis at the promoter region. Capping permease (y), transacetylase (a). z, y and a are three
involves the addition of unusual nucleotide at the 5' structural genes which produces three enzymes for
end of hn RNA. Tailing involves the addition of the degradation of lactose to glucose and galactose.
adenylate residues at 3' end in a template Label X (b- galactosidase) is primarily responsible
independently. Coding strand (also called leading for the hydrolysis of disaccharide lactose into
strand) is a strand synthesized by an enzyme in galactose and glucose.
continuous piece in 5' - 3' direction. 72. (b) In the given figure, the step shown is termination of
63. (b) Alec Jeffreys developed techniques for DNA transcription in bacteria. The label A, B and C are
fingerprinting and DNA profiling. These techniques respectively RNA, RNA polymerase and rho factor.
are now used worldwide in forensic science. F. RNA polymerase is an enzyme that synthesizes the
Sanger worked on protein sequencing and DNA formation of RNA from a DNA template during
sequencing and got Noble prize for the same. Jacob transcription. Rho factor is a termination factor which
and Monad proposed the lac (lactose) operon. Avery, releases RNA from the DNA template.
McLeod and McCarty expanded the work of Griffith
73. (b) tRNA or transfer RNA is a single stranded molecule
on the process of transformation.
and takes the shape of a clover leaf. In the process
64. (a) Nucleosome is a structural unit of a eukaryotic
of transcription tRNA brings amino acid and reads
chromosome which consists of a length of DNA coiled
the genetic code and acts as an adapter molecule. In
around a core of histones and are thought to be
the given structure of tRNA, the labels A, B, C and D
present only during interphase of cell cycle. In the
are respectively AA binding site (amino acid binding
given figure of nucleosome structure, the parts
site), TyC loop, anticodon loop (codon recognition
marked as A, B and C are respectively DNA, H1
site) and DHU loop (amino acid recognition site).
histones and histone octamer.
74. (a) A histone octamer is a complex of eight positively
65. (a) Replication fork is a site on a DNA molecule at which
charged histone proteins (two of each H2A, H2B,
both unwinding of the helices and synthesis of
H3 and H4) that aid in the packaging of DNA.
daughter molecules occurs. In the given figure of
Negatively charged DNA wraps around these
replication fork of DNA, the A and B synthesis are
histone octamers to form the nucleosome. The DNA
respectively called continuous (the template with
is held there by ionic bonds. Linker histone H1 binds
polarity 3' - 5') and discontinuous (the template with
to each nucleosome where the DNA enters and exits
polarity 5' - 3') synthesis.
and this draws a string of nucleosomes closer
66. (d) together to form the 10 nm fibre. The nucleosomes in
DNA
A
mRNA
B
Protein chromatin are seen as beads-on string structure when
67. (b)
viewed under electron microscope.
The given figure shows the concept of central dogma 75. (b)
of molecular biology. In this question A is 76. (a) Ribosomal RNA or rRNA is the most abundant types
transcription, B - translation C - Francis Crick. It is of RNA (about 80%) in the cell. It is found to be a
unidirectional flow of information DNA to mRNA catalytic element for protein synthesis.
(transcription) and then decoding the information 77. (c) A nucleoside is made up of a sugar molecules and a
present in mRNA in the formation of polypeptide heterocyclic base while a nucleotide is made up of a
chain or protein (translation). sugar molecule, phosphate group and a heterocyclic
68. (d) The given figure represents the figure of replication base.
fork of DNA. The new strands of DNA are formed in 78. (b) Double stranded DNA is much more stable than
the 5' ® 3' direction from the 3' ® 5' template DNA single stranded RNA and this helps to protect our
by the addition of deoxyribonucleotides to the 3' end genetic code. Having a second copy of our genetic
of primer RNA. code means that there is a reference for repair in the
69. (a) A, B and C represent the pitch (a complete turn) of event of a mutation or damage.
helix, distance between a base pair in a helix and 79. (a) Messelson and Stahl (1958) cultured (Escherichia
distance between two strand of DNA molecule coli) bacteria in a culture medium containing 15N.
respectively. After these had been replicated for a few generations
in the medium both the strands of their DNA
S-160 Biology
contained 15 N as constituents of purines and acids. Three types of degenerate codons are UAG
pyrimidines. When these bacteria with 15N were (amber), UAA (ochre) and UGA (opal).
transferred in cultural medium containing 14N, it was 89. (d) The two strands of a double helix model of DNA are
found that DNA separated from fresh generation of held together by hydrogen bonds between
bacteria possesses one strand heavier than the other. nitrogenous bases which help to stabilize the
The heavier strand represents the parental strand interaction. Adenine - thymine pair has two hydrogen
and lighter one is the new one synthesized from the bonds while guanine - cytosine pair has three
culture indicating semi conservative mode of DNA hydrogen bonds.
replication. 90. (c) Formation of mRNA from DNA is called as
80. (b) transcription. The segment of DNA involved in
81. (a) The function of sigma factor is to confer the transcriptions is cistron, which have a promoter
specificity of RNA synthesis at the promoter site. region where initiation is started and terminator
But during elongation of polypeptide chain, sigma region where transcription ends. Enzyme involved
factor is functionless. in transcription is RNA polymerase-II.
82. (a) Degeneracy of codons is the redundancy of the 91. (c)
genetic code. A single amino acid may be specified 92. (b) Nucleotides have three components - a nitrogenous
by many codon i.e., called degeneracy. Degeneracy base, pentose sugar and a phosphate group. The
is due to the last base in codon (which is known as phosphate group of one nucleotide is linked by
wobble base). Thus, first two codon are more phosphodiester bonds with the pentose sugar of
important to determine the amino acid and third one the other nucleotide.
differ without affecting the coding i.e., known
93. (c) The mRNA formed after transcription of a gene is
wobble hypothesis, proposed by Crick which
shorter than the DNA because the intervening
establishes an economy of tRNA molecule. sequences called introns are removed through
83. (b) Operons are segments of genetic material (DNA) which splicing.
functions as regulated unit or units that can be switched
94. (b) When a DNA strand with the sequence
on or switched off. It is a sequence of closely placed
AACGTAACG is transcribed, the resultant sequence
genes regulating a metabolic pathway in prokaryotes.
of the mRNA molecule synthesized is UUGCAUUGC.
84. (b) Clover leaf secondary structure of tRNA has a loop This is based on the paring of nitrogenous bases -
for three unpaired bases (triplet of base) whose adenine pairs with thymine (in DNA) and uracil (in
sequence is complementary with a codon in mRNA. RNA) and guanine with cytosine.
85. (c) Replication is the formation of exact carbon copy or 95. (c) Translation is the process of decoding of the
replica. According to semi -conservative method of messages from mRNA to protein with the help of
DNA replication, the two strands of DNA molecule tRNA, ribosome and enzyme.
separate and the complementary strand is 96. (c) The process of protein synthesis is catalyzed by
synthesized from the medium. After the completion ribosomal RNA. Messenger RNA provides the
of replication, each DNA molecule would have one genetic blueprint for the protein. Transfer RNA is
parental and one newly synthesized strand. responsible for translating the triplet code into a
86. (c) In frame shift mutation the reading of the frame of specific amino acid. Messenger RNA molecules are
the base sequence shifts laterally either in forward modified prior to protein synthesis by small nuclear
direction due to addition of one or more nucleotides RNA.
or in backward direction due to deletion of one or 97. (c) tRNA (or transfer RNA) is a single stranded RNA
more nucleotides. Whereas in base pair substitution molecule which brings amino acid and reads the
a base pair is replaced by another base which results genetic code in the process of transcription. It helps
in change of nucleotide sequence. decode a messenger RNA (mRNA) sequence into a
87. (c) DNA (deoxyribose nucleic acid) consists of 3 protein. It functions at specific sites in the ribosome
different molecules-phosphate, 5-carbon during translation, which is a process that
deoxyribose sugar and nitrogenous base. The synthesizes a protein from a mRNA molecule.
nitrogen ous base may be a 98. (b) mRNA carries the coded information for synthesis
9-membered, double purine, i.e., adenine (A) or of one (monocistronic) or more polypeptides
guanine (G), or a 6-membered, single -ringed (polycistronic). Its codons are recognized by tRNAs.
pyrimidine, i.e., thymine (T) or cytosine (C). 99. (a) The first mRNA codon to specify an amino acid is
88. (a) Degenerate codons (also called as non - sense codons always AUG. A DNA strand with the sequence TAC
or terminator codons) do not code for any amino will corresponds to the first amino acid i.e., AUG. On
EBD_7209
Hints & Solutions S-161

DNA strand A always pairs with T while on RNA 2. (a) Oparin and Haldane of England proposed that the
strand A always pairs with U. first form of life come from pre-existing non- living
100. (a) Inducible system includes a repressor protein which molecules (eg, RNA, protein etc. and that formation
is bound to DNA in the absence of any other factor. of life was preceded by chemical evolution i.e.
101. (c) A low level of Lac Z expression is required for formation of diverse organic molecule from inorganic
conversion of lactose to the inducer, allolactose. molecules.
3. (b) Extinction is represented by a branch which ends in
102. (b) Human genome project was launched in the year
a family tree.
1990. It is an international scientific research project
4. (c) The primitive atmosphere was reducing due to lack
having the goal to determine the sequence of base of free molecular oxygen. The early atmosphere
pairs which make up human DNA, and to identify contained ammonia (NH3), water vapour (H2O),
and map all of the genes of the human genome. hydrogen (H2), methane (CH4).
103. (d) Many non-human model organisms such as bacteria, 5. (a) The analogous organs are different in origin or basic
Saccharomyces cerevisiae (yeast), Caenorhabditis plan, but have similar functions or adaptations.
elegans (a free living non-pathogenic nematode) Wings of butterfly and wings of bird and fins of
Drosophila (the fruit fly), plants (Oryza sativa and fishes and flippers of whale are some of the examples
Ararbidopsis thalliana), etc. have also been of analogous organs.
sequences. 6. (a) The homologous organs are similar in origin or
104. (d) Minisatellites are inherently unstable and fundamental structure or basic plan, but may or may
susceptible to mutation at a higher rate than other not be similar in function, i.e. can differ in functions.
sequences of DNA. Thus, due to difference in Examples are wings of bat and arms of monkey and
number, location and size of minisatellites on gills of fishes and lungs of rabbit. In these animals,
chromosomes, each individual has a unique DNA the same structure developed along different
fingerprint. directions due to adaptations to different needs,
therefore the homologous organs show divergent
105. (d) The technique of DNA fingerprinting was initially
evolution.
developed by Alec Jeffrey's. He used a satellite DNA
7. (d) The analogous organs show convergent evolution
as probe that shows high degree of polymorphisms.
due to similar adaptation. They do not support
DNA fingerprinting using variable number tandem organic evolution. Whale, seal and shark shows
repeats is based on the observation that VNTR loci convergent evolution due to similar habitats.
are highly polymorphic. 8. (c) Thorn of Bougainvillea and tendrils of Cucurbita
106. (d) On template strand which has 5' ® 3' orientation, are the examples of homologous organ. Homologous
DNA polymerase synthesizes short pairs on new organs are those organs which are similar in their
DNA (about 1000 nucleotide long) in 5' ® 3' direction morphology, anatomy and embryology but dissimilar
and then joins these piece together. These small in their functions
framments are called okazaki fragments and new 9. (c) Industrial melanism (In England) is an example of
DNA strand made in this discontinuous manner is natural selection. Natural selection is the process by
called lagging strand. Okazaki fragments are joined which the organisms that are best suited for their
by means of DNA ligase. environment survive and reproduce.
3 Okazaki fragment Before industrialization (1850s), it was observed that
5 3 there were more white winged moths (Biston
5
mRNA betularia) on trees than dark winged or melanised
5
Leading strand 3 moths (Biston carbonaria). Because there were more
white coloured lichens that covered the trees. Hence,
Chapter 29 : Evolution white winged moths were able to camouflage them-
1. (a) A British scientist, J.B.S. Haldane, suggested that selves and survived. But the dark coloured moths
life originated from simple inorganic molecules. Two were picked out by predators.
scientists, Stanley Miller and Harold Urey were the After industrialization (1920), there were more dark
first to put to test, the Oparin - Haldane theory, in the winged moths and less white winged moths. Be-
year 1953. They created a condition similar to that of cause, the tree trunks became dark due to industrial
primitive earth (i.e. high temperature, volcanic storms,
smoke and soot. Under this condition the white
reducing atmosphere) containing ammonia (NH3),
winged moth did not survive as the predators could
water vapour (H2O), hydrogen (H2), methane (CH4).
They made electric discharge in a closed flask identify them easily. However, dark winged moth
containing CH4, NH3, H2 and water vapour at 800o survived because of suitable dark background.
C. As a result, some amino acids are formed. 10. (d) Darwin finches are a good example of adaptive
radiation. Adaptive radiation is a process of evolution
S-162 Biology
of different species in a given geographical area where, p2 = frequency of AA (homozygous dominant)
starting from a point and radiating to other areas of individuals, 2pq = frequency of Aa (heterozygous)
geography. individuals and q2 = frequency of aa (homozygous
11. (c) Adaptive radiation refers to evolution of different recessive) individuals. The equation can be used to
species from a common ancestor. The mammals are calculate allele frequencies if the numbers of
adapted for different mode of life i.e. they show homozygous recessive individuals in the population
adaptive radiation. They can be aerial (bat), aquatic is known.
(whale and dolphins), burrowing or fossorial (rat), Here, p = 0.7 and q = 0.3 (given)
cursorial ( horse), scantorial (squarrel) or arboreal \ 2pq (frequency of heterozygote)
(monkey). The adaptive radiation, the term by = 2 × 0.7 × 0.3 = 0.42
Osborn, is also known as divergent evolution. 18. (a) Frequency of dominant allele (p) = 60% = 0.6
12. (a) The Finches of Galapagos islands provide an example The frequency of recessive allele, according to (p +
of biogeographical evolution. Biogeography is the q) = 1, will be 0.4
study of th e distribution of life forms over The value of pq = 0.6 × 0.4 = 0.24
geographical areas. It not only provides significant The frequency of heterozygous individuals,
inferential evidence for evolution and common therefore, will be
descent, but also provides what creationists like to 2pq = 2 × 0.24 = 0.48 or 48%.
deny is possible in evolution. 19. (a) Random genetic drift in a population probably
13. (b) The term survival of the fittest was used by Herbert results from highly genetically variable individuals.
Spencer. "Survival of the fittest" is a phrase that Random genetic drift refers to random fluctuations
originated in evolutionary theory as a method to in the numbers of gene variants in a population. It
describe the mechanism of natural selection. Herbert takes place when the occurrence of variant forms
Spencer first used the phrase - after reading Charles of a gene, called alleles, increases and decreases
Darwin's On the Origin of Species - in his Principles by chance over time. These variations in the
of Biology (1864), in which he drew parallels between presence of alleles are measured as changes in
his own economic theories and Darwin's biological allele frequencies.
ones, writing. 20. (d) There are three types of natural selection i.e.
14. (a) Marsuvpials in Australia and placental mammals in stabilizing, directional and disruptive.
North America show convergent evolution. These
Stabilizing selection - In this type of selection, the
two subclasses of mammal have adapted in similar
frequency of average sized individuals increases
ways to a particular food supply, locomotor skill or
further but the frequency of smaller and larger sized
climate. individuals is reduced and finally becomes negligible.
15. (d) When Charles Darwin returned from the voyage in
Directional selection - In such selection, the longest
1836, he got a chance to read the books ‘The
or the shortest individuals are selected by nature
principles of Geology’ by Charles Lyell and ‘Essays
and increases in frequency in the forth-coming
on principles of (human) population’ by T.H.
generations. In directional selection it is always the
Malthus. Malthus in his book described that human
‘extreme’ (end one) which is favoured.
population increases in geometric ratio while food
Disruptive selection - In this case, the individuals of
increases in arithmetic ratio. This book of Malthus
both the extremes (ends), the smallest and the largest
influenced the work of Charles Darwin most.
ones, are favoured, and thus increase in the
16. (b) Hugo de Vries proposed mutation theory of
frequency or percentage.
evolution. He conducted some experiments on
21. (c) Mammals evolved during the Triassic period, about
Oenothera lamarckiana (evening primrose) and
the same time that the first dinosaurs appeared. The
believed that evolution takes place through mutation
first placental mammals appeared at the beginning
and not by minor variation and hence called it
of the cretaceous period. The earliest mammals were
saltation (single step large mutation).
tiny, shrew-like mammals. The shrew is a small,
17. (b) For a gene with two alleles, A (dominant) and a
mouse-sized mammal with an elongated snout, a
(recessive), if the frequency of A is p and the
dense fur of uniform colour, small eyes, and five
frequency of a is q, then the the frequencies of the
clawed toes on each foot. Shrews are among the
three possible genotypes (AA, Aa, and aa) can be
world's smallest mammal.
expressed by the Hardy-Weinberg equation :
22. (b) Australopithecus evolved 2 mya and had a cranial
p 2 + 2 pq + q 2 = 1 capacity of 450 cm3. They had full erect posture with
EBD_7209
Hints & Solutions S-163

height 1.5 m. They were herbivorous, hunted with organisms in Primitive ocean were protocell or
stone weapons, lived at trees, canines and incisors eobiont or protobiont.
teeth were small. 33. (c) When more individuals of a population acquire mean
23. (b) Australopithecus is one of the longest-lived and character value, the type of natural selection is called
best-known early human species whose remains were stabilizing selection. It favours the average or normal
found between 3.85 and 2.95 million years ago in phenotypes and eliminates the extreme variants.
Eastern Africa (Ethiopia, Kenya, and Tanzania). Disruptive selection is just the opposite of stabilizing
Evidences shows that they hunted with stone selection, i.e., the extremes have more adaptable
weapons but essentially ate fruits. phenotypes than the average ones. Hardy-Weinberg
24. (c) All the given statements are correct. equilibrium is defined as the relative frequencies of
25. (b) Lichens can be used as industrial pollution various kinds of genes in a large and randomly mating
indicators. sexual population which tend to remain constant from
26. (d) Homo erectus is an extinct species of hominid that generation to generation in the absence of mutation,
lived throughout most of the Pleistocene, with the selection and gene flow.
earliest first fossil evidence dating to around 1.5 34. (d) Genetic recombination leads to variation results in
million years ago and the most recent to around 70,000 changed frequency of genes and alleles in future
years ago. They had a large brain capacity around generation.
900cc and probably ate meat. 35. (a) i. Wings of butterfly and birds look alike and are the
27. (c) Neanderthal was the first specimen to be recognized results of convergent (A) evolution.
as an early human fossil. They had a brain size of ii. Miller showed that CH4, H2, NH3 and water vapour
1400cc and lived in near east and central Asia (A) when exposed to electric discharge in a flask
between 100000 - 40000 years back. Neanderthals resulted in the formation of amino acids (B).
made and used a diverse set of cultured tools, iii. Vermiform appendix is a vestigial (A) organ and
controlled fire and lived in shelters, made and wore an anatomical or morphological (B) evidence of
clothing, were skilled hunters of large animals and evolution.
also ate plant foods, and occasionally made symbolic iv. According to Darwin, evolution took place due to
or ornamental objects. There is evidence that small variation (A) and survival (B) of the fittest.
Neanderthals deliberately buried their dead and 36. (c) The given statements are the evidences for Darwin's
occasionally even marked their graves with offerings, theory of common descent. A group of organisms
such as flowers. share common descent, if they have a common
28. (d) All the given statements are correct. ancestor. There is strong quantitative support for
29. (a) The potato (stem modification) and sweet potato the theory that all living organisms on Earth are
(root modification) are the examples of analogy. descended from a common ancestor. Charles Darwin
Analogy is the similarity of function and superficial proposed the theory of universal common descent
resemblance of structures that have different origins. through an evolutionary process in the Origin of
30. (c) The evolution of the peppered moth is an Species,
evolutionary instance of colour variation in the moth 37. (c) The theory of natural selection states that those
population as a consequence of Industrial individuals that are better adapted to their
Revolution. It is a concept of increase in number of environment will have greater reproductive success.
dark-coloured moths due to industrial pollution, and 38. (b) Organic compounds first evolved in earth required
reciprocal decrease of the population under clean for origin of life were protein and nucleic acid. All life
environment. Hence, the phenomenon is called forms were in water environment only.
industrial melanism. Therefore, after industrialization 39. (a) The banding pattern seen on stained chromosomes
(i.e in 1920), the white winged moths did not survive from humans and chimpanzee show striking
due to predation. similarities which indicates that they have
31. (c) Neanderthal man lived in east and central Asia and evolutionary relationships (cytogenetic evidence).
used hides to protect their bodies. Agriculture came
40. (a) Natural selection acts on the variations found in a
around 10000 years back and human settlement
population. The individuals with favourable
started.
variations are selected and adaptive forms of a given
32. (d) Life originated in the ocean (water) presumably about
trait tend to become less common or disappear.
3.7 million years ago in Precambrian era. Earth was
formed about 4.5 million years ago. First living The Darwin Wallace theory of Natural Selection can
S-164 Biology
be generalised as the change in species by the metres tall. He had erect posture. His skull was flatter
survival of an organismal type exhibiting a natural than that of modern man. He had protruding brow
variation that gives it an adaptive advantage in an ridges, small canines and large molar teeth. He made
environment. That leads to a new environmental more elaborate tools of stones and bones, hunted
equilibrium. The idea of the survival of the fittest big game and perhaps knew use of fire.
explain the above evolution by natural selection. 45. (d)
According to survival of fittest, some of the 46. (c) Human embryo have gills shows ontogeny repeats
variations exhibited by living things make it easier phylogeny. Ontogeny is the life history of an
for them to survive and reproduce thus more organism while phylogeny is the evolutionary history
adaptive forms increase. Those which are not fit (or of the race of that organism. Modern theory of origin
less adaptive) become eliminated. of life was propounded by Oparin and Haldane which
is based on chemical evolution. Chemical evolution,
41. (b) The earliest organisms that appeared on the earth
also called chemogeny, involves the synthesis of
were heterotrophic (taking number from outside)
simple organic molecules. Millers and Urey
because of reducing atmosphere and the first
experimentally supported Oparin and Haldane theory
autotrophs were chemoautotrophs.
with the help of stimulation experiment. Analogous
An anaerobic organism does not require oxygen for organ are those organs which are similar in shape
growth and may even die in its presence. and function but their origin, basic plan and
Chemotrophs are the first organisms that appeared development are dissimilar. Example - wings of
on earth & that obtain energy by the oxidation of butterfly, bird and bat. Such similarities are because
electron donating molecules in their environments. of convergent evolution for adaptation to a common
These molecules can be organic (organotrophs) or condition.
47. (a) 48. (b)
inorganic (lithotrophs). The chemotroph utilize solar
49. (a) Saltation theory was given by de Vries who was a
energy and can be either autotrophic or heterotrophic.
Dutch botanist working with primroses. He believed
42. (b) Darwing finches found on Galapagos islands differ that a new species had suddenly sprung into
primarily in body size, feather colour, bill shape as existence as a mutation. He theorized that new species
adaptation to type of food available. It is a type of "saltated" (leaped), that is continually sprung into
divergent evolution. existence. Oparin and Haldane give origin of life.
43. (d) The lightest atoms of nitrogen, carbon etc. formed According to their theory, life evolved in the oceans
the primitive atmosphere, Hydrogen atoms were most during a period when the atmosphere was reducing
numerous and most reactive in primitive atmosphere. - containing H2, H2O, NH3, CH4, and CO2, but no
Hydrogen atoms combined with all oxygen stoms to free O2. Reproductive fitness was given by Darwin.
form water leaving no free oxygen. Thus primitive Louis Pasteur was a French chemist and
atmosphere was reducing atmosphere (without free microbiologist renowned for his discoveries of the
oxygen) unlike the present oxidizing atmosphere principles of vaccination, microbial fermentation and
(with free oxygen). pasteurization. According to him, life comes from
pre-existing life.
Fromation of ozone layer is the consequence of
50. (d) Alfred Russel Wallace is best known for
modern oxidizing atmosphere leaving plenty of free
independently conceiving the theory of evolution
oxygen. As more oxygen accumulated in the
through natural selection. His paper on the subject
atmosphere (due to photosynthesis) ozone began
was jointly published with some of Charles Darwin's
to appear in the top layers.
writings in 1858. Malthus wrote an Essay on the
44. (b) The fossil of Java Ape-man was discovered from Principle of Population, which describes how
pleistocene rocks in central Java. The fossil of Peking unchecked population growth is exponential while
man was discovered from the lime stone caves of the growth of the food supply was expected to be
Choukoution near Peking while that of Heidelberg arithmetical. The Hardy-Weinberg principle states
man was discovered in mid pleistocene. All these that allele and genotype frequencies in a population
three fossils come under th e categor y of will remain constant from generation to generation
Homoerectus. Homo erectus appeared about 1.7 in the absence of other evolutionary influences. The
million years ago in middle pleistocene. H. erectus equation for gen otype fr equen cies is:
evolved from Homo habilis. He was about 1.5-1.8 p2+ 2pq + q2 = 1. Industrial melanism is an effect of
EBD_7209
Hints & Solutions S-165

urban pollution prominent in Biston betularia. It is 57. (b) The graph A, B and C shows stabilizing, directional
the phenomenon of an organism evolving dark and disruptive traits of natural selection. In stabilizing
pigmentation when exposed to an environment selection, the median phenotype is selected during
polluted by dark soot deposit and sulphuric build- natural selection and which does not tilt the bell curve
up from industrial pollution. In this type of industrial in any way. Instead, it makes the peak of the bell
melanism, the darker pigmented individuals develop curve even higher than what would be considered
a higher fitness and are favoured by natural selection. normal. Directional selection of natural selection
51. (a) The theory of the continuity of the germplasm derives its name from the shape of the approximate
published by August Weismann (1834-1914) in 1886. bell curve that is produced when all individuals' traits
It proposes that the contents of the reproductive are plotted. Instead of the bell curve falling directly
cells (sperms and ova) are passed on unchanged in the middle of the axes on which they are plotted, it
from one generation to the next, unaffected by any tilts either to the left or the right by varying degrees.
changes undergone by the rest of the body. It thus Hence, it has moved in one direction or the other. In
rules out any possibility of the inheritance of disruptive selection instead of the bell curve having
acquired characteristics, and has become one peak in the middle, it has two peaks with a valley
fundamental to Neo-Darwinian theory. Inheritance in the middle of them.
58. (a) In the given diagrammatic representation of the
of acquired characters was proposed by Lamarck.
evolutionary history of vertebrates th rough
Natural selection and theory of pangenesis was geological periods, the geological periods marked
proposed by Darwin. as A, B, C and D are respectively carboniferous,
52. (d) Homo sapiens - Arose in Africa and moved across triassic, cretaceous and quaternary. A geologic
continents and developed into distinct races. period is a sub-division of geologic time enabling
53. (d) Adaptive radiations - Darwin finches and marsupials. cross-referencing of rocks and geologic events from
Vertebrate's hearts and brains are the examples of place to place.
homologous organ. 59. (d) The given figure of Bougainvillea and Cucurbita
54. (a) In 1953, scientist Stanley Miller performed an sows an example of homologous organ or divergent
evolution. Homologous organ are those organs which
experiment to explain what occurred on primitive Earth
are dissimilar in shape, size and function but their
billions of years ago. He sent an electrical charge origin, basic plan and development are similar. Other
through a flask of a chemical solution of methane, examples in animals are fore limbs of frog, reptile,
ammonia, hydrogen and water. This created organic birds and mammals. Such differences are due to
compounds including amino acids. In the given divergent evolution or adaptation for varied
diagram of Millers experiment, the correct conditions.
combination of the labels marked as A, B, C and D 60. (d) The diagrams of two skulls of two different mammals
are respectively electrodes, NH3 + H2 + H2O + CH4, show that skull A is of an ape and skull B is of human.
liquid water in trap, vacuum pump. 61. (d)
62. (b) The given figures show the forelimbs of three
55. (d) The given diagram explains the process of natural
mammals which indicate the homology among
selection and adaptive radiation in the wild bird species
themselves. Homologous organs are those organs
diversity. Adaptive radiation s a process in which which are dissimilar in shape, size and function but
organisms diversify rapidly into a multitude of new their origin, basic plan and development are similar.
forms, particularly when a change in the environment Such differences are due to divergent evolution or
makes new resources available, creates new challenges adaptation for varied conditions.
and opens environmental niches. Darwin finches 63. (b) Birds having variety of beaks were found in the
represent one of the best examples of this phenomenon. Galapagos Islands and show the process of adaptive
56. (d) The given diagram of marsupials of Australia radiation. Over time, an abundance of seeds for food
provides an example of divergent evolution. led to increased differences between the species.
Divergent evolution is the accumulation of Adaptive radiation is a process in which organisms
differences between groups which can lead to the diversify rapidly into a mass of new forms, especially
formation of new species. It is usually due to when a change in the environment makes new
resources available, creates new challenges and
diffusion of the same species to different and
opens environmental niches.
isolated environments which blocks the gene flow
64. (c) Homologous organs are those organs which are
among the distinct populations allowing
anatomical similar but dissimilar in shape, size and
differentiated fixation of characteristics through
function.
genetic drift and natural selection.
S-166 Biology
65. (b) The term evolution was given by Herbert Spencer of geographical areas is called adaptive radiations.
that is “descent with modifications”. Evolution helps Example : Darwin’s finches, Australian marsupials.
us to understand the history of life. 71. (b) The eye of octopus and eye of cat show different
Evolution is a process in which something changes patterns of structure, yet they perform similar
into a different and usually more complete or better functions. This is an example of analogous organs.
form over time and in response to environment. This Analogous organs have evolved due to convergent
results in descendents becoming different from evolution. Analogous organs have developed in the
ancestors. evolutionary process through adaptation of quite
66. (c) The correct geological time scale of earth is different organisms to similar mode of life.
72. (d) Archaeopteryx (meaning "ancient wing") is a very
Precambrian (Era/Age of early life) early prehistoric bird, dating from about 150 million
¯ years ago during the Jurassic period, when many
Palaeozoic (Age of fishes) dinosaurs lived. It is one of the oldest-known birds.
¯
Therefore fossils remains of Archaeopteryx indicate
Mesozoic (Age of reptiles dinosaurs and cycads) that reptiles gave rise to birds during Jurassic.
¯
73. (b) Archaeopteryx as a transition between dinosaurs
Coenozoic (Age of mammals)
and modern birds. It is known as a missing
connecting link because it is a fossil and shows the
A geological time scale is a diagram that details the
characters of birds and reptiles.
history of earths geology, noting major events like
74. (a) Convergent evolution is the process whereby
the formation of the earth, the first life forms and
organisms which are not closely related
mass extinctions. The first geological time scale was
independently evolve similar traits as a result of
proposed in 1913 by the British geologist Arthur
capability to adapt to similar environments or
Holmes (1890-1965).
ecological niches. It is the independent evolution of
The history of the earth has been subdivided into
similar features in species of different lineages.
eras, eras into periods and periods into major
Convergent evolution creates analogous structures
divisions.
that have similar form or function, but that were not
67. (b) Chemoheterotrophs were the first organisms. They
present in the last common ancestor of those groups.
were prokaryotic like bacteria, anaerobes, as 75. (a) The concept of chemical evolution is based on
molecular oxygen was absent. They obtained energy possible origin of life by combination of chemicals
by fermentation of some of the organic molecules under suitable environmental conditions. Chemical
present in the broth. Thus, they absorbed organic evolution describes chemical changes on the
molecules from outside for body building and energy. primitive Earth that gave rise to the first forms of life.
• Chemoautotrophs are those organisms that are The first living things on Earth were prokaryotes
capable to synthesize organic molecules from with a type of cell similar to present day bacteria.
inorganic molecules. e.g., Nitrifying bacteria, Prokaryote fossils have been found in 3.4-million-
sulphur reducing bacteria etc. year old rock in the southern part of Africa, and in
• Autotrophs are photosynthesizing plants / even older rocks in Australia, including some that
organisms. appear to be photosynthetic.
68. (d) Nature selects those set of characters that are best 76. (b) Natural selection is the Darwin's most famous theory
adapted to the environment. This has resulted in the of evolution. Natural selection is the gradual process
great diversity seen in the population of finches. by which transmissible biological traits become either
69. (b) The first scientific explanation of origin of life was more or less common in a population as a function of
put forward by A.I. Oparin in 1923. J.B.S. Haldane the effect of inherited traits on the differential
(1928), also made similar observation regarding the reproductive success of organisms interacting with
origin of life. According to them primitive atmosphere their environment. It is a key mechanism of evolution.
was reducing atmosphere because hydrogen atoms The shape of heal of Darwin finches, industrial
(most numerous and most reactive) combined with melanism or the changes in horse teeth are all
all available oxygen atoms to form water and leaving examples of natural selection.
no free oxygen. Therefore oxygen was not present 77. (b) Stabilizing selection or balancing selection acts in
in the primitive atmosphere of the earth. the absence of large scale environmental change or
70. (a) Process of evolution of different species in a given directional change for long period. It favours an
area starting from a point and radiating to other area intermediate form and eliminates the extreme variants.
EBD_7209
Hints & Solutions S-167

78. (d) Living fossils is a living species of organism that individuals with the trait under study occurs
appears to be similar to a species otherwise known according to the product rule of probability.
only from fossils without having any close living
relatives. 86. (d) Speciation through reproductive isolation is one of
79. (b) Founder's effect is the loss of genetic variation that the important consequences of geographical isolation.
occurs when a new population is established by a Geographical isolation is a group of plants, animals or
very small number of individuals from a larger other living individuals being separated from mixing
population. As a result of the loss of genetic genes within their same species. Geographical
variation, the new population may be distinctively isolation may ultimately lead to the populations
different, both genotypically and phenotypically, becoming separate species by adaptive radiation.
from the parent population from which it is derived. 87. (a) Darwin fitness is the ability to survive and reproduce.
In extreme cases, the founder effect is thought to It is a type of natural selection that considers the
lead to the speciation and subsequent evolution of role relatives play when evaluating the genetic fitness
new species. of a given individual. It is based on the concept of
80. (c) According to Lamarckism (or Lamarckian inclusive fitness, which is made up of individual
inheritance) an organism can pass on the survival and reproduction (direct fitness) and any
characteristics that it acquired during its lifetime. impact that an individual has on the survival and
According to Lamarck, a giraffe has a long neck reproduction of relatives (indirect fitness).
because its ancestors stretched their necks to get 88. (b) In ancient period, hands were used to collect food
the food. and to save themselves. Gradually men learnt to cook
81. (c) Biogeographical evolution is a process in which gene food, make tools for their own purpose, this change in
pool of a population gradually changes in response habit brings perfection in their hand. Similarly, there is
to environmental pressures, natural selection and an increase in the ability to comunicate with other and
genetic mutations develop community behaviour. Loss of tail took a great
82. (c) Hugo de Vries, a Dutch botanist, one of the role in course of evolution. But as in ancient period
independent rediscoverers of Mendelism, put forward men still eat hard nuts and hard roots (though they
his views regarding the formation of new species in often used to take soft food also). This change in diet
1901. According to him, new species are not formed is the most irrelevant change in the evolution of man.
by continuous variations but by sudden appearance 89. (a) Occurrence of natural selection and small size of
of variations, which he named as mutations. Hugo de population do not met for a population to reach
Vries stated that mutations are heritable and persist in Hardy-Weinberg Equilibrium. For Hardy-Weinberg
successive gen erations. He conducted his equilibrium to be reached natural selection cannot
experiments on Oenothera lamarckiana (evening be occurring. If populations are undergoing natural
primrose). selection at the locus under consideration allele
83. (c) Binocular vision, smaller jaw and upright posture frequencies will be changing in a specific direction
are the main adaptations that led to evolution of man and changing continuously, Hardy-Weinberg
from its ancestors. But during human evolution major Equilibrium predicts that allele frequencies will stay
and most significant changes occurred in the cranial constant. Hardy-Weinberg Equilibrium assumes that
capacity of man. In living modern man it is about population size is very large.
1450 cc compared to 500 cc in Australopithecus. 90. (d) p2 + 2pq +q2 = 1is the equation associated with
Increased cranial capacity accommodates larger brain Hardy-Weinberg Equilibrium. In this equation p2
and forms the basis of social, cultural and represents the frequency of individuals who are
educational evolution of modern man. homozygous for one allele, 2pq is the frequency of
84. (c) According to Darwin, two different areas within a heterozygous individuals, and q2 is the frequency of
continent have different species because they have individuals who are homozygous for the other allele.
different environments.
85. (d) The tendency of population to remain in genetic p q
2
equilibrium may be disturbed by lack of random p p pq
mating. Random mating is a mating between 2
q q = pq q
individuals where the choice of partner is not
influenced by the genotypes. The mating of Adding the cells of the table together will provides
individuals in a population such that the union of with, p2 + pq + pq + q2 = p2 + 2pq + q2.
S-168 Biology

Chapter 30 : Human Health and Disease also through blood or other bodily fluid. The terms
infectious and contagious are also used to describe
1. (b) Widal test is used for the confirmation of typhoid communicable disease. Communicable diseases
disease. includes malaria, typhoid etc.
2. (b) Malaria is a protozoan disease, caused by 11. (a) Black water fever disease is caused by Plasmodium
Plasmodium species. In case of malaria, there is falciparum which is also known as deadly tertian
release of a toxic substance (called haemozoin) from malaria.
the ruptured RBCs which is responsible for the chill 12. (a) Mumps is a viral disease caused by RNA containing
and high fever. Paramyxo mumps virus. It involves painful swelling
3. (d) Malaria is a serious infectious disease caused by of parotid glands due to which patient has difficulty
four different species of a parasite belonging to genus in swallowing and opening of mouth.
Plasmodium: Plasmodium falciparum (the most 13. (b) Diphtheria is not transmitted through contaminated
deadly), Plasmodium vivax, Plasmodium malariae, and water. Diphtheria, caused by Corynebacterium
Plasmodium ovale. It is characterized by recurrent diphtheriae is spread through droplet infection and
symptoms of chills, fever, and an enlarged spleen. direct contact.
Malignant malaria is caused by P. falciparum. 14. (a) AIDS is a disease in which there is a severe loss of
4. (c) Pathogens for fungal disease called ring worm are the body's cellular immunity, greatly lowering the
Microsporum, Trichophyton and Epidermophyton. resistance to infection and malignancy. There is
They are seen in groin, between toes, etc. always a time lag between the infection and
5. (a) Wuchereria (W. bancrofti and W. malayi), also called appearance of AIDS symptoms. This period may vary
filarial worms chronic inflammation of the organs in from a few months to many years (usually 5 – 10
which they live for many years, usually the lymphatic years).
vessels of the lower limbs. The disease caused by 15. (c) Infection is the invasion and multiplication of
them is known as elephantiasis or filariasis. The microorganisms such as bacteria, viruses, and
genital organs are mainly affected, resulting in gross parasites that are not normally present within the
deformities. The pathogens are transmitted to a body. An infection may remain localized, or it may
healthy person through the bite by the female spread through the blood or lymphatic vessels to
mosquito vectors. become systemic. Microorganisms that live naturally
6. (c) Food poisoning is an illness caused by bacteria or in the body are not considered infections. For
other toxins in food. Symptoms which generally begin example, bacteria that normally live within the mouth
within 2 to 6 hours include abdominal cramping, and intestine are not infections.
diarrhoea, fever, headache, nausea, vomiting, and 16. (d) Ascaris, an intestinal parasite causes ascariasis.
weakness. Food poisoning is caused by Clostridium Symptoms of these disease include internal bleeding,
botulinum. muscular pain, fever, anaemia and blockage of the
7. (d) Amoebiasis (amoebic dysentery) is caused by intestinal passage. The eggs of the parasite are
intestinal endoparasitic protozoan, Entamoeba excreted along with the faeces of infected persons
histolytica. Infection is transmitted by contamination which contaminate soil, water, plants, etc. A healthy
8. (a) Kala azar is a chronic and potentially fatal parasitic person acquires this infection through contaminated
disease of the viscera (the internal organs, particularly water vegetables, fruits, etc.
the liver, spleen, bone marrow and lymph nodes) due 17. (d) Leukemia is a group of cancers that usually begins
to infection caused by Leishmania donovani. in the bone marrow and results in high numbers of
Leishmania donovani is transmitted by sand-fly abnormal white blood cells. These white blood cells
bites. Kala-azar is associated with fever, loss of are not fully developed and are called blasts or
appetite (anorexia), fatigue, enlargement of the liver, leukemia cells. Symptoms may include bleeding and
spleen and nodes and suppression of the bone bruising problems, tiredness, fever and an increased
marrow. risk of infections. These symptoms occur due to a
9. (a) Tetanus, typhoid and tuberculosis are bacterial lack of normal blood cells.
diseases. Tetanus is caused by Clostridium tetani, 18. (d) The examples of physiological barriers are
typhoid is caused by Salmonella typhi and hydrochloric acid in stomach, saliva in mouth,
tuberculosis is caused by Mycobacterium lysozyme in tears, saliva, etc.
tuberculosis 19. (c) The examples of physiological barriers are
10. (a) Communicable diseases spread from one person to hydrochloric acid in stomach, saliva in mouth,
another or from an animal to a person. The spread lysozyme in tears, saliva, etc.
often happens via airborne viruses or bacteria, but
EBD_7209
Hints & Solutions S-169

20. (b) Immunoglobulins (Ig), also known as antibodies, is 33. (a) The principle of vaccination is based upon the
a large Y-shape protein. It is produced by plasma memory of immune system. When a pathogen for
cells and used by the immune system to identify and the first time infects a person; it produces primary
neutralize foreign objects such as bacteria and immune response which is of low intensity. When
viruses. The antibody recognizes a unique part of the same pathogen attacks again, highly intensified
the foreign target, called an antigen. secondary (anamnestic) response is generated,
21. (b) Antigen is a toxin or other foreign substance which thereby preventing the occurrence of disease.
induces an immune response in the body, especially 34. (a) Pyrogen is released in body during disease.
in the production of antibodies. Antigens include 35. (c) Transfer of antibodies present in the mother’s milk
toxins, bacteria, foreign blood cells, and the cells of to the infant is an example of passive immunity.
transplanted organs. Antigens are present on the 36. (c) Preformed antibodies are derived from the blood
cell surface. serum of previously infected people or animals. They
22. (a) Interferon is a protein releases usually in response are often administered in an antiserum to another
to the entry of a virus. It has the property of inhibiting person in order to provide immediate, passive
virus replication. Interferon prevents the virus from immunization against fast-acting toxins or microbes,
reproducing within the infected cells and can also such as those in snakebites or tetanus infections.
induce resistance to the virus in other cells. 37. (b) Vaccine for human hepatitis B virus was prepared
23. (a) Cytotoxic cells a subset of T lymphocytes that bind using antigen produced by recombinant technology
to other cells through MHCs and are involved in in yeast (Saccharomyces cerevisiae). Yeast-derived
their destruction. Cytotoxic T lymphocytes kills vaccine was safe and was equally immunogenic and
cancer cells, cells that are infected (particularly with protective against hepatitis B as plasma-derived
viruses), or cells that are damaged in other ways. vaccine.
24. (a) The letter T in T lymphocytes refers to thymus. 38. (a) Immune response by the B-cells by production of
25. (d) Allergy is the exaggerated response of the immune antibody is called antibody mediated immune
system to certain antigens present in the response or humoral immune response. Here,
environment. The substances to which such as antibodies are found in blood plasma. So, it is called
immune is produced are called allergens. as humoral immune response.
26. (c) Passive immunity is a type of acquired immunity in 39. (c) The spleen is the site of interaction of lymphocytes
which readymade antibodies are transferred from one with the antigen. It is the organ that is responsible
individual to another. It is acquired through first for purifying the blood as well as storing blood cells.
exposure to the disease. It is the largest lymphatic organ in the body and
27. (c) Autoimmunity is the memory based acquired serves a valuable role in immune function because it
immunity, which is not able to distinguish foreign purifies the blood and helps the immune system with
molecules or cells (pathogen) from self-cells. recognize and attack foreign antibodies and disease.
28. (b) Passive immunity is the short-term immunity which 40. (a) Antigens trapped in the lymph nodes are responsible
results from the introduction of antibodies from for the activation of lymphocytes present there and
another person or animal, from the transfer of cause the immune response. Antigen is a toxin or
antibodies through the placenta or from colostrum. other foreign substance which induces an immune
29. (c) Immunosuppressive drugs are those drugs that response in the body, especially the production of
inhibit or prevent activity of the immune system. They antibodies.
are used in immunosuppressive therapy to: prevent 41. (b) Rheumatoid arthritis is an autoimmune disease.
the rejection of transplanted organs and tissues Autoimmune disease results when the immune
(e.g., bone marrow, heart, kidney and liver etc). system attacks and destroys self cells and molecules.
30. (c) Antigen binding site is a region on an antibody that Other examples are multiple sclerosis, insulin
binds to antigens. It is composed of one constant dependent diabetes etc.
and one variable domain of each of the heavy and 42. (d) Hepatitis B and AIDS are transmitted through
the light chain. Antigen binding site in an antibody infected blood. Hepatitis B, poputarly called serum
is found in between one heavy and one light chain. hepatitis or transfusion hepatitis is caused by
31. (a) Allergy is due to the release of chemicals like hepatitis B virus. It is also called Dane parlicle.
histamine and serotonin from the mast cells. Transmission of hepatites B includes blood (blood
32. (c) Resistance in body against diseases is given by transfusion), tattoos etc. AIDS is caused by HIV
antibodies or immunoglobulins. which belongs to retrovirus. It is transmitted through
body fluids by sexual contact, infected blood shared
needles etc.
S-170 Biology

43. (b) The human immunodeficiency virus is a lentivirus exhibited by malignant tumors. These cells divide
that causes th e acquir ed immunodeficiency uncontrollably, forming a mass of cells called tumor.
syndrome, a condition in humans in which 51. (d) Cancer is a disease caused by an uncontrolled
progressive failure of the immune system allows life- division of abnormal cells in the body. Techniques
threatening opportunistic infections and cancers to like radiography (use of X-rays), CT scanning and
thrive. HIV has a protein coat and a genetic material MRI are very useful to detect cancers of internal
which is single stranded RNA. organs. Computed tomography uses X–rays to
44. (d) HIV (human immunodeficiency virus) is a virus that generate a three dimensional image of the internal
attacks the immune system, the body's natural defense objects. MRI uses strong magnetic fields and non -
system. White blood cells are an important part of the ionizing radiations to accurately detect pathological
immune system. HIV infects vital cells in the human and physiological changes in the living tissue.
immune system such as helper T cells (specifically CD4+ 52. (c) Drugs like anti-histamine, adrenaline and steroids
T cells), macrophages, and dendritic cells. quickly reduce the symptoms of allergy.
45. (c) Cancer is not an immunity related disease. Cancer is 53. (c) Rifampin is antibacterial prescribed in the treatment
a disease caused by an uncontrolled division of of tuberculosis.
abnormal cells in the body. It may spread through 54. (c) Lysergic acid diethylamide (LSD), discovered in 1938,
the lymphatic system or blood stream to other parts is a hallucinogen. It is manufactured from lysergic
of the body. It is a progressive disease, and goes acid, which is found in ergot, a fungus (Claviceps
through several stages. Each stage may produce a purpurea) that grows on rye and other grains. It is
number of symptoms. Some symptoms are produced odourless, colorless, and has a slightly bitter taste.
early and may occur due to a tumor that is growing
55. (b) Belladonna is a drug prepared from the dried leaves
within an organ or a gland. As the tumor grows, it
and root of deadly nightshade poisonous plant,
may press on the nearby nerves, organs, and blood
Atropa belladonna, containing atropine and related
vessels. This causes pain and some pressure which
alkaloids. It is used in medicine to check secretions
may be the earliest warning signs of cancer.
and spasms, to relieve pain or dizziness, and as a
46. (c) Thymus gland is related with AIDS (the final stage cardiac and respiratory stimulant.
of HIV infection), because HIV decreases natural
56. (d) Bhang, ganja, charas (hashish), marijuana etc. are
immunity of the body by attacking T - lymphocytes.
hallucinogens.
T lymphocyte is a type of lymphocyte which is
57. (d) Narcotic drug are psychoactive compound with sleep-
produced or processed by the thymus gland and
inducing properties. They include mainly plant-based
actively participating in the immune response.
products such as opium and its derivatives morphine,
47. (a) ‘Helper T’ lymphocytes play a great role in regulating
codeine and heroin, but also synthetic narcotics such
the immune system. Damages to or destruction of
as methadone and pethidine, as well as cannabis,
‘helper-T’ lymphocytes lead to the development of a
coca and cocaine.
cellular immune deficiency which makes the patient
58. (b) Cyclosporin is used as immunosuppresant.
susceptible to wide variety of infections.
It (Cyclosporin A) is a biologically active metabolite
48. (c) Cancer is an abnormal, uncontrolled and unwanted
obtained from Tolypocladium infatum and certain
growth of cells. e.g. Breast cancer, leukemia. Cancer
other fungi. It has shown to prolong graft survival in
is not a contagious disease. It is the major cause of
kidney, liver, heart and lung transplants.
death in human beings.
59. (a) Opium is obtained from dried latex of unripe capsules
49. (d) Cancer cells are more easily damaged by radiation
of Poppy plant (Papaver somniferum).
than normal cells because they are undergoing rapid
60. (d) Belladonna is a drug prepared from the dried leaves
division. Radiation is energy that's carried by waves
and root of deadly nightshade poisonous plant,
or a stream of particles. Radiation works by damaging
Atropa belladonna, containing atropine and related
the genes (DNA) in cells which control how cells
alkaloids.
grow and divide. When radiation damages the genes
of cancer cells, they can't grow and divide any more. 61. (a) Natural cannabinoids are obtained from Cannabis
Over time, the cells die. This means radiation can be sativa (Hemp plant). Its flower tops, leaves and resin
used to kill cancer cells and shrink tumors. Radiation are used to produce bhang, ganja, charas (hashish),
therapy is always a balance between destroying the marijuana etc.
cancer cells and minimizing damage to the normal 62. (b) Hashish and charas are obtained from Cannabis
cells. satia. Cannabis sativa (bhang) is an annual
50. (d) Metastasis is the pathological process of spreading herbaceous plant in the Cannabis genus, a species
cancerous cells to the different part of the body. It is of the cannabaceae family.
EBD_7209
Hints & Solutions S-171

63. (c) Opium is a highly addictive narcotic drug acquired There are three main types of T lymphocytes: helper
in the dried latex form from the seed pod of opium T cell which affects the production of antibodies by
poppy (Papaver somniferum). Traditionally the B cells; suppressor T cell which suppresses B cell
unripened pod is slit open and the sap seeps out activity; and cytotoxic T cells which kills cancer cells,
and dries on the outer surface of the pod. The cells that are infected (particularly with viruses), or
resulting yellow-brown latex, which is scraped off of cells that are damaged in other ways.
the pod, is bitter in taste and contains varying 71. (b) Barbiturates are used to cope with mental illnesses
amounts of alkaloids such as morphine, codeine, the like depression and insomnia. Opium is the dried latex
baine and papaverine. obtained from the opium poppy (Papaver
64. (c) Ergot is a disease of rye and other cereal grasses, somniferum). It is a highly narcotic drug. Morphine
caused by a fungus of the genus Claviceps, is a very effective sedative and painkiller. It is very
especially C. purpurea, which replaces the affected useful in the patients who undergone surgery.
grain with a long, hard, blackish sclerotial body. 72. (a) Metastasis is the pathological process of spreading
Therefore its drug is derived from the Claviceps. cancerous cells to the different part of the body. It is
65. (c) Cirrhosis is a complication of liver diseases which is exhibited by malignant tumors.
characterized by abnormal structure and function of 73. (a) (iii) The B lymphocyte is responsible for producing
the liver. Cirrhosis of liver is caused by alcohol. There antibodies in response to pathogens into our
are many causes of cirrhosis including chemicals blood to fight with them.
(such as alcohol, fat, and certain medications), (iv) The acceptance or rejection of a kidney
viruses, toxic metals (such as iron and copper that transplant does not depend on the specific
accumulate in the liver as a result of genetic diseases), interferon, but on immune system. Interferon
and autoimmune liver disease in which the body's is a protein released in response to the entry of
immune system attacks the liver. a virus, which has the property of inhibiting
66. (b) virus replication. Interferon prevents the virus
67. (d) LSD is obtained from Ergot fungus (Claviceps from reproducing within the infected cells and
purpurea). It is one of the most dangerous can also induce resistance to the virus in other
hallucenogens of modern times. cells.
68. (b) Injecting microbes during immunization induces 74. (a) (i) The HIV can be transmitted by sexual contact
active immunity. Colostrum secreted from the mother with infected person, transfusion of
during the initial days of lactation has abundant contaminated blood and blood products,
antibodies (IgA) to protect th e foetus. This sharing infected needles as in case of
protection provides passive immunity. intravenous drug abusers and infected mother
to her child through placenta.
69. (b) (i) Heroine, commonly called as smack, is a white
colourless, bitter crystalline compound. This is (ii) Drugs addicts are highly susceptible to HIV
obtained by acetylation of morphine which is infection.
extracted from the latex of the poppy plant, (iii) AIDS patients can- not be fully cured after
Papaver somniferum. taking proper care and nutrition.
(ii) Cocaine is obtained from the coca plant, 75. (c) Morphine is a strong analgesic and sedative
Erthyroxylum coca. Cocaine is commonly extracted from the latex of poppy plant. They are
called as coke or crack which is usually snored. useful during surgery.
It has a potent stimulating action of euphoria 76. (b) Life cycle of Plasmodium starts with inoculation of
and increased energy. Its increased dosages sporozoites (infective stage) through the bite of
cause hallucinations. infected female Anopheles mosquitoes. Sporozoites
(iii) Marijuana interferes with the transmission of are found in saliva of infected female Anopheles
endogenous cannabinoids (anandamide). mosquitoes.
(iv) Morphine is a very effective sedative and 77. (d) When HIV replicates rapidly in helper T lymphocytes
painkiller. It is very useful in the patients who and damages large number of these, then one can
undergone surgery. shows the symptoms of AIDS.
70. (d) T lymphocyte is produced or processed by the 78. (c) Yeast has been used to produce hepatitis B vaccine
thymus gland and actively participating in the via recombinant DNA technology. Injection of snake
immune response including the identification of antivenom (containing preformed antibodies)
specific foreign antigens in the body and the against snake bite is an example of passive
activation and deactivation of other immune cells. immunization. Injection of dead/inactivated
S-172 Biology
pathogens causes active immunity. infection. Besides defending against viruses, they
79. (b) Pneumonia and common cold both are communicable also help to regulate the immune response.
diseases. Pneumonia is an infection that inflames 87. (b) Artificially acquired passive immunity results when
the alveolar air sacs in one or both lungs. The air antibodies or lymphocytes that have been produced
sacs may fill with fluid or pus (purulent material), outside the host are introduced into a host. This
causing cough with phlegm or pus, fever, chills, and type of immunity is immediate short lived, lasting
difficulty breathing. A variety of organisms, including only a few weeks to a few months. An example is
bacteria, viruses and fungi, can cause pneumonia. bone marrow transplant given to a patient with
The most common cause of bacterial pneumonia is genetic immunodeficiency.
Streptococcus pneumoniae. 88. (d): Treatment of AIDS with anti-retroviral drugs is only
Common cold, caused by rhino virus, is a viral partially effective. They can only prolong the life of
infection of upper respiratory tract (nose and throat). the patient but cannot prevent death. AIDS virus is
A common cold is usually harmless, although it may a retro virus with ssRNA as a genetic material.
not feel that way at the time. If it's not a runny nose, 89. (d) IgA is the most abundant class, constituting about
sore throat and cough, it's the watery eyes, sneezing 10 to 15 per cent of antibodies of serum. It is mainly
and congestion, or maybe all of the above. There's found in sweat, tears, saliva, mucus, colostrum (first
no cure for the common cold. Antibiotics are of no milk secreted by a mother) and gastrointestinal
use against cold viruses secretions. IgG is the second most abundant class
80. (b) Cancerous cells appear to have lost the property of of Ig in the body constituting approximately 8% of
contact inhibition. the total Igs. It is found in the blood, lymph and
81. (c) In general, as the concentration of HIV increases the intestine.
number of helper T cells destroyed increases causing 90. (c) Tuberculosis is caused by Mycobacterium
the cell count to fall. tuberculosis.
82. (b) The use of drugs like antihistamine, adrenaline, and 91. (c) Malaria is a vector borne disease that spreads by
steroids quickly reduces the symptoms of allergy. biting of the female Anopheles mosquito.
Allergy is a damaging immune response by the body 92. (d) (a) Bhang – It is a preparation from the leaves and
to a substance, especially a particular food, pollen, flowers (buds) of the female cannabis plant,
fur, or dust, to which it has become hypersensitive. consumed as a beverage.
83. (c) Cocaine is obtained from the coca plant, (b) Cocaine – Cannabinoids, it is obtained from
Erthyroxylum coca. Cocaine is commonly called as the coca plant, Erthyroxylum coca. It interferes
coke or crack which is usually snored. It has a potent with the transport of the neurotransmitter
stimulating action of euphoria and increased energy. dopamine. Cocaine is commonly called as coke
It interferes with the transport of the neurotransmitter or crack.
dopamine. Its increased dosages cause hallucinations. (c) Morphine – It is an opioid analgesic drug. It is
84. (b) Active immunity is the immunity which results from the main psychoactive chemical in opium and
the production of antibodies by the immune system used as sedative and painkiller.
in response to the presence of an antigen. Active 93. (b) AIDS - HIV (human deficiency virus - retrovirus)
immunity is slow and takes time to give its full Malaria - Plasmodium (protozoan)
effective response. Injecting the microbes
Gonorrhoea - Neisseria gonorrhoeae (bacteria)
intentionally during immunization or infectious
94. (c) A : Active natural immunity occurs when the person
organisms gaining access into body during natural
is exposed to a live pathogen, develops the disease,
infection induces active immunity.
and becomes immune as a result of the primary
85. (a) Humans are exposed to large number of infectious
immune response.
agents. However, only a few of these exposures
B : First line of defense includes physical and
result in disease due to the fact that the body is able
chemical barriers that are always ready and prepared
to defend itself from most of these foreign agents.
to defend the body from infection. These include
This overall ability of the host to fight the disease-
skin, tears, mucus, cilia, stomach acid, urine flow,
causing organisms conferred by the immune system
'friendly' bacteria and white blood cells called
is called immunity.
neutrophils.
86. (c) Virus-infected cells secrete proteins called interferons
C : Passive natural immunity refers to antibody-
which protect non-infected cells from further viral
mediated immunity conveyed to a foetus by its
infection. Interferons are a group of related low
mother during pregnancy. This type of immunity is
molecular weight regulatory cytokines produced by
short-lived, lasting the first six months of the
certain eukaryotic cells in response to a viral
newborn's life.
EBD_7209
Hints & Solutions S-173

D : Second line of defense is carried on by WBC, 99. (d) A : Appearance of dry, scaly lesions on various parts
macrophages, inflammatory reactions, fever, of the body such as skin nails and scalp is the
interferons and complement system. All these devices symptoms of ringworm disease caused by
operate together to check damage to the body by Microsporum. Ringworm is a fungal disorder.
pathogens. B : Chronic inflammation of the lymphatic vessel of
95. (a) A : Amoebiasis refers to infection caused by lower limbs is the symptoms of elephantiasis disorder
Entamoeba histolytica. It may cause dysentery and which is caused by Wuchereria bancrofti.
invasive extra-intestinal disease. C : Fever, chills, cough, headache and in severe cases
B : Diphtheria is a bacterial (Corynebacterium the lips and finger nails may turn gray to bluish in
diphtheria) infection and spreads through colour are the symptoms of pneumonia. Pneumonia
respiratory droplets (such as from a cough or sneeze) is caused by Haemophilus influenzae and
of an infected person or someone who carries the Streptococcus pneumonia.
bacteria but has no symptoms. D : Constipation, abdominal pain and cramps, stool
C : Cholera is an infectious disease that causes severe with excess mucous and blood clots are the
watery diarrhea, which can lead to dehydration and symptoms of amoebiasis. Amoebiasis is caused by
even death if untreated. It is caused by eating food Entamoeba histolytica.
or drinking water contaminated with a bacterium called E : Internal bleeding, muscular pain, fever, anaemia
Vibrio cholerae. and blockage of intestinal passage are the symptoms
D : Rabies is a viral illness spread via the saliva of an of ascariasis which is caused by common roundworm,
infected animal. It is caused by the rabies (lassa) Ascaris lumbricoides.
virus which infects the brain and ultimately leads to 100. (d) Molecule (i) represents structure of morphine.
death. Morphine is the most abundant alkaloid found in opium,
96. (d) (a) Elephantiasis is caused by Wuchereria the dried sap (latex) derived from shallowly slicing the
bancrofti which is transmitted from human to unripe seedpods of the opium, or common and/or edible,
human via the female mosquito when it takes a poppy Papaver somniferum. Morphine is a potent opiate
blood meal. The parasite grows into an adult analgesic drug that is used to relieve severe pain.
worm that lives in the lymphatic system of 101. (c) In the given figure showing the mode of action of
humans. AIDS virus, the labeled sequence marked as A, B, C
(b) Malaria is caused by protozoan species called and D are respectively, viral RNA introduced into
Plasomodium that is passed from one human cell, viral DNA, viral DNA incorporates into the host
to another by the bite of infected female DNA and new viral RNA produced.
Anopheles mosquitoes. 102. (c) Lymphatic system is the network of vessels through
(c) Typhoid fever (also known as enteric fever) is which lymph drains from the tissues into the blood. In
an infectious disease caused by the bacteria the given figure of human lymphatic system, the parts
Salmonella typhi. It easily spreads through marked as A, B, C and D are respectively lymph nodes
contaminated food and water supplies and close (secondary lymphoid organ), thymus (primary
contact with others who are infected. It is lymphoid organ), spleen (secondary lymphoid organ),
characterized by very high fever, sweating, and bone marrow (primary lymphoid organ).
gastroenteritis, and diarrhoea. 103. (a) In the given diagram of an antibody molecule the part
97. (c) Physical barriers : E.g. skin and mucous coating of marked as A, B, C, D, E and F are respectively antigen
the respiratory, gastro-intestinal and urinogenital binding site, variable regions (of L chain), constant
tracts prevent the entry of microbes into the body. region ( of L chain), light polypeptide chain (L chain),
Physiological barriers : E.g. hydrochloric acid in heavy polypeptide chain (H chain) and disulfide bond.
stomach, saliva in mouth, lysozyme in tears and saliva 104. (c) Rabies and mumps are caused by virus. Mumps is
etc. caused by mumps virus while rabies is caused by
Cellular barriers : E.g. polymorphonuclear rabies virus.
leucocytes (PMNL-neutrophils), macrophages, and
105. (b) ELISA test (Enzyme-linked immune-sorbent Assay) is
natural killer cells in the blood and tissues kill
done to diagnose AIDS. ELISA is a technique which
pathogen by phagocytosis.
can detect and even quantitate extremely small. amount
Cytokine barriers : Virus infected cells secrete of proteins antibodies or antigens with the help of
proteins called interferon which protect non-infected enzymes (peroxidases and alkaline phosphatase).
cells from spread of viral infection.
106. (c) Life cycle of Plasmodium starts with inoculation of
98. (d) Tetanus is caused by Clostridium tetani while plague sporozoites (infective stage) through the bite of
is caused by Pasteurella pestis. infected female Anopheles mosquitoes.The parasite
S-174 Biology
initially multiplies within the liver cells and then chemistry by altering levels of neurotransmitters
attacks the red blood cells (RBCs), resulting in their (chemical messengers) that transmit the signals
rupture.There is release of a toxic substance called throughout the body that control thought processes,
haemozoin from the ruptured RBCs which is behaviour and emotion.
responsible for the chill and high fever. 114. (d) Smoking addiction is harmful because it produces
107. (c) Hormone produced during allergic reaction is polycyclic aromatic hydrocarbons which cause
glucocorticoids. Allergic reactions typically have an cancer.
adrenal component. Cortisol, one of the primary 115. (c) In alcoholism liver gets damaged as it accumulates
hormones produced by the adrenal glands, is a strong excess of fats.
anti-inflammatory agent. For this reason proper 116. (c) Diphtheria is a bacterial (Corynebacterium
adrenal function plays an important role in mediating diphtheria) infection and spreads through respiratory
the histamine release and inflammatory reactions that droplets (such as from a cough or sneeze) of an infected
produce the symptoms experienced with allergies. person or someone who carries the bacteria but has no
Glucocorticoids are part of the feedback mechanism symptoms. It is a serious infection usually affecting the
in the immune system that turns immune activity mucous membranes of nose and throat.
(inflammation) down. They are therefore used in 117. (d) Mumps, cholera and TB, all are communicable
medicine to treat diseases caused by an overactive diseases. Mumps is caused by mumps virus, Cholera
immune system, such as allergies, asth ma, is caused by a bacterium called, Vibrio cholerae,
autoimmune diseases, and sepsis. and TB is caused by bacterium called
108. (c) Vaccines produced through genetic engineering are Mycobacterium tuberculosis.
considered safe because they contain antibodies for 118. (b) Cholera is an infectious disease, caused by eating
coat proteins only. food or drinking water contaminated with a bacterium
109. (a) Cholera is transmitted by food and water called Vibrio cholerae. It causes severe watery
contaminated with faeces. Hence, if you keep the diarrhoea, which can lead to dehydration and even
sanitary system around you clean, then it would not death if untreated. Therefore saline is given to
probably break out. cholera patients because giving saline to a cholera
110. (b) Poliomyelitis (polio) is a highly infectious viral patient is a rehydration treatment.
disease, which mainly affects young children. The 119. (c) A person is injected with globulin against hepatitis.
virus is transmitted by person-to-person and spread This is an example of artificially acquired active
mainly through the faecal-oral route or, less immunity. Artificially acquired active immunity can be
frequently, by a common vehicle (e.g. contaminated induced by a vaccine, a substance that contains antigen.
water or food) and multiplies in the intestine, from A vaccine stimulates a primary response against the
where it can invade the nervous system and can antigen without causing symptoms of the disease.
cause paralysis. In polio, the legs get paralyzed and 120. (d) Damage to thymus in a child may lead to loss of cell-
atrophied due to death of some muscles. Initial mediated immunity.
symptoms of polio include fever, fatigue, headache,
121. (c) The best HLA match for transplants in order of
vomiting, stiffness in the neck, and pain in the limbs.
preference is:
111. (d) Alcohol consumption dilates blood vessels and
Twin > siblings > parent > unrelated donor
increases loss of body heat from the skin and may
cause a life-threatening decrease in core temperature The human leukocyte antigen (HLA) system is the
of the body called hypothermia. locus of genes that encode for proteins on the surface
of cells that are responsible for regulation of the
112. (a) The antibodies could not solve all the problems of
immune system in humans. This group of genes
bacteria mediated diseases because of the
resides on chromosome 6, and encodes cell-surface
development of mutant strains resistant to
antigen-presenting proteins and has many other
antibodies.
functions. It is used to match the patient with a donor
113. (d) Pons varolii is not affected by alcohol. Alcohol not for bone marrow or cord transplantation.
broken down by the liver goes to the rest of the
122. (d) Yellow fever and dengue are viral diseases, as well
body, including the brain. Alcohol can affect parts
as they are transmitted by mosquitoes.
of the brain (like cerebral cortex, cerebellum,
123. (a) Ringworm is a skin infection caused by a fungus.
hypothalamus, pituitary and medulla) that control
The Microsporum, pathogen responsible for
movement, speech, judgment, and memory. These
ringworm disease in human beings belongs to the
effects lead to the familiar signs of drunkenness:
same kingdom as that of Rhizopus, a mould.
difficulty walking, slurred speech, memory lapses,
Rhizopus is a genus of common saprophytic fungi
and impulsive behavior. Alcohol affects brain
on plants and specialized parasites on animals. They
EBD_7209
Hints & Solutions S-175

are found on a wide variety of organic substrates, improvement.


including "mature fruits and vegetables", jellies, 8. (c) Apiculture or bee-keeping is rearing of honey bees
syrups, leather, bread, peanuts and tobacco. for the production of honey and wax at commercial
124. (d) Health is the state of being free from illness or injury. level. Apis indica is the most commonly domesticated
Mind and mental state can affect the health. Health honey bee in India.
is also affected by genetic disorders (deficiencies 9. (a) The raising and care of bees for commercial or
with which a child is born and deficiencies or defects agricultural purposes is called apiculture. Apis indica
which the child inherits from their parents); is reared in artificial hives or beehive because of its
infections; and life style including food and water docile nature and friendly temperature.
we take, rest and exercise we give to our bodies, 10. (d) Labeo rohita (Rohu), Catla catla (Catla), Mystus
habits that we have or lack. cinghala (Singhara), Clarias batracus (Magur),
125. (a) Along the symptoms, facts (i) and (iii) confirmed that Wallago attu (Malli), Heteropneustes (Singhi) and
the person is suffering from typhoid fever. Typhoid common carps (fam. Cyprinidae) are all fresh water
fever (also known as enteric fever) is an infectious fishes.
disease caused by the bacteria Salmonella typhi. It 11. (d) Harpodon (Bombay duck), Hilsa, salmon, mackerel,
easily spreads through contaminated food and water sardine, Stromateus (Pomphret) are marine fishes.
supplies and close contact with others who are Marine fishes deals with fishery aspects of the sea
infected. It is characterized by very high fever, water or oceans.
sweating, weakness, stomach pain, constipation, 12. (b) India is mainly an agricultural country. Agriculture
headache and loss of appetite. accounts for approximately 33 percent of India's GDP
Chapter 31 : Strategies for Enhancement in and employs nearly 62 percent of the population.
After India's independence, one of the main
Food Production
challenges facing the country was that of producing
1. (b) It is estimated that more than 70 percent of the world enough food for the increasing population.
livestock population is in India and China. However, 13. (a) Plant breeding refers to the modification and
the contribution to the world farm produce is only 25 improvement of genetic material of plants resulting
percent i.e the productivity per unit is very low. in the development of crops which are more beneficial
2. (b) When breeding is between animals of the same breed it to human beings.
is called inbreeding which is for 4 - 6 generation, while Crossing between the two genetically diverse parents
crosses between different breeds are called outbreeding. to obtain a progeny with the desired traits is called
3. (c) Inbreeding depression is the reduced biological hybridization.
fitness in a given population as a result of inbreeding 14. (c) Pusa swarnim is a variety of Brassica which is
i.e., breeding of related individuals. However, resistant to white rust disease. Himgiri is a variety
continued inbreeding, especially close inbreeding of wheat and resistant to leaf and stripe rust. Pusa
usually reduce fertility and even productivity. komal is a variety of cowpea, resistant to bacterial blight.
4. (a) Hisardale is a new breed of sheep developed in Pusa sadabahar is a variety of chilli, resistant to chilli
Punjab by the method of cross-breeding. In this mosaic virus, tobacco mosaic virus and leaf curl.
method, superior males of one breed are mated with 15 (d) Pusa Snowball K–1 and Pusa Shubhra are varieties
superior females of another breed. of cauliflower and resistant to black rot and curl blight
5. (d) The mule is a hybrid between male donkey and black rot diseases.
female horse. It is the result of interspecific 16. (c) Mutation is a sudden stable and heritable change
hybridization in which crossing is done between the which alters the genotype of an organism. In mung
members of different species. Mules show hybrid bean, resistance to yellow mosaic virus and powdery
vigour. They are sturdier than the horse and larger mildew were induced by mutations.
than donkeys. Both male and female mules are 17. (c) Cobalt 60 is the synthetic radioactive isotope of
infertile. cobalt. Gamma rays are produced when an unstable
6. (a) To improve the chances of successful mating for the atomic nucleus like cobalt-60 releases energy to gain
production of better breeds, a new technology, called stability. Sharbati Sonora and Pusa Lerma are the
Multiple Ovulation and Embryo Transfer technology two important varieties of wheat that are produced
(MOET) is being used for cattle and other animals. by gamma ray treatment of Sonora-64 and Lerma
7. (c) Stage of 8 to 32 celled embryos are transferred to Rojo-64 which are Mexican dwarf wheat varieties.
surrogate mothers in livestock breeding experiments. 18. (d) Plant breeding refers to the modification and
It is a stage under MOET (multiple ovulation embryo improvement of genetic material of plants resulting
transfer technology). MOET is a programme for herd in the development of crops which are more beneficial
S-176 Biology
to human beings. Out-crossing, out-breeding and is called somatic hybridization.
cross-breeding comes under animal breeding. 27. (a) Inbreeding is mating of individuals closely related
19. (c) Germplasm collection is the entire collection of by ancestry. Inbreeding is used in developing pure
plants/ seeds having all the diverse alleles for all lines or homozygous lines.
genes in a given crop. Germplasm is a term used to 28. (d) All the given statements are correct.
describe living genetic resources such as seeds or Haploid culture, developed by Guha and
tissue, maintained for the purpose of breeding, Maheshwari is highly useful for immediate expression
preservation, and other research uses. of mutations and fast formation of pure lines. A line
20. (d) A. esculentus (called prabhani kranti) is a yellow vein consists of a group of individuals which are related
mosaic resistant new variety which is obtained when to descent and have similar genotypes. Mutation is
resistance gene are transferred from a wild species the changing of the structure of a gene, resulting in
against yellow mosaic virus in Abelmoschus a variant form due to alteration of single base units
esculentus. It is derived from the back cross of in DNA or the deletion, insertion, or rearrangement
Abelmoschus manihot x A. esculentus. Two back of larger sections of genes or chromosomes. Such
crosses were made using Pusa sawani (A. esculentus) variants may be transmitted to subsequent
as a recurrent parent followed by selection in the generations.
subsequent generations. It is a yellow vein mosaic 29. (d) All the given statements are correct.
virus (YVMy) resistant variety isolated in F8 30. (d) In 2000, maize hybrids that had twice the amount of
generation. the amino acids, lysine and tryptophan, compared
21. (d) Maize stem borer is the most notorious pest of maize to existing maize hybrids were developed. Some of
crop which damages the crop considerably and the diseases caused by fungi are brown rust of wheat,
reduces the yield by 10-20% but sometimes the losses red rot of sugarcane, late blight of potato, etc. Black
may be up to 80%. High aspartic acid, low nitrogen rot of crucifers is caused by bacteria and tobacco
content and low sugar content are some factors which mosaic and turnip mosaic are caused by viruses.
are responsible for resistance to maize stem borer. 31. (d) Spirulina is a single cell protein rich in protein,
22. (b) Atlas 66 has been used as a donor for improving vitamins and minerals. 250 gram biomass of
cultivated wheat. It is a cultivar i.e a plant variety Methylophilus methylotrophus produces 25 tonn
that has been produced in cultivation by selective protein/day while a cow of 250 kg. produces only
breeding so it contains desired characters such as 200 gm. protein/day. Common button mushrooms are
more than one Al tolerance gene. So plant breeders a very rich source of vitamin D. A rice variety has
can use seeds of Atlas 66 and then cross breed it been developed which is very rich in iron content.
with another plant having another desired character. 32. (c) Pusa Sawani variety of Okra is resistant to shoot
In this way there would be improvement in cultivated and fruit borer.
wheat. 33. (c) Animal husbandary is the agricultural practice of
23. (b) Totipotency is the ability of a cell, such as an egg, to breeding and raising livestock by applying scientific
give rise to unlike cells and to develop into or principles.
generate a new organism or part. Plant cells are also 34. (d) Inbreeding increases homozygosity to develop a pure
totipotent, which helps to explain why a graft of a line animal.
plant can generate a whole new individual out of
35. (c) All the given statements are related to outcrossing.
just a small branch cutting.
Outcrossing refers to mating of unrelated individuals.
24. (c) The part of the plant which is used in tissue culture is Inbreeding is the production of offspring from the
called explant. It may be excised root tips, shoot bud, leaf mating or breeding of individuals or organisms that
petiole, inflorescence, anther, ovule, ovary or embryo. are closely related genetically. Outbreeding is the
25. (a) Micropropagation is the practice of rapidly practice of introducing unrelated genetic material into
multiplying stock plant material to produce a large a breeding line. Cross breeding is the process of
number of progeny plants, using modern plant tissue mating of animals with another species, or of causing
culture methods under controlled aseptic physical plants to reproduce with another species.
conditions. Types or processes of micropropagation 36. (a) Pisciculture refers to the breeding, hatching and
are: micropropagation by axillary and apical buds; rearing of fishes under controlled conditions whereas
by axillary shoots (buds, bulbs and protocorms); aquaculture is the cultivation of aquatic organisms
through callus culture; artificial seeds and such as algae, fishes, prawns, shrimps, crabs and
somaclonal variations. pearl-oysters etc. at commercial level by proper
26. (b) Culture of fused protoplasts produce somatic hybrid utilization of small and large water bodies. The
cells and the process of producing somatic hybrids development and flourishing of the fishery industry
EBD_7209
Hints & Solutions S-177

is called blue revolution. to black rot and curl blight black rot diseases.
37. (a) 44. (b) Pusa sadabahar is a variety of chilli while Pusa komal
38. (c) Sonalika and Kalyan Sona were high yielding and is a variety of cowpea.
disease resistant crops. Saccharum barberi had poor 45. (c) Cytokinins is a plant hormone. It is a derivative of
sugar content and yield, but was originally grown in purine adenine. Made in the roots (also seeds and
north India. fruits), cytokinins travel up the xylem and promote
39. (b) Pisciculture is the controlled breeding and rearing of lateral growth. Cytokinins have been found in all
fish. Apiculture is the raising and caring of bees for complex plants, as well as mosses, fungi, and
commercial or agricultural purposes. Tissue culture bacteria. Cytokinins promote cell division in plant
is the process or technique of making body tissue roots and shoots, and they promote the growth of
which grow in a culture medium outside the organism. buds.
Green revolution is a large increase in crop 46. (d) The raising and care of bees for commercial or agri-
production in developing countries achieved by the cultural purposes is called apiculture.
use of artificial fertilizers, pesticides, and high- Sericulture is the breeding and management of silk-
yielding crop varieties. Blue revolution is the rapid worm and the raw silk they produce.
increase of fish production in small ponds and water Pisciculture is the controlled breeding and rearing of
bodies. fish. Aquaculture (also known as fish or shellfish
40. (c) Hidden hunger is a nutritional deficiency which is farming) refers to the breeding, rearing, and
caused by lack of balance in a full diet. Bio- harvesting of plants and animals in all types of water
fortification is the method of breeding staple crops environments including ponds, rivers, lakes, and the
to obtain higher levels of essential nutrients, either ocean.
through selective breeding or genetic modification. 47. (a) Somatic embryos are non- zygotic embryo like
Single cell protein refers to those proteins which are structures that develop into from any type of tissue
extracted from pure culture or mixed cultures of in plant tissue culture.
microorganisms such as algae, yeast, fungi or 48. (b) Fish meal obtained from the non-edible parts is a
bacteria. These extracted proteins are used as a good source of protein for poultry and cattle.
substitute. Totipotency is the ability of a cell that is 49. (d) Protoplast culture results in the production of somatic
capable of developing into a complete organism or hybrid plants, an important technique of tissue
differentiating into any of its cells or tissues. culture.
Micropropagation is the propagation of plants by
50. (c) Meristem tissue culture involves removal of meristem
growing plantlets in tissue culture and then planting
and growth in vitro. This is how virus free plants can
them out. Somaclones are genetically induced plants
be obtained.
which are produced by a genetic engineering
51. (b) An important technique of tissue culture, somatic
technique. In this process single cells or protoplasts
hybridization results in the production of somatic
are cultured to produce individuals which are
hybrid plants. Two different plant varieties each with
genetically variable from their genetically stable
a desirable character can be made to undergo
parent.
protoplast fusion, which further can be grown into a
41. (d) Semi dwarf wheat was developed by Nobel laureate
new plant.
Norman E. Borlaug. Sonalika and Kalyan Sona are
52. (b) The above figures of maize, wheat and garden pea
the varieties of semi dwarf wheat which are high
are some Indian hybrid crops. The term "hybrid"
yielding and disease resistance. Semi dwarf rice was
refers to a plant variety which is developed through
derived from IR - 8. Examples are IR 8, Jaya, Taichung
the interbreeding of two or more varieties, genera or
Native - 1 and Ratna.
species. Though hybrids contain the best proper-
42. (b) Selection and hybridization are the conventional
ties of the parent plants, they usually do not breed
method of breeding for disease resistance. Some
true and often revert to one of the parent plants.
crops were bred by the above mentioned technique
Hybrids are favoured for greater disease resistance,
for disease resistance to fungi, bacterial and viral
more vigorous growth, earlier maturity, higher qual-
diseases. For examples: wheat (leaf and stripe rust,
ity of vegetables, better uniformity and improved
hill bunt), Brassica (white rust), cowpea (bacterial
flavour.
blight), cauliflower (black rot and curl blight black
53. (c) Callus culture is a type of in vitro plant tissue culture.
rot), chilli (tobacco mosaic virus and leaf curl).
The process is carried out under controlled
43. (c) Pusa komal is a variety of cowpea which is resistant
conditions. The selected cell, tissue or organ is called
to bacterial blight disease. Pusa shubhra and Pusa
explant. The number of cells increases through cell
Snowball K-1 are varieties of cauliflower resistant
division. However, these cells are unorganized and
S-178 Biology
collectively constitute a callus. They are maintained 60. (d) Any alteration from the healthy structure and normal
on agar-agar gel. Growth promoters like auxin and physiological activities of the plant body is called
cytokinins are added to the culture. Under these plant disease. The consequences of the plant
conditions, the cells become meristematic and begin diseases are reduced yield, lower quality of produce,
to divide. Callus is obtained within 2 to 3 weeks. in creased cost of production and poisonous
54. (a) For sugarcane crop, farmer looks for its thick stem, long produce.
internodes, high sugar content and disease resistance. 61. (c) Sharbati Sonora is mutant yielding variety of wheat
55. (c) The biggest compulsion of plant breeding is transfer developed from Sonora 64 by irradiation with gamma
of genes from unrelated sources. Plant breeding is rays.
the purposeful manipulation of plant species to create 62. (c) Plants derived sexually from the same plant are
desired genotypes and phenotypes for specific different while those derived from somatic tissue
purposes. This manipulation involves either from the same plant are identical.
controlled pollination, genetic engineering, or both, 63. (a) Callus is an unorganised mass of parenchyma cells.
followed by artificial selection of progeny. Callus cells are those cells that cover a plant wound
56. (d) High milk yielding varieties of cows are obtained by and whose formation is induced from plant tissues
super ovulation, artificial insemination and use of after surface sterilization and plating onto in vitro
surrogate mothers. In artificial insemination, semen tissue culture medium. Plant growth regulators, such
of a selected male is artifically deposited into the as auxins, cytokinins, and gibberellins are
vagina of a selected female. Multiple Ovulation supplemented into the medium to initiate callus
Embryo Transfer (MOET) is a program which formation or somatic embryogenesis.
improves the chances of successful production of 64. (d) In crop improvement programme, haploids are very
hybrids. It involves the stimulation of healthy and important because they form perfect homozygotes
high-milk yielding breed of female animals by which are helpful to get a pure line.
injecting gonadotrophic hormones to release more 65. (c) Animal biotechnology is the use of science and en-
eggs from the ovaries. Embryo at 8-32 celled stage gineering to modify living organisms. This technique
can be recovered non-surgically from the female and makes products to improve animals and to develop
then transferred into the surrogate females for microorganisms for specific agricultural uses. In vitro
implantation and further development. fertilization and embryo transfer are the techniques
57. (a) Improvement of crop varieties is a permanent measure which are used in animal biotechnology to require
which creates genetic potentiality in crop plants for the rapid multiplication and production of animals
higher & better yield. Steps of crop improvement are with a desirable trait.
as follow : In vitro fertilization is the joining of a woman's egg
Plants with superior traits are indentified and a man's sperm in a laboratory dish. Embryo trans-
¯ fer refers to a step in the process of assisted repro-
Cross-hybridization between the selected plants duction in which embryos are placed into the uterus
¯ of a female with the intent to establish pregnancy.
Plants with superior traits among the progeny are 66. (d) Sustainable agriculture is the preparation, manage-
selected ment and successful use of bio - resources in agri-
¯ culture to meet the present demand without compro-
Again cross hybridization between the superior mising the ability of future generations to meet their
hybrids is carried out while eliminating the plants own needs. The way in which biotechnology has
with less beneficial traits. contributed to sustainable agriculture is biofertilizers,
bio - pesticides, single cell protein and disease and
58. (a) Explants that contain pre-existing shoot meristems
insect resistant varieties.
and produce shoots from them are used in meristem
culture. The virus free clones can be obtained from a 67. (b) An improved variety of transgenic basmati rice gives
virus infected plant by tissue culture since virus is high yield and is rich in vitamin A.
trans-located through sieve tubes. The apical 68. (a) Cellular totipotency is the ability, shown by many
meristem of virus infected plant remains free of virus. living cells to form all types of tissue that constitute
The shoot apex of such plant can be cultured. the mature organism.
59. (c) Selective breeding (also called artificial selection) is 69. (a) Plant can be made disease resistant by breeding with
the process by which humans can breed animals and their wild relatives.
plants for particular traits. Typically, strains that are 70. (c) Genetic diversity refers to the variety of genes within
selectively bred are domesticated, and the breeding a species. Each species is made up of individuals
is normally done by a professional breeder. having their own particular genetic composition.
EBD_7209
Hints & Solutions S-179

Within a species there may also be discrete popula- Escherichia. It is commonly found in the lower
tions with distinctive genes. Introduction of high intestine of warm-blooded organisms (also called
yielding varieties is the greatest threat to genetic endotherms).
diversity in agricultural crops. 5. (a) The dough, which is used for making bread, is
71. (a) Crop improvement proposes to obtain crops with fermented by using baker’s yeast (Saccharomyces
higher yield, better quality, resistance to disease and cerevisiae). The puffed up appearance of dough is
shorter duration which are suitable to particular due to the production of CO2 gas.
environmental conditions. Introduction, hybridization 6. (c) Cheese is prepared by the coagulation of casein and
and mutations are methods of plant breeding. other minor milk proteins by an enzyme rennin.
72. (d) Somaclonal variation is a genetic variation present Rennin is extracted from the calf gastric mucosa.
among plant cells of a culture. It has been used to Streptococcus and Lactobacillus species are
develop several useful varieties. It usually appears involved in the manufacture of most cheese. In
in tissue culture raised plants. cheese manufacture, these microorganisms are
73. (b) Totipotency is the capability of a cell to develop into important in both souring and ripening processes.
a complete organism or differentiating into any of its 7. (a) Louis Pasteur was a French chemist and
cells or tissues. Totipotent cells serve the same role microbiologist renowned for his discoveries of the
in plants that stem cells do in animals. They are found principles of vaccination, microbial fermentation and
in shoot and root growing tips as meristems, and in pasteurization.
the cambium layer (the layer of cells between the 8. (d) Saccharomyces is a genus in the fungi kingdom that
bark and the wood) of woody plants and trees. All of includes many species of yeast. Many members of
the structures found in a mature or growing plant are this genus are considered very important in food
the result of cellular material produced by meristem- production. It is known as the brewer's yeast or
atic tissue. baker's yeast. This type of yeast is unicellular and
74. (d) Hisardale is a new breed of sheep developed in saprophytic fungi, example Saccharomyces
Punjab by crossing Bikaneri ewes and Marino rams. cerevisiae, which is used in making wine, bread, idli
Cross breeding is a method in which superior male and beer.
of one breed is mated with superior females of an- 9. (a) Large holes in ‘Swiss cheese’ are due to production
other breed. It allows the desirable qualities of two of CO 2 by Propionibacterium sharmanii (a
different breeds to be combined. bacterium).
75. (a) The process by which protoplasts of two different 10. (c) Yeast is used in the making of alcohol. For many
plant species fuse together to form hybrids is known years, the only source of ethanol for industrial use
as somatic hybridization and the hybrids so produced was from the fermentation of sugars by yeast. The
are known as somatic hybrids. Somatic hybrids are basic reaction of this conversion involves the
used in gene transfer, transfer of cytoplasm and breakdown of glucose into carbon dioxide and ethanol.
production of useful alloploids. The glucose is converted to pyruvic acid and the
pyruvic acid may be converted into many different
Chapter 32 : Microbes in Human Welfare end products such as ethanol, lactic acid, etc.
1. (b) Curd contains numerous lactic acid bacteria (LAB) 11. (c) Zymase is an enzyme complex that catalyzes the
or Lactobacillus. These bacteria produce acids that fermentation of sugar into ethanol and carbon
coagulate and digest milk proteins. dioxide. It occurs naturally in yeasts. Its activity
2. (a) Role of lactic acid bacteria (LAB) in stomach is varies among yeast strains.
beneficial. These microorganisms have the specific 12. (a) Fermentation is defined as an energy yielding
property of transforming sugars almost completely process whereby organic molecules serve as both
into lactic acid and acetic acid that decrease the pH electron donors and electron accepters. The molecule
of the intestines and produce substances that being metabolized does not have all its potential
suppress harmful bacteria. They are abundant in energy extracted from it. Beer and wine are produced
nature and are essential for human and animal by fermenting glucose with yeast. Yeast contains
survival. enzymes that catalyse the breakdown of glucose to
3. (b) Curd contains numerous lactic acid bacteria (LAB) ethanol and carbon dioxide.
or Lactobacillus. These bacteria produce acids that 13. (a) Streptokinase are produced by Streptococcus and
coagulate and digest milk proteins. It improves its are used as a ‘clot buster’ to remove clots from the
nutritional quality by increasing vitamin B12. blood vessels of patients who have myocardial
4. (c) Escherichia coli is a gram-negative, facultatively infarction.
anaerobic, rod-shaped bacterium of the genus
S-180 Biology
14. (c) Cyclosporin A, a powerful immunosuppressive 26. (b) BOD refers to biological oxygen demand. It is the
agent, is used in the treatment of organ transplant method of determining the amount of oxygen
patients. required by microorganisms to decompose the waste
15. (d) Cyclosporin A, a powerful immunosuppressive present in the water supply.
agent, is used in the treatment of organ transplant 27. (d) Activated sludge is a mass of microorganisms
patients. It has been used with initial success in cultivated in the treatment process to break down
recipients of kidney, liver, bone marrow and pancreas organic matter into carbon dioxide, water, and other
transplants, and it may also have clinical application inorganic compounds. The activated sludge process
in the treatment of autoimmune disorders. has three basic components: i) a reactor in which the
16. (c) Statin is a lipid-lowering drug that reduces serum microorganisms are kept in suspension, aerated, and
cholesterol levels by inhibiting a key enzyme in contact with the waste they are treating; ii. liquid-
involved in the biosynthesis of cholesterol. Statins solid separation and iii. a sludge recycling system
are obtained from Monascus purpureus. for returning activated sludge back to the beginning
17. (d) Statins are produced by Monascus purpureus, yeast of the process.
and used as blood-cholesterol lowering agents. It 28. (d) The major component of biogas is methane (about
inhibits the enzymes responsible for synthesis of 50-68%) which is highly inflammable . The other
cholesterol. gases are carbon dioxide (25 - 35%), hydrogen (1 -
18. (d) Monascus purpureus is a yeast used in the 5%), nitrogen (2 - 7%), oxygen (0 - 0.1%) and rarely
production of statins. Statins are used in lowering hydrogen sulfide. Biogas is a “mixture of gases”
blood cholestrol. produced from degradable organic matter by the
19. (d) Fermentor is an apparatus that maintains optimal activity of various anaerobic bacteria that offers a
conditions for the growth of microorganisms, used low cost alternative for energy requirements.
in large-scale fermentation and in the commercial 29. (a) The Ministry of Environment and Forests has
production of antibiotics and hormones. initiated Ganga Action Plan and Yamuna Action Plan
20. (a) Sewage is a waste material (like human urine and to save major rivers of India from water pollution.
faeces) that is carried away from homes and other 30. (c) Methanobacterium, a common bacter ia of
buildings in a system of pipes. Sewage treatment is methanogens, is found in the anaerobic sludge during
the process of removing contamin ants from sewage treatment and rumen of cattle (for cellulose
wastewater which is done by heterotrophic microbes digestion).
naturally present in sewage. 31. (a) The technology of biogas production was developed
21. (c) Antibiotics are medicines that are produced by in India mainly due to the efforts of Indian Agricultural
certain microorganisms to kill other disease causing Research Institute (IARI) and Khadi and Village
microorganisms. These medicines are commonly Industries Commission (KVIC).
obtained from bacteria and fungi. 32. (c) Insect resistant transgenic cotton has been
22. (a) Primary treatment of sewage is a physical process produced by inserting a piece of DNA from
and concerned mainly with the removal of coarse bacterium. Pest resistant genetically modified crops
solid materials through filtration and sedimentation. (primarily cotton and maize) have been genetically
23. (d) Primary sludge is used for the preparation of compost, modified so they are toxic to certain insects. They
manure and biogas production. Primary sludge is a are often called Bt crops because the introduced
result of the capture of suspended solids and genes were originally identified in a bacterial species,
organics in the primary treatment process through Bacillus thuringiensis. These bacteria produce a
gravitational sedimentation. The secondary group of toxins called Cry toxins.
treatment process uses microorganisms to consume 33. (c) Microbial biocontrol agent that can be introduced in
the organic matter in the wastewater. The order to control butterfly caterpillars is the bacteria
microorganisms feed on the biodegradable material Bacillus thuringiensis (Bt). They are sprayed onto
in the wastewater in the aeration tank then flow into vulnerable plants, where they are eaten by the insect
a secondary clarifier where the biomass settles out larvae. In the gut of the larvae, the toxin is released
and removed as secondary sludge. and the larvae get killed. The bacterial disease will kill
24. (b) The primary effluent is passed into large aeration the caterpillars, but leave other insects unharmed.
tanks and constantly agitated to allow vigorous 34. (b) Trichoderma is a free-living saprophytic fungi that
growth of useful aerobic microbes into flocs. most commonly lives on dead organic matter in the
25. (b) BOD is the method of determining the amount of soil and rhizosphere (root ecosystem). It inhibits
oxygen required by microorganisms to decompose pathogens through release of gliotoxin, viridin,
the waste present in the water supply. It is a measure gliovirin and trichodermin like substances.
of organic matter present in the water.
EBD_7209
Hints & Solutions S-181

35. (c) Baculoviruses are a family of rod-shaped viruses that 46. (d) Organic farming is a method of agriculture. It
belong to genera-Nucleopolyhedrovirus. comprises the production of crop and livestock that
Baculoviruses attacks insects and other arthropods involves much more than selecting not to use
and the majority of them are used as biological pesticides, fertilizers, genetically modified organisms,
control. They are extremely small and composed of antibiotics and growth hormones. It supports the
circular, double stranded DNA that codes for genes use of crop rotations and cover crops, and
needed for virus establishment and reproduction. encourages balanced host/predator relationships.
36. (c) Cyanobacteria (blue-green algae) is one of the major Organic residues and nutrients produced on the farm
components of the nitrogen fixing biomass in the are recycled back to the soil. Cover crops and
paddy (rice) field. The agricultural importance of composted manure are used to maintain soil organic
cyanobacteria in rice cultivation is directly related matter and fertility.
with their ability to fix nitrogen and other positive 47. (c) Controlled dissolved oxygen is not involved in stirred
effects for plants and soil. tank fermentation.
37. (a) Anabaena is the free-living nitrogen fixing 48. (c) Activated sludge sediment in settlement tanks of sewage
cyanobacteria that forms symbiotic association with treatment plants is rich source of aerobic bacteria.
Azolla. Anaerobic bacteria digest the bacteria and fungi in the
38. (a) Biofertilizers are living organisms, which help sludge by producing gases like methane, hydrogen
increase the fertility of soil. It involves the selection sulphide and carbon dioxide. These gases form biogas.
of beneficial microorganisms like bacteria, fungi, 49. (c) Biological controls of plant diseases are the most
cyanobacteria, etc. important role of microorganisms for the well - being
39. (d) Symbiotic association is exhibited by mycorrhiza and of humans.
Rhizobium. 50. (c) Statement (ii) and (iii) are the correct statements
40. (b) Mycorrhiza is a symbiotic association of fungi (e.g. regarding the organic farming. Organic farming
the genus of Glomus) with plants. The fungus gets comprises the production of crop and livestock that
food from the plant while the fungal symbionts involves much more than selecting not to use
absorb phosphorous from soil and passes it to the pesticides, fertilizers, genetically modified organisms,
plant. Also, they give resistance to root-borne antibiotics and growth hormones. It supports the
pathogens and tolerance to salinity and drought. use of crop rotations and cover crops, and
Hence, they give an overall increase in plant growth encourages balanced host/predator relationships.
and development. 51. (d) Gobar gas (or biogas) is a mixture of methane and
41. (c) Baculoviruses are suitable for species-specific, carbon dioxide produced by bacterial degradation
narrow spectrum insecticidal applications. of organic matter and used as a fuel. Gobar gas is an
42. (a) BOD is the method of determining the amount of efficient source of energy and used as good
oxygen required by microorganisms to decompose fertilizers. It also reduces the chances of spreading
the waste present in the water supply. It is a measure of pathogens.
of organic matter present in the water. If the quantity 52. (b) Besides cereals, green vegetables, brewer's yeast,
of organic wastes in the water supply is high then egg white, vitamin B12 is also produced by intestinal
the number of decomposing bacteria present in the bacteria. The vitamin was first obtained in 1938 using
water will also be high. As a result, BOD value will wild strain of mould Ashbhya gossypii. Vitamin B12
increase. is essential for normal growth and reproduction in a
43. (d) Antibiotic is a chemical substance, produced by some number of laboratory animals.
microbes and kills the growth of other disease 53. (b) Irrigation relates to the supply of water to the crops.
causing microbes. Each antibiotic is effective against Fertilizers which are mainly NPK are required to
all kinds of germs. increase the harvest of crops.
44. (b) Animal excreta and organic waste from kitchen lead 54. (c) Leguminous plants like beans, peas, groundnut are
to pollution which can be most profitably minimised sown in between the seasons of cereal crops during
by using them or producing biogas. Biogas is a crop rotation. These plants help in restoring the
mixture of gases produced by microbial activity and fertility of the soil by fixing the atmospheric nitrogen
which may be used as fuel. through the bacteria Rhizobium present in their root
45. (d) Mycorrhiza is an association between fungus with nodules. These plants thus help in overcoming the
the roots of a plant in a symbiotic or mildly nitrogen deficiency of the soil.
pathogenic relationship in which the hyphae form a 55. (b) Biopesticides pose less threat than conventional
closely woven mass around the rootlets or penetrate pesticides. Only targeted pest and closely related
the cells of the root. organisms are affected by biopesticides in contrast
S-182 Biology
to synthetic pesticides that affect a large spectrums large tanks called anaerobic sludge digesters. Here,
of animals. some anaerobic bacteria digest the bacteria and fungi
56. (b) Biogas is used for various purposes, as fuel for in the sludge by producing gases like methane,
heating, cooking, lighting, power for irrigation and hydrogen sulphide and carbon dioxide. These gases
as an alternative to kerosene, firewood dung cakes. form biogas.
57. (c) Curd contains numerous lactic acid bacteria (LAB) 68. (a) Biofertilizers are living organisms, which help in
or Lactobacillus. These bacteria produce acids (lactic increasing the fertility of soil. Example includes Azolla
acid) that coagulate and digest milk proteins. Amylase and blue green algae.
is used for the production of beer. 69. (a) Microbes are a very important component of life on
58. (c) A – III; B – IV; C – I; D – V; E – II earth. Microbes are diverse group which include
59. (b) A – III; B – IV; C – II; D – I bacteria, algae, fungi, and protozoa.
60. (a) A – III; B – I; C – V; D – II 70. (b) Microbes are single-cell organisms and so tiny that
61. (b) A : Cyanobacteria are used as fertilizers. millions can fit into the eye of a needle. Microbes
B : Mycorrhiza is an association between fungi with include bacteria and archaebacteria, protists, some
the roots of a plant. fungi and even some very tiny animals that are too
D : Single cell protein is a protein derived from a small to be seen without the aid of a microscope.
culture of single-celled organisms and used Microbes are present everywhere in soil, water, air,
especially as a food supplement. inside our bodies and that of the animals and plants.
62. (a) The bacterium involved in lactic acid production is 71. (d) Rhizobium is a symbiotic bacteria found in the root
Lactobacillus. The fungus involved in Cyclosporin nodules of leguminous plants that has the ability to
A production is Trichoderma polysporum. Penicillin fix atmospheric nitrogen. Azospirillum and
is obtained from Penicillium notatum, a fungus. Azotobacter are free-living bacteria found in the soil.
63. (a) The correct name of A, B, C and D are respectively They also enrich the nitrogen content of the soil.
Streptococcus, fungus, Cyclosporin A and Lactobacillus or Lactic acid bacteria grow in milk to
Clostridium butylicum. convert it into curd.
Streptokinase enzyme, produced by Streptococcus 72. (c) A biofertilizer is a substance which contains living
bacterium, is used as a clot buster for removing clots microorganisms which, when applied to seed, plant
from blood vessels of the patients who had gone surfaces, or soil, colonizes the rhizosphere or the
under myocardial infarction. Aspergillus niger is a interior of the plant and promotes growth by
fungus which produces citric acid. Cyclosporin A is increasing the supply or availability of primary
produced from Trichoderma polysporum, and used nutrients to the host plant. Cyanobacteria (blue
as a immuno-suppressive agent. Butyric acid is green algae), symbiotic bacteria (Rhizobium,
produced from bacterium, Clostridium butylicum. Anabaena) and free living bacteria (Azotobacter,
64. (a) The blank spaces marked as A, B, C and D are Clostridium) are used as biofertilizer. Yeast is a
respectively Trichoderma polyspora, organ microscopic fungus which consists of single oval
transplant patients, yeast and lowering of blood cells that reproduce by budding, and capable of
cholesterol. (For more refer answer 34 and 68). converting sugar into alcohol and carbon dioxide.
65. (a) 73. (b) All the given industrial products are synthesized from
66. (b) The label A represents sludge, label B represents microbes. A microbe is a microscopic organism, such as
methane and carbon dioxide, and the label C a bacterium, virus or parasite (excluding the large ones).
represents dung and water. 74. (a) Methanogens grow anaerobically on cellulosic
67. (b) The primary effluent is passed into large aeration material and produce methane, carbon dioxide and
tanks and mechanically agitated to allow vigorous hydrogen.
growth of useful aerobic microbes into flocs.These 75. (b) Cheese and yogurt are products are fermentation.
microbes consume the major part of the organic Fermentation is a metabolic process that converts
matter in the effluent. This significantly reduces the sugar to acids, gases, and/or alcohol. It occurs in
BOD (biochemical oxygen demand) of the effluent. yeast and bacteria, but also in oxygen-starved
Once the BOD of sewage water is reduced muscle cells, as in the case of lactic acid fermentation.
significantly, the effluent is then passed into a settling French microbiologist Louis Pasteur is often
tank where the bacterial ‘flocs’ are allowed to remembered for his visions into fermentation and its
sediment. This sediment is called activated sludge.A microbial causes.
small part of the activated sludge is pumped back 76. (d) Antibiotics are drugs used to treat bacterial
into the aeration tank to serve as the inoculum.The infections. Antibiotics have no effect on viral
remaining major part of the sludge is pumped into infections. An antibiotic is a substance produced by
EBD_7209
Hints & Solutions S-183

one microorganism that selectively inhibits the bioreactive molecules.


growth of another. In 1926, Alexander Fleming 87. (b) In the secondary treatment of sewage, if oxygen
discovered penicillin, a substance produced by fungi availability to activated sludge flocs (masses of
that appeared able to inhibit bacterial growth. bacteria associated with fungal filaments to form
77. (c) Azospirillum, Azotobacter, Anabaena, Nostoc, and mesh like structure) is reduced; the centre of flocs
Oscillatoria are all nitrogen fixing biofertilizers. They will becomes anoxic, which would cause death of
are all prokaryotes. bacteria and eventually breakage of flocs.
78. (b) Lactobacillus produce acids that coagulate and 88. (c) A lake with an inflow of domestic sewage rich
digest milk proteins. inorganic waste may result in death of fish due to
79. (a) Crystals of Bt toxin produced by some bacteria do lack of oxygen.
not kill the bacteria themselves because bacteria are
Chapter 33 : Biotechnology : Principles and
resistant to the toxin. Bt crystals, sometimes referred
to as insecticidal crystal proteins (ICP), are protein Processes
crystals formed during sporulation in some Bt strains. 1. (c) Plasmids is a genetic structure in a cell that can
Bt produces proteins that aggregate to form a crystal. replicate independently of the chromosomes. It is
These crystal proteins are toxic to very specific typically a small circular DNA strand in the cytoplasm
species of insects yet harmless to humans and are of a bacterium or protozoan. Plasmids are much used
the natural enemies of many crop pests. in the laboratory manipulation of genes.
80. (d) Biogas is a mixture of gases produced by microbial 2. (a) pBR 322 is an artificially constructed vector plasmid.
activity and which may be used as fuel. The correct It is widely used in gene cloning experiments.
sequence of microbes involved in biogas production 3. (c) Restriction endonuclease are called “molecular
is decomposers, fermentative microbes an d scissors or biological scissors”. They recognize and
methanogens. cut double stranded DNA at specific points and are
81. (b) Trichoderma is a species of filamentous fungi. called palindromic sequences. Palindromic sequence
Cyclosporin A is immunosuppressive drug obtained are the ones which read same on both the strand in
from Trichoderma and used in organ transplantation. 5¢ ® 3¢ direction. Same is true about 3¢ ® 5¢
82. (c) Aspergillus niger is used for the production of citric direction.
acid in industries. Citric acid is the most important 5¢ —— G¯AATTC —— 3¢
organic acid and is extensively used in food and
3¢ —— CTTAA­G —— 5¢
pharmaceutical industries. It is produced mainly by
submerged fermentation using Aspergillus niger or 4. (a) The first restriction endonuclease reported was Hind
Candida sp. from different sources of carbohydrates, II. Hind II is the first type II restriction endonuclease
such as molasses and starch based media. identified, by Hamilton Smith in 1970. It is isolated
83. (a) Biogas is a fuel gas, a mixture consisting of 60% from Haemophilus influenzae. It cleaves the
methane (CH4) and of 40% CO2. It is a renewable sequence GTPyPuAC between the unspecified
energy resulting from biomass. Biogas typically refers pyrimidine and purine generating blunt ends.
to a mixture of different gases produced by the 5. (c) Restriction enzyme belongs to nucleases (group of
breakdown of organic matter in the absence of enzymes that split nucleic acids into nucleotides and
oxygen. It can be a good substitute for fuel wood. other products). A restriction enzyme acts as a
84. (a) Pseudomonas is a genus of gram-negative, aerobic biochemical scissor and is an essential tool in
bacteria. Some members of the genus are able to recombinant DNA technology and genetic
metabolise chemical pollutants in the environment, engineering. Each restriction enzyme recognizes a
and as a result, can be used for bioremediation. short, specific sequence of nucleotide bases.
Species suitable for use as bioremediation agents Restriction enzymes were originally discovered and
include: P. alcaligenes (which can degrade characterized by the molecular biologists Werner
polycyclic aromatic hydrocarbons) and P. mendocina Arber, Hamilton O. Smith, and Daniel Nathans who
(which is able to degrade toluene etc). shared the 1978 Nobel Prize in medicine.
85. (d) Bacillus thuringiensis (Bt) is an insecticide with 6. (d) EcoRI is an endonuclease enzyme isolated from
unusual properties that make it useful for pest control strains of E.coli and a part of restriction modified
in certain situations. Bt is a naturally occurring bacterium system. So, .co. part stands for coli.
common in soils throughout the world. Several strains 7. (a) Restriction endonuclease-Hind II, always cuts DNA
can infect and kill insects. Because of this property, Bt molecules at a particular point by recognizing a
has been developed for insect control. specific sequence of six base pairs. This specific base
86. (c) Microbes are used in secondary treatment of sewage, sequence is known as the recognition sequence for
anaerobic sludge digesters and production of Hind II.
S-184 Biology
8. (a) DNA ligase is the enzyme which helps in joining two yeast) cell and produces many copies of itself and
fragments of DNA. The enzyme is used in DNA the foreign DNA. Origin of replication, selectable
replication as it joints the Okazaki segments. It also marker and cloning sites are required to facilitate
finds its use in genetic engineering as it can join two cloning of vector.
or more desired nucleotide sequences of DNA. 20. (d) Agarose gel electrophoresis plays a critical role in
9. (c) Palindromic sequences in DNA molecule are group of analysing DNA in laboratory experiments. It is a
bases that forms the same sequence when read in both method of separating biological molecules by using
forward and backward direction. In the given question, an electrical current. Agarose gel electrophoresis
only option (c) represents a palindromic sequence. provides a means of analysing DNA by separating
10. (b) DNA fragments generated by restriction molecules based on size. However, agarose gel
endonucleases in a chemical reaction can be electrophoresis does not provide a means of
separated by gel electrophoresis. Since DNA visualizing the DNA but that role is played by two
fragments are negatively charged molecules they can dyes: ethidium bromide and loading buffer. Ethidium
be separated by forcing them to move towards the bromide, known as an intercalating agent. It allows
anode under an electric field through a medium/ to intercalate, or insert, between nitrogenous bases
matrix. The DNA fragments separate according to of a DNA molecule. When it is exposed to ultraviolet
their size through sieving effect provided by matrix. light, ethidium bromide fluoresces. Thus, this
11. (d) In gel electrophoresis, agarose extracted from sea chemical provides both a means of tagging DNA
weed is used as gel agarose which is made up of molecules and a means of visualizing them.
0.7% gel which shows good resolution of large DNA 21. (c) For transformation with recombinant DNA, the
and 2% gel will show good resolution of small bacterial cells must first be made competent which
fragments. means increasing the efficiency with which the DNA
12. (a) Selectable markers help in identifying and eliminating enters the bacterium.
non transformants and selectively permitting the 22. (d) For gene transfer into the host cell without using
growth of the transformants. Antibiotics are powerful vector microparticles made of tungsten and gold
medicines that fight bacterial infections. They either coated with foregin DNA are bombarded into target
kill bacteria or keep them from reproducing. In genetic cells at a very high velocity. This method is called
engineering, the antibiotics are used as selectable biolistics or gene gun which is suitable for plants.
markers. 23. (b) The enzyme used in PCR is Taq polymerase. Taq
13. (d) During heat shock to the bacterium, the temperature polymerase (isolated from bacterium thermus
used for giving thermal shock is 42º C. This enables aquaticus) which remains active during the high
the bacteria to take up the recombinant DNA. temperature, usually amplifies DNA segments of upto
14. (c) Chitinase is an enzyme that cleaves the glycosidic 2 kb.
bonds in chitin, thereby breaking down the structural 24. (a) PCR is a technique used to amplify a small amount of
polysaccharide component of the hard outer DNA without using a living organism, such as E.coli
covering of many animals and of the cell wall of fungi. or yeast. It is followed in a sequence where denaturation,
15. (a) During isolation of DNA, addition of chilled ethanol primer annealing and primer extension occurs.
causes precipitation of purified DNA. This can be 25. (d) Agarose gel electrophoresis is employed to check
seen as collection of fine threads in the suspension. the progression of a restriction enzyme digestion, to
16. (b) A. tumefaciens (a pathogen of several dicot plants) quickly determine the yield and purity of a DNA
has natural ability to transfer T-DNA of their Ti (tumor isolation, and to fractionate DNA molecules
inducing) plasmids into plant genome upon infection according to size which then could be purified from
of cells at the wound site and cause an unorganised the agarose gel if necessary.
growth of a cell mass known as crown gall. The 26. (d) A stirred-tank bioreactors is usually cylindrical or
transformed host genome produces galls. Because with a curved base to facilitate the mixing of the
of this. A tumefaciens functions as natural genetic reaction contents. The stirrer facilitates even mixing
engineer of plants. and oxygen availability throughout the bioreactor.
17. (b) Alternatively air can be bubbled through the reactor.
18. (d) Retrovirus has the ability to transform normal cells 27. (c) Bioreactor is an apparatus in which a biological
into cancerous cells. Hence, it can used as a vector reaction or process is carried out, especially on an
for cloning desirable genes into animal cells. industrial scale. Conversion of organic waste such
19. (d) Cloning vector is a DNA molecule that carries foreign as compost or solid waste is a common application
DNA into a host cell, replicates inside a bacterial (or for bioreactors.
EBD_7209
Hints & Solutions S-185

28. (b) Downstream processing refers to the recovery and 38. (c) In order to force bacteria to take up the plasmid, the
purification of biosynthetic products, particularly bacterial cells must first be made ‘competent’ to take
pharmaceuticals. It is an essential step in the up DNA.
manufacture of pharmaceuticals such as antibiotics, 39. (b) Bacteriophage is used in constructing vectors for
hormones (e.g. insulin and human growth hormone), gene cloning. Ethidium bromide is used for staining
antibodies and vaccines; antibodies and enzymes DNA.
used in diagnostics; industrial enzymes; and natural 40. (a) A stirred – tank reactor is usually cylindrical with a
fragrance and flavour compounds. curved base to facilitate the mixing of the reactor
29. (c) Taq polymerase is a thermostable DNA polymerase contents. Small volume cultures cannot yield
named after the thermophilic bacterium Thermus appreciable quantities of products. To produce in
aquaticus from which it was originally isolated. large quantities, the development of bioreactors was
30. (c) When cut by the same restriction enzyme, the required, where large volumes (100 -1000 litres) of
resultant DNA fragments have the same kind of culture can be processed.
‘sticky-ends’ and, these can be joined together (end- 41. (c) Molecular probe is a group of atoms or molecules
to-end) using DNA ligases. which are used in molecular biology or chemistry to
31. (d) Restriction enzymes belong to a larger class of study the properties of other molecules or structures.
enzymes called nucleases. These are of two kinds– 42. (a) Transformation does not involve passive entry of
exonucleases and endonucleases. DNA molecules through permeable cell walls and
32. (d) Selectable markers have been developed which membranes. It does not occur 'naturally' in all species
differentiate recombinants from non-recombinants of bacteria, but only in those species possessing
on the basis of their ability to produce colour in the the enzymatic machinery involved in the active
presence of a chromogenic substrate. uptake and recombination processes. Even in these
33. (b) Selectable marker selectively permitting the growth species, all cells in a given population are not capable
of the transformants. of active uptake of DNA. Only competent cells,
34. (c) The tumor inducing (Ti) plasmid of Agrobacterium which possess a so called competence factor are
tumefaciens has now been modified into a cloning capable of serving as recipients in transformation.
vector which is no more pathogenic to the plants. 43. (a) Bacteria and yeast easily grow in culture medium
35. (a) Hind II always cut DNA molecules at a particular and multiply very fast so they are the best vectors
point by recognizing a specific sequence of six base for making the many copies of recombinant DNA,
pairs. and express character of the desired gene.
36. (d) Polymerase chain reaction (PCR) is a technique used 44. (d) All endonuleases do not cut DNA at specific sites.
to make numerous copies of a specific segment of They were discovered in bacteria, not viruses.
DNA quickly and accurately. This technique enables Restriction endonucleases is a type of endonucleases
investigators to obtain the large quantities of DNA that cleaves DNA at specific point.
that are required for various experiments and 45. (b) Restriction endonucleases cleaves DNA molecule
procedures in molecular biology, forensic analysis, within a specific restriction site. Thus, they are called
evolutionary biology, and medical diagnostics. In as molecular scissors. DNA fragments generated by
the pr ocedure, DNA polymerase had to be restriction endonucleases upon mixing join due to
replenished after every cycle due to unstability at free 5' phosphate and 3' OH group.
the high temperatures which are needed for 46. (a) Alternative markers have been developed that can
denaturation. This problem was solved in 1987 with differentiate recombinants from non-recombinants
the discovery of a heat-stable DNA polymerase based upon their ability to produce colour in presence
called Taq, an enzyme isolated from the thermophilic of a chromogenic substrate. The plasmid in the
bacterium Thermus aquaticus, which inhabits hot bacteria, lacking an insert produces blue coloured
springs. colonies, while those plasmids with an insert do not
37. (d) Agarose gel electrophoresis does not provide a produce any colour due to insertional inactivation
means of visualizing the DNA but that role is played of enzyme, b-galactosidase.
by two dyes: ethidium bromide and loading buffer. 47. (b) A : A plasmid is a small, circular, extra chromosomal
Ethidium bromide, known as an intercalating agent double-stranded DNA molecule that is distinct from a cell's
allows it to intercalate, or insert, between nitrogenous chromosomal DNA. It naturally exists in bacterial cells,
bases of a DNA molecule. When it is exposed to and also occurs in some eukaryotes.
ultraviolet light, ethidium bromide fluoresces. Thus, B : amp is a selectable marker and is essential for the
this chemical provides both a means of tagging DNA identification of bacteria containing recombinant plasmids.
molecules and a means of visualizing them. C : Ti plasmids (Ti or tumour inducing plasmid) of
S-186 Biology
Agrobacterium tumefaciens has been developed as a eg, MALAYALAM.
vehicle for introducing foreign genes into plants. 51. (a) A : A microinjection which is used to inject a liquid
D : Chitinase is an enzyme that cleaves the glycosidic substan ce at a microscopic or border lin e
bonds in chitin, thereby breaking down the macroscopic level with the help of glass micropipette
polysaccharide structural component of the hard outer to make cells competent
covering of many animals and of the cell wall of fungi. B : Taq DNA polymerase is a heat stable enzyme
48. (c) A : Recombinant technology is a series of procedures used in the polymerase chain reaction (PCR) to
that are used to join together (recombine) DNA amplify segments of DNA in the lab. It was
segments. A recombinant DNA molecule is discovered in the heat-loving bacterium Thermus
constructed from segments of two or more different aquaticus, and without it, DNA cannot by amplified.
DNA molecules. C : Ampicillin is an antibiotic. It is a semi-synthetic
B : Cloning vehicle is the central component of a form of penicillin used to treat infections of the
gene cloning experiment, which transports the gene urinary and respiratory tracts.
into the host cell and is responsible for its replication. D : Ethidium bromide is a molecule commonly used
Two types of DNA molecules that act as vectors are to visualize DNA in agarose gel electrophoresis
plasmids and bacteriophage (virus chromosomes). experiments. It both binds to DNA and fluoresces
C : Macromolecular separation occurs in under proper conditions. It is known as an
electrophoresis. Electrophoresis is a technique for intercalating agent and when it is exposed to
separating the components of a mixture of charged ultraviolet light, ethidium bromide fluoresces.
molecules (proteins, DNAs, or RNAs) in an electric 52. (a) A : PCR is a technique used to make numerous copies
field within a gel or other support. of a specific segment of DNA quickly and accurately.
D : DNA ligase is a specific type of enzyme that B : Bioreactor is an apparatus in which a biological
facilitates the joining of DNA strands together by reaction or process is carried out, especially on a
catalysing the formation of a phosphodiester bond. large industrial scale. Conversion of organic waste
49. (a) A : EcoRI is an endonuclease enzyme isolated from such as compost or solid waste is a common
strains of E. coli, and is part of the restriction application for bioreactors.
modification system. C : Gene gun is a device used for high-velocity
B : BamHI (from Bacillus amyloliquefaciens) is a injection of DNA-coated fragments of gold directly
type II restriction endonuclease, having the capacity into host tissues, cells, or organelles which results
for recognizing short sequences of DNA and in the transformation of any cell. DNA shot into a
specifically cleaving them at a target site. cell can evoke an immune response protective against
C : Hind III is a type II site-specific deoxyribonuclease an antigen or organism of interest.
restriction enzyme isolated from Haemophilus D : EcoRI is an endonuclease enzyme isolated from
influenzae that cleaves the DNA palindromic the strains of E. coli, and is part of the restriction
sequence AAGCTT in the presence of the cofactor modification system. In molecular biology it is used
Mg2+ via hydrolysis. as a restriction enzyme.
D : pBR 322 is a plasmid and was one of the first 53. (d)
widely used E. coli cloning vectors. 54. (b) Agrobacterium tumefaciens, a pathogen of several
50. (a) A : Hind III is a type II site-specific deoxyribonuclease dicot plants is able to deliver a piece of DNA known
restriction enzyme isolated from Haemophilus as ‘T-DNA’ to transform normal plant cells into a
influenzae. tumor and direct these tumor cells to produce the
B : Transposons (also called jumping gene) is a chemicals required by the pathogen.
chromosomal segment that can undergo 55. (d) A-Eco RI; B-Bam HI; C-ampR; D-ori. The given figure
transposition, especially a segment of bacterial DNA represents a physical map of plasmid pBR322. It is
that can be translocated as a whole between the first artificial and most widely used cloning
chromosomal, phage, and plasmid DNA in the vector. pBR322 is constructed from the plasmids of
absence of a complementary sequence in the host E.coli, pBR 318 and pBR 320. It contains origin of
DNA. replication (Ori), antibiotic resistance genes e.g.,
C : Bacteriophage is a virus which parasitizes a ampicillin (amp r ) and tetracycline (tet r), and unique
bacterium by infecting it and reproducing inside it. recognition sites for 20 restriction endonucleases.
D : Palindrome is a segment of double-stranded DNA Hind III, Eco R I, Bam H, Sal I, Pvu II, Pst I, Cla I are
in which the nucleotide sequence of one strand reads some restriction sites. Rop codes for the proteins
in reverse order to that of the complementary strand, involved in the replication of the plasmid.
EBD_7209
Hints & Solutions S-187

56. (d) The given figure shows a recombinant DNA C – Extension or elongation of the strands using the
technology where the vector DNA (e.g., plasmid DNA primer with heat-stable DNA polymerases, most
DNA) and alien (foreign) DNA carrying gene of frequently Taq (Thermus aquaticus) polymerase at
interest are cut by the same restriction endonuclease 72ºC.
to produce complementary sticky ends. With the help 61. (c) Viruses that infect bacterial cells are called
of DNA ligase enzyme, the complementary sticky bacteriophages. Often, a viral infection results in the
ends of the two DNAs are joined to produce a death of the host. A major protective strategy for the
recombinant DNA (rDNA). This rDNA is inserted host is to use restriction endonucleases (restriction
into host bacterium by transformation using cold enzymes) to degrade the viral DNA on its
CaCl2 solution. introduction into a cell. These enzymes recognize
57. (a) In the given figure of simple stirred tank bioreactor, particular base sequences called recognition
the part marked as A, B, C and D are respectively sequences or recognition sites, in their target DNA
motor, pH control, foam braker and sterile air. A stirred and cleave that DNA at defined positions.
tank reactor is usually cylindrical with a curved base 62. (d) The fact that DNA is structured the same way in all
to facilitate the mixing of the reactor contents. The known organisms means that similar methods can
stirrer facilitates even mixing and oxygen availability be used to study the hereditary material.
throughout the bioreactor. 63. (d) Although cloned genes may be used as part of, or in
58. (c) The given apparatus shows sparged stirred tank association with, the other choices, the definition of
bioreactor. This apparatus carries out the a cloned gene is one that has been isolated and
fermentation process by transferring oxygen to mass repeatedly duplicated.
bioreactors. The rate of oxygen mass transfer into a 64. (d) Genetic engineering is the science of gene
culture in a bioreactor is affected by operational manipulation. Genetic engineering, genetic
conditions and geometrical parameters as well as the modification (GM) and gene splicing are terms for
physicochemical properties of th e medium the process of manipulating genes, generally
(containing nutrients, substances excreted by the implying that the process is outside the organism’s
micro-organism, and surface active agents that are natural reproductive process. It involves the
often added to the medium) and the presence of the isolation, manipulation, transfer, and reintroduction
micro-organism. Thus, oxygen mass transfer of DNA into cells or model organisms usually to
coefficient values in fermentation broths often differ express a protein. The aim is to introduce new
substantially from values estimated for simple characteristics or attributes physiologically or
aqueous solutions. physically, such as making a crop resistant to a
59. (a) The given apparatus shows the technique of herbicide, introducing a novel trait, or producing a
electrophoresis. Electrophoresis is the migration of new protein or enzyme, along with altering the
electrically charged molecules through a fluid or gel organism to produce more of certain traits.
under th e influence of an electric field. 65. (c) Gel electrophoresis is the standard lab procedure for
Electrophoresis is used especially to separate separating DNA by size (e.g., length in base pairs)
combinations of compounds, such as fragments of for visualization and purification.
DNA, for the purpose of studying their components. 66. (c) Ligases are used to connect short DNA fragments
60. (c) PCR is a technique for enzymatically replicating DNA to form longer segments, a process essential for
without using a living organism such as E. coli or inserting DNA segments into vectors.
Yeast. It is commonly used in medical and biological 67. (a) Migration through the electrophoresis gel is a
research labs for a variety of tasks like detection of function of the size of the DNA fragments, with small
hereditary diseases, identification of genetic fragments moving farthest as they are able to
fingerprints etc. “squeeze” through the gel matrix more easily.
The correct steps shown in the given figure are: 68. (b) PCR is a way to increase the numbers of a particular
A – Denaturation at a temperature of about 94° to sequence of DNA (or an entire gene). This technique
98°C. During the denaturation, the double strand is most useful when cloning DNA since prior
melts open to single stranded DNA, and all enzymatic knowledge of the DNA sequence must first be
reactions stop. obtained.
B – Annealing (binding of DNA primer to the 69. (b) Gene gun or biolistic is a method in which cells are
separated strands occurs at 50° to 65°C, which is bombarded with high velocity microparticles of gold
lower than the optimal temperature of the DNA or tungsten coated with DNA. Besides the gene gun,
polymerase). other methods of genetic transfer between cells or
S-188 Biology
organisms include the use of vectors such as viruses synthesis, isolation, modification, combination,
and plasmids. addition and repair of the genetic material (DNA) to
70. (d) Polymerase chain reaction (PCR) is a technique of alter the phenotype of the host organism to suit
synthesizing multiple copies of the desired gene (or human needs.
DNA) in vitro. Three techniques of genetic engineering are:
At the start of PCR, the DNA from which a segment (i) rDNA (recombinant DNA technology)
is to be amplified, an excess of the two primer (ii) gene cloning/gene amplification
molecules, the four deoxynucleoside triphosphates (iii) gene therapy
and the DNA polymerase are mixed together in 78. (d) Restriction endonuclease is an enzyme produced
reaction mixture that has appropriate quantities of chiefly by certain bacteria that has the property of
Mg2+. The PCR operation is followed in a sequence cleaving DNA molecules at or near a specific
where denaturation, primer extention occurs. sequence of bases. It hydrolyses polynucleotide
71. (d) Biolistic (gene gun) is direct gene transferred method from a phosphodiester bond within a specific
for constructing recombinant DNA. The gene gun sequence.
was invented by John C. Sanford with Edward Wolf. 79. (a) Restriction modification (R-M) systems of bacteria
A gene gun can be used to genetically infect cells or exist to protect bacteria from invading foreign DNA.
whole organisms with foreign DNA by aiming the This system is universal and is an important
barrel of the gun and firing. The microshot projectiles component of prokaryotic defense mechanisms
in the biolistic gene gun are made of microscopic (or against invading genomes. They occur in a wide
nano) sized gold or platinum powders. These variety of unicellular organisms, including eubacteria
expensive powders are soaked in DNA or RNA (in and archaea and comprise two contrasting enzymatic
raw or plasmid form) that are engineered for insertion activities: a restriction endonuclease (REase) and a
into the genome of the cells or organisms under the methyltransferase (MTase).
gun.
80. (b) Agrobacterium tumefacieans is a gram negative
72. (a) The linking of antibiotic resistance gene with the bacteria which is well known for its ability to transfer
plasmid vector became possible with DNA ligase. DNA between itself and plant, and for this reason it
DNA ligase is an enzyme that is able to join together has become an important tool for genetic engineering.
two cut portions of DNA and therefore plays an
81. (a) DNA ligase is an enzyme which can join the ends of
important role in DNA repair. DNA ligase is also used
two DNA chains by catalysing the synthesis of
in recombinant DNA technology as it ensures that
phosphodiester bond between 3¢ —OH at the one
the foreign DNA is bound to the plasmid into which
end of chain, and a 5¢ —phosphate group at the end
it is incorporated.
of the other. Normally, joining of two molecules by
73. (a) Plasmid is a genetic structure in a cell that can the action of ligase is coupled with the breakdown
replicate independently of the chromosomes. It is of a phosphate bond as the formation of new
typically a small circular DNA strand in the cytoplasm phosphodiester bond requires energy.
of a bacterium or protozoan. Plasmids are much used
82. (b) Agarose gel electrophoresis provides a means of
in the laboratory manipulation of genes.
analysing DNA by separating molecules based on their
74. (b) Before placing DNA into the electrophoretic chamber size. Two dyes: ethidium bromide and loading buffer
it must be cut by restriction endonuclease. are used to provide a means of visualizing the DNA.
75. (b) Retrovirus is a virus composed of RNA. Retroviruses 83. (a) Restriction endonucleases are enzymes that makes
have an enzyme, called reverse transcriptase that cuts at specific positions within the DNA molecule.
gives them the unique property of transcribing their They acts as molecular scissors. They recognize
RNA into DNA after entering a cell. The retroviral specific base sequence at palindrome sites in DNA
DNA can then integrate into the chromosomal DNA duplex and cut its strands.
of the host cell, to be expressed there. Most
84. (b) An esterase enzyme cleaves ester bonds. The
retroviruses can cause cancer also.
restriction enzyme cleaves sugar phosphate bonds in
76. (b) Vector is an organism that does not cause disease itself DNA. The lipase enzyme breaks down fats. The ligase
but which spreads infection by conveying pathogens enzyme reforms sugar phosphate bonds after annealing.
from one host to another. Species of mosquito, for
85. (d) Alternative selectable markers have been developed
example, serve as vectors for the deadly disease malaria.
which differentiate recombinant from non-
The four major types of vectors are plasmids, viral
recombinants on the basis of their ability to produce
vectors, cosmids, and artificial chromosomes.
colour in the presence of chromogenic substrate. In
77. (b) Genetic engineering is the manipulation of genes by this , a recombinant DNA is inserted within the coding
man in vitro. Genetic engineering refers to artificial
EBD_7209
Hints & Solutions S-189

sequence of an enzyme b-galactosidase. This results established. The Cry proteins exist as inactive
into inactivation of the enzyme, which is referred to protoxins and get converted into active toxin when
as insertional inactivation. The presence of ingested by the insect, as the alkaline pH of gut
chromogenic substrate gives blue coloured colonies solubilises the crystals.
of the plasmid in the bacteria which does not have 6. (d) Cry genes encoding the protein (Bt protein) are
an insert. Presence of insert results in insertional isolated from the bacterium and incorporated into
inactivation of the galactosidase and the colonies several crop plants like cotton, tomato, corn, rice,
do not produce any colour and these are identified soyabean, etc. The proteins encoded by the
as recombinant colonies. following cry genes control the pest given against
86. (b) Genetic engineering is scientific alteration of the them:
structure of genetic material in a living organism. Genetic - cry I Ac and cry II Ab control cotton bollworms.
engineering is possible because restriction - cry I Ab controls corn borer.
endonuclease purified from the bacteria can be used in - cry III Ab controls Colorado potato beetle.
vitro. It involves the production and use of recombinant - cry III Bb controls corn rootworm.
DNA and has been employed to create bacteria that 7. (d) Bt toxins is harmful to insects like lepidopteron
synthesize insulin and other human proteins. (tobacco budworm, army worm), coleopterans
Chapter 34 : Biotechnology and its (beetles) and dipterans (flies and mosquito) because
they kill larvae of certain insects by binding the
Applications
activated toxins on midgut epithelial cells, creating
1. (c) Golden rice is vitamin A rich variety developed by pores which causes swelling and lysis of the cells
rDNA technology and used in the treatment of vitamin leading to the death of the insect larva.
A deficiency. 8. (c) RNA interference (RNAi) technique has been
2. (d) Bt cotton, a transgenic crop variety has been developed to protect the plants from nematode which
introduced in India. The Bt cotton variety contains a is silenced by dsRNA produced by the host plant. It
foreign gene obtained from Bacillus thuringiensis. is a biological process in which RNA molecules
This bacterial gene protects cotton from the ball inhibit gene expression, typically by causing the
worm, a major pest of cotton. destruction of specific mRNA molecules.
3. (a) The genetically modified brinjal in India has been 9. (a) Several nematodes parasitise a wide variety of plants
developed for insect resistance. Bt-brinjal is a & animals including human beings. A nematode
transgenic brinjal that is developed by inserting a Meloidogyne incognitia infects the roots of tobacco
crystal gene from the Bacillus thuringiensis into the plants & causes a great reduction in yield. In RNA
brinjal’s genome. This process of insertion is interference technique, sense & antisense RNA fuse
accomplished using Agrobacterium mediated to form dsRNA that silents the expression of m-RNA
recombination. of nematode. RNA interference is a novel strategy
4. (a) Bacillus thuringiensis (or Bt) is a gram-positive, soil- adopted to prevent infestation of nematode
dwelling bacterium and commonly used as a Meloidogyne incognitia in roots of tobacco plants.
biological pesticide. It also occurs naturally in the 10. (c) Insulin is made up of 51 amino acids arranged in two
gut of caterpillars of various types of moths and polypeptide chains, A having 21 amino acids and B
butterflies, as well on leaf surfaces, aquatic with 30 amino acids that are linked together by
environments, animal faeces, insect-rich disulphide bridges. In mammals, including humans,
environments, and flour mills and grain-storage insulin is synthesized as a pro-hormone (like a pro-
facilities. Bacillus thuringiensis forms protein crystal enzyme, the pro-hormone also needs to be processed
(which contains insecticidal protein) during a before it becomes a fully mature and functional
particular phase of their growth. Bt toxin kills larvae hormone) which contains an extra stretch called the
of certain insects by binding the activated toxin on C peptide. This C peptide is not present in the mature
mid gut epithelial cells, creating pores which causes insulin and is removed during maturation into insulin.
swelling and lysis of the cells leading to the death of The main challenge for production of insulin using
the insect larva. rDNA techniques was getting the insulin assembled
5. (a) Cry protein is a large family of crystalline toxins into a mature form.
produced from soil bacterium, Bacillus 11. (c) Recombinant DNA technology is the joining
thuringiensis. These proteins are harmless to together of DNA molecules from two different species
vertebrates, but they are highly toxic to insects and that are inserted into a host organism to produce
nematodes. Their value in controlling insects that new genetic combinations that are of significance to
destroy crops and transmit human diseases is well science, medicine, agriculture, and industry. Insulin
S-190 Biology
was the first human drug made using recombinant genome.
DNA technology. In 1983, Eli lilly, an American 19. (d) Transgenic animals play a number of critical roles in
company, prepared two DNA sequences coding for drug discovery and development. Importantly, they
chains A and B of human insulin and introduced enable scientists to study the function of specific
them into plasmids of Escherichia coli to produce genes at the level of the whole organism which has
insulin. Recombinant insulin is synthesized by enhanced the study of physiology and disease
inserting the human insulin gene into E. coli, or yeast biology and facilitated the identification of new drug
(Saccharomyces cerevisiae) which then produces targets. Mice are being used as models, for example,
insulin for human use. to study obesity, heart disease, diabetes, arthritis,
12. (d) E. coli is used in production of interferon. Interferon cancer, cystic fibrosa, substance abuse, anxiety,
is a protein released by animal cells, usually in ageing, Alzheimer's disease and Parkinson's disease.
response to the entry of a virus, which has the They are also used to study different forms of cancer.
property of inhibiting virus replication. 20. (d) Emphysema is a condition in which the air sacs of
13. (a) Gene therapy is an experimental technique that uses the lungs are damaged and enlarged, causing
genes to treat or prevent disease. The first clinical breathlessness. The protein alpha - 1 antitrypsin is
gene therapy was given for treating adenosine used to treat the emphysema. Alpha - 1 antitrypsin is
deaminase deficiency. A four-year old girl became a protein made by cells in the liver and passes out
the first gene therapy patient on September 14, 1990 into the bloodstream and can travel to the lungs. Its
at the NIH Clinical Center. Adenosine deaminase main function is to protect the lungs from damage
deficiency, also called ADA deficiency or ADA-SCID caused by other types of proteins.
is an autosomal recessive metabolic disorder that 21. (b) Animals that have their DNA manipulated to possess
causes immunodeficiency. ADA deficiency is due to and express an extra (foreign) gene are known as
a lack of the enzyme adenosine deaminase. transgenic animals. Transgenic mice, rabbits, pigs,
14. (c) Adenosine deaminase deficiency (also called ADA sheep, cows and fish have been produced, although
deficiency or ADA-SCID) is an autosomal recessive over 95 per cent of all existing transgenic animals are
metabolic disorder that damages the immune system mice.
and causes severe combined immunodeficiency 22. (c) The Indian Government has set up organizations
(SCID). The main symptoms of ADA deficiency are such as GEAC (Genetic Engineering Approval
pneumonia, chronic diarrhoea, and widespread skin Committee), which will make decisions regarding the
rashes. ADA deficiency can be cured by bone validity of GM research and the safety of introducing
marrow transplantation and enzyme replacement GM organisms for public services.
therapy. 23. (c) Around 200000 varieties of rice has been estimated
15. (b) ADA is a genetic (inherited) condition that results to be present in India.
in an immune deficiency disorder called severe 24. (c) Basmati rice was patented by a US company even
combined immunodeficiency disease. The main site though the highest number of varieties of this rice is
of production of ADA in the body is lymphocytes [a found in India.
form of small leucocyte (white blood cell) with a 25. (d) Biopiracy is the practice of commercially exploiting
single round nucleus, occurring especially in the naturally occurring biochemical or genetic material,
lymphatic system and which plays a large role in especially by obtaining patents that restrict its future
defending the body against disease]. use, while failing to pay fair compensation to the
16. (c) A hybridization probe is a fragment of DNA of community from which it originates. Biopiracy is
variable length which is used in DNA samples to related to bioresearches, traditional knowledge and
detect the presence of nucleotide sequence (the DNA biomolecules and genes discovered.
target) that are complementary to the sequence in 26. (b) Refer to answer 25.
the probe. The probe hybridize to single–stranded 27. (b) The Indian Parliament has recently cleared the second
DNA whose base sequence allow probe target base- amendment of the Indian Patents Bill, that takes such
pairing due to complementary between the probe issues into consideration, including patent terms
and target. emergency provisions and research and development
17. (b) DNA or RNA segment tagged with a radioactive initiative.
molecule is called probe. They are used to detect the 28. (d) About Bt toxin, it is true, that the inactive protoxin
presence of complementary sequences in nucleic acid gets converted into active form in the insect gut due
samples. Probes are used for identification and to the alkaline pH of the gut which solubilises the
isolation of DNA and RNA. crystals. There are several advantages in expressing
18. (b) Transgenic animals are those animals which have Bt toxins in transgenic Bt crops. The level of toxin
had a foreign gene intentionally inserted into their expression can be very high, thus delivering sufficient
EBD_7209
Hints & Solutions S-191

dosage to the pest. affect the way in which the insulin works inside the
The toxin expression is contained within the plant human body. Pork insulin is structurally closer to
system and hence only those insects that feed on human insulin than is beef insulin. These days, animal
the crop perish. The toxin expression can be insulins are made from highly purified pancreas
modulated by using tissue-specific promoters and extracts and are marketed as 'natural' insulins.
replaces the use of synthetic pesticides in the 40. (a) All the statements regarding transgenic animals are
environment. correct.
29. (b) Insulin chain A and B are produced separately in 41. (a) All the given steps are performed for gene therapy
plasmid of E. coli, extracted and combined by creating in the treatment for defective ADA. Adenosine
disulfide bond to make it human insulin (active form) deaminase deficiency (also called ADA deficiency
called humulin. or ADA-SCID) is an autosomal recessive metabolic
30. (d) All the statements regarding GEAC are correct. The disorder that that damages the immune system and
Genetic Engineering Approval Committee (GEAC) causes severe combined immunodeficiency (SCID).
permitted certain companies as well as the research ADA deficiency can be cured by bone marrow
institutes for conducting the field trials of 5 GM transplantation and enzyme replacement therapy.
(genetically modified) crops. The trials for 42. (b) Gene therapy is the introduction of normal genes
development of genetically modified cotton, maize, into cells in place of missing or defective ones in
castor, wheat and rice were permitted. order to correct genetic disorders. The correct
31. (c) Statement (a) and (b) are correct. (d) Proteins encoded sequence of the steps in gene therapy is: (iv), (iii),
by cry 1 Ac and cry 1 Ab controls cotton bollworms (ii), (i).
and cry1Ab controls corn borer. 43. (b) Hirudin protein prevents blood clotting. The gene
32. (c) Hirudin extracted from leeches was the first that codes for this protein has been synthesized
anticoagulant used in humans in 1905. Its gene was chemically and then transferred to Brassica napus
chemically synthesized and was transferred into for it to accumulate in the seeds.
Brassica napus where hirudin accumulates in seeds. 44. (b) Production of transgenic plants is an application of
The hirudin is extracted and purified and used as plant tissue culture in various fields of biology. An
medicine. Today hirudins ar e produced as organism that contains genes, transferred through
recombinant proteins based on the leech genetic organism is a transgenic organism.
an ticoagulant pr otein sequence. It prevents 45. (a) Flavr Savr Tomato remains fresh by blocking the
coagulation by acting as an antithrombin. protein degrading enzyme, polygalactouronase.
33. (d) All the statements are correct. 46. (a) Enzyme Linked Immuno Sorbent Assay is used to
34. (d) All the statements are correct. detect diseases such as AIDS. Presence of pathogen
35. (c) Statement (c) is correct. The first step in PCR usually causes secretion of antibodies by the host
(polymerase chain reaction) is heat which is used to to act against the pathogen which is detected by
separate both the strands of target DNA. PCR is a ELISA.
laboratory technique used to make multiple copies 47. (a) The GEAC set up by the Government of India makes
of a segment of DNA. PCR is very precise and can discussions upon the validity of GM research and
be used to amplify, or copy, a specific DNA target assesses the associated safety and risks of
from a mixture of DNA molecules. introduction of GM organisms for public services.
36. (c) B thuringiensis forms protein crystals during a 48. (c) A: E. coli is used in production of interferon.
particular phase of their growth. Interferon is a protein released by animal cells,
37. (d) Transgenic Rosie is actually cow. Restriction usually in response to the entry of a virus, which
enzymes cut the DNA at specific sites. has the property of inhibiting virus replication.
38. (b) Statement (ii) and (iii) are correct. B: Bacillus thuringiensis is a bacterium that
39. (c) Statement (iii) is incorrect. Insulin is a hormone made produces proteins which are toxic to insects.
by the beta cells in the pancreas. Animal insulin is C: Rhizobium meliloti is a soil bacterium that forms
not identical to human insulin. Animal insulin was nitrogen-fixing nodules on the roots of certain
extracted from the pancreases of cattle and pigs. The genera of leguminous plants. The nif genes are
sequence of amino acids (the building blocks that genes encoding enzymes involved in the
make up the protein) is slightly different in insulins fixation of atmospheric nitrogen into a form of
from the different species. Compared to human nitrogen available to living organisms. The nif
insulin, porcine (pork) insulin has one different genes are found in both free-living nitrogen-
amino acid and bovine (beef) insulin three different fixing bacteria and in symbiotic bacteria
amino acids. These very slight differences do not associated with various plants.
S-192 Biology
D: Agrobacterium tumefaciens is a small motile C: RNAi is a biological process in which RNA
bacterial rod that can reduce nitrates and cause molecules inhibit gene expression, typically by
galls on plant stems (called crown gall disease causing the destruction of specific mRNA
of plants). Agrobacterium potentially might be molecules. RNA interference (RNAi) technique
a very useful vector for introducing any desired has been devised to protect the plants from
DNA into plants. nematode is silenced by dsRNA produced by
49. (a) A: GMO (genetically modified organism) is any the host plant.
organism whose genetic material has been D: Rosie is the first transgenic cow (genetically
altered using genetic engineering techniques modified cow). Rosie's milk contains the human
(recombinant DNA). gene alpha-lactalbumin into their DNA. The
B: Flavr - Savr tomato is a genetically-modified extra gene may come from the same species or
tomato which was the first genetically from a different species.
engineered whole food to be granted a licence 52. (d) A: Forensic science is the scientific method of
for human consumption. It has increased shelf gathering and examining information about the
life. past which is then used in a court of law. It is
C: Bio piracy is related to bioresearches, traditional related to DNA fingerprinting.
knowledge and biomolecules an d genes B: ELISA (Enzyme-linked immunosorbent
discovered. assay),used in AIDS, is a rapid immunochemical
D: E coli are used in the production of insulin. test that involves an enzyme used for measuring
Recombinant insulin is synthesized by inserting a wide variety of tests of body fluids. ELISA
the human insulin gene into E. coli, or yeast tests detect substances that have antigenic
(Saccharomyces cerevisiae) which then properties, primarily proteins rather than small
produces insulin for human use. molecules and ions, such as glucose and
50. (b) A: Gene therapy is the first clinical therapy which potassium.
is given to a 4 year old girl with adenosine C: Probe is a radioactive DNA/RNA.
deaminase deficiency (caused due to deletion D: Alpha 1 antitrypsin is used to treat emphysema
of the gene for adenosine deaminase). (a condition in which the air sacs of the lungs
B: Biofertilizer is a substance which contains living are damaged and enlarged, causin g
microorganisms which, promotes growth by breathlessness). Alpha - 1 antitrypsin is a
increasing the supply or availability of primary protein made by cells in the liver and protect
nutrients to the host plant. Bio-fertilizers such the lungs from damage caused by other types
as Rhizobium, Azotobacter, Azospirilium and of proteins called enzymes.
blue green algae (BGA) have been in use a long 53. (a) The given figure shows the maturation of pro- insulin
time. into insulin. The parts marked a s A, B, C and D are
C: Bt cotton is a genetically modified variety of respectively pro-insulin, cell peptideres, insulin and
cotton producing an insecticide. Bt cotton was free C- peptide.
created through the addition of genes encoding 54. (a) Transgenic plants are the ones generated by
toxin crystals in the Cry group of endotoxin. introducing foreign DNA into a cell and regenerating
When insects attack and eat the cotton plant a plant from that cell.
the Cry toxins are dissolved due to the high pH 55. (a) Transgenic animals have been successfully used for
level of the insects stomach. producing transgenic mice through genetic
D: Humulin is used for a preparation of insulin engineering. Many transgenic mice are designed to
produced by genetic engineerin g an d increase our understanding of how genes contribute
structurally identical to insulin made by the to development of diseases. Transgenic mice are
human pancreas. It is used to treat diabetes. being developed for use in testing the safety of
51. (a) A: Golden rice is genetically modified rice that has vaccine before they are used in human beings. For
been engineered to have high levels of beta example, transgenic mice are being used to test the
carotene in it. It is a pre cursor of vitamin A, safety of polio vaccine.
which gives it a characteristic golden colour. 56. (b) In this technique, nematode specific genes are
B: Bt toxin is a pesticidal toxins (e.g., CryAb1) introduced in the host plant in such a way that it
produced by the soil bacterium Bacillus produces both sense and antisense RNA. The two
thuringiensis, which are lethal to corn earworms, RNA’s being complementary to each other formed a
Colorado potato bugs and others. double stranded RNA (dsRNA) that initiated RNA
interference (RNA i). This (dsRNA) bind to and
EBD_7209
Hints & Solutions S-193

prevent translation of specific mRNA of nematode production of antibiotics, hormones, etc.


(gene silencing). Thus, transgenic plants based on 65. (a) Bt toxin genes were isolated from Bacillus
RNAi technology are resistant to nematode. thuringiensis and incorporated into several crop
57. (a) Recombinant DNA technology is the process of plants such as cotton. The choice of genes depends
joining together two DNA molecules from two upon the crop and the targeted pest, as most of Bt
different species that are inserted into a host toxins are insect group specific. The toxin is coded
organism to produce new genetic combination. by gene named cry. Two cry genes - cry I Ac and cry
58. (a) Bt cotton is being grown in India by the farmers. Bt II Ab have been incorporated in cotton. The proteins
cotton is pest resistant plant which decreases the encoded by genes cry II Ab and cry I Ac control
pesticides use. Bt toxin is produced by a bacterium cotton bollworms and that of cry I Ab controls
Bacillus thuringiensis (Bt for short). Bt toxin gene cornborer.
has been cloned from the bacteria and been expressed 66. (b) Bt toxin (obtained from Bacillus thuringiensis) is
in plants to provide resistance to insects without intracellular crystalline protein which are toxic to
the need for insecticides. insects. It is widely used as a biological pesticide.
59. (a) A transgenic food crop which may help in solving 67. (c) RNAi is a method of cellular defense in all
the problem of night blindness in developing eukaryotes. It is a system within living cells that
countries is golden rice. Golden rice is genetically helps to control the activity of specific genes. This
modified rice that has been engineered to have method involves silencing of mRNA due to
elevated levels of beta carotene in it. It is a pre cursor complementary double stranded RNA that prevents
of vitamin A, which gives it a characteristic golden translation of target gene or mRNA [silencing].
colour. Source of ds RNA is retrovirus (having RNA genome)
60. (c) In gene therapy of SCID, WBCs are extracted from or transposons (mobile genetic material).
bone marrow of patients and a good copy of human 68. (c) RNAi stands for RNA interference. It is a process
gene encoding ADA is introduced via retrovirus as within living cells that moderates the activity of their
vector. genes. It has an important role in defending cells
61. (d) The disorder ADA deficiency is caused due to against parasitic nucleotide sequences - viruses and
deletion of the gene for adenosine deaminase. ADA transposons but also in directing development as
deficiency can be cured by bone marrow well as gene expression.
transplantation or by enzyme replacement therapy, 69. (c) Human insulin (humulin) was the first genetically
in which functional ADA is given to the patient by engineered product produced by an American firm
injection. But both these approaches are not Eli-Lilly (5th July 1983). Insulin was earlier extracted
completely curative as the patient requires periodic from pancreas of slaughtered cattle and pigs. Such
infusion of such genetically engineered lymphocytes insulin however caused some patients to develop
or hormonal injections. However, if the gene isolated allergy and other type of reactions to the foreign
from marrow cells producing ADA is introduced into proteins.
cells at early embryonic stages, it could be a 70. (b) Gene therapy is the technique of genetic engineering
permanent cure. to replace a faulty gene by a normal healthy functional
62. (a) Human insulin has 53 amino acids in two gene. Gene therapy is being tried for sickle cell
polypeptides (A and B) connected by two S -S anaemia and severe combined immuno-deficiency
(disulphide) linkages. In 1983, American company (SCID).
Eli Lilly prepar ed two DNA sequences As a first step towards gene therapy, lymphocytes
corresponding to A and B insulin chains. When are extracted from the bone marrow of the patient
introduced in plasmids of E.coli, insulin chains were and are grown in a culture outside the body. A
formed. They were extracted and fused to produce functional ADA cDNA (using a retroviral vector) is
humulin (human insulin). then introduced into these lymphocytes, which are
63. (d) Main objective of production/use of herbicide resistant reinjected to the patient’s bone marrow. But as these
GM crops is to reduce herbicide accumulation in food cells do not always remain alive, the patient requires
articles for health safety. GM plants have been useful periodic infusion of such genetically engineered
in many ways. Genetic modifications has made crops more lymphocytes. However, if the isolated gene from bone
tolerant to abiotic stresses, reduced reliance on chemical marrow cells producing ADA is introduced into cells
pesticides and enhanced nutritional value of food. at early embryonic stages, it can be a permanent cure.
64. (d) Biotechnology is the exploitation of biological 71. (c) In order for gene therapy to be most effective, genes
processes for industrial and other purposes, especially should be inserted in stem cells because stem cells
the genetic manipulation of microorganisms (like have the ability to self-renew. For each organ in the
microbes, fungi, plants and animals) for the
S-194 Biology
mature body, there are specific stem cells that can most readily. Likewise, plants absorb soil water from the
make all the different kinds of cells in that organ. For larger pores first because it takes less energy to pull
example, in the blood system, hematopoietic ("blood- water from large pores than from small pores. Therefore
forming") stem cells (HSC) give rise to each of the water holding capacity is dependent on the soil
different types of blood cells such as red blood cells composition, grain size and aggregation.
(RBC), white blood cells (WBC) and platelets. 9. (a) The benthic community is made up of organisms
72. (b) The ELISA is a fundamental tool of clinical that live in and on the bottom of the ocean floor.
immunology, and is used as an initial screen for HIV These organisms are known as benthos. Benthos
delection. Based on the principle of antigen- include worms, clams, crabs, lobsters, sponges, and
antibody interaction, this test allows for easy other tiny organisms that live in the bottom sediments.
visualization of results. 10. (a) Homeostasis is the property of a system that regulate its
73. (d) Gene therapy is a rapidly growing field of medicine interval and tends to maintain a stable relatively constant
in which genes are introduced into the body to conditions of properties such as temperature or pH.
provide treatment for a particular disease. Genes 11. (d) Diapause is a period during which growth or
control heredity and provide the basic biological development is suspended and physiological activity
code for determining the specific function of a cell. is diminished, as in certain insects in response to
Gene therapy seeks to provide genes that correct adverse environmental conditions.
the disease-controlling functions of cells that are 12. (c) Aestivation is the cessation or slowing of metabolic
not doing their job. activity during the summer period to avoid problems
of heat and desiccation etc.
Chapter 35 : Organisms and Populations 13. (c) Adaptation is an outcome of natural selection. It is a
1. (d) Precipitation is any form of any water (such as rain, dew, process in which animal or plant species can adapt
snow, sleet or hail) formed by condensation of water to a certain environment by adjusting themselves in
vapour in the atmosphere and falls to the earth's surface. great varieties of ways to survive and reproduce in
2. (d) Temperature, water, light and soil are non - living that particular habitat.
abiotic factors which affect plants and animal species 14. (d) Blubber is the fatty layer present between the skin
in an environment. These factors play an important and muscle of whales and other cetaceans. It acts as
role in leading variations in the physical or chemical an insulator and protects the animal from heat loss
conditions of different habitats. and serves as a food reserve.
15. (b) Archaebacteria is a group of microorganisms (like
3. (a) Stenothermal organisms are those organisms which
methanogens and certain h alophiles and
are capable to live or survive within a limited range
thermacidophiles) which flourishes in hot springs
of temperature.
and deep sea hydrovents where temperatures
4. (b) Eurythermal organisms are those organisms which
exceed 100ºC.
are capable to live or survive within a wide range of
16. (d) Age pyramids are graphical representations that
temperature in the environment.
show the distribution of various age groups in a
5. (b) Sea water typically has a salinity of around 35 g/kg population which forms the shape of a pyramid of a
although lower values are typical near coasts where growing population. It determines the overall age
rivers enter the ocean. Rivers and lakes can have a distribution of a population, an indication of the
wide range of salinities, from less than 0-0.1 g/kg to reproductive capabilities and likelihood of the
a few g/kg, although there are many places where continuation of a species.
even higher salinities are found. The dead sea has a 17. (b) The age of pyramid with narrow base indicates a low
salinity of more than 200 g/kg. percentage of young individuals. If the birth rate is
6. (d) Stenohaline organisms are incapable to withstand drastically reduced, the pre-reproductive group
wide variation in salinity of the surrounding water. dwindles in proportion to the other two groups and
7. (c) Deep inside the oceans, the environment is it results in an urn-shaped pyramid, which indicates
continuously dark and its inhabitants are not aware of that population is dying off.
the existence of a celestial source of energy called sun. 18. (c) Population growth is the increase in the number of
8. (d) Water holding capacity is the amount of water held individuals in a population. Natality, immigration,
between field capacity and wilting point. Accessible water mortality and emigration are the processes which
is held in soil pores via forces which depend on the pore affect the density of population in a given habitat
size and the surface tension of water. The closer together during a given period of time. The former two
soil particles or aggregates are, the smaller the pores and processes contribute an increase in population
the stronger the force holding water in the soil. Because density whereas the latter two processes to a
the water in large pores is held with little force, it drains decrease.
EBD_7209
Hints & Solutions S-195

19. (d) Emigration may be defined as the movement of them to be hatched and the young raised by the
individuals from one place to another to establish hosts, regularly at the cost of the hosts' own young.
permanent or temporary habitation. Examples include cuckoos and cowbirds.
20. (c) A population growing in a habitat with limited 28. (c) Commensalism is a symbiotic association between
resources show initially a lag phase, followed by two organisms in which one benefits and the other
phases of acceleration and deceleration and finally derives neither benefit nor harm. Some of these
an asymptote, when the population density reaches include clownfish and sea anemones, fleas and dogs,
the carrying capacity. A plot of N in relation to time sharks and remoras, and epiphyte/orchid on mango
(t) results in a sigmoid curve. This type of curve is branch. Association between liver flukes and
called verhulst pearl logistic curve. tapeworms is a type of endosymbiosis in which one
symbiont lives within the body of another.
dN æK-Nö
= rN ç ÷ 29. (d) Interaction between two species, where both suffer
dt è K ø adverse effects is known as competition. Competition is
where, N = Population density at time ‘t’ the relationship in which each population adversely affects
r = intrinsic rate of natural increase the other in the struggle for resources short in supply.
K = carrying capacity Competition is of two types – interspecific and
21. (c) In growth pattern, environmental resistance (1 - N/ intraspecific.
K) is the action of limiting abiotic and biotic factors (a) Interspecific competition occurs between two
that prohibit the growth of a population as it would individuals of two different species occurring
grow according to its biotic potential. in a habitat.
22. (c) Interspecific interaction arise from the interaction of (b) Intraspecific competition occurs between
populations of two different species. They could be individuals of the same species.
beneficial, detrimental or neutral (neither harm nor 30. (d) Mutualism is a symbiotic association between two
benefit) to one of the species or both. species of organisms in which both the species
23. (c) Thorns of Acacia and cactus are the most common benefit and takes advantages from each other and
morphological means of defence. cannot survive without each other. Fig wasps are
24. (d) Secondary compounds produced by plants are toxic the sole pollinators of fig flowers/trees and in turn,
or repellent to herbivores and microbes, and help fig wasps can breed nowhere else but inside figs.
defend plants producing them. Production increases 31. (c) In amensalism, one species is inhibited by toxic
when a plant is attacked by herbivores or pathogens. secretion of another species. Inhibitor species is
Strychnine, caffeine and quinine are some secondary neither benefitted nor harmed.
metabolites which help them against grazers and 32. (b) A: Lichen represents an intimate mutualistic (i)
browsers. relationship between a fungus and
25. (b) Competitive release is the growth of the species cyanobacteria (ii).
range when a competitor for its niche is eliminated. It B: The mycorrhizae (iii) are the associations
usually occurs when one of two species competing between fungi and roots of higher plants.
for the same resource disappears, thereby allowing C: Plants needs the help of animals (iv) for pollinating
the remaining competitor to utilize the resource more their flowers and dispersing their seeds.
fully than it could in the presence of the first species. D: The wasp (v) pollinates the fig inflorescence
Joseph Connell's (1961) demonstrates competitive while searching for suitable egg laying sites.
release for study of competition for space between 33. (b)
the barnacle species Balanus and Chathamalus in 34. (c) Abiotic components are physical factors which affect
the intertidal zone on the rocky Scottish coast. the structure, behaviour and life history of organisms.
26. (a) Cuscuta is a total stem parasite which is a good These components (such as soil, moisture, range of
example of ectoparasitism. It is commonly found temperature, and availability of light) along with
growing on hedge plants. It has lost chlorophyll and biotic factors (such as the availability of food and
leaves in the course of evolution. It attaches and the presence of predators) can characterise the
wraps itself around the stem of host plant and habitat of an organism.
produces haustoria that gets inserted into the 35. (c) Abiotic factor (which include water, sunlight,
vascular system of host. The parasitic plant sucks oxygen, soil and temperature) are the non-living parts
all the nutrients from the host plant with the help of of the environment that can have a major influence
haustoria. Cuscuta is known to receive even the on living organisms. Temperature is strongly
flower inducing hormone or florigen from the host. influenced by sunlight and plays an important role
27. (c) Brood parasitism is a type of social parasitism in for animals that cannot regulate their own body
which eggs are laid in the nests of other birds, causing temperature. A few organisms can tolerate and
S-196 Biology
flourish in wide range of temperature and few temperature fairly constant by behavioural means.
organisms are limited to narrow range of temperature. They bask in the sun and absorb heat when their
And this level of thermal tolerance of different body temperature drops below the comfort zone, but
species determines their geographical distribution move into shade when the ambient temperature starts
to a large extent. Therefore, temperature is responsible increasing.
for all the given statements. 44. (d) The given examples show behavioural adaptations.
(i) Mango trees cannot grow in temperate Behavioural adaptation is the process by which an
countries like Canada and Germany. organism or a species changes its pattern of action
(ii) Tuna fish are rarely caught beyond tropical to better suit its environment.
latitude in the ocean. 45. (b) Expanding population is a population containing a
(iii) Snow leopard is not found in Kerala forest large proportion of young individuals. Pyramid
36. (b) Statement (i) and (iv) are incorrect. shaped age structure and urn shaped age structure
(i) Temperature progressively increases from pole to are the characteristics of expanding population.
equator. (iv) Temperature ranges from sub-zero levels 46. (d) Predator is an organism that exists by preying upon
in polar areas and high altitudes to >50ºC in tropical other organisms. Predator in nature is prudent (means
deserts in summer. sensible) because they do not exploit their prey.
37. (d) An overexposure to UV-B radiation can cause Such predator would maintain the prey population
sunburn and some form of skin cancer. In humans, at the density that gives the maximum rate of
prolonged exposure to solar UV-radiation may result production of new prey biomass.
in acute and chronic health effects on the skin, eye 47. (c) Head louse living on the human scalp as well as
and immune system. Moreover, UV-C can cause laying egg on human hair is categorised as a parasite
adverse effects that can variously be mutagenic or because they survive by living on human scalp by
carcinogenic. taking nourishment from there.
38. (d) Plants donot have mechanism of thermoregulation 48. (c) Parasites that feed on the external surface of the host
to maintain that internal temperatures. organism are called ectoparasites. The most familiar
39. (d) Small animals have a large surface area relative to examples of this group are the lice on humans and
their volume. They tend to lose body heat very fast ticks on dogs.
when it is cold outside; then they have to spend 49. (a) Statement (a) is incorrect. An overwhelming majority
much energy to generate body heat through (around 99 percent) of animals and nearly all plants
metabolism. This is the main reason why very small cannot maintain a constant internal temperature.
animals are rarely found in polar regions. Hibernation 50. (c) In sigmoid growth curve, finally, growth rate becomes
is a time when animals ‘sleep’ through cold weather. stable because mortality and natality rates become
The familiar case of bears going into hibernation equal to each other and finally the population shows
during winter is an example of escape in time. zero growth rate as birth rate equals death rate.
40. (c) Thermoregulation is energetically expensive for 51. (a) Cyclomorphosis, or cyclic change in morphology is
many organisms particularly true for small animals observed. The morphology is Dependent upon
like shrews and humming birds. During the course variation of temperature in water so, Daphnia shows
of evolution, the costs benefits of maintaining a different morphology in different seasons.
constant internal environment are taken into 52. (a) The interaction between predator and prey is
consideration. interspecific. Predator population usually depends
41. (a) Statements (i), (ii) and (iii) are correct. upon the number of prey which in turn is controlled
(iv) All adaptation are not genetically fixed in all by predators.
organisms. 53. (b) A group of individuals resembling each other in
(v) In Opuntia, the pathway of photosynthesis is morphological, physiological, biochemical and
through C4 cycle. behavioural characters constitute a species such
42. (b) Allen's rule states that the limbs, ears, and other individuals can breed among themselves but cannot
appendages of the animals living in cold climates breed with members other than their own to produce
tend to be shorter than in animals of the same species fertile offsprings. New species are formed mainly due
living in warm climates. Shorter and more compact to reproductive isolation.
body parts have less surface area than elongated 54. (a) Cold blooded organisms utilize their stored food at
ones and thus radiate less body heat. the time of hibernation and aestivation.
43. (d) Desert lizards lack the physiological ability that 55. (a)
mammals love to deal with the high temperature of 56. (a) Predation is a relation between two organisms in
their habitat but manage to keep their body which one organism captures and feeds on other.
EBD_7209
Hints & Solutions S-197

Commensalism is a relation between two organisms place and what kinds of organisms the biomes can
in which one benefits and the other derives neither sustain. Along with temperature and precipitation,
benefit nor harms. Parasitism is a relation between these are factors that distinguish one biome from
organisms in which one lives as a parasite on another another and influence the dominant types of
and harm the host. Competition is process in which vegetation and animals that have adapted to a
the fitness and survival ability of one species is biome’s unique characteristics.
significantly lower in the presence of another species. 67. (c) Many freshwater fishes cannot live for long in sea-
57. (d) water and vice-versa mainly because of osmosis.
58. (a) In the given figure of biome distribution with respect Freshwater fishes are adapted to reduce the amount
to annual temperature and precipitation, the parts of water reaching into their bodies. Freshwater fish
marked as A, B and C is respectively tropical forest, differ physiologically from salt water fish in several
temperate forest and coniferous forest. A tropical aspects. Their gills must be able to diffuse water
forest is found in areas with high average temperature while simultaneously keeping the salts of the bodily
and significant amount of rain fall. This forest fluids inside. The scales of the fish also play a part in
consists of a completely closed canopy of trees that the process; freshwater fish that have lost too many
prevents penetration of sunlight to the ground. The scales get a surplus of water diffused in through the
temperate forests are found in rather mild climatic skin, causing the fish to die.
area within the temperate zone that receives heavy 68. (d) Chemical composition of water and pH of water are
rainfall and usually includes numerous kinds of trees. the factors that are important for aquatic organisms.
Coniferous forest is a terrestrial biome found in 69. (d)
temperate regions of the world with warm summers 70. (c) pH, mineral composition and topography are the
and cool winters and adequate rainfall to sustain a important factors which determine to a large extent
forest. of vegetation in an area.
59. (c) In the given figure of organismic response, the types 71. (b) Regarding temperature and osmotic concentration
of organism in response to abiotic factors marked as nearly all plants are conformers. Conformers are those
A, B and C are respectively conformers, regulator organisms whose internal conditions are controlled
and partial regulator. primarily by environmental conditions.
60. (d) The given age pyramid represents the declining 72. (d) Very small animals are rarely found in polar regions
population of humans. It is an Urn shaped pyramid because they have a larger surface area relative to
with least number of pre-reproductive individuals. their volume.
Low birth rate, stable death rate and increased 73. (d) The Kangaroo rats of North American deserts do
immigrations can lead to declining population of not need to drink water because of the following
humans. reasons, like- they meet their water requirements
61. (d) ‘A’ is more recent and shows slight reduction in through internal oxidation fats when water is a
growth rate. byproduct, they are able to concentrate their urine
62. (a) Population density due to changes in the following thereby minimizing the loss of water from their body
basic process - A: Natality + Immigration; B: and also they do not have sweat glands so no
Mortality + Emigration perspiration occurs in the animals.
Natality is the proportion of births to the total 74. (c) At the high altitudes, the atmospheric pressure of
population in a place in a given time. Immigration is O2 will be too low so the solubility of oxygen in the
the number of individuals of the same species that blood will be very less hence the oxygen carried by
have come into the habitat from elsewhere during each RBC will be too less. But to fulfill the oxygen
the time period under consideration. Mortality is the requirement of the body blood has to carry more
number of deaths in the population during a given oxygen to the body tissue and this is done by the
period of time. Emigration is the number of individuals increased number of RBCs.
of the population who left the habitat and have gone 75. (c) Population ecology is the branch of ecology that
elsewhere during a given period of time. studies the structure and dynamics of populations.
63. (c) According to the given figure, I, II, III and IV (which Population ecology is an important area of ecology
affects the basic process of population density) are because it links ecology of population genetics and
respectively increase, increase, decrease and evolution.
decrease. 76. (d) Natural selection operates to evolve the desired traits
64. (b) 65. (c) at population level, because it is the organisms only
66. (d) Soil moisture, soil nutrients and length of growing which has to cope with a changed environment.
season affect what kinds of plants can grow in a Therefore, population ecology links ecology to
population genetics and evolution.
S-198 Biology
77. (b) Birth rate or natality rate is a measure of the extent to 84. (c) Carrying capacity of a population in any environment
which a population replenishes itself through births. is the maximum population size of that particular
species that the environment can tolerate
No. of births
Birth rate = indefinitely, given the food, habitat, water, and other
Total population necessities available in the environment.
8
85. (b) The intrinsic rate of increase is inversely related to
= = 0.4 offspring per lotus per year generation time, T. Therefore, organisms with very
20
high intrinsic growth rates have short generation time.
78. (b) Mortality or death rate refers to the death of 86. (c) In exponential growth, the increase or decrease in
individuals in a population. population size during a unit period is N × (b – d).
No. of deaths 87. (d) Exotic species when introduced into the environment
Death rate = where they are not native, they become invasive and
Total population
start spreading fast because the invaded land does
4 not have their natural predators.
= = 0.1 individuals per fruitfly per week
40 88. (d) Different feeding habit of finches does not lessen
79. (a) When in an age pyramid, the number of individuals the impact of predation.
of reproductive age is lesser than pre - reproductive 89. (b) This type of mechanism is known as resource
but higher than post - reproductive ones, then it partitioning. If two species compete for the same
shows that population is growing. resource, they could avoid competition by choosing,
80. (b) Age distribution is important, as it influences both, for instance, different times for feeding or different
natality and mortality of the population. From an foraging patterns.
ecological view point there are three major ecological 90. (b) The Abingdon tortoise in Galapagos Island became
ages (age groups) in any population. These are-pre- extinct within a decade after goats were introduced
reproductive, reproductive and post-reproductive. on the islands, apparently due to the greater browsing
The relative duration of these age groups in efficiency of the goats. The whole incidence shows
proportion to the life span varies greatly with different the process of competition among the species.
organisms. 91. (d) Gause's principle states that similar species cannot
81. (c) Population density will increase if the number of coexist for long in the same ecological niche because
births plus the number of immigrant (B + I) is more competing for the same critical resources within an
than the number of deaths plus the number of environment, one of them will eventually outcompete
emigrants (D + E), otherwise it will decrease. Births and displace the other.
and deaths are the most important factors influencing Chapter 36 : Ecosystem
population density.
82. (b) Natality and immigration are the two basic process 1. (c) Any unit which includes all the organisms interacting
which contribute to an increase in population with the physical environment so that a flow of energy
density. Natality is the proportion of births to the leads to clearly defined trophic structure, biotic
total population in a place in a given time. Immigration diversity & material cycle within the system is called
is the number of individuals of the same species that ecosystem.
have come into the habitat from elsewhere during 2. (d) Man - made ecosystems are the artificial ecosystems
the time period under consideration. which rely on the human efforts to sustain. They do
83. (a) The integral form of the exponential growth equation not possess a self - regulating mechanism and have
is Nt = N0 e–rt almost no diversity and have simple food webs. The
where, cycling of nutrients is negligible. The man made
Nt = Population density after time t. ecosystems include the villages, towns, cities, rivers,
N0 = Population density at time zero. orchids, dams, gardens, lakes, crop field, aquarium
r = intrinsic rate of natural increase. and agriculture.
e = the base of natural logarithms (2.71828). 3. (b) Stratification is the way in which plants of different
The equation describes the exponential or geometric species are arranged in different vertical layers in
growth pattern of a population and results in a J- order to make full use of the available physical and
shaped curve. The J-shaped curve of exponential physiological requirements.
growth is characteristic of some population that are 4. (b) Abiotic components (non living) include inorganic
introduced into a new or unfilled environment or substances or minerals, organic substances &
whose numbers have been drastically reduced by a different climatic conditions like temperature, pH,
catastrophic event and are rebounding. light etc. Decomposers are heterotrophic organisms,
EBD_7209
Hints & Solutions S-199

mainly fungi and bacteria, who meet their energy photosynthetically active radiation (PAR).
and nutrient requirements by degrading dead organic The light energy is converted into chemical energy
matter or detritus. in the form of sugar by photosynthesis.
5. (d) Ecosystem is a biological community of interacting 6H2O + 6CO2 + Light ® 6C6H12O6 + 6O2
organisms and their physical environment. It includes Plants utilize 2-10% of PAR in photosynthesis.
both biotic and abiotic components, their 14. (a) The flow of energy in an ecosystem is unidirectional.
interactions, and some source of energy. Biotic That is, it flows from the producer level to the
factors are the living parts of an ecosystem-the consumer level and never in the reverse direction.
animals, plants and microorganisms. Abiotic factors Hence, energy can be used only once in the
are non-living chemical and physical parts of the ecosystem. But the minerals circulate and recirculate
environment that affect living organisms and the many times in the ecosystem.
functioning of ecosystems. Abiotic factors can 15. (b) The standing crop is measured as the mass of living
include water, light, radiation, temperature, humidity, organisms (biomass) or the number in a unit area
atmosphere, soil, flow of energy and cycling of which is expressed in terms of fresh or dry weight. It
materials. indicates the productivity and luxuriance of growth.
6. (a) Species composition and stratification are the two 16. (a) Ecosystem is a system or a group of interconnected
main structural features of an ecosystem. Species elements, formed by the interaction of a community
composition is the identity of all the different of organisms with their environment such that energy
organisms that make up a community. Stratification is exchanged and system-level processes, such as
is the vertical distribution of different species the cycling of elements, develop. Ecosystem is open
occupying different levels. For e.g., tree occupy top because resources can move from one ecosystem to
vertical strata or layer of a forest, shrubs the second another. Ecosystems require a continuous flow of high-
and herbs and grasses occupy the bottom layers. quality energy to maintain their structure and function.
7. (c) Identification and enumeration of plants and animal For this reason, all ecosystems are open systems
species of an ecosystem gives its species requiring a net flow of energy to persist over time.
composition. 17. (b) The energy flow in an ecosystem is from the producer
8. (c) Primary productivity is associated with the level to the consumer level. At each trophic level 80
producers which are autotrophic, most of which are to 90% of energy is lost. Hence, the amount of energy
photosynthetic and to a much lesser extent the decreases from the producer level to the consumer
chemosynthetic micro-organisms. It is defined as the level.
rate of production of biomass or organic matter per Pyramid of energy is always upright because during
unit area over a time period by plants during the flow of energy from one trophic level to the next
photosynthesis. one, there always occurs a loss of energy.
9. (d) Gross primary productivity of an ecosystem is the 18. (d) Two types of nutrient cycles are – gaseous cycles
rate of production of organic matter during (nitrogen, oxygen, carbon cycles) and sedimentary
photosynthesis. cycles (phosphorus, sulphur cycles). In gaseous
10. (a) Primary productivity is the rate at which energy is cycle, the main reservoirs of chemicals are the
converted by photosynthetic and chemosynthetic atmosphere and ocean. In sedimentary cycles, the
autotrophs to organic substances. It is expressed in main reservoirs are soils and rocks. In sedimentary
terms of weight or energy. The total amount of cycles the nutrients do not enter the atmosphere.
productivity in a region or system is gross primary 19. (d) In sedimentary cycles of matter, materials involved
productivity. in circulation between biotic and abiotic components
11. (a) The rate of biomass production is called productivity. of biosphere are non-gaseous and the reservoir pool
Net primary productivity is the available biomass for is lithosphere, e.g., phosphorus, calcium, magnesium.
the consumption to heterotrophs (herbivores and Sulphur has both sedimentary and gaseous phases.
decomposers). The annual net primary productivity 20. (d) Statements (i) and (iii) are correct.
of the whole biosphere is approximately 170 billion (ii) Abiotic factors are non-living chemical and physical
tons (dry weight) of organic matter. parts of the environment that affect living organisms
12. (b) In the process of leaching, water soluble substances and the functioning of ecosystems. Abiotic factors
(formed as a result of decomposition) are leached or can include water, light, radiation, temperature,
go down to deeper layers of soil. humidity, atmosphere, and soil, flow of energy and
13. (d) The main source of energy for an ecosystem is the cycling of materials.
radiant energy or light energy derived from the sun. (iv) Energy flow is unidirectional.
50% of the total solar radiation that falls on earth is
S-200 Biology
21. (a) Decomposition is an oxygen - requiring process. The Primary succession occurs in essentially lifeless
rate of decomposition is controlled by chemical areas-regions in which the soil is incapable of
composition of detritus and climatic factors. sustaining life as a result of factors such as lava
22. (a) Temperature and soil moisture are the most important flows, newly formed sand dunes, or rocks left from a
climatic factors that regulate decomposition through retreating glacier. Secondary succession occurs in
the effects on the activities of soil microbes. Warm areas where a community that previously existed has
and moist environment favour decomposition been removed; it is typified by smaller-scale
whereas low temperature and anaerobiosis inhibit disturbances that do not eliminate all life and nutrients
decomposition resulting in build - up of organic from the environment.
materials. 30. (d) The climax community remains stable as long as the
23. (a) Food chain is a series of organisms interrelated in environment remains unchanged.
their feeding habits, the smallest being fed upon by 31. (d) Human beings benefit from a multitude of resources
a larger one, which in turn feeds a still larger one, etc. and processes that are supplied by natural
A food chain begins with a producer, usually a green ecosystem. Collectively, these benefits are known
plant or alga that creates its own food through as ecosystem services, for example, healthy forest
photosynthesis. Limitation of the length of the food ecosystems purify air and water, mitigate droughts
chain does not depend on one organisms ability to and floods, cycle nutrients, generate fertile soils,
consume another. provide wildlife habitat, maintain biodiversity,
24. (c) In aquatic ecosystem such as pond, the producers pollinate crops, provide storage site for carbon and
(macrophytes, phytoplankton) are small organisms. also provide aesthetic, cultural and spiritual values.
Their biomass is least, and this value gradually 32. (d) Communities is an assemblage of interacting
shows an increase towards the apex of the pyramid, populations occupying a given area. Climate, species
thus making the pyramid inverted in shape. interaction, feeding relationship among organisms
25. (b) In most ecosystems, all the pyramids of number, and succession are the factors which influence
energy and biomass are upright i.e., producers are communities.
more in number and biomass than the herbivores 33. (c) Any calculations of energy content, biomass, or
and herbivores are more in number and biomass than numbers has to include all organisms at that trophic
the carnivores. Also energy at a lower trophic level level. Pyramid of energy is always upright, can never
is always more than that at a higher level. be inverted, because when energy flows from a
26. (c) Statement (ii) and (iii), regarding food chain, are particular trophic level to the next trophic level, some
correct. energy is always lost as heat at each step.
27. (a) The rate of biomass production is called productivity 34. (b) Regarding biogeochemical cycles, statement (i), (ii),
and is expressed in terms of g–2 yr–1 or (k cal m–2) yr– and (iv) are correct.
1 to compare the productivity of different Biogeochemical cycle is the flow of chemical elements
ecosystems. Gross primary productivity of an and compounds between living organisms and the
ecosystem is the rate of production of organic matter physical environment. Chemicals absorbed or
during photosynthesis. Primary productivity is ingested by organisms are passed through the food
associated with the producers which are autotrophic, chain and returned to the soil, air, and water by such
most of which are photosynthetic and to a much mechanisms as respiration, excretion, and
lesser extent the chemosynthetic micro-organisms. decomposition. As an element moves through this
cycle, it often forms compounds with other elements
28. (a) The transfer of food energy from the producers, as a result of metabolic processes in living tissues
through a series of organisms (herbivores to and of natural reactions in the atmosphere,
carnivores to decomposers) with repeated eating and hydrosphere, or lithosphere.
being eaten, is known as a food chain. Thus, food 35. (d) Energy flow in the ecosystem in a unidirectional
chain is the flow of energy in a community. Under manner. There is a decline in the amount of energy
natural conditions, the linear arrangement of food passing from one trophic level to the next. Thus pyra-
chains, hardly occurs and these remain indeed mid of energy is always upright. According to
interconnected with each other through different Lindemann, only 10% of energy goes to next trophic
types of organisms at different trophic levels. level.
29. (c) Ecological succession is the process by which the 36. (b) Net primary productivity is the rate of organic matter
structure of a biological community changes and built up or stored by producers in their bodies per
evolves over time. Two different types of succession unit time and area. Net productivity is equal to gross
(primary and secondary) have been distinguished. primary productivity minus loss due to respiration
EBD_7209
Hints & Solutions S-201

and other reasons. Rate of increase in energy air or the oceans (via evaporation).
containing organic matter or biomass by C : Standing crop is the total biomass of an ecosystem
heterotrophs or consumers per unit time and area is or any of its components at a given time. The
known as secondary productivity. standing crop is measured as the mass of living
37. (d) When food is made available automatically the next organisms or the number in a unit area.
higher level of organism in the hierarchy should in- D : Sedimentary cycles include those of iron, calcium,
crease. This is because when the forest cover got phosphorus, and other more earthbound elements.
depleted it led to the increase in the number of en- In this cycle, the reservoir is the Earth's crust.
dangered species. If the deer population is more, it
42. (b)
automatically leads to an increase in the tiger popu-
lation. 43. (c) A : Pioneer community on lithosphere is crustose
lichen. Pioneer species are the first to occupy bare
38. (c) In the food web differen t food ch ains are
ground. These plants are often intolerant of
interconnected. Each chain consists of
competition and especially of shading, and may be
interconnected. Each chain consists of different
crowded out as the community develops.
trophic levels i.e. producers, consumers and
B : Ecological succession is the process by which
detrivorous. So, kite can also be a part of food web.
the structure of a biological community changes and
39. (c) The organisms of all the species that live in a evolves over time.
particular area and interact in various ways with one
C : Climax community is the final stage of biotic
another form biotic community. Biotic community is
succession attainable by a plant community in an
a grouping that is higher than population in ecological
area under the environmental conditions present at
hierarchy. It is an assemblage of all the populations
a particular time.
of different organisms occurring in an area. The
different populations of a community do not remain D : Ecological pyramid is a graphical representation
isolated. They show interactions and inter- designed to show the biomass or bio productivity at
dependence. each trophic level in a given ecosystem. Ecological
pyramids were developed by Charles Elton (1927)
40. (a) A : Primary succession is the series of community
and are, therefore, also called Eltonian pyramids.
changes which occur on an entirely new habitat
44. (d) A : Presence of 3 - 4 storeys of plant grown in a
which has never been colonized before. For example,
forest s called stratification.
facesa newly quarried rock or sand dunes.
B : A biome having grasses with scattered trees is
B : Climax community is the final stage of biotic
called savannah.
succession which is attained by a plant community
in an area under the environmental conditions C : Man made ecosystem is dam. Man - made
present at a particular time. The species composition ecosystem are the artificial ecosystems which rely
of the climax community remains the same because on the human efforts to sustain.
all the species present successfully reproduce D : Pioneer in hydrosere is blue green algae.
themselves and invading species fail to gain a 45. (a) Fungi is not autotrophic. Autotrophs are producers
foothold. which make their own food through the process of
C : Consumers are organisms of an ecological food photosynthesis.
chain that receive energy by consuming other organisms. 46. (d) In the given diagrammatic representation of trophic
These organisms are formally referred to as heterotrophs, levels in an ecosystem, species marked a s A, B,C and
which include animals, bacteria and fungus. . D are respectively secondary consumer, primary
producer, plants and man/lion.
D : Producer is an autotrophic organism that serves
as a source of food for other organisms in a food 47. (c) The given diagram represents a simplified model of
chain. Producers include green plants, which produce phosphorus cycle in a terrestrial ecosystem. The
food through photosynthesis, and certain bacteria natural reservoir of phosphorus is rock, which
that are capable of converting inorganic substances contains the phosphorus in the form of phosphates.
into food through chemosynthesis. 48. (a) Food web is a network of food chains or feeding
relationships by which energy and nutrients are
41. (a) A : Standing state refers to the amount of nutrients
passed on from one species of living organisms to
such as carbon, nitrogen, phosphorous, calcium etc,
another.
present in the soil at any given time. It varies in different
49. (a)
kinds of ecosystems and also on a seasonal basis.
50. (c) In aquatic ecosystem such as pond, as the producers
B : Gaseous cycle include those of nitrogen, oxygen,
(macrophytes, phytoplankton) are small organisms.
carbon, and water. In this cycle, the reservoir is the
Their biomass is least, and this value gradually
S-202 Biology
shows an increase towards the apex of the pyramid, plants (shoots and leaves), and the belowground
thus making the pyramid inverted in shape. parts (roots). In the next step, animals eat the plants,
51. (c) Pyramid given in option (c) represents the variation breath in the oxygen, and exhale carbon dioxide. The
in biomass at different trophic levels in pond carbon dioxide created by animals is then available
ecosystem. Biomass means the dry mass of living for plants to use in photosynthesis. Carbon stored
material at a stage in a food chain. The biomass goes in plants that are not eaten by animals eventually
down as you go from one stage to the next, just like decomposes after the plants die, and is either
the amount of energy. released into the atmosphere or stored in the soil.
52. (c) Option (c) is a pyramid of energy which represents Therefore, If CO2 is removed totally from the
both the food chain. Pyramid of energy is a graphic biosphere; secondary consumers will be affected first.
representation of amount of energy trapped per unit 57. (d) Decomposition involves breakdown of complex
time and area in different trophic levels of food chain organic matter by decomposer to inorganic raw
with producers forming the base and top carnivores materials like CO2, water & various nutrients. It
the tip. The energy content is expressed as Kcal/m 2/ consists of the following processes:
yr. The pyramids of energy indicates not only the Fragmentation : It is the formation of smaller pieces
amount of energy flow at each level, but more of dead organic matter or detritus by detritivores.
importantly the actual role the various organisms
Catabolism : Chemical conversion of detritus into
play in the transfer of energy. In shape it is always
simpler inorganic substances with the help of
upright, as in most of the cases there is always a
bacterial and fungal enzymes is called catabolism.
gradual decrease in the energy content at successive
trophic levels from the producers to various Leaching : Water soluble substances (formed as a
consumers. result of decomposition) are leached to deeper layers
of soil.
TC Humification : If decomposition leads to the
formation of colloidal organic matter (humus), the
SC
process is called humification.
PC Mineralization : Formation of simpler inorganic
P substances (like CO2, water and minerals) is termed
mineralization.
Pyramid of energy (Kcal/m2/yr) in any ecosystem 58. (b) According to the 10% law, at each trophic level, only
(P = Producers; PC = primary consumers (herbivores); 10% of the energy received is transferred to the next
SC = secondary consumers (carnivores); TC = trophic level. And rest of the energy is used up in
Tertiary consumers (carnivores) biological activity and gets wasted in the
53. (c) Arrow III is incorrect. environment.
54. (d) The given food chain represents the grazing food 59. (a) Sparrow feeds upon grains hence called primary
chain. This food chain starts from plants, goes consumer and can also feed on insects hence also
through herbivores and ends in carnivores. called secondary consumer at the same time in the
Plant ® Herbivores ® Pri. Carnivores ® Sec. same ecosystem.
Carnivores ® Top carnivores 60. (b) According to the 10% law, at each trophic level, only
10% of the energy received is transfered to the next
55. (c) In parasitic food chain, the pyramid of number is trophic level.
always inverted, because a single plant may support 61. (b) The phosphorus cycle is the biogeochemical cycle
the growth of many herbivore and each herbivore that describes the movement of phosphorus through
provides nutrition to several parasites, which the lithosphere, hydrosphere, and biosphere. The
supports many hyperparasites. Thus, the number of carbon cycle is the circulation and transformation of
organisms gradually shows an increase. carbon back and forth between living things and the
56. (b) Secondary consumers are organisms, primarily environment. The phosphorous cycle differs from
animals, which eat primary consumers. Through carbon cycle in that the phosphorous cycle does
photosynthesis (the process by which plants not include a gaseous phase whereas the carbon
captures the sun's energy and uses it to grow), plants cycle does.
take carbon dioxide out of the atmosphere and release 62. (d) Grassland can support greater grazing rates by
oxygen. The carbon dioxide is converted into carbon herbivores than forests because grassland produces
compounds that make up the body of the plant, which less woody plant tissue.
are stored in both the aboveground parts of the 63. (b) An ecosystem includes the environment and the
EBD_7209
Hints & Solutions S-203

biological community living there. A coral reef environments along with their interactions and processes.
ecosystem would include much more than just the 2. (a) Alpha diversity, also known as within community
fish. diversity, means diversity of organisms sharing the
64. (b) Herbivorous animals are primary consumers in same community habitat.
terrestrial ecosystem. 3. (b) All the three diversities [alpha diversity (a-diversity),
65. (d) Respir ation and photosynth esis; fossil fuel beta diversity (b-diversity) and gamma diversity (g-
combustion and decomposition of dead organisms diversity)] were introduced by R. H. Whittaker. Beta
contribute to the carbon cycle. The carbon cycle is diversity may be defined as the rate of replacement
the circulation and transformation of carbon back of species along a gradient of habitats or communities
and forth between living things and the environment. due to presence of different microhabitat, niches and
66. (b) The pyramid of biomass is upright, in which the differences in environmental condition between
number of organism decreases from first trophic level, community diversity.
i.e., producers to second trophic level, i.e., herbivores 4. (b) Gamma diversity refers to the total species richness
to the last trophic level, i.e., carnivores. over a large area or region.
67. (d) Primary succession is the ecological succession 5. (c) Eastern Himalayan and Western Ghats are the
occurring in an area where no organisms are found, hotspot area which shows maximum biodiversity in
like bare rocks, sand dunes etc. our country.
68. (d) Unlike their behaviour in air and water, elements on 6. (b) The International Union for Conservation of Nature
land move slowly, and usually only for short (IUCN) is an international organization which works
distances. in the field of nature conservation and sustainable
69. (c) Fresh weight is not used for the construction of use of natural resources. It is also involved in data
ecological pyramids because the total fresh weight gathering and analysis, research, field projects and
does not change into energy. Hence we can say that education.
fresh weight is not continuous in the tropic levels. 7. (c) According to IUCN (2004), the total numbers of plant
70. (d) Four important functional aspects of the ecocystem and animals species are more than 1.5 million species
are (i) productivity (ii) decomposition, (iii) energy described so far.
flow, (iv) nutrient cycling. Stratification is the occurrence 8. (c) Animals are more diverse (above 70%) than plants
of vertical zonation in the ecosystem & indicates the including plantae and fungi (22%).
presence of favourable environmental conditions. 9. (d) Exotic species usually reduces biodiversity because
71. (b) In an ecosystem, producers (e.g., plants/ these species which is not native to a particular area,
phytoplankton) belong to first trophic level, arrives (usually with the help of human) and
herbivores or primary consumer (e.g., grasshopper) establishes a population, and spreads on its own
to the second and carnivores or secondary consumer kind.
(e.g., lion, wolf) to the third trophic level. 10. (c) The number of fungal species in the world is more
72. (b) In food web, each successive trophic level has less than the combined total species of fishes, amphibians,
total energy content as per law of Lindeman. Some reptiles and mammals. Therefore it represents the
energy is lost in tranfer from one to another level. highest number of species in the world.
73. (a) Green plants are called producer because they can
11. (d) Pigeon is not an example of recent extinction because
synthesize food material in presence of light
extinction is a natural phenomenon in which a species
(autotrophs), therefore for any food chain first link
goes extinct if it is not able to adapt to changes in it's
is the green plants.
environment, or compete effectively with other
74. (d) 2/3 parts of earth is ocean, here various types of
organisms. Examples of recent extinctions include
food chains form food webs. This ecosystem is most
dodo (Mauritius), quagga (Africa), Steller's sea cow
stable due to buffering action of water.
(Russia), thylacine (Australia) and three subspecies
75. (b) Secondary productivity is the rate of storage of organic
of tiger (Bali, Java, Caspian).
matter by consumers per unit area per unit time.
12. (b) Amphibians are more vulnerable to extinction because
76. (d) In ecological succession from pioneer to climax
they are highly sensitive to environmental changes
community, the biomass shall increase continuously.
which lead to extinction. Threatened by habitat
Chapter 37 : Biodiversity and Conservation destruction and degradation, the spread of virulent
diseases, climate change and trade, amphibians may
1. (d) Biodiversity is the variety of living forms present in be on the verge of a near total disappearance from
various ecosystems. It is the occurrence of different types global ecosystems.
of ecosystems different species of organisms with their 13. (d)
biotypes and genes adapted to different climates, 14. (d) Hunting is not the cause of “The Evil Quartet of
S-204 Biology
biodiversity loss.” Western Ghats, Indo-Burma and Himalaya.
15. (a) 24. (a) Hot spots are the areas or regions of high endemism
16. (c) Vulnerable species are those species that already and very high levels of species richness. These are
existed in low number and are likely to move into the richest and the most threatened reservoirs of
endangered category in the near future, if causal plant and animal life on earth.
factors such as habitat destruction, over exploitation 25. (b) Endemic plants and animals are those species which
and other environmental disturbances continue over are restricted to a particular area. This makes them
a period of time. E.g. Musk deer, Sambhar deer, Black both extremely special as well as more vulnerable to
buck. extinction. As they are only found in certain
17. (b) Nile perch is a large predator fish, introduced in locations, they require special conservation efforts.
Lake Victoria of South Africa. It was endemic to the 26. (d) National Park is a protected area, which is strictly
aquatic system because it begins to threaten the reserved for the welfare of the wildlife and where
entire fresh water ecosystem by feeding on small activities like forestry, grazing and cultivation are
herbivorous and detrivorous cichlid fish species (of not permitted. Their boundaries are well marked and
Lake victoria). circumscribed. In national parks, the emphasis is on
18. (b) Global warming is predicted to be a major cause of the preservation of a single plant or animal species.
extinction in the future. Global warming is the regular 27. (b) There are 83 national parks and 421 sanctuaries found
increase in the overall temperature of the earth's in India. Their total area is about 1,41,295 sq. km. It is
atmosphere which is generally credited to the about 4% of India’s total geographic area.
greenhouse effect caused by increased levels of 28. (c) Seed bank is an example of ex - situ conservation. Ex
harmful gases like carbon dioxide, CFCs, and other situ conservation is the conservation outside the
pollutants. (organisms) habitats by preserving sample
19. (c) Habitat loss is the most serious threat to biodiversity population in genetic resources, centres, zoos,
leading to extinction of animals and plants species. botanical gardens etc. Seed bank is a type of gene
All over the world habitats are being turned into bank which stores seeds (may be of food crops or
agricultural land, harvested for wood and fuel, and those of rare species which were destroyed) to
destroyed or changed to build roads, schools, malls protect biodiversity.
and other human developments. Since human 29. (d) Cryopreservation is a process where cells, whole
population is growing so rapidly and consuming so tissues, or any other substances susceptible to
many natural resources, habitat loss is occurring at damage caused by chemical reactivity or time are
a rapid pace. preserved by cooling to sub-zero temperatures. At
20. (a) A country with a high proportion of endemic species low enough temperatures, any enzymatic or chemical
should receive high conservation priority because if activity which might cause damage to the material in
the endemic species are lost there, they generally question is effectively stopped. The germ plasm in
become completely extinct. cryopreservation is maintained at –196°C (not °F).
21. (c) In situ conservation (on site) is the conservation of 30. (d) All the given statements are correct.
genetic resources within natural or human-made 31. (c) Wildlife conservation is the practice of protecting
ecosystems in which they occur. E.g. protected areas wild plant and animal species in their natural habitats.
such as national parks, sanctuaries, biosphere 32. (c) A botanical garden is a place where plants are grown
reserves, cultural landscapes, natural monuments. and revealed for the purposes of research and
22. (c) Biosphere reserve is an area created to protect the education and allows ex situ conservation for germ
biological and cultural diversity of a region. It is a plasm. Botanical gardens distinguish them from other
place of collaboration, education an d parks and gardens where plants, with beautiful
experimentation, where scientists etc can share flowers, are grown for public convenience. Botanical
research data to better understand man's impact on gardens specialize in trees are referred to as arboreta.
nature, and where local communities, environmental 33. (a) Wild life sanctuary is a protected area which is
groups, and economic interests can work reserved for the conservation of animals only.
collaboratively on conservation and development 34. (d) Biodiversity is the variety of living forms present in
issue. various ecosystems. It includes variability among life
23. (d) Hot spots are the areas or regions of high endemism forms from all sources including air, water and land. It is
and very high levels of species richness. These are the diversity of biological organisation ranging from
the richest and the most threatened reservoirs of cellular macromolecules to biomes.
plant and animal life on earth. There are 34 hot spots 35. (a) Accor ding to the concept of species-area
in the world, of which three are in India; namely relationship, within a region, species richness gets
EBD_7209
Hints & Solutions S-205

increased when explored area is increased, but only cm. They are also called jungles and cover one
up to a limit. twelveth of earth's surface but contain more than
36. (a) Genetic diversity is the diversity shown by a single half of the earth's flora and fauna (i.e., rich in
species at genetic level. biodiversity). Now-a-days th ese forests ar e
37. (d) Quantifying or measuring the species genome does disappearing due to excessive cutting of forests for
not help to recognise biodiversity. Biodiversity is domestic purposes like fuel, furnitur es,
the variety of plant and animal life found in a accomodations, cloths, resin, gum, etc.
particular habitat. 47. (c) Biodiversity varies with change in latitude or altitude.
38. (a) Habitat loss and its alteration is the most important The diversity increases as we move from high to low
human activity leading to extinction of wildlife. The latitudes (i.e., from poles of equator). In the temperate
transformation of the natural areas determines not region, the climate is severe with short growing
only the loss of the vegetable species, but also a period for plants while in tropical rain forests the
decrease in the animal species associated with them. conditions are favourable for growth throughout the
Habitat alteration and destruction of the natural year. This makes it possible for a large number of
habitat cause the destruction of breeding grounds, species to occur and grow. Gamma diversity refers
shelter and sources of food. to the total biodiversity over a large area or region. It
39. (d) A sacred grove or sacred wood is a grove of trees of is the total of a and b diversity.
special religious importance to a particular culture. 48. (b) A - IV, B - I, C - II, D - III.
40. (d) Ex situ conservation means off-site conservation. It Nile perch introduced in Lake Victoria acts as an alien
protects an endangered species of plant or animal species and leads to the extinction of cichlid fish
outside its natural habitat; for example, by removing present in the lake. Narrowly utilitarian is the obvious
part of the population from a threatened habitat and reason for biodiversity conservation. Habitat
placing it in a new location, either a wild area or within destruction is the main cause for loss of biodiversity.
the care of humans. While ex situ conservation Hot spot is a biogeographic region with a major
comprises some of the oldest and best known reservoir of biodiversity that is under threat from
conservation methods, it also involves newer humans, hence it show high species diversity as well
methods also like cryopreservation etc.. as high endemism.
41. (d) Biodiversity refers to the variety found in biota from 49. (c) A - I, B - III, C - II, D - IV
genetic make-up of plants and animals to cultural Biodiversity term (given by Edward Wilson) is used
diversity. The main cause of the loss of biodiversity to describe the variety of plant and animal species in
can be attributed to the influence of human beings a given particular area. In - situ conservation (also
on the world's ecosystem. The important factors called as on site conservation) protects an
causing loss of biodiversity are - habitat loss, habitat endangered plant or animal species in its natural
fragmentation, disturbances, over exploitation of habitat by different methods. Plant pollinator or
resources, pollution, exotic species, co-extinction, mutualism shows co - extinctions (phenomena of the
alien species invasion, intensive agriculture and loss or decline of a host species resulting in the loss
forestry. or endangering of other species that depend on it).
42. (b) Fragmentation and loss of habitat are the major cause Ex - situ conservation (also called as off - site
of loss of numbers of migratory birds. conservation) preserves the components of
43. (a) Communities with higher number of species are more biological diversity outside their natural habitats.
stable as it can resist occasional disturbances. A 50. (a) A - III, B - V, C - I, D - II, E - IV
stable community should show less variation in 51. (b) A - III, B - I, C - II
productivity from year to year and resistant towards 52. (b) A - V, B - IV, C - II, D - I, E - III
by alien species. 53. (a) In the given pie diagrams (A, B and C), the
44. (a) Latitudinal and altitudinal gradation is displayed by proportionate number of species of major taxa of
biodiversity. A decrease in in variety of species is invertebrates, vertebrates and plants respectively
seen as we assend a high mountain due to a drop in marked as I (mollusca), II (amphibians), III (Fungi),
temperature and greater variations in seasons. IV (Angiosperms)
45. (c) Fossil records indicate that there had been five
54. (d)
episodes of mass extinctions since life originated
55. (a) A, B and C respectively show minimum, greater and
and diversified on earth mostly due to natural caves,
maximum diversity. Since the number of individuals
the sixth or current mass extinction is on going and
in all the three boxes are same but the variety of animals
different from earlier extinctions as it is man-made.
in all the boxes are different, like box A shows diversity
46. (c) Tropical rain forests are located in the equatorial
of single animal - bird, box B shows diversity of birds
regions where ever the annual rainfall exceeds 140
S-206 Biology
and mammals whereas box C shows diversity of three which is usually the area in which they breed. These
animals - birds, mammals and insects. species present special preservation challenges
56. (d) Fungi > Angiosperms > Algae > Pteridophytes because their conservation may need international
57. (a) India has only 2.4% of the land area of the world; it cooperation when they require habitats or
has 8.1% of the global species biodiversity. environments in different countries where they
58. (d) On a logarithmic scale, the relationship is a straight line migrate to survive.
and the equation is described as Log S = log C + Z log 70. (b) Biosphere reserves are a special category of
A, where, S= Species richness, A= Area, C= Y-intercept protected areas of land or coastal environments
and Z= slope of the line (regression co-efficient). where in people are an integral component of the
59. (d) Cheetah has become extinct from India in the mid- system.
20th century, about the time that India gained its 71. (c) Sacred groves help in protection of many rare,
independence in 1947. And it is the only animal to threatened and endemic species of plants and animals
become extinct from India due to unnatural causes. found in an area. The process of deforestation is
60. (d) Threatened species are species that are likely to strictly prohibited in this region by tribals. Hence,
disappear from the world sooner or later. In the Red the sacred grove is a biodiversity rich area.
list, all species listed under the categories critically
are endangered, vulnerable and endangered are Chapter 38 : Environmental Issues
together described as threatened species.
1. (d) Pollution causes undesirable changes in the
61. (d) Creating biosphere reserve is the most effective characteristics of air, water, land or soil. Pollution is
approach to conserve the plant diversity in an area. the contamination of air, water, or soil by substances
A biosphere reserve is an ecosystem with plants and that are harmful to living organisms. Pollution can
animals of unusual scientific and natural interest. occur naturally, for example through volcanic
62. (d) Animal species should be preserved mainly because eruptions, or as the result of human activities, such
man cannot recreate a species of animals after its as the spilling of oil or disposal of industrial waste.
destruction and also preservation of these animal 2. (b) Pollution is any undesirable change in physical
will provide better knowledge and understanding of chemical or biological characteristics of air, land,
their own species. water or soil. In order to control environmental
63. (b) Conservation is the systematic study of the nature pollution, the Govt. of India has passed the
and of Earth's biodiversity with the aim of protecting Environment (Protection) Act, 1986 to protect and
species, their habitats, and ecosystems from excessive improve the quality of our environment.
rates of extinction and the erosion of biotic interactions. 3. (d) The electrostat precipitator can remove over 99
64. (a) All life forms should be conserved because, they percent particulate matter present in the exhaust from
maintain diverse genetic resources. a thermal power plant. The electrostatic precipitator
65. (a) The variety in the number and richness of the species has electrode wires which are maintained at several
of a region is called species diversity. Species thousand of volts. The electrode wires produce a
diversity increases from high altitude to low altitude Corona that releases electrons. These electrons get
and from high latitude to low latitude. attached to the dust particles by giving them a net-
66. (c) Lantana camara and water hyacinth are exotic negative charge. The collecting plates are grounded
species introduced in India. Exotic species are those to attract the charged dust –particles. The velocity
species which are not native to an ecosystem, but of air between the plates is kept low enough to allow
often established purposefully or inadvertently by the dust particles to fall. However, the particulate
human activity. matter that are very small are not removed by these
67. (c) Information given in the table shows that area III precipitators.
has the maximum species diversity. Species diversity 4. (b) To remove harmful gases, like sulphur-dioxide, from
is a extent of the diversity within their ecological gaseous air pollutants the scrubbers are used. In scrubber
community that includes both species richness (the the harmful gases (exhaust) are passed through a spray
number of species in a community) and the evenness of water or the lime to separate the clean air.
of species abundances. 5. (a) According to Central Pollution Control Board (CPCB),
68. (d) Alteration of habitat by human beings is the greatest particulate size 2.5 micrometers or less in diameter
threat to global biodiversity. Natural habitats are (PM 2.5) are responsible for causing the greatest
increasingly changed by human activities. harm to human health.
69. (b) Migratory species perform cyclical movements 6. (a) Catalytic converter is a device incorporated in the
between two distinct geographical areas, one of exhaust system of a motor vehicle, containing a
EBD_7209
Hints & Solutions S-207

catalyst for converting pollutant gases into less brown, or red. Bright green blooms may also occur.
harmful ones. In its reaction chamber, it contains a These are a result of blue-green algae, which are
finely divided catalyst (usually platinum and other actually bacteria (cyanobacteria). Algal bloom causes
metals such as rhodium, palladium, and gold) into deterioration of the water quality and fish mortality.
which exhaust gases from an automotive engine are 15. (b) The excessive growth of a floating plant, Water
passed together with excess air so that carbon Hyacinth (Eichhornia crassipes) has caused havoc
monoxide and hydrocarbon pollutants are oxidized in India by blocking our water- ways. In India it is
to carbon dioxide and water, and nitrogen oxides are also known as ‘Terror of Bengal’. It grows abundantly
reduced to nitrogen and oxygen gases. in nutrient rich water bodies and causes an imbalance
7. (d) Noise is undesirable sound of high level that causes in the aquatic ecosystem.
psychological and physiological disorders in human. 16. (a) Unlike domestic sewage, waste water from industries
A brief exposure to extremely high level of sound like petroleum, paper manufacturing, metal extraction
( ³ 150 dB) generated by taking off of a jet plane or and processing, chemical manufacturing, etc. often
rocket, may damage ear drums and hence can impair contain toxic substances, notably heavy metals
hearing ability permanently. (defined as elements with density > 5g/cm3); such
8. (a) The Govt. of India through a new auto fuel policy as mercury, cadmium, copper, lead etc.) and a variety
has laid out a roadmap to cut down the vehicular of organic compounds.
pollution in Indian cities. For example, Euro II norms 17. (a) Increase in concentration of a pollutant from one
stipulate that sulphur be controlled at 350 ppm in link in a food chain to another is called
diesel and 150 ppm in petrol. biomagnification. In aquatic food chain,
9. (c) All automobiles and fuel - petrol and diesel were to biomagnification of DDT shows that the
meet the Euro III emission specification in some cities concentration of DDT is increased at successive
from 1 April 2008 and have to meet the Euro IV norms trophic level. Concentration of DDT for first trophic
by 1 April 2010. level (phytoplankton) and top trophic level (fish
10. (c) All domestic sewage and industrial effluents, without eating birds) is respectively 0.025ppm and 25ppm in
being treated are dumped into nearby river. Even aquatic food chain if DDT is 0.03 pb in water. High
0.1% of impurites by sewage, makes the water unfit concentration of DDT disturb calcium metabolism in
for human use. birds which causes thinning of eggshell and their
11. (a) The domestic sewage primarily contains premature breaking eventually causing decline in bird
biodegradable organic matter which is decomposed population.
by bacteria and other micro-organisms. The amount 18. (b) DDT causes egg shell thinning in birds because it
of such organic matter in sewage water is measured inhibits calcium ATPase.
by BOD (Biochemical Oxygen Demand) method. 19. (b) Eutrophication is a natural process which literally
12. (c) Full form of BOD is biochemical oxygen demand. It means well nourished or enriched. It is a natural state
is the amount of dissolved oxygen needed by aerobic in many lakes and ponds which have a rich supply
biological organisms in a body of water to break down of nutrients. Eutrophication becomes excessive when
organic material present in a given water sample at abnormally high amount of nutrient from sewage,
certain temperature over a specific time period. fertilizers, animal wastage and detergent, enter
13. (c) BOD may be defined as the amount of oxygen streams and lakes causing excessive growth or
required by aerobic microorganisms to decompose blooms of microorganisms. With increasing
the organic matter in a sample of water, such as that eutrophication, the diversity of the phytoplankton
polluted by sewage. It is used as a measure of the community of a lake increases and the lake finally
degree of water pollution. becomes dominated by blue - green algae.
14. (c) An algal bloom is a relatively rapid increase in the 20. (b) Friends of the Arcata Marsh (FOAM) is a non-profit
population of (usually) phytoplankton algae in an organization which for the last twenty years has
aquatic system. Typically only one or a few species advanced knowledge and educated the public about
are involved and the bloom is recognized by treatment and reuse of wastewater. The purpose of
discoloration of the water resulting from the high the FOAM is to stimulate understanding of the
density of pigmented cells. Although there is no Arcata Marsh and Wildlife Sanctuary, its relationship
officially recognized threshold level, algae are unlikely with Arcata's integrated wastewater treatment system,
to be considered to be blooming unless more than the surrounding watersheds and bay, and their link
10,000 cells per millilitre occur. Algal bloom with the earth's water cycle.
concentrations may reach millions of cells per 21. (a) Polyblend is a fine powder of recycled modified
millilitre. Colours observed are green, yellowish- plastic, developed by Ahmed khan’s company. This
S-208 Biology
mixture is mixed with bitumen that is used to lay temperature and may enhance the growth of plants
roads. and fish in extremely cold areas, only after causing
22. (a) Main green house gases are CO2, CH4, N2O, CFC damage to indigenous flora and fauna.
excluding this SO2, NO2, O3, water vapour are also 31. (d) All the statements regarding Eco San are correct.
released from industries and agriculture which are Eco san is a sustainable system for handling human
responsible for to increasing the green house effect. excreta or faecal matter by using composting toilets.
23. (d) The green house effect is a naturally occuring These are very useful in the rural areas where sewer
phenomenon that is responsible for heating of Earth’s system are not possible. These toilets are hygienic,
surface and atmosphere. Without a green house efficient, practical and most effective for the disposal
effect, the average temperature at earth’s surface of human excreta.
would have been around – 18°C, rather than the 32. (a) Ozone layer is a deep layer in earth's atmosphere
present average of +15°C. that contains ozone which is a naturally occurring
24. (a) Jhum cultivation is a local name for slash and burn molecule containing three oxygen atoms. The good
agriculture practiced by the tribal groups in the north ozone is found in the upper part of the atmosphere
eastern states of India like Arunachal Pradesh, called the stratosphere. The ozone layer forms a thick
Meghalaya, Mizoram and Nagaland and also in the layer in stratosphere, encircling the earth that has
districts of Bangladesh like Khagrachari and Sylhet. large amount, of ozone in it. It protects the Earth
This system involves clearing a piece of land by from the harmful radiations that come from the sun.
setting fire or clear felling and using the area for 33. (d) All the statements regarding noise pollution are
growing crops of agricultural importance such as correct. Noise pollution is the unwanted sound which
upland rice, vegetables or fruits. After a few cycles, is released into the environment. It disturbs the
the land loses fertility and a new area is chosen. human being and cause an adverse effect on the
25. (c) Integrated organic farming refers to a cylindrical zero mental and psychological well - being. It is measured
waste procedure, where waste products from one in the units of decibels and is denoted by dB. The
process are recycled in as nutrients for the other noise which is more than 115 dB is tolerant. It affects
processes. This allows the maximum utilization of the general health and hearing power of the human
resources and increases the efficiency of production. beings.
26. (a) The presence of a large amount of nutrients in water 34. (a) Integrated waste water treatment involves
causes excessive growth of planktons or free- conventional sedimentation, filtering and chlorine
floating algae to produce algal bloom. Such algal treatment. After this a lot of dangerous pollutants
blooms deteriorate the quality of water, cause the like dissolved heavy metals still remain. To combat
mortality of fishes and impart distinct colour to water this situation, the biologists developed a series of
bodies. six connected marshes over 60 hectares of marshland
27. (b) Pollution in large cities can be controlled to a large where appropriate plants, algae, fungi and bacteria
extent by road side plantation and proper disposal were seeded which neutralize, absorb and assimilate
of garbage and domestic as well as municipal wastes. the pollutants. Hence, as the water flows through
28. (d) All the statements regarding eutrophication are the marshes, it gets purified naturally.
correct. Eutrophication is a natural process that 35. (c) The major contribution in green houses is water
occurs in an aging lake or pond as that body of water vapour, carbon dioxide, methane, nitrous oxide,
gradually builds up its concentration of plant ozone, and any fluorocarbons. Global warming is
nutrients. because of green - house gases. Global warming is a
29. (d) Eco friendly disposal of municipal wastes should be gradual increase in the overall temperature of the
sorted out into biodegradable, non - biodegradable earth's atmosphere generally attributed to the
and recyclable wastes and treated separately. greenhouse effect caused by increased levels of
30. (a) Cultural eutrophication is an accelerated form of carbon dioxide, CFCs, CH4, and other pollutants.
eutrophication. Cultural or artificial eutrophication 36. (b) Montreal protocol on substances that deplete the
occurs when human activity introduces increased ozone layer is a protocol to the Vienna Convention
amounts of these nutrients, which speed up plant for the Protection of the Ozone Layer. It is an
growth and eventually choke the lake of all of its international treaty designed to protect the ozone
animal life. layer by phasing out the production of a number of
The prime contaminates are nitrates and phosphates substances believed to be responsible for ozone
which act like plant nutrients. depletion. The treaty was opened for signature on
September 16, 1987, and entered into force on
The water from electricity generating units reduces
January 1, 1989, followed by a first meeting in Helsinki,
the number of organisms sensitive to high
May 1989.
EBD_7209
Hints & Solutions S-209

37. (d) Unlike other pollution, noise pollution is such that it biomagnification of DDT shows that the
does not show its presence by leaving any kind of concentration of DDT is increased at successive
residue in the environment. Yet it badly affects the trophic level.
human health, as apart from hampering the hearing B : Platinum - Palladium and Rhodium - These act as
ability, it also causes nervous disorders in man. Green catalyst in catalytic converter which is a device
plantation along the noisy roads is a technique (green incorporated in the exhaust system of a motor vehicle
mufflers) to counteract the noise pollution because for converting pollutant gases into less harmful ones.
plants may absorb noise of low frequency. Motor vehicles equipped with catalytic converter
38. (d) The greenhouse effect is a process by which thermal are advised to use unleaded petrol because lead
radiation from a planetary surface is absorbed by causes inactivation of catalyst (platinum, palladium,
atmospheric greenhouse gases, and is re-radiated in and rhodium) in a catalytic converter.
all directions. Re designing land-fill dumps to allow C : Acid rain- Acid rain is a rainfall made by
methane to be collected slows down the green house atmospheric pollution, it causes environmental harm,
effect. chiefly to forests and lakes. The main cause is the
39. (b) Methane is produced by incomplete biomass industrial burning of coal and other fossil fuels, the
combustion, incomplete decomposition mostly by waste gases containing sulphur and nitrogen oxides
anaerobic methanogens. which combine with atmospheric water to form acids.
Carbon dioxide contributes about 60% of the total D : Global warming - Global warming is the increase
global warming and share of methane (CH4) and of Earth's average surface temperature due to effect
chlorofluoro carbons (CFCs) is 20% and 14% of greenhouse gases, such as carbon dioxide
respectively. N2O also contributes 6% in total global emissions from burning fossil fuels or from
warming. Efficient engine such as multi point fuel deforestation, CFCs, CH4, and other pollutants,
injection engine can reduce the unburnt which trap heat that would otherwise escape from
hydrocarbon (methane) in auto-emissions. Earth.
40. (b) Catalytic converters are involved in reducing 45. (b) A : Catalytic converter is a device incorporated in
gaseous pollutions by converting the exhaust system of a motor vehicle, containing a
CO ¾¾ ® CO2, NO2 ¾¾ ® N2 etc. Thus, decrease catalyst for converting pollutant gases into less
the amount of pollutant. They can not reduce harmful ones. As the exhaust passes through the
emission of SPM. converter, unburnt hydrocarbons are converted into
41. (b) Global warming is due to the increase in concentration carbon dioxide, water, carbon monoxide and nitric
of green house gases resulted in increase in global oxide are changed to carbon dioxide and nitrogen
temperature. These global gases prevent the escape gas respectively.
of long wave radiations into space. B : Electrostatic precipitator is a device that removes
42. (a) Water pollutants are commonly measured by their suspended dust particles from a gas or exhaust by
main common denominator, called BOD (Biochemical applying a high-voltage electrostatic charge and
Oxygen Demand), i.e., the amount of free oxygen collect the particles on charged plates.
absorbed by extraneous substances from water. If C : Earmuffs is a pair of soft fabric coverings,
water is polluted, it will consume more oxygen, connected by a band across the top of the head, that
thereby enhancing the BOD of water. are worn over the ears to protect them from cold or
43. (b) Eutrophication is a natural process which literally high noise level.
means well nourished or enriched. It is a natural state D : Land - fill is a site for the disposal of solid waste
in many lakes and ponds which have a rich supply in which refuse is buried between layers of dirt so as
of nutrients. Eutrophication become excessive, to fill in or reclaim low-lying ground.
however when abnormally high amount of nutrient 46. (b) A : Environment (protection) Act was enacted in 1986
from sewage, fertilizers, animal wastage and with the objective of providing for the protection
detergent, enter streams and lakes causing excessive and improvement of the environment. The Act was
growth or blooms of microorganisms. With last amended in 1991.
increasing eutrophication, the diversity of the B : Air (Prevention and Control of Pollution) Act,
phytoplankton community of a lake increases and 1981 is an Act of the Parliament of India to control
the lake finally becomes dominated by blue - green and prevent air pollution. It was amended in 1987.
algae. C : Water (Prevention and control of pollution) Act,
44. (d) A : DDT - Increase in concentration of a pollutant 1974 is established for water pollution control. This
from one link in a food chain to another is called act prevents and controls water pollution, maintains
biomagnification. In aquatic food ch ain, or restores the wholesomeness of water, establish
S-210 Biology
boards for the prevention and control of water chlorinated aromatic hydrocarbons, DDT tends to
pollution and confer on and assign to the boards, persist in the environment and becomes concentrated
the power and functions relating to the above in animals at the head of the food chain.
mentioned. D : Phosphates - Eutrophication : Eutrophication is a
D : Concept of Joint Forest Management of Govt. of natural process that occurs in an aging lake or pond
India, 1980 to work closely with the local communities as that body of water gradually builds up its
for protecting and managing forests. concentration of plant nutrients such as nitrogen
47. (a) DDT is a synthetic organic compound used as an and phosphorus.
insecticide. Like other chlorinated aromatic 51. (c) A : Colloidal material - Faecal matter bacteria, cloth
hydrocarbons, DDT tends to persist in the and paper fibres.
environment and becomes concentrated in animals
at the head of the food chain. Its use is now banned B : Waterborne disease - Typhoid, Jaundice, Cholera.
in many countries. Waterborne diseases are caused by pathogenic
microorganisms that most commonly are transmitted
A : Zooplankton - 0.04 ppm
in contaminated fresh water.
B : Small fish - 0.5 ppm
C : Large fish - 2 ppm C : E - wastes - Irreparable computers and other
D : Fish eating birds - 25 ppm electronic goods. It is the term used to describe old,
end-of-life or discarded appliances using electricity. It
48. (a) A : Ahmed khan, aged 57 years old is a plastic sac
includes computers, consumer electronics, fridges etc
manufacturer in Bangalore and has managed to find
which have been disposed of by their original users.
the ideal solution to the ever increasing problem of
accumulating plastic waste. He has been producing D : Manure - Cattle excreta (dung). Manure is animal
plastic sacks for 20 years. excrement that is used to fertilize.
B : Ramesh Chandra Dagar is an organic farmer in E : Bad ozone - Troposphere. In the Earth's lower
Sonipat, follows integrated organic farming. atmosphere (troposphere), near ground level, bad
C : Amrita Devi Bishnoi is involved in protecting ozone is formed when pollutants emitted by cars,
wildlife. In 1730, 363 Bishnoi men, women and power plants, industrial boilers, refineries, chemical
children led by Amrita Devi died protecting trees plants, and other sources react chemically in the
from cutting by the king's men. This incident presence of sunlight. Ozone at ground level is a
happened in Khejarli which is a village in Jodhpur harmful pollutant.
district of Rajasthan, 26 km south-east of the city of 52. (c) The given diagram in question shows a device known
Jodhpur. The Bishnois sacrificed their lives while as scrubber that can remove gases like sulphur
protecting trees by hugging to them. dioxide. In a scrubber, the exhaust is passed through
49. (c) Solar energy coming to the earth is not responsible a spray of water or lime.
for green house effect. It is the increase in green 53. (a) Figure (a) shows the correct relative contribution of
house gases in atmosphere like CO2 which is released greenhouse gases to global warming.
by complete combustion of fossil fuels or biomass 54. (a) The given figure shows changes occur after the
in industries or transportation vehicles that prevent discharge of sewage into a river. Micro-organisms
the re-radiation of infrared radiation from the earth involved in biodegradation of organic matter in the
and result in increase in the temperature of the earth. receiving water body consume a lot of oxygen, and
50. (c) A : UV - Snow blindness : Snow blindness is a painful as a result there is a sharp decline in dissolved oxygen
eye condition that occurs when your eye is exposed downstream from the point of sewage discharge.
to invisible rays of energy called ultraviolet (UV) 55. (a) In the given diagram of the effect of polluting river
rays, either from the sun or from a man-made source. with untreated whey, the graph marked as "X"
B : Biodegradable organic matter - BOD : represents the bacterias count.
Biodegradable organic matter such as plant and 56. (a) Electrostatic precipitator is a device that removes
animal matter and other substances originating from suspended dust particles from a gas or exhaust by
living organisms, or artificial materials that are similar applying a high-voltage electrostatic charge and
enough to plant and animal matter to be put to use collecting the particles on charged plates. In the given
by microorganisms. It is possible to estimate the diagram of electrostatic precipitator, the parts marked
amount of biodegradable organic matter in sewage as A, B and C are respectively discharge corona,
water by measuring biochemical oxygen demand negatively charged ions, and collection plate grounded.
(BOD). 57. (c) In the given figure ‘X’ represents ozone hole. Ozone
C : DDT - Biomagnification : DDT is a synthetic hole is the area above Antarctica, where the ozone
organic compound used as an insecticide. Like other layer is the thinnest.
EBD_7209
Hints & Solutions S-211

58. (a) The given figure shows the phenomenon of green oxygen dissolution.
house effect. Green house effect is a naturally 65. (d) Greenhouse gases contribute to the greenhouse
occurring phenomenon that is responsible for effect by absorbing infrared radiation. Greenhouse
heating of earth surface and atmosphere. gases in the earth's atmosphere absorb IR from the
The sunlight or solar energy when enters sun and release it. Some of the heat released reaches
atmosphere, about ¼ of it is reflected back due to the earth, along with heat from the sun that has
clouds and gases in the outermost atmosphere, some penetrated the atmosphere. Both the solar heat and
of it is absorbed, and about half of solar radiation the radiated heat are absorbed by the earth and
falls on the earth surface for heating it, though a released; some are reabsorbed by greenhouse gases
small proportion of it is reflected back from earth. to preserve the cycle. Common examples of
59. (d) CNG is a readily available alternative to gasoline greenhouse gases include: water vapour, carbon
that's made by compressing natural gas to less than dioxide, methane, nitrous oxide, ozone, and any
1% of its volume at standard atmospheric pressure. fluorocarbons. Although water vapour is the most
It consists mostly of methane. It is odourless, abundant greenhouse gas but it is a relatively
colourless and tasteless. CNG is better than petrol/ ineffective one. Human activities are responsible for
diesel because it has very low moisture content, almost all of the increase in greenhouse gases in the
cheap, easy to transport, burns more efficiently, atmosphere over the last 150 years. Today the
completely and can not be adulterated. concentration of green - houses gases is very high
60. (a) In clean water, the concentration of BOD because of use of refrigerator, increased combustion
(biochemical oxygen demand) is low but the DO of oils and coal and deforestation.
(dissolved oxygen) is high because BOD may be 66. (b) One of the major effects of deforestation is increased
defined as the amount of oxygen required by aerobic CO2 concentration in the atmosphere because trees
microorganisms to decompose the organic matter in hold a lot of carbon in their biomass that is lost with
a sample of water, such as that polluted by sewage. deforestation.
It is used as a measure of the degree of water pollution. 67. (c) If the forest cover is reduced to half, large areas will
61. (a) Water pollution is the contamination of water bodies become deserts, on a long term basis.
(e.g. lakes, rivers, oceans, aquifers and groundwater) 68. (d) The main problem with switching over to
by chemical, physical, radioactive or pathogenic compressed nautral gas (CNG) is the difficulty of
microbial substances. This form of environmental laying down pipelines to deliver CNG through
degradation occurs when pollutants are directly or distribution points/pumps and ensuring
indirectly discharged into water bodies without uninterrupted supply.
adequate treatment to remove harmful compounds. 69. (a) Catalytic converter is a device incorporated in the
Water pollution can be stopped best by treating exhaust system of a motor vehicle, containing a
effluents to remove injurious chemicals. catalyst for converting pollutant gases into less
62. (c) Sewage is a waste material (such as human urine and harmful ones. Motor vehicles equipped with catalytic
faeces) that is carried away from homes and other converter are advised to use unleaded petrol because
buildings in a system of pipes. Fishes die by sewage lead causes inactivation of catalyst (platinum,
due to increase oxygen competition among them palladium, and rhodium) in a catalytic converter.
which leads to suffocation from low levels of 70. (b) Emissions that are released directly into the
dissolved oxygen in the water. atmosphere from the tailpipes of cars and trucks are
63. (a) Eutrophication is a natural process that occurs in an the primary source of vehicular pollution. But motor
aging lake or pond as that body of water gradually vehicles also pollute the air during the processes of
builds up its concentration of plant nutrients. manufacturing, refuelling, and from the emissions
Artificial or cultural eutrophication occurs when associated with oil refining and distribution of the
human activity introduces increased amounts of fuel they burn. Use of high sulphur petrol and diesel
these nutrients, which speed up plant growth and is not taken for reducing vehicular pollution.
eventually choke the lake of all of its animal life. 71. (d) All the statements regarding biomagnification are
Therefore, lake affected by high levels of artificial correct. Biomagnification is the concentration of
eutrophication will have high nutrient levels, large toxins in an organism as a result of its ingesting other
phytoplankton populations and low oxygen levels plants or animals in which the toxins are more widely
at depth. dispersed.
64. (b) Lake A and B are identical in all aspects except that 72. (d)
Lake A has higher temperature. On the basis of this 73. (a) Excess fluorine in drinking water causes hardening
difference it is clear that Lake B has higher rate of of bones and stiffness of joints, black foot disease is
S-212 Biology
due to arsenic and Itai-Itai disease is due to cadmium deaths.
in contaminated water. 83. (a) An algal bloom is a rapid increase in the population
74. (c) The effect of today's radioactive fallout will be of phytoplankton algae in an aquatic system.
harmful to children of future generation because its Typically only one or a few species are involved and
affects the DNA structure. the bloom is recognized by discoloration of the water
75. (d) Soil erosion is the loss of protective vegetation resulting from the high density of pigmented cells.
through deforestation, over grazing, ploughing and 84. (d) Ozone layer is a deep layer in earth's atmosphere
fire makes soil vulnerable to being swept away by that contains ozone which is a naturally occurring
wind and water. Without soil and plants, the land molecule containing three oxygen atoms. Ozone layer
becomes desert like and unable to support life knanv absorbs most of the ultraviolet radiation reaching
as desertification. Irrigation without proper drainage the earth from the sun. Depletion of ozone layer is
of water leads to water logging in the soil. But the destruction of the upper atmospheric layer of
reforestation is a method of conservation of forest ozone gas, caused by substances formed from
by developing forest cover in the area which has breakdown of ozone depleting substances. Skin
been damaged or cleared during exploitation. related disorders will be more common due to
76. (a) UV-radiations are harmful for living beings. These depletion of ozone layer. It causes aging of skin,
radiations cause harm to the DNA, RNA and protein. damage to skin cell and various types of skin cancer.
It also causes ageing of skin, damage to skin cells 85. (d) Balance in an ecosystem is necessary. When one
and various types of skin cancers. part is upset, the results spread to a much wider
77. (a) Increase in the level of green house gases leads to arena, affecting not only that ecosystem, but others
global warming. This rise in temperature is leading as well.
to deleterious changes in the environment and result 86. (a) Biomagnification refers to the increase in the
in odd climatic conditions, thus leading to rise concentration of a persistant chemical by the
melting of polar ice caps and himalayan snow caps. organisms at successive trophic levels in a food
Over many years, this will result in a rise in sea level chain. The maximum biomagnification is seen among
that can submerge many coastal areas. fishes in an aquatic ecosystem.
78. (a) Radioactive pollution causes gene mutation. It can 87. (d) Soil fertility is the ability of level of soil to grow and
change the base sequence of DNA. support plant life. Fertile soil contains the sufficient
79. (c) In 1962, American biologist, Rachel Carson published minerals and nutrients needed for plant growth. It is
the book Silent Spring, which alleged that DDT often composed of large amounts of topsoil. An
causes cancer and harmed bird reproduction by important characteristic of soil fertility is its sufficient
thinning egg shells. This followed the principles of amount of microorganisms such as bacteria, fungi,
biological magnification, killing higher level earthworms, protozoa and nematodes. Soil fertility
organisms like the birds. DDT is a persistent organic can be destroyed by cutting down forests, acid rain
pollutant with a reported half life of between 2-15 and over grazing and over irrigation.
years, and is immobile in most soils. 88. (b) Ozone is a colourless unstable toxic gas with a
80. (d) Global warming is the increase of Earth's average pungent odour and powerful oxidizing properties. It
surface temperature due to effect of greenhouse is formed from oxygen by electrical discharges or
gases, such as carbon dioxide emissions from ultraviolet light. The ozone layer forms a thick layer
burning fossil fuels or from deforestation, which trap in stratosphere, encircling the earth that has large
heat that would otherwise escape from Earth. It can amount of ozone in it. It protects Earth from the
be controlled by reducing deforestation, cutting harmful radiations that come from the sun. The ozone
down use of fossil fuels. layer has the capability to absorb almost
81. (c) The increase in the level of green house gases (e.g., 97-99% of the harmful ultraviolet radiations that the
CO2, CH4, CFC, N2O etc.) has led to the considerable sun emits and which can produce long term
heating of earth surface leading to global warming. devastating effects on humans beings as well as
The relative contributions of various green house plants and animals.
gases to global warming is 89. (a) Thermal pollution can occur when water is used as a
CO2 (60%) > CH4 (20%) > CFC (14%) > N2O (6%) coolant in nuclear power or industrial plant and then
82. (c) According to Centrol Pollution Control Board (CPCB) returned to the aquatic environment at a higher
particulate size 2.5 micrometers or less in diameter temperature than it is originally. Thermal pollution
are responsible for causing various breathing and can lead to a decrease in the dissolved oxygen level
respiratory problems. They can also cause irritation, in the water while increasing the biological demand
inflammations, damage to the lungs and premature of aquatic organisms for oxygen.
EBD_7209
MOCK TESTS

Mock Test

1
BOTANY 7. Plant cells that are photosynthetically active are found in
the ___________ layer of leaf and are ___________cells.
1. The sum total of chemical reactions occurring in our body is (a) epidermis, parenchymatous
called (b) mesophyll, parenchymatous
(a) Metabolism (b) Homeostasis (c) mesophyll, sclerenchymatous
(c) Irritability (d) Catabolism (d) aerenchyma, collenchymatous
2. An important criterion for modern day classification is
8. Which of the following statements is/are true?
(a) Resemblances in morphology
(i) Uneven thickening of cell wall is characteristic of
(b) Anatomical and physiological traits
sclerenchyma.
(c) Breeding habits
(d) Presence or absence of notochord (ii) Periblem forms the cortex of the stem and the root.
3. In which of the following, all listed genera belong to the (iii) Tracheids are the chief water transporting elements in
same class of Algae? gymnosperms.
(a) Chara, Fucus, Polysiphonia (iv) Companion cell is devoid of nucleus at maturity.
(b) Volvox, Spirogyra, Chlamydomonas (v) The commercial cork is obtained from Quercus suber.
(c) Porphyra, Ectocarpus, Ulothrix (a) (i) and (iv) (b) (ii) and (v)
(d) Sargassum, Laminaria, Gracillaria (c) (iii) and (iv) (d) (ii), (iii) and (v)
4. In the class Phaeophyceae, the plant body is usually 9. Which plant hormone promotes seed, bud dormancy and
attached to the substratum by a _____(A)_____ and has a causes stomatal closure ?
stalk, the _____(B)_____ and leaf like photosynthetic organ
(a) IAA (b) Abscisic acid
the _____(C)_____.
(c) GAl (d) Cytokinin
(a) A – holdfast, B – stipe, C – frond
10. Which of the following divisions of fungi includes club fungi?
(b) A – stipe, B – holdfast, C – frond
(a) Zygomycota (b) Ascomycota
(c) A – frond, B – stipe, C – holdfast (c) Deuteromycota (d) Basidiomycota
(d) A – stipe, B – frond, C – holdfast 11. Identify the factors which affect the rate of diffusion.
5. The region of the stem where leaves are born are called (i) Gradient of concentration.
____________ while ___________ are the portions (ii) Permeability of the membrane.
between two _____________ . (iii) Temperature.
(a) nodes, nodes, internodes (iv) Pressure.
(b) nodes, internodes, nodes (v) Size of diffusing material.
(c) internodes, nodes, nodes (a) (i), (iii) and (iv) (b) (i) and (v)
(d) nodes, internodes, internodes (c) All of these (d) Only (v) is correct
6. Which option is correctly matched with the diagrams? 12. What is the number and positions of insertions of flagella in
class Rhodophyceae?
(a) 2 – 8, equal, apical (b) 2, unequal, lateral
(c) 2 – 6, equal, lateral (d) Absent
(a) A-Valvate, B-Twisted, C-Imbricate, D-Vexillary 13. In biological taxonomy, a ________is a comprehensive
(b) A-Vexillary, B-Valvate, C-Twisted, D-Imbricate treatment of a taxon.
(c) A-Imbricate, B-Vexillary, C-Valvate, D-Twisted (a) flora (b) manuals
(d) A-Twisted, B-Imbricate, C-Vexillary, D-Valvate (c) monograph (d) monogram
MT-2 Mock Test - 1

14. Assertion: A simple leaf has undivided lamina. 20. R.Q. for glucose (Carbohydrates) is
Reason : Leaves showing pinnate and palmate venations (a) 1 (b) 0.5
have various type of incisions. (c) 2 (d) 0.05
(a) If both Assertion and Reason are true and the Reason 21. Which one is the correct option for the labelled diagrams A,
is the correct explanation of the Assertion. B, C and D ?
(b) If both Assertion and Reason are true but the Reason
is not the correct explanation of the Assertion.
(c) If Assertion is true but Reason is false.
(d) If both Assertion and Reason are false.
15. When a meristematic tissue cambium is present inside a
vascular bundle, the bundle is said to be
(a) conjoint (b) open
(c) closed (d) collateral
16. Which one does not differ between a C3 and a C4 plant?
(i) Initial CO2 acceptor
(ii) Extent of photorespiration (a) A – Telocentric chromosome, B – Acrocentric chromo-
some, C – Submetacentric chromosome, D – Metacen-
(iii) Enzyme catalyzing reaction that fixes CO2
tric chromosome
(iv) Presence of Calvin cycle (b) A – Acrocentric chromosome, B – Telocentric chromo-
(v) Leaf anatomy some, C – Metacentric chromosome, D – Submetacen-
(a) (i) and (v) (b) Only (iv) tric chromosome
(c) (ii) and (iii) (d) Only (ii) (c) A – Submetacentric chromosome, B – Metacentric chro-
17. Identify the following figures. mosome, C – Telocentric chromosome, D – Acrocentric
chromosome
(d) A – Metacentric chromosome, B – Submetacentric chro-
mosome, C– Acrocentric chromosome, D – Telocentric
chromosome
22. Match column-I with column-II and choose the correct
option.
Column-I Column-II
A. Algae I. Solanum tuberosum
(a) A – Euglena, B – Paramecium, C – Agaricus B. Fungi II. Equisetum
(b) A – Euglena, B – Planaria, C – Agaricus C. Angiosperm III. Cycas
D. Pteridophyte IV. Chlamydomonas
(c) A – Planaria, B – Paramecium, C – Agaricus
E. Gymnosperm V. Rhizopus
(d) A – Euglena, B – Paramecium, C – Aspergillus
(a) A ® V; B ® IV; C ® I; D ® II; E ® III
18. Nuclear DNA exists as a complex of proteins called ______
(b) A ® IV; B ® V; C ® I; D ® II; E ® III
that condenses into ______ during cellular division.
(a) chromosomes, chromatin (c) A ® IV; B ® I; C ® V;; D ® II; E ® III
(b) chromatids, chromosomes (d) A ® IV; B ® I; C ® V;; D ® III; E ® II
(c) chromophores, chromatin 23. In animals, glucose is stored as ________ while in plants
(d) chromatin, chromosomes stored as ________________.
19. The flower is the reproductive unit in the ________ meant (a) cellulose, starch (b) starch, glycogen
for ____________ reproduction. (c) cellulose, glycogen (d) glycogen, starch
(a) angiosperms and sexual 24. Vessels are absent in
(b) gymnosperms and sexual (a) teak wood (b) shisham wood
(c) algae and asexual (c) Pinus wood (d) sal wood
(d) pteridophytes and asexual
EBD_7209
Mock Test - 1 MT-3

25. Identify the phases from the graph given below that shows (a) A ® I; B ® II; C ® III; D ® V
the change in DNA content during various phases (A to D) (b) A ® II; B ® I; C ® IV;; D ® III
of mitotic cell cycle. (c) A ® III; B ® IV; C ® V;; D ® I
(d) A ® IV; B ® V; C ® II; D ® III
28. The genetically-modified (GM) brinjal in India has been
DNA content 4C developed for
2C (a) insect-resistance
A B C D (b) enhancing shelf life
(c) enhancing mineral content
Time
(d) drought-resistance
A B C D 29. Match column-I with column-II and choose the correct
option.
(a) G2 G1 S M
Column-I Column-II
A. Radial vascular bundle I. Cucurbita pepo
(b) G2 S G1 M
B. Collateral vascular bundle II. Dracaena
C. Bicollateral vascular bundle III. Roots of
(c) G1 S G2 M angiosperms
D. Amphicribral vascular IV. Sunflower stem
(d) M G1 S G2 bundle
E. Amphivasal vascular bundle V. Fern
26. Match column-I with column-II and Column-IIII and choose (a) A ® III; B ® IV; C ® I; D ® V;; E ® II
the correct option. (b) A ® II; B ® III; C ® I; D ® V:: E ® IV
Column-I Column-II Column-III (c) A ® III; B ® IV; C ® V;; D ® I: E ® II
(Element) (Function) (Deficiency (d) A ® III; B ® I; C ® II; D ® IV;; E ® V
Symptom) 30. How many different kinds of gametes will be produced by a
A. Calcium I. Required for ionic- (i) Grey plant having the genotype AABbCC ?
balance. blot on (a) Four (b) Nine
leaves. (c) Two (d) Three
B. Boron II. Essential for constit- (ii) Fruit-yield 31. Acid rain is caused due to increase in concentration of
ution of nucleic acid decreases. (in atmosphere)
C. PhosphorusIII. Required for (iii) Red blots (a) SO2 and NO2 (b) CO and CO2
absorption of calcium. on leaves. (c) CO and SO3 (d) O3 and dust
D. Chlorine IV. Required to activate (iv) Fruit-size 32. Which of the following is considered a hot-spot of
respiratory enzyme. diminishes. biodiversity in India ?
E. Manganese V. Required for synthesis (v) Young (a) Indo-Gangetic Plain (b) Eastern Ghats
(c) Aravalli Hills (d) Western Ghats
of bipolar spindle. root tip
33. Measuring Biochemical Oxygen Demand (BOD) is a method
begin to used for
die. (a) estimating the amount of organic matter in sewage water.
(a) A ® I-iv; B ® II-v; C ® III-iii; D ® IV-i; E ® V-ii (b) working out the efficiency of oil driven automobile
(b) A ® V-v; B ® IV-iv; C ® III-i; D ® II-iii; E ® I-ii engines.
(c) A ® IV-iii; B ® I-iv; C ® V-v; D ® III-ii; E ® II-i (c) measuring the activity of Saccharomyces cerevisiae in
(d) A ® V-v; B ® III-iv; C ® II-iii; D ® I-ii; E ® IV-i producing curd on a commercial scale.
27. Match column-I with column-II and choose the correct (d) working out the efficiency of RBCs about their capacity
option. to carry oxygen.
Column-I Column-II 34. Function of companion cells is
A. Transpiration I. Uses energy to pump (a) providing energy to sieve elements for active transport
molecules against a (b) providing water to phloem
concentration gradient. (c) loading of sucrose into sieve elements by passive
B. Guttation II. Active absorption of water. transport
C. Exudation III. Loss of water vapour from (d) loading of sucrose into sieve elements
plant parts. 35. Which one of the following ecosystem types has the highest
D. Active transport IV. Loss of liquid water from annual net primary productivity?
leaves. (a) Tropical deciduous forest
V. Loss of water from injured (b) Temperate evergreen forest
(c) Temperate deciduous forest
plant parts.
(d) Tropical rain forest
MT-4 Mock Test - 1
36. The impacts of loss of biodiversity include 44. Which of the following statement(s) is/are false?
(i) Decrease in plant production. (i) Pollen grains represent immature male
(ii) Lowered resistance to environmental perturbation gametophyte.etophyte.
(iii) Increased variability in ecosystem processes like water (ii) In angiosperms partially developed male gametophytes
use, pest/disease cycle, plants productivity. are pollinated.
(iv) None of these (iii) Generative cell is siponogenous while vegetative cell
(a) (i) and (ii) (b) (i), (ii), and (iii) is spermatogenous.
(iv) Formation and differentiation of pollen grains is called
(c) (ii) and (iii) (d) Only (iv)
microsporogenesis.
37. Assertion : DNA is associated with proteins.
(v) Pollen grains of some plants produce severe allergy
Reason : DNA binds around histone proteins that form a and respiratory or bronchial diseases.
pool and the entire structure is called a nucleosome. (vi) Pollen grains are poor in nutrients.
(a) If both Assertion and Reason are true and the Reason (a) (i) and (vi) (b) (iii) and (vi)
is the correct explanation of the Assertion. (c) (iv) and (v) (d) (v) and (vi)
(b) If both Assertion and Reason are true but the Reason 45. Assertion : In case of vegetatively propagated crops, pure-
is not the correct explanation of the Assertion. line selection is not required.
(c) If Assertion is true but Reason is false. Reason : Hybrid vigour is mostly used in vegetatively
(d) If both Assertion and Reason are false. propagated plants.
38. What will happen if decomposers are removed from the (a) If both Assertion and Reason are true and the Reason
ecosystem?
is the correct explanation of the Assertion.
(a) Energy cycle is stopped
(b) If both Assertion and Reason are true but the Reason
(b) Mineral cycle is stopped
(c) Consumers cannot absorb solar energy is not the correct explanation of the Assertion.
(d) Rate of decomposition of mineral increases (c) If Assertion is true but Reason is false.
39. The formula for exponential population growth is (d) If both Assertion and Reason are false.
(a) dN/rN = dt (b) rN / dN = dt
(c) dN / dt = rN (d) dt / dN = rN ZOOLOGY
40. Osmotic pressure of a solution is
(a) greater than pure solvent. 46. The practical purpose of classification of living organisms
(b) less than pure solvent. is to
(c) equal to pure solvent. (a) explain the origin of living organsims
(d) less than or greater than pure solvent. (b) trace the evolution of living organsims
41. Himgiri developed by hybridisation and selection for disease (c) name the living organisms
resistance against rust pathogens is a variety of (d) facilitate identification of unknown organisms
(a) Chilli (b) Maize 47. Which one of the following animals is correctly matched
(c) Sugarcane (d) Wheat with its particular named taxonomic category ?
42. The age of pyramid with broad base indicates (a) Tiger - tigris, the species
(a) High percentage of young individuals (b) Cuttle fish - mollusca, a class
(b) Low percentage of young individuals (c) Humans - primata, the family
(c) High percentage of old individuals (d) Housefly - musca, an order
(d) Low percentage of old individuals 48. Glucagon is secreted by
43. Match column-I with column-II and choose the correct (a) b (beta) cells of islets of langerhans
option. (b) a (alpha) cells of islets of langerhans
(c) b cells of pancreas
Column-I Column-II (d) Adrenal cortex
A. Auxin I. Herring sperm DNA
49. Choose the correct names of the different bacteria according
to their shapes.
B. Cytokinin II. Inhibitor of growth
C. Gibberellin III. Apical dominance
D. Ethylene IV. Epinasty
Abscisic
E. V. Induces amylase synthesis
acid
(a) A ® III; B ® I; C ® V; D ® IV; E ® II (a) A – Cocci, B – Bacilli, C – Spirilla, D – Vibrio
(b) A ® IV; B ® V; C ® I; D ® III; E ® II (b) A – Bacilli, B – Cocci, C – Spirilla, D – Vibrio
(c) A ® II; B ® I; C ® V; D ® III; E ® IV (c) A – Spirilla, B – Bacilli, C – Cocci, D – Vibrio
(d) A ® III; B ® I; C ® V; D ® II; E ® IV (d) A – Spirilla, B – Vibrio, C – Cocci, D – Bacilli
EBD_7209
Mock Test - 1 MT-5

50. Which of following is not a protein hormone ? 59. Which one of the following is a wrong matching of a microbe
(a) Relaxin (b) HCG and its industrial product, while the remaining three are correct?
(c) Placental lactogen (d) Estradiol (a) Yeast - Statins
51. Match column-I with column-II and choose the correct (b) Acetobacter aceti - Acetic acid
option. (c) Clostridium butylicum - Lactic acid
Column-I Column-II (d) Aspergillus niger - Citric acid
(Epithelial tissue) (Location) 60. Which of the following is false about Columnar epithelium?
A. Cuboidal I. Epidermis of skin (i) It is made of tall and slender cells.
B. Ciliated II. Inner lining of blood vessels (ii) Free surface may have microvilli.
C. Columnar III. Inner surface of gall bladder (iii) They are found in stomach and intestine and help in
D. Squamous IV. Inner lining of fallopian tube secretion and absorption.
E. Keratinized squamous V. Lining of pancreatic duct (iv) Ciliated epithelium is mainly present in hollow structure
(a) A ® V; B ® IV; C ® II; D ® III; E ® I like bronchioles and fallopian tubes/ products.
(b) A ® III; B ® IV; C ® V;; D ® II; E ® I (v) They have apical nuclei.
(c) A ® V; B ® IV; C ® III; D ® II; E ® I (a) Only (i) (b) Only (iv)
(d) A ® III; B ® IV; C ® V;; D ® I; E ® II (c) (ii) and (iv) (d) (ii) and (iii)
52. Chloramphenicol and Erythromycin (broad spectrum 61. Select the correct statement from the ones given below.
antibiotics) are produced by (a) Barbiturates when given to criminals make them tell the
(a) Streptomyces (b) Nitrobacter truth.
(c) Rhizobium (d) Penicillium (b) Morphine is often given as a pain killer to persons who
53. Which one of the following statements about Human sperm
have undergone surgery.
is correct?
(a) Acrosome has a conical pointed structure used for (c) Chewing tobacco lowers blood pressure and heart rate.
piercing and penetrating the egg, resulting in (d) Cocaine is given to patients after surgery as it stimulates
fertilisation. recovery.
(b) The sperm lysins in the acrosome dissolve the egg 62. The cell junctions called tight, adhering and gap junctions
envelope facilitating fertilisation. are found in
(c) Acrosome serves as a sensory structure leading the (a) muscular tissue (b) connective tissue
sperm towards the ovum. (c) epithelial tissue (d) neural tissue
(d) Acrosome serves no particular function. 63. Which of the following is a pair of viral diseases?
54. Assertion : Sponges belong to Porifera. (a) Common cold, AIDS
Reason : Sponges have canal system. (b) Dysentery, Common cold
(a) If both Assertion and Reason are true and the Reason (c) Typhoid, Tuberculosis
is the correct explanation of the Assertion. (d) Ringworm, AIDS
(b) If both Assertion and Reason are true but the Reason 64. The part of the virus which gives to it the hereditary feature,
is not the correct explanation of the Assertion. is
(c) If Assertion is true but Reason is false. (a) capsid (b) capsomere
(d) If both Assertion and Reason are false. (c) nucleic acid (d) None of these
55. Which one of the following organisms may respire in the 65. A population of organisms has a gene for which there are
absence of oxygen ? two alleles, D and d. The allele frequency of D = 0.8. If this
(a) Azotobacter (b) Clostridium population satisfies all five of the Hardy-Weinberg
(c) Rhizobium (d) Lactobacillus conditions, what are the genotype frequencies that are expected
56. Vectors include in the next generation?
(a) bacterial and plant plasmids (a) DD = 0.04; Dd = 0.32; dd = 0.64
(b) viruses (b) DD = 0.64; Dd = 0.32; dd = 0.64
(c) artificial chromosomes (c) DD = 0.04; Dd = 0.64; dd = 0.32
(d) DD = 0.64; Dd = 0.32; dd = 0.04
(d) All of the above
66. Which of the following organs can be called as a sort of
57. Which of the following is not a step in understanding "Blood bank" ?
Biodiversity ? (a) Lungs (b) Heart
(a) Naming the species (c) Liver (d) Spleen
(b) Looking at other related species
67. Which one of the following groups of animals is Bilaterally
(c) Assessing the species geographic range
symmetrical and Triploblastic?
(d) Quantifying the species genome
(a) Aschelminthes (Round worms)
58. Which is part of pectoral girdle?
(b) Ctenophores
(a) Glenoid cavity (b) Sternum
(c) Sponges
(c) Ilium (d) Acetabulum (d) Coelenterates (Cnidarians)
MT-6 Mock Test - 1
68. Haemophilia is more commonly seen in human males than in 72. Which of the following pair(s) is/are correct?
human females because (a) Semicircular canal - Balancing
(a) a greater proportion of girls die in infancy. (b) Cochlea - Hearing
(b) this disease is due to a Y-linked recessive mutation. (c) Utriculus & sacculus - Balancing & hearing
(d) All of the above
(c) this disease is due to an X-linked recessive mutation.
73. The Ribosomes are made up of
(d) this disease is due to an X-linked dominant mutation.
(a) DNA + Protein (b) RNA + Protein
69. Which of the followin g statement is wrong about
(c) DNA + RNA (d) None of these
Chylomicrons?
74. Match column-I with column-II and choose the correct
(i) Chylomicrons are produced in the epithelial cells of option.
small intestine. Column-I Column-II
(ii) It contains triglycerides, cholesterol and phospholipids. A. Pinna I. Collects vibrations in the air
(iii) It is protein coated small vesicles. which produces sound
B. Ear canal II. Passage for sound wave from
(iv) Chylomicrons are released from the epithelial cell into
pinna to ear drum
lacteals. C. Tympanic membrane III. Transfers sound wave to ear
(a) (i) and (iv) (b) (ii) and (iii) ossicles
(c) (i), (ii), (iii) and (iv) (d) None of these D. Ear Ossicles IV. Increases the efficiency of
70. The following graph is of relative concentrations of the four transmission of sound
hormones present in the blood plasma of a woman during waves to the innear ear
E. Cochlea V. Has hearing receptors
her menstrual cycle. Identify the hormones.
F. Eustachian tube VI. Equalizes the pressure on
both sides of ear drum
(a) A ® I; B ® II; C ® III; D ® IV; E ® V; F ® VI
(b) A ® I; B ® VI; C ® V; D ® IV; E ® III; F ® II
increasing concentration

C D (c) A ® I; B ® II; C ® IV; D ® III; E ® V; F ® VI


(d) A ® I; B ® VI; C ® V; D ® IV; E ® III; F ® II
of hormone

B
75. Air is breathed through
A
(a) Trachea — lungs — larynx — pharynx — alveoli
(b) Nose — larynx — pharynx — bronchus — alveoli —
bronchioles
(c) Nostrils — pharynx — larynx — trachea — bronchi —
day bronchioles — alveoli
(d) Nose — mouth — lungs
76. Match column-I with column-II and choose the correct
A B C D option.
(a) FSH Progesterone LH Oestrogen Column-I Column-II
(b) LH Progesterone FSH Oestrogen A. Limbless reptiles I. Elephant
(c) FSH Oestrogen LH Progesterone B. Jawless vertebrates II. Lamprey
(d) LH Oestrogen FSH Progesterone C. Flightless bird III. Ichthiophis
71. Animal A and B show symmetry D. Largest IV. Ostrich
terrestrial animal V. Cobra
A B E. Blind worm VI. Penguin
(a) A ® II; B ® V; C ® IV; D ® I; E ® III
(b) A ® V; B ® II; C ® IV; D ® I; E ® III
(c) A ® V; B ® II; C ® I; D ® VI; E ® III
(d) A ® V; B ® VI; C ® II; D ® VI; E ® III
77. Match column-I with column-II and choose the correct
option.
Column-I Column-II
A. Tidal volume I. 2500 to 3000 ml
B. Inspiratory II. 1000 ml of air
(a) Bilateral, Asymetrical
reserve volume
(b) Radial, Bilateral
C. Expiratory reserve III. 500 ml of air
(c) Bilateral, Bilateral
D. Residual volume IV. 3400 to 4800 ml of air
(d) Radial, Radial E. Vital capacity V. 1200 ml of air
EBD_7209
Mock Test - 1 MT-7

(a) A ® III; B ® IV; C ® II; D ® I; E ® V 82. At metaphase of mitosis, each chromosome consists of
(b) A ® III; B ® I; C ® II; D ® V;; E ® IV __________ chromatid(s) _________ centromere(s)
(c) A ® III; B ® I; C ® IV;; D ® V; E ® IV _______ kenetochore(s) and _______ molecules of DNA
(d) A ® IV; B ® IV; C ® II; D ® I; E ® III (a) 2, 1, 2, 2 (b) 2, 2, 2, 2
78. In the diagram of excretory system of humna beings given
(c) 2, 1, 1, 1 (d) 2, 1, 2, 1
below, different parts have been indicated by alphabets;
83. Function of Gall bladder is
choose the answer in which these alphabets have been
correctly matched with the parts which they represent (a) storage of bile (b) formation of enzymes
B (c) synthesis of bile (d) formation of bile salts
A 84. The technique called Gamete Intrafallopian Transfer (GIFT)
is recommended for those females
(a) who cannot produce an ovum.
F (b) who cannot retain the foetus inside uterus.
(c) whose cervical canal is too narrow to allow passage for
C the sperms.
(d) who cannot provide suitable environment for
fertilisation.
D 85. Following is the figure of actin (thin) filaments. Identify A, B
and C.
E
(a) A = Kidney, B = Abdominal aorta, C = Ureters,
D = Urinary bladder, E = Urethra, F = Renal pelvis
(b) A = Kidney, B = Abdominal aorta, C = Urethra,
D = Urinary bladder, E = Ureters , F = Renal pelvis
(c) A = Kidney, B = Renal pelvis, C = Urethra, D = Urinary
bladder, E = Ureters, F = Abdominal aorta
(d) A = Kidney, B = Abdominal aorta, C = Urethra, (a) A-Tropomyosin, B-Troponin, C-F-actin
D = Urinary bladder, E = Renal pelvis, F = Ureters (b) A-Troponin, B-Myosin, C-Tropomyosin
79. Given below is the diagrammatic representation of one of (c) A-Troponin, B-Tropomyosin, C-Myosin
the categories of small molecular weight organic compounds (d) A-Troponin, B-Tropomyosin, C-F-actin
in the living tissues. Identify the category shown and the
one blank component “X” in it.
O 86.
HOCH 2 “X”

OH OH
Category Component
(a) Cholesterol Guanine
(b) Amino acid NH2
(c) Nucleotide Adenine
(d) Nucleoside Uracil
80. Which of the following is totally reabsorbed in renal
tubules ? Identify A to E .
(a) Na (b) K (a) A- Pronotum, B-Mesothorax, C-Metathorax,
(c) H2O (d) C6H12O6 D-Tegmina, E-Pleura
81. In a standard ECG which one of the following alphabets is
(b) A- Pronotum, B-Mesothorax, C-Metathorax,
the correct representation of the respective activity of the
D-Tegmina, E-Sterna
human heart?
(c) A- Pronotum, B-Mesothorax, C-Metathorax,
(a) S - start of systole
D-Tegmina, E-Anal cerci
(b) T - end of diastole
(d) A- Pronotum, B-Mesothorax, C-Metathorax,
(c) P - depolarisation of the atria
D-Tegmina, E-Anal style
(d) R - repolarisation of ventricles
MT-8 Mock Test - 1
87. Match column-I with column-II and choose the correct (c) If Assertion is true but Reason is false.
option. (d) If both Assertion and Reason are false.
Column-I Column - II 89. The primary reason why the same basic techniques can be
A. Somatostain I. Pineal gland used to analyze the DNA from species as diverse as bacteria
B. Melatonin II. Corpus luteum and humans is that
C. Aldosterone III. Placenta (a) all cells are identical.
D. Progesterone IV. Adrenal cortex (b) every organism has the same amount of DNA.
E. hCG V. Islet of Langerhans (c) the DNA sequences of all organisms are the same.
VI. Adenohypophysis
(d) DNA has a consistent structure in all organisms.
(a) A ® V; B ® I; C ® VI; D ® III; E ® II
90. Which one of the following statement is true?
(b) A ® I; B ® II; C ® IV;; D ® III; E ® V
(a) The greater the BOD of waste water, more is its polluting
(c) A ® II; B ® VI; C ® IV;; D ® V; E ® III
potential.
(d) A ® V; B ® I; C ® IV;; D ® II; E ® III
(b) The greater the BOD of waste water, less is its polluting
88. Assertion : Chiasmata is formed during diplotene.
Reason : Chiasmata are formed due to deposition of potential.
nucleoproteins. (c) The lesser the BOD of waste water, more is its polluting
(a) If both Assertion and Reason are true and the Reason potential.
is the correct explanation of the Assertion. (d) The lesser the BOD of waste water, less is its
(b) If both Assertion and Reason are true but the Reason polluting potential.
is not the correct explanation of the Assertion.
EBD_7209
Mock Test - 1 MT-9

ANSWER KEY

1 (a) 13 (c) 25 (c) 37 (a) 49 (a) 61 (b) 73 (b) 85 (d)

2 (b) 14 (b) 26 (d) 38 (b) 50 (d) 62 (c) 74 (a) 86 (c)

3 (b) 15 (b) 27 (c) 39 (c) 51 (c) 63 (a) 75 (c) 87 (d)

4 (a) 16 (b) 28 (a) 40 (b) 52 (a) 64 (c) 76 (b) 88 (c)

5 (b) 17 (a) 29 (a) 41 (d) 53 (b) 65 (d) 77 (b) 89 (d)

6 (a) 18 (d) 30 (c) 42 (a) 54 (b) 66 (d) 78 (a) 90 (a)

7 (b) 19 (a) 31 (a) 43 (a) 55 (b) 67 (a) 79 (d)

8 (d) 20 (a) 32 (d) 44 (b) 56 (d) 68 (c) 80 (d)

9 (b) 21 (a) 33 (a) 45 (b) 57 (d) 69 (d) 81 (c)

10 (d) 22 (b) 34 (d) 46 (d) 58 (a) 70 (c) 82 (a)

11 (c) 23 (d) 35 (d) 47 (a) 59 (c) 71 (b) 83 (a)

12 (d) 24 (c) 36 (b) 48 (b) 60 (b) 72 (d) 84 (a)

BOTANY 20. (a) C6 H12 O6 + 6O 2 ¾¾


® 6CO 2 + 6H 2 O
2. (b) Modern day classification is new systematics or
biosystematics which includes all the characteristics Vol.of CO 2 evolved 6
R.Q. = = =1
of organisms gathered from the study of different Vol.of O2 absorbed 6
sections like physiology, ecology, anatomy,
biochemistry, cytology. 28. (a) The genetically modified brinjal in India has been
10. (d) The members of division basidiomycota (class developed for insect resistance. Bt brinjal is a transgenic
basidiomycetes) are commonly called club fungi brinjal that is developed by inserting a crystal gene
because the basidia are club shaped.
from the Bacillus thuringiensis into the brinjal’s genome.
14. (b) A leaf having a single or undivided lamina is called
simple leaf. The lamina can have different types of This process of insertion is accomplished using
incisions, which may reach upto half (-fid), more than Agrobacterium mediated recombination.
half (-partite) or near the base or midrib (-sect). 30. (c) It would make only two types of gametes, these are
Depending upon the pinnate or palmate venation, the ABC & AbC.
incisions ar e known as pinnatifid palmatifid, 31. (a) The main precursors of acid rain are SO2 and NO2 in
pinnatipartite, palmatipartite, pinnatisect and
atmosphere which form H2SO4 (Sulphuric acid) and
palmatisect, etc.
HNO3 (nitric acid) with H2O and these come down with
18. (d) The complex of protein and DNA is called chromatin.
Chromosomes are only present during cell rain. Such rains are called acid rains.
reproduction.
MT-10 Mock Test - 1
32. (d) Hot spots are those ar eas which were rich in 59. (c) Clostridium butylicum industrially produces butyric
biodiversity but are now under threat due to direct or acid.
indirect interference of human activites. These regions 61. (b) Morphine is potent opioid analgesic that is often given
are on the edge to get some of their species extinct due to persons (who have undergone surgery) as a pain
to humans. Western Ghats in India are under threat killer. It is mainly used to relieve severe and persistent
due to continuous developmental activities and Doon pain. It is administrated through mouth, injection or
valley is under threat due to continuous mining activities.
suppositories.
33. (a) BOD is a measure of organic matter present in water. It
refers to amount of O2 consumed by microbes to 63. (a) Common cold, AIDS is a pair of viral diseases. Viruses
decompose all the organic matter in 1 L of water at 20°C are a very common type of agents. Viruses are the
for 5 days. The greater the BOD of waste water, more is smallest life-form existing, since they are not even a
its polluting potential. single cell. It is almost like they are not alive at all. They
34. (d) Function of companion cell is to load sugar and amino are small strands of DNA-like cell material. A virus
acids into sieve elements. These cells use transmem- consists mostly of RNA and cannot survive without
brane proteins to take up by active transport. host cells.
35. (d) They are found in the equatorial regions where rainfall 65. (d) We are told that the allele frequency for D = 0.8;
exceeds 140 cm. The warm humid climate supports broad therefore, the frequency of d = 0.2. These values are
leaved evergreen plants. Productivity is very high the p and q that we need to calculate the genotype
(12000 k. cal/m 2 / year). The vegetation show frequencies in the next generation. Using the Hardy-
stratification into two or more well defined layers. Weinberg equation, p2 (DD) = 0.64, 2pq (Dd) = 0.32,
3 and q2 (dd) = 0.04.
37. (a) A chain of DNA has 140 base pairs, make 1 turns and
4 67. (a) Aschelminthes is bilaterally symmetrical and
twist around a histone octamer forming nucleosome. triploblastic. These are mostly aquatic, free living or
The core of nucleosome consists of 4 histones H2A,
parasitic. Their body is three layered which is ectoderm,
H2B, H3 and H4.
dN mesoderm and endoderm.
39. (c) The formula of exponential growth is = rN where 68. (c) This disease is due to an X-linked recessive mutation.
dt
Males suffer this disorder since they have only one X
dN chromosome and hence express any trait present on
is the rate of change in population size, r is the
dt this chromosome.
biotic potential and N is the population size. 75. (c) The pathway of inhaled air is - Nostrils - pharynx
41. (d) ‘Himgiri’ has been developed by hybridisation and (common passage for food & air) - larynx (voice box) -
selection for disease resistance against rust pathogens
trachea (the wind pipe) - bronchi (2 for each side lungs)
is a variety of wheat. It is resistant to leaf/stripe rust
and hill bunt. - bronchioles (give arise to alveolar ducts) - alveoli (the
45. (b) In case of vegetatively propagated crops, pure line exchange site for gases in the form of small sacs or pouches).
selection is not required. Pure-line selection is useful 79. (d) A combination of a nitrogen base with a pentose
only for sexually reproducing plants. Hybrid vigour is sugar is known as nucleoside. The nitrogen base
most profitably used in vegetatively propagated crops combines with the sugar molecule at its carbon atom 1¢
because they do not involve sexual reproduction and in a glycosidic bond (C – N – C) by one of its nitrogen
hence loss of hybrid superiority. atoms (usually 1 in pyrimidines and 9 in purines).
Dependin g upon th e type of pentose sugar,
ZOOLOGY nucleosides are differentiated into ribonucleosides and
46. (d) Biological classification is the scientific arrangement deoxyribonucleosides.
of organisms in a hierarchial series of groups and 81. (c) In a standard ECG, the P-wave is a small upward wave
subgroups on the basis of similarities and differences that indicates the depolarisation of the atria. This is
in their traits. It helps in building evolutionary pathways caused by the activation of SA node.
and in identifying new organisms. 84. (a) Gamete intrafallopian transfer (GIFT) is recommended for
47. (a) Tiger and tigris both are from same genus with particular those females who cannot produce an ovum. In this
taxonomic category. process, the eggs of the donor woman are removed and
48. (b) Glucagon is secreted by a (alpha) cells of islets of transferred into fallopian tube of another woman in the
langerhans. form of mixture with sperm who cannot produce ovum,
53. (b) Acrosome is a small pointed structure at the tip of but can provide suitable environment for fertilization. Thus
nucleus. It breaks down just before fertilization, in GIFT, site of fertilization is fallopian tube, not laboratory.
releasing hydrolytic enzymes that assist penetration 88. (c) The point of attach ment between homologous
between follicle cells that surrounds the ovum, thus chromosomes after the partial dissolution of
facilitating fertilization. nucleoprotein complex are called chiasmata. It occurs
54. (b) Sponges belong to Porifera and they have characteristic during diplotene substage of prophase I.
canal system. 89. (d) The fact that DNA is structured the same way in all
58. (a) Glenoid cavity is a shallow concavity on the lateral
known organisms means that similar methods can be
side of pectoral girdle in which the head of humerus
used to study the hereditary material.
fits making the shoulder joint.
EBD_7209
Mock Test

2
BOTANY 6. The most important feature of all living systems is to
(a) utilize oxygen to generate energy
1. Binomial nomenclature means (b) replicate the genetic information
(a) one name given by two scientists (c) produce gametes
(b) one scientific name consisting of a generic and specific (d) utilize solar energy for metabolic activities
epithet 7. Consider the following statements regarding
(c) two names, one latinised, other of a person Photosynthesis.
(d) two names of the same plant
(i) ATP formation during photosynthesis is termed as
2. Which pair of the following belongs to basidiomycetes ?
photophosphorylation.
(a) Puffballs and Claviceps
(b) Peziza and Stink borns (ii) Kranz anatomy pertains to leaf.
(c) Morchella and Mushrooms (iii) Reduction of NADP to NADPH occurs during Calvin
(d) Birds nest fungi and Puffballs. cycle.
3. Which two points are known as the twin characteristics of (iv) In a chlorophyll molecule magnesium is present in
growth? phytol tail.
(i) Increase in mass Of the above statements.
(ii) Differentiation
(a) (i) and (ii) are correct
(iii) Increase in number of individuals
(iv) Response to stimuli (b) (iii) and (iv) are correct
(a) (i) and (ii) (b) (i) and (iv) (c) (i) and (iii) are correct
(c) (ii) and (iii) (d) (i) and (iii) (d) (i) and (iv) are correct
4. Choose the collection of terms that completes the following 8. Soil can easily become deficient in____________
sentence : Plants are____ organisms, they frequently show because these ions are negatively charged and do not
a ______ spatial distribution, and their population density stick to negatively charged clay particles.
is most appropriately expressed in terms of ______ . (a) Nitrate (b) Calcium
(a) modular; clumped; biomass (c) Ammonium (d) Magnesium
(b) modular; random; individuals per unit area
9. In a dicotyledonous stem, the sequence of tissues from the
(c) modular; uniform; biomass
outside to the inside is-
(d) unitary; uniform; biomass
(a) Phellem-pericycle-endodermis-phloem
5. Which of the following statement(s) is/are correct?
(b) Phellem-phloem-endodermis-pericycle
(i) Organisms living in oceans, lakes and rivers do not (c) Phellem-endodermis-pericycle-phloem
face any water-related problems. (d) Pericycle-phellem-endodermis-phloem
(ii) Euryhaline can tolerate a wide range of salinities. 10. Which one of the following generally acts as an antagonist
(iii) Stenohaline are restricted to a narrow range of salinities. to gibberellins?
(a) Zeatin (b) Ethylene
(iv) No fresh water animals cannot live for long in sea water
(c) ABA (d) IAA
but sea animals can live in fresh water for long time 11. Which is the correct option for the all given characteristics
because of osmotic balance of Fungi ?
(a) All are correct (b) All are false (i) It includes unicellular as well as multicellular fungi.
(c) Only (iv) (d) (i), (iii) and (iv) (ii) In multicellular forms hyphae are branched and septate.
MT-12 Mock Test - 2
(iii) Conidiophore produces conidia (spores) exogenously 18. Find out the pairs which are correctly matched.
in chain. Column-I Column-II
(iv) Sexual spores are ascopores produced endogenously A. Primary succession I. Autotrophs
in chain. B. Climax community II. Community that has
(v) Fruiting body is called ascocarp. completed succession
(a) Phycomycetes (b) Sac fungi C. Consumer III. Colonization of a new
(c) Club fungi (d) Fungi imperfecti environment
12. Assertion : Net primary productivity is gross primary D. Producer IV. Animals
productivity minus respiration. (a) A ® (III); B ® (II); C ® (IV); D ® (I)
Reason : Secondar y productivity is produced by (b) A ® (III); B ® (I); C ® (IV); D ® (II)
heterotrophs. (c) A ® (I); B ® (III); C ® (II); D ® (IV)
(a) If both Assertion and Reason are true and the Reason (d) A ® (II); B ® (III); C ® (IV); D ® (I)
is the correct explanation of the Assertion. 19. The expressions given below shows the summary equations.
(b) If both Assertion and Reason are true but the Reason +
+ NAD
NADH + H
is not the correct explanation of the Assertion.
(A) Pyruvate C2H5OH + CO2
(c) If Assertion is true but Reason is false.
(d) If both Assertion and Reason are false. (B) C6H12O6 + NAD+ + 2ADP + 2 iP + 2C3H4O3 + 2ATP +
13. If the forest cover is reduced to half, what is most likely to 2NADH + 2H+
happen on a long term basis? (C) Pyruvic acid + 4NAD+ + FAD+ + 2H2O + ADP + Pi ?
3CO2 + 4NADH + 4H+ + ATP + FADH2
(a) Tribals living in these areas will starve to death.
(b) Cattle in these and adjoining areas will die due to lack Categorise the summary equations under respective phases.
of fodder. A B C
(c) Large areas will become deserts. (a) Krebs’ cycle Glycolysis Fermentation
(d) Crop breeding programmes will suffer due to a reduced (b) Glycolysis Krebs’ cycle Fermentation
availability of variety of germplasm. (c) Fermentation Krebs’ cycle Glycolysis
14. Increase in CO2 concentration around leaf results in (d) Fermentation Glycolysis Krebs’ cycle
(a) Rapid opening of stomata 20. Water potential of pure water and its solution are
(b) Partial closure of stomata (a) 0 and 1 (b) 0 and 0
(c) Complete closure of stomata (c) 0 and more than 1 (d) 0 and less than 1
(d) No effect on stomatal opening 21. Double fertilisation leading to initiation of endosperm in
15. Green house gases include angiosperms require
(a) CO2, CFC, CH4 and NO2 (a) fusion of one polar nucleus and the second male gamete
only
(b) CO2, O2, N2, NO2 and NH3
(b) fusion of two polar nuclei and the second male gamete
(c) CH4, N2, CO2 and NH3 (c) fusion of four or more polar nuclei and the second male
(d) CFC, CO2, NH3 and N2 gamete only
16. Sacred groves are found in (d) all the above kinds of fusion in different angiosperms
(i) Khasi and Jaintia Hills in Meghalaya 22. In bryophytes, male and female sex organs are called ______
(ii) Aravalli Hills of Rajasthan and ______ respectively.
(iii) Western ghat regions of Karnataka and Maharashtra (a) microsporangia; macrosporangia
and Sarguja, Chanda and Bastar areas of Madhya (b) male strobili; female strobili
Pradesh (c) antheridia; archegonia
(iv) None of these (d) androecium; gynoecium
(a) (i) and (ii) (b) (i), (ii), and (iii) 23. Seeds are regarded to be the product of sexual reproduction
(c) (ii), (iii) and (iv) (d) Only (iv) because they
17. In Gymnosperm, endosperm is formed by (a) can be stored for a long period.
(a) fusion between a male gamete and two polar nuclei. (b) give rise to new plants.
(b) fusion between a male gamete and a polar nuclei. (c) are the result of fusion of male gamete with the female
(c) fusion between egg and male gamete. gamete.
(d) germination of megaspore. (d) None of these
EBD_7209
Mock Test - 2 MT-13

24. What is common between Chloroplasts, Chromoplasts and 29. Assertion : The product of the first reaction of the Kreb's
Leucoplasts? cycle is citric acid, a six carbon compound.
(a) Presence of pigments Reason : The first reaction of the Kreb's cycle is the
(b) Possession of thylakoids and grana condensation of acetyl CoA with oxaloacetate.
(c) Storage of starch, proteins and lipids (a) If both Assertion and Reason are true and the Reason
(d) Ability to multiply by a fission-like process is the correct explanation of the Assertion.
25. In Krebs’ cycle GTP is formed in (b) If both Assertion and Reason are true but the Reason
(a) substrate level phosphorylation is not the correct explanation of the Assertion.
(c) If Assertion is true but Reason is false.
(b) oxidative phosphorylation
(d) If both Assertion and Reason are false.
(c) photophosphorylation 30. Which of the following statement(s) is/are true about
(d) decarboxylation gemmae?
26. Select the characters which are not applicable to the family (i) These are specialised structures by which asexual
Solanaceae? reproduction take place in liverworts.
(i) Epipetalous and Syngenesious anthers (ii) They are green and multicellular.
(ii) Bicarpellary and Syncarpous ovary (iii) They develop in small receptacles called gemma cups.
(iii) Oblique overy with Axile placentation (iv) They detach from parent body and germinate to form
(iv) Stamens six, arranged in two whorls. new individuals.
(v) Bicarpellary, Syncarpous and Inferior ovary (a) (i) and (ii) (b) (ii) and (iii)
(a) (ii) and (iii) are correct (c) (i), (ii) and (iii) (d) All of these
(b) (i), (iv) and (v) are correct 31. Which of the following shows the correct graph of arithmetic
(c) (ii), (iv) and (v) are correct growth?
(d) (i) and (iii) are correct
(a) (b)
27. Assertion : Ethylene causes climacteric ripening of fruits.
Reason : Climacteric fruits show a rise in respiration at the
Growth Growth
time of ripening.
(a) If both Assertion and Reason are true and the Reason
is the correct explanation of the Assertion. Time Time
(b) If both Assertion and Reason are true but the Reason
(c) (d)
is not the correct explanation of the Assertion.
(c) If Assertion is true but Reason is false.
(d) If both Assertion and Reason are false. Growth Growth
28. Identify the types of simple tissue indicated by A, B, C and D.
Time Time

32. Translocation of sugars in flowering plants occurs in the


form of
(a) Glucose (b) Sucrose
(c) Fructose (d) Maltose
33. Refer the figure given below and select the option which
gives correct words for A, B, C and D.

(a) A – Parenchyma, B – Collenchyma, C – Fibre


(Sclerenchyma), D – Sclereid (Sclerenchyma)
(b) A – Collenchyma, B – Parenchyma, C – Fibre
(Sclerenchyma), D – Sclereid (Sclerenchyma)
(c) A – Parenchyma, B – Collenchyma, C – Sclereid
(Sclerenchyma), D – Fibre (Sclerenchyma)
(d) A – Collenchyma, B – Parenchyma, C – Sclereid
(Sclerenchyma), D – Fibre (Sclerenchyma)
MT-14 Mock Test - 2
A B C D 38. Conversion of sugar into alcohol during fermentation is due
(a) K Ammonification Animal biomass Plant biomass to the direct action of
(b) NH3 Ammonification Plant biomass Animal (a) temperature
biomass
(b) micro-organisms
(c) CO2 Denitrification Animal biomass Plant biomass
(d) CHO Nitrification Plant biomass Animal (c) concentration of sugar solution
biomass (d) enzyme zymase
34. Plants have supporting roots coming out of the lower nodes 39. Identify the phyllotaxy.
of the stem known as __________ roots.
(a) prop (b) stilt
(c) tap (d) adventitious
35. The diagram below shows ATP synthesis through
chemiosmosis.
(a) A-Alternate, B - Opposite, C - Whorled
(b) A- Whorled, B - Opposite, C -Alternate
Cyt. (c) A-Alternate, B - Whorled, C - Opposite
b&f
(d) A-Whorled, B -Alternate, C - Opposite
40. Match Column-I with Column-II
Column-I Column-II
A. Diffusion I. Hydrophilic substances
lumen
B. Osmosis II. Shrinkage of protoplasm
C. Imbibition III. Semipermeable membrane
D. Plasmolysis IV. Free movement of ions and
gases
(a) A ® (II); B ® (I); C ® (IV); D ® (III)
(b) A ® (IV); B ® (III); C ® (I); D ® (II)
(c) A ® (III); B ® (I); C ® (IV); D ® (II)
(d) A ® (II); B ® (III); C ® (IV); D ® (I)
Which option shows the correct labelling of A, B, C and D in 41. Which of the following statements are correct about Calyx?
the diagram ? (a) Calyx is the outermost whorl of the flower and are called
(a) A - F1, B - Thylakoid membrane, C - Photosystem (I), sepals.
D - Photosystem (II) (b) Sepals are green, leaf like and protect the flower in the
(b) A - F0, B - Thylakoid membrane, C - Photosystem (I), bud stage.
D - Photosystem (II) (c) The calyx may be gamosepalous (sepals free) or
(c) A - F1, B - Thylakoid membrane, C - Photosystem (II), polysepalous (sepals united)
D - Photosystem (I) (d) Both (a) and (b).
(d) A - F0, B - Thylakoid membrane, C - Photosystem (II), 42. Assertion : In Mirabilis, selfing of F1 pink flower plants
D - Photosystem (I) produces same phenotypic and genotypic ratio.
36. The process of mitosis is divided into 4 phases. Identify the Reason : Flower colour gene shows incomplete dominance.
correct order in which these phases appear in mitosis (a) If both Assertion and Reason are true and the Reason
(a) Anaphase, Metaphase, Telophase and Prophase is the correct explanation of the Assertion.
(b) Telophase, Anaphase, Metaphase and Prophase (b) If both Assertion and Reason are true but the Reason
(c) Metaphase, Prophase, Anaphase and Telophase is not the correct explanation of the Assertion.
(d) Prophase, Metaphase, Anaphase and Telophase (c) If Assertion is true but Reason is false.
37. RQ (respiratory quotient) is defined as (d) If both Assertion and Reason are false.
(a) Volume of CO2 evolved = volume of O2 consumed 43. Which of the following statements are not true?
Volumeof O2 consumed (i) Cork cambium is otherwise called phellogen.
(b) Volumeof CO 2 evolved (ii) Cork is otherwise called phellem.
Volumeof CO2 evolved (iii) Secondary cortex is otherwise called periderm.
(c) Volumeof O2 consumed (iv) Cork cambium, cork and secondary cortex are
collectively called phelloderm.
Volume of O2 evolved
(d) (a) (iii) and (iv) (b) (i) and (ii)
Volumeof CO2 consumed (c) (ii) and (iii) (d) (ii) and (iv)
EBD_7209
Mock Test - 2 MT-15

44. Assertion : Each molecule of ribulose-1, 5-bisphosphate 52. About which day in a normal human menstrual cycle does
fixesonemolecule of CO2. rapid secretion of LH (Popularly called LH-surge) normally
Reason : Three molecules of NADPH and two ATP are occurs?
required for fixation of one molecule of CO2. (a) 14th day (b) 20th day
(c) 5 day
th
(d) 11th day
(a) If both Assertion and Reason are true and the Reason
53. In the ABO system of blood groups, if both antigens are
is the correct explanation of the Assertion. present but no antibody, the blood group of the individual
(b) If both Assertion and Reason are true but the Reason would be
is not the correct explanation of the Assertion. (a) B (b) O
(c) If Assertion is true but Reason is false. (c) AB (d) A
54. The chemical method of contraception includes
(d) If both Assertion and Reason are false.
(a) jellies only
45. Which of the following structures in Pinus are haploid?
(b) creams and foams only
(a) Megaspore, integument, root
(c) oral contraceptives only
(b) Endosperm, megaspore, pollen grain
(c) Pollen grain, leaf, root (d) all of the above
(d) Megaspore, endosperm, embryo 55. What is the correct sequence of sperm formation?
(a) Spermatogonia, Spermatocyte, Spermatozoa, Spermatid
(b) Spermatogonia; Spermatozoa, Spermatocyte, Spermatid
ZOOLOGY (c) Spermatogonia, Spermatocyte, Spermatid, Spermatozoa
46. Two common characters found in centipede, cockroach and (d) Spermatid, Spermatocyte, Spermatogonia, Spermatozoa
crab are 56. Which of the following conditions is responsible for increase
in ventilation rate of lungs ?
(a) jointed legs and chitinous exoskeleton
(a) Increase of CO2 content in inhaled air
(b) green gland and tracheae
(b) Increase of CO2 content in exhaled air
(c) book lungs and antennae
(c) Decrease of O2 content in inhaled air
(d) compound eyes and anal cerci (d) Decrease of O2 content in exhaled air
47. The most abundant prokaryotes helpful to humans in making 57. The diagram shows an important concept in the genetic
curd from milk and in production of antibiotics are the ones implication of DNA. Fill in the blanks A to C.
categorised as
(a) Cyanobacteria
(b) Archaebacteria (a) A-translation B - transciption C-Erwin Chargaff
(c) Chemosynthetic autotrophs (b) A-transcription B - translation C-Francis Crick
(d) Heterotrophic bacteria (c) A-translation B - extension C-Rosalind Franklin
48. Dikaryon formation is characteristic of (d) A-transcription B - replication C-James Watson
(a) Ascomycetes and Basidiomycetes 58. Parkinson’s disease (characterized by tremors and
(b) Phycomycetes and Basidiomycetes progressive rigidity of limbs) is caused by degeneration of
(c) Ascomycetes and Phycomycetes brain neurons that are involved in movement control and
(d) Phycomycetes and Zygomycetes make use of neurotransmitter
49. Which gas is responsible for the puffed-up appearance of (a) acetylcholine (b) norepinephrine
dough ? (c) dopamine (d) GABA
(a) CO2 (b) O2 59. Which one of the following is commonly used in transfer of
(c) SO2 (d) NO2 foreign DNA into crop plants?
50. Neurons receive signals through their __________ and send (a) Meloidogyne incognita
signals to other neurons through their __________. (b) Agrobacterium tumefaciens
(a) dendrites ... receptors (c) Penicillium expansum
(b) end feet ... cell bodies and dendrites (d) Trichoderma harzianum
(c) cell bodies and dendrites ... axons 60. Select the answer which correctly matches the endocrine
(d) transmitter vesicles ... axons gland with the hormone it secrets and its function/deficiency
51. Which one of the following pairs of structures distinguishes symptom:
a nerve cell from other types of cells? Endocrine Hormone Function/deficiency
(a) Vacuoles and Fibres gland symptom
(b) Flagellum and Medullary sheath (a) Posterior Growth Oversecretion
(c) Nucleus and Mitochondria pituitary Hormone Stimulates abnormal
(d) Perikaryon and Dendrites. (GH) growth
MT-16 Mock Test - 2
(b) Thyroid Thyroxine Lack of iodine in diet 66. Which option is true for A, B, C and D?
gland results in goitre
(c) Corpus Testosterone Stimulates spermato-
luteum genesis
(d) Anterior Oxytocin Stimulates uterus
pituitary contraction during child
birth
61. The type of epithelium found in oesophagus, cornea, vagina
and urethra is
(a) stratified squamous epithelium
(b) ciliated epithelium
(c) stratified columnar epithelium
(d) glandular epithelium
62. Which one of the following is an example of Ex-situ A B C D
conservation? (a) Tail fibres Head Sheath Collar
(a) Wildlife sanctuary (b) Seed bank (b) Sheath Collar Head Tail fibres
(c) Sacred groves (d) National park (c) Head Sheath Collar Tail fibres
63. Match column-I with column-II. (d) Collar Tail fibres Head Sheath
Column-I Column-II 67. Roquefort cheese is produced with the help of
A. Somatostain I. Pineal gland (a) Yeast
B. Melatonin II. Corpus luteum (b) Rhizopus nigricans
C. Aldosterone III. Placenta (c) Aspergillus niger
D. Progesterone IV. Adrenal cortex (d) Penicillium roquefortii
68. Which of the following statements are correct?
E. hCG V. Islet of Langerhans
(i) Ca+2 is necessary for blood coagulation.
VI. Adenohypophysis
(ii) Coagulation in blood vessel is prevented during normal
(a) A ® (V); B ® (I); C ® (VI); D ® (III); E ® (II)
condition by heparin.
(b) A ® (I); B ® (II); C ® (IV); D ® (III); E ® (V) (iii) Clotting of blood involves changes of fibrinogen to
(c) A ® (II); B ® (VI); C ® (IV); D ® (V); E ® (III) fibrin by thrombin.
(d) A ® (V); B ® (I); C ® (IV); D ® (II); E ® (III) (iv) Blood clotting involves cascading process involving a
64. The diagram represents the reproductive organ of male number of factors present in the active form always.
cockroach. Choose the correct combination of labelling (a) (i), (iii) and (iv) (b) (ii) and (iv)
(c) (i), (ii) and (iii) (d) (iii) and (iv)
69. Which one of the following combination is mismatched?
A
(a) Glycocalyx - May be capsule or slime layer
B (b) Pili - Reproduction
C
D (c) Cell wall - Protective, determines shape, prevents from
bursting
(a) A – 8th sternum, B - Anal cercus, C - 10th tergum, (d) Flagella, Pili and Fimbriae - Surface structures of
D - Anal style bacterial cell
(b) A - 10th tergum, B - Anal cercus, C - Anal style, D - 8th 70. A sagittal section of human-brain is shown here. Identify at
sternum least two labels from A-D.
(c) A - Anal style, B - Anal cercus, C - 10th tergum, D- 8th
sternum
(d) A - Anal cercus, B - 8th sternum, C - 10th tergum,
D - Anal style.
65. Which of the following statements about Restriction
enzymes is false?
(a) They work on DNA extracted from all types of organ-
isms.
(b) They are used to glue together short segments of DNA. (a) A – Cerebral hemispheres;
B – Cerebellum
(c) They come in many varieties, each with its own DNA
(b) C – Mid brain; D – Cerebellum
target sequence. (c) A – Cerebrum; C – Pons
(d) They are highly specific for their DNA target sequences. (d) B – Corpus callosum; D – Medulla
EBD_7209
Mock Test - 2 MT-17

71. If concentration of CO2 is more the curve of oxygen will 77. Consider the diagram given below :
shift towards
(a) Right (b) Left
(c) Central (d) None of these
72. The figure shows a section of human ovary. Select the option
which gives the correct identification of A and B with
function/characteristic:

(a) A – Primary oocyte – it is the prophase – I of the meiotic


division
(b) B – Corpus luteum – secretes progesterone
Parts labelled as 'A', 'B', 'C', 'D', and 'E' respectively indicate
(c) A – Tertiary follicle – forms Graafian follicle
(d) B – Corpus luteum – secretes estrogen (a) Femur, Ilium, Tibia, Pubis and Sacrum
Ectoderm (b) Pubis, Tibia, Femur, Ilium and Sacrum
73. Mesoglea
Endoderm (c) Ilium, Femur, Tibia, Pubis and Sacrum
(d) Pubis, Femur, Tibia, Ilium and Sacrum
78. Which of the following statements is/are not true?
(i) In Urochordata, notochord is present only in larval tail.
(ii) In Cephalochordata, notochord extends from head to
tail region.
(iii) Branchiostoma belongs to Hemichordata.
Mesoderm
A B (iv) Only one class of living members, class cyclostomata
The above diagram shows the germs layer. The animals represents the super class agnatha
having structures shown in the figures A and B are (a) (i), (ii) and (iv) only
respectively called (b) (iii), (iv) and (i) only
(a) Diploblastic, Triploblastic (c) (iii) only
(b) Triploblastic, Diploblastic (d) (i) and (iv) only
(c) Diploblastic, Diploblastic 79. Which one of the following pairs of items correctly belongs
(d) Triploblastic, Triploblastic to the category of organs mentioned against it?
74. In ECG, what does ‘T’ wave represent? (a) Thorn of Bougainvillea and tendril of Cucurbita -
(a) Diastole of auricles Analogous organs
(b) Diastole of ventricles (b) Nictitating membrane and blind spot in human eye -
(c) Systole of ventricles Vestigial organs
(d) Diastole of auricles and ventricles (c) Nephridia of earthworm and malpighian tubules of
75. The effectiveness of an enzyme is affected least by Cockroach - Excretory organs
(a) temperature (d) Wings of honey bee and wings of crow - Homologous
(b) concentration of the substrate organs
(c) original activation energy of the system 80. The number of chromatids in a chromosome at anaphase is
(d) concentration of the enzyme (a) 2 in mitosis and 1 in meiosis
76. Which one of the following statements is true regarding (b) 1 in mitosis and 2 in meiosis
Digestion and Absorption of food in humans? (c) 2 each in mitosis and meiosis
(a) Fructose and amino acids are absorbed through (d) 2 in mitosis and 4 in meiosis
intestinal mucosa with the help of carrier ions like Na+. 81. When a person is suffering from poor renal reabsorption
(b) Chylomicrons are small lipoprotein particles that are then which of the following will not help in the maintenance
transported from intestine into blood capillaries. of blood volume
(a) Decreased glomerular filtration
(c) About 60% of starch is hydrolysed by salivary amylase
in our mouth (b) Increased ADH secretion
(c) Decreased arterial pressure in kidney
(d) Oxyntic cells in our stomach secrete the proenzyme
pepsinogen. (d) Increased arterial pressure in kidney
MT-18 Mock Test - 2
82. A ribose (but not deoxyribose) nucleotide is 87. If both parents are carriers for thalassemia, which is an auto-
(a) Cytosine - pentose sugar - phosphate somal recessive disorder, what are the chances of pregnancy
(b) Guanine - pentose sugar - phosphate resulting in an affected child?
(c) Thymine - pentose sugar - phosphate (a) 50% (b) 25%
(c) 100% (d) no chance
(d) Uracil - pentose sugar - phosphate
83. Which enzymes are likely to act on the baked potatoes eaten 88. O
|| O
by a man, starting from the mouth and as it moves down the HO – P – OCH2 N-base
|
alimentary canal? OH A
(a) Salivary maltase ® carboxypeptidase ® trypsinogen
(b) Pancreatic amylase ® salivary amylase ® lipases
(c) Disaccharidase like maltase ® lipases ® nucleases
(d) Salivary amylase ® pancreatic amylase ® Identify A.
disaccharidases (a) Glycosidic bond (b) Phosphate bond
(c) Ester bond (d) Ionic bond
84.
89. The blood leaving the lungs has all its haemoglobin
oxygenated and gives up oxygen to the tissues, because
(a) the tissues can absorb O2 from oxyhaemoglobin
(b) O2 -concentration in tissues is higher and CO 2
concentration lower as compared to lungs
(c) oxyhaemoglobin undergoes reduction
(d) O 2 -concentration in tissues is lower and CO 2
concentration higher than in lungs.
Anatomical regions of human stomach are – 90. Match column - I with column -II
(a) A - Fundus; B - Pyloric; C - Cardiac Column-I Column-II
(b) A - Cardiac; B - Fundus; C - Pyloric A. Basophils I. Phagocytes
(c) A - Fundus; B - Cardiac; C - Pyloric
B. Neutrophils II. Secrete histamin,
(d) A - Pyloric; B - Fundus; C - Cardiac
serotonin, heparin and
85. Assertion : HIV infection can be avoided by use of condoms.
involved in inflammatory
Reason : Condoms secrete anti-viral interferons.
response
(a) If both Assertion and Reason are true and the Reason
C. Monocytes III. Resist infections and are
is the correct explanation of the Assertion. also involved in allergic
(b) If both Assertion and Reason are true but the Reason reaction
is not the correct explanation of the Assertion. D. Eosinophils IV. Immunity
(c) If Assertion is true but Reason is false. E. Lymphocytes V. 60 - 65
(d) If both Assertion and Reason are false.
(a) A ® (II); B and C ® (I); D ® (III); E ® (IV)
86. Which one of the following is correct pairing of a body part
and the kind of muscle tissue that moves it ? (b) A ® (II); B and C ® (III); D ® (I); E ® (IV)
(a) Biceps of upper arm–Smooth muscle fibres (c) A ® (III); B and C ® (I); D ® (II); E ® (IV)
(b) Abdominal wall–Smooth muscle (d) A ® (IV); B and C ® (III); D ® (I); E ® (II)
(c) Iris–Involuntary smooth muscle
(d) Heart wall–Involuntary unstriated muscle
EBD_7209
Mock Test - 2 MT-19

ANS W ER KEY
1 (b ) 14 (b) 27 (b) 40 (b ) 53 (c) 66 (c) 79 (c)
2 (d ) 15 (a) 28 (a) 41 (d ) 54 (d) 67 (d) 80 (b )
3 (d ) 16 (d) 29 (a) 42 (a) 55 (c) 68 (c) 81 (d )
4 (c) 17 (d) 30 (d) 43 (a) 56 (a) 69 (b) 82 (d )
5 (c) 18 (a) 31 (b) 44 (c) 57 (b) 70 (c) 83 (d )
6 (b) 19 (d) 32 (b) 45 (b ) 58 (c) 71 (a) 84 (c)
7 (a) 20 (d) 33 (b) 46 (a) 59 (b) 72 (b) 85 (c)
8 (a) 21 (b) 34 (b) 47 (d ) 60 (b) 73 (a) 86 (b )
9 (c) 22 (c) 35 (d) 48 (a) 61 (c) 74 (d) 87 (b )
10 (c) 23 (c) 36 (d) 49 (a) 62 (b) 75 (c) 88 (c)
11 (b ) 24 (d) 37 (c) 50 (c) 63 (d) 76 (a) 89 (d )
12 (b ) 25 (a) 38 (d) 51 (d ) 64 (a) 77 (d) 90 (a)
13 (c) 26 (b) 39 (a) 52 (a) 65 (b) 78 (d)

BOTANY 24. (d) Though the flagella are of different structure, they serve
the same role in prokaryotes and eukaryotes.
1. (b) Binomial Nomenclature means the scientific name of any 27. (b) In most fruits the rate of respiration will undergo a sharp
organism consisting of a generic epithet and a specific rise and then fall near the end of ripening. Kidd and West
epithet. termed this phenomenon "climacteric rise". The climacteric
2. (d) The class Basidiomycetes includes those members that acts as a trigger that sets in progress those changes that
produce their basidia and basidiospores on or in a rapidly transform the fruit from an unripe to a ripe condition.
basidiocarp. Finally, application of ethylene to unripe fruit will bring on
6. (b) Replication of the genetic information causes transfer a premature climactric and accelerate ripening.
of genetic information from one generation to the next. 29. (a) The first reaction of the Krebs cycle is the condensation
8. (a) NO3–, is negatively charged and not tightly bound to of acetyl CoA (2 C compound) with oxaloacetate
soil particles. (4 C compound). Citric acid, a 6 carbon-compound is
10. (c) Gibberellins and ABA are antagonistic with each other. the first product of Krebs cycle.
ABA counteracts many effects of gibberellins like 42. (a) F2 phenotypic and genotypic ratio in monohybrid cross
induction of hydrolases and a-amylases in barley involving incomplete dominance is
seedlings. 1 : 2 : 1
12. (b) Net primary productivity is the rate of organic matter RR Rr rr
built up or stored by producers in their bodies per unit (red) (pink) (white)
time and area. Net productivity is equal to gross primary 44. (c) Each molecule of ribulose-1, 5-biphosphate fixes one
productivity minus loss due to respiration and other molecule of carbon dioxide with the addition of water,
reasons. Rate of increase in energy containing organic thereby resulting in the formation of two molecules of
matter or biomass by heterotrophs or consumers per 3-phosphoglyceric acid (3-PGA). The fixation and
unit time and area is known as secondary productivity. reduction of one molecule of CO2 requires three
13. (c) If the forest cover is reduced to half, large areas will molecules of ATP and two of NADPH, coming from the
become deserts, on a long term basis. photochemical reactions.
21. (b) The second male gamete entering the ovary fuses with 45. (b) Because endosperm is haploid (n) and formed before
two haploid polar nuclei to form triploid primary fertilization and megaspore mother cell divides
endosperm nucleus which develops into endosperm. reductionally to form a linear tetrad of haploid
This fusion of two male gametes with two different megaspores and microspore (= pollen grain) is the first
structures (egg and secondary nucleus) in the same stage of the gametophyte (n).
female gametophyte is called double fertilization.
MT-20 Mock Test - 2
spherical projection, which is situated below the mid-
ZOOLOGY
brain and upper side of the medulla oblongata. It acts as
46. (a) Jointed legs and chitinous exoskeleton are the common a relay centre among different parts of brain. B and D
characters found in centipede, cockroach and crab. are thalamus and spinal cord respectively.
47. (d) The most abundant prokaryotes are helpful to humans 71. (a) Oxygen tends to displace CO2 so that the curve shifts
in making curd from milk and in production of antibiotics more to the right.
are the heterotrophic bacteria. Lactobacillis converts 72. (b) The corpus luteum is a temporary endocrine structure in
milk into curd. female mammals that is involved in the production of rela-
50. (c) Dendrites generally receive inputs and conduct signals tively high levels of progesterone. ‘A’ marked in the figure
towards the cell body, whereas axons conduct signals shows primary follicle, a layer of granulosa cells, surrounds
away from the cell body. each primary oocyte. A large number of these follicles de-
51. (d) The cytoplasm immediately surrounding the nucleus is generate during the phase from birth to puberty.
loaded with protein synthetic machinery and is called 76. (a) Fructose and amino acids are absorbed through
perikaryon, dendrites are usually shorter, tapering and intestinal mucosa with the help of carrier ions like Na +.
much branched processes which may be one to several. Carbohydrates are absorbed as monosaccharides
These two are only present in nerve cells. (simple sugars such as glucose, fructose, and galactose
52. (a) At 14th day of normal human menstrual cycle rapid that cannot be further broken down by hydrolysis) or
secretion of LH hormone normally occurs. as disaccharides (such as sucrose, lactose, maltose, and
53. (c) Blood group AB is also known as the universal recipient. dextrin that can be hydrolyzed to two monosaccharides).
55. (c) In testis, the immature male germ cells or spermatogonia These simpler molecules, however, must be obtained
(2n) multiply by mitotic divide and increase in number. by the breaking down of polysaccharides, (complex
Some spermatogonia (2n) known as primary spermato- carbohydrates) that contain many monosaccharides.
cytes divide meiotic division to form secondary sper- Chief among these is amylase, a starch that accounts
matocytes (n). The secondary spermatocytes undergo for 20 percent of dietary carbohydrate.
second meiotic division to produce spermatid which
79. (c) Nephridia of earthworm and malphigian tubules of
are transformed into spermatozoa (sperms) by the pro-
cockroch belong to excretory organs. Earthworm has a
cess called spermiogenesis.
well developed excretory system which is composed of
57. (b) large number of minute, coiled and glandular,
segmentally arranged excretory tubules called the
In this question A is transcription, B - translation C - nephridia. Malphigian tubules of cockroach are
Francis Crick (central dogma) It is unidirectional flow of extremely fine yellowish unbranched thread like
information DNA to mRNA (transcription) and then structures present at the junction of midgut and hindgut.
decoding the information present in mRNA in the 81. (d) Increased arterial pressure in kidney will promote the
formation of polypeptide chain or protein (translation). filtration but the person is suffering from poor renal
59. (b) Agrobacterium tumefaciens is the causal agent of crown
reabsorption, so the volume of glomerular filtrate and
gall disease (the formation of tumours) in over 140 species
urine will increase and person would be unable to
of dicot. This disease is caused by a DNA plasmid
maintain the volume of blood.
(Ti plasmid) carried by bacterium and transferred to the
plant cells. Ti plasmid is widely used in plant engineering 83. (d) Chemical process of digestion started in the oral cavity
as a vector in order to inject gene in host plant to form by the hydrolytic action of the carbohydrate (potato
transgenic plant. contains starch) splitting enzyme, the salivary amylase.
60. (b) • Growth hormone secreted by Anterior pituitary Carbohydrates in the chyme are hydrolysed by
• Corpus leutum secrete Progesterone pancreatic amylase into disaccharides.
• Oxytocin is secreted by Posterior pituitary 85. (c) The use of condoms has shown a decrease in the
transmission of AIDS because condoms are contraceptive.
62. (b) Ex-situ conservation is the conservation of selected
organism in places outside their natural homes. They 86. (b) The structure of the abdominal wall is similar in principle
include off site collection and gene banks. to the thoracic wall. There are three layers, an external,
In situ conservation, on the other hand, is the internal and innermost layer. The vessels and nerves lie
conservation of endangered species in their natural between the internal and innermost layers. The abdomen
habitat. Biosphere reserves, National parks, Wildlife can be divided into quadrants or nine abdominal regions.
sanctuaries and sacred groves all are examples of In Pain felt in these regions may be considered to be direct
situ conservation. or referred to abdominal wall made up of smooth muscles.
65. (b) Ligases are the enzymes used to glue together DNA 87. (b) Genotype of carrier parents is –
fragments. Aa (male parent) × Aa (female parent)
67. (d) Roquefort cheese is produced with the help Penicillium AA ® normal child (25%)
roquefortii. Aa ® carrier child (50%)
70. (c) Cerebrum is the first and most developed part of fore aa ® affected child (25%)
brain. It makes 2/3 part of total brain. Pons is a small
EBD_7209
Mock Test

3
BOTANY (b) In Gymnosperms and Angiosperms they have no
independent free-living existence.
1. An important function of Botanical gardens is (c) Both (a) and (b)
(a) Providing beautiful area for recreation (d) In bryophytes, pteridophytes and angiosperms they
(b) One can observe tropical plants over there have free-living life. They remain in sporangia which
(c) They allow ex-situ conservation of germ plasm are retained on sporophytes.
(d) They provide natural habitat to wildlife 8. Assertion : Magnesium is important in photosynthesis and
2. Which of the following cell membrane components serve as carbohydrate metabolism.
recognition signals for interactions between cells? Reason : Mg++ is involved in the synthesis of nucleic acids.
(a) Recognition proteins (a) If both Assertion and Reason are true and the Reason
(b) Glycolipids or glycoproteins is the correct explanation of the Assertion.
(c) Phospholipids (b) If both Assertion and Reason are true but the Reason
(d) Integral proteins is not the correct explanation of the Assertion.
3. Which of the following meristems is responsible for (c) If Assertion is true but Reason is false.
extrastelar secondary growth in dicotyledonous stem?
(d) If both Assertion and Reason are false.
(a) Phellogen
9. Which of the following is the most significant difference
(b) Intrafascicular cambium
between Mitosis and Meiosis?
(c) Interfascicular cambium
(a) Chromosomes are duplicated before mitosis.
(d) Intercalary meristem (b) Meiosis is not followed by cytokinesis.
4. Nitrogenase enzymes are extremely sensitive to _________ (c) Homologous pairs of chromosomes are split up in
molecules. meiosis.
(a) hydrogen (b) oxygen (d) A spindle formed of microtubules moves the
(c) water (d) CO2 chromosomes in mitosis.
5. Which of the following statements are correct regarding 10. What does the shape of the given age pyramids
facilitated diffusion? (I to III) reflect about the growth status of populations ?
(i) It is a very specific process.
(ii) It is a passive process.
Post-reproductive
(iii) It helps the substances, hydrophilic in nature, to be
transported across the membrane Reproductive
(iv) It is faster than active process.
(a) All are correct Pre-reproductive
(b) (i) and (iv) are correct (a) (b) (c)
I II III
(c) (i) (ii) and (iv) are correct
(d) All are correct except (iv) I II III
6. Na+ / K+ pump in a cell is an example of (a) Declining Stable Expanding
(a) osmosis (b) diffusion (b) Stable Expanding Declining
(c) passive transport (d) active transport (c) Expanding Stable Declining
7. What is true about male and female gametophyte in plant (d) Declining Expanding Stable
kingdom ? 11. Cyclic photophosphorylation results in the formation of
(a) In bryophytes an d pteridophytes they have (a) ATP and NADPH (b) ATP, NADPH and O2
independent free-living existence. (c) ATP (d) NADPH
MT-22 Mock Test - 3
12. Which of the following statements about herbarium is correct? 19. Denitrification is carried by bacteria
(a) It is a store house of collected plant specimens that are (a) Pseudomonas and Thiobacillus
dried and preserved on sheets. (b) Nitrosomonas and Nitrococcus
(b) Herbarium sheets contain information about date and (c) Nitrosomonas and Nitrobacter
place of collection, names, family, collector’s name etc. (d) Pseudomonas and Nitrococcus
(c) It serves as quick referral systems in taxonomical 20. Phellogen and Phellem respectively denote
studies. (a) Cork and Cork cambium
(b) Cork cambium and Cork
(d) All of the these
(c) Secondary cortex and Cork
13. Which of the following arrangement is correct from the point
(d) Cork and Secondary cortex
of view of decreasing biodiversity in angiosperms (N), fungi 21. Which one of the following areas in India, is a hot spot of
(F), pteridophytes (P) and algae (A) biodiversity ?
(a) N > F > P > A (b) N > F > A > P (a) Eastern Ghats (b) Gangetic Plain
(c) F > N > P > A (d) F > N > A > P (c) Sunderbans (d) Western Ghats
14. A bicollateral vascular bundle has which of the following 22. During the post-fertilisation period the ovules develop into
arrangement of tissues ? __________ and the ovary matures into a _________.
(a) Outer phloem - Outer xylem - Middle cambium - Inner (a) A – seeds; B – fruit (b) A – fruit; B – seeds
xylem - Inner phloem (c) A – flower; B – seed (d) A – seeds; B – flower
(b) Outer xylem - Outer cambium - Middle phloem - Inner 23. If a colour blind woman marries a normal visioned man, their
cambium - Inner xylem sons will be
(a) one-half colour blind and one-half normal
(c) Outer cambium - Outer phloem - Middle xylem - Inner
(b) three-fourths colour blind and one-fourth normal
phloem - Inner cambium (c) all colour blind sons
(d) Outer phloem - Outer cambium- Middle xylem - Inner (d) all normal visioned
cambium - Inner phloem 24. Which of the following statments are true about virues ?
15. Assertion : The two cotyledons in seed are embryonic leaves. (i) Viruses are obligate parasites.
Reason : The embryo contains radicle and plumule. (ii) Viruses can multiply only when they are inside the living
(a) If both Assertion and Reason are true and the Reason cells.
is the correct explanation of the Assertion. (iii) Viruses cannot pass through bacterial proof filters.
(b) If both Assertion and Reason are true but the Reason (iv) Viruses are made up of protein + DNA or RNA (never
is not the correct explanation of the Assertion. both DNA and RNA).
(c) If Assertion is true but Reason is false. Choose the answer from the following options
(a) (i) and (ii) (b) (i), (ii) and (iii)
(d) If both Assertion and Reason are false.
(c) (i), (ii) and (iv) (d) All of these
16. Assertion : Water potential is new term for diffusion pressure
25. Assertion : A Sanctuary is formed for the conservation of
deficit. animals only.
Reason : Both diffusion pressure deficit and water potential Reason : Restricted human activities are allowed in
have a negative value. sanctuaries.
(a) If both Assertion and Reason are true and the Reason (a) If both Assertion and Reason are true and the Reason
is the correct explanation of the Assertion. is the correct explanation of the Assertion.
(b) If both Assertion and Reason are true but the Reason (b) If both Assertion and Reason are true but the Reason
is not the correct explanation of the Assertion. is not the correct explanation of the Assertion.
(c) If Assertion is true but Reason is false. (c) If Assertion is true but Reason is false.
(d) If both Assertion and Reason are false. (d) If both Assertion and Reason are false.
17. Which of the following pair is a combination of lowest and 26. The Km value of the enzyme is the value of the substrate
highest energy molecules ? concentration at which the reaction reaches to
(a) zero (b) 2 Vmax
(a) Glucose and Pyruvic acid
(c) ½ Vmax (d) ¼ Vmax
(b) Acetyl CoA and Palmitic acid 27. Bryophytes are different from fungi in having
(c) Glucose and Malic acid (a) land habit
(d) Malic acid and Acetyl CoA (b) sterile jacket layers
18. Which of the following pair comes under the group (c) multiflagellate gametes
Chrysophytes? (d) gametophytic plant body
(a) Diatoms and Euglena 28. Desert regions are characterized by ___ centimeters of rainfall
per year.
(b) Euglena and Trypanosoma
(a) less than 5 (b) less than 15
(c) Diatoms and Desmids (c) less than 25 (d) over 50
(d) Gonyaulax and Desmids
EBD_7209
Mock Test - 3 MT-23

29. Match the following and choose the correct option. 30. Which is not the function of Cell wall?
Column-I Column-II I. Provides shape to the cell.
A. Chromosomes are moved I. Pachytene II. Protects the cell from mechanical damage and infection.
to spindle equator
III. Helps in cell to cell interaction.
B. Centromere splits and II. Zygotene
chromatids apart IV. Provides barrier to undesirable macromolecules.
C. Pairing between homologous III. Anaphase (a) Only (i) (b) Only (iv)
chromosomes takes place (c) Only (ii), (iii) and (iv) (d) None of these
D. Crossing between homologous IV. Metaphase 31. Which one o23f the following pairs, is not correctly matched?
chromosomes (a) IAA - Cell wall elongation
(a) A ® I; B ® II; C ® III; D ® IV (b) Abscissic acid - Bolting
(b) A ® II; B ® III; C ® IV;; D ® I (c) Gibberellic acid - Stem elongation
(c) A ® IV; B ® III; C ® II; D ® I
(d) Cytokinin - Cell division
(d) A ® III; B ® I; C ® IV;; D ® II

32. Which one of the following option correctly represents the type of Life cycle patterns?

Sporophyte
(2n) Zygote Sporophyte
(2n) (2n)
Syngamy Meiosis Zygote (2n)
Meiosis
A Syngamy C
Zygote (2n) B Spores (n)
Gametogenesis Spores (n)
Gametogenesis Gametogenesis
Syngamy
Meiosis Gametophyte (n)
Gametophyte (n)
(n)

(a) A - Diplontic, B - Haplodiplontic, C - Haplontic (b) A - Haplodiplontic, B - Haplontic, C - Diplontic


(c) A - Haplontic, B - Diplontic, C - Haplodiplontic (d) A - Diplontic, B - Haplontic, C - Haplodiplontic

33. The conditions necessary for vernalization are (c) Antipodal cells, Secondary nuclei, Stigma, Chalaza,
(i) high temperature and water Style
(ii) low temperature (d) Antipodal cells, Secondary nuclei, Chalaza, Stigma,
(iii) water and carbon dioxide Style
(iv) oxygen and water 35. Which of the following statements are correct about Leaf?
(a) (i), (ii), and (iii) (b) (i) and (ii) (i) Leaf is a lateral, generally flattened structure borne on
(c) (ii) and (iv) (d) (i) and (iii) the stem.
34. Identify A, B, C, D and E respectively- (ii) It develops at the node and bears a bud in its axil.
(iii) Leaves originate from root apical meristems and
arranged in an acropetal order.
C (iv) They are the most important vegetative organs for
reproduction.
D
(a) (i) and (ii) (b) (ii) and (iii)
Pollen tube (c) (i), (ii) and (iii) (d) All of these
36. Select the correct statement(s)
E A-cells
(i) IARI has released a mustard variety rich in vitamin C.
B
(ii) Pusa Sawani variety of Okra is resistant to aphids.
Egg cell
(iii) Hairiness of leaves provides resistance to insect pests.
Synergid
(iv) Agriculture accounts for approximately 33% of India's
GDP and employs nearly 62% of the population.
(a) (i) and (ii)
(a) Antipodal cells, Secondary nuclei, Stigma, Style,
(b) (ii) and (iii)
Chalaza
(c) (i), (iii) and (iv)
(b) Antipodal cells, Secondary nuclei, Style, Stigma,
(d) None of these
Chalaza
MT-24 Mock Test - 3
37. Assertion : UAA, UAG and UGA terminate protein synthesis. 44. Assertion : Wilting occurs due to loss in turgidity.
Reason : They are not recognised by tRNA. Reason : Turgor pressure checks the excessive entry of water
(a) If both Assertion and Reason are true and the Reason into cells.
is the correct explanation of the Assertion. (a) If both Assertion and Reason are true and the Reason
(b) If both Assertion and Reason are true but the Reason is the correct explanation of the Assertion.
is not the correct explanation of the Assertion. (b) If both Assertion and Reason are true but the Reason
(c) If Assertion is true but Reason is false. is not the correct explanation of the Assertion.
(d) If both Assertion and Reason are false. (c) If Assertion is true but Reason is false.
38. Match column-I with column-II (d) If both Assertion and Reason are false.
Column-I Column-II
45. Which one of the following processes during decomposition
A. Grana of chloroplast I. Kreb's cycle
is correctly described?
B. Stroma of chloroplast II. Light reaction
C. Cytoplasm III. Dark reaction (a) Humification-Leads to the accumulation of a dark
D. Mitochondrial matrix IV. Glycolysis coloured substance humus which undergoes microbial
(a) A ® IV; B ® III; C ® II; D ® I action at a very fast rate.
(b) A ® I; B ® II; C ® IV;; D ® III (b) Catabolism-Last step decomposition under fully
(c) A ® IV; B ® I; C ® III; D ® II anaerobic condition.
(d) A ® II; B ® III; C ® IV;; D ® I (c) Leaching-Water soluble inorganic nutrients rise to the
39. If you consider the combined processes of photosynthesis top layers of soil.
and cellular respiration, the electrons found in H2O at the
(d) Fragmentation-Carried out by organisms such as
beginning of the light reactions end up attaching to
_______ during respiration. earthworm.
(a) O2 to make new H2O molecule.
(b) NADPH to make new glucose molecule. ZOOLOGY
(c) pyruvate to make ethanol.
(d) electron transport carriers to make O2. 46. Biological organisation starts with
40. Match the following and choose the correct option (a) cellular level
Column-I Column-II
(b) organism level
(Symbols used in floral (Parts of the flower)
formula) (c) atomic level
(A) Å I. Zygomorphic (d) sub-microscopic molecular level
(B) K II. Corolla 47. Which of the following statements regarding nomenclature
(C) C III. Androecium is correct ?
(D) A IV. Actinomorphic (a) Generic name always begins with capital letter whereas
(E) G V. Calyx specific name with small letter.
(F) % VI. Superior ovary (b) Scientific name should be printed in italics.
(a) A - IV; B - V; C - VI; D - I; E - III; F - II (c) Scientific name when typed or handwritten should be
(b) A - IV; B - V; C - III; D - II; E - I; F - VI underlined.
(c) A - V; B - IV; C - III; D - II; E - VI; F - I
(d) All the above
(d) A - IV; B - V; C - II; D - III; E - VI; F - I
48. Which of the following is an example of Amoeboid protozoan?
41. Food chains differ from food webs in that
(i) food chains are single sequence of who eats whom in a (a) Trypanosoma (b) Paramoecium
community. (c) Gonyaulax (d) Entamoeba
(ii) food chains better represent the entire community. 49. Which one of the following phyla is correctly matched with
(iii) food webs represent the complex interaction among its two general characteristics?
food chains. (a) Arthropoda - Body divided into head, thorax
(iv) food chain is the flow of energy in a population. and abdomen and respiration by
(a) (i) and (iii) (b) (i) and (iv)
tracheae
(c) (i), (ii) and (iii) (d) None of these
42. In electron transport system (ETS), which of the following (b) Chordata - Notochord at some stage and
cytochrome reacts with oxygen ? separate anal and urinary
(a) Cyt b (b) Cyt a3 openings to the outside
(c) Cyt b6 (d) Cyt f (c) Echinodermata - Pentamerous radial symmetry
43. Which of the following class of algae is rarely found in fresh and mostly internal fertilization
water? (d) Mollusca - Normally oviparous and
(a) Chlorophyceae (b) Phaeophyceae development thr ough a
(c) Rhodophyceae (d) Both (a) and (b) trochophore or veliger larva
EBD_7209
Mock Test - 3 MT-25

50. Assertion : Blood is colourless in the insects. 57. Assertion : Enzymes lower the activation energy.
Reason : Insect blood has no role in O2 transport. Reason : A substrate molecule can be acted upon by a
(a) If both Assertion and Reason are true and the Reason particular enzyme.
is the correct explanation of the Assertion. (a) If both Assertion and Reason are true and the Reason
(b) If both Assertion and Reason are true but the Reason is the correct explanation of the Assertion.
is not the correct explanation of the Assertion. (b) If both Assertion and Reason are true but the Reason
(c) If Assertion is true but Reason is false. is not the correct explanation of the Assertion.
(d) If both Assertion and Reason are false. (c) If Assertion is true but Reason is false.
51. Adaptive radiation refers to (d) If both Assertion and Reason are false.
(a) evolution of different species from a common ancestor 58. Which one of the following genes is defective in patients
(b) migration of members of a species to different suffering from Severe Combined Immuno-deficiency
geographical areas Syndrome (SCID) ?
(c) power of adaptation in an individual to a variety of
(a) RNAase
environments
(d) adaptations due to geographical isolation (b) ADA
52. Which part of human brain is concerned with the regulation (c) Ribonucleotide reductase
of body temperature? (d) DNAase
(a) Cerebellum (b) Cerebrum 59. Recombinant DNA can be transferred into host cell by
(c) Hypothalamus (d) Medulla Oblongata (a) growing the host cell in growth medium containing
53. Find out the pairs, which are correctly matched.
ampicillin.
Column-I Column-II
A. PCR I. Large scale (b) coating the DNA with carbohydrates so that the cells
culture will engulf the DNA.
B. Bioreactor II. To induce alien (c) treating cells with calcium ions or electrical pulses to
DNA in host cell
increase cell permeability.
C. Gene gun III. Restriction
endonuclease (d) injecting proteins into host cells to make them more
D. Eco RI IV. Amplification of gene permeable.
(a) A ® (IV); B ® (I); C ® (II); D ® (III) 60. Prophase of reduction division is divided into number of
(b) A ® (II); B ® (I); C ® (IV); D ® (III) stages. The correct chronological sequence is
(c) A ® (IV); B ® (I); C ® (III); D ® (II)
(a) Leptotene - Pachytene - Zygotene - Diplotene -
(d) A ® (I); B ® (IV); C ® (II); D ® (III)
Diakinesis
54. Which of the following processes is helped by Bile salts?
(b) Leptotene - Diplotene - Pachytene - Zygotene -
(a) Nucleic acid ¾¾¾¾¾
Nuclease
® Nucleotides ¾¾¾¾¾¾®
Nucleotidase Diakinesis
Nucleosides ¾¾¾¾¾¾¾ ® Sugar + bases (c) Leptotene - Zygotene - Diplotene - Pachytene -
Nucleosidase
Diakinesis
(b) Sucrose ¾¾¾¾®
Sucrase Glucose + Fructose (d) Leptotene - Zygotene - Pachytene - Diplotene -
Diakinesis
(c) Fats ¾¾¾¾ ® Diglycerides ¾¾¾
® Monoglycerides
Lipase 61. Progestasert and LNG-20 are
(a) Implants
Proteins ü
ï Trypsin/Chymotrypsin (b) Copper releasing IUDs
(d) Peptones ý ¾¾¾¾¾¾¾¾¾¾® Dipeptides
Carboxypeptidase (c) Non-medicated IUDs
Proteosesïþ
(d) Hormone releasing IUDs
55. A biologist studied the population of rats in 62. At which stage of HIV infection does one usually show
a barn. He found that the average natality was 250, average
symptoms of AIDS?
mortality 240, immigration 20 and emigration 30. The net
increase in population is (a) When the infecting retrovirus enters host cells.
(a) 15 (b) 05 (b) When viral DNA is produced by reverse trancriptase.
(c) zero (d) 10 (c) When HIV replicates rapidly in helper T-lymphocytes
56. Which National Park is the new home of the Indian one- and damages large number of these cells.
horned rhinoceros ?
(a) Dudhwa (b) Jim Corbett (d) Within 15 day of sexual contact with an infected
(c) Kanha (d) Bandhavgarh person
MT-26 Mock Test - 3
63. Which of the following are necessary for evolution by natural 68. Which one of the following has its own DNA?
selection to take place? (a) Mitochondria (b) Dictyosome
(i) Offspring resemble their parents more than other (c) Lysosome (d) Peroxisome
individuals in the population. 69. Down’s syndrome is caused by an extra copy of chromosome
(ii) Differences among individuals exist and lead to different number 21. What percentage of offspring produced by an
number of successful offspring being produced. affected mother and a normal father would be affected by
(iii) Individuals adjust their development depending on the this disorder?
environment. (a) 25% (b) 100%
(iv) Every individual has a desire to have many offspring. (c) 75% (d) 50%
(v) Populations tend to grow faster than their food supplies. 70. Match column-I with column-II
(a) (i) and (ii) (b) (i) and (v) Column-I Column-II
(c) (ii), (iii) and (iv) (d) (iii) and (iv) A. Ribs are attached I. True ribs
64. Which of the following is a chordate feature, not shared by to the sternum
the non-chordates? ventrally and to
(a) Metamerism (b) Axial organization the vertebrae
(c) Bilateral symmetry (d) Pharyngeal gill slits dorsally.
65. Match column-I with column-II B. Ribs are attached II. False ribs
to sternum
Column-I Column-II
through coastal
A. Operator site I. Binding site for
cartilage (hyaline)
RNA polymerase
of 7th rib
B. Promoter site II. Binding site for
C. Ribs are not III. Floating ribs
repressor molecule
attached to sternum
C. Structural gene III. Codes for enzyme protein
(a) A ® I; B ® II; C ® III
D. Regulator gene IV. Codes for repressor
molecules (b) A ® I; B ® III; C ® II
(a) A ® (II); B ® (I); C ® (III); D ® (IV) (c) A ® II; B ® I; C ® III
(b) A ® (II); B ® (I); C ® (IV); D ® (III) (d) A ® III; B ® II; C ® I
(c) A ® (IV); B ® (III); C ® (I); D ® (II) 71. The diagram given below repesents the T.S. of Gut. Identify
(d) A ® (II); B ® (III); C ® (I); D ® (IV) A, B, C and D
66. Which of the following statements are true/false ?
(i) The blood transports CO 2 comparatively easily
because of its higher solubility.
(ii) Approximately 8.9% of CO2 is transported being
dissolved in the plasma of blood.
(iii) The carbon dioxide produced by the tissues, diffuses
passively into the blood stream and passes into red
blood corpsucles and react with water to form H2CO3.
(iv) The oxyhaemoglobin (HbO2) of the erythrocytes is (a) A - Serosa; B - Muscularis; C - Submucosa; D - Mucosa
basic. (b) A - Muscularis; B - Serosa; C - Submucosa; D - Mucosa
(v) The chloride ions diffuse from plasma into the (c) A - Serosa; B - Muscularis; C - Mucosa; D - Submucosa
erythrocytes to maintain ionic balance. (d) A - Serosa; B - Submucosa; C - Muscularis; D - Mucosa
(a) (i), (iii) and (v) are true, (ii) and (iv) are false. 72. Which of the following represents the action of insulin?
(b) (i), (iii) and (v) are false, (ii) and (iv) are true. (a) Increases blood glucose levels by hydrolysis of
(c) (i), (ii) and (iv) are true, (iii) and (v) are false. glycogen.
(d) (i), (ii) and (iv) are false, (iii) and (v) are true. (b) Increases blood glucose levels by stimulating glucagon
67. Match column -I with column - II. production.
Column-I Column-II (c) Decreases blood glucose levels of forming glycogen.
A. Endometrium I. Copulation chamber in female (d) Increases blood glucose level by promoting cellular
B. Menopause II. Site of implantation of zygote uptake of glucose.
C. Fallopian tube III. Cessation of menstrual cycle 73. Which of the following possesses electric organ and belongs
in female to class Chondrichthyes?
D. Vagina IV. Site of fertilization in female (a) Torpedo (b) Petromyzon
(a) A ® (II); B ® (III); C ® (IV); D ® (I) (c) Trygon (d) Exocoetus
(b) A ® (IV); B ® (II); C ® (III); D ® (I) 74. Which of the following is the contractile protein of a muscle?
(c) A ® (IV); B ® (III); C ® (II); D ® (I) (a) Myosin (b) Tropomyosin
(d) A ® (IV); B ® (III); C ® (I); D ® (II) (c) Actin (d) Tubulin
EBD_7209
Mock Test - 3 MT-27

75. Although much CO2 is carried in blood, yet blood does not (d) A-Receptor, C-Synaptic vesicles
become acidic, because
(a) it is absorbed by the leucocytes. 81.
(b) blood buffers play an important role in CO2 transport.
(c) it combines with water to form H2CO3 which is
neutralized by NaCO3.
(d) it is continuously diffused through tissues and is not
allowed to accumulate.
76. Consider the following four statements (i-iv) regarding
kidney transplant and select the two correct ones out of
these.
(i) Even if a kidney transplant is proper the recipient may
need to take immune suppresants for a long time.
(ii) The cell mediated immune response is responsible for
the graft rejection.
(iii) The B lymphocytes are responsible for rejection of the
graft.
(iv) The acceptance or rejection of a kidney transplant
depends on specific interferons. Identify structures A to D
The two correct statements are: (a) A-Gizzard, B-Crop, C-Hepatic caecae, D-Malpighian
(a) (ii) and (iii) (b) (iii) and (iv) tubules
(c) (i) and (iii) (d) (i) and (ii) (b) A-Crop, B-Gizzard, C-Hepatic caecae, D-Malpighian
77. Which of the following ion is necessary for the contraction tubules
of a muscle and nerve impulse transmission? (c) A- Crop, B-Gizzard, C-Malpighian tubules, D-Hepatic
(a) Na+ (b) K+ caecae
(c) Ca++ and Mg++ ions (d) None of these
(d) A- Gizzard, B-Crop, C-Malpighian tubules, D-Hepatic
78. Which one of the following pairs correctly matches a
caecae
hormone with a disease resulting from its deficiency?
82. The cardiac pacemaker in a patient fails to function normally.
(a) Luteinizing – Failure of ovulation
The doctors find that an artificial pacemaker is to be grafted
(b) Insulin – Diabetes insipidus
in him. It is likely that it will be grafted at the site of
(c) Thyroxine – Tetany
(a) Atrioventricular bundle
(d) Parathyroid – Diabetes mellitus
79. If due to some injury the chordae tendinae of the tricuspid (b) Purkinje system
valve of the human heart is partially non - functional, what (c) Sinuatrial node
will be the immediate effect? (d) Atrioventricular node
(a) The flow of blood into the aorta will be slowed down. 83. The figure shows a diagrammatic view of human respiratory
(b) The ‘pacemaker’ will stop working. system with labels A, B, C and D. Select the option which
(c) The blood will tend to flow back into the left atrium. gives correct identification and main function and/or
(d) The flow of blood into the pulmonary artery will be characteristics.
reduced.
80. A diagram showing axon terminal and synapse is given
below. Identify at least two of the labels A, B, C, D correctly.

(a) B-Pleural membrane-surrounds ribs on both sides to


provides cushion against rubbing.
(b) C-Alveoli-thin walled vascular bag like structures for
exchange of gases.
(c) D-Lower end of lungs-diaphragm pulls it down during
(a) B-Synaptic connection, D-K+
inspiration.
(b) A-Neurotransmitter, B-Synaptic cleft
(d) A-Trachea-long tube supported by complete
(c) C- Neurotransmitter, D-Ca++
cartilaginous rings for conducting inspired air.
MT-28 Mock Test - 3
84. Refer the following diagram and identify the parts of a kidney (v) Fats are absorbed through lymph in the lacteals lpresent
indicated in the intestinal villi.
(a) Only (i) (b) (iii) and (iv)
A (c) (ii) and (iii) (d) Only (iv)
B 87. Match column-I with column-II
C Column-I Column-II
A. Hypothalamus I. Sperm lysins
D B. Acrosome II. Estrogen
C. Graafian follicle III. Relaxin
E D. Leydig cells IV. GnRH
E. Parturition V. Testosterone
(a) A ® (IV); B ® (I); C ® (II); D ® (III); E ® (V)
(b) A ® (II); B ® (I); C ® (IV); D ® (III); E ® (V)
(a) A - cortex, B - nephron, C - pelvis, D - medulla,
(c) A ® (I); B ® (I); C ® (V); D ® (IV); E ® (III)
E - ureter
(d) A ® (IV); B ® (I); C ® (II); D ® (V); E ® (III)
(b) A - cortex, B - medulla, C - nephron, D - pelvis,
88. Antiparallel strands of a DNA molecule means that
E - ureter
(a) one strand turns anti-clockwise
(c) A - nephron, B - cortex, C - medulla, D - ureter,
(b) the phosphate groups of two DNA strands, at their
E - pelvis
ends, share the same position
(d) A - nephron, B - cortex, C - medulla, D - pelvis,
(c) the phosphate groups at the start of two DNA strands
E - ureter
are in opposite position (pole)
85. Assertion : Intercalated discs are important regions of
(d) one strand turns clockwise
cardiac muscle cells.
89. If for some reason, the vasa efferentia in the human
Reason : Intercalated discs function as boosters for muscle
reproductive system gets blocked, the gametes will not be
contraction waves.
transported from
(a) If both Assertion and Reason are true and the Reason
(a) testes to epididymis
is the correct explanation of the Assertion.
(b) epididymis to vas deferens
(b) If both Assertion and Reason are true but the Reason
is not the correct explanation of the Assertion. (c) ovary to uterus
(c) If Assertion is true but Reason is false. (d) vagina to uterus
(d) If both Assertion and Reason are false. 90. Which of the following statements about the neural basis of
86. Which of the following statement(s) is/are wrong about the knee-jerk reflex in humans is false ?
Lymph ? (a) The leg extensor is stimulated to contract by the motor
(i) Lymph is colourful as it has haemoglobin but no RBC. neuron in this reflex loop.
(ii) The fluid present in lymphatic system is called lymph. (b) The only synapses between neurons involved in this
(iii) It contains specialized lymphocytes which are loop occur in the spinal cord.
responsible for immunity of the body. (c) The basic response is a polysynaptic circuit containing
(iv) Lymph is an important carrier for nutrients and several interneurons.
hormones. (d) Conscious action can modify the basic reflex because
of the action of interneurons.
EBD_7209
Mock Test - 3 MT-29

ANSWER KEY
1 (c) 13 (d) 25 (a) 37 (a) 49 (c) 61 (d) 73 (a) 85 (a)
2 (b) 14 (d) 26 (c) 38 (d) 50 (b) 62 (c) 74 (a) 86 (a)
3 (a) 15 (b) 27 (b) 39 (a) 51 (a) 63 (a) 75 (b) 87 (d)
4 (b) 16 (c) 28 (c) 40 (d) 52 (c) 64 (d) 76 (d) 88 (c)
5 (d) 17 (b) 29 (c) 41 (a) 53 (a) 65 (a) 77 (c) 89 (a)
6 (d) 18 (b) 30 (d) 42 (b) 54 (c) 66 (a) 78 (a) 90 (c)
7 (c) 19 (a) 31 (b) 43 (b) 55 (c) 67 (a) 79 (d)
8 (b) 20 (b) 32 (d) 44 (b) 56 (a) 68 (a) 80 (d)
9 (c) 21 (d) 33 (c) 45 (d) 57 (b) 69 (d) 81 (b)
10 (c) 22 (a) 34 (a) 46 (d) 58 (b) 70 (a) 82 (c)
11 (c) 23 (c) 35 (a) 47 (d) 59 (c) 71 (a) 83 (b)
12 (d) 24 (c) 36 (c) 48 (d) 60 (d) 72 (c) 84 (d)

BOTANY has positive value. Water potential is a modern term


coined by Slatyer and Taylor (1960) which is equivalent
2. (b) Both glycolipids and glycoproteins serve as to DPD, but it has a negative value.
recognition signals. 21. (d) Hotspots are the geographical areas where biodiversity
8. (b) Magnesium is a constituent of the chlorophyll is maximum. Two hot spots is India are Western Ghats
molecule, without which photosynthesis would not and North eastern himlayan region.
occur. Many of the enzymes involved in carbohydrate 23. (c) Colour blindness is a X-chromosome linked character.
metabolism require magnesium as an activator. So they’ll be having all colour blind sons and carrier
Magnesium is also an activator for those enzymes daughters.
involved in the synthesis of nucleic acids (DNA, RNA)
from nucleotide polyphosphate.
9. (c) In mitosis, each daughter cell ends up with two of
each chromosome.
11. (c) Cyclic photophosphorylation results in the formation
of ATP. This process is called photophosphorylation,
which occurs in two different ways. Adenosine
triphosphate (ATP) is considered by biologists to be
the energy currency of life. It is the high-energy 25. (a) A sanctuary is an area which is reserved for the
molecule that stores the energy we need to do just protection of wild animals only. The activities like
about everything we do. It is present in the cytoplasm harvesting of timber, collection of minor forest products
and nucleoplasm of every cell, and essentially all the and private ownership rights are allowed, however, such
physiological mechanisms that require energy for activities should not have any adverse effect on animals.
operation obtain it directly from the stored ATP. 27. (b) Bryophytes are different from fungi in having sterile
14. (d) In a bicollateral vascular bundle, the middle xylem is jacket layers.
bounded by outer phloem and outer cambium. The inner 28. (c) Deserts have less than 25 centimeters of rainfall per
face of the xylem has inner cambium and inner phloem. year.
The bicollateral bundle is always open. 44. (b) Flowers, young stems and other softer organs are able
15. (b) In angiosperms, cotyledons are embryonic leaves. to maintain their form due to turgidity or TP (turgor
Embryo also has radicle and plumule which gives rise
pressure). In case of loss of turgidity, the shoots droop
to root and shoot respectively.
down and the leaves show wilting. Turgor pressure
16. (c) The reduction in the diffusion pressure of water in a
(pressure potential or hydrostatic pressure) keeps a
solution over its pure state is called diffusion pressure
check on the excessive entry of water into cells.
deficit or DPD. It is a term coined by Meyer (1938). It
MT-30 Mock Test - 3
45. (d) Humification, catabolism, leaching and fragmentation 72. (c) Insulin is a peptide hormone, which plays a major role
are the steps of decomposition which operate in the regulation of glucose homeostasis. Insulin acts
simultaneously on the detritus. Fragmentation is mainly on hepatocytes and adipocytes (cells of adipose
breaking down detritus into smaller particles by tissue), and enhances cellular glucose uptake and
detritivores like earthworm. By the process of leaching, utilization. Insulin also stimulates conversion of glucose
water soluble inorganic nutrients go down into soil to glycogen (glycogenesis) in the target cells.
horizon and get precipitated as unavailable salts. 74. (a) Actin and tropomyosin are part of thin filaments of
Humification occurs at a very slow rate. skeletal muscle. Tubulin is present in microtubules.
Myosin is muscle protein.
ZOOLOGY 75. (b) CO2 enters RBC and reacts with water to form carbonic
acid. Carbonic acid dissociates to form bicarbonate and
46. (d) Biological organisation starts with sub-microscopic
hydrogen ions. Some bicarbonate ions are transported
moleculer level like viruses, bacteria etc. These
in erythrocytes while some diffuse into the blood
organisms are unable to be seen by naked eyes without
plasma. Exit of bicarbonate ions change the ionic
the help of microscope or even electron microscope.
balance between the plasma and erythrocytes. To
49. (c) Pseudocoelomate is any invertebrate animal whose
restore this balance chloride ions diffuse from plasma
body cavity is a pseudocoel, a cavity between the gut
into erythrocytes. Due to this the pH of blood is
and the outer body wall derived from a persistent
maintained.
blastocoel, rather than a true coelom. Pseudocoelomate
76. (d) Tissue and blood group matching are essential before
animal include the Rotifera and Nematoda.
undertaking kidney transplant. Even if kidney transplant
50. (b) Insect blood is colourless and does not play any role
is proper, the recipient may need to take immuno
in transport of oxygen. Insects have tracheal
suppresant all his/her life. The ability of body to
respiration.
51. (a) Adaptive radiation refers to evolution of different differentiate self and nonself and the cell-mediated
species from a common ancestor. The mammals are immune response is responsible for graft rejection.
adapted for different mode of life i.e. they show adaptive 77. (c) Ca++ is an essential element for the contraction of
radiation. They can be aerial (bat), aquatic (whale and muscles because release of Ca++ ions from sarcoplasmic
dolphins), burrowing or fossorial (rat), cursorial (horse), reticulum trigger the muscle contraction process.
scantorial (squarrel) or arboreal (monkey). The adaptive 78. (a) Luteinizing hormone (LH) stimulates ovulation.
radiation, the term by osborn, is also known as Divergent Deficiency of insulin causes diabetes mellitus.
evolution. Deficiency of ADH or vasopressin causes diabetes
55. (c) Net increase in population : (Natality + Immigration) – insipidus. Deficiency of parathormone causes tetany.
(Mortality + Emigration) Deficiency of thyroxine causes cretinism in infants and
(250 + 20) – (240 + 30) = 270 – 270 = 0 myxoedema in adults.
57. (b) Activation energy is an external supply of energy which 79. (d) Tricuspid valve is the valve in the heart between the
is needed for the initiation of the chemical reaction. right atrium and right ventricle. The valve opens to
Activation energy required for such a large number of allow blood to flow from atrium into the ventricle. Thus
reactions cannot be provided by living systems. if tricuspid valve is partially non-functional, then the
Enzymes lower the activation energy required for a flow of blood into the pulmonary artery will be reduced.
reaction. Enzymes are generally specific for their 80. (d) A-Receptor, C-Synaptic vesicles B is synaptic cleft. A
substrates. synapse is formed by the membranes of a synaptic
59. (c) Treating cells with ampicillin does not make them more neuron and post synaptic neuron, which may or may
permeable to DNA. Coating DNA with lipids (not not be separated by a gap called synaptic cleft. It is
carbohydrates) is used to introduce recombinant DNA filled by fluid called neurotransmitter which are in-
molecules into host cells, injecting DNA (not protein) volved in transmission of impulse at these synapses.
is a method used to place recombinant DNA into best 82. (c) SA node is the natural pacemaker located in the right
cells. atrium. SA node initiates the cardiac impulse. So, artificial
62. (c) When HIV replicates rapidly in helper T-lymphocytes pacemaker will be grafted at the site of SA node.
and damages large number of these cells, at this stage 83. (b) Alveoli are very thin, irregular walled bag like structures
infected persons start showing symptoms of AIDS. for gaseous exchange. Tracheae, bronchi and
64. (d) Chordates show the presence of nerve cord, notochord bronchioles are supported by incomplete cartilaginous
and pharyngeal gill slits. rings. Double layered pleural membrane surrounds the
68. (a) Mitochondria has its own DNA. It is a structure within lungs with pleural fluid between them. It reduces friction
cytoplasm of eukaryotic cells that carries out aerobic on the lung surface.
respiration. It is the site of Kreb’s cycle and ETS. 89. (a) If the vasa efferentia in the human reproductive system
Therefore, it is also called as cell’s energy production gets blocked the gametes will not be transformed from
site. testes to epididymis.
69. (d) 50% of ova will have (n + 1) chromosome which would, 90. (c) The basic knee-jerk response is controlled by a
on fertilisation, yield abnormal zygotes (n + 1) + (n) monosynaptic circuit. Interneurons are only involved
= 2n + 1. in the voluntary or conscious modification of this reflex.
EBD_7209
Mock Test

4
BOTANY 8. Match column-I with column-II
Column-I Column-II
1. Species are considered as A. Apple I. Outer portion of receptacle
(a) real units of classification devised by taxonomists
B. Coconut II. Fleshy thalamus
(b) real basic units of classification
C. Jackfruit III. Thalamus & pericarp
(c) the lowest units of classification
(d) artificial concept of human mind which cannot be D. Guava IV. Endosperm
defined in absolute terms E. Pineapple V. Bract, perianth & seeds
2. Which one of the following is primarily concerned with Cell (a) A ® (II); B ® (III); C ® (IV); D ® (V); E ® (I)
division ? (b) A ® (V); B ® (III); C ® (I); D ® (VI); E ® (II)
(a) GA3 (b) IAA (c) A ® (II); B ® (III); C ® (I); D ® (V); E ® (IV)
(c) Cytokinin (d) IBA (d) A ® (II); B ® (IV); C ® (V), D ® (III); E ® (I)
3. Which of the following factors most often limits the primary
9. The biomass available for consumption by the herbivores
productivity of the Ecosystem ?
and the decomposers is called
(a) Solar radiation/light (b) Oxygen
(a) Net primary productivity
(c) Consumers (d) Nitrogen
(b) Secondary productivity
4. If the pressure potential is +0.16 megapascals (mPa) and the (c) Standing crop
osmotic potential is –0.24 megapascals, then the water
(d) Gross primary productivity
potential would be
10. How much portion of the PAR is captured by the plants?
(a) + 0.4 mPa (b) +0.08 mPa
(a) 5 – 10%
(c) – 0.08 mPa (d) + 0.16 mPa
(b) 7 – 10%
5. Vacuole in a plant cell
(c) 8 – 10%
(a) is membrane-bound and contains storage proteins and
(d) 2 – 10%
lipids
11. Which one of the following organisms is scientifically
(b) is membrane-bound and contains water and excretory
correctly named, correctly printed according to the
substances
International Rules of Nomenclature and correctly described?
(c) lacks membrane and contains air
(a) Musca domestica - The common house lizard, a reptile.
(d) lacks membrane and contains water and excretory
(b) Plasmodium falciparum – A protozoan pathogen
substances
causing the most serious type of malaria.
6. Which one is wrong about the Guard cell?
(c) Felis tigris - The Indian tiger, well protected in Gir
(a) They are modified ground tissue.
forests.
(b) They are chlorophyllous.
(d) E.coli - Full name Entamoeba coli, a commonly
(c) Its outer wall is thin and inner wall is highly thickened.
occurring bacterium in human intestine.
(d) They regulate stomatal movement for transpiration and
12. Pyrenoids in green algal cells are related to
gaseous exchange.
(a) starch formation
7. The cork cambium, cork and secondary cortex are
(b) protein storage
collectively called
(c) general metabolism
(a) Phelloderm (b) Phellogen
(d) enzyme secretion
(c) Periderm (d) Phellem
MT-32 Mock Test - 4
13. Examine the figures given below and select the right options (a) A ® (V); B ® (II); C ® (III); D ® (IV); E ® (I)
out of (a-d); in which all the 4 items A, B, C and D are identified (b) A ® (II); B ® (V); C ® (I); D ® (III); E ® (IV)
correctly (c) A ® (III); B ® (I); C ® (IV); D ® (V); E ® (II)
(d) A ® (V); B ® (IV); C ® (III); D ® (II); E ® (I)
Bud Nodes
Leaf scar 19. Two lakes, A and B are identical in all aspects except that
Buds
Node lake A has higher temperature. Which of the following is
true ?
(a) A has higher rate of Oxygen dissolution
(A) Adventitious (B) (b) B has higher rate of Oxygen dissolution
root (c) Oxygen dissolution of both is the same
(d) Both have same BOD
20. Which of the following is not a product of light reaction of
photosynthesis?
(a) O2 (b) ATP and NADPH2
(c) High-energy electrons (d) Sugar
21. Assertion : Systematics is the branch of biology that deals
with classification of living organisms.
Reason : The aim of classification is to group the organisms.
(a) If both Assertion and Reason are true and the Reason
(C) (D) is the correct explanation of the Assertion.
A B C D (b) If both Assertion and Reason are true but the Reason
(a) Tuber Rhizome Bulb Leaf buds is not the correct explanation of the Assertion.
(b) Offset Sucker Stolon Leaf buds (c) If Assertion is true but Reason is false.
(c) Offset Stolon Sucker Leaf buds (d) If both Assertion and Reason are false.
22. Which of the following statement(s) is/are correct?
(d) Tuber Rhizome Bulbil Leaf buds
14. The Scrubber is used mainly to remove the following gas/es (i) The deficiency of any element can cause multiple
from the exhaust after spraying water/ lime symptoms.
(ii) Same symptoms may be caused by the deficiency of
(a) CO2 (b) SO2
one or several different elements.
(c) O2 & N2 (d) CO & CO2
(iii) The concentration of the essential element below which
15. Growth rings are formed due to activity of plant growth is retarded is termed as critical
(a) extrastelar cambium (b) intrastelar cambium concentration.
(c) interstelar cambium (d) both (b) and (c)
(iv) Chlorosis is the loss of chlorophyll due to deficiency
16. Mark the statement that describes the eco-friendly disposal
of N, K, Mg, Fe, S, Mn, Zn, Mo.
of municipal solid-waste
(v) Different plants respond differently to the deficiency
(a) It should be burnt to completion.
of the same element.
(b) It should be dumped in open waste land. (a) (i), (ii), (iii), (iv) and (v) (b) (i) and (iv)
(c) It should be dumped in sanitary landfills. (c) (i) and (iii) (d) (iii) and (iv)
(d) It should be sorted out into bio-degradable, 23. In the DNA molecule
non- biodegradable and recyclable wastes and treated (a) the total amount of purine nucleotides and pyrimidine
separately. nucleotides is not always equal
17. A population would grow exponentially (b) there are two strands which run parallel in the 5¢ ® 3¢
(a) if it were limited only by density-dependent factors. direction
(b) until it reaches carrying capacity. (c) the proportion of adenine in relation to thymine varies
(c) if there were no limiting factors. with the organism
(d) if it were a population with an equilibrial life history. (d) there are two strands which run anti-parallel one in
18. Match the column-I (Component) with column-II (% of the 5¢ ® 3¢ direction and other in 3¢ ® 5¢
total cellular mass) and identify the correct options 24. Leaves of dicotyledonous plants possess ________
Column-I Column-II venation, while ____________ venation is the characteristic
(Component) (% of the total cellular mass) of most monocotyledons.
A. Water I. 3
(a) reticulate and parallel
B. Proteins II. 70-90
(b) parallel and reticulate
C. Carbohydrates III. 2
D. Lipids IV. 5-7 (c) reticulate and perpendicular
E. Nucleic acids V. 10-15 (d) obliquely and parallel
EBD_7209
Mock Test - 4 MT-33

25. In C4 plant, during photosynthesis C4 acid undergoes 29. Osmosis means movement of
decarboxylation in __________ to produce C3 acid (pyruvic (a) Solute from low concentration to higher concentration
acid) and __________. (b) Solute from higher concentration to low concentration
(a) mesophyll, O2 (b) bundle sheath, CO2 (c) Solvent from low concentration of solution to higher
concentration of solution
(c) grana, CO2 (d) bundle sheath, CO2
(d) Solvent from higher concentration of solution to low
26. Assertion : Leaves of Bryophyllum, Begonia help in concentration of solution
vegetative multiplication. 30. Consider the following statements
Reason : Leaves of these plants possess adventitious buds. (i) In a dicot root, the vascular bundles are collateral and
(a) If both Assertion and Reason are true and the Reason endarch.
is the correct explanation of the Assertion. (ii) The inner most layer of cortex in a dicot root is
(b) If both Assertion and Reason are true but the Reason endodermis.
is not the correct explanation of the Assertion. (iii) In a dicot root, the phloem masses are separated from
(c) If Assertion is true but Reason is false. the xylem by parenchymatous cells that are known as
(d) If both Assertion and Reason are false. the conjunctive tissue.
27. Diagram given below shows stages in embryogenesis in a Of the statements given above
typical dicot (Capsella). Identify structures A to D (a) (i) is true, but (ii) and (iii) are false
respectively (b) (ii) is true, but (i) and (iii) are false
(c) (i) is false, but (ii) and (iii) are true
(d) (iii) is false, but (i) and (iii) are true
A 31. Root pressure is maximum when
B (a) Transpiration is high and Absorption is very low
Zygote (b) Transpiration is very low and Absorption is high
D
(c) Transpiration is very high and Absorption is also high
Heart-Shaped (d) Transpiration and Absorption both are slow
Embryo C 32. Match the growth regulators in column-I with the processes
Globular Mature
Embryo in column-II and choose the correct combination.
Embryo
(a) Suspensor, Radicle, Plumule, Cotyledons Column-I Column-II
(b) Hypophysis, Radicle, Plumule, Cotyledons A. Auxin I. Colouring test in lemon
(c) Suspensor, Plumule, Radicle, Cotyledons
(d) Suspensor, Radicule, Plumule, Hypocotyls B. Gibberellin II. Cell division test in plants
28. Aristotle used simple_______characters to classify plants C. Cytokinin III. Avena curvature test
into trees, shrubs and herbs.
(a) anatomical D. Ethylene IV. Dwarf corn test
(b) biochemical (a) A – (III), B – (IV), C – (II), D – (I)
(c) morphological (b) A – (I), B – (IV), C – (II), D – (III)
(d) physiological (c) A – (IV), B – (III), C – (I), D – (II)
(d) A – (II), B – (I), C – (IV), D – (III)

33. Which of the following is the correct option ?


MT-34 Mock Test - 4

S. No. A B C D
(a) Porphyra Fucus Dictyota Polysiphonia

(b) Polysiphonia Porphyra Dictyota Fucus

(c) Fucus Dictyota Porphyra Polysiphonia

(d) Porphyra Polysiphonia Fucus Dictyota

34. The oxidation of a molecule of FADH2 yields less ATP than (a) A - Hilum, B - Micropyle, C - Radicle, D - Cotyledon,
a molecue of NADH yields because FADH2 E - Plumule
(a) carries fewer electrons. (b) A - Hilum, B - Micropyle, C - Plumule, D - Cotyledon,
(b) is formed in the cytosol and energy is lost when it E - Radicle
shuttles its electrons across the mitochondrial
(c) A - Micropyle, B - Hilum, C - Plumule, D - Cotyledon,
membrane.
E - Radicle
(c) passes its electrons to a transport molecule later in the
chain and at a lower energy level. (d) A - Hilum, B - Micropyle, C - Plumule, D - Radicle,
(d) is the last molecule produced by the Krebs cycle, and E - Cotyledon
little energy is left to be captured. 39. The picture below shows a graph drawn on the parameters
35. Assertion : Mosses are evolved from algae. of growth versus time. A, B, C respectively represent
A
Reason : Protonema of mosses is similar to some green algae.
(a) If both Assertion and Reason are true and the Reason
is the correct explanation of the Assertion. Size C
(b) If both Assertion and Reason are true but the Reason
is not the correct explanation of the Assertion.
(c) If Assertion is true but Reason is false. B
(d) If both Assertion and Reason are false. Time
36. Which of the following options is correct? (a) Exponential Log phase Steady state phase
(a) Pollination gives the guarantee of the promotion of phase
post-pollination events that lead to fertilization.
(b) Steady state Log phase Log phase
(b) The events – “from pollen deposition on stigma until
pollen tubes enter the ovule” are together referred to phase
as pollen-pistil interaction. (c) Log phase Steady Logarithmic phase
(c) Pollen-pistil interaction is a dynamic process involv- state
ing pollen recognition followed by only promotion (not phase
rejection) of the pollen.
(d) Pistil has no ability to recognise the pollen, whether (d) Log phase Lag phase Steady state phase
right or wrong type.
37. In which of the following reaction of glycolysis, a molecule 40. Which one of the following statement(s) is/are false?
of water is removed from the substrate ? (i) C2H4 promotes leaf senescence.
(a) Frucoste-6-phosphate ® Fructose 1, 6-phosphate (ii) C2 H4 speeds the ripening of fruits.
(b) 3-phosphate glyceraldehyde ® 1, 3-biphosphoglyceric (iii) C2H4 causes apical hook formation.
acid (iv) C2H4 promotes horizontal growth of seedling and
(c) PEP ® Pyruvic acid swelling of axis.
(d) 2-phosphoglycerate ® PEP (v) C2H4 promotes male flowers in cucumber thereby
38. Which one of the option is correct? increasing the yield.
(a) All the statements (b) (i) and (v)
C (c) (ii) and (iv) (d) Only (v)
Seed coat
41. Read the following statements and choose the correct option.
Raphe D
(i) Green algae occur in fresh water, brackish water, salt
A E water.
B (ii) Habitat of Brown algae-fresh water (rare), brackish
water, salt water
Seed Seed opened
(iii) Some red algae are found in fresh water, mostly occur
in salt water, some are in brackish water.
EBD_7209
Mock Test - 4 MT-35

(iv) Most of the red algae are multicellular. ZOOLOGY


(v) Red alga may occur in both well lighted regions close
to water-surface and also at great depths in oceans 46. The ‘Birds’ taxonomically represent
where light penetration is little. (a) Family (b) Order
(vi) Cell wall of red algae consists of cellulose + agar. (c) Class (d) Phylum
(vii) 2 – 8, equal and apical flagella in green algae 47. 'Project Tiger' in India was started in
(a) All are correct (a) 1970 (b) 1972
(b) All are false (c) 1981 (d) 1985
(c) (i) and (vi) are correct 48. Ganga and Yamuna Action plan is initiated by
(d) (ii), (iii) and (v) are correct (a) ministry of environment and forest.
42. Assertion : Enzymes are defined as biological proteins. (b) ministry of agriculture.
Reason : Chemically all enzymes are globular proteins. (c) ministry of wild-life conservation.
(a) If both Assertion and Reason are true and the Reason (d) none of these
is the correct explanation of the Assertion. 49. Industrial melanism as observed in Peppered moth proves
(b) If both Assertion and Reason are true but the Reason that
is not the correct explanation of the Assertion. (a) the melanic form of the moth has no selective
(c) If Assertion is true but Reason is false. advantage over lighter form in industrial area
(d) If both Assertion and Reason are false. (b) the lighter-form moth has no selective advantage either
43. Meiosis and Mitosis differ from each other because in meiosis in polluted industrial area or non-polluted area
(a) the four nuclei formed are not similar to parental ones (c) melanism is a pollution-generated feature
(b) homologous chromosomes pair exchange parts (d) the true black melanic forms arise by a recurring random
(c) number of chromosomes gets halved mutation
(d) all of the above 50. Which of these is not correctly matched ?
44. Choose the correct option (a) Gene gun—Biolistic gun
A. Semi Dwarf Wheat I. Sonalika (b) Plasmids—Extrachromosomal DNA
B. Semi Dwarf Rice II. Kalyan sona (c) DNA ligase—Biological scissors
III. IR-8 (d) Bacteriophages—Viruses
IV. Jaya 51. AIDS is caused by HIV that principally infects
V. Taichung Native-1 (a) all lymphocytes (b) activator B cells
VI. Ratna (c) cytotoxic T cells (d) T4 lymphocytes
(a) A ® (I), (III), (V); B ® (II), (IV), (VI) 52. A genetically engineered micro-organism used successfully
in bioremediation of oil spills is a species of
(b) A ® (III), (IV), (V), (VI); B ® (I), (II)
(a) Pseudomonas (b) Trichoderma
(c) A ® (I), (II), (IV); B ® (III), (V), (VI)
(d) A ® (I), (II); B ® (III), (IV), (V), (VI) (c) Xanthomonas (d) Bacillus
45. Identify A, B, C and D of a nutrient cycle. 53. Meiosis in AaBb will produce gametes
(a) AB, aB, Ab, ab (b) AB, ab
B A (c) Aa, bb (d) Aa, Bb
54. Baculoviruses are excellent candidates for
(a) species-specific narrow spectrum pesticidal applications.
Litter
Detritus (b) species-specific broad spectrum pesticidal applica-
tions.
C
(c) species-specific narrow spectrum insecticidal applica-
Uptake
Soil solution tions.
Run off (d) species-specific broad spectrum insecticidal applications.
D
55. What type of human population is represented by the
Rock minerals following age pyramid?

(a) A – Consumers; B – Decomposition; C – Producers


D – Weathering
(b) A – Consumers; B – Weathering; C – Producers;
D – Decomposition
(c) A – Producers; B – Consumers; C – Decomposition;
D – Weathering
(d) A – Consumers; B – Producers; C – Decomposition
D – Weathering
(a) Vanishing population (b) Stable population
(c) Declining population (d) Expanding population
MT-36 Mock Test - 4
56. The genetic material in viruses is (a) If both Assertion and Reason are true and the Reason
(a) Only RNA is the correct explanation of the Assertion.
(b) Only DNA (b) If both Assertion and Reason are true but the Reason
is not the correct explanation of the Assertion.
(c) RNA and DNA both
(c) If Assertion is true but Reason is false.
(d) RNA or DNA i.e. one nucleic acid in a virus (d) If both Assertion and Reason are false.
57. Which one of the following option gives the correct matching 62. Which one of the following is a correct statement?
of a disease with its causative organism and mode of infection? (a) "Bt" in "Bt-cotton" indicates that it is a genetically
Disease Causative Mode of modified organism produced through biotechnology.
Organisms Infection (b) Somatic hybridization involves fusion of two complete
(a) Typhoid Salmonella typhii With inspired air plant cells carrying desired genes.
(b) Pneumonia Streptococcus Droplet (c) The anticoagulant hirudin is being produced from
pneumoniae Infection transgenic Brassica napus seeds.
(c) Elephantiasis Wuchereria Infected water (d) "Flavr Savr" variety of tomato has enhanced the
bancrofti and food production of ethylene which improves its taste.
(d) Malaria Plasmodium Bite of male 63. Select the option which correctly matches the endocrine
vivax Anopheles gland with its hormone and its function:
mosquito Endocrine Hormone Function
58. A normal- visioned man whose father was colour blind, gland
marries a woman whose father was also colour blind. They (a) Ovary FSH stimulates follicular
have their first child as a daughter. What are the chances development and the
that this child would be colour blind ? secretion of estrogens.
(a) 100% (b) zero percent (b) Placenta estrogen initiates secretion of the milk.
(c) Corpus estrogen essential for maintenance
(c) 25% (d) 50%
luteum of endometerium
59. Select the correct statement(s)
(d) Leydig androgen initiates the cells production
(i) The essence of Darwinian theory about evolution is
of sperms.
natural selection.
64. What is true about Ribosomes?
(ii) The rate of appearance of new forms is not linked to
the life cycle or the life span. (a) The prokaryotic ribosomes are 80S, where “S” stands
(iii) Adaptative ability is not inherited. for sedimentation coefficient.
(iv) Mutation is random and directionless. (b) These are composed of ribonucleic acid and proteins.
(a) (ii) and (iii) (b) (i) and (iv) (c) These are found only in eukaryotic cells.
(c) None of these (d) All of these (d) These are self-splicing introns of some RNAs.
60. Identify A, B and C of a nucleosome. 65. Monocondylic skull, warm blooded animals with air sacs are
present in
A B (a) reptilia (b) birds
(c) amphibia (d) mammalia
66. The diagram represents the Human larynx. Choose the
C correct combination of labelling from the options given

Core of histone molecules

(a) A – DNA; B – H1 histone; C – Histone octamer


(b) A – H1 histone; B – DNA; C – Histone octamer
(c) A – Histone octamer; B – RNA; C – H1 histone
(d) A – RNA; B – H1 histone; C – Histone octamer
61. Assertion : Cartilage (protein matrix) and bone (calcium
matrix) are rigid connective tissue.
Reason : Blood is connective tissue in which plasma is the
matrix.
EBD_7209
Mock Test - 4 MT-37

(a) A – Larynx, B – Parathyroid, C – Tracheal cartilage, 71. Match the column-I with column-II.
D – Trachea Column-I Column-II
(b) A – Naso Larynx, B – Thyroid, C – Tracheal cartilage, A. Animals which give I. Hydra
D – Trachea birth to young one
(c) A – Trachea, B – Thyroid, C – Bronchiole, D – Tracheal B. Animal which produces II. Planaria
cartilage bud
(d) A – Epiglottis, B – Thyroid, C – Tracheal cartilage, C. An animal which III. Viviparous
D – Trachea shows regeneration
67. Which of the following statement(s) is/are correct about D. Provides nutrition IV. Placenta
Mycoplasma ? to the developing
(i) Mycoplasma has no cell wall. embryo from the mother
(ii) Mycoplasma is the smallest living organism. (a) A ® (I); B ® (III); C ® (II); D ® (IV)
(iii) Mycoplasma cannot survive without O2. (b) A ® (III); B ® (I); C ® (II); D ® (IV)
(iv) Mycoplasma are pathogenic in animals and plants. (c) A ® (III); B ® (II); C ® (IV); D ® (II)
(v) True sexuality is not found in bacteria. (d) A ® (III); B ® (IV); C ® (I); D ® (II)
(vi) A sort of sexual reproduction by adopting a primitive 72. Combining of haemoglobin with O2 in lungs can be
DNA transfer from one bacterium to the other occurs. promoted by
Choose the answer from the following options (a) decreasing O2 concentration in blood
(a) All of these (b) increasing O2 concentation in blood
(b) Only (iii) (c) increasing CO2 concentration in blood
(c) (i), (ii), (iv), (v) and (vi) (d) introducing CO into blood.
(d) (i), (iii) and (vi) 73. Which one of the following groups of three animals each is
68. Assertion : Copper-T is an effective contraceptive device in correctly matched with their one characteristic morphological
human females. feature?
Reason : Copper-T prevents passage of sperms from vagina Animals Morphological feature
upwards into fallopian tubes. (a) Liver fluke, Sea - Bilateral
(a) If both Assertion and Reason are true and the Reason anemone, Sea symmetry
is the correct explanation of the Assertion. cucumber
(b) If both Assertion and Reason are true but the Reason (b) Centipede, Prawn, - Jointed appendages
is not the correct explanation of the Assertion. Sea urchin
(c) If Assertion is true but Reason is false. (c) Scorpion, Spider, - Ventral solid central
(d) If both Assertion and Reason are false. Cockroach nervous system
69. Match column-I with column-II (d) Cockroach, - Metameric
Column-I Column-II Locust, Taenia segmentation
A. Ultrafiltration I. Henle's loop 74. The blood calcium level is lowered by the deficiency of
B. Concentration of urine II. Ureter (a) Parathormone
C. Transport of urine III. Urinary bladder (b) Thyroxine
(c) Both calcitonin and parathormone
D. Storage of urine IV. Malpighian corpuscles
(d) Calcitonin
V. Proximal convoluted 75. Which of the following statements is incorrect?
tubules (i) Absorption of simple sugar, alcohol, some water and
(a) A ® (IV); B ® (I); C ® (II); D ® (III) medicines takes place in stomach.
(b) A ® (IV); B ® (III); C ® (II); D ® (I) (ii) Maximum water absorption occurs in small intestine.
(c) A ® (V); B ® (IV); C ® (II); D ® (III) (iii) Small intestine is the major site of digestion and
(d) A ® (V); B ® (IV); C ® (I); D ® (II) absorption of food.
(e) A ® (VI); B ® (I); C ® (III); D ® (II) (iv) Fatty acid and glycerol are absorbed by lacteals.
70. Tendons and Ligaments belong to (v) Nothing is absorbed in mouth and large intestine.
(a) Muscular tissue (a) (i), (iv) and (v) (b) only (v)
(b) Epithelial tissue (c) only (iv) (d) (ii) and (iii)
(c) Fibrous connective tissue 76. The wishbone of the bird is derived from
(a) skull (b) pectoral girdle
(d) Areolar connective tissue
(c) pelvic girdle (d) hindlimb
MT-38 Mock Test - 4
77. Select the correct statement regarding the specific disorder of 81. Which one of the following pairs is incorrectly matched?
muscular or skeletal system (a) Glucagon - Beta cells (source)
(a) Muscular dystrophy-Age related shortening or muscles. (b) Somatostatin - Delta cells (source)
(b) Osteoporosis-Decrease in bone mass and higher
(c) Corpus luteum - Relaxin (secretion)
chance of fractures with advancing age.
(d) Insulin - Diabetes mellitus (disease)
(c) Myasthenia gravis-Autoimmune disorder which
inhibits sliding of myosin filaments. 82. Assertion : Presence of HCl in stomach is necessary for the
(d) Gout - Inflammation of joints due to extra deposition of process of digestion.
calcium. Reason : HCl kills and inhibits the growth of bacteria in the
78. Study the diagram of Synapse stomach.
(a) If both Assertion and Reason are true and the Reason
is the correct explanation of the Assertion.
(b) If both Assertion and Reason are true but the Reason
is not the correct explanation of the Assertion.
(c) If Assertion is true but Reason is false.
(d) If both Assertion and Reason are false.
83. Which of the following is correctly stated as it happens in
the common Cockroach ?
(a) Malpighian tubules are excretory organs projecting out
from the colon
(b) Oxygen is transported by haemoglobin in blood
I. Which numbered label indicate the location of the
receptor molecules? (c) Nitrogenous excretory product is urea
II. Which number points to a synaptic vesicles? (d) The food is grounded by mandibles and gizzard
III. Which number point to neurotransmitter? Cockroach takes the food after grounding by its
IV. Which number points to synaptic cleft? mandibles and gizzard.
I II III IV
84. Which one of the following statements in regard to the
(a) C A B D
excretion by the human kidneys is correct?
(b) B A C D
(c) C A D B (a) Descending limb of Loop of Henle is impermeable to
(d) C D A B water
79. Which of the following statements are true/false? (b) Distal convoluted tubule is incapable of reabsorbing
(i) In Torpedo the electric organs are capable of generating HCO3
strong electric shock to paralyze the prey. (c) Nearly 99 per cent of the glomerular filtrate is
(ii) Bony fishes use pectoral, pelvic, dorsal anal and caudal reabsorbed by the renal tubules
fins in swimming.
(d) Ascending limb of Loop of Henle is impermeable to
(iii) Amphibian skin is moist and has thick scales.
(iv) Birds are poikilothermous animals. electrolytes
(v) The most unique mammalian characteristic is the 85. Column-I Column-II
presence of milk producing mammary glands by which A. Initiation of I. Anaphase-I
the young ones are nourished. spindle fibres
(a) (i), (ii) and (iii) are true; (iv), E are false
B. Synthesis of II. Zygotene
(b) (i), (ii) and (v) are true; (iii) and (iv) are false
(c) (i), (iv) and (v) are true; (ii) and (iii) are false RNA and protein
(d) (i), (ii) and (iv) are false; (iii) and (v) are true C. Action of endonuclease III. G1 phase
80. Which of the following statements about Hormones is/are D. Movement of chromatids IV. Pachytene
correct? towards opposite poles
(i) Hormones are non-nutrient chemicals.
V. Anaphase-II
(ii) Hormones act as intercellular messengers.
(iii) Hormones are produced in trace amount. The correct match is
(iv) Hormones may be proteins, steroids, glycoproteins and (a) A ® (II); B ® (III); C ® (IV); D ® (V)
bigenic amines. (b) A ® (III); B ® (II); C ® (I); D ® (V)
(a) All of these (b) (i), (ii) and (iii) (c) A ® (I); B ® (III); C ® (V); D ® (IV)
(c) Only (iv) (d) (i) and (iii)
(d) A ® (V); B ® (III); C ® (I); D ® (II)
EBD_7209
Mock Test - 4 MT-39

86. Figure refers to reproductive system of female cockroach. 88. An enzyme increases the rate of a reaction by
The correct labellings indicated by alphabets are (a) supplying the energy required to start the reaction.
respectively (b) increasing the rate of random collisions of molecules.
(c) removing the product of the reaction so allowing it to
continue.
(d) bringing the reacting molecules into precise orientation
with each other.
89. Assertion : Implantation is the process of attachment of
blastocyst on uterine endometrium.
Reason : Implantation is controlled by trophoblast and
occurs by decidual cell reaction.
(a) If both Assertion and Reason are true and the Reason
(a) A-Spermatheca, B-Collateral glands, C-Gonapophyses is the correct explanation of the Assertion.
(b) A-Phallic gland, B-Collateral glands, C-Gonapophyses (b) If both Assertion and Reason are true but the Reason
(c) A-Spermatheca, B-Seminal vesicle, C-Gonapophyses is not the correct explanation of the Assertion.
(d) A-Spermatheca, B-Collateral glands, C-Tegmina (c) If Assertion is true but Reason is false.
87. Assertion : EEG is of immense diagnostic value in the cardiac (d) If both Assertion and Reason are false.
diseases. 90. Assertion : Medulla oblongata causes reflex actions like
vomiting, coughing and sneezing.
Reason : Defects in cardiac functions can be reflected in Reason : It has many nerve cells which control autonomic
changes in the pattern of electrical potentials recorded in reflexes.
the EEG. (a) If both Assertion and Reason are true and the Reason
(a) If both Assertion and Reason are true and the Reason is the correct explanation of the Assertion.
is the correct explanation of the Assertion. (b) If both Assertion and Reason are true but the Reason
(b) If both Assertion and Reason are true but the Reason is not the correct explanation of the Assertion.
is not the correct explanation of the Assertion. (c) If Assertion is true but Reason is false.
(c) If Assertion is true but Reason is false. (d) If both Assertion and Reason are false.
(d) If both Assertion and Reason are false.
MT-40 Mock Test - 4

ANS W ER KEY
1 (c) 14 (b) 27 (b) 40 (d) 53 (a) 66 (d) 79 (c)
2 (c) 15 (d) 28 (c) 41 (a) 54 (c) 67 (c) 80 (a)
3 (a) 16 (d) 29 (c) 42 (a) 55 (c) 68 (c) 81 (a)
4 (b) 17 (c) 30 (c) 43 (d) 56 (d) 69 (a) 82 (b)
5 (a) 18 (b) 31 (b) 44 (d) 57 (b) 70 (c) 83 (d)
6 (a) 19 (b) 32 (a) 45 (c) 58 (b) 71 (b) 84 (c)
7 (c) 20 (d) 33 (a) 46 (c) 59 (b) 72 (a) 85 (a)
8 (d) 21 (b) 34 (c) 47 (b) 60 (a) 73 (c) 86 (a)
9 (a) 22 (a) 35 (a) 48 (a) 61 (b) 74 (a) 87 (d)
10 (d) 23 (d) 36 (b) 49 (d) 62 (c) 75 (b) 88 (d)
11 (c) 24 (a) 37 (d) 50 (c) 63 (d) 76 (b) 89 (b)
12 (a) 25 (d) 38 (b) 51 (d) 64 (b) 77 (b) 90 (a)
13 (d) 26 (a) 39 (b) 52 (a) 65 (b) 78 (a)

BOTANY chain is exactly opposite thymine (T), a pyramidine of


the other chain. Similarly, cytosine (C), a pyrimidine lies
2. (c) Cytokinins induce cell division in plants and thus
opposite guanine (G), a purine. This allows a sort of lock
influence the plant growth by controlling cell division.
and key arrangement between large sized purine and small
4. (c) The water potential is the sum of the osmotic potential
sized pyrimidine. It is strengthened by the appearance of
(usually negative) and the pressure potential (usually
hydrogen bonds between the two.
positive), so W.P. = – 0.24 + 0.16 = –0.08 mPa.
31. (b) Stephentiales (1727) coined the term root pressure, Root
7. (c) Phellem, phellogen and phelloderm are collectively called
pressure is developed when rate of water absorption is
periderm.
more than the rate of transpiration.
9. (a) The biomass available for consumption by the herbivores
35. (a) According to some Botanists, Mosses originated from
and the decomposers is called net primary productivity.
algae. Protonema of mosses is similar to certain algae.
It is equal to the rate of organic matter created by
42. (a) We know that all biological reactions are catalysed by
photosynthesis minus the rate of respiration and other
special catalysts called enzyme, thus enzymes are defined
losses.
as biological proteins. We also know that enzymes are
12. (a) Pyrenoids are the rounded bodies found in the chloroplast
small organic molecules which are weakly held to the
of green algae and are the centres of conversion of
protein and can be easily separated by dialysis. Therefore
glucose to starch and also collection of starch.
chemically all enzymes are globular proteins. A few
21. (b) Systematics is related with classification of organisms.
exceptions are ribozymes.
In classification the organisms are grouped on the basis
of their characters or phylogeny, etc. 43. (d) Four nuclei formed are not similar to parent ones because
23. (d) In the DNA molecule, there are two strands which run they are haploid in nature. Homologous chromosome
anti-parallel one in 5' - 3' direction and other in 3' -5' pairs exchange parts because in crossing over exchange
direction, the two chains are held together by hydrogen of chromatid arms takes place and number of
bonds between their bases. Adenine (A), a purine of one chromosomes gets halved.
EBD_7209
Mock Test - 4 MT-41

ZOOLOGY – Spider belongs to class-Archnida


– Scorpion belongs to class-Archnida
49. (d) During the period when the number of coal-burning
– Cockroach belongs to class-Insecta
factories in England was increasing (during the Industrial 74. (a) A peptide hormone secreted by the parathyroid gland in
Revolution) it was noticed that the number of melanic response to low levels of calcium in the blood. It acts to
individuals of the species of Peppered Moth (Biston maintain normal blood levels of calcium by increasing
betularia) was becoming more common. Originally rare in the number of osteoclasts, which break down the bone
the population of normally light-colored moths, the matrix and release calcium into the blood.
frequency of the melanic form increased in polluted areas 77. (b) Major causative factors of osteoporosis are imbalances
until it was over 90%. This change in color has come to of hormones like calcitonin of thyroid, parathormone of
be known as “industrial melanism.” parathyroids, and sex hormones and deficiencies of
51. (d) AIDS virus infects T4 lymphocytes (also called Helper calcium and vitamin D.
cells). Cytotoxic T cells are called T8 lymphocytes. 81. (a) Glucagon is secreted by a-cells of the islets of Langerhans
55. (c) This age pyramid represents the declining population of in the pancreas. It increases the concentration of glucose
any organism. Population decline is the reduction over in the blood by stimulating the breakdown of glycogen.
time in region’s census. It can be caused for several 82. (b) Presence of hydrochloric acid in stomach is necessary
reasons that includes heavy immigration disease, famine for digestion because acidic medium activates the action
or sub-replacement fertility. of gastric juice. HCl maintains a strong acidic pH of about
57. (b) Pneumonia disease is spreaded by the organism 1-2 in the stomach. At this acidic pH inactive pepsinogen
Streptococcus pneumoniae and the mode of infection is is spontaneously hydrolysed to active pepsin and inactive
by droplet infection. pro-rennin is converted to active rennin. Pepsin and
58. (b) If a normal visioned man marries a woman whose father rennin digest proteins to peptones and proteoses. In
was also colourblind. Then his wife would be carrier of addition, HCl helps to kill and inhibit the growth of
this disease if her mother was normal. This trait passed bacteria and other harmful organisms that may enter in
on to children but daughters produced by this couple the stomach along with the food.
are carrier not colourblind. 50% of sons would be 83. (d) Cockroach takes the food after grounding by its
colourblind. mandibles and gizzard.
61. (b) Cartilage comprises of mucopolysaccharide called 84. (c) Urine formation involves three main process called,
chondroctin sulphate. Bone is a hard connective tissue. glomerular filtration, reabsorption and secretion. A
Blood is a fluid connective tissue. comparison of the volume of the filtrate formed per day
63. (d) Leydig cells or interstitial cells, which are present in the (which is 180 litres per day) with that of urine released
intertubular spaces produce a group of hormones called (about 1.5 litres) suggest that nearly 99 percent of the
androgens mainly testosterone. Androgens play a major glomerular filtrate is reabsorbed by the renal tubules. The
stimulatory role in the process of spermatogenesis (for- descending limb of loop of Henle is permeable to water
mation of spermatozoa). but impermeable to electrolytes. The ascending limb is
64. (b) Ribosomes are amembranous (i.e. without membrane) cell impermeable to water but allows transport of electrolytes.
organelle composed of rRNA and protein. These are Reabsorption of sodium ions and water takes place in
found in both prokaryotic and eukaryotic cells. In distal convoluted tubule.
prokaryotes, ribosomes are 70S type while in eukaryotes, 87. (d) An electrocardiogram is the recording of the various
it is 80S type. events of the cardiac cycle. Defects in cardiac functions
68. (c) Intra-uterine device (IUD) Copper-T is plastic or metal or structures are reflected in the ECG. The ECG is, therefore
object placed in the uterus by a doctor. Copper-T prevent of immense diagnostic value in cardiac diseases. On the
the fertilization of the egg or implantation of the embryo. other hand, EEG is an index of the brain functions. EEG or
73. (c) Scorpion, spider and cockroach have ventral solid central electroencephalogram represents the spontaneous
nervous system. All three belong to phylum-Arthopoda. electrical activity of the brain as recorded from the
MT-42 Mock Test - 4

electrodes placed on the scalp. EEG wave pattern obtained multiple substrates in a way that brings them together in
shows certain characterstic feature of the brain like the a precise orientation so they can react readily with one
frequency amplitude of the signals of brain. Thus any another.
deviation from the normal caused either by brain disease 89. (b) The process of attachment of the blastocyst (mammalian
or change in the physiological state of the brain can be blastula) on the endometrium of uterus is called
easily detected by EEG. implantation.
88. (d) An enzyme reduces the activation energy needed for the 90. (a) Medulla oblongata consists of accumulation of nerve
reaction to occur by binding with a substrate and straining cells and act as vital centres of many autonomic reflexes
its bonds so allowing for easier reaction, or binding like vomiting, coughing and sneezing.
EBD_7209
Mock Test

5
BOTANY 8. The symbiotic association of fungi and algae is called
(a) Lichen (b) Mycorrhiza
1. Taxonomic hierarchy refers to (c) Rhizome (d) Endomycorrhiza
(a) step-wise arrangement of all categories for 9. What is the function of the filiform apparatus present at the
classification of plants and animals. entrance of ovule?
(b) a group of senior taxonomists who decide the (a) It helps in the entry of pollen tube into a synergid.
nomenclature of plants and animals. (b) It prevents entry of more than one pollen tube into the
(c) a list of botanists or zoologists who have worked on embryo sac.
taxonomy of a species or group. (c) It brings about opening of the pollen tube.
(d) classification of a species based on fossil record.
(d) It guides pollen tube from a synergid to egg.
2. The living organisms can be unexceptionally distinguished
10. Which element is essential for the stability of chromosome
from the non-living things on the basis of their ability for
structure?
(a) interaction with the environment and progressive
evolution (a) Zn (b) Ca
(b) reproduction (c) Mo (d) Fe
(c) growth and Movement 11. Which one of the following cellular parts is correctly
(d) responsiveness to touch described?
3. Which one of the following animals may occupy more than (a) Centrioles - Sites for active RNA synthesis.
one trophic levels in the same ecosystem at the same time? (b) Lysosomes - Optimally active at a pH of about 8.5.
(a) Sparrow (b) Lion (c) Thylakoids - Flattened membranous sacs forming the
(c) Goat (d) Frog grana of chloroplasts.
4. Which one of the following acids is a derivative of (d) Ribosomes - Those on chloroplasts are larger (80S) while
carotenoids? those in the cytoplasm are smaller (70S).
(a) Indole-3 -acetic acid (b) Gibberellic acid
12. Assertion : Hydroponics is used for solution culture.
(c) Abscisic acid (d) Indole butyric acid
5. Which combination of tissues acts together to provide Reason : A balanced nutrient solution contains both essential
support to the hypocotyl of seedling? and non-essential elements.
(a) Xylem and Phloem fibres (a) If both Assertion and Reason are true and the Reason
is the correct explanation of the Assertion.
(b) Epidermis and Parenchyma
(b) If both Assertion and Reason are true but the Reason
(c) Xylem and Parenchyma
is not the correct explanation of the Assertion.
(d) Epidermis and Collenchyma
(c) If Assertion is true but Reason is false.
6. Jaya and Ratna developed for green revolution in India are
(d) If both Assertion and Reason are false.
the varieties of
13. A graph that plots the rate at which CO2 is converted to
(a) Maize (b) Rice
glucose versus the wavelength of light illuminating a leaf is
(c) Wheat (d) Bajra
called
7. Which one of the following is correctly matched ?
(a) Passive transport of nutrients - ATP (a) An absorption spectrum
(b) Apoplast - Plasmodesmata (b) An action spectrum
(c) Potassium - Readily immobilisation (c) A planck constant
(d) Bakane of rice seedlings - F. Skoog (d) Enzyme kinetics
MT-44 Mock Test - 5
14. Choose the right option.

S . No. A B C D

(a) Vo lvo x Chlamydo mon as Lamina ria C h ara

(b) C ha ra La min aria Vo lvo x C h la myd omo na s

(c) Lamin aria Volvox C h la myd omo na s C h ara

(d) C h la myd omon as Cha ra Lamina ria Volvox

15. Which of the following statements about the process of 18. Select the correct statement(s)-
DNA replication is false? (i) IARI has released a mustard variety rich in vitamin C.
(a) Many different enzymes are needed for the process to
(ii) Pusa Sawani variety of Okra is resistant to aphids.
function properly.
(b) Mistakes can be corrected at multiple steps in the (iii) Hairiness of leaves provides resistance to insect pests.
process. (iv) Agriculture accounts for approximately 33% of India's
(c) Uncorrected mistakes introduce mutations into the GDP and employs nearly 62% of the population.
DNA base sequence. (a) (i) and (ii) (b) (ii) and (iii)
(d) Mistakes in the copying process are very common
(c) (i), (iii) and (iv) (d) None of these
occurrences.
16. Which of the following statements about Pollen and 19. Which of the following statement(s) is/are correct?
Pollination is false ? (i) Light reaction occurs in stroma.
(a) Evolution of the pollen grain rejected the need for (ii) Light reaction occurs in grana and ATP + NADPH2 are
swimming sperm in flowering plants. formed.
(b) At maturity, the pollen grain consists of two sperm
(iii) In stroma dark reaction occurs.
nuclei and a tube nucleus.
(c) The pollen tube enters the female gametophyte through (iv) Dark reaction is not directly light driven but is
the style. dependent on the products (ATP + NADPH2) formed
(d) The pollen grain makes twice the genetic contribution to in light reaction.
endosperm cells than it does to the cells of the embryo. (a) (i), (ii) and (iv) are correct.
17. In meiosis (b) (ii), (iii) and (iv) are correct.
(a) division of nucleus occurs twice but replication of DNA
only once (c) All are correct.
(b) division of nucleus occurs twice and replication of (d) Only (ii) is correct.
DNA twice 20. Which of the following class of Algae is mostly found in salt
(c) division of nucleus occurs once and replication of DNA water?
is also once (a) Phaeophyceae (b) Rhodophyceae
(d) division of nucleus occurs once and DNA - replication
is twice (c) Chlorophyceae (d) Both (a) and (b)
EBD_7209
Mock Test - 5 MT-45

21. Pollution in big cities can be controlled to a large extent by (c) Sporogenous tissue, epidermis, middle layer, tapetum,
(i) Improving traffic condition and road. endothecium
(ii) Road side plantation (d) Sporogenous tissue, tapetum, middle layer, epidermis,
(iii) Proper disposal of garbage and domestic as well as endothecium
municipal wastes. 27. Which of the following groups of plants have Underground
(iv) Cannot be controlled stems?
(a) (i) and (ii) (b) (ii) and (iii)
(a) Potato, ginger, turmeric, Euphorbia, zaminkand
(c) (i) and (ii) (d) All of these
22. Which of the following statements about Nitrification is not (b) Potato, ginger, turmeric, zaminkand, Colocasia
correct ? (c) Potato, Citrus, Opuntia, zaminand, Colocasia
(a) Nitrobacter oxidizes nitrite to nitrate. (d) Potato, cucumber, watermelon, zaminkand, Colocasia
(b) Nitrosomonas and Nitrosococcus convert ammonium 28. Match column-I with column-II
ions to nitrite.
Column-I Column-II
(c) Nitrification reactions are energy-producing (exergonic)
reactions. A. 4C Compound I. Acetyl CoA
(d) Heterotrophic plants are more directly dependent on B. 2C Compound II. Pyruvate
the nitrifying bacteria for usable nitrogen than C. 5C Compound III. Citric acid
autotrophic plants. D. 3C Compound IV. a-ketoglutaric acid
23. Match column-I with column-II
V. Malic acid
Column-I Column-II
A. Parenchyma I. Shoot apex (a) A ® (II); B ® (V); C ® (III); D ® (I)
B. Sclerenchyma II. Mechanical tissue (b) A ® (V); B ® (I); C ® (IV); D ® (II)
C. Xylem III. Water conduction (c) A ® (III); B ® (I); C ® (IV); D ® (II)
D. Apical meristem IV. Universal tissue (d) A ® (V); B ® (III); C ® (I); D ® (II)
E. Phloem V. Sieve cells 29. Refer the experiment given below
(a) A ® (II); B ® (IV); C ® (II); D ® (I); E ® (V)
(b) A ® (IV); B ® (II); C ® (III); D ® (I); E ® (V)
(c) A ® (IV); B ® (III); C ® (I); D ® (II); E ® (V)
(d) A ® (IV); B ® (I); C ® (III); D ® (II); E ® (V)
24. Interactions in which the consumer lives within the body of
the host and slowly damages the host are referred to as
(a) Commensalism (b) Parasitism After a few days, which of the following will have occurred?
(c) Mutualism (d) Competition (a) A rise in level X and a drop in level Y.
25. Which of the following regarding water transport is true ?
(b) A drop in level X and a drop in level Y.
(a) Root pressure is sufficient to drive xylem sap movement.
(b) Bulk flow is not a mechanism by which water and (c) A rise in level X and a rise in level Y.
minerals are transported. (d) A drop in level X and a rise in level Y.
(c) The cohesive nature of water is central to water 30. One of the most important functions of Botanical gardens is
movement in a plant. that
(d) None of the above (a) they provide a beautiful area for recreation
(b) one can observe tropical plants there
26.
(c) they allow ex-situ conservation of germ plasm
Connective
C (d) they provide the natural habitat for wildlife
E 31. Which two distinct microbial processes are responsible for
A the release of fixed nitrogen as dinitrogen gas (N2) to the
D
atmosphere?
B
(a) Anaerobic ammonium oxidation and Denitrification
The above diagram refers to a T. S. of anther. Identify A to E
(b) Aerobic nitrate oxidation and Nitrite reduction
respectively-
(c) Decomposition of organic nitrogen and Conversion of
(a) Sporogenous tissue, tapetum, epidermis, middle layer,
endothecium dinitrogen to ammonium compounds
(b) Sporogenous tissue, epidermis, tapetum, middle layer, (d) Enteric fermentation in cattle and Nitrogen fixation by
endothecium Rhizobium in root nodules of legumes
MT-46 Mock Test - 5
32. Choose the correct option. 36. The okazaki fragments in DNA chain growth
(a) polymerize in the 3’ - to - 5’ direction and forms replication
fork
(b) prove semi-conservative nature of DNA replication
(c) polymerize in the 5’ - to - 3’ direction and explain
3’ - to - 5’ DNA replication
(d) result in transcription.
37. In PS-I the reaction centre Chl a has absorption maxima at
_____________, while in PS-II the reaction centre Chl a
has absorption maxima at ___________.
(a) A-Epicarp, B-Mesocarp, C-Seed, D-Endocarp
(a) P680, P700 (b) P700, P680
(b) A-Epicarp, B-Mesocarp, C-Ovule, D-Endocarp
(c) P800, P600 (d) P700, P900
(c) A-Epicarp, B-Mesocarp, C-Ovary, D-Endocarp
38. Product of biotechnology is
(d) A-Epicarp, B-Mesocarp, C-Embryo, D- Endocarp
(a) transgenic crop (GM crop)
33. Two genes R and Y are located ver y close on the
(b) humulin
chromosomal linkage map of maize plant. When RRYY and
(c) biofertilizer
rryy genotypes are hybridized, the F2 generation will show
(d) All of these
(a) segregation in the expected 9 : 3 : 3 : 1 ratio
39. The base pairs of DNA are correctly shown as
(b) segregation in 3 : 1 ratio
(a) A º T and C = G (b) A = T and C = G
(c) higher number of the parental types
(c) A = T and C º G (d) A º T and C º G
(d) higher number of the recombinant types.
40. It shows correct chronological order of the events occuring
34. Which of the following statements are false ?
during callus culture
(i) C2H4 breaks seed and bud dormancy.
(ii) ABA stimulates the opening of stomata. (a) Callus ® Cell division ® Explant ® Addition of
(iii) Cytokinin is primarily concerned with cell division. cytokinin ® Acquire meristematic property
(iv) ABA is synergistic to GA. (b) Explant ® Callus ® Cell division ® Addition of
(a) (i), (ii) and (iii) (b) (i) and (ii) cytokinin ® Cells acquire meristematic property
(c) (ii) and (iv) (d) (i) and (iii) (c) Explant ® Cell division ® Callus ® Addition of
35. The following diagram is of a typical cell cycle. cytokinin ® Cells acquire meristematic property
(d) Callus ® Explant ® Cell division ® Addition of
cytokinin ® Cells acquire meristematic property
41. Assertion: Many plants are propagated vegetatively even
though they bear seeds.

X Y
Reason: Potatoes multiply by tubers, apple by cutting.
G0 (a) If both Assertion and Reason are true and the Reason
is the correct explanation of the Assertion.
(b) If both Assertion and Reason are true but the Reason
es is is not the correct explanation of the Assertion.
Cytokin se
ha
p e
Telo phas ase (c) If Assertion is true but Reason is false.
a h
An etap (d) If both Assertion and Reason are false.
se
ha
M

M
op

Z 42. Assertion : Ginger has a prostrate-growing rhizome.


Ph

Pr
as
e

Reason : Shoot growth is not effected by gravity.


(a) If both Assertion and Reason are true and the Reason
is the correct explanation of the Assertion.
(b) If both Assertion and Reason are true but the Reason
Mark the correct option. is not the correct explanation of the Assertion.
(a) X - G1; Y - S; Z - G2 (b) X - G2; Y - S; Z - G1 (c) If Assertion is true but Reason is false.
(c) X - G0; Y - S; Z - G2 (d) None of these (d) If both Assertion and Reason are false.
EBD_7209
Mock Test - 5 MT-47

43. When huge amount of sewage is dumped into a river, the 51. Which one of the following four glands is correcly matched
BOD will with the accompanying description?
(a) Increase (b) Remain unchanged (a) Thyroid — Hyperactivity in young
(c) Slightly decrease (d) Decrease children causes cretinism
44. Assertion : No two species can occupy the same ecological (b) Thymus — Starts undergoing atrophy
niche in a habitat. after puberty
(c) Parathyroid — Secretes parathormone,
Reason: A habitat can contain only one ecological niche.
which promotes movement of
(a) If both Assertion and Reason are true and the Reason
is the correct explanation of the Assertion. calcium ions from blood into
(b) If both Assertion and Reason are true but the Reason bones during classification
is not the correct explanation of the Assertion. (d) Pancreas — Delta cells of the islets of
(c) If Assertion is true but Reason is false. Langerhans secrete a hormone,
(d) If both Assertion and Reason are false. which stimulates glycolysis in liver
45. Main objective of production/use of herbicide resistant GM 52. Mature mammalian sperm are stored in the ____ prior to
crops is to their release during ejaculation.
(a) eliminate weeds from the field without the use of manual (a) seminiferous tubules
labour. (b) scrotum
(b) eliminate weeds from the field without the use of (c) vas deferens
herbicides. (d) epididymis
(c) encourage eco-friendly herbicides. 53. Consider the statements given below regarding
(d) reduce herbicide accumulation in food articles for health contraception and answer as directed thereafter:
safety. (i) Medical Termination of Pregnancy (MTP) during first
trimester is generally safe.
ZOOLOGY (ii) Generally chances of conception are nil until mother
breast-feeds the infant upto two years.
46. An organism is in the same class but not in the same family. (iii) Intrauterine devices like copper-T are effective
It may belong to same contraceptives.
(a) Genus (b) Species (iv) Contraception pills may be taken upto one week after
(c) Variety (d) Order coitus to prevent conception.
47. Which one of the following statements about Mycoplasma Which two of the above statements are correct?
is wrong ? (a) (i) and (ii) (b) (iii) and (iv)
(a) They are pleomorphic. (c) (i) and (iii) (d) (i) and (ii)
(b) They are sensitive to penicillin. 54. The foetal ejection reflex in humans triggers release of
(c) They cause diseases in plants. (a) oxytocin from maternal pituitary
(d) They are also called PPLO. (b) oxytocin from foetal pituitary
48. An organic substance bound to an enzyme and essential for (c) human Chorionic Gonadotropin (hCG) from placenta
its activity is called (d) human Placental Lactogen (hPL) from placenta
(a) Apoenzyme (b) Isoenzyme 55. Which one of the following pairs of animals comprises
(c) Coenzyme (d) Holoenzyme ‘Jawless fishes’?
49. Which one of the following statements about certain given (a) Mackerals and Rohu
animals is correct? (b) Lampreys and Hag fishes
(a) Round worms (Aschelminthes) are pseudocoelomates. (c) Guppies and Hag fishes
(b) Molluscs are acoelomates. (d) Lampreys and Eels
(c) Insects are pseudocoelomates. 56. The processes by which DNA forms mRNA and mRNA
(d) Flatworms (Platyhelminthes) are coelomates. forms protein are respectively
50. A patient is generally advised to specially, consume more (a) Translation and Transcription
meat, lentils, milk and eggs in diet only when he suffers from (b) Transcription and Replication
(a) Scurvy (b) Kwashiorkor (c) Transcription and Translation
(c) Rickets (d) Anemia (d) Replication and Translation.
MT-48 Mock Test - 5

57. When CO2 concentration in blood increases, breathing 63. A healthy person eats the following diet - 5 gm raw sugar, 4
becomes gm albumin, 10 gm pure buffalo ghee adultrated with 2 gm
(a) shallower and slow vegetable ghee (hydrogenated vegetable oil) and 5 gm lignin.
(b) there is no effect on breathing How many calories he is likely to get?
(c) slow and deep (a) 144 (b) 126
(d) faster and deeper (c) 164 (d) 112
58. Match the column-I with column-II and choose the right 64. Diversity in the type of breaks of finches adapted to different
option feeding habits on the Galapagos Islands, as observed by
Column-I Column-II Darwin, provides evidence for
A. Statins I. Yeast (a) Intraspecific variations.
B. Ethanol II. Blood-cholesterol (b) Intraspecific competition.
(c) Interspecific competition.
lowering agent
(d) Origin of species by natural selection.
C. Dung III. Insect-resistant plant
65. Which one of the following is the true description about an
D. Bt-cotton IV. Biogas
animal concerned?
(a) A ® (II); B ® (I); C ® (IV); D ® (III)
(a) Earthworm - The alimentary canal c on s i s t s
(b) A ® (III); B ® (IV); C ® (I); D ® (II)
of a sequence of pharynx,
(c) A ® (I); B ® (II); C ® (III); D ® (IV)
oesophagus, stomach, gizzard and
(d) A ® (IV); B ® (II); C ® (I); D ® (III)
intestine
59. Birth control pills check ovulation in female by inhibiting (b) Frog - Body divisible into three regions -
the secretion of head, neck and trunk
(a) follicle stimulating hormone (FSH) (c) Rat - Left kidney is slightly higher in
(b) luteinizing hormone (LH) position than the right one
(c) both (a) and (b) (d) Cockroach - 10 pairs of spiracles (2 pairs on thorax
(d) none of these and 8 pairs on abdomen)
60. When number of chromosomes is already reduced to half in 66. Genetic engineering has been successfully used for
the first reductional division of meiosis, where is the producing
necessity of second meiotic division? (a) transgenic mice for testing safety of polio vaccine before
(a) The division is required for the formation of four use in humans
gametes (b) transgenic models for studying new treatments for
(b) Division ensures equal distribution of haploid certain cardiac diseases
chromosomes (c) transgenic cow – rosie which produces high fat milk
(c) Division ensures equal distribution of genes on for making ghee
chromosomes (d) animals like bulls for farm work as they have super
(d) Division is required for segregation of replicated power
chromosomes 67. The figure given below shows human urinary system with
61. Bell-shaped polygonal pyramid indicates structures labelled A to D. Select option which correctly
(a) High percentage of young individuals identifies them and gives their characteristics and /or
(b) Moderate percentage of young individuals functions.
(c) Low percentage of young individuals
(d) Low percentage of old individuals
62. Lactose operon produces enzymes
(a) b-galactosidase, permease and glycogen synthetase.
(b) b-galactosidase, permease and transacetylase.
(c) Permease, glycogen synthetase and transacetylase.
(d) b-galactosidase, permease and phosphoglucose
isomerase.
EBD_7209
Mock Test - 5 MT-49

(a) B-pelvis-broad funnel shaped space inner to hilum, C. Vasectomy III. Prevents ovulation
directly connected to loops of Henle. D. Copper T IV. Semen contains no
(b) C-Medulla-inner zone of kidney and contains complex sperms
nephrons. (a) A ® (III); B ® (I); C ® (IV); D ® (II)
(c) D - Cortex - outer part of kidney and do not contain any
(b) A ® (IV); B ® (I); C ® (II); D ® (III)
part of nephrons
(c) A ® (III); B ® (IV); C ® (I); D ® (II)
(d) A-Adrenal gland - located at the anterior part of kidney.
Secrete Catecholamines which stimulate glycogen (d) A ® (II); B ® (III); C ® (I); D ® (IV)
breakdown. 71. Assertion : Symptoms of emphysema develops when a
68. Identify the correct match for the given apparatus. person living on plains ascends and stays on a mountain.
Reason : Air pressure and partial pressure of oxygen falls
with the rise in altitude.
(a) If both Assertion and Reason are true and the Reason
is the correct explanation of the Assertion.
(b) If both Assertion and Reason are true but the Reason
is not the correct explanation of the Assertion.
(c) If Assertion is true but Reason is false.
(d) If both Assertion and Reason are false.
72. Which one of the following statement is correct in relation
to Honey bees?
(a) Apis indica is the largest wild honey bee in India
(b) Honey is predominantly sucrose and arabinose
Apparatus Function (c) Beewax is a waste product of honey bees
(d) Communication in honey bees was discovered by Karl
(a) Gene gun Vectorless direct gene transfer
Von Frisch
(b) Column Separation of chlorophyll
73. Male and Female Cockroaches can be distinguished
chromatograph pigments
externally through
(c) Stirred tank Carry out fermentation
bioreactor process (a) anal styles in male
(d) Respirometer Finding out rate of respiration (b) anal cerci in female
69. Match column-I with column-II (c) anal style and antennae in female
Column-I Column-II (d) both (b) and (c)
A. Collagen I. Glucose transport 74. Inheritances of skin colour in humans is an example of
B. Trypsin II. Binding with some (a) Point mutation
chemical like for smell, taste
(b) Polygenic inheritance
and hormones
(c) Codominance
C. Insulin III. Hormone
D. Antibody IV. Enzyme (d) Chromosomal aberration
E. Receptor V. Intercellular ground 75. A stage in cell division is shown in the figure. Select the
substance answer which gives correct identification of the stage with
F. GLUT-4 VI. Fight infectious agents its characteristics.
(a) A ® V; B ® IV;C;C ® III; D ® VI; E ® II; F ® I (a) Late anaphase Chromosomes move away from
(b) A ® II; B ® III; C ® IV;; D ® V; E ® VI; F ® I equatorial plate, Golgi complex
(c) A ® VI; B ® II; C ® I; D ® V;; E ® VI; F ® III not present
(d) A ® I; B ® IV; C ® III; D ® VI; E ® II; F ® V (b) Cytokinesis Cell plate formed, mitochondria
70. Match column-I with column-II. distributed between two
Column-I Column-II daughter cells
Method Mode of Action (c) Telophase Endoplasmic reticulum and
A. The pill I. Prevents sperms nucleolus not reformed yet
reaching cervix (d) Telophase Nuclear envelop reforms, golgi
B. Condom II. Prevents implantation complex reforms
MT-50 Mock Test - 5
76. RNA primers are necessary in DNA synthesis because 81. Which one of the following statements is correct with respect
(a) DNA polymerase can only add to an existing strand of to Immunity?
nucleotides. (a) Preformed antibodies need to be injected to treat the
(b) DNA polymerase can only add to an existing DNA bite by a viper snake.
strand. (b) The antibodies against small pox pathogen are
(c) DNA primase is the first enzyme in the replication produced by T – lymphocytes.
complex. (c) Antibodies are protein molecules, each of which has
(d) All of the above four light chains.
77. Which of the following statement about Cell junctions is (d) Rejection of a kidney graft is the function of
false? B-lymphocytes.
(i) All the cells of the epithelium are held together with 82. Which of the following statement (s) is/are true ?
little intercellular materials. (i) Urine is hypertonic in distal convoluted tubule.
(ii) In almost all animal tissues specialized junction provide (ii) When the urine passes into collecting tubule, it becomes
both structural and functional link between its individual hypotonic.
cells. (iii) Urine is isotonic in proximal convoluted tubule.
(iv) Urine becomes more and more hypotoinic as it passes
(iii) Tight junctions help to stop substances from leaking
through Henle's loop.
across a tissue.
(a) (i) and (iv) (b) (i), (ii) and (iii)
(iv) Adhering junctions provide cementing to keep
(c) (ii) and (iii) (d) (iii) only
neighbouring cells together.
83. The diagram given here is the standard ECG of
(v) Gap junctions provide cytoplasmic channels between
a normal person, the P-wave represents the
cells for passage of ions, small molecules and sometimes
big molecules.
(a) (ii) and (iii) (b) (i) and (ii)
(c) Only (v) (d) None of these
78. Read the following statements and identify the correct
options.
Na+ – K+ pump (a) Initiation of the ventricular contraction
(i) Needs energy (ATP) to work (b) Beginning of the systole
(ii) Expels 3 Na+ for every 2K+ ions imported (c) End of systole
(iii) Works against a concentration gradient (d) Contraction of both the atria
(iv) Maintains resting potential 84. The accompanying diagram shows the structure of a neuron.
Identify A to E.
(a) All are corect
(b) Only (ii) and (iii) are correct
(c) Only (i) and (iii) are correct
(d) None is correct
79. Which of the following statement(s) is/are false about
internal fertilization?
(i) Male gametes are motile.
(ii) Male gametes are non-motile.
(iii) Male gametes are produced in large number.
(iv) Male gametes are produced in small number.
(v) There is a significant reduction in the number of eggs
produced.
(a) (i), (iii) and (v) (b) (iii) and (iv)
(c) (ii) and (iv) (d) Only (v)
80. The class of enzymes that catalyze the removal of a group
from a substrate without addition of water, leaving double
bonds, is
(a) transferases (b) dehydrogenases
(c) hydrolases (d) lyases
EBD_7209
Mock Test - 5 MT-51

A B C 88. Assertion : Muscle contraction force increases with rise in


(a) Nerve fibre Cyton or cell body Schwann cell strength of stimulus.
(b) Dendrites Cyton or cell body Schwann cell Reason : This is due to increased contraction of individual
muscle fibres with increase in stimulus strength.
(c) Dendrites Nerve cell Schwann cell
(a) If both Assertion and Reason are true and the Reason
(d) Dendrites Cyton or cell body Nerve cell
is the correct explanation of the Assertion.
85. The most important function of Endoplasmic Reticulum is (b) If both Assertion and Reason are true but the Reason
(a) Protein synthesis is not the correct explanation of the Assertion.
(c) If Assertion is true but Reason is false.
(b Nourishing the nucleus (d) If both Assertion and Reason are false.
(c) Secretion of materials 89. The eye of octopus and eye of cat show different patterns
(d) To give shape to the cell of structure, yet they perform similar function. This is an
example of
86. Match column-I with column-II.
(a) Homologous organs that have evolved due to divergent
Column-I Column-II
evolution.
A. Adrenohypophysis I. Epinephrine (b) Analogous organ s th at h ave evolved due to
B. Adrenal medulla II. Somatotropin convergent evolution.
C. Parathyroid gland III. Thymosin (c) Analogous organs that have evolved due to divergent
D. Thymus gland IV. Calcitonin evolution.
(d) Homologous organs that have evolved due to
(a) A ® (III); B ® (I); C ® (IV); D ® (II)
convergent evolution.
(b) A ® (I); B ® (II); C ® (III); D ® (IV)
90. Assertion : Urinary bladder and ureters are lined by
(c) A ® (II); B ® (I); C ® (IV); D ® (III) transitional epithelium.
(d) A ® (IV); B ® (III); C ® (II); D ® (I) Reason : Ureters carry the urine to urinary bladder where it
87. Pulmonary artery carries is stored temporarily.
(a) If both Assertion and Reason are true and the Reason
(a) deoxygenated blood from heart to lungs
is the correct explanation of the Assertion.
(b) deoxygenated blood from lungs to heart (b) If both Assertion and Reason are true but the Reason
(c) oxygenated blood from heart to lungs is not the correct explanation of the Assertion.
(c) If Assertion is true but Reason is false.
(d) oxygenated blood from lungs to heart (d) If both Assertion and Reason are false.
MT-52 Mock Test - 5

ANSWER KEY
1 (a) 13 (b) 25 (c) 37 (b) 49 (a) 61 (b) 73 (a) 85 (a)

2 (b) 14 (a) 26 (a) 38 (d) 50 (b) 62 (b) 74 (b) 86 (c)

3 (a) 15 (d) 27 (b) 39 (c) 51 (b) 63 (a) 75 (d) 87 (a)

4 (c) 16 (d) 28 (b) 40 (c) 52 (d) 64 (d) 76 (a) 88 (c)

5 (d) 17 (a) 29 (a) 41 (a) 53 (c) 65 (d) 77 (d) 89 (b)

6 (b) 18 (c) 30 (c) 42 (b) 54 (a) 66 (a) 78 (a) 90 (b)

7 (c) 19 (b) 31 (b) 43 (a) 55 (b) 67 (d) 79 (c)

8 (a) 20 (a) 32 (a) 44 (c) 56 (c) 68 (c) 80 (d)

9 (a) 21 (b) 33 (c) 45 (d) 57 (d) 69 (a) 81 (a)

10 (c) 22 (d) 34 (c) 46 (d) 58 (a) 70 (a) 82 (d)

11 (c) 23 (b) 35 (a) 47 (b) 59 (c) 71 (a) 83 (d)

12 (c) 24 (b) 36 (c) 48 (c) 60 (d) 72 (d) 84 (b)

BOTANY 9. (a) Filiform apparatus helps in the entry of pollen tube into
a synergid in ovule. Filiform apparatus is in the form of
2. (b) There are several factors and processes which finger like projection comprising a core of microfibrils
differentiate living beings with non-living beings like enclosed in a sheath. The filiform apparatus resembles
reproducction, respiration, growth, etc. But among them transfer cells meant for short distance movement of
reproduction is the only difference which differentiate metabolites. The filiform apparatus is responsible for
without any exception living beings with non-living
the absorption of food from the nucleus.
beings.
12. (c) Solution culture is being used for raising flowers and
3. (a) Sparrow feeds upon grains hence called primary
vegetables at home. This soilless production of plants
consumer and can also feed on insects hence called
secondary consumer at the same time in the same is called hydroponics. A solution having all the essential
ecosystem. elements in proper proportion is called normal or
4. (c) Abscisic acid (ABA), also known as abscisin II and balanced nutrient solution.
dormin, is a plant hormone. It functions in many plant 15. (d) Even though DNA replication typically occurs millions
developmental processes, including bud dormancy. of times during the life of a multicellular organism, it is
Abscisic acid is a derivative of carotenoids. It was remarkably error-free. Those errors that do occur are
called “abscisin II” originally because it was thought usually corrected with a high degree of reliability.
to play a major role in abscission of fruits. At about the 22. (d) Heterotrophic plants are less dependent on nitrogen
same time another group was calling it “dormin” obtained from nitrification since they receive some
because they thought it had a major role in bud nitrite and nitrate through their parasitic or carnivorous
dormancy. The name abscisic acid (ABA) was coined
nutritional modes.
by a compromise between the two groups.
24. (b) This is the classic definition of a parasitic interaction.
6. (b) Jaya and Ratna are two rice varieties developed for
25. (c) Water movement depends on the cohesive nature of
green revolution in India.
water to withstand the tension placed on the water
8. (a) The symbiotic association of fungi and algae is called
Lichen. column by transpiration.
EBD_7209
Mock Test - 5 MT-53

30. (c) Ex-situ conservation means off site conservation. It is ZOOLOGY


the process of protecting an endangered species of
plant or animal by removing it from an unsafe or 47. (b) While working at the Rockefeller Institute, Brown
threatened habitat and placing it or part of it under the reported isolation of a PPLO from human arthritic joint
care of man. Botanical garden is one of the method of tissue in 1938. In discussing the significance of this
ex-situ conservation. observation, Brown reported successful treatment of
33. (c) When the linked genes are situated quite close, the arthritic patients in 1949 with a new antibiotic called
chances of crossing over are highly reduced. Due to
aureomycin (Clark, 1997).
this, large number of parental gametes are formed and
48. (c) An organic substance bound to an enzyme and
only few recombinant gametes are formed. This results
essential for its activity is called coenzyme.
in higher number of parental types in F2 generation as
compared to recombinants. 49. (a) Acoelomates are animals that have no body cavity or
36. (c) Okazaki fragments in DNA are linked up by the enzyme coelom. The examples are poriferans, coelenterates,
DNA ligase. Replication always occur in 5' - 3' direction. ctenophore, platyhelminthes. Pseudo-coelomates are
Okazaki fragments synthesized on 3' - 5' DNA template, animals that have false or pseudo coelom. Examples
join to form lagging strand which grows in 3' - 5' are aschelminthes. Coelomates are animals that have
direction. true coelom enclosed by mesoderm on both sides.
41. (a) Vegetative reproduction is asexual type of reproduction Examples: from annelida to arthropoda are coelomates.
Potato, Sugarcane, Apple, etc., are multiplied by asexual
Hence roundworms are pseudocoelomates, molluscs and
means.
insects are coelomates while flatworms are acoelomates.
42. (b) Ginger is horizontal in position and generally branched
50. (b) A child may have a diet containing sufficient
and producing aerial leaves or shoots aboveground
and adventitious roots on lower side in favourable carbohydrates and fats but still suffers a serious form
season. Thus shoot growth is not effected by gravity. of malnutrition. This form of malnutrition is known as
43. (a) The degree of pollution is directly proportional to BOD, Kwashiorkar. It develops in children whose diets are
therefore more the organic pollution (specially sewage), deficient in protein.
the more would be BOD of water. 52. (d) Located on the surface of each testis, the epididymis
44. (c) The place where an organism lives is called its habitat. stores mature sperm until they are ready to be emitted
Habitats are characterised by conspicuous physical just prior to ejaculation.
features, which may include the dominant forms of plant
54. (a) Parturition is induced by a complex neuroendocrine
and animal life. Habitat may also refer to the place
mechanism. The signals for parturition originate from
occupied by an entire biological community. For
the fully developed foetus and the placenta which
example, a large number of species are found in a forest
habitat. On the other hand, the ecological niche of an induce mild uterine contractions called foetal ejection
organism represents (i) the range of conditions it can reflex. This triggers release of oxytocin from the maternal
tolerate (ii) the resources it utilises, and (iii) its functional pituitary.
role in the ecological system. A habitat can contain 55. (b) Lampreys and hag fishes are unusual, jawless fishes
many ecological niches and support a variety of species. that comprise the order Cyclostomata, so named
Each species has a distinct niche, and no two species because of the circular shape of the mouth. The brains
are believed to occupy exactly the same niche. of lampreys and hagfishes differ a lot, but they also
45. (d) Main objective of production/use of herbicide resistant
show a large number of similarities, as do all craniate
GM crops is to reduce herbicide accumulation in food
brains.
articles for health safety. GM plants have been useful
57. (d) The breathing becomes faster and deeper in order to
in many ways. Genetic modifications has made crops
more tolerant to abiotic stresses, reduced reliance on oxygenate the blood at a fast rate. Shallow and slow
chemical pesticides, enhanced nutritional value of food. breathing occurs during rest.
MT-54 Mock Test - 5

63. (a) Physiological value of carbohydrates is 4.0 kcal/g, 75. (d) Telophase is the stage of reconstitution of nuclei. The
proteins 4.0 kcal/g and of fats is 9.0 kcal/g. Hence, chromosomes that have reached their respective poles
5 g raw sugar will yield decondense and lose their individuality and collect in
5 × 4.0 = 20.0 kcal a mass in the two poles. Nuclear envelope assemble
4 g albumin (protein) will yield around chromatin mass. Nucleolus, Golgi complex and
ER reform.
4 × 4.0 = 16.0 kcal
76. (a) DNA polymerase cannot initiate the building of a
10 + 2 g of fat will yield
nucleotide strand; it can only add to an existing strand.
12 × 9.0 = 108.0 kcal
Thus, RNA primers are necessary to begin DNA
Total yield = 144 kcal.
synthesis.
64. (d) Nature select those set of characters that are best
81. (a) Preformed antibodies need to be injected to treat the
adapted to the environment. This has resulted in the bite by a viper snake. It is also a type of immunization
great diversity seen in the population of finches. which is called as passive immunization.
65. (d) Mollusca mostly oviparous and a few viviparous. The Antibodies, produced by B-cells, are typically made of
development may be direct or indirect with trochophore, basic structural units—each with two large heavy
velliger and glochidium. chains and two small light chains. B cells differentiate
66. (a) Genetic engineering has been successfully used for into plasma cells that secrete antibodies. Antibodies
producing transgenic mice. Many transgenic mice are are proteins that bind to specific antigens and mark
designed to increase our understanding of how genes them for destruction by, for example, marking them more
contribute to development of diseases. Transgenic mice recognizable to phagocytic cells. Rejection of a kidney
are being developed for use in testing the safety of graft is not a function of B lymphocyte.
82. (d) The filtrate is isotonic to blood plasma (in proximal
vaccine before they are used in human beings. For
convoluted tubule) and the filtrate becomes hypertonic
example transgenic mice are being used to test the safety
to blood plasma (in descending limb of loop of Henle).
of polio vaccine.
The filtrate is hypotonic to blood plasma (in ascending
71. (a) When a person living on plains ascend and stays on a
limb of loop of Henle) in distal convoluted tubule. ADH
mountain above 8000 feet from the sea level, he develops
make the filtrate isotonic to blood plasma.
symptoms of mountain sickness which includes 83. (d) The P-wave represents the electrical excitation (or
breathlessness, headache, dizziness, irritability, nausea, depolarisation) of the atria, which leads to the
vomiting, mental fatigue and a bluish ting on the skin, contraction of both the atria. The QRS complex
nails and lips. The rise in altitude, consequently lowers represents the depolarisation of the ventricles, which
the partial pressure of oxygen. This lowers the alveolar initiates the ventricular contraction. The contraction
partial pressure of oxygen which causes reduction in starts shortly after Q and marks the beginning of the
the diffusion of oxygen from the alveolar air to the systole.
blood. so oxygenation of blood is decr eased 85. (a) Because E.R. possesses ribosomes on their membrane
progressively, which produces the symptoms of which are responsible for protein synthesis.
mountain sickness, Emphysema. 88. (c) The muscle fibre always contracts with the maximum
72. (d) Karl Von Frisch, carried out many experiments and force and this force doesn't rise on increasing the
determined that when a foraging bee returns to the strength of the stimulus. If the stimulus is of strength
hive, it performs a waggle dance. Honey is below the threshold, then the muscle fibre doesn't
contract at all. This is known as All or None law. But
predominantly glucose and fructose. Dorsata is a bigger
the entire muscle doesn't obey this law, it means that
bee than Apis indica (a medium sized bee). Bees wax is
force of contraction of muscle increase with rise in
secreted by special wax glands to make compartments.
strength of the stimulus. This is due to the fact that the
74. (b) Inheritance of skin colour in human is controlled by
strength of the threshold stimulus varies from muscle
three genes, A, B and C which is polygenic inheritance.
fibre to muscle fibre in a muscle.
EBD_7209
Mock Test - 5 MT-55

89. (b) The eye of octopus and eye of cat show different ureters may be considerably stretched without getting
patterns of structure, yet they perform similar functions. torn when they are filled with urine. Ureters are thin
This is an example of analogous organs. Analogous muscular tubes which emerge from the hilum of each
organs have evolved due to convergent evolution . kidney. Urine enters the ureters from the renal pelvis
Analogous organs have developed in the evolutionary and is conducted along the ureters by peristaltic waves
process through adaptation of quite on their walls. Ureters from both the kidneys finally
different organisms to similar mode of life. open into urinary bladder which is a hollow muscular
90. (b) Urinary bladder and ureters of excretory system are sac. In this way urine from both the kidneys is drained
lined by transitional epithelium because it is a into the urinary bladder which stores it temporarily.
stretchable epithelium, hence the urinary bladder and

Vous aimerez peut-être aussi